53
GRE AWA John박 박박박박박 1그그: Statistical & Study Errors I.The following appeared in a memo from the vice president of a food distribution company with food storage warehouses in several cities. "Recently, we signed a contract with the Fly-Away Pest Control Company to provide pest control services at our fast-food warehouse in Palm City, but last month we discovered that over $20,000 worth of food there had been destroyed by pest damage. Meanwhile, the Buzzoff Pest Control Company, which we have used for many years, continued to service our warehouse in Wintervale, and last month only $10,000 worth of the food stored there had been destroyed by pest damage. Even though the price charged by Fly-Away is considerably lower, our best means of saving money is to return to Buzzoff for all our pest control services." Write a response in which you discuss what specific evidence is needed to evaluate the argument and explain how the evidence would weaken or strengthen the argument. II.The following appeared in a memo from the vice president of a food distribution company with food storage warehouses in several cities. "Recently, we signed a contract with the Fly-Away Pest Control Company to provide pest control services at our warehouse in Palm City, but last month we discovered that over $20,000 worth of food there had been destroyed by pest damage. Meanwhile, the Buzzoff Pest Control Company, which we have used for many years in Palm City, continued to service our warehouse in Wintervale, and last month only $10,000 worth of the food stored there had been destroyed by pest damage. Even though the price charged by Fly-Away is considerably lower, our best means of saving money is to return to Buzzoff for all our pest control services." Write a response in which you discuss what questions would need to be answered in order to decide whether the recommendation and the argument on which it is based are reasonable. Be sure to explain how the answers to these questions would help to evaluate the recommendation. However, the vice president does not answer crucial questions which if answered may undermine his recommendation. III.The following appeared in a memo from the vice president of a food distribution company with food storage warehouses in several cities. "Recently, we signed a contract with the Fly-Away Pest Control Company to provide pest control services at our warehouse in Palm City, but last month we discovered that over $20,000 worth of food there had been destroyed by pest damage. Meanwhile, the Buzzoff Pest Control Company, which we have used for many years in Palm City, continued to service our warehouse in Wintervale, and last month only $10,000 worth of the food stored there had been destroyed by pest damage. This difference in pest damage is best explained by the negligence of Fly-Away." Write a response in which you discuss one or more alternative explanations that could rival the proposed explanation and explain how your explanation(s) can plausibly account for the facts presented in the argument.

GRE writing argument brain storm

Embed Size (px)

DESCRIPTION

argument essay topics

Citation preview

Page 1: GRE writing argument brain storm

GRE AWA John박 박정어학원

1그룹 Statistical amp Study Errors

IThe following appeared in a memo from the vice president of a food distribution company with food storage warehouses in several cities

Recently we signed a contract with the Fly-Away Pest Control Company to provide pest control services at our fast-food warehouse in Palm City but last month we discovered that over $20000 worth of food there had been destroyed by pest damage Meanwhile the Buzzoff Pest Control Company which we have used for many years continued to service our warehouse in Wintervale and last month only $10000 worth of the food stored there had been destroyed by pest damage Even though the price charged by Fly-Away is considerably lower our best means of saving money is to return to Buzzoff for all our pest control services

Write a response in which you discuss what specific evidence is needed to evaluate the argument and explain how the evidence would weaken or strengthen the argument

IIThe following appeared in a memo from the vice president of a food distribution company with food storage warehouses in several cities

Recently we signed a contract with the Fly-Away Pest Control Company to provide pest control services at our warehouse in Palm City but last month we discovered that over $20000 worth of food there had been destroyed by pest damage Meanwhile the Buzzoff Pest Control Company which we have used for many years in Palm City continued to service our warehouse in Wintervale and last month only $10000 worth of the food stored there had been destroyed by pest damage Even though the price charged by Fly-Away is considerably lower our best means of saving money is to return to Buzzoff for all our pest control services

Write a response in which you discuss what questions would need to be answered in order to decide whether the recommendation and the argument on which it is based are reasonable Be sure to explain how the answers to these questions would help to evaluate the recommendation

However the vice president does not answer crucial questions which if answered may undermine his recommendation

IIIThe following appeared in a memo from the vice president of a food distribution company with food storage warehouses in several cities

Recently we signed a contract with the Fly-Away Pest Control Company to provide pest control services at our warehouse in Palm City but last month we discovered that over $20000 worth of food there had been destroyed by pest damage Meanwhile the Buzzoff Pest Control Company which we have used for many years in Palm City continued to service our warehouse in Wintervale and last month only $10000 worth of the food stored there had been destroyed by pest damage This difference in pest damage is best explained by the negligence of Fly-Away

Write a response in which you discuss one or more alternative explanations that could rival the proposed explanation and explain how your explanation(s) can plausibly account for the facts presented in the argument

However the author does not consider other alternative explanations

First the author assumes that the Analogy Palm City and Wintervalemdashstored food might be different climate proportion of damage to the actual amount of food kept in the warehouse The amount of money in proportion to its real value Saving moneyHasty decision making evidence of one month

최근 우리 회사는 The Fly-Away Pest-Control 회사와 Palm 시의 패스트 푸드 창고에서 병균퇴치 서비스와

GRE AWA John박 박정어학원

관련한 계약을 했으나 지난달 이곳의 식료품중 2만불어치 이상이 병균침해로 폐기되었다는 것을 알았다 그러는 동안 Buzzoff Pest-Control 회사는 우리 회사에서도 수년간 이곳회사의 제품을 사용해 온 회사로써 Wintervale에 있는 저장고에서 같은 서비스를 시작했는데 지난달 역시 1만불어치의 식료품이 폐기처분되었다 Fly-Away회사와 계약한 금액이 상당히 적기는 했으나 손실을 줄이려면 Buzzoff사에게 이 일을 맡겨야 한다주장 손실을 줄이려면 buzzoff사에 pest-control service를 맡겨야 한다

1 bad analogy 두 상황이 다르다 단순히 수치로 비교할 수없다 다른 요인 작용가능(지역도 다름)2 사업은 많은 데이터를 가지고 신중히 결정을 내려야하는데 단순한 하나의 사건의 결론만을 놓고

판단하는 것은 잘못된 결론을 가져올 수 있다(물류비용 등등) 3 이번에 손실이 적었다고 앞으로도 계속 적을 것이라고 볼 수만은 없다

In the memo the vice president of a food-distribution concludes that in order to save their companyrsquos money they should return to Buzzoff Company of all their pest-control services To support this argument the vice president of a food-distribution should provide objective information that proves both cities were in the same environment The vice president of a food distribution relies on unsubstantiated assumption is therefore unconvincing as it stands

First of all the vice president of the food distribution provides no information about the weather in two cites It is quite possible that the fast-food warehouse in Palm City had been destroyed by pest damage because of bad weather condition and the warehouse in Wintervale had been less destroyed by pest damage because of good weather condition Therefore without considering the weather condition in two the cites the vice presidentrsquos argument is not reliable

Second of all the vice president of the food distribution unfairly assumes that both food conditions were exactly the same condition however it is possible that the foodrsquos condition in warehouse in Palm City was originally worse than the one in Wintervale Food in bad condition quickly influences other food around it so in this case it is not all The Fly-Away Pest-Control Companyrsquos fault to make over $20000 worth of food damage

Third of all the vice president of the food distribution provides no information about how much each company charged for the pest-control service It is possible that the total price including the worth of food destroyed by pest damage in Palm City and the pest-control service is less expensive than paying for Buzzoff Company In this case Buzzoff Company charges more money therefore the company could not save their money

Fourth of all it is also possible that Palm City is naturally or geographically vulnerable to pest damage In this case it does not matter which company they hire If they just hire Buzzoff Company for the pest-control service the result would not be different Moreover they would be spending more than hiring The Fly-Away Pest-Control Company

In short this memo fails to provide the key evidence need to support its claims To make the claim stronger the vice president of a food distribution should provide information about weather in two cites and the original food conditions The vice president of the food distribution also needs to provide information about the price charged by the two companies and natural and geographical information With providing that information the vice president of a food distributionrsquos argument would be more convincing

Good transitions and organization Work on smoothening awkward sentence structuresScore 40

Hospital statistics regarding people who go to the emergency room after roller-skating accidents indicate the need for more protective equipment Within that group of people 75 percent of those who had accidents in streets or parking lots had not been wearing any protective clothing (helmets knee pads etc) or any light-reflecting material (clip-on lights glow-in-the-dark wrist pads etc) Clearly the statistics indicate that by investing in high-quality protective gear and reflective equipment roller skaters will greatly reduce their risk of being severely injured in an accident

Write a response in which you examine the stated andor unstated assumptions of the argument Be

GRE AWA John박 박정어학원

sure to explain how the argument depends on these assumptions and what the implications are for the argument if the assumptions prove unwarranted

Assumptions The author is assuming that 75 percent is a significant number protective clothing and light reflecting material mitigate severe injuries (the cautious nature of those who wear such equipment might have prevented accidents or the skills and reflex of the roller skaters drivers are the main reason for accidents)

ETS Essay Response ndash Score 5

The argument presented is limited but useful It indicates a possible relationship between a high percentageof accidents and a lack of protective equipment The statistics cited compel a further investigation of theusefulness of protective gear in preventing or mitigating roller-skating related injuries However theconclusion that protective gear and reflective equipment would greatly reducerisk of being severelyinjured is premature Data is lacking with reference to the total population of skaters and the relativelevels of experience skill and physical coordination of that population It is entirely possible that furtherresearch would indicate that most serious injury is averted by the skaters ability to react quickly andskillfully in emergency situationsAnother area of investigation necessary before conclusions can be reached is identification of the types ofinjuries that occur and the various causes of those injuries The article fails to identify the most prevalenttypes of roller-skating related injuries It also fails to correlate the absence of protective gear and reflectiveequipment to those injuries For example if the majority of injuries are skin abrasions and closed-headinjuries then a case can be made for the usefulness of protective clothing mentioned Likewise if injuriesare caused by collision with vehicles (eg bicycles cars) or pedestrians then light-reflective equipmentmight mitigate the occurences However if the primary types of injuries are soft-tissue injuries such astorn ligaments and muscles back injuries and the like then a greater case could be made for training andexperience as preventative measures

Reader Commentary for Essay Response ndash Score 5This strong response gets right to the work of critiquing the argument observing that it indicates apossible relationship but that its conclusion is premature It raises three central questions that ifanswered might undermine the soundness of the argumentbull What are the characteristics of the total population of skatersbull What is the usefulness of protective or reflective gear in preventing or mitigating roller skatingrelatedinjuriesbull What are the types of injuries sustained and their causesThe writer develops each of these questions by considering possible answers that would either strengthen orweaken the argument The paper does not analyze the argument as insightfully or develop the critique asfully as required for a 6 paper but the clear organization strong control of language and substantial degreeof development warrant more than a score of 4

Milk and dairy products are rich in vitamin D and calcium substances essential for building and maintaining bones Many people therefore believe that a diet rich in dairy products can help prevent osteoporosis a disease in which the bones weaken significantly with age and that is linked to both environmental and genetic factors But a long-term study of a large number of people has found that those who have consistently consumed dairy products throughout the years of the study have a higher rate of bone fractures than any other participants in the study Since bone fractures are a symptom of osteoporosis this study result shows that a diet rich in dairy products may actually increase rather than decrease the risk of osteoporosis

Write a response in which you discuss what specific evidence is needed to evaluate the argument and explain how the evidence would weaken or strengthen the argument

Proven fact vitamin D and calcium in milk help prevent osteoporosis

GRE AWA John박 박정어학원

Challenge long-term study of a lot of people consistently consumed dairy products throughout the years=gt higher rate of bone fractures than other participantsThus bone fractures(symptom of osteoporosis) increased by diet rich in dairy products

-The author uses unscientific and imprecise terms in his study to support his claim-Control group-The author tries to refute a well established fact through a single study-Other factors bring about osteoporosis For example occupation hazards genetics dietary habits-Bone fracture could have might have may have been caused by other factors work environment etc and not specifically osteoporosis-The lack of a controlled environment of the study group The researchers only noted the consumption of milk and did not control other variantsmdashlifestyle and diet -The author bases his conclusion only by noting that the study samples consumed milk The researchers only noted the consumption of milk and did not control other variantsmdashlifestyle and diet-Dairy products might have had adverse effects on some of the participants in the study for example lactose intolerance

In this study the author claims that the consumption of milk actually increases the risk of osteoporosisTo support this claim the author refers to a long-term study he conducted ThesisHowever careful scrutiny of this study reveals that it provides little credible support for the authorrsquos claim

First the author states that the bone fractures were caused by osteoporosis However there could be other reasons for the bone fractures For example it is entirely possible that working conditions might have contributed to the bone fracture A person working at a construction has a much higher chance of bone fracture than a person that works in an office The author needs to provide the reasons for the occurrence of the study participantsrsquo bone fractures Thus the author should provide more sufficient evidence that

The author observes a correlation between osteoporosis and bone fracture then concludes that the former is the cause of the latter However the author fails to rule out other possible explanations For example it is entirely possible that working

The authorrsquos study includes an indefinite amount of people whose backgrounds arenrsquot defined genetic(family) history and environment the age at which the participants partook in the study It may be that the participants had genetic predisposition that were more prone to osteoporosis

The author does not sufficiently define what types of people were included in the study Bones are affected as noted in the prompt by various factors genes and environment The author however does not hellipFinally the author asserts his claim definitively While it is a proven fact that dairy products help people maintain healthy bones the author overthrows this long standing fact with a single long-term study that he did 우유와 유지방 제품에는 뼈를 형성시키고 강화시키는데 아주 중요한 역할을 하는 비타임 D와 칼슘이 많다 때문에 많은 사람들이 유지방이 풍부한 식이요법을 하면 환경 및 유전적 요인에 의한 손상과 노화에 의해 뼈가 심각하게 손상을 입는 질병인 오스티오포로시스를 억제하는데 도움이 된다고 믿고 있다 그러나 상당수의 사람을 대상으로 한 장기간 실험을 통해서 볼 때 이들 대상자중 실험 기간동안 유제품을 꾸준히 섭취한 사람들은 그렇지 않은 사람에 비해 훨씬 높은 골절현상을 보였다 이러한 실험결과를 볼 때 골절증상은 오스티오포로시스가 원인이기 때문에 이러한 식이요법이 오스티오포로시스 질병을 줄이기보다는 오히려 증가시킬 것이라는 것을 알수 있다결론 우유와 유지방 제품을 섭취하는 것은 오스티스오포로시스 질병을 줄이기봐는 오히려 증가시킬 것이다

1 실험은 변수에 의해 상당히 다른 결과를 얻을 수 있다 변수(생활 습관 운동)가 효과적으로 통제가 되었나 알 수 없다 상당수의 사람 장기간의 실험이 너무 주관적이다 객관적인 데이터가 주어져야한다

2 상당수와 장기간이 얼마만큼인지 알 수 없다 샘플링이랑 기간등 여러가지로 다르게 적용되었을 수 있다(실험에 충분한 기간과 인원이였는지)

3 골절 증상이 골다공증이 원인이 되어 나타났다고 했는데 그렇지 않을 수 있다다른 원인이 골절증상을 가져온 것일 수 있다

4 한가지 실험가지고 일반화했다 이미 밝혀진 사실을 단 한가지 실험으로 뒤엎었다 두 비교군이 동일 했는지에 대한 자료 없음(한쪽이 원래 뼈가 약했을수도있음)

GRE AWA John박 박정어학원

A recently issued twenty-year study on headaches suffered by the residents of Mentia investigated the possible therapeutic effect of consuming salicylates Salicylates are members of the same chemical family as aspirin a medicine used to treat headaches Although many foods are naturally rich in salicylates food-processing companies also add salicylates to foods as preservatives The twenty-year study found a correlation between the rise in the commercial use of salicylates and a steady decline in the average number of headaches reported by study participants At the time when the study concluded food-processing companies had just discovered that salicylates can also be used as flavor additives for foods and as a result many companies plan to do so Based on these study results some health experts predict that residents of Mentia will suffer even fewer headaches in the future

Write a response in which you discuss what questions would need to be answered in order to decide whether the prediction and the argument on which it is based are reasonable Be sure to explain how the answers to these questions would help to evaluate the prediction

In this study the author asserts that the number of headaches suffered by the average citizen of Mentia will steadily decline though the use of salicylates as flavor additives To support this assertion the author provides several evidences to support his claim However his argument is specious

Unscientific data correlation the specific amount used as preservatives Other factors could have caused the decline in headaches20 year study Was it a controlled study Lack of controlled environment exposes the participants to other factors

1) The decline in the average number of headaches does not necessarily substantiate that it was indeed caused by the commercial use of salicylates It is entirely possible that other factors are responsible for the decline Perhaps other substances in the food remedied the headaches of the participants Or the decline in headache may have been caused by other dietary consumption or lifestyle habits like exercising Accordingly if these are true the author cannot draw any firm conclusion that the commercial use of salicylates caused the decline in the average number of headaches

살리실산염은 아스피린과 같은 동일 화학물질 계열로써 두통을 치료하는데 사용되는 약물이다 많은 음식에 이 살리실산염이 다량 함유되어 있지만 지난 수십년간 식품가공 업체들은 가공식품 부패방지용으로 살리실산염을 첨가해왔다 이러한 살리실산염의 산업용 사용의 증가는 본 연구원들에 의해 보고된 평균 두통 횟수를 꾸준히 감소시키는데 일조한것으로 이해되어 왔다 최근 식품가공 회사들은 이 살리실산염이 맛을 내는 첨가물로 사용될 수 있음을 발견했다 이러한 새로운 용도로 Mentia에 사는 주민들이 겪고 있는 두통의 평균 횟수를 앞으로도 계속 줄여나갈 수 있을 것으로 보인다결론 살리산염의 사용증가는 주민들의 두통 횟수를 계속 줄여줄 것이다

가정 1 조사받은 사람들은 단지 Salicylate에 의해 headache가 감소된 것이다 -gt 오류 다른 노력을 기울였을 수 있다 가정 2 나중에 Salicylate의 사용증가가 headache의 감소를 가져올 것이다 -gt 오류 또다른 원인으로 얼마든지 headache는 증가가능하다가정 3 음식물에서 headache 감소 원인은 Salicylate 이다 -gt 오류 또다른 성분이 있었을 수 있다가정 4 Flavor additive 로 사용된 Salicylate도 headache를 줄일것이다 -gt 오류 Flavor additive로 사용시 같은 효과를 가질지 밝혀진바 없다 상업적으로 사용되었다고 다 먹었을까

The following appeared as part of an article in a business magazine

GRE AWA John박 박정어학원

A recent study rating 300 male and female Mentian advertising executives according to the average number of hours they sleep per night showed an association between the amount of sleep the executives need and the success of their firms Of the advertising firms studied those whose executives reported needing no more than 6 hours of sleep per night had higher profit margins and faster growth These results suggest that if a business wants to prosper it should hire only people who need less than 6 hours of sleep per night

Write a response in which you examine the stated andor unstated assumptions of the argument Be sure to explain how the argument depends on these assumptions and what the implications are for the argument if the assumptions prove unwarranted

Assumptions 1 Correlation of hours of sleep with success The author assumes that the executives used their wake-up hours on work Could there be other factors 2 Executives versus employeesmdashcriteria for hiring is too simpleminded 3 Characteristics of the 300 male and female executives 4 The study was about advertising executives but the author applies this case to all businesses 5 The author should consider long-term stability rather that sharp growth and profit 6 Average number of hours of sleep is insufficient to support the authorrsquos recommendation 7 The author assumes that this lifestyle pattern will remain consistent

The following appeared in a memo from the president of a company that makes breakfast cereals

In a recent study subjects who ate soybeans at least five times per week had significantly lower cholesterol levels than subjects who ate no soy products By fortifying our Wheat-O cereal with soy protein we can increase sales by appealing to additional consumers who are concerned about their health This new version of Wheat-O should increase company profits and at the same time improve the health of our customers

Write a response in which you examine the stated andor unstated assumptions of the argument Be sure to explain how the argument depends on these assumptions and what the implications are for the argument if the assumptions prove unwarranted

최근의 조사에서 일주에 최소 5회 정도 콩을 먹었던 사람들은 전혀 먹지 않았던 사람들에 비해 콜레스테롤이 상당히 낮은것으로 조사되었다 따라서 Wheat-O 시리얼에 콩 단백질의 함유를 강화시킴으로써 건강에 관심이 있는 더 많은 소비자를 상대로 매출을 올릴 수 있을 것이다 이 신제품이 회사 수입 증대를 가져오는 것 뿐만아니라 소비자의 건강을 향상시킬 수 있다

GRE AWA John박 박정어학원

주장 By increasing our Wheat-O cereal with soy protein we can multiply sales

1 콩 단백질의 함유를 강화해도 그수치가 미흡하거나 흡수가 잘 안되거나 할 수 있다 다른 성분과 섞여서 만들어지는 것이므로 다른 성분에 의해서 그 흡수가 잘 안되거나 효과가 낮을 수 있다 2 강화한 것이 건강에 관심이 있는 소비자에게 어필한다고 해도 그 수가 적어서 매출에 큰 영향을 안줄 수 있다3 다른 음식에 의해서 콜레스테롤이 더 낮아질수 있다 4 콜레스테롤을 낮추는 것이 사람들에게 그들의 건강생활을 위해 크게 어필 안할수도 있다( 저지방식이라든지 다른 건강을 높이는 방법이 더 관심이 많을수 있다)

In a study of the reading habits of Waymarsh citizens conducted by the University of Waymarsh most respondents said that they preferred literary classics as reading material However a second study conducted by the same researchers found that the type of book most frequently checked out of each of the public libraries in Waymarsh was the mystery novel Therefore it can be concluded that the respondents in the first study had misrepresented their reading habits

Write a response in which you discuss what specific evidence is needed to evaluate the argument and explain how the evidence would weaken or strengthen the argument

Evidence needed -Information about the respondentsmdashage gender class precise number of respondents-Time-shift After how many years was the second study conductedmdashtaste in books may change over time-Can library records sufficiently evidence the reading habits of Waymarsh citizens

1)2)3)4)5)6) First the author provides no evidence that the surveyrsquos studyrsquos results are statistically reliable Lacking

information about the precise methodology of the study the number of (customers)respondents surveyed and the number of respondentsvarious information such as gender age and social background which are essential to bolster the conclusion it is impossible to assess the validity of the results It is possible that people who feel inclined to( take low fat and low cholesterol foods ) read literary classics were more willing to respond to the survey than were others Another problem is the representativeness of the respondents Were they representative of all the customersWaymarsh citizens Were they chosen for the survey randomly Lastly the survey results must depend on the honesty and integrity of the respondents Without more information about the survey the author cannot simply conclude that (most of customers want to take low fat and low cholesterol foods)Waymarsh citizens misrepresented their reading habit on the first study on the basis of this surveystudy(survey reliability)

The exact number of books checked-out should be notedMystery novels could have been in trend at the time of the studyPublic libraries vs other libraries such as Leeville University library

Leeville 대학에서 실시한 Leeville 주민의 독서습관에 대한 조사에서 대부분의 응답자는 독서용으로 문학작품을 선호한다고 응답했다 그러나 같은 조사팀에서 이어 실시한 조사에서는 Leeville에 있는 도서관마다 가장 빈번하게 연람된 책의 종류를 보면 미스테리 소설이었던 것으로 조사되었다 따라서 최초 조사 응답자들이 독서 습관에 대해 잘못 말했다고 결론지을 수 있다

결론 it can be concluded that the respondents in the first study had misrepresented their reading habits1 첫번째 조사 그룹하고 두번째 조사 그룹하고 다른 그룹일 수 있다

GRE AWA John박 박정어학원

2 도서관에 비치된 책이 미스터리 소설이 더 많아서 선택의 여지가 없을 수 있다

High Frequency Group 2 National-gtLocal Local-gtNational amp Group Member Error

The following appeared in a letter to the editor of a Batavia newspaper

The department of agriculture in Batavia reports that the number of dairy farms throughout the country is now 25 percent greater than it was 10 years ago During this same time period however the price of milk at the local Excello Food Market has increased from $150 to over $300 per gallon To prevent farmers from continuing to receive excessive profits on an apparently increased supply of milk the Batavia government should begin to regulate retail milk prices Such regulation is necessary to ensure fair prices for consumers

Write a response in which you discuss what questions would need to be answered in order to decide whether the recommendation is likely to have the predicted result Be sure to explain how the answers to these questions would help to evaluate the recommendation

First the author points to the fact that hellip25 increase of dairy farms may be(could be might be) due to population increaseThe milk price increase could be reflecting the increase in cost of livingThe actual price of milk might be cheaper if the economy is experiencing inflationOne market Excello Food cannot reflect the price increase of every market in BataviaThe author claims that farmers are receiving excessive profits however this might not be the caseThe author claims that by regulating milk prices the government will successfully ensure both lower prices and an adequate supply of milk for consumers However this might not be the best solutionThe writer adduces the information released by the department of agriculture that the number of dairy farms has

increased by 25 percent over the last 10 years and thus he claims that dairy farmers are earning excessive profits from milk However the information is filled with loops and holes which need to be filled First what percentage of the milk are sold as milk

Are the dairy farms processing the milk into other dairy products Or are they selling them as milkmdashWhat percentage of the milk is sold as milk Could this be a natural increase

Can the price at the local Excello Food Market represent the price of milk in stores throughout all of Batavia

Are the farmers receiving excessive profits What is the cost of producing milk Could such regulation offer fair prices to consumers

Topic SentenceTo begin with the author assumes that the price of Excello Food Market which has increased from $150 to over $300 per gallon can be applied to the whole nation of Batavia however the author fails to provide evidence to substantiate this assumptionExampleFirstIn additionTherefore in order to make the argument more reliable the author should consider the prices of a large number markets all over the countryThere could be an economic inflation

GRE AWA John박 박정어학원

ldquoBatavia 지역의 농림부의 보고에 따르면 전국적으로 낙농업자의 숫자가 과거 10 년전에 비해 25나 증가했다고 합니다 그러나 동기간 동안 이 지역에 있는 Excello Food Market 에서 우유의 가격은 갤런당 15 불에서 3 불로 증가했습니다 증가된 우유 공급량에 맞춘 낙농업자들의 수익률 증대를 보장하기 위해서는 주 정부가 소매 공급가에 대한 규제를 해야 합니다 소비자에게 보다 저렴한 가격에 안정된 물량을 공급할 수 있도록 하기 위해서는 이러한 조치가 필요합니다

결론 the Batavia government should begin to regulate retail milk prices

반박 ( Excello Food Market doesnrsquot reflect the whole of Batavia) 우유값 증가가 단순히 inflation 을 따라간것일수도 있다 가격상승이 수익을 보장하지 않는다( 원자재 상승등으로 인해서)우유값 규제만이 최선의 방법이 아니다( 물량 조절)낙농업자의 수익보호도 필요하다

A recent sales study indicates that consumption of seafood dishes in Bay City restaurants has increased by 30 percent during the past five years Yet there are no currently operating city restaurants whose specialty is seafood Moreover the majority of families in Bay City are two-income families and a nationwide study has shown that such families eat significantly fewer home-cooked meals than they did a decade ago but at the same time express more concern about healthful eating Therefore the new Captain Seafood restaurant that specializes in seafood should be quite popular and profitable

Write a response in which you discuss what questions would need to be addressed in order to decide whether the conclusion and the argument on which it is based are reasonable Be sure to explain how the answers to the questions would help to evaluate the conclusion

BAY CITY The name implies that this is a port city which would have traditionally consumed seafood 30--what type of seafood Unprocessed or processed cans of tuna and sardines If the latter increased Captain Seafood may not profit 30--natural inflation Are the non-seafood restaurants selling seafood If yes this may suffice since patrons would not easily dine at an unfamiliar restaurant Nationwide studyrsquos representativeness Desire does not lead to action

Nationwide study applied to Bay City Eating fewer home cooked meals than they did a decade agoHealthy food The author needs to show that 30 -gt population increaseOther foods may be more popularCurrent number of restaurants that serve seafood might sufficeWhat kind of healthy food are they interested in Surely not just seafoodThe locals may still patronize the current restaurants that serve seafood dishes

lt national-gt local에 적용 local-gtnational에 적용gt1) The author assumes that the nationwide trend reflects the general trend upon which the argument relies

Yet the author fails to provide evidence to substantiate this crucial assumption The nationwide trend of two income families eating fewer home-cooked and concern for health my not reflect the lifestyle and interests of Bay City citizens Therefore to make his argument stronger the author needs to provide evidence that two income families of Bay City maintains the nationwide trend

GRE AWA John박 박정어학원

최근의 매상에 관한 조사에서 Bay 시에 있는 레스토랑의 해산물 요리의 소비가 지난 5년동안 30 증가했다고 보고되었다 그러나 현재 해산물을 전문으로 취급하고 있는 레스토랑이 없다 더군다나 대다수의 가정이 맞벌이 가정인데다가 전국 조사에서도 나와있듯이 이러한 맞벌이 가정은 10년전의 가정에서 했듯이 집에서 식사를 챙겨먹는 것이 급격하게 줄어 들었고 동시에 건강식과 관련한 지출이 늘고 있다 따라서 해산물 전문 레스토랑이 아주 인기를 끌것이고 그만큼 수익도 많을 것이다

1 해산물이 건강식인지에 대한 언급이 없다2 외식하는데 해산물 요리만 먹지 않을 것이다 집에서 챙겨먹지 않는다고 해산물을 먹는다고 할 수 없다3 현재 해산물 취급하는 식당이 현재까지 없다고 해서 계속 없을 수만은 없다 수익이 만약 늘어난다면

Bay시 주변의 도시의 레스토랑이 체인점을 내거나 새로운 식당이 생길 수 있고 오히려 수익이 줄어들 수도 있다

Scientists studying historical weather patterns have discovered that in the mid-sixth century Earth suddenly became significantly cooler Although few historical records survive from that time some accounts found both in Asia and Europe mention a dimming of the sun and extremely cold temperatures Either a huge volcanic eruption or a large meteorite colliding with Earth could have created a large dust cloud throughout Earths atmosphere that would have been capable of blocking enough sunlight to lower global temperatures significantly A large meteorite collision however would probably create a sudden bright flash of light and no extant historical records of the time mention such a flash Some surviving Asian historical records of the time however mention a loud boom that would be consistent with a volcanic eruption Therefore the cooling was probably caused by a volcanic eruption

Write a response in which you discuss what questions would need to be addressed in order to decide whether the conclusion and the argument on which it is based are reasonable Be sure to explain how the answers to the questions would help to evaluate the conclusion

Historical records may not be enough to explain the global cooling phenomenonThere may be reasons other than the two hypothesis to explain the global coolingA volcanic eruption big enough to produce dust clouds that envelop the earth to cause global cooling would have countless recordsThe author argues that a large meteorite collision is not a feasible explanation for the global cooling because there are no historical records that indicate a flash The authorrsquos logic is flawed in eliminating the meteorite hypothesis by claiming that there was no historical record of a bright flash of light According to common sense a meteorite collision big enough to produce dust that could envelop the earth would result in catastrophe even to the extinction of major species of animals

Loud boom would be insufficient evidence to conclude it was a volcanic eruption If could hear a loud boom there should be records of a volcanic eruption because the author claims that the dust cloud from this gigantic eruption was significant enough to cause global coolingCooling could have been caused by factors besides volcanic eruption and meteor collisionThe absence of historical records that indicate a bright flash of light Collision could have occurred at a place where there no witnesses Could have fell out in the seaBoom might have been caused by things other than a volcanic eruptionHistorical records from Asia and Europe may not be applicable to earth as a whole

과거의 날씨 주기를 연구하는 과학자들은 6세기 중엽 지구가 갑자기 혹한기가 되었던 것을 알게되었다 이 당시의 몇몇 기록들이 아직도 남아있긴 하지만 아시아와 유럽에서 발견되는 몇가지 자료에서 태양 빛의 수축과 그에따른 엄청난 추위가 있었음을 언급하고 있다 거대한 화산 폭발이나 거대 운석의 지구 충돌중 어느것에

GRE AWA John박 박정어학원

의해서든 엄청난 먼지구름을 형성시켜 지구전체에 태양빛을 차단해서 지구의 기온을 뚝 떨어뜨릴수 있을 수도 있다 그러나 이러한 운석 충돌은 순간 섬광을 발산시키게 될 것인데 이 시기의 어느 기록에서도 이러한 섬광은 언급되지 않고 있다 그러나 이 시기에 남아 있는 일부 아시아의 기록문서에서는 연속적으로 화산폭발음일수 있는 엄청난 폭발음이 있었음을 기록하고 있다 따라서 이러한 혹한은 아마도 화산 폭발에 의한 것이었을 것이다

결론 the cooling was probably caused by a volcanic eruption

반박 자료의 부족( 몇몇 자료가지고 그 당시의 기후를 예상하기에는 무리가 있다)다른 원인에 의해서 기후가 떨어졌을수도 있다 (빙하기 다른 기상원인)기록되지 않았다고 해서 그 사실이 없었던 것은 아니다(기록은 했으나 자료가 없어졌을수도 있고 모든 사실이 전부다 기록되지는 않는다 )기록된 폭발음이 꼭 확산 폭발음이 아닐수도 있다( 화산 폭발음이 너무 먼거리여서 들리지 않을수도 있고 다른 소리가 기록된 것이였을수도 있다)부족한 자료를 신빙할수 있는가 기록의 소멸도 예상 할수 있다 실제로 운석이 떨어졌는데 안들렸을수 있다

The following appeared in a memo from the president of Bower Builders a company that constructs new homes

A nationwide survey reveals that the two most-desired home features are a large family room and a large well-appointed kitchen A number of homes in our area built by our competitor Domus Construction have such features and have sold much faster and at significantly higher prices than the national average To boost sales and profits we should increase the size of the family rooms and kitchens in all the homes we build and should make state-of-the-art kitchens a standard feature Moreover our larger family rooms and kitchens can come at the expense of the dining room since many of our recent buyers say they do not need a separate dining room for family meals

Write a response in which you examine the stated andor unstated assumptions of the argument Be sure to explain how the argument depends on these assumptions and what the implications are for the argument if the assumptions prove unwarranted

The presidentrsquos first unstated but apparent assumption is that a nationwide survey can be applied to local areas There is no guarantee that the area in which Bower Builders builds houses will follow the national consumer trend Since he only mentions the overall result of the nationwide survey it is difficult to determine more specific conditions such whether people in urban and rural areas have the same housing preferences Such uncertainty is further exacerbated by the nature of the survey question itselfmdash it does not deal with current trends in actual home purchases but with desired home features The naiumlve assumption that consumer desire will directly result in consumer action underlies the presidentrsquos interpretation and application of the survey results to his company plan Not everyone who wishes for a large family room and kitchen will or can actually buy a house with those features mainly due to financial reasons Furthermore the president also assumes for no evident reason that consumers will not hesitate to purchase houses with state-of-the-art kitchens as a standard rather than optional feature when it is clear that such a feature will raise the overall cost significantly He continues to blunder in his mistaken assumptions about consumer behavior in assuming that the taste of recent buyers can represent the concern of future prospective buyers The fact that recent buyers have claimed no need for separate dining rooms does not mean future buyers will feel the same way as well

The president of Bower Builders recommends that in order to raise company profits the company should build houses with large family rooms and large state-of-the-art kitchens He uses the results of a nationwide survey on desired home features and the example of Bower Buildersrsquo competitor Domus Construction to support his plan His memo manifests several assumptions about surveys consumer behavior and Domus Constructionrsquos houses that do not necessarily bolsterbuttress his argument

GRE AWA John박 박정어학원

The writer assumes 1 the nationwide survey results can be applied to the local area (Desire cannot reflect consumer trend) 2 Domus Construction was profitable because they had such features (The writer should investigate what other features the DC provides and the actual number of homes sold) 3 State-of-the-art kitchens will sell well (no evidence to buttress his assumption furthermore they would need to spend more money which would increase the price of the housesmdashtheir market will be limited to the affluent) 4 The recent buyers represent the concern of most buyersmdashthat they are ok with a house that is without a dining room

Nationwide survey might not be applicable to this regionDomus Construction could have other superior qualities than Bower Builders Ex DesignSelling fast doesnrsquot indicate quantitySmall yards complaints could be voiced in the future

First the author concludes that a nationwide survey reveals that the two most desired home features are a bathroom with a whirlpool tub and a large kitchen However this nationwide survey may not reflect the trends of the customers that Bower Builders target The author assumes that the nationwide trend reflects customer trends The national study would lead support only if the nationwide trend reflect the home-purchasing trends The nationwide trend may just be a trend of desire which does not result in the act of purchasing However the author does not provide credible evidence that this is the case

전국 조사에서 가장 인기있는 집안 구조 2가지는 훨풀 튜브가 마련된 욕실과 커다른 부엌으로 조사되었다 경쟁업체 Domus Construction이 신축한 완공예정인 주택은 이러한 욕실을 갖추고 있어서 분양이 조기에 이루어지고 있고 가격도 평균치보다 상당히 웃돌고 있다 우리도 집을 많이 팔도 그만큼 이윤을 많이 남기려면 신규 주택에는 위의 2가지 사항을 갖추어야 한다 최근 우리가 지은 주택 수요자들이 적은 평수에 대해서는 별다른 불만사항이 없었기 때문에 앞으로 평수를 더 줄여서 이익을 극대화 할 수 있을 것이다

주장 We should include whirlpool tubs and a large kitchen

1 경쟁업체 Domus Construction이 신축한 완공예정인 주택은 이러한 욕실을 갖추고 있어서라기 보다는 위치가 좋거나 다른 마감재(construction material)나 인테리어가 뛰어나서 분양이 조기에 이루어진것이라고 볼 수 있다 2 전국조사가 건물이 지어지는 지역에 항상 적용되리라고 볼 수 없다 3 평수를 줄이는 것에 불만이 없었다는 것은 editor의 견해일 수 있다 사람들이 표현하지 않은 것일 수도 있고 다른 장점이 많아서 그런 단점을 드러내지 않은 것일 수 있기 때문에 속단할 수 없다 4 다른것을 함으로써 더 잘 팔릴수도 있다

2011 7 19 화요일 GRE AWA 실전반이OO

Argument 94

The president of Bower Builders contends recommends that in order to raise company profits the company should build houses with large family rooms and large state-of-the-art kitchens He uses the results of a nationwide survey on desired home features and the example of Bower Buildersrsquo competitor Domus Construction to support his plan His memo manifests several assumptions about surveys consumer behavior and Domus Constructionrsquos houses that do not

GRE AWA John박 박정어학원

necessarily bolsterbuttress his argumentThe presidentrsquos first unstated but apparent assumption is that a nationwide survey can be

applied to local areas There is no guarantee that the area in which Bower Builders builds houses will follow the national consumer trend Since he only mentions the overall result of the nationwide survey it is difficult to determine more specific conditions such whether people in urban and rural areas have the same housing preferences Such uncertainty is further exacerbated by the nature of the survey question itselfmdash it does not deal with current trends in actual home purchases but with desired home features The naiumlve assumption that consumer desire will directly result in consumer action underlies the presidentrsquos interpretation and application of the survey results to his company plan Not everyone who wishes for a large family room and kitchen will or can actually buy a house with those features mainly due to financial reasons Furthermore the president also assumes for no evident reason that consumers will not hesitate to purchase houses with state-of-the-art kitchens as a standard rather than optional feature when it is clear that such a feature will raise the overall cost significantly He continues to blunder in his mistaken assumptions about consumer behavior in assuming that the taste of recent buyers can represent the concern of future prospective buyers The fact that recent buyers have claimed no need for separate dining rooms does not mean future buyers will feel the same way as well

In addition the president finds a real-life actualization of the nationwide survey results in the recent sales of Domus Construction However he easily assumes that large family rooms and kitchens are the only reasons the houses of Domus Construction sell well He does not take into account other features and selling points of the competitorrsquos houses There may well be other explanations for its houses selling more quickly and expensively including additional home features as well as external factors such as proximity to better schools or superior financial solvency of its clientele If Bower Builders merely added larger family rooms and kitchens without taking into consideration the other factors they may lose rather than gain profits

Clearly the presidentrsquos assertion that Bower Builders make houses with large family rooms and high-tech kitchens at the expense of dining rooms rests on a number of assumptions that are ill-informed and naiumlve If Bower Builders undertakes the proposed plan without further research into local consumer desires purchasing trends and the marketing and sales of competing companies the company will risk losing money by building big new houses that people cannot afford to or will not wish to buy

The following appeared in a letter to the editor of a journal on environmental issues

Over the past year the Crust Copper Company (CCC) has purchased over 10000 square miles of land in the tropical nation of West Fredonia Mining copper on this land will inevitably result in pollution and since West Fredonia is the home of several endangered animal species in environmental disaster But such disasters can be prevented if consumers simply refuse to purchase products that are made with CCCs copper unless the company abandons its mining plans

Write a response in which you examine the stated andor unstated assumptions of the argument Be sure to explain how the argument depends on these assumptions and what the implications are for the argument if the assumptions prove unwarranted

The writer assumes 1 The writer is trying to avoid an inevitability 2 Mining copper will result pollution (they could make preventative measures) 3 The writer may be too late from stopping CCC from developing the area into a copper mine 4 Since mining is an underground enterprise the surface may not be affected that much therefore endangered species may not be affected 4 Consumers wonrsquot buy CCC products if the journal publishes a negative review about CCC (How many readers) CCC could a company that has ties with many IT companies and industries in that their copper is almost ubiquitous in various products

GRE AWA John박 박정어학원

지난 한해동안 CCC(Consolidated Copper 회사)는 서부 플로리다의 열대 지역에 1백만 마일이 넘는 땅을 사들였다 이 곳에서의 채광 활동은 서부 플로리다가 몇몇 멸종 위기에 처한 동물의 서식지이기 때문에 분명 오염과 환경파괴를 가져올 것이다 그러나 이러한 파괴는 CCC 회사가 채광을 포기할 때까지 이 회사가 채굴한 구리로 제조된 제품을 구매하지 않으면 막을 수 있을 것이다

결론 such disaster can be prevented if consumers simply refuse to purchase products that are made with CCCs copper until the company abandons its mining plans

1 구리로 제조된 물건이 생활에 필수적인 것이 많은 만큼 불매가 쉽지 않을 수 있다(전선이나 각종 전자제품에 필수적으로 들어가기 때문에)

2 불매를 유도한다고 해서 소비자들이 구매를 안하는 것은 아니다3 적절한 채굴로 환경파괴를 가져 오지 않을 수 있다 (땅속에 있는 물질을 채굴하는 만큼 생물에 영향을 안

미칠 수도 있다)4 이미 채굴이 다 끝나서 더 이상의 채굴이 없을 수도 있다 5 땅을 구입한다고 해서 채광하는건 아니다 (다른 용도로 샀을수도 있다)

The following is a letter to the editor of an environmental magazine

In 1975 a wildlife census found that there were seven species of amphibians in Xanadu National Park with abundant numbers of each species However in 2002 only four species of amphibians were observed in the park and the numbers of each species were drastically reduced There has been a substantial decline in the numbers of amphibians worldwide and global pollution of water and air is clearly implicated The decline of amphibians in Xanadu National Park however almost certainly has a different cause in 1975 troutmdashwhich are known to eat amphibian eggsmdashwere introduced into the park

Write a response in which you discuss what specific evidence is needed to evaluate the argument and explain how the evidence would weaken or strengthen the argument

Evidence needed the identity of the sender and the census taker of rsquo75 and lsquo02mdasha scientist or an environmentalist credibility issue census methodology vs mere observationmdashthe absence of evidence is not an evidence of absence the season of when the census was taken time-shiftmdashconditions may have changed worldwide decline may include Xanadu other species of predators that prey on amphibians because trout is only one species that prey on amphibians the number of troutmdashhave they increased significantly since rsquo75 First the author needs to be more overt about the credibility of the census and observation There were to accounts that notes the population of amphibiansmdashthe first a census and the second an observation The author needs to bolster his conclusion with the evidence that indicate that the census and second observation were done using scientific methodologies This could either could strengthen or weaken his claim In addition he

GRE AWA John박 박정어학원

needs to provide the specific season of when the census and observation occurred In this letter the writer is informing an editor of an environmental magazine that the number of amphibians was greatly reduced since 1975 and he points out the introduction of trout as the only reason for the decline However the author fails to provide crucial evidences that could strengthen or weaken his conclusion

The following appeared in a memorandum from the president of Hyper-Go Toy Company

Last year sales of our Fierce Fighter toy airplane declined sharply even though the toy had been a top seller for three years Our customer surveys show that parents are now more worried about youthful violence and are concerned about better education for their children Therefore to maintain profits we should discontinue all our action toys and focus exclusively on a new line of educational toys Several other toy companies have already begun marketing educational toys and report sales increases last year of 200 percent And since the average family income is growing sales of new Hyper-Go toys should also increase

Write a response in which you discuss what specific evidence is needed to evaluate the argument and explain how the evidence would weaken or strengthen the argument

First the president indicates that the sales of Fierce Fighter toy airplane declined sharply However he fails to consider the fact that toys are a fad Since FFT enjoyed a three year success it may be natural that the trend would subsideSecond Customer survey is this representative of most toy consumers This trend is not newThird other companies may have profited not through educational but other toys Or educational toy profit may be small in proportion to their sale of other toys 200Parents are not the customers companies should concentrate on what the children want to increase profitLastly parents worry about youthful violence and concern for better education are not new trends

The presidentrsquos decision to discontinue all action toys and focus only on educational toys is too extreme If this decision is taken into effect its procedure may be an onerous task because it would require major shifts in human resources and company image Therefore further data should be considered before following up on this decision

우리 회사에서 지난 3년간 최고 매출을 일으켰던 Fierce Fighter 장난감 비행기가 지난해에는 매출이 급격히 떨어졌다 자체 고객 조사에서는 부모들이 현재 청소년 폭력에 걱정을 하고 있어서 아이들의 양질 교육에 더 관심을 가지고 있는 것으로 나타났다 따라서 회사의 수익을 유지하기 위해서는 모든 자사 전투용 장난감 생산을 중단하고 오로지 교육적인 장난감 생산에 집중해야 한다 몇몇 여타 장난감 회사들도 이미 교육용 장난감 마케팅을 시작해서 지난해에는 200의 매출신장을 가져왔다고 한다 그리고 평균 가계 수입이 점점 늘고 있기 때문에 신형 Hyper-Go 장난감의 매출도 늘어날 것이다

3그룹 Time-shift Error

Woven baskets characterized by a particular distinctive pattern have previously been found only in the immediate vicinity of the prehistoric village of Palea and therefore were believed to have been unique to the Palean people Recently however archaeologists discovered such a Palean basket in Lithos an ancient village across the Brim River from Palea The Brim River is very deep and broad and so the ancient Paleans could only have crossed it by boat but there is no evidence that the Paleans had boats And boats capable of carrying groups of people and cargo were not developed until thousands of years after the Palean people disappeared Moreover Paleans would have had no need to cross the rivermdashthe woods around Palea are full of nuts berries and small game It follows that the so-called Palean baskets were not unique to Palea

GRE AWA John박 박정어학원

Write a response in which you discuss what specific evidence is needed to evaluate the argument and explain how the evidence would weaken or strengthen the argument

RefutationPrehistoric time shift-gtbrim river could have been narrow and shallow or it might have not existed Indigenous patterns may exist in other disconnected remote placesNuts berries small game(hunting) may not have existed or the author should be proved these existed at that time Abundance of resources doesnrsquot support the reason for seclusionBoats not yet found baskets may have been carried across by the river current without the help of a boat Lithos might have crossed the river for commercial purposeOne Palean basket does not substantiate the authorrsquos claimThe absence of evidence is not an evidence of absence

The author assumes without justification that present conditions are the same as at the prehistoric era The author unfairly infers from the presence of Brim River which exist today that it would have existed in the past However the author fails to offer any evidence to substantiate this inference It is very likely that the Brim River might not have existed in prehistoric times or if it did exist may have been shallow and narrow enough for the Paleans to easily cross Any of these scenarios if true would serve to undermine the claim thathelliphelliphelliphelliphellip

First the author claims that the Brim River was very deep and broad so the Paleans could not have crossed it However the author fails to offer any evidence to substantiate that this was true in the prehistoric time of the Paleans For all we know the Brim River might not have existed in the prehistoric era or if it did exist could have been a shallow and narrow river For example scientists believe that thousands of years ago an ice-bridge existed on the Bering Sea connecting Eurasia to what is now North America to explain how the Eskimos and the Asian inhabitants of America came to migrate all over the Americas In this example the absence of evidence is not an evidence of absence Therefore to sufficiently support his claim the author needs to substantiate the fact that the Brim River really did exist and was broad and wide in the Prehistoric Era proving that the Paleans could not have influenced or traded with other groups of people

First the author claims that the Brim River was very deep and broad so the Paleans could not have crossed it However the author fails to offer any evidence to substantiate that this was true in the prehistoric times of the Paleans For all we know the Brim River might not have existed in the prehistoric era or if it did exist could have been a shallow and narrow river For example rivers are created by natural erosion over thousands of years Niagara Falls carved its way from the mouth of Lake Ottawa and created a long river Likewise the current Brim Riverrsquos physical features may not have been wide and deep Therefore to sufficiently support his claim the author needs to substantiate the fact that the Brim River really did exist and was broad and wide in the Prehistoric Era proving that the Paleans could not have influenced or traded with other groups of people

예전에는 실로짠 특이한 무늬 바구니가 Palea의 선사시대 지역의 인근마을에서만 발견되어왔기 때문에 Palea 마을 사람들의 특징이라고 여겨졌었다 그러나 최근들어 고고학자들이 Lithos지역에서 Palean 바구니를 발견하였는데 그 당시 지역은 Brim 강을 가로질러 Palea까지 닿아있었다 이 강은 수심이 아주 깊고 강폭이 넓었으며 때문에 고대의 Palea인들은 배를 이용해서 강을 건널수 있었을 것이다 그러나 이들이 배를 가지고 있었다는 증거는 발견되지 않고 있다 더군다나 이들이 멸명한 이후 수천년이 지난뒤에도 수많은 물자와 사람을 실어 나를수 있는 용적을 가진 배는 개발되지 않았다 이와더불어 Palea인들은 강을 건널필요가 없었는데 그것은 너트나무 장과열매 그리고 작은 사냥감들이 주변숲에 풍부했기때문이다 따라서 Palean 바구니라고 하는 것도 Palea인들만의 전유물이 아니라는 결론을 얻을 수 있다

결론 if follows that the so-called Palean baskets were not unique to Palea이번문제는 굿이 causal Bad analogy 로 구분해서 찾기가 힘드내요 배를 발견했다는 증거가 없는것이다 (앞으로도 발견될수 있음)

GRE AWA John박 박정어학원

계절의 영향으로 겨울에 얼음이 두껍게 언다든지 여름에 가뭄으로 인해서 건널수 있다물자가 풍부한 것이 이동하지 않을 조건이 아니다 다른 것에 의해서 이동가능(의약품등)

Thirteen years ago researchers studied a group of 25 infants who showed signs of mild distress when exposed to unfamiliar stimuli such as an unusual odor or a tape recording of an unknown voice They discovered that these infants were more likely than other infants to have been conceived in early autumn a time when their mothers production of melatonin hormone known to affect some brain functions would naturally increase in response to decreased daylight In a follow-up study conducted earlier this year more than half of these children now teenagers who had shown signs of distress identified themselves as shy Clearly increased levels of melatonin before birth cause shyness during infancy and this shyness continues into later life

Write a response in which you examine the stated andor unstated assumptions of the argument Be sure to explain how the argument depends on these assumptions and what the implications are for the argument if the assumptions prove unwarranted

Any baby exposed to unpleasant stimuli would react in such wayFirst of all the author states 25 infants as his evidence However this research sample is too small to prove his claimSecond the author states that 25 infants were conceived in early autumn which he claims lead to a shy disposition However this is faulty evidenceThird the research study was a long term study done in the span of 13 years However the author only writes about the initial and final stages of the study and leaves out evidences of what could have happened during the 13 years which could be more evidential factors of influenceFourth neither the infantsrsquo genetic predisposition nor their environment were taken into accountFinally the author concludes that his shyness continues into later life (Other factors could alter this disposition epigenetic theory)

13 년전 학자들은 25명의 유아를 대상으로 이상한 냄새나 특이한 소리를 녹음한 테잎등으로 낯선 자극을 주었을때 보이는 미미한 압박감 증상을 조사하였다 이들은 성숙기가 막 지났을 즈음에 보통의 유아들이 비슷한 증상을 보이는 정도 보다는 다소 민감한 반응을 보였는데 이 시기는 아이의 엄마가 뇌의 일부 기능에 영향을 미치는 것으로 알려진 멜라토니아 호르몬을 생산하는 시기로써 이 호르몬은 낯 시간이 짧을때 자연적으로 증가할 수도 있다 금년초에 실시된 추가연구에서 현재 10대로 성장한 당시 조사대상의 절반 이상의 아이들이 부끄럼을 잘타는 것으로 여기고 있었다 따라서 분명한 것은 출산전 멜라토닌 수치의 증가가 유아기에 수줍음 등의 영향을 미치게 되며 이러한 영향이 성장후에도 작용한다는 것이다

주장 Clearly increased levels of melatonin before birth cause shyness during infancy and this shyness continues into later life

1 25명의 아기로 결론 내리기에 샘플이 작다2 과학적 사실들에 대한 명확한 근거가 엇음3 다른 영향을 간과했다(다른 호르몬에 의한 영향 후천적인 성격형성의 영향)

GRE AWA John박 박정어학원

The following is a letter to the editor of the Atticus City newspaper

Former Mayor Durant owes an apology to the city of Atticus Both the damage to the River Bridge which connects Atticus to Hartley and the traffic problems we have long experienced on the bridge were actually caused 20 years ago by Durant After all he is the one who approved the construction of the bridge If he had approved a wider and better-designed bridge on which approximately the same amount of public money would have been spent none of the damage or problems would have occurred Instead the River Bridge has deteriorated far more rapidly over the past 20 years than has the much longer Derby Bridge up the river Even though the winters have been severe in the past several years this is no excuse for the negligence and wastefulness of Durant

Write a response in which you discuss what questions would need to be answered in order to decide whether the recommendation is likely to have the predicted result Be sure to explain how the answers to these questions would help to evaluate the recommendation

전임 시장인 Durant 씨는 Atticus 시에 대해 사과할 의무가 있습니다 Atticus와 Hartley를 잇는 River Bridge 교량에 대한 피해와 이 교량에서 오랫동안 주민들이 겪어오고 있는 교통 혼잡 문제들은 실제로 20년 전부터 시작된 것이었습니다 결정적으로 그가 교량 공사를 허가했던 바로 그 장본인입니다 당시 비슷한 공사비용으로 폭이 더 넓고 튼튼하게 설계된 교량을 허가했다면 이러한 문제나 피해는 발생하지 않았을 겁니다 더군다나 이 다리는 지난 20년 동안 상류에 건설된 훨씬 오래된 Derby 다리보다도 빠르게 부식되어 갔습니다 지난 수년동안 심지어 혹한이 있었다 하더라도 이러한 태만과 국고 손실에 대한 책임을 회피할 길이 없는 것입니다

결론 Former Mayor Durant owes an apology to the city of Atticus

1 디자인이 문제가 아닐수 있다 (디자인은 좋았으나 건설과정에 문제가 있었을 수 있다)2 그 당시의 시예산이 적어서 더 큰 다리를 짓기가 불가능했을 수도 있다3 교통량이 많거나 다른 상황으로 인해서 부식이 빨리 됐을 수 있다4 그 당시에는 최선의 선택이였지만 갑자기 변한 상황에 의해서 이런 문제점들이 발생했을 수 있다

GRE AWA John박 박정어학원

4그룹 거짓인과관계 오류 (False Cause) 빈출

Fifteen years ago Omega University implemented a new procedure that encouraged students to evaluate the teaching effectiveness of all their professors Since that time Omega professors have begun to assign higher grades in their classes and overall student grade averages at Omega have risen by 30 percent Potential employers looking at this dramatic rise in grades believe that grades at Omega are inflated and do not accurately reflect student achievement as a result Omega graduates have not been as successful at getting jobs as have graduates from nearby Alpha University To enable its graduates to secure better jobs Omega University should terminate student evaluation of professors

Write a response in which you discuss what specific evidence is needed to evaluate the argument and explain how the evidence would weaken or strengthen the argument

Omega professor evaluation implemented 15 years ago =gt Omega prof assign higher grades 30Employers believe therersquos grade inflation

Thus unsuccessful employment than AlphaTherefore to secure jobs Omega should end evaluating profs

Specific evidence neededRelationship between higher grades and evaluationRelationship between GPA and unsuccessful employmentAlpharsquos education could just be better than OmegaldquoFifteen years agordquo is a long time other factors could have influenced Why is the inflation a problem just now How much is Alpha better Is the comparison just How much gap is thereOmegarsquos student could just be doing better in their studiesComparison to other universities다른 대안 없나hellip Could Omega alleviate the employment problem by implementing a different procedure or program

15 년전 우리 대학은 학생들로 하여금 교수평가를 하도록 한 새로운 조치를 시행했었습니다 이후 교수들은 자신의 학과 학생들에게 높은 학점을 주었으며 그에따라 학생들의 전체 평점이 30나 올랐습니다 외부의 기업체들은 분명 점수가 지나치게 부풀려졌다고 믿고 있습니다 결국 본 대학 졸업생들이 인근 Alpha 대학의 졸업자들보다 구직률이 떨어지는 이유를 잘 보여주고 있는 것입니다 이를 해결하기 위해 이제부터는 학생들에 의한 교수평가제를 중단해야 합니다

결론 Omega University should now terminate student evaluation of professors

반박 교수 평가와 학점 인플레의 연관성이 적다( 교수 평가를 먼저하고 학점을 나중에 매길수도 있다)채용기준에 성적만 있는게 아니다 학업성취의 결과 일수도 있다 Alpha 가 원래 유능했다 Alpha 의 교육내용이 좋았다

GRE AWA John박 박정어학원

In this memo the dean of Omega University(OU) recommends OU to terminate professor evaluation to secure better jobs for the students To support this recommendation the dean offers several reasons However this argument contains several logical flaws which render it unconvincing

A threshold problem with the argument involves the voluntary nature of the evaluationprocedure The dean provides no evidence about the number or percentage of Omegastudents who participate in the procedure Lacking such evidence it is entirely possible thatthose numbers are insignificant in which case terminating the procedure is unlikely to haveany effect on the grade average of Omega students or their success in getting jobs aftergraduationThe argument also assumes unfairly that the grade-average increase is the result of theevaluation procedure--rather than some other phenomenon The dean ignores a host of otherpossible explanations for the increase--such as a trend at Omega toward higher admissionstandards or higher quality instruction or facilities Without ruling out all other possibleexplanations for the grade-average increase the dean cannot convince me that by terminatingthe evaluation procedure Omega would curb its perceived grade inflation let alone help itsgraduates get jobsEven if the evaluation procedure has resulted in grade inflation at Omega the deans claimthat grade inflation explains why Omega graduates are less successful than Alpha graduatesin getting jobs is unjustified The dean overlooks a myriad of other possible reasons forOmegas comparatively poor job-placement record Perhaps Omegas career services areinadequate or perhaps Omegas curriculum does not prepare students for the job market aseffectively as Alphas In short without accounting for other factors that might contribute toOmega graduates comparative lack of success in getting jobs the dean cannot justify theclaim that if Omega curbs its grade inflation employers will be more likely to hire OmegagraduatesFinally even if the dean can substantiate all of the foregoing assumptions the deansassertion that Omega must terminate its evaluation procedure to enable its graduates to findbetter jobs is still unwarranted in two respects First the dean ignores other possible ways bywhich Omega can increase its job-placement record--for example by improving its publicrelations or career-counseling services Second the dean unfairly equates more jobs withbetter jobs In other words even if more Omega graduates are able to find jobs as a result ofthe deans recommended course of action the kinds of jobs Omega graduates find would notnecessarily be better onesIn sum the deans argument is unpersuasive as it stands To strengthen it the dean mustprovide better evidence that the increase in grade average is attributable to Omegasprofessor-evaluation procedure and that the end result is a perception on the part ofemployers that Omega graduates are less qualified for jobs than Alpha graduates To betterassess the argument I would need to analyze 15-year trends in (l) the percentage of Omegastudents participating in the evaluation procedure (2) Omegas admission standards andquality of education and (3) Omegas emphasis on job training and career preparation I wouldalso need to know what other means are available to Omega for enabling its graduates to findbetter jobs

GRE AWA John박 박정어학원

The following appeared in a memo from a vice president of Quiot Manufacturing

During the past year Quiot Manufacturing had 30 percent more on-the-job accidents than at the nearby Panoply Industries plant where the work shifts are one hour shorter than ours Experts say that significant contributing factors in many on-the-job accidents are fatigue and sleep deprivation among workers Therefore to reduce the number of on-the-job accidents at Quiot and thereby increase productivity we should shorten each of our three work shifts by one hour so that employees will get adequate amounts of sleep

Write a response in which you examine the stated andor unstated assumptions of the argument Be sure to explain how the argument depends on these assumptions and what the implications are for the argument if the assumptions prove unwarranted

The following appeared in a memo from a vice president of Alta Manufacturing

During the past year Alta Manufacturing had thirty percent more on-the-job accidents than nearby Panoply Industries where the work shifts are one hour shorter than ours Experts believe that a significant contributing factor in many accidents is fatigue caused by sleep deprivation among workers Therefore to reduce the number of on-the-job accidents at Alta we recommend shortening each of our three work shifts by one hour If we do this our employees will get adequate amounts of sleep

Write a response in which you discuss what questions would need to be answered in order to decide whether the recommendation and the argument on which it is based are reasonable Be sure to explain how the answers to these questions would help to evaluate the recommendation

The following appeared in a memo from the vice president of Butler Manufacturing

During the past year workers at Butler Manufacturing reported 30 percent more on-the-job accidents than workers at nearby Panoply Industries where the work shifts are one hour shorter than ours A recent government study reports that fatigue and sleep deprivation among workers are significant contributing factors in many on-the-job accidents If we shorten each of our work shifts by one hour we can improve Butler Manufacturings safety record by ensuring that our employees are adequately rested

1 Write a response in which you discuss what specific evidence is needed to evaluate the argument and explain how the evidence would weaken or strengthen the argument

2 Write a response in which you discuss what questions would need to be answered in order to decide whether the recommendation is likely to have the predicted result Be sure to explain how the answers to these questions would help to evaluate the recommendation

4번 반복됨

Alta has 30 more job accidents than Panoply(work shifts one hour shorter)Experts Job accidents caused by fatigue and sleep deprivationTherefore to reduce job accidents and increase productivity shorten three work shifts by one hour for adequate sleep

지난해 우리 회사는 인근 Panoply Industries보다 업무상 재해가 30나 더 많았다 그 회사는 우리보다 근무 교대시간이 1시간 정도 짧았다 전문가들은 대부분의 업무상 재해에 있어서 가장 중요한 요인이 과로와 수면부족으로 보고있다 따라서 우리 회사에서 높은 산업재해를 줄이고 아울러 생산성을 높이기 위해서는 근로자들이 충분한 수면을 취할 수 있도록 1시간씩 3교대 시간을 줄여야 한다

In this memo the (author) vice president of Alta Manufacturing (AM) recommends that to reduce on-the-job accidents and increase productivity AM should shorten its three work shifts by one hour so that employees can

GRE AWA John박 박정어학원

get more sleep To support this recommendation the author provides several evidences However careful scrutiny of each of the facts reveals that it provides little credible support for the authorrsquos recommendation QuestionsThe number of accidents What kind of accidents The seriousness of the accidents is importantHow many employees are in each company What are their productsFalse cause Sleep may not be the reason for the on-the-job accidents What do Alta and Panoply manufacture

First of all the author believes that fatigue caused the on-the job accidents However there could be other reasons The author observes a correlation between sleep deprivation and on-the-job accidents then concludes that the former is the cause of the latter However the author fails to rule out other possible explanations For example it is entirely possible that Alta factories require more strenuous and dangerous labor than Panoply Without ruling out all other such factors it is unfair to conclude that fatigue is responsible for the accidents In addition the work-shifts may not be the cause of the sleep deprivation and fatigue It is possiblehellip Thus the author should provide what exactly Panoply and Alta manufacture and more precise data about their working conditions to be more convincing

Shortening the shift by one hour does not necessarily lead to more sleep And is one hour enoughLess accidents does not mean increased productivity

결론 We should shorten each of out three work shifts by one hour

반박 경쟁사에 비해서 시간당 하는 업무량이 많아서 더욱 피곤할 수도 있다 시간이 문제가 아니라 노후된 시설 설비 자체의 문제 작업 자체가 원래 위험한 것이여서 사고가 많을 수도 있다 다른 회사는 더욱 많은 작업시간에도 불구하고 안정한 작업여건으로 인해서 사고율이 오히려 더 작을 수도 있다비교사의 재해감소가 다른 요인일수 있다(안전 교육 철저)줄인 시간이 피로회복이나 수면으로 연결 안될수 있음(술을 마실 수도 있고 그 시간에 휴식을 취하지 않고 다른일을 함으로써 더욱 피로해질수 있다)

This editorial recommends that Alta Manufacturing reduce its work shifts by one hour each inorder to reduce its on-the-job accident rate and thereby increase Altas productivity To supportthis recommendation the author points out that last year the number of accidents at Alta was30 greater than at Panoply Industries where work shifts were one hour shorter The authoralso cites certain experts who believe that many on-the-job accidents are caused by fatigueand sleep deprivation I find this the argument unconvincing for several reasonsFirst and foremost the author provides absolutely no evidence that overall workerproductivity is attributable in part to the number of on-the-job accidents Although commonsense informs me that such a relationship exists the author must provide some evidence ofthis cause-and-effect relationship before I can accept the authors final conclusion that theproposed course of action would in fact increase Altas productivitySecondly the author assumes that some accidents at Alta are caused by fatigue or sleepdeprivation However the author overlooks other possible causes such as inadequateequipment maintenance or worker training or the inherent hazards of Altas manufacturingprocesses By the same token Panoplys comparatively low accident rate might be attributablenot to the length of its work shifts but rather to other factors such as superior equipmentmaintenance or worker training In other words without ruling out alternative causes ofon-the-job accidents at both companies the author cannot justifmbly conclude that merely byemulating Panoplys work-shift policy Alta would reduce the number of such accidentsThirdly even assuming that Altas workers are fatigued or sleep-deprived and that this is thecause of some of Altas on-the-job accidents in order to accept the authors solution to thisproblem we must assume that Altas workers would use the additional hour of free time tosleep or rest However the author provides no evidence that they would use the time in thismanner It is entirely possible that Altas workers would use that extra hour to engage in someother fatiguing activity Without ruling out this possibility the author cannot convincinglyconclude that reducing Altas work shifts by one hour would reduce Altas accident rateFinally a series of problems with the argument arise from the scant statistical information onwhich it relies In comparing the number of accidents at Alta and Panoply the author fails toconsider that the per-worker accident rate might reveal that Alta is actually safer than Panoplydepending on the total number of workers at each company Second perhaps accident rates

GRE AWA John박 박정어학원

at the two companies last year were aberrations and during other years Altas accident ratewas no greater or even lower than Panoplys rate Or perhaps Panoply is not representativeof industrial companies generally and that other companies with shorter work shifts have evenhigher accident rates In short since the argument relies on very limited statistical information Icannot take the authors recommendation seriouslyIn conclusion the recommendation for emulating Panoplys work-shift policy is not wellsupported To convince me that shorter work shifts would reduce Altas on-the-job accidentrate the author must provide clear evidence that work-shift length is responsible for some ofAltas accidents The author must also supply evidence to support her final conclusion that alower accident rate would in fact increase overall worker productivity

The following appeared in a memo from the vice president of marketing at Dura-Sock Inc

A recent study of our customers suggests that our company is wasting the money it spends on its patented Endure manufacturing process which ensures that our socks are strong enough to last for two years We have always advertised our use of the Endure process but the new study shows that despite our socks durability our average customer actually purchases new Dura-Socks every three months Furthermore our customers surveyed in our largest market northeastern United States cities say that they most value Dura-Socks stylish appearance and availability in many colors These findings suggest that we can increase our profits by discontinuing use of the Endure manufacturing process

1 Write a response in which you examine the stated andor unstated assumptions of the argument Be sure to explain how the argument depends on these assumptions and what the implications are for the argument if the assumptions prove unwarranted

2 Write a response in which you discuss what specific evidence is needed to evaluate the argument and explain how the evidence would weaken or strengthen the argument

3 Write a response in which you discuss what questions would need to be answered in order to decide whether the recommendation and the argument on which it is based are reasonable Be sure to explain how the answers to these questions would help to evaluate the recommendation

Intro The vice president of marketing at Dura-Sock Inc is offering a potentially harmful investment recommendation by claiming that Dura-Sock should discontinue its use of the ldquoEndurerdquo process To support his recommendation he points out a study that Dura-Sock customers actually purchase the socks every three months and a survey that reveals that Dura-Sock customers like the sockrsquos stylish appearance and availability in many colors The study and survey however are insufficient in supporting his proposal and the VP makes several unwarranted assumptionsIntro (simplified) The VP states that though Dura-Socks last for two years customers buy the socks every three months Therefore he assumes that the consumersrsquo motive for buying the produce is not its durabilityHowever the author fails to rule out other possible motivation for consumption

Even if the survey is reliable the author should consider the rest of the market Vague terms ldquowasting moneyrdquomdashprecisely how much are they wasting Studysurvey errorThe company must calculate the outcome of such momentous decisionStudy participantsrsquo comment that they prefer Dura-Sock for its stylishness and availability might take Dura-Sockrsquos enduring quality for granted

우리회사 제품 소비자들에 대한 최근 조사에서 지난 2년여간 양말의 내구성을 강하게 하는 필수공정이었던 자사 특허의 Endure 공정에 들어가는 비용이 낭비라고 말하고 있다 우리 회사는 항상 이 공정 처리에 대한 광고를 내보냈으나 이에 대한 시장 조사에서 실제로 고객들은 이 신제품을 평균 석달마다 구매하는 것으로 나타났다 더군다나 북동부지역에서 실시한 대규모 시장조사에 응답한 고객들은 양말의 모양과 색상등에 더

GRE AWA John박 박정어학원

관심을 나타냈다 이러한 결과는 우리회사가 신기술 공법을 중단하면 그에 따라 수익이 늘어날 것이라는 것을 말해주고 있는 것이다주장 These findings suggest that Dura0Sock can increase its profits by discontinuing its use of the ldquoEndurerdquo manufacturing process

1 survey가 정확한 소비자의 의견을 나타낸 것인가 다른 선택없이 양자택일과 같은 방법의 survey였는지2 북동부지역의 시장조사가 전체 의견을 대표할 수 있나3 사람들이 모양이나 색상에 앞서 내구성을 먼저 평가했을 수도 있다 내구성을 갖추었다는 전제하에 모양과 색상에 관심을 드러낸 것일 수 있다4 소비자가 도매상(retail)인지 소매상(whole)인지가 없다

The following appeared in a business magazine

As a result of numerous complaints of dizziness and nausea on the part of consumers of Promofoods tuna the company requested that eight million cans of its tuna be returned for testing Promofoods concluded that the canned tuna did not after all pose a health risk This conclusion is based on tests performed on samples of the recalled cans by chemists from Promofoods the chemists found that of the eight food chemicals most commonly blamed for causing symptoms of dizziness and nausea five were not found in any of the tested cans The chemists did find small amounts of the three remaining suspected chemicals but pointed out that these occur naturally in all canned foods

Write a response in which you discuss what questions would need to be addressed in order to decide whether the conclusion and the argument on which it is based are reasonable Be sure to explain how the answers to the questions would help to evaluate the conclusion

Representativeness of the tested cansThey should conduct a comparative studyThe testing could be biased because Promofoods employees conducted the testingHow much (quantity) of the five and three suspected chemicals were in the canned foodsFalse cause The substance that caused dizziness and nausea may not be one of the eight common chemicals

많은 소비자들의 현기증과 구역질 불만에 따라 Promofoods사는 지난해 참치 캔 8백만 개를 테스트하기 위해 반품시켰다 그 결과 캔에서는 건강에 위험이 될 수 있는 화합물질이 없었던 것으로 회사측은 결론지었다 이러한 결론은 회사측 화학연구자들이 회수된 캔의 샘플을 테스트해서 이들 증상의 원인이 되는 8가지 화합물 중에서 5가지가 실험된 캔에서 발견되지 않았다는 사실에 근거한 것이다 이들 화학자들은 나머지 3개가지 화합물이 모든 캔 식료품에서 흔히 발견되는 것이라고 언급했다 결론 Promofoods concluded that the cans did not after all contain chemicals that posed a health risk

1 공인된 기간에서 테스트를 한 것이 아니고 자사에서 직접 테스트를 했기에 신뢰성이 안간다 2 이런 증상을 일으키는 8개의 물질 말고 다른 물질들이 캔속에 많이 포함됬을수 있다 3 나머지 3개의 물질들의 함유량이 많아서 다른 종류의 캔들은 문제를 일으키지 않지만 참치캔은 문제를

일으킬 수 있다

This magazine article concludes that the 8 million cans of tuna Promofoods recalled due tocomplaints about nausea and dizziness do not after ail contain any chemicals that pose a

GRE AWA John박 박정어학원

health risk To support this conclusion the author cites the fact that five of eight chemicalscommonly causing these symptoms were not found in the recalled cans while the other threealso occur naturally in other canned foods For several reasons this evidence lends littlecredible support to the authors conclusionTo begin with the author relies partly on the fact that although three of the eight chemicalsmost commonly blamed for nausea and dizziness appeared in Promofoods recalled tunathese chemicals also occur naturally in other canned foods However this fact alone lends nosupport to the authors conclusion for two reasons First the author might be ignoring animportant distinction between naturally occurring chemicals and those not occurring naturallyIt is entirely possible that these three chemicals do not occur naturally in Promofoods tunaand that it is for this reason that the chemicals cause nausea and dizziness Secondly it isentirely possible that even when they occur naturally these chemicals cause the samesymptoms Unless the author rules out both possibilities he cannot reliably conclude that therecalled tuna would not cause these symptomsAnother problem with the argument is that the authors conclusion is too broad Based onevidence about certain chemicals that might cause two particular heath-related symptoms theauthor concludes that the recalled tuna contains no chemicals that pose a health risk Howeverthe author fails to account for the myriad of other possible health risks that the recalled tunamight potentially pose Without ruling out all other such risks the author cannot justifiablyreach his conclusionA third problem with the argument involves that fact that the eight particular chemicals withwhich the test was concerned are only the eight most commonly blamed for nausea anddizziness It is entirely possibly that other chemicals might also cause these symptoms andthat one or more of these other chemicals actually caused the symptoms Without ruling outthis possibility the author cannot jusufiably conclude that the recalled tuna would not causenausea and dizzinessA final problem with the argument involves thetesting procedure itself The author providesno information about the number of recaUed cans tested or the selection method used Unlessthe number of cans is a sufficiently large sample and is statistically repre sentative of all therecalled cans the studys results are not statistically reliableIn conclusion the article is unconvincing as it stands To strengthen the assertion that therecalled tuna would not cause nausea and dizziness the author must provide evidence thatthe three chemicals mentioned that occur naturally in other canned foods also appear naturallyin Promofoods tuna The author must also provide evidence that ingesting other canned foodscontaining these three chemicals does not cause these symptoms To better evaluate theargument we would need to know whether the sample used in the tests was statisticallysignificant and representative of all the recalled tuna We would also need to know what otherchemicals in the recalled tuna might pose any health risk at all

5그룹 불충분 조건오류 빈출

Natures Way a chain of stores selling health food and other health-related products is opening its next franchise in the town of Plainsville The store should prove to be very successful Natures Way franchises tend to be most profitable in areas where residents lead healthy lives and clearly Plainsville is such an area Plainsville merchants report that sales of running shoes and exercise clothing are at all-time highs The local health club has more members than ever and the weight training and aerobics classes are always full Finally Plainsvilles schoolchildren represent a new generation of potential customers these schoolchildren are required to participate in a fitness-for-life program which emphasizes the benefits of regular exercise at an early age

Write a response in which you examine the stated andor unstated assumptions of the argument Be sure to

GRE AWA John박 박정어학원

explain how the argument depends on these assumptions and what the implications are for the argument if the assumptions prove unwarranted

False cause

First of all the author believes that the Increased sales of running shoes and exercise clothing indicates

Plainesville residentsrsquo interest in leading healthy lives However this assumption is not logically convincing for

several reasons could be a fashion trendTime shift ldquoFitness for liferdquo might not have any influence on schoolchildren as they growFalse cause There could be other reasons for member increase in the health clubAll of the above are insufficient condition

The author has to prove that local residents are interested in leading healthy lives However he supports his conclusion with insufficient evidence Nevertheless even if the residents are concerned with health naturersquos way may not be successful First

그 동안의 경험을 토대로 볼 때 건강생활과 밀접히 관련되어 있는 거주 지역에서 본 상점들이 아주 호응을 얻고 있다 따라서 이러한 주민들이 많이 거주하고 있는 Plainsville 에 새로운 상점들을 계속 세워야 한다 이 지역 상인들은 런닝화와 운동복 판매가 가장 높다고 말한다 불과 5 년전에는 거의 전무하다시피하던 지역 헬스 클럽의 경우도 엄청나게 많은 회원을 확보하고 있으며 웨이트 트레이닝과 에어로빅 강좌들도 항상 만원이라고 한다 새로운 고객층을 예측해 보는 것도 가능하다 이 지역의 학생들의 경우 Fitness for Life프로그램을 받게 되는데 이러한 프로그램을 통해서 유년시절부터 정규적인 운동 습관을 들이게 하고 있는 것이 그것이다

결론 We should therefore build our next new store in Plainsville

반박 그동안의 경험에 의한 과거 통계가 꼭 여기에도 적용되는건 아니다 5 년전 헬스 클럽이 잘 안되었던게 다른 원인이였을수 있다(강사수준 미달 강좌미비)tourist 에 의한 원인 일수 있다 어렸을때부터 운동을 했다고 해서 커서도 관심이 있지는 않다 (오히려 반감이 있을수 있다 혹은 건강하기에 건강에 관심이 적을수도 있다)운동복이나 신발의 판매가 육체노동에 의한 것일수도 있다

IntroductionSupport1049896In this memorandum the author asserts that Naturersquos Way should build its next newstore in Plainsville To support this assertion the author states that Plainsvillesmerchantsrsquo sales of exercise clothing are going well the local health club has moremembers than ever and a new generation of customers will help to ensure NaturersquosWayrsquos success At first glance the authorrsquos assumption seems convincing but in-depth scrutiny revealsthat it lacks substantial evidence as it stands

Body 1-SamplingTopic Sentence 1To begin with the author assumes that the merchantsrsquo report indicates that the residentsare concerned about their health However this assumption is based on unsubstantiated

GRE AWA John박 박정어학원

data Example 1 (Rebuttal1) First if we do not know the total volume of items sold and the price of the goods exactly we cannot infer whether the residents are actually buying many goods Example 2 (Rebuttal2)In addition to that the report emphasizes the rising sales of running shoes and exerciseclothing however these may not be hot-selling items for Naturersquos Way or may not be theproducts the company is planning to sell Concluding Sentence Therefore in order to make the argument reliable the author should reconsider themerchantsrsquo report with more detailed data

Body 2-CausalTopic Sentence 2Second the author contends that the health clubs classes are full yet this does not meanthat many people actually use the health club other factors may be the real cause forthose closed classes Example 1 (Rebuttal1) To begin with if the health club is very small the number of people working out wouldnot be a large one In fact regular gym-going may just be a vogue among a smallunrepresentative segment of Plainsvilles population Example 2 (Rebuttal2) Moreover it is possible that most of the people who exercise in the health club do weight training and aerobics only to look good and to meet other singles not for their health In that case there would be little demand for health products Concluding SentenceThus the author should not hasten to presume what really caused people to be interested in a healthier lifestyle and enroll in the health club

Body 3-Time-ShiftTopic Sentence 3Finally the author highlights that Naturersquos Way can expect a new generation of customersin Plainsville that will help the company in the long term This notion is mistaken in that itassumes the conditions of the present will continue unchanged in the future Although theschool children are required to participate in the fitness for life program they may notnecessarily buy Naturersquos Ways products Example 1 (Rebuttal1) In the first instance they may suffer a fall in purchasing power arising from future economic difficulties this would cause reluctance to spend a considerable amount of money on health products which tend to be more expensive Example 2 (Rebuttal2)Another possibility is that there may emerge many competitor companies vying with Naturersquos Way so that in the future the school children may not feel the necessity to purchase one companyrsquos health products over anotherrsquosConcluding Sentence Thus the authorrsquos assumption is highly speculative since it relies heavily on unknowablefuture circumstances

ConclusionThesis In sum the author uses many assumptions that are insufficient in supporting his claimsSupportIn order for the authorrsquos claims to be convincing he needs to advance more persuasiveevidence that people in Plainsville really are concerned with their health and health foodThe following was written as a part of an application for a small-business loan by a group of developers in the city of Monroe

A jazz music club in Monroe would be a tremendously profitable enterprise Currently the nearest jazz club is 65 miles away thus the proposed new jazz club in Monroe the C-Note would have the local market all to itself Plus jazz is extremely popular in Monroe over 100000 people attended Monroes annual jazz festival last summer several well-known jazz musicians live in Monroe and the highest-rated radio program in Monroe is Jazz Nightly which airs every weeknight at 7 PM Finally a nationwide study indicates that the typical jazz fan spends close to $1000 per year on jazz entertainment

1 Write a response in which you discuss what specific evidence is needed to evaluate the argument and explain how the evidence would weaken or strengthen the argument

2 Write a response in which you examine the stated andor unstated assumptions of the argument Be

GRE AWA John박 박정어학원

sure to explain how the argument depends on these assumptions and what the implications are for the argument if the assumptions prove unwarranted

3 Write a response in which you discuss what questions would need to be answered in order to decide whether the prediction and the argument on which it is based are reasonable Be sure to explain how the answers to these questions would help to evaluate the prediction

Group error nationwide survey may not reflect local trends Is the nationwide jazz fan population substantialInsufficient non-residents of Monroe may have attended the jazz festival (Body alternative explanation last year may have been an anomaly The author should consider data from various years) The author should indicate how many out of 100000 were Monroe residentsNationwide study Does this reflect Insufficient Citizens of Monroe may continue to go to the jazz club 65 miles away

Are the people in Monroe really interested in jazzMajority of the people who attended the jazz festival might not be Monroe residentsSurvey error nationwide study may not be applicable to MonroeJazz musicians who live in MonroeMonopolyRadio station

In this business application the author claims that the proposed jazz club C Note will be very profitable in Monroe To support this claim the author argues for his case with several evidences At first glance the authorrsquos argument seems convincing however careful scrutiny reveals that his argument in specious

To begin with the author claims that Monroersquos citizens are interested in jazz He presents three evidences First Secondhellip Thirdhellip Howeverhellip

Monroe 시에 있는 재즈 음악 클럽은 수익성이 좋은 사업이다 현재 가장 가까이에 있는 클럽은 65 마일 정도 떨어져 있다 따라서 이번에 세우려고 하는 C Note 는 독보적인 위치를 점할것이다 더군다나 재즈는 이 시에서 가장 인기있는 음악이다 지난 여름 재즈 축제에서는 10 만명 이상의 Morone 시 주민이 참석하였고 몇몇 유명한 재즈 음악가들도 이곳에 살고 있으며 저녁때 방영되는 라디오 프로그램중에서 최고의 시청률을 보이고 있는 것도 Jazz Nightly 이다 전국조사에서도 전형적인 재즈 팬들은 재즈 분야에 년간 1천 달러 가까이 지출하고 있는 것으로 보고되고 있다 따라서 C Note 클럽이 돈을 벌 수 있는 사업이라는 것은 확실한 것이다

결과 It is clear that the C Note cannot help but make money반박 nearest jazz club 이 양질의 써비스로 여전히 손님을 끌수도 있다Festival 에 얼마나 참여하는지가 jazz 의 인기를 반영하지 않는다 뮤지션이 많이 사는거랑 jazz 의 인기가 상관없다라디오 프로그램이 다른 요인에 의해서 인기일수도 있다 (진행자때문)전국 통계 적용 불가화목 실전반_Ms Noh6In this application the author suggests that a jazz club in Monroe will make a number of profits To support this suggestion the author exemplifies the local condition popularity of jazz in Monroe and nationwide study However careful scrutiny of each of the facts reveals that it provides little credible support for the authorrsquos recommendation Good clear intro

First the author assumes that jazz is popular in Monroe because of several facts the jazz festival last year had high participation some famous jazz musicians live in Monroe and the high-rated radio program is lsquoJazz Nightlyrsquo However this assumption has many drawbacks that must be seriously considered(Good topic sentences) If many attendants in the last-yearrsquos festival came from other cities and not Monroe it is hard to conclude that Monroersquos people like jazz Therefore the author must examine how many Monroe residents actually attended the festival On top of that there is little relationship between habitation of famous jazz musician and the popularity of jazz in Monroe Although several well-known musicians live there if they do not take part in any jazz performance of Monroe this might have no effect to the interest of Monroersquos residents

GRE AWA John박 박정어학원

about jazz Finally in the case of radio program this is also not suitable reason why jazz is popular in Monroe It might be possible that people cannot help choosing lsquoJazz Nightlyrsquo because there are few radio programs at Night The fact that the radio program is the highest rating program is not a germane evidence The approximate number of listeners would be the more crucial evidence Therefore the author needs to seriously deliberate the correlation between jazzrsquos popularity in Monroe and his examples (Good logical flow and clarity)

Second the author uses as evidence the nationwide study that jazz fans spend much money on jazz entertainment to substantiate why starting a jazz club in Monroe will be profitable In other words the author assumes that the characteristics of a nationwide study can be applied to Monroe The national study would lend support to the applicantrsquos claim only if residents in Monroe typify national jazz fans However the author does not provide credible evidence that this is the case Moreover the populations of jazz fans nationwide may be insubstantial Thus the author should not infer hastily that Monroersquos residents will spend much money on enjoying jazz from the nationwide study

Lastly even if jazz is popular in Monroe C Note may not be successful It is entirely possible that residents might still prefer other clubs where they have always went In addition there is another possibility that the nearest jazz club will attract many of Monroersquos people because it serves fine performances and is equipped with favorite facilities Without considering these other possibilities the author cannot make his argument convincing In sum the author presents many reasons that are insufficient in supporting his or her claim In order for the authorrsquos claims to be convincing he needs to advance more persuasive evidence such as the total number of Monroe residents who attended the jazz festival the effects on the popularity of jazz by the musicians living in Monroe and the actual number of residents who would typify themselves to be jazz fans through a local survey Without substantial evidence that C Note will be successful in Monroe the businessmen may be overinvesting in what might lead to a business failureExcellent clarity Score 50

The following appeared in a newsletter offering advice to investors

Over 80 percent of the respondents to a recent survey indicated a desire to reduce their intake of foods containing fats and cholesterol and today low-fat products abound in many food stores Since many of the food products currently marketed by Old Dairy Industries are high in fat and cholesterol the companys sales are likely to diminish greatly and company profits will no doubt decrease We therefore advise Old Dairy stockholders to sell their shares and other investors not to purchase stock in this company

Write a response in which you discuss what questions would need to be answered in order to decide whether the advice and the argument on which it is based are reasonable Be sure to explain how the answers to these questions would help to evaluate the advice

Survey 80

GRE AWA John박 박정어학원

Old Dairy could change their products and manufacture low fat dairy foodsLess competing companies Old Dairy could eventually be the only company that produces hellipImprecise numbers and measurementsCustomers may still buy high fat dairy products

The author of the newsletter is offering potentially dangerous advice by recommending Old Dairy stockholders to withdraw investment and stop purchase What is more the authorrsquos prediction debases the reputation and business of Old Dairy and if false could devoid the investment opportunity of the newsletter readers Therefore investors should examine whether the authorrsquos evidences are substantial

To begin with the author states that 80 percent of the respondents in a survey indicated a desire to reduce their intake of foods He therefore argues that Old Dairyrsquos high fat and cholesterol products would decrease in sales However the author makes a crucial error in this argument First the author provides no evidence that the surveyrsquos results are statistically reliable Were they representative of all the customers Were they chosen for the survey randomly Furthermore the desire to reduce fat and cholesterol intake is a pervasive trend in todayrsquos opulent society however the author erroneously identifies this as a new phenomenon which will affect consumer trends Second having a desire to reduce fat and cholesterol intake does not necessarily indicate that people who have this desire will actually reduce consuming these types of products It is entirely possible that they may continue buying Old Dairy products for its quality and taste Accordingly the author cannot draw any firm conclusion that people will not buy Old Dairy products Therefore if any of these cases are true the author may be offering investors a detrimental investment advice

최근 조사에 대한 응답자중 80 이상이 자신이 먹는 음식에서 지방과 콜레스테롤의 함유량을 줄이고 싶다고 한다 아울러 요즘은 많은 식료품 가계에서 저지방 제품들을 많이 취급하고 있다 현재 Old Dairy Industries가 판매하고 있는 많은 음식제품들은 지방과 콜레스테롤이 높기 때문에 이 회사의 매출이 격감할 것으로 보이며 당연히 매출이익도 줄어들것이다 따라서 이 회사의 주주들은 주식을 매각하고 다른 주식 투자가들도 이 회사의 주식을 매입하지 않는 것이 좋다

결론 Old Dairy stockholders to sell their shares and other investors not to purchase stock in this company

반박 모든 상품이 다 고 지방 고 칼로리는 아니다(비록 많을지라도) 일부의 식품의 경우 기호에 맞어서 히트해서 전체적인 수입이 증가할 수도 있다국내시장만 생각할 수 없다( 외국시장에서 호황을 누릴수 있다 )입맛이라는게 즉각 바뀌는게 아니다

The following appeared in a letter to the editor of the Balmer Island Gazette

On Balmer Island where mopeds serve as a popular form of transportation the population increases to 100000 during the summer months To reduce the number of accidents involving mopeds and pedestrians the town council of Balmer Island should limit the number of mopeds rented by the islands moped rental companies from 50 per day to 25 per day during the summer season By limiting the number of rentals the town council will attain the 50 percent annual reduction in moped accidents that was achieved last year on the neighboring island of Seaville when Seavilles town council enforced similar limits on moped rentals

1 Write a response in which you discuss what questions would need to be answered in order to decide whether the recommendation is likely to have the predicted result Be sure to explain how the answers to these questions would help to evaluate the recommendation

2 Write a response in which you discuss what questions would need to be answered in order to decide whether the prediction and the argument on which it is based are reasonable Be sure to explain how the answers to these questions would help to evaluate the prediction

3 Write a response in which you examine the stated andor unstated assumptions of the argument Be sure to explain how the argument depends on these assumptions and what the implications are for the argument if the assumptions prove unwarranted

Whatrsquos the actual population of Balmer Island 100000mdashis this a significant increase What kind of accidents Skin abrasions or serious injury And compared to Seaville how serious are the accidents and the actual number of accidents Did Seaville enforce other restrictions like safety signsHow different are the conditions of Balmer

GRE AWA John박 박정어학원

and Seaville regarding population road (safety) conditions topography other town-government regulation How much will the economy of Balmer be affected do to this restriction Could it cause an economic recession due to the fact that these rental companiesrsquo chance to make money is only during the summer thereby weakening the economic infrastructure Are there any other ways that could better alleviate the accident rate

Statistics 50-impreciseAnalogy Balmer compared with TorseauFalse Cause Accidents might have occurred because of reasons other than mopeds False Cause population increase may not be part of the cause of the accidentsOther explanations for the accident pedestrians few road safety regulations narrow roadsThere could be other better solutionshellip

Balmer Island의 인구가 여름철에는 십만명으로 늘어난다 2륜차와 보행자간 사고를 줄이기 위해 시의회는 6개의 자전거를 포함한 2륜차 대여업체에게 이 기간동안에는 대여숫자를 일일 50에서 30으로 제한하도록 할 것이다 대여숫자를 줄임으로써 시 의회는 지난해 이웃한 Torseau섬에서 이와 동일한 규제를 시행해서 50나 줄인 결과를 보고 마찬가지로 50를 줄일수 있다고 확신하고 있다

결론 The town council of Balmer Island should linit the number

반박 보행자의 부실에 의해서 사고가 많이 일어날수도 있다렌탈수의 줄임만이 대책은 아니다(대부분의 사람들이 렌탈 보다는 소유하고 있을 수도 있다)옆섬과는 상황이 다를수도 있다(그 섬에서는 사고의 원인이 많은 자전거 수로 인한것일수있다) 하지만 이 섬은 좁은 도로가 원인일 수도 있고 도로 안전 장치의미비가 원일일수 있다

In this letter the author recommends that Balmer Island should limit the number moped rentals from 50 to 30 per day To support this recommendation the author points out several reasons However careful scrutiny of each of the facts reveals that it is filled with unanswered questions that could significantly weaken the authorrsquos recommendation with loops and holes which are answered

The recommendation depends on the assumption that no alternative means of reducing the number of accidents are available However the author fails to offer any evidence to substantiate this crucial assumption It is highly possible that means other than this would better solve the problem Perhaps they could widen the roads or put-up more safety signs Or perhaps the accidents were due to the lack of skills in which case proper safety training would significantly alleviate the problem Without considering and ruling out these and other alternative means of reducing accidetns the author cannot confidently conclude that merely emulating Torseau would suffice Moreover the author is advising a recommendation which could potentially harm the economy of Balmer Island sincehellip Moreover the Balmer Island should alternative means to reduce accidents because limiting moped rentals during the summer could harm the economy of Balmerhellip

First of all the author believes that increase in population and the number of moped rentals are responsible for the accidents It is entirely possible that other factors are responsible for the accidents Perhaps Balmer Islandrsquos lack of safety signs was a major factor Or maybe the roads are narrow and dangerous on the Island therefore the town council could enforce stricter traffic regulations to alleviate the problem Accordingly if either of these scenarios is true the author cannot draw any firm conclusion that increase in the number of population and moped rentals are the cause of the accidents

The author of this editorial recommends that to reduce accidents involving mopeds andpedestrians Balmer Islands city council should restrict moped rentals to 30 per day down from50 at each of the islands six rental outlets To support this recommendation the author citesthe fact that last year when nearby Torseau Islands town council enforced similar measuresTorseaus rate of moped accidents fell by 50 For several reasons this evidence providesscant support for the authors recommendationTo begin with the author assumes that all other conditions in Balmer that might affect therate of moped-pedestrian accidents will remain unchanged after the restrictions are enactedHowever with a restricted supply of rental mopeds people in Balmer might purchase mopedsinstead Also the number of pedestrians might increase in the future with more pedestriansespecially tourists the risk of moped-pedestrian accidents would probably increase For thatmatter the number of rental outlets might increase to make up for the artificial supplyrestriction per outlet--a likely scenario assuming moped rental demand does not declineWithout considering and ruling out these and other possible changes that might contribute to ahigh incidence of moped-pedestrian accidents the author cannot convince me that theproposed restrictions will necessarily have the desired effect

GRE AWA John박 박정어학원

Next the author fails to consider other possible explanations for the 50 decline inTorseaus moped accident rate last year Perhaps last year Torseau experienced unusually fairweather during which moped accidents are less likely Perhaps fewer tourists visited Tot seanlast year than during most years thereby diminishing the demand for rental mopeds to belowthe allowed limits Perhaps last year some of Torseaus moped rental outlets purchased newmopeds that are safer to drive Or perhaps the restrictions were already in effect but were notenforced until last year In any event a decline in Torseaus moped accident rate during onlyone year is scarcely sufficient to draw any reliable conclusions about what might have causedthe decline or about what the accident rate will be in years aheadAdditionally in asserting that the same phenomenon that caused a 50 decline in mopedaccidents in Torseau would cause a similar decline in Balmer the author relies on what mightamount to an unfair analogy between Balmer and Torseau Perhaps Balmers ability to enforcemoped-rental restrictions does not meet Torseaus ability if not then the mere enactment ofsimilar restrictions in Balmer is no guarantee of a similar result Or perhaps the demand formopeds in Torseau is always greater than in Balmer Specifically if fewer than all availablemopeds are currently rented per day from the average Balmer outlet while in Torseau everyavailable moped is rented each day then the proposed restriction is likely to have less impacton the accident rate in Balmer than in TorseauFinally the author provides no evidence that the same restrictions that served to reduce theincidence of all moped accidents by 50 would also serve to reduce the incidence ofaccidents involving mopeds and pedestrians by 50 Lacking such evidence it is entirelypossible that the number of moped accidents not involving pedestrians decreased by a greaterpercentage while the number of moped-pedestrian accidents decreased by a smallerpercentage or even increased Since the author has not accounted for these possibilities theeditorials recommendation cannot be taken seriouslyIn conclusion the recommendation is not well supported To convince me that the proposedrestriction would achieve the desired outcome the author would have to assure me that nochanges serving to increase Balmers moped-pedestrian accident rate will occur in theforeseeable future The author must also provide dear evidence that last years decline inmoped accidents in Torseau was attributable primarily to its moped rental restrictions ratherthan to one or more other factors In order to better evaluate the recommendation I wouldneed more information comparing the supply of and demand for moped rentals on the twoislands I would also need to know the rate of mopedpedestrian accidents in Torseau both priorto and after the restrictions were enforced in TorseauThe following appeared in a magazine article about planning for retirement

Clearview should be a top choice for anyone seeking a place to retire because it has spectacular natural beauty and a consistent climate Another advantage is that housing costs in Clearview have fallen significantly during the past year and taxes remain lower than those in neighboring towns Moreover Clearviews mayor promises many new programs to improve schools streets and public services And best of all retirees in Clearview can also expect excellent health care as they grow older since the number of physicians in the area is far greater than the national average

Write a response in which you discuss what specific evidence is needed to evaluate the argument and explain how the evidence would weaken or strengthen the argument

-Natural beauty and consistent climate may not be the most wanted qualities-Housing costs could have lowered on a national level wealthy retirees may not care about costs-Taxes may be high compared to the nationrsquos average tax rate-What about other qualities of Clearview Crime rate what qualities would retirees want -If schools streets and public services need improvement then this is proof that the current condition of Clearview is low Or due to budgetary reasons the mayor may not follow-up on his promise because of lowered tax rate -Schools and people who are retired no relationship-Physicians What kind of physicians Number is irrelevant Are these physicians capable of addressing the illnesses of old people

This author argues that anyone seeking a place to retire should choose Clearview To supportthis argument the article cites Clearviews consistent climate and natural beauty its fallinghousing costs its low property taxes compared to nearby towns and the mayors promise toimprove schools streets and services The article also claims that retirees can expectexcellent health care because the number of physicians in Clearview greatly exceeds thenational average This argument is flawed in several critical respectsTo begin with although consistent climate and natural beauty might be attractive to manyretirees these features are probably not important to all retirees For many retirees it isprobably more important to live near relatives or even to enjoy changing seasons Thus I

GRE AWA John박 박정어학원

cannot accept the authors sweeping recommendation for all retirees on this basisAlso Clearviews declining housing costs do not necessarily make Clearview the best placeto retire for two reasons First despite the decline Clearviews housing costs might be highcompared to housing costs in other cities Secondly for wealthier retirees housing costs arenot likely to be a factor in choosing a place to retire Thus the mere fact that housing costshave been in decline lends scant support to the recommendationThe articles reliance on Clearviews property-tax rates is also problematic in two respectsFirst retirees obviously have innumerable choices about where to retire besides Clear viewand nearby towns Secondly for retirees who are well-off financially property taxes are notlikely to be an important concern in choosing a place to retire Thus it is unfair to infer fromClearviews property-tax rates that retirees would prefer ClearviewYet another problem with the argument involves the mayors promises In light of Clearviewslow property-tax rates whether the mayor can follow through on those promises is highlyquestionable Absent any explanation of how the city can spend more money in the areas citedwithout raising property taxes I simply cannot accept the editorials recommendation on thebasis of those promises Besides even if the city makes the improvements promised thoseimprovements--particular the ones to schools--would not necessarily be important to retireesFinally although the number of physicians in Clearview is relatively high the per capitanumber might be relatively low Moreover it would be fairer to compare this per capita numberwith the per capita number for other attractive retirement towns--rather than the nationalaverage After all retirees are likely to place a relatively heavy burden on health-careresources Besides the article provides no assurances that the number of physicians inClearview will remain high in the foreseeable futureIn conclusion the recommendation is poorly supported To strengthen it the author mustconvince me--perhaps by way of a reliable survey--that the key features that the vast majorityof retirees look for in choosing a place to live are consistent climate natural beauty and lowhousing costs The author must also provide better evidence that Clear views property taxesare lower than the those of cities in other areas The author must also explain how the city canmake its promised improvements without raising property taxes Finally to better assess theargument I would need to now how the per capita number of physicians in Clearview wouldcompare to the national average in the futureThe following appeared as a letter to the editor from a Central Plaza store owner

Over the past two years the number of shoppers in Central Plaza has been steadily decreasing while the popularity of skateboarding has increased dramatically Many Central Plaza store owners believe that the decrease in their business is due to the number of skateboard users in the plaza There has also been a dramatic increase in the amount of litter and vandalism throughout the plaza Thus we recommend that the city prohibit skateboarding in Central Plaza If skateboarding is prohibited here we predict that business in Central Plaza will return to its previously high levels

Write a response in which you discuss what questions would need to be answered in order to decide whether the recommendation is likely to have the predicted result Be sure to explain how the answers to these questions would help to evaluate the recommendation

Why two years ago What happened two years ago which started this declineIs the dramatic increase in the ldquopopularityrdquo of skateboarding the cause of the steady decline of shoppers Are there any malls nearby Were there any changes nearby which could affect the decline in customersmdasha big mall perhaps Could the decline be due to the shop ownersHow many skateboarders use the plazaWhere do they skateboardDo they shop and are they customersAre the increase in litter and vandalism due to skateboarders Could this be alleviated by installing CCTVs and hiring security

This editorial concludes that the city should ban skateboarding from its downtown CentralPlaza in order to attract visitors to that area to return the area to its former glory and to makeit a place where people can congregate for fun and relaxation To justify this conclusion theeditorial points out that skateboarders are nearly the only people one sees anymore at CentralPlaza and that the Plaza is littered and its property defaced The editorial also points out thatthe majority of downtown merchants support the skate boarding ban This argument is flawedin several critical respectsFirst the editorials author falsely assumes that a ban on skateboarding is both necessaryand sufficient to achieve the three stated objectives Perhaps the city can achieve thoseobjectives by other means as well--for example by creating a new mall that incorporates anattractive new skateboard park Even if banning skateboarders altogether is necessary to meetthe citys goals the author has not shown that this action by itself would suffice Assuming thatthe Plazas reputation is now tarnished restoring that reputation and in turn enticing peopleback to the Plaza might require additional measures--such as removing litter and graffiti

GRE AWA John박 박정어학원

promoting the Plaza to the public or enticing popular restaurant or retail chains to the PlazaSecondly the editorial assumes too hastily that the Plazas decline is attributable to theskateboarders--rather than to some other phenomenon Perhaps the Plazas primary appeal inits glory days had to do with particular shops or eateries which were eventually replaced byless appealing ones Or perhaps the crime rate in surrounding areas has risen dramatically forreasons unrelated to the skateboarders presence at the Plaza Without ruling out these andother alternative explanations for the Plazas decline the editorials author cannot convince methat a skateboard ban would reverse that declineThirdly the editorials author might be confusing cause with effect--by assuming that theskateboarders caused the abandonment of the Plaza rather than vice versa It is entirelypossible that skateboarders did not frequent the Plaza until it was largely abandoned--andbecause it had been abandoned In fact this scenario makes good sense since skateboardingis most enjoyable where there are few pedestrians or motorists to get in the wayFourth it is unreasonable to infer from the mere fact that most merchants favor the ban thatthe ban would be effective in achieving the citys objectives Admittedly perhaps thesemerchants would be more likely to help dean up the Plaza area and promote their businesseswere the city to act in accordance with their preference Yet lacking any supporting evidencethe author cannot convince me of this Thus the survey amounts to scant evidence at best thatthe proposed ban would carry the intended resultFinally the author recommends a course of action that might actually defeat the citysobjective of providing a fun and relaxing place for people to congregate In my experienceskateboarding contributes to an atmosphere of fun and relaxation for adults and children alikemore so than many other types of ambiance Without considering that continuing to allowskateboarding--or even encouraging this activity--might achieve the citys goal more effectivelythan banning the activity the author cannot convincingly conclude that the ban would be in thecitys best interestsIn sum the argument is a specious one To strengthen it the editorials author must providedear evidence that skateboarding and not some other factor is responsible for the conditionsmarking the Plazas decline The author must also convince me that no alternative means ofrestoring the Plaza are available to the city and that the proposed ban by itself would suffice toattract tourists and restore the Plaza to its former glory Finally to better assess the argument itwould be useful to know the circumstances under which the downtown merchants would bewilling to help the city achieve its objectives

6그룹 약한 비유 빈출

The following recommendation appeared in a memo from the mayor of the town of Hopewell

Two years ago the nearby town of Ocean View built a new municipal golf course and resort hotel During the past two years tourism in Ocean View has increased new businesses have opened there and Ocean Views tax revenues have risen by 30 percent Therefore the best way to improve Hopewells economymdashand generate additional tax revenuesmdashis to build a golf course and resort hotel similar to those in Ocean View

Write a response in which you examine the stated andor unstated assumptions of the argument Be sure to explain how the argument depends on these assumptions and what the implications are for the argument if the assumptions prove unwarranted

GRE AWA John박 박정어학원

Assumptions The author assumes that OVrsquos municipal golf course and resort hotel caused tourism new businesses and increased tax revenues There may be other reasons advertising promo He assumes that this will continueAssumes that Ocean View and Hopewell are similar in many waysmdashthe name suggests otherwise OV may have always been a tourist attractions for its beaches We need to know the topography

2년전 Ocean View 시는 시정 소유 골프 및 휴양지 호텔을 신축했다 그리고 지난 2년동안 이 시의 관광객이 증가했으며 새로운 사업들이 생겨났다 그에따라 시의 세수도 30나 증가했다 Hopewell의 경제를 향상시키고 아울러 세수를 늘릴 수 있는 가장 좋은 방법은 Ocean View에 세워진 것과 같은 골프 시설과 휴양지 호텔을 신축하는 것이다

1 다른 요인으로 관광 산업이 발전했을 수도 있다 문화 유적이 발견이 되었거나 도로의 정비등으로 여행자가 늘었을 수도 있다

2 관광 산업의증가가 늘어난 세수의 원인이 아니라 새로 유입된 인구의 증가나 다른 공장에서 발생한 것일 수 있다

3 2년동안 한참 골프가 붐을 이루었을 수 있다 경제상황이 나빠지거나 다른 레포츠가 인근 지역에 생겨난다면 골프하는 사람이 줄어들 수 있다

In this memo HopeweUs mayor recommends that in order to stimulate the towns economyand boost tax revenues HopeweU should build a new golf course and resort hotel just as thetown of Ocean View did two years ago To support this recommendation the mayor points outthat in Ocean View during the last two years tourism has increased new businesses haveopened and tax revenues have increased by 30 I find the mayors argument unconvincingin several important respectsFirst of all it is possible that the mayor has confused cause with effect respecting the recentdevelopments in Ocean View Perhaps Ocean Views construction of a new golf course andhotel was a response to previous increases in tourism and business development increasesthat have simply continued during the most recent two years Since the mayor has failed toaccount for this possibility the claim that Hopewell would boost its economy by alsoconstructing a golf course and hotel is completely unwarrantedSecondly the mayor fails to account for other possible causes of the trends in Ocean Viewduring the last two years The increase in tourism might have been due to improving economicconditions nationwide or to unusually pleasant weather in the region The new businessesthat have opened in Ocean View might have opened there irrespective of the new golf courseand hotel And the 30 increase in tax revenues might have been the result of an increase intax rates or the addition of a new type of municipal taxWithout ruling out these and other alternative explanations for the three recent trends inOcean View the mayor cannot reasonably infer based on those trends that Hopewellseconomy would benefit by following Ocean Views exampleThirdly even if the recent trends in Ocean View are attributable to the construction of the newgolf course and hotel there the mayor assumes too hastily that the golf course and hotel willcontinue to benefit that towns overall economy The mayor has not accounted for thepossibility that increased tourism will begin to drive residents away during tourist season orthat new business development will result in the towns losing its appeal as a place to visit or tolive Unless the mayor can convince me that these scenarios are unlikely I cannot accept themayors recommendation that Hopewell follow Ocean Views exampleFinally the mayors argument rests on the unsubstantiated assumption that Hopewell andOcean View are sufficiently alike in ways that might affect the economic impact of a new golfcourse and hotel Hopewell might lack the sort of natural environment that would attract moretourists and new businesses to the town--regardless of its new golf course and hotel For thatmatter perhaps Hopewell already contains several resort hotels and golf courses that are notutilized to their capacity If so building yet another golf course and hotel might amount to amisallocation of the towns resources--and actually harm the towns overall economyIn sum the mayors recommendation is not well supported To bolster it the mayor mustprovide better evidence that Ocean Views new golf course and hotel and not some otherphenomenon--has been responsible for boosting Ocean Views economy during the last twoyears To better assess the recommendation I would need to know why Ocean View decidedto construct its new golf course and hotel in the first place--specifically what events prior toconstruction might have prompted that decision I would also need to thoroughly compare

GRE AWA John박 박정어학원

HopeweU with Ocean View--especially in terms of their appeal to tourists and businesses--todetermine whether the same course of action that appears to have boosted Ocean Viewseconomy would also boost Hopewells economy

The following is part of a memorandum from the president of Humana University

Last year the number of students who enrolled in online degree programs offered by nearby Omni University increased by 50 percent During the same year Omni showed a significant decrease from prior years in expenditures for dormitory and classroom space most likely because instruction in the online programs takes place via the Internet In contrast over the past three years enrollment at Humana University has failed to grow and the cost of maintaining buildings has increased along with our budget deficit To address these problems Humana University will begin immediately to create and actively promote online degree programs like those at Omni We predict that instituting these online degree programs will help Humana both increase its total enrollment and solve its budget problems

Write a response in which you discuss what questions would need to be answered in order to decide whether the prediction and the argument on which it is based are reasonable Be sure to explain how the answers to these questions would help to evaluate the prediction

Is Omni University successful due to the online degree program 50 Is the decrease in expenditures for dormitory and classroom space due to the decrease in of on-campus students Which classes were successful Does HU have those classes

Even if the long-distance degree programs at Omni University benefited the school the presidentrsquos recommendation that Human College should emulate Omni University is too hasty First OUrsquos name implies that the school would have more majors than Humanahellip the president should examine which degrees were in the long-distance programhellip

지난해에는 Omni 대학에서 개강했던 원거리 학생 학점 취득 프로그램을 등록했던 학생들의 숫자가 50나 증가했다 같은해 기간동안 Omni 대학에서는 그 전년도부터 기숙사와 학급의 공간 확충을 위한 예산을 대폭 줄였는데 이는 이 원거리 학점 취득 프로그램이 양방향 비디오 컴퓨터 접속을 통해서만 가능한 수업지도 방식이기때문인 것으로 보인다 반면 지난 3개년 동안 Humana 대학에서의 수강률은 감소한데다가 건물

GRE AWA John박 박정어학원

유지비도 올랐다 따라서 Humana대학의 수강을 늘리고 예산손실을 회복하기 위해서는 Omni 대학에서 취한 조치와 같은 능동적인 프로그램을 추진해야 한다

결론 we should initiate and actively promote long-distance degree programs like those at Omni 반박 원거리 학생 취득 프로그램 숫자가 증가한거하고 예산이 줄어드는 것 사이에 연관이 약하다 (causal 학생의 증가로 관리비용 증가할수 있음 원거리 수업가능 장비도입에의한 비용발생)bad analogy(omni university 하고 같은 조건이 아니다 )-gt omni college 가 강좌내용이 좋아서 학생의 등록이 많을수 있다 Humana 대학에서 만들었다 하더라도 인기 없을수 있음다른 요인에 의해서 Humana 대학의 수강 인원이 증가할수 있음(비록 과거엔 인기가 없었을지라도)

The following appeared as part of a business plan developed by the manager of the Rialto Movie Theater

Despite its downtown location the Rialto Movie Theater a local institution for five decades must make big changes or close its doors forever It should follow the example of the new Apex Theater in the mall outside of town When the Apex opened last year it featured a video arcade plush carpeting and seats and a state-of-the-art sound system Furthermore in a recent survey over 85 percent of respondents reported that the high price of newly released movies prevents them from going to the movies more than five times per year Thus if the Rialto intends to hold on to its share of a decreasing pool of moviegoers it must offer the same features as Apex

Write a response in which you discuss what questions would need to be answered in order to decide whether the recommendation is likely to have the predicted result Be sure to explain how the answers to these questions would help to evaluate the recommendation

Before following through this business plan the manager should investigate the cause of Rialtorsquos unsuccessful business

The author provides no evidence that the surveyrsquos results are statistically reliable The surveyrsquos sample of 85 percent must be sufficient in size and representative of overall population of the city where Rialto and Apex is serving Lacking evidence of a sufficiently representative sample the author cannot justifiably rely on the survey to draw any conclusion whatsoever The author does not indicate that Apex is indeed currently successful However even if Apex is enjoying success the argument relies on what might be a false analogy between Rialto and Apex In order for Apex to serve as a model that Rialto should emulate the author must assume that all relevant circumstances are essentially the same However this assumption is unwarranted For example the argument overlooks the face that Apex is located in a strategic placemdashbeside a mall where customers can not only watch a movie but also enjoy shopping Therefore simply changing the facility to that of Apex may not lead to success

The author does not mention whether Apex is successful or not Nevertheless even if Apex is currently successful the argument relies on what might be a false analogy between Rialto and Apex In order for Apex to serve as a model that Rialto should emulate the author must assume that all relevant circumstances are essentially the same However this assumption is unwarranted For example the argument overlooks the fact that these two institutions are located in different locations Rialto in downtown and Apex in a mall outside of town Although Apex opened with state-of-the-art facilities the decisive factor in its success could be due to its strategic location of being in a mall People could enjoy both shopping and movies at one location thus they may prefer Apex over Rialto Furthermore the place where people enjoy leisure activities has shifted in the past decades for most cities from downtown to the suburbs Therefore Rialto may not be successful even if it emulates Apexrsquos facilities A better business plan may be relocating Apex to the thriving section of the downtown

Rialto 극장은 지난 50여년간 지역 회관으로써 시내에 위치해 있으면서도 이제 변화를 꾀하지 않으면 문을 닫을

GRE AWA John박 박정어학원

판이다 이 극장은 시외 쇼핑타운에 새로 들어선 Apex 극장의 사례를 본받아야 했다 Apex가 지난해 개업했을 당시 이 극장은 비디오 아케이드 플러쉬 카펫트 바닥과 좌석 그리고 최신 음향시설을 갖추었다 더군다나 최근 조사에서는 응답자의 85 이상이 새로 출시된 영화 입장료가 비싼 탓으로 지난해보다 5배이상의 관람객이 줄어들었다고 나타났다 따라서 Rialto 극장이 줄어들고 있는 관람객을 뺐기지 않고 유지하려면 Apex와 같은 시설들을 갖추어야 할 것이다주장 리알토 극장이 줄어들고 있는 관람객을 뺐기지 않고 유지하려면 Apex와 같은 시설들을 갖추어야 할 것이다

1 조사에서 응답자가 전체를 대표할 수 없다 2 apex 극장이 좋은 시설을 갖추고 있지만 그로 인해 수익이 많이 발생했다는 말이 없으므로 시설투자를

하고도 좋은 결과를 얻을 수 있을지 그 근거가 미흡하다3 좋은 영화가 출시된다면 입장료가 비싸도 영화관에서 꼭 보려고 할 수 있다 4 rialto 가 시설이 아닌 다른 요인에 의해 장사가 안될수도 있다( 우범 지역이라든지)

The following is a recommendation from the business manager of Monarch Books

Since its opening in Collegeville twenty years ago Monarch Books has developed a large customer base due to its reader-friendly atmosphere and wide selection of books on all subjects Last month Book and Bean a combination bookstore and coffee shop announced its intention to open a Collegeville store Monarch Books should open its own in-store cafeacute in the space currently devoted to childrens books Given recent national census data indicating a significant decline in the percentage of the population under age ten sales of childrens books are likely to decline By replacing its childrens books section with a cafeacute Monarch Books can increase profits and ward off competition from Book and Bean

Write a response in which you examine the stated andor unstated assumptions of the argument Be sure to explain how the argument depends on these assumptions and what the implications are for the argument if the assumptions prove unwarranted

The following is a recommendation from the business manager of Monarch Books

Since its opening in Collegeville twenty years ago Monarch Books has developed a large customer base due to its reader-friendly atmosphere and wide selection of books on all subjects Last month Book and Bean a combination bookstore and coffee shop announced its intention to open a Collegeville store Monarch Books should open its own in-store cafeacute in the space currently devoted to childrens books Given recent national census data indicating a significant decline in the percentage of the population under age ten sales of childrens books are likely to decline By replacing its childrens books section with a cafeacute Monarch Books can increase profits and ward off competition from Book and Bean

1 Write a response in which you discuss what questions would need to be answered in order to decide whether the recommendation is likely to have the predicted result Be sure to explain how the answers to these questions would help to evaluate the recommendation

2 Write a response in which you discuss what specific evidence is needed to evaluate the argument and explain how the evidence would weaken or strengthen the argument

No evidence regarding Monarch Bookrsquos successEven if Regal Bookrsquos is successful this may not be attributable to the cafeacute False analogy Emulating may not lead to success Other factors may be involvedInsufficient condition The national census is not enough evidence that childrenrsquos book sales will decline Can

GRE AWA John박 박정어학원

the national census represent the local child populationDid opening a cafeacute boost sales for Regal Books Even assuming Regal is successful by opening a cafeacute this may not be suitable for Monarch which plans to close the childrenrsquos book section to establish a cafe Imprecise language ldquorelatively little spacerdquo how smallThe managerrsquos recommendation contradicts what he says Since Monarch is popular for its wide selection of books closing a selection which targets a major group of readers may hurt Monarchrsquos salesIs this the best way to compete

When Stanley Park first opened it was the largest most heavily used public park in town It is still the largest park but it is no longer heavily used Video cameras mounted in the parks parking lots last month revealed the parks drop in popularity the recordings showed an average of only 50 cars per day In contrast tiny Carlton Park in the heart of the business district is visited by more than 150 people on a typical weekday An obvious difference is that Carlton Park unlike Stanley Park provides ample seating Thus if Stanley Park is ever to be as popular with our citizens as Carlton Park the town will obviously need to provide more benches thereby converting some of the unused open areas into spaces suitable for socializing

Write a response in which you examine the stated andor unstated assumptions of the argument Be sure to explain how the argument depends on these assumptions and what the implications are for the argument if the assumptions prove unwarranted

Stanley 파크가 처음 개장했을 당시 가장 크고 가장 많이 이용되는 공원이었다 아직도 공원중에서는 가장 크지만 이용률은 상당히 떨어졌다 지난달 공원 주차장에 설치해놓은 비디오 카메라를 통해 보면 drop(주차장으로 여겨짐) 이용률이 가장 높았다 수치상으로는 하루 평균 50대의 차량만이 이용하였다 반면 직장 중심거리에 위치한 작은 규모의 Carlton 파크는 주당 무려 150여명 이상이 이용하고 있다 Stanley 파크와는 달리 Carlton 파크에는 의자가 있다는 것이 가장 뚜렷한 차이점이다 따라서 Stanley 파크가 Carlton 파크처럼 시민들이 자주 이용하는 공원이 되기 위해서는 벤치를 설치할 필요가 있으며 이렇게 사용되지 않는 일부 공간을 활용해서 사교를 위한 공간으로 바꾸어야 한다 ===gtdrop 에 대한 첨부사항 (영영사전내용입니다)---- a place or central depository to which something (as mail money or stolen property) is brought for distribution or transmission also the act of depositing something at such a place dropgt

주장 if Stanley Park is ever to be as popular with our citizens as is Carlton Park the town will obviously need to provide more benches thereby converting some of the unused open areas into spaces suitable for socializing1 조사가 언제 이루어진 것인가 조사가 언제 실시되었느냐에 따라 결과가 다를 수있다 현재는 다시 스탠리 파크가 늘어났었을 수 있다 2 벤치를 많이 설치했다고 해서 많은 관광객이 오지 않을수 있다(사람들이 벤치나 사교 공간을 원한다는 어떠한 자료도 없다)3스탠리 파크 주변에 교통 상황이 악화가 되었거나 칼튼 파크에서 문화행사등을 많이 가져서 이용객이 줄어든것일 수도 있다 4 칼튼 파크가 중심지에 있어서 접근성이 좋을수 있다5 조사가 같은 시간을 기준으로 한게 아니다(하나는 주중이고 하나는 주말이다)6사람의 수와 차의 대수를 같은것으로 비교할수 없다 (차안에 몇 명이 타고 있는지 모르고 대중교통을 이용해서 왔을수도 있다)

Page 2: GRE writing argument brain storm

GRE AWA John박 박정어학원

관련한 계약을 했으나 지난달 이곳의 식료품중 2만불어치 이상이 병균침해로 폐기되었다는 것을 알았다 그러는 동안 Buzzoff Pest-Control 회사는 우리 회사에서도 수년간 이곳회사의 제품을 사용해 온 회사로써 Wintervale에 있는 저장고에서 같은 서비스를 시작했는데 지난달 역시 1만불어치의 식료품이 폐기처분되었다 Fly-Away회사와 계약한 금액이 상당히 적기는 했으나 손실을 줄이려면 Buzzoff사에게 이 일을 맡겨야 한다주장 손실을 줄이려면 buzzoff사에 pest-control service를 맡겨야 한다

1 bad analogy 두 상황이 다르다 단순히 수치로 비교할 수없다 다른 요인 작용가능(지역도 다름)2 사업은 많은 데이터를 가지고 신중히 결정을 내려야하는데 단순한 하나의 사건의 결론만을 놓고

판단하는 것은 잘못된 결론을 가져올 수 있다(물류비용 등등) 3 이번에 손실이 적었다고 앞으로도 계속 적을 것이라고 볼 수만은 없다

In the memo the vice president of a food-distribution concludes that in order to save their companyrsquos money they should return to Buzzoff Company of all their pest-control services To support this argument the vice president of a food-distribution should provide objective information that proves both cities were in the same environment The vice president of a food distribution relies on unsubstantiated assumption is therefore unconvincing as it stands

First of all the vice president of the food distribution provides no information about the weather in two cites It is quite possible that the fast-food warehouse in Palm City had been destroyed by pest damage because of bad weather condition and the warehouse in Wintervale had been less destroyed by pest damage because of good weather condition Therefore without considering the weather condition in two the cites the vice presidentrsquos argument is not reliable

Second of all the vice president of the food distribution unfairly assumes that both food conditions were exactly the same condition however it is possible that the foodrsquos condition in warehouse in Palm City was originally worse than the one in Wintervale Food in bad condition quickly influences other food around it so in this case it is not all The Fly-Away Pest-Control Companyrsquos fault to make over $20000 worth of food damage

Third of all the vice president of the food distribution provides no information about how much each company charged for the pest-control service It is possible that the total price including the worth of food destroyed by pest damage in Palm City and the pest-control service is less expensive than paying for Buzzoff Company In this case Buzzoff Company charges more money therefore the company could not save their money

Fourth of all it is also possible that Palm City is naturally or geographically vulnerable to pest damage In this case it does not matter which company they hire If they just hire Buzzoff Company for the pest-control service the result would not be different Moreover they would be spending more than hiring The Fly-Away Pest-Control Company

In short this memo fails to provide the key evidence need to support its claims To make the claim stronger the vice president of a food distribution should provide information about weather in two cites and the original food conditions The vice president of the food distribution also needs to provide information about the price charged by the two companies and natural and geographical information With providing that information the vice president of a food distributionrsquos argument would be more convincing

Good transitions and organization Work on smoothening awkward sentence structuresScore 40

Hospital statistics regarding people who go to the emergency room after roller-skating accidents indicate the need for more protective equipment Within that group of people 75 percent of those who had accidents in streets or parking lots had not been wearing any protective clothing (helmets knee pads etc) or any light-reflecting material (clip-on lights glow-in-the-dark wrist pads etc) Clearly the statistics indicate that by investing in high-quality protective gear and reflective equipment roller skaters will greatly reduce their risk of being severely injured in an accident

Write a response in which you examine the stated andor unstated assumptions of the argument Be

GRE AWA John박 박정어학원

sure to explain how the argument depends on these assumptions and what the implications are for the argument if the assumptions prove unwarranted

Assumptions The author is assuming that 75 percent is a significant number protective clothing and light reflecting material mitigate severe injuries (the cautious nature of those who wear such equipment might have prevented accidents or the skills and reflex of the roller skaters drivers are the main reason for accidents)

ETS Essay Response ndash Score 5

The argument presented is limited but useful It indicates a possible relationship between a high percentageof accidents and a lack of protective equipment The statistics cited compel a further investigation of theusefulness of protective gear in preventing or mitigating roller-skating related injuries However theconclusion that protective gear and reflective equipment would greatly reducerisk of being severelyinjured is premature Data is lacking with reference to the total population of skaters and the relativelevels of experience skill and physical coordination of that population It is entirely possible that furtherresearch would indicate that most serious injury is averted by the skaters ability to react quickly andskillfully in emergency situationsAnother area of investigation necessary before conclusions can be reached is identification of the types ofinjuries that occur and the various causes of those injuries The article fails to identify the most prevalenttypes of roller-skating related injuries It also fails to correlate the absence of protective gear and reflectiveequipment to those injuries For example if the majority of injuries are skin abrasions and closed-headinjuries then a case can be made for the usefulness of protective clothing mentioned Likewise if injuriesare caused by collision with vehicles (eg bicycles cars) or pedestrians then light-reflective equipmentmight mitigate the occurences However if the primary types of injuries are soft-tissue injuries such astorn ligaments and muscles back injuries and the like then a greater case could be made for training andexperience as preventative measures

Reader Commentary for Essay Response ndash Score 5This strong response gets right to the work of critiquing the argument observing that it indicates apossible relationship but that its conclusion is premature It raises three central questions that ifanswered might undermine the soundness of the argumentbull What are the characteristics of the total population of skatersbull What is the usefulness of protective or reflective gear in preventing or mitigating roller skatingrelatedinjuriesbull What are the types of injuries sustained and their causesThe writer develops each of these questions by considering possible answers that would either strengthen orweaken the argument The paper does not analyze the argument as insightfully or develop the critique asfully as required for a 6 paper but the clear organization strong control of language and substantial degreeof development warrant more than a score of 4

Milk and dairy products are rich in vitamin D and calcium substances essential for building and maintaining bones Many people therefore believe that a diet rich in dairy products can help prevent osteoporosis a disease in which the bones weaken significantly with age and that is linked to both environmental and genetic factors But a long-term study of a large number of people has found that those who have consistently consumed dairy products throughout the years of the study have a higher rate of bone fractures than any other participants in the study Since bone fractures are a symptom of osteoporosis this study result shows that a diet rich in dairy products may actually increase rather than decrease the risk of osteoporosis

Write a response in which you discuss what specific evidence is needed to evaluate the argument and explain how the evidence would weaken or strengthen the argument

Proven fact vitamin D and calcium in milk help prevent osteoporosis

GRE AWA John박 박정어학원

Challenge long-term study of a lot of people consistently consumed dairy products throughout the years=gt higher rate of bone fractures than other participantsThus bone fractures(symptom of osteoporosis) increased by diet rich in dairy products

-The author uses unscientific and imprecise terms in his study to support his claim-Control group-The author tries to refute a well established fact through a single study-Other factors bring about osteoporosis For example occupation hazards genetics dietary habits-Bone fracture could have might have may have been caused by other factors work environment etc and not specifically osteoporosis-The lack of a controlled environment of the study group The researchers only noted the consumption of milk and did not control other variantsmdashlifestyle and diet -The author bases his conclusion only by noting that the study samples consumed milk The researchers only noted the consumption of milk and did not control other variantsmdashlifestyle and diet-Dairy products might have had adverse effects on some of the participants in the study for example lactose intolerance

In this study the author claims that the consumption of milk actually increases the risk of osteoporosisTo support this claim the author refers to a long-term study he conducted ThesisHowever careful scrutiny of this study reveals that it provides little credible support for the authorrsquos claim

First the author states that the bone fractures were caused by osteoporosis However there could be other reasons for the bone fractures For example it is entirely possible that working conditions might have contributed to the bone fracture A person working at a construction has a much higher chance of bone fracture than a person that works in an office The author needs to provide the reasons for the occurrence of the study participantsrsquo bone fractures Thus the author should provide more sufficient evidence that

The author observes a correlation between osteoporosis and bone fracture then concludes that the former is the cause of the latter However the author fails to rule out other possible explanations For example it is entirely possible that working

The authorrsquos study includes an indefinite amount of people whose backgrounds arenrsquot defined genetic(family) history and environment the age at which the participants partook in the study It may be that the participants had genetic predisposition that were more prone to osteoporosis

The author does not sufficiently define what types of people were included in the study Bones are affected as noted in the prompt by various factors genes and environment The author however does not hellipFinally the author asserts his claim definitively While it is a proven fact that dairy products help people maintain healthy bones the author overthrows this long standing fact with a single long-term study that he did 우유와 유지방 제품에는 뼈를 형성시키고 강화시키는데 아주 중요한 역할을 하는 비타임 D와 칼슘이 많다 때문에 많은 사람들이 유지방이 풍부한 식이요법을 하면 환경 및 유전적 요인에 의한 손상과 노화에 의해 뼈가 심각하게 손상을 입는 질병인 오스티오포로시스를 억제하는데 도움이 된다고 믿고 있다 그러나 상당수의 사람을 대상으로 한 장기간 실험을 통해서 볼 때 이들 대상자중 실험 기간동안 유제품을 꾸준히 섭취한 사람들은 그렇지 않은 사람에 비해 훨씬 높은 골절현상을 보였다 이러한 실험결과를 볼 때 골절증상은 오스티오포로시스가 원인이기 때문에 이러한 식이요법이 오스티오포로시스 질병을 줄이기보다는 오히려 증가시킬 것이라는 것을 알수 있다결론 우유와 유지방 제품을 섭취하는 것은 오스티스오포로시스 질병을 줄이기봐는 오히려 증가시킬 것이다

1 실험은 변수에 의해 상당히 다른 결과를 얻을 수 있다 변수(생활 습관 운동)가 효과적으로 통제가 되었나 알 수 없다 상당수의 사람 장기간의 실험이 너무 주관적이다 객관적인 데이터가 주어져야한다

2 상당수와 장기간이 얼마만큼인지 알 수 없다 샘플링이랑 기간등 여러가지로 다르게 적용되었을 수 있다(실험에 충분한 기간과 인원이였는지)

3 골절 증상이 골다공증이 원인이 되어 나타났다고 했는데 그렇지 않을 수 있다다른 원인이 골절증상을 가져온 것일 수 있다

4 한가지 실험가지고 일반화했다 이미 밝혀진 사실을 단 한가지 실험으로 뒤엎었다 두 비교군이 동일 했는지에 대한 자료 없음(한쪽이 원래 뼈가 약했을수도있음)

GRE AWA John박 박정어학원

A recently issued twenty-year study on headaches suffered by the residents of Mentia investigated the possible therapeutic effect of consuming salicylates Salicylates are members of the same chemical family as aspirin a medicine used to treat headaches Although many foods are naturally rich in salicylates food-processing companies also add salicylates to foods as preservatives The twenty-year study found a correlation between the rise in the commercial use of salicylates and a steady decline in the average number of headaches reported by study participants At the time when the study concluded food-processing companies had just discovered that salicylates can also be used as flavor additives for foods and as a result many companies plan to do so Based on these study results some health experts predict that residents of Mentia will suffer even fewer headaches in the future

Write a response in which you discuss what questions would need to be answered in order to decide whether the prediction and the argument on which it is based are reasonable Be sure to explain how the answers to these questions would help to evaluate the prediction

In this study the author asserts that the number of headaches suffered by the average citizen of Mentia will steadily decline though the use of salicylates as flavor additives To support this assertion the author provides several evidences to support his claim However his argument is specious

Unscientific data correlation the specific amount used as preservatives Other factors could have caused the decline in headaches20 year study Was it a controlled study Lack of controlled environment exposes the participants to other factors

1) The decline in the average number of headaches does not necessarily substantiate that it was indeed caused by the commercial use of salicylates It is entirely possible that other factors are responsible for the decline Perhaps other substances in the food remedied the headaches of the participants Or the decline in headache may have been caused by other dietary consumption or lifestyle habits like exercising Accordingly if these are true the author cannot draw any firm conclusion that the commercial use of salicylates caused the decline in the average number of headaches

살리실산염은 아스피린과 같은 동일 화학물질 계열로써 두통을 치료하는데 사용되는 약물이다 많은 음식에 이 살리실산염이 다량 함유되어 있지만 지난 수십년간 식품가공 업체들은 가공식품 부패방지용으로 살리실산염을 첨가해왔다 이러한 살리실산염의 산업용 사용의 증가는 본 연구원들에 의해 보고된 평균 두통 횟수를 꾸준히 감소시키는데 일조한것으로 이해되어 왔다 최근 식품가공 회사들은 이 살리실산염이 맛을 내는 첨가물로 사용될 수 있음을 발견했다 이러한 새로운 용도로 Mentia에 사는 주민들이 겪고 있는 두통의 평균 횟수를 앞으로도 계속 줄여나갈 수 있을 것으로 보인다결론 살리산염의 사용증가는 주민들의 두통 횟수를 계속 줄여줄 것이다

가정 1 조사받은 사람들은 단지 Salicylate에 의해 headache가 감소된 것이다 -gt 오류 다른 노력을 기울였을 수 있다 가정 2 나중에 Salicylate의 사용증가가 headache의 감소를 가져올 것이다 -gt 오류 또다른 원인으로 얼마든지 headache는 증가가능하다가정 3 음식물에서 headache 감소 원인은 Salicylate 이다 -gt 오류 또다른 성분이 있었을 수 있다가정 4 Flavor additive 로 사용된 Salicylate도 headache를 줄일것이다 -gt 오류 Flavor additive로 사용시 같은 효과를 가질지 밝혀진바 없다 상업적으로 사용되었다고 다 먹었을까

The following appeared as part of an article in a business magazine

GRE AWA John박 박정어학원

A recent study rating 300 male and female Mentian advertising executives according to the average number of hours they sleep per night showed an association between the amount of sleep the executives need and the success of their firms Of the advertising firms studied those whose executives reported needing no more than 6 hours of sleep per night had higher profit margins and faster growth These results suggest that if a business wants to prosper it should hire only people who need less than 6 hours of sleep per night

Write a response in which you examine the stated andor unstated assumptions of the argument Be sure to explain how the argument depends on these assumptions and what the implications are for the argument if the assumptions prove unwarranted

Assumptions 1 Correlation of hours of sleep with success The author assumes that the executives used their wake-up hours on work Could there be other factors 2 Executives versus employeesmdashcriteria for hiring is too simpleminded 3 Characteristics of the 300 male and female executives 4 The study was about advertising executives but the author applies this case to all businesses 5 The author should consider long-term stability rather that sharp growth and profit 6 Average number of hours of sleep is insufficient to support the authorrsquos recommendation 7 The author assumes that this lifestyle pattern will remain consistent

The following appeared in a memo from the president of a company that makes breakfast cereals

In a recent study subjects who ate soybeans at least five times per week had significantly lower cholesterol levels than subjects who ate no soy products By fortifying our Wheat-O cereal with soy protein we can increase sales by appealing to additional consumers who are concerned about their health This new version of Wheat-O should increase company profits and at the same time improve the health of our customers

Write a response in which you examine the stated andor unstated assumptions of the argument Be sure to explain how the argument depends on these assumptions and what the implications are for the argument if the assumptions prove unwarranted

최근의 조사에서 일주에 최소 5회 정도 콩을 먹었던 사람들은 전혀 먹지 않았던 사람들에 비해 콜레스테롤이 상당히 낮은것으로 조사되었다 따라서 Wheat-O 시리얼에 콩 단백질의 함유를 강화시킴으로써 건강에 관심이 있는 더 많은 소비자를 상대로 매출을 올릴 수 있을 것이다 이 신제품이 회사 수입 증대를 가져오는 것 뿐만아니라 소비자의 건강을 향상시킬 수 있다

GRE AWA John박 박정어학원

주장 By increasing our Wheat-O cereal with soy protein we can multiply sales

1 콩 단백질의 함유를 강화해도 그수치가 미흡하거나 흡수가 잘 안되거나 할 수 있다 다른 성분과 섞여서 만들어지는 것이므로 다른 성분에 의해서 그 흡수가 잘 안되거나 효과가 낮을 수 있다 2 강화한 것이 건강에 관심이 있는 소비자에게 어필한다고 해도 그 수가 적어서 매출에 큰 영향을 안줄 수 있다3 다른 음식에 의해서 콜레스테롤이 더 낮아질수 있다 4 콜레스테롤을 낮추는 것이 사람들에게 그들의 건강생활을 위해 크게 어필 안할수도 있다( 저지방식이라든지 다른 건강을 높이는 방법이 더 관심이 많을수 있다)

In a study of the reading habits of Waymarsh citizens conducted by the University of Waymarsh most respondents said that they preferred literary classics as reading material However a second study conducted by the same researchers found that the type of book most frequently checked out of each of the public libraries in Waymarsh was the mystery novel Therefore it can be concluded that the respondents in the first study had misrepresented their reading habits

Write a response in which you discuss what specific evidence is needed to evaluate the argument and explain how the evidence would weaken or strengthen the argument

Evidence needed -Information about the respondentsmdashage gender class precise number of respondents-Time-shift After how many years was the second study conductedmdashtaste in books may change over time-Can library records sufficiently evidence the reading habits of Waymarsh citizens

1)2)3)4)5)6) First the author provides no evidence that the surveyrsquos studyrsquos results are statistically reliable Lacking

information about the precise methodology of the study the number of (customers)respondents surveyed and the number of respondentsvarious information such as gender age and social background which are essential to bolster the conclusion it is impossible to assess the validity of the results It is possible that people who feel inclined to( take low fat and low cholesterol foods ) read literary classics were more willing to respond to the survey than were others Another problem is the representativeness of the respondents Were they representative of all the customersWaymarsh citizens Were they chosen for the survey randomly Lastly the survey results must depend on the honesty and integrity of the respondents Without more information about the survey the author cannot simply conclude that (most of customers want to take low fat and low cholesterol foods)Waymarsh citizens misrepresented their reading habit on the first study on the basis of this surveystudy(survey reliability)

The exact number of books checked-out should be notedMystery novels could have been in trend at the time of the studyPublic libraries vs other libraries such as Leeville University library

Leeville 대학에서 실시한 Leeville 주민의 독서습관에 대한 조사에서 대부분의 응답자는 독서용으로 문학작품을 선호한다고 응답했다 그러나 같은 조사팀에서 이어 실시한 조사에서는 Leeville에 있는 도서관마다 가장 빈번하게 연람된 책의 종류를 보면 미스테리 소설이었던 것으로 조사되었다 따라서 최초 조사 응답자들이 독서 습관에 대해 잘못 말했다고 결론지을 수 있다

결론 it can be concluded that the respondents in the first study had misrepresented their reading habits1 첫번째 조사 그룹하고 두번째 조사 그룹하고 다른 그룹일 수 있다

GRE AWA John박 박정어학원

2 도서관에 비치된 책이 미스터리 소설이 더 많아서 선택의 여지가 없을 수 있다

High Frequency Group 2 National-gtLocal Local-gtNational amp Group Member Error

The following appeared in a letter to the editor of a Batavia newspaper

The department of agriculture in Batavia reports that the number of dairy farms throughout the country is now 25 percent greater than it was 10 years ago During this same time period however the price of milk at the local Excello Food Market has increased from $150 to over $300 per gallon To prevent farmers from continuing to receive excessive profits on an apparently increased supply of milk the Batavia government should begin to regulate retail milk prices Such regulation is necessary to ensure fair prices for consumers

Write a response in which you discuss what questions would need to be answered in order to decide whether the recommendation is likely to have the predicted result Be sure to explain how the answers to these questions would help to evaluate the recommendation

First the author points to the fact that hellip25 increase of dairy farms may be(could be might be) due to population increaseThe milk price increase could be reflecting the increase in cost of livingThe actual price of milk might be cheaper if the economy is experiencing inflationOne market Excello Food cannot reflect the price increase of every market in BataviaThe author claims that farmers are receiving excessive profits however this might not be the caseThe author claims that by regulating milk prices the government will successfully ensure both lower prices and an adequate supply of milk for consumers However this might not be the best solutionThe writer adduces the information released by the department of agriculture that the number of dairy farms has

increased by 25 percent over the last 10 years and thus he claims that dairy farmers are earning excessive profits from milk However the information is filled with loops and holes which need to be filled First what percentage of the milk are sold as milk

Are the dairy farms processing the milk into other dairy products Or are they selling them as milkmdashWhat percentage of the milk is sold as milk Could this be a natural increase

Can the price at the local Excello Food Market represent the price of milk in stores throughout all of Batavia

Are the farmers receiving excessive profits What is the cost of producing milk Could such regulation offer fair prices to consumers

Topic SentenceTo begin with the author assumes that the price of Excello Food Market which has increased from $150 to over $300 per gallon can be applied to the whole nation of Batavia however the author fails to provide evidence to substantiate this assumptionExampleFirstIn additionTherefore in order to make the argument more reliable the author should consider the prices of a large number markets all over the countryThere could be an economic inflation

GRE AWA John박 박정어학원

ldquoBatavia 지역의 농림부의 보고에 따르면 전국적으로 낙농업자의 숫자가 과거 10 년전에 비해 25나 증가했다고 합니다 그러나 동기간 동안 이 지역에 있는 Excello Food Market 에서 우유의 가격은 갤런당 15 불에서 3 불로 증가했습니다 증가된 우유 공급량에 맞춘 낙농업자들의 수익률 증대를 보장하기 위해서는 주 정부가 소매 공급가에 대한 규제를 해야 합니다 소비자에게 보다 저렴한 가격에 안정된 물량을 공급할 수 있도록 하기 위해서는 이러한 조치가 필요합니다

결론 the Batavia government should begin to regulate retail milk prices

반박 ( Excello Food Market doesnrsquot reflect the whole of Batavia) 우유값 증가가 단순히 inflation 을 따라간것일수도 있다 가격상승이 수익을 보장하지 않는다( 원자재 상승등으로 인해서)우유값 규제만이 최선의 방법이 아니다( 물량 조절)낙농업자의 수익보호도 필요하다

A recent sales study indicates that consumption of seafood dishes in Bay City restaurants has increased by 30 percent during the past five years Yet there are no currently operating city restaurants whose specialty is seafood Moreover the majority of families in Bay City are two-income families and a nationwide study has shown that such families eat significantly fewer home-cooked meals than they did a decade ago but at the same time express more concern about healthful eating Therefore the new Captain Seafood restaurant that specializes in seafood should be quite popular and profitable

Write a response in which you discuss what questions would need to be addressed in order to decide whether the conclusion and the argument on which it is based are reasonable Be sure to explain how the answers to the questions would help to evaluate the conclusion

BAY CITY The name implies that this is a port city which would have traditionally consumed seafood 30--what type of seafood Unprocessed or processed cans of tuna and sardines If the latter increased Captain Seafood may not profit 30--natural inflation Are the non-seafood restaurants selling seafood If yes this may suffice since patrons would not easily dine at an unfamiliar restaurant Nationwide studyrsquos representativeness Desire does not lead to action

Nationwide study applied to Bay City Eating fewer home cooked meals than they did a decade agoHealthy food The author needs to show that 30 -gt population increaseOther foods may be more popularCurrent number of restaurants that serve seafood might sufficeWhat kind of healthy food are they interested in Surely not just seafoodThe locals may still patronize the current restaurants that serve seafood dishes

lt national-gt local에 적용 local-gtnational에 적용gt1) The author assumes that the nationwide trend reflects the general trend upon which the argument relies

Yet the author fails to provide evidence to substantiate this crucial assumption The nationwide trend of two income families eating fewer home-cooked and concern for health my not reflect the lifestyle and interests of Bay City citizens Therefore to make his argument stronger the author needs to provide evidence that two income families of Bay City maintains the nationwide trend

GRE AWA John박 박정어학원

최근의 매상에 관한 조사에서 Bay 시에 있는 레스토랑의 해산물 요리의 소비가 지난 5년동안 30 증가했다고 보고되었다 그러나 현재 해산물을 전문으로 취급하고 있는 레스토랑이 없다 더군다나 대다수의 가정이 맞벌이 가정인데다가 전국 조사에서도 나와있듯이 이러한 맞벌이 가정은 10년전의 가정에서 했듯이 집에서 식사를 챙겨먹는 것이 급격하게 줄어 들었고 동시에 건강식과 관련한 지출이 늘고 있다 따라서 해산물 전문 레스토랑이 아주 인기를 끌것이고 그만큼 수익도 많을 것이다

1 해산물이 건강식인지에 대한 언급이 없다2 외식하는데 해산물 요리만 먹지 않을 것이다 집에서 챙겨먹지 않는다고 해산물을 먹는다고 할 수 없다3 현재 해산물 취급하는 식당이 현재까지 없다고 해서 계속 없을 수만은 없다 수익이 만약 늘어난다면

Bay시 주변의 도시의 레스토랑이 체인점을 내거나 새로운 식당이 생길 수 있고 오히려 수익이 줄어들 수도 있다

Scientists studying historical weather patterns have discovered that in the mid-sixth century Earth suddenly became significantly cooler Although few historical records survive from that time some accounts found both in Asia and Europe mention a dimming of the sun and extremely cold temperatures Either a huge volcanic eruption or a large meteorite colliding with Earth could have created a large dust cloud throughout Earths atmosphere that would have been capable of blocking enough sunlight to lower global temperatures significantly A large meteorite collision however would probably create a sudden bright flash of light and no extant historical records of the time mention such a flash Some surviving Asian historical records of the time however mention a loud boom that would be consistent with a volcanic eruption Therefore the cooling was probably caused by a volcanic eruption

Write a response in which you discuss what questions would need to be addressed in order to decide whether the conclusion and the argument on which it is based are reasonable Be sure to explain how the answers to the questions would help to evaluate the conclusion

Historical records may not be enough to explain the global cooling phenomenonThere may be reasons other than the two hypothesis to explain the global coolingA volcanic eruption big enough to produce dust clouds that envelop the earth to cause global cooling would have countless recordsThe author argues that a large meteorite collision is not a feasible explanation for the global cooling because there are no historical records that indicate a flash The authorrsquos logic is flawed in eliminating the meteorite hypothesis by claiming that there was no historical record of a bright flash of light According to common sense a meteorite collision big enough to produce dust that could envelop the earth would result in catastrophe even to the extinction of major species of animals

Loud boom would be insufficient evidence to conclude it was a volcanic eruption If could hear a loud boom there should be records of a volcanic eruption because the author claims that the dust cloud from this gigantic eruption was significant enough to cause global coolingCooling could have been caused by factors besides volcanic eruption and meteor collisionThe absence of historical records that indicate a bright flash of light Collision could have occurred at a place where there no witnesses Could have fell out in the seaBoom might have been caused by things other than a volcanic eruptionHistorical records from Asia and Europe may not be applicable to earth as a whole

과거의 날씨 주기를 연구하는 과학자들은 6세기 중엽 지구가 갑자기 혹한기가 되었던 것을 알게되었다 이 당시의 몇몇 기록들이 아직도 남아있긴 하지만 아시아와 유럽에서 발견되는 몇가지 자료에서 태양 빛의 수축과 그에따른 엄청난 추위가 있었음을 언급하고 있다 거대한 화산 폭발이나 거대 운석의 지구 충돌중 어느것에

GRE AWA John박 박정어학원

의해서든 엄청난 먼지구름을 형성시켜 지구전체에 태양빛을 차단해서 지구의 기온을 뚝 떨어뜨릴수 있을 수도 있다 그러나 이러한 운석 충돌은 순간 섬광을 발산시키게 될 것인데 이 시기의 어느 기록에서도 이러한 섬광은 언급되지 않고 있다 그러나 이 시기에 남아 있는 일부 아시아의 기록문서에서는 연속적으로 화산폭발음일수 있는 엄청난 폭발음이 있었음을 기록하고 있다 따라서 이러한 혹한은 아마도 화산 폭발에 의한 것이었을 것이다

결론 the cooling was probably caused by a volcanic eruption

반박 자료의 부족( 몇몇 자료가지고 그 당시의 기후를 예상하기에는 무리가 있다)다른 원인에 의해서 기후가 떨어졌을수도 있다 (빙하기 다른 기상원인)기록되지 않았다고 해서 그 사실이 없었던 것은 아니다(기록은 했으나 자료가 없어졌을수도 있고 모든 사실이 전부다 기록되지는 않는다 )기록된 폭발음이 꼭 확산 폭발음이 아닐수도 있다( 화산 폭발음이 너무 먼거리여서 들리지 않을수도 있고 다른 소리가 기록된 것이였을수도 있다)부족한 자료를 신빙할수 있는가 기록의 소멸도 예상 할수 있다 실제로 운석이 떨어졌는데 안들렸을수 있다

The following appeared in a memo from the president of Bower Builders a company that constructs new homes

A nationwide survey reveals that the two most-desired home features are a large family room and a large well-appointed kitchen A number of homes in our area built by our competitor Domus Construction have such features and have sold much faster and at significantly higher prices than the national average To boost sales and profits we should increase the size of the family rooms and kitchens in all the homes we build and should make state-of-the-art kitchens a standard feature Moreover our larger family rooms and kitchens can come at the expense of the dining room since many of our recent buyers say they do not need a separate dining room for family meals

Write a response in which you examine the stated andor unstated assumptions of the argument Be sure to explain how the argument depends on these assumptions and what the implications are for the argument if the assumptions prove unwarranted

The presidentrsquos first unstated but apparent assumption is that a nationwide survey can be applied to local areas There is no guarantee that the area in which Bower Builders builds houses will follow the national consumer trend Since he only mentions the overall result of the nationwide survey it is difficult to determine more specific conditions such whether people in urban and rural areas have the same housing preferences Such uncertainty is further exacerbated by the nature of the survey question itselfmdash it does not deal with current trends in actual home purchases but with desired home features The naiumlve assumption that consumer desire will directly result in consumer action underlies the presidentrsquos interpretation and application of the survey results to his company plan Not everyone who wishes for a large family room and kitchen will or can actually buy a house with those features mainly due to financial reasons Furthermore the president also assumes for no evident reason that consumers will not hesitate to purchase houses with state-of-the-art kitchens as a standard rather than optional feature when it is clear that such a feature will raise the overall cost significantly He continues to blunder in his mistaken assumptions about consumer behavior in assuming that the taste of recent buyers can represent the concern of future prospective buyers The fact that recent buyers have claimed no need for separate dining rooms does not mean future buyers will feel the same way as well

The president of Bower Builders recommends that in order to raise company profits the company should build houses with large family rooms and large state-of-the-art kitchens He uses the results of a nationwide survey on desired home features and the example of Bower Buildersrsquo competitor Domus Construction to support his plan His memo manifests several assumptions about surveys consumer behavior and Domus Constructionrsquos houses that do not necessarily bolsterbuttress his argument

GRE AWA John박 박정어학원

The writer assumes 1 the nationwide survey results can be applied to the local area (Desire cannot reflect consumer trend) 2 Domus Construction was profitable because they had such features (The writer should investigate what other features the DC provides and the actual number of homes sold) 3 State-of-the-art kitchens will sell well (no evidence to buttress his assumption furthermore they would need to spend more money which would increase the price of the housesmdashtheir market will be limited to the affluent) 4 The recent buyers represent the concern of most buyersmdashthat they are ok with a house that is without a dining room

Nationwide survey might not be applicable to this regionDomus Construction could have other superior qualities than Bower Builders Ex DesignSelling fast doesnrsquot indicate quantitySmall yards complaints could be voiced in the future

First the author concludes that a nationwide survey reveals that the two most desired home features are a bathroom with a whirlpool tub and a large kitchen However this nationwide survey may not reflect the trends of the customers that Bower Builders target The author assumes that the nationwide trend reflects customer trends The national study would lead support only if the nationwide trend reflect the home-purchasing trends The nationwide trend may just be a trend of desire which does not result in the act of purchasing However the author does not provide credible evidence that this is the case

전국 조사에서 가장 인기있는 집안 구조 2가지는 훨풀 튜브가 마련된 욕실과 커다른 부엌으로 조사되었다 경쟁업체 Domus Construction이 신축한 완공예정인 주택은 이러한 욕실을 갖추고 있어서 분양이 조기에 이루어지고 있고 가격도 평균치보다 상당히 웃돌고 있다 우리도 집을 많이 팔도 그만큼 이윤을 많이 남기려면 신규 주택에는 위의 2가지 사항을 갖추어야 한다 최근 우리가 지은 주택 수요자들이 적은 평수에 대해서는 별다른 불만사항이 없었기 때문에 앞으로 평수를 더 줄여서 이익을 극대화 할 수 있을 것이다

주장 We should include whirlpool tubs and a large kitchen

1 경쟁업체 Domus Construction이 신축한 완공예정인 주택은 이러한 욕실을 갖추고 있어서라기 보다는 위치가 좋거나 다른 마감재(construction material)나 인테리어가 뛰어나서 분양이 조기에 이루어진것이라고 볼 수 있다 2 전국조사가 건물이 지어지는 지역에 항상 적용되리라고 볼 수 없다 3 평수를 줄이는 것에 불만이 없었다는 것은 editor의 견해일 수 있다 사람들이 표현하지 않은 것일 수도 있고 다른 장점이 많아서 그런 단점을 드러내지 않은 것일 수 있기 때문에 속단할 수 없다 4 다른것을 함으로써 더 잘 팔릴수도 있다

2011 7 19 화요일 GRE AWA 실전반이OO

Argument 94

The president of Bower Builders contends recommends that in order to raise company profits the company should build houses with large family rooms and large state-of-the-art kitchens He uses the results of a nationwide survey on desired home features and the example of Bower Buildersrsquo competitor Domus Construction to support his plan His memo manifests several assumptions about surveys consumer behavior and Domus Constructionrsquos houses that do not

GRE AWA John박 박정어학원

necessarily bolsterbuttress his argumentThe presidentrsquos first unstated but apparent assumption is that a nationwide survey can be

applied to local areas There is no guarantee that the area in which Bower Builders builds houses will follow the national consumer trend Since he only mentions the overall result of the nationwide survey it is difficult to determine more specific conditions such whether people in urban and rural areas have the same housing preferences Such uncertainty is further exacerbated by the nature of the survey question itselfmdash it does not deal with current trends in actual home purchases but with desired home features The naiumlve assumption that consumer desire will directly result in consumer action underlies the presidentrsquos interpretation and application of the survey results to his company plan Not everyone who wishes for a large family room and kitchen will or can actually buy a house with those features mainly due to financial reasons Furthermore the president also assumes for no evident reason that consumers will not hesitate to purchase houses with state-of-the-art kitchens as a standard rather than optional feature when it is clear that such a feature will raise the overall cost significantly He continues to blunder in his mistaken assumptions about consumer behavior in assuming that the taste of recent buyers can represent the concern of future prospective buyers The fact that recent buyers have claimed no need for separate dining rooms does not mean future buyers will feel the same way as well

In addition the president finds a real-life actualization of the nationwide survey results in the recent sales of Domus Construction However he easily assumes that large family rooms and kitchens are the only reasons the houses of Domus Construction sell well He does not take into account other features and selling points of the competitorrsquos houses There may well be other explanations for its houses selling more quickly and expensively including additional home features as well as external factors such as proximity to better schools or superior financial solvency of its clientele If Bower Builders merely added larger family rooms and kitchens without taking into consideration the other factors they may lose rather than gain profits

Clearly the presidentrsquos assertion that Bower Builders make houses with large family rooms and high-tech kitchens at the expense of dining rooms rests on a number of assumptions that are ill-informed and naiumlve If Bower Builders undertakes the proposed plan without further research into local consumer desires purchasing trends and the marketing and sales of competing companies the company will risk losing money by building big new houses that people cannot afford to or will not wish to buy

The following appeared in a letter to the editor of a journal on environmental issues

Over the past year the Crust Copper Company (CCC) has purchased over 10000 square miles of land in the tropical nation of West Fredonia Mining copper on this land will inevitably result in pollution and since West Fredonia is the home of several endangered animal species in environmental disaster But such disasters can be prevented if consumers simply refuse to purchase products that are made with CCCs copper unless the company abandons its mining plans

Write a response in which you examine the stated andor unstated assumptions of the argument Be sure to explain how the argument depends on these assumptions and what the implications are for the argument if the assumptions prove unwarranted

The writer assumes 1 The writer is trying to avoid an inevitability 2 Mining copper will result pollution (they could make preventative measures) 3 The writer may be too late from stopping CCC from developing the area into a copper mine 4 Since mining is an underground enterprise the surface may not be affected that much therefore endangered species may not be affected 4 Consumers wonrsquot buy CCC products if the journal publishes a negative review about CCC (How many readers) CCC could a company that has ties with many IT companies and industries in that their copper is almost ubiquitous in various products

GRE AWA John박 박정어학원

지난 한해동안 CCC(Consolidated Copper 회사)는 서부 플로리다의 열대 지역에 1백만 마일이 넘는 땅을 사들였다 이 곳에서의 채광 활동은 서부 플로리다가 몇몇 멸종 위기에 처한 동물의 서식지이기 때문에 분명 오염과 환경파괴를 가져올 것이다 그러나 이러한 파괴는 CCC 회사가 채광을 포기할 때까지 이 회사가 채굴한 구리로 제조된 제품을 구매하지 않으면 막을 수 있을 것이다

결론 such disaster can be prevented if consumers simply refuse to purchase products that are made with CCCs copper until the company abandons its mining plans

1 구리로 제조된 물건이 생활에 필수적인 것이 많은 만큼 불매가 쉽지 않을 수 있다(전선이나 각종 전자제품에 필수적으로 들어가기 때문에)

2 불매를 유도한다고 해서 소비자들이 구매를 안하는 것은 아니다3 적절한 채굴로 환경파괴를 가져 오지 않을 수 있다 (땅속에 있는 물질을 채굴하는 만큼 생물에 영향을 안

미칠 수도 있다)4 이미 채굴이 다 끝나서 더 이상의 채굴이 없을 수도 있다 5 땅을 구입한다고 해서 채광하는건 아니다 (다른 용도로 샀을수도 있다)

The following is a letter to the editor of an environmental magazine

In 1975 a wildlife census found that there were seven species of amphibians in Xanadu National Park with abundant numbers of each species However in 2002 only four species of amphibians were observed in the park and the numbers of each species were drastically reduced There has been a substantial decline in the numbers of amphibians worldwide and global pollution of water and air is clearly implicated The decline of amphibians in Xanadu National Park however almost certainly has a different cause in 1975 troutmdashwhich are known to eat amphibian eggsmdashwere introduced into the park

Write a response in which you discuss what specific evidence is needed to evaluate the argument and explain how the evidence would weaken or strengthen the argument

Evidence needed the identity of the sender and the census taker of rsquo75 and lsquo02mdasha scientist or an environmentalist credibility issue census methodology vs mere observationmdashthe absence of evidence is not an evidence of absence the season of when the census was taken time-shiftmdashconditions may have changed worldwide decline may include Xanadu other species of predators that prey on amphibians because trout is only one species that prey on amphibians the number of troutmdashhave they increased significantly since rsquo75 First the author needs to be more overt about the credibility of the census and observation There were to accounts that notes the population of amphibiansmdashthe first a census and the second an observation The author needs to bolster his conclusion with the evidence that indicate that the census and second observation were done using scientific methodologies This could either could strengthen or weaken his claim In addition he

GRE AWA John박 박정어학원

needs to provide the specific season of when the census and observation occurred In this letter the writer is informing an editor of an environmental magazine that the number of amphibians was greatly reduced since 1975 and he points out the introduction of trout as the only reason for the decline However the author fails to provide crucial evidences that could strengthen or weaken his conclusion

The following appeared in a memorandum from the president of Hyper-Go Toy Company

Last year sales of our Fierce Fighter toy airplane declined sharply even though the toy had been a top seller for three years Our customer surveys show that parents are now more worried about youthful violence and are concerned about better education for their children Therefore to maintain profits we should discontinue all our action toys and focus exclusively on a new line of educational toys Several other toy companies have already begun marketing educational toys and report sales increases last year of 200 percent And since the average family income is growing sales of new Hyper-Go toys should also increase

Write a response in which you discuss what specific evidence is needed to evaluate the argument and explain how the evidence would weaken or strengthen the argument

First the president indicates that the sales of Fierce Fighter toy airplane declined sharply However he fails to consider the fact that toys are a fad Since FFT enjoyed a three year success it may be natural that the trend would subsideSecond Customer survey is this representative of most toy consumers This trend is not newThird other companies may have profited not through educational but other toys Or educational toy profit may be small in proportion to their sale of other toys 200Parents are not the customers companies should concentrate on what the children want to increase profitLastly parents worry about youthful violence and concern for better education are not new trends

The presidentrsquos decision to discontinue all action toys and focus only on educational toys is too extreme If this decision is taken into effect its procedure may be an onerous task because it would require major shifts in human resources and company image Therefore further data should be considered before following up on this decision

우리 회사에서 지난 3년간 최고 매출을 일으켰던 Fierce Fighter 장난감 비행기가 지난해에는 매출이 급격히 떨어졌다 자체 고객 조사에서는 부모들이 현재 청소년 폭력에 걱정을 하고 있어서 아이들의 양질 교육에 더 관심을 가지고 있는 것으로 나타났다 따라서 회사의 수익을 유지하기 위해서는 모든 자사 전투용 장난감 생산을 중단하고 오로지 교육적인 장난감 생산에 집중해야 한다 몇몇 여타 장난감 회사들도 이미 교육용 장난감 마케팅을 시작해서 지난해에는 200의 매출신장을 가져왔다고 한다 그리고 평균 가계 수입이 점점 늘고 있기 때문에 신형 Hyper-Go 장난감의 매출도 늘어날 것이다

3그룹 Time-shift Error

Woven baskets characterized by a particular distinctive pattern have previously been found only in the immediate vicinity of the prehistoric village of Palea and therefore were believed to have been unique to the Palean people Recently however archaeologists discovered such a Palean basket in Lithos an ancient village across the Brim River from Palea The Brim River is very deep and broad and so the ancient Paleans could only have crossed it by boat but there is no evidence that the Paleans had boats And boats capable of carrying groups of people and cargo were not developed until thousands of years after the Palean people disappeared Moreover Paleans would have had no need to cross the rivermdashthe woods around Palea are full of nuts berries and small game It follows that the so-called Palean baskets were not unique to Palea

GRE AWA John박 박정어학원

Write a response in which you discuss what specific evidence is needed to evaluate the argument and explain how the evidence would weaken or strengthen the argument

RefutationPrehistoric time shift-gtbrim river could have been narrow and shallow or it might have not existed Indigenous patterns may exist in other disconnected remote placesNuts berries small game(hunting) may not have existed or the author should be proved these existed at that time Abundance of resources doesnrsquot support the reason for seclusionBoats not yet found baskets may have been carried across by the river current without the help of a boat Lithos might have crossed the river for commercial purposeOne Palean basket does not substantiate the authorrsquos claimThe absence of evidence is not an evidence of absence

The author assumes without justification that present conditions are the same as at the prehistoric era The author unfairly infers from the presence of Brim River which exist today that it would have existed in the past However the author fails to offer any evidence to substantiate this inference It is very likely that the Brim River might not have existed in prehistoric times or if it did exist may have been shallow and narrow enough for the Paleans to easily cross Any of these scenarios if true would serve to undermine the claim thathelliphelliphelliphelliphellip

First the author claims that the Brim River was very deep and broad so the Paleans could not have crossed it However the author fails to offer any evidence to substantiate that this was true in the prehistoric time of the Paleans For all we know the Brim River might not have existed in the prehistoric era or if it did exist could have been a shallow and narrow river For example scientists believe that thousands of years ago an ice-bridge existed on the Bering Sea connecting Eurasia to what is now North America to explain how the Eskimos and the Asian inhabitants of America came to migrate all over the Americas In this example the absence of evidence is not an evidence of absence Therefore to sufficiently support his claim the author needs to substantiate the fact that the Brim River really did exist and was broad and wide in the Prehistoric Era proving that the Paleans could not have influenced or traded with other groups of people

First the author claims that the Brim River was very deep and broad so the Paleans could not have crossed it However the author fails to offer any evidence to substantiate that this was true in the prehistoric times of the Paleans For all we know the Brim River might not have existed in the prehistoric era or if it did exist could have been a shallow and narrow river For example rivers are created by natural erosion over thousands of years Niagara Falls carved its way from the mouth of Lake Ottawa and created a long river Likewise the current Brim Riverrsquos physical features may not have been wide and deep Therefore to sufficiently support his claim the author needs to substantiate the fact that the Brim River really did exist and was broad and wide in the Prehistoric Era proving that the Paleans could not have influenced or traded with other groups of people

예전에는 실로짠 특이한 무늬 바구니가 Palea의 선사시대 지역의 인근마을에서만 발견되어왔기 때문에 Palea 마을 사람들의 특징이라고 여겨졌었다 그러나 최근들어 고고학자들이 Lithos지역에서 Palean 바구니를 발견하였는데 그 당시 지역은 Brim 강을 가로질러 Palea까지 닿아있었다 이 강은 수심이 아주 깊고 강폭이 넓었으며 때문에 고대의 Palea인들은 배를 이용해서 강을 건널수 있었을 것이다 그러나 이들이 배를 가지고 있었다는 증거는 발견되지 않고 있다 더군다나 이들이 멸명한 이후 수천년이 지난뒤에도 수많은 물자와 사람을 실어 나를수 있는 용적을 가진 배는 개발되지 않았다 이와더불어 Palea인들은 강을 건널필요가 없었는데 그것은 너트나무 장과열매 그리고 작은 사냥감들이 주변숲에 풍부했기때문이다 따라서 Palean 바구니라고 하는 것도 Palea인들만의 전유물이 아니라는 결론을 얻을 수 있다

결론 if follows that the so-called Palean baskets were not unique to Palea이번문제는 굿이 causal Bad analogy 로 구분해서 찾기가 힘드내요 배를 발견했다는 증거가 없는것이다 (앞으로도 발견될수 있음)

GRE AWA John박 박정어학원

계절의 영향으로 겨울에 얼음이 두껍게 언다든지 여름에 가뭄으로 인해서 건널수 있다물자가 풍부한 것이 이동하지 않을 조건이 아니다 다른 것에 의해서 이동가능(의약품등)

Thirteen years ago researchers studied a group of 25 infants who showed signs of mild distress when exposed to unfamiliar stimuli such as an unusual odor or a tape recording of an unknown voice They discovered that these infants were more likely than other infants to have been conceived in early autumn a time when their mothers production of melatonin hormone known to affect some brain functions would naturally increase in response to decreased daylight In a follow-up study conducted earlier this year more than half of these children now teenagers who had shown signs of distress identified themselves as shy Clearly increased levels of melatonin before birth cause shyness during infancy and this shyness continues into later life

Write a response in which you examine the stated andor unstated assumptions of the argument Be sure to explain how the argument depends on these assumptions and what the implications are for the argument if the assumptions prove unwarranted

Any baby exposed to unpleasant stimuli would react in such wayFirst of all the author states 25 infants as his evidence However this research sample is too small to prove his claimSecond the author states that 25 infants were conceived in early autumn which he claims lead to a shy disposition However this is faulty evidenceThird the research study was a long term study done in the span of 13 years However the author only writes about the initial and final stages of the study and leaves out evidences of what could have happened during the 13 years which could be more evidential factors of influenceFourth neither the infantsrsquo genetic predisposition nor their environment were taken into accountFinally the author concludes that his shyness continues into later life (Other factors could alter this disposition epigenetic theory)

13 년전 학자들은 25명의 유아를 대상으로 이상한 냄새나 특이한 소리를 녹음한 테잎등으로 낯선 자극을 주었을때 보이는 미미한 압박감 증상을 조사하였다 이들은 성숙기가 막 지났을 즈음에 보통의 유아들이 비슷한 증상을 보이는 정도 보다는 다소 민감한 반응을 보였는데 이 시기는 아이의 엄마가 뇌의 일부 기능에 영향을 미치는 것으로 알려진 멜라토니아 호르몬을 생산하는 시기로써 이 호르몬은 낯 시간이 짧을때 자연적으로 증가할 수도 있다 금년초에 실시된 추가연구에서 현재 10대로 성장한 당시 조사대상의 절반 이상의 아이들이 부끄럼을 잘타는 것으로 여기고 있었다 따라서 분명한 것은 출산전 멜라토닌 수치의 증가가 유아기에 수줍음 등의 영향을 미치게 되며 이러한 영향이 성장후에도 작용한다는 것이다

주장 Clearly increased levels of melatonin before birth cause shyness during infancy and this shyness continues into later life

1 25명의 아기로 결론 내리기에 샘플이 작다2 과학적 사실들에 대한 명확한 근거가 엇음3 다른 영향을 간과했다(다른 호르몬에 의한 영향 후천적인 성격형성의 영향)

GRE AWA John박 박정어학원

The following is a letter to the editor of the Atticus City newspaper

Former Mayor Durant owes an apology to the city of Atticus Both the damage to the River Bridge which connects Atticus to Hartley and the traffic problems we have long experienced on the bridge were actually caused 20 years ago by Durant After all he is the one who approved the construction of the bridge If he had approved a wider and better-designed bridge on which approximately the same amount of public money would have been spent none of the damage or problems would have occurred Instead the River Bridge has deteriorated far more rapidly over the past 20 years than has the much longer Derby Bridge up the river Even though the winters have been severe in the past several years this is no excuse for the negligence and wastefulness of Durant

Write a response in which you discuss what questions would need to be answered in order to decide whether the recommendation is likely to have the predicted result Be sure to explain how the answers to these questions would help to evaluate the recommendation

전임 시장인 Durant 씨는 Atticus 시에 대해 사과할 의무가 있습니다 Atticus와 Hartley를 잇는 River Bridge 교량에 대한 피해와 이 교량에서 오랫동안 주민들이 겪어오고 있는 교통 혼잡 문제들은 실제로 20년 전부터 시작된 것이었습니다 결정적으로 그가 교량 공사를 허가했던 바로 그 장본인입니다 당시 비슷한 공사비용으로 폭이 더 넓고 튼튼하게 설계된 교량을 허가했다면 이러한 문제나 피해는 발생하지 않았을 겁니다 더군다나 이 다리는 지난 20년 동안 상류에 건설된 훨씬 오래된 Derby 다리보다도 빠르게 부식되어 갔습니다 지난 수년동안 심지어 혹한이 있었다 하더라도 이러한 태만과 국고 손실에 대한 책임을 회피할 길이 없는 것입니다

결론 Former Mayor Durant owes an apology to the city of Atticus

1 디자인이 문제가 아닐수 있다 (디자인은 좋았으나 건설과정에 문제가 있었을 수 있다)2 그 당시의 시예산이 적어서 더 큰 다리를 짓기가 불가능했을 수도 있다3 교통량이 많거나 다른 상황으로 인해서 부식이 빨리 됐을 수 있다4 그 당시에는 최선의 선택이였지만 갑자기 변한 상황에 의해서 이런 문제점들이 발생했을 수 있다

GRE AWA John박 박정어학원

4그룹 거짓인과관계 오류 (False Cause) 빈출

Fifteen years ago Omega University implemented a new procedure that encouraged students to evaluate the teaching effectiveness of all their professors Since that time Omega professors have begun to assign higher grades in their classes and overall student grade averages at Omega have risen by 30 percent Potential employers looking at this dramatic rise in grades believe that grades at Omega are inflated and do not accurately reflect student achievement as a result Omega graduates have not been as successful at getting jobs as have graduates from nearby Alpha University To enable its graduates to secure better jobs Omega University should terminate student evaluation of professors

Write a response in which you discuss what specific evidence is needed to evaluate the argument and explain how the evidence would weaken or strengthen the argument

Omega professor evaluation implemented 15 years ago =gt Omega prof assign higher grades 30Employers believe therersquos grade inflation

Thus unsuccessful employment than AlphaTherefore to secure jobs Omega should end evaluating profs

Specific evidence neededRelationship between higher grades and evaluationRelationship between GPA and unsuccessful employmentAlpharsquos education could just be better than OmegaldquoFifteen years agordquo is a long time other factors could have influenced Why is the inflation a problem just now How much is Alpha better Is the comparison just How much gap is thereOmegarsquos student could just be doing better in their studiesComparison to other universities다른 대안 없나hellip Could Omega alleviate the employment problem by implementing a different procedure or program

15 년전 우리 대학은 학생들로 하여금 교수평가를 하도록 한 새로운 조치를 시행했었습니다 이후 교수들은 자신의 학과 학생들에게 높은 학점을 주었으며 그에따라 학생들의 전체 평점이 30나 올랐습니다 외부의 기업체들은 분명 점수가 지나치게 부풀려졌다고 믿고 있습니다 결국 본 대학 졸업생들이 인근 Alpha 대학의 졸업자들보다 구직률이 떨어지는 이유를 잘 보여주고 있는 것입니다 이를 해결하기 위해 이제부터는 학생들에 의한 교수평가제를 중단해야 합니다

결론 Omega University should now terminate student evaluation of professors

반박 교수 평가와 학점 인플레의 연관성이 적다( 교수 평가를 먼저하고 학점을 나중에 매길수도 있다)채용기준에 성적만 있는게 아니다 학업성취의 결과 일수도 있다 Alpha 가 원래 유능했다 Alpha 의 교육내용이 좋았다

GRE AWA John박 박정어학원

In this memo the dean of Omega University(OU) recommends OU to terminate professor evaluation to secure better jobs for the students To support this recommendation the dean offers several reasons However this argument contains several logical flaws which render it unconvincing

A threshold problem with the argument involves the voluntary nature of the evaluationprocedure The dean provides no evidence about the number or percentage of Omegastudents who participate in the procedure Lacking such evidence it is entirely possible thatthose numbers are insignificant in which case terminating the procedure is unlikely to haveany effect on the grade average of Omega students or their success in getting jobs aftergraduationThe argument also assumes unfairly that the grade-average increase is the result of theevaluation procedure--rather than some other phenomenon The dean ignores a host of otherpossible explanations for the increase--such as a trend at Omega toward higher admissionstandards or higher quality instruction or facilities Without ruling out all other possibleexplanations for the grade-average increase the dean cannot convince me that by terminatingthe evaluation procedure Omega would curb its perceived grade inflation let alone help itsgraduates get jobsEven if the evaluation procedure has resulted in grade inflation at Omega the deans claimthat grade inflation explains why Omega graduates are less successful than Alpha graduatesin getting jobs is unjustified The dean overlooks a myriad of other possible reasons forOmegas comparatively poor job-placement record Perhaps Omegas career services areinadequate or perhaps Omegas curriculum does not prepare students for the job market aseffectively as Alphas In short without accounting for other factors that might contribute toOmega graduates comparative lack of success in getting jobs the dean cannot justify theclaim that if Omega curbs its grade inflation employers will be more likely to hire OmegagraduatesFinally even if the dean can substantiate all of the foregoing assumptions the deansassertion that Omega must terminate its evaluation procedure to enable its graduates to findbetter jobs is still unwarranted in two respects First the dean ignores other possible ways bywhich Omega can increase its job-placement record--for example by improving its publicrelations or career-counseling services Second the dean unfairly equates more jobs withbetter jobs In other words even if more Omega graduates are able to find jobs as a result ofthe deans recommended course of action the kinds of jobs Omega graduates find would notnecessarily be better onesIn sum the deans argument is unpersuasive as it stands To strengthen it the dean mustprovide better evidence that the increase in grade average is attributable to Omegasprofessor-evaluation procedure and that the end result is a perception on the part ofemployers that Omega graduates are less qualified for jobs than Alpha graduates To betterassess the argument I would need to analyze 15-year trends in (l) the percentage of Omegastudents participating in the evaluation procedure (2) Omegas admission standards andquality of education and (3) Omegas emphasis on job training and career preparation I wouldalso need to know what other means are available to Omega for enabling its graduates to findbetter jobs

GRE AWA John박 박정어학원

The following appeared in a memo from a vice president of Quiot Manufacturing

During the past year Quiot Manufacturing had 30 percent more on-the-job accidents than at the nearby Panoply Industries plant where the work shifts are one hour shorter than ours Experts say that significant contributing factors in many on-the-job accidents are fatigue and sleep deprivation among workers Therefore to reduce the number of on-the-job accidents at Quiot and thereby increase productivity we should shorten each of our three work shifts by one hour so that employees will get adequate amounts of sleep

Write a response in which you examine the stated andor unstated assumptions of the argument Be sure to explain how the argument depends on these assumptions and what the implications are for the argument if the assumptions prove unwarranted

The following appeared in a memo from a vice president of Alta Manufacturing

During the past year Alta Manufacturing had thirty percent more on-the-job accidents than nearby Panoply Industries where the work shifts are one hour shorter than ours Experts believe that a significant contributing factor in many accidents is fatigue caused by sleep deprivation among workers Therefore to reduce the number of on-the-job accidents at Alta we recommend shortening each of our three work shifts by one hour If we do this our employees will get adequate amounts of sleep

Write a response in which you discuss what questions would need to be answered in order to decide whether the recommendation and the argument on which it is based are reasonable Be sure to explain how the answers to these questions would help to evaluate the recommendation

The following appeared in a memo from the vice president of Butler Manufacturing

During the past year workers at Butler Manufacturing reported 30 percent more on-the-job accidents than workers at nearby Panoply Industries where the work shifts are one hour shorter than ours A recent government study reports that fatigue and sleep deprivation among workers are significant contributing factors in many on-the-job accidents If we shorten each of our work shifts by one hour we can improve Butler Manufacturings safety record by ensuring that our employees are adequately rested

1 Write a response in which you discuss what specific evidence is needed to evaluate the argument and explain how the evidence would weaken or strengthen the argument

2 Write a response in which you discuss what questions would need to be answered in order to decide whether the recommendation is likely to have the predicted result Be sure to explain how the answers to these questions would help to evaluate the recommendation

4번 반복됨

Alta has 30 more job accidents than Panoply(work shifts one hour shorter)Experts Job accidents caused by fatigue and sleep deprivationTherefore to reduce job accidents and increase productivity shorten three work shifts by one hour for adequate sleep

지난해 우리 회사는 인근 Panoply Industries보다 업무상 재해가 30나 더 많았다 그 회사는 우리보다 근무 교대시간이 1시간 정도 짧았다 전문가들은 대부분의 업무상 재해에 있어서 가장 중요한 요인이 과로와 수면부족으로 보고있다 따라서 우리 회사에서 높은 산업재해를 줄이고 아울러 생산성을 높이기 위해서는 근로자들이 충분한 수면을 취할 수 있도록 1시간씩 3교대 시간을 줄여야 한다

In this memo the (author) vice president of Alta Manufacturing (AM) recommends that to reduce on-the-job accidents and increase productivity AM should shorten its three work shifts by one hour so that employees can

GRE AWA John박 박정어학원

get more sleep To support this recommendation the author provides several evidences However careful scrutiny of each of the facts reveals that it provides little credible support for the authorrsquos recommendation QuestionsThe number of accidents What kind of accidents The seriousness of the accidents is importantHow many employees are in each company What are their productsFalse cause Sleep may not be the reason for the on-the-job accidents What do Alta and Panoply manufacture

First of all the author believes that fatigue caused the on-the job accidents However there could be other reasons The author observes a correlation between sleep deprivation and on-the-job accidents then concludes that the former is the cause of the latter However the author fails to rule out other possible explanations For example it is entirely possible that Alta factories require more strenuous and dangerous labor than Panoply Without ruling out all other such factors it is unfair to conclude that fatigue is responsible for the accidents In addition the work-shifts may not be the cause of the sleep deprivation and fatigue It is possiblehellip Thus the author should provide what exactly Panoply and Alta manufacture and more precise data about their working conditions to be more convincing

Shortening the shift by one hour does not necessarily lead to more sleep And is one hour enoughLess accidents does not mean increased productivity

결론 We should shorten each of out three work shifts by one hour

반박 경쟁사에 비해서 시간당 하는 업무량이 많아서 더욱 피곤할 수도 있다 시간이 문제가 아니라 노후된 시설 설비 자체의 문제 작업 자체가 원래 위험한 것이여서 사고가 많을 수도 있다 다른 회사는 더욱 많은 작업시간에도 불구하고 안정한 작업여건으로 인해서 사고율이 오히려 더 작을 수도 있다비교사의 재해감소가 다른 요인일수 있다(안전 교육 철저)줄인 시간이 피로회복이나 수면으로 연결 안될수 있음(술을 마실 수도 있고 그 시간에 휴식을 취하지 않고 다른일을 함으로써 더욱 피로해질수 있다)

This editorial recommends that Alta Manufacturing reduce its work shifts by one hour each inorder to reduce its on-the-job accident rate and thereby increase Altas productivity To supportthis recommendation the author points out that last year the number of accidents at Alta was30 greater than at Panoply Industries where work shifts were one hour shorter The authoralso cites certain experts who believe that many on-the-job accidents are caused by fatigueand sleep deprivation I find this the argument unconvincing for several reasonsFirst and foremost the author provides absolutely no evidence that overall workerproductivity is attributable in part to the number of on-the-job accidents Although commonsense informs me that such a relationship exists the author must provide some evidence ofthis cause-and-effect relationship before I can accept the authors final conclusion that theproposed course of action would in fact increase Altas productivitySecondly the author assumes that some accidents at Alta are caused by fatigue or sleepdeprivation However the author overlooks other possible causes such as inadequateequipment maintenance or worker training or the inherent hazards of Altas manufacturingprocesses By the same token Panoplys comparatively low accident rate might be attributablenot to the length of its work shifts but rather to other factors such as superior equipmentmaintenance or worker training In other words without ruling out alternative causes ofon-the-job accidents at both companies the author cannot justifmbly conclude that merely byemulating Panoplys work-shift policy Alta would reduce the number of such accidentsThirdly even assuming that Altas workers are fatigued or sleep-deprived and that this is thecause of some of Altas on-the-job accidents in order to accept the authors solution to thisproblem we must assume that Altas workers would use the additional hour of free time tosleep or rest However the author provides no evidence that they would use the time in thismanner It is entirely possible that Altas workers would use that extra hour to engage in someother fatiguing activity Without ruling out this possibility the author cannot convincinglyconclude that reducing Altas work shifts by one hour would reduce Altas accident rateFinally a series of problems with the argument arise from the scant statistical information onwhich it relies In comparing the number of accidents at Alta and Panoply the author fails toconsider that the per-worker accident rate might reveal that Alta is actually safer than Panoplydepending on the total number of workers at each company Second perhaps accident rates

GRE AWA John박 박정어학원

at the two companies last year were aberrations and during other years Altas accident ratewas no greater or even lower than Panoplys rate Or perhaps Panoply is not representativeof industrial companies generally and that other companies with shorter work shifts have evenhigher accident rates In short since the argument relies on very limited statistical information Icannot take the authors recommendation seriouslyIn conclusion the recommendation for emulating Panoplys work-shift policy is not wellsupported To convince me that shorter work shifts would reduce Altas on-the-job accidentrate the author must provide clear evidence that work-shift length is responsible for some ofAltas accidents The author must also supply evidence to support her final conclusion that alower accident rate would in fact increase overall worker productivity

The following appeared in a memo from the vice president of marketing at Dura-Sock Inc

A recent study of our customers suggests that our company is wasting the money it spends on its patented Endure manufacturing process which ensures that our socks are strong enough to last for two years We have always advertised our use of the Endure process but the new study shows that despite our socks durability our average customer actually purchases new Dura-Socks every three months Furthermore our customers surveyed in our largest market northeastern United States cities say that they most value Dura-Socks stylish appearance and availability in many colors These findings suggest that we can increase our profits by discontinuing use of the Endure manufacturing process

1 Write a response in which you examine the stated andor unstated assumptions of the argument Be sure to explain how the argument depends on these assumptions and what the implications are for the argument if the assumptions prove unwarranted

2 Write a response in which you discuss what specific evidence is needed to evaluate the argument and explain how the evidence would weaken or strengthen the argument

3 Write a response in which you discuss what questions would need to be answered in order to decide whether the recommendation and the argument on which it is based are reasonable Be sure to explain how the answers to these questions would help to evaluate the recommendation

Intro The vice president of marketing at Dura-Sock Inc is offering a potentially harmful investment recommendation by claiming that Dura-Sock should discontinue its use of the ldquoEndurerdquo process To support his recommendation he points out a study that Dura-Sock customers actually purchase the socks every three months and a survey that reveals that Dura-Sock customers like the sockrsquos stylish appearance and availability in many colors The study and survey however are insufficient in supporting his proposal and the VP makes several unwarranted assumptionsIntro (simplified) The VP states that though Dura-Socks last for two years customers buy the socks every three months Therefore he assumes that the consumersrsquo motive for buying the produce is not its durabilityHowever the author fails to rule out other possible motivation for consumption

Even if the survey is reliable the author should consider the rest of the market Vague terms ldquowasting moneyrdquomdashprecisely how much are they wasting Studysurvey errorThe company must calculate the outcome of such momentous decisionStudy participantsrsquo comment that they prefer Dura-Sock for its stylishness and availability might take Dura-Sockrsquos enduring quality for granted

우리회사 제품 소비자들에 대한 최근 조사에서 지난 2년여간 양말의 내구성을 강하게 하는 필수공정이었던 자사 특허의 Endure 공정에 들어가는 비용이 낭비라고 말하고 있다 우리 회사는 항상 이 공정 처리에 대한 광고를 내보냈으나 이에 대한 시장 조사에서 실제로 고객들은 이 신제품을 평균 석달마다 구매하는 것으로 나타났다 더군다나 북동부지역에서 실시한 대규모 시장조사에 응답한 고객들은 양말의 모양과 색상등에 더

GRE AWA John박 박정어학원

관심을 나타냈다 이러한 결과는 우리회사가 신기술 공법을 중단하면 그에 따라 수익이 늘어날 것이라는 것을 말해주고 있는 것이다주장 These findings suggest that Dura0Sock can increase its profits by discontinuing its use of the ldquoEndurerdquo manufacturing process

1 survey가 정확한 소비자의 의견을 나타낸 것인가 다른 선택없이 양자택일과 같은 방법의 survey였는지2 북동부지역의 시장조사가 전체 의견을 대표할 수 있나3 사람들이 모양이나 색상에 앞서 내구성을 먼저 평가했을 수도 있다 내구성을 갖추었다는 전제하에 모양과 색상에 관심을 드러낸 것일 수 있다4 소비자가 도매상(retail)인지 소매상(whole)인지가 없다

The following appeared in a business magazine

As a result of numerous complaints of dizziness and nausea on the part of consumers of Promofoods tuna the company requested that eight million cans of its tuna be returned for testing Promofoods concluded that the canned tuna did not after all pose a health risk This conclusion is based on tests performed on samples of the recalled cans by chemists from Promofoods the chemists found that of the eight food chemicals most commonly blamed for causing symptoms of dizziness and nausea five were not found in any of the tested cans The chemists did find small amounts of the three remaining suspected chemicals but pointed out that these occur naturally in all canned foods

Write a response in which you discuss what questions would need to be addressed in order to decide whether the conclusion and the argument on which it is based are reasonable Be sure to explain how the answers to the questions would help to evaluate the conclusion

Representativeness of the tested cansThey should conduct a comparative studyThe testing could be biased because Promofoods employees conducted the testingHow much (quantity) of the five and three suspected chemicals were in the canned foodsFalse cause The substance that caused dizziness and nausea may not be one of the eight common chemicals

많은 소비자들의 현기증과 구역질 불만에 따라 Promofoods사는 지난해 참치 캔 8백만 개를 테스트하기 위해 반품시켰다 그 결과 캔에서는 건강에 위험이 될 수 있는 화합물질이 없었던 것으로 회사측은 결론지었다 이러한 결론은 회사측 화학연구자들이 회수된 캔의 샘플을 테스트해서 이들 증상의 원인이 되는 8가지 화합물 중에서 5가지가 실험된 캔에서 발견되지 않았다는 사실에 근거한 것이다 이들 화학자들은 나머지 3개가지 화합물이 모든 캔 식료품에서 흔히 발견되는 것이라고 언급했다 결론 Promofoods concluded that the cans did not after all contain chemicals that posed a health risk

1 공인된 기간에서 테스트를 한 것이 아니고 자사에서 직접 테스트를 했기에 신뢰성이 안간다 2 이런 증상을 일으키는 8개의 물질 말고 다른 물질들이 캔속에 많이 포함됬을수 있다 3 나머지 3개의 물질들의 함유량이 많아서 다른 종류의 캔들은 문제를 일으키지 않지만 참치캔은 문제를

일으킬 수 있다

This magazine article concludes that the 8 million cans of tuna Promofoods recalled due tocomplaints about nausea and dizziness do not after ail contain any chemicals that pose a

GRE AWA John박 박정어학원

health risk To support this conclusion the author cites the fact that five of eight chemicalscommonly causing these symptoms were not found in the recalled cans while the other threealso occur naturally in other canned foods For several reasons this evidence lends littlecredible support to the authors conclusionTo begin with the author relies partly on the fact that although three of the eight chemicalsmost commonly blamed for nausea and dizziness appeared in Promofoods recalled tunathese chemicals also occur naturally in other canned foods However this fact alone lends nosupport to the authors conclusion for two reasons First the author might be ignoring animportant distinction between naturally occurring chemicals and those not occurring naturallyIt is entirely possible that these three chemicals do not occur naturally in Promofoods tunaand that it is for this reason that the chemicals cause nausea and dizziness Secondly it isentirely possible that even when they occur naturally these chemicals cause the samesymptoms Unless the author rules out both possibilities he cannot reliably conclude that therecalled tuna would not cause these symptomsAnother problem with the argument is that the authors conclusion is too broad Based onevidence about certain chemicals that might cause two particular heath-related symptoms theauthor concludes that the recalled tuna contains no chemicals that pose a health risk Howeverthe author fails to account for the myriad of other possible health risks that the recalled tunamight potentially pose Without ruling out all other such risks the author cannot justifiablyreach his conclusionA third problem with the argument involves that fact that the eight particular chemicals withwhich the test was concerned are only the eight most commonly blamed for nausea anddizziness It is entirely possibly that other chemicals might also cause these symptoms andthat one or more of these other chemicals actually caused the symptoms Without ruling outthis possibility the author cannot jusufiably conclude that the recalled tuna would not causenausea and dizzinessA final problem with the argument involves thetesting procedure itself The author providesno information about the number of recaUed cans tested or the selection method used Unlessthe number of cans is a sufficiently large sample and is statistically repre sentative of all therecalled cans the studys results are not statistically reliableIn conclusion the article is unconvincing as it stands To strengthen the assertion that therecalled tuna would not cause nausea and dizziness the author must provide evidence thatthe three chemicals mentioned that occur naturally in other canned foods also appear naturallyin Promofoods tuna The author must also provide evidence that ingesting other canned foodscontaining these three chemicals does not cause these symptoms To better evaluate theargument we would need to know whether the sample used in the tests was statisticallysignificant and representative of all the recalled tuna We would also need to know what otherchemicals in the recalled tuna might pose any health risk at all

5그룹 불충분 조건오류 빈출

Natures Way a chain of stores selling health food and other health-related products is opening its next franchise in the town of Plainsville The store should prove to be very successful Natures Way franchises tend to be most profitable in areas where residents lead healthy lives and clearly Plainsville is such an area Plainsville merchants report that sales of running shoes and exercise clothing are at all-time highs The local health club has more members than ever and the weight training and aerobics classes are always full Finally Plainsvilles schoolchildren represent a new generation of potential customers these schoolchildren are required to participate in a fitness-for-life program which emphasizes the benefits of regular exercise at an early age

Write a response in which you examine the stated andor unstated assumptions of the argument Be sure to

GRE AWA John박 박정어학원

explain how the argument depends on these assumptions and what the implications are for the argument if the assumptions prove unwarranted

False cause

First of all the author believes that the Increased sales of running shoes and exercise clothing indicates

Plainesville residentsrsquo interest in leading healthy lives However this assumption is not logically convincing for

several reasons could be a fashion trendTime shift ldquoFitness for liferdquo might not have any influence on schoolchildren as they growFalse cause There could be other reasons for member increase in the health clubAll of the above are insufficient condition

The author has to prove that local residents are interested in leading healthy lives However he supports his conclusion with insufficient evidence Nevertheless even if the residents are concerned with health naturersquos way may not be successful First

그 동안의 경험을 토대로 볼 때 건강생활과 밀접히 관련되어 있는 거주 지역에서 본 상점들이 아주 호응을 얻고 있다 따라서 이러한 주민들이 많이 거주하고 있는 Plainsville 에 새로운 상점들을 계속 세워야 한다 이 지역 상인들은 런닝화와 운동복 판매가 가장 높다고 말한다 불과 5 년전에는 거의 전무하다시피하던 지역 헬스 클럽의 경우도 엄청나게 많은 회원을 확보하고 있으며 웨이트 트레이닝과 에어로빅 강좌들도 항상 만원이라고 한다 새로운 고객층을 예측해 보는 것도 가능하다 이 지역의 학생들의 경우 Fitness for Life프로그램을 받게 되는데 이러한 프로그램을 통해서 유년시절부터 정규적인 운동 습관을 들이게 하고 있는 것이 그것이다

결론 We should therefore build our next new store in Plainsville

반박 그동안의 경험에 의한 과거 통계가 꼭 여기에도 적용되는건 아니다 5 년전 헬스 클럽이 잘 안되었던게 다른 원인이였을수 있다(강사수준 미달 강좌미비)tourist 에 의한 원인 일수 있다 어렸을때부터 운동을 했다고 해서 커서도 관심이 있지는 않다 (오히려 반감이 있을수 있다 혹은 건강하기에 건강에 관심이 적을수도 있다)운동복이나 신발의 판매가 육체노동에 의한 것일수도 있다

IntroductionSupport1049896In this memorandum the author asserts that Naturersquos Way should build its next newstore in Plainsville To support this assertion the author states that Plainsvillesmerchantsrsquo sales of exercise clothing are going well the local health club has moremembers than ever and a new generation of customers will help to ensure NaturersquosWayrsquos success At first glance the authorrsquos assumption seems convincing but in-depth scrutiny revealsthat it lacks substantial evidence as it stands

Body 1-SamplingTopic Sentence 1To begin with the author assumes that the merchantsrsquo report indicates that the residentsare concerned about their health However this assumption is based on unsubstantiated

GRE AWA John박 박정어학원

data Example 1 (Rebuttal1) First if we do not know the total volume of items sold and the price of the goods exactly we cannot infer whether the residents are actually buying many goods Example 2 (Rebuttal2)In addition to that the report emphasizes the rising sales of running shoes and exerciseclothing however these may not be hot-selling items for Naturersquos Way or may not be theproducts the company is planning to sell Concluding Sentence Therefore in order to make the argument reliable the author should reconsider themerchantsrsquo report with more detailed data

Body 2-CausalTopic Sentence 2Second the author contends that the health clubs classes are full yet this does not meanthat many people actually use the health club other factors may be the real cause forthose closed classes Example 1 (Rebuttal1) To begin with if the health club is very small the number of people working out wouldnot be a large one In fact regular gym-going may just be a vogue among a smallunrepresentative segment of Plainsvilles population Example 2 (Rebuttal2) Moreover it is possible that most of the people who exercise in the health club do weight training and aerobics only to look good and to meet other singles not for their health In that case there would be little demand for health products Concluding SentenceThus the author should not hasten to presume what really caused people to be interested in a healthier lifestyle and enroll in the health club

Body 3-Time-ShiftTopic Sentence 3Finally the author highlights that Naturersquos Way can expect a new generation of customersin Plainsville that will help the company in the long term This notion is mistaken in that itassumes the conditions of the present will continue unchanged in the future Although theschool children are required to participate in the fitness for life program they may notnecessarily buy Naturersquos Ways products Example 1 (Rebuttal1) In the first instance they may suffer a fall in purchasing power arising from future economic difficulties this would cause reluctance to spend a considerable amount of money on health products which tend to be more expensive Example 2 (Rebuttal2)Another possibility is that there may emerge many competitor companies vying with Naturersquos Way so that in the future the school children may not feel the necessity to purchase one companyrsquos health products over anotherrsquosConcluding Sentence Thus the authorrsquos assumption is highly speculative since it relies heavily on unknowablefuture circumstances

ConclusionThesis In sum the author uses many assumptions that are insufficient in supporting his claimsSupportIn order for the authorrsquos claims to be convincing he needs to advance more persuasiveevidence that people in Plainsville really are concerned with their health and health foodThe following was written as a part of an application for a small-business loan by a group of developers in the city of Monroe

A jazz music club in Monroe would be a tremendously profitable enterprise Currently the nearest jazz club is 65 miles away thus the proposed new jazz club in Monroe the C-Note would have the local market all to itself Plus jazz is extremely popular in Monroe over 100000 people attended Monroes annual jazz festival last summer several well-known jazz musicians live in Monroe and the highest-rated radio program in Monroe is Jazz Nightly which airs every weeknight at 7 PM Finally a nationwide study indicates that the typical jazz fan spends close to $1000 per year on jazz entertainment

1 Write a response in which you discuss what specific evidence is needed to evaluate the argument and explain how the evidence would weaken or strengthen the argument

2 Write a response in which you examine the stated andor unstated assumptions of the argument Be

GRE AWA John박 박정어학원

sure to explain how the argument depends on these assumptions and what the implications are for the argument if the assumptions prove unwarranted

3 Write a response in which you discuss what questions would need to be answered in order to decide whether the prediction and the argument on which it is based are reasonable Be sure to explain how the answers to these questions would help to evaluate the prediction

Group error nationwide survey may not reflect local trends Is the nationwide jazz fan population substantialInsufficient non-residents of Monroe may have attended the jazz festival (Body alternative explanation last year may have been an anomaly The author should consider data from various years) The author should indicate how many out of 100000 were Monroe residentsNationwide study Does this reflect Insufficient Citizens of Monroe may continue to go to the jazz club 65 miles away

Are the people in Monroe really interested in jazzMajority of the people who attended the jazz festival might not be Monroe residentsSurvey error nationwide study may not be applicable to MonroeJazz musicians who live in MonroeMonopolyRadio station

In this business application the author claims that the proposed jazz club C Note will be very profitable in Monroe To support this claim the author argues for his case with several evidences At first glance the authorrsquos argument seems convincing however careful scrutiny reveals that his argument in specious

To begin with the author claims that Monroersquos citizens are interested in jazz He presents three evidences First Secondhellip Thirdhellip Howeverhellip

Monroe 시에 있는 재즈 음악 클럽은 수익성이 좋은 사업이다 현재 가장 가까이에 있는 클럽은 65 마일 정도 떨어져 있다 따라서 이번에 세우려고 하는 C Note 는 독보적인 위치를 점할것이다 더군다나 재즈는 이 시에서 가장 인기있는 음악이다 지난 여름 재즈 축제에서는 10 만명 이상의 Morone 시 주민이 참석하였고 몇몇 유명한 재즈 음악가들도 이곳에 살고 있으며 저녁때 방영되는 라디오 프로그램중에서 최고의 시청률을 보이고 있는 것도 Jazz Nightly 이다 전국조사에서도 전형적인 재즈 팬들은 재즈 분야에 년간 1천 달러 가까이 지출하고 있는 것으로 보고되고 있다 따라서 C Note 클럽이 돈을 벌 수 있는 사업이라는 것은 확실한 것이다

결과 It is clear that the C Note cannot help but make money반박 nearest jazz club 이 양질의 써비스로 여전히 손님을 끌수도 있다Festival 에 얼마나 참여하는지가 jazz 의 인기를 반영하지 않는다 뮤지션이 많이 사는거랑 jazz 의 인기가 상관없다라디오 프로그램이 다른 요인에 의해서 인기일수도 있다 (진행자때문)전국 통계 적용 불가화목 실전반_Ms Noh6In this application the author suggests that a jazz club in Monroe will make a number of profits To support this suggestion the author exemplifies the local condition popularity of jazz in Monroe and nationwide study However careful scrutiny of each of the facts reveals that it provides little credible support for the authorrsquos recommendation Good clear intro

First the author assumes that jazz is popular in Monroe because of several facts the jazz festival last year had high participation some famous jazz musicians live in Monroe and the high-rated radio program is lsquoJazz Nightlyrsquo However this assumption has many drawbacks that must be seriously considered(Good topic sentences) If many attendants in the last-yearrsquos festival came from other cities and not Monroe it is hard to conclude that Monroersquos people like jazz Therefore the author must examine how many Monroe residents actually attended the festival On top of that there is little relationship between habitation of famous jazz musician and the popularity of jazz in Monroe Although several well-known musicians live there if they do not take part in any jazz performance of Monroe this might have no effect to the interest of Monroersquos residents

GRE AWA John박 박정어학원

about jazz Finally in the case of radio program this is also not suitable reason why jazz is popular in Monroe It might be possible that people cannot help choosing lsquoJazz Nightlyrsquo because there are few radio programs at Night The fact that the radio program is the highest rating program is not a germane evidence The approximate number of listeners would be the more crucial evidence Therefore the author needs to seriously deliberate the correlation between jazzrsquos popularity in Monroe and his examples (Good logical flow and clarity)

Second the author uses as evidence the nationwide study that jazz fans spend much money on jazz entertainment to substantiate why starting a jazz club in Monroe will be profitable In other words the author assumes that the characteristics of a nationwide study can be applied to Monroe The national study would lend support to the applicantrsquos claim only if residents in Monroe typify national jazz fans However the author does not provide credible evidence that this is the case Moreover the populations of jazz fans nationwide may be insubstantial Thus the author should not infer hastily that Monroersquos residents will spend much money on enjoying jazz from the nationwide study

Lastly even if jazz is popular in Monroe C Note may not be successful It is entirely possible that residents might still prefer other clubs where they have always went In addition there is another possibility that the nearest jazz club will attract many of Monroersquos people because it serves fine performances and is equipped with favorite facilities Without considering these other possibilities the author cannot make his argument convincing In sum the author presents many reasons that are insufficient in supporting his or her claim In order for the authorrsquos claims to be convincing he needs to advance more persuasive evidence such as the total number of Monroe residents who attended the jazz festival the effects on the popularity of jazz by the musicians living in Monroe and the actual number of residents who would typify themselves to be jazz fans through a local survey Without substantial evidence that C Note will be successful in Monroe the businessmen may be overinvesting in what might lead to a business failureExcellent clarity Score 50

The following appeared in a newsletter offering advice to investors

Over 80 percent of the respondents to a recent survey indicated a desire to reduce their intake of foods containing fats and cholesterol and today low-fat products abound in many food stores Since many of the food products currently marketed by Old Dairy Industries are high in fat and cholesterol the companys sales are likely to diminish greatly and company profits will no doubt decrease We therefore advise Old Dairy stockholders to sell their shares and other investors not to purchase stock in this company

Write a response in which you discuss what questions would need to be answered in order to decide whether the advice and the argument on which it is based are reasonable Be sure to explain how the answers to these questions would help to evaluate the advice

Survey 80

GRE AWA John박 박정어학원

Old Dairy could change their products and manufacture low fat dairy foodsLess competing companies Old Dairy could eventually be the only company that produces hellipImprecise numbers and measurementsCustomers may still buy high fat dairy products

The author of the newsletter is offering potentially dangerous advice by recommending Old Dairy stockholders to withdraw investment and stop purchase What is more the authorrsquos prediction debases the reputation and business of Old Dairy and if false could devoid the investment opportunity of the newsletter readers Therefore investors should examine whether the authorrsquos evidences are substantial

To begin with the author states that 80 percent of the respondents in a survey indicated a desire to reduce their intake of foods He therefore argues that Old Dairyrsquos high fat and cholesterol products would decrease in sales However the author makes a crucial error in this argument First the author provides no evidence that the surveyrsquos results are statistically reliable Were they representative of all the customers Were they chosen for the survey randomly Furthermore the desire to reduce fat and cholesterol intake is a pervasive trend in todayrsquos opulent society however the author erroneously identifies this as a new phenomenon which will affect consumer trends Second having a desire to reduce fat and cholesterol intake does not necessarily indicate that people who have this desire will actually reduce consuming these types of products It is entirely possible that they may continue buying Old Dairy products for its quality and taste Accordingly the author cannot draw any firm conclusion that people will not buy Old Dairy products Therefore if any of these cases are true the author may be offering investors a detrimental investment advice

최근 조사에 대한 응답자중 80 이상이 자신이 먹는 음식에서 지방과 콜레스테롤의 함유량을 줄이고 싶다고 한다 아울러 요즘은 많은 식료품 가계에서 저지방 제품들을 많이 취급하고 있다 현재 Old Dairy Industries가 판매하고 있는 많은 음식제품들은 지방과 콜레스테롤이 높기 때문에 이 회사의 매출이 격감할 것으로 보이며 당연히 매출이익도 줄어들것이다 따라서 이 회사의 주주들은 주식을 매각하고 다른 주식 투자가들도 이 회사의 주식을 매입하지 않는 것이 좋다

결론 Old Dairy stockholders to sell their shares and other investors not to purchase stock in this company

반박 모든 상품이 다 고 지방 고 칼로리는 아니다(비록 많을지라도) 일부의 식품의 경우 기호에 맞어서 히트해서 전체적인 수입이 증가할 수도 있다국내시장만 생각할 수 없다( 외국시장에서 호황을 누릴수 있다 )입맛이라는게 즉각 바뀌는게 아니다

The following appeared in a letter to the editor of the Balmer Island Gazette

On Balmer Island where mopeds serve as a popular form of transportation the population increases to 100000 during the summer months To reduce the number of accidents involving mopeds and pedestrians the town council of Balmer Island should limit the number of mopeds rented by the islands moped rental companies from 50 per day to 25 per day during the summer season By limiting the number of rentals the town council will attain the 50 percent annual reduction in moped accidents that was achieved last year on the neighboring island of Seaville when Seavilles town council enforced similar limits on moped rentals

1 Write a response in which you discuss what questions would need to be answered in order to decide whether the recommendation is likely to have the predicted result Be sure to explain how the answers to these questions would help to evaluate the recommendation

2 Write a response in which you discuss what questions would need to be answered in order to decide whether the prediction and the argument on which it is based are reasonable Be sure to explain how the answers to these questions would help to evaluate the prediction

3 Write a response in which you examine the stated andor unstated assumptions of the argument Be sure to explain how the argument depends on these assumptions and what the implications are for the argument if the assumptions prove unwarranted

Whatrsquos the actual population of Balmer Island 100000mdashis this a significant increase What kind of accidents Skin abrasions or serious injury And compared to Seaville how serious are the accidents and the actual number of accidents Did Seaville enforce other restrictions like safety signsHow different are the conditions of Balmer

GRE AWA John박 박정어학원

and Seaville regarding population road (safety) conditions topography other town-government regulation How much will the economy of Balmer be affected do to this restriction Could it cause an economic recession due to the fact that these rental companiesrsquo chance to make money is only during the summer thereby weakening the economic infrastructure Are there any other ways that could better alleviate the accident rate

Statistics 50-impreciseAnalogy Balmer compared with TorseauFalse Cause Accidents might have occurred because of reasons other than mopeds False Cause population increase may not be part of the cause of the accidentsOther explanations for the accident pedestrians few road safety regulations narrow roadsThere could be other better solutionshellip

Balmer Island의 인구가 여름철에는 십만명으로 늘어난다 2륜차와 보행자간 사고를 줄이기 위해 시의회는 6개의 자전거를 포함한 2륜차 대여업체에게 이 기간동안에는 대여숫자를 일일 50에서 30으로 제한하도록 할 것이다 대여숫자를 줄임으로써 시 의회는 지난해 이웃한 Torseau섬에서 이와 동일한 규제를 시행해서 50나 줄인 결과를 보고 마찬가지로 50를 줄일수 있다고 확신하고 있다

결론 The town council of Balmer Island should linit the number

반박 보행자의 부실에 의해서 사고가 많이 일어날수도 있다렌탈수의 줄임만이 대책은 아니다(대부분의 사람들이 렌탈 보다는 소유하고 있을 수도 있다)옆섬과는 상황이 다를수도 있다(그 섬에서는 사고의 원인이 많은 자전거 수로 인한것일수있다) 하지만 이 섬은 좁은 도로가 원인일 수도 있고 도로 안전 장치의미비가 원일일수 있다

In this letter the author recommends that Balmer Island should limit the number moped rentals from 50 to 30 per day To support this recommendation the author points out several reasons However careful scrutiny of each of the facts reveals that it is filled with unanswered questions that could significantly weaken the authorrsquos recommendation with loops and holes which are answered

The recommendation depends on the assumption that no alternative means of reducing the number of accidents are available However the author fails to offer any evidence to substantiate this crucial assumption It is highly possible that means other than this would better solve the problem Perhaps they could widen the roads or put-up more safety signs Or perhaps the accidents were due to the lack of skills in which case proper safety training would significantly alleviate the problem Without considering and ruling out these and other alternative means of reducing accidetns the author cannot confidently conclude that merely emulating Torseau would suffice Moreover the author is advising a recommendation which could potentially harm the economy of Balmer Island sincehellip Moreover the Balmer Island should alternative means to reduce accidents because limiting moped rentals during the summer could harm the economy of Balmerhellip

First of all the author believes that increase in population and the number of moped rentals are responsible for the accidents It is entirely possible that other factors are responsible for the accidents Perhaps Balmer Islandrsquos lack of safety signs was a major factor Or maybe the roads are narrow and dangerous on the Island therefore the town council could enforce stricter traffic regulations to alleviate the problem Accordingly if either of these scenarios is true the author cannot draw any firm conclusion that increase in the number of population and moped rentals are the cause of the accidents

The author of this editorial recommends that to reduce accidents involving mopeds andpedestrians Balmer Islands city council should restrict moped rentals to 30 per day down from50 at each of the islands six rental outlets To support this recommendation the author citesthe fact that last year when nearby Torseau Islands town council enforced similar measuresTorseaus rate of moped accidents fell by 50 For several reasons this evidence providesscant support for the authors recommendationTo begin with the author assumes that all other conditions in Balmer that might affect therate of moped-pedestrian accidents will remain unchanged after the restrictions are enactedHowever with a restricted supply of rental mopeds people in Balmer might purchase mopedsinstead Also the number of pedestrians might increase in the future with more pedestriansespecially tourists the risk of moped-pedestrian accidents would probably increase For thatmatter the number of rental outlets might increase to make up for the artificial supplyrestriction per outlet--a likely scenario assuming moped rental demand does not declineWithout considering and ruling out these and other possible changes that might contribute to ahigh incidence of moped-pedestrian accidents the author cannot convince me that theproposed restrictions will necessarily have the desired effect

GRE AWA John박 박정어학원

Next the author fails to consider other possible explanations for the 50 decline inTorseaus moped accident rate last year Perhaps last year Torseau experienced unusually fairweather during which moped accidents are less likely Perhaps fewer tourists visited Tot seanlast year than during most years thereby diminishing the demand for rental mopeds to belowthe allowed limits Perhaps last year some of Torseaus moped rental outlets purchased newmopeds that are safer to drive Or perhaps the restrictions were already in effect but were notenforced until last year In any event a decline in Torseaus moped accident rate during onlyone year is scarcely sufficient to draw any reliable conclusions about what might have causedthe decline or about what the accident rate will be in years aheadAdditionally in asserting that the same phenomenon that caused a 50 decline in mopedaccidents in Torseau would cause a similar decline in Balmer the author relies on what mightamount to an unfair analogy between Balmer and Torseau Perhaps Balmers ability to enforcemoped-rental restrictions does not meet Torseaus ability if not then the mere enactment ofsimilar restrictions in Balmer is no guarantee of a similar result Or perhaps the demand formopeds in Torseau is always greater than in Balmer Specifically if fewer than all availablemopeds are currently rented per day from the average Balmer outlet while in Torseau everyavailable moped is rented each day then the proposed restriction is likely to have less impacton the accident rate in Balmer than in TorseauFinally the author provides no evidence that the same restrictions that served to reduce theincidence of all moped accidents by 50 would also serve to reduce the incidence ofaccidents involving mopeds and pedestrians by 50 Lacking such evidence it is entirelypossible that the number of moped accidents not involving pedestrians decreased by a greaterpercentage while the number of moped-pedestrian accidents decreased by a smallerpercentage or even increased Since the author has not accounted for these possibilities theeditorials recommendation cannot be taken seriouslyIn conclusion the recommendation is not well supported To convince me that the proposedrestriction would achieve the desired outcome the author would have to assure me that nochanges serving to increase Balmers moped-pedestrian accident rate will occur in theforeseeable future The author must also provide dear evidence that last years decline inmoped accidents in Torseau was attributable primarily to its moped rental restrictions ratherthan to one or more other factors In order to better evaluate the recommendation I wouldneed more information comparing the supply of and demand for moped rentals on the twoislands I would also need to know the rate of mopedpedestrian accidents in Torseau both priorto and after the restrictions were enforced in TorseauThe following appeared in a magazine article about planning for retirement

Clearview should be a top choice for anyone seeking a place to retire because it has spectacular natural beauty and a consistent climate Another advantage is that housing costs in Clearview have fallen significantly during the past year and taxes remain lower than those in neighboring towns Moreover Clearviews mayor promises many new programs to improve schools streets and public services And best of all retirees in Clearview can also expect excellent health care as they grow older since the number of physicians in the area is far greater than the national average

Write a response in which you discuss what specific evidence is needed to evaluate the argument and explain how the evidence would weaken or strengthen the argument

-Natural beauty and consistent climate may not be the most wanted qualities-Housing costs could have lowered on a national level wealthy retirees may not care about costs-Taxes may be high compared to the nationrsquos average tax rate-What about other qualities of Clearview Crime rate what qualities would retirees want -If schools streets and public services need improvement then this is proof that the current condition of Clearview is low Or due to budgetary reasons the mayor may not follow-up on his promise because of lowered tax rate -Schools and people who are retired no relationship-Physicians What kind of physicians Number is irrelevant Are these physicians capable of addressing the illnesses of old people

This author argues that anyone seeking a place to retire should choose Clearview To supportthis argument the article cites Clearviews consistent climate and natural beauty its fallinghousing costs its low property taxes compared to nearby towns and the mayors promise toimprove schools streets and services The article also claims that retirees can expectexcellent health care because the number of physicians in Clearview greatly exceeds thenational average This argument is flawed in several critical respectsTo begin with although consistent climate and natural beauty might be attractive to manyretirees these features are probably not important to all retirees For many retirees it isprobably more important to live near relatives or even to enjoy changing seasons Thus I

GRE AWA John박 박정어학원

cannot accept the authors sweeping recommendation for all retirees on this basisAlso Clearviews declining housing costs do not necessarily make Clearview the best placeto retire for two reasons First despite the decline Clearviews housing costs might be highcompared to housing costs in other cities Secondly for wealthier retirees housing costs arenot likely to be a factor in choosing a place to retire Thus the mere fact that housing costshave been in decline lends scant support to the recommendationThe articles reliance on Clearviews property-tax rates is also problematic in two respectsFirst retirees obviously have innumerable choices about where to retire besides Clear viewand nearby towns Secondly for retirees who are well-off financially property taxes are notlikely to be an important concern in choosing a place to retire Thus it is unfair to infer fromClearviews property-tax rates that retirees would prefer ClearviewYet another problem with the argument involves the mayors promises In light of Clearviewslow property-tax rates whether the mayor can follow through on those promises is highlyquestionable Absent any explanation of how the city can spend more money in the areas citedwithout raising property taxes I simply cannot accept the editorials recommendation on thebasis of those promises Besides even if the city makes the improvements promised thoseimprovements--particular the ones to schools--would not necessarily be important to retireesFinally although the number of physicians in Clearview is relatively high the per capitanumber might be relatively low Moreover it would be fairer to compare this per capita numberwith the per capita number for other attractive retirement towns--rather than the nationalaverage After all retirees are likely to place a relatively heavy burden on health-careresources Besides the article provides no assurances that the number of physicians inClearview will remain high in the foreseeable futureIn conclusion the recommendation is poorly supported To strengthen it the author mustconvince me--perhaps by way of a reliable survey--that the key features that the vast majorityof retirees look for in choosing a place to live are consistent climate natural beauty and lowhousing costs The author must also provide better evidence that Clear views property taxesare lower than the those of cities in other areas The author must also explain how the city canmake its promised improvements without raising property taxes Finally to better assess theargument I would need to now how the per capita number of physicians in Clearview wouldcompare to the national average in the futureThe following appeared as a letter to the editor from a Central Plaza store owner

Over the past two years the number of shoppers in Central Plaza has been steadily decreasing while the popularity of skateboarding has increased dramatically Many Central Plaza store owners believe that the decrease in their business is due to the number of skateboard users in the plaza There has also been a dramatic increase in the amount of litter and vandalism throughout the plaza Thus we recommend that the city prohibit skateboarding in Central Plaza If skateboarding is prohibited here we predict that business in Central Plaza will return to its previously high levels

Write a response in which you discuss what questions would need to be answered in order to decide whether the recommendation is likely to have the predicted result Be sure to explain how the answers to these questions would help to evaluate the recommendation

Why two years ago What happened two years ago which started this declineIs the dramatic increase in the ldquopopularityrdquo of skateboarding the cause of the steady decline of shoppers Are there any malls nearby Were there any changes nearby which could affect the decline in customersmdasha big mall perhaps Could the decline be due to the shop ownersHow many skateboarders use the plazaWhere do they skateboardDo they shop and are they customersAre the increase in litter and vandalism due to skateboarders Could this be alleviated by installing CCTVs and hiring security

This editorial concludes that the city should ban skateboarding from its downtown CentralPlaza in order to attract visitors to that area to return the area to its former glory and to makeit a place where people can congregate for fun and relaxation To justify this conclusion theeditorial points out that skateboarders are nearly the only people one sees anymore at CentralPlaza and that the Plaza is littered and its property defaced The editorial also points out thatthe majority of downtown merchants support the skate boarding ban This argument is flawedin several critical respectsFirst the editorials author falsely assumes that a ban on skateboarding is both necessaryand sufficient to achieve the three stated objectives Perhaps the city can achieve thoseobjectives by other means as well--for example by creating a new mall that incorporates anattractive new skateboard park Even if banning skateboarders altogether is necessary to meetthe citys goals the author has not shown that this action by itself would suffice Assuming thatthe Plazas reputation is now tarnished restoring that reputation and in turn enticing peopleback to the Plaza might require additional measures--such as removing litter and graffiti

GRE AWA John박 박정어학원

promoting the Plaza to the public or enticing popular restaurant or retail chains to the PlazaSecondly the editorial assumes too hastily that the Plazas decline is attributable to theskateboarders--rather than to some other phenomenon Perhaps the Plazas primary appeal inits glory days had to do with particular shops or eateries which were eventually replaced byless appealing ones Or perhaps the crime rate in surrounding areas has risen dramatically forreasons unrelated to the skateboarders presence at the Plaza Without ruling out these andother alternative explanations for the Plazas decline the editorials author cannot convince methat a skateboard ban would reverse that declineThirdly the editorials author might be confusing cause with effect--by assuming that theskateboarders caused the abandonment of the Plaza rather than vice versa It is entirelypossible that skateboarders did not frequent the Plaza until it was largely abandoned--andbecause it had been abandoned In fact this scenario makes good sense since skateboardingis most enjoyable where there are few pedestrians or motorists to get in the wayFourth it is unreasonable to infer from the mere fact that most merchants favor the ban thatthe ban would be effective in achieving the citys objectives Admittedly perhaps thesemerchants would be more likely to help dean up the Plaza area and promote their businesseswere the city to act in accordance with their preference Yet lacking any supporting evidencethe author cannot convince me of this Thus the survey amounts to scant evidence at best thatthe proposed ban would carry the intended resultFinally the author recommends a course of action that might actually defeat the citysobjective of providing a fun and relaxing place for people to congregate In my experienceskateboarding contributes to an atmosphere of fun and relaxation for adults and children alikemore so than many other types of ambiance Without considering that continuing to allowskateboarding--or even encouraging this activity--might achieve the citys goal more effectivelythan banning the activity the author cannot convincingly conclude that the ban would be in thecitys best interestsIn sum the argument is a specious one To strengthen it the editorials author must providedear evidence that skateboarding and not some other factor is responsible for the conditionsmarking the Plazas decline The author must also convince me that no alternative means ofrestoring the Plaza are available to the city and that the proposed ban by itself would suffice toattract tourists and restore the Plaza to its former glory Finally to better assess the argument itwould be useful to know the circumstances under which the downtown merchants would bewilling to help the city achieve its objectives

6그룹 약한 비유 빈출

The following recommendation appeared in a memo from the mayor of the town of Hopewell

Two years ago the nearby town of Ocean View built a new municipal golf course and resort hotel During the past two years tourism in Ocean View has increased new businesses have opened there and Ocean Views tax revenues have risen by 30 percent Therefore the best way to improve Hopewells economymdashand generate additional tax revenuesmdashis to build a golf course and resort hotel similar to those in Ocean View

Write a response in which you examine the stated andor unstated assumptions of the argument Be sure to explain how the argument depends on these assumptions and what the implications are for the argument if the assumptions prove unwarranted

GRE AWA John박 박정어학원

Assumptions The author assumes that OVrsquos municipal golf course and resort hotel caused tourism new businesses and increased tax revenues There may be other reasons advertising promo He assumes that this will continueAssumes that Ocean View and Hopewell are similar in many waysmdashthe name suggests otherwise OV may have always been a tourist attractions for its beaches We need to know the topography

2년전 Ocean View 시는 시정 소유 골프 및 휴양지 호텔을 신축했다 그리고 지난 2년동안 이 시의 관광객이 증가했으며 새로운 사업들이 생겨났다 그에따라 시의 세수도 30나 증가했다 Hopewell의 경제를 향상시키고 아울러 세수를 늘릴 수 있는 가장 좋은 방법은 Ocean View에 세워진 것과 같은 골프 시설과 휴양지 호텔을 신축하는 것이다

1 다른 요인으로 관광 산업이 발전했을 수도 있다 문화 유적이 발견이 되었거나 도로의 정비등으로 여행자가 늘었을 수도 있다

2 관광 산업의증가가 늘어난 세수의 원인이 아니라 새로 유입된 인구의 증가나 다른 공장에서 발생한 것일 수 있다

3 2년동안 한참 골프가 붐을 이루었을 수 있다 경제상황이 나빠지거나 다른 레포츠가 인근 지역에 생겨난다면 골프하는 사람이 줄어들 수 있다

In this memo HopeweUs mayor recommends that in order to stimulate the towns economyand boost tax revenues HopeweU should build a new golf course and resort hotel just as thetown of Ocean View did two years ago To support this recommendation the mayor points outthat in Ocean View during the last two years tourism has increased new businesses haveopened and tax revenues have increased by 30 I find the mayors argument unconvincingin several important respectsFirst of all it is possible that the mayor has confused cause with effect respecting the recentdevelopments in Ocean View Perhaps Ocean Views construction of a new golf course andhotel was a response to previous increases in tourism and business development increasesthat have simply continued during the most recent two years Since the mayor has failed toaccount for this possibility the claim that Hopewell would boost its economy by alsoconstructing a golf course and hotel is completely unwarrantedSecondly the mayor fails to account for other possible causes of the trends in Ocean Viewduring the last two years The increase in tourism might have been due to improving economicconditions nationwide or to unusually pleasant weather in the region The new businessesthat have opened in Ocean View might have opened there irrespective of the new golf courseand hotel And the 30 increase in tax revenues might have been the result of an increase intax rates or the addition of a new type of municipal taxWithout ruling out these and other alternative explanations for the three recent trends inOcean View the mayor cannot reasonably infer based on those trends that Hopewellseconomy would benefit by following Ocean Views exampleThirdly even if the recent trends in Ocean View are attributable to the construction of the newgolf course and hotel there the mayor assumes too hastily that the golf course and hotel willcontinue to benefit that towns overall economy The mayor has not accounted for thepossibility that increased tourism will begin to drive residents away during tourist season orthat new business development will result in the towns losing its appeal as a place to visit or tolive Unless the mayor can convince me that these scenarios are unlikely I cannot accept themayors recommendation that Hopewell follow Ocean Views exampleFinally the mayors argument rests on the unsubstantiated assumption that Hopewell andOcean View are sufficiently alike in ways that might affect the economic impact of a new golfcourse and hotel Hopewell might lack the sort of natural environment that would attract moretourists and new businesses to the town--regardless of its new golf course and hotel For thatmatter perhaps Hopewell already contains several resort hotels and golf courses that are notutilized to their capacity If so building yet another golf course and hotel might amount to amisallocation of the towns resources--and actually harm the towns overall economyIn sum the mayors recommendation is not well supported To bolster it the mayor mustprovide better evidence that Ocean Views new golf course and hotel and not some otherphenomenon--has been responsible for boosting Ocean Views economy during the last twoyears To better assess the recommendation I would need to know why Ocean View decidedto construct its new golf course and hotel in the first place--specifically what events prior toconstruction might have prompted that decision I would also need to thoroughly compare

GRE AWA John박 박정어학원

HopeweU with Ocean View--especially in terms of their appeal to tourists and businesses--todetermine whether the same course of action that appears to have boosted Ocean Viewseconomy would also boost Hopewells economy

The following is part of a memorandum from the president of Humana University

Last year the number of students who enrolled in online degree programs offered by nearby Omni University increased by 50 percent During the same year Omni showed a significant decrease from prior years in expenditures for dormitory and classroom space most likely because instruction in the online programs takes place via the Internet In contrast over the past three years enrollment at Humana University has failed to grow and the cost of maintaining buildings has increased along with our budget deficit To address these problems Humana University will begin immediately to create and actively promote online degree programs like those at Omni We predict that instituting these online degree programs will help Humana both increase its total enrollment and solve its budget problems

Write a response in which you discuss what questions would need to be answered in order to decide whether the prediction and the argument on which it is based are reasonable Be sure to explain how the answers to these questions would help to evaluate the prediction

Is Omni University successful due to the online degree program 50 Is the decrease in expenditures for dormitory and classroom space due to the decrease in of on-campus students Which classes were successful Does HU have those classes

Even if the long-distance degree programs at Omni University benefited the school the presidentrsquos recommendation that Human College should emulate Omni University is too hasty First OUrsquos name implies that the school would have more majors than Humanahellip the president should examine which degrees were in the long-distance programhellip

지난해에는 Omni 대학에서 개강했던 원거리 학생 학점 취득 프로그램을 등록했던 학생들의 숫자가 50나 증가했다 같은해 기간동안 Omni 대학에서는 그 전년도부터 기숙사와 학급의 공간 확충을 위한 예산을 대폭 줄였는데 이는 이 원거리 학점 취득 프로그램이 양방향 비디오 컴퓨터 접속을 통해서만 가능한 수업지도 방식이기때문인 것으로 보인다 반면 지난 3개년 동안 Humana 대학에서의 수강률은 감소한데다가 건물

GRE AWA John박 박정어학원

유지비도 올랐다 따라서 Humana대학의 수강을 늘리고 예산손실을 회복하기 위해서는 Omni 대학에서 취한 조치와 같은 능동적인 프로그램을 추진해야 한다

결론 we should initiate and actively promote long-distance degree programs like those at Omni 반박 원거리 학생 취득 프로그램 숫자가 증가한거하고 예산이 줄어드는 것 사이에 연관이 약하다 (causal 학생의 증가로 관리비용 증가할수 있음 원거리 수업가능 장비도입에의한 비용발생)bad analogy(omni university 하고 같은 조건이 아니다 )-gt omni college 가 강좌내용이 좋아서 학생의 등록이 많을수 있다 Humana 대학에서 만들었다 하더라도 인기 없을수 있음다른 요인에 의해서 Humana 대학의 수강 인원이 증가할수 있음(비록 과거엔 인기가 없었을지라도)

The following appeared as part of a business plan developed by the manager of the Rialto Movie Theater

Despite its downtown location the Rialto Movie Theater a local institution for five decades must make big changes or close its doors forever It should follow the example of the new Apex Theater in the mall outside of town When the Apex opened last year it featured a video arcade plush carpeting and seats and a state-of-the-art sound system Furthermore in a recent survey over 85 percent of respondents reported that the high price of newly released movies prevents them from going to the movies more than five times per year Thus if the Rialto intends to hold on to its share of a decreasing pool of moviegoers it must offer the same features as Apex

Write a response in which you discuss what questions would need to be answered in order to decide whether the recommendation is likely to have the predicted result Be sure to explain how the answers to these questions would help to evaluate the recommendation

Before following through this business plan the manager should investigate the cause of Rialtorsquos unsuccessful business

The author provides no evidence that the surveyrsquos results are statistically reliable The surveyrsquos sample of 85 percent must be sufficient in size and representative of overall population of the city where Rialto and Apex is serving Lacking evidence of a sufficiently representative sample the author cannot justifiably rely on the survey to draw any conclusion whatsoever The author does not indicate that Apex is indeed currently successful However even if Apex is enjoying success the argument relies on what might be a false analogy between Rialto and Apex In order for Apex to serve as a model that Rialto should emulate the author must assume that all relevant circumstances are essentially the same However this assumption is unwarranted For example the argument overlooks the face that Apex is located in a strategic placemdashbeside a mall where customers can not only watch a movie but also enjoy shopping Therefore simply changing the facility to that of Apex may not lead to success

The author does not mention whether Apex is successful or not Nevertheless even if Apex is currently successful the argument relies on what might be a false analogy between Rialto and Apex In order for Apex to serve as a model that Rialto should emulate the author must assume that all relevant circumstances are essentially the same However this assumption is unwarranted For example the argument overlooks the fact that these two institutions are located in different locations Rialto in downtown and Apex in a mall outside of town Although Apex opened with state-of-the-art facilities the decisive factor in its success could be due to its strategic location of being in a mall People could enjoy both shopping and movies at one location thus they may prefer Apex over Rialto Furthermore the place where people enjoy leisure activities has shifted in the past decades for most cities from downtown to the suburbs Therefore Rialto may not be successful even if it emulates Apexrsquos facilities A better business plan may be relocating Apex to the thriving section of the downtown

Rialto 극장은 지난 50여년간 지역 회관으로써 시내에 위치해 있으면서도 이제 변화를 꾀하지 않으면 문을 닫을

GRE AWA John박 박정어학원

판이다 이 극장은 시외 쇼핑타운에 새로 들어선 Apex 극장의 사례를 본받아야 했다 Apex가 지난해 개업했을 당시 이 극장은 비디오 아케이드 플러쉬 카펫트 바닥과 좌석 그리고 최신 음향시설을 갖추었다 더군다나 최근 조사에서는 응답자의 85 이상이 새로 출시된 영화 입장료가 비싼 탓으로 지난해보다 5배이상의 관람객이 줄어들었다고 나타났다 따라서 Rialto 극장이 줄어들고 있는 관람객을 뺐기지 않고 유지하려면 Apex와 같은 시설들을 갖추어야 할 것이다주장 리알토 극장이 줄어들고 있는 관람객을 뺐기지 않고 유지하려면 Apex와 같은 시설들을 갖추어야 할 것이다

1 조사에서 응답자가 전체를 대표할 수 없다 2 apex 극장이 좋은 시설을 갖추고 있지만 그로 인해 수익이 많이 발생했다는 말이 없으므로 시설투자를

하고도 좋은 결과를 얻을 수 있을지 그 근거가 미흡하다3 좋은 영화가 출시된다면 입장료가 비싸도 영화관에서 꼭 보려고 할 수 있다 4 rialto 가 시설이 아닌 다른 요인에 의해 장사가 안될수도 있다( 우범 지역이라든지)

The following is a recommendation from the business manager of Monarch Books

Since its opening in Collegeville twenty years ago Monarch Books has developed a large customer base due to its reader-friendly atmosphere and wide selection of books on all subjects Last month Book and Bean a combination bookstore and coffee shop announced its intention to open a Collegeville store Monarch Books should open its own in-store cafeacute in the space currently devoted to childrens books Given recent national census data indicating a significant decline in the percentage of the population under age ten sales of childrens books are likely to decline By replacing its childrens books section with a cafeacute Monarch Books can increase profits and ward off competition from Book and Bean

Write a response in which you examine the stated andor unstated assumptions of the argument Be sure to explain how the argument depends on these assumptions and what the implications are for the argument if the assumptions prove unwarranted

The following is a recommendation from the business manager of Monarch Books

Since its opening in Collegeville twenty years ago Monarch Books has developed a large customer base due to its reader-friendly atmosphere and wide selection of books on all subjects Last month Book and Bean a combination bookstore and coffee shop announced its intention to open a Collegeville store Monarch Books should open its own in-store cafeacute in the space currently devoted to childrens books Given recent national census data indicating a significant decline in the percentage of the population under age ten sales of childrens books are likely to decline By replacing its childrens books section with a cafeacute Monarch Books can increase profits and ward off competition from Book and Bean

1 Write a response in which you discuss what questions would need to be answered in order to decide whether the recommendation is likely to have the predicted result Be sure to explain how the answers to these questions would help to evaluate the recommendation

2 Write a response in which you discuss what specific evidence is needed to evaluate the argument and explain how the evidence would weaken or strengthen the argument

No evidence regarding Monarch Bookrsquos successEven if Regal Bookrsquos is successful this may not be attributable to the cafeacute False analogy Emulating may not lead to success Other factors may be involvedInsufficient condition The national census is not enough evidence that childrenrsquos book sales will decline Can

GRE AWA John박 박정어학원

the national census represent the local child populationDid opening a cafeacute boost sales for Regal Books Even assuming Regal is successful by opening a cafeacute this may not be suitable for Monarch which plans to close the childrenrsquos book section to establish a cafe Imprecise language ldquorelatively little spacerdquo how smallThe managerrsquos recommendation contradicts what he says Since Monarch is popular for its wide selection of books closing a selection which targets a major group of readers may hurt Monarchrsquos salesIs this the best way to compete

When Stanley Park first opened it was the largest most heavily used public park in town It is still the largest park but it is no longer heavily used Video cameras mounted in the parks parking lots last month revealed the parks drop in popularity the recordings showed an average of only 50 cars per day In contrast tiny Carlton Park in the heart of the business district is visited by more than 150 people on a typical weekday An obvious difference is that Carlton Park unlike Stanley Park provides ample seating Thus if Stanley Park is ever to be as popular with our citizens as Carlton Park the town will obviously need to provide more benches thereby converting some of the unused open areas into spaces suitable for socializing

Write a response in which you examine the stated andor unstated assumptions of the argument Be sure to explain how the argument depends on these assumptions and what the implications are for the argument if the assumptions prove unwarranted

Stanley 파크가 처음 개장했을 당시 가장 크고 가장 많이 이용되는 공원이었다 아직도 공원중에서는 가장 크지만 이용률은 상당히 떨어졌다 지난달 공원 주차장에 설치해놓은 비디오 카메라를 통해 보면 drop(주차장으로 여겨짐) 이용률이 가장 높았다 수치상으로는 하루 평균 50대의 차량만이 이용하였다 반면 직장 중심거리에 위치한 작은 규모의 Carlton 파크는 주당 무려 150여명 이상이 이용하고 있다 Stanley 파크와는 달리 Carlton 파크에는 의자가 있다는 것이 가장 뚜렷한 차이점이다 따라서 Stanley 파크가 Carlton 파크처럼 시민들이 자주 이용하는 공원이 되기 위해서는 벤치를 설치할 필요가 있으며 이렇게 사용되지 않는 일부 공간을 활용해서 사교를 위한 공간으로 바꾸어야 한다 ===gtdrop 에 대한 첨부사항 (영영사전내용입니다)---- a place or central depository to which something (as mail money or stolen property) is brought for distribution or transmission also the act of depositing something at such a place dropgt

주장 if Stanley Park is ever to be as popular with our citizens as is Carlton Park the town will obviously need to provide more benches thereby converting some of the unused open areas into spaces suitable for socializing1 조사가 언제 이루어진 것인가 조사가 언제 실시되었느냐에 따라 결과가 다를 수있다 현재는 다시 스탠리 파크가 늘어났었을 수 있다 2 벤치를 많이 설치했다고 해서 많은 관광객이 오지 않을수 있다(사람들이 벤치나 사교 공간을 원한다는 어떠한 자료도 없다)3스탠리 파크 주변에 교통 상황이 악화가 되었거나 칼튼 파크에서 문화행사등을 많이 가져서 이용객이 줄어든것일 수도 있다 4 칼튼 파크가 중심지에 있어서 접근성이 좋을수 있다5 조사가 같은 시간을 기준으로 한게 아니다(하나는 주중이고 하나는 주말이다)6사람의 수와 차의 대수를 같은것으로 비교할수 없다 (차안에 몇 명이 타고 있는지 모르고 대중교통을 이용해서 왔을수도 있다)

Page 3: GRE writing argument brain storm

GRE AWA John박 박정어학원

sure to explain how the argument depends on these assumptions and what the implications are for the argument if the assumptions prove unwarranted

Assumptions The author is assuming that 75 percent is a significant number protective clothing and light reflecting material mitigate severe injuries (the cautious nature of those who wear such equipment might have prevented accidents or the skills and reflex of the roller skaters drivers are the main reason for accidents)

ETS Essay Response ndash Score 5

The argument presented is limited but useful It indicates a possible relationship between a high percentageof accidents and a lack of protective equipment The statistics cited compel a further investigation of theusefulness of protective gear in preventing or mitigating roller-skating related injuries However theconclusion that protective gear and reflective equipment would greatly reducerisk of being severelyinjured is premature Data is lacking with reference to the total population of skaters and the relativelevels of experience skill and physical coordination of that population It is entirely possible that furtherresearch would indicate that most serious injury is averted by the skaters ability to react quickly andskillfully in emergency situationsAnother area of investigation necessary before conclusions can be reached is identification of the types ofinjuries that occur and the various causes of those injuries The article fails to identify the most prevalenttypes of roller-skating related injuries It also fails to correlate the absence of protective gear and reflectiveequipment to those injuries For example if the majority of injuries are skin abrasions and closed-headinjuries then a case can be made for the usefulness of protective clothing mentioned Likewise if injuriesare caused by collision with vehicles (eg bicycles cars) or pedestrians then light-reflective equipmentmight mitigate the occurences However if the primary types of injuries are soft-tissue injuries such astorn ligaments and muscles back injuries and the like then a greater case could be made for training andexperience as preventative measures

Reader Commentary for Essay Response ndash Score 5This strong response gets right to the work of critiquing the argument observing that it indicates apossible relationship but that its conclusion is premature It raises three central questions that ifanswered might undermine the soundness of the argumentbull What are the characteristics of the total population of skatersbull What is the usefulness of protective or reflective gear in preventing or mitigating roller skatingrelatedinjuriesbull What are the types of injuries sustained and their causesThe writer develops each of these questions by considering possible answers that would either strengthen orweaken the argument The paper does not analyze the argument as insightfully or develop the critique asfully as required for a 6 paper but the clear organization strong control of language and substantial degreeof development warrant more than a score of 4

Milk and dairy products are rich in vitamin D and calcium substances essential for building and maintaining bones Many people therefore believe that a diet rich in dairy products can help prevent osteoporosis a disease in which the bones weaken significantly with age and that is linked to both environmental and genetic factors But a long-term study of a large number of people has found that those who have consistently consumed dairy products throughout the years of the study have a higher rate of bone fractures than any other participants in the study Since bone fractures are a symptom of osteoporosis this study result shows that a diet rich in dairy products may actually increase rather than decrease the risk of osteoporosis

Write a response in which you discuss what specific evidence is needed to evaluate the argument and explain how the evidence would weaken or strengthen the argument

Proven fact vitamin D and calcium in milk help prevent osteoporosis

GRE AWA John박 박정어학원

Challenge long-term study of a lot of people consistently consumed dairy products throughout the years=gt higher rate of bone fractures than other participantsThus bone fractures(symptom of osteoporosis) increased by diet rich in dairy products

-The author uses unscientific and imprecise terms in his study to support his claim-Control group-The author tries to refute a well established fact through a single study-Other factors bring about osteoporosis For example occupation hazards genetics dietary habits-Bone fracture could have might have may have been caused by other factors work environment etc and not specifically osteoporosis-The lack of a controlled environment of the study group The researchers only noted the consumption of milk and did not control other variantsmdashlifestyle and diet -The author bases his conclusion only by noting that the study samples consumed milk The researchers only noted the consumption of milk and did not control other variantsmdashlifestyle and diet-Dairy products might have had adverse effects on some of the participants in the study for example lactose intolerance

In this study the author claims that the consumption of milk actually increases the risk of osteoporosisTo support this claim the author refers to a long-term study he conducted ThesisHowever careful scrutiny of this study reveals that it provides little credible support for the authorrsquos claim

First the author states that the bone fractures were caused by osteoporosis However there could be other reasons for the bone fractures For example it is entirely possible that working conditions might have contributed to the bone fracture A person working at a construction has a much higher chance of bone fracture than a person that works in an office The author needs to provide the reasons for the occurrence of the study participantsrsquo bone fractures Thus the author should provide more sufficient evidence that

The author observes a correlation between osteoporosis and bone fracture then concludes that the former is the cause of the latter However the author fails to rule out other possible explanations For example it is entirely possible that working

The authorrsquos study includes an indefinite amount of people whose backgrounds arenrsquot defined genetic(family) history and environment the age at which the participants partook in the study It may be that the participants had genetic predisposition that were more prone to osteoporosis

The author does not sufficiently define what types of people were included in the study Bones are affected as noted in the prompt by various factors genes and environment The author however does not hellipFinally the author asserts his claim definitively While it is a proven fact that dairy products help people maintain healthy bones the author overthrows this long standing fact with a single long-term study that he did 우유와 유지방 제품에는 뼈를 형성시키고 강화시키는데 아주 중요한 역할을 하는 비타임 D와 칼슘이 많다 때문에 많은 사람들이 유지방이 풍부한 식이요법을 하면 환경 및 유전적 요인에 의한 손상과 노화에 의해 뼈가 심각하게 손상을 입는 질병인 오스티오포로시스를 억제하는데 도움이 된다고 믿고 있다 그러나 상당수의 사람을 대상으로 한 장기간 실험을 통해서 볼 때 이들 대상자중 실험 기간동안 유제품을 꾸준히 섭취한 사람들은 그렇지 않은 사람에 비해 훨씬 높은 골절현상을 보였다 이러한 실험결과를 볼 때 골절증상은 오스티오포로시스가 원인이기 때문에 이러한 식이요법이 오스티오포로시스 질병을 줄이기보다는 오히려 증가시킬 것이라는 것을 알수 있다결론 우유와 유지방 제품을 섭취하는 것은 오스티스오포로시스 질병을 줄이기봐는 오히려 증가시킬 것이다

1 실험은 변수에 의해 상당히 다른 결과를 얻을 수 있다 변수(생활 습관 운동)가 효과적으로 통제가 되었나 알 수 없다 상당수의 사람 장기간의 실험이 너무 주관적이다 객관적인 데이터가 주어져야한다

2 상당수와 장기간이 얼마만큼인지 알 수 없다 샘플링이랑 기간등 여러가지로 다르게 적용되었을 수 있다(실험에 충분한 기간과 인원이였는지)

3 골절 증상이 골다공증이 원인이 되어 나타났다고 했는데 그렇지 않을 수 있다다른 원인이 골절증상을 가져온 것일 수 있다

4 한가지 실험가지고 일반화했다 이미 밝혀진 사실을 단 한가지 실험으로 뒤엎었다 두 비교군이 동일 했는지에 대한 자료 없음(한쪽이 원래 뼈가 약했을수도있음)

GRE AWA John박 박정어학원

A recently issued twenty-year study on headaches suffered by the residents of Mentia investigated the possible therapeutic effect of consuming salicylates Salicylates are members of the same chemical family as aspirin a medicine used to treat headaches Although many foods are naturally rich in salicylates food-processing companies also add salicylates to foods as preservatives The twenty-year study found a correlation between the rise in the commercial use of salicylates and a steady decline in the average number of headaches reported by study participants At the time when the study concluded food-processing companies had just discovered that salicylates can also be used as flavor additives for foods and as a result many companies plan to do so Based on these study results some health experts predict that residents of Mentia will suffer even fewer headaches in the future

Write a response in which you discuss what questions would need to be answered in order to decide whether the prediction and the argument on which it is based are reasonable Be sure to explain how the answers to these questions would help to evaluate the prediction

In this study the author asserts that the number of headaches suffered by the average citizen of Mentia will steadily decline though the use of salicylates as flavor additives To support this assertion the author provides several evidences to support his claim However his argument is specious

Unscientific data correlation the specific amount used as preservatives Other factors could have caused the decline in headaches20 year study Was it a controlled study Lack of controlled environment exposes the participants to other factors

1) The decline in the average number of headaches does not necessarily substantiate that it was indeed caused by the commercial use of salicylates It is entirely possible that other factors are responsible for the decline Perhaps other substances in the food remedied the headaches of the participants Or the decline in headache may have been caused by other dietary consumption or lifestyle habits like exercising Accordingly if these are true the author cannot draw any firm conclusion that the commercial use of salicylates caused the decline in the average number of headaches

살리실산염은 아스피린과 같은 동일 화학물질 계열로써 두통을 치료하는데 사용되는 약물이다 많은 음식에 이 살리실산염이 다량 함유되어 있지만 지난 수십년간 식품가공 업체들은 가공식품 부패방지용으로 살리실산염을 첨가해왔다 이러한 살리실산염의 산업용 사용의 증가는 본 연구원들에 의해 보고된 평균 두통 횟수를 꾸준히 감소시키는데 일조한것으로 이해되어 왔다 최근 식품가공 회사들은 이 살리실산염이 맛을 내는 첨가물로 사용될 수 있음을 발견했다 이러한 새로운 용도로 Mentia에 사는 주민들이 겪고 있는 두통의 평균 횟수를 앞으로도 계속 줄여나갈 수 있을 것으로 보인다결론 살리산염의 사용증가는 주민들의 두통 횟수를 계속 줄여줄 것이다

가정 1 조사받은 사람들은 단지 Salicylate에 의해 headache가 감소된 것이다 -gt 오류 다른 노력을 기울였을 수 있다 가정 2 나중에 Salicylate의 사용증가가 headache의 감소를 가져올 것이다 -gt 오류 또다른 원인으로 얼마든지 headache는 증가가능하다가정 3 음식물에서 headache 감소 원인은 Salicylate 이다 -gt 오류 또다른 성분이 있었을 수 있다가정 4 Flavor additive 로 사용된 Salicylate도 headache를 줄일것이다 -gt 오류 Flavor additive로 사용시 같은 효과를 가질지 밝혀진바 없다 상업적으로 사용되었다고 다 먹었을까

The following appeared as part of an article in a business magazine

GRE AWA John박 박정어학원

A recent study rating 300 male and female Mentian advertising executives according to the average number of hours they sleep per night showed an association between the amount of sleep the executives need and the success of their firms Of the advertising firms studied those whose executives reported needing no more than 6 hours of sleep per night had higher profit margins and faster growth These results suggest that if a business wants to prosper it should hire only people who need less than 6 hours of sleep per night

Write a response in which you examine the stated andor unstated assumptions of the argument Be sure to explain how the argument depends on these assumptions and what the implications are for the argument if the assumptions prove unwarranted

Assumptions 1 Correlation of hours of sleep with success The author assumes that the executives used their wake-up hours on work Could there be other factors 2 Executives versus employeesmdashcriteria for hiring is too simpleminded 3 Characteristics of the 300 male and female executives 4 The study was about advertising executives but the author applies this case to all businesses 5 The author should consider long-term stability rather that sharp growth and profit 6 Average number of hours of sleep is insufficient to support the authorrsquos recommendation 7 The author assumes that this lifestyle pattern will remain consistent

The following appeared in a memo from the president of a company that makes breakfast cereals

In a recent study subjects who ate soybeans at least five times per week had significantly lower cholesterol levels than subjects who ate no soy products By fortifying our Wheat-O cereal with soy protein we can increase sales by appealing to additional consumers who are concerned about their health This new version of Wheat-O should increase company profits and at the same time improve the health of our customers

Write a response in which you examine the stated andor unstated assumptions of the argument Be sure to explain how the argument depends on these assumptions and what the implications are for the argument if the assumptions prove unwarranted

최근의 조사에서 일주에 최소 5회 정도 콩을 먹었던 사람들은 전혀 먹지 않았던 사람들에 비해 콜레스테롤이 상당히 낮은것으로 조사되었다 따라서 Wheat-O 시리얼에 콩 단백질의 함유를 강화시킴으로써 건강에 관심이 있는 더 많은 소비자를 상대로 매출을 올릴 수 있을 것이다 이 신제품이 회사 수입 증대를 가져오는 것 뿐만아니라 소비자의 건강을 향상시킬 수 있다

GRE AWA John박 박정어학원

주장 By increasing our Wheat-O cereal with soy protein we can multiply sales

1 콩 단백질의 함유를 강화해도 그수치가 미흡하거나 흡수가 잘 안되거나 할 수 있다 다른 성분과 섞여서 만들어지는 것이므로 다른 성분에 의해서 그 흡수가 잘 안되거나 효과가 낮을 수 있다 2 강화한 것이 건강에 관심이 있는 소비자에게 어필한다고 해도 그 수가 적어서 매출에 큰 영향을 안줄 수 있다3 다른 음식에 의해서 콜레스테롤이 더 낮아질수 있다 4 콜레스테롤을 낮추는 것이 사람들에게 그들의 건강생활을 위해 크게 어필 안할수도 있다( 저지방식이라든지 다른 건강을 높이는 방법이 더 관심이 많을수 있다)

In a study of the reading habits of Waymarsh citizens conducted by the University of Waymarsh most respondents said that they preferred literary classics as reading material However a second study conducted by the same researchers found that the type of book most frequently checked out of each of the public libraries in Waymarsh was the mystery novel Therefore it can be concluded that the respondents in the first study had misrepresented their reading habits

Write a response in which you discuss what specific evidence is needed to evaluate the argument and explain how the evidence would weaken or strengthen the argument

Evidence needed -Information about the respondentsmdashage gender class precise number of respondents-Time-shift After how many years was the second study conductedmdashtaste in books may change over time-Can library records sufficiently evidence the reading habits of Waymarsh citizens

1)2)3)4)5)6) First the author provides no evidence that the surveyrsquos studyrsquos results are statistically reliable Lacking

information about the precise methodology of the study the number of (customers)respondents surveyed and the number of respondentsvarious information such as gender age and social background which are essential to bolster the conclusion it is impossible to assess the validity of the results It is possible that people who feel inclined to( take low fat and low cholesterol foods ) read literary classics were more willing to respond to the survey than were others Another problem is the representativeness of the respondents Were they representative of all the customersWaymarsh citizens Were they chosen for the survey randomly Lastly the survey results must depend on the honesty and integrity of the respondents Without more information about the survey the author cannot simply conclude that (most of customers want to take low fat and low cholesterol foods)Waymarsh citizens misrepresented their reading habit on the first study on the basis of this surveystudy(survey reliability)

The exact number of books checked-out should be notedMystery novels could have been in trend at the time of the studyPublic libraries vs other libraries such as Leeville University library

Leeville 대학에서 실시한 Leeville 주민의 독서습관에 대한 조사에서 대부분의 응답자는 독서용으로 문학작품을 선호한다고 응답했다 그러나 같은 조사팀에서 이어 실시한 조사에서는 Leeville에 있는 도서관마다 가장 빈번하게 연람된 책의 종류를 보면 미스테리 소설이었던 것으로 조사되었다 따라서 최초 조사 응답자들이 독서 습관에 대해 잘못 말했다고 결론지을 수 있다

결론 it can be concluded that the respondents in the first study had misrepresented their reading habits1 첫번째 조사 그룹하고 두번째 조사 그룹하고 다른 그룹일 수 있다

GRE AWA John박 박정어학원

2 도서관에 비치된 책이 미스터리 소설이 더 많아서 선택의 여지가 없을 수 있다

High Frequency Group 2 National-gtLocal Local-gtNational amp Group Member Error

The following appeared in a letter to the editor of a Batavia newspaper

The department of agriculture in Batavia reports that the number of dairy farms throughout the country is now 25 percent greater than it was 10 years ago During this same time period however the price of milk at the local Excello Food Market has increased from $150 to over $300 per gallon To prevent farmers from continuing to receive excessive profits on an apparently increased supply of milk the Batavia government should begin to regulate retail milk prices Such regulation is necessary to ensure fair prices for consumers

Write a response in which you discuss what questions would need to be answered in order to decide whether the recommendation is likely to have the predicted result Be sure to explain how the answers to these questions would help to evaluate the recommendation

First the author points to the fact that hellip25 increase of dairy farms may be(could be might be) due to population increaseThe milk price increase could be reflecting the increase in cost of livingThe actual price of milk might be cheaper if the economy is experiencing inflationOne market Excello Food cannot reflect the price increase of every market in BataviaThe author claims that farmers are receiving excessive profits however this might not be the caseThe author claims that by regulating milk prices the government will successfully ensure both lower prices and an adequate supply of milk for consumers However this might not be the best solutionThe writer adduces the information released by the department of agriculture that the number of dairy farms has

increased by 25 percent over the last 10 years and thus he claims that dairy farmers are earning excessive profits from milk However the information is filled with loops and holes which need to be filled First what percentage of the milk are sold as milk

Are the dairy farms processing the milk into other dairy products Or are they selling them as milkmdashWhat percentage of the milk is sold as milk Could this be a natural increase

Can the price at the local Excello Food Market represent the price of milk in stores throughout all of Batavia

Are the farmers receiving excessive profits What is the cost of producing milk Could such regulation offer fair prices to consumers

Topic SentenceTo begin with the author assumes that the price of Excello Food Market which has increased from $150 to over $300 per gallon can be applied to the whole nation of Batavia however the author fails to provide evidence to substantiate this assumptionExampleFirstIn additionTherefore in order to make the argument more reliable the author should consider the prices of a large number markets all over the countryThere could be an economic inflation

GRE AWA John박 박정어학원

ldquoBatavia 지역의 농림부의 보고에 따르면 전국적으로 낙농업자의 숫자가 과거 10 년전에 비해 25나 증가했다고 합니다 그러나 동기간 동안 이 지역에 있는 Excello Food Market 에서 우유의 가격은 갤런당 15 불에서 3 불로 증가했습니다 증가된 우유 공급량에 맞춘 낙농업자들의 수익률 증대를 보장하기 위해서는 주 정부가 소매 공급가에 대한 규제를 해야 합니다 소비자에게 보다 저렴한 가격에 안정된 물량을 공급할 수 있도록 하기 위해서는 이러한 조치가 필요합니다

결론 the Batavia government should begin to regulate retail milk prices

반박 ( Excello Food Market doesnrsquot reflect the whole of Batavia) 우유값 증가가 단순히 inflation 을 따라간것일수도 있다 가격상승이 수익을 보장하지 않는다( 원자재 상승등으로 인해서)우유값 규제만이 최선의 방법이 아니다( 물량 조절)낙농업자의 수익보호도 필요하다

A recent sales study indicates that consumption of seafood dishes in Bay City restaurants has increased by 30 percent during the past five years Yet there are no currently operating city restaurants whose specialty is seafood Moreover the majority of families in Bay City are two-income families and a nationwide study has shown that such families eat significantly fewer home-cooked meals than they did a decade ago but at the same time express more concern about healthful eating Therefore the new Captain Seafood restaurant that specializes in seafood should be quite popular and profitable

Write a response in which you discuss what questions would need to be addressed in order to decide whether the conclusion and the argument on which it is based are reasonable Be sure to explain how the answers to the questions would help to evaluate the conclusion

BAY CITY The name implies that this is a port city which would have traditionally consumed seafood 30--what type of seafood Unprocessed or processed cans of tuna and sardines If the latter increased Captain Seafood may not profit 30--natural inflation Are the non-seafood restaurants selling seafood If yes this may suffice since patrons would not easily dine at an unfamiliar restaurant Nationwide studyrsquos representativeness Desire does not lead to action

Nationwide study applied to Bay City Eating fewer home cooked meals than they did a decade agoHealthy food The author needs to show that 30 -gt population increaseOther foods may be more popularCurrent number of restaurants that serve seafood might sufficeWhat kind of healthy food are they interested in Surely not just seafoodThe locals may still patronize the current restaurants that serve seafood dishes

lt national-gt local에 적용 local-gtnational에 적용gt1) The author assumes that the nationwide trend reflects the general trend upon which the argument relies

Yet the author fails to provide evidence to substantiate this crucial assumption The nationwide trend of two income families eating fewer home-cooked and concern for health my not reflect the lifestyle and interests of Bay City citizens Therefore to make his argument stronger the author needs to provide evidence that two income families of Bay City maintains the nationwide trend

GRE AWA John박 박정어학원

최근의 매상에 관한 조사에서 Bay 시에 있는 레스토랑의 해산물 요리의 소비가 지난 5년동안 30 증가했다고 보고되었다 그러나 현재 해산물을 전문으로 취급하고 있는 레스토랑이 없다 더군다나 대다수의 가정이 맞벌이 가정인데다가 전국 조사에서도 나와있듯이 이러한 맞벌이 가정은 10년전의 가정에서 했듯이 집에서 식사를 챙겨먹는 것이 급격하게 줄어 들었고 동시에 건강식과 관련한 지출이 늘고 있다 따라서 해산물 전문 레스토랑이 아주 인기를 끌것이고 그만큼 수익도 많을 것이다

1 해산물이 건강식인지에 대한 언급이 없다2 외식하는데 해산물 요리만 먹지 않을 것이다 집에서 챙겨먹지 않는다고 해산물을 먹는다고 할 수 없다3 현재 해산물 취급하는 식당이 현재까지 없다고 해서 계속 없을 수만은 없다 수익이 만약 늘어난다면

Bay시 주변의 도시의 레스토랑이 체인점을 내거나 새로운 식당이 생길 수 있고 오히려 수익이 줄어들 수도 있다

Scientists studying historical weather patterns have discovered that in the mid-sixth century Earth suddenly became significantly cooler Although few historical records survive from that time some accounts found both in Asia and Europe mention a dimming of the sun and extremely cold temperatures Either a huge volcanic eruption or a large meteorite colliding with Earth could have created a large dust cloud throughout Earths atmosphere that would have been capable of blocking enough sunlight to lower global temperatures significantly A large meteorite collision however would probably create a sudden bright flash of light and no extant historical records of the time mention such a flash Some surviving Asian historical records of the time however mention a loud boom that would be consistent with a volcanic eruption Therefore the cooling was probably caused by a volcanic eruption

Write a response in which you discuss what questions would need to be addressed in order to decide whether the conclusion and the argument on which it is based are reasonable Be sure to explain how the answers to the questions would help to evaluate the conclusion

Historical records may not be enough to explain the global cooling phenomenonThere may be reasons other than the two hypothesis to explain the global coolingA volcanic eruption big enough to produce dust clouds that envelop the earth to cause global cooling would have countless recordsThe author argues that a large meteorite collision is not a feasible explanation for the global cooling because there are no historical records that indicate a flash The authorrsquos logic is flawed in eliminating the meteorite hypothesis by claiming that there was no historical record of a bright flash of light According to common sense a meteorite collision big enough to produce dust that could envelop the earth would result in catastrophe even to the extinction of major species of animals

Loud boom would be insufficient evidence to conclude it was a volcanic eruption If could hear a loud boom there should be records of a volcanic eruption because the author claims that the dust cloud from this gigantic eruption was significant enough to cause global coolingCooling could have been caused by factors besides volcanic eruption and meteor collisionThe absence of historical records that indicate a bright flash of light Collision could have occurred at a place where there no witnesses Could have fell out in the seaBoom might have been caused by things other than a volcanic eruptionHistorical records from Asia and Europe may not be applicable to earth as a whole

과거의 날씨 주기를 연구하는 과학자들은 6세기 중엽 지구가 갑자기 혹한기가 되었던 것을 알게되었다 이 당시의 몇몇 기록들이 아직도 남아있긴 하지만 아시아와 유럽에서 발견되는 몇가지 자료에서 태양 빛의 수축과 그에따른 엄청난 추위가 있었음을 언급하고 있다 거대한 화산 폭발이나 거대 운석의 지구 충돌중 어느것에

GRE AWA John박 박정어학원

의해서든 엄청난 먼지구름을 형성시켜 지구전체에 태양빛을 차단해서 지구의 기온을 뚝 떨어뜨릴수 있을 수도 있다 그러나 이러한 운석 충돌은 순간 섬광을 발산시키게 될 것인데 이 시기의 어느 기록에서도 이러한 섬광은 언급되지 않고 있다 그러나 이 시기에 남아 있는 일부 아시아의 기록문서에서는 연속적으로 화산폭발음일수 있는 엄청난 폭발음이 있었음을 기록하고 있다 따라서 이러한 혹한은 아마도 화산 폭발에 의한 것이었을 것이다

결론 the cooling was probably caused by a volcanic eruption

반박 자료의 부족( 몇몇 자료가지고 그 당시의 기후를 예상하기에는 무리가 있다)다른 원인에 의해서 기후가 떨어졌을수도 있다 (빙하기 다른 기상원인)기록되지 않았다고 해서 그 사실이 없었던 것은 아니다(기록은 했으나 자료가 없어졌을수도 있고 모든 사실이 전부다 기록되지는 않는다 )기록된 폭발음이 꼭 확산 폭발음이 아닐수도 있다( 화산 폭발음이 너무 먼거리여서 들리지 않을수도 있고 다른 소리가 기록된 것이였을수도 있다)부족한 자료를 신빙할수 있는가 기록의 소멸도 예상 할수 있다 실제로 운석이 떨어졌는데 안들렸을수 있다

The following appeared in a memo from the president of Bower Builders a company that constructs new homes

A nationwide survey reveals that the two most-desired home features are a large family room and a large well-appointed kitchen A number of homes in our area built by our competitor Domus Construction have such features and have sold much faster and at significantly higher prices than the national average To boost sales and profits we should increase the size of the family rooms and kitchens in all the homes we build and should make state-of-the-art kitchens a standard feature Moreover our larger family rooms and kitchens can come at the expense of the dining room since many of our recent buyers say they do not need a separate dining room for family meals

Write a response in which you examine the stated andor unstated assumptions of the argument Be sure to explain how the argument depends on these assumptions and what the implications are for the argument if the assumptions prove unwarranted

The presidentrsquos first unstated but apparent assumption is that a nationwide survey can be applied to local areas There is no guarantee that the area in which Bower Builders builds houses will follow the national consumer trend Since he only mentions the overall result of the nationwide survey it is difficult to determine more specific conditions such whether people in urban and rural areas have the same housing preferences Such uncertainty is further exacerbated by the nature of the survey question itselfmdash it does not deal with current trends in actual home purchases but with desired home features The naiumlve assumption that consumer desire will directly result in consumer action underlies the presidentrsquos interpretation and application of the survey results to his company plan Not everyone who wishes for a large family room and kitchen will or can actually buy a house with those features mainly due to financial reasons Furthermore the president also assumes for no evident reason that consumers will not hesitate to purchase houses with state-of-the-art kitchens as a standard rather than optional feature when it is clear that such a feature will raise the overall cost significantly He continues to blunder in his mistaken assumptions about consumer behavior in assuming that the taste of recent buyers can represent the concern of future prospective buyers The fact that recent buyers have claimed no need for separate dining rooms does not mean future buyers will feel the same way as well

The president of Bower Builders recommends that in order to raise company profits the company should build houses with large family rooms and large state-of-the-art kitchens He uses the results of a nationwide survey on desired home features and the example of Bower Buildersrsquo competitor Domus Construction to support his plan His memo manifests several assumptions about surveys consumer behavior and Domus Constructionrsquos houses that do not necessarily bolsterbuttress his argument

GRE AWA John박 박정어학원

The writer assumes 1 the nationwide survey results can be applied to the local area (Desire cannot reflect consumer trend) 2 Domus Construction was profitable because they had such features (The writer should investigate what other features the DC provides and the actual number of homes sold) 3 State-of-the-art kitchens will sell well (no evidence to buttress his assumption furthermore they would need to spend more money which would increase the price of the housesmdashtheir market will be limited to the affluent) 4 The recent buyers represent the concern of most buyersmdashthat they are ok with a house that is without a dining room

Nationwide survey might not be applicable to this regionDomus Construction could have other superior qualities than Bower Builders Ex DesignSelling fast doesnrsquot indicate quantitySmall yards complaints could be voiced in the future

First the author concludes that a nationwide survey reveals that the two most desired home features are a bathroom with a whirlpool tub and a large kitchen However this nationwide survey may not reflect the trends of the customers that Bower Builders target The author assumes that the nationwide trend reflects customer trends The national study would lead support only if the nationwide trend reflect the home-purchasing trends The nationwide trend may just be a trend of desire which does not result in the act of purchasing However the author does not provide credible evidence that this is the case

전국 조사에서 가장 인기있는 집안 구조 2가지는 훨풀 튜브가 마련된 욕실과 커다른 부엌으로 조사되었다 경쟁업체 Domus Construction이 신축한 완공예정인 주택은 이러한 욕실을 갖추고 있어서 분양이 조기에 이루어지고 있고 가격도 평균치보다 상당히 웃돌고 있다 우리도 집을 많이 팔도 그만큼 이윤을 많이 남기려면 신규 주택에는 위의 2가지 사항을 갖추어야 한다 최근 우리가 지은 주택 수요자들이 적은 평수에 대해서는 별다른 불만사항이 없었기 때문에 앞으로 평수를 더 줄여서 이익을 극대화 할 수 있을 것이다

주장 We should include whirlpool tubs and a large kitchen

1 경쟁업체 Domus Construction이 신축한 완공예정인 주택은 이러한 욕실을 갖추고 있어서라기 보다는 위치가 좋거나 다른 마감재(construction material)나 인테리어가 뛰어나서 분양이 조기에 이루어진것이라고 볼 수 있다 2 전국조사가 건물이 지어지는 지역에 항상 적용되리라고 볼 수 없다 3 평수를 줄이는 것에 불만이 없었다는 것은 editor의 견해일 수 있다 사람들이 표현하지 않은 것일 수도 있고 다른 장점이 많아서 그런 단점을 드러내지 않은 것일 수 있기 때문에 속단할 수 없다 4 다른것을 함으로써 더 잘 팔릴수도 있다

2011 7 19 화요일 GRE AWA 실전반이OO

Argument 94

The president of Bower Builders contends recommends that in order to raise company profits the company should build houses with large family rooms and large state-of-the-art kitchens He uses the results of a nationwide survey on desired home features and the example of Bower Buildersrsquo competitor Domus Construction to support his plan His memo manifests several assumptions about surveys consumer behavior and Domus Constructionrsquos houses that do not

GRE AWA John박 박정어학원

necessarily bolsterbuttress his argumentThe presidentrsquos first unstated but apparent assumption is that a nationwide survey can be

applied to local areas There is no guarantee that the area in which Bower Builders builds houses will follow the national consumer trend Since he only mentions the overall result of the nationwide survey it is difficult to determine more specific conditions such whether people in urban and rural areas have the same housing preferences Such uncertainty is further exacerbated by the nature of the survey question itselfmdash it does not deal with current trends in actual home purchases but with desired home features The naiumlve assumption that consumer desire will directly result in consumer action underlies the presidentrsquos interpretation and application of the survey results to his company plan Not everyone who wishes for a large family room and kitchen will or can actually buy a house with those features mainly due to financial reasons Furthermore the president also assumes for no evident reason that consumers will not hesitate to purchase houses with state-of-the-art kitchens as a standard rather than optional feature when it is clear that such a feature will raise the overall cost significantly He continues to blunder in his mistaken assumptions about consumer behavior in assuming that the taste of recent buyers can represent the concern of future prospective buyers The fact that recent buyers have claimed no need for separate dining rooms does not mean future buyers will feel the same way as well

In addition the president finds a real-life actualization of the nationwide survey results in the recent sales of Domus Construction However he easily assumes that large family rooms and kitchens are the only reasons the houses of Domus Construction sell well He does not take into account other features and selling points of the competitorrsquos houses There may well be other explanations for its houses selling more quickly and expensively including additional home features as well as external factors such as proximity to better schools or superior financial solvency of its clientele If Bower Builders merely added larger family rooms and kitchens without taking into consideration the other factors they may lose rather than gain profits

Clearly the presidentrsquos assertion that Bower Builders make houses with large family rooms and high-tech kitchens at the expense of dining rooms rests on a number of assumptions that are ill-informed and naiumlve If Bower Builders undertakes the proposed plan without further research into local consumer desires purchasing trends and the marketing and sales of competing companies the company will risk losing money by building big new houses that people cannot afford to or will not wish to buy

The following appeared in a letter to the editor of a journal on environmental issues

Over the past year the Crust Copper Company (CCC) has purchased over 10000 square miles of land in the tropical nation of West Fredonia Mining copper on this land will inevitably result in pollution and since West Fredonia is the home of several endangered animal species in environmental disaster But such disasters can be prevented if consumers simply refuse to purchase products that are made with CCCs copper unless the company abandons its mining plans

Write a response in which you examine the stated andor unstated assumptions of the argument Be sure to explain how the argument depends on these assumptions and what the implications are for the argument if the assumptions prove unwarranted

The writer assumes 1 The writer is trying to avoid an inevitability 2 Mining copper will result pollution (they could make preventative measures) 3 The writer may be too late from stopping CCC from developing the area into a copper mine 4 Since mining is an underground enterprise the surface may not be affected that much therefore endangered species may not be affected 4 Consumers wonrsquot buy CCC products if the journal publishes a negative review about CCC (How many readers) CCC could a company that has ties with many IT companies and industries in that their copper is almost ubiquitous in various products

GRE AWA John박 박정어학원

지난 한해동안 CCC(Consolidated Copper 회사)는 서부 플로리다의 열대 지역에 1백만 마일이 넘는 땅을 사들였다 이 곳에서의 채광 활동은 서부 플로리다가 몇몇 멸종 위기에 처한 동물의 서식지이기 때문에 분명 오염과 환경파괴를 가져올 것이다 그러나 이러한 파괴는 CCC 회사가 채광을 포기할 때까지 이 회사가 채굴한 구리로 제조된 제품을 구매하지 않으면 막을 수 있을 것이다

결론 such disaster can be prevented if consumers simply refuse to purchase products that are made with CCCs copper until the company abandons its mining plans

1 구리로 제조된 물건이 생활에 필수적인 것이 많은 만큼 불매가 쉽지 않을 수 있다(전선이나 각종 전자제품에 필수적으로 들어가기 때문에)

2 불매를 유도한다고 해서 소비자들이 구매를 안하는 것은 아니다3 적절한 채굴로 환경파괴를 가져 오지 않을 수 있다 (땅속에 있는 물질을 채굴하는 만큼 생물에 영향을 안

미칠 수도 있다)4 이미 채굴이 다 끝나서 더 이상의 채굴이 없을 수도 있다 5 땅을 구입한다고 해서 채광하는건 아니다 (다른 용도로 샀을수도 있다)

The following is a letter to the editor of an environmental magazine

In 1975 a wildlife census found that there were seven species of amphibians in Xanadu National Park with abundant numbers of each species However in 2002 only four species of amphibians were observed in the park and the numbers of each species were drastically reduced There has been a substantial decline in the numbers of amphibians worldwide and global pollution of water and air is clearly implicated The decline of amphibians in Xanadu National Park however almost certainly has a different cause in 1975 troutmdashwhich are known to eat amphibian eggsmdashwere introduced into the park

Write a response in which you discuss what specific evidence is needed to evaluate the argument and explain how the evidence would weaken or strengthen the argument

Evidence needed the identity of the sender and the census taker of rsquo75 and lsquo02mdasha scientist or an environmentalist credibility issue census methodology vs mere observationmdashthe absence of evidence is not an evidence of absence the season of when the census was taken time-shiftmdashconditions may have changed worldwide decline may include Xanadu other species of predators that prey on amphibians because trout is only one species that prey on amphibians the number of troutmdashhave they increased significantly since rsquo75 First the author needs to be more overt about the credibility of the census and observation There were to accounts that notes the population of amphibiansmdashthe first a census and the second an observation The author needs to bolster his conclusion with the evidence that indicate that the census and second observation were done using scientific methodologies This could either could strengthen or weaken his claim In addition he

GRE AWA John박 박정어학원

needs to provide the specific season of when the census and observation occurred In this letter the writer is informing an editor of an environmental magazine that the number of amphibians was greatly reduced since 1975 and he points out the introduction of trout as the only reason for the decline However the author fails to provide crucial evidences that could strengthen or weaken his conclusion

The following appeared in a memorandum from the president of Hyper-Go Toy Company

Last year sales of our Fierce Fighter toy airplane declined sharply even though the toy had been a top seller for three years Our customer surveys show that parents are now more worried about youthful violence and are concerned about better education for their children Therefore to maintain profits we should discontinue all our action toys and focus exclusively on a new line of educational toys Several other toy companies have already begun marketing educational toys and report sales increases last year of 200 percent And since the average family income is growing sales of new Hyper-Go toys should also increase

Write a response in which you discuss what specific evidence is needed to evaluate the argument and explain how the evidence would weaken or strengthen the argument

First the president indicates that the sales of Fierce Fighter toy airplane declined sharply However he fails to consider the fact that toys are a fad Since FFT enjoyed a three year success it may be natural that the trend would subsideSecond Customer survey is this representative of most toy consumers This trend is not newThird other companies may have profited not through educational but other toys Or educational toy profit may be small in proportion to their sale of other toys 200Parents are not the customers companies should concentrate on what the children want to increase profitLastly parents worry about youthful violence and concern for better education are not new trends

The presidentrsquos decision to discontinue all action toys and focus only on educational toys is too extreme If this decision is taken into effect its procedure may be an onerous task because it would require major shifts in human resources and company image Therefore further data should be considered before following up on this decision

우리 회사에서 지난 3년간 최고 매출을 일으켰던 Fierce Fighter 장난감 비행기가 지난해에는 매출이 급격히 떨어졌다 자체 고객 조사에서는 부모들이 현재 청소년 폭력에 걱정을 하고 있어서 아이들의 양질 교육에 더 관심을 가지고 있는 것으로 나타났다 따라서 회사의 수익을 유지하기 위해서는 모든 자사 전투용 장난감 생산을 중단하고 오로지 교육적인 장난감 생산에 집중해야 한다 몇몇 여타 장난감 회사들도 이미 교육용 장난감 마케팅을 시작해서 지난해에는 200의 매출신장을 가져왔다고 한다 그리고 평균 가계 수입이 점점 늘고 있기 때문에 신형 Hyper-Go 장난감의 매출도 늘어날 것이다

3그룹 Time-shift Error

Woven baskets characterized by a particular distinctive pattern have previously been found only in the immediate vicinity of the prehistoric village of Palea and therefore were believed to have been unique to the Palean people Recently however archaeologists discovered such a Palean basket in Lithos an ancient village across the Brim River from Palea The Brim River is very deep and broad and so the ancient Paleans could only have crossed it by boat but there is no evidence that the Paleans had boats And boats capable of carrying groups of people and cargo were not developed until thousands of years after the Palean people disappeared Moreover Paleans would have had no need to cross the rivermdashthe woods around Palea are full of nuts berries and small game It follows that the so-called Palean baskets were not unique to Palea

GRE AWA John박 박정어학원

Write a response in which you discuss what specific evidence is needed to evaluate the argument and explain how the evidence would weaken or strengthen the argument

RefutationPrehistoric time shift-gtbrim river could have been narrow and shallow or it might have not existed Indigenous patterns may exist in other disconnected remote placesNuts berries small game(hunting) may not have existed or the author should be proved these existed at that time Abundance of resources doesnrsquot support the reason for seclusionBoats not yet found baskets may have been carried across by the river current without the help of a boat Lithos might have crossed the river for commercial purposeOne Palean basket does not substantiate the authorrsquos claimThe absence of evidence is not an evidence of absence

The author assumes without justification that present conditions are the same as at the prehistoric era The author unfairly infers from the presence of Brim River which exist today that it would have existed in the past However the author fails to offer any evidence to substantiate this inference It is very likely that the Brim River might not have existed in prehistoric times or if it did exist may have been shallow and narrow enough for the Paleans to easily cross Any of these scenarios if true would serve to undermine the claim thathelliphelliphelliphelliphellip

First the author claims that the Brim River was very deep and broad so the Paleans could not have crossed it However the author fails to offer any evidence to substantiate that this was true in the prehistoric time of the Paleans For all we know the Brim River might not have existed in the prehistoric era or if it did exist could have been a shallow and narrow river For example scientists believe that thousands of years ago an ice-bridge existed on the Bering Sea connecting Eurasia to what is now North America to explain how the Eskimos and the Asian inhabitants of America came to migrate all over the Americas In this example the absence of evidence is not an evidence of absence Therefore to sufficiently support his claim the author needs to substantiate the fact that the Brim River really did exist and was broad and wide in the Prehistoric Era proving that the Paleans could not have influenced or traded with other groups of people

First the author claims that the Brim River was very deep and broad so the Paleans could not have crossed it However the author fails to offer any evidence to substantiate that this was true in the prehistoric times of the Paleans For all we know the Brim River might not have existed in the prehistoric era or if it did exist could have been a shallow and narrow river For example rivers are created by natural erosion over thousands of years Niagara Falls carved its way from the mouth of Lake Ottawa and created a long river Likewise the current Brim Riverrsquos physical features may not have been wide and deep Therefore to sufficiently support his claim the author needs to substantiate the fact that the Brim River really did exist and was broad and wide in the Prehistoric Era proving that the Paleans could not have influenced or traded with other groups of people

예전에는 실로짠 특이한 무늬 바구니가 Palea의 선사시대 지역의 인근마을에서만 발견되어왔기 때문에 Palea 마을 사람들의 특징이라고 여겨졌었다 그러나 최근들어 고고학자들이 Lithos지역에서 Palean 바구니를 발견하였는데 그 당시 지역은 Brim 강을 가로질러 Palea까지 닿아있었다 이 강은 수심이 아주 깊고 강폭이 넓었으며 때문에 고대의 Palea인들은 배를 이용해서 강을 건널수 있었을 것이다 그러나 이들이 배를 가지고 있었다는 증거는 발견되지 않고 있다 더군다나 이들이 멸명한 이후 수천년이 지난뒤에도 수많은 물자와 사람을 실어 나를수 있는 용적을 가진 배는 개발되지 않았다 이와더불어 Palea인들은 강을 건널필요가 없었는데 그것은 너트나무 장과열매 그리고 작은 사냥감들이 주변숲에 풍부했기때문이다 따라서 Palean 바구니라고 하는 것도 Palea인들만의 전유물이 아니라는 결론을 얻을 수 있다

결론 if follows that the so-called Palean baskets were not unique to Palea이번문제는 굿이 causal Bad analogy 로 구분해서 찾기가 힘드내요 배를 발견했다는 증거가 없는것이다 (앞으로도 발견될수 있음)

GRE AWA John박 박정어학원

계절의 영향으로 겨울에 얼음이 두껍게 언다든지 여름에 가뭄으로 인해서 건널수 있다물자가 풍부한 것이 이동하지 않을 조건이 아니다 다른 것에 의해서 이동가능(의약품등)

Thirteen years ago researchers studied a group of 25 infants who showed signs of mild distress when exposed to unfamiliar stimuli such as an unusual odor or a tape recording of an unknown voice They discovered that these infants were more likely than other infants to have been conceived in early autumn a time when their mothers production of melatonin hormone known to affect some brain functions would naturally increase in response to decreased daylight In a follow-up study conducted earlier this year more than half of these children now teenagers who had shown signs of distress identified themselves as shy Clearly increased levels of melatonin before birth cause shyness during infancy and this shyness continues into later life

Write a response in which you examine the stated andor unstated assumptions of the argument Be sure to explain how the argument depends on these assumptions and what the implications are for the argument if the assumptions prove unwarranted

Any baby exposed to unpleasant stimuli would react in such wayFirst of all the author states 25 infants as his evidence However this research sample is too small to prove his claimSecond the author states that 25 infants were conceived in early autumn which he claims lead to a shy disposition However this is faulty evidenceThird the research study was a long term study done in the span of 13 years However the author only writes about the initial and final stages of the study and leaves out evidences of what could have happened during the 13 years which could be more evidential factors of influenceFourth neither the infantsrsquo genetic predisposition nor their environment were taken into accountFinally the author concludes that his shyness continues into later life (Other factors could alter this disposition epigenetic theory)

13 년전 학자들은 25명의 유아를 대상으로 이상한 냄새나 특이한 소리를 녹음한 테잎등으로 낯선 자극을 주었을때 보이는 미미한 압박감 증상을 조사하였다 이들은 성숙기가 막 지났을 즈음에 보통의 유아들이 비슷한 증상을 보이는 정도 보다는 다소 민감한 반응을 보였는데 이 시기는 아이의 엄마가 뇌의 일부 기능에 영향을 미치는 것으로 알려진 멜라토니아 호르몬을 생산하는 시기로써 이 호르몬은 낯 시간이 짧을때 자연적으로 증가할 수도 있다 금년초에 실시된 추가연구에서 현재 10대로 성장한 당시 조사대상의 절반 이상의 아이들이 부끄럼을 잘타는 것으로 여기고 있었다 따라서 분명한 것은 출산전 멜라토닌 수치의 증가가 유아기에 수줍음 등의 영향을 미치게 되며 이러한 영향이 성장후에도 작용한다는 것이다

주장 Clearly increased levels of melatonin before birth cause shyness during infancy and this shyness continues into later life

1 25명의 아기로 결론 내리기에 샘플이 작다2 과학적 사실들에 대한 명확한 근거가 엇음3 다른 영향을 간과했다(다른 호르몬에 의한 영향 후천적인 성격형성의 영향)

GRE AWA John박 박정어학원

The following is a letter to the editor of the Atticus City newspaper

Former Mayor Durant owes an apology to the city of Atticus Both the damage to the River Bridge which connects Atticus to Hartley and the traffic problems we have long experienced on the bridge were actually caused 20 years ago by Durant After all he is the one who approved the construction of the bridge If he had approved a wider and better-designed bridge on which approximately the same amount of public money would have been spent none of the damage or problems would have occurred Instead the River Bridge has deteriorated far more rapidly over the past 20 years than has the much longer Derby Bridge up the river Even though the winters have been severe in the past several years this is no excuse for the negligence and wastefulness of Durant

Write a response in which you discuss what questions would need to be answered in order to decide whether the recommendation is likely to have the predicted result Be sure to explain how the answers to these questions would help to evaluate the recommendation

전임 시장인 Durant 씨는 Atticus 시에 대해 사과할 의무가 있습니다 Atticus와 Hartley를 잇는 River Bridge 교량에 대한 피해와 이 교량에서 오랫동안 주민들이 겪어오고 있는 교통 혼잡 문제들은 실제로 20년 전부터 시작된 것이었습니다 결정적으로 그가 교량 공사를 허가했던 바로 그 장본인입니다 당시 비슷한 공사비용으로 폭이 더 넓고 튼튼하게 설계된 교량을 허가했다면 이러한 문제나 피해는 발생하지 않았을 겁니다 더군다나 이 다리는 지난 20년 동안 상류에 건설된 훨씬 오래된 Derby 다리보다도 빠르게 부식되어 갔습니다 지난 수년동안 심지어 혹한이 있었다 하더라도 이러한 태만과 국고 손실에 대한 책임을 회피할 길이 없는 것입니다

결론 Former Mayor Durant owes an apology to the city of Atticus

1 디자인이 문제가 아닐수 있다 (디자인은 좋았으나 건설과정에 문제가 있었을 수 있다)2 그 당시의 시예산이 적어서 더 큰 다리를 짓기가 불가능했을 수도 있다3 교통량이 많거나 다른 상황으로 인해서 부식이 빨리 됐을 수 있다4 그 당시에는 최선의 선택이였지만 갑자기 변한 상황에 의해서 이런 문제점들이 발생했을 수 있다

GRE AWA John박 박정어학원

4그룹 거짓인과관계 오류 (False Cause) 빈출

Fifteen years ago Omega University implemented a new procedure that encouraged students to evaluate the teaching effectiveness of all their professors Since that time Omega professors have begun to assign higher grades in their classes and overall student grade averages at Omega have risen by 30 percent Potential employers looking at this dramatic rise in grades believe that grades at Omega are inflated and do not accurately reflect student achievement as a result Omega graduates have not been as successful at getting jobs as have graduates from nearby Alpha University To enable its graduates to secure better jobs Omega University should terminate student evaluation of professors

Write a response in which you discuss what specific evidence is needed to evaluate the argument and explain how the evidence would weaken or strengthen the argument

Omega professor evaluation implemented 15 years ago =gt Omega prof assign higher grades 30Employers believe therersquos grade inflation

Thus unsuccessful employment than AlphaTherefore to secure jobs Omega should end evaluating profs

Specific evidence neededRelationship between higher grades and evaluationRelationship between GPA and unsuccessful employmentAlpharsquos education could just be better than OmegaldquoFifteen years agordquo is a long time other factors could have influenced Why is the inflation a problem just now How much is Alpha better Is the comparison just How much gap is thereOmegarsquos student could just be doing better in their studiesComparison to other universities다른 대안 없나hellip Could Omega alleviate the employment problem by implementing a different procedure or program

15 년전 우리 대학은 학생들로 하여금 교수평가를 하도록 한 새로운 조치를 시행했었습니다 이후 교수들은 자신의 학과 학생들에게 높은 학점을 주었으며 그에따라 학생들의 전체 평점이 30나 올랐습니다 외부의 기업체들은 분명 점수가 지나치게 부풀려졌다고 믿고 있습니다 결국 본 대학 졸업생들이 인근 Alpha 대학의 졸업자들보다 구직률이 떨어지는 이유를 잘 보여주고 있는 것입니다 이를 해결하기 위해 이제부터는 학생들에 의한 교수평가제를 중단해야 합니다

결론 Omega University should now terminate student evaluation of professors

반박 교수 평가와 학점 인플레의 연관성이 적다( 교수 평가를 먼저하고 학점을 나중에 매길수도 있다)채용기준에 성적만 있는게 아니다 학업성취의 결과 일수도 있다 Alpha 가 원래 유능했다 Alpha 의 교육내용이 좋았다

GRE AWA John박 박정어학원

In this memo the dean of Omega University(OU) recommends OU to terminate professor evaluation to secure better jobs for the students To support this recommendation the dean offers several reasons However this argument contains several logical flaws which render it unconvincing

A threshold problem with the argument involves the voluntary nature of the evaluationprocedure The dean provides no evidence about the number or percentage of Omegastudents who participate in the procedure Lacking such evidence it is entirely possible thatthose numbers are insignificant in which case terminating the procedure is unlikely to haveany effect on the grade average of Omega students or their success in getting jobs aftergraduationThe argument also assumes unfairly that the grade-average increase is the result of theevaluation procedure--rather than some other phenomenon The dean ignores a host of otherpossible explanations for the increase--such as a trend at Omega toward higher admissionstandards or higher quality instruction or facilities Without ruling out all other possibleexplanations for the grade-average increase the dean cannot convince me that by terminatingthe evaluation procedure Omega would curb its perceived grade inflation let alone help itsgraduates get jobsEven if the evaluation procedure has resulted in grade inflation at Omega the deans claimthat grade inflation explains why Omega graduates are less successful than Alpha graduatesin getting jobs is unjustified The dean overlooks a myriad of other possible reasons forOmegas comparatively poor job-placement record Perhaps Omegas career services areinadequate or perhaps Omegas curriculum does not prepare students for the job market aseffectively as Alphas In short without accounting for other factors that might contribute toOmega graduates comparative lack of success in getting jobs the dean cannot justify theclaim that if Omega curbs its grade inflation employers will be more likely to hire OmegagraduatesFinally even if the dean can substantiate all of the foregoing assumptions the deansassertion that Omega must terminate its evaluation procedure to enable its graduates to findbetter jobs is still unwarranted in two respects First the dean ignores other possible ways bywhich Omega can increase its job-placement record--for example by improving its publicrelations or career-counseling services Second the dean unfairly equates more jobs withbetter jobs In other words even if more Omega graduates are able to find jobs as a result ofthe deans recommended course of action the kinds of jobs Omega graduates find would notnecessarily be better onesIn sum the deans argument is unpersuasive as it stands To strengthen it the dean mustprovide better evidence that the increase in grade average is attributable to Omegasprofessor-evaluation procedure and that the end result is a perception on the part ofemployers that Omega graduates are less qualified for jobs than Alpha graduates To betterassess the argument I would need to analyze 15-year trends in (l) the percentage of Omegastudents participating in the evaluation procedure (2) Omegas admission standards andquality of education and (3) Omegas emphasis on job training and career preparation I wouldalso need to know what other means are available to Omega for enabling its graduates to findbetter jobs

GRE AWA John박 박정어학원

The following appeared in a memo from a vice president of Quiot Manufacturing

During the past year Quiot Manufacturing had 30 percent more on-the-job accidents than at the nearby Panoply Industries plant where the work shifts are one hour shorter than ours Experts say that significant contributing factors in many on-the-job accidents are fatigue and sleep deprivation among workers Therefore to reduce the number of on-the-job accidents at Quiot and thereby increase productivity we should shorten each of our three work shifts by one hour so that employees will get adequate amounts of sleep

Write a response in which you examine the stated andor unstated assumptions of the argument Be sure to explain how the argument depends on these assumptions and what the implications are for the argument if the assumptions prove unwarranted

The following appeared in a memo from a vice president of Alta Manufacturing

During the past year Alta Manufacturing had thirty percent more on-the-job accidents than nearby Panoply Industries where the work shifts are one hour shorter than ours Experts believe that a significant contributing factor in many accidents is fatigue caused by sleep deprivation among workers Therefore to reduce the number of on-the-job accidents at Alta we recommend shortening each of our three work shifts by one hour If we do this our employees will get adequate amounts of sleep

Write a response in which you discuss what questions would need to be answered in order to decide whether the recommendation and the argument on which it is based are reasonable Be sure to explain how the answers to these questions would help to evaluate the recommendation

The following appeared in a memo from the vice president of Butler Manufacturing

During the past year workers at Butler Manufacturing reported 30 percent more on-the-job accidents than workers at nearby Panoply Industries where the work shifts are one hour shorter than ours A recent government study reports that fatigue and sleep deprivation among workers are significant contributing factors in many on-the-job accidents If we shorten each of our work shifts by one hour we can improve Butler Manufacturings safety record by ensuring that our employees are adequately rested

1 Write a response in which you discuss what specific evidence is needed to evaluate the argument and explain how the evidence would weaken or strengthen the argument

2 Write a response in which you discuss what questions would need to be answered in order to decide whether the recommendation is likely to have the predicted result Be sure to explain how the answers to these questions would help to evaluate the recommendation

4번 반복됨

Alta has 30 more job accidents than Panoply(work shifts one hour shorter)Experts Job accidents caused by fatigue and sleep deprivationTherefore to reduce job accidents and increase productivity shorten three work shifts by one hour for adequate sleep

지난해 우리 회사는 인근 Panoply Industries보다 업무상 재해가 30나 더 많았다 그 회사는 우리보다 근무 교대시간이 1시간 정도 짧았다 전문가들은 대부분의 업무상 재해에 있어서 가장 중요한 요인이 과로와 수면부족으로 보고있다 따라서 우리 회사에서 높은 산업재해를 줄이고 아울러 생산성을 높이기 위해서는 근로자들이 충분한 수면을 취할 수 있도록 1시간씩 3교대 시간을 줄여야 한다

In this memo the (author) vice president of Alta Manufacturing (AM) recommends that to reduce on-the-job accidents and increase productivity AM should shorten its three work shifts by one hour so that employees can

GRE AWA John박 박정어학원

get more sleep To support this recommendation the author provides several evidences However careful scrutiny of each of the facts reveals that it provides little credible support for the authorrsquos recommendation QuestionsThe number of accidents What kind of accidents The seriousness of the accidents is importantHow many employees are in each company What are their productsFalse cause Sleep may not be the reason for the on-the-job accidents What do Alta and Panoply manufacture

First of all the author believes that fatigue caused the on-the job accidents However there could be other reasons The author observes a correlation between sleep deprivation and on-the-job accidents then concludes that the former is the cause of the latter However the author fails to rule out other possible explanations For example it is entirely possible that Alta factories require more strenuous and dangerous labor than Panoply Without ruling out all other such factors it is unfair to conclude that fatigue is responsible for the accidents In addition the work-shifts may not be the cause of the sleep deprivation and fatigue It is possiblehellip Thus the author should provide what exactly Panoply and Alta manufacture and more precise data about their working conditions to be more convincing

Shortening the shift by one hour does not necessarily lead to more sleep And is one hour enoughLess accidents does not mean increased productivity

결론 We should shorten each of out three work shifts by one hour

반박 경쟁사에 비해서 시간당 하는 업무량이 많아서 더욱 피곤할 수도 있다 시간이 문제가 아니라 노후된 시설 설비 자체의 문제 작업 자체가 원래 위험한 것이여서 사고가 많을 수도 있다 다른 회사는 더욱 많은 작업시간에도 불구하고 안정한 작업여건으로 인해서 사고율이 오히려 더 작을 수도 있다비교사의 재해감소가 다른 요인일수 있다(안전 교육 철저)줄인 시간이 피로회복이나 수면으로 연결 안될수 있음(술을 마실 수도 있고 그 시간에 휴식을 취하지 않고 다른일을 함으로써 더욱 피로해질수 있다)

This editorial recommends that Alta Manufacturing reduce its work shifts by one hour each inorder to reduce its on-the-job accident rate and thereby increase Altas productivity To supportthis recommendation the author points out that last year the number of accidents at Alta was30 greater than at Panoply Industries where work shifts were one hour shorter The authoralso cites certain experts who believe that many on-the-job accidents are caused by fatigueand sleep deprivation I find this the argument unconvincing for several reasonsFirst and foremost the author provides absolutely no evidence that overall workerproductivity is attributable in part to the number of on-the-job accidents Although commonsense informs me that such a relationship exists the author must provide some evidence ofthis cause-and-effect relationship before I can accept the authors final conclusion that theproposed course of action would in fact increase Altas productivitySecondly the author assumes that some accidents at Alta are caused by fatigue or sleepdeprivation However the author overlooks other possible causes such as inadequateequipment maintenance or worker training or the inherent hazards of Altas manufacturingprocesses By the same token Panoplys comparatively low accident rate might be attributablenot to the length of its work shifts but rather to other factors such as superior equipmentmaintenance or worker training In other words without ruling out alternative causes ofon-the-job accidents at both companies the author cannot justifmbly conclude that merely byemulating Panoplys work-shift policy Alta would reduce the number of such accidentsThirdly even assuming that Altas workers are fatigued or sleep-deprived and that this is thecause of some of Altas on-the-job accidents in order to accept the authors solution to thisproblem we must assume that Altas workers would use the additional hour of free time tosleep or rest However the author provides no evidence that they would use the time in thismanner It is entirely possible that Altas workers would use that extra hour to engage in someother fatiguing activity Without ruling out this possibility the author cannot convincinglyconclude that reducing Altas work shifts by one hour would reduce Altas accident rateFinally a series of problems with the argument arise from the scant statistical information onwhich it relies In comparing the number of accidents at Alta and Panoply the author fails toconsider that the per-worker accident rate might reveal that Alta is actually safer than Panoplydepending on the total number of workers at each company Second perhaps accident rates

GRE AWA John박 박정어학원

at the two companies last year were aberrations and during other years Altas accident ratewas no greater or even lower than Panoplys rate Or perhaps Panoply is not representativeof industrial companies generally and that other companies with shorter work shifts have evenhigher accident rates In short since the argument relies on very limited statistical information Icannot take the authors recommendation seriouslyIn conclusion the recommendation for emulating Panoplys work-shift policy is not wellsupported To convince me that shorter work shifts would reduce Altas on-the-job accidentrate the author must provide clear evidence that work-shift length is responsible for some ofAltas accidents The author must also supply evidence to support her final conclusion that alower accident rate would in fact increase overall worker productivity

The following appeared in a memo from the vice president of marketing at Dura-Sock Inc

A recent study of our customers suggests that our company is wasting the money it spends on its patented Endure manufacturing process which ensures that our socks are strong enough to last for two years We have always advertised our use of the Endure process but the new study shows that despite our socks durability our average customer actually purchases new Dura-Socks every three months Furthermore our customers surveyed in our largest market northeastern United States cities say that they most value Dura-Socks stylish appearance and availability in many colors These findings suggest that we can increase our profits by discontinuing use of the Endure manufacturing process

1 Write a response in which you examine the stated andor unstated assumptions of the argument Be sure to explain how the argument depends on these assumptions and what the implications are for the argument if the assumptions prove unwarranted

2 Write a response in which you discuss what specific evidence is needed to evaluate the argument and explain how the evidence would weaken or strengthen the argument

3 Write a response in which you discuss what questions would need to be answered in order to decide whether the recommendation and the argument on which it is based are reasonable Be sure to explain how the answers to these questions would help to evaluate the recommendation

Intro The vice president of marketing at Dura-Sock Inc is offering a potentially harmful investment recommendation by claiming that Dura-Sock should discontinue its use of the ldquoEndurerdquo process To support his recommendation he points out a study that Dura-Sock customers actually purchase the socks every three months and a survey that reveals that Dura-Sock customers like the sockrsquos stylish appearance and availability in many colors The study and survey however are insufficient in supporting his proposal and the VP makes several unwarranted assumptionsIntro (simplified) The VP states that though Dura-Socks last for two years customers buy the socks every three months Therefore he assumes that the consumersrsquo motive for buying the produce is not its durabilityHowever the author fails to rule out other possible motivation for consumption

Even if the survey is reliable the author should consider the rest of the market Vague terms ldquowasting moneyrdquomdashprecisely how much are they wasting Studysurvey errorThe company must calculate the outcome of such momentous decisionStudy participantsrsquo comment that they prefer Dura-Sock for its stylishness and availability might take Dura-Sockrsquos enduring quality for granted

우리회사 제품 소비자들에 대한 최근 조사에서 지난 2년여간 양말의 내구성을 강하게 하는 필수공정이었던 자사 특허의 Endure 공정에 들어가는 비용이 낭비라고 말하고 있다 우리 회사는 항상 이 공정 처리에 대한 광고를 내보냈으나 이에 대한 시장 조사에서 실제로 고객들은 이 신제품을 평균 석달마다 구매하는 것으로 나타났다 더군다나 북동부지역에서 실시한 대규모 시장조사에 응답한 고객들은 양말의 모양과 색상등에 더

GRE AWA John박 박정어학원

관심을 나타냈다 이러한 결과는 우리회사가 신기술 공법을 중단하면 그에 따라 수익이 늘어날 것이라는 것을 말해주고 있는 것이다주장 These findings suggest that Dura0Sock can increase its profits by discontinuing its use of the ldquoEndurerdquo manufacturing process

1 survey가 정확한 소비자의 의견을 나타낸 것인가 다른 선택없이 양자택일과 같은 방법의 survey였는지2 북동부지역의 시장조사가 전체 의견을 대표할 수 있나3 사람들이 모양이나 색상에 앞서 내구성을 먼저 평가했을 수도 있다 내구성을 갖추었다는 전제하에 모양과 색상에 관심을 드러낸 것일 수 있다4 소비자가 도매상(retail)인지 소매상(whole)인지가 없다

The following appeared in a business magazine

As a result of numerous complaints of dizziness and nausea on the part of consumers of Promofoods tuna the company requested that eight million cans of its tuna be returned for testing Promofoods concluded that the canned tuna did not after all pose a health risk This conclusion is based on tests performed on samples of the recalled cans by chemists from Promofoods the chemists found that of the eight food chemicals most commonly blamed for causing symptoms of dizziness and nausea five were not found in any of the tested cans The chemists did find small amounts of the three remaining suspected chemicals but pointed out that these occur naturally in all canned foods

Write a response in which you discuss what questions would need to be addressed in order to decide whether the conclusion and the argument on which it is based are reasonable Be sure to explain how the answers to the questions would help to evaluate the conclusion

Representativeness of the tested cansThey should conduct a comparative studyThe testing could be biased because Promofoods employees conducted the testingHow much (quantity) of the five and three suspected chemicals were in the canned foodsFalse cause The substance that caused dizziness and nausea may not be one of the eight common chemicals

많은 소비자들의 현기증과 구역질 불만에 따라 Promofoods사는 지난해 참치 캔 8백만 개를 테스트하기 위해 반품시켰다 그 결과 캔에서는 건강에 위험이 될 수 있는 화합물질이 없었던 것으로 회사측은 결론지었다 이러한 결론은 회사측 화학연구자들이 회수된 캔의 샘플을 테스트해서 이들 증상의 원인이 되는 8가지 화합물 중에서 5가지가 실험된 캔에서 발견되지 않았다는 사실에 근거한 것이다 이들 화학자들은 나머지 3개가지 화합물이 모든 캔 식료품에서 흔히 발견되는 것이라고 언급했다 결론 Promofoods concluded that the cans did not after all contain chemicals that posed a health risk

1 공인된 기간에서 테스트를 한 것이 아니고 자사에서 직접 테스트를 했기에 신뢰성이 안간다 2 이런 증상을 일으키는 8개의 물질 말고 다른 물질들이 캔속에 많이 포함됬을수 있다 3 나머지 3개의 물질들의 함유량이 많아서 다른 종류의 캔들은 문제를 일으키지 않지만 참치캔은 문제를

일으킬 수 있다

This magazine article concludes that the 8 million cans of tuna Promofoods recalled due tocomplaints about nausea and dizziness do not after ail contain any chemicals that pose a

GRE AWA John박 박정어학원

health risk To support this conclusion the author cites the fact that five of eight chemicalscommonly causing these symptoms were not found in the recalled cans while the other threealso occur naturally in other canned foods For several reasons this evidence lends littlecredible support to the authors conclusionTo begin with the author relies partly on the fact that although three of the eight chemicalsmost commonly blamed for nausea and dizziness appeared in Promofoods recalled tunathese chemicals also occur naturally in other canned foods However this fact alone lends nosupport to the authors conclusion for two reasons First the author might be ignoring animportant distinction between naturally occurring chemicals and those not occurring naturallyIt is entirely possible that these three chemicals do not occur naturally in Promofoods tunaand that it is for this reason that the chemicals cause nausea and dizziness Secondly it isentirely possible that even when they occur naturally these chemicals cause the samesymptoms Unless the author rules out both possibilities he cannot reliably conclude that therecalled tuna would not cause these symptomsAnother problem with the argument is that the authors conclusion is too broad Based onevidence about certain chemicals that might cause two particular heath-related symptoms theauthor concludes that the recalled tuna contains no chemicals that pose a health risk Howeverthe author fails to account for the myriad of other possible health risks that the recalled tunamight potentially pose Without ruling out all other such risks the author cannot justifiablyreach his conclusionA third problem with the argument involves that fact that the eight particular chemicals withwhich the test was concerned are only the eight most commonly blamed for nausea anddizziness It is entirely possibly that other chemicals might also cause these symptoms andthat one or more of these other chemicals actually caused the symptoms Without ruling outthis possibility the author cannot jusufiably conclude that the recalled tuna would not causenausea and dizzinessA final problem with the argument involves thetesting procedure itself The author providesno information about the number of recaUed cans tested or the selection method used Unlessthe number of cans is a sufficiently large sample and is statistically repre sentative of all therecalled cans the studys results are not statistically reliableIn conclusion the article is unconvincing as it stands To strengthen the assertion that therecalled tuna would not cause nausea and dizziness the author must provide evidence thatthe three chemicals mentioned that occur naturally in other canned foods also appear naturallyin Promofoods tuna The author must also provide evidence that ingesting other canned foodscontaining these three chemicals does not cause these symptoms To better evaluate theargument we would need to know whether the sample used in the tests was statisticallysignificant and representative of all the recalled tuna We would also need to know what otherchemicals in the recalled tuna might pose any health risk at all

5그룹 불충분 조건오류 빈출

Natures Way a chain of stores selling health food and other health-related products is opening its next franchise in the town of Plainsville The store should prove to be very successful Natures Way franchises tend to be most profitable in areas where residents lead healthy lives and clearly Plainsville is such an area Plainsville merchants report that sales of running shoes and exercise clothing are at all-time highs The local health club has more members than ever and the weight training and aerobics classes are always full Finally Plainsvilles schoolchildren represent a new generation of potential customers these schoolchildren are required to participate in a fitness-for-life program which emphasizes the benefits of regular exercise at an early age

Write a response in which you examine the stated andor unstated assumptions of the argument Be sure to

GRE AWA John박 박정어학원

explain how the argument depends on these assumptions and what the implications are for the argument if the assumptions prove unwarranted

False cause

First of all the author believes that the Increased sales of running shoes and exercise clothing indicates

Plainesville residentsrsquo interest in leading healthy lives However this assumption is not logically convincing for

several reasons could be a fashion trendTime shift ldquoFitness for liferdquo might not have any influence on schoolchildren as they growFalse cause There could be other reasons for member increase in the health clubAll of the above are insufficient condition

The author has to prove that local residents are interested in leading healthy lives However he supports his conclusion with insufficient evidence Nevertheless even if the residents are concerned with health naturersquos way may not be successful First

그 동안의 경험을 토대로 볼 때 건강생활과 밀접히 관련되어 있는 거주 지역에서 본 상점들이 아주 호응을 얻고 있다 따라서 이러한 주민들이 많이 거주하고 있는 Plainsville 에 새로운 상점들을 계속 세워야 한다 이 지역 상인들은 런닝화와 운동복 판매가 가장 높다고 말한다 불과 5 년전에는 거의 전무하다시피하던 지역 헬스 클럽의 경우도 엄청나게 많은 회원을 확보하고 있으며 웨이트 트레이닝과 에어로빅 강좌들도 항상 만원이라고 한다 새로운 고객층을 예측해 보는 것도 가능하다 이 지역의 학생들의 경우 Fitness for Life프로그램을 받게 되는데 이러한 프로그램을 통해서 유년시절부터 정규적인 운동 습관을 들이게 하고 있는 것이 그것이다

결론 We should therefore build our next new store in Plainsville

반박 그동안의 경험에 의한 과거 통계가 꼭 여기에도 적용되는건 아니다 5 년전 헬스 클럽이 잘 안되었던게 다른 원인이였을수 있다(강사수준 미달 강좌미비)tourist 에 의한 원인 일수 있다 어렸을때부터 운동을 했다고 해서 커서도 관심이 있지는 않다 (오히려 반감이 있을수 있다 혹은 건강하기에 건강에 관심이 적을수도 있다)운동복이나 신발의 판매가 육체노동에 의한 것일수도 있다

IntroductionSupport1049896In this memorandum the author asserts that Naturersquos Way should build its next newstore in Plainsville To support this assertion the author states that Plainsvillesmerchantsrsquo sales of exercise clothing are going well the local health club has moremembers than ever and a new generation of customers will help to ensure NaturersquosWayrsquos success At first glance the authorrsquos assumption seems convincing but in-depth scrutiny revealsthat it lacks substantial evidence as it stands

Body 1-SamplingTopic Sentence 1To begin with the author assumes that the merchantsrsquo report indicates that the residentsare concerned about their health However this assumption is based on unsubstantiated

GRE AWA John박 박정어학원

data Example 1 (Rebuttal1) First if we do not know the total volume of items sold and the price of the goods exactly we cannot infer whether the residents are actually buying many goods Example 2 (Rebuttal2)In addition to that the report emphasizes the rising sales of running shoes and exerciseclothing however these may not be hot-selling items for Naturersquos Way or may not be theproducts the company is planning to sell Concluding Sentence Therefore in order to make the argument reliable the author should reconsider themerchantsrsquo report with more detailed data

Body 2-CausalTopic Sentence 2Second the author contends that the health clubs classes are full yet this does not meanthat many people actually use the health club other factors may be the real cause forthose closed classes Example 1 (Rebuttal1) To begin with if the health club is very small the number of people working out wouldnot be a large one In fact regular gym-going may just be a vogue among a smallunrepresentative segment of Plainsvilles population Example 2 (Rebuttal2) Moreover it is possible that most of the people who exercise in the health club do weight training and aerobics only to look good and to meet other singles not for their health In that case there would be little demand for health products Concluding SentenceThus the author should not hasten to presume what really caused people to be interested in a healthier lifestyle and enroll in the health club

Body 3-Time-ShiftTopic Sentence 3Finally the author highlights that Naturersquos Way can expect a new generation of customersin Plainsville that will help the company in the long term This notion is mistaken in that itassumes the conditions of the present will continue unchanged in the future Although theschool children are required to participate in the fitness for life program they may notnecessarily buy Naturersquos Ways products Example 1 (Rebuttal1) In the first instance they may suffer a fall in purchasing power arising from future economic difficulties this would cause reluctance to spend a considerable amount of money on health products which tend to be more expensive Example 2 (Rebuttal2)Another possibility is that there may emerge many competitor companies vying with Naturersquos Way so that in the future the school children may not feel the necessity to purchase one companyrsquos health products over anotherrsquosConcluding Sentence Thus the authorrsquos assumption is highly speculative since it relies heavily on unknowablefuture circumstances

ConclusionThesis In sum the author uses many assumptions that are insufficient in supporting his claimsSupportIn order for the authorrsquos claims to be convincing he needs to advance more persuasiveevidence that people in Plainsville really are concerned with their health and health foodThe following was written as a part of an application for a small-business loan by a group of developers in the city of Monroe

A jazz music club in Monroe would be a tremendously profitable enterprise Currently the nearest jazz club is 65 miles away thus the proposed new jazz club in Monroe the C-Note would have the local market all to itself Plus jazz is extremely popular in Monroe over 100000 people attended Monroes annual jazz festival last summer several well-known jazz musicians live in Monroe and the highest-rated radio program in Monroe is Jazz Nightly which airs every weeknight at 7 PM Finally a nationwide study indicates that the typical jazz fan spends close to $1000 per year on jazz entertainment

1 Write a response in which you discuss what specific evidence is needed to evaluate the argument and explain how the evidence would weaken or strengthen the argument

2 Write a response in which you examine the stated andor unstated assumptions of the argument Be

GRE AWA John박 박정어학원

sure to explain how the argument depends on these assumptions and what the implications are for the argument if the assumptions prove unwarranted

3 Write a response in which you discuss what questions would need to be answered in order to decide whether the prediction and the argument on which it is based are reasonable Be sure to explain how the answers to these questions would help to evaluate the prediction

Group error nationwide survey may not reflect local trends Is the nationwide jazz fan population substantialInsufficient non-residents of Monroe may have attended the jazz festival (Body alternative explanation last year may have been an anomaly The author should consider data from various years) The author should indicate how many out of 100000 were Monroe residentsNationwide study Does this reflect Insufficient Citizens of Monroe may continue to go to the jazz club 65 miles away

Are the people in Monroe really interested in jazzMajority of the people who attended the jazz festival might not be Monroe residentsSurvey error nationwide study may not be applicable to MonroeJazz musicians who live in MonroeMonopolyRadio station

In this business application the author claims that the proposed jazz club C Note will be very profitable in Monroe To support this claim the author argues for his case with several evidences At first glance the authorrsquos argument seems convincing however careful scrutiny reveals that his argument in specious

To begin with the author claims that Monroersquos citizens are interested in jazz He presents three evidences First Secondhellip Thirdhellip Howeverhellip

Monroe 시에 있는 재즈 음악 클럽은 수익성이 좋은 사업이다 현재 가장 가까이에 있는 클럽은 65 마일 정도 떨어져 있다 따라서 이번에 세우려고 하는 C Note 는 독보적인 위치를 점할것이다 더군다나 재즈는 이 시에서 가장 인기있는 음악이다 지난 여름 재즈 축제에서는 10 만명 이상의 Morone 시 주민이 참석하였고 몇몇 유명한 재즈 음악가들도 이곳에 살고 있으며 저녁때 방영되는 라디오 프로그램중에서 최고의 시청률을 보이고 있는 것도 Jazz Nightly 이다 전국조사에서도 전형적인 재즈 팬들은 재즈 분야에 년간 1천 달러 가까이 지출하고 있는 것으로 보고되고 있다 따라서 C Note 클럽이 돈을 벌 수 있는 사업이라는 것은 확실한 것이다

결과 It is clear that the C Note cannot help but make money반박 nearest jazz club 이 양질의 써비스로 여전히 손님을 끌수도 있다Festival 에 얼마나 참여하는지가 jazz 의 인기를 반영하지 않는다 뮤지션이 많이 사는거랑 jazz 의 인기가 상관없다라디오 프로그램이 다른 요인에 의해서 인기일수도 있다 (진행자때문)전국 통계 적용 불가화목 실전반_Ms Noh6In this application the author suggests that a jazz club in Monroe will make a number of profits To support this suggestion the author exemplifies the local condition popularity of jazz in Monroe and nationwide study However careful scrutiny of each of the facts reveals that it provides little credible support for the authorrsquos recommendation Good clear intro

First the author assumes that jazz is popular in Monroe because of several facts the jazz festival last year had high participation some famous jazz musicians live in Monroe and the high-rated radio program is lsquoJazz Nightlyrsquo However this assumption has many drawbacks that must be seriously considered(Good topic sentences) If many attendants in the last-yearrsquos festival came from other cities and not Monroe it is hard to conclude that Monroersquos people like jazz Therefore the author must examine how many Monroe residents actually attended the festival On top of that there is little relationship between habitation of famous jazz musician and the popularity of jazz in Monroe Although several well-known musicians live there if they do not take part in any jazz performance of Monroe this might have no effect to the interest of Monroersquos residents

GRE AWA John박 박정어학원

about jazz Finally in the case of radio program this is also not suitable reason why jazz is popular in Monroe It might be possible that people cannot help choosing lsquoJazz Nightlyrsquo because there are few radio programs at Night The fact that the radio program is the highest rating program is not a germane evidence The approximate number of listeners would be the more crucial evidence Therefore the author needs to seriously deliberate the correlation between jazzrsquos popularity in Monroe and his examples (Good logical flow and clarity)

Second the author uses as evidence the nationwide study that jazz fans spend much money on jazz entertainment to substantiate why starting a jazz club in Monroe will be profitable In other words the author assumes that the characteristics of a nationwide study can be applied to Monroe The national study would lend support to the applicantrsquos claim only if residents in Monroe typify national jazz fans However the author does not provide credible evidence that this is the case Moreover the populations of jazz fans nationwide may be insubstantial Thus the author should not infer hastily that Monroersquos residents will spend much money on enjoying jazz from the nationwide study

Lastly even if jazz is popular in Monroe C Note may not be successful It is entirely possible that residents might still prefer other clubs where they have always went In addition there is another possibility that the nearest jazz club will attract many of Monroersquos people because it serves fine performances and is equipped with favorite facilities Without considering these other possibilities the author cannot make his argument convincing In sum the author presents many reasons that are insufficient in supporting his or her claim In order for the authorrsquos claims to be convincing he needs to advance more persuasive evidence such as the total number of Monroe residents who attended the jazz festival the effects on the popularity of jazz by the musicians living in Monroe and the actual number of residents who would typify themselves to be jazz fans through a local survey Without substantial evidence that C Note will be successful in Monroe the businessmen may be overinvesting in what might lead to a business failureExcellent clarity Score 50

The following appeared in a newsletter offering advice to investors

Over 80 percent of the respondents to a recent survey indicated a desire to reduce their intake of foods containing fats and cholesterol and today low-fat products abound in many food stores Since many of the food products currently marketed by Old Dairy Industries are high in fat and cholesterol the companys sales are likely to diminish greatly and company profits will no doubt decrease We therefore advise Old Dairy stockholders to sell their shares and other investors not to purchase stock in this company

Write a response in which you discuss what questions would need to be answered in order to decide whether the advice and the argument on which it is based are reasonable Be sure to explain how the answers to these questions would help to evaluate the advice

Survey 80

GRE AWA John박 박정어학원

Old Dairy could change their products and manufacture low fat dairy foodsLess competing companies Old Dairy could eventually be the only company that produces hellipImprecise numbers and measurementsCustomers may still buy high fat dairy products

The author of the newsletter is offering potentially dangerous advice by recommending Old Dairy stockholders to withdraw investment and stop purchase What is more the authorrsquos prediction debases the reputation and business of Old Dairy and if false could devoid the investment opportunity of the newsletter readers Therefore investors should examine whether the authorrsquos evidences are substantial

To begin with the author states that 80 percent of the respondents in a survey indicated a desire to reduce their intake of foods He therefore argues that Old Dairyrsquos high fat and cholesterol products would decrease in sales However the author makes a crucial error in this argument First the author provides no evidence that the surveyrsquos results are statistically reliable Were they representative of all the customers Were they chosen for the survey randomly Furthermore the desire to reduce fat and cholesterol intake is a pervasive trend in todayrsquos opulent society however the author erroneously identifies this as a new phenomenon which will affect consumer trends Second having a desire to reduce fat and cholesterol intake does not necessarily indicate that people who have this desire will actually reduce consuming these types of products It is entirely possible that they may continue buying Old Dairy products for its quality and taste Accordingly the author cannot draw any firm conclusion that people will not buy Old Dairy products Therefore if any of these cases are true the author may be offering investors a detrimental investment advice

최근 조사에 대한 응답자중 80 이상이 자신이 먹는 음식에서 지방과 콜레스테롤의 함유량을 줄이고 싶다고 한다 아울러 요즘은 많은 식료품 가계에서 저지방 제품들을 많이 취급하고 있다 현재 Old Dairy Industries가 판매하고 있는 많은 음식제품들은 지방과 콜레스테롤이 높기 때문에 이 회사의 매출이 격감할 것으로 보이며 당연히 매출이익도 줄어들것이다 따라서 이 회사의 주주들은 주식을 매각하고 다른 주식 투자가들도 이 회사의 주식을 매입하지 않는 것이 좋다

결론 Old Dairy stockholders to sell their shares and other investors not to purchase stock in this company

반박 모든 상품이 다 고 지방 고 칼로리는 아니다(비록 많을지라도) 일부의 식품의 경우 기호에 맞어서 히트해서 전체적인 수입이 증가할 수도 있다국내시장만 생각할 수 없다( 외국시장에서 호황을 누릴수 있다 )입맛이라는게 즉각 바뀌는게 아니다

The following appeared in a letter to the editor of the Balmer Island Gazette

On Balmer Island where mopeds serve as a popular form of transportation the population increases to 100000 during the summer months To reduce the number of accidents involving mopeds and pedestrians the town council of Balmer Island should limit the number of mopeds rented by the islands moped rental companies from 50 per day to 25 per day during the summer season By limiting the number of rentals the town council will attain the 50 percent annual reduction in moped accidents that was achieved last year on the neighboring island of Seaville when Seavilles town council enforced similar limits on moped rentals

1 Write a response in which you discuss what questions would need to be answered in order to decide whether the recommendation is likely to have the predicted result Be sure to explain how the answers to these questions would help to evaluate the recommendation

2 Write a response in which you discuss what questions would need to be answered in order to decide whether the prediction and the argument on which it is based are reasonable Be sure to explain how the answers to these questions would help to evaluate the prediction

3 Write a response in which you examine the stated andor unstated assumptions of the argument Be sure to explain how the argument depends on these assumptions and what the implications are for the argument if the assumptions prove unwarranted

Whatrsquos the actual population of Balmer Island 100000mdashis this a significant increase What kind of accidents Skin abrasions or serious injury And compared to Seaville how serious are the accidents and the actual number of accidents Did Seaville enforce other restrictions like safety signsHow different are the conditions of Balmer

GRE AWA John박 박정어학원

and Seaville regarding population road (safety) conditions topography other town-government regulation How much will the economy of Balmer be affected do to this restriction Could it cause an economic recession due to the fact that these rental companiesrsquo chance to make money is only during the summer thereby weakening the economic infrastructure Are there any other ways that could better alleviate the accident rate

Statistics 50-impreciseAnalogy Balmer compared with TorseauFalse Cause Accidents might have occurred because of reasons other than mopeds False Cause population increase may not be part of the cause of the accidentsOther explanations for the accident pedestrians few road safety regulations narrow roadsThere could be other better solutionshellip

Balmer Island의 인구가 여름철에는 십만명으로 늘어난다 2륜차와 보행자간 사고를 줄이기 위해 시의회는 6개의 자전거를 포함한 2륜차 대여업체에게 이 기간동안에는 대여숫자를 일일 50에서 30으로 제한하도록 할 것이다 대여숫자를 줄임으로써 시 의회는 지난해 이웃한 Torseau섬에서 이와 동일한 규제를 시행해서 50나 줄인 결과를 보고 마찬가지로 50를 줄일수 있다고 확신하고 있다

결론 The town council of Balmer Island should linit the number

반박 보행자의 부실에 의해서 사고가 많이 일어날수도 있다렌탈수의 줄임만이 대책은 아니다(대부분의 사람들이 렌탈 보다는 소유하고 있을 수도 있다)옆섬과는 상황이 다를수도 있다(그 섬에서는 사고의 원인이 많은 자전거 수로 인한것일수있다) 하지만 이 섬은 좁은 도로가 원인일 수도 있고 도로 안전 장치의미비가 원일일수 있다

In this letter the author recommends that Balmer Island should limit the number moped rentals from 50 to 30 per day To support this recommendation the author points out several reasons However careful scrutiny of each of the facts reveals that it is filled with unanswered questions that could significantly weaken the authorrsquos recommendation with loops and holes which are answered

The recommendation depends on the assumption that no alternative means of reducing the number of accidents are available However the author fails to offer any evidence to substantiate this crucial assumption It is highly possible that means other than this would better solve the problem Perhaps they could widen the roads or put-up more safety signs Or perhaps the accidents were due to the lack of skills in which case proper safety training would significantly alleviate the problem Without considering and ruling out these and other alternative means of reducing accidetns the author cannot confidently conclude that merely emulating Torseau would suffice Moreover the author is advising a recommendation which could potentially harm the economy of Balmer Island sincehellip Moreover the Balmer Island should alternative means to reduce accidents because limiting moped rentals during the summer could harm the economy of Balmerhellip

First of all the author believes that increase in population and the number of moped rentals are responsible for the accidents It is entirely possible that other factors are responsible for the accidents Perhaps Balmer Islandrsquos lack of safety signs was a major factor Or maybe the roads are narrow and dangerous on the Island therefore the town council could enforce stricter traffic regulations to alleviate the problem Accordingly if either of these scenarios is true the author cannot draw any firm conclusion that increase in the number of population and moped rentals are the cause of the accidents

The author of this editorial recommends that to reduce accidents involving mopeds andpedestrians Balmer Islands city council should restrict moped rentals to 30 per day down from50 at each of the islands six rental outlets To support this recommendation the author citesthe fact that last year when nearby Torseau Islands town council enforced similar measuresTorseaus rate of moped accidents fell by 50 For several reasons this evidence providesscant support for the authors recommendationTo begin with the author assumes that all other conditions in Balmer that might affect therate of moped-pedestrian accidents will remain unchanged after the restrictions are enactedHowever with a restricted supply of rental mopeds people in Balmer might purchase mopedsinstead Also the number of pedestrians might increase in the future with more pedestriansespecially tourists the risk of moped-pedestrian accidents would probably increase For thatmatter the number of rental outlets might increase to make up for the artificial supplyrestriction per outlet--a likely scenario assuming moped rental demand does not declineWithout considering and ruling out these and other possible changes that might contribute to ahigh incidence of moped-pedestrian accidents the author cannot convince me that theproposed restrictions will necessarily have the desired effect

GRE AWA John박 박정어학원

Next the author fails to consider other possible explanations for the 50 decline inTorseaus moped accident rate last year Perhaps last year Torseau experienced unusually fairweather during which moped accidents are less likely Perhaps fewer tourists visited Tot seanlast year than during most years thereby diminishing the demand for rental mopeds to belowthe allowed limits Perhaps last year some of Torseaus moped rental outlets purchased newmopeds that are safer to drive Or perhaps the restrictions were already in effect but were notenforced until last year In any event a decline in Torseaus moped accident rate during onlyone year is scarcely sufficient to draw any reliable conclusions about what might have causedthe decline or about what the accident rate will be in years aheadAdditionally in asserting that the same phenomenon that caused a 50 decline in mopedaccidents in Torseau would cause a similar decline in Balmer the author relies on what mightamount to an unfair analogy between Balmer and Torseau Perhaps Balmers ability to enforcemoped-rental restrictions does not meet Torseaus ability if not then the mere enactment ofsimilar restrictions in Balmer is no guarantee of a similar result Or perhaps the demand formopeds in Torseau is always greater than in Balmer Specifically if fewer than all availablemopeds are currently rented per day from the average Balmer outlet while in Torseau everyavailable moped is rented each day then the proposed restriction is likely to have less impacton the accident rate in Balmer than in TorseauFinally the author provides no evidence that the same restrictions that served to reduce theincidence of all moped accidents by 50 would also serve to reduce the incidence ofaccidents involving mopeds and pedestrians by 50 Lacking such evidence it is entirelypossible that the number of moped accidents not involving pedestrians decreased by a greaterpercentage while the number of moped-pedestrian accidents decreased by a smallerpercentage or even increased Since the author has not accounted for these possibilities theeditorials recommendation cannot be taken seriouslyIn conclusion the recommendation is not well supported To convince me that the proposedrestriction would achieve the desired outcome the author would have to assure me that nochanges serving to increase Balmers moped-pedestrian accident rate will occur in theforeseeable future The author must also provide dear evidence that last years decline inmoped accidents in Torseau was attributable primarily to its moped rental restrictions ratherthan to one or more other factors In order to better evaluate the recommendation I wouldneed more information comparing the supply of and demand for moped rentals on the twoislands I would also need to know the rate of mopedpedestrian accidents in Torseau both priorto and after the restrictions were enforced in TorseauThe following appeared in a magazine article about planning for retirement

Clearview should be a top choice for anyone seeking a place to retire because it has spectacular natural beauty and a consistent climate Another advantage is that housing costs in Clearview have fallen significantly during the past year and taxes remain lower than those in neighboring towns Moreover Clearviews mayor promises many new programs to improve schools streets and public services And best of all retirees in Clearview can also expect excellent health care as they grow older since the number of physicians in the area is far greater than the national average

Write a response in which you discuss what specific evidence is needed to evaluate the argument and explain how the evidence would weaken or strengthen the argument

-Natural beauty and consistent climate may not be the most wanted qualities-Housing costs could have lowered on a national level wealthy retirees may not care about costs-Taxes may be high compared to the nationrsquos average tax rate-What about other qualities of Clearview Crime rate what qualities would retirees want -If schools streets and public services need improvement then this is proof that the current condition of Clearview is low Or due to budgetary reasons the mayor may not follow-up on his promise because of lowered tax rate -Schools and people who are retired no relationship-Physicians What kind of physicians Number is irrelevant Are these physicians capable of addressing the illnesses of old people

This author argues that anyone seeking a place to retire should choose Clearview To supportthis argument the article cites Clearviews consistent climate and natural beauty its fallinghousing costs its low property taxes compared to nearby towns and the mayors promise toimprove schools streets and services The article also claims that retirees can expectexcellent health care because the number of physicians in Clearview greatly exceeds thenational average This argument is flawed in several critical respectsTo begin with although consistent climate and natural beauty might be attractive to manyretirees these features are probably not important to all retirees For many retirees it isprobably more important to live near relatives or even to enjoy changing seasons Thus I

GRE AWA John박 박정어학원

cannot accept the authors sweeping recommendation for all retirees on this basisAlso Clearviews declining housing costs do not necessarily make Clearview the best placeto retire for two reasons First despite the decline Clearviews housing costs might be highcompared to housing costs in other cities Secondly for wealthier retirees housing costs arenot likely to be a factor in choosing a place to retire Thus the mere fact that housing costshave been in decline lends scant support to the recommendationThe articles reliance on Clearviews property-tax rates is also problematic in two respectsFirst retirees obviously have innumerable choices about where to retire besides Clear viewand nearby towns Secondly for retirees who are well-off financially property taxes are notlikely to be an important concern in choosing a place to retire Thus it is unfair to infer fromClearviews property-tax rates that retirees would prefer ClearviewYet another problem with the argument involves the mayors promises In light of Clearviewslow property-tax rates whether the mayor can follow through on those promises is highlyquestionable Absent any explanation of how the city can spend more money in the areas citedwithout raising property taxes I simply cannot accept the editorials recommendation on thebasis of those promises Besides even if the city makes the improvements promised thoseimprovements--particular the ones to schools--would not necessarily be important to retireesFinally although the number of physicians in Clearview is relatively high the per capitanumber might be relatively low Moreover it would be fairer to compare this per capita numberwith the per capita number for other attractive retirement towns--rather than the nationalaverage After all retirees are likely to place a relatively heavy burden on health-careresources Besides the article provides no assurances that the number of physicians inClearview will remain high in the foreseeable futureIn conclusion the recommendation is poorly supported To strengthen it the author mustconvince me--perhaps by way of a reliable survey--that the key features that the vast majorityof retirees look for in choosing a place to live are consistent climate natural beauty and lowhousing costs The author must also provide better evidence that Clear views property taxesare lower than the those of cities in other areas The author must also explain how the city canmake its promised improvements without raising property taxes Finally to better assess theargument I would need to now how the per capita number of physicians in Clearview wouldcompare to the national average in the futureThe following appeared as a letter to the editor from a Central Plaza store owner

Over the past two years the number of shoppers in Central Plaza has been steadily decreasing while the popularity of skateboarding has increased dramatically Many Central Plaza store owners believe that the decrease in their business is due to the number of skateboard users in the plaza There has also been a dramatic increase in the amount of litter and vandalism throughout the plaza Thus we recommend that the city prohibit skateboarding in Central Plaza If skateboarding is prohibited here we predict that business in Central Plaza will return to its previously high levels

Write a response in which you discuss what questions would need to be answered in order to decide whether the recommendation is likely to have the predicted result Be sure to explain how the answers to these questions would help to evaluate the recommendation

Why two years ago What happened two years ago which started this declineIs the dramatic increase in the ldquopopularityrdquo of skateboarding the cause of the steady decline of shoppers Are there any malls nearby Were there any changes nearby which could affect the decline in customersmdasha big mall perhaps Could the decline be due to the shop ownersHow many skateboarders use the plazaWhere do they skateboardDo they shop and are they customersAre the increase in litter and vandalism due to skateboarders Could this be alleviated by installing CCTVs and hiring security

This editorial concludes that the city should ban skateboarding from its downtown CentralPlaza in order to attract visitors to that area to return the area to its former glory and to makeit a place where people can congregate for fun and relaxation To justify this conclusion theeditorial points out that skateboarders are nearly the only people one sees anymore at CentralPlaza and that the Plaza is littered and its property defaced The editorial also points out thatthe majority of downtown merchants support the skate boarding ban This argument is flawedin several critical respectsFirst the editorials author falsely assumes that a ban on skateboarding is both necessaryand sufficient to achieve the three stated objectives Perhaps the city can achieve thoseobjectives by other means as well--for example by creating a new mall that incorporates anattractive new skateboard park Even if banning skateboarders altogether is necessary to meetthe citys goals the author has not shown that this action by itself would suffice Assuming thatthe Plazas reputation is now tarnished restoring that reputation and in turn enticing peopleback to the Plaza might require additional measures--such as removing litter and graffiti

GRE AWA John박 박정어학원

promoting the Plaza to the public or enticing popular restaurant or retail chains to the PlazaSecondly the editorial assumes too hastily that the Plazas decline is attributable to theskateboarders--rather than to some other phenomenon Perhaps the Plazas primary appeal inits glory days had to do with particular shops or eateries which were eventually replaced byless appealing ones Or perhaps the crime rate in surrounding areas has risen dramatically forreasons unrelated to the skateboarders presence at the Plaza Without ruling out these andother alternative explanations for the Plazas decline the editorials author cannot convince methat a skateboard ban would reverse that declineThirdly the editorials author might be confusing cause with effect--by assuming that theskateboarders caused the abandonment of the Plaza rather than vice versa It is entirelypossible that skateboarders did not frequent the Plaza until it was largely abandoned--andbecause it had been abandoned In fact this scenario makes good sense since skateboardingis most enjoyable where there are few pedestrians or motorists to get in the wayFourth it is unreasonable to infer from the mere fact that most merchants favor the ban thatthe ban would be effective in achieving the citys objectives Admittedly perhaps thesemerchants would be more likely to help dean up the Plaza area and promote their businesseswere the city to act in accordance with their preference Yet lacking any supporting evidencethe author cannot convince me of this Thus the survey amounts to scant evidence at best thatthe proposed ban would carry the intended resultFinally the author recommends a course of action that might actually defeat the citysobjective of providing a fun and relaxing place for people to congregate In my experienceskateboarding contributes to an atmosphere of fun and relaxation for adults and children alikemore so than many other types of ambiance Without considering that continuing to allowskateboarding--or even encouraging this activity--might achieve the citys goal more effectivelythan banning the activity the author cannot convincingly conclude that the ban would be in thecitys best interestsIn sum the argument is a specious one To strengthen it the editorials author must providedear evidence that skateboarding and not some other factor is responsible for the conditionsmarking the Plazas decline The author must also convince me that no alternative means ofrestoring the Plaza are available to the city and that the proposed ban by itself would suffice toattract tourists and restore the Plaza to its former glory Finally to better assess the argument itwould be useful to know the circumstances under which the downtown merchants would bewilling to help the city achieve its objectives

6그룹 약한 비유 빈출

The following recommendation appeared in a memo from the mayor of the town of Hopewell

Two years ago the nearby town of Ocean View built a new municipal golf course and resort hotel During the past two years tourism in Ocean View has increased new businesses have opened there and Ocean Views tax revenues have risen by 30 percent Therefore the best way to improve Hopewells economymdashand generate additional tax revenuesmdashis to build a golf course and resort hotel similar to those in Ocean View

Write a response in which you examine the stated andor unstated assumptions of the argument Be sure to explain how the argument depends on these assumptions and what the implications are for the argument if the assumptions prove unwarranted

GRE AWA John박 박정어학원

Assumptions The author assumes that OVrsquos municipal golf course and resort hotel caused tourism new businesses and increased tax revenues There may be other reasons advertising promo He assumes that this will continueAssumes that Ocean View and Hopewell are similar in many waysmdashthe name suggests otherwise OV may have always been a tourist attractions for its beaches We need to know the topography

2년전 Ocean View 시는 시정 소유 골프 및 휴양지 호텔을 신축했다 그리고 지난 2년동안 이 시의 관광객이 증가했으며 새로운 사업들이 생겨났다 그에따라 시의 세수도 30나 증가했다 Hopewell의 경제를 향상시키고 아울러 세수를 늘릴 수 있는 가장 좋은 방법은 Ocean View에 세워진 것과 같은 골프 시설과 휴양지 호텔을 신축하는 것이다

1 다른 요인으로 관광 산업이 발전했을 수도 있다 문화 유적이 발견이 되었거나 도로의 정비등으로 여행자가 늘었을 수도 있다

2 관광 산업의증가가 늘어난 세수의 원인이 아니라 새로 유입된 인구의 증가나 다른 공장에서 발생한 것일 수 있다

3 2년동안 한참 골프가 붐을 이루었을 수 있다 경제상황이 나빠지거나 다른 레포츠가 인근 지역에 생겨난다면 골프하는 사람이 줄어들 수 있다

In this memo HopeweUs mayor recommends that in order to stimulate the towns economyand boost tax revenues HopeweU should build a new golf course and resort hotel just as thetown of Ocean View did two years ago To support this recommendation the mayor points outthat in Ocean View during the last two years tourism has increased new businesses haveopened and tax revenues have increased by 30 I find the mayors argument unconvincingin several important respectsFirst of all it is possible that the mayor has confused cause with effect respecting the recentdevelopments in Ocean View Perhaps Ocean Views construction of a new golf course andhotel was a response to previous increases in tourism and business development increasesthat have simply continued during the most recent two years Since the mayor has failed toaccount for this possibility the claim that Hopewell would boost its economy by alsoconstructing a golf course and hotel is completely unwarrantedSecondly the mayor fails to account for other possible causes of the trends in Ocean Viewduring the last two years The increase in tourism might have been due to improving economicconditions nationwide or to unusually pleasant weather in the region The new businessesthat have opened in Ocean View might have opened there irrespective of the new golf courseand hotel And the 30 increase in tax revenues might have been the result of an increase intax rates or the addition of a new type of municipal taxWithout ruling out these and other alternative explanations for the three recent trends inOcean View the mayor cannot reasonably infer based on those trends that Hopewellseconomy would benefit by following Ocean Views exampleThirdly even if the recent trends in Ocean View are attributable to the construction of the newgolf course and hotel there the mayor assumes too hastily that the golf course and hotel willcontinue to benefit that towns overall economy The mayor has not accounted for thepossibility that increased tourism will begin to drive residents away during tourist season orthat new business development will result in the towns losing its appeal as a place to visit or tolive Unless the mayor can convince me that these scenarios are unlikely I cannot accept themayors recommendation that Hopewell follow Ocean Views exampleFinally the mayors argument rests on the unsubstantiated assumption that Hopewell andOcean View are sufficiently alike in ways that might affect the economic impact of a new golfcourse and hotel Hopewell might lack the sort of natural environment that would attract moretourists and new businesses to the town--regardless of its new golf course and hotel For thatmatter perhaps Hopewell already contains several resort hotels and golf courses that are notutilized to their capacity If so building yet another golf course and hotel might amount to amisallocation of the towns resources--and actually harm the towns overall economyIn sum the mayors recommendation is not well supported To bolster it the mayor mustprovide better evidence that Ocean Views new golf course and hotel and not some otherphenomenon--has been responsible for boosting Ocean Views economy during the last twoyears To better assess the recommendation I would need to know why Ocean View decidedto construct its new golf course and hotel in the first place--specifically what events prior toconstruction might have prompted that decision I would also need to thoroughly compare

GRE AWA John박 박정어학원

HopeweU with Ocean View--especially in terms of their appeal to tourists and businesses--todetermine whether the same course of action that appears to have boosted Ocean Viewseconomy would also boost Hopewells economy

The following is part of a memorandum from the president of Humana University

Last year the number of students who enrolled in online degree programs offered by nearby Omni University increased by 50 percent During the same year Omni showed a significant decrease from prior years in expenditures for dormitory and classroom space most likely because instruction in the online programs takes place via the Internet In contrast over the past three years enrollment at Humana University has failed to grow and the cost of maintaining buildings has increased along with our budget deficit To address these problems Humana University will begin immediately to create and actively promote online degree programs like those at Omni We predict that instituting these online degree programs will help Humana both increase its total enrollment and solve its budget problems

Write a response in which you discuss what questions would need to be answered in order to decide whether the prediction and the argument on which it is based are reasonable Be sure to explain how the answers to these questions would help to evaluate the prediction

Is Omni University successful due to the online degree program 50 Is the decrease in expenditures for dormitory and classroom space due to the decrease in of on-campus students Which classes were successful Does HU have those classes

Even if the long-distance degree programs at Omni University benefited the school the presidentrsquos recommendation that Human College should emulate Omni University is too hasty First OUrsquos name implies that the school would have more majors than Humanahellip the president should examine which degrees were in the long-distance programhellip

지난해에는 Omni 대학에서 개강했던 원거리 학생 학점 취득 프로그램을 등록했던 학생들의 숫자가 50나 증가했다 같은해 기간동안 Omni 대학에서는 그 전년도부터 기숙사와 학급의 공간 확충을 위한 예산을 대폭 줄였는데 이는 이 원거리 학점 취득 프로그램이 양방향 비디오 컴퓨터 접속을 통해서만 가능한 수업지도 방식이기때문인 것으로 보인다 반면 지난 3개년 동안 Humana 대학에서의 수강률은 감소한데다가 건물

GRE AWA John박 박정어학원

유지비도 올랐다 따라서 Humana대학의 수강을 늘리고 예산손실을 회복하기 위해서는 Omni 대학에서 취한 조치와 같은 능동적인 프로그램을 추진해야 한다

결론 we should initiate and actively promote long-distance degree programs like those at Omni 반박 원거리 학생 취득 프로그램 숫자가 증가한거하고 예산이 줄어드는 것 사이에 연관이 약하다 (causal 학생의 증가로 관리비용 증가할수 있음 원거리 수업가능 장비도입에의한 비용발생)bad analogy(omni university 하고 같은 조건이 아니다 )-gt omni college 가 강좌내용이 좋아서 학생의 등록이 많을수 있다 Humana 대학에서 만들었다 하더라도 인기 없을수 있음다른 요인에 의해서 Humana 대학의 수강 인원이 증가할수 있음(비록 과거엔 인기가 없었을지라도)

The following appeared as part of a business plan developed by the manager of the Rialto Movie Theater

Despite its downtown location the Rialto Movie Theater a local institution for five decades must make big changes or close its doors forever It should follow the example of the new Apex Theater in the mall outside of town When the Apex opened last year it featured a video arcade plush carpeting and seats and a state-of-the-art sound system Furthermore in a recent survey over 85 percent of respondents reported that the high price of newly released movies prevents them from going to the movies more than five times per year Thus if the Rialto intends to hold on to its share of a decreasing pool of moviegoers it must offer the same features as Apex

Write a response in which you discuss what questions would need to be answered in order to decide whether the recommendation is likely to have the predicted result Be sure to explain how the answers to these questions would help to evaluate the recommendation

Before following through this business plan the manager should investigate the cause of Rialtorsquos unsuccessful business

The author provides no evidence that the surveyrsquos results are statistically reliable The surveyrsquos sample of 85 percent must be sufficient in size and representative of overall population of the city where Rialto and Apex is serving Lacking evidence of a sufficiently representative sample the author cannot justifiably rely on the survey to draw any conclusion whatsoever The author does not indicate that Apex is indeed currently successful However even if Apex is enjoying success the argument relies on what might be a false analogy between Rialto and Apex In order for Apex to serve as a model that Rialto should emulate the author must assume that all relevant circumstances are essentially the same However this assumption is unwarranted For example the argument overlooks the face that Apex is located in a strategic placemdashbeside a mall where customers can not only watch a movie but also enjoy shopping Therefore simply changing the facility to that of Apex may not lead to success

The author does not mention whether Apex is successful or not Nevertheless even if Apex is currently successful the argument relies on what might be a false analogy between Rialto and Apex In order for Apex to serve as a model that Rialto should emulate the author must assume that all relevant circumstances are essentially the same However this assumption is unwarranted For example the argument overlooks the fact that these two institutions are located in different locations Rialto in downtown and Apex in a mall outside of town Although Apex opened with state-of-the-art facilities the decisive factor in its success could be due to its strategic location of being in a mall People could enjoy both shopping and movies at one location thus they may prefer Apex over Rialto Furthermore the place where people enjoy leisure activities has shifted in the past decades for most cities from downtown to the suburbs Therefore Rialto may not be successful even if it emulates Apexrsquos facilities A better business plan may be relocating Apex to the thriving section of the downtown

Rialto 극장은 지난 50여년간 지역 회관으로써 시내에 위치해 있으면서도 이제 변화를 꾀하지 않으면 문을 닫을

GRE AWA John박 박정어학원

판이다 이 극장은 시외 쇼핑타운에 새로 들어선 Apex 극장의 사례를 본받아야 했다 Apex가 지난해 개업했을 당시 이 극장은 비디오 아케이드 플러쉬 카펫트 바닥과 좌석 그리고 최신 음향시설을 갖추었다 더군다나 최근 조사에서는 응답자의 85 이상이 새로 출시된 영화 입장료가 비싼 탓으로 지난해보다 5배이상의 관람객이 줄어들었다고 나타났다 따라서 Rialto 극장이 줄어들고 있는 관람객을 뺐기지 않고 유지하려면 Apex와 같은 시설들을 갖추어야 할 것이다주장 리알토 극장이 줄어들고 있는 관람객을 뺐기지 않고 유지하려면 Apex와 같은 시설들을 갖추어야 할 것이다

1 조사에서 응답자가 전체를 대표할 수 없다 2 apex 극장이 좋은 시설을 갖추고 있지만 그로 인해 수익이 많이 발생했다는 말이 없으므로 시설투자를

하고도 좋은 결과를 얻을 수 있을지 그 근거가 미흡하다3 좋은 영화가 출시된다면 입장료가 비싸도 영화관에서 꼭 보려고 할 수 있다 4 rialto 가 시설이 아닌 다른 요인에 의해 장사가 안될수도 있다( 우범 지역이라든지)

The following is a recommendation from the business manager of Monarch Books

Since its opening in Collegeville twenty years ago Monarch Books has developed a large customer base due to its reader-friendly atmosphere and wide selection of books on all subjects Last month Book and Bean a combination bookstore and coffee shop announced its intention to open a Collegeville store Monarch Books should open its own in-store cafeacute in the space currently devoted to childrens books Given recent national census data indicating a significant decline in the percentage of the population under age ten sales of childrens books are likely to decline By replacing its childrens books section with a cafeacute Monarch Books can increase profits and ward off competition from Book and Bean

Write a response in which you examine the stated andor unstated assumptions of the argument Be sure to explain how the argument depends on these assumptions and what the implications are for the argument if the assumptions prove unwarranted

The following is a recommendation from the business manager of Monarch Books

Since its opening in Collegeville twenty years ago Monarch Books has developed a large customer base due to its reader-friendly atmosphere and wide selection of books on all subjects Last month Book and Bean a combination bookstore and coffee shop announced its intention to open a Collegeville store Monarch Books should open its own in-store cafeacute in the space currently devoted to childrens books Given recent national census data indicating a significant decline in the percentage of the population under age ten sales of childrens books are likely to decline By replacing its childrens books section with a cafeacute Monarch Books can increase profits and ward off competition from Book and Bean

1 Write a response in which you discuss what questions would need to be answered in order to decide whether the recommendation is likely to have the predicted result Be sure to explain how the answers to these questions would help to evaluate the recommendation

2 Write a response in which you discuss what specific evidence is needed to evaluate the argument and explain how the evidence would weaken or strengthen the argument

No evidence regarding Monarch Bookrsquos successEven if Regal Bookrsquos is successful this may not be attributable to the cafeacute False analogy Emulating may not lead to success Other factors may be involvedInsufficient condition The national census is not enough evidence that childrenrsquos book sales will decline Can

GRE AWA John박 박정어학원

the national census represent the local child populationDid opening a cafeacute boost sales for Regal Books Even assuming Regal is successful by opening a cafeacute this may not be suitable for Monarch which plans to close the childrenrsquos book section to establish a cafe Imprecise language ldquorelatively little spacerdquo how smallThe managerrsquos recommendation contradicts what he says Since Monarch is popular for its wide selection of books closing a selection which targets a major group of readers may hurt Monarchrsquos salesIs this the best way to compete

When Stanley Park first opened it was the largest most heavily used public park in town It is still the largest park but it is no longer heavily used Video cameras mounted in the parks parking lots last month revealed the parks drop in popularity the recordings showed an average of only 50 cars per day In contrast tiny Carlton Park in the heart of the business district is visited by more than 150 people on a typical weekday An obvious difference is that Carlton Park unlike Stanley Park provides ample seating Thus if Stanley Park is ever to be as popular with our citizens as Carlton Park the town will obviously need to provide more benches thereby converting some of the unused open areas into spaces suitable for socializing

Write a response in which you examine the stated andor unstated assumptions of the argument Be sure to explain how the argument depends on these assumptions and what the implications are for the argument if the assumptions prove unwarranted

Stanley 파크가 처음 개장했을 당시 가장 크고 가장 많이 이용되는 공원이었다 아직도 공원중에서는 가장 크지만 이용률은 상당히 떨어졌다 지난달 공원 주차장에 설치해놓은 비디오 카메라를 통해 보면 drop(주차장으로 여겨짐) 이용률이 가장 높았다 수치상으로는 하루 평균 50대의 차량만이 이용하였다 반면 직장 중심거리에 위치한 작은 규모의 Carlton 파크는 주당 무려 150여명 이상이 이용하고 있다 Stanley 파크와는 달리 Carlton 파크에는 의자가 있다는 것이 가장 뚜렷한 차이점이다 따라서 Stanley 파크가 Carlton 파크처럼 시민들이 자주 이용하는 공원이 되기 위해서는 벤치를 설치할 필요가 있으며 이렇게 사용되지 않는 일부 공간을 활용해서 사교를 위한 공간으로 바꾸어야 한다 ===gtdrop 에 대한 첨부사항 (영영사전내용입니다)---- a place or central depository to which something (as mail money or stolen property) is brought for distribution or transmission also the act of depositing something at such a place dropgt

주장 if Stanley Park is ever to be as popular with our citizens as is Carlton Park the town will obviously need to provide more benches thereby converting some of the unused open areas into spaces suitable for socializing1 조사가 언제 이루어진 것인가 조사가 언제 실시되었느냐에 따라 결과가 다를 수있다 현재는 다시 스탠리 파크가 늘어났었을 수 있다 2 벤치를 많이 설치했다고 해서 많은 관광객이 오지 않을수 있다(사람들이 벤치나 사교 공간을 원한다는 어떠한 자료도 없다)3스탠리 파크 주변에 교통 상황이 악화가 되었거나 칼튼 파크에서 문화행사등을 많이 가져서 이용객이 줄어든것일 수도 있다 4 칼튼 파크가 중심지에 있어서 접근성이 좋을수 있다5 조사가 같은 시간을 기준으로 한게 아니다(하나는 주중이고 하나는 주말이다)6사람의 수와 차의 대수를 같은것으로 비교할수 없다 (차안에 몇 명이 타고 있는지 모르고 대중교통을 이용해서 왔을수도 있다)

Page 4: GRE writing argument brain storm

GRE AWA John박 박정어학원

Challenge long-term study of a lot of people consistently consumed dairy products throughout the years=gt higher rate of bone fractures than other participantsThus bone fractures(symptom of osteoporosis) increased by diet rich in dairy products

-The author uses unscientific and imprecise terms in his study to support his claim-Control group-The author tries to refute a well established fact through a single study-Other factors bring about osteoporosis For example occupation hazards genetics dietary habits-Bone fracture could have might have may have been caused by other factors work environment etc and not specifically osteoporosis-The lack of a controlled environment of the study group The researchers only noted the consumption of milk and did not control other variantsmdashlifestyle and diet -The author bases his conclusion only by noting that the study samples consumed milk The researchers only noted the consumption of milk and did not control other variantsmdashlifestyle and diet-Dairy products might have had adverse effects on some of the participants in the study for example lactose intolerance

In this study the author claims that the consumption of milk actually increases the risk of osteoporosisTo support this claim the author refers to a long-term study he conducted ThesisHowever careful scrutiny of this study reveals that it provides little credible support for the authorrsquos claim

First the author states that the bone fractures were caused by osteoporosis However there could be other reasons for the bone fractures For example it is entirely possible that working conditions might have contributed to the bone fracture A person working at a construction has a much higher chance of bone fracture than a person that works in an office The author needs to provide the reasons for the occurrence of the study participantsrsquo bone fractures Thus the author should provide more sufficient evidence that

The author observes a correlation between osteoporosis and bone fracture then concludes that the former is the cause of the latter However the author fails to rule out other possible explanations For example it is entirely possible that working

The authorrsquos study includes an indefinite amount of people whose backgrounds arenrsquot defined genetic(family) history and environment the age at which the participants partook in the study It may be that the participants had genetic predisposition that were more prone to osteoporosis

The author does not sufficiently define what types of people were included in the study Bones are affected as noted in the prompt by various factors genes and environment The author however does not hellipFinally the author asserts his claim definitively While it is a proven fact that dairy products help people maintain healthy bones the author overthrows this long standing fact with a single long-term study that he did 우유와 유지방 제품에는 뼈를 형성시키고 강화시키는데 아주 중요한 역할을 하는 비타임 D와 칼슘이 많다 때문에 많은 사람들이 유지방이 풍부한 식이요법을 하면 환경 및 유전적 요인에 의한 손상과 노화에 의해 뼈가 심각하게 손상을 입는 질병인 오스티오포로시스를 억제하는데 도움이 된다고 믿고 있다 그러나 상당수의 사람을 대상으로 한 장기간 실험을 통해서 볼 때 이들 대상자중 실험 기간동안 유제품을 꾸준히 섭취한 사람들은 그렇지 않은 사람에 비해 훨씬 높은 골절현상을 보였다 이러한 실험결과를 볼 때 골절증상은 오스티오포로시스가 원인이기 때문에 이러한 식이요법이 오스티오포로시스 질병을 줄이기보다는 오히려 증가시킬 것이라는 것을 알수 있다결론 우유와 유지방 제품을 섭취하는 것은 오스티스오포로시스 질병을 줄이기봐는 오히려 증가시킬 것이다

1 실험은 변수에 의해 상당히 다른 결과를 얻을 수 있다 변수(생활 습관 운동)가 효과적으로 통제가 되었나 알 수 없다 상당수의 사람 장기간의 실험이 너무 주관적이다 객관적인 데이터가 주어져야한다

2 상당수와 장기간이 얼마만큼인지 알 수 없다 샘플링이랑 기간등 여러가지로 다르게 적용되었을 수 있다(실험에 충분한 기간과 인원이였는지)

3 골절 증상이 골다공증이 원인이 되어 나타났다고 했는데 그렇지 않을 수 있다다른 원인이 골절증상을 가져온 것일 수 있다

4 한가지 실험가지고 일반화했다 이미 밝혀진 사실을 단 한가지 실험으로 뒤엎었다 두 비교군이 동일 했는지에 대한 자료 없음(한쪽이 원래 뼈가 약했을수도있음)

GRE AWA John박 박정어학원

A recently issued twenty-year study on headaches suffered by the residents of Mentia investigated the possible therapeutic effect of consuming salicylates Salicylates are members of the same chemical family as aspirin a medicine used to treat headaches Although many foods are naturally rich in salicylates food-processing companies also add salicylates to foods as preservatives The twenty-year study found a correlation between the rise in the commercial use of salicylates and a steady decline in the average number of headaches reported by study participants At the time when the study concluded food-processing companies had just discovered that salicylates can also be used as flavor additives for foods and as a result many companies plan to do so Based on these study results some health experts predict that residents of Mentia will suffer even fewer headaches in the future

Write a response in which you discuss what questions would need to be answered in order to decide whether the prediction and the argument on which it is based are reasonable Be sure to explain how the answers to these questions would help to evaluate the prediction

In this study the author asserts that the number of headaches suffered by the average citizen of Mentia will steadily decline though the use of salicylates as flavor additives To support this assertion the author provides several evidences to support his claim However his argument is specious

Unscientific data correlation the specific amount used as preservatives Other factors could have caused the decline in headaches20 year study Was it a controlled study Lack of controlled environment exposes the participants to other factors

1) The decline in the average number of headaches does not necessarily substantiate that it was indeed caused by the commercial use of salicylates It is entirely possible that other factors are responsible for the decline Perhaps other substances in the food remedied the headaches of the participants Or the decline in headache may have been caused by other dietary consumption or lifestyle habits like exercising Accordingly if these are true the author cannot draw any firm conclusion that the commercial use of salicylates caused the decline in the average number of headaches

살리실산염은 아스피린과 같은 동일 화학물질 계열로써 두통을 치료하는데 사용되는 약물이다 많은 음식에 이 살리실산염이 다량 함유되어 있지만 지난 수십년간 식품가공 업체들은 가공식품 부패방지용으로 살리실산염을 첨가해왔다 이러한 살리실산염의 산업용 사용의 증가는 본 연구원들에 의해 보고된 평균 두통 횟수를 꾸준히 감소시키는데 일조한것으로 이해되어 왔다 최근 식품가공 회사들은 이 살리실산염이 맛을 내는 첨가물로 사용될 수 있음을 발견했다 이러한 새로운 용도로 Mentia에 사는 주민들이 겪고 있는 두통의 평균 횟수를 앞으로도 계속 줄여나갈 수 있을 것으로 보인다결론 살리산염의 사용증가는 주민들의 두통 횟수를 계속 줄여줄 것이다

가정 1 조사받은 사람들은 단지 Salicylate에 의해 headache가 감소된 것이다 -gt 오류 다른 노력을 기울였을 수 있다 가정 2 나중에 Salicylate의 사용증가가 headache의 감소를 가져올 것이다 -gt 오류 또다른 원인으로 얼마든지 headache는 증가가능하다가정 3 음식물에서 headache 감소 원인은 Salicylate 이다 -gt 오류 또다른 성분이 있었을 수 있다가정 4 Flavor additive 로 사용된 Salicylate도 headache를 줄일것이다 -gt 오류 Flavor additive로 사용시 같은 효과를 가질지 밝혀진바 없다 상업적으로 사용되었다고 다 먹었을까

The following appeared as part of an article in a business magazine

GRE AWA John박 박정어학원

A recent study rating 300 male and female Mentian advertising executives according to the average number of hours they sleep per night showed an association between the amount of sleep the executives need and the success of their firms Of the advertising firms studied those whose executives reported needing no more than 6 hours of sleep per night had higher profit margins and faster growth These results suggest that if a business wants to prosper it should hire only people who need less than 6 hours of sleep per night

Write a response in which you examine the stated andor unstated assumptions of the argument Be sure to explain how the argument depends on these assumptions and what the implications are for the argument if the assumptions prove unwarranted

Assumptions 1 Correlation of hours of sleep with success The author assumes that the executives used their wake-up hours on work Could there be other factors 2 Executives versus employeesmdashcriteria for hiring is too simpleminded 3 Characteristics of the 300 male and female executives 4 The study was about advertising executives but the author applies this case to all businesses 5 The author should consider long-term stability rather that sharp growth and profit 6 Average number of hours of sleep is insufficient to support the authorrsquos recommendation 7 The author assumes that this lifestyle pattern will remain consistent

The following appeared in a memo from the president of a company that makes breakfast cereals

In a recent study subjects who ate soybeans at least five times per week had significantly lower cholesterol levels than subjects who ate no soy products By fortifying our Wheat-O cereal with soy protein we can increase sales by appealing to additional consumers who are concerned about their health This new version of Wheat-O should increase company profits and at the same time improve the health of our customers

Write a response in which you examine the stated andor unstated assumptions of the argument Be sure to explain how the argument depends on these assumptions and what the implications are for the argument if the assumptions prove unwarranted

최근의 조사에서 일주에 최소 5회 정도 콩을 먹었던 사람들은 전혀 먹지 않았던 사람들에 비해 콜레스테롤이 상당히 낮은것으로 조사되었다 따라서 Wheat-O 시리얼에 콩 단백질의 함유를 강화시킴으로써 건강에 관심이 있는 더 많은 소비자를 상대로 매출을 올릴 수 있을 것이다 이 신제품이 회사 수입 증대를 가져오는 것 뿐만아니라 소비자의 건강을 향상시킬 수 있다

GRE AWA John박 박정어학원

주장 By increasing our Wheat-O cereal with soy protein we can multiply sales

1 콩 단백질의 함유를 강화해도 그수치가 미흡하거나 흡수가 잘 안되거나 할 수 있다 다른 성분과 섞여서 만들어지는 것이므로 다른 성분에 의해서 그 흡수가 잘 안되거나 효과가 낮을 수 있다 2 강화한 것이 건강에 관심이 있는 소비자에게 어필한다고 해도 그 수가 적어서 매출에 큰 영향을 안줄 수 있다3 다른 음식에 의해서 콜레스테롤이 더 낮아질수 있다 4 콜레스테롤을 낮추는 것이 사람들에게 그들의 건강생활을 위해 크게 어필 안할수도 있다( 저지방식이라든지 다른 건강을 높이는 방법이 더 관심이 많을수 있다)

In a study of the reading habits of Waymarsh citizens conducted by the University of Waymarsh most respondents said that they preferred literary classics as reading material However a second study conducted by the same researchers found that the type of book most frequently checked out of each of the public libraries in Waymarsh was the mystery novel Therefore it can be concluded that the respondents in the first study had misrepresented their reading habits

Write a response in which you discuss what specific evidence is needed to evaluate the argument and explain how the evidence would weaken or strengthen the argument

Evidence needed -Information about the respondentsmdashage gender class precise number of respondents-Time-shift After how many years was the second study conductedmdashtaste in books may change over time-Can library records sufficiently evidence the reading habits of Waymarsh citizens

1)2)3)4)5)6) First the author provides no evidence that the surveyrsquos studyrsquos results are statistically reliable Lacking

information about the precise methodology of the study the number of (customers)respondents surveyed and the number of respondentsvarious information such as gender age and social background which are essential to bolster the conclusion it is impossible to assess the validity of the results It is possible that people who feel inclined to( take low fat and low cholesterol foods ) read literary classics were more willing to respond to the survey than were others Another problem is the representativeness of the respondents Were they representative of all the customersWaymarsh citizens Were they chosen for the survey randomly Lastly the survey results must depend on the honesty and integrity of the respondents Without more information about the survey the author cannot simply conclude that (most of customers want to take low fat and low cholesterol foods)Waymarsh citizens misrepresented their reading habit on the first study on the basis of this surveystudy(survey reliability)

The exact number of books checked-out should be notedMystery novels could have been in trend at the time of the studyPublic libraries vs other libraries such as Leeville University library

Leeville 대학에서 실시한 Leeville 주민의 독서습관에 대한 조사에서 대부분의 응답자는 독서용으로 문학작품을 선호한다고 응답했다 그러나 같은 조사팀에서 이어 실시한 조사에서는 Leeville에 있는 도서관마다 가장 빈번하게 연람된 책의 종류를 보면 미스테리 소설이었던 것으로 조사되었다 따라서 최초 조사 응답자들이 독서 습관에 대해 잘못 말했다고 결론지을 수 있다

결론 it can be concluded that the respondents in the first study had misrepresented their reading habits1 첫번째 조사 그룹하고 두번째 조사 그룹하고 다른 그룹일 수 있다

GRE AWA John박 박정어학원

2 도서관에 비치된 책이 미스터리 소설이 더 많아서 선택의 여지가 없을 수 있다

High Frequency Group 2 National-gtLocal Local-gtNational amp Group Member Error

The following appeared in a letter to the editor of a Batavia newspaper

The department of agriculture in Batavia reports that the number of dairy farms throughout the country is now 25 percent greater than it was 10 years ago During this same time period however the price of milk at the local Excello Food Market has increased from $150 to over $300 per gallon To prevent farmers from continuing to receive excessive profits on an apparently increased supply of milk the Batavia government should begin to regulate retail milk prices Such regulation is necessary to ensure fair prices for consumers

Write a response in which you discuss what questions would need to be answered in order to decide whether the recommendation is likely to have the predicted result Be sure to explain how the answers to these questions would help to evaluate the recommendation

First the author points to the fact that hellip25 increase of dairy farms may be(could be might be) due to population increaseThe milk price increase could be reflecting the increase in cost of livingThe actual price of milk might be cheaper if the economy is experiencing inflationOne market Excello Food cannot reflect the price increase of every market in BataviaThe author claims that farmers are receiving excessive profits however this might not be the caseThe author claims that by regulating milk prices the government will successfully ensure both lower prices and an adequate supply of milk for consumers However this might not be the best solutionThe writer adduces the information released by the department of agriculture that the number of dairy farms has

increased by 25 percent over the last 10 years and thus he claims that dairy farmers are earning excessive profits from milk However the information is filled with loops and holes which need to be filled First what percentage of the milk are sold as milk

Are the dairy farms processing the milk into other dairy products Or are they selling them as milkmdashWhat percentage of the milk is sold as milk Could this be a natural increase

Can the price at the local Excello Food Market represent the price of milk in stores throughout all of Batavia

Are the farmers receiving excessive profits What is the cost of producing milk Could such regulation offer fair prices to consumers

Topic SentenceTo begin with the author assumes that the price of Excello Food Market which has increased from $150 to over $300 per gallon can be applied to the whole nation of Batavia however the author fails to provide evidence to substantiate this assumptionExampleFirstIn additionTherefore in order to make the argument more reliable the author should consider the prices of a large number markets all over the countryThere could be an economic inflation

GRE AWA John박 박정어학원

ldquoBatavia 지역의 농림부의 보고에 따르면 전국적으로 낙농업자의 숫자가 과거 10 년전에 비해 25나 증가했다고 합니다 그러나 동기간 동안 이 지역에 있는 Excello Food Market 에서 우유의 가격은 갤런당 15 불에서 3 불로 증가했습니다 증가된 우유 공급량에 맞춘 낙농업자들의 수익률 증대를 보장하기 위해서는 주 정부가 소매 공급가에 대한 규제를 해야 합니다 소비자에게 보다 저렴한 가격에 안정된 물량을 공급할 수 있도록 하기 위해서는 이러한 조치가 필요합니다

결론 the Batavia government should begin to regulate retail milk prices

반박 ( Excello Food Market doesnrsquot reflect the whole of Batavia) 우유값 증가가 단순히 inflation 을 따라간것일수도 있다 가격상승이 수익을 보장하지 않는다( 원자재 상승등으로 인해서)우유값 규제만이 최선의 방법이 아니다( 물량 조절)낙농업자의 수익보호도 필요하다

A recent sales study indicates that consumption of seafood dishes in Bay City restaurants has increased by 30 percent during the past five years Yet there are no currently operating city restaurants whose specialty is seafood Moreover the majority of families in Bay City are two-income families and a nationwide study has shown that such families eat significantly fewer home-cooked meals than they did a decade ago but at the same time express more concern about healthful eating Therefore the new Captain Seafood restaurant that specializes in seafood should be quite popular and profitable

Write a response in which you discuss what questions would need to be addressed in order to decide whether the conclusion and the argument on which it is based are reasonable Be sure to explain how the answers to the questions would help to evaluate the conclusion

BAY CITY The name implies that this is a port city which would have traditionally consumed seafood 30--what type of seafood Unprocessed or processed cans of tuna and sardines If the latter increased Captain Seafood may not profit 30--natural inflation Are the non-seafood restaurants selling seafood If yes this may suffice since patrons would not easily dine at an unfamiliar restaurant Nationwide studyrsquos representativeness Desire does not lead to action

Nationwide study applied to Bay City Eating fewer home cooked meals than they did a decade agoHealthy food The author needs to show that 30 -gt population increaseOther foods may be more popularCurrent number of restaurants that serve seafood might sufficeWhat kind of healthy food are they interested in Surely not just seafoodThe locals may still patronize the current restaurants that serve seafood dishes

lt national-gt local에 적용 local-gtnational에 적용gt1) The author assumes that the nationwide trend reflects the general trend upon which the argument relies

Yet the author fails to provide evidence to substantiate this crucial assumption The nationwide trend of two income families eating fewer home-cooked and concern for health my not reflect the lifestyle and interests of Bay City citizens Therefore to make his argument stronger the author needs to provide evidence that two income families of Bay City maintains the nationwide trend

GRE AWA John박 박정어학원

최근의 매상에 관한 조사에서 Bay 시에 있는 레스토랑의 해산물 요리의 소비가 지난 5년동안 30 증가했다고 보고되었다 그러나 현재 해산물을 전문으로 취급하고 있는 레스토랑이 없다 더군다나 대다수의 가정이 맞벌이 가정인데다가 전국 조사에서도 나와있듯이 이러한 맞벌이 가정은 10년전의 가정에서 했듯이 집에서 식사를 챙겨먹는 것이 급격하게 줄어 들었고 동시에 건강식과 관련한 지출이 늘고 있다 따라서 해산물 전문 레스토랑이 아주 인기를 끌것이고 그만큼 수익도 많을 것이다

1 해산물이 건강식인지에 대한 언급이 없다2 외식하는데 해산물 요리만 먹지 않을 것이다 집에서 챙겨먹지 않는다고 해산물을 먹는다고 할 수 없다3 현재 해산물 취급하는 식당이 현재까지 없다고 해서 계속 없을 수만은 없다 수익이 만약 늘어난다면

Bay시 주변의 도시의 레스토랑이 체인점을 내거나 새로운 식당이 생길 수 있고 오히려 수익이 줄어들 수도 있다

Scientists studying historical weather patterns have discovered that in the mid-sixth century Earth suddenly became significantly cooler Although few historical records survive from that time some accounts found both in Asia and Europe mention a dimming of the sun and extremely cold temperatures Either a huge volcanic eruption or a large meteorite colliding with Earth could have created a large dust cloud throughout Earths atmosphere that would have been capable of blocking enough sunlight to lower global temperatures significantly A large meteorite collision however would probably create a sudden bright flash of light and no extant historical records of the time mention such a flash Some surviving Asian historical records of the time however mention a loud boom that would be consistent with a volcanic eruption Therefore the cooling was probably caused by a volcanic eruption

Write a response in which you discuss what questions would need to be addressed in order to decide whether the conclusion and the argument on which it is based are reasonable Be sure to explain how the answers to the questions would help to evaluate the conclusion

Historical records may not be enough to explain the global cooling phenomenonThere may be reasons other than the two hypothesis to explain the global coolingA volcanic eruption big enough to produce dust clouds that envelop the earth to cause global cooling would have countless recordsThe author argues that a large meteorite collision is not a feasible explanation for the global cooling because there are no historical records that indicate a flash The authorrsquos logic is flawed in eliminating the meteorite hypothesis by claiming that there was no historical record of a bright flash of light According to common sense a meteorite collision big enough to produce dust that could envelop the earth would result in catastrophe even to the extinction of major species of animals

Loud boom would be insufficient evidence to conclude it was a volcanic eruption If could hear a loud boom there should be records of a volcanic eruption because the author claims that the dust cloud from this gigantic eruption was significant enough to cause global coolingCooling could have been caused by factors besides volcanic eruption and meteor collisionThe absence of historical records that indicate a bright flash of light Collision could have occurred at a place where there no witnesses Could have fell out in the seaBoom might have been caused by things other than a volcanic eruptionHistorical records from Asia and Europe may not be applicable to earth as a whole

과거의 날씨 주기를 연구하는 과학자들은 6세기 중엽 지구가 갑자기 혹한기가 되었던 것을 알게되었다 이 당시의 몇몇 기록들이 아직도 남아있긴 하지만 아시아와 유럽에서 발견되는 몇가지 자료에서 태양 빛의 수축과 그에따른 엄청난 추위가 있었음을 언급하고 있다 거대한 화산 폭발이나 거대 운석의 지구 충돌중 어느것에

GRE AWA John박 박정어학원

의해서든 엄청난 먼지구름을 형성시켜 지구전체에 태양빛을 차단해서 지구의 기온을 뚝 떨어뜨릴수 있을 수도 있다 그러나 이러한 운석 충돌은 순간 섬광을 발산시키게 될 것인데 이 시기의 어느 기록에서도 이러한 섬광은 언급되지 않고 있다 그러나 이 시기에 남아 있는 일부 아시아의 기록문서에서는 연속적으로 화산폭발음일수 있는 엄청난 폭발음이 있었음을 기록하고 있다 따라서 이러한 혹한은 아마도 화산 폭발에 의한 것이었을 것이다

결론 the cooling was probably caused by a volcanic eruption

반박 자료의 부족( 몇몇 자료가지고 그 당시의 기후를 예상하기에는 무리가 있다)다른 원인에 의해서 기후가 떨어졌을수도 있다 (빙하기 다른 기상원인)기록되지 않았다고 해서 그 사실이 없었던 것은 아니다(기록은 했으나 자료가 없어졌을수도 있고 모든 사실이 전부다 기록되지는 않는다 )기록된 폭발음이 꼭 확산 폭발음이 아닐수도 있다( 화산 폭발음이 너무 먼거리여서 들리지 않을수도 있고 다른 소리가 기록된 것이였을수도 있다)부족한 자료를 신빙할수 있는가 기록의 소멸도 예상 할수 있다 실제로 운석이 떨어졌는데 안들렸을수 있다

The following appeared in a memo from the president of Bower Builders a company that constructs new homes

A nationwide survey reveals that the two most-desired home features are a large family room and a large well-appointed kitchen A number of homes in our area built by our competitor Domus Construction have such features and have sold much faster and at significantly higher prices than the national average To boost sales and profits we should increase the size of the family rooms and kitchens in all the homes we build and should make state-of-the-art kitchens a standard feature Moreover our larger family rooms and kitchens can come at the expense of the dining room since many of our recent buyers say they do not need a separate dining room for family meals

Write a response in which you examine the stated andor unstated assumptions of the argument Be sure to explain how the argument depends on these assumptions and what the implications are for the argument if the assumptions prove unwarranted

The presidentrsquos first unstated but apparent assumption is that a nationwide survey can be applied to local areas There is no guarantee that the area in which Bower Builders builds houses will follow the national consumer trend Since he only mentions the overall result of the nationwide survey it is difficult to determine more specific conditions such whether people in urban and rural areas have the same housing preferences Such uncertainty is further exacerbated by the nature of the survey question itselfmdash it does not deal with current trends in actual home purchases but with desired home features The naiumlve assumption that consumer desire will directly result in consumer action underlies the presidentrsquos interpretation and application of the survey results to his company plan Not everyone who wishes for a large family room and kitchen will or can actually buy a house with those features mainly due to financial reasons Furthermore the president also assumes for no evident reason that consumers will not hesitate to purchase houses with state-of-the-art kitchens as a standard rather than optional feature when it is clear that such a feature will raise the overall cost significantly He continues to blunder in his mistaken assumptions about consumer behavior in assuming that the taste of recent buyers can represent the concern of future prospective buyers The fact that recent buyers have claimed no need for separate dining rooms does not mean future buyers will feel the same way as well

The president of Bower Builders recommends that in order to raise company profits the company should build houses with large family rooms and large state-of-the-art kitchens He uses the results of a nationwide survey on desired home features and the example of Bower Buildersrsquo competitor Domus Construction to support his plan His memo manifests several assumptions about surveys consumer behavior and Domus Constructionrsquos houses that do not necessarily bolsterbuttress his argument

GRE AWA John박 박정어학원

The writer assumes 1 the nationwide survey results can be applied to the local area (Desire cannot reflect consumer trend) 2 Domus Construction was profitable because they had such features (The writer should investigate what other features the DC provides and the actual number of homes sold) 3 State-of-the-art kitchens will sell well (no evidence to buttress his assumption furthermore they would need to spend more money which would increase the price of the housesmdashtheir market will be limited to the affluent) 4 The recent buyers represent the concern of most buyersmdashthat they are ok with a house that is without a dining room

Nationwide survey might not be applicable to this regionDomus Construction could have other superior qualities than Bower Builders Ex DesignSelling fast doesnrsquot indicate quantitySmall yards complaints could be voiced in the future

First the author concludes that a nationwide survey reveals that the two most desired home features are a bathroom with a whirlpool tub and a large kitchen However this nationwide survey may not reflect the trends of the customers that Bower Builders target The author assumes that the nationwide trend reflects customer trends The national study would lead support only if the nationwide trend reflect the home-purchasing trends The nationwide trend may just be a trend of desire which does not result in the act of purchasing However the author does not provide credible evidence that this is the case

전국 조사에서 가장 인기있는 집안 구조 2가지는 훨풀 튜브가 마련된 욕실과 커다른 부엌으로 조사되었다 경쟁업체 Domus Construction이 신축한 완공예정인 주택은 이러한 욕실을 갖추고 있어서 분양이 조기에 이루어지고 있고 가격도 평균치보다 상당히 웃돌고 있다 우리도 집을 많이 팔도 그만큼 이윤을 많이 남기려면 신규 주택에는 위의 2가지 사항을 갖추어야 한다 최근 우리가 지은 주택 수요자들이 적은 평수에 대해서는 별다른 불만사항이 없었기 때문에 앞으로 평수를 더 줄여서 이익을 극대화 할 수 있을 것이다

주장 We should include whirlpool tubs and a large kitchen

1 경쟁업체 Domus Construction이 신축한 완공예정인 주택은 이러한 욕실을 갖추고 있어서라기 보다는 위치가 좋거나 다른 마감재(construction material)나 인테리어가 뛰어나서 분양이 조기에 이루어진것이라고 볼 수 있다 2 전국조사가 건물이 지어지는 지역에 항상 적용되리라고 볼 수 없다 3 평수를 줄이는 것에 불만이 없었다는 것은 editor의 견해일 수 있다 사람들이 표현하지 않은 것일 수도 있고 다른 장점이 많아서 그런 단점을 드러내지 않은 것일 수 있기 때문에 속단할 수 없다 4 다른것을 함으로써 더 잘 팔릴수도 있다

2011 7 19 화요일 GRE AWA 실전반이OO

Argument 94

The president of Bower Builders contends recommends that in order to raise company profits the company should build houses with large family rooms and large state-of-the-art kitchens He uses the results of a nationwide survey on desired home features and the example of Bower Buildersrsquo competitor Domus Construction to support his plan His memo manifests several assumptions about surveys consumer behavior and Domus Constructionrsquos houses that do not

GRE AWA John박 박정어학원

necessarily bolsterbuttress his argumentThe presidentrsquos first unstated but apparent assumption is that a nationwide survey can be

applied to local areas There is no guarantee that the area in which Bower Builders builds houses will follow the national consumer trend Since he only mentions the overall result of the nationwide survey it is difficult to determine more specific conditions such whether people in urban and rural areas have the same housing preferences Such uncertainty is further exacerbated by the nature of the survey question itselfmdash it does not deal with current trends in actual home purchases but with desired home features The naiumlve assumption that consumer desire will directly result in consumer action underlies the presidentrsquos interpretation and application of the survey results to his company plan Not everyone who wishes for a large family room and kitchen will or can actually buy a house with those features mainly due to financial reasons Furthermore the president also assumes for no evident reason that consumers will not hesitate to purchase houses with state-of-the-art kitchens as a standard rather than optional feature when it is clear that such a feature will raise the overall cost significantly He continues to blunder in his mistaken assumptions about consumer behavior in assuming that the taste of recent buyers can represent the concern of future prospective buyers The fact that recent buyers have claimed no need for separate dining rooms does not mean future buyers will feel the same way as well

In addition the president finds a real-life actualization of the nationwide survey results in the recent sales of Domus Construction However he easily assumes that large family rooms and kitchens are the only reasons the houses of Domus Construction sell well He does not take into account other features and selling points of the competitorrsquos houses There may well be other explanations for its houses selling more quickly and expensively including additional home features as well as external factors such as proximity to better schools or superior financial solvency of its clientele If Bower Builders merely added larger family rooms and kitchens without taking into consideration the other factors they may lose rather than gain profits

Clearly the presidentrsquos assertion that Bower Builders make houses with large family rooms and high-tech kitchens at the expense of dining rooms rests on a number of assumptions that are ill-informed and naiumlve If Bower Builders undertakes the proposed plan without further research into local consumer desires purchasing trends and the marketing and sales of competing companies the company will risk losing money by building big new houses that people cannot afford to or will not wish to buy

The following appeared in a letter to the editor of a journal on environmental issues

Over the past year the Crust Copper Company (CCC) has purchased over 10000 square miles of land in the tropical nation of West Fredonia Mining copper on this land will inevitably result in pollution and since West Fredonia is the home of several endangered animal species in environmental disaster But such disasters can be prevented if consumers simply refuse to purchase products that are made with CCCs copper unless the company abandons its mining plans

Write a response in which you examine the stated andor unstated assumptions of the argument Be sure to explain how the argument depends on these assumptions and what the implications are for the argument if the assumptions prove unwarranted

The writer assumes 1 The writer is trying to avoid an inevitability 2 Mining copper will result pollution (they could make preventative measures) 3 The writer may be too late from stopping CCC from developing the area into a copper mine 4 Since mining is an underground enterprise the surface may not be affected that much therefore endangered species may not be affected 4 Consumers wonrsquot buy CCC products if the journal publishes a negative review about CCC (How many readers) CCC could a company that has ties with many IT companies and industries in that their copper is almost ubiquitous in various products

GRE AWA John박 박정어학원

지난 한해동안 CCC(Consolidated Copper 회사)는 서부 플로리다의 열대 지역에 1백만 마일이 넘는 땅을 사들였다 이 곳에서의 채광 활동은 서부 플로리다가 몇몇 멸종 위기에 처한 동물의 서식지이기 때문에 분명 오염과 환경파괴를 가져올 것이다 그러나 이러한 파괴는 CCC 회사가 채광을 포기할 때까지 이 회사가 채굴한 구리로 제조된 제품을 구매하지 않으면 막을 수 있을 것이다

결론 such disaster can be prevented if consumers simply refuse to purchase products that are made with CCCs copper until the company abandons its mining plans

1 구리로 제조된 물건이 생활에 필수적인 것이 많은 만큼 불매가 쉽지 않을 수 있다(전선이나 각종 전자제품에 필수적으로 들어가기 때문에)

2 불매를 유도한다고 해서 소비자들이 구매를 안하는 것은 아니다3 적절한 채굴로 환경파괴를 가져 오지 않을 수 있다 (땅속에 있는 물질을 채굴하는 만큼 생물에 영향을 안

미칠 수도 있다)4 이미 채굴이 다 끝나서 더 이상의 채굴이 없을 수도 있다 5 땅을 구입한다고 해서 채광하는건 아니다 (다른 용도로 샀을수도 있다)

The following is a letter to the editor of an environmental magazine

In 1975 a wildlife census found that there were seven species of amphibians in Xanadu National Park with abundant numbers of each species However in 2002 only four species of amphibians were observed in the park and the numbers of each species were drastically reduced There has been a substantial decline in the numbers of amphibians worldwide and global pollution of water and air is clearly implicated The decline of amphibians in Xanadu National Park however almost certainly has a different cause in 1975 troutmdashwhich are known to eat amphibian eggsmdashwere introduced into the park

Write a response in which you discuss what specific evidence is needed to evaluate the argument and explain how the evidence would weaken or strengthen the argument

Evidence needed the identity of the sender and the census taker of rsquo75 and lsquo02mdasha scientist or an environmentalist credibility issue census methodology vs mere observationmdashthe absence of evidence is not an evidence of absence the season of when the census was taken time-shiftmdashconditions may have changed worldwide decline may include Xanadu other species of predators that prey on amphibians because trout is only one species that prey on amphibians the number of troutmdashhave they increased significantly since rsquo75 First the author needs to be more overt about the credibility of the census and observation There were to accounts that notes the population of amphibiansmdashthe first a census and the second an observation The author needs to bolster his conclusion with the evidence that indicate that the census and second observation were done using scientific methodologies This could either could strengthen or weaken his claim In addition he

GRE AWA John박 박정어학원

needs to provide the specific season of when the census and observation occurred In this letter the writer is informing an editor of an environmental magazine that the number of amphibians was greatly reduced since 1975 and he points out the introduction of trout as the only reason for the decline However the author fails to provide crucial evidences that could strengthen or weaken his conclusion

The following appeared in a memorandum from the president of Hyper-Go Toy Company

Last year sales of our Fierce Fighter toy airplane declined sharply even though the toy had been a top seller for three years Our customer surveys show that parents are now more worried about youthful violence and are concerned about better education for their children Therefore to maintain profits we should discontinue all our action toys and focus exclusively on a new line of educational toys Several other toy companies have already begun marketing educational toys and report sales increases last year of 200 percent And since the average family income is growing sales of new Hyper-Go toys should also increase

Write a response in which you discuss what specific evidence is needed to evaluate the argument and explain how the evidence would weaken or strengthen the argument

First the president indicates that the sales of Fierce Fighter toy airplane declined sharply However he fails to consider the fact that toys are a fad Since FFT enjoyed a three year success it may be natural that the trend would subsideSecond Customer survey is this representative of most toy consumers This trend is not newThird other companies may have profited not through educational but other toys Or educational toy profit may be small in proportion to their sale of other toys 200Parents are not the customers companies should concentrate on what the children want to increase profitLastly parents worry about youthful violence and concern for better education are not new trends

The presidentrsquos decision to discontinue all action toys and focus only on educational toys is too extreme If this decision is taken into effect its procedure may be an onerous task because it would require major shifts in human resources and company image Therefore further data should be considered before following up on this decision

우리 회사에서 지난 3년간 최고 매출을 일으켰던 Fierce Fighter 장난감 비행기가 지난해에는 매출이 급격히 떨어졌다 자체 고객 조사에서는 부모들이 현재 청소년 폭력에 걱정을 하고 있어서 아이들의 양질 교육에 더 관심을 가지고 있는 것으로 나타났다 따라서 회사의 수익을 유지하기 위해서는 모든 자사 전투용 장난감 생산을 중단하고 오로지 교육적인 장난감 생산에 집중해야 한다 몇몇 여타 장난감 회사들도 이미 교육용 장난감 마케팅을 시작해서 지난해에는 200의 매출신장을 가져왔다고 한다 그리고 평균 가계 수입이 점점 늘고 있기 때문에 신형 Hyper-Go 장난감의 매출도 늘어날 것이다

3그룹 Time-shift Error

Woven baskets characterized by a particular distinctive pattern have previously been found only in the immediate vicinity of the prehistoric village of Palea and therefore were believed to have been unique to the Palean people Recently however archaeologists discovered such a Palean basket in Lithos an ancient village across the Brim River from Palea The Brim River is very deep and broad and so the ancient Paleans could only have crossed it by boat but there is no evidence that the Paleans had boats And boats capable of carrying groups of people and cargo were not developed until thousands of years after the Palean people disappeared Moreover Paleans would have had no need to cross the rivermdashthe woods around Palea are full of nuts berries and small game It follows that the so-called Palean baskets were not unique to Palea

GRE AWA John박 박정어학원

Write a response in which you discuss what specific evidence is needed to evaluate the argument and explain how the evidence would weaken or strengthen the argument

RefutationPrehistoric time shift-gtbrim river could have been narrow and shallow or it might have not existed Indigenous patterns may exist in other disconnected remote placesNuts berries small game(hunting) may not have existed or the author should be proved these existed at that time Abundance of resources doesnrsquot support the reason for seclusionBoats not yet found baskets may have been carried across by the river current without the help of a boat Lithos might have crossed the river for commercial purposeOne Palean basket does not substantiate the authorrsquos claimThe absence of evidence is not an evidence of absence

The author assumes without justification that present conditions are the same as at the prehistoric era The author unfairly infers from the presence of Brim River which exist today that it would have existed in the past However the author fails to offer any evidence to substantiate this inference It is very likely that the Brim River might not have existed in prehistoric times or if it did exist may have been shallow and narrow enough for the Paleans to easily cross Any of these scenarios if true would serve to undermine the claim thathelliphelliphelliphelliphellip

First the author claims that the Brim River was very deep and broad so the Paleans could not have crossed it However the author fails to offer any evidence to substantiate that this was true in the prehistoric time of the Paleans For all we know the Brim River might not have existed in the prehistoric era or if it did exist could have been a shallow and narrow river For example scientists believe that thousands of years ago an ice-bridge existed on the Bering Sea connecting Eurasia to what is now North America to explain how the Eskimos and the Asian inhabitants of America came to migrate all over the Americas In this example the absence of evidence is not an evidence of absence Therefore to sufficiently support his claim the author needs to substantiate the fact that the Brim River really did exist and was broad and wide in the Prehistoric Era proving that the Paleans could not have influenced or traded with other groups of people

First the author claims that the Brim River was very deep and broad so the Paleans could not have crossed it However the author fails to offer any evidence to substantiate that this was true in the prehistoric times of the Paleans For all we know the Brim River might not have existed in the prehistoric era or if it did exist could have been a shallow and narrow river For example rivers are created by natural erosion over thousands of years Niagara Falls carved its way from the mouth of Lake Ottawa and created a long river Likewise the current Brim Riverrsquos physical features may not have been wide and deep Therefore to sufficiently support his claim the author needs to substantiate the fact that the Brim River really did exist and was broad and wide in the Prehistoric Era proving that the Paleans could not have influenced or traded with other groups of people

예전에는 실로짠 특이한 무늬 바구니가 Palea의 선사시대 지역의 인근마을에서만 발견되어왔기 때문에 Palea 마을 사람들의 특징이라고 여겨졌었다 그러나 최근들어 고고학자들이 Lithos지역에서 Palean 바구니를 발견하였는데 그 당시 지역은 Brim 강을 가로질러 Palea까지 닿아있었다 이 강은 수심이 아주 깊고 강폭이 넓었으며 때문에 고대의 Palea인들은 배를 이용해서 강을 건널수 있었을 것이다 그러나 이들이 배를 가지고 있었다는 증거는 발견되지 않고 있다 더군다나 이들이 멸명한 이후 수천년이 지난뒤에도 수많은 물자와 사람을 실어 나를수 있는 용적을 가진 배는 개발되지 않았다 이와더불어 Palea인들은 강을 건널필요가 없었는데 그것은 너트나무 장과열매 그리고 작은 사냥감들이 주변숲에 풍부했기때문이다 따라서 Palean 바구니라고 하는 것도 Palea인들만의 전유물이 아니라는 결론을 얻을 수 있다

결론 if follows that the so-called Palean baskets were not unique to Palea이번문제는 굿이 causal Bad analogy 로 구분해서 찾기가 힘드내요 배를 발견했다는 증거가 없는것이다 (앞으로도 발견될수 있음)

GRE AWA John박 박정어학원

계절의 영향으로 겨울에 얼음이 두껍게 언다든지 여름에 가뭄으로 인해서 건널수 있다물자가 풍부한 것이 이동하지 않을 조건이 아니다 다른 것에 의해서 이동가능(의약품등)

Thirteen years ago researchers studied a group of 25 infants who showed signs of mild distress when exposed to unfamiliar stimuli such as an unusual odor or a tape recording of an unknown voice They discovered that these infants were more likely than other infants to have been conceived in early autumn a time when their mothers production of melatonin hormone known to affect some brain functions would naturally increase in response to decreased daylight In a follow-up study conducted earlier this year more than half of these children now teenagers who had shown signs of distress identified themselves as shy Clearly increased levels of melatonin before birth cause shyness during infancy and this shyness continues into later life

Write a response in which you examine the stated andor unstated assumptions of the argument Be sure to explain how the argument depends on these assumptions and what the implications are for the argument if the assumptions prove unwarranted

Any baby exposed to unpleasant stimuli would react in such wayFirst of all the author states 25 infants as his evidence However this research sample is too small to prove his claimSecond the author states that 25 infants were conceived in early autumn which he claims lead to a shy disposition However this is faulty evidenceThird the research study was a long term study done in the span of 13 years However the author only writes about the initial and final stages of the study and leaves out evidences of what could have happened during the 13 years which could be more evidential factors of influenceFourth neither the infantsrsquo genetic predisposition nor their environment were taken into accountFinally the author concludes that his shyness continues into later life (Other factors could alter this disposition epigenetic theory)

13 년전 학자들은 25명의 유아를 대상으로 이상한 냄새나 특이한 소리를 녹음한 테잎등으로 낯선 자극을 주었을때 보이는 미미한 압박감 증상을 조사하였다 이들은 성숙기가 막 지났을 즈음에 보통의 유아들이 비슷한 증상을 보이는 정도 보다는 다소 민감한 반응을 보였는데 이 시기는 아이의 엄마가 뇌의 일부 기능에 영향을 미치는 것으로 알려진 멜라토니아 호르몬을 생산하는 시기로써 이 호르몬은 낯 시간이 짧을때 자연적으로 증가할 수도 있다 금년초에 실시된 추가연구에서 현재 10대로 성장한 당시 조사대상의 절반 이상의 아이들이 부끄럼을 잘타는 것으로 여기고 있었다 따라서 분명한 것은 출산전 멜라토닌 수치의 증가가 유아기에 수줍음 등의 영향을 미치게 되며 이러한 영향이 성장후에도 작용한다는 것이다

주장 Clearly increased levels of melatonin before birth cause shyness during infancy and this shyness continues into later life

1 25명의 아기로 결론 내리기에 샘플이 작다2 과학적 사실들에 대한 명확한 근거가 엇음3 다른 영향을 간과했다(다른 호르몬에 의한 영향 후천적인 성격형성의 영향)

GRE AWA John박 박정어학원

The following is a letter to the editor of the Atticus City newspaper

Former Mayor Durant owes an apology to the city of Atticus Both the damage to the River Bridge which connects Atticus to Hartley and the traffic problems we have long experienced on the bridge were actually caused 20 years ago by Durant After all he is the one who approved the construction of the bridge If he had approved a wider and better-designed bridge on which approximately the same amount of public money would have been spent none of the damage or problems would have occurred Instead the River Bridge has deteriorated far more rapidly over the past 20 years than has the much longer Derby Bridge up the river Even though the winters have been severe in the past several years this is no excuse for the negligence and wastefulness of Durant

Write a response in which you discuss what questions would need to be answered in order to decide whether the recommendation is likely to have the predicted result Be sure to explain how the answers to these questions would help to evaluate the recommendation

전임 시장인 Durant 씨는 Atticus 시에 대해 사과할 의무가 있습니다 Atticus와 Hartley를 잇는 River Bridge 교량에 대한 피해와 이 교량에서 오랫동안 주민들이 겪어오고 있는 교통 혼잡 문제들은 실제로 20년 전부터 시작된 것이었습니다 결정적으로 그가 교량 공사를 허가했던 바로 그 장본인입니다 당시 비슷한 공사비용으로 폭이 더 넓고 튼튼하게 설계된 교량을 허가했다면 이러한 문제나 피해는 발생하지 않았을 겁니다 더군다나 이 다리는 지난 20년 동안 상류에 건설된 훨씬 오래된 Derby 다리보다도 빠르게 부식되어 갔습니다 지난 수년동안 심지어 혹한이 있었다 하더라도 이러한 태만과 국고 손실에 대한 책임을 회피할 길이 없는 것입니다

결론 Former Mayor Durant owes an apology to the city of Atticus

1 디자인이 문제가 아닐수 있다 (디자인은 좋았으나 건설과정에 문제가 있었을 수 있다)2 그 당시의 시예산이 적어서 더 큰 다리를 짓기가 불가능했을 수도 있다3 교통량이 많거나 다른 상황으로 인해서 부식이 빨리 됐을 수 있다4 그 당시에는 최선의 선택이였지만 갑자기 변한 상황에 의해서 이런 문제점들이 발생했을 수 있다

GRE AWA John박 박정어학원

4그룹 거짓인과관계 오류 (False Cause) 빈출

Fifteen years ago Omega University implemented a new procedure that encouraged students to evaluate the teaching effectiveness of all their professors Since that time Omega professors have begun to assign higher grades in their classes and overall student grade averages at Omega have risen by 30 percent Potential employers looking at this dramatic rise in grades believe that grades at Omega are inflated and do not accurately reflect student achievement as a result Omega graduates have not been as successful at getting jobs as have graduates from nearby Alpha University To enable its graduates to secure better jobs Omega University should terminate student evaluation of professors

Write a response in which you discuss what specific evidence is needed to evaluate the argument and explain how the evidence would weaken or strengthen the argument

Omega professor evaluation implemented 15 years ago =gt Omega prof assign higher grades 30Employers believe therersquos grade inflation

Thus unsuccessful employment than AlphaTherefore to secure jobs Omega should end evaluating profs

Specific evidence neededRelationship between higher grades and evaluationRelationship between GPA and unsuccessful employmentAlpharsquos education could just be better than OmegaldquoFifteen years agordquo is a long time other factors could have influenced Why is the inflation a problem just now How much is Alpha better Is the comparison just How much gap is thereOmegarsquos student could just be doing better in their studiesComparison to other universities다른 대안 없나hellip Could Omega alleviate the employment problem by implementing a different procedure or program

15 년전 우리 대학은 학생들로 하여금 교수평가를 하도록 한 새로운 조치를 시행했었습니다 이후 교수들은 자신의 학과 학생들에게 높은 학점을 주었으며 그에따라 학생들의 전체 평점이 30나 올랐습니다 외부의 기업체들은 분명 점수가 지나치게 부풀려졌다고 믿고 있습니다 결국 본 대학 졸업생들이 인근 Alpha 대학의 졸업자들보다 구직률이 떨어지는 이유를 잘 보여주고 있는 것입니다 이를 해결하기 위해 이제부터는 학생들에 의한 교수평가제를 중단해야 합니다

결론 Omega University should now terminate student evaluation of professors

반박 교수 평가와 학점 인플레의 연관성이 적다( 교수 평가를 먼저하고 학점을 나중에 매길수도 있다)채용기준에 성적만 있는게 아니다 학업성취의 결과 일수도 있다 Alpha 가 원래 유능했다 Alpha 의 교육내용이 좋았다

GRE AWA John박 박정어학원

In this memo the dean of Omega University(OU) recommends OU to terminate professor evaluation to secure better jobs for the students To support this recommendation the dean offers several reasons However this argument contains several logical flaws which render it unconvincing

A threshold problem with the argument involves the voluntary nature of the evaluationprocedure The dean provides no evidence about the number or percentage of Omegastudents who participate in the procedure Lacking such evidence it is entirely possible thatthose numbers are insignificant in which case terminating the procedure is unlikely to haveany effect on the grade average of Omega students or their success in getting jobs aftergraduationThe argument also assumes unfairly that the grade-average increase is the result of theevaluation procedure--rather than some other phenomenon The dean ignores a host of otherpossible explanations for the increase--such as a trend at Omega toward higher admissionstandards or higher quality instruction or facilities Without ruling out all other possibleexplanations for the grade-average increase the dean cannot convince me that by terminatingthe evaluation procedure Omega would curb its perceived grade inflation let alone help itsgraduates get jobsEven if the evaluation procedure has resulted in grade inflation at Omega the deans claimthat grade inflation explains why Omega graduates are less successful than Alpha graduatesin getting jobs is unjustified The dean overlooks a myriad of other possible reasons forOmegas comparatively poor job-placement record Perhaps Omegas career services areinadequate or perhaps Omegas curriculum does not prepare students for the job market aseffectively as Alphas In short without accounting for other factors that might contribute toOmega graduates comparative lack of success in getting jobs the dean cannot justify theclaim that if Omega curbs its grade inflation employers will be more likely to hire OmegagraduatesFinally even if the dean can substantiate all of the foregoing assumptions the deansassertion that Omega must terminate its evaluation procedure to enable its graduates to findbetter jobs is still unwarranted in two respects First the dean ignores other possible ways bywhich Omega can increase its job-placement record--for example by improving its publicrelations or career-counseling services Second the dean unfairly equates more jobs withbetter jobs In other words even if more Omega graduates are able to find jobs as a result ofthe deans recommended course of action the kinds of jobs Omega graduates find would notnecessarily be better onesIn sum the deans argument is unpersuasive as it stands To strengthen it the dean mustprovide better evidence that the increase in grade average is attributable to Omegasprofessor-evaluation procedure and that the end result is a perception on the part ofemployers that Omega graduates are less qualified for jobs than Alpha graduates To betterassess the argument I would need to analyze 15-year trends in (l) the percentage of Omegastudents participating in the evaluation procedure (2) Omegas admission standards andquality of education and (3) Omegas emphasis on job training and career preparation I wouldalso need to know what other means are available to Omega for enabling its graduates to findbetter jobs

GRE AWA John박 박정어학원

The following appeared in a memo from a vice president of Quiot Manufacturing

During the past year Quiot Manufacturing had 30 percent more on-the-job accidents than at the nearby Panoply Industries plant where the work shifts are one hour shorter than ours Experts say that significant contributing factors in many on-the-job accidents are fatigue and sleep deprivation among workers Therefore to reduce the number of on-the-job accidents at Quiot and thereby increase productivity we should shorten each of our three work shifts by one hour so that employees will get adequate amounts of sleep

Write a response in which you examine the stated andor unstated assumptions of the argument Be sure to explain how the argument depends on these assumptions and what the implications are for the argument if the assumptions prove unwarranted

The following appeared in a memo from a vice president of Alta Manufacturing

During the past year Alta Manufacturing had thirty percent more on-the-job accidents than nearby Panoply Industries where the work shifts are one hour shorter than ours Experts believe that a significant contributing factor in many accidents is fatigue caused by sleep deprivation among workers Therefore to reduce the number of on-the-job accidents at Alta we recommend shortening each of our three work shifts by one hour If we do this our employees will get adequate amounts of sleep

Write a response in which you discuss what questions would need to be answered in order to decide whether the recommendation and the argument on which it is based are reasonable Be sure to explain how the answers to these questions would help to evaluate the recommendation

The following appeared in a memo from the vice president of Butler Manufacturing

During the past year workers at Butler Manufacturing reported 30 percent more on-the-job accidents than workers at nearby Panoply Industries where the work shifts are one hour shorter than ours A recent government study reports that fatigue and sleep deprivation among workers are significant contributing factors in many on-the-job accidents If we shorten each of our work shifts by one hour we can improve Butler Manufacturings safety record by ensuring that our employees are adequately rested

1 Write a response in which you discuss what specific evidence is needed to evaluate the argument and explain how the evidence would weaken or strengthen the argument

2 Write a response in which you discuss what questions would need to be answered in order to decide whether the recommendation is likely to have the predicted result Be sure to explain how the answers to these questions would help to evaluate the recommendation

4번 반복됨

Alta has 30 more job accidents than Panoply(work shifts one hour shorter)Experts Job accidents caused by fatigue and sleep deprivationTherefore to reduce job accidents and increase productivity shorten three work shifts by one hour for adequate sleep

지난해 우리 회사는 인근 Panoply Industries보다 업무상 재해가 30나 더 많았다 그 회사는 우리보다 근무 교대시간이 1시간 정도 짧았다 전문가들은 대부분의 업무상 재해에 있어서 가장 중요한 요인이 과로와 수면부족으로 보고있다 따라서 우리 회사에서 높은 산업재해를 줄이고 아울러 생산성을 높이기 위해서는 근로자들이 충분한 수면을 취할 수 있도록 1시간씩 3교대 시간을 줄여야 한다

In this memo the (author) vice president of Alta Manufacturing (AM) recommends that to reduce on-the-job accidents and increase productivity AM should shorten its three work shifts by one hour so that employees can

GRE AWA John박 박정어학원

get more sleep To support this recommendation the author provides several evidences However careful scrutiny of each of the facts reveals that it provides little credible support for the authorrsquos recommendation QuestionsThe number of accidents What kind of accidents The seriousness of the accidents is importantHow many employees are in each company What are their productsFalse cause Sleep may not be the reason for the on-the-job accidents What do Alta and Panoply manufacture

First of all the author believes that fatigue caused the on-the job accidents However there could be other reasons The author observes a correlation between sleep deprivation and on-the-job accidents then concludes that the former is the cause of the latter However the author fails to rule out other possible explanations For example it is entirely possible that Alta factories require more strenuous and dangerous labor than Panoply Without ruling out all other such factors it is unfair to conclude that fatigue is responsible for the accidents In addition the work-shifts may not be the cause of the sleep deprivation and fatigue It is possiblehellip Thus the author should provide what exactly Panoply and Alta manufacture and more precise data about their working conditions to be more convincing

Shortening the shift by one hour does not necessarily lead to more sleep And is one hour enoughLess accidents does not mean increased productivity

결론 We should shorten each of out three work shifts by one hour

반박 경쟁사에 비해서 시간당 하는 업무량이 많아서 더욱 피곤할 수도 있다 시간이 문제가 아니라 노후된 시설 설비 자체의 문제 작업 자체가 원래 위험한 것이여서 사고가 많을 수도 있다 다른 회사는 더욱 많은 작업시간에도 불구하고 안정한 작업여건으로 인해서 사고율이 오히려 더 작을 수도 있다비교사의 재해감소가 다른 요인일수 있다(안전 교육 철저)줄인 시간이 피로회복이나 수면으로 연결 안될수 있음(술을 마실 수도 있고 그 시간에 휴식을 취하지 않고 다른일을 함으로써 더욱 피로해질수 있다)

This editorial recommends that Alta Manufacturing reduce its work shifts by one hour each inorder to reduce its on-the-job accident rate and thereby increase Altas productivity To supportthis recommendation the author points out that last year the number of accidents at Alta was30 greater than at Panoply Industries where work shifts were one hour shorter The authoralso cites certain experts who believe that many on-the-job accidents are caused by fatigueand sleep deprivation I find this the argument unconvincing for several reasonsFirst and foremost the author provides absolutely no evidence that overall workerproductivity is attributable in part to the number of on-the-job accidents Although commonsense informs me that such a relationship exists the author must provide some evidence ofthis cause-and-effect relationship before I can accept the authors final conclusion that theproposed course of action would in fact increase Altas productivitySecondly the author assumes that some accidents at Alta are caused by fatigue or sleepdeprivation However the author overlooks other possible causes such as inadequateequipment maintenance or worker training or the inherent hazards of Altas manufacturingprocesses By the same token Panoplys comparatively low accident rate might be attributablenot to the length of its work shifts but rather to other factors such as superior equipmentmaintenance or worker training In other words without ruling out alternative causes ofon-the-job accidents at both companies the author cannot justifmbly conclude that merely byemulating Panoplys work-shift policy Alta would reduce the number of such accidentsThirdly even assuming that Altas workers are fatigued or sleep-deprived and that this is thecause of some of Altas on-the-job accidents in order to accept the authors solution to thisproblem we must assume that Altas workers would use the additional hour of free time tosleep or rest However the author provides no evidence that they would use the time in thismanner It is entirely possible that Altas workers would use that extra hour to engage in someother fatiguing activity Without ruling out this possibility the author cannot convincinglyconclude that reducing Altas work shifts by one hour would reduce Altas accident rateFinally a series of problems with the argument arise from the scant statistical information onwhich it relies In comparing the number of accidents at Alta and Panoply the author fails toconsider that the per-worker accident rate might reveal that Alta is actually safer than Panoplydepending on the total number of workers at each company Second perhaps accident rates

GRE AWA John박 박정어학원

at the two companies last year were aberrations and during other years Altas accident ratewas no greater or even lower than Panoplys rate Or perhaps Panoply is not representativeof industrial companies generally and that other companies with shorter work shifts have evenhigher accident rates In short since the argument relies on very limited statistical information Icannot take the authors recommendation seriouslyIn conclusion the recommendation for emulating Panoplys work-shift policy is not wellsupported To convince me that shorter work shifts would reduce Altas on-the-job accidentrate the author must provide clear evidence that work-shift length is responsible for some ofAltas accidents The author must also supply evidence to support her final conclusion that alower accident rate would in fact increase overall worker productivity

The following appeared in a memo from the vice president of marketing at Dura-Sock Inc

A recent study of our customers suggests that our company is wasting the money it spends on its patented Endure manufacturing process which ensures that our socks are strong enough to last for two years We have always advertised our use of the Endure process but the new study shows that despite our socks durability our average customer actually purchases new Dura-Socks every three months Furthermore our customers surveyed in our largest market northeastern United States cities say that they most value Dura-Socks stylish appearance and availability in many colors These findings suggest that we can increase our profits by discontinuing use of the Endure manufacturing process

1 Write a response in which you examine the stated andor unstated assumptions of the argument Be sure to explain how the argument depends on these assumptions and what the implications are for the argument if the assumptions prove unwarranted

2 Write a response in which you discuss what specific evidence is needed to evaluate the argument and explain how the evidence would weaken or strengthen the argument

3 Write a response in which you discuss what questions would need to be answered in order to decide whether the recommendation and the argument on which it is based are reasonable Be sure to explain how the answers to these questions would help to evaluate the recommendation

Intro The vice president of marketing at Dura-Sock Inc is offering a potentially harmful investment recommendation by claiming that Dura-Sock should discontinue its use of the ldquoEndurerdquo process To support his recommendation he points out a study that Dura-Sock customers actually purchase the socks every three months and a survey that reveals that Dura-Sock customers like the sockrsquos stylish appearance and availability in many colors The study and survey however are insufficient in supporting his proposal and the VP makes several unwarranted assumptionsIntro (simplified) The VP states that though Dura-Socks last for two years customers buy the socks every three months Therefore he assumes that the consumersrsquo motive for buying the produce is not its durabilityHowever the author fails to rule out other possible motivation for consumption

Even if the survey is reliable the author should consider the rest of the market Vague terms ldquowasting moneyrdquomdashprecisely how much are they wasting Studysurvey errorThe company must calculate the outcome of such momentous decisionStudy participantsrsquo comment that they prefer Dura-Sock for its stylishness and availability might take Dura-Sockrsquos enduring quality for granted

우리회사 제품 소비자들에 대한 최근 조사에서 지난 2년여간 양말의 내구성을 강하게 하는 필수공정이었던 자사 특허의 Endure 공정에 들어가는 비용이 낭비라고 말하고 있다 우리 회사는 항상 이 공정 처리에 대한 광고를 내보냈으나 이에 대한 시장 조사에서 실제로 고객들은 이 신제품을 평균 석달마다 구매하는 것으로 나타났다 더군다나 북동부지역에서 실시한 대규모 시장조사에 응답한 고객들은 양말의 모양과 색상등에 더

GRE AWA John박 박정어학원

관심을 나타냈다 이러한 결과는 우리회사가 신기술 공법을 중단하면 그에 따라 수익이 늘어날 것이라는 것을 말해주고 있는 것이다주장 These findings suggest that Dura0Sock can increase its profits by discontinuing its use of the ldquoEndurerdquo manufacturing process

1 survey가 정확한 소비자의 의견을 나타낸 것인가 다른 선택없이 양자택일과 같은 방법의 survey였는지2 북동부지역의 시장조사가 전체 의견을 대표할 수 있나3 사람들이 모양이나 색상에 앞서 내구성을 먼저 평가했을 수도 있다 내구성을 갖추었다는 전제하에 모양과 색상에 관심을 드러낸 것일 수 있다4 소비자가 도매상(retail)인지 소매상(whole)인지가 없다

The following appeared in a business magazine

As a result of numerous complaints of dizziness and nausea on the part of consumers of Promofoods tuna the company requested that eight million cans of its tuna be returned for testing Promofoods concluded that the canned tuna did not after all pose a health risk This conclusion is based on tests performed on samples of the recalled cans by chemists from Promofoods the chemists found that of the eight food chemicals most commonly blamed for causing symptoms of dizziness and nausea five were not found in any of the tested cans The chemists did find small amounts of the three remaining suspected chemicals but pointed out that these occur naturally in all canned foods

Write a response in which you discuss what questions would need to be addressed in order to decide whether the conclusion and the argument on which it is based are reasonable Be sure to explain how the answers to the questions would help to evaluate the conclusion

Representativeness of the tested cansThey should conduct a comparative studyThe testing could be biased because Promofoods employees conducted the testingHow much (quantity) of the five and three suspected chemicals were in the canned foodsFalse cause The substance that caused dizziness and nausea may not be one of the eight common chemicals

많은 소비자들의 현기증과 구역질 불만에 따라 Promofoods사는 지난해 참치 캔 8백만 개를 테스트하기 위해 반품시켰다 그 결과 캔에서는 건강에 위험이 될 수 있는 화합물질이 없었던 것으로 회사측은 결론지었다 이러한 결론은 회사측 화학연구자들이 회수된 캔의 샘플을 테스트해서 이들 증상의 원인이 되는 8가지 화합물 중에서 5가지가 실험된 캔에서 발견되지 않았다는 사실에 근거한 것이다 이들 화학자들은 나머지 3개가지 화합물이 모든 캔 식료품에서 흔히 발견되는 것이라고 언급했다 결론 Promofoods concluded that the cans did not after all contain chemicals that posed a health risk

1 공인된 기간에서 테스트를 한 것이 아니고 자사에서 직접 테스트를 했기에 신뢰성이 안간다 2 이런 증상을 일으키는 8개의 물질 말고 다른 물질들이 캔속에 많이 포함됬을수 있다 3 나머지 3개의 물질들의 함유량이 많아서 다른 종류의 캔들은 문제를 일으키지 않지만 참치캔은 문제를

일으킬 수 있다

This magazine article concludes that the 8 million cans of tuna Promofoods recalled due tocomplaints about nausea and dizziness do not after ail contain any chemicals that pose a

GRE AWA John박 박정어학원

health risk To support this conclusion the author cites the fact that five of eight chemicalscommonly causing these symptoms were not found in the recalled cans while the other threealso occur naturally in other canned foods For several reasons this evidence lends littlecredible support to the authors conclusionTo begin with the author relies partly on the fact that although three of the eight chemicalsmost commonly blamed for nausea and dizziness appeared in Promofoods recalled tunathese chemicals also occur naturally in other canned foods However this fact alone lends nosupport to the authors conclusion for two reasons First the author might be ignoring animportant distinction between naturally occurring chemicals and those not occurring naturallyIt is entirely possible that these three chemicals do not occur naturally in Promofoods tunaand that it is for this reason that the chemicals cause nausea and dizziness Secondly it isentirely possible that even when they occur naturally these chemicals cause the samesymptoms Unless the author rules out both possibilities he cannot reliably conclude that therecalled tuna would not cause these symptomsAnother problem with the argument is that the authors conclusion is too broad Based onevidence about certain chemicals that might cause two particular heath-related symptoms theauthor concludes that the recalled tuna contains no chemicals that pose a health risk Howeverthe author fails to account for the myriad of other possible health risks that the recalled tunamight potentially pose Without ruling out all other such risks the author cannot justifiablyreach his conclusionA third problem with the argument involves that fact that the eight particular chemicals withwhich the test was concerned are only the eight most commonly blamed for nausea anddizziness It is entirely possibly that other chemicals might also cause these symptoms andthat one or more of these other chemicals actually caused the symptoms Without ruling outthis possibility the author cannot jusufiably conclude that the recalled tuna would not causenausea and dizzinessA final problem with the argument involves thetesting procedure itself The author providesno information about the number of recaUed cans tested or the selection method used Unlessthe number of cans is a sufficiently large sample and is statistically repre sentative of all therecalled cans the studys results are not statistically reliableIn conclusion the article is unconvincing as it stands To strengthen the assertion that therecalled tuna would not cause nausea and dizziness the author must provide evidence thatthe three chemicals mentioned that occur naturally in other canned foods also appear naturallyin Promofoods tuna The author must also provide evidence that ingesting other canned foodscontaining these three chemicals does not cause these symptoms To better evaluate theargument we would need to know whether the sample used in the tests was statisticallysignificant and representative of all the recalled tuna We would also need to know what otherchemicals in the recalled tuna might pose any health risk at all

5그룹 불충분 조건오류 빈출

Natures Way a chain of stores selling health food and other health-related products is opening its next franchise in the town of Plainsville The store should prove to be very successful Natures Way franchises tend to be most profitable in areas where residents lead healthy lives and clearly Plainsville is such an area Plainsville merchants report that sales of running shoes and exercise clothing are at all-time highs The local health club has more members than ever and the weight training and aerobics classes are always full Finally Plainsvilles schoolchildren represent a new generation of potential customers these schoolchildren are required to participate in a fitness-for-life program which emphasizes the benefits of regular exercise at an early age

Write a response in which you examine the stated andor unstated assumptions of the argument Be sure to

GRE AWA John박 박정어학원

explain how the argument depends on these assumptions and what the implications are for the argument if the assumptions prove unwarranted

False cause

First of all the author believes that the Increased sales of running shoes and exercise clothing indicates

Plainesville residentsrsquo interest in leading healthy lives However this assumption is not logically convincing for

several reasons could be a fashion trendTime shift ldquoFitness for liferdquo might not have any influence on schoolchildren as they growFalse cause There could be other reasons for member increase in the health clubAll of the above are insufficient condition

The author has to prove that local residents are interested in leading healthy lives However he supports his conclusion with insufficient evidence Nevertheless even if the residents are concerned with health naturersquos way may not be successful First

그 동안의 경험을 토대로 볼 때 건강생활과 밀접히 관련되어 있는 거주 지역에서 본 상점들이 아주 호응을 얻고 있다 따라서 이러한 주민들이 많이 거주하고 있는 Plainsville 에 새로운 상점들을 계속 세워야 한다 이 지역 상인들은 런닝화와 운동복 판매가 가장 높다고 말한다 불과 5 년전에는 거의 전무하다시피하던 지역 헬스 클럽의 경우도 엄청나게 많은 회원을 확보하고 있으며 웨이트 트레이닝과 에어로빅 강좌들도 항상 만원이라고 한다 새로운 고객층을 예측해 보는 것도 가능하다 이 지역의 학생들의 경우 Fitness for Life프로그램을 받게 되는데 이러한 프로그램을 통해서 유년시절부터 정규적인 운동 습관을 들이게 하고 있는 것이 그것이다

결론 We should therefore build our next new store in Plainsville

반박 그동안의 경험에 의한 과거 통계가 꼭 여기에도 적용되는건 아니다 5 년전 헬스 클럽이 잘 안되었던게 다른 원인이였을수 있다(강사수준 미달 강좌미비)tourist 에 의한 원인 일수 있다 어렸을때부터 운동을 했다고 해서 커서도 관심이 있지는 않다 (오히려 반감이 있을수 있다 혹은 건강하기에 건강에 관심이 적을수도 있다)운동복이나 신발의 판매가 육체노동에 의한 것일수도 있다

IntroductionSupport1049896In this memorandum the author asserts that Naturersquos Way should build its next newstore in Plainsville To support this assertion the author states that Plainsvillesmerchantsrsquo sales of exercise clothing are going well the local health club has moremembers than ever and a new generation of customers will help to ensure NaturersquosWayrsquos success At first glance the authorrsquos assumption seems convincing but in-depth scrutiny revealsthat it lacks substantial evidence as it stands

Body 1-SamplingTopic Sentence 1To begin with the author assumes that the merchantsrsquo report indicates that the residentsare concerned about their health However this assumption is based on unsubstantiated

GRE AWA John박 박정어학원

data Example 1 (Rebuttal1) First if we do not know the total volume of items sold and the price of the goods exactly we cannot infer whether the residents are actually buying many goods Example 2 (Rebuttal2)In addition to that the report emphasizes the rising sales of running shoes and exerciseclothing however these may not be hot-selling items for Naturersquos Way or may not be theproducts the company is planning to sell Concluding Sentence Therefore in order to make the argument reliable the author should reconsider themerchantsrsquo report with more detailed data

Body 2-CausalTopic Sentence 2Second the author contends that the health clubs classes are full yet this does not meanthat many people actually use the health club other factors may be the real cause forthose closed classes Example 1 (Rebuttal1) To begin with if the health club is very small the number of people working out wouldnot be a large one In fact regular gym-going may just be a vogue among a smallunrepresentative segment of Plainsvilles population Example 2 (Rebuttal2) Moreover it is possible that most of the people who exercise in the health club do weight training and aerobics only to look good and to meet other singles not for their health In that case there would be little demand for health products Concluding SentenceThus the author should not hasten to presume what really caused people to be interested in a healthier lifestyle and enroll in the health club

Body 3-Time-ShiftTopic Sentence 3Finally the author highlights that Naturersquos Way can expect a new generation of customersin Plainsville that will help the company in the long term This notion is mistaken in that itassumes the conditions of the present will continue unchanged in the future Although theschool children are required to participate in the fitness for life program they may notnecessarily buy Naturersquos Ways products Example 1 (Rebuttal1) In the first instance they may suffer a fall in purchasing power arising from future economic difficulties this would cause reluctance to spend a considerable amount of money on health products which tend to be more expensive Example 2 (Rebuttal2)Another possibility is that there may emerge many competitor companies vying with Naturersquos Way so that in the future the school children may not feel the necessity to purchase one companyrsquos health products over anotherrsquosConcluding Sentence Thus the authorrsquos assumption is highly speculative since it relies heavily on unknowablefuture circumstances

ConclusionThesis In sum the author uses many assumptions that are insufficient in supporting his claimsSupportIn order for the authorrsquos claims to be convincing he needs to advance more persuasiveevidence that people in Plainsville really are concerned with their health and health foodThe following was written as a part of an application for a small-business loan by a group of developers in the city of Monroe

A jazz music club in Monroe would be a tremendously profitable enterprise Currently the nearest jazz club is 65 miles away thus the proposed new jazz club in Monroe the C-Note would have the local market all to itself Plus jazz is extremely popular in Monroe over 100000 people attended Monroes annual jazz festival last summer several well-known jazz musicians live in Monroe and the highest-rated radio program in Monroe is Jazz Nightly which airs every weeknight at 7 PM Finally a nationwide study indicates that the typical jazz fan spends close to $1000 per year on jazz entertainment

1 Write a response in which you discuss what specific evidence is needed to evaluate the argument and explain how the evidence would weaken or strengthen the argument

2 Write a response in which you examine the stated andor unstated assumptions of the argument Be

GRE AWA John박 박정어학원

sure to explain how the argument depends on these assumptions and what the implications are for the argument if the assumptions prove unwarranted

3 Write a response in which you discuss what questions would need to be answered in order to decide whether the prediction and the argument on which it is based are reasonable Be sure to explain how the answers to these questions would help to evaluate the prediction

Group error nationwide survey may not reflect local trends Is the nationwide jazz fan population substantialInsufficient non-residents of Monroe may have attended the jazz festival (Body alternative explanation last year may have been an anomaly The author should consider data from various years) The author should indicate how many out of 100000 were Monroe residentsNationwide study Does this reflect Insufficient Citizens of Monroe may continue to go to the jazz club 65 miles away

Are the people in Monroe really interested in jazzMajority of the people who attended the jazz festival might not be Monroe residentsSurvey error nationwide study may not be applicable to MonroeJazz musicians who live in MonroeMonopolyRadio station

In this business application the author claims that the proposed jazz club C Note will be very profitable in Monroe To support this claim the author argues for his case with several evidences At first glance the authorrsquos argument seems convincing however careful scrutiny reveals that his argument in specious

To begin with the author claims that Monroersquos citizens are interested in jazz He presents three evidences First Secondhellip Thirdhellip Howeverhellip

Monroe 시에 있는 재즈 음악 클럽은 수익성이 좋은 사업이다 현재 가장 가까이에 있는 클럽은 65 마일 정도 떨어져 있다 따라서 이번에 세우려고 하는 C Note 는 독보적인 위치를 점할것이다 더군다나 재즈는 이 시에서 가장 인기있는 음악이다 지난 여름 재즈 축제에서는 10 만명 이상의 Morone 시 주민이 참석하였고 몇몇 유명한 재즈 음악가들도 이곳에 살고 있으며 저녁때 방영되는 라디오 프로그램중에서 최고의 시청률을 보이고 있는 것도 Jazz Nightly 이다 전국조사에서도 전형적인 재즈 팬들은 재즈 분야에 년간 1천 달러 가까이 지출하고 있는 것으로 보고되고 있다 따라서 C Note 클럽이 돈을 벌 수 있는 사업이라는 것은 확실한 것이다

결과 It is clear that the C Note cannot help but make money반박 nearest jazz club 이 양질의 써비스로 여전히 손님을 끌수도 있다Festival 에 얼마나 참여하는지가 jazz 의 인기를 반영하지 않는다 뮤지션이 많이 사는거랑 jazz 의 인기가 상관없다라디오 프로그램이 다른 요인에 의해서 인기일수도 있다 (진행자때문)전국 통계 적용 불가화목 실전반_Ms Noh6In this application the author suggests that a jazz club in Monroe will make a number of profits To support this suggestion the author exemplifies the local condition popularity of jazz in Monroe and nationwide study However careful scrutiny of each of the facts reveals that it provides little credible support for the authorrsquos recommendation Good clear intro

First the author assumes that jazz is popular in Monroe because of several facts the jazz festival last year had high participation some famous jazz musicians live in Monroe and the high-rated radio program is lsquoJazz Nightlyrsquo However this assumption has many drawbacks that must be seriously considered(Good topic sentences) If many attendants in the last-yearrsquos festival came from other cities and not Monroe it is hard to conclude that Monroersquos people like jazz Therefore the author must examine how many Monroe residents actually attended the festival On top of that there is little relationship between habitation of famous jazz musician and the popularity of jazz in Monroe Although several well-known musicians live there if they do not take part in any jazz performance of Monroe this might have no effect to the interest of Monroersquos residents

GRE AWA John박 박정어학원

about jazz Finally in the case of radio program this is also not suitable reason why jazz is popular in Monroe It might be possible that people cannot help choosing lsquoJazz Nightlyrsquo because there are few radio programs at Night The fact that the radio program is the highest rating program is not a germane evidence The approximate number of listeners would be the more crucial evidence Therefore the author needs to seriously deliberate the correlation between jazzrsquos popularity in Monroe and his examples (Good logical flow and clarity)

Second the author uses as evidence the nationwide study that jazz fans spend much money on jazz entertainment to substantiate why starting a jazz club in Monroe will be profitable In other words the author assumes that the characteristics of a nationwide study can be applied to Monroe The national study would lend support to the applicantrsquos claim only if residents in Monroe typify national jazz fans However the author does not provide credible evidence that this is the case Moreover the populations of jazz fans nationwide may be insubstantial Thus the author should not infer hastily that Monroersquos residents will spend much money on enjoying jazz from the nationwide study

Lastly even if jazz is popular in Monroe C Note may not be successful It is entirely possible that residents might still prefer other clubs where they have always went In addition there is another possibility that the nearest jazz club will attract many of Monroersquos people because it serves fine performances and is equipped with favorite facilities Without considering these other possibilities the author cannot make his argument convincing In sum the author presents many reasons that are insufficient in supporting his or her claim In order for the authorrsquos claims to be convincing he needs to advance more persuasive evidence such as the total number of Monroe residents who attended the jazz festival the effects on the popularity of jazz by the musicians living in Monroe and the actual number of residents who would typify themselves to be jazz fans through a local survey Without substantial evidence that C Note will be successful in Monroe the businessmen may be overinvesting in what might lead to a business failureExcellent clarity Score 50

The following appeared in a newsletter offering advice to investors

Over 80 percent of the respondents to a recent survey indicated a desire to reduce their intake of foods containing fats and cholesterol and today low-fat products abound in many food stores Since many of the food products currently marketed by Old Dairy Industries are high in fat and cholesterol the companys sales are likely to diminish greatly and company profits will no doubt decrease We therefore advise Old Dairy stockholders to sell their shares and other investors not to purchase stock in this company

Write a response in which you discuss what questions would need to be answered in order to decide whether the advice and the argument on which it is based are reasonable Be sure to explain how the answers to these questions would help to evaluate the advice

Survey 80

GRE AWA John박 박정어학원

Old Dairy could change their products and manufacture low fat dairy foodsLess competing companies Old Dairy could eventually be the only company that produces hellipImprecise numbers and measurementsCustomers may still buy high fat dairy products

The author of the newsletter is offering potentially dangerous advice by recommending Old Dairy stockholders to withdraw investment and stop purchase What is more the authorrsquos prediction debases the reputation and business of Old Dairy and if false could devoid the investment opportunity of the newsletter readers Therefore investors should examine whether the authorrsquos evidences are substantial

To begin with the author states that 80 percent of the respondents in a survey indicated a desire to reduce their intake of foods He therefore argues that Old Dairyrsquos high fat and cholesterol products would decrease in sales However the author makes a crucial error in this argument First the author provides no evidence that the surveyrsquos results are statistically reliable Were they representative of all the customers Were they chosen for the survey randomly Furthermore the desire to reduce fat and cholesterol intake is a pervasive trend in todayrsquos opulent society however the author erroneously identifies this as a new phenomenon which will affect consumer trends Second having a desire to reduce fat and cholesterol intake does not necessarily indicate that people who have this desire will actually reduce consuming these types of products It is entirely possible that they may continue buying Old Dairy products for its quality and taste Accordingly the author cannot draw any firm conclusion that people will not buy Old Dairy products Therefore if any of these cases are true the author may be offering investors a detrimental investment advice

최근 조사에 대한 응답자중 80 이상이 자신이 먹는 음식에서 지방과 콜레스테롤의 함유량을 줄이고 싶다고 한다 아울러 요즘은 많은 식료품 가계에서 저지방 제품들을 많이 취급하고 있다 현재 Old Dairy Industries가 판매하고 있는 많은 음식제품들은 지방과 콜레스테롤이 높기 때문에 이 회사의 매출이 격감할 것으로 보이며 당연히 매출이익도 줄어들것이다 따라서 이 회사의 주주들은 주식을 매각하고 다른 주식 투자가들도 이 회사의 주식을 매입하지 않는 것이 좋다

결론 Old Dairy stockholders to sell their shares and other investors not to purchase stock in this company

반박 모든 상품이 다 고 지방 고 칼로리는 아니다(비록 많을지라도) 일부의 식품의 경우 기호에 맞어서 히트해서 전체적인 수입이 증가할 수도 있다국내시장만 생각할 수 없다( 외국시장에서 호황을 누릴수 있다 )입맛이라는게 즉각 바뀌는게 아니다

The following appeared in a letter to the editor of the Balmer Island Gazette

On Balmer Island where mopeds serve as a popular form of transportation the population increases to 100000 during the summer months To reduce the number of accidents involving mopeds and pedestrians the town council of Balmer Island should limit the number of mopeds rented by the islands moped rental companies from 50 per day to 25 per day during the summer season By limiting the number of rentals the town council will attain the 50 percent annual reduction in moped accidents that was achieved last year on the neighboring island of Seaville when Seavilles town council enforced similar limits on moped rentals

1 Write a response in which you discuss what questions would need to be answered in order to decide whether the recommendation is likely to have the predicted result Be sure to explain how the answers to these questions would help to evaluate the recommendation

2 Write a response in which you discuss what questions would need to be answered in order to decide whether the prediction and the argument on which it is based are reasonable Be sure to explain how the answers to these questions would help to evaluate the prediction

3 Write a response in which you examine the stated andor unstated assumptions of the argument Be sure to explain how the argument depends on these assumptions and what the implications are for the argument if the assumptions prove unwarranted

Whatrsquos the actual population of Balmer Island 100000mdashis this a significant increase What kind of accidents Skin abrasions or serious injury And compared to Seaville how serious are the accidents and the actual number of accidents Did Seaville enforce other restrictions like safety signsHow different are the conditions of Balmer

GRE AWA John박 박정어학원

and Seaville regarding population road (safety) conditions topography other town-government regulation How much will the economy of Balmer be affected do to this restriction Could it cause an economic recession due to the fact that these rental companiesrsquo chance to make money is only during the summer thereby weakening the economic infrastructure Are there any other ways that could better alleviate the accident rate

Statistics 50-impreciseAnalogy Balmer compared with TorseauFalse Cause Accidents might have occurred because of reasons other than mopeds False Cause population increase may not be part of the cause of the accidentsOther explanations for the accident pedestrians few road safety regulations narrow roadsThere could be other better solutionshellip

Balmer Island의 인구가 여름철에는 십만명으로 늘어난다 2륜차와 보행자간 사고를 줄이기 위해 시의회는 6개의 자전거를 포함한 2륜차 대여업체에게 이 기간동안에는 대여숫자를 일일 50에서 30으로 제한하도록 할 것이다 대여숫자를 줄임으로써 시 의회는 지난해 이웃한 Torseau섬에서 이와 동일한 규제를 시행해서 50나 줄인 결과를 보고 마찬가지로 50를 줄일수 있다고 확신하고 있다

결론 The town council of Balmer Island should linit the number

반박 보행자의 부실에 의해서 사고가 많이 일어날수도 있다렌탈수의 줄임만이 대책은 아니다(대부분의 사람들이 렌탈 보다는 소유하고 있을 수도 있다)옆섬과는 상황이 다를수도 있다(그 섬에서는 사고의 원인이 많은 자전거 수로 인한것일수있다) 하지만 이 섬은 좁은 도로가 원인일 수도 있고 도로 안전 장치의미비가 원일일수 있다

In this letter the author recommends that Balmer Island should limit the number moped rentals from 50 to 30 per day To support this recommendation the author points out several reasons However careful scrutiny of each of the facts reveals that it is filled with unanswered questions that could significantly weaken the authorrsquos recommendation with loops and holes which are answered

The recommendation depends on the assumption that no alternative means of reducing the number of accidents are available However the author fails to offer any evidence to substantiate this crucial assumption It is highly possible that means other than this would better solve the problem Perhaps they could widen the roads or put-up more safety signs Or perhaps the accidents were due to the lack of skills in which case proper safety training would significantly alleviate the problem Without considering and ruling out these and other alternative means of reducing accidetns the author cannot confidently conclude that merely emulating Torseau would suffice Moreover the author is advising a recommendation which could potentially harm the economy of Balmer Island sincehellip Moreover the Balmer Island should alternative means to reduce accidents because limiting moped rentals during the summer could harm the economy of Balmerhellip

First of all the author believes that increase in population and the number of moped rentals are responsible for the accidents It is entirely possible that other factors are responsible for the accidents Perhaps Balmer Islandrsquos lack of safety signs was a major factor Or maybe the roads are narrow and dangerous on the Island therefore the town council could enforce stricter traffic regulations to alleviate the problem Accordingly if either of these scenarios is true the author cannot draw any firm conclusion that increase in the number of population and moped rentals are the cause of the accidents

The author of this editorial recommends that to reduce accidents involving mopeds andpedestrians Balmer Islands city council should restrict moped rentals to 30 per day down from50 at each of the islands six rental outlets To support this recommendation the author citesthe fact that last year when nearby Torseau Islands town council enforced similar measuresTorseaus rate of moped accidents fell by 50 For several reasons this evidence providesscant support for the authors recommendationTo begin with the author assumes that all other conditions in Balmer that might affect therate of moped-pedestrian accidents will remain unchanged after the restrictions are enactedHowever with a restricted supply of rental mopeds people in Balmer might purchase mopedsinstead Also the number of pedestrians might increase in the future with more pedestriansespecially tourists the risk of moped-pedestrian accidents would probably increase For thatmatter the number of rental outlets might increase to make up for the artificial supplyrestriction per outlet--a likely scenario assuming moped rental demand does not declineWithout considering and ruling out these and other possible changes that might contribute to ahigh incidence of moped-pedestrian accidents the author cannot convince me that theproposed restrictions will necessarily have the desired effect

GRE AWA John박 박정어학원

Next the author fails to consider other possible explanations for the 50 decline inTorseaus moped accident rate last year Perhaps last year Torseau experienced unusually fairweather during which moped accidents are less likely Perhaps fewer tourists visited Tot seanlast year than during most years thereby diminishing the demand for rental mopeds to belowthe allowed limits Perhaps last year some of Torseaus moped rental outlets purchased newmopeds that are safer to drive Or perhaps the restrictions were already in effect but were notenforced until last year In any event a decline in Torseaus moped accident rate during onlyone year is scarcely sufficient to draw any reliable conclusions about what might have causedthe decline or about what the accident rate will be in years aheadAdditionally in asserting that the same phenomenon that caused a 50 decline in mopedaccidents in Torseau would cause a similar decline in Balmer the author relies on what mightamount to an unfair analogy between Balmer and Torseau Perhaps Balmers ability to enforcemoped-rental restrictions does not meet Torseaus ability if not then the mere enactment ofsimilar restrictions in Balmer is no guarantee of a similar result Or perhaps the demand formopeds in Torseau is always greater than in Balmer Specifically if fewer than all availablemopeds are currently rented per day from the average Balmer outlet while in Torseau everyavailable moped is rented each day then the proposed restriction is likely to have less impacton the accident rate in Balmer than in TorseauFinally the author provides no evidence that the same restrictions that served to reduce theincidence of all moped accidents by 50 would also serve to reduce the incidence ofaccidents involving mopeds and pedestrians by 50 Lacking such evidence it is entirelypossible that the number of moped accidents not involving pedestrians decreased by a greaterpercentage while the number of moped-pedestrian accidents decreased by a smallerpercentage or even increased Since the author has not accounted for these possibilities theeditorials recommendation cannot be taken seriouslyIn conclusion the recommendation is not well supported To convince me that the proposedrestriction would achieve the desired outcome the author would have to assure me that nochanges serving to increase Balmers moped-pedestrian accident rate will occur in theforeseeable future The author must also provide dear evidence that last years decline inmoped accidents in Torseau was attributable primarily to its moped rental restrictions ratherthan to one or more other factors In order to better evaluate the recommendation I wouldneed more information comparing the supply of and demand for moped rentals on the twoislands I would also need to know the rate of mopedpedestrian accidents in Torseau both priorto and after the restrictions were enforced in TorseauThe following appeared in a magazine article about planning for retirement

Clearview should be a top choice for anyone seeking a place to retire because it has spectacular natural beauty and a consistent climate Another advantage is that housing costs in Clearview have fallen significantly during the past year and taxes remain lower than those in neighboring towns Moreover Clearviews mayor promises many new programs to improve schools streets and public services And best of all retirees in Clearview can also expect excellent health care as they grow older since the number of physicians in the area is far greater than the national average

Write a response in which you discuss what specific evidence is needed to evaluate the argument and explain how the evidence would weaken or strengthen the argument

-Natural beauty and consistent climate may not be the most wanted qualities-Housing costs could have lowered on a national level wealthy retirees may not care about costs-Taxes may be high compared to the nationrsquos average tax rate-What about other qualities of Clearview Crime rate what qualities would retirees want -If schools streets and public services need improvement then this is proof that the current condition of Clearview is low Or due to budgetary reasons the mayor may not follow-up on his promise because of lowered tax rate -Schools and people who are retired no relationship-Physicians What kind of physicians Number is irrelevant Are these physicians capable of addressing the illnesses of old people

This author argues that anyone seeking a place to retire should choose Clearview To supportthis argument the article cites Clearviews consistent climate and natural beauty its fallinghousing costs its low property taxes compared to nearby towns and the mayors promise toimprove schools streets and services The article also claims that retirees can expectexcellent health care because the number of physicians in Clearview greatly exceeds thenational average This argument is flawed in several critical respectsTo begin with although consistent climate and natural beauty might be attractive to manyretirees these features are probably not important to all retirees For many retirees it isprobably more important to live near relatives or even to enjoy changing seasons Thus I

GRE AWA John박 박정어학원

cannot accept the authors sweeping recommendation for all retirees on this basisAlso Clearviews declining housing costs do not necessarily make Clearview the best placeto retire for two reasons First despite the decline Clearviews housing costs might be highcompared to housing costs in other cities Secondly for wealthier retirees housing costs arenot likely to be a factor in choosing a place to retire Thus the mere fact that housing costshave been in decline lends scant support to the recommendationThe articles reliance on Clearviews property-tax rates is also problematic in two respectsFirst retirees obviously have innumerable choices about where to retire besides Clear viewand nearby towns Secondly for retirees who are well-off financially property taxes are notlikely to be an important concern in choosing a place to retire Thus it is unfair to infer fromClearviews property-tax rates that retirees would prefer ClearviewYet another problem with the argument involves the mayors promises In light of Clearviewslow property-tax rates whether the mayor can follow through on those promises is highlyquestionable Absent any explanation of how the city can spend more money in the areas citedwithout raising property taxes I simply cannot accept the editorials recommendation on thebasis of those promises Besides even if the city makes the improvements promised thoseimprovements--particular the ones to schools--would not necessarily be important to retireesFinally although the number of physicians in Clearview is relatively high the per capitanumber might be relatively low Moreover it would be fairer to compare this per capita numberwith the per capita number for other attractive retirement towns--rather than the nationalaverage After all retirees are likely to place a relatively heavy burden on health-careresources Besides the article provides no assurances that the number of physicians inClearview will remain high in the foreseeable futureIn conclusion the recommendation is poorly supported To strengthen it the author mustconvince me--perhaps by way of a reliable survey--that the key features that the vast majorityof retirees look for in choosing a place to live are consistent climate natural beauty and lowhousing costs The author must also provide better evidence that Clear views property taxesare lower than the those of cities in other areas The author must also explain how the city canmake its promised improvements without raising property taxes Finally to better assess theargument I would need to now how the per capita number of physicians in Clearview wouldcompare to the national average in the futureThe following appeared as a letter to the editor from a Central Plaza store owner

Over the past two years the number of shoppers in Central Plaza has been steadily decreasing while the popularity of skateboarding has increased dramatically Many Central Plaza store owners believe that the decrease in their business is due to the number of skateboard users in the plaza There has also been a dramatic increase in the amount of litter and vandalism throughout the plaza Thus we recommend that the city prohibit skateboarding in Central Plaza If skateboarding is prohibited here we predict that business in Central Plaza will return to its previously high levels

Write a response in which you discuss what questions would need to be answered in order to decide whether the recommendation is likely to have the predicted result Be sure to explain how the answers to these questions would help to evaluate the recommendation

Why two years ago What happened two years ago which started this declineIs the dramatic increase in the ldquopopularityrdquo of skateboarding the cause of the steady decline of shoppers Are there any malls nearby Were there any changes nearby which could affect the decline in customersmdasha big mall perhaps Could the decline be due to the shop ownersHow many skateboarders use the plazaWhere do they skateboardDo they shop and are they customersAre the increase in litter and vandalism due to skateboarders Could this be alleviated by installing CCTVs and hiring security

This editorial concludes that the city should ban skateboarding from its downtown CentralPlaza in order to attract visitors to that area to return the area to its former glory and to makeit a place where people can congregate for fun and relaxation To justify this conclusion theeditorial points out that skateboarders are nearly the only people one sees anymore at CentralPlaza and that the Plaza is littered and its property defaced The editorial also points out thatthe majority of downtown merchants support the skate boarding ban This argument is flawedin several critical respectsFirst the editorials author falsely assumes that a ban on skateboarding is both necessaryand sufficient to achieve the three stated objectives Perhaps the city can achieve thoseobjectives by other means as well--for example by creating a new mall that incorporates anattractive new skateboard park Even if banning skateboarders altogether is necessary to meetthe citys goals the author has not shown that this action by itself would suffice Assuming thatthe Plazas reputation is now tarnished restoring that reputation and in turn enticing peopleback to the Plaza might require additional measures--such as removing litter and graffiti

GRE AWA John박 박정어학원

promoting the Plaza to the public or enticing popular restaurant or retail chains to the PlazaSecondly the editorial assumes too hastily that the Plazas decline is attributable to theskateboarders--rather than to some other phenomenon Perhaps the Plazas primary appeal inits glory days had to do with particular shops or eateries which were eventually replaced byless appealing ones Or perhaps the crime rate in surrounding areas has risen dramatically forreasons unrelated to the skateboarders presence at the Plaza Without ruling out these andother alternative explanations for the Plazas decline the editorials author cannot convince methat a skateboard ban would reverse that declineThirdly the editorials author might be confusing cause with effect--by assuming that theskateboarders caused the abandonment of the Plaza rather than vice versa It is entirelypossible that skateboarders did not frequent the Plaza until it was largely abandoned--andbecause it had been abandoned In fact this scenario makes good sense since skateboardingis most enjoyable where there are few pedestrians or motorists to get in the wayFourth it is unreasonable to infer from the mere fact that most merchants favor the ban thatthe ban would be effective in achieving the citys objectives Admittedly perhaps thesemerchants would be more likely to help dean up the Plaza area and promote their businesseswere the city to act in accordance with their preference Yet lacking any supporting evidencethe author cannot convince me of this Thus the survey amounts to scant evidence at best thatthe proposed ban would carry the intended resultFinally the author recommends a course of action that might actually defeat the citysobjective of providing a fun and relaxing place for people to congregate In my experienceskateboarding contributes to an atmosphere of fun and relaxation for adults and children alikemore so than many other types of ambiance Without considering that continuing to allowskateboarding--or even encouraging this activity--might achieve the citys goal more effectivelythan banning the activity the author cannot convincingly conclude that the ban would be in thecitys best interestsIn sum the argument is a specious one To strengthen it the editorials author must providedear evidence that skateboarding and not some other factor is responsible for the conditionsmarking the Plazas decline The author must also convince me that no alternative means ofrestoring the Plaza are available to the city and that the proposed ban by itself would suffice toattract tourists and restore the Plaza to its former glory Finally to better assess the argument itwould be useful to know the circumstances under which the downtown merchants would bewilling to help the city achieve its objectives

6그룹 약한 비유 빈출

The following recommendation appeared in a memo from the mayor of the town of Hopewell

Two years ago the nearby town of Ocean View built a new municipal golf course and resort hotel During the past two years tourism in Ocean View has increased new businesses have opened there and Ocean Views tax revenues have risen by 30 percent Therefore the best way to improve Hopewells economymdashand generate additional tax revenuesmdashis to build a golf course and resort hotel similar to those in Ocean View

Write a response in which you examine the stated andor unstated assumptions of the argument Be sure to explain how the argument depends on these assumptions and what the implications are for the argument if the assumptions prove unwarranted

GRE AWA John박 박정어학원

Assumptions The author assumes that OVrsquos municipal golf course and resort hotel caused tourism new businesses and increased tax revenues There may be other reasons advertising promo He assumes that this will continueAssumes that Ocean View and Hopewell are similar in many waysmdashthe name suggests otherwise OV may have always been a tourist attractions for its beaches We need to know the topography

2년전 Ocean View 시는 시정 소유 골프 및 휴양지 호텔을 신축했다 그리고 지난 2년동안 이 시의 관광객이 증가했으며 새로운 사업들이 생겨났다 그에따라 시의 세수도 30나 증가했다 Hopewell의 경제를 향상시키고 아울러 세수를 늘릴 수 있는 가장 좋은 방법은 Ocean View에 세워진 것과 같은 골프 시설과 휴양지 호텔을 신축하는 것이다

1 다른 요인으로 관광 산업이 발전했을 수도 있다 문화 유적이 발견이 되었거나 도로의 정비등으로 여행자가 늘었을 수도 있다

2 관광 산업의증가가 늘어난 세수의 원인이 아니라 새로 유입된 인구의 증가나 다른 공장에서 발생한 것일 수 있다

3 2년동안 한참 골프가 붐을 이루었을 수 있다 경제상황이 나빠지거나 다른 레포츠가 인근 지역에 생겨난다면 골프하는 사람이 줄어들 수 있다

In this memo HopeweUs mayor recommends that in order to stimulate the towns economyand boost tax revenues HopeweU should build a new golf course and resort hotel just as thetown of Ocean View did two years ago To support this recommendation the mayor points outthat in Ocean View during the last two years tourism has increased new businesses haveopened and tax revenues have increased by 30 I find the mayors argument unconvincingin several important respectsFirst of all it is possible that the mayor has confused cause with effect respecting the recentdevelopments in Ocean View Perhaps Ocean Views construction of a new golf course andhotel was a response to previous increases in tourism and business development increasesthat have simply continued during the most recent two years Since the mayor has failed toaccount for this possibility the claim that Hopewell would boost its economy by alsoconstructing a golf course and hotel is completely unwarrantedSecondly the mayor fails to account for other possible causes of the trends in Ocean Viewduring the last two years The increase in tourism might have been due to improving economicconditions nationwide or to unusually pleasant weather in the region The new businessesthat have opened in Ocean View might have opened there irrespective of the new golf courseand hotel And the 30 increase in tax revenues might have been the result of an increase intax rates or the addition of a new type of municipal taxWithout ruling out these and other alternative explanations for the three recent trends inOcean View the mayor cannot reasonably infer based on those trends that Hopewellseconomy would benefit by following Ocean Views exampleThirdly even if the recent trends in Ocean View are attributable to the construction of the newgolf course and hotel there the mayor assumes too hastily that the golf course and hotel willcontinue to benefit that towns overall economy The mayor has not accounted for thepossibility that increased tourism will begin to drive residents away during tourist season orthat new business development will result in the towns losing its appeal as a place to visit or tolive Unless the mayor can convince me that these scenarios are unlikely I cannot accept themayors recommendation that Hopewell follow Ocean Views exampleFinally the mayors argument rests on the unsubstantiated assumption that Hopewell andOcean View are sufficiently alike in ways that might affect the economic impact of a new golfcourse and hotel Hopewell might lack the sort of natural environment that would attract moretourists and new businesses to the town--regardless of its new golf course and hotel For thatmatter perhaps Hopewell already contains several resort hotels and golf courses that are notutilized to their capacity If so building yet another golf course and hotel might amount to amisallocation of the towns resources--and actually harm the towns overall economyIn sum the mayors recommendation is not well supported To bolster it the mayor mustprovide better evidence that Ocean Views new golf course and hotel and not some otherphenomenon--has been responsible for boosting Ocean Views economy during the last twoyears To better assess the recommendation I would need to know why Ocean View decidedto construct its new golf course and hotel in the first place--specifically what events prior toconstruction might have prompted that decision I would also need to thoroughly compare

GRE AWA John박 박정어학원

HopeweU with Ocean View--especially in terms of their appeal to tourists and businesses--todetermine whether the same course of action that appears to have boosted Ocean Viewseconomy would also boost Hopewells economy

The following is part of a memorandum from the president of Humana University

Last year the number of students who enrolled in online degree programs offered by nearby Omni University increased by 50 percent During the same year Omni showed a significant decrease from prior years in expenditures for dormitory and classroom space most likely because instruction in the online programs takes place via the Internet In contrast over the past three years enrollment at Humana University has failed to grow and the cost of maintaining buildings has increased along with our budget deficit To address these problems Humana University will begin immediately to create and actively promote online degree programs like those at Omni We predict that instituting these online degree programs will help Humana both increase its total enrollment and solve its budget problems

Write a response in which you discuss what questions would need to be answered in order to decide whether the prediction and the argument on which it is based are reasonable Be sure to explain how the answers to these questions would help to evaluate the prediction

Is Omni University successful due to the online degree program 50 Is the decrease in expenditures for dormitory and classroom space due to the decrease in of on-campus students Which classes were successful Does HU have those classes

Even if the long-distance degree programs at Omni University benefited the school the presidentrsquos recommendation that Human College should emulate Omni University is too hasty First OUrsquos name implies that the school would have more majors than Humanahellip the president should examine which degrees were in the long-distance programhellip

지난해에는 Omni 대학에서 개강했던 원거리 학생 학점 취득 프로그램을 등록했던 학생들의 숫자가 50나 증가했다 같은해 기간동안 Omni 대학에서는 그 전년도부터 기숙사와 학급의 공간 확충을 위한 예산을 대폭 줄였는데 이는 이 원거리 학점 취득 프로그램이 양방향 비디오 컴퓨터 접속을 통해서만 가능한 수업지도 방식이기때문인 것으로 보인다 반면 지난 3개년 동안 Humana 대학에서의 수강률은 감소한데다가 건물

GRE AWA John박 박정어학원

유지비도 올랐다 따라서 Humana대학의 수강을 늘리고 예산손실을 회복하기 위해서는 Omni 대학에서 취한 조치와 같은 능동적인 프로그램을 추진해야 한다

결론 we should initiate and actively promote long-distance degree programs like those at Omni 반박 원거리 학생 취득 프로그램 숫자가 증가한거하고 예산이 줄어드는 것 사이에 연관이 약하다 (causal 학생의 증가로 관리비용 증가할수 있음 원거리 수업가능 장비도입에의한 비용발생)bad analogy(omni university 하고 같은 조건이 아니다 )-gt omni college 가 강좌내용이 좋아서 학생의 등록이 많을수 있다 Humana 대학에서 만들었다 하더라도 인기 없을수 있음다른 요인에 의해서 Humana 대학의 수강 인원이 증가할수 있음(비록 과거엔 인기가 없었을지라도)

The following appeared as part of a business plan developed by the manager of the Rialto Movie Theater

Despite its downtown location the Rialto Movie Theater a local institution for five decades must make big changes or close its doors forever It should follow the example of the new Apex Theater in the mall outside of town When the Apex opened last year it featured a video arcade plush carpeting and seats and a state-of-the-art sound system Furthermore in a recent survey over 85 percent of respondents reported that the high price of newly released movies prevents them from going to the movies more than five times per year Thus if the Rialto intends to hold on to its share of a decreasing pool of moviegoers it must offer the same features as Apex

Write a response in which you discuss what questions would need to be answered in order to decide whether the recommendation is likely to have the predicted result Be sure to explain how the answers to these questions would help to evaluate the recommendation

Before following through this business plan the manager should investigate the cause of Rialtorsquos unsuccessful business

The author provides no evidence that the surveyrsquos results are statistically reliable The surveyrsquos sample of 85 percent must be sufficient in size and representative of overall population of the city where Rialto and Apex is serving Lacking evidence of a sufficiently representative sample the author cannot justifiably rely on the survey to draw any conclusion whatsoever The author does not indicate that Apex is indeed currently successful However even if Apex is enjoying success the argument relies on what might be a false analogy between Rialto and Apex In order for Apex to serve as a model that Rialto should emulate the author must assume that all relevant circumstances are essentially the same However this assumption is unwarranted For example the argument overlooks the face that Apex is located in a strategic placemdashbeside a mall where customers can not only watch a movie but also enjoy shopping Therefore simply changing the facility to that of Apex may not lead to success

The author does not mention whether Apex is successful or not Nevertheless even if Apex is currently successful the argument relies on what might be a false analogy between Rialto and Apex In order for Apex to serve as a model that Rialto should emulate the author must assume that all relevant circumstances are essentially the same However this assumption is unwarranted For example the argument overlooks the fact that these two institutions are located in different locations Rialto in downtown and Apex in a mall outside of town Although Apex opened with state-of-the-art facilities the decisive factor in its success could be due to its strategic location of being in a mall People could enjoy both shopping and movies at one location thus they may prefer Apex over Rialto Furthermore the place where people enjoy leisure activities has shifted in the past decades for most cities from downtown to the suburbs Therefore Rialto may not be successful even if it emulates Apexrsquos facilities A better business plan may be relocating Apex to the thriving section of the downtown

Rialto 극장은 지난 50여년간 지역 회관으로써 시내에 위치해 있으면서도 이제 변화를 꾀하지 않으면 문을 닫을

GRE AWA John박 박정어학원

판이다 이 극장은 시외 쇼핑타운에 새로 들어선 Apex 극장의 사례를 본받아야 했다 Apex가 지난해 개업했을 당시 이 극장은 비디오 아케이드 플러쉬 카펫트 바닥과 좌석 그리고 최신 음향시설을 갖추었다 더군다나 최근 조사에서는 응답자의 85 이상이 새로 출시된 영화 입장료가 비싼 탓으로 지난해보다 5배이상의 관람객이 줄어들었다고 나타났다 따라서 Rialto 극장이 줄어들고 있는 관람객을 뺐기지 않고 유지하려면 Apex와 같은 시설들을 갖추어야 할 것이다주장 리알토 극장이 줄어들고 있는 관람객을 뺐기지 않고 유지하려면 Apex와 같은 시설들을 갖추어야 할 것이다

1 조사에서 응답자가 전체를 대표할 수 없다 2 apex 극장이 좋은 시설을 갖추고 있지만 그로 인해 수익이 많이 발생했다는 말이 없으므로 시설투자를

하고도 좋은 결과를 얻을 수 있을지 그 근거가 미흡하다3 좋은 영화가 출시된다면 입장료가 비싸도 영화관에서 꼭 보려고 할 수 있다 4 rialto 가 시설이 아닌 다른 요인에 의해 장사가 안될수도 있다( 우범 지역이라든지)

The following is a recommendation from the business manager of Monarch Books

Since its opening in Collegeville twenty years ago Monarch Books has developed a large customer base due to its reader-friendly atmosphere and wide selection of books on all subjects Last month Book and Bean a combination bookstore and coffee shop announced its intention to open a Collegeville store Monarch Books should open its own in-store cafeacute in the space currently devoted to childrens books Given recent national census data indicating a significant decline in the percentage of the population under age ten sales of childrens books are likely to decline By replacing its childrens books section with a cafeacute Monarch Books can increase profits and ward off competition from Book and Bean

Write a response in which you examine the stated andor unstated assumptions of the argument Be sure to explain how the argument depends on these assumptions and what the implications are for the argument if the assumptions prove unwarranted

The following is a recommendation from the business manager of Monarch Books

Since its opening in Collegeville twenty years ago Monarch Books has developed a large customer base due to its reader-friendly atmosphere and wide selection of books on all subjects Last month Book and Bean a combination bookstore and coffee shop announced its intention to open a Collegeville store Monarch Books should open its own in-store cafeacute in the space currently devoted to childrens books Given recent national census data indicating a significant decline in the percentage of the population under age ten sales of childrens books are likely to decline By replacing its childrens books section with a cafeacute Monarch Books can increase profits and ward off competition from Book and Bean

1 Write a response in which you discuss what questions would need to be answered in order to decide whether the recommendation is likely to have the predicted result Be sure to explain how the answers to these questions would help to evaluate the recommendation

2 Write a response in which you discuss what specific evidence is needed to evaluate the argument and explain how the evidence would weaken or strengthen the argument

No evidence regarding Monarch Bookrsquos successEven if Regal Bookrsquos is successful this may not be attributable to the cafeacute False analogy Emulating may not lead to success Other factors may be involvedInsufficient condition The national census is not enough evidence that childrenrsquos book sales will decline Can

GRE AWA John박 박정어학원

the national census represent the local child populationDid opening a cafeacute boost sales for Regal Books Even assuming Regal is successful by opening a cafeacute this may not be suitable for Monarch which plans to close the childrenrsquos book section to establish a cafe Imprecise language ldquorelatively little spacerdquo how smallThe managerrsquos recommendation contradicts what he says Since Monarch is popular for its wide selection of books closing a selection which targets a major group of readers may hurt Monarchrsquos salesIs this the best way to compete

When Stanley Park first opened it was the largest most heavily used public park in town It is still the largest park but it is no longer heavily used Video cameras mounted in the parks parking lots last month revealed the parks drop in popularity the recordings showed an average of only 50 cars per day In contrast tiny Carlton Park in the heart of the business district is visited by more than 150 people on a typical weekday An obvious difference is that Carlton Park unlike Stanley Park provides ample seating Thus if Stanley Park is ever to be as popular with our citizens as Carlton Park the town will obviously need to provide more benches thereby converting some of the unused open areas into spaces suitable for socializing

Write a response in which you examine the stated andor unstated assumptions of the argument Be sure to explain how the argument depends on these assumptions and what the implications are for the argument if the assumptions prove unwarranted

Stanley 파크가 처음 개장했을 당시 가장 크고 가장 많이 이용되는 공원이었다 아직도 공원중에서는 가장 크지만 이용률은 상당히 떨어졌다 지난달 공원 주차장에 설치해놓은 비디오 카메라를 통해 보면 drop(주차장으로 여겨짐) 이용률이 가장 높았다 수치상으로는 하루 평균 50대의 차량만이 이용하였다 반면 직장 중심거리에 위치한 작은 규모의 Carlton 파크는 주당 무려 150여명 이상이 이용하고 있다 Stanley 파크와는 달리 Carlton 파크에는 의자가 있다는 것이 가장 뚜렷한 차이점이다 따라서 Stanley 파크가 Carlton 파크처럼 시민들이 자주 이용하는 공원이 되기 위해서는 벤치를 설치할 필요가 있으며 이렇게 사용되지 않는 일부 공간을 활용해서 사교를 위한 공간으로 바꾸어야 한다 ===gtdrop 에 대한 첨부사항 (영영사전내용입니다)---- a place or central depository to which something (as mail money or stolen property) is brought for distribution or transmission also the act of depositing something at such a place dropgt

주장 if Stanley Park is ever to be as popular with our citizens as is Carlton Park the town will obviously need to provide more benches thereby converting some of the unused open areas into spaces suitable for socializing1 조사가 언제 이루어진 것인가 조사가 언제 실시되었느냐에 따라 결과가 다를 수있다 현재는 다시 스탠리 파크가 늘어났었을 수 있다 2 벤치를 많이 설치했다고 해서 많은 관광객이 오지 않을수 있다(사람들이 벤치나 사교 공간을 원한다는 어떠한 자료도 없다)3스탠리 파크 주변에 교통 상황이 악화가 되었거나 칼튼 파크에서 문화행사등을 많이 가져서 이용객이 줄어든것일 수도 있다 4 칼튼 파크가 중심지에 있어서 접근성이 좋을수 있다5 조사가 같은 시간을 기준으로 한게 아니다(하나는 주중이고 하나는 주말이다)6사람의 수와 차의 대수를 같은것으로 비교할수 없다 (차안에 몇 명이 타고 있는지 모르고 대중교통을 이용해서 왔을수도 있다)

Page 5: GRE writing argument brain storm

GRE AWA John박 박정어학원

A recently issued twenty-year study on headaches suffered by the residents of Mentia investigated the possible therapeutic effect of consuming salicylates Salicylates are members of the same chemical family as aspirin a medicine used to treat headaches Although many foods are naturally rich in salicylates food-processing companies also add salicylates to foods as preservatives The twenty-year study found a correlation between the rise in the commercial use of salicylates and a steady decline in the average number of headaches reported by study participants At the time when the study concluded food-processing companies had just discovered that salicylates can also be used as flavor additives for foods and as a result many companies plan to do so Based on these study results some health experts predict that residents of Mentia will suffer even fewer headaches in the future

Write a response in which you discuss what questions would need to be answered in order to decide whether the prediction and the argument on which it is based are reasonable Be sure to explain how the answers to these questions would help to evaluate the prediction

In this study the author asserts that the number of headaches suffered by the average citizen of Mentia will steadily decline though the use of salicylates as flavor additives To support this assertion the author provides several evidences to support his claim However his argument is specious

Unscientific data correlation the specific amount used as preservatives Other factors could have caused the decline in headaches20 year study Was it a controlled study Lack of controlled environment exposes the participants to other factors

1) The decline in the average number of headaches does not necessarily substantiate that it was indeed caused by the commercial use of salicylates It is entirely possible that other factors are responsible for the decline Perhaps other substances in the food remedied the headaches of the participants Or the decline in headache may have been caused by other dietary consumption or lifestyle habits like exercising Accordingly if these are true the author cannot draw any firm conclusion that the commercial use of salicylates caused the decline in the average number of headaches

살리실산염은 아스피린과 같은 동일 화학물질 계열로써 두통을 치료하는데 사용되는 약물이다 많은 음식에 이 살리실산염이 다량 함유되어 있지만 지난 수십년간 식품가공 업체들은 가공식품 부패방지용으로 살리실산염을 첨가해왔다 이러한 살리실산염의 산업용 사용의 증가는 본 연구원들에 의해 보고된 평균 두통 횟수를 꾸준히 감소시키는데 일조한것으로 이해되어 왔다 최근 식품가공 회사들은 이 살리실산염이 맛을 내는 첨가물로 사용될 수 있음을 발견했다 이러한 새로운 용도로 Mentia에 사는 주민들이 겪고 있는 두통의 평균 횟수를 앞으로도 계속 줄여나갈 수 있을 것으로 보인다결론 살리산염의 사용증가는 주민들의 두통 횟수를 계속 줄여줄 것이다

가정 1 조사받은 사람들은 단지 Salicylate에 의해 headache가 감소된 것이다 -gt 오류 다른 노력을 기울였을 수 있다 가정 2 나중에 Salicylate의 사용증가가 headache의 감소를 가져올 것이다 -gt 오류 또다른 원인으로 얼마든지 headache는 증가가능하다가정 3 음식물에서 headache 감소 원인은 Salicylate 이다 -gt 오류 또다른 성분이 있었을 수 있다가정 4 Flavor additive 로 사용된 Salicylate도 headache를 줄일것이다 -gt 오류 Flavor additive로 사용시 같은 효과를 가질지 밝혀진바 없다 상업적으로 사용되었다고 다 먹었을까

The following appeared as part of an article in a business magazine

GRE AWA John박 박정어학원

A recent study rating 300 male and female Mentian advertising executives according to the average number of hours they sleep per night showed an association between the amount of sleep the executives need and the success of their firms Of the advertising firms studied those whose executives reported needing no more than 6 hours of sleep per night had higher profit margins and faster growth These results suggest that if a business wants to prosper it should hire only people who need less than 6 hours of sleep per night

Write a response in which you examine the stated andor unstated assumptions of the argument Be sure to explain how the argument depends on these assumptions and what the implications are for the argument if the assumptions prove unwarranted

Assumptions 1 Correlation of hours of sleep with success The author assumes that the executives used their wake-up hours on work Could there be other factors 2 Executives versus employeesmdashcriteria for hiring is too simpleminded 3 Characteristics of the 300 male and female executives 4 The study was about advertising executives but the author applies this case to all businesses 5 The author should consider long-term stability rather that sharp growth and profit 6 Average number of hours of sleep is insufficient to support the authorrsquos recommendation 7 The author assumes that this lifestyle pattern will remain consistent

The following appeared in a memo from the president of a company that makes breakfast cereals

In a recent study subjects who ate soybeans at least five times per week had significantly lower cholesterol levels than subjects who ate no soy products By fortifying our Wheat-O cereal with soy protein we can increase sales by appealing to additional consumers who are concerned about their health This new version of Wheat-O should increase company profits and at the same time improve the health of our customers

Write a response in which you examine the stated andor unstated assumptions of the argument Be sure to explain how the argument depends on these assumptions and what the implications are for the argument if the assumptions prove unwarranted

최근의 조사에서 일주에 최소 5회 정도 콩을 먹었던 사람들은 전혀 먹지 않았던 사람들에 비해 콜레스테롤이 상당히 낮은것으로 조사되었다 따라서 Wheat-O 시리얼에 콩 단백질의 함유를 강화시킴으로써 건강에 관심이 있는 더 많은 소비자를 상대로 매출을 올릴 수 있을 것이다 이 신제품이 회사 수입 증대를 가져오는 것 뿐만아니라 소비자의 건강을 향상시킬 수 있다

GRE AWA John박 박정어학원

주장 By increasing our Wheat-O cereal with soy protein we can multiply sales

1 콩 단백질의 함유를 강화해도 그수치가 미흡하거나 흡수가 잘 안되거나 할 수 있다 다른 성분과 섞여서 만들어지는 것이므로 다른 성분에 의해서 그 흡수가 잘 안되거나 효과가 낮을 수 있다 2 강화한 것이 건강에 관심이 있는 소비자에게 어필한다고 해도 그 수가 적어서 매출에 큰 영향을 안줄 수 있다3 다른 음식에 의해서 콜레스테롤이 더 낮아질수 있다 4 콜레스테롤을 낮추는 것이 사람들에게 그들의 건강생활을 위해 크게 어필 안할수도 있다( 저지방식이라든지 다른 건강을 높이는 방법이 더 관심이 많을수 있다)

In a study of the reading habits of Waymarsh citizens conducted by the University of Waymarsh most respondents said that they preferred literary classics as reading material However a second study conducted by the same researchers found that the type of book most frequently checked out of each of the public libraries in Waymarsh was the mystery novel Therefore it can be concluded that the respondents in the first study had misrepresented their reading habits

Write a response in which you discuss what specific evidence is needed to evaluate the argument and explain how the evidence would weaken or strengthen the argument

Evidence needed -Information about the respondentsmdashage gender class precise number of respondents-Time-shift After how many years was the second study conductedmdashtaste in books may change over time-Can library records sufficiently evidence the reading habits of Waymarsh citizens

1)2)3)4)5)6) First the author provides no evidence that the surveyrsquos studyrsquos results are statistically reliable Lacking

information about the precise methodology of the study the number of (customers)respondents surveyed and the number of respondentsvarious information such as gender age and social background which are essential to bolster the conclusion it is impossible to assess the validity of the results It is possible that people who feel inclined to( take low fat and low cholesterol foods ) read literary classics were more willing to respond to the survey than were others Another problem is the representativeness of the respondents Were they representative of all the customersWaymarsh citizens Were they chosen for the survey randomly Lastly the survey results must depend on the honesty and integrity of the respondents Without more information about the survey the author cannot simply conclude that (most of customers want to take low fat and low cholesterol foods)Waymarsh citizens misrepresented their reading habit on the first study on the basis of this surveystudy(survey reliability)

The exact number of books checked-out should be notedMystery novels could have been in trend at the time of the studyPublic libraries vs other libraries such as Leeville University library

Leeville 대학에서 실시한 Leeville 주민의 독서습관에 대한 조사에서 대부분의 응답자는 독서용으로 문학작품을 선호한다고 응답했다 그러나 같은 조사팀에서 이어 실시한 조사에서는 Leeville에 있는 도서관마다 가장 빈번하게 연람된 책의 종류를 보면 미스테리 소설이었던 것으로 조사되었다 따라서 최초 조사 응답자들이 독서 습관에 대해 잘못 말했다고 결론지을 수 있다

결론 it can be concluded that the respondents in the first study had misrepresented their reading habits1 첫번째 조사 그룹하고 두번째 조사 그룹하고 다른 그룹일 수 있다

GRE AWA John박 박정어학원

2 도서관에 비치된 책이 미스터리 소설이 더 많아서 선택의 여지가 없을 수 있다

High Frequency Group 2 National-gtLocal Local-gtNational amp Group Member Error

The following appeared in a letter to the editor of a Batavia newspaper

The department of agriculture in Batavia reports that the number of dairy farms throughout the country is now 25 percent greater than it was 10 years ago During this same time period however the price of milk at the local Excello Food Market has increased from $150 to over $300 per gallon To prevent farmers from continuing to receive excessive profits on an apparently increased supply of milk the Batavia government should begin to regulate retail milk prices Such regulation is necessary to ensure fair prices for consumers

Write a response in which you discuss what questions would need to be answered in order to decide whether the recommendation is likely to have the predicted result Be sure to explain how the answers to these questions would help to evaluate the recommendation

First the author points to the fact that hellip25 increase of dairy farms may be(could be might be) due to population increaseThe milk price increase could be reflecting the increase in cost of livingThe actual price of milk might be cheaper if the economy is experiencing inflationOne market Excello Food cannot reflect the price increase of every market in BataviaThe author claims that farmers are receiving excessive profits however this might not be the caseThe author claims that by regulating milk prices the government will successfully ensure both lower prices and an adequate supply of milk for consumers However this might not be the best solutionThe writer adduces the information released by the department of agriculture that the number of dairy farms has

increased by 25 percent over the last 10 years and thus he claims that dairy farmers are earning excessive profits from milk However the information is filled with loops and holes which need to be filled First what percentage of the milk are sold as milk

Are the dairy farms processing the milk into other dairy products Or are they selling them as milkmdashWhat percentage of the milk is sold as milk Could this be a natural increase

Can the price at the local Excello Food Market represent the price of milk in stores throughout all of Batavia

Are the farmers receiving excessive profits What is the cost of producing milk Could such regulation offer fair prices to consumers

Topic SentenceTo begin with the author assumes that the price of Excello Food Market which has increased from $150 to over $300 per gallon can be applied to the whole nation of Batavia however the author fails to provide evidence to substantiate this assumptionExampleFirstIn additionTherefore in order to make the argument more reliable the author should consider the prices of a large number markets all over the countryThere could be an economic inflation

GRE AWA John박 박정어학원

ldquoBatavia 지역의 농림부의 보고에 따르면 전국적으로 낙농업자의 숫자가 과거 10 년전에 비해 25나 증가했다고 합니다 그러나 동기간 동안 이 지역에 있는 Excello Food Market 에서 우유의 가격은 갤런당 15 불에서 3 불로 증가했습니다 증가된 우유 공급량에 맞춘 낙농업자들의 수익률 증대를 보장하기 위해서는 주 정부가 소매 공급가에 대한 규제를 해야 합니다 소비자에게 보다 저렴한 가격에 안정된 물량을 공급할 수 있도록 하기 위해서는 이러한 조치가 필요합니다

결론 the Batavia government should begin to regulate retail milk prices

반박 ( Excello Food Market doesnrsquot reflect the whole of Batavia) 우유값 증가가 단순히 inflation 을 따라간것일수도 있다 가격상승이 수익을 보장하지 않는다( 원자재 상승등으로 인해서)우유값 규제만이 최선의 방법이 아니다( 물량 조절)낙농업자의 수익보호도 필요하다

A recent sales study indicates that consumption of seafood dishes in Bay City restaurants has increased by 30 percent during the past five years Yet there are no currently operating city restaurants whose specialty is seafood Moreover the majority of families in Bay City are two-income families and a nationwide study has shown that such families eat significantly fewer home-cooked meals than they did a decade ago but at the same time express more concern about healthful eating Therefore the new Captain Seafood restaurant that specializes in seafood should be quite popular and profitable

Write a response in which you discuss what questions would need to be addressed in order to decide whether the conclusion and the argument on which it is based are reasonable Be sure to explain how the answers to the questions would help to evaluate the conclusion

BAY CITY The name implies that this is a port city which would have traditionally consumed seafood 30--what type of seafood Unprocessed or processed cans of tuna and sardines If the latter increased Captain Seafood may not profit 30--natural inflation Are the non-seafood restaurants selling seafood If yes this may suffice since patrons would not easily dine at an unfamiliar restaurant Nationwide studyrsquos representativeness Desire does not lead to action

Nationwide study applied to Bay City Eating fewer home cooked meals than they did a decade agoHealthy food The author needs to show that 30 -gt population increaseOther foods may be more popularCurrent number of restaurants that serve seafood might sufficeWhat kind of healthy food are they interested in Surely not just seafoodThe locals may still patronize the current restaurants that serve seafood dishes

lt national-gt local에 적용 local-gtnational에 적용gt1) The author assumes that the nationwide trend reflects the general trend upon which the argument relies

Yet the author fails to provide evidence to substantiate this crucial assumption The nationwide trend of two income families eating fewer home-cooked and concern for health my not reflect the lifestyle and interests of Bay City citizens Therefore to make his argument stronger the author needs to provide evidence that two income families of Bay City maintains the nationwide trend

GRE AWA John박 박정어학원

최근의 매상에 관한 조사에서 Bay 시에 있는 레스토랑의 해산물 요리의 소비가 지난 5년동안 30 증가했다고 보고되었다 그러나 현재 해산물을 전문으로 취급하고 있는 레스토랑이 없다 더군다나 대다수의 가정이 맞벌이 가정인데다가 전국 조사에서도 나와있듯이 이러한 맞벌이 가정은 10년전의 가정에서 했듯이 집에서 식사를 챙겨먹는 것이 급격하게 줄어 들었고 동시에 건강식과 관련한 지출이 늘고 있다 따라서 해산물 전문 레스토랑이 아주 인기를 끌것이고 그만큼 수익도 많을 것이다

1 해산물이 건강식인지에 대한 언급이 없다2 외식하는데 해산물 요리만 먹지 않을 것이다 집에서 챙겨먹지 않는다고 해산물을 먹는다고 할 수 없다3 현재 해산물 취급하는 식당이 현재까지 없다고 해서 계속 없을 수만은 없다 수익이 만약 늘어난다면

Bay시 주변의 도시의 레스토랑이 체인점을 내거나 새로운 식당이 생길 수 있고 오히려 수익이 줄어들 수도 있다

Scientists studying historical weather patterns have discovered that in the mid-sixth century Earth suddenly became significantly cooler Although few historical records survive from that time some accounts found both in Asia and Europe mention a dimming of the sun and extremely cold temperatures Either a huge volcanic eruption or a large meteorite colliding with Earth could have created a large dust cloud throughout Earths atmosphere that would have been capable of blocking enough sunlight to lower global temperatures significantly A large meteorite collision however would probably create a sudden bright flash of light and no extant historical records of the time mention such a flash Some surviving Asian historical records of the time however mention a loud boom that would be consistent with a volcanic eruption Therefore the cooling was probably caused by a volcanic eruption

Write a response in which you discuss what questions would need to be addressed in order to decide whether the conclusion and the argument on which it is based are reasonable Be sure to explain how the answers to the questions would help to evaluate the conclusion

Historical records may not be enough to explain the global cooling phenomenonThere may be reasons other than the two hypothesis to explain the global coolingA volcanic eruption big enough to produce dust clouds that envelop the earth to cause global cooling would have countless recordsThe author argues that a large meteorite collision is not a feasible explanation for the global cooling because there are no historical records that indicate a flash The authorrsquos logic is flawed in eliminating the meteorite hypothesis by claiming that there was no historical record of a bright flash of light According to common sense a meteorite collision big enough to produce dust that could envelop the earth would result in catastrophe even to the extinction of major species of animals

Loud boom would be insufficient evidence to conclude it was a volcanic eruption If could hear a loud boom there should be records of a volcanic eruption because the author claims that the dust cloud from this gigantic eruption was significant enough to cause global coolingCooling could have been caused by factors besides volcanic eruption and meteor collisionThe absence of historical records that indicate a bright flash of light Collision could have occurred at a place where there no witnesses Could have fell out in the seaBoom might have been caused by things other than a volcanic eruptionHistorical records from Asia and Europe may not be applicable to earth as a whole

과거의 날씨 주기를 연구하는 과학자들은 6세기 중엽 지구가 갑자기 혹한기가 되었던 것을 알게되었다 이 당시의 몇몇 기록들이 아직도 남아있긴 하지만 아시아와 유럽에서 발견되는 몇가지 자료에서 태양 빛의 수축과 그에따른 엄청난 추위가 있었음을 언급하고 있다 거대한 화산 폭발이나 거대 운석의 지구 충돌중 어느것에

GRE AWA John박 박정어학원

의해서든 엄청난 먼지구름을 형성시켜 지구전체에 태양빛을 차단해서 지구의 기온을 뚝 떨어뜨릴수 있을 수도 있다 그러나 이러한 운석 충돌은 순간 섬광을 발산시키게 될 것인데 이 시기의 어느 기록에서도 이러한 섬광은 언급되지 않고 있다 그러나 이 시기에 남아 있는 일부 아시아의 기록문서에서는 연속적으로 화산폭발음일수 있는 엄청난 폭발음이 있었음을 기록하고 있다 따라서 이러한 혹한은 아마도 화산 폭발에 의한 것이었을 것이다

결론 the cooling was probably caused by a volcanic eruption

반박 자료의 부족( 몇몇 자료가지고 그 당시의 기후를 예상하기에는 무리가 있다)다른 원인에 의해서 기후가 떨어졌을수도 있다 (빙하기 다른 기상원인)기록되지 않았다고 해서 그 사실이 없었던 것은 아니다(기록은 했으나 자료가 없어졌을수도 있고 모든 사실이 전부다 기록되지는 않는다 )기록된 폭발음이 꼭 확산 폭발음이 아닐수도 있다( 화산 폭발음이 너무 먼거리여서 들리지 않을수도 있고 다른 소리가 기록된 것이였을수도 있다)부족한 자료를 신빙할수 있는가 기록의 소멸도 예상 할수 있다 실제로 운석이 떨어졌는데 안들렸을수 있다

The following appeared in a memo from the president of Bower Builders a company that constructs new homes

A nationwide survey reveals that the two most-desired home features are a large family room and a large well-appointed kitchen A number of homes in our area built by our competitor Domus Construction have such features and have sold much faster and at significantly higher prices than the national average To boost sales and profits we should increase the size of the family rooms and kitchens in all the homes we build and should make state-of-the-art kitchens a standard feature Moreover our larger family rooms and kitchens can come at the expense of the dining room since many of our recent buyers say they do not need a separate dining room for family meals

Write a response in which you examine the stated andor unstated assumptions of the argument Be sure to explain how the argument depends on these assumptions and what the implications are for the argument if the assumptions prove unwarranted

The presidentrsquos first unstated but apparent assumption is that a nationwide survey can be applied to local areas There is no guarantee that the area in which Bower Builders builds houses will follow the national consumer trend Since he only mentions the overall result of the nationwide survey it is difficult to determine more specific conditions such whether people in urban and rural areas have the same housing preferences Such uncertainty is further exacerbated by the nature of the survey question itselfmdash it does not deal with current trends in actual home purchases but with desired home features The naiumlve assumption that consumer desire will directly result in consumer action underlies the presidentrsquos interpretation and application of the survey results to his company plan Not everyone who wishes for a large family room and kitchen will or can actually buy a house with those features mainly due to financial reasons Furthermore the president also assumes for no evident reason that consumers will not hesitate to purchase houses with state-of-the-art kitchens as a standard rather than optional feature when it is clear that such a feature will raise the overall cost significantly He continues to blunder in his mistaken assumptions about consumer behavior in assuming that the taste of recent buyers can represent the concern of future prospective buyers The fact that recent buyers have claimed no need for separate dining rooms does not mean future buyers will feel the same way as well

The president of Bower Builders recommends that in order to raise company profits the company should build houses with large family rooms and large state-of-the-art kitchens He uses the results of a nationwide survey on desired home features and the example of Bower Buildersrsquo competitor Domus Construction to support his plan His memo manifests several assumptions about surveys consumer behavior and Domus Constructionrsquos houses that do not necessarily bolsterbuttress his argument

GRE AWA John박 박정어학원

The writer assumes 1 the nationwide survey results can be applied to the local area (Desire cannot reflect consumer trend) 2 Domus Construction was profitable because they had such features (The writer should investigate what other features the DC provides and the actual number of homes sold) 3 State-of-the-art kitchens will sell well (no evidence to buttress his assumption furthermore they would need to spend more money which would increase the price of the housesmdashtheir market will be limited to the affluent) 4 The recent buyers represent the concern of most buyersmdashthat they are ok with a house that is without a dining room

Nationwide survey might not be applicable to this regionDomus Construction could have other superior qualities than Bower Builders Ex DesignSelling fast doesnrsquot indicate quantitySmall yards complaints could be voiced in the future

First the author concludes that a nationwide survey reveals that the two most desired home features are a bathroom with a whirlpool tub and a large kitchen However this nationwide survey may not reflect the trends of the customers that Bower Builders target The author assumes that the nationwide trend reflects customer trends The national study would lead support only if the nationwide trend reflect the home-purchasing trends The nationwide trend may just be a trend of desire which does not result in the act of purchasing However the author does not provide credible evidence that this is the case

전국 조사에서 가장 인기있는 집안 구조 2가지는 훨풀 튜브가 마련된 욕실과 커다른 부엌으로 조사되었다 경쟁업체 Domus Construction이 신축한 완공예정인 주택은 이러한 욕실을 갖추고 있어서 분양이 조기에 이루어지고 있고 가격도 평균치보다 상당히 웃돌고 있다 우리도 집을 많이 팔도 그만큼 이윤을 많이 남기려면 신규 주택에는 위의 2가지 사항을 갖추어야 한다 최근 우리가 지은 주택 수요자들이 적은 평수에 대해서는 별다른 불만사항이 없었기 때문에 앞으로 평수를 더 줄여서 이익을 극대화 할 수 있을 것이다

주장 We should include whirlpool tubs and a large kitchen

1 경쟁업체 Domus Construction이 신축한 완공예정인 주택은 이러한 욕실을 갖추고 있어서라기 보다는 위치가 좋거나 다른 마감재(construction material)나 인테리어가 뛰어나서 분양이 조기에 이루어진것이라고 볼 수 있다 2 전국조사가 건물이 지어지는 지역에 항상 적용되리라고 볼 수 없다 3 평수를 줄이는 것에 불만이 없었다는 것은 editor의 견해일 수 있다 사람들이 표현하지 않은 것일 수도 있고 다른 장점이 많아서 그런 단점을 드러내지 않은 것일 수 있기 때문에 속단할 수 없다 4 다른것을 함으로써 더 잘 팔릴수도 있다

2011 7 19 화요일 GRE AWA 실전반이OO

Argument 94

The president of Bower Builders contends recommends that in order to raise company profits the company should build houses with large family rooms and large state-of-the-art kitchens He uses the results of a nationwide survey on desired home features and the example of Bower Buildersrsquo competitor Domus Construction to support his plan His memo manifests several assumptions about surveys consumer behavior and Domus Constructionrsquos houses that do not

GRE AWA John박 박정어학원

necessarily bolsterbuttress his argumentThe presidentrsquos first unstated but apparent assumption is that a nationwide survey can be

applied to local areas There is no guarantee that the area in which Bower Builders builds houses will follow the national consumer trend Since he only mentions the overall result of the nationwide survey it is difficult to determine more specific conditions such whether people in urban and rural areas have the same housing preferences Such uncertainty is further exacerbated by the nature of the survey question itselfmdash it does not deal with current trends in actual home purchases but with desired home features The naiumlve assumption that consumer desire will directly result in consumer action underlies the presidentrsquos interpretation and application of the survey results to his company plan Not everyone who wishes for a large family room and kitchen will or can actually buy a house with those features mainly due to financial reasons Furthermore the president also assumes for no evident reason that consumers will not hesitate to purchase houses with state-of-the-art kitchens as a standard rather than optional feature when it is clear that such a feature will raise the overall cost significantly He continues to blunder in his mistaken assumptions about consumer behavior in assuming that the taste of recent buyers can represent the concern of future prospective buyers The fact that recent buyers have claimed no need for separate dining rooms does not mean future buyers will feel the same way as well

In addition the president finds a real-life actualization of the nationwide survey results in the recent sales of Domus Construction However he easily assumes that large family rooms and kitchens are the only reasons the houses of Domus Construction sell well He does not take into account other features and selling points of the competitorrsquos houses There may well be other explanations for its houses selling more quickly and expensively including additional home features as well as external factors such as proximity to better schools or superior financial solvency of its clientele If Bower Builders merely added larger family rooms and kitchens without taking into consideration the other factors they may lose rather than gain profits

Clearly the presidentrsquos assertion that Bower Builders make houses with large family rooms and high-tech kitchens at the expense of dining rooms rests on a number of assumptions that are ill-informed and naiumlve If Bower Builders undertakes the proposed plan without further research into local consumer desires purchasing trends and the marketing and sales of competing companies the company will risk losing money by building big new houses that people cannot afford to or will not wish to buy

The following appeared in a letter to the editor of a journal on environmental issues

Over the past year the Crust Copper Company (CCC) has purchased over 10000 square miles of land in the tropical nation of West Fredonia Mining copper on this land will inevitably result in pollution and since West Fredonia is the home of several endangered animal species in environmental disaster But such disasters can be prevented if consumers simply refuse to purchase products that are made with CCCs copper unless the company abandons its mining plans

Write a response in which you examine the stated andor unstated assumptions of the argument Be sure to explain how the argument depends on these assumptions and what the implications are for the argument if the assumptions prove unwarranted

The writer assumes 1 The writer is trying to avoid an inevitability 2 Mining copper will result pollution (they could make preventative measures) 3 The writer may be too late from stopping CCC from developing the area into a copper mine 4 Since mining is an underground enterprise the surface may not be affected that much therefore endangered species may not be affected 4 Consumers wonrsquot buy CCC products if the journal publishes a negative review about CCC (How many readers) CCC could a company that has ties with many IT companies and industries in that their copper is almost ubiquitous in various products

GRE AWA John박 박정어학원

지난 한해동안 CCC(Consolidated Copper 회사)는 서부 플로리다의 열대 지역에 1백만 마일이 넘는 땅을 사들였다 이 곳에서의 채광 활동은 서부 플로리다가 몇몇 멸종 위기에 처한 동물의 서식지이기 때문에 분명 오염과 환경파괴를 가져올 것이다 그러나 이러한 파괴는 CCC 회사가 채광을 포기할 때까지 이 회사가 채굴한 구리로 제조된 제품을 구매하지 않으면 막을 수 있을 것이다

결론 such disaster can be prevented if consumers simply refuse to purchase products that are made with CCCs copper until the company abandons its mining plans

1 구리로 제조된 물건이 생활에 필수적인 것이 많은 만큼 불매가 쉽지 않을 수 있다(전선이나 각종 전자제품에 필수적으로 들어가기 때문에)

2 불매를 유도한다고 해서 소비자들이 구매를 안하는 것은 아니다3 적절한 채굴로 환경파괴를 가져 오지 않을 수 있다 (땅속에 있는 물질을 채굴하는 만큼 생물에 영향을 안

미칠 수도 있다)4 이미 채굴이 다 끝나서 더 이상의 채굴이 없을 수도 있다 5 땅을 구입한다고 해서 채광하는건 아니다 (다른 용도로 샀을수도 있다)

The following is a letter to the editor of an environmental magazine

In 1975 a wildlife census found that there were seven species of amphibians in Xanadu National Park with abundant numbers of each species However in 2002 only four species of amphibians were observed in the park and the numbers of each species were drastically reduced There has been a substantial decline in the numbers of amphibians worldwide and global pollution of water and air is clearly implicated The decline of amphibians in Xanadu National Park however almost certainly has a different cause in 1975 troutmdashwhich are known to eat amphibian eggsmdashwere introduced into the park

Write a response in which you discuss what specific evidence is needed to evaluate the argument and explain how the evidence would weaken or strengthen the argument

Evidence needed the identity of the sender and the census taker of rsquo75 and lsquo02mdasha scientist or an environmentalist credibility issue census methodology vs mere observationmdashthe absence of evidence is not an evidence of absence the season of when the census was taken time-shiftmdashconditions may have changed worldwide decline may include Xanadu other species of predators that prey on amphibians because trout is only one species that prey on amphibians the number of troutmdashhave they increased significantly since rsquo75 First the author needs to be more overt about the credibility of the census and observation There were to accounts that notes the population of amphibiansmdashthe first a census and the second an observation The author needs to bolster his conclusion with the evidence that indicate that the census and second observation were done using scientific methodologies This could either could strengthen or weaken his claim In addition he

GRE AWA John박 박정어학원

needs to provide the specific season of when the census and observation occurred In this letter the writer is informing an editor of an environmental magazine that the number of amphibians was greatly reduced since 1975 and he points out the introduction of trout as the only reason for the decline However the author fails to provide crucial evidences that could strengthen or weaken his conclusion

The following appeared in a memorandum from the president of Hyper-Go Toy Company

Last year sales of our Fierce Fighter toy airplane declined sharply even though the toy had been a top seller for three years Our customer surveys show that parents are now more worried about youthful violence and are concerned about better education for their children Therefore to maintain profits we should discontinue all our action toys and focus exclusively on a new line of educational toys Several other toy companies have already begun marketing educational toys and report sales increases last year of 200 percent And since the average family income is growing sales of new Hyper-Go toys should also increase

Write a response in which you discuss what specific evidence is needed to evaluate the argument and explain how the evidence would weaken or strengthen the argument

First the president indicates that the sales of Fierce Fighter toy airplane declined sharply However he fails to consider the fact that toys are a fad Since FFT enjoyed a three year success it may be natural that the trend would subsideSecond Customer survey is this representative of most toy consumers This trend is not newThird other companies may have profited not through educational but other toys Or educational toy profit may be small in proportion to their sale of other toys 200Parents are not the customers companies should concentrate on what the children want to increase profitLastly parents worry about youthful violence and concern for better education are not new trends

The presidentrsquos decision to discontinue all action toys and focus only on educational toys is too extreme If this decision is taken into effect its procedure may be an onerous task because it would require major shifts in human resources and company image Therefore further data should be considered before following up on this decision

우리 회사에서 지난 3년간 최고 매출을 일으켰던 Fierce Fighter 장난감 비행기가 지난해에는 매출이 급격히 떨어졌다 자체 고객 조사에서는 부모들이 현재 청소년 폭력에 걱정을 하고 있어서 아이들의 양질 교육에 더 관심을 가지고 있는 것으로 나타났다 따라서 회사의 수익을 유지하기 위해서는 모든 자사 전투용 장난감 생산을 중단하고 오로지 교육적인 장난감 생산에 집중해야 한다 몇몇 여타 장난감 회사들도 이미 교육용 장난감 마케팅을 시작해서 지난해에는 200의 매출신장을 가져왔다고 한다 그리고 평균 가계 수입이 점점 늘고 있기 때문에 신형 Hyper-Go 장난감의 매출도 늘어날 것이다

3그룹 Time-shift Error

Woven baskets characterized by a particular distinctive pattern have previously been found only in the immediate vicinity of the prehistoric village of Palea and therefore were believed to have been unique to the Palean people Recently however archaeologists discovered such a Palean basket in Lithos an ancient village across the Brim River from Palea The Brim River is very deep and broad and so the ancient Paleans could only have crossed it by boat but there is no evidence that the Paleans had boats And boats capable of carrying groups of people and cargo were not developed until thousands of years after the Palean people disappeared Moreover Paleans would have had no need to cross the rivermdashthe woods around Palea are full of nuts berries and small game It follows that the so-called Palean baskets were not unique to Palea

GRE AWA John박 박정어학원

Write a response in which you discuss what specific evidence is needed to evaluate the argument and explain how the evidence would weaken or strengthen the argument

RefutationPrehistoric time shift-gtbrim river could have been narrow and shallow or it might have not existed Indigenous patterns may exist in other disconnected remote placesNuts berries small game(hunting) may not have existed or the author should be proved these existed at that time Abundance of resources doesnrsquot support the reason for seclusionBoats not yet found baskets may have been carried across by the river current without the help of a boat Lithos might have crossed the river for commercial purposeOne Palean basket does not substantiate the authorrsquos claimThe absence of evidence is not an evidence of absence

The author assumes without justification that present conditions are the same as at the prehistoric era The author unfairly infers from the presence of Brim River which exist today that it would have existed in the past However the author fails to offer any evidence to substantiate this inference It is very likely that the Brim River might not have existed in prehistoric times or if it did exist may have been shallow and narrow enough for the Paleans to easily cross Any of these scenarios if true would serve to undermine the claim thathelliphelliphelliphelliphellip

First the author claims that the Brim River was very deep and broad so the Paleans could not have crossed it However the author fails to offer any evidence to substantiate that this was true in the prehistoric time of the Paleans For all we know the Brim River might not have existed in the prehistoric era or if it did exist could have been a shallow and narrow river For example scientists believe that thousands of years ago an ice-bridge existed on the Bering Sea connecting Eurasia to what is now North America to explain how the Eskimos and the Asian inhabitants of America came to migrate all over the Americas In this example the absence of evidence is not an evidence of absence Therefore to sufficiently support his claim the author needs to substantiate the fact that the Brim River really did exist and was broad and wide in the Prehistoric Era proving that the Paleans could not have influenced or traded with other groups of people

First the author claims that the Brim River was very deep and broad so the Paleans could not have crossed it However the author fails to offer any evidence to substantiate that this was true in the prehistoric times of the Paleans For all we know the Brim River might not have existed in the prehistoric era or if it did exist could have been a shallow and narrow river For example rivers are created by natural erosion over thousands of years Niagara Falls carved its way from the mouth of Lake Ottawa and created a long river Likewise the current Brim Riverrsquos physical features may not have been wide and deep Therefore to sufficiently support his claim the author needs to substantiate the fact that the Brim River really did exist and was broad and wide in the Prehistoric Era proving that the Paleans could not have influenced or traded with other groups of people

예전에는 실로짠 특이한 무늬 바구니가 Palea의 선사시대 지역의 인근마을에서만 발견되어왔기 때문에 Palea 마을 사람들의 특징이라고 여겨졌었다 그러나 최근들어 고고학자들이 Lithos지역에서 Palean 바구니를 발견하였는데 그 당시 지역은 Brim 강을 가로질러 Palea까지 닿아있었다 이 강은 수심이 아주 깊고 강폭이 넓었으며 때문에 고대의 Palea인들은 배를 이용해서 강을 건널수 있었을 것이다 그러나 이들이 배를 가지고 있었다는 증거는 발견되지 않고 있다 더군다나 이들이 멸명한 이후 수천년이 지난뒤에도 수많은 물자와 사람을 실어 나를수 있는 용적을 가진 배는 개발되지 않았다 이와더불어 Palea인들은 강을 건널필요가 없었는데 그것은 너트나무 장과열매 그리고 작은 사냥감들이 주변숲에 풍부했기때문이다 따라서 Palean 바구니라고 하는 것도 Palea인들만의 전유물이 아니라는 결론을 얻을 수 있다

결론 if follows that the so-called Palean baskets were not unique to Palea이번문제는 굿이 causal Bad analogy 로 구분해서 찾기가 힘드내요 배를 발견했다는 증거가 없는것이다 (앞으로도 발견될수 있음)

GRE AWA John박 박정어학원

계절의 영향으로 겨울에 얼음이 두껍게 언다든지 여름에 가뭄으로 인해서 건널수 있다물자가 풍부한 것이 이동하지 않을 조건이 아니다 다른 것에 의해서 이동가능(의약품등)

Thirteen years ago researchers studied a group of 25 infants who showed signs of mild distress when exposed to unfamiliar stimuli such as an unusual odor or a tape recording of an unknown voice They discovered that these infants were more likely than other infants to have been conceived in early autumn a time when their mothers production of melatonin hormone known to affect some brain functions would naturally increase in response to decreased daylight In a follow-up study conducted earlier this year more than half of these children now teenagers who had shown signs of distress identified themselves as shy Clearly increased levels of melatonin before birth cause shyness during infancy and this shyness continues into later life

Write a response in which you examine the stated andor unstated assumptions of the argument Be sure to explain how the argument depends on these assumptions and what the implications are for the argument if the assumptions prove unwarranted

Any baby exposed to unpleasant stimuli would react in such wayFirst of all the author states 25 infants as his evidence However this research sample is too small to prove his claimSecond the author states that 25 infants were conceived in early autumn which he claims lead to a shy disposition However this is faulty evidenceThird the research study was a long term study done in the span of 13 years However the author only writes about the initial and final stages of the study and leaves out evidences of what could have happened during the 13 years which could be more evidential factors of influenceFourth neither the infantsrsquo genetic predisposition nor their environment were taken into accountFinally the author concludes that his shyness continues into later life (Other factors could alter this disposition epigenetic theory)

13 년전 학자들은 25명의 유아를 대상으로 이상한 냄새나 특이한 소리를 녹음한 테잎등으로 낯선 자극을 주었을때 보이는 미미한 압박감 증상을 조사하였다 이들은 성숙기가 막 지났을 즈음에 보통의 유아들이 비슷한 증상을 보이는 정도 보다는 다소 민감한 반응을 보였는데 이 시기는 아이의 엄마가 뇌의 일부 기능에 영향을 미치는 것으로 알려진 멜라토니아 호르몬을 생산하는 시기로써 이 호르몬은 낯 시간이 짧을때 자연적으로 증가할 수도 있다 금년초에 실시된 추가연구에서 현재 10대로 성장한 당시 조사대상의 절반 이상의 아이들이 부끄럼을 잘타는 것으로 여기고 있었다 따라서 분명한 것은 출산전 멜라토닌 수치의 증가가 유아기에 수줍음 등의 영향을 미치게 되며 이러한 영향이 성장후에도 작용한다는 것이다

주장 Clearly increased levels of melatonin before birth cause shyness during infancy and this shyness continues into later life

1 25명의 아기로 결론 내리기에 샘플이 작다2 과학적 사실들에 대한 명확한 근거가 엇음3 다른 영향을 간과했다(다른 호르몬에 의한 영향 후천적인 성격형성의 영향)

GRE AWA John박 박정어학원

The following is a letter to the editor of the Atticus City newspaper

Former Mayor Durant owes an apology to the city of Atticus Both the damage to the River Bridge which connects Atticus to Hartley and the traffic problems we have long experienced on the bridge were actually caused 20 years ago by Durant After all he is the one who approved the construction of the bridge If he had approved a wider and better-designed bridge on which approximately the same amount of public money would have been spent none of the damage or problems would have occurred Instead the River Bridge has deteriorated far more rapidly over the past 20 years than has the much longer Derby Bridge up the river Even though the winters have been severe in the past several years this is no excuse for the negligence and wastefulness of Durant

Write a response in which you discuss what questions would need to be answered in order to decide whether the recommendation is likely to have the predicted result Be sure to explain how the answers to these questions would help to evaluate the recommendation

전임 시장인 Durant 씨는 Atticus 시에 대해 사과할 의무가 있습니다 Atticus와 Hartley를 잇는 River Bridge 교량에 대한 피해와 이 교량에서 오랫동안 주민들이 겪어오고 있는 교통 혼잡 문제들은 실제로 20년 전부터 시작된 것이었습니다 결정적으로 그가 교량 공사를 허가했던 바로 그 장본인입니다 당시 비슷한 공사비용으로 폭이 더 넓고 튼튼하게 설계된 교량을 허가했다면 이러한 문제나 피해는 발생하지 않았을 겁니다 더군다나 이 다리는 지난 20년 동안 상류에 건설된 훨씬 오래된 Derby 다리보다도 빠르게 부식되어 갔습니다 지난 수년동안 심지어 혹한이 있었다 하더라도 이러한 태만과 국고 손실에 대한 책임을 회피할 길이 없는 것입니다

결론 Former Mayor Durant owes an apology to the city of Atticus

1 디자인이 문제가 아닐수 있다 (디자인은 좋았으나 건설과정에 문제가 있었을 수 있다)2 그 당시의 시예산이 적어서 더 큰 다리를 짓기가 불가능했을 수도 있다3 교통량이 많거나 다른 상황으로 인해서 부식이 빨리 됐을 수 있다4 그 당시에는 최선의 선택이였지만 갑자기 변한 상황에 의해서 이런 문제점들이 발생했을 수 있다

GRE AWA John박 박정어학원

4그룹 거짓인과관계 오류 (False Cause) 빈출

Fifteen years ago Omega University implemented a new procedure that encouraged students to evaluate the teaching effectiveness of all their professors Since that time Omega professors have begun to assign higher grades in their classes and overall student grade averages at Omega have risen by 30 percent Potential employers looking at this dramatic rise in grades believe that grades at Omega are inflated and do not accurately reflect student achievement as a result Omega graduates have not been as successful at getting jobs as have graduates from nearby Alpha University To enable its graduates to secure better jobs Omega University should terminate student evaluation of professors

Write a response in which you discuss what specific evidence is needed to evaluate the argument and explain how the evidence would weaken or strengthen the argument

Omega professor evaluation implemented 15 years ago =gt Omega prof assign higher grades 30Employers believe therersquos grade inflation

Thus unsuccessful employment than AlphaTherefore to secure jobs Omega should end evaluating profs

Specific evidence neededRelationship between higher grades and evaluationRelationship between GPA and unsuccessful employmentAlpharsquos education could just be better than OmegaldquoFifteen years agordquo is a long time other factors could have influenced Why is the inflation a problem just now How much is Alpha better Is the comparison just How much gap is thereOmegarsquos student could just be doing better in their studiesComparison to other universities다른 대안 없나hellip Could Omega alleviate the employment problem by implementing a different procedure or program

15 년전 우리 대학은 학생들로 하여금 교수평가를 하도록 한 새로운 조치를 시행했었습니다 이후 교수들은 자신의 학과 학생들에게 높은 학점을 주었으며 그에따라 학생들의 전체 평점이 30나 올랐습니다 외부의 기업체들은 분명 점수가 지나치게 부풀려졌다고 믿고 있습니다 결국 본 대학 졸업생들이 인근 Alpha 대학의 졸업자들보다 구직률이 떨어지는 이유를 잘 보여주고 있는 것입니다 이를 해결하기 위해 이제부터는 학생들에 의한 교수평가제를 중단해야 합니다

결론 Omega University should now terminate student evaluation of professors

반박 교수 평가와 학점 인플레의 연관성이 적다( 교수 평가를 먼저하고 학점을 나중에 매길수도 있다)채용기준에 성적만 있는게 아니다 학업성취의 결과 일수도 있다 Alpha 가 원래 유능했다 Alpha 의 교육내용이 좋았다

GRE AWA John박 박정어학원

In this memo the dean of Omega University(OU) recommends OU to terminate professor evaluation to secure better jobs for the students To support this recommendation the dean offers several reasons However this argument contains several logical flaws which render it unconvincing

A threshold problem with the argument involves the voluntary nature of the evaluationprocedure The dean provides no evidence about the number or percentage of Omegastudents who participate in the procedure Lacking such evidence it is entirely possible thatthose numbers are insignificant in which case terminating the procedure is unlikely to haveany effect on the grade average of Omega students or their success in getting jobs aftergraduationThe argument also assumes unfairly that the grade-average increase is the result of theevaluation procedure--rather than some other phenomenon The dean ignores a host of otherpossible explanations for the increase--such as a trend at Omega toward higher admissionstandards or higher quality instruction or facilities Without ruling out all other possibleexplanations for the grade-average increase the dean cannot convince me that by terminatingthe evaluation procedure Omega would curb its perceived grade inflation let alone help itsgraduates get jobsEven if the evaluation procedure has resulted in grade inflation at Omega the deans claimthat grade inflation explains why Omega graduates are less successful than Alpha graduatesin getting jobs is unjustified The dean overlooks a myriad of other possible reasons forOmegas comparatively poor job-placement record Perhaps Omegas career services areinadequate or perhaps Omegas curriculum does not prepare students for the job market aseffectively as Alphas In short without accounting for other factors that might contribute toOmega graduates comparative lack of success in getting jobs the dean cannot justify theclaim that if Omega curbs its grade inflation employers will be more likely to hire OmegagraduatesFinally even if the dean can substantiate all of the foregoing assumptions the deansassertion that Omega must terminate its evaluation procedure to enable its graduates to findbetter jobs is still unwarranted in two respects First the dean ignores other possible ways bywhich Omega can increase its job-placement record--for example by improving its publicrelations or career-counseling services Second the dean unfairly equates more jobs withbetter jobs In other words even if more Omega graduates are able to find jobs as a result ofthe deans recommended course of action the kinds of jobs Omega graduates find would notnecessarily be better onesIn sum the deans argument is unpersuasive as it stands To strengthen it the dean mustprovide better evidence that the increase in grade average is attributable to Omegasprofessor-evaluation procedure and that the end result is a perception on the part ofemployers that Omega graduates are less qualified for jobs than Alpha graduates To betterassess the argument I would need to analyze 15-year trends in (l) the percentage of Omegastudents participating in the evaluation procedure (2) Omegas admission standards andquality of education and (3) Omegas emphasis on job training and career preparation I wouldalso need to know what other means are available to Omega for enabling its graduates to findbetter jobs

GRE AWA John박 박정어학원

The following appeared in a memo from a vice president of Quiot Manufacturing

During the past year Quiot Manufacturing had 30 percent more on-the-job accidents than at the nearby Panoply Industries plant where the work shifts are one hour shorter than ours Experts say that significant contributing factors in many on-the-job accidents are fatigue and sleep deprivation among workers Therefore to reduce the number of on-the-job accidents at Quiot and thereby increase productivity we should shorten each of our three work shifts by one hour so that employees will get adequate amounts of sleep

Write a response in which you examine the stated andor unstated assumptions of the argument Be sure to explain how the argument depends on these assumptions and what the implications are for the argument if the assumptions prove unwarranted

The following appeared in a memo from a vice president of Alta Manufacturing

During the past year Alta Manufacturing had thirty percent more on-the-job accidents than nearby Panoply Industries where the work shifts are one hour shorter than ours Experts believe that a significant contributing factor in many accidents is fatigue caused by sleep deprivation among workers Therefore to reduce the number of on-the-job accidents at Alta we recommend shortening each of our three work shifts by one hour If we do this our employees will get adequate amounts of sleep

Write a response in which you discuss what questions would need to be answered in order to decide whether the recommendation and the argument on which it is based are reasonable Be sure to explain how the answers to these questions would help to evaluate the recommendation

The following appeared in a memo from the vice president of Butler Manufacturing

During the past year workers at Butler Manufacturing reported 30 percent more on-the-job accidents than workers at nearby Panoply Industries where the work shifts are one hour shorter than ours A recent government study reports that fatigue and sleep deprivation among workers are significant contributing factors in many on-the-job accidents If we shorten each of our work shifts by one hour we can improve Butler Manufacturings safety record by ensuring that our employees are adequately rested

1 Write a response in which you discuss what specific evidence is needed to evaluate the argument and explain how the evidence would weaken or strengthen the argument

2 Write a response in which you discuss what questions would need to be answered in order to decide whether the recommendation is likely to have the predicted result Be sure to explain how the answers to these questions would help to evaluate the recommendation

4번 반복됨

Alta has 30 more job accidents than Panoply(work shifts one hour shorter)Experts Job accidents caused by fatigue and sleep deprivationTherefore to reduce job accidents and increase productivity shorten three work shifts by one hour for adequate sleep

지난해 우리 회사는 인근 Panoply Industries보다 업무상 재해가 30나 더 많았다 그 회사는 우리보다 근무 교대시간이 1시간 정도 짧았다 전문가들은 대부분의 업무상 재해에 있어서 가장 중요한 요인이 과로와 수면부족으로 보고있다 따라서 우리 회사에서 높은 산업재해를 줄이고 아울러 생산성을 높이기 위해서는 근로자들이 충분한 수면을 취할 수 있도록 1시간씩 3교대 시간을 줄여야 한다

In this memo the (author) vice president of Alta Manufacturing (AM) recommends that to reduce on-the-job accidents and increase productivity AM should shorten its three work shifts by one hour so that employees can

GRE AWA John박 박정어학원

get more sleep To support this recommendation the author provides several evidences However careful scrutiny of each of the facts reveals that it provides little credible support for the authorrsquos recommendation QuestionsThe number of accidents What kind of accidents The seriousness of the accidents is importantHow many employees are in each company What are their productsFalse cause Sleep may not be the reason for the on-the-job accidents What do Alta and Panoply manufacture

First of all the author believes that fatigue caused the on-the job accidents However there could be other reasons The author observes a correlation between sleep deprivation and on-the-job accidents then concludes that the former is the cause of the latter However the author fails to rule out other possible explanations For example it is entirely possible that Alta factories require more strenuous and dangerous labor than Panoply Without ruling out all other such factors it is unfair to conclude that fatigue is responsible for the accidents In addition the work-shifts may not be the cause of the sleep deprivation and fatigue It is possiblehellip Thus the author should provide what exactly Panoply and Alta manufacture and more precise data about their working conditions to be more convincing

Shortening the shift by one hour does not necessarily lead to more sleep And is one hour enoughLess accidents does not mean increased productivity

결론 We should shorten each of out three work shifts by one hour

반박 경쟁사에 비해서 시간당 하는 업무량이 많아서 더욱 피곤할 수도 있다 시간이 문제가 아니라 노후된 시설 설비 자체의 문제 작업 자체가 원래 위험한 것이여서 사고가 많을 수도 있다 다른 회사는 더욱 많은 작업시간에도 불구하고 안정한 작업여건으로 인해서 사고율이 오히려 더 작을 수도 있다비교사의 재해감소가 다른 요인일수 있다(안전 교육 철저)줄인 시간이 피로회복이나 수면으로 연결 안될수 있음(술을 마실 수도 있고 그 시간에 휴식을 취하지 않고 다른일을 함으로써 더욱 피로해질수 있다)

This editorial recommends that Alta Manufacturing reduce its work shifts by one hour each inorder to reduce its on-the-job accident rate and thereby increase Altas productivity To supportthis recommendation the author points out that last year the number of accidents at Alta was30 greater than at Panoply Industries where work shifts were one hour shorter The authoralso cites certain experts who believe that many on-the-job accidents are caused by fatigueand sleep deprivation I find this the argument unconvincing for several reasonsFirst and foremost the author provides absolutely no evidence that overall workerproductivity is attributable in part to the number of on-the-job accidents Although commonsense informs me that such a relationship exists the author must provide some evidence ofthis cause-and-effect relationship before I can accept the authors final conclusion that theproposed course of action would in fact increase Altas productivitySecondly the author assumes that some accidents at Alta are caused by fatigue or sleepdeprivation However the author overlooks other possible causes such as inadequateequipment maintenance or worker training or the inherent hazards of Altas manufacturingprocesses By the same token Panoplys comparatively low accident rate might be attributablenot to the length of its work shifts but rather to other factors such as superior equipmentmaintenance or worker training In other words without ruling out alternative causes ofon-the-job accidents at both companies the author cannot justifmbly conclude that merely byemulating Panoplys work-shift policy Alta would reduce the number of such accidentsThirdly even assuming that Altas workers are fatigued or sleep-deprived and that this is thecause of some of Altas on-the-job accidents in order to accept the authors solution to thisproblem we must assume that Altas workers would use the additional hour of free time tosleep or rest However the author provides no evidence that they would use the time in thismanner It is entirely possible that Altas workers would use that extra hour to engage in someother fatiguing activity Without ruling out this possibility the author cannot convincinglyconclude that reducing Altas work shifts by one hour would reduce Altas accident rateFinally a series of problems with the argument arise from the scant statistical information onwhich it relies In comparing the number of accidents at Alta and Panoply the author fails toconsider that the per-worker accident rate might reveal that Alta is actually safer than Panoplydepending on the total number of workers at each company Second perhaps accident rates

GRE AWA John박 박정어학원

at the two companies last year were aberrations and during other years Altas accident ratewas no greater or even lower than Panoplys rate Or perhaps Panoply is not representativeof industrial companies generally and that other companies with shorter work shifts have evenhigher accident rates In short since the argument relies on very limited statistical information Icannot take the authors recommendation seriouslyIn conclusion the recommendation for emulating Panoplys work-shift policy is not wellsupported To convince me that shorter work shifts would reduce Altas on-the-job accidentrate the author must provide clear evidence that work-shift length is responsible for some ofAltas accidents The author must also supply evidence to support her final conclusion that alower accident rate would in fact increase overall worker productivity

The following appeared in a memo from the vice president of marketing at Dura-Sock Inc

A recent study of our customers suggests that our company is wasting the money it spends on its patented Endure manufacturing process which ensures that our socks are strong enough to last for two years We have always advertised our use of the Endure process but the new study shows that despite our socks durability our average customer actually purchases new Dura-Socks every three months Furthermore our customers surveyed in our largest market northeastern United States cities say that they most value Dura-Socks stylish appearance and availability in many colors These findings suggest that we can increase our profits by discontinuing use of the Endure manufacturing process

1 Write a response in which you examine the stated andor unstated assumptions of the argument Be sure to explain how the argument depends on these assumptions and what the implications are for the argument if the assumptions prove unwarranted

2 Write a response in which you discuss what specific evidence is needed to evaluate the argument and explain how the evidence would weaken or strengthen the argument

3 Write a response in which you discuss what questions would need to be answered in order to decide whether the recommendation and the argument on which it is based are reasonable Be sure to explain how the answers to these questions would help to evaluate the recommendation

Intro The vice president of marketing at Dura-Sock Inc is offering a potentially harmful investment recommendation by claiming that Dura-Sock should discontinue its use of the ldquoEndurerdquo process To support his recommendation he points out a study that Dura-Sock customers actually purchase the socks every three months and a survey that reveals that Dura-Sock customers like the sockrsquos stylish appearance and availability in many colors The study and survey however are insufficient in supporting his proposal and the VP makes several unwarranted assumptionsIntro (simplified) The VP states that though Dura-Socks last for two years customers buy the socks every three months Therefore he assumes that the consumersrsquo motive for buying the produce is not its durabilityHowever the author fails to rule out other possible motivation for consumption

Even if the survey is reliable the author should consider the rest of the market Vague terms ldquowasting moneyrdquomdashprecisely how much are they wasting Studysurvey errorThe company must calculate the outcome of such momentous decisionStudy participantsrsquo comment that they prefer Dura-Sock for its stylishness and availability might take Dura-Sockrsquos enduring quality for granted

우리회사 제품 소비자들에 대한 최근 조사에서 지난 2년여간 양말의 내구성을 강하게 하는 필수공정이었던 자사 특허의 Endure 공정에 들어가는 비용이 낭비라고 말하고 있다 우리 회사는 항상 이 공정 처리에 대한 광고를 내보냈으나 이에 대한 시장 조사에서 실제로 고객들은 이 신제품을 평균 석달마다 구매하는 것으로 나타났다 더군다나 북동부지역에서 실시한 대규모 시장조사에 응답한 고객들은 양말의 모양과 색상등에 더

GRE AWA John박 박정어학원

관심을 나타냈다 이러한 결과는 우리회사가 신기술 공법을 중단하면 그에 따라 수익이 늘어날 것이라는 것을 말해주고 있는 것이다주장 These findings suggest that Dura0Sock can increase its profits by discontinuing its use of the ldquoEndurerdquo manufacturing process

1 survey가 정확한 소비자의 의견을 나타낸 것인가 다른 선택없이 양자택일과 같은 방법의 survey였는지2 북동부지역의 시장조사가 전체 의견을 대표할 수 있나3 사람들이 모양이나 색상에 앞서 내구성을 먼저 평가했을 수도 있다 내구성을 갖추었다는 전제하에 모양과 색상에 관심을 드러낸 것일 수 있다4 소비자가 도매상(retail)인지 소매상(whole)인지가 없다

The following appeared in a business magazine

As a result of numerous complaints of dizziness and nausea on the part of consumers of Promofoods tuna the company requested that eight million cans of its tuna be returned for testing Promofoods concluded that the canned tuna did not after all pose a health risk This conclusion is based on tests performed on samples of the recalled cans by chemists from Promofoods the chemists found that of the eight food chemicals most commonly blamed for causing symptoms of dizziness and nausea five were not found in any of the tested cans The chemists did find small amounts of the three remaining suspected chemicals but pointed out that these occur naturally in all canned foods

Write a response in which you discuss what questions would need to be addressed in order to decide whether the conclusion and the argument on which it is based are reasonable Be sure to explain how the answers to the questions would help to evaluate the conclusion

Representativeness of the tested cansThey should conduct a comparative studyThe testing could be biased because Promofoods employees conducted the testingHow much (quantity) of the five and three suspected chemicals were in the canned foodsFalse cause The substance that caused dizziness and nausea may not be one of the eight common chemicals

많은 소비자들의 현기증과 구역질 불만에 따라 Promofoods사는 지난해 참치 캔 8백만 개를 테스트하기 위해 반품시켰다 그 결과 캔에서는 건강에 위험이 될 수 있는 화합물질이 없었던 것으로 회사측은 결론지었다 이러한 결론은 회사측 화학연구자들이 회수된 캔의 샘플을 테스트해서 이들 증상의 원인이 되는 8가지 화합물 중에서 5가지가 실험된 캔에서 발견되지 않았다는 사실에 근거한 것이다 이들 화학자들은 나머지 3개가지 화합물이 모든 캔 식료품에서 흔히 발견되는 것이라고 언급했다 결론 Promofoods concluded that the cans did not after all contain chemicals that posed a health risk

1 공인된 기간에서 테스트를 한 것이 아니고 자사에서 직접 테스트를 했기에 신뢰성이 안간다 2 이런 증상을 일으키는 8개의 물질 말고 다른 물질들이 캔속에 많이 포함됬을수 있다 3 나머지 3개의 물질들의 함유량이 많아서 다른 종류의 캔들은 문제를 일으키지 않지만 참치캔은 문제를

일으킬 수 있다

This magazine article concludes that the 8 million cans of tuna Promofoods recalled due tocomplaints about nausea and dizziness do not after ail contain any chemicals that pose a

GRE AWA John박 박정어학원

health risk To support this conclusion the author cites the fact that five of eight chemicalscommonly causing these symptoms were not found in the recalled cans while the other threealso occur naturally in other canned foods For several reasons this evidence lends littlecredible support to the authors conclusionTo begin with the author relies partly on the fact that although three of the eight chemicalsmost commonly blamed for nausea and dizziness appeared in Promofoods recalled tunathese chemicals also occur naturally in other canned foods However this fact alone lends nosupport to the authors conclusion for two reasons First the author might be ignoring animportant distinction between naturally occurring chemicals and those not occurring naturallyIt is entirely possible that these three chemicals do not occur naturally in Promofoods tunaand that it is for this reason that the chemicals cause nausea and dizziness Secondly it isentirely possible that even when they occur naturally these chemicals cause the samesymptoms Unless the author rules out both possibilities he cannot reliably conclude that therecalled tuna would not cause these symptomsAnother problem with the argument is that the authors conclusion is too broad Based onevidence about certain chemicals that might cause two particular heath-related symptoms theauthor concludes that the recalled tuna contains no chemicals that pose a health risk Howeverthe author fails to account for the myriad of other possible health risks that the recalled tunamight potentially pose Without ruling out all other such risks the author cannot justifiablyreach his conclusionA third problem with the argument involves that fact that the eight particular chemicals withwhich the test was concerned are only the eight most commonly blamed for nausea anddizziness It is entirely possibly that other chemicals might also cause these symptoms andthat one or more of these other chemicals actually caused the symptoms Without ruling outthis possibility the author cannot jusufiably conclude that the recalled tuna would not causenausea and dizzinessA final problem with the argument involves thetesting procedure itself The author providesno information about the number of recaUed cans tested or the selection method used Unlessthe number of cans is a sufficiently large sample and is statistically repre sentative of all therecalled cans the studys results are not statistically reliableIn conclusion the article is unconvincing as it stands To strengthen the assertion that therecalled tuna would not cause nausea and dizziness the author must provide evidence thatthe three chemicals mentioned that occur naturally in other canned foods also appear naturallyin Promofoods tuna The author must also provide evidence that ingesting other canned foodscontaining these three chemicals does not cause these symptoms To better evaluate theargument we would need to know whether the sample used in the tests was statisticallysignificant and representative of all the recalled tuna We would also need to know what otherchemicals in the recalled tuna might pose any health risk at all

5그룹 불충분 조건오류 빈출

Natures Way a chain of stores selling health food and other health-related products is opening its next franchise in the town of Plainsville The store should prove to be very successful Natures Way franchises tend to be most profitable in areas where residents lead healthy lives and clearly Plainsville is such an area Plainsville merchants report that sales of running shoes and exercise clothing are at all-time highs The local health club has more members than ever and the weight training and aerobics classes are always full Finally Plainsvilles schoolchildren represent a new generation of potential customers these schoolchildren are required to participate in a fitness-for-life program which emphasizes the benefits of regular exercise at an early age

Write a response in which you examine the stated andor unstated assumptions of the argument Be sure to

GRE AWA John박 박정어학원

explain how the argument depends on these assumptions and what the implications are for the argument if the assumptions prove unwarranted

False cause

First of all the author believes that the Increased sales of running shoes and exercise clothing indicates

Plainesville residentsrsquo interest in leading healthy lives However this assumption is not logically convincing for

several reasons could be a fashion trendTime shift ldquoFitness for liferdquo might not have any influence on schoolchildren as they growFalse cause There could be other reasons for member increase in the health clubAll of the above are insufficient condition

The author has to prove that local residents are interested in leading healthy lives However he supports his conclusion with insufficient evidence Nevertheless even if the residents are concerned with health naturersquos way may not be successful First

그 동안의 경험을 토대로 볼 때 건강생활과 밀접히 관련되어 있는 거주 지역에서 본 상점들이 아주 호응을 얻고 있다 따라서 이러한 주민들이 많이 거주하고 있는 Plainsville 에 새로운 상점들을 계속 세워야 한다 이 지역 상인들은 런닝화와 운동복 판매가 가장 높다고 말한다 불과 5 년전에는 거의 전무하다시피하던 지역 헬스 클럽의 경우도 엄청나게 많은 회원을 확보하고 있으며 웨이트 트레이닝과 에어로빅 강좌들도 항상 만원이라고 한다 새로운 고객층을 예측해 보는 것도 가능하다 이 지역의 학생들의 경우 Fitness for Life프로그램을 받게 되는데 이러한 프로그램을 통해서 유년시절부터 정규적인 운동 습관을 들이게 하고 있는 것이 그것이다

결론 We should therefore build our next new store in Plainsville

반박 그동안의 경험에 의한 과거 통계가 꼭 여기에도 적용되는건 아니다 5 년전 헬스 클럽이 잘 안되었던게 다른 원인이였을수 있다(강사수준 미달 강좌미비)tourist 에 의한 원인 일수 있다 어렸을때부터 운동을 했다고 해서 커서도 관심이 있지는 않다 (오히려 반감이 있을수 있다 혹은 건강하기에 건강에 관심이 적을수도 있다)운동복이나 신발의 판매가 육체노동에 의한 것일수도 있다

IntroductionSupport1049896In this memorandum the author asserts that Naturersquos Way should build its next newstore in Plainsville To support this assertion the author states that Plainsvillesmerchantsrsquo sales of exercise clothing are going well the local health club has moremembers than ever and a new generation of customers will help to ensure NaturersquosWayrsquos success At first glance the authorrsquos assumption seems convincing but in-depth scrutiny revealsthat it lacks substantial evidence as it stands

Body 1-SamplingTopic Sentence 1To begin with the author assumes that the merchantsrsquo report indicates that the residentsare concerned about their health However this assumption is based on unsubstantiated

GRE AWA John박 박정어학원

data Example 1 (Rebuttal1) First if we do not know the total volume of items sold and the price of the goods exactly we cannot infer whether the residents are actually buying many goods Example 2 (Rebuttal2)In addition to that the report emphasizes the rising sales of running shoes and exerciseclothing however these may not be hot-selling items for Naturersquos Way or may not be theproducts the company is planning to sell Concluding Sentence Therefore in order to make the argument reliable the author should reconsider themerchantsrsquo report with more detailed data

Body 2-CausalTopic Sentence 2Second the author contends that the health clubs classes are full yet this does not meanthat many people actually use the health club other factors may be the real cause forthose closed classes Example 1 (Rebuttal1) To begin with if the health club is very small the number of people working out wouldnot be a large one In fact regular gym-going may just be a vogue among a smallunrepresentative segment of Plainsvilles population Example 2 (Rebuttal2) Moreover it is possible that most of the people who exercise in the health club do weight training and aerobics only to look good and to meet other singles not for their health In that case there would be little demand for health products Concluding SentenceThus the author should not hasten to presume what really caused people to be interested in a healthier lifestyle and enroll in the health club

Body 3-Time-ShiftTopic Sentence 3Finally the author highlights that Naturersquos Way can expect a new generation of customersin Plainsville that will help the company in the long term This notion is mistaken in that itassumes the conditions of the present will continue unchanged in the future Although theschool children are required to participate in the fitness for life program they may notnecessarily buy Naturersquos Ways products Example 1 (Rebuttal1) In the first instance they may suffer a fall in purchasing power arising from future economic difficulties this would cause reluctance to spend a considerable amount of money on health products which tend to be more expensive Example 2 (Rebuttal2)Another possibility is that there may emerge many competitor companies vying with Naturersquos Way so that in the future the school children may not feel the necessity to purchase one companyrsquos health products over anotherrsquosConcluding Sentence Thus the authorrsquos assumption is highly speculative since it relies heavily on unknowablefuture circumstances

ConclusionThesis In sum the author uses many assumptions that are insufficient in supporting his claimsSupportIn order for the authorrsquos claims to be convincing he needs to advance more persuasiveevidence that people in Plainsville really are concerned with their health and health foodThe following was written as a part of an application for a small-business loan by a group of developers in the city of Monroe

A jazz music club in Monroe would be a tremendously profitable enterprise Currently the nearest jazz club is 65 miles away thus the proposed new jazz club in Monroe the C-Note would have the local market all to itself Plus jazz is extremely popular in Monroe over 100000 people attended Monroes annual jazz festival last summer several well-known jazz musicians live in Monroe and the highest-rated radio program in Monroe is Jazz Nightly which airs every weeknight at 7 PM Finally a nationwide study indicates that the typical jazz fan spends close to $1000 per year on jazz entertainment

1 Write a response in which you discuss what specific evidence is needed to evaluate the argument and explain how the evidence would weaken or strengthen the argument

2 Write a response in which you examine the stated andor unstated assumptions of the argument Be

GRE AWA John박 박정어학원

sure to explain how the argument depends on these assumptions and what the implications are for the argument if the assumptions prove unwarranted

3 Write a response in which you discuss what questions would need to be answered in order to decide whether the prediction and the argument on which it is based are reasonable Be sure to explain how the answers to these questions would help to evaluate the prediction

Group error nationwide survey may not reflect local trends Is the nationwide jazz fan population substantialInsufficient non-residents of Monroe may have attended the jazz festival (Body alternative explanation last year may have been an anomaly The author should consider data from various years) The author should indicate how many out of 100000 were Monroe residentsNationwide study Does this reflect Insufficient Citizens of Monroe may continue to go to the jazz club 65 miles away

Are the people in Monroe really interested in jazzMajority of the people who attended the jazz festival might not be Monroe residentsSurvey error nationwide study may not be applicable to MonroeJazz musicians who live in MonroeMonopolyRadio station

In this business application the author claims that the proposed jazz club C Note will be very profitable in Monroe To support this claim the author argues for his case with several evidences At first glance the authorrsquos argument seems convincing however careful scrutiny reveals that his argument in specious

To begin with the author claims that Monroersquos citizens are interested in jazz He presents three evidences First Secondhellip Thirdhellip Howeverhellip

Monroe 시에 있는 재즈 음악 클럽은 수익성이 좋은 사업이다 현재 가장 가까이에 있는 클럽은 65 마일 정도 떨어져 있다 따라서 이번에 세우려고 하는 C Note 는 독보적인 위치를 점할것이다 더군다나 재즈는 이 시에서 가장 인기있는 음악이다 지난 여름 재즈 축제에서는 10 만명 이상의 Morone 시 주민이 참석하였고 몇몇 유명한 재즈 음악가들도 이곳에 살고 있으며 저녁때 방영되는 라디오 프로그램중에서 최고의 시청률을 보이고 있는 것도 Jazz Nightly 이다 전국조사에서도 전형적인 재즈 팬들은 재즈 분야에 년간 1천 달러 가까이 지출하고 있는 것으로 보고되고 있다 따라서 C Note 클럽이 돈을 벌 수 있는 사업이라는 것은 확실한 것이다

결과 It is clear that the C Note cannot help but make money반박 nearest jazz club 이 양질의 써비스로 여전히 손님을 끌수도 있다Festival 에 얼마나 참여하는지가 jazz 의 인기를 반영하지 않는다 뮤지션이 많이 사는거랑 jazz 의 인기가 상관없다라디오 프로그램이 다른 요인에 의해서 인기일수도 있다 (진행자때문)전국 통계 적용 불가화목 실전반_Ms Noh6In this application the author suggests that a jazz club in Monroe will make a number of profits To support this suggestion the author exemplifies the local condition popularity of jazz in Monroe and nationwide study However careful scrutiny of each of the facts reveals that it provides little credible support for the authorrsquos recommendation Good clear intro

First the author assumes that jazz is popular in Monroe because of several facts the jazz festival last year had high participation some famous jazz musicians live in Monroe and the high-rated radio program is lsquoJazz Nightlyrsquo However this assumption has many drawbacks that must be seriously considered(Good topic sentences) If many attendants in the last-yearrsquos festival came from other cities and not Monroe it is hard to conclude that Monroersquos people like jazz Therefore the author must examine how many Monroe residents actually attended the festival On top of that there is little relationship between habitation of famous jazz musician and the popularity of jazz in Monroe Although several well-known musicians live there if they do not take part in any jazz performance of Monroe this might have no effect to the interest of Monroersquos residents

GRE AWA John박 박정어학원

about jazz Finally in the case of radio program this is also not suitable reason why jazz is popular in Monroe It might be possible that people cannot help choosing lsquoJazz Nightlyrsquo because there are few radio programs at Night The fact that the radio program is the highest rating program is not a germane evidence The approximate number of listeners would be the more crucial evidence Therefore the author needs to seriously deliberate the correlation between jazzrsquos popularity in Monroe and his examples (Good logical flow and clarity)

Second the author uses as evidence the nationwide study that jazz fans spend much money on jazz entertainment to substantiate why starting a jazz club in Monroe will be profitable In other words the author assumes that the characteristics of a nationwide study can be applied to Monroe The national study would lend support to the applicantrsquos claim only if residents in Monroe typify national jazz fans However the author does not provide credible evidence that this is the case Moreover the populations of jazz fans nationwide may be insubstantial Thus the author should not infer hastily that Monroersquos residents will spend much money on enjoying jazz from the nationwide study

Lastly even if jazz is popular in Monroe C Note may not be successful It is entirely possible that residents might still prefer other clubs where they have always went In addition there is another possibility that the nearest jazz club will attract many of Monroersquos people because it serves fine performances and is equipped with favorite facilities Without considering these other possibilities the author cannot make his argument convincing In sum the author presents many reasons that are insufficient in supporting his or her claim In order for the authorrsquos claims to be convincing he needs to advance more persuasive evidence such as the total number of Monroe residents who attended the jazz festival the effects on the popularity of jazz by the musicians living in Monroe and the actual number of residents who would typify themselves to be jazz fans through a local survey Without substantial evidence that C Note will be successful in Monroe the businessmen may be overinvesting in what might lead to a business failureExcellent clarity Score 50

The following appeared in a newsletter offering advice to investors

Over 80 percent of the respondents to a recent survey indicated a desire to reduce their intake of foods containing fats and cholesterol and today low-fat products abound in many food stores Since many of the food products currently marketed by Old Dairy Industries are high in fat and cholesterol the companys sales are likely to diminish greatly and company profits will no doubt decrease We therefore advise Old Dairy stockholders to sell their shares and other investors not to purchase stock in this company

Write a response in which you discuss what questions would need to be answered in order to decide whether the advice and the argument on which it is based are reasonable Be sure to explain how the answers to these questions would help to evaluate the advice

Survey 80

GRE AWA John박 박정어학원

Old Dairy could change their products and manufacture low fat dairy foodsLess competing companies Old Dairy could eventually be the only company that produces hellipImprecise numbers and measurementsCustomers may still buy high fat dairy products

The author of the newsletter is offering potentially dangerous advice by recommending Old Dairy stockholders to withdraw investment and stop purchase What is more the authorrsquos prediction debases the reputation and business of Old Dairy and if false could devoid the investment opportunity of the newsletter readers Therefore investors should examine whether the authorrsquos evidences are substantial

To begin with the author states that 80 percent of the respondents in a survey indicated a desire to reduce their intake of foods He therefore argues that Old Dairyrsquos high fat and cholesterol products would decrease in sales However the author makes a crucial error in this argument First the author provides no evidence that the surveyrsquos results are statistically reliable Were they representative of all the customers Were they chosen for the survey randomly Furthermore the desire to reduce fat and cholesterol intake is a pervasive trend in todayrsquos opulent society however the author erroneously identifies this as a new phenomenon which will affect consumer trends Second having a desire to reduce fat and cholesterol intake does not necessarily indicate that people who have this desire will actually reduce consuming these types of products It is entirely possible that they may continue buying Old Dairy products for its quality and taste Accordingly the author cannot draw any firm conclusion that people will not buy Old Dairy products Therefore if any of these cases are true the author may be offering investors a detrimental investment advice

최근 조사에 대한 응답자중 80 이상이 자신이 먹는 음식에서 지방과 콜레스테롤의 함유량을 줄이고 싶다고 한다 아울러 요즘은 많은 식료품 가계에서 저지방 제품들을 많이 취급하고 있다 현재 Old Dairy Industries가 판매하고 있는 많은 음식제품들은 지방과 콜레스테롤이 높기 때문에 이 회사의 매출이 격감할 것으로 보이며 당연히 매출이익도 줄어들것이다 따라서 이 회사의 주주들은 주식을 매각하고 다른 주식 투자가들도 이 회사의 주식을 매입하지 않는 것이 좋다

결론 Old Dairy stockholders to sell their shares and other investors not to purchase stock in this company

반박 모든 상품이 다 고 지방 고 칼로리는 아니다(비록 많을지라도) 일부의 식품의 경우 기호에 맞어서 히트해서 전체적인 수입이 증가할 수도 있다국내시장만 생각할 수 없다( 외국시장에서 호황을 누릴수 있다 )입맛이라는게 즉각 바뀌는게 아니다

The following appeared in a letter to the editor of the Balmer Island Gazette

On Balmer Island where mopeds serve as a popular form of transportation the population increases to 100000 during the summer months To reduce the number of accidents involving mopeds and pedestrians the town council of Balmer Island should limit the number of mopeds rented by the islands moped rental companies from 50 per day to 25 per day during the summer season By limiting the number of rentals the town council will attain the 50 percent annual reduction in moped accidents that was achieved last year on the neighboring island of Seaville when Seavilles town council enforced similar limits on moped rentals

1 Write a response in which you discuss what questions would need to be answered in order to decide whether the recommendation is likely to have the predicted result Be sure to explain how the answers to these questions would help to evaluate the recommendation

2 Write a response in which you discuss what questions would need to be answered in order to decide whether the prediction and the argument on which it is based are reasonable Be sure to explain how the answers to these questions would help to evaluate the prediction

3 Write a response in which you examine the stated andor unstated assumptions of the argument Be sure to explain how the argument depends on these assumptions and what the implications are for the argument if the assumptions prove unwarranted

Whatrsquos the actual population of Balmer Island 100000mdashis this a significant increase What kind of accidents Skin abrasions or serious injury And compared to Seaville how serious are the accidents and the actual number of accidents Did Seaville enforce other restrictions like safety signsHow different are the conditions of Balmer

GRE AWA John박 박정어학원

and Seaville regarding population road (safety) conditions topography other town-government regulation How much will the economy of Balmer be affected do to this restriction Could it cause an economic recession due to the fact that these rental companiesrsquo chance to make money is only during the summer thereby weakening the economic infrastructure Are there any other ways that could better alleviate the accident rate

Statistics 50-impreciseAnalogy Balmer compared with TorseauFalse Cause Accidents might have occurred because of reasons other than mopeds False Cause population increase may not be part of the cause of the accidentsOther explanations for the accident pedestrians few road safety regulations narrow roadsThere could be other better solutionshellip

Balmer Island의 인구가 여름철에는 십만명으로 늘어난다 2륜차와 보행자간 사고를 줄이기 위해 시의회는 6개의 자전거를 포함한 2륜차 대여업체에게 이 기간동안에는 대여숫자를 일일 50에서 30으로 제한하도록 할 것이다 대여숫자를 줄임으로써 시 의회는 지난해 이웃한 Torseau섬에서 이와 동일한 규제를 시행해서 50나 줄인 결과를 보고 마찬가지로 50를 줄일수 있다고 확신하고 있다

결론 The town council of Balmer Island should linit the number

반박 보행자의 부실에 의해서 사고가 많이 일어날수도 있다렌탈수의 줄임만이 대책은 아니다(대부분의 사람들이 렌탈 보다는 소유하고 있을 수도 있다)옆섬과는 상황이 다를수도 있다(그 섬에서는 사고의 원인이 많은 자전거 수로 인한것일수있다) 하지만 이 섬은 좁은 도로가 원인일 수도 있고 도로 안전 장치의미비가 원일일수 있다

In this letter the author recommends that Balmer Island should limit the number moped rentals from 50 to 30 per day To support this recommendation the author points out several reasons However careful scrutiny of each of the facts reveals that it is filled with unanswered questions that could significantly weaken the authorrsquos recommendation with loops and holes which are answered

The recommendation depends on the assumption that no alternative means of reducing the number of accidents are available However the author fails to offer any evidence to substantiate this crucial assumption It is highly possible that means other than this would better solve the problem Perhaps they could widen the roads or put-up more safety signs Or perhaps the accidents were due to the lack of skills in which case proper safety training would significantly alleviate the problem Without considering and ruling out these and other alternative means of reducing accidetns the author cannot confidently conclude that merely emulating Torseau would suffice Moreover the author is advising a recommendation which could potentially harm the economy of Balmer Island sincehellip Moreover the Balmer Island should alternative means to reduce accidents because limiting moped rentals during the summer could harm the economy of Balmerhellip

First of all the author believes that increase in population and the number of moped rentals are responsible for the accidents It is entirely possible that other factors are responsible for the accidents Perhaps Balmer Islandrsquos lack of safety signs was a major factor Or maybe the roads are narrow and dangerous on the Island therefore the town council could enforce stricter traffic regulations to alleviate the problem Accordingly if either of these scenarios is true the author cannot draw any firm conclusion that increase in the number of population and moped rentals are the cause of the accidents

The author of this editorial recommends that to reduce accidents involving mopeds andpedestrians Balmer Islands city council should restrict moped rentals to 30 per day down from50 at each of the islands six rental outlets To support this recommendation the author citesthe fact that last year when nearby Torseau Islands town council enforced similar measuresTorseaus rate of moped accidents fell by 50 For several reasons this evidence providesscant support for the authors recommendationTo begin with the author assumes that all other conditions in Balmer that might affect therate of moped-pedestrian accidents will remain unchanged after the restrictions are enactedHowever with a restricted supply of rental mopeds people in Balmer might purchase mopedsinstead Also the number of pedestrians might increase in the future with more pedestriansespecially tourists the risk of moped-pedestrian accidents would probably increase For thatmatter the number of rental outlets might increase to make up for the artificial supplyrestriction per outlet--a likely scenario assuming moped rental demand does not declineWithout considering and ruling out these and other possible changes that might contribute to ahigh incidence of moped-pedestrian accidents the author cannot convince me that theproposed restrictions will necessarily have the desired effect

GRE AWA John박 박정어학원

Next the author fails to consider other possible explanations for the 50 decline inTorseaus moped accident rate last year Perhaps last year Torseau experienced unusually fairweather during which moped accidents are less likely Perhaps fewer tourists visited Tot seanlast year than during most years thereby diminishing the demand for rental mopeds to belowthe allowed limits Perhaps last year some of Torseaus moped rental outlets purchased newmopeds that are safer to drive Or perhaps the restrictions were already in effect but were notenforced until last year In any event a decline in Torseaus moped accident rate during onlyone year is scarcely sufficient to draw any reliable conclusions about what might have causedthe decline or about what the accident rate will be in years aheadAdditionally in asserting that the same phenomenon that caused a 50 decline in mopedaccidents in Torseau would cause a similar decline in Balmer the author relies on what mightamount to an unfair analogy between Balmer and Torseau Perhaps Balmers ability to enforcemoped-rental restrictions does not meet Torseaus ability if not then the mere enactment ofsimilar restrictions in Balmer is no guarantee of a similar result Or perhaps the demand formopeds in Torseau is always greater than in Balmer Specifically if fewer than all availablemopeds are currently rented per day from the average Balmer outlet while in Torseau everyavailable moped is rented each day then the proposed restriction is likely to have less impacton the accident rate in Balmer than in TorseauFinally the author provides no evidence that the same restrictions that served to reduce theincidence of all moped accidents by 50 would also serve to reduce the incidence ofaccidents involving mopeds and pedestrians by 50 Lacking such evidence it is entirelypossible that the number of moped accidents not involving pedestrians decreased by a greaterpercentage while the number of moped-pedestrian accidents decreased by a smallerpercentage or even increased Since the author has not accounted for these possibilities theeditorials recommendation cannot be taken seriouslyIn conclusion the recommendation is not well supported To convince me that the proposedrestriction would achieve the desired outcome the author would have to assure me that nochanges serving to increase Balmers moped-pedestrian accident rate will occur in theforeseeable future The author must also provide dear evidence that last years decline inmoped accidents in Torseau was attributable primarily to its moped rental restrictions ratherthan to one or more other factors In order to better evaluate the recommendation I wouldneed more information comparing the supply of and demand for moped rentals on the twoislands I would also need to know the rate of mopedpedestrian accidents in Torseau both priorto and after the restrictions were enforced in TorseauThe following appeared in a magazine article about planning for retirement

Clearview should be a top choice for anyone seeking a place to retire because it has spectacular natural beauty and a consistent climate Another advantage is that housing costs in Clearview have fallen significantly during the past year and taxes remain lower than those in neighboring towns Moreover Clearviews mayor promises many new programs to improve schools streets and public services And best of all retirees in Clearview can also expect excellent health care as they grow older since the number of physicians in the area is far greater than the national average

Write a response in which you discuss what specific evidence is needed to evaluate the argument and explain how the evidence would weaken or strengthen the argument

-Natural beauty and consistent climate may not be the most wanted qualities-Housing costs could have lowered on a national level wealthy retirees may not care about costs-Taxes may be high compared to the nationrsquos average tax rate-What about other qualities of Clearview Crime rate what qualities would retirees want -If schools streets and public services need improvement then this is proof that the current condition of Clearview is low Or due to budgetary reasons the mayor may not follow-up on his promise because of lowered tax rate -Schools and people who are retired no relationship-Physicians What kind of physicians Number is irrelevant Are these physicians capable of addressing the illnesses of old people

This author argues that anyone seeking a place to retire should choose Clearview To supportthis argument the article cites Clearviews consistent climate and natural beauty its fallinghousing costs its low property taxes compared to nearby towns and the mayors promise toimprove schools streets and services The article also claims that retirees can expectexcellent health care because the number of physicians in Clearview greatly exceeds thenational average This argument is flawed in several critical respectsTo begin with although consistent climate and natural beauty might be attractive to manyretirees these features are probably not important to all retirees For many retirees it isprobably more important to live near relatives or even to enjoy changing seasons Thus I

GRE AWA John박 박정어학원

cannot accept the authors sweeping recommendation for all retirees on this basisAlso Clearviews declining housing costs do not necessarily make Clearview the best placeto retire for two reasons First despite the decline Clearviews housing costs might be highcompared to housing costs in other cities Secondly for wealthier retirees housing costs arenot likely to be a factor in choosing a place to retire Thus the mere fact that housing costshave been in decline lends scant support to the recommendationThe articles reliance on Clearviews property-tax rates is also problematic in two respectsFirst retirees obviously have innumerable choices about where to retire besides Clear viewand nearby towns Secondly for retirees who are well-off financially property taxes are notlikely to be an important concern in choosing a place to retire Thus it is unfair to infer fromClearviews property-tax rates that retirees would prefer ClearviewYet another problem with the argument involves the mayors promises In light of Clearviewslow property-tax rates whether the mayor can follow through on those promises is highlyquestionable Absent any explanation of how the city can spend more money in the areas citedwithout raising property taxes I simply cannot accept the editorials recommendation on thebasis of those promises Besides even if the city makes the improvements promised thoseimprovements--particular the ones to schools--would not necessarily be important to retireesFinally although the number of physicians in Clearview is relatively high the per capitanumber might be relatively low Moreover it would be fairer to compare this per capita numberwith the per capita number for other attractive retirement towns--rather than the nationalaverage After all retirees are likely to place a relatively heavy burden on health-careresources Besides the article provides no assurances that the number of physicians inClearview will remain high in the foreseeable futureIn conclusion the recommendation is poorly supported To strengthen it the author mustconvince me--perhaps by way of a reliable survey--that the key features that the vast majorityof retirees look for in choosing a place to live are consistent climate natural beauty and lowhousing costs The author must also provide better evidence that Clear views property taxesare lower than the those of cities in other areas The author must also explain how the city canmake its promised improvements without raising property taxes Finally to better assess theargument I would need to now how the per capita number of physicians in Clearview wouldcompare to the national average in the futureThe following appeared as a letter to the editor from a Central Plaza store owner

Over the past two years the number of shoppers in Central Plaza has been steadily decreasing while the popularity of skateboarding has increased dramatically Many Central Plaza store owners believe that the decrease in their business is due to the number of skateboard users in the plaza There has also been a dramatic increase in the amount of litter and vandalism throughout the plaza Thus we recommend that the city prohibit skateboarding in Central Plaza If skateboarding is prohibited here we predict that business in Central Plaza will return to its previously high levels

Write a response in which you discuss what questions would need to be answered in order to decide whether the recommendation is likely to have the predicted result Be sure to explain how the answers to these questions would help to evaluate the recommendation

Why two years ago What happened two years ago which started this declineIs the dramatic increase in the ldquopopularityrdquo of skateboarding the cause of the steady decline of shoppers Are there any malls nearby Were there any changes nearby which could affect the decline in customersmdasha big mall perhaps Could the decline be due to the shop ownersHow many skateboarders use the plazaWhere do they skateboardDo they shop and are they customersAre the increase in litter and vandalism due to skateboarders Could this be alleviated by installing CCTVs and hiring security

This editorial concludes that the city should ban skateboarding from its downtown CentralPlaza in order to attract visitors to that area to return the area to its former glory and to makeit a place where people can congregate for fun and relaxation To justify this conclusion theeditorial points out that skateboarders are nearly the only people one sees anymore at CentralPlaza and that the Plaza is littered and its property defaced The editorial also points out thatthe majority of downtown merchants support the skate boarding ban This argument is flawedin several critical respectsFirst the editorials author falsely assumes that a ban on skateboarding is both necessaryand sufficient to achieve the three stated objectives Perhaps the city can achieve thoseobjectives by other means as well--for example by creating a new mall that incorporates anattractive new skateboard park Even if banning skateboarders altogether is necessary to meetthe citys goals the author has not shown that this action by itself would suffice Assuming thatthe Plazas reputation is now tarnished restoring that reputation and in turn enticing peopleback to the Plaza might require additional measures--such as removing litter and graffiti

GRE AWA John박 박정어학원

promoting the Plaza to the public or enticing popular restaurant or retail chains to the PlazaSecondly the editorial assumes too hastily that the Plazas decline is attributable to theskateboarders--rather than to some other phenomenon Perhaps the Plazas primary appeal inits glory days had to do with particular shops or eateries which were eventually replaced byless appealing ones Or perhaps the crime rate in surrounding areas has risen dramatically forreasons unrelated to the skateboarders presence at the Plaza Without ruling out these andother alternative explanations for the Plazas decline the editorials author cannot convince methat a skateboard ban would reverse that declineThirdly the editorials author might be confusing cause with effect--by assuming that theskateboarders caused the abandonment of the Plaza rather than vice versa It is entirelypossible that skateboarders did not frequent the Plaza until it was largely abandoned--andbecause it had been abandoned In fact this scenario makes good sense since skateboardingis most enjoyable where there are few pedestrians or motorists to get in the wayFourth it is unreasonable to infer from the mere fact that most merchants favor the ban thatthe ban would be effective in achieving the citys objectives Admittedly perhaps thesemerchants would be more likely to help dean up the Plaza area and promote their businesseswere the city to act in accordance with their preference Yet lacking any supporting evidencethe author cannot convince me of this Thus the survey amounts to scant evidence at best thatthe proposed ban would carry the intended resultFinally the author recommends a course of action that might actually defeat the citysobjective of providing a fun and relaxing place for people to congregate In my experienceskateboarding contributes to an atmosphere of fun and relaxation for adults and children alikemore so than many other types of ambiance Without considering that continuing to allowskateboarding--or even encouraging this activity--might achieve the citys goal more effectivelythan banning the activity the author cannot convincingly conclude that the ban would be in thecitys best interestsIn sum the argument is a specious one To strengthen it the editorials author must providedear evidence that skateboarding and not some other factor is responsible for the conditionsmarking the Plazas decline The author must also convince me that no alternative means ofrestoring the Plaza are available to the city and that the proposed ban by itself would suffice toattract tourists and restore the Plaza to its former glory Finally to better assess the argument itwould be useful to know the circumstances under which the downtown merchants would bewilling to help the city achieve its objectives

6그룹 약한 비유 빈출

The following recommendation appeared in a memo from the mayor of the town of Hopewell

Two years ago the nearby town of Ocean View built a new municipal golf course and resort hotel During the past two years tourism in Ocean View has increased new businesses have opened there and Ocean Views tax revenues have risen by 30 percent Therefore the best way to improve Hopewells economymdashand generate additional tax revenuesmdashis to build a golf course and resort hotel similar to those in Ocean View

Write a response in which you examine the stated andor unstated assumptions of the argument Be sure to explain how the argument depends on these assumptions and what the implications are for the argument if the assumptions prove unwarranted

GRE AWA John박 박정어학원

Assumptions The author assumes that OVrsquos municipal golf course and resort hotel caused tourism new businesses and increased tax revenues There may be other reasons advertising promo He assumes that this will continueAssumes that Ocean View and Hopewell are similar in many waysmdashthe name suggests otherwise OV may have always been a tourist attractions for its beaches We need to know the topography

2년전 Ocean View 시는 시정 소유 골프 및 휴양지 호텔을 신축했다 그리고 지난 2년동안 이 시의 관광객이 증가했으며 새로운 사업들이 생겨났다 그에따라 시의 세수도 30나 증가했다 Hopewell의 경제를 향상시키고 아울러 세수를 늘릴 수 있는 가장 좋은 방법은 Ocean View에 세워진 것과 같은 골프 시설과 휴양지 호텔을 신축하는 것이다

1 다른 요인으로 관광 산업이 발전했을 수도 있다 문화 유적이 발견이 되었거나 도로의 정비등으로 여행자가 늘었을 수도 있다

2 관광 산업의증가가 늘어난 세수의 원인이 아니라 새로 유입된 인구의 증가나 다른 공장에서 발생한 것일 수 있다

3 2년동안 한참 골프가 붐을 이루었을 수 있다 경제상황이 나빠지거나 다른 레포츠가 인근 지역에 생겨난다면 골프하는 사람이 줄어들 수 있다

In this memo HopeweUs mayor recommends that in order to stimulate the towns economyand boost tax revenues HopeweU should build a new golf course and resort hotel just as thetown of Ocean View did two years ago To support this recommendation the mayor points outthat in Ocean View during the last two years tourism has increased new businesses haveopened and tax revenues have increased by 30 I find the mayors argument unconvincingin several important respectsFirst of all it is possible that the mayor has confused cause with effect respecting the recentdevelopments in Ocean View Perhaps Ocean Views construction of a new golf course andhotel was a response to previous increases in tourism and business development increasesthat have simply continued during the most recent two years Since the mayor has failed toaccount for this possibility the claim that Hopewell would boost its economy by alsoconstructing a golf course and hotel is completely unwarrantedSecondly the mayor fails to account for other possible causes of the trends in Ocean Viewduring the last two years The increase in tourism might have been due to improving economicconditions nationwide or to unusually pleasant weather in the region The new businessesthat have opened in Ocean View might have opened there irrespective of the new golf courseand hotel And the 30 increase in tax revenues might have been the result of an increase intax rates or the addition of a new type of municipal taxWithout ruling out these and other alternative explanations for the three recent trends inOcean View the mayor cannot reasonably infer based on those trends that Hopewellseconomy would benefit by following Ocean Views exampleThirdly even if the recent trends in Ocean View are attributable to the construction of the newgolf course and hotel there the mayor assumes too hastily that the golf course and hotel willcontinue to benefit that towns overall economy The mayor has not accounted for thepossibility that increased tourism will begin to drive residents away during tourist season orthat new business development will result in the towns losing its appeal as a place to visit or tolive Unless the mayor can convince me that these scenarios are unlikely I cannot accept themayors recommendation that Hopewell follow Ocean Views exampleFinally the mayors argument rests on the unsubstantiated assumption that Hopewell andOcean View are sufficiently alike in ways that might affect the economic impact of a new golfcourse and hotel Hopewell might lack the sort of natural environment that would attract moretourists and new businesses to the town--regardless of its new golf course and hotel For thatmatter perhaps Hopewell already contains several resort hotels and golf courses that are notutilized to their capacity If so building yet another golf course and hotel might amount to amisallocation of the towns resources--and actually harm the towns overall economyIn sum the mayors recommendation is not well supported To bolster it the mayor mustprovide better evidence that Ocean Views new golf course and hotel and not some otherphenomenon--has been responsible for boosting Ocean Views economy during the last twoyears To better assess the recommendation I would need to know why Ocean View decidedto construct its new golf course and hotel in the first place--specifically what events prior toconstruction might have prompted that decision I would also need to thoroughly compare

GRE AWA John박 박정어학원

HopeweU with Ocean View--especially in terms of their appeal to tourists and businesses--todetermine whether the same course of action that appears to have boosted Ocean Viewseconomy would also boost Hopewells economy

The following is part of a memorandum from the president of Humana University

Last year the number of students who enrolled in online degree programs offered by nearby Omni University increased by 50 percent During the same year Omni showed a significant decrease from prior years in expenditures for dormitory and classroom space most likely because instruction in the online programs takes place via the Internet In contrast over the past three years enrollment at Humana University has failed to grow and the cost of maintaining buildings has increased along with our budget deficit To address these problems Humana University will begin immediately to create and actively promote online degree programs like those at Omni We predict that instituting these online degree programs will help Humana both increase its total enrollment and solve its budget problems

Write a response in which you discuss what questions would need to be answered in order to decide whether the prediction and the argument on which it is based are reasonable Be sure to explain how the answers to these questions would help to evaluate the prediction

Is Omni University successful due to the online degree program 50 Is the decrease in expenditures for dormitory and classroom space due to the decrease in of on-campus students Which classes were successful Does HU have those classes

Even if the long-distance degree programs at Omni University benefited the school the presidentrsquos recommendation that Human College should emulate Omni University is too hasty First OUrsquos name implies that the school would have more majors than Humanahellip the president should examine which degrees were in the long-distance programhellip

지난해에는 Omni 대학에서 개강했던 원거리 학생 학점 취득 프로그램을 등록했던 학생들의 숫자가 50나 증가했다 같은해 기간동안 Omni 대학에서는 그 전년도부터 기숙사와 학급의 공간 확충을 위한 예산을 대폭 줄였는데 이는 이 원거리 학점 취득 프로그램이 양방향 비디오 컴퓨터 접속을 통해서만 가능한 수업지도 방식이기때문인 것으로 보인다 반면 지난 3개년 동안 Humana 대학에서의 수강률은 감소한데다가 건물

GRE AWA John박 박정어학원

유지비도 올랐다 따라서 Humana대학의 수강을 늘리고 예산손실을 회복하기 위해서는 Omni 대학에서 취한 조치와 같은 능동적인 프로그램을 추진해야 한다

결론 we should initiate and actively promote long-distance degree programs like those at Omni 반박 원거리 학생 취득 프로그램 숫자가 증가한거하고 예산이 줄어드는 것 사이에 연관이 약하다 (causal 학생의 증가로 관리비용 증가할수 있음 원거리 수업가능 장비도입에의한 비용발생)bad analogy(omni university 하고 같은 조건이 아니다 )-gt omni college 가 강좌내용이 좋아서 학생의 등록이 많을수 있다 Humana 대학에서 만들었다 하더라도 인기 없을수 있음다른 요인에 의해서 Humana 대학의 수강 인원이 증가할수 있음(비록 과거엔 인기가 없었을지라도)

The following appeared as part of a business plan developed by the manager of the Rialto Movie Theater

Despite its downtown location the Rialto Movie Theater a local institution for five decades must make big changes or close its doors forever It should follow the example of the new Apex Theater in the mall outside of town When the Apex opened last year it featured a video arcade plush carpeting and seats and a state-of-the-art sound system Furthermore in a recent survey over 85 percent of respondents reported that the high price of newly released movies prevents them from going to the movies more than five times per year Thus if the Rialto intends to hold on to its share of a decreasing pool of moviegoers it must offer the same features as Apex

Write a response in which you discuss what questions would need to be answered in order to decide whether the recommendation is likely to have the predicted result Be sure to explain how the answers to these questions would help to evaluate the recommendation

Before following through this business plan the manager should investigate the cause of Rialtorsquos unsuccessful business

The author provides no evidence that the surveyrsquos results are statistically reliable The surveyrsquos sample of 85 percent must be sufficient in size and representative of overall population of the city where Rialto and Apex is serving Lacking evidence of a sufficiently representative sample the author cannot justifiably rely on the survey to draw any conclusion whatsoever The author does not indicate that Apex is indeed currently successful However even if Apex is enjoying success the argument relies on what might be a false analogy between Rialto and Apex In order for Apex to serve as a model that Rialto should emulate the author must assume that all relevant circumstances are essentially the same However this assumption is unwarranted For example the argument overlooks the face that Apex is located in a strategic placemdashbeside a mall where customers can not only watch a movie but also enjoy shopping Therefore simply changing the facility to that of Apex may not lead to success

The author does not mention whether Apex is successful or not Nevertheless even if Apex is currently successful the argument relies on what might be a false analogy between Rialto and Apex In order for Apex to serve as a model that Rialto should emulate the author must assume that all relevant circumstances are essentially the same However this assumption is unwarranted For example the argument overlooks the fact that these two institutions are located in different locations Rialto in downtown and Apex in a mall outside of town Although Apex opened with state-of-the-art facilities the decisive factor in its success could be due to its strategic location of being in a mall People could enjoy both shopping and movies at one location thus they may prefer Apex over Rialto Furthermore the place where people enjoy leisure activities has shifted in the past decades for most cities from downtown to the suburbs Therefore Rialto may not be successful even if it emulates Apexrsquos facilities A better business plan may be relocating Apex to the thriving section of the downtown

Rialto 극장은 지난 50여년간 지역 회관으로써 시내에 위치해 있으면서도 이제 변화를 꾀하지 않으면 문을 닫을

GRE AWA John박 박정어학원

판이다 이 극장은 시외 쇼핑타운에 새로 들어선 Apex 극장의 사례를 본받아야 했다 Apex가 지난해 개업했을 당시 이 극장은 비디오 아케이드 플러쉬 카펫트 바닥과 좌석 그리고 최신 음향시설을 갖추었다 더군다나 최근 조사에서는 응답자의 85 이상이 새로 출시된 영화 입장료가 비싼 탓으로 지난해보다 5배이상의 관람객이 줄어들었다고 나타났다 따라서 Rialto 극장이 줄어들고 있는 관람객을 뺐기지 않고 유지하려면 Apex와 같은 시설들을 갖추어야 할 것이다주장 리알토 극장이 줄어들고 있는 관람객을 뺐기지 않고 유지하려면 Apex와 같은 시설들을 갖추어야 할 것이다

1 조사에서 응답자가 전체를 대표할 수 없다 2 apex 극장이 좋은 시설을 갖추고 있지만 그로 인해 수익이 많이 발생했다는 말이 없으므로 시설투자를

하고도 좋은 결과를 얻을 수 있을지 그 근거가 미흡하다3 좋은 영화가 출시된다면 입장료가 비싸도 영화관에서 꼭 보려고 할 수 있다 4 rialto 가 시설이 아닌 다른 요인에 의해 장사가 안될수도 있다( 우범 지역이라든지)

The following is a recommendation from the business manager of Monarch Books

Since its opening in Collegeville twenty years ago Monarch Books has developed a large customer base due to its reader-friendly atmosphere and wide selection of books on all subjects Last month Book and Bean a combination bookstore and coffee shop announced its intention to open a Collegeville store Monarch Books should open its own in-store cafeacute in the space currently devoted to childrens books Given recent national census data indicating a significant decline in the percentage of the population under age ten sales of childrens books are likely to decline By replacing its childrens books section with a cafeacute Monarch Books can increase profits and ward off competition from Book and Bean

Write a response in which you examine the stated andor unstated assumptions of the argument Be sure to explain how the argument depends on these assumptions and what the implications are for the argument if the assumptions prove unwarranted

The following is a recommendation from the business manager of Monarch Books

Since its opening in Collegeville twenty years ago Monarch Books has developed a large customer base due to its reader-friendly atmosphere and wide selection of books on all subjects Last month Book and Bean a combination bookstore and coffee shop announced its intention to open a Collegeville store Monarch Books should open its own in-store cafeacute in the space currently devoted to childrens books Given recent national census data indicating a significant decline in the percentage of the population under age ten sales of childrens books are likely to decline By replacing its childrens books section with a cafeacute Monarch Books can increase profits and ward off competition from Book and Bean

1 Write a response in which you discuss what questions would need to be answered in order to decide whether the recommendation is likely to have the predicted result Be sure to explain how the answers to these questions would help to evaluate the recommendation

2 Write a response in which you discuss what specific evidence is needed to evaluate the argument and explain how the evidence would weaken or strengthen the argument

No evidence regarding Monarch Bookrsquos successEven if Regal Bookrsquos is successful this may not be attributable to the cafeacute False analogy Emulating may not lead to success Other factors may be involvedInsufficient condition The national census is not enough evidence that childrenrsquos book sales will decline Can

GRE AWA John박 박정어학원

the national census represent the local child populationDid opening a cafeacute boost sales for Regal Books Even assuming Regal is successful by opening a cafeacute this may not be suitable for Monarch which plans to close the childrenrsquos book section to establish a cafe Imprecise language ldquorelatively little spacerdquo how smallThe managerrsquos recommendation contradicts what he says Since Monarch is popular for its wide selection of books closing a selection which targets a major group of readers may hurt Monarchrsquos salesIs this the best way to compete

When Stanley Park first opened it was the largest most heavily used public park in town It is still the largest park but it is no longer heavily used Video cameras mounted in the parks parking lots last month revealed the parks drop in popularity the recordings showed an average of only 50 cars per day In contrast tiny Carlton Park in the heart of the business district is visited by more than 150 people on a typical weekday An obvious difference is that Carlton Park unlike Stanley Park provides ample seating Thus if Stanley Park is ever to be as popular with our citizens as Carlton Park the town will obviously need to provide more benches thereby converting some of the unused open areas into spaces suitable for socializing

Write a response in which you examine the stated andor unstated assumptions of the argument Be sure to explain how the argument depends on these assumptions and what the implications are for the argument if the assumptions prove unwarranted

Stanley 파크가 처음 개장했을 당시 가장 크고 가장 많이 이용되는 공원이었다 아직도 공원중에서는 가장 크지만 이용률은 상당히 떨어졌다 지난달 공원 주차장에 설치해놓은 비디오 카메라를 통해 보면 drop(주차장으로 여겨짐) 이용률이 가장 높았다 수치상으로는 하루 평균 50대의 차량만이 이용하였다 반면 직장 중심거리에 위치한 작은 규모의 Carlton 파크는 주당 무려 150여명 이상이 이용하고 있다 Stanley 파크와는 달리 Carlton 파크에는 의자가 있다는 것이 가장 뚜렷한 차이점이다 따라서 Stanley 파크가 Carlton 파크처럼 시민들이 자주 이용하는 공원이 되기 위해서는 벤치를 설치할 필요가 있으며 이렇게 사용되지 않는 일부 공간을 활용해서 사교를 위한 공간으로 바꾸어야 한다 ===gtdrop 에 대한 첨부사항 (영영사전내용입니다)---- a place or central depository to which something (as mail money or stolen property) is brought for distribution or transmission also the act of depositing something at such a place dropgt

주장 if Stanley Park is ever to be as popular with our citizens as is Carlton Park the town will obviously need to provide more benches thereby converting some of the unused open areas into spaces suitable for socializing1 조사가 언제 이루어진 것인가 조사가 언제 실시되었느냐에 따라 결과가 다를 수있다 현재는 다시 스탠리 파크가 늘어났었을 수 있다 2 벤치를 많이 설치했다고 해서 많은 관광객이 오지 않을수 있다(사람들이 벤치나 사교 공간을 원한다는 어떠한 자료도 없다)3스탠리 파크 주변에 교통 상황이 악화가 되었거나 칼튼 파크에서 문화행사등을 많이 가져서 이용객이 줄어든것일 수도 있다 4 칼튼 파크가 중심지에 있어서 접근성이 좋을수 있다5 조사가 같은 시간을 기준으로 한게 아니다(하나는 주중이고 하나는 주말이다)6사람의 수와 차의 대수를 같은것으로 비교할수 없다 (차안에 몇 명이 타고 있는지 모르고 대중교통을 이용해서 왔을수도 있다)

Page 6: GRE writing argument brain storm

GRE AWA John박 박정어학원

A recent study rating 300 male and female Mentian advertising executives according to the average number of hours they sleep per night showed an association between the amount of sleep the executives need and the success of their firms Of the advertising firms studied those whose executives reported needing no more than 6 hours of sleep per night had higher profit margins and faster growth These results suggest that if a business wants to prosper it should hire only people who need less than 6 hours of sleep per night

Write a response in which you examine the stated andor unstated assumptions of the argument Be sure to explain how the argument depends on these assumptions and what the implications are for the argument if the assumptions prove unwarranted

Assumptions 1 Correlation of hours of sleep with success The author assumes that the executives used their wake-up hours on work Could there be other factors 2 Executives versus employeesmdashcriteria for hiring is too simpleminded 3 Characteristics of the 300 male and female executives 4 The study was about advertising executives but the author applies this case to all businesses 5 The author should consider long-term stability rather that sharp growth and profit 6 Average number of hours of sleep is insufficient to support the authorrsquos recommendation 7 The author assumes that this lifestyle pattern will remain consistent

The following appeared in a memo from the president of a company that makes breakfast cereals

In a recent study subjects who ate soybeans at least five times per week had significantly lower cholesterol levels than subjects who ate no soy products By fortifying our Wheat-O cereal with soy protein we can increase sales by appealing to additional consumers who are concerned about their health This new version of Wheat-O should increase company profits and at the same time improve the health of our customers

Write a response in which you examine the stated andor unstated assumptions of the argument Be sure to explain how the argument depends on these assumptions and what the implications are for the argument if the assumptions prove unwarranted

최근의 조사에서 일주에 최소 5회 정도 콩을 먹었던 사람들은 전혀 먹지 않았던 사람들에 비해 콜레스테롤이 상당히 낮은것으로 조사되었다 따라서 Wheat-O 시리얼에 콩 단백질의 함유를 강화시킴으로써 건강에 관심이 있는 더 많은 소비자를 상대로 매출을 올릴 수 있을 것이다 이 신제품이 회사 수입 증대를 가져오는 것 뿐만아니라 소비자의 건강을 향상시킬 수 있다

GRE AWA John박 박정어학원

주장 By increasing our Wheat-O cereal with soy protein we can multiply sales

1 콩 단백질의 함유를 강화해도 그수치가 미흡하거나 흡수가 잘 안되거나 할 수 있다 다른 성분과 섞여서 만들어지는 것이므로 다른 성분에 의해서 그 흡수가 잘 안되거나 효과가 낮을 수 있다 2 강화한 것이 건강에 관심이 있는 소비자에게 어필한다고 해도 그 수가 적어서 매출에 큰 영향을 안줄 수 있다3 다른 음식에 의해서 콜레스테롤이 더 낮아질수 있다 4 콜레스테롤을 낮추는 것이 사람들에게 그들의 건강생활을 위해 크게 어필 안할수도 있다( 저지방식이라든지 다른 건강을 높이는 방법이 더 관심이 많을수 있다)

In a study of the reading habits of Waymarsh citizens conducted by the University of Waymarsh most respondents said that they preferred literary classics as reading material However a second study conducted by the same researchers found that the type of book most frequently checked out of each of the public libraries in Waymarsh was the mystery novel Therefore it can be concluded that the respondents in the first study had misrepresented their reading habits

Write a response in which you discuss what specific evidence is needed to evaluate the argument and explain how the evidence would weaken or strengthen the argument

Evidence needed -Information about the respondentsmdashage gender class precise number of respondents-Time-shift After how many years was the second study conductedmdashtaste in books may change over time-Can library records sufficiently evidence the reading habits of Waymarsh citizens

1)2)3)4)5)6) First the author provides no evidence that the surveyrsquos studyrsquos results are statistically reliable Lacking

information about the precise methodology of the study the number of (customers)respondents surveyed and the number of respondentsvarious information such as gender age and social background which are essential to bolster the conclusion it is impossible to assess the validity of the results It is possible that people who feel inclined to( take low fat and low cholesterol foods ) read literary classics were more willing to respond to the survey than were others Another problem is the representativeness of the respondents Were they representative of all the customersWaymarsh citizens Were they chosen for the survey randomly Lastly the survey results must depend on the honesty and integrity of the respondents Without more information about the survey the author cannot simply conclude that (most of customers want to take low fat and low cholesterol foods)Waymarsh citizens misrepresented their reading habit on the first study on the basis of this surveystudy(survey reliability)

The exact number of books checked-out should be notedMystery novels could have been in trend at the time of the studyPublic libraries vs other libraries such as Leeville University library

Leeville 대학에서 실시한 Leeville 주민의 독서습관에 대한 조사에서 대부분의 응답자는 독서용으로 문학작품을 선호한다고 응답했다 그러나 같은 조사팀에서 이어 실시한 조사에서는 Leeville에 있는 도서관마다 가장 빈번하게 연람된 책의 종류를 보면 미스테리 소설이었던 것으로 조사되었다 따라서 최초 조사 응답자들이 독서 습관에 대해 잘못 말했다고 결론지을 수 있다

결론 it can be concluded that the respondents in the first study had misrepresented their reading habits1 첫번째 조사 그룹하고 두번째 조사 그룹하고 다른 그룹일 수 있다

GRE AWA John박 박정어학원

2 도서관에 비치된 책이 미스터리 소설이 더 많아서 선택의 여지가 없을 수 있다

High Frequency Group 2 National-gtLocal Local-gtNational amp Group Member Error

The following appeared in a letter to the editor of a Batavia newspaper

The department of agriculture in Batavia reports that the number of dairy farms throughout the country is now 25 percent greater than it was 10 years ago During this same time period however the price of milk at the local Excello Food Market has increased from $150 to over $300 per gallon To prevent farmers from continuing to receive excessive profits on an apparently increased supply of milk the Batavia government should begin to regulate retail milk prices Such regulation is necessary to ensure fair prices for consumers

Write a response in which you discuss what questions would need to be answered in order to decide whether the recommendation is likely to have the predicted result Be sure to explain how the answers to these questions would help to evaluate the recommendation

First the author points to the fact that hellip25 increase of dairy farms may be(could be might be) due to population increaseThe milk price increase could be reflecting the increase in cost of livingThe actual price of milk might be cheaper if the economy is experiencing inflationOne market Excello Food cannot reflect the price increase of every market in BataviaThe author claims that farmers are receiving excessive profits however this might not be the caseThe author claims that by regulating milk prices the government will successfully ensure both lower prices and an adequate supply of milk for consumers However this might not be the best solutionThe writer adduces the information released by the department of agriculture that the number of dairy farms has

increased by 25 percent over the last 10 years and thus he claims that dairy farmers are earning excessive profits from milk However the information is filled with loops and holes which need to be filled First what percentage of the milk are sold as milk

Are the dairy farms processing the milk into other dairy products Or are they selling them as milkmdashWhat percentage of the milk is sold as milk Could this be a natural increase

Can the price at the local Excello Food Market represent the price of milk in stores throughout all of Batavia

Are the farmers receiving excessive profits What is the cost of producing milk Could such regulation offer fair prices to consumers

Topic SentenceTo begin with the author assumes that the price of Excello Food Market which has increased from $150 to over $300 per gallon can be applied to the whole nation of Batavia however the author fails to provide evidence to substantiate this assumptionExampleFirstIn additionTherefore in order to make the argument more reliable the author should consider the prices of a large number markets all over the countryThere could be an economic inflation

GRE AWA John박 박정어학원

ldquoBatavia 지역의 농림부의 보고에 따르면 전국적으로 낙농업자의 숫자가 과거 10 년전에 비해 25나 증가했다고 합니다 그러나 동기간 동안 이 지역에 있는 Excello Food Market 에서 우유의 가격은 갤런당 15 불에서 3 불로 증가했습니다 증가된 우유 공급량에 맞춘 낙농업자들의 수익률 증대를 보장하기 위해서는 주 정부가 소매 공급가에 대한 규제를 해야 합니다 소비자에게 보다 저렴한 가격에 안정된 물량을 공급할 수 있도록 하기 위해서는 이러한 조치가 필요합니다

결론 the Batavia government should begin to regulate retail milk prices

반박 ( Excello Food Market doesnrsquot reflect the whole of Batavia) 우유값 증가가 단순히 inflation 을 따라간것일수도 있다 가격상승이 수익을 보장하지 않는다( 원자재 상승등으로 인해서)우유값 규제만이 최선의 방법이 아니다( 물량 조절)낙농업자의 수익보호도 필요하다

A recent sales study indicates that consumption of seafood dishes in Bay City restaurants has increased by 30 percent during the past five years Yet there are no currently operating city restaurants whose specialty is seafood Moreover the majority of families in Bay City are two-income families and a nationwide study has shown that such families eat significantly fewer home-cooked meals than they did a decade ago but at the same time express more concern about healthful eating Therefore the new Captain Seafood restaurant that specializes in seafood should be quite popular and profitable

Write a response in which you discuss what questions would need to be addressed in order to decide whether the conclusion and the argument on which it is based are reasonable Be sure to explain how the answers to the questions would help to evaluate the conclusion

BAY CITY The name implies that this is a port city which would have traditionally consumed seafood 30--what type of seafood Unprocessed or processed cans of tuna and sardines If the latter increased Captain Seafood may not profit 30--natural inflation Are the non-seafood restaurants selling seafood If yes this may suffice since patrons would not easily dine at an unfamiliar restaurant Nationwide studyrsquos representativeness Desire does not lead to action

Nationwide study applied to Bay City Eating fewer home cooked meals than they did a decade agoHealthy food The author needs to show that 30 -gt population increaseOther foods may be more popularCurrent number of restaurants that serve seafood might sufficeWhat kind of healthy food are they interested in Surely not just seafoodThe locals may still patronize the current restaurants that serve seafood dishes

lt national-gt local에 적용 local-gtnational에 적용gt1) The author assumes that the nationwide trend reflects the general trend upon which the argument relies

Yet the author fails to provide evidence to substantiate this crucial assumption The nationwide trend of two income families eating fewer home-cooked and concern for health my not reflect the lifestyle and interests of Bay City citizens Therefore to make his argument stronger the author needs to provide evidence that two income families of Bay City maintains the nationwide trend

GRE AWA John박 박정어학원

최근의 매상에 관한 조사에서 Bay 시에 있는 레스토랑의 해산물 요리의 소비가 지난 5년동안 30 증가했다고 보고되었다 그러나 현재 해산물을 전문으로 취급하고 있는 레스토랑이 없다 더군다나 대다수의 가정이 맞벌이 가정인데다가 전국 조사에서도 나와있듯이 이러한 맞벌이 가정은 10년전의 가정에서 했듯이 집에서 식사를 챙겨먹는 것이 급격하게 줄어 들었고 동시에 건강식과 관련한 지출이 늘고 있다 따라서 해산물 전문 레스토랑이 아주 인기를 끌것이고 그만큼 수익도 많을 것이다

1 해산물이 건강식인지에 대한 언급이 없다2 외식하는데 해산물 요리만 먹지 않을 것이다 집에서 챙겨먹지 않는다고 해산물을 먹는다고 할 수 없다3 현재 해산물 취급하는 식당이 현재까지 없다고 해서 계속 없을 수만은 없다 수익이 만약 늘어난다면

Bay시 주변의 도시의 레스토랑이 체인점을 내거나 새로운 식당이 생길 수 있고 오히려 수익이 줄어들 수도 있다

Scientists studying historical weather patterns have discovered that in the mid-sixth century Earth suddenly became significantly cooler Although few historical records survive from that time some accounts found both in Asia and Europe mention a dimming of the sun and extremely cold temperatures Either a huge volcanic eruption or a large meteorite colliding with Earth could have created a large dust cloud throughout Earths atmosphere that would have been capable of blocking enough sunlight to lower global temperatures significantly A large meteorite collision however would probably create a sudden bright flash of light and no extant historical records of the time mention such a flash Some surviving Asian historical records of the time however mention a loud boom that would be consistent with a volcanic eruption Therefore the cooling was probably caused by a volcanic eruption

Write a response in which you discuss what questions would need to be addressed in order to decide whether the conclusion and the argument on which it is based are reasonable Be sure to explain how the answers to the questions would help to evaluate the conclusion

Historical records may not be enough to explain the global cooling phenomenonThere may be reasons other than the two hypothesis to explain the global coolingA volcanic eruption big enough to produce dust clouds that envelop the earth to cause global cooling would have countless recordsThe author argues that a large meteorite collision is not a feasible explanation for the global cooling because there are no historical records that indicate a flash The authorrsquos logic is flawed in eliminating the meteorite hypothesis by claiming that there was no historical record of a bright flash of light According to common sense a meteorite collision big enough to produce dust that could envelop the earth would result in catastrophe even to the extinction of major species of animals

Loud boom would be insufficient evidence to conclude it was a volcanic eruption If could hear a loud boom there should be records of a volcanic eruption because the author claims that the dust cloud from this gigantic eruption was significant enough to cause global coolingCooling could have been caused by factors besides volcanic eruption and meteor collisionThe absence of historical records that indicate a bright flash of light Collision could have occurred at a place where there no witnesses Could have fell out in the seaBoom might have been caused by things other than a volcanic eruptionHistorical records from Asia and Europe may not be applicable to earth as a whole

과거의 날씨 주기를 연구하는 과학자들은 6세기 중엽 지구가 갑자기 혹한기가 되었던 것을 알게되었다 이 당시의 몇몇 기록들이 아직도 남아있긴 하지만 아시아와 유럽에서 발견되는 몇가지 자료에서 태양 빛의 수축과 그에따른 엄청난 추위가 있었음을 언급하고 있다 거대한 화산 폭발이나 거대 운석의 지구 충돌중 어느것에

GRE AWA John박 박정어학원

의해서든 엄청난 먼지구름을 형성시켜 지구전체에 태양빛을 차단해서 지구의 기온을 뚝 떨어뜨릴수 있을 수도 있다 그러나 이러한 운석 충돌은 순간 섬광을 발산시키게 될 것인데 이 시기의 어느 기록에서도 이러한 섬광은 언급되지 않고 있다 그러나 이 시기에 남아 있는 일부 아시아의 기록문서에서는 연속적으로 화산폭발음일수 있는 엄청난 폭발음이 있었음을 기록하고 있다 따라서 이러한 혹한은 아마도 화산 폭발에 의한 것이었을 것이다

결론 the cooling was probably caused by a volcanic eruption

반박 자료의 부족( 몇몇 자료가지고 그 당시의 기후를 예상하기에는 무리가 있다)다른 원인에 의해서 기후가 떨어졌을수도 있다 (빙하기 다른 기상원인)기록되지 않았다고 해서 그 사실이 없었던 것은 아니다(기록은 했으나 자료가 없어졌을수도 있고 모든 사실이 전부다 기록되지는 않는다 )기록된 폭발음이 꼭 확산 폭발음이 아닐수도 있다( 화산 폭발음이 너무 먼거리여서 들리지 않을수도 있고 다른 소리가 기록된 것이였을수도 있다)부족한 자료를 신빙할수 있는가 기록의 소멸도 예상 할수 있다 실제로 운석이 떨어졌는데 안들렸을수 있다

The following appeared in a memo from the president of Bower Builders a company that constructs new homes

A nationwide survey reveals that the two most-desired home features are a large family room and a large well-appointed kitchen A number of homes in our area built by our competitor Domus Construction have such features and have sold much faster and at significantly higher prices than the national average To boost sales and profits we should increase the size of the family rooms and kitchens in all the homes we build and should make state-of-the-art kitchens a standard feature Moreover our larger family rooms and kitchens can come at the expense of the dining room since many of our recent buyers say they do not need a separate dining room for family meals

Write a response in which you examine the stated andor unstated assumptions of the argument Be sure to explain how the argument depends on these assumptions and what the implications are for the argument if the assumptions prove unwarranted

The presidentrsquos first unstated but apparent assumption is that a nationwide survey can be applied to local areas There is no guarantee that the area in which Bower Builders builds houses will follow the national consumer trend Since he only mentions the overall result of the nationwide survey it is difficult to determine more specific conditions such whether people in urban and rural areas have the same housing preferences Such uncertainty is further exacerbated by the nature of the survey question itselfmdash it does not deal with current trends in actual home purchases but with desired home features The naiumlve assumption that consumer desire will directly result in consumer action underlies the presidentrsquos interpretation and application of the survey results to his company plan Not everyone who wishes for a large family room and kitchen will or can actually buy a house with those features mainly due to financial reasons Furthermore the president also assumes for no evident reason that consumers will not hesitate to purchase houses with state-of-the-art kitchens as a standard rather than optional feature when it is clear that such a feature will raise the overall cost significantly He continues to blunder in his mistaken assumptions about consumer behavior in assuming that the taste of recent buyers can represent the concern of future prospective buyers The fact that recent buyers have claimed no need for separate dining rooms does not mean future buyers will feel the same way as well

The president of Bower Builders recommends that in order to raise company profits the company should build houses with large family rooms and large state-of-the-art kitchens He uses the results of a nationwide survey on desired home features and the example of Bower Buildersrsquo competitor Domus Construction to support his plan His memo manifests several assumptions about surveys consumer behavior and Domus Constructionrsquos houses that do not necessarily bolsterbuttress his argument

GRE AWA John박 박정어학원

The writer assumes 1 the nationwide survey results can be applied to the local area (Desire cannot reflect consumer trend) 2 Domus Construction was profitable because they had such features (The writer should investigate what other features the DC provides and the actual number of homes sold) 3 State-of-the-art kitchens will sell well (no evidence to buttress his assumption furthermore they would need to spend more money which would increase the price of the housesmdashtheir market will be limited to the affluent) 4 The recent buyers represent the concern of most buyersmdashthat they are ok with a house that is without a dining room

Nationwide survey might not be applicable to this regionDomus Construction could have other superior qualities than Bower Builders Ex DesignSelling fast doesnrsquot indicate quantitySmall yards complaints could be voiced in the future

First the author concludes that a nationwide survey reveals that the two most desired home features are a bathroom with a whirlpool tub and a large kitchen However this nationwide survey may not reflect the trends of the customers that Bower Builders target The author assumes that the nationwide trend reflects customer trends The national study would lead support only if the nationwide trend reflect the home-purchasing trends The nationwide trend may just be a trend of desire which does not result in the act of purchasing However the author does not provide credible evidence that this is the case

전국 조사에서 가장 인기있는 집안 구조 2가지는 훨풀 튜브가 마련된 욕실과 커다른 부엌으로 조사되었다 경쟁업체 Domus Construction이 신축한 완공예정인 주택은 이러한 욕실을 갖추고 있어서 분양이 조기에 이루어지고 있고 가격도 평균치보다 상당히 웃돌고 있다 우리도 집을 많이 팔도 그만큼 이윤을 많이 남기려면 신규 주택에는 위의 2가지 사항을 갖추어야 한다 최근 우리가 지은 주택 수요자들이 적은 평수에 대해서는 별다른 불만사항이 없었기 때문에 앞으로 평수를 더 줄여서 이익을 극대화 할 수 있을 것이다

주장 We should include whirlpool tubs and a large kitchen

1 경쟁업체 Domus Construction이 신축한 완공예정인 주택은 이러한 욕실을 갖추고 있어서라기 보다는 위치가 좋거나 다른 마감재(construction material)나 인테리어가 뛰어나서 분양이 조기에 이루어진것이라고 볼 수 있다 2 전국조사가 건물이 지어지는 지역에 항상 적용되리라고 볼 수 없다 3 평수를 줄이는 것에 불만이 없었다는 것은 editor의 견해일 수 있다 사람들이 표현하지 않은 것일 수도 있고 다른 장점이 많아서 그런 단점을 드러내지 않은 것일 수 있기 때문에 속단할 수 없다 4 다른것을 함으로써 더 잘 팔릴수도 있다

2011 7 19 화요일 GRE AWA 실전반이OO

Argument 94

The president of Bower Builders contends recommends that in order to raise company profits the company should build houses with large family rooms and large state-of-the-art kitchens He uses the results of a nationwide survey on desired home features and the example of Bower Buildersrsquo competitor Domus Construction to support his plan His memo manifests several assumptions about surveys consumer behavior and Domus Constructionrsquos houses that do not

GRE AWA John박 박정어학원

necessarily bolsterbuttress his argumentThe presidentrsquos first unstated but apparent assumption is that a nationwide survey can be

applied to local areas There is no guarantee that the area in which Bower Builders builds houses will follow the national consumer trend Since he only mentions the overall result of the nationwide survey it is difficult to determine more specific conditions such whether people in urban and rural areas have the same housing preferences Such uncertainty is further exacerbated by the nature of the survey question itselfmdash it does not deal with current trends in actual home purchases but with desired home features The naiumlve assumption that consumer desire will directly result in consumer action underlies the presidentrsquos interpretation and application of the survey results to his company plan Not everyone who wishes for a large family room and kitchen will or can actually buy a house with those features mainly due to financial reasons Furthermore the president also assumes for no evident reason that consumers will not hesitate to purchase houses with state-of-the-art kitchens as a standard rather than optional feature when it is clear that such a feature will raise the overall cost significantly He continues to blunder in his mistaken assumptions about consumer behavior in assuming that the taste of recent buyers can represent the concern of future prospective buyers The fact that recent buyers have claimed no need for separate dining rooms does not mean future buyers will feel the same way as well

In addition the president finds a real-life actualization of the nationwide survey results in the recent sales of Domus Construction However he easily assumes that large family rooms and kitchens are the only reasons the houses of Domus Construction sell well He does not take into account other features and selling points of the competitorrsquos houses There may well be other explanations for its houses selling more quickly and expensively including additional home features as well as external factors such as proximity to better schools or superior financial solvency of its clientele If Bower Builders merely added larger family rooms and kitchens without taking into consideration the other factors they may lose rather than gain profits

Clearly the presidentrsquos assertion that Bower Builders make houses with large family rooms and high-tech kitchens at the expense of dining rooms rests on a number of assumptions that are ill-informed and naiumlve If Bower Builders undertakes the proposed plan without further research into local consumer desires purchasing trends and the marketing and sales of competing companies the company will risk losing money by building big new houses that people cannot afford to or will not wish to buy

The following appeared in a letter to the editor of a journal on environmental issues

Over the past year the Crust Copper Company (CCC) has purchased over 10000 square miles of land in the tropical nation of West Fredonia Mining copper on this land will inevitably result in pollution and since West Fredonia is the home of several endangered animal species in environmental disaster But such disasters can be prevented if consumers simply refuse to purchase products that are made with CCCs copper unless the company abandons its mining plans

Write a response in which you examine the stated andor unstated assumptions of the argument Be sure to explain how the argument depends on these assumptions and what the implications are for the argument if the assumptions prove unwarranted

The writer assumes 1 The writer is trying to avoid an inevitability 2 Mining copper will result pollution (they could make preventative measures) 3 The writer may be too late from stopping CCC from developing the area into a copper mine 4 Since mining is an underground enterprise the surface may not be affected that much therefore endangered species may not be affected 4 Consumers wonrsquot buy CCC products if the journal publishes a negative review about CCC (How many readers) CCC could a company that has ties with many IT companies and industries in that their copper is almost ubiquitous in various products

GRE AWA John박 박정어학원

지난 한해동안 CCC(Consolidated Copper 회사)는 서부 플로리다의 열대 지역에 1백만 마일이 넘는 땅을 사들였다 이 곳에서의 채광 활동은 서부 플로리다가 몇몇 멸종 위기에 처한 동물의 서식지이기 때문에 분명 오염과 환경파괴를 가져올 것이다 그러나 이러한 파괴는 CCC 회사가 채광을 포기할 때까지 이 회사가 채굴한 구리로 제조된 제품을 구매하지 않으면 막을 수 있을 것이다

결론 such disaster can be prevented if consumers simply refuse to purchase products that are made with CCCs copper until the company abandons its mining plans

1 구리로 제조된 물건이 생활에 필수적인 것이 많은 만큼 불매가 쉽지 않을 수 있다(전선이나 각종 전자제품에 필수적으로 들어가기 때문에)

2 불매를 유도한다고 해서 소비자들이 구매를 안하는 것은 아니다3 적절한 채굴로 환경파괴를 가져 오지 않을 수 있다 (땅속에 있는 물질을 채굴하는 만큼 생물에 영향을 안

미칠 수도 있다)4 이미 채굴이 다 끝나서 더 이상의 채굴이 없을 수도 있다 5 땅을 구입한다고 해서 채광하는건 아니다 (다른 용도로 샀을수도 있다)

The following is a letter to the editor of an environmental magazine

In 1975 a wildlife census found that there were seven species of amphibians in Xanadu National Park with abundant numbers of each species However in 2002 only four species of amphibians were observed in the park and the numbers of each species were drastically reduced There has been a substantial decline in the numbers of amphibians worldwide and global pollution of water and air is clearly implicated The decline of amphibians in Xanadu National Park however almost certainly has a different cause in 1975 troutmdashwhich are known to eat amphibian eggsmdashwere introduced into the park

Write a response in which you discuss what specific evidence is needed to evaluate the argument and explain how the evidence would weaken or strengthen the argument

Evidence needed the identity of the sender and the census taker of rsquo75 and lsquo02mdasha scientist or an environmentalist credibility issue census methodology vs mere observationmdashthe absence of evidence is not an evidence of absence the season of when the census was taken time-shiftmdashconditions may have changed worldwide decline may include Xanadu other species of predators that prey on amphibians because trout is only one species that prey on amphibians the number of troutmdashhave they increased significantly since rsquo75 First the author needs to be more overt about the credibility of the census and observation There were to accounts that notes the population of amphibiansmdashthe first a census and the second an observation The author needs to bolster his conclusion with the evidence that indicate that the census and second observation were done using scientific methodologies This could either could strengthen or weaken his claim In addition he

GRE AWA John박 박정어학원

needs to provide the specific season of when the census and observation occurred In this letter the writer is informing an editor of an environmental magazine that the number of amphibians was greatly reduced since 1975 and he points out the introduction of trout as the only reason for the decline However the author fails to provide crucial evidences that could strengthen or weaken his conclusion

The following appeared in a memorandum from the president of Hyper-Go Toy Company

Last year sales of our Fierce Fighter toy airplane declined sharply even though the toy had been a top seller for three years Our customer surveys show that parents are now more worried about youthful violence and are concerned about better education for their children Therefore to maintain profits we should discontinue all our action toys and focus exclusively on a new line of educational toys Several other toy companies have already begun marketing educational toys and report sales increases last year of 200 percent And since the average family income is growing sales of new Hyper-Go toys should also increase

Write a response in which you discuss what specific evidence is needed to evaluate the argument and explain how the evidence would weaken or strengthen the argument

First the president indicates that the sales of Fierce Fighter toy airplane declined sharply However he fails to consider the fact that toys are a fad Since FFT enjoyed a three year success it may be natural that the trend would subsideSecond Customer survey is this representative of most toy consumers This trend is not newThird other companies may have profited not through educational but other toys Or educational toy profit may be small in proportion to their sale of other toys 200Parents are not the customers companies should concentrate on what the children want to increase profitLastly parents worry about youthful violence and concern for better education are not new trends

The presidentrsquos decision to discontinue all action toys and focus only on educational toys is too extreme If this decision is taken into effect its procedure may be an onerous task because it would require major shifts in human resources and company image Therefore further data should be considered before following up on this decision

우리 회사에서 지난 3년간 최고 매출을 일으켰던 Fierce Fighter 장난감 비행기가 지난해에는 매출이 급격히 떨어졌다 자체 고객 조사에서는 부모들이 현재 청소년 폭력에 걱정을 하고 있어서 아이들의 양질 교육에 더 관심을 가지고 있는 것으로 나타났다 따라서 회사의 수익을 유지하기 위해서는 모든 자사 전투용 장난감 생산을 중단하고 오로지 교육적인 장난감 생산에 집중해야 한다 몇몇 여타 장난감 회사들도 이미 교육용 장난감 마케팅을 시작해서 지난해에는 200의 매출신장을 가져왔다고 한다 그리고 평균 가계 수입이 점점 늘고 있기 때문에 신형 Hyper-Go 장난감의 매출도 늘어날 것이다

3그룹 Time-shift Error

Woven baskets characterized by a particular distinctive pattern have previously been found only in the immediate vicinity of the prehistoric village of Palea and therefore were believed to have been unique to the Palean people Recently however archaeologists discovered such a Palean basket in Lithos an ancient village across the Brim River from Palea The Brim River is very deep and broad and so the ancient Paleans could only have crossed it by boat but there is no evidence that the Paleans had boats And boats capable of carrying groups of people and cargo were not developed until thousands of years after the Palean people disappeared Moreover Paleans would have had no need to cross the rivermdashthe woods around Palea are full of nuts berries and small game It follows that the so-called Palean baskets were not unique to Palea

GRE AWA John박 박정어학원

Write a response in which you discuss what specific evidence is needed to evaluate the argument and explain how the evidence would weaken or strengthen the argument

RefutationPrehistoric time shift-gtbrim river could have been narrow and shallow or it might have not existed Indigenous patterns may exist in other disconnected remote placesNuts berries small game(hunting) may not have existed or the author should be proved these existed at that time Abundance of resources doesnrsquot support the reason for seclusionBoats not yet found baskets may have been carried across by the river current without the help of a boat Lithos might have crossed the river for commercial purposeOne Palean basket does not substantiate the authorrsquos claimThe absence of evidence is not an evidence of absence

The author assumes without justification that present conditions are the same as at the prehistoric era The author unfairly infers from the presence of Brim River which exist today that it would have existed in the past However the author fails to offer any evidence to substantiate this inference It is very likely that the Brim River might not have existed in prehistoric times or if it did exist may have been shallow and narrow enough for the Paleans to easily cross Any of these scenarios if true would serve to undermine the claim thathelliphelliphelliphelliphellip

First the author claims that the Brim River was very deep and broad so the Paleans could not have crossed it However the author fails to offer any evidence to substantiate that this was true in the prehistoric time of the Paleans For all we know the Brim River might not have existed in the prehistoric era or if it did exist could have been a shallow and narrow river For example scientists believe that thousands of years ago an ice-bridge existed on the Bering Sea connecting Eurasia to what is now North America to explain how the Eskimos and the Asian inhabitants of America came to migrate all over the Americas In this example the absence of evidence is not an evidence of absence Therefore to sufficiently support his claim the author needs to substantiate the fact that the Brim River really did exist and was broad and wide in the Prehistoric Era proving that the Paleans could not have influenced or traded with other groups of people

First the author claims that the Brim River was very deep and broad so the Paleans could not have crossed it However the author fails to offer any evidence to substantiate that this was true in the prehistoric times of the Paleans For all we know the Brim River might not have existed in the prehistoric era or if it did exist could have been a shallow and narrow river For example rivers are created by natural erosion over thousands of years Niagara Falls carved its way from the mouth of Lake Ottawa and created a long river Likewise the current Brim Riverrsquos physical features may not have been wide and deep Therefore to sufficiently support his claim the author needs to substantiate the fact that the Brim River really did exist and was broad and wide in the Prehistoric Era proving that the Paleans could not have influenced or traded with other groups of people

예전에는 실로짠 특이한 무늬 바구니가 Palea의 선사시대 지역의 인근마을에서만 발견되어왔기 때문에 Palea 마을 사람들의 특징이라고 여겨졌었다 그러나 최근들어 고고학자들이 Lithos지역에서 Palean 바구니를 발견하였는데 그 당시 지역은 Brim 강을 가로질러 Palea까지 닿아있었다 이 강은 수심이 아주 깊고 강폭이 넓었으며 때문에 고대의 Palea인들은 배를 이용해서 강을 건널수 있었을 것이다 그러나 이들이 배를 가지고 있었다는 증거는 발견되지 않고 있다 더군다나 이들이 멸명한 이후 수천년이 지난뒤에도 수많은 물자와 사람을 실어 나를수 있는 용적을 가진 배는 개발되지 않았다 이와더불어 Palea인들은 강을 건널필요가 없었는데 그것은 너트나무 장과열매 그리고 작은 사냥감들이 주변숲에 풍부했기때문이다 따라서 Palean 바구니라고 하는 것도 Palea인들만의 전유물이 아니라는 결론을 얻을 수 있다

결론 if follows that the so-called Palean baskets were not unique to Palea이번문제는 굿이 causal Bad analogy 로 구분해서 찾기가 힘드내요 배를 발견했다는 증거가 없는것이다 (앞으로도 발견될수 있음)

GRE AWA John박 박정어학원

계절의 영향으로 겨울에 얼음이 두껍게 언다든지 여름에 가뭄으로 인해서 건널수 있다물자가 풍부한 것이 이동하지 않을 조건이 아니다 다른 것에 의해서 이동가능(의약품등)

Thirteen years ago researchers studied a group of 25 infants who showed signs of mild distress when exposed to unfamiliar stimuli such as an unusual odor or a tape recording of an unknown voice They discovered that these infants were more likely than other infants to have been conceived in early autumn a time when their mothers production of melatonin hormone known to affect some brain functions would naturally increase in response to decreased daylight In a follow-up study conducted earlier this year more than half of these children now teenagers who had shown signs of distress identified themselves as shy Clearly increased levels of melatonin before birth cause shyness during infancy and this shyness continues into later life

Write a response in which you examine the stated andor unstated assumptions of the argument Be sure to explain how the argument depends on these assumptions and what the implications are for the argument if the assumptions prove unwarranted

Any baby exposed to unpleasant stimuli would react in such wayFirst of all the author states 25 infants as his evidence However this research sample is too small to prove his claimSecond the author states that 25 infants were conceived in early autumn which he claims lead to a shy disposition However this is faulty evidenceThird the research study was a long term study done in the span of 13 years However the author only writes about the initial and final stages of the study and leaves out evidences of what could have happened during the 13 years which could be more evidential factors of influenceFourth neither the infantsrsquo genetic predisposition nor their environment were taken into accountFinally the author concludes that his shyness continues into later life (Other factors could alter this disposition epigenetic theory)

13 년전 학자들은 25명의 유아를 대상으로 이상한 냄새나 특이한 소리를 녹음한 테잎등으로 낯선 자극을 주었을때 보이는 미미한 압박감 증상을 조사하였다 이들은 성숙기가 막 지났을 즈음에 보통의 유아들이 비슷한 증상을 보이는 정도 보다는 다소 민감한 반응을 보였는데 이 시기는 아이의 엄마가 뇌의 일부 기능에 영향을 미치는 것으로 알려진 멜라토니아 호르몬을 생산하는 시기로써 이 호르몬은 낯 시간이 짧을때 자연적으로 증가할 수도 있다 금년초에 실시된 추가연구에서 현재 10대로 성장한 당시 조사대상의 절반 이상의 아이들이 부끄럼을 잘타는 것으로 여기고 있었다 따라서 분명한 것은 출산전 멜라토닌 수치의 증가가 유아기에 수줍음 등의 영향을 미치게 되며 이러한 영향이 성장후에도 작용한다는 것이다

주장 Clearly increased levels of melatonin before birth cause shyness during infancy and this shyness continues into later life

1 25명의 아기로 결론 내리기에 샘플이 작다2 과학적 사실들에 대한 명확한 근거가 엇음3 다른 영향을 간과했다(다른 호르몬에 의한 영향 후천적인 성격형성의 영향)

GRE AWA John박 박정어학원

The following is a letter to the editor of the Atticus City newspaper

Former Mayor Durant owes an apology to the city of Atticus Both the damage to the River Bridge which connects Atticus to Hartley and the traffic problems we have long experienced on the bridge were actually caused 20 years ago by Durant After all he is the one who approved the construction of the bridge If he had approved a wider and better-designed bridge on which approximately the same amount of public money would have been spent none of the damage or problems would have occurred Instead the River Bridge has deteriorated far more rapidly over the past 20 years than has the much longer Derby Bridge up the river Even though the winters have been severe in the past several years this is no excuse for the negligence and wastefulness of Durant

Write a response in which you discuss what questions would need to be answered in order to decide whether the recommendation is likely to have the predicted result Be sure to explain how the answers to these questions would help to evaluate the recommendation

전임 시장인 Durant 씨는 Atticus 시에 대해 사과할 의무가 있습니다 Atticus와 Hartley를 잇는 River Bridge 교량에 대한 피해와 이 교량에서 오랫동안 주민들이 겪어오고 있는 교통 혼잡 문제들은 실제로 20년 전부터 시작된 것이었습니다 결정적으로 그가 교량 공사를 허가했던 바로 그 장본인입니다 당시 비슷한 공사비용으로 폭이 더 넓고 튼튼하게 설계된 교량을 허가했다면 이러한 문제나 피해는 발생하지 않았을 겁니다 더군다나 이 다리는 지난 20년 동안 상류에 건설된 훨씬 오래된 Derby 다리보다도 빠르게 부식되어 갔습니다 지난 수년동안 심지어 혹한이 있었다 하더라도 이러한 태만과 국고 손실에 대한 책임을 회피할 길이 없는 것입니다

결론 Former Mayor Durant owes an apology to the city of Atticus

1 디자인이 문제가 아닐수 있다 (디자인은 좋았으나 건설과정에 문제가 있었을 수 있다)2 그 당시의 시예산이 적어서 더 큰 다리를 짓기가 불가능했을 수도 있다3 교통량이 많거나 다른 상황으로 인해서 부식이 빨리 됐을 수 있다4 그 당시에는 최선의 선택이였지만 갑자기 변한 상황에 의해서 이런 문제점들이 발생했을 수 있다

GRE AWA John박 박정어학원

4그룹 거짓인과관계 오류 (False Cause) 빈출

Fifteen years ago Omega University implemented a new procedure that encouraged students to evaluate the teaching effectiveness of all their professors Since that time Omega professors have begun to assign higher grades in their classes and overall student grade averages at Omega have risen by 30 percent Potential employers looking at this dramatic rise in grades believe that grades at Omega are inflated and do not accurately reflect student achievement as a result Omega graduates have not been as successful at getting jobs as have graduates from nearby Alpha University To enable its graduates to secure better jobs Omega University should terminate student evaluation of professors

Write a response in which you discuss what specific evidence is needed to evaluate the argument and explain how the evidence would weaken or strengthen the argument

Omega professor evaluation implemented 15 years ago =gt Omega prof assign higher grades 30Employers believe therersquos grade inflation

Thus unsuccessful employment than AlphaTherefore to secure jobs Omega should end evaluating profs

Specific evidence neededRelationship between higher grades and evaluationRelationship between GPA and unsuccessful employmentAlpharsquos education could just be better than OmegaldquoFifteen years agordquo is a long time other factors could have influenced Why is the inflation a problem just now How much is Alpha better Is the comparison just How much gap is thereOmegarsquos student could just be doing better in their studiesComparison to other universities다른 대안 없나hellip Could Omega alleviate the employment problem by implementing a different procedure or program

15 년전 우리 대학은 학생들로 하여금 교수평가를 하도록 한 새로운 조치를 시행했었습니다 이후 교수들은 자신의 학과 학생들에게 높은 학점을 주었으며 그에따라 학생들의 전체 평점이 30나 올랐습니다 외부의 기업체들은 분명 점수가 지나치게 부풀려졌다고 믿고 있습니다 결국 본 대학 졸업생들이 인근 Alpha 대학의 졸업자들보다 구직률이 떨어지는 이유를 잘 보여주고 있는 것입니다 이를 해결하기 위해 이제부터는 학생들에 의한 교수평가제를 중단해야 합니다

결론 Omega University should now terminate student evaluation of professors

반박 교수 평가와 학점 인플레의 연관성이 적다( 교수 평가를 먼저하고 학점을 나중에 매길수도 있다)채용기준에 성적만 있는게 아니다 학업성취의 결과 일수도 있다 Alpha 가 원래 유능했다 Alpha 의 교육내용이 좋았다

GRE AWA John박 박정어학원

In this memo the dean of Omega University(OU) recommends OU to terminate professor evaluation to secure better jobs for the students To support this recommendation the dean offers several reasons However this argument contains several logical flaws which render it unconvincing

A threshold problem with the argument involves the voluntary nature of the evaluationprocedure The dean provides no evidence about the number or percentage of Omegastudents who participate in the procedure Lacking such evidence it is entirely possible thatthose numbers are insignificant in which case terminating the procedure is unlikely to haveany effect on the grade average of Omega students or their success in getting jobs aftergraduationThe argument also assumes unfairly that the grade-average increase is the result of theevaluation procedure--rather than some other phenomenon The dean ignores a host of otherpossible explanations for the increase--such as a trend at Omega toward higher admissionstandards or higher quality instruction or facilities Without ruling out all other possibleexplanations for the grade-average increase the dean cannot convince me that by terminatingthe evaluation procedure Omega would curb its perceived grade inflation let alone help itsgraduates get jobsEven if the evaluation procedure has resulted in grade inflation at Omega the deans claimthat grade inflation explains why Omega graduates are less successful than Alpha graduatesin getting jobs is unjustified The dean overlooks a myriad of other possible reasons forOmegas comparatively poor job-placement record Perhaps Omegas career services areinadequate or perhaps Omegas curriculum does not prepare students for the job market aseffectively as Alphas In short without accounting for other factors that might contribute toOmega graduates comparative lack of success in getting jobs the dean cannot justify theclaim that if Omega curbs its grade inflation employers will be more likely to hire OmegagraduatesFinally even if the dean can substantiate all of the foregoing assumptions the deansassertion that Omega must terminate its evaluation procedure to enable its graduates to findbetter jobs is still unwarranted in two respects First the dean ignores other possible ways bywhich Omega can increase its job-placement record--for example by improving its publicrelations or career-counseling services Second the dean unfairly equates more jobs withbetter jobs In other words even if more Omega graduates are able to find jobs as a result ofthe deans recommended course of action the kinds of jobs Omega graduates find would notnecessarily be better onesIn sum the deans argument is unpersuasive as it stands To strengthen it the dean mustprovide better evidence that the increase in grade average is attributable to Omegasprofessor-evaluation procedure and that the end result is a perception on the part ofemployers that Omega graduates are less qualified for jobs than Alpha graduates To betterassess the argument I would need to analyze 15-year trends in (l) the percentage of Omegastudents participating in the evaluation procedure (2) Omegas admission standards andquality of education and (3) Omegas emphasis on job training and career preparation I wouldalso need to know what other means are available to Omega for enabling its graduates to findbetter jobs

GRE AWA John박 박정어학원

The following appeared in a memo from a vice president of Quiot Manufacturing

During the past year Quiot Manufacturing had 30 percent more on-the-job accidents than at the nearby Panoply Industries plant where the work shifts are one hour shorter than ours Experts say that significant contributing factors in many on-the-job accidents are fatigue and sleep deprivation among workers Therefore to reduce the number of on-the-job accidents at Quiot and thereby increase productivity we should shorten each of our three work shifts by one hour so that employees will get adequate amounts of sleep

Write a response in which you examine the stated andor unstated assumptions of the argument Be sure to explain how the argument depends on these assumptions and what the implications are for the argument if the assumptions prove unwarranted

The following appeared in a memo from a vice president of Alta Manufacturing

During the past year Alta Manufacturing had thirty percent more on-the-job accidents than nearby Panoply Industries where the work shifts are one hour shorter than ours Experts believe that a significant contributing factor in many accidents is fatigue caused by sleep deprivation among workers Therefore to reduce the number of on-the-job accidents at Alta we recommend shortening each of our three work shifts by one hour If we do this our employees will get adequate amounts of sleep

Write a response in which you discuss what questions would need to be answered in order to decide whether the recommendation and the argument on which it is based are reasonable Be sure to explain how the answers to these questions would help to evaluate the recommendation

The following appeared in a memo from the vice president of Butler Manufacturing

During the past year workers at Butler Manufacturing reported 30 percent more on-the-job accidents than workers at nearby Panoply Industries where the work shifts are one hour shorter than ours A recent government study reports that fatigue and sleep deprivation among workers are significant contributing factors in many on-the-job accidents If we shorten each of our work shifts by one hour we can improve Butler Manufacturings safety record by ensuring that our employees are adequately rested

1 Write a response in which you discuss what specific evidence is needed to evaluate the argument and explain how the evidence would weaken or strengthen the argument

2 Write a response in which you discuss what questions would need to be answered in order to decide whether the recommendation is likely to have the predicted result Be sure to explain how the answers to these questions would help to evaluate the recommendation

4번 반복됨

Alta has 30 more job accidents than Panoply(work shifts one hour shorter)Experts Job accidents caused by fatigue and sleep deprivationTherefore to reduce job accidents and increase productivity shorten three work shifts by one hour for adequate sleep

지난해 우리 회사는 인근 Panoply Industries보다 업무상 재해가 30나 더 많았다 그 회사는 우리보다 근무 교대시간이 1시간 정도 짧았다 전문가들은 대부분의 업무상 재해에 있어서 가장 중요한 요인이 과로와 수면부족으로 보고있다 따라서 우리 회사에서 높은 산업재해를 줄이고 아울러 생산성을 높이기 위해서는 근로자들이 충분한 수면을 취할 수 있도록 1시간씩 3교대 시간을 줄여야 한다

In this memo the (author) vice president of Alta Manufacturing (AM) recommends that to reduce on-the-job accidents and increase productivity AM should shorten its three work shifts by one hour so that employees can

GRE AWA John박 박정어학원

get more sleep To support this recommendation the author provides several evidences However careful scrutiny of each of the facts reveals that it provides little credible support for the authorrsquos recommendation QuestionsThe number of accidents What kind of accidents The seriousness of the accidents is importantHow many employees are in each company What are their productsFalse cause Sleep may not be the reason for the on-the-job accidents What do Alta and Panoply manufacture

First of all the author believes that fatigue caused the on-the job accidents However there could be other reasons The author observes a correlation between sleep deprivation and on-the-job accidents then concludes that the former is the cause of the latter However the author fails to rule out other possible explanations For example it is entirely possible that Alta factories require more strenuous and dangerous labor than Panoply Without ruling out all other such factors it is unfair to conclude that fatigue is responsible for the accidents In addition the work-shifts may not be the cause of the sleep deprivation and fatigue It is possiblehellip Thus the author should provide what exactly Panoply and Alta manufacture and more precise data about their working conditions to be more convincing

Shortening the shift by one hour does not necessarily lead to more sleep And is one hour enoughLess accidents does not mean increased productivity

결론 We should shorten each of out three work shifts by one hour

반박 경쟁사에 비해서 시간당 하는 업무량이 많아서 더욱 피곤할 수도 있다 시간이 문제가 아니라 노후된 시설 설비 자체의 문제 작업 자체가 원래 위험한 것이여서 사고가 많을 수도 있다 다른 회사는 더욱 많은 작업시간에도 불구하고 안정한 작업여건으로 인해서 사고율이 오히려 더 작을 수도 있다비교사의 재해감소가 다른 요인일수 있다(안전 교육 철저)줄인 시간이 피로회복이나 수면으로 연결 안될수 있음(술을 마실 수도 있고 그 시간에 휴식을 취하지 않고 다른일을 함으로써 더욱 피로해질수 있다)

This editorial recommends that Alta Manufacturing reduce its work shifts by one hour each inorder to reduce its on-the-job accident rate and thereby increase Altas productivity To supportthis recommendation the author points out that last year the number of accidents at Alta was30 greater than at Panoply Industries where work shifts were one hour shorter The authoralso cites certain experts who believe that many on-the-job accidents are caused by fatigueand sleep deprivation I find this the argument unconvincing for several reasonsFirst and foremost the author provides absolutely no evidence that overall workerproductivity is attributable in part to the number of on-the-job accidents Although commonsense informs me that such a relationship exists the author must provide some evidence ofthis cause-and-effect relationship before I can accept the authors final conclusion that theproposed course of action would in fact increase Altas productivitySecondly the author assumes that some accidents at Alta are caused by fatigue or sleepdeprivation However the author overlooks other possible causes such as inadequateequipment maintenance or worker training or the inherent hazards of Altas manufacturingprocesses By the same token Panoplys comparatively low accident rate might be attributablenot to the length of its work shifts but rather to other factors such as superior equipmentmaintenance or worker training In other words without ruling out alternative causes ofon-the-job accidents at both companies the author cannot justifmbly conclude that merely byemulating Panoplys work-shift policy Alta would reduce the number of such accidentsThirdly even assuming that Altas workers are fatigued or sleep-deprived and that this is thecause of some of Altas on-the-job accidents in order to accept the authors solution to thisproblem we must assume that Altas workers would use the additional hour of free time tosleep or rest However the author provides no evidence that they would use the time in thismanner It is entirely possible that Altas workers would use that extra hour to engage in someother fatiguing activity Without ruling out this possibility the author cannot convincinglyconclude that reducing Altas work shifts by one hour would reduce Altas accident rateFinally a series of problems with the argument arise from the scant statistical information onwhich it relies In comparing the number of accidents at Alta and Panoply the author fails toconsider that the per-worker accident rate might reveal that Alta is actually safer than Panoplydepending on the total number of workers at each company Second perhaps accident rates

GRE AWA John박 박정어학원

at the two companies last year were aberrations and during other years Altas accident ratewas no greater or even lower than Panoplys rate Or perhaps Panoply is not representativeof industrial companies generally and that other companies with shorter work shifts have evenhigher accident rates In short since the argument relies on very limited statistical information Icannot take the authors recommendation seriouslyIn conclusion the recommendation for emulating Panoplys work-shift policy is not wellsupported To convince me that shorter work shifts would reduce Altas on-the-job accidentrate the author must provide clear evidence that work-shift length is responsible for some ofAltas accidents The author must also supply evidence to support her final conclusion that alower accident rate would in fact increase overall worker productivity

The following appeared in a memo from the vice president of marketing at Dura-Sock Inc

A recent study of our customers suggests that our company is wasting the money it spends on its patented Endure manufacturing process which ensures that our socks are strong enough to last for two years We have always advertised our use of the Endure process but the new study shows that despite our socks durability our average customer actually purchases new Dura-Socks every three months Furthermore our customers surveyed in our largest market northeastern United States cities say that they most value Dura-Socks stylish appearance and availability in many colors These findings suggest that we can increase our profits by discontinuing use of the Endure manufacturing process

1 Write a response in which you examine the stated andor unstated assumptions of the argument Be sure to explain how the argument depends on these assumptions and what the implications are for the argument if the assumptions prove unwarranted

2 Write a response in which you discuss what specific evidence is needed to evaluate the argument and explain how the evidence would weaken or strengthen the argument

3 Write a response in which you discuss what questions would need to be answered in order to decide whether the recommendation and the argument on which it is based are reasonable Be sure to explain how the answers to these questions would help to evaluate the recommendation

Intro The vice president of marketing at Dura-Sock Inc is offering a potentially harmful investment recommendation by claiming that Dura-Sock should discontinue its use of the ldquoEndurerdquo process To support his recommendation he points out a study that Dura-Sock customers actually purchase the socks every three months and a survey that reveals that Dura-Sock customers like the sockrsquos stylish appearance and availability in many colors The study and survey however are insufficient in supporting his proposal and the VP makes several unwarranted assumptionsIntro (simplified) The VP states that though Dura-Socks last for two years customers buy the socks every three months Therefore he assumes that the consumersrsquo motive for buying the produce is not its durabilityHowever the author fails to rule out other possible motivation for consumption

Even if the survey is reliable the author should consider the rest of the market Vague terms ldquowasting moneyrdquomdashprecisely how much are they wasting Studysurvey errorThe company must calculate the outcome of such momentous decisionStudy participantsrsquo comment that they prefer Dura-Sock for its stylishness and availability might take Dura-Sockrsquos enduring quality for granted

우리회사 제품 소비자들에 대한 최근 조사에서 지난 2년여간 양말의 내구성을 강하게 하는 필수공정이었던 자사 특허의 Endure 공정에 들어가는 비용이 낭비라고 말하고 있다 우리 회사는 항상 이 공정 처리에 대한 광고를 내보냈으나 이에 대한 시장 조사에서 실제로 고객들은 이 신제품을 평균 석달마다 구매하는 것으로 나타났다 더군다나 북동부지역에서 실시한 대규모 시장조사에 응답한 고객들은 양말의 모양과 색상등에 더

GRE AWA John박 박정어학원

관심을 나타냈다 이러한 결과는 우리회사가 신기술 공법을 중단하면 그에 따라 수익이 늘어날 것이라는 것을 말해주고 있는 것이다주장 These findings suggest that Dura0Sock can increase its profits by discontinuing its use of the ldquoEndurerdquo manufacturing process

1 survey가 정확한 소비자의 의견을 나타낸 것인가 다른 선택없이 양자택일과 같은 방법의 survey였는지2 북동부지역의 시장조사가 전체 의견을 대표할 수 있나3 사람들이 모양이나 색상에 앞서 내구성을 먼저 평가했을 수도 있다 내구성을 갖추었다는 전제하에 모양과 색상에 관심을 드러낸 것일 수 있다4 소비자가 도매상(retail)인지 소매상(whole)인지가 없다

The following appeared in a business magazine

As a result of numerous complaints of dizziness and nausea on the part of consumers of Promofoods tuna the company requested that eight million cans of its tuna be returned for testing Promofoods concluded that the canned tuna did not after all pose a health risk This conclusion is based on tests performed on samples of the recalled cans by chemists from Promofoods the chemists found that of the eight food chemicals most commonly blamed for causing symptoms of dizziness and nausea five were not found in any of the tested cans The chemists did find small amounts of the three remaining suspected chemicals but pointed out that these occur naturally in all canned foods

Write a response in which you discuss what questions would need to be addressed in order to decide whether the conclusion and the argument on which it is based are reasonable Be sure to explain how the answers to the questions would help to evaluate the conclusion

Representativeness of the tested cansThey should conduct a comparative studyThe testing could be biased because Promofoods employees conducted the testingHow much (quantity) of the five and three suspected chemicals were in the canned foodsFalse cause The substance that caused dizziness and nausea may not be one of the eight common chemicals

많은 소비자들의 현기증과 구역질 불만에 따라 Promofoods사는 지난해 참치 캔 8백만 개를 테스트하기 위해 반품시켰다 그 결과 캔에서는 건강에 위험이 될 수 있는 화합물질이 없었던 것으로 회사측은 결론지었다 이러한 결론은 회사측 화학연구자들이 회수된 캔의 샘플을 테스트해서 이들 증상의 원인이 되는 8가지 화합물 중에서 5가지가 실험된 캔에서 발견되지 않았다는 사실에 근거한 것이다 이들 화학자들은 나머지 3개가지 화합물이 모든 캔 식료품에서 흔히 발견되는 것이라고 언급했다 결론 Promofoods concluded that the cans did not after all contain chemicals that posed a health risk

1 공인된 기간에서 테스트를 한 것이 아니고 자사에서 직접 테스트를 했기에 신뢰성이 안간다 2 이런 증상을 일으키는 8개의 물질 말고 다른 물질들이 캔속에 많이 포함됬을수 있다 3 나머지 3개의 물질들의 함유량이 많아서 다른 종류의 캔들은 문제를 일으키지 않지만 참치캔은 문제를

일으킬 수 있다

This magazine article concludes that the 8 million cans of tuna Promofoods recalled due tocomplaints about nausea and dizziness do not after ail contain any chemicals that pose a

GRE AWA John박 박정어학원

health risk To support this conclusion the author cites the fact that five of eight chemicalscommonly causing these symptoms were not found in the recalled cans while the other threealso occur naturally in other canned foods For several reasons this evidence lends littlecredible support to the authors conclusionTo begin with the author relies partly on the fact that although three of the eight chemicalsmost commonly blamed for nausea and dizziness appeared in Promofoods recalled tunathese chemicals also occur naturally in other canned foods However this fact alone lends nosupport to the authors conclusion for two reasons First the author might be ignoring animportant distinction between naturally occurring chemicals and those not occurring naturallyIt is entirely possible that these three chemicals do not occur naturally in Promofoods tunaand that it is for this reason that the chemicals cause nausea and dizziness Secondly it isentirely possible that even when they occur naturally these chemicals cause the samesymptoms Unless the author rules out both possibilities he cannot reliably conclude that therecalled tuna would not cause these symptomsAnother problem with the argument is that the authors conclusion is too broad Based onevidence about certain chemicals that might cause two particular heath-related symptoms theauthor concludes that the recalled tuna contains no chemicals that pose a health risk Howeverthe author fails to account for the myriad of other possible health risks that the recalled tunamight potentially pose Without ruling out all other such risks the author cannot justifiablyreach his conclusionA third problem with the argument involves that fact that the eight particular chemicals withwhich the test was concerned are only the eight most commonly blamed for nausea anddizziness It is entirely possibly that other chemicals might also cause these symptoms andthat one or more of these other chemicals actually caused the symptoms Without ruling outthis possibility the author cannot jusufiably conclude that the recalled tuna would not causenausea and dizzinessA final problem with the argument involves thetesting procedure itself The author providesno information about the number of recaUed cans tested or the selection method used Unlessthe number of cans is a sufficiently large sample and is statistically repre sentative of all therecalled cans the studys results are not statistically reliableIn conclusion the article is unconvincing as it stands To strengthen the assertion that therecalled tuna would not cause nausea and dizziness the author must provide evidence thatthe three chemicals mentioned that occur naturally in other canned foods also appear naturallyin Promofoods tuna The author must also provide evidence that ingesting other canned foodscontaining these three chemicals does not cause these symptoms To better evaluate theargument we would need to know whether the sample used in the tests was statisticallysignificant and representative of all the recalled tuna We would also need to know what otherchemicals in the recalled tuna might pose any health risk at all

5그룹 불충분 조건오류 빈출

Natures Way a chain of stores selling health food and other health-related products is opening its next franchise in the town of Plainsville The store should prove to be very successful Natures Way franchises tend to be most profitable in areas where residents lead healthy lives and clearly Plainsville is such an area Plainsville merchants report that sales of running shoes and exercise clothing are at all-time highs The local health club has more members than ever and the weight training and aerobics classes are always full Finally Plainsvilles schoolchildren represent a new generation of potential customers these schoolchildren are required to participate in a fitness-for-life program which emphasizes the benefits of regular exercise at an early age

Write a response in which you examine the stated andor unstated assumptions of the argument Be sure to

GRE AWA John박 박정어학원

explain how the argument depends on these assumptions and what the implications are for the argument if the assumptions prove unwarranted

False cause

First of all the author believes that the Increased sales of running shoes and exercise clothing indicates

Plainesville residentsrsquo interest in leading healthy lives However this assumption is not logically convincing for

several reasons could be a fashion trendTime shift ldquoFitness for liferdquo might not have any influence on schoolchildren as they growFalse cause There could be other reasons for member increase in the health clubAll of the above are insufficient condition

The author has to prove that local residents are interested in leading healthy lives However he supports his conclusion with insufficient evidence Nevertheless even if the residents are concerned with health naturersquos way may not be successful First

그 동안의 경험을 토대로 볼 때 건강생활과 밀접히 관련되어 있는 거주 지역에서 본 상점들이 아주 호응을 얻고 있다 따라서 이러한 주민들이 많이 거주하고 있는 Plainsville 에 새로운 상점들을 계속 세워야 한다 이 지역 상인들은 런닝화와 운동복 판매가 가장 높다고 말한다 불과 5 년전에는 거의 전무하다시피하던 지역 헬스 클럽의 경우도 엄청나게 많은 회원을 확보하고 있으며 웨이트 트레이닝과 에어로빅 강좌들도 항상 만원이라고 한다 새로운 고객층을 예측해 보는 것도 가능하다 이 지역의 학생들의 경우 Fitness for Life프로그램을 받게 되는데 이러한 프로그램을 통해서 유년시절부터 정규적인 운동 습관을 들이게 하고 있는 것이 그것이다

결론 We should therefore build our next new store in Plainsville

반박 그동안의 경험에 의한 과거 통계가 꼭 여기에도 적용되는건 아니다 5 년전 헬스 클럽이 잘 안되었던게 다른 원인이였을수 있다(강사수준 미달 강좌미비)tourist 에 의한 원인 일수 있다 어렸을때부터 운동을 했다고 해서 커서도 관심이 있지는 않다 (오히려 반감이 있을수 있다 혹은 건강하기에 건강에 관심이 적을수도 있다)운동복이나 신발의 판매가 육체노동에 의한 것일수도 있다

IntroductionSupport1049896In this memorandum the author asserts that Naturersquos Way should build its next newstore in Plainsville To support this assertion the author states that Plainsvillesmerchantsrsquo sales of exercise clothing are going well the local health club has moremembers than ever and a new generation of customers will help to ensure NaturersquosWayrsquos success At first glance the authorrsquos assumption seems convincing but in-depth scrutiny revealsthat it lacks substantial evidence as it stands

Body 1-SamplingTopic Sentence 1To begin with the author assumes that the merchantsrsquo report indicates that the residentsare concerned about their health However this assumption is based on unsubstantiated

GRE AWA John박 박정어학원

data Example 1 (Rebuttal1) First if we do not know the total volume of items sold and the price of the goods exactly we cannot infer whether the residents are actually buying many goods Example 2 (Rebuttal2)In addition to that the report emphasizes the rising sales of running shoes and exerciseclothing however these may not be hot-selling items for Naturersquos Way or may not be theproducts the company is planning to sell Concluding Sentence Therefore in order to make the argument reliable the author should reconsider themerchantsrsquo report with more detailed data

Body 2-CausalTopic Sentence 2Second the author contends that the health clubs classes are full yet this does not meanthat many people actually use the health club other factors may be the real cause forthose closed classes Example 1 (Rebuttal1) To begin with if the health club is very small the number of people working out wouldnot be a large one In fact regular gym-going may just be a vogue among a smallunrepresentative segment of Plainsvilles population Example 2 (Rebuttal2) Moreover it is possible that most of the people who exercise in the health club do weight training and aerobics only to look good and to meet other singles not for their health In that case there would be little demand for health products Concluding SentenceThus the author should not hasten to presume what really caused people to be interested in a healthier lifestyle and enroll in the health club

Body 3-Time-ShiftTopic Sentence 3Finally the author highlights that Naturersquos Way can expect a new generation of customersin Plainsville that will help the company in the long term This notion is mistaken in that itassumes the conditions of the present will continue unchanged in the future Although theschool children are required to participate in the fitness for life program they may notnecessarily buy Naturersquos Ways products Example 1 (Rebuttal1) In the first instance they may suffer a fall in purchasing power arising from future economic difficulties this would cause reluctance to spend a considerable amount of money on health products which tend to be more expensive Example 2 (Rebuttal2)Another possibility is that there may emerge many competitor companies vying with Naturersquos Way so that in the future the school children may not feel the necessity to purchase one companyrsquos health products over anotherrsquosConcluding Sentence Thus the authorrsquos assumption is highly speculative since it relies heavily on unknowablefuture circumstances

ConclusionThesis In sum the author uses many assumptions that are insufficient in supporting his claimsSupportIn order for the authorrsquos claims to be convincing he needs to advance more persuasiveevidence that people in Plainsville really are concerned with their health and health foodThe following was written as a part of an application for a small-business loan by a group of developers in the city of Monroe

A jazz music club in Monroe would be a tremendously profitable enterprise Currently the nearest jazz club is 65 miles away thus the proposed new jazz club in Monroe the C-Note would have the local market all to itself Plus jazz is extremely popular in Monroe over 100000 people attended Monroes annual jazz festival last summer several well-known jazz musicians live in Monroe and the highest-rated radio program in Monroe is Jazz Nightly which airs every weeknight at 7 PM Finally a nationwide study indicates that the typical jazz fan spends close to $1000 per year on jazz entertainment

1 Write a response in which you discuss what specific evidence is needed to evaluate the argument and explain how the evidence would weaken or strengthen the argument

2 Write a response in which you examine the stated andor unstated assumptions of the argument Be

GRE AWA John박 박정어학원

sure to explain how the argument depends on these assumptions and what the implications are for the argument if the assumptions prove unwarranted

3 Write a response in which you discuss what questions would need to be answered in order to decide whether the prediction and the argument on which it is based are reasonable Be sure to explain how the answers to these questions would help to evaluate the prediction

Group error nationwide survey may not reflect local trends Is the nationwide jazz fan population substantialInsufficient non-residents of Monroe may have attended the jazz festival (Body alternative explanation last year may have been an anomaly The author should consider data from various years) The author should indicate how many out of 100000 were Monroe residentsNationwide study Does this reflect Insufficient Citizens of Monroe may continue to go to the jazz club 65 miles away

Are the people in Monroe really interested in jazzMajority of the people who attended the jazz festival might not be Monroe residentsSurvey error nationwide study may not be applicable to MonroeJazz musicians who live in MonroeMonopolyRadio station

In this business application the author claims that the proposed jazz club C Note will be very profitable in Monroe To support this claim the author argues for his case with several evidences At first glance the authorrsquos argument seems convincing however careful scrutiny reveals that his argument in specious

To begin with the author claims that Monroersquos citizens are interested in jazz He presents three evidences First Secondhellip Thirdhellip Howeverhellip

Monroe 시에 있는 재즈 음악 클럽은 수익성이 좋은 사업이다 현재 가장 가까이에 있는 클럽은 65 마일 정도 떨어져 있다 따라서 이번에 세우려고 하는 C Note 는 독보적인 위치를 점할것이다 더군다나 재즈는 이 시에서 가장 인기있는 음악이다 지난 여름 재즈 축제에서는 10 만명 이상의 Morone 시 주민이 참석하였고 몇몇 유명한 재즈 음악가들도 이곳에 살고 있으며 저녁때 방영되는 라디오 프로그램중에서 최고의 시청률을 보이고 있는 것도 Jazz Nightly 이다 전국조사에서도 전형적인 재즈 팬들은 재즈 분야에 년간 1천 달러 가까이 지출하고 있는 것으로 보고되고 있다 따라서 C Note 클럽이 돈을 벌 수 있는 사업이라는 것은 확실한 것이다

결과 It is clear that the C Note cannot help but make money반박 nearest jazz club 이 양질의 써비스로 여전히 손님을 끌수도 있다Festival 에 얼마나 참여하는지가 jazz 의 인기를 반영하지 않는다 뮤지션이 많이 사는거랑 jazz 의 인기가 상관없다라디오 프로그램이 다른 요인에 의해서 인기일수도 있다 (진행자때문)전국 통계 적용 불가화목 실전반_Ms Noh6In this application the author suggests that a jazz club in Monroe will make a number of profits To support this suggestion the author exemplifies the local condition popularity of jazz in Monroe and nationwide study However careful scrutiny of each of the facts reveals that it provides little credible support for the authorrsquos recommendation Good clear intro

First the author assumes that jazz is popular in Monroe because of several facts the jazz festival last year had high participation some famous jazz musicians live in Monroe and the high-rated radio program is lsquoJazz Nightlyrsquo However this assumption has many drawbacks that must be seriously considered(Good topic sentences) If many attendants in the last-yearrsquos festival came from other cities and not Monroe it is hard to conclude that Monroersquos people like jazz Therefore the author must examine how many Monroe residents actually attended the festival On top of that there is little relationship between habitation of famous jazz musician and the popularity of jazz in Monroe Although several well-known musicians live there if they do not take part in any jazz performance of Monroe this might have no effect to the interest of Monroersquos residents

GRE AWA John박 박정어학원

about jazz Finally in the case of radio program this is also not suitable reason why jazz is popular in Monroe It might be possible that people cannot help choosing lsquoJazz Nightlyrsquo because there are few radio programs at Night The fact that the radio program is the highest rating program is not a germane evidence The approximate number of listeners would be the more crucial evidence Therefore the author needs to seriously deliberate the correlation between jazzrsquos popularity in Monroe and his examples (Good logical flow and clarity)

Second the author uses as evidence the nationwide study that jazz fans spend much money on jazz entertainment to substantiate why starting a jazz club in Monroe will be profitable In other words the author assumes that the characteristics of a nationwide study can be applied to Monroe The national study would lend support to the applicantrsquos claim only if residents in Monroe typify national jazz fans However the author does not provide credible evidence that this is the case Moreover the populations of jazz fans nationwide may be insubstantial Thus the author should not infer hastily that Monroersquos residents will spend much money on enjoying jazz from the nationwide study

Lastly even if jazz is popular in Monroe C Note may not be successful It is entirely possible that residents might still prefer other clubs where they have always went In addition there is another possibility that the nearest jazz club will attract many of Monroersquos people because it serves fine performances and is equipped with favorite facilities Without considering these other possibilities the author cannot make his argument convincing In sum the author presents many reasons that are insufficient in supporting his or her claim In order for the authorrsquos claims to be convincing he needs to advance more persuasive evidence such as the total number of Monroe residents who attended the jazz festival the effects on the popularity of jazz by the musicians living in Monroe and the actual number of residents who would typify themselves to be jazz fans through a local survey Without substantial evidence that C Note will be successful in Monroe the businessmen may be overinvesting in what might lead to a business failureExcellent clarity Score 50

The following appeared in a newsletter offering advice to investors

Over 80 percent of the respondents to a recent survey indicated a desire to reduce their intake of foods containing fats and cholesterol and today low-fat products abound in many food stores Since many of the food products currently marketed by Old Dairy Industries are high in fat and cholesterol the companys sales are likely to diminish greatly and company profits will no doubt decrease We therefore advise Old Dairy stockholders to sell their shares and other investors not to purchase stock in this company

Write a response in which you discuss what questions would need to be answered in order to decide whether the advice and the argument on which it is based are reasonable Be sure to explain how the answers to these questions would help to evaluate the advice

Survey 80

GRE AWA John박 박정어학원

Old Dairy could change their products and manufacture low fat dairy foodsLess competing companies Old Dairy could eventually be the only company that produces hellipImprecise numbers and measurementsCustomers may still buy high fat dairy products

The author of the newsletter is offering potentially dangerous advice by recommending Old Dairy stockholders to withdraw investment and stop purchase What is more the authorrsquos prediction debases the reputation and business of Old Dairy and if false could devoid the investment opportunity of the newsletter readers Therefore investors should examine whether the authorrsquos evidences are substantial

To begin with the author states that 80 percent of the respondents in a survey indicated a desire to reduce their intake of foods He therefore argues that Old Dairyrsquos high fat and cholesterol products would decrease in sales However the author makes a crucial error in this argument First the author provides no evidence that the surveyrsquos results are statistically reliable Were they representative of all the customers Were they chosen for the survey randomly Furthermore the desire to reduce fat and cholesterol intake is a pervasive trend in todayrsquos opulent society however the author erroneously identifies this as a new phenomenon which will affect consumer trends Second having a desire to reduce fat and cholesterol intake does not necessarily indicate that people who have this desire will actually reduce consuming these types of products It is entirely possible that they may continue buying Old Dairy products for its quality and taste Accordingly the author cannot draw any firm conclusion that people will not buy Old Dairy products Therefore if any of these cases are true the author may be offering investors a detrimental investment advice

최근 조사에 대한 응답자중 80 이상이 자신이 먹는 음식에서 지방과 콜레스테롤의 함유량을 줄이고 싶다고 한다 아울러 요즘은 많은 식료품 가계에서 저지방 제품들을 많이 취급하고 있다 현재 Old Dairy Industries가 판매하고 있는 많은 음식제품들은 지방과 콜레스테롤이 높기 때문에 이 회사의 매출이 격감할 것으로 보이며 당연히 매출이익도 줄어들것이다 따라서 이 회사의 주주들은 주식을 매각하고 다른 주식 투자가들도 이 회사의 주식을 매입하지 않는 것이 좋다

결론 Old Dairy stockholders to sell their shares and other investors not to purchase stock in this company

반박 모든 상품이 다 고 지방 고 칼로리는 아니다(비록 많을지라도) 일부의 식품의 경우 기호에 맞어서 히트해서 전체적인 수입이 증가할 수도 있다국내시장만 생각할 수 없다( 외국시장에서 호황을 누릴수 있다 )입맛이라는게 즉각 바뀌는게 아니다

The following appeared in a letter to the editor of the Balmer Island Gazette

On Balmer Island where mopeds serve as a popular form of transportation the population increases to 100000 during the summer months To reduce the number of accidents involving mopeds and pedestrians the town council of Balmer Island should limit the number of mopeds rented by the islands moped rental companies from 50 per day to 25 per day during the summer season By limiting the number of rentals the town council will attain the 50 percent annual reduction in moped accidents that was achieved last year on the neighboring island of Seaville when Seavilles town council enforced similar limits on moped rentals

1 Write a response in which you discuss what questions would need to be answered in order to decide whether the recommendation is likely to have the predicted result Be sure to explain how the answers to these questions would help to evaluate the recommendation

2 Write a response in which you discuss what questions would need to be answered in order to decide whether the prediction and the argument on which it is based are reasonable Be sure to explain how the answers to these questions would help to evaluate the prediction

3 Write a response in which you examine the stated andor unstated assumptions of the argument Be sure to explain how the argument depends on these assumptions and what the implications are for the argument if the assumptions prove unwarranted

Whatrsquos the actual population of Balmer Island 100000mdashis this a significant increase What kind of accidents Skin abrasions or serious injury And compared to Seaville how serious are the accidents and the actual number of accidents Did Seaville enforce other restrictions like safety signsHow different are the conditions of Balmer

GRE AWA John박 박정어학원

and Seaville regarding population road (safety) conditions topography other town-government regulation How much will the economy of Balmer be affected do to this restriction Could it cause an economic recession due to the fact that these rental companiesrsquo chance to make money is only during the summer thereby weakening the economic infrastructure Are there any other ways that could better alleviate the accident rate

Statistics 50-impreciseAnalogy Balmer compared with TorseauFalse Cause Accidents might have occurred because of reasons other than mopeds False Cause population increase may not be part of the cause of the accidentsOther explanations for the accident pedestrians few road safety regulations narrow roadsThere could be other better solutionshellip

Balmer Island의 인구가 여름철에는 십만명으로 늘어난다 2륜차와 보행자간 사고를 줄이기 위해 시의회는 6개의 자전거를 포함한 2륜차 대여업체에게 이 기간동안에는 대여숫자를 일일 50에서 30으로 제한하도록 할 것이다 대여숫자를 줄임으로써 시 의회는 지난해 이웃한 Torseau섬에서 이와 동일한 규제를 시행해서 50나 줄인 결과를 보고 마찬가지로 50를 줄일수 있다고 확신하고 있다

결론 The town council of Balmer Island should linit the number

반박 보행자의 부실에 의해서 사고가 많이 일어날수도 있다렌탈수의 줄임만이 대책은 아니다(대부분의 사람들이 렌탈 보다는 소유하고 있을 수도 있다)옆섬과는 상황이 다를수도 있다(그 섬에서는 사고의 원인이 많은 자전거 수로 인한것일수있다) 하지만 이 섬은 좁은 도로가 원인일 수도 있고 도로 안전 장치의미비가 원일일수 있다

In this letter the author recommends that Balmer Island should limit the number moped rentals from 50 to 30 per day To support this recommendation the author points out several reasons However careful scrutiny of each of the facts reveals that it is filled with unanswered questions that could significantly weaken the authorrsquos recommendation with loops and holes which are answered

The recommendation depends on the assumption that no alternative means of reducing the number of accidents are available However the author fails to offer any evidence to substantiate this crucial assumption It is highly possible that means other than this would better solve the problem Perhaps they could widen the roads or put-up more safety signs Or perhaps the accidents were due to the lack of skills in which case proper safety training would significantly alleviate the problem Without considering and ruling out these and other alternative means of reducing accidetns the author cannot confidently conclude that merely emulating Torseau would suffice Moreover the author is advising a recommendation which could potentially harm the economy of Balmer Island sincehellip Moreover the Balmer Island should alternative means to reduce accidents because limiting moped rentals during the summer could harm the economy of Balmerhellip

First of all the author believes that increase in population and the number of moped rentals are responsible for the accidents It is entirely possible that other factors are responsible for the accidents Perhaps Balmer Islandrsquos lack of safety signs was a major factor Or maybe the roads are narrow and dangerous on the Island therefore the town council could enforce stricter traffic regulations to alleviate the problem Accordingly if either of these scenarios is true the author cannot draw any firm conclusion that increase in the number of population and moped rentals are the cause of the accidents

The author of this editorial recommends that to reduce accidents involving mopeds andpedestrians Balmer Islands city council should restrict moped rentals to 30 per day down from50 at each of the islands six rental outlets To support this recommendation the author citesthe fact that last year when nearby Torseau Islands town council enforced similar measuresTorseaus rate of moped accidents fell by 50 For several reasons this evidence providesscant support for the authors recommendationTo begin with the author assumes that all other conditions in Balmer that might affect therate of moped-pedestrian accidents will remain unchanged after the restrictions are enactedHowever with a restricted supply of rental mopeds people in Balmer might purchase mopedsinstead Also the number of pedestrians might increase in the future with more pedestriansespecially tourists the risk of moped-pedestrian accidents would probably increase For thatmatter the number of rental outlets might increase to make up for the artificial supplyrestriction per outlet--a likely scenario assuming moped rental demand does not declineWithout considering and ruling out these and other possible changes that might contribute to ahigh incidence of moped-pedestrian accidents the author cannot convince me that theproposed restrictions will necessarily have the desired effect

GRE AWA John박 박정어학원

Next the author fails to consider other possible explanations for the 50 decline inTorseaus moped accident rate last year Perhaps last year Torseau experienced unusually fairweather during which moped accidents are less likely Perhaps fewer tourists visited Tot seanlast year than during most years thereby diminishing the demand for rental mopeds to belowthe allowed limits Perhaps last year some of Torseaus moped rental outlets purchased newmopeds that are safer to drive Or perhaps the restrictions were already in effect but were notenforced until last year In any event a decline in Torseaus moped accident rate during onlyone year is scarcely sufficient to draw any reliable conclusions about what might have causedthe decline or about what the accident rate will be in years aheadAdditionally in asserting that the same phenomenon that caused a 50 decline in mopedaccidents in Torseau would cause a similar decline in Balmer the author relies on what mightamount to an unfair analogy between Balmer and Torseau Perhaps Balmers ability to enforcemoped-rental restrictions does not meet Torseaus ability if not then the mere enactment ofsimilar restrictions in Balmer is no guarantee of a similar result Or perhaps the demand formopeds in Torseau is always greater than in Balmer Specifically if fewer than all availablemopeds are currently rented per day from the average Balmer outlet while in Torseau everyavailable moped is rented each day then the proposed restriction is likely to have less impacton the accident rate in Balmer than in TorseauFinally the author provides no evidence that the same restrictions that served to reduce theincidence of all moped accidents by 50 would also serve to reduce the incidence ofaccidents involving mopeds and pedestrians by 50 Lacking such evidence it is entirelypossible that the number of moped accidents not involving pedestrians decreased by a greaterpercentage while the number of moped-pedestrian accidents decreased by a smallerpercentage or even increased Since the author has not accounted for these possibilities theeditorials recommendation cannot be taken seriouslyIn conclusion the recommendation is not well supported To convince me that the proposedrestriction would achieve the desired outcome the author would have to assure me that nochanges serving to increase Balmers moped-pedestrian accident rate will occur in theforeseeable future The author must also provide dear evidence that last years decline inmoped accidents in Torseau was attributable primarily to its moped rental restrictions ratherthan to one or more other factors In order to better evaluate the recommendation I wouldneed more information comparing the supply of and demand for moped rentals on the twoislands I would also need to know the rate of mopedpedestrian accidents in Torseau both priorto and after the restrictions were enforced in TorseauThe following appeared in a magazine article about planning for retirement

Clearview should be a top choice for anyone seeking a place to retire because it has spectacular natural beauty and a consistent climate Another advantage is that housing costs in Clearview have fallen significantly during the past year and taxes remain lower than those in neighboring towns Moreover Clearviews mayor promises many new programs to improve schools streets and public services And best of all retirees in Clearview can also expect excellent health care as they grow older since the number of physicians in the area is far greater than the national average

Write a response in which you discuss what specific evidence is needed to evaluate the argument and explain how the evidence would weaken or strengthen the argument

-Natural beauty and consistent climate may not be the most wanted qualities-Housing costs could have lowered on a national level wealthy retirees may not care about costs-Taxes may be high compared to the nationrsquos average tax rate-What about other qualities of Clearview Crime rate what qualities would retirees want -If schools streets and public services need improvement then this is proof that the current condition of Clearview is low Or due to budgetary reasons the mayor may not follow-up on his promise because of lowered tax rate -Schools and people who are retired no relationship-Physicians What kind of physicians Number is irrelevant Are these physicians capable of addressing the illnesses of old people

This author argues that anyone seeking a place to retire should choose Clearview To supportthis argument the article cites Clearviews consistent climate and natural beauty its fallinghousing costs its low property taxes compared to nearby towns and the mayors promise toimprove schools streets and services The article also claims that retirees can expectexcellent health care because the number of physicians in Clearview greatly exceeds thenational average This argument is flawed in several critical respectsTo begin with although consistent climate and natural beauty might be attractive to manyretirees these features are probably not important to all retirees For many retirees it isprobably more important to live near relatives or even to enjoy changing seasons Thus I

GRE AWA John박 박정어학원

cannot accept the authors sweeping recommendation for all retirees on this basisAlso Clearviews declining housing costs do not necessarily make Clearview the best placeto retire for two reasons First despite the decline Clearviews housing costs might be highcompared to housing costs in other cities Secondly for wealthier retirees housing costs arenot likely to be a factor in choosing a place to retire Thus the mere fact that housing costshave been in decline lends scant support to the recommendationThe articles reliance on Clearviews property-tax rates is also problematic in two respectsFirst retirees obviously have innumerable choices about where to retire besides Clear viewand nearby towns Secondly for retirees who are well-off financially property taxes are notlikely to be an important concern in choosing a place to retire Thus it is unfair to infer fromClearviews property-tax rates that retirees would prefer ClearviewYet another problem with the argument involves the mayors promises In light of Clearviewslow property-tax rates whether the mayor can follow through on those promises is highlyquestionable Absent any explanation of how the city can spend more money in the areas citedwithout raising property taxes I simply cannot accept the editorials recommendation on thebasis of those promises Besides even if the city makes the improvements promised thoseimprovements--particular the ones to schools--would not necessarily be important to retireesFinally although the number of physicians in Clearview is relatively high the per capitanumber might be relatively low Moreover it would be fairer to compare this per capita numberwith the per capita number for other attractive retirement towns--rather than the nationalaverage After all retirees are likely to place a relatively heavy burden on health-careresources Besides the article provides no assurances that the number of physicians inClearview will remain high in the foreseeable futureIn conclusion the recommendation is poorly supported To strengthen it the author mustconvince me--perhaps by way of a reliable survey--that the key features that the vast majorityof retirees look for in choosing a place to live are consistent climate natural beauty and lowhousing costs The author must also provide better evidence that Clear views property taxesare lower than the those of cities in other areas The author must also explain how the city canmake its promised improvements without raising property taxes Finally to better assess theargument I would need to now how the per capita number of physicians in Clearview wouldcompare to the national average in the futureThe following appeared as a letter to the editor from a Central Plaza store owner

Over the past two years the number of shoppers in Central Plaza has been steadily decreasing while the popularity of skateboarding has increased dramatically Many Central Plaza store owners believe that the decrease in their business is due to the number of skateboard users in the plaza There has also been a dramatic increase in the amount of litter and vandalism throughout the plaza Thus we recommend that the city prohibit skateboarding in Central Plaza If skateboarding is prohibited here we predict that business in Central Plaza will return to its previously high levels

Write a response in which you discuss what questions would need to be answered in order to decide whether the recommendation is likely to have the predicted result Be sure to explain how the answers to these questions would help to evaluate the recommendation

Why two years ago What happened two years ago which started this declineIs the dramatic increase in the ldquopopularityrdquo of skateboarding the cause of the steady decline of shoppers Are there any malls nearby Were there any changes nearby which could affect the decline in customersmdasha big mall perhaps Could the decline be due to the shop ownersHow many skateboarders use the plazaWhere do they skateboardDo they shop and are they customersAre the increase in litter and vandalism due to skateboarders Could this be alleviated by installing CCTVs and hiring security

This editorial concludes that the city should ban skateboarding from its downtown CentralPlaza in order to attract visitors to that area to return the area to its former glory and to makeit a place where people can congregate for fun and relaxation To justify this conclusion theeditorial points out that skateboarders are nearly the only people one sees anymore at CentralPlaza and that the Plaza is littered and its property defaced The editorial also points out thatthe majority of downtown merchants support the skate boarding ban This argument is flawedin several critical respectsFirst the editorials author falsely assumes that a ban on skateboarding is both necessaryand sufficient to achieve the three stated objectives Perhaps the city can achieve thoseobjectives by other means as well--for example by creating a new mall that incorporates anattractive new skateboard park Even if banning skateboarders altogether is necessary to meetthe citys goals the author has not shown that this action by itself would suffice Assuming thatthe Plazas reputation is now tarnished restoring that reputation and in turn enticing peopleback to the Plaza might require additional measures--such as removing litter and graffiti

GRE AWA John박 박정어학원

promoting the Plaza to the public or enticing popular restaurant or retail chains to the PlazaSecondly the editorial assumes too hastily that the Plazas decline is attributable to theskateboarders--rather than to some other phenomenon Perhaps the Plazas primary appeal inits glory days had to do with particular shops or eateries which were eventually replaced byless appealing ones Or perhaps the crime rate in surrounding areas has risen dramatically forreasons unrelated to the skateboarders presence at the Plaza Without ruling out these andother alternative explanations for the Plazas decline the editorials author cannot convince methat a skateboard ban would reverse that declineThirdly the editorials author might be confusing cause with effect--by assuming that theskateboarders caused the abandonment of the Plaza rather than vice versa It is entirelypossible that skateboarders did not frequent the Plaza until it was largely abandoned--andbecause it had been abandoned In fact this scenario makes good sense since skateboardingis most enjoyable where there are few pedestrians or motorists to get in the wayFourth it is unreasonable to infer from the mere fact that most merchants favor the ban thatthe ban would be effective in achieving the citys objectives Admittedly perhaps thesemerchants would be more likely to help dean up the Plaza area and promote their businesseswere the city to act in accordance with their preference Yet lacking any supporting evidencethe author cannot convince me of this Thus the survey amounts to scant evidence at best thatthe proposed ban would carry the intended resultFinally the author recommends a course of action that might actually defeat the citysobjective of providing a fun and relaxing place for people to congregate In my experienceskateboarding contributes to an atmosphere of fun and relaxation for adults and children alikemore so than many other types of ambiance Without considering that continuing to allowskateboarding--or even encouraging this activity--might achieve the citys goal more effectivelythan banning the activity the author cannot convincingly conclude that the ban would be in thecitys best interestsIn sum the argument is a specious one To strengthen it the editorials author must providedear evidence that skateboarding and not some other factor is responsible for the conditionsmarking the Plazas decline The author must also convince me that no alternative means ofrestoring the Plaza are available to the city and that the proposed ban by itself would suffice toattract tourists and restore the Plaza to its former glory Finally to better assess the argument itwould be useful to know the circumstances under which the downtown merchants would bewilling to help the city achieve its objectives

6그룹 약한 비유 빈출

The following recommendation appeared in a memo from the mayor of the town of Hopewell

Two years ago the nearby town of Ocean View built a new municipal golf course and resort hotel During the past two years tourism in Ocean View has increased new businesses have opened there and Ocean Views tax revenues have risen by 30 percent Therefore the best way to improve Hopewells economymdashand generate additional tax revenuesmdashis to build a golf course and resort hotel similar to those in Ocean View

Write a response in which you examine the stated andor unstated assumptions of the argument Be sure to explain how the argument depends on these assumptions and what the implications are for the argument if the assumptions prove unwarranted

GRE AWA John박 박정어학원

Assumptions The author assumes that OVrsquos municipal golf course and resort hotel caused tourism new businesses and increased tax revenues There may be other reasons advertising promo He assumes that this will continueAssumes that Ocean View and Hopewell are similar in many waysmdashthe name suggests otherwise OV may have always been a tourist attractions for its beaches We need to know the topography

2년전 Ocean View 시는 시정 소유 골프 및 휴양지 호텔을 신축했다 그리고 지난 2년동안 이 시의 관광객이 증가했으며 새로운 사업들이 생겨났다 그에따라 시의 세수도 30나 증가했다 Hopewell의 경제를 향상시키고 아울러 세수를 늘릴 수 있는 가장 좋은 방법은 Ocean View에 세워진 것과 같은 골프 시설과 휴양지 호텔을 신축하는 것이다

1 다른 요인으로 관광 산업이 발전했을 수도 있다 문화 유적이 발견이 되었거나 도로의 정비등으로 여행자가 늘었을 수도 있다

2 관광 산업의증가가 늘어난 세수의 원인이 아니라 새로 유입된 인구의 증가나 다른 공장에서 발생한 것일 수 있다

3 2년동안 한참 골프가 붐을 이루었을 수 있다 경제상황이 나빠지거나 다른 레포츠가 인근 지역에 생겨난다면 골프하는 사람이 줄어들 수 있다

In this memo HopeweUs mayor recommends that in order to stimulate the towns economyand boost tax revenues HopeweU should build a new golf course and resort hotel just as thetown of Ocean View did two years ago To support this recommendation the mayor points outthat in Ocean View during the last two years tourism has increased new businesses haveopened and tax revenues have increased by 30 I find the mayors argument unconvincingin several important respectsFirst of all it is possible that the mayor has confused cause with effect respecting the recentdevelopments in Ocean View Perhaps Ocean Views construction of a new golf course andhotel was a response to previous increases in tourism and business development increasesthat have simply continued during the most recent two years Since the mayor has failed toaccount for this possibility the claim that Hopewell would boost its economy by alsoconstructing a golf course and hotel is completely unwarrantedSecondly the mayor fails to account for other possible causes of the trends in Ocean Viewduring the last two years The increase in tourism might have been due to improving economicconditions nationwide or to unusually pleasant weather in the region The new businessesthat have opened in Ocean View might have opened there irrespective of the new golf courseand hotel And the 30 increase in tax revenues might have been the result of an increase intax rates or the addition of a new type of municipal taxWithout ruling out these and other alternative explanations for the three recent trends inOcean View the mayor cannot reasonably infer based on those trends that Hopewellseconomy would benefit by following Ocean Views exampleThirdly even if the recent trends in Ocean View are attributable to the construction of the newgolf course and hotel there the mayor assumes too hastily that the golf course and hotel willcontinue to benefit that towns overall economy The mayor has not accounted for thepossibility that increased tourism will begin to drive residents away during tourist season orthat new business development will result in the towns losing its appeal as a place to visit or tolive Unless the mayor can convince me that these scenarios are unlikely I cannot accept themayors recommendation that Hopewell follow Ocean Views exampleFinally the mayors argument rests on the unsubstantiated assumption that Hopewell andOcean View are sufficiently alike in ways that might affect the economic impact of a new golfcourse and hotel Hopewell might lack the sort of natural environment that would attract moretourists and new businesses to the town--regardless of its new golf course and hotel For thatmatter perhaps Hopewell already contains several resort hotels and golf courses that are notutilized to their capacity If so building yet another golf course and hotel might amount to amisallocation of the towns resources--and actually harm the towns overall economyIn sum the mayors recommendation is not well supported To bolster it the mayor mustprovide better evidence that Ocean Views new golf course and hotel and not some otherphenomenon--has been responsible for boosting Ocean Views economy during the last twoyears To better assess the recommendation I would need to know why Ocean View decidedto construct its new golf course and hotel in the first place--specifically what events prior toconstruction might have prompted that decision I would also need to thoroughly compare

GRE AWA John박 박정어학원

HopeweU with Ocean View--especially in terms of their appeal to tourists and businesses--todetermine whether the same course of action that appears to have boosted Ocean Viewseconomy would also boost Hopewells economy

The following is part of a memorandum from the president of Humana University

Last year the number of students who enrolled in online degree programs offered by nearby Omni University increased by 50 percent During the same year Omni showed a significant decrease from prior years in expenditures for dormitory and classroom space most likely because instruction in the online programs takes place via the Internet In contrast over the past three years enrollment at Humana University has failed to grow and the cost of maintaining buildings has increased along with our budget deficit To address these problems Humana University will begin immediately to create and actively promote online degree programs like those at Omni We predict that instituting these online degree programs will help Humana both increase its total enrollment and solve its budget problems

Write a response in which you discuss what questions would need to be answered in order to decide whether the prediction and the argument on which it is based are reasonable Be sure to explain how the answers to these questions would help to evaluate the prediction

Is Omni University successful due to the online degree program 50 Is the decrease in expenditures for dormitory and classroom space due to the decrease in of on-campus students Which classes were successful Does HU have those classes

Even if the long-distance degree programs at Omni University benefited the school the presidentrsquos recommendation that Human College should emulate Omni University is too hasty First OUrsquos name implies that the school would have more majors than Humanahellip the president should examine which degrees were in the long-distance programhellip

지난해에는 Omni 대학에서 개강했던 원거리 학생 학점 취득 프로그램을 등록했던 학생들의 숫자가 50나 증가했다 같은해 기간동안 Omni 대학에서는 그 전년도부터 기숙사와 학급의 공간 확충을 위한 예산을 대폭 줄였는데 이는 이 원거리 학점 취득 프로그램이 양방향 비디오 컴퓨터 접속을 통해서만 가능한 수업지도 방식이기때문인 것으로 보인다 반면 지난 3개년 동안 Humana 대학에서의 수강률은 감소한데다가 건물

GRE AWA John박 박정어학원

유지비도 올랐다 따라서 Humana대학의 수강을 늘리고 예산손실을 회복하기 위해서는 Omni 대학에서 취한 조치와 같은 능동적인 프로그램을 추진해야 한다

결론 we should initiate and actively promote long-distance degree programs like those at Omni 반박 원거리 학생 취득 프로그램 숫자가 증가한거하고 예산이 줄어드는 것 사이에 연관이 약하다 (causal 학생의 증가로 관리비용 증가할수 있음 원거리 수업가능 장비도입에의한 비용발생)bad analogy(omni university 하고 같은 조건이 아니다 )-gt omni college 가 강좌내용이 좋아서 학생의 등록이 많을수 있다 Humana 대학에서 만들었다 하더라도 인기 없을수 있음다른 요인에 의해서 Humana 대학의 수강 인원이 증가할수 있음(비록 과거엔 인기가 없었을지라도)

The following appeared as part of a business plan developed by the manager of the Rialto Movie Theater

Despite its downtown location the Rialto Movie Theater a local institution for five decades must make big changes or close its doors forever It should follow the example of the new Apex Theater in the mall outside of town When the Apex opened last year it featured a video arcade plush carpeting and seats and a state-of-the-art sound system Furthermore in a recent survey over 85 percent of respondents reported that the high price of newly released movies prevents them from going to the movies more than five times per year Thus if the Rialto intends to hold on to its share of a decreasing pool of moviegoers it must offer the same features as Apex

Write a response in which you discuss what questions would need to be answered in order to decide whether the recommendation is likely to have the predicted result Be sure to explain how the answers to these questions would help to evaluate the recommendation

Before following through this business plan the manager should investigate the cause of Rialtorsquos unsuccessful business

The author provides no evidence that the surveyrsquos results are statistically reliable The surveyrsquos sample of 85 percent must be sufficient in size and representative of overall population of the city where Rialto and Apex is serving Lacking evidence of a sufficiently representative sample the author cannot justifiably rely on the survey to draw any conclusion whatsoever The author does not indicate that Apex is indeed currently successful However even if Apex is enjoying success the argument relies on what might be a false analogy between Rialto and Apex In order for Apex to serve as a model that Rialto should emulate the author must assume that all relevant circumstances are essentially the same However this assumption is unwarranted For example the argument overlooks the face that Apex is located in a strategic placemdashbeside a mall where customers can not only watch a movie but also enjoy shopping Therefore simply changing the facility to that of Apex may not lead to success

The author does not mention whether Apex is successful or not Nevertheless even if Apex is currently successful the argument relies on what might be a false analogy between Rialto and Apex In order for Apex to serve as a model that Rialto should emulate the author must assume that all relevant circumstances are essentially the same However this assumption is unwarranted For example the argument overlooks the fact that these two institutions are located in different locations Rialto in downtown and Apex in a mall outside of town Although Apex opened with state-of-the-art facilities the decisive factor in its success could be due to its strategic location of being in a mall People could enjoy both shopping and movies at one location thus they may prefer Apex over Rialto Furthermore the place where people enjoy leisure activities has shifted in the past decades for most cities from downtown to the suburbs Therefore Rialto may not be successful even if it emulates Apexrsquos facilities A better business plan may be relocating Apex to the thriving section of the downtown

Rialto 극장은 지난 50여년간 지역 회관으로써 시내에 위치해 있으면서도 이제 변화를 꾀하지 않으면 문을 닫을

GRE AWA John박 박정어학원

판이다 이 극장은 시외 쇼핑타운에 새로 들어선 Apex 극장의 사례를 본받아야 했다 Apex가 지난해 개업했을 당시 이 극장은 비디오 아케이드 플러쉬 카펫트 바닥과 좌석 그리고 최신 음향시설을 갖추었다 더군다나 최근 조사에서는 응답자의 85 이상이 새로 출시된 영화 입장료가 비싼 탓으로 지난해보다 5배이상의 관람객이 줄어들었다고 나타났다 따라서 Rialto 극장이 줄어들고 있는 관람객을 뺐기지 않고 유지하려면 Apex와 같은 시설들을 갖추어야 할 것이다주장 리알토 극장이 줄어들고 있는 관람객을 뺐기지 않고 유지하려면 Apex와 같은 시설들을 갖추어야 할 것이다

1 조사에서 응답자가 전체를 대표할 수 없다 2 apex 극장이 좋은 시설을 갖추고 있지만 그로 인해 수익이 많이 발생했다는 말이 없으므로 시설투자를

하고도 좋은 결과를 얻을 수 있을지 그 근거가 미흡하다3 좋은 영화가 출시된다면 입장료가 비싸도 영화관에서 꼭 보려고 할 수 있다 4 rialto 가 시설이 아닌 다른 요인에 의해 장사가 안될수도 있다( 우범 지역이라든지)

The following is a recommendation from the business manager of Monarch Books

Since its opening in Collegeville twenty years ago Monarch Books has developed a large customer base due to its reader-friendly atmosphere and wide selection of books on all subjects Last month Book and Bean a combination bookstore and coffee shop announced its intention to open a Collegeville store Monarch Books should open its own in-store cafeacute in the space currently devoted to childrens books Given recent national census data indicating a significant decline in the percentage of the population under age ten sales of childrens books are likely to decline By replacing its childrens books section with a cafeacute Monarch Books can increase profits and ward off competition from Book and Bean

Write a response in which you examine the stated andor unstated assumptions of the argument Be sure to explain how the argument depends on these assumptions and what the implications are for the argument if the assumptions prove unwarranted

The following is a recommendation from the business manager of Monarch Books

Since its opening in Collegeville twenty years ago Monarch Books has developed a large customer base due to its reader-friendly atmosphere and wide selection of books on all subjects Last month Book and Bean a combination bookstore and coffee shop announced its intention to open a Collegeville store Monarch Books should open its own in-store cafeacute in the space currently devoted to childrens books Given recent national census data indicating a significant decline in the percentage of the population under age ten sales of childrens books are likely to decline By replacing its childrens books section with a cafeacute Monarch Books can increase profits and ward off competition from Book and Bean

1 Write a response in which you discuss what questions would need to be answered in order to decide whether the recommendation is likely to have the predicted result Be sure to explain how the answers to these questions would help to evaluate the recommendation

2 Write a response in which you discuss what specific evidence is needed to evaluate the argument and explain how the evidence would weaken or strengthen the argument

No evidence regarding Monarch Bookrsquos successEven if Regal Bookrsquos is successful this may not be attributable to the cafeacute False analogy Emulating may not lead to success Other factors may be involvedInsufficient condition The national census is not enough evidence that childrenrsquos book sales will decline Can

GRE AWA John박 박정어학원

the national census represent the local child populationDid opening a cafeacute boost sales for Regal Books Even assuming Regal is successful by opening a cafeacute this may not be suitable for Monarch which plans to close the childrenrsquos book section to establish a cafe Imprecise language ldquorelatively little spacerdquo how smallThe managerrsquos recommendation contradicts what he says Since Monarch is popular for its wide selection of books closing a selection which targets a major group of readers may hurt Monarchrsquos salesIs this the best way to compete

When Stanley Park first opened it was the largest most heavily used public park in town It is still the largest park but it is no longer heavily used Video cameras mounted in the parks parking lots last month revealed the parks drop in popularity the recordings showed an average of only 50 cars per day In contrast tiny Carlton Park in the heart of the business district is visited by more than 150 people on a typical weekday An obvious difference is that Carlton Park unlike Stanley Park provides ample seating Thus if Stanley Park is ever to be as popular with our citizens as Carlton Park the town will obviously need to provide more benches thereby converting some of the unused open areas into spaces suitable for socializing

Write a response in which you examine the stated andor unstated assumptions of the argument Be sure to explain how the argument depends on these assumptions and what the implications are for the argument if the assumptions prove unwarranted

Stanley 파크가 처음 개장했을 당시 가장 크고 가장 많이 이용되는 공원이었다 아직도 공원중에서는 가장 크지만 이용률은 상당히 떨어졌다 지난달 공원 주차장에 설치해놓은 비디오 카메라를 통해 보면 drop(주차장으로 여겨짐) 이용률이 가장 높았다 수치상으로는 하루 평균 50대의 차량만이 이용하였다 반면 직장 중심거리에 위치한 작은 규모의 Carlton 파크는 주당 무려 150여명 이상이 이용하고 있다 Stanley 파크와는 달리 Carlton 파크에는 의자가 있다는 것이 가장 뚜렷한 차이점이다 따라서 Stanley 파크가 Carlton 파크처럼 시민들이 자주 이용하는 공원이 되기 위해서는 벤치를 설치할 필요가 있으며 이렇게 사용되지 않는 일부 공간을 활용해서 사교를 위한 공간으로 바꾸어야 한다 ===gtdrop 에 대한 첨부사항 (영영사전내용입니다)---- a place or central depository to which something (as mail money or stolen property) is brought for distribution or transmission also the act of depositing something at such a place dropgt

주장 if Stanley Park is ever to be as popular with our citizens as is Carlton Park the town will obviously need to provide more benches thereby converting some of the unused open areas into spaces suitable for socializing1 조사가 언제 이루어진 것인가 조사가 언제 실시되었느냐에 따라 결과가 다를 수있다 현재는 다시 스탠리 파크가 늘어났었을 수 있다 2 벤치를 많이 설치했다고 해서 많은 관광객이 오지 않을수 있다(사람들이 벤치나 사교 공간을 원한다는 어떠한 자료도 없다)3스탠리 파크 주변에 교통 상황이 악화가 되었거나 칼튼 파크에서 문화행사등을 많이 가져서 이용객이 줄어든것일 수도 있다 4 칼튼 파크가 중심지에 있어서 접근성이 좋을수 있다5 조사가 같은 시간을 기준으로 한게 아니다(하나는 주중이고 하나는 주말이다)6사람의 수와 차의 대수를 같은것으로 비교할수 없다 (차안에 몇 명이 타고 있는지 모르고 대중교통을 이용해서 왔을수도 있다)

Page 7: GRE writing argument brain storm

GRE AWA John박 박정어학원

주장 By increasing our Wheat-O cereal with soy protein we can multiply sales

1 콩 단백질의 함유를 강화해도 그수치가 미흡하거나 흡수가 잘 안되거나 할 수 있다 다른 성분과 섞여서 만들어지는 것이므로 다른 성분에 의해서 그 흡수가 잘 안되거나 효과가 낮을 수 있다 2 강화한 것이 건강에 관심이 있는 소비자에게 어필한다고 해도 그 수가 적어서 매출에 큰 영향을 안줄 수 있다3 다른 음식에 의해서 콜레스테롤이 더 낮아질수 있다 4 콜레스테롤을 낮추는 것이 사람들에게 그들의 건강생활을 위해 크게 어필 안할수도 있다( 저지방식이라든지 다른 건강을 높이는 방법이 더 관심이 많을수 있다)

In a study of the reading habits of Waymarsh citizens conducted by the University of Waymarsh most respondents said that they preferred literary classics as reading material However a second study conducted by the same researchers found that the type of book most frequently checked out of each of the public libraries in Waymarsh was the mystery novel Therefore it can be concluded that the respondents in the first study had misrepresented their reading habits

Write a response in which you discuss what specific evidence is needed to evaluate the argument and explain how the evidence would weaken or strengthen the argument

Evidence needed -Information about the respondentsmdashage gender class precise number of respondents-Time-shift After how many years was the second study conductedmdashtaste in books may change over time-Can library records sufficiently evidence the reading habits of Waymarsh citizens

1)2)3)4)5)6) First the author provides no evidence that the surveyrsquos studyrsquos results are statistically reliable Lacking

information about the precise methodology of the study the number of (customers)respondents surveyed and the number of respondentsvarious information such as gender age and social background which are essential to bolster the conclusion it is impossible to assess the validity of the results It is possible that people who feel inclined to( take low fat and low cholesterol foods ) read literary classics were more willing to respond to the survey than were others Another problem is the representativeness of the respondents Were they representative of all the customersWaymarsh citizens Were they chosen for the survey randomly Lastly the survey results must depend on the honesty and integrity of the respondents Without more information about the survey the author cannot simply conclude that (most of customers want to take low fat and low cholesterol foods)Waymarsh citizens misrepresented their reading habit on the first study on the basis of this surveystudy(survey reliability)

The exact number of books checked-out should be notedMystery novels could have been in trend at the time of the studyPublic libraries vs other libraries such as Leeville University library

Leeville 대학에서 실시한 Leeville 주민의 독서습관에 대한 조사에서 대부분의 응답자는 독서용으로 문학작품을 선호한다고 응답했다 그러나 같은 조사팀에서 이어 실시한 조사에서는 Leeville에 있는 도서관마다 가장 빈번하게 연람된 책의 종류를 보면 미스테리 소설이었던 것으로 조사되었다 따라서 최초 조사 응답자들이 독서 습관에 대해 잘못 말했다고 결론지을 수 있다

결론 it can be concluded that the respondents in the first study had misrepresented their reading habits1 첫번째 조사 그룹하고 두번째 조사 그룹하고 다른 그룹일 수 있다

GRE AWA John박 박정어학원

2 도서관에 비치된 책이 미스터리 소설이 더 많아서 선택의 여지가 없을 수 있다

High Frequency Group 2 National-gtLocal Local-gtNational amp Group Member Error

The following appeared in a letter to the editor of a Batavia newspaper

The department of agriculture in Batavia reports that the number of dairy farms throughout the country is now 25 percent greater than it was 10 years ago During this same time period however the price of milk at the local Excello Food Market has increased from $150 to over $300 per gallon To prevent farmers from continuing to receive excessive profits on an apparently increased supply of milk the Batavia government should begin to regulate retail milk prices Such regulation is necessary to ensure fair prices for consumers

Write a response in which you discuss what questions would need to be answered in order to decide whether the recommendation is likely to have the predicted result Be sure to explain how the answers to these questions would help to evaluate the recommendation

First the author points to the fact that hellip25 increase of dairy farms may be(could be might be) due to population increaseThe milk price increase could be reflecting the increase in cost of livingThe actual price of milk might be cheaper if the economy is experiencing inflationOne market Excello Food cannot reflect the price increase of every market in BataviaThe author claims that farmers are receiving excessive profits however this might not be the caseThe author claims that by regulating milk prices the government will successfully ensure both lower prices and an adequate supply of milk for consumers However this might not be the best solutionThe writer adduces the information released by the department of agriculture that the number of dairy farms has

increased by 25 percent over the last 10 years and thus he claims that dairy farmers are earning excessive profits from milk However the information is filled with loops and holes which need to be filled First what percentage of the milk are sold as milk

Are the dairy farms processing the milk into other dairy products Or are they selling them as milkmdashWhat percentage of the milk is sold as milk Could this be a natural increase

Can the price at the local Excello Food Market represent the price of milk in stores throughout all of Batavia

Are the farmers receiving excessive profits What is the cost of producing milk Could such regulation offer fair prices to consumers

Topic SentenceTo begin with the author assumes that the price of Excello Food Market which has increased from $150 to over $300 per gallon can be applied to the whole nation of Batavia however the author fails to provide evidence to substantiate this assumptionExampleFirstIn additionTherefore in order to make the argument more reliable the author should consider the prices of a large number markets all over the countryThere could be an economic inflation

GRE AWA John박 박정어학원

ldquoBatavia 지역의 농림부의 보고에 따르면 전국적으로 낙농업자의 숫자가 과거 10 년전에 비해 25나 증가했다고 합니다 그러나 동기간 동안 이 지역에 있는 Excello Food Market 에서 우유의 가격은 갤런당 15 불에서 3 불로 증가했습니다 증가된 우유 공급량에 맞춘 낙농업자들의 수익률 증대를 보장하기 위해서는 주 정부가 소매 공급가에 대한 규제를 해야 합니다 소비자에게 보다 저렴한 가격에 안정된 물량을 공급할 수 있도록 하기 위해서는 이러한 조치가 필요합니다

결론 the Batavia government should begin to regulate retail milk prices

반박 ( Excello Food Market doesnrsquot reflect the whole of Batavia) 우유값 증가가 단순히 inflation 을 따라간것일수도 있다 가격상승이 수익을 보장하지 않는다( 원자재 상승등으로 인해서)우유값 규제만이 최선의 방법이 아니다( 물량 조절)낙농업자의 수익보호도 필요하다

A recent sales study indicates that consumption of seafood dishes in Bay City restaurants has increased by 30 percent during the past five years Yet there are no currently operating city restaurants whose specialty is seafood Moreover the majority of families in Bay City are two-income families and a nationwide study has shown that such families eat significantly fewer home-cooked meals than they did a decade ago but at the same time express more concern about healthful eating Therefore the new Captain Seafood restaurant that specializes in seafood should be quite popular and profitable

Write a response in which you discuss what questions would need to be addressed in order to decide whether the conclusion and the argument on which it is based are reasonable Be sure to explain how the answers to the questions would help to evaluate the conclusion

BAY CITY The name implies that this is a port city which would have traditionally consumed seafood 30--what type of seafood Unprocessed or processed cans of tuna and sardines If the latter increased Captain Seafood may not profit 30--natural inflation Are the non-seafood restaurants selling seafood If yes this may suffice since patrons would not easily dine at an unfamiliar restaurant Nationwide studyrsquos representativeness Desire does not lead to action

Nationwide study applied to Bay City Eating fewer home cooked meals than they did a decade agoHealthy food The author needs to show that 30 -gt population increaseOther foods may be more popularCurrent number of restaurants that serve seafood might sufficeWhat kind of healthy food are they interested in Surely not just seafoodThe locals may still patronize the current restaurants that serve seafood dishes

lt national-gt local에 적용 local-gtnational에 적용gt1) The author assumes that the nationwide trend reflects the general trend upon which the argument relies

Yet the author fails to provide evidence to substantiate this crucial assumption The nationwide trend of two income families eating fewer home-cooked and concern for health my not reflect the lifestyle and interests of Bay City citizens Therefore to make his argument stronger the author needs to provide evidence that two income families of Bay City maintains the nationwide trend

GRE AWA John박 박정어학원

최근의 매상에 관한 조사에서 Bay 시에 있는 레스토랑의 해산물 요리의 소비가 지난 5년동안 30 증가했다고 보고되었다 그러나 현재 해산물을 전문으로 취급하고 있는 레스토랑이 없다 더군다나 대다수의 가정이 맞벌이 가정인데다가 전국 조사에서도 나와있듯이 이러한 맞벌이 가정은 10년전의 가정에서 했듯이 집에서 식사를 챙겨먹는 것이 급격하게 줄어 들었고 동시에 건강식과 관련한 지출이 늘고 있다 따라서 해산물 전문 레스토랑이 아주 인기를 끌것이고 그만큼 수익도 많을 것이다

1 해산물이 건강식인지에 대한 언급이 없다2 외식하는데 해산물 요리만 먹지 않을 것이다 집에서 챙겨먹지 않는다고 해산물을 먹는다고 할 수 없다3 현재 해산물 취급하는 식당이 현재까지 없다고 해서 계속 없을 수만은 없다 수익이 만약 늘어난다면

Bay시 주변의 도시의 레스토랑이 체인점을 내거나 새로운 식당이 생길 수 있고 오히려 수익이 줄어들 수도 있다

Scientists studying historical weather patterns have discovered that in the mid-sixth century Earth suddenly became significantly cooler Although few historical records survive from that time some accounts found both in Asia and Europe mention a dimming of the sun and extremely cold temperatures Either a huge volcanic eruption or a large meteorite colliding with Earth could have created a large dust cloud throughout Earths atmosphere that would have been capable of blocking enough sunlight to lower global temperatures significantly A large meteorite collision however would probably create a sudden bright flash of light and no extant historical records of the time mention such a flash Some surviving Asian historical records of the time however mention a loud boom that would be consistent with a volcanic eruption Therefore the cooling was probably caused by a volcanic eruption

Write a response in which you discuss what questions would need to be addressed in order to decide whether the conclusion and the argument on which it is based are reasonable Be sure to explain how the answers to the questions would help to evaluate the conclusion

Historical records may not be enough to explain the global cooling phenomenonThere may be reasons other than the two hypothesis to explain the global coolingA volcanic eruption big enough to produce dust clouds that envelop the earth to cause global cooling would have countless recordsThe author argues that a large meteorite collision is not a feasible explanation for the global cooling because there are no historical records that indicate a flash The authorrsquos logic is flawed in eliminating the meteorite hypothesis by claiming that there was no historical record of a bright flash of light According to common sense a meteorite collision big enough to produce dust that could envelop the earth would result in catastrophe even to the extinction of major species of animals

Loud boom would be insufficient evidence to conclude it was a volcanic eruption If could hear a loud boom there should be records of a volcanic eruption because the author claims that the dust cloud from this gigantic eruption was significant enough to cause global coolingCooling could have been caused by factors besides volcanic eruption and meteor collisionThe absence of historical records that indicate a bright flash of light Collision could have occurred at a place where there no witnesses Could have fell out in the seaBoom might have been caused by things other than a volcanic eruptionHistorical records from Asia and Europe may not be applicable to earth as a whole

과거의 날씨 주기를 연구하는 과학자들은 6세기 중엽 지구가 갑자기 혹한기가 되었던 것을 알게되었다 이 당시의 몇몇 기록들이 아직도 남아있긴 하지만 아시아와 유럽에서 발견되는 몇가지 자료에서 태양 빛의 수축과 그에따른 엄청난 추위가 있었음을 언급하고 있다 거대한 화산 폭발이나 거대 운석의 지구 충돌중 어느것에

GRE AWA John박 박정어학원

의해서든 엄청난 먼지구름을 형성시켜 지구전체에 태양빛을 차단해서 지구의 기온을 뚝 떨어뜨릴수 있을 수도 있다 그러나 이러한 운석 충돌은 순간 섬광을 발산시키게 될 것인데 이 시기의 어느 기록에서도 이러한 섬광은 언급되지 않고 있다 그러나 이 시기에 남아 있는 일부 아시아의 기록문서에서는 연속적으로 화산폭발음일수 있는 엄청난 폭발음이 있었음을 기록하고 있다 따라서 이러한 혹한은 아마도 화산 폭발에 의한 것이었을 것이다

결론 the cooling was probably caused by a volcanic eruption

반박 자료의 부족( 몇몇 자료가지고 그 당시의 기후를 예상하기에는 무리가 있다)다른 원인에 의해서 기후가 떨어졌을수도 있다 (빙하기 다른 기상원인)기록되지 않았다고 해서 그 사실이 없었던 것은 아니다(기록은 했으나 자료가 없어졌을수도 있고 모든 사실이 전부다 기록되지는 않는다 )기록된 폭발음이 꼭 확산 폭발음이 아닐수도 있다( 화산 폭발음이 너무 먼거리여서 들리지 않을수도 있고 다른 소리가 기록된 것이였을수도 있다)부족한 자료를 신빙할수 있는가 기록의 소멸도 예상 할수 있다 실제로 운석이 떨어졌는데 안들렸을수 있다

The following appeared in a memo from the president of Bower Builders a company that constructs new homes

A nationwide survey reveals that the two most-desired home features are a large family room and a large well-appointed kitchen A number of homes in our area built by our competitor Domus Construction have such features and have sold much faster and at significantly higher prices than the national average To boost sales and profits we should increase the size of the family rooms and kitchens in all the homes we build and should make state-of-the-art kitchens a standard feature Moreover our larger family rooms and kitchens can come at the expense of the dining room since many of our recent buyers say they do not need a separate dining room for family meals

Write a response in which you examine the stated andor unstated assumptions of the argument Be sure to explain how the argument depends on these assumptions and what the implications are for the argument if the assumptions prove unwarranted

The presidentrsquos first unstated but apparent assumption is that a nationwide survey can be applied to local areas There is no guarantee that the area in which Bower Builders builds houses will follow the national consumer trend Since he only mentions the overall result of the nationwide survey it is difficult to determine more specific conditions such whether people in urban and rural areas have the same housing preferences Such uncertainty is further exacerbated by the nature of the survey question itselfmdash it does not deal with current trends in actual home purchases but with desired home features The naiumlve assumption that consumer desire will directly result in consumer action underlies the presidentrsquos interpretation and application of the survey results to his company plan Not everyone who wishes for a large family room and kitchen will or can actually buy a house with those features mainly due to financial reasons Furthermore the president also assumes for no evident reason that consumers will not hesitate to purchase houses with state-of-the-art kitchens as a standard rather than optional feature when it is clear that such a feature will raise the overall cost significantly He continues to blunder in his mistaken assumptions about consumer behavior in assuming that the taste of recent buyers can represent the concern of future prospective buyers The fact that recent buyers have claimed no need for separate dining rooms does not mean future buyers will feel the same way as well

The president of Bower Builders recommends that in order to raise company profits the company should build houses with large family rooms and large state-of-the-art kitchens He uses the results of a nationwide survey on desired home features and the example of Bower Buildersrsquo competitor Domus Construction to support his plan His memo manifests several assumptions about surveys consumer behavior and Domus Constructionrsquos houses that do not necessarily bolsterbuttress his argument

GRE AWA John박 박정어학원

The writer assumes 1 the nationwide survey results can be applied to the local area (Desire cannot reflect consumer trend) 2 Domus Construction was profitable because they had such features (The writer should investigate what other features the DC provides and the actual number of homes sold) 3 State-of-the-art kitchens will sell well (no evidence to buttress his assumption furthermore they would need to spend more money which would increase the price of the housesmdashtheir market will be limited to the affluent) 4 The recent buyers represent the concern of most buyersmdashthat they are ok with a house that is without a dining room

Nationwide survey might not be applicable to this regionDomus Construction could have other superior qualities than Bower Builders Ex DesignSelling fast doesnrsquot indicate quantitySmall yards complaints could be voiced in the future

First the author concludes that a nationwide survey reveals that the two most desired home features are a bathroom with a whirlpool tub and a large kitchen However this nationwide survey may not reflect the trends of the customers that Bower Builders target The author assumes that the nationwide trend reflects customer trends The national study would lead support only if the nationwide trend reflect the home-purchasing trends The nationwide trend may just be a trend of desire which does not result in the act of purchasing However the author does not provide credible evidence that this is the case

전국 조사에서 가장 인기있는 집안 구조 2가지는 훨풀 튜브가 마련된 욕실과 커다른 부엌으로 조사되었다 경쟁업체 Domus Construction이 신축한 완공예정인 주택은 이러한 욕실을 갖추고 있어서 분양이 조기에 이루어지고 있고 가격도 평균치보다 상당히 웃돌고 있다 우리도 집을 많이 팔도 그만큼 이윤을 많이 남기려면 신규 주택에는 위의 2가지 사항을 갖추어야 한다 최근 우리가 지은 주택 수요자들이 적은 평수에 대해서는 별다른 불만사항이 없었기 때문에 앞으로 평수를 더 줄여서 이익을 극대화 할 수 있을 것이다

주장 We should include whirlpool tubs and a large kitchen

1 경쟁업체 Domus Construction이 신축한 완공예정인 주택은 이러한 욕실을 갖추고 있어서라기 보다는 위치가 좋거나 다른 마감재(construction material)나 인테리어가 뛰어나서 분양이 조기에 이루어진것이라고 볼 수 있다 2 전국조사가 건물이 지어지는 지역에 항상 적용되리라고 볼 수 없다 3 평수를 줄이는 것에 불만이 없었다는 것은 editor의 견해일 수 있다 사람들이 표현하지 않은 것일 수도 있고 다른 장점이 많아서 그런 단점을 드러내지 않은 것일 수 있기 때문에 속단할 수 없다 4 다른것을 함으로써 더 잘 팔릴수도 있다

2011 7 19 화요일 GRE AWA 실전반이OO

Argument 94

The president of Bower Builders contends recommends that in order to raise company profits the company should build houses with large family rooms and large state-of-the-art kitchens He uses the results of a nationwide survey on desired home features and the example of Bower Buildersrsquo competitor Domus Construction to support his plan His memo manifests several assumptions about surveys consumer behavior and Domus Constructionrsquos houses that do not

GRE AWA John박 박정어학원

necessarily bolsterbuttress his argumentThe presidentrsquos first unstated but apparent assumption is that a nationwide survey can be

applied to local areas There is no guarantee that the area in which Bower Builders builds houses will follow the national consumer trend Since he only mentions the overall result of the nationwide survey it is difficult to determine more specific conditions such whether people in urban and rural areas have the same housing preferences Such uncertainty is further exacerbated by the nature of the survey question itselfmdash it does not deal with current trends in actual home purchases but with desired home features The naiumlve assumption that consumer desire will directly result in consumer action underlies the presidentrsquos interpretation and application of the survey results to his company plan Not everyone who wishes for a large family room and kitchen will or can actually buy a house with those features mainly due to financial reasons Furthermore the president also assumes for no evident reason that consumers will not hesitate to purchase houses with state-of-the-art kitchens as a standard rather than optional feature when it is clear that such a feature will raise the overall cost significantly He continues to blunder in his mistaken assumptions about consumer behavior in assuming that the taste of recent buyers can represent the concern of future prospective buyers The fact that recent buyers have claimed no need for separate dining rooms does not mean future buyers will feel the same way as well

In addition the president finds a real-life actualization of the nationwide survey results in the recent sales of Domus Construction However he easily assumes that large family rooms and kitchens are the only reasons the houses of Domus Construction sell well He does not take into account other features and selling points of the competitorrsquos houses There may well be other explanations for its houses selling more quickly and expensively including additional home features as well as external factors such as proximity to better schools or superior financial solvency of its clientele If Bower Builders merely added larger family rooms and kitchens without taking into consideration the other factors they may lose rather than gain profits

Clearly the presidentrsquos assertion that Bower Builders make houses with large family rooms and high-tech kitchens at the expense of dining rooms rests on a number of assumptions that are ill-informed and naiumlve If Bower Builders undertakes the proposed plan without further research into local consumer desires purchasing trends and the marketing and sales of competing companies the company will risk losing money by building big new houses that people cannot afford to or will not wish to buy

The following appeared in a letter to the editor of a journal on environmental issues

Over the past year the Crust Copper Company (CCC) has purchased over 10000 square miles of land in the tropical nation of West Fredonia Mining copper on this land will inevitably result in pollution and since West Fredonia is the home of several endangered animal species in environmental disaster But such disasters can be prevented if consumers simply refuse to purchase products that are made with CCCs copper unless the company abandons its mining plans

Write a response in which you examine the stated andor unstated assumptions of the argument Be sure to explain how the argument depends on these assumptions and what the implications are for the argument if the assumptions prove unwarranted

The writer assumes 1 The writer is trying to avoid an inevitability 2 Mining copper will result pollution (they could make preventative measures) 3 The writer may be too late from stopping CCC from developing the area into a copper mine 4 Since mining is an underground enterprise the surface may not be affected that much therefore endangered species may not be affected 4 Consumers wonrsquot buy CCC products if the journal publishes a negative review about CCC (How many readers) CCC could a company that has ties with many IT companies and industries in that their copper is almost ubiquitous in various products

GRE AWA John박 박정어학원

지난 한해동안 CCC(Consolidated Copper 회사)는 서부 플로리다의 열대 지역에 1백만 마일이 넘는 땅을 사들였다 이 곳에서의 채광 활동은 서부 플로리다가 몇몇 멸종 위기에 처한 동물의 서식지이기 때문에 분명 오염과 환경파괴를 가져올 것이다 그러나 이러한 파괴는 CCC 회사가 채광을 포기할 때까지 이 회사가 채굴한 구리로 제조된 제품을 구매하지 않으면 막을 수 있을 것이다

결론 such disaster can be prevented if consumers simply refuse to purchase products that are made with CCCs copper until the company abandons its mining plans

1 구리로 제조된 물건이 생활에 필수적인 것이 많은 만큼 불매가 쉽지 않을 수 있다(전선이나 각종 전자제품에 필수적으로 들어가기 때문에)

2 불매를 유도한다고 해서 소비자들이 구매를 안하는 것은 아니다3 적절한 채굴로 환경파괴를 가져 오지 않을 수 있다 (땅속에 있는 물질을 채굴하는 만큼 생물에 영향을 안

미칠 수도 있다)4 이미 채굴이 다 끝나서 더 이상의 채굴이 없을 수도 있다 5 땅을 구입한다고 해서 채광하는건 아니다 (다른 용도로 샀을수도 있다)

The following is a letter to the editor of an environmental magazine

In 1975 a wildlife census found that there were seven species of amphibians in Xanadu National Park with abundant numbers of each species However in 2002 only four species of amphibians were observed in the park and the numbers of each species were drastically reduced There has been a substantial decline in the numbers of amphibians worldwide and global pollution of water and air is clearly implicated The decline of amphibians in Xanadu National Park however almost certainly has a different cause in 1975 troutmdashwhich are known to eat amphibian eggsmdashwere introduced into the park

Write a response in which you discuss what specific evidence is needed to evaluate the argument and explain how the evidence would weaken or strengthen the argument

Evidence needed the identity of the sender and the census taker of rsquo75 and lsquo02mdasha scientist or an environmentalist credibility issue census methodology vs mere observationmdashthe absence of evidence is not an evidence of absence the season of when the census was taken time-shiftmdashconditions may have changed worldwide decline may include Xanadu other species of predators that prey on amphibians because trout is only one species that prey on amphibians the number of troutmdashhave they increased significantly since rsquo75 First the author needs to be more overt about the credibility of the census and observation There were to accounts that notes the population of amphibiansmdashthe first a census and the second an observation The author needs to bolster his conclusion with the evidence that indicate that the census and second observation were done using scientific methodologies This could either could strengthen or weaken his claim In addition he

GRE AWA John박 박정어학원

needs to provide the specific season of when the census and observation occurred In this letter the writer is informing an editor of an environmental magazine that the number of amphibians was greatly reduced since 1975 and he points out the introduction of trout as the only reason for the decline However the author fails to provide crucial evidences that could strengthen or weaken his conclusion

The following appeared in a memorandum from the president of Hyper-Go Toy Company

Last year sales of our Fierce Fighter toy airplane declined sharply even though the toy had been a top seller for three years Our customer surveys show that parents are now more worried about youthful violence and are concerned about better education for their children Therefore to maintain profits we should discontinue all our action toys and focus exclusively on a new line of educational toys Several other toy companies have already begun marketing educational toys and report sales increases last year of 200 percent And since the average family income is growing sales of new Hyper-Go toys should also increase

Write a response in which you discuss what specific evidence is needed to evaluate the argument and explain how the evidence would weaken or strengthen the argument

First the president indicates that the sales of Fierce Fighter toy airplane declined sharply However he fails to consider the fact that toys are a fad Since FFT enjoyed a three year success it may be natural that the trend would subsideSecond Customer survey is this representative of most toy consumers This trend is not newThird other companies may have profited not through educational but other toys Or educational toy profit may be small in proportion to their sale of other toys 200Parents are not the customers companies should concentrate on what the children want to increase profitLastly parents worry about youthful violence and concern for better education are not new trends

The presidentrsquos decision to discontinue all action toys and focus only on educational toys is too extreme If this decision is taken into effect its procedure may be an onerous task because it would require major shifts in human resources and company image Therefore further data should be considered before following up on this decision

우리 회사에서 지난 3년간 최고 매출을 일으켰던 Fierce Fighter 장난감 비행기가 지난해에는 매출이 급격히 떨어졌다 자체 고객 조사에서는 부모들이 현재 청소년 폭력에 걱정을 하고 있어서 아이들의 양질 교육에 더 관심을 가지고 있는 것으로 나타났다 따라서 회사의 수익을 유지하기 위해서는 모든 자사 전투용 장난감 생산을 중단하고 오로지 교육적인 장난감 생산에 집중해야 한다 몇몇 여타 장난감 회사들도 이미 교육용 장난감 마케팅을 시작해서 지난해에는 200의 매출신장을 가져왔다고 한다 그리고 평균 가계 수입이 점점 늘고 있기 때문에 신형 Hyper-Go 장난감의 매출도 늘어날 것이다

3그룹 Time-shift Error

Woven baskets characterized by a particular distinctive pattern have previously been found only in the immediate vicinity of the prehistoric village of Palea and therefore were believed to have been unique to the Palean people Recently however archaeologists discovered such a Palean basket in Lithos an ancient village across the Brim River from Palea The Brim River is very deep and broad and so the ancient Paleans could only have crossed it by boat but there is no evidence that the Paleans had boats And boats capable of carrying groups of people and cargo were not developed until thousands of years after the Palean people disappeared Moreover Paleans would have had no need to cross the rivermdashthe woods around Palea are full of nuts berries and small game It follows that the so-called Palean baskets were not unique to Palea

GRE AWA John박 박정어학원

Write a response in which you discuss what specific evidence is needed to evaluate the argument and explain how the evidence would weaken or strengthen the argument

RefutationPrehistoric time shift-gtbrim river could have been narrow and shallow or it might have not existed Indigenous patterns may exist in other disconnected remote placesNuts berries small game(hunting) may not have existed or the author should be proved these existed at that time Abundance of resources doesnrsquot support the reason for seclusionBoats not yet found baskets may have been carried across by the river current without the help of a boat Lithos might have crossed the river for commercial purposeOne Palean basket does not substantiate the authorrsquos claimThe absence of evidence is not an evidence of absence

The author assumes without justification that present conditions are the same as at the prehistoric era The author unfairly infers from the presence of Brim River which exist today that it would have existed in the past However the author fails to offer any evidence to substantiate this inference It is very likely that the Brim River might not have existed in prehistoric times or if it did exist may have been shallow and narrow enough for the Paleans to easily cross Any of these scenarios if true would serve to undermine the claim thathelliphelliphelliphelliphellip

First the author claims that the Brim River was very deep and broad so the Paleans could not have crossed it However the author fails to offer any evidence to substantiate that this was true in the prehistoric time of the Paleans For all we know the Brim River might not have existed in the prehistoric era or if it did exist could have been a shallow and narrow river For example scientists believe that thousands of years ago an ice-bridge existed on the Bering Sea connecting Eurasia to what is now North America to explain how the Eskimos and the Asian inhabitants of America came to migrate all over the Americas In this example the absence of evidence is not an evidence of absence Therefore to sufficiently support his claim the author needs to substantiate the fact that the Brim River really did exist and was broad and wide in the Prehistoric Era proving that the Paleans could not have influenced or traded with other groups of people

First the author claims that the Brim River was very deep and broad so the Paleans could not have crossed it However the author fails to offer any evidence to substantiate that this was true in the prehistoric times of the Paleans For all we know the Brim River might not have existed in the prehistoric era or if it did exist could have been a shallow and narrow river For example rivers are created by natural erosion over thousands of years Niagara Falls carved its way from the mouth of Lake Ottawa and created a long river Likewise the current Brim Riverrsquos physical features may not have been wide and deep Therefore to sufficiently support his claim the author needs to substantiate the fact that the Brim River really did exist and was broad and wide in the Prehistoric Era proving that the Paleans could not have influenced or traded with other groups of people

예전에는 실로짠 특이한 무늬 바구니가 Palea의 선사시대 지역의 인근마을에서만 발견되어왔기 때문에 Palea 마을 사람들의 특징이라고 여겨졌었다 그러나 최근들어 고고학자들이 Lithos지역에서 Palean 바구니를 발견하였는데 그 당시 지역은 Brim 강을 가로질러 Palea까지 닿아있었다 이 강은 수심이 아주 깊고 강폭이 넓었으며 때문에 고대의 Palea인들은 배를 이용해서 강을 건널수 있었을 것이다 그러나 이들이 배를 가지고 있었다는 증거는 발견되지 않고 있다 더군다나 이들이 멸명한 이후 수천년이 지난뒤에도 수많은 물자와 사람을 실어 나를수 있는 용적을 가진 배는 개발되지 않았다 이와더불어 Palea인들은 강을 건널필요가 없었는데 그것은 너트나무 장과열매 그리고 작은 사냥감들이 주변숲에 풍부했기때문이다 따라서 Palean 바구니라고 하는 것도 Palea인들만의 전유물이 아니라는 결론을 얻을 수 있다

결론 if follows that the so-called Palean baskets were not unique to Palea이번문제는 굿이 causal Bad analogy 로 구분해서 찾기가 힘드내요 배를 발견했다는 증거가 없는것이다 (앞으로도 발견될수 있음)

GRE AWA John박 박정어학원

계절의 영향으로 겨울에 얼음이 두껍게 언다든지 여름에 가뭄으로 인해서 건널수 있다물자가 풍부한 것이 이동하지 않을 조건이 아니다 다른 것에 의해서 이동가능(의약품등)

Thirteen years ago researchers studied a group of 25 infants who showed signs of mild distress when exposed to unfamiliar stimuli such as an unusual odor or a tape recording of an unknown voice They discovered that these infants were more likely than other infants to have been conceived in early autumn a time when their mothers production of melatonin hormone known to affect some brain functions would naturally increase in response to decreased daylight In a follow-up study conducted earlier this year more than half of these children now teenagers who had shown signs of distress identified themselves as shy Clearly increased levels of melatonin before birth cause shyness during infancy and this shyness continues into later life

Write a response in which you examine the stated andor unstated assumptions of the argument Be sure to explain how the argument depends on these assumptions and what the implications are for the argument if the assumptions prove unwarranted

Any baby exposed to unpleasant stimuli would react in such wayFirst of all the author states 25 infants as his evidence However this research sample is too small to prove his claimSecond the author states that 25 infants were conceived in early autumn which he claims lead to a shy disposition However this is faulty evidenceThird the research study was a long term study done in the span of 13 years However the author only writes about the initial and final stages of the study and leaves out evidences of what could have happened during the 13 years which could be more evidential factors of influenceFourth neither the infantsrsquo genetic predisposition nor their environment were taken into accountFinally the author concludes that his shyness continues into later life (Other factors could alter this disposition epigenetic theory)

13 년전 학자들은 25명의 유아를 대상으로 이상한 냄새나 특이한 소리를 녹음한 테잎등으로 낯선 자극을 주었을때 보이는 미미한 압박감 증상을 조사하였다 이들은 성숙기가 막 지났을 즈음에 보통의 유아들이 비슷한 증상을 보이는 정도 보다는 다소 민감한 반응을 보였는데 이 시기는 아이의 엄마가 뇌의 일부 기능에 영향을 미치는 것으로 알려진 멜라토니아 호르몬을 생산하는 시기로써 이 호르몬은 낯 시간이 짧을때 자연적으로 증가할 수도 있다 금년초에 실시된 추가연구에서 현재 10대로 성장한 당시 조사대상의 절반 이상의 아이들이 부끄럼을 잘타는 것으로 여기고 있었다 따라서 분명한 것은 출산전 멜라토닌 수치의 증가가 유아기에 수줍음 등의 영향을 미치게 되며 이러한 영향이 성장후에도 작용한다는 것이다

주장 Clearly increased levels of melatonin before birth cause shyness during infancy and this shyness continues into later life

1 25명의 아기로 결론 내리기에 샘플이 작다2 과학적 사실들에 대한 명확한 근거가 엇음3 다른 영향을 간과했다(다른 호르몬에 의한 영향 후천적인 성격형성의 영향)

GRE AWA John박 박정어학원

The following is a letter to the editor of the Atticus City newspaper

Former Mayor Durant owes an apology to the city of Atticus Both the damage to the River Bridge which connects Atticus to Hartley and the traffic problems we have long experienced on the bridge were actually caused 20 years ago by Durant After all he is the one who approved the construction of the bridge If he had approved a wider and better-designed bridge on which approximately the same amount of public money would have been spent none of the damage or problems would have occurred Instead the River Bridge has deteriorated far more rapidly over the past 20 years than has the much longer Derby Bridge up the river Even though the winters have been severe in the past several years this is no excuse for the negligence and wastefulness of Durant

Write a response in which you discuss what questions would need to be answered in order to decide whether the recommendation is likely to have the predicted result Be sure to explain how the answers to these questions would help to evaluate the recommendation

전임 시장인 Durant 씨는 Atticus 시에 대해 사과할 의무가 있습니다 Atticus와 Hartley를 잇는 River Bridge 교량에 대한 피해와 이 교량에서 오랫동안 주민들이 겪어오고 있는 교통 혼잡 문제들은 실제로 20년 전부터 시작된 것이었습니다 결정적으로 그가 교량 공사를 허가했던 바로 그 장본인입니다 당시 비슷한 공사비용으로 폭이 더 넓고 튼튼하게 설계된 교량을 허가했다면 이러한 문제나 피해는 발생하지 않았을 겁니다 더군다나 이 다리는 지난 20년 동안 상류에 건설된 훨씬 오래된 Derby 다리보다도 빠르게 부식되어 갔습니다 지난 수년동안 심지어 혹한이 있었다 하더라도 이러한 태만과 국고 손실에 대한 책임을 회피할 길이 없는 것입니다

결론 Former Mayor Durant owes an apology to the city of Atticus

1 디자인이 문제가 아닐수 있다 (디자인은 좋았으나 건설과정에 문제가 있었을 수 있다)2 그 당시의 시예산이 적어서 더 큰 다리를 짓기가 불가능했을 수도 있다3 교통량이 많거나 다른 상황으로 인해서 부식이 빨리 됐을 수 있다4 그 당시에는 최선의 선택이였지만 갑자기 변한 상황에 의해서 이런 문제점들이 발생했을 수 있다

GRE AWA John박 박정어학원

4그룹 거짓인과관계 오류 (False Cause) 빈출

Fifteen years ago Omega University implemented a new procedure that encouraged students to evaluate the teaching effectiveness of all their professors Since that time Omega professors have begun to assign higher grades in their classes and overall student grade averages at Omega have risen by 30 percent Potential employers looking at this dramatic rise in grades believe that grades at Omega are inflated and do not accurately reflect student achievement as a result Omega graduates have not been as successful at getting jobs as have graduates from nearby Alpha University To enable its graduates to secure better jobs Omega University should terminate student evaluation of professors

Write a response in which you discuss what specific evidence is needed to evaluate the argument and explain how the evidence would weaken or strengthen the argument

Omega professor evaluation implemented 15 years ago =gt Omega prof assign higher grades 30Employers believe therersquos grade inflation

Thus unsuccessful employment than AlphaTherefore to secure jobs Omega should end evaluating profs

Specific evidence neededRelationship between higher grades and evaluationRelationship between GPA and unsuccessful employmentAlpharsquos education could just be better than OmegaldquoFifteen years agordquo is a long time other factors could have influenced Why is the inflation a problem just now How much is Alpha better Is the comparison just How much gap is thereOmegarsquos student could just be doing better in their studiesComparison to other universities다른 대안 없나hellip Could Omega alleviate the employment problem by implementing a different procedure or program

15 년전 우리 대학은 학생들로 하여금 교수평가를 하도록 한 새로운 조치를 시행했었습니다 이후 교수들은 자신의 학과 학생들에게 높은 학점을 주었으며 그에따라 학생들의 전체 평점이 30나 올랐습니다 외부의 기업체들은 분명 점수가 지나치게 부풀려졌다고 믿고 있습니다 결국 본 대학 졸업생들이 인근 Alpha 대학의 졸업자들보다 구직률이 떨어지는 이유를 잘 보여주고 있는 것입니다 이를 해결하기 위해 이제부터는 학생들에 의한 교수평가제를 중단해야 합니다

결론 Omega University should now terminate student evaluation of professors

반박 교수 평가와 학점 인플레의 연관성이 적다( 교수 평가를 먼저하고 학점을 나중에 매길수도 있다)채용기준에 성적만 있는게 아니다 학업성취의 결과 일수도 있다 Alpha 가 원래 유능했다 Alpha 의 교육내용이 좋았다

GRE AWA John박 박정어학원

In this memo the dean of Omega University(OU) recommends OU to terminate professor evaluation to secure better jobs for the students To support this recommendation the dean offers several reasons However this argument contains several logical flaws which render it unconvincing

A threshold problem with the argument involves the voluntary nature of the evaluationprocedure The dean provides no evidence about the number or percentage of Omegastudents who participate in the procedure Lacking such evidence it is entirely possible thatthose numbers are insignificant in which case terminating the procedure is unlikely to haveany effect on the grade average of Omega students or their success in getting jobs aftergraduationThe argument also assumes unfairly that the grade-average increase is the result of theevaluation procedure--rather than some other phenomenon The dean ignores a host of otherpossible explanations for the increase--such as a trend at Omega toward higher admissionstandards or higher quality instruction or facilities Without ruling out all other possibleexplanations for the grade-average increase the dean cannot convince me that by terminatingthe evaluation procedure Omega would curb its perceived grade inflation let alone help itsgraduates get jobsEven if the evaluation procedure has resulted in grade inflation at Omega the deans claimthat grade inflation explains why Omega graduates are less successful than Alpha graduatesin getting jobs is unjustified The dean overlooks a myriad of other possible reasons forOmegas comparatively poor job-placement record Perhaps Omegas career services areinadequate or perhaps Omegas curriculum does not prepare students for the job market aseffectively as Alphas In short without accounting for other factors that might contribute toOmega graduates comparative lack of success in getting jobs the dean cannot justify theclaim that if Omega curbs its grade inflation employers will be more likely to hire OmegagraduatesFinally even if the dean can substantiate all of the foregoing assumptions the deansassertion that Omega must terminate its evaluation procedure to enable its graduates to findbetter jobs is still unwarranted in two respects First the dean ignores other possible ways bywhich Omega can increase its job-placement record--for example by improving its publicrelations or career-counseling services Second the dean unfairly equates more jobs withbetter jobs In other words even if more Omega graduates are able to find jobs as a result ofthe deans recommended course of action the kinds of jobs Omega graduates find would notnecessarily be better onesIn sum the deans argument is unpersuasive as it stands To strengthen it the dean mustprovide better evidence that the increase in grade average is attributable to Omegasprofessor-evaluation procedure and that the end result is a perception on the part ofemployers that Omega graduates are less qualified for jobs than Alpha graduates To betterassess the argument I would need to analyze 15-year trends in (l) the percentage of Omegastudents participating in the evaluation procedure (2) Omegas admission standards andquality of education and (3) Omegas emphasis on job training and career preparation I wouldalso need to know what other means are available to Omega for enabling its graduates to findbetter jobs

GRE AWA John박 박정어학원

The following appeared in a memo from a vice president of Quiot Manufacturing

During the past year Quiot Manufacturing had 30 percent more on-the-job accidents than at the nearby Panoply Industries plant where the work shifts are one hour shorter than ours Experts say that significant contributing factors in many on-the-job accidents are fatigue and sleep deprivation among workers Therefore to reduce the number of on-the-job accidents at Quiot and thereby increase productivity we should shorten each of our three work shifts by one hour so that employees will get adequate amounts of sleep

Write a response in which you examine the stated andor unstated assumptions of the argument Be sure to explain how the argument depends on these assumptions and what the implications are for the argument if the assumptions prove unwarranted

The following appeared in a memo from a vice president of Alta Manufacturing

During the past year Alta Manufacturing had thirty percent more on-the-job accidents than nearby Panoply Industries where the work shifts are one hour shorter than ours Experts believe that a significant contributing factor in many accidents is fatigue caused by sleep deprivation among workers Therefore to reduce the number of on-the-job accidents at Alta we recommend shortening each of our three work shifts by one hour If we do this our employees will get adequate amounts of sleep

Write a response in which you discuss what questions would need to be answered in order to decide whether the recommendation and the argument on which it is based are reasonable Be sure to explain how the answers to these questions would help to evaluate the recommendation

The following appeared in a memo from the vice president of Butler Manufacturing

During the past year workers at Butler Manufacturing reported 30 percent more on-the-job accidents than workers at nearby Panoply Industries where the work shifts are one hour shorter than ours A recent government study reports that fatigue and sleep deprivation among workers are significant contributing factors in many on-the-job accidents If we shorten each of our work shifts by one hour we can improve Butler Manufacturings safety record by ensuring that our employees are adequately rested

1 Write a response in which you discuss what specific evidence is needed to evaluate the argument and explain how the evidence would weaken or strengthen the argument

2 Write a response in which you discuss what questions would need to be answered in order to decide whether the recommendation is likely to have the predicted result Be sure to explain how the answers to these questions would help to evaluate the recommendation

4번 반복됨

Alta has 30 more job accidents than Panoply(work shifts one hour shorter)Experts Job accidents caused by fatigue and sleep deprivationTherefore to reduce job accidents and increase productivity shorten three work shifts by one hour for adequate sleep

지난해 우리 회사는 인근 Panoply Industries보다 업무상 재해가 30나 더 많았다 그 회사는 우리보다 근무 교대시간이 1시간 정도 짧았다 전문가들은 대부분의 업무상 재해에 있어서 가장 중요한 요인이 과로와 수면부족으로 보고있다 따라서 우리 회사에서 높은 산업재해를 줄이고 아울러 생산성을 높이기 위해서는 근로자들이 충분한 수면을 취할 수 있도록 1시간씩 3교대 시간을 줄여야 한다

In this memo the (author) vice president of Alta Manufacturing (AM) recommends that to reduce on-the-job accidents and increase productivity AM should shorten its three work shifts by one hour so that employees can

GRE AWA John박 박정어학원

get more sleep To support this recommendation the author provides several evidences However careful scrutiny of each of the facts reveals that it provides little credible support for the authorrsquos recommendation QuestionsThe number of accidents What kind of accidents The seriousness of the accidents is importantHow many employees are in each company What are their productsFalse cause Sleep may not be the reason for the on-the-job accidents What do Alta and Panoply manufacture

First of all the author believes that fatigue caused the on-the job accidents However there could be other reasons The author observes a correlation between sleep deprivation and on-the-job accidents then concludes that the former is the cause of the latter However the author fails to rule out other possible explanations For example it is entirely possible that Alta factories require more strenuous and dangerous labor than Panoply Without ruling out all other such factors it is unfair to conclude that fatigue is responsible for the accidents In addition the work-shifts may not be the cause of the sleep deprivation and fatigue It is possiblehellip Thus the author should provide what exactly Panoply and Alta manufacture and more precise data about their working conditions to be more convincing

Shortening the shift by one hour does not necessarily lead to more sleep And is one hour enoughLess accidents does not mean increased productivity

결론 We should shorten each of out three work shifts by one hour

반박 경쟁사에 비해서 시간당 하는 업무량이 많아서 더욱 피곤할 수도 있다 시간이 문제가 아니라 노후된 시설 설비 자체의 문제 작업 자체가 원래 위험한 것이여서 사고가 많을 수도 있다 다른 회사는 더욱 많은 작업시간에도 불구하고 안정한 작업여건으로 인해서 사고율이 오히려 더 작을 수도 있다비교사의 재해감소가 다른 요인일수 있다(안전 교육 철저)줄인 시간이 피로회복이나 수면으로 연결 안될수 있음(술을 마실 수도 있고 그 시간에 휴식을 취하지 않고 다른일을 함으로써 더욱 피로해질수 있다)

This editorial recommends that Alta Manufacturing reduce its work shifts by one hour each inorder to reduce its on-the-job accident rate and thereby increase Altas productivity To supportthis recommendation the author points out that last year the number of accidents at Alta was30 greater than at Panoply Industries where work shifts were one hour shorter The authoralso cites certain experts who believe that many on-the-job accidents are caused by fatigueand sleep deprivation I find this the argument unconvincing for several reasonsFirst and foremost the author provides absolutely no evidence that overall workerproductivity is attributable in part to the number of on-the-job accidents Although commonsense informs me that such a relationship exists the author must provide some evidence ofthis cause-and-effect relationship before I can accept the authors final conclusion that theproposed course of action would in fact increase Altas productivitySecondly the author assumes that some accidents at Alta are caused by fatigue or sleepdeprivation However the author overlooks other possible causes such as inadequateequipment maintenance or worker training or the inherent hazards of Altas manufacturingprocesses By the same token Panoplys comparatively low accident rate might be attributablenot to the length of its work shifts but rather to other factors such as superior equipmentmaintenance or worker training In other words without ruling out alternative causes ofon-the-job accidents at both companies the author cannot justifmbly conclude that merely byemulating Panoplys work-shift policy Alta would reduce the number of such accidentsThirdly even assuming that Altas workers are fatigued or sleep-deprived and that this is thecause of some of Altas on-the-job accidents in order to accept the authors solution to thisproblem we must assume that Altas workers would use the additional hour of free time tosleep or rest However the author provides no evidence that they would use the time in thismanner It is entirely possible that Altas workers would use that extra hour to engage in someother fatiguing activity Without ruling out this possibility the author cannot convincinglyconclude that reducing Altas work shifts by one hour would reduce Altas accident rateFinally a series of problems with the argument arise from the scant statistical information onwhich it relies In comparing the number of accidents at Alta and Panoply the author fails toconsider that the per-worker accident rate might reveal that Alta is actually safer than Panoplydepending on the total number of workers at each company Second perhaps accident rates

GRE AWA John박 박정어학원

at the two companies last year were aberrations and during other years Altas accident ratewas no greater or even lower than Panoplys rate Or perhaps Panoply is not representativeof industrial companies generally and that other companies with shorter work shifts have evenhigher accident rates In short since the argument relies on very limited statistical information Icannot take the authors recommendation seriouslyIn conclusion the recommendation for emulating Panoplys work-shift policy is not wellsupported To convince me that shorter work shifts would reduce Altas on-the-job accidentrate the author must provide clear evidence that work-shift length is responsible for some ofAltas accidents The author must also supply evidence to support her final conclusion that alower accident rate would in fact increase overall worker productivity

The following appeared in a memo from the vice president of marketing at Dura-Sock Inc

A recent study of our customers suggests that our company is wasting the money it spends on its patented Endure manufacturing process which ensures that our socks are strong enough to last for two years We have always advertised our use of the Endure process but the new study shows that despite our socks durability our average customer actually purchases new Dura-Socks every three months Furthermore our customers surveyed in our largest market northeastern United States cities say that they most value Dura-Socks stylish appearance and availability in many colors These findings suggest that we can increase our profits by discontinuing use of the Endure manufacturing process

1 Write a response in which you examine the stated andor unstated assumptions of the argument Be sure to explain how the argument depends on these assumptions and what the implications are for the argument if the assumptions prove unwarranted

2 Write a response in which you discuss what specific evidence is needed to evaluate the argument and explain how the evidence would weaken or strengthen the argument

3 Write a response in which you discuss what questions would need to be answered in order to decide whether the recommendation and the argument on which it is based are reasonable Be sure to explain how the answers to these questions would help to evaluate the recommendation

Intro The vice president of marketing at Dura-Sock Inc is offering a potentially harmful investment recommendation by claiming that Dura-Sock should discontinue its use of the ldquoEndurerdquo process To support his recommendation he points out a study that Dura-Sock customers actually purchase the socks every three months and a survey that reveals that Dura-Sock customers like the sockrsquos stylish appearance and availability in many colors The study and survey however are insufficient in supporting his proposal and the VP makes several unwarranted assumptionsIntro (simplified) The VP states that though Dura-Socks last for two years customers buy the socks every three months Therefore he assumes that the consumersrsquo motive for buying the produce is not its durabilityHowever the author fails to rule out other possible motivation for consumption

Even if the survey is reliable the author should consider the rest of the market Vague terms ldquowasting moneyrdquomdashprecisely how much are they wasting Studysurvey errorThe company must calculate the outcome of such momentous decisionStudy participantsrsquo comment that they prefer Dura-Sock for its stylishness and availability might take Dura-Sockrsquos enduring quality for granted

우리회사 제품 소비자들에 대한 최근 조사에서 지난 2년여간 양말의 내구성을 강하게 하는 필수공정이었던 자사 특허의 Endure 공정에 들어가는 비용이 낭비라고 말하고 있다 우리 회사는 항상 이 공정 처리에 대한 광고를 내보냈으나 이에 대한 시장 조사에서 실제로 고객들은 이 신제품을 평균 석달마다 구매하는 것으로 나타났다 더군다나 북동부지역에서 실시한 대규모 시장조사에 응답한 고객들은 양말의 모양과 색상등에 더

GRE AWA John박 박정어학원

관심을 나타냈다 이러한 결과는 우리회사가 신기술 공법을 중단하면 그에 따라 수익이 늘어날 것이라는 것을 말해주고 있는 것이다주장 These findings suggest that Dura0Sock can increase its profits by discontinuing its use of the ldquoEndurerdquo manufacturing process

1 survey가 정확한 소비자의 의견을 나타낸 것인가 다른 선택없이 양자택일과 같은 방법의 survey였는지2 북동부지역의 시장조사가 전체 의견을 대표할 수 있나3 사람들이 모양이나 색상에 앞서 내구성을 먼저 평가했을 수도 있다 내구성을 갖추었다는 전제하에 모양과 색상에 관심을 드러낸 것일 수 있다4 소비자가 도매상(retail)인지 소매상(whole)인지가 없다

The following appeared in a business magazine

As a result of numerous complaints of dizziness and nausea on the part of consumers of Promofoods tuna the company requested that eight million cans of its tuna be returned for testing Promofoods concluded that the canned tuna did not after all pose a health risk This conclusion is based on tests performed on samples of the recalled cans by chemists from Promofoods the chemists found that of the eight food chemicals most commonly blamed for causing symptoms of dizziness and nausea five were not found in any of the tested cans The chemists did find small amounts of the three remaining suspected chemicals but pointed out that these occur naturally in all canned foods

Write a response in which you discuss what questions would need to be addressed in order to decide whether the conclusion and the argument on which it is based are reasonable Be sure to explain how the answers to the questions would help to evaluate the conclusion

Representativeness of the tested cansThey should conduct a comparative studyThe testing could be biased because Promofoods employees conducted the testingHow much (quantity) of the five and three suspected chemicals were in the canned foodsFalse cause The substance that caused dizziness and nausea may not be one of the eight common chemicals

많은 소비자들의 현기증과 구역질 불만에 따라 Promofoods사는 지난해 참치 캔 8백만 개를 테스트하기 위해 반품시켰다 그 결과 캔에서는 건강에 위험이 될 수 있는 화합물질이 없었던 것으로 회사측은 결론지었다 이러한 결론은 회사측 화학연구자들이 회수된 캔의 샘플을 테스트해서 이들 증상의 원인이 되는 8가지 화합물 중에서 5가지가 실험된 캔에서 발견되지 않았다는 사실에 근거한 것이다 이들 화학자들은 나머지 3개가지 화합물이 모든 캔 식료품에서 흔히 발견되는 것이라고 언급했다 결론 Promofoods concluded that the cans did not after all contain chemicals that posed a health risk

1 공인된 기간에서 테스트를 한 것이 아니고 자사에서 직접 테스트를 했기에 신뢰성이 안간다 2 이런 증상을 일으키는 8개의 물질 말고 다른 물질들이 캔속에 많이 포함됬을수 있다 3 나머지 3개의 물질들의 함유량이 많아서 다른 종류의 캔들은 문제를 일으키지 않지만 참치캔은 문제를

일으킬 수 있다

This magazine article concludes that the 8 million cans of tuna Promofoods recalled due tocomplaints about nausea and dizziness do not after ail contain any chemicals that pose a

GRE AWA John박 박정어학원

health risk To support this conclusion the author cites the fact that five of eight chemicalscommonly causing these symptoms were not found in the recalled cans while the other threealso occur naturally in other canned foods For several reasons this evidence lends littlecredible support to the authors conclusionTo begin with the author relies partly on the fact that although three of the eight chemicalsmost commonly blamed for nausea and dizziness appeared in Promofoods recalled tunathese chemicals also occur naturally in other canned foods However this fact alone lends nosupport to the authors conclusion for two reasons First the author might be ignoring animportant distinction between naturally occurring chemicals and those not occurring naturallyIt is entirely possible that these three chemicals do not occur naturally in Promofoods tunaand that it is for this reason that the chemicals cause nausea and dizziness Secondly it isentirely possible that even when they occur naturally these chemicals cause the samesymptoms Unless the author rules out both possibilities he cannot reliably conclude that therecalled tuna would not cause these symptomsAnother problem with the argument is that the authors conclusion is too broad Based onevidence about certain chemicals that might cause two particular heath-related symptoms theauthor concludes that the recalled tuna contains no chemicals that pose a health risk Howeverthe author fails to account for the myriad of other possible health risks that the recalled tunamight potentially pose Without ruling out all other such risks the author cannot justifiablyreach his conclusionA third problem with the argument involves that fact that the eight particular chemicals withwhich the test was concerned are only the eight most commonly blamed for nausea anddizziness It is entirely possibly that other chemicals might also cause these symptoms andthat one or more of these other chemicals actually caused the symptoms Without ruling outthis possibility the author cannot jusufiably conclude that the recalled tuna would not causenausea and dizzinessA final problem with the argument involves thetesting procedure itself The author providesno information about the number of recaUed cans tested or the selection method used Unlessthe number of cans is a sufficiently large sample and is statistically repre sentative of all therecalled cans the studys results are not statistically reliableIn conclusion the article is unconvincing as it stands To strengthen the assertion that therecalled tuna would not cause nausea and dizziness the author must provide evidence thatthe three chemicals mentioned that occur naturally in other canned foods also appear naturallyin Promofoods tuna The author must also provide evidence that ingesting other canned foodscontaining these three chemicals does not cause these symptoms To better evaluate theargument we would need to know whether the sample used in the tests was statisticallysignificant and representative of all the recalled tuna We would also need to know what otherchemicals in the recalled tuna might pose any health risk at all

5그룹 불충분 조건오류 빈출

Natures Way a chain of stores selling health food and other health-related products is opening its next franchise in the town of Plainsville The store should prove to be very successful Natures Way franchises tend to be most profitable in areas where residents lead healthy lives and clearly Plainsville is such an area Plainsville merchants report that sales of running shoes and exercise clothing are at all-time highs The local health club has more members than ever and the weight training and aerobics classes are always full Finally Plainsvilles schoolchildren represent a new generation of potential customers these schoolchildren are required to participate in a fitness-for-life program which emphasizes the benefits of regular exercise at an early age

Write a response in which you examine the stated andor unstated assumptions of the argument Be sure to

GRE AWA John박 박정어학원

explain how the argument depends on these assumptions and what the implications are for the argument if the assumptions prove unwarranted

False cause

First of all the author believes that the Increased sales of running shoes and exercise clothing indicates

Plainesville residentsrsquo interest in leading healthy lives However this assumption is not logically convincing for

several reasons could be a fashion trendTime shift ldquoFitness for liferdquo might not have any influence on schoolchildren as they growFalse cause There could be other reasons for member increase in the health clubAll of the above are insufficient condition

The author has to prove that local residents are interested in leading healthy lives However he supports his conclusion with insufficient evidence Nevertheless even if the residents are concerned with health naturersquos way may not be successful First

그 동안의 경험을 토대로 볼 때 건강생활과 밀접히 관련되어 있는 거주 지역에서 본 상점들이 아주 호응을 얻고 있다 따라서 이러한 주민들이 많이 거주하고 있는 Plainsville 에 새로운 상점들을 계속 세워야 한다 이 지역 상인들은 런닝화와 운동복 판매가 가장 높다고 말한다 불과 5 년전에는 거의 전무하다시피하던 지역 헬스 클럽의 경우도 엄청나게 많은 회원을 확보하고 있으며 웨이트 트레이닝과 에어로빅 강좌들도 항상 만원이라고 한다 새로운 고객층을 예측해 보는 것도 가능하다 이 지역의 학생들의 경우 Fitness for Life프로그램을 받게 되는데 이러한 프로그램을 통해서 유년시절부터 정규적인 운동 습관을 들이게 하고 있는 것이 그것이다

결론 We should therefore build our next new store in Plainsville

반박 그동안의 경험에 의한 과거 통계가 꼭 여기에도 적용되는건 아니다 5 년전 헬스 클럽이 잘 안되었던게 다른 원인이였을수 있다(강사수준 미달 강좌미비)tourist 에 의한 원인 일수 있다 어렸을때부터 운동을 했다고 해서 커서도 관심이 있지는 않다 (오히려 반감이 있을수 있다 혹은 건강하기에 건강에 관심이 적을수도 있다)운동복이나 신발의 판매가 육체노동에 의한 것일수도 있다

IntroductionSupport1049896In this memorandum the author asserts that Naturersquos Way should build its next newstore in Plainsville To support this assertion the author states that Plainsvillesmerchantsrsquo sales of exercise clothing are going well the local health club has moremembers than ever and a new generation of customers will help to ensure NaturersquosWayrsquos success At first glance the authorrsquos assumption seems convincing but in-depth scrutiny revealsthat it lacks substantial evidence as it stands

Body 1-SamplingTopic Sentence 1To begin with the author assumes that the merchantsrsquo report indicates that the residentsare concerned about their health However this assumption is based on unsubstantiated

GRE AWA John박 박정어학원

data Example 1 (Rebuttal1) First if we do not know the total volume of items sold and the price of the goods exactly we cannot infer whether the residents are actually buying many goods Example 2 (Rebuttal2)In addition to that the report emphasizes the rising sales of running shoes and exerciseclothing however these may not be hot-selling items for Naturersquos Way or may not be theproducts the company is planning to sell Concluding Sentence Therefore in order to make the argument reliable the author should reconsider themerchantsrsquo report with more detailed data

Body 2-CausalTopic Sentence 2Second the author contends that the health clubs classes are full yet this does not meanthat many people actually use the health club other factors may be the real cause forthose closed classes Example 1 (Rebuttal1) To begin with if the health club is very small the number of people working out wouldnot be a large one In fact regular gym-going may just be a vogue among a smallunrepresentative segment of Plainsvilles population Example 2 (Rebuttal2) Moreover it is possible that most of the people who exercise in the health club do weight training and aerobics only to look good and to meet other singles not for their health In that case there would be little demand for health products Concluding SentenceThus the author should not hasten to presume what really caused people to be interested in a healthier lifestyle and enroll in the health club

Body 3-Time-ShiftTopic Sentence 3Finally the author highlights that Naturersquos Way can expect a new generation of customersin Plainsville that will help the company in the long term This notion is mistaken in that itassumes the conditions of the present will continue unchanged in the future Although theschool children are required to participate in the fitness for life program they may notnecessarily buy Naturersquos Ways products Example 1 (Rebuttal1) In the first instance they may suffer a fall in purchasing power arising from future economic difficulties this would cause reluctance to spend a considerable amount of money on health products which tend to be more expensive Example 2 (Rebuttal2)Another possibility is that there may emerge many competitor companies vying with Naturersquos Way so that in the future the school children may not feel the necessity to purchase one companyrsquos health products over anotherrsquosConcluding Sentence Thus the authorrsquos assumption is highly speculative since it relies heavily on unknowablefuture circumstances

ConclusionThesis In sum the author uses many assumptions that are insufficient in supporting his claimsSupportIn order for the authorrsquos claims to be convincing he needs to advance more persuasiveevidence that people in Plainsville really are concerned with their health and health foodThe following was written as a part of an application for a small-business loan by a group of developers in the city of Monroe

A jazz music club in Monroe would be a tremendously profitable enterprise Currently the nearest jazz club is 65 miles away thus the proposed new jazz club in Monroe the C-Note would have the local market all to itself Plus jazz is extremely popular in Monroe over 100000 people attended Monroes annual jazz festival last summer several well-known jazz musicians live in Monroe and the highest-rated radio program in Monroe is Jazz Nightly which airs every weeknight at 7 PM Finally a nationwide study indicates that the typical jazz fan spends close to $1000 per year on jazz entertainment

1 Write a response in which you discuss what specific evidence is needed to evaluate the argument and explain how the evidence would weaken or strengthen the argument

2 Write a response in which you examine the stated andor unstated assumptions of the argument Be

GRE AWA John박 박정어학원

sure to explain how the argument depends on these assumptions and what the implications are for the argument if the assumptions prove unwarranted

3 Write a response in which you discuss what questions would need to be answered in order to decide whether the prediction and the argument on which it is based are reasonable Be sure to explain how the answers to these questions would help to evaluate the prediction

Group error nationwide survey may not reflect local trends Is the nationwide jazz fan population substantialInsufficient non-residents of Monroe may have attended the jazz festival (Body alternative explanation last year may have been an anomaly The author should consider data from various years) The author should indicate how many out of 100000 were Monroe residentsNationwide study Does this reflect Insufficient Citizens of Monroe may continue to go to the jazz club 65 miles away

Are the people in Monroe really interested in jazzMajority of the people who attended the jazz festival might not be Monroe residentsSurvey error nationwide study may not be applicable to MonroeJazz musicians who live in MonroeMonopolyRadio station

In this business application the author claims that the proposed jazz club C Note will be very profitable in Monroe To support this claim the author argues for his case with several evidences At first glance the authorrsquos argument seems convincing however careful scrutiny reveals that his argument in specious

To begin with the author claims that Monroersquos citizens are interested in jazz He presents three evidences First Secondhellip Thirdhellip Howeverhellip

Monroe 시에 있는 재즈 음악 클럽은 수익성이 좋은 사업이다 현재 가장 가까이에 있는 클럽은 65 마일 정도 떨어져 있다 따라서 이번에 세우려고 하는 C Note 는 독보적인 위치를 점할것이다 더군다나 재즈는 이 시에서 가장 인기있는 음악이다 지난 여름 재즈 축제에서는 10 만명 이상의 Morone 시 주민이 참석하였고 몇몇 유명한 재즈 음악가들도 이곳에 살고 있으며 저녁때 방영되는 라디오 프로그램중에서 최고의 시청률을 보이고 있는 것도 Jazz Nightly 이다 전국조사에서도 전형적인 재즈 팬들은 재즈 분야에 년간 1천 달러 가까이 지출하고 있는 것으로 보고되고 있다 따라서 C Note 클럽이 돈을 벌 수 있는 사업이라는 것은 확실한 것이다

결과 It is clear that the C Note cannot help but make money반박 nearest jazz club 이 양질의 써비스로 여전히 손님을 끌수도 있다Festival 에 얼마나 참여하는지가 jazz 의 인기를 반영하지 않는다 뮤지션이 많이 사는거랑 jazz 의 인기가 상관없다라디오 프로그램이 다른 요인에 의해서 인기일수도 있다 (진행자때문)전국 통계 적용 불가화목 실전반_Ms Noh6In this application the author suggests that a jazz club in Monroe will make a number of profits To support this suggestion the author exemplifies the local condition popularity of jazz in Monroe and nationwide study However careful scrutiny of each of the facts reveals that it provides little credible support for the authorrsquos recommendation Good clear intro

First the author assumes that jazz is popular in Monroe because of several facts the jazz festival last year had high participation some famous jazz musicians live in Monroe and the high-rated radio program is lsquoJazz Nightlyrsquo However this assumption has many drawbacks that must be seriously considered(Good topic sentences) If many attendants in the last-yearrsquos festival came from other cities and not Monroe it is hard to conclude that Monroersquos people like jazz Therefore the author must examine how many Monroe residents actually attended the festival On top of that there is little relationship between habitation of famous jazz musician and the popularity of jazz in Monroe Although several well-known musicians live there if they do not take part in any jazz performance of Monroe this might have no effect to the interest of Monroersquos residents

GRE AWA John박 박정어학원

about jazz Finally in the case of radio program this is also not suitable reason why jazz is popular in Monroe It might be possible that people cannot help choosing lsquoJazz Nightlyrsquo because there are few radio programs at Night The fact that the radio program is the highest rating program is not a germane evidence The approximate number of listeners would be the more crucial evidence Therefore the author needs to seriously deliberate the correlation between jazzrsquos popularity in Monroe and his examples (Good logical flow and clarity)

Second the author uses as evidence the nationwide study that jazz fans spend much money on jazz entertainment to substantiate why starting a jazz club in Monroe will be profitable In other words the author assumes that the characteristics of a nationwide study can be applied to Monroe The national study would lend support to the applicantrsquos claim only if residents in Monroe typify national jazz fans However the author does not provide credible evidence that this is the case Moreover the populations of jazz fans nationwide may be insubstantial Thus the author should not infer hastily that Monroersquos residents will spend much money on enjoying jazz from the nationwide study

Lastly even if jazz is popular in Monroe C Note may not be successful It is entirely possible that residents might still prefer other clubs where they have always went In addition there is another possibility that the nearest jazz club will attract many of Monroersquos people because it serves fine performances and is equipped with favorite facilities Without considering these other possibilities the author cannot make his argument convincing In sum the author presents many reasons that are insufficient in supporting his or her claim In order for the authorrsquos claims to be convincing he needs to advance more persuasive evidence such as the total number of Monroe residents who attended the jazz festival the effects on the popularity of jazz by the musicians living in Monroe and the actual number of residents who would typify themselves to be jazz fans through a local survey Without substantial evidence that C Note will be successful in Monroe the businessmen may be overinvesting in what might lead to a business failureExcellent clarity Score 50

The following appeared in a newsletter offering advice to investors

Over 80 percent of the respondents to a recent survey indicated a desire to reduce their intake of foods containing fats and cholesterol and today low-fat products abound in many food stores Since many of the food products currently marketed by Old Dairy Industries are high in fat and cholesterol the companys sales are likely to diminish greatly and company profits will no doubt decrease We therefore advise Old Dairy stockholders to sell their shares and other investors not to purchase stock in this company

Write a response in which you discuss what questions would need to be answered in order to decide whether the advice and the argument on which it is based are reasonable Be sure to explain how the answers to these questions would help to evaluate the advice

Survey 80

GRE AWA John박 박정어학원

Old Dairy could change their products and manufacture low fat dairy foodsLess competing companies Old Dairy could eventually be the only company that produces hellipImprecise numbers and measurementsCustomers may still buy high fat dairy products

The author of the newsletter is offering potentially dangerous advice by recommending Old Dairy stockholders to withdraw investment and stop purchase What is more the authorrsquos prediction debases the reputation and business of Old Dairy and if false could devoid the investment opportunity of the newsletter readers Therefore investors should examine whether the authorrsquos evidences are substantial

To begin with the author states that 80 percent of the respondents in a survey indicated a desire to reduce their intake of foods He therefore argues that Old Dairyrsquos high fat and cholesterol products would decrease in sales However the author makes a crucial error in this argument First the author provides no evidence that the surveyrsquos results are statistically reliable Were they representative of all the customers Were they chosen for the survey randomly Furthermore the desire to reduce fat and cholesterol intake is a pervasive trend in todayrsquos opulent society however the author erroneously identifies this as a new phenomenon which will affect consumer trends Second having a desire to reduce fat and cholesterol intake does not necessarily indicate that people who have this desire will actually reduce consuming these types of products It is entirely possible that they may continue buying Old Dairy products for its quality and taste Accordingly the author cannot draw any firm conclusion that people will not buy Old Dairy products Therefore if any of these cases are true the author may be offering investors a detrimental investment advice

최근 조사에 대한 응답자중 80 이상이 자신이 먹는 음식에서 지방과 콜레스테롤의 함유량을 줄이고 싶다고 한다 아울러 요즘은 많은 식료품 가계에서 저지방 제품들을 많이 취급하고 있다 현재 Old Dairy Industries가 판매하고 있는 많은 음식제품들은 지방과 콜레스테롤이 높기 때문에 이 회사의 매출이 격감할 것으로 보이며 당연히 매출이익도 줄어들것이다 따라서 이 회사의 주주들은 주식을 매각하고 다른 주식 투자가들도 이 회사의 주식을 매입하지 않는 것이 좋다

결론 Old Dairy stockholders to sell their shares and other investors not to purchase stock in this company

반박 모든 상품이 다 고 지방 고 칼로리는 아니다(비록 많을지라도) 일부의 식품의 경우 기호에 맞어서 히트해서 전체적인 수입이 증가할 수도 있다국내시장만 생각할 수 없다( 외국시장에서 호황을 누릴수 있다 )입맛이라는게 즉각 바뀌는게 아니다

The following appeared in a letter to the editor of the Balmer Island Gazette

On Balmer Island where mopeds serve as a popular form of transportation the population increases to 100000 during the summer months To reduce the number of accidents involving mopeds and pedestrians the town council of Balmer Island should limit the number of mopeds rented by the islands moped rental companies from 50 per day to 25 per day during the summer season By limiting the number of rentals the town council will attain the 50 percent annual reduction in moped accidents that was achieved last year on the neighboring island of Seaville when Seavilles town council enforced similar limits on moped rentals

1 Write a response in which you discuss what questions would need to be answered in order to decide whether the recommendation is likely to have the predicted result Be sure to explain how the answers to these questions would help to evaluate the recommendation

2 Write a response in which you discuss what questions would need to be answered in order to decide whether the prediction and the argument on which it is based are reasonable Be sure to explain how the answers to these questions would help to evaluate the prediction

3 Write a response in which you examine the stated andor unstated assumptions of the argument Be sure to explain how the argument depends on these assumptions and what the implications are for the argument if the assumptions prove unwarranted

Whatrsquos the actual population of Balmer Island 100000mdashis this a significant increase What kind of accidents Skin abrasions or serious injury And compared to Seaville how serious are the accidents and the actual number of accidents Did Seaville enforce other restrictions like safety signsHow different are the conditions of Balmer

GRE AWA John박 박정어학원

and Seaville regarding population road (safety) conditions topography other town-government regulation How much will the economy of Balmer be affected do to this restriction Could it cause an economic recession due to the fact that these rental companiesrsquo chance to make money is only during the summer thereby weakening the economic infrastructure Are there any other ways that could better alleviate the accident rate

Statistics 50-impreciseAnalogy Balmer compared with TorseauFalse Cause Accidents might have occurred because of reasons other than mopeds False Cause population increase may not be part of the cause of the accidentsOther explanations for the accident pedestrians few road safety regulations narrow roadsThere could be other better solutionshellip

Balmer Island의 인구가 여름철에는 십만명으로 늘어난다 2륜차와 보행자간 사고를 줄이기 위해 시의회는 6개의 자전거를 포함한 2륜차 대여업체에게 이 기간동안에는 대여숫자를 일일 50에서 30으로 제한하도록 할 것이다 대여숫자를 줄임으로써 시 의회는 지난해 이웃한 Torseau섬에서 이와 동일한 규제를 시행해서 50나 줄인 결과를 보고 마찬가지로 50를 줄일수 있다고 확신하고 있다

결론 The town council of Balmer Island should linit the number

반박 보행자의 부실에 의해서 사고가 많이 일어날수도 있다렌탈수의 줄임만이 대책은 아니다(대부분의 사람들이 렌탈 보다는 소유하고 있을 수도 있다)옆섬과는 상황이 다를수도 있다(그 섬에서는 사고의 원인이 많은 자전거 수로 인한것일수있다) 하지만 이 섬은 좁은 도로가 원인일 수도 있고 도로 안전 장치의미비가 원일일수 있다

In this letter the author recommends that Balmer Island should limit the number moped rentals from 50 to 30 per day To support this recommendation the author points out several reasons However careful scrutiny of each of the facts reveals that it is filled with unanswered questions that could significantly weaken the authorrsquos recommendation with loops and holes which are answered

The recommendation depends on the assumption that no alternative means of reducing the number of accidents are available However the author fails to offer any evidence to substantiate this crucial assumption It is highly possible that means other than this would better solve the problem Perhaps they could widen the roads or put-up more safety signs Or perhaps the accidents were due to the lack of skills in which case proper safety training would significantly alleviate the problem Without considering and ruling out these and other alternative means of reducing accidetns the author cannot confidently conclude that merely emulating Torseau would suffice Moreover the author is advising a recommendation which could potentially harm the economy of Balmer Island sincehellip Moreover the Balmer Island should alternative means to reduce accidents because limiting moped rentals during the summer could harm the economy of Balmerhellip

First of all the author believes that increase in population and the number of moped rentals are responsible for the accidents It is entirely possible that other factors are responsible for the accidents Perhaps Balmer Islandrsquos lack of safety signs was a major factor Or maybe the roads are narrow and dangerous on the Island therefore the town council could enforce stricter traffic regulations to alleviate the problem Accordingly if either of these scenarios is true the author cannot draw any firm conclusion that increase in the number of population and moped rentals are the cause of the accidents

The author of this editorial recommends that to reduce accidents involving mopeds andpedestrians Balmer Islands city council should restrict moped rentals to 30 per day down from50 at each of the islands six rental outlets To support this recommendation the author citesthe fact that last year when nearby Torseau Islands town council enforced similar measuresTorseaus rate of moped accidents fell by 50 For several reasons this evidence providesscant support for the authors recommendationTo begin with the author assumes that all other conditions in Balmer that might affect therate of moped-pedestrian accidents will remain unchanged after the restrictions are enactedHowever with a restricted supply of rental mopeds people in Balmer might purchase mopedsinstead Also the number of pedestrians might increase in the future with more pedestriansespecially tourists the risk of moped-pedestrian accidents would probably increase For thatmatter the number of rental outlets might increase to make up for the artificial supplyrestriction per outlet--a likely scenario assuming moped rental demand does not declineWithout considering and ruling out these and other possible changes that might contribute to ahigh incidence of moped-pedestrian accidents the author cannot convince me that theproposed restrictions will necessarily have the desired effect

GRE AWA John박 박정어학원

Next the author fails to consider other possible explanations for the 50 decline inTorseaus moped accident rate last year Perhaps last year Torseau experienced unusually fairweather during which moped accidents are less likely Perhaps fewer tourists visited Tot seanlast year than during most years thereby diminishing the demand for rental mopeds to belowthe allowed limits Perhaps last year some of Torseaus moped rental outlets purchased newmopeds that are safer to drive Or perhaps the restrictions were already in effect but were notenforced until last year In any event a decline in Torseaus moped accident rate during onlyone year is scarcely sufficient to draw any reliable conclusions about what might have causedthe decline or about what the accident rate will be in years aheadAdditionally in asserting that the same phenomenon that caused a 50 decline in mopedaccidents in Torseau would cause a similar decline in Balmer the author relies on what mightamount to an unfair analogy between Balmer and Torseau Perhaps Balmers ability to enforcemoped-rental restrictions does not meet Torseaus ability if not then the mere enactment ofsimilar restrictions in Balmer is no guarantee of a similar result Or perhaps the demand formopeds in Torseau is always greater than in Balmer Specifically if fewer than all availablemopeds are currently rented per day from the average Balmer outlet while in Torseau everyavailable moped is rented each day then the proposed restriction is likely to have less impacton the accident rate in Balmer than in TorseauFinally the author provides no evidence that the same restrictions that served to reduce theincidence of all moped accidents by 50 would also serve to reduce the incidence ofaccidents involving mopeds and pedestrians by 50 Lacking such evidence it is entirelypossible that the number of moped accidents not involving pedestrians decreased by a greaterpercentage while the number of moped-pedestrian accidents decreased by a smallerpercentage or even increased Since the author has not accounted for these possibilities theeditorials recommendation cannot be taken seriouslyIn conclusion the recommendation is not well supported To convince me that the proposedrestriction would achieve the desired outcome the author would have to assure me that nochanges serving to increase Balmers moped-pedestrian accident rate will occur in theforeseeable future The author must also provide dear evidence that last years decline inmoped accidents in Torseau was attributable primarily to its moped rental restrictions ratherthan to one or more other factors In order to better evaluate the recommendation I wouldneed more information comparing the supply of and demand for moped rentals on the twoislands I would also need to know the rate of mopedpedestrian accidents in Torseau both priorto and after the restrictions were enforced in TorseauThe following appeared in a magazine article about planning for retirement

Clearview should be a top choice for anyone seeking a place to retire because it has spectacular natural beauty and a consistent climate Another advantage is that housing costs in Clearview have fallen significantly during the past year and taxes remain lower than those in neighboring towns Moreover Clearviews mayor promises many new programs to improve schools streets and public services And best of all retirees in Clearview can also expect excellent health care as they grow older since the number of physicians in the area is far greater than the national average

Write a response in which you discuss what specific evidence is needed to evaluate the argument and explain how the evidence would weaken or strengthen the argument

-Natural beauty and consistent climate may not be the most wanted qualities-Housing costs could have lowered on a national level wealthy retirees may not care about costs-Taxes may be high compared to the nationrsquos average tax rate-What about other qualities of Clearview Crime rate what qualities would retirees want -If schools streets and public services need improvement then this is proof that the current condition of Clearview is low Or due to budgetary reasons the mayor may not follow-up on his promise because of lowered tax rate -Schools and people who are retired no relationship-Physicians What kind of physicians Number is irrelevant Are these physicians capable of addressing the illnesses of old people

This author argues that anyone seeking a place to retire should choose Clearview To supportthis argument the article cites Clearviews consistent climate and natural beauty its fallinghousing costs its low property taxes compared to nearby towns and the mayors promise toimprove schools streets and services The article also claims that retirees can expectexcellent health care because the number of physicians in Clearview greatly exceeds thenational average This argument is flawed in several critical respectsTo begin with although consistent climate and natural beauty might be attractive to manyretirees these features are probably not important to all retirees For many retirees it isprobably more important to live near relatives or even to enjoy changing seasons Thus I

GRE AWA John박 박정어학원

cannot accept the authors sweeping recommendation for all retirees on this basisAlso Clearviews declining housing costs do not necessarily make Clearview the best placeto retire for two reasons First despite the decline Clearviews housing costs might be highcompared to housing costs in other cities Secondly for wealthier retirees housing costs arenot likely to be a factor in choosing a place to retire Thus the mere fact that housing costshave been in decline lends scant support to the recommendationThe articles reliance on Clearviews property-tax rates is also problematic in two respectsFirst retirees obviously have innumerable choices about where to retire besides Clear viewand nearby towns Secondly for retirees who are well-off financially property taxes are notlikely to be an important concern in choosing a place to retire Thus it is unfair to infer fromClearviews property-tax rates that retirees would prefer ClearviewYet another problem with the argument involves the mayors promises In light of Clearviewslow property-tax rates whether the mayor can follow through on those promises is highlyquestionable Absent any explanation of how the city can spend more money in the areas citedwithout raising property taxes I simply cannot accept the editorials recommendation on thebasis of those promises Besides even if the city makes the improvements promised thoseimprovements--particular the ones to schools--would not necessarily be important to retireesFinally although the number of physicians in Clearview is relatively high the per capitanumber might be relatively low Moreover it would be fairer to compare this per capita numberwith the per capita number for other attractive retirement towns--rather than the nationalaverage After all retirees are likely to place a relatively heavy burden on health-careresources Besides the article provides no assurances that the number of physicians inClearview will remain high in the foreseeable futureIn conclusion the recommendation is poorly supported To strengthen it the author mustconvince me--perhaps by way of a reliable survey--that the key features that the vast majorityof retirees look for in choosing a place to live are consistent climate natural beauty and lowhousing costs The author must also provide better evidence that Clear views property taxesare lower than the those of cities in other areas The author must also explain how the city canmake its promised improvements without raising property taxes Finally to better assess theargument I would need to now how the per capita number of physicians in Clearview wouldcompare to the national average in the futureThe following appeared as a letter to the editor from a Central Plaza store owner

Over the past two years the number of shoppers in Central Plaza has been steadily decreasing while the popularity of skateboarding has increased dramatically Many Central Plaza store owners believe that the decrease in their business is due to the number of skateboard users in the plaza There has also been a dramatic increase in the amount of litter and vandalism throughout the plaza Thus we recommend that the city prohibit skateboarding in Central Plaza If skateboarding is prohibited here we predict that business in Central Plaza will return to its previously high levels

Write a response in which you discuss what questions would need to be answered in order to decide whether the recommendation is likely to have the predicted result Be sure to explain how the answers to these questions would help to evaluate the recommendation

Why two years ago What happened two years ago which started this declineIs the dramatic increase in the ldquopopularityrdquo of skateboarding the cause of the steady decline of shoppers Are there any malls nearby Were there any changes nearby which could affect the decline in customersmdasha big mall perhaps Could the decline be due to the shop ownersHow many skateboarders use the plazaWhere do they skateboardDo they shop and are they customersAre the increase in litter and vandalism due to skateboarders Could this be alleviated by installing CCTVs and hiring security

This editorial concludes that the city should ban skateboarding from its downtown CentralPlaza in order to attract visitors to that area to return the area to its former glory and to makeit a place where people can congregate for fun and relaxation To justify this conclusion theeditorial points out that skateboarders are nearly the only people one sees anymore at CentralPlaza and that the Plaza is littered and its property defaced The editorial also points out thatthe majority of downtown merchants support the skate boarding ban This argument is flawedin several critical respectsFirst the editorials author falsely assumes that a ban on skateboarding is both necessaryand sufficient to achieve the three stated objectives Perhaps the city can achieve thoseobjectives by other means as well--for example by creating a new mall that incorporates anattractive new skateboard park Even if banning skateboarders altogether is necessary to meetthe citys goals the author has not shown that this action by itself would suffice Assuming thatthe Plazas reputation is now tarnished restoring that reputation and in turn enticing peopleback to the Plaza might require additional measures--such as removing litter and graffiti

GRE AWA John박 박정어학원

promoting the Plaza to the public or enticing popular restaurant or retail chains to the PlazaSecondly the editorial assumes too hastily that the Plazas decline is attributable to theskateboarders--rather than to some other phenomenon Perhaps the Plazas primary appeal inits glory days had to do with particular shops or eateries which were eventually replaced byless appealing ones Or perhaps the crime rate in surrounding areas has risen dramatically forreasons unrelated to the skateboarders presence at the Plaza Without ruling out these andother alternative explanations for the Plazas decline the editorials author cannot convince methat a skateboard ban would reverse that declineThirdly the editorials author might be confusing cause with effect--by assuming that theskateboarders caused the abandonment of the Plaza rather than vice versa It is entirelypossible that skateboarders did not frequent the Plaza until it was largely abandoned--andbecause it had been abandoned In fact this scenario makes good sense since skateboardingis most enjoyable where there are few pedestrians or motorists to get in the wayFourth it is unreasonable to infer from the mere fact that most merchants favor the ban thatthe ban would be effective in achieving the citys objectives Admittedly perhaps thesemerchants would be more likely to help dean up the Plaza area and promote their businesseswere the city to act in accordance with their preference Yet lacking any supporting evidencethe author cannot convince me of this Thus the survey amounts to scant evidence at best thatthe proposed ban would carry the intended resultFinally the author recommends a course of action that might actually defeat the citysobjective of providing a fun and relaxing place for people to congregate In my experienceskateboarding contributes to an atmosphere of fun and relaxation for adults and children alikemore so than many other types of ambiance Without considering that continuing to allowskateboarding--or even encouraging this activity--might achieve the citys goal more effectivelythan banning the activity the author cannot convincingly conclude that the ban would be in thecitys best interestsIn sum the argument is a specious one To strengthen it the editorials author must providedear evidence that skateboarding and not some other factor is responsible for the conditionsmarking the Plazas decline The author must also convince me that no alternative means ofrestoring the Plaza are available to the city and that the proposed ban by itself would suffice toattract tourists and restore the Plaza to its former glory Finally to better assess the argument itwould be useful to know the circumstances under which the downtown merchants would bewilling to help the city achieve its objectives

6그룹 약한 비유 빈출

The following recommendation appeared in a memo from the mayor of the town of Hopewell

Two years ago the nearby town of Ocean View built a new municipal golf course and resort hotel During the past two years tourism in Ocean View has increased new businesses have opened there and Ocean Views tax revenues have risen by 30 percent Therefore the best way to improve Hopewells economymdashand generate additional tax revenuesmdashis to build a golf course and resort hotel similar to those in Ocean View

Write a response in which you examine the stated andor unstated assumptions of the argument Be sure to explain how the argument depends on these assumptions and what the implications are for the argument if the assumptions prove unwarranted

GRE AWA John박 박정어학원

Assumptions The author assumes that OVrsquos municipal golf course and resort hotel caused tourism new businesses and increased tax revenues There may be other reasons advertising promo He assumes that this will continueAssumes that Ocean View and Hopewell are similar in many waysmdashthe name suggests otherwise OV may have always been a tourist attractions for its beaches We need to know the topography

2년전 Ocean View 시는 시정 소유 골프 및 휴양지 호텔을 신축했다 그리고 지난 2년동안 이 시의 관광객이 증가했으며 새로운 사업들이 생겨났다 그에따라 시의 세수도 30나 증가했다 Hopewell의 경제를 향상시키고 아울러 세수를 늘릴 수 있는 가장 좋은 방법은 Ocean View에 세워진 것과 같은 골프 시설과 휴양지 호텔을 신축하는 것이다

1 다른 요인으로 관광 산업이 발전했을 수도 있다 문화 유적이 발견이 되었거나 도로의 정비등으로 여행자가 늘었을 수도 있다

2 관광 산업의증가가 늘어난 세수의 원인이 아니라 새로 유입된 인구의 증가나 다른 공장에서 발생한 것일 수 있다

3 2년동안 한참 골프가 붐을 이루었을 수 있다 경제상황이 나빠지거나 다른 레포츠가 인근 지역에 생겨난다면 골프하는 사람이 줄어들 수 있다

In this memo HopeweUs mayor recommends that in order to stimulate the towns economyand boost tax revenues HopeweU should build a new golf course and resort hotel just as thetown of Ocean View did two years ago To support this recommendation the mayor points outthat in Ocean View during the last two years tourism has increased new businesses haveopened and tax revenues have increased by 30 I find the mayors argument unconvincingin several important respectsFirst of all it is possible that the mayor has confused cause with effect respecting the recentdevelopments in Ocean View Perhaps Ocean Views construction of a new golf course andhotel was a response to previous increases in tourism and business development increasesthat have simply continued during the most recent two years Since the mayor has failed toaccount for this possibility the claim that Hopewell would boost its economy by alsoconstructing a golf course and hotel is completely unwarrantedSecondly the mayor fails to account for other possible causes of the trends in Ocean Viewduring the last two years The increase in tourism might have been due to improving economicconditions nationwide or to unusually pleasant weather in the region The new businessesthat have opened in Ocean View might have opened there irrespective of the new golf courseand hotel And the 30 increase in tax revenues might have been the result of an increase intax rates or the addition of a new type of municipal taxWithout ruling out these and other alternative explanations for the three recent trends inOcean View the mayor cannot reasonably infer based on those trends that Hopewellseconomy would benefit by following Ocean Views exampleThirdly even if the recent trends in Ocean View are attributable to the construction of the newgolf course and hotel there the mayor assumes too hastily that the golf course and hotel willcontinue to benefit that towns overall economy The mayor has not accounted for thepossibility that increased tourism will begin to drive residents away during tourist season orthat new business development will result in the towns losing its appeal as a place to visit or tolive Unless the mayor can convince me that these scenarios are unlikely I cannot accept themayors recommendation that Hopewell follow Ocean Views exampleFinally the mayors argument rests on the unsubstantiated assumption that Hopewell andOcean View are sufficiently alike in ways that might affect the economic impact of a new golfcourse and hotel Hopewell might lack the sort of natural environment that would attract moretourists and new businesses to the town--regardless of its new golf course and hotel For thatmatter perhaps Hopewell already contains several resort hotels and golf courses that are notutilized to their capacity If so building yet another golf course and hotel might amount to amisallocation of the towns resources--and actually harm the towns overall economyIn sum the mayors recommendation is not well supported To bolster it the mayor mustprovide better evidence that Ocean Views new golf course and hotel and not some otherphenomenon--has been responsible for boosting Ocean Views economy during the last twoyears To better assess the recommendation I would need to know why Ocean View decidedto construct its new golf course and hotel in the first place--specifically what events prior toconstruction might have prompted that decision I would also need to thoroughly compare

GRE AWA John박 박정어학원

HopeweU with Ocean View--especially in terms of their appeal to tourists and businesses--todetermine whether the same course of action that appears to have boosted Ocean Viewseconomy would also boost Hopewells economy

The following is part of a memorandum from the president of Humana University

Last year the number of students who enrolled in online degree programs offered by nearby Omni University increased by 50 percent During the same year Omni showed a significant decrease from prior years in expenditures for dormitory and classroom space most likely because instruction in the online programs takes place via the Internet In contrast over the past three years enrollment at Humana University has failed to grow and the cost of maintaining buildings has increased along with our budget deficit To address these problems Humana University will begin immediately to create and actively promote online degree programs like those at Omni We predict that instituting these online degree programs will help Humana both increase its total enrollment and solve its budget problems

Write a response in which you discuss what questions would need to be answered in order to decide whether the prediction and the argument on which it is based are reasonable Be sure to explain how the answers to these questions would help to evaluate the prediction

Is Omni University successful due to the online degree program 50 Is the decrease in expenditures for dormitory and classroom space due to the decrease in of on-campus students Which classes were successful Does HU have those classes

Even if the long-distance degree programs at Omni University benefited the school the presidentrsquos recommendation that Human College should emulate Omni University is too hasty First OUrsquos name implies that the school would have more majors than Humanahellip the president should examine which degrees were in the long-distance programhellip

지난해에는 Omni 대학에서 개강했던 원거리 학생 학점 취득 프로그램을 등록했던 학생들의 숫자가 50나 증가했다 같은해 기간동안 Omni 대학에서는 그 전년도부터 기숙사와 학급의 공간 확충을 위한 예산을 대폭 줄였는데 이는 이 원거리 학점 취득 프로그램이 양방향 비디오 컴퓨터 접속을 통해서만 가능한 수업지도 방식이기때문인 것으로 보인다 반면 지난 3개년 동안 Humana 대학에서의 수강률은 감소한데다가 건물

GRE AWA John박 박정어학원

유지비도 올랐다 따라서 Humana대학의 수강을 늘리고 예산손실을 회복하기 위해서는 Omni 대학에서 취한 조치와 같은 능동적인 프로그램을 추진해야 한다

결론 we should initiate and actively promote long-distance degree programs like those at Omni 반박 원거리 학생 취득 프로그램 숫자가 증가한거하고 예산이 줄어드는 것 사이에 연관이 약하다 (causal 학생의 증가로 관리비용 증가할수 있음 원거리 수업가능 장비도입에의한 비용발생)bad analogy(omni university 하고 같은 조건이 아니다 )-gt omni college 가 강좌내용이 좋아서 학생의 등록이 많을수 있다 Humana 대학에서 만들었다 하더라도 인기 없을수 있음다른 요인에 의해서 Humana 대학의 수강 인원이 증가할수 있음(비록 과거엔 인기가 없었을지라도)

The following appeared as part of a business plan developed by the manager of the Rialto Movie Theater

Despite its downtown location the Rialto Movie Theater a local institution for five decades must make big changes or close its doors forever It should follow the example of the new Apex Theater in the mall outside of town When the Apex opened last year it featured a video arcade plush carpeting and seats and a state-of-the-art sound system Furthermore in a recent survey over 85 percent of respondents reported that the high price of newly released movies prevents them from going to the movies more than five times per year Thus if the Rialto intends to hold on to its share of a decreasing pool of moviegoers it must offer the same features as Apex

Write a response in which you discuss what questions would need to be answered in order to decide whether the recommendation is likely to have the predicted result Be sure to explain how the answers to these questions would help to evaluate the recommendation

Before following through this business plan the manager should investigate the cause of Rialtorsquos unsuccessful business

The author provides no evidence that the surveyrsquos results are statistically reliable The surveyrsquos sample of 85 percent must be sufficient in size and representative of overall population of the city where Rialto and Apex is serving Lacking evidence of a sufficiently representative sample the author cannot justifiably rely on the survey to draw any conclusion whatsoever The author does not indicate that Apex is indeed currently successful However even if Apex is enjoying success the argument relies on what might be a false analogy between Rialto and Apex In order for Apex to serve as a model that Rialto should emulate the author must assume that all relevant circumstances are essentially the same However this assumption is unwarranted For example the argument overlooks the face that Apex is located in a strategic placemdashbeside a mall where customers can not only watch a movie but also enjoy shopping Therefore simply changing the facility to that of Apex may not lead to success

The author does not mention whether Apex is successful or not Nevertheless even if Apex is currently successful the argument relies on what might be a false analogy between Rialto and Apex In order for Apex to serve as a model that Rialto should emulate the author must assume that all relevant circumstances are essentially the same However this assumption is unwarranted For example the argument overlooks the fact that these two institutions are located in different locations Rialto in downtown and Apex in a mall outside of town Although Apex opened with state-of-the-art facilities the decisive factor in its success could be due to its strategic location of being in a mall People could enjoy both shopping and movies at one location thus they may prefer Apex over Rialto Furthermore the place where people enjoy leisure activities has shifted in the past decades for most cities from downtown to the suburbs Therefore Rialto may not be successful even if it emulates Apexrsquos facilities A better business plan may be relocating Apex to the thriving section of the downtown

Rialto 극장은 지난 50여년간 지역 회관으로써 시내에 위치해 있으면서도 이제 변화를 꾀하지 않으면 문을 닫을

GRE AWA John박 박정어학원

판이다 이 극장은 시외 쇼핑타운에 새로 들어선 Apex 극장의 사례를 본받아야 했다 Apex가 지난해 개업했을 당시 이 극장은 비디오 아케이드 플러쉬 카펫트 바닥과 좌석 그리고 최신 음향시설을 갖추었다 더군다나 최근 조사에서는 응답자의 85 이상이 새로 출시된 영화 입장료가 비싼 탓으로 지난해보다 5배이상의 관람객이 줄어들었다고 나타났다 따라서 Rialto 극장이 줄어들고 있는 관람객을 뺐기지 않고 유지하려면 Apex와 같은 시설들을 갖추어야 할 것이다주장 리알토 극장이 줄어들고 있는 관람객을 뺐기지 않고 유지하려면 Apex와 같은 시설들을 갖추어야 할 것이다

1 조사에서 응답자가 전체를 대표할 수 없다 2 apex 극장이 좋은 시설을 갖추고 있지만 그로 인해 수익이 많이 발생했다는 말이 없으므로 시설투자를

하고도 좋은 결과를 얻을 수 있을지 그 근거가 미흡하다3 좋은 영화가 출시된다면 입장료가 비싸도 영화관에서 꼭 보려고 할 수 있다 4 rialto 가 시설이 아닌 다른 요인에 의해 장사가 안될수도 있다( 우범 지역이라든지)

The following is a recommendation from the business manager of Monarch Books

Since its opening in Collegeville twenty years ago Monarch Books has developed a large customer base due to its reader-friendly atmosphere and wide selection of books on all subjects Last month Book and Bean a combination bookstore and coffee shop announced its intention to open a Collegeville store Monarch Books should open its own in-store cafeacute in the space currently devoted to childrens books Given recent national census data indicating a significant decline in the percentage of the population under age ten sales of childrens books are likely to decline By replacing its childrens books section with a cafeacute Monarch Books can increase profits and ward off competition from Book and Bean

Write a response in which you examine the stated andor unstated assumptions of the argument Be sure to explain how the argument depends on these assumptions and what the implications are for the argument if the assumptions prove unwarranted

The following is a recommendation from the business manager of Monarch Books

Since its opening in Collegeville twenty years ago Monarch Books has developed a large customer base due to its reader-friendly atmosphere and wide selection of books on all subjects Last month Book and Bean a combination bookstore and coffee shop announced its intention to open a Collegeville store Monarch Books should open its own in-store cafeacute in the space currently devoted to childrens books Given recent national census data indicating a significant decline in the percentage of the population under age ten sales of childrens books are likely to decline By replacing its childrens books section with a cafeacute Monarch Books can increase profits and ward off competition from Book and Bean

1 Write a response in which you discuss what questions would need to be answered in order to decide whether the recommendation is likely to have the predicted result Be sure to explain how the answers to these questions would help to evaluate the recommendation

2 Write a response in which you discuss what specific evidence is needed to evaluate the argument and explain how the evidence would weaken or strengthen the argument

No evidence regarding Monarch Bookrsquos successEven if Regal Bookrsquos is successful this may not be attributable to the cafeacute False analogy Emulating may not lead to success Other factors may be involvedInsufficient condition The national census is not enough evidence that childrenrsquos book sales will decline Can

GRE AWA John박 박정어학원

the national census represent the local child populationDid opening a cafeacute boost sales for Regal Books Even assuming Regal is successful by opening a cafeacute this may not be suitable for Monarch which plans to close the childrenrsquos book section to establish a cafe Imprecise language ldquorelatively little spacerdquo how smallThe managerrsquos recommendation contradicts what he says Since Monarch is popular for its wide selection of books closing a selection which targets a major group of readers may hurt Monarchrsquos salesIs this the best way to compete

When Stanley Park first opened it was the largest most heavily used public park in town It is still the largest park but it is no longer heavily used Video cameras mounted in the parks parking lots last month revealed the parks drop in popularity the recordings showed an average of only 50 cars per day In contrast tiny Carlton Park in the heart of the business district is visited by more than 150 people on a typical weekday An obvious difference is that Carlton Park unlike Stanley Park provides ample seating Thus if Stanley Park is ever to be as popular with our citizens as Carlton Park the town will obviously need to provide more benches thereby converting some of the unused open areas into spaces suitable for socializing

Write a response in which you examine the stated andor unstated assumptions of the argument Be sure to explain how the argument depends on these assumptions and what the implications are for the argument if the assumptions prove unwarranted

Stanley 파크가 처음 개장했을 당시 가장 크고 가장 많이 이용되는 공원이었다 아직도 공원중에서는 가장 크지만 이용률은 상당히 떨어졌다 지난달 공원 주차장에 설치해놓은 비디오 카메라를 통해 보면 drop(주차장으로 여겨짐) 이용률이 가장 높았다 수치상으로는 하루 평균 50대의 차량만이 이용하였다 반면 직장 중심거리에 위치한 작은 규모의 Carlton 파크는 주당 무려 150여명 이상이 이용하고 있다 Stanley 파크와는 달리 Carlton 파크에는 의자가 있다는 것이 가장 뚜렷한 차이점이다 따라서 Stanley 파크가 Carlton 파크처럼 시민들이 자주 이용하는 공원이 되기 위해서는 벤치를 설치할 필요가 있으며 이렇게 사용되지 않는 일부 공간을 활용해서 사교를 위한 공간으로 바꾸어야 한다 ===gtdrop 에 대한 첨부사항 (영영사전내용입니다)---- a place or central depository to which something (as mail money or stolen property) is brought for distribution or transmission also the act of depositing something at such a place dropgt

주장 if Stanley Park is ever to be as popular with our citizens as is Carlton Park the town will obviously need to provide more benches thereby converting some of the unused open areas into spaces suitable for socializing1 조사가 언제 이루어진 것인가 조사가 언제 실시되었느냐에 따라 결과가 다를 수있다 현재는 다시 스탠리 파크가 늘어났었을 수 있다 2 벤치를 많이 설치했다고 해서 많은 관광객이 오지 않을수 있다(사람들이 벤치나 사교 공간을 원한다는 어떠한 자료도 없다)3스탠리 파크 주변에 교통 상황이 악화가 되었거나 칼튼 파크에서 문화행사등을 많이 가져서 이용객이 줄어든것일 수도 있다 4 칼튼 파크가 중심지에 있어서 접근성이 좋을수 있다5 조사가 같은 시간을 기준으로 한게 아니다(하나는 주중이고 하나는 주말이다)6사람의 수와 차의 대수를 같은것으로 비교할수 없다 (차안에 몇 명이 타고 있는지 모르고 대중교통을 이용해서 왔을수도 있다)

Page 8: GRE writing argument brain storm

GRE AWA John박 박정어학원

2 도서관에 비치된 책이 미스터리 소설이 더 많아서 선택의 여지가 없을 수 있다

High Frequency Group 2 National-gtLocal Local-gtNational amp Group Member Error

The following appeared in a letter to the editor of a Batavia newspaper

The department of agriculture in Batavia reports that the number of dairy farms throughout the country is now 25 percent greater than it was 10 years ago During this same time period however the price of milk at the local Excello Food Market has increased from $150 to over $300 per gallon To prevent farmers from continuing to receive excessive profits on an apparently increased supply of milk the Batavia government should begin to regulate retail milk prices Such regulation is necessary to ensure fair prices for consumers

Write a response in which you discuss what questions would need to be answered in order to decide whether the recommendation is likely to have the predicted result Be sure to explain how the answers to these questions would help to evaluate the recommendation

First the author points to the fact that hellip25 increase of dairy farms may be(could be might be) due to population increaseThe milk price increase could be reflecting the increase in cost of livingThe actual price of milk might be cheaper if the economy is experiencing inflationOne market Excello Food cannot reflect the price increase of every market in BataviaThe author claims that farmers are receiving excessive profits however this might not be the caseThe author claims that by regulating milk prices the government will successfully ensure both lower prices and an adequate supply of milk for consumers However this might not be the best solutionThe writer adduces the information released by the department of agriculture that the number of dairy farms has

increased by 25 percent over the last 10 years and thus he claims that dairy farmers are earning excessive profits from milk However the information is filled with loops and holes which need to be filled First what percentage of the milk are sold as milk

Are the dairy farms processing the milk into other dairy products Or are they selling them as milkmdashWhat percentage of the milk is sold as milk Could this be a natural increase

Can the price at the local Excello Food Market represent the price of milk in stores throughout all of Batavia

Are the farmers receiving excessive profits What is the cost of producing milk Could such regulation offer fair prices to consumers

Topic SentenceTo begin with the author assumes that the price of Excello Food Market which has increased from $150 to over $300 per gallon can be applied to the whole nation of Batavia however the author fails to provide evidence to substantiate this assumptionExampleFirstIn additionTherefore in order to make the argument more reliable the author should consider the prices of a large number markets all over the countryThere could be an economic inflation

GRE AWA John박 박정어학원

ldquoBatavia 지역의 농림부의 보고에 따르면 전국적으로 낙농업자의 숫자가 과거 10 년전에 비해 25나 증가했다고 합니다 그러나 동기간 동안 이 지역에 있는 Excello Food Market 에서 우유의 가격은 갤런당 15 불에서 3 불로 증가했습니다 증가된 우유 공급량에 맞춘 낙농업자들의 수익률 증대를 보장하기 위해서는 주 정부가 소매 공급가에 대한 규제를 해야 합니다 소비자에게 보다 저렴한 가격에 안정된 물량을 공급할 수 있도록 하기 위해서는 이러한 조치가 필요합니다

결론 the Batavia government should begin to regulate retail milk prices

반박 ( Excello Food Market doesnrsquot reflect the whole of Batavia) 우유값 증가가 단순히 inflation 을 따라간것일수도 있다 가격상승이 수익을 보장하지 않는다( 원자재 상승등으로 인해서)우유값 규제만이 최선의 방법이 아니다( 물량 조절)낙농업자의 수익보호도 필요하다

A recent sales study indicates that consumption of seafood dishes in Bay City restaurants has increased by 30 percent during the past five years Yet there are no currently operating city restaurants whose specialty is seafood Moreover the majority of families in Bay City are two-income families and a nationwide study has shown that such families eat significantly fewer home-cooked meals than they did a decade ago but at the same time express more concern about healthful eating Therefore the new Captain Seafood restaurant that specializes in seafood should be quite popular and profitable

Write a response in which you discuss what questions would need to be addressed in order to decide whether the conclusion and the argument on which it is based are reasonable Be sure to explain how the answers to the questions would help to evaluate the conclusion

BAY CITY The name implies that this is a port city which would have traditionally consumed seafood 30--what type of seafood Unprocessed or processed cans of tuna and sardines If the latter increased Captain Seafood may not profit 30--natural inflation Are the non-seafood restaurants selling seafood If yes this may suffice since patrons would not easily dine at an unfamiliar restaurant Nationwide studyrsquos representativeness Desire does not lead to action

Nationwide study applied to Bay City Eating fewer home cooked meals than they did a decade agoHealthy food The author needs to show that 30 -gt population increaseOther foods may be more popularCurrent number of restaurants that serve seafood might sufficeWhat kind of healthy food are they interested in Surely not just seafoodThe locals may still patronize the current restaurants that serve seafood dishes

lt national-gt local에 적용 local-gtnational에 적용gt1) The author assumes that the nationwide trend reflects the general trend upon which the argument relies

Yet the author fails to provide evidence to substantiate this crucial assumption The nationwide trend of two income families eating fewer home-cooked and concern for health my not reflect the lifestyle and interests of Bay City citizens Therefore to make his argument stronger the author needs to provide evidence that two income families of Bay City maintains the nationwide trend

GRE AWA John박 박정어학원

최근의 매상에 관한 조사에서 Bay 시에 있는 레스토랑의 해산물 요리의 소비가 지난 5년동안 30 증가했다고 보고되었다 그러나 현재 해산물을 전문으로 취급하고 있는 레스토랑이 없다 더군다나 대다수의 가정이 맞벌이 가정인데다가 전국 조사에서도 나와있듯이 이러한 맞벌이 가정은 10년전의 가정에서 했듯이 집에서 식사를 챙겨먹는 것이 급격하게 줄어 들었고 동시에 건강식과 관련한 지출이 늘고 있다 따라서 해산물 전문 레스토랑이 아주 인기를 끌것이고 그만큼 수익도 많을 것이다

1 해산물이 건강식인지에 대한 언급이 없다2 외식하는데 해산물 요리만 먹지 않을 것이다 집에서 챙겨먹지 않는다고 해산물을 먹는다고 할 수 없다3 현재 해산물 취급하는 식당이 현재까지 없다고 해서 계속 없을 수만은 없다 수익이 만약 늘어난다면

Bay시 주변의 도시의 레스토랑이 체인점을 내거나 새로운 식당이 생길 수 있고 오히려 수익이 줄어들 수도 있다

Scientists studying historical weather patterns have discovered that in the mid-sixth century Earth suddenly became significantly cooler Although few historical records survive from that time some accounts found both in Asia and Europe mention a dimming of the sun and extremely cold temperatures Either a huge volcanic eruption or a large meteorite colliding with Earth could have created a large dust cloud throughout Earths atmosphere that would have been capable of blocking enough sunlight to lower global temperatures significantly A large meteorite collision however would probably create a sudden bright flash of light and no extant historical records of the time mention such a flash Some surviving Asian historical records of the time however mention a loud boom that would be consistent with a volcanic eruption Therefore the cooling was probably caused by a volcanic eruption

Write a response in which you discuss what questions would need to be addressed in order to decide whether the conclusion and the argument on which it is based are reasonable Be sure to explain how the answers to the questions would help to evaluate the conclusion

Historical records may not be enough to explain the global cooling phenomenonThere may be reasons other than the two hypothesis to explain the global coolingA volcanic eruption big enough to produce dust clouds that envelop the earth to cause global cooling would have countless recordsThe author argues that a large meteorite collision is not a feasible explanation for the global cooling because there are no historical records that indicate a flash The authorrsquos logic is flawed in eliminating the meteorite hypothesis by claiming that there was no historical record of a bright flash of light According to common sense a meteorite collision big enough to produce dust that could envelop the earth would result in catastrophe even to the extinction of major species of animals

Loud boom would be insufficient evidence to conclude it was a volcanic eruption If could hear a loud boom there should be records of a volcanic eruption because the author claims that the dust cloud from this gigantic eruption was significant enough to cause global coolingCooling could have been caused by factors besides volcanic eruption and meteor collisionThe absence of historical records that indicate a bright flash of light Collision could have occurred at a place where there no witnesses Could have fell out in the seaBoom might have been caused by things other than a volcanic eruptionHistorical records from Asia and Europe may not be applicable to earth as a whole

과거의 날씨 주기를 연구하는 과학자들은 6세기 중엽 지구가 갑자기 혹한기가 되었던 것을 알게되었다 이 당시의 몇몇 기록들이 아직도 남아있긴 하지만 아시아와 유럽에서 발견되는 몇가지 자료에서 태양 빛의 수축과 그에따른 엄청난 추위가 있었음을 언급하고 있다 거대한 화산 폭발이나 거대 운석의 지구 충돌중 어느것에

GRE AWA John박 박정어학원

의해서든 엄청난 먼지구름을 형성시켜 지구전체에 태양빛을 차단해서 지구의 기온을 뚝 떨어뜨릴수 있을 수도 있다 그러나 이러한 운석 충돌은 순간 섬광을 발산시키게 될 것인데 이 시기의 어느 기록에서도 이러한 섬광은 언급되지 않고 있다 그러나 이 시기에 남아 있는 일부 아시아의 기록문서에서는 연속적으로 화산폭발음일수 있는 엄청난 폭발음이 있었음을 기록하고 있다 따라서 이러한 혹한은 아마도 화산 폭발에 의한 것이었을 것이다

결론 the cooling was probably caused by a volcanic eruption

반박 자료의 부족( 몇몇 자료가지고 그 당시의 기후를 예상하기에는 무리가 있다)다른 원인에 의해서 기후가 떨어졌을수도 있다 (빙하기 다른 기상원인)기록되지 않았다고 해서 그 사실이 없었던 것은 아니다(기록은 했으나 자료가 없어졌을수도 있고 모든 사실이 전부다 기록되지는 않는다 )기록된 폭발음이 꼭 확산 폭발음이 아닐수도 있다( 화산 폭발음이 너무 먼거리여서 들리지 않을수도 있고 다른 소리가 기록된 것이였을수도 있다)부족한 자료를 신빙할수 있는가 기록의 소멸도 예상 할수 있다 실제로 운석이 떨어졌는데 안들렸을수 있다

The following appeared in a memo from the president of Bower Builders a company that constructs new homes

A nationwide survey reveals that the two most-desired home features are a large family room and a large well-appointed kitchen A number of homes in our area built by our competitor Domus Construction have such features and have sold much faster and at significantly higher prices than the national average To boost sales and profits we should increase the size of the family rooms and kitchens in all the homes we build and should make state-of-the-art kitchens a standard feature Moreover our larger family rooms and kitchens can come at the expense of the dining room since many of our recent buyers say they do not need a separate dining room for family meals

Write a response in which you examine the stated andor unstated assumptions of the argument Be sure to explain how the argument depends on these assumptions and what the implications are for the argument if the assumptions prove unwarranted

The presidentrsquos first unstated but apparent assumption is that a nationwide survey can be applied to local areas There is no guarantee that the area in which Bower Builders builds houses will follow the national consumer trend Since he only mentions the overall result of the nationwide survey it is difficult to determine more specific conditions such whether people in urban and rural areas have the same housing preferences Such uncertainty is further exacerbated by the nature of the survey question itselfmdash it does not deal with current trends in actual home purchases but with desired home features The naiumlve assumption that consumer desire will directly result in consumer action underlies the presidentrsquos interpretation and application of the survey results to his company plan Not everyone who wishes for a large family room and kitchen will or can actually buy a house with those features mainly due to financial reasons Furthermore the president also assumes for no evident reason that consumers will not hesitate to purchase houses with state-of-the-art kitchens as a standard rather than optional feature when it is clear that such a feature will raise the overall cost significantly He continues to blunder in his mistaken assumptions about consumer behavior in assuming that the taste of recent buyers can represent the concern of future prospective buyers The fact that recent buyers have claimed no need for separate dining rooms does not mean future buyers will feel the same way as well

The president of Bower Builders recommends that in order to raise company profits the company should build houses with large family rooms and large state-of-the-art kitchens He uses the results of a nationwide survey on desired home features and the example of Bower Buildersrsquo competitor Domus Construction to support his plan His memo manifests several assumptions about surveys consumer behavior and Domus Constructionrsquos houses that do not necessarily bolsterbuttress his argument

GRE AWA John박 박정어학원

The writer assumes 1 the nationwide survey results can be applied to the local area (Desire cannot reflect consumer trend) 2 Domus Construction was profitable because they had such features (The writer should investigate what other features the DC provides and the actual number of homes sold) 3 State-of-the-art kitchens will sell well (no evidence to buttress his assumption furthermore they would need to spend more money which would increase the price of the housesmdashtheir market will be limited to the affluent) 4 The recent buyers represent the concern of most buyersmdashthat they are ok with a house that is without a dining room

Nationwide survey might not be applicable to this regionDomus Construction could have other superior qualities than Bower Builders Ex DesignSelling fast doesnrsquot indicate quantitySmall yards complaints could be voiced in the future

First the author concludes that a nationwide survey reveals that the two most desired home features are a bathroom with a whirlpool tub and a large kitchen However this nationwide survey may not reflect the trends of the customers that Bower Builders target The author assumes that the nationwide trend reflects customer trends The national study would lead support only if the nationwide trend reflect the home-purchasing trends The nationwide trend may just be a trend of desire which does not result in the act of purchasing However the author does not provide credible evidence that this is the case

전국 조사에서 가장 인기있는 집안 구조 2가지는 훨풀 튜브가 마련된 욕실과 커다른 부엌으로 조사되었다 경쟁업체 Domus Construction이 신축한 완공예정인 주택은 이러한 욕실을 갖추고 있어서 분양이 조기에 이루어지고 있고 가격도 평균치보다 상당히 웃돌고 있다 우리도 집을 많이 팔도 그만큼 이윤을 많이 남기려면 신규 주택에는 위의 2가지 사항을 갖추어야 한다 최근 우리가 지은 주택 수요자들이 적은 평수에 대해서는 별다른 불만사항이 없었기 때문에 앞으로 평수를 더 줄여서 이익을 극대화 할 수 있을 것이다

주장 We should include whirlpool tubs and a large kitchen

1 경쟁업체 Domus Construction이 신축한 완공예정인 주택은 이러한 욕실을 갖추고 있어서라기 보다는 위치가 좋거나 다른 마감재(construction material)나 인테리어가 뛰어나서 분양이 조기에 이루어진것이라고 볼 수 있다 2 전국조사가 건물이 지어지는 지역에 항상 적용되리라고 볼 수 없다 3 평수를 줄이는 것에 불만이 없었다는 것은 editor의 견해일 수 있다 사람들이 표현하지 않은 것일 수도 있고 다른 장점이 많아서 그런 단점을 드러내지 않은 것일 수 있기 때문에 속단할 수 없다 4 다른것을 함으로써 더 잘 팔릴수도 있다

2011 7 19 화요일 GRE AWA 실전반이OO

Argument 94

The president of Bower Builders contends recommends that in order to raise company profits the company should build houses with large family rooms and large state-of-the-art kitchens He uses the results of a nationwide survey on desired home features and the example of Bower Buildersrsquo competitor Domus Construction to support his plan His memo manifests several assumptions about surveys consumer behavior and Domus Constructionrsquos houses that do not

GRE AWA John박 박정어학원

necessarily bolsterbuttress his argumentThe presidentrsquos first unstated but apparent assumption is that a nationwide survey can be

applied to local areas There is no guarantee that the area in which Bower Builders builds houses will follow the national consumer trend Since he only mentions the overall result of the nationwide survey it is difficult to determine more specific conditions such whether people in urban and rural areas have the same housing preferences Such uncertainty is further exacerbated by the nature of the survey question itselfmdash it does not deal with current trends in actual home purchases but with desired home features The naiumlve assumption that consumer desire will directly result in consumer action underlies the presidentrsquos interpretation and application of the survey results to his company plan Not everyone who wishes for a large family room and kitchen will or can actually buy a house with those features mainly due to financial reasons Furthermore the president also assumes for no evident reason that consumers will not hesitate to purchase houses with state-of-the-art kitchens as a standard rather than optional feature when it is clear that such a feature will raise the overall cost significantly He continues to blunder in his mistaken assumptions about consumer behavior in assuming that the taste of recent buyers can represent the concern of future prospective buyers The fact that recent buyers have claimed no need for separate dining rooms does not mean future buyers will feel the same way as well

In addition the president finds a real-life actualization of the nationwide survey results in the recent sales of Domus Construction However he easily assumes that large family rooms and kitchens are the only reasons the houses of Domus Construction sell well He does not take into account other features and selling points of the competitorrsquos houses There may well be other explanations for its houses selling more quickly and expensively including additional home features as well as external factors such as proximity to better schools or superior financial solvency of its clientele If Bower Builders merely added larger family rooms and kitchens without taking into consideration the other factors they may lose rather than gain profits

Clearly the presidentrsquos assertion that Bower Builders make houses with large family rooms and high-tech kitchens at the expense of dining rooms rests on a number of assumptions that are ill-informed and naiumlve If Bower Builders undertakes the proposed plan without further research into local consumer desires purchasing trends and the marketing and sales of competing companies the company will risk losing money by building big new houses that people cannot afford to or will not wish to buy

The following appeared in a letter to the editor of a journal on environmental issues

Over the past year the Crust Copper Company (CCC) has purchased over 10000 square miles of land in the tropical nation of West Fredonia Mining copper on this land will inevitably result in pollution and since West Fredonia is the home of several endangered animal species in environmental disaster But such disasters can be prevented if consumers simply refuse to purchase products that are made with CCCs copper unless the company abandons its mining plans

Write a response in which you examine the stated andor unstated assumptions of the argument Be sure to explain how the argument depends on these assumptions and what the implications are for the argument if the assumptions prove unwarranted

The writer assumes 1 The writer is trying to avoid an inevitability 2 Mining copper will result pollution (they could make preventative measures) 3 The writer may be too late from stopping CCC from developing the area into a copper mine 4 Since mining is an underground enterprise the surface may not be affected that much therefore endangered species may not be affected 4 Consumers wonrsquot buy CCC products if the journal publishes a negative review about CCC (How many readers) CCC could a company that has ties with many IT companies and industries in that their copper is almost ubiquitous in various products

GRE AWA John박 박정어학원

지난 한해동안 CCC(Consolidated Copper 회사)는 서부 플로리다의 열대 지역에 1백만 마일이 넘는 땅을 사들였다 이 곳에서의 채광 활동은 서부 플로리다가 몇몇 멸종 위기에 처한 동물의 서식지이기 때문에 분명 오염과 환경파괴를 가져올 것이다 그러나 이러한 파괴는 CCC 회사가 채광을 포기할 때까지 이 회사가 채굴한 구리로 제조된 제품을 구매하지 않으면 막을 수 있을 것이다

결론 such disaster can be prevented if consumers simply refuse to purchase products that are made with CCCs copper until the company abandons its mining plans

1 구리로 제조된 물건이 생활에 필수적인 것이 많은 만큼 불매가 쉽지 않을 수 있다(전선이나 각종 전자제품에 필수적으로 들어가기 때문에)

2 불매를 유도한다고 해서 소비자들이 구매를 안하는 것은 아니다3 적절한 채굴로 환경파괴를 가져 오지 않을 수 있다 (땅속에 있는 물질을 채굴하는 만큼 생물에 영향을 안

미칠 수도 있다)4 이미 채굴이 다 끝나서 더 이상의 채굴이 없을 수도 있다 5 땅을 구입한다고 해서 채광하는건 아니다 (다른 용도로 샀을수도 있다)

The following is a letter to the editor of an environmental magazine

In 1975 a wildlife census found that there were seven species of amphibians in Xanadu National Park with abundant numbers of each species However in 2002 only four species of amphibians were observed in the park and the numbers of each species were drastically reduced There has been a substantial decline in the numbers of amphibians worldwide and global pollution of water and air is clearly implicated The decline of amphibians in Xanadu National Park however almost certainly has a different cause in 1975 troutmdashwhich are known to eat amphibian eggsmdashwere introduced into the park

Write a response in which you discuss what specific evidence is needed to evaluate the argument and explain how the evidence would weaken or strengthen the argument

Evidence needed the identity of the sender and the census taker of rsquo75 and lsquo02mdasha scientist or an environmentalist credibility issue census methodology vs mere observationmdashthe absence of evidence is not an evidence of absence the season of when the census was taken time-shiftmdashconditions may have changed worldwide decline may include Xanadu other species of predators that prey on amphibians because trout is only one species that prey on amphibians the number of troutmdashhave they increased significantly since rsquo75 First the author needs to be more overt about the credibility of the census and observation There were to accounts that notes the population of amphibiansmdashthe first a census and the second an observation The author needs to bolster his conclusion with the evidence that indicate that the census and second observation were done using scientific methodologies This could either could strengthen or weaken his claim In addition he

GRE AWA John박 박정어학원

needs to provide the specific season of when the census and observation occurred In this letter the writer is informing an editor of an environmental magazine that the number of amphibians was greatly reduced since 1975 and he points out the introduction of trout as the only reason for the decline However the author fails to provide crucial evidences that could strengthen or weaken his conclusion

The following appeared in a memorandum from the president of Hyper-Go Toy Company

Last year sales of our Fierce Fighter toy airplane declined sharply even though the toy had been a top seller for three years Our customer surveys show that parents are now more worried about youthful violence and are concerned about better education for their children Therefore to maintain profits we should discontinue all our action toys and focus exclusively on a new line of educational toys Several other toy companies have already begun marketing educational toys and report sales increases last year of 200 percent And since the average family income is growing sales of new Hyper-Go toys should also increase

Write a response in which you discuss what specific evidence is needed to evaluate the argument and explain how the evidence would weaken or strengthen the argument

First the president indicates that the sales of Fierce Fighter toy airplane declined sharply However he fails to consider the fact that toys are a fad Since FFT enjoyed a three year success it may be natural that the trend would subsideSecond Customer survey is this representative of most toy consumers This trend is not newThird other companies may have profited not through educational but other toys Or educational toy profit may be small in proportion to their sale of other toys 200Parents are not the customers companies should concentrate on what the children want to increase profitLastly parents worry about youthful violence and concern for better education are not new trends

The presidentrsquos decision to discontinue all action toys and focus only on educational toys is too extreme If this decision is taken into effect its procedure may be an onerous task because it would require major shifts in human resources and company image Therefore further data should be considered before following up on this decision

우리 회사에서 지난 3년간 최고 매출을 일으켰던 Fierce Fighter 장난감 비행기가 지난해에는 매출이 급격히 떨어졌다 자체 고객 조사에서는 부모들이 현재 청소년 폭력에 걱정을 하고 있어서 아이들의 양질 교육에 더 관심을 가지고 있는 것으로 나타났다 따라서 회사의 수익을 유지하기 위해서는 모든 자사 전투용 장난감 생산을 중단하고 오로지 교육적인 장난감 생산에 집중해야 한다 몇몇 여타 장난감 회사들도 이미 교육용 장난감 마케팅을 시작해서 지난해에는 200의 매출신장을 가져왔다고 한다 그리고 평균 가계 수입이 점점 늘고 있기 때문에 신형 Hyper-Go 장난감의 매출도 늘어날 것이다

3그룹 Time-shift Error

Woven baskets characterized by a particular distinctive pattern have previously been found only in the immediate vicinity of the prehistoric village of Palea and therefore were believed to have been unique to the Palean people Recently however archaeologists discovered such a Palean basket in Lithos an ancient village across the Brim River from Palea The Brim River is very deep and broad and so the ancient Paleans could only have crossed it by boat but there is no evidence that the Paleans had boats And boats capable of carrying groups of people and cargo were not developed until thousands of years after the Palean people disappeared Moreover Paleans would have had no need to cross the rivermdashthe woods around Palea are full of nuts berries and small game It follows that the so-called Palean baskets were not unique to Palea

GRE AWA John박 박정어학원

Write a response in which you discuss what specific evidence is needed to evaluate the argument and explain how the evidence would weaken or strengthen the argument

RefutationPrehistoric time shift-gtbrim river could have been narrow and shallow or it might have not existed Indigenous patterns may exist in other disconnected remote placesNuts berries small game(hunting) may not have existed or the author should be proved these existed at that time Abundance of resources doesnrsquot support the reason for seclusionBoats not yet found baskets may have been carried across by the river current without the help of a boat Lithos might have crossed the river for commercial purposeOne Palean basket does not substantiate the authorrsquos claimThe absence of evidence is not an evidence of absence

The author assumes without justification that present conditions are the same as at the prehistoric era The author unfairly infers from the presence of Brim River which exist today that it would have existed in the past However the author fails to offer any evidence to substantiate this inference It is very likely that the Brim River might not have existed in prehistoric times or if it did exist may have been shallow and narrow enough for the Paleans to easily cross Any of these scenarios if true would serve to undermine the claim thathelliphelliphelliphelliphellip

First the author claims that the Brim River was very deep and broad so the Paleans could not have crossed it However the author fails to offer any evidence to substantiate that this was true in the prehistoric time of the Paleans For all we know the Brim River might not have existed in the prehistoric era or if it did exist could have been a shallow and narrow river For example scientists believe that thousands of years ago an ice-bridge existed on the Bering Sea connecting Eurasia to what is now North America to explain how the Eskimos and the Asian inhabitants of America came to migrate all over the Americas In this example the absence of evidence is not an evidence of absence Therefore to sufficiently support his claim the author needs to substantiate the fact that the Brim River really did exist and was broad and wide in the Prehistoric Era proving that the Paleans could not have influenced or traded with other groups of people

First the author claims that the Brim River was very deep and broad so the Paleans could not have crossed it However the author fails to offer any evidence to substantiate that this was true in the prehistoric times of the Paleans For all we know the Brim River might not have existed in the prehistoric era or if it did exist could have been a shallow and narrow river For example rivers are created by natural erosion over thousands of years Niagara Falls carved its way from the mouth of Lake Ottawa and created a long river Likewise the current Brim Riverrsquos physical features may not have been wide and deep Therefore to sufficiently support his claim the author needs to substantiate the fact that the Brim River really did exist and was broad and wide in the Prehistoric Era proving that the Paleans could not have influenced or traded with other groups of people

예전에는 실로짠 특이한 무늬 바구니가 Palea의 선사시대 지역의 인근마을에서만 발견되어왔기 때문에 Palea 마을 사람들의 특징이라고 여겨졌었다 그러나 최근들어 고고학자들이 Lithos지역에서 Palean 바구니를 발견하였는데 그 당시 지역은 Brim 강을 가로질러 Palea까지 닿아있었다 이 강은 수심이 아주 깊고 강폭이 넓었으며 때문에 고대의 Palea인들은 배를 이용해서 강을 건널수 있었을 것이다 그러나 이들이 배를 가지고 있었다는 증거는 발견되지 않고 있다 더군다나 이들이 멸명한 이후 수천년이 지난뒤에도 수많은 물자와 사람을 실어 나를수 있는 용적을 가진 배는 개발되지 않았다 이와더불어 Palea인들은 강을 건널필요가 없었는데 그것은 너트나무 장과열매 그리고 작은 사냥감들이 주변숲에 풍부했기때문이다 따라서 Palean 바구니라고 하는 것도 Palea인들만의 전유물이 아니라는 결론을 얻을 수 있다

결론 if follows that the so-called Palean baskets were not unique to Palea이번문제는 굿이 causal Bad analogy 로 구분해서 찾기가 힘드내요 배를 발견했다는 증거가 없는것이다 (앞으로도 발견될수 있음)

GRE AWA John박 박정어학원

계절의 영향으로 겨울에 얼음이 두껍게 언다든지 여름에 가뭄으로 인해서 건널수 있다물자가 풍부한 것이 이동하지 않을 조건이 아니다 다른 것에 의해서 이동가능(의약품등)

Thirteen years ago researchers studied a group of 25 infants who showed signs of mild distress when exposed to unfamiliar stimuli such as an unusual odor or a tape recording of an unknown voice They discovered that these infants were more likely than other infants to have been conceived in early autumn a time when their mothers production of melatonin hormone known to affect some brain functions would naturally increase in response to decreased daylight In a follow-up study conducted earlier this year more than half of these children now teenagers who had shown signs of distress identified themselves as shy Clearly increased levels of melatonin before birth cause shyness during infancy and this shyness continues into later life

Write a response in which you examine the stated andor unstated assumptions of the argument Be sure to explain how the argument depends on these assumptions and what the implications are for the argument if the assumptions prove unwarranted

Any baby exposed to unpleasant stimuli would react in such wayFirst of all the author states 25 infants as his evidence However this research sample is too small to prove his claimSecond the author states that 25 infants were conceived in early autumn which he claims lead to a shy disposition However this is faulty evidenceThird the research study was a long term study done in the span of 13 years However the author only writes about the initial and final stages of the study and leaves out evidences of what could have happened during the 13 years which could be more evidential factors of influenceFourth neither the infantsrsquo genetic predisposition nor their environment were taken into accountFinally the author concludes that his shyness continues into later life (Other factors could alter this disposition epigenetic theory)

13 년전 학자들은 25명의 유아를 대상으로 이상한 냄새나 특이한 소리를 녹음한 테잎등으로 낯선 자극을 주었을때 보이는 미미한 압박감 증상을 조사하였다 이들은 성숙기가 막 지났을 즈음에 보통의 유아들이 비슷한 증상을 보이는 정도 보다는 다소 민감한 반응을 보였는데 이 시기는 아이의 엄마가 뇌의 일부 기능에 영향을 미치는 것으로 알려진 멜라토니아 호르몬을 생산하는 시기로써 이 호르몬은 낯 시간이 짧을때 자연적으로 증가할 수도 있다 금년초에 실시된 추가연구에서 현재 10대로 성장한 당시 조사대상의 절반 이상의 아이들이 부끄럼을 잘타는 것으로 여기고 있었다 따라서 분명한 것은 출산전 멜라토닌 수치의 증가가 유아기에 수줍음 등의 영향을 미치게 되며 이러한 영향이 성장후에도 작용한다는 것이다

주장 Clearly increased levels of melatonin before birth cause shyness during infancy and this shyness continues into later life

1 25명의 아기로 결론 내리기에 샘플이 작다2 과학적 사실들에 대한 명확한 근거가 엇음3 다른 영향을 간과했다(다른 호르몬에 의한 영향 후천적인 성격형성의 영향)

GRE AWA John박 박정어학원

The following is a letter to the editor of the Atticus City newspaper

Former Mayor Durant owes an apology to the city of Atticus Both the damage to the River Bridge which connects Atticus to Hartley and the traffic problems we have long experienced on the bridge were actually caused 20 years ago by Durant After all he is the one who approved the construction of the bridge If he had approved a wider and better-designed bridge on which approximately the same amount of public money would have been spent none of the damage or problems would have occurred Instead the River Bridge has deteriorated far more rapidly over the past 20 years than has the much longer Derby Bridge up the river Even though the winters have been severe in the past several years this is no excuse for the negligence and wastefulness of Durant

Write a response in which you discuss what questions would need to be answered in order to decide whether the recommendation is likely to have the predicted result Be sure to explain how the answers to these questions would help to evaluate the recommendation

전임 시장인 Durant 씨는 Atticus 시에 대해 사과할 의무가 있습니다 Atticus와 Hartley를 잇는 River Bridge 교량에 대한 피해와 이 교량에서 오랫동안 주민들이 겪어오고 있는 교통 혼잡 문제들은 실제로 20년 전부터 시작된 것이었습니다 결정적으로 그가 교량 공사를 허가했던 바로 그 장본인입니다 당시 비슷한 공사비용으로 폭이 더 넓고 튼튼하게 설계된 교량을 허가했다면 이러한 문제나 피해는 발생하지 않았을 겁니다 더군다나 이 다리는 지난 20년 동안 상류에 건설된 훨씬 오래된 Derby 다리보다도 빠르게 부식되어 갔습니다 지난 수년동안 심지어 혹한이 있었다 하더라도 이러한 태만과 국고 손실에 대한 책임을 회피할 길이 없는 것입니다

결론 Former Mayor Durant owes an apology to the city of Atticus

1 디자인이 문제가 아닐수 있다 (디자인은 좋았으나 건설과정에 문제가 있었을 수 있다)2 그 당시의 시예산이 적어서 더 큰 다리를 짓기가 불가능했을 수도 있다3 교통량이 많거나 다른 상황으로 인해서 부식이 빨리 됐을 수 있다4 그 당시에는 최선의 선택이였지만 갑자기 변한 상황에 의해서 이런 문제점들이 발생했을 수 있다

GRE AWA John박 박정어학원

4그룹 거짓인과관계 오류 (False Cause) 빈출

Fifteen years ago Omega University implemented a new procedure that encouraged students to evaluate the teaching effectiveness of all their professors Since that time Omega professors have begun to assign higher grades in their classes and overall student grade averages at Omega have risen by 30 percent Potential employers looking at this dramatic rise in grades believe that grades at Omega are inflated and do not accurately reflect student achievement as a result Omega graduates have not been as successful at getting jobs as have graduates from nearby Alpha University To enable its graduates to secure better jobs Omega University should terminate student evaluation of professors

Write a response in which you discuss what specific evidence is needed to evaluate the argument and explain how the evidence would weaken or strengthen the argument

Omega professor evaluation implemented 15 years ago =gt Omega prof assign higher grades 30Employers believe therersquos grade inflation

Thus unsuccessful employment than AlphaTherefore to secure jobs Omega should end evaluating profs

Specific evidence neededRelationship between higher grades and evaluationRelationship between GPA and unsuccessful employmentAlpharsquos education could just be better than OmegaldquoFifteen years agordquo is a long time other factors could have influenced Why is the inflation a problem just now How much is Alpha better Is the comparison just How much gap is thereOmegarsquos student could just be doing better in their studiesComparison to other universities다른 대안 없나hellip Could Omega alleviate the employment problem by implementing a different procedure or program

15 년전 우리 대학은 학생들로 하여금 교수평가를 하도록 한 새로운 조치를 시행했었습니다 이후 교수들은 자신의 학과 학생들에게 높은 학점을 주었으며 그에따라 학생들의 전체 평점이 30나 올랐습니다 외부의 기업체들은 분명 점수가 지나치게 부풀려졌다고 믿고 있습니다 결국 본 대학 졸업생들이 인근 Alpha 대학의 졸업자들보다 구직률이 떨어지는 이유를 잘 보여주고 있는 것입니다 이를 해결하기 위해 이제부터는 학생들에 의한 교수평가제를 중단해야 합니다

결론 Omega University should now terminate student evaluation of professors

반박 교수 평가와 학점 인플레의 연관성이 적다( 교수 평가를 먼저하고 학점을 나중에 매길수도 있다)채용기준에 성적만 있는게 아니다 학업성취의 결과 일수도 있다 Alpha 가 원래 유능했다 Alpha 의 교육내용이 좋았다

GRE AWA John박 박정어학원

In this memo the dean of Omega University(OU) recommends OU to terminate professor evaluation to secure better jobs for the students To support this recommendation the dean offers several reasons However this argument contains several logical flaws which render it unconvincing

A threshold problem with the argument involves the voluntary nature of the evaluationprocedure The dean provides no evidence about the number or percentage of Omegastudents who participate in the procedure Lacking such evidence it is entirely possible thatthose numbers are insignificant in which case terminating the procedure is unlikely to haveany effect on the grade average of Omega students or their success in getting jobs aftergraduationThe argument also assumes unfairly that the grade-average increase is the result of theevaluation procedure--rather than some other phenomenon The dean ignores a host of otherpossible explanations for the increase--such as a trend at Omega toward higher admissionstandards or higher quality instruction or facilities Without ruling out all other possibleexplanations for the grade-average increase the dean cannot convince me that by terminatingthe evaluation procedure Omega would curb its perceived grade inflation let alone help itsgraduates get jobsEven if the evaluation procedure has resulted in grade inflation at Omega the deans claimthat grade inflation explains why Omega graduates are less successful than Alpha graduatesin getting jobs is unjustified The dean overlooks a myriad of other possible reasons forOmegas comparatively poor job-placement record Perhaps Omegas career services areinadequate or perhaps Omegas curriculum does not prepare students for the job market aseffectively as Alphas In short without accounting for other factors that might contribute toOmega graduates comparative lack of success in getting jobs the dean cannot justify theclaim that if Omega curbs its grade inflation employers will be more likely to hire OmegagraduatesFinally even if the dean can substantiate all of the foregoing assumptions the deansassertion that Omega must terminate its evaluation procedure to enable its graduates to findbetter jobs is still unwarranted in two respects First the dean ignores other possible ways bywhich Omega can increase its job-placement record--for example by improving its publicrelations or career-counseling services Second the dean unfairly equates more jobs withbetter jobs In other words even if more Omega graduates are able to find jobs as a result ofthe deans recommended course of action the kinds of jobs Omega graduates find would notnecessarily be better onesIn sum the deans argument is unpersuasive as it stands To strengthen it the dean mustprovide better evidence that the increase in grade average is attributable to Omegasprofessor-evaluation procedure and that the end result is a perception on the part ofemployers that Omega graduates are less qualified for jobs than Alpha graduates To betterassess the argument I would need to analyze 15-year trends in (l) the percentage of Omegastudents participating in the evaluation procedure (2) Omegas admission standards andquality of education and (3) Omegas emphasis on job training and career preparation I wouldalso need to know what other means are available to Omega for enabling its graduates to findbetter jobs

GRE AWA John박 박정어학원

The following appeared in a memo from a vice president of Quiot Manufacturing

During the past year Quiot Manufacturing had 30 percent more on-the-job accidents than at the nearby Panoply Industries plant where the work shifts are one hour shorter than ours Experts say that significant contributing factors in many on-the-job accidents are fatigue and sleep deprivation among workers Therefore to reduce the number of on-the-job accidents at Quiot and thereby increase productivity we should shorten each of our three work shifts by one hour so that employees will get adequate amounts of sleep

Write a response in which you examine the stated andor unstated assumptions of the argument Be sure to explain how the argument depends on these assumptions and what the implications are for the argument if the assumptions prove unwarranted

The following appeared in a memo from a vice president of Alta Manufacturing

During the past year Alta Manufacturing had thirty percent more on-the-job accidents than nearby Panoply Industries where the work shifts are one hour shorter than ours Experts believe that a significant contributing factor in many accidents is fatigue caused by sleep deprivation among workers Therefore to reduce the number of on-the-job accidents at Alta we recommend shortening each of our three work shifts by one hour If we do this our employees will get adequate amounts of sleep

Write a response in which you discuss what questions would need to be answered in order to decide whether the recommendation and the argument on which it is based are reasonable Be sure to explain how the answers to these questions would help to evaluate the recommendation

The following appeared in a memo from the vice president of Butler Manufacturing

During the past year workers at Butler Manufacturing reported 30 percent more on-the-job accidents than workers at nearby Panoply Industries where the work shifts are one hour shorter than ours A recent government study reports that fatigue and sleep deprivation among workers are significant contributing factors in many on-the-job accidents If we shorten each of our work shifts by one hour we can improve Butler Manufacturings safety record by ensuring that our employees are adequately rested

1 Write a response in which you discuss what specific evidence is needed to evaluate the argument and explain how the evidence would weaken or strengthen the argument

2 Write a response in which you discuss what questions would need to be answered in order to decide whether the recommendation is likely to have the predicted result Be sure to explain how the answers to these questions would help to evaluate the recommendation

4번 반복됨

Alta has 30 more job accidents than Panoply(work shifts one hour shorter)Experts Job accidents caused by fatigue and sleep deprivationTherefore to reduce job accidents and increase productivity shorten three work shifts by one hour for adequate sleep

지난해 우리 회사는 인근 Panoply Industries보다 업무상 재해가 30나 더 많았다 그 회사는 우리보다 근무 교대시간이 1시간 정도 짧았다 전문가들은 대부분의 업무상 재해에 있어서 가장 중요한 요인이 과로와 수면부족으로 보고있다 따라서 우리 회사에서 높은 산업재해를 줄이고 아울러 생산성을 높이기 위해서는 근로자들이 충분한 수면을 취할 수 있도록 1시간씩 3교대 시간을 줄여야 한다

In this memo the (author) vice president of Alta Manufacturing (AM) recommends that to reduce on-the-job accidents and increase productivity AM should shorten its three work shifts by one hour so that employees can

GRE AWA John박 박정어학원

get more sleep To support this recommendation the author provides several evidences However careful scrutiny of each of the facts reveals that it provides little credible support for the authorrsquos recommendation QuestionsThe number of accidents What kind of accidents The seriousness of the accidents is importantHow many employees are in each company What are their productsFalse cause Sleep may not be the reason for the on-the-job accidents What do Alta and Panoply manufacture

First of all the author believes that fatigue caused the on-the job accidents However there could be other reasons The author observes a correlation between sleep deprivation and on-the-job accidents then concludes that the former is the cause of the latter However the author fails to rule out other possible explanations For example it is entirely possible that Alta factories require more strenuous and dangerous labor than Panoply Without ruling out all other such factors it is unfair to conclude that fatigue is responsible for the accidents In addition the work-shifts may not be the cause of the sleep deprivation and fatigue It is possiblehellip Thus the author should provide what exactly Panoply and Alta manufacture and more precise data about their working conditions to be more convincing

Shortening the shift by one hour does not necessarily lead to more sleep And is one hour enoughLess accidents does not mean increased productivity

결론 We should shorten each of out three work shifts by one hour

반박 경쟁사에 비해서 시간당 하는 업무량이 많아서 더욱 피곤할 수도 있다 시간이 문제가 아니라 노후된 시설 설비 자체의 문제 작업 자체가 원래 위험한 것이여서 사고가 많을 수도 있다 다른 회사는 더욱 많은 작업시간에도 불구하고 안정한 작업여건으로 인해서 사고율이 오히려 더 작을 수도 있다비교사의 재해감소가 다른 요인일수 있다(안전 교육 철저)줄인 시간이 피로회복이나 수면으로 연결 안될수 있음(술을 마실 수도 있고 그 시간에 휴식을 취하지 않고 다른일을 함으로써 더욱 피로해질수 있다)

This editorial recommends that Alta Manufacturing reduce its work shifts by one hour each inorder to reduce its on-the-job accident rate and thereby increase Altas productivity To supportthis recommendation the author points out that last year the number of accidents at Alta was30 greater than at Panoply Industries where work shifts were one hour shorter The authoralso cites certain experts who believe that many on-the-job accidents are caused by fatigueand sleep deprivation I find this the argument unconvincing for several reasonsFirst and foremost the author provides absolutely no evidence that overall workerproductivity is attributable in part to the number of on-the-job accidents Although commonsense informs me that such a relationship exists the author must provide some evidence ofthis cause-and-effect relationship before I can accept the authors final conclusion that theproposed course of action would in fact increase Altas productivitySecondly the author assumes that some accidents at Alta are caused by fatigue or sleepdeprivation However the author overlooks other possible causes such as inadequateequipment maintenance or worker training or the inherent hazards of Altas manufacturingprocesses By the same token Panoplys comparatively low accident rate might be attributablenot to the length of its work shifts but rather to other factors such as superior equipmentmaintenance or worker training In other words without ruling out alternative causes ofon-the-job accidents at both companies the author cannot justifmbly conclude that merely byemulating Panoplys work-shift policy Alta would reduce the number of such accidentsThirdly even assuming that Altas workers are fatigued or sleep-deprived and that this is thecause of some of Altas on-the-job accidents in order to accept the authors solution to thisproblem we must assume that Altas workers would use the additional hour of free time tosleep or rest However the author provides no evidence that they would use the time in thismanner It is entirely possible that Altas workers would use that extra hour to engage in someother fatiguing activity Without ruling out this possibility the author cannot convincinglyconclude that reducing Altas work shifts by one hour would reduce Altas accident rateFinally a series of problems with the argument arise from the scant statistical information onwhich it relies In comparing the number of accidents at Alta and Panoply the author fails toconsider that the per-worker accident rate might reveal that Alta is actually safer than Panoplydepending on the total number of workers at each company Second perhaps accident rates

GRE AWA John박 박정어학원

at the two companies last year were aberrations and during other years Altas accident ratewas no greater or even lower than Panoplys rate Or perhaps Panoply is not representativeof industrial companies generally and that other companies with shorter work shifts have evenhigher accident rates In short since the argument relies on very limited statistical information Icannot take the authors recommendation seriouslyIn conclusion the recommendation for emulating Panoplys work-shift policy is not wellsupported To convince me that shorter work shifts would reduce Altas on-the-job accidentrate the author must provide clear evidence that work-shift length is responsible for some ofAltas accidents The author must also supply evidence to support her final conclusion that alower accident rate would in fact increase overall worker productivity

The following appeared in a memo from the vice president of marketing at Dura-Sock Inc

A recent study of our customers suggests that our company is wasting the money it spends on its patented Endure manufacturing process which ensures that our socks are strong enough to last for two years We have always advertised our use of the Endure process but the new study shows that despite our socks durability our average customer actually purchases new Dura-Socks every three months Furthermore our customers surveyed in our largest market northeastern United States cities say that they most value Dura-Socks stylish appearance and availability in many colors These findings suggest that we can increase our profits by discontinuing use of the Endure manufacturing process

1 Write a response in which you examine the stated andor unstated assumptions of the argument Be sure to explain how the argument depends on these assumptions and what the implications are for the argument if the assumptions prove unwarranted

2 Write a response in which you discuss what specific evidence is needed to evaluate the argument and explain how the evidence would weaken or strengthen the argument

3 Write a response in which you discuss what questions would need to be answered in order to decide whether the recommendation and the argument on which it is based are reasonable Be sure to explain how the answers to these questions would help to evaluate the recommendation

Intro The vice president of marketing at Dura-Sock Inc is offering a potentially harmful investment recommendation by claiming that Dura-Sock should discontinue its use of the ldquoEndurerdquo process To support his recommendation he points out a study that Dura-Sock customers actually purchase the socks every three months and a survey that reveals that Dura-Sock customers like the sockrsquos stylish appearance and availability in many colors The study and survey however are insufficient in supporting his proposal and the VP makes several unwarranted assumptionsIntro (simplified) The VP states that though Dura-Socks last for two years customers buy the socks every three months Therefore he assumes that the consumersrsquo motive for buying the produce is not its durabilityHowever the author fails to rule out other possible motivation for consumption

Even if the survey is reliable the author should consider the rest of the market Vague terms ldquowasting moneyrdquomdashprecisely how much are they wasting Studysurvey errorThe company must calculate the outcome of such momentous decisionStudy participantsrsquo comment that they prefer Dura-Sock for its stylishness and availability might take Dura-Sockrsquos enduring quality for granted

우리회사 제품 소비자들에 대한 최근 조사에서 지난 2년여간 양말의 내구성을 강하게 하는 필수공정이었던 자사 특허의 Endure 공정에 들어가는 비용이 낭비라고 말하고 있다 우리 회사는 항상 이 공정 처리에 대한 광고를 내보냈으나 이에 대한 시장 조사에서 실제로 고객들은 이 신제품을 평균 석달마다 구매하는 것으로 나타났다 더군다나 북동부지역에서 실시한 대규모 시장조사에 응답한 고객들은 양말의 모양과 색상등에 더

GRE AWA John박 박정어학원

관심을 나타냈다 이러한 결과는 우리회사가 신기술 공법을 중단하면 그에 따라 수익이 늘어날 것이라는 것을 말해주고 있는 것이다주장 These findings suggest that Dura0Sock can increase its profits by discontinuing its use of the ldquoEndurerdquo manufacturing process

1 survey가 정확한 소비자의 의견을 나타낸 것인가 다른 선택없이 양자택일과 같은 방법의 survey였는지2 북동부지역의 시장조사가 전체 의견을 대표할 수 있나3 사람들이 모양이나 색상에 앞서 내구성을 먼저 평가했을 수도 있다 내구성을 갖추었다는 전제하에 모양과 색상에 관심을 드러낸 것일 수 있다4 소비자가 도매상(retail)인지 소매상(whole)인지가 없다

The following appeared in a business magazine

As a result of numerous complaints of dizziness and nausea on the part of consumers of Promofoods tuna the company requested that eight million cans of its tuna be returned for testing Promofoods concluded that the canned tuna did not after all pose a health risk This conclusion is based on tests performed on samples of the recalled cans by chemists from Promofoods the chemists found that of the eight food chemicals most commonly blamed for causing symptoms of dizziness and nausea five were not found in any of the tested cans The chemists did find small amounts of the three remaining suspected chemicals but pointed out that these occur naturally in all canned foods

Write a response in which you discuss what questions would need to be addressed in order to decide whether the conclusion and the argument on which it is based are reasonable Be sure to explain how the answers to the questions would help to evaluate the conclusion

Representativeness of the tested cansThey should conduct a comparative studyThe testing could be biased because Promofoods employees conducted the testingHow much (quantity) of the five and three suspected chemicals were in the canned foodsFalse cause The substance that caused dizziness and nausea may not be one of the eight common chemicals

많은 소비자들의 현기증과 구역질 불만에 따라 Promofoods사는 지난해 참치 캔 8백만 개를 테스트하기 위해 반품시켰다 그 결과 캔에서는 건강에 위험이 될 수 있는 화합물질이 없었던 것으로 회사측은 결론지었다 이러한 결론은 회사측 화학연구자들이 회수된 캔의 샘플을 테스트해서 이들 증상의 원인이 되는 8가지 화합물 중에서 5가지가 실험된 캔에서 발견되지 않았다는 사실에 근거한 것이다 이들 화학자들은 나머지 3개가지 화합물이 모든 캔 식료품에서 흔히 발견되는 것이라고 언급했다 결론 Promofoods concluded that the cans did not after all contain chemicals that posed a health risk

1 공인된 기간에서 테스트를 한 것이 아니고 자사에서 직접 테스트를 했기에 신뢰성이 안간다 2 이런 증상을 일으키는 8개의 물질 말고 다른 물질들이 캔속에 많이 포함됬을수 있다 3 나머지 3개의 물질들의 함유량이 많아서 다른 종류의 캔들은 문제를 일으키지 않지만 참치캔은 문제를

일으킬 수 있다

This magazine article concludes that the 8 million cans of tuna Promofoods recalled due tocomplaints about nausea and dizziness do not after ail contain any chemicals that pose a

GRE AWA John박 박정어학원

health risk To support this conclusion the author cites the fact that five of eight chemicalscommonly causing these symptoms were not found in the recalled cans while the other threealso occur naturally in other canned foods For several reasons this evidence lends littlecredible support to the authors conclusionTo begin with the author relies partly on the fact that although three of the eight chemicalsmost commonly blamed for nausea and dizziness appeared in Promofoods recalled tunathese chemicals also occur naturally in other canned foods However this fact alone lends nosupport to the authors conclusion for two reasons First the author might be ignoring animportant distinction between naturally occurring chemicals and those not occurring naturallyIt is entirely possible that these three chemicals do not occur naturally in Promofoods tunaand that it is for this reason that the chemicals cause nausea and dizziness Secondly it isentirely possible that even when they occur naturally these chemicals cause the samesymptoms Unless the author rules out both possibilities he cannot reliably conclude that therecalled tuna would not cause these symptomsAnother problem with the argument is that the authors conclusion is too broad Based onevidence about certain chemicals that might cause two particular heath-related symptoms theauthor concludes that the recalled tuna contains no chemicals that pose a health risk Howeverthe author fails to account for the myriad of other possible health risks that the recalled tunamight potentially pose Without ruling out all other such risks the author cannot justifiablyreach his conclusionA third problem with the argument involves that fact that the eight particular chemicals withwhich the test was concerned are only the eight most commonly blamed for nausea anddizziness It is entirely possibly that other chemicals might also cause these symptoms andthat one or more of these other chemicals actually caused the symptoms Without ruling outthis possibility the author cannot jusufiably conclude that the recalled tuna would not causenausea and dizzinessA final problem with the argument involves thetesting procedure itself The author providesno information about the number of recaUed cans tested or the selection method used Unlessthe number of cans is a sufficiently large sample and is statistically repre sentative of all therecalled cans the studys results are not statistically reliableIn conclusion the article is unconvincing as it stands To strengthen the assertion that therecalled tuna would not cause nausea and dizziness the author must provide evidence thatthe three chemicals mentioned that occur naturally in other canned foods also appear naturallyin Promofoods tuna The author must also provide evidence that ingesting other canned foodscontaining these three chemicals does not cause these symptoms To better evaluate theargument we would need to know whether the sample used in the tests was statisticallysignificant and representative of all the recalled tuna We would also need to know what otherchemicals in the recalled tuna might pose any health risk at all

5그룹 불충분 조건오류 빈출

Natures Way a chain of stores selling health food and other health-related products is opening its next franchise in the town of Plainsville The store should prove to be very successful Natures Way franchises tend to be most profitable in areas where residents lead healthy lives and clearly Plainsville is such an area Plainsville merchants report that sales of running shoes and exercise clothing are at all-time highs The local health club has more members than ever and the weight training and aerobics classes are always full Finally Plainsvilles schoolchildren represent a new generation of potential customers these schoolchildren are required to participate in a fitness-for-life program which emphasizes the benefits of regular exercise at an early age

Write a response in which you examine the stated andor unstated assumptions of the argument Be sure to

GRE AWA John박 박정어학원

explain how the argument depends on these assumptions and what the implications are for the argument if the assumptions prove unwarranted

False cause

First of all the author believes that the Increased sales of running shoes and exercise clothing indicates

Plainesville residentsrsquo interest in leading healthy lives However this assumption is not logically convincing for

several reasons could be a fashion trendTime shift ldquoFitness for liferdquo might not have any influence on schoolchildren as they growFalse cause There could be other reasons for member increase in the health clubAll of the above are insufficient condition

The author has to prove that local residents are interested in leading healthy lives However he supports his conclusion with insufficient evidence Nevertheless even if the residents are concerned with health naturersquos way may not be successful First

그 동안의 경험을 토대로 볼 때 건강생활과 밀접히 관련되어 있는 거주 지역에서 본 상점들이 아주 호응을 얻고 있다 따라서 이러한 주민들이 많이 거주하고 있는 Plainsville 에 새로운 상점들을 계속 세워야 한다 이 지역 상인들은 런닝화와 운동복 판매가 가장 높다고 말한다 불과 5 년전에는 거의 전무하다시피하던 지역 헬스 클럽의 경우도 엄청나게 많은 회원을 확보하고 있으며 웨이트 트레이닝과 에어로빅 강좌들도 항상 만원이라고 한다 새로운 고객층을 예측해 보는 것도 가능하다 이 지역의 학생들의 경우 Fitness for Life프로그램을 받게 되는데 이러한 프로그램을 통해서 유년시절부터 정규적인 운동 습관을 들이게 하고 있는 것이 그것이다

결론 We should therefore build our next new store in Plainsville

반박 그동안의 경험에 의한 과거 통계가 꼭 여기에도 적용되는건 아니다 5 년전 헬스 클럽이 잘 안되었던게 다른 원인이였을수 있다(강사수준 미달 강좌미비)tourist 에 의한 원인 일수 있다 어렸을때부터 운동을 했다고 해서 커서도 관심이 있지는 않다 (오히려 반감이 있을수 있다 혹은 건강하기에 건강에 관심이 적을수도 있다)운동복이나 신발의 판매가 육체노동에 의한 것일수도 있다

IntroductionSupport1049896In this memorandum the author asserts that Naturersquos Way should build its next newstore in Plainsville To support this assertion the author states that Plainsvillesmerchantsrsquo sales of exercise clothing are going well the local health club has moremembers than ever and a new generation of customers will help to ensure NaturersquosWayrsquos success At first glance the authorrsquos assumption seems convincing but in-depth scrutiny revealsthat it lacks substantial evidence as it stands

Body 1-SamplingTopic Sentence 1To begin with the author assumes that the merchantsrsquo report indicates that the residentsare concerned about their health However this assumption is based on unsubstantiated

GRE AWA John박 박정어학원

data Example 1 (Rebuttal1) First if we do not know the total volume of items sold and the price of the goods exactly we cannot infer whether the residents are actually buying many goods Example 2 (Rebuttal2)In addition to that the report emphasizes the rising sales of running shoes and exerciseclothing however these may not be hot-selling items for Naturersquos Way or may not be theproducts the company is planning to sell Concluding Sentence Therefore in order to make the argument reliable the author should reconsider themerchantsrsquo report with more detailed data

Body 2-CausalTopic Sentence 2Second the author contends that the health clubs classes are full yet this does not meanthat many people actually use the health club other factors may be the real cause forthose closed classes Example 1 (Rebuttal1) To begin with if the health club is very small the number of people working out wouldnot be a large one In fact regular gym-going may just be a vogue among a smallunrepresentative segment of Plainsvilles population Example 2 (Rebuttal2) Moreover it is possible that most of the people who exercise in the health club do weight training and aerobics only to look good and to meet other singles not for their health In that case there would be little demand for health products Concluding SentenceThus the author should not hasten to presume what really caused people to be interested in a healthier lifestyle and enroll in the health club

Body 3-Time-ShiftTopic Sentence 3Finally the author highlights that Naturersquos Way can expect a new generation of customersin Plainsville that will help the company in the long term This notion is mistaken in that itassumes the conditions of the present will continue unchanged in the future Although theschool children are required to participate in the fitness for life program they may notnecessarily buy Naturersquos Ways products Example 1 (Rebuttal1) In the first instance they may suffer a fall in purchasing power arising from future economic difficulties this would cause reluctance to spend a considerable amount of money on health products which tend to be more expensive Example 2 (Rebuttal2)Another possibility is that there may emerge many competitor companies vying with Naturersquos Way so that in the future the school children may not feel the necessity to purchase one companyrsquos health products over anotherrsquosConcluding Sentence Thus the authorrsquos assumption is highly speculative since it relies heavily on unknowablefuture circumstances

ConclusionThesis In sum the author uses many assumptions that are insufficient in supporting his claimsSupportIn order for the authorrsquos claims to be convincing he needs to advance more persuasiveevidence that people in Plainsville really are concerned with their health and health foodThe following was written as a part of an application for a small-business loan by a group of developers in the city of Monroe

A jazz music club in Monroe would be a tremendously profitable enterprise Currently the nearest jazz club is 65 miles away thus the proposed new jazz club in Monroe the C-Note would have the local market all to itself Plus jazz is extremely popular in Monroe over 100000 people attended Monroes annual jazz festival last summer several well-known jazz musicians live in Monroe and the highest-rated radio program in Monroe is Jazz Nightly which airs every weeknight at 7 PM Finally a nationwide study indicates that the typical jazz fan spends close to $1000 per year on jazz entertainment

1 Write a response in which you discuss what specific evidence is needed to evaluate the argument and explain how the evidence would weaken or strengthen the argument

2 Write a response in which you examine the stated andor unstated assumptions of the argument Be

GRE AWA John박 박정어학원

sure to explain how the argument depends on these assumptions and what the implications are for the argument if the assumptions prove unwarranted

3 Write a response in which you discuss what questions would need to be answered in order to decide whether the prediction and the argument on which it is based are reasonable Be sure to explain how the answers to these questions would help to evaluate the prediction

Group error nationwide survey may not reflect local trends Is the nationwide jazz fan population substantialInsufficient non-residents of Monroe may have attended the jazz festival (Body alternative explanation last year may have been an anomaly The author should consider data from various years) The author should indicate how many out of 100000 were Monroe residentsNationwide study Does this reflect Insufficient Citizens of Monroe may continue to go to the jazz club 65 miles away

Are the people in Monroe really interested in jazzMajority of the people who attended the jazz festival might not be Monroe residentsSurvey error nationwide study may not be applicable to MonroeJazz musicians who live in MonroeMonopolyRadio station

In this business application the author claims that the proposed jazz club C Note will be very profitable in Monroe To support this claim the author argues for his case with several evidences At first glance the authorrsquos argument seems convincing however careful scrutiny reveals that his argument in specious

To begin with the author claims that Monroersquos citizens are interested in jazz He presents three evidences First Secondhellip Thirdhellip Howeverhellip

Monroe 시에 있는 재즈 음악 클럽은 수익성이 좋은 사업이다 현재 가장 가까이에 있는 클럽은 65 마일 정도 떨어져 있다 따라서 이번에 세우려고 하는 C Note 는 독보적인 위치를 점할것이다 더군다나 재즈는 이 시에서 가장 인기있는 음악이다 지난 여름 재즈 축제에서는 10 만명 이상의 Morone 시 주민이 참석하였고 몇몇 유명한 재즈 음악가들도 이곳에 살고 있으며 저녁때 방영되는 라디오 프로그램중에서 최고의 시청률을 보이고 있는 것도 Jazz Nightly 이다 전국조사에서도 전형적인 재즈 팬들은 재즈 분야에 년간 1천 달러 가까이 지출하고 있는 것으로 보고되고 있다 따라서 C Note 클럽이 돈을 벌 수 있는 사업이라는 것은 확실한 것이다

결과 It is clear that the C Note cannot help but make money반박 nearest jazz club 이 양질의 써비스로 여전히 손님을 끌수도 있다Festival 에 얼마나 참여하는지가 jazz 의 인기를 반영하지 않는다 뮤지션이 많이 사는거랑 jazz 의 인기가 상관없다라디오 프로그램이 다른 요인에 의해서 인기일수도 있다 (진행자때문)전국 통계 적용 불가화목 실전반_Ms Noh6In this application the author suggests that a jazz club in Monroe will make a number of profits To support this suggestion the author exemplifies the local condition popularity of jazz in Monroe and nationwide study However careful scrutiny of each of the facts reveals that it provides little credible support for the authorrsquos recommendation Good clear intro

First the author assumes that jazz is popular in Monroe because of several facts the jazz festival last year had high participation some famous jazz musicians live in Monroe and the high-rated radio program is lsquoJazz Nightlyrsquo However this assumption has many drawbacks that must be seriously considered(Good topic sentences) If many attendants in the last-yearrsquos festival came from other cities and not Monroe it is hard to conclude that Monroersquos people like jazz Therefore the author must examine how many Monroe residents actually attended the festival On top of that there is little relationship between habitation of famous jazz musician and the popularity of jazz in Monroe Although several well-known musicians live there if they do not take part in any jazz performance of Monroe this might have no effect to the interest of Monroersquos residents

GRE AWA John박 박정어학원

about jazz Finally in the case of radio program this is also not suitable reason why jazz is popular in Monroe It might be possible that people cannot help choosing lsquoJazz Nightlyrsquo because there are few radio programs at Night The fact that the radio program is the highest rating program is not a germane evidence The approximate number of listeners would be the more crucial evidence Therefore the author needs to seriously deliberate the correlation between jazzrsquos popularity in Monroe and his examples (Good logical flow and clarity)

Second the author uses as evidence the nationwide study that jazz fans spend much money on jazz entertainment to substantiate why starting a jazz club in Monroe will be profitable In other words the author assumes that the characteristics of a nationwide study can be applied to Monroe The national study would lend support to the applicantrsquos claim only if residents in Monroe typify national jazz fans However the author does not provide credible evidence that this is the case Moreover the populations of jazz fans nationwide may be insubstantial Thus the author should not infer hastily that Monroersquos residents will spend much money on enjoying jazz from the nationwide study

Lastly even if jazz is popular in Monroe C Note may not be successful It is entirely possible that residents might still prefer other clubs where they have always went In addition there is another possibility that the nearest jazz club will attract many of Monroersquos people because it serves fine performances and is equipped with favorite facilities Without considering these other possibilities the author cannot make his argument convincing In sum the author presents many reasons that are insufficient in supporting his or her claim In order for the authorrsquos claims to be convincing he needs to advance more persuasive evidence such as the total number of Monroe residents who attended the jazz festival the effects on the popularity of jazz by the musicians living in Monroe and the actual number of residents who would typify themselves to be jazz fans through a local survey Without substantial evidence that C Note will be successful in Monroe the businessmen may be overinvesting in what might lead to a business failureExcellent clarity Score 50

The following appeared in a newsletter offering advice to investors

Over 80 percent of the respondents to a recent survey indicated a desire to reduce their intake of foods containing fats and cholesterol and today low-fat products abound in many food stores Since many of the food products currently marketed by Old Dairy Industries are high in fat and cholesterol the companys sales are likely to diminish greatly and company profits will no doubt decrease We therefore advise Old Dairy stockholders to sell their shares and other investors not to purchase stock in this company

Write a response in which you discuss what questions would need to be answered in order to decide whether the advice and the argument on which it is based are reasonable Be sure to explain how the answers to these questions would help to evaluate the advice

Survey 80

GRE AWA John박 박정어학원

Old Dairy could change their products and manufacture low fat dairy foodsLess competing companies Old Dairy could eventually be the only company that produces hellipImprecise numbers and measurementsCustomers may still buy high fat dairy products

The author of the newsletter is offering potentially dangerous advice by recommending Old Dairy stockholders to withdraw investment and stop purchase What is more the authorrsquos prediction debases the reputation and business of Old Dairy and if false could devoid the investment opportunity of the newsletter readers Therefore investors should examine whether the authorrsquos evidences are substantial

To begin with the author states that 80 percent of the respondents in a survey indicated a desire to reduce their intake of foods He therefore argues that Old Dairyrsquos high fat and cholesterol products would decrease in sales However the author makes a crucial error in this argument First the author provides no evidence that the surveyrsquos results are statistically reliable Were they representative of all the customers Were they chosen for the survey randomly Furthermore the desire to reduce fat and cholesterol intake is a pervasive trend in todayrsquos opulent society however the author erroneously identifies this as a new phenomenon which will affect consumer trends Second having a desire to reduce fat and cholesterol intake does not necessarily indicate that people who have this desire will actually reduce consuming these types of products It is entirely possible that they may continue buying Old Dairy products for its quality and taste Accordingly the author cannot draw any firm conclusion that people will not buy Old Dairy products Therefore if any of these cases are true the author may be offering investors a detrimental investment advice

최근 조사에 대한 응답자중 80 이상이 자신이 먹는 음식에서 지방과 콜레스테롤의 함유량을 줄이고 싶다고 한다 아울러 요즘은 많은 식료품 가계에서 저지방 제품들을 많이 취급하고 있다 현재 Old Dairy Industries가 판매하고 있는 많은 음식제품들은 지방과 콜레스테롤이 높기 때문에 이 회사의 매출이 격감할 것으로 보이며 당연히 매출이익도 줄어들것이다 따라서 이 회사의 주주들은 주식을 매각하고 다른 주식 투자가들도 이 회사의 주식을 매입하지 않는 것이 좋다

결론 Old Dairy stockholders to sell their shares and other investors not to purchase stock in this company

반박 모든 상품이 다 고 지방 고 칼로리는 아니다(비록 많을지라도) 일부의 식품의 경우 기호에 맞어서 히트해서 전체적인 수입이 증가할 수도 있다국내시장만 생각할 수 없다( 외국시장에서 호황을 누릴수 있다 )입맛이라는게 즉각 바뀌는게 아니다

The following appeared in a letter to the editor of the Balmer Island Gazette

On Balmer Island where mopeds serve as a popular form of transportation the population increases to 100000 during the summer months To reduce the number of accidents involving mopeds and pedestrians the town council of Balmer Island should limit the number of mopeds rented by the islands moped rental companies from 50 per day to 25 per day during the summer season By limiting the number of rentals the town council will attain the 50 percent annual reduction in moped accidents that was achieved last year on the neighboring island of Seaville when Seavilles town council enforced similar limits on moped rentals

1 Write a response in which you discuss what questions would need to be answered in order to decide whether the recommendation is likely to have the predicted result Be sure to explain how the answers to these questions would help to evaluate the recommendation

2 Write a response in which you discuss what questions would need to be answered in order to decide whether the prediction and the argument on which it is based are reasonable Be sure to explain how the answers to these questions would help to evaluate the prediction

3 Write a response in which you examine the stated andor unstated assumptions of the argument Be sure to explain how the argument depends on these assumptions and what the implications are for the argument if the assumptions prove unwarranted

Whatrsquos the actual population of Balmer Island 100000mdashis this a significant increase What kind of accidents Skin abrasions or serious injury And compared to Seaville how serious are the accidents and the actual number of accidents Did Seaville enforce other restrictions like safety signsHow different are the conditions of Balmer

GRE AWA John박 박정어학원

and Seaville regarding population road (safety) conditions topography other town-government regulation How much will the economy of Balmer be affected do to this restriction Could it cause an economic recession due to the fact that these rental companiesrsquo chance to make money is only during the summer thereby weakening the economic infrastructure Are there any other ways that could better alleviate the accident rate

Statistics 50-impreciseAnalogy Balmer compared with TorseauFalse Cause Accidents might have occurred because of reasons other than mopeds False Cause population increase may not be part of the cause of the accidentsOther explanations for the accident pedestrians few road safety regulations narrow roadsThere could be other better solutionshellip

Balmer Island의 인구가 여름철에는 십만명으로 늘어난다 2륜차와 보행자간 사고를 줄이기 위해 시의회는 6개의 자전거를 포함한 2륜차 대여업체에게 이 기간동안에는 대여숫자를 일일 50에서 30으로 제한하도록 할 것이다 대여숫자를 줄임으로써 시 의회는 지난해 이웃한 Torseau섬에서 이와 동일한 규제를 시행해서 50나 줄인 결과를 보고 마찬가지로 50를 줄일수 있다고 확신하고 있다

결론 The town council of Balmer Island should linit the number

반박 보행자의 부실에 의해서 사고가 많이 일어날수도 있다렌탈수의 줄임만이 대책은 아니다(대부분의 사람들이 렌탈 보다는 소유하고 있을 수도 있다)옆섬과는 상황이 다를수도 있다(그 섬에서는 사고의 원인이 많은 자전거 수로 인한것일수있다) 하지만 이 섬은 좁은 도로가 원인일 수도 있고 도로 안전 장치의미비가 원일일수 있다

In this letter the author recommends that Balmer Island should limit the number moped rentals from 50 to 30 per day To support this recommendation the author points out several reasons However careful scrutiny of each of the facts reveals that it is filled with unanswered questions that could significantly weaken the authorrsquos recommendation with loops and holes which are answered

The recommendation depends on the assumption that no alternative means of reducing the number of accidents are available However the author fails to offer any evidence to substantiate this crucial assumption It is highly possible that means other than this would better solve the problem Perhaps they could widen the roads or put-up more safety signs Or perhaps the accidents were due to the lack of skills in which case proper safety training would significantly alleviate the problem Without considering and ruling out these and other alternative means of reducing accidetns the author cannot confidently conclude that merely emulating Torseau would suffice Moreover the author is advising a recommendation which could potentially harm the economy of Balmer Island sincehellip Moreover the Balmer Island should alternative means to reduce accidents because limiting moped rentals during the summer could harm the economy of Balmerhellip

First of all the author believes that increase in population and the number of moped rentals are responsible for the accidents It is entirely possible that other factors are responsible for the accidents Perhaps Balmer Islandrsquos lack of safety signs was a major factor Or maybe the roads are narrow and dangerous on the Island therefore the town council could enforce stricter traffic regulations to alleviate the problem Accordingly if either of these scenarios is true the author cannot draw any firm conclusion that increase in the number of population and moped rentals are the cause of the accidents

The author of this editorial recommends that to reduce accidents involving mopeds andpedestrians Balmer Islands city council should restrict moped rentals to 30 per day down from50 at each of the islands six rental outlets To support this recommendation the author citesthe fact that last year when nearby Torseau Islands town council enforced similar measuresTorseaus rate of moped accidents fell by 50 For several reasons this evidence providesscant support for the authors recommendationTo begin with the author assumes that all other conditions in Balmer that might affect therate of moped-pedestrian accidents will remain unchanged after the restrictions are enactedHowever with a restricted supply of rental mopeds people in Balmer might purchase mopedsinstead Also the number of pedestrians might increase in the future with more pedestriansespecially tourists the risk of moped-pedestrian accidents would probably increase For thatmatter the number of rental outlets might increase to make up for the artificial supplyrestriction per outlet--a likely scenario assuming moped rental demand does not declineWithout considering and ruling out these and other possible changes that might contribute to ahigh incidence of moped-pedestrian accidents the author cannot convince me that theproposed restrictions will necessarily have the desired effect

GRE AWA John박 박정어학원

Next the author fails to consider other possible explanations for the 50 decline inTorseaus moped accident rate last year Perhaps last year Torseau experienced unusually fairweather during which moped accidents are less likely Perhaps fewer tourists visited Tot seanlast year than during most years thereby diminishing the demand for rental mopeds to belowthe allowed limits Perhaps last year some of Torseaus moped rental outlets purchased newmopeds that are safer to drive Or perhaps the restrictions were already in effect but were notenforced until last year In any event a decline in Torseaus moped accident rate during onlyone year is scarcely sufficient to draw any reliable conclusions about what might have causedthe decline or about what the accident rate will be in years aheadAdditionally in asserting that the same phenomenon that caused a 50 decline in mopedaccidents in Torseau would cause a similar decline in Balmer the author relies on what mightamount to an unfair analogy between Balmer and Torseau Perhaps Balmers ability to enforcemoped-rental restrictions does not meet Torseaus ability if not then the mere enactment ofsimilar restrictions in Balmer is no guarantee of a similar result Or perhaps the demand formopeds in Torseau is always greater than in Balmer Specifically if fewer than all availablemopeds are currently rented per day from the average Balmer outlet while in Torseau everyavailable moped is rented each day then the proposed restriction is likely to have less impacton the accident rate in Balmer than in TorseauFinally the author provides no evidence that the same restrictions that served to reduce theincidence of all moped accidents by 50 would also serve to reduce the incidence ofaccidents involving mopeds and pedestrians by 50 Lacking such evidence it is entirelypossible that the number of moped accidents not involving pedestrians decreased by a greaterpercentage while the number of moped-pedestrian accidents decreased by a smallerpercentage or even increased Since the author has not accounted for these possibilities theeditorials recommendation cannot be taken seriouslyIn conclusion the recommendation is not well supported To convince me that the proposedrestriction would achieve the desired outcome the author would have to assure me that nochanges serving to increase Balmers moped-pedestrian accident rate will occur in theforeseeable future The author must also provide dear evidence that last years decline inmoped accidents in Torseau was attributable primarily to its moped rental restrictions ratherthan to one or more other factors In order to better evaluate the recommendation I wouldneed more information comparing the supply of and demand for moped rentals on the twoislands I would also need to know the rate of mopedpedestrian accidents in Torseau both priorto and after the restrictions were enforced in TorseauThe following appeared in a magazine article about planning for retirement

Clearview should be a top choice for anyone seeking a place to retire because it has spectacular natural beauty and a consistent climate Another advantage is that housing costs in Clearview have fallen significantly during the past year and taxes remain lower than those in neighboring towns Moreover Clearviews mayor promises many new programs to improve schools streets and public services And best of all retirees in Clearview can also expect excellent health care as they grow older since the number of physicians in the area is far greater than the national average

Write a response in which you discuss what specific evidence is needed to evaluate the argument and explain how the evidence would weaken or strengthen the argument

-Natural beauty and consistent climate may not be the most wanted qualities-Housing costs could have lowered on a national level wealthy retirees may not care about costs-Taxes may be high compared to the nationrsquos average tax rate-What about other qualities of Clearview Crime rate what qualities would retirees want -If schools streets and public services need improvement then this is proof that the current condition of Clearview is low Or due to budgetary reasons the mayor may not follow-up on his promise because of lowered tax rate -Schools and people who are retired no relationship-Physicians What kind of physicians Number is irrelevant Are these physicians capable of addressing the illnesses of old people

This author argues that anyone seeking a place to retire should choose Clearview To supportthis argument the article cites Clearviews consistent climate and natural beauty its fallinghousing costs its low property taxes compared to nearby towns and the mayors promise toimprove schools streets and services The article also claims that retirees can expectexcellent health care because the number of physicians in Clearview greatly exceeds thenational average This argument is flawed in several critical respectsTo begin with although consistent climate and natural beauty might be attractive to manyretirees these features are probably not important to all retirees For many retirees it isprobably more important to live near relatives or even to enjoy changing seasons Thus I

GRE AWA John박 박정어학원

cannot accept the authors sweeping recommendation for all retirees on this basisAlso Clearviews declining housing costs do not necessarily make Clearview the best placeto retire for two reasons First despite the decline Clearviews housing costs might be highcompared to housing costs in other cities Secondly for wealthier retirees housing costs arenot likely to be a factor in choosing a place to retire Thus the mere fact that housing costshave been in decline lends scant support to the recommendationThe articles reliance on Clearviews property-tax rates is also problematic in two respectsFirst retirees obviously have innumerable choices about where to retire besides Clear viewand nearby towns Secondly for retirees who are well-off financially property taxes are notlikely to be an important concern in choosing a place to retire Thus it is unfair to infer fromClearviews property-tax rates that retirees would prefer ClearviewYet another problem with the argument involves the mayors promises In light of Clearviewslow property-tax rates whether the mayor can follow through on those promises is highlyquestionable Absent any explanation of how the city can spend more money in the areas citedwithout raising property taxes I simply cannot accept the editorials recommendation on thebasis of those promises Besides even if the city makes the improvements promised thoseimprovements--particular the ones to schools--would not necessarily be important to retireesFinally although the number of physicians in Clearview is relatively high the per capitanumber might be relatively low Moreover it would be fairer to compare this per capita numberwith the per capita number for other attractive retirement towns--rather than the nationalaverage After all retirees are likely to place a relatively heavy burden on health-careresources Besides the article provides no assurances that the number of physicians inClearview will remain high in the foreseeable futureIn conclusion the recommendation is poorly supported To strengthen it the author mustconvince me--perhaps by way of a reliable survey--that the key features that the vast majorityof retirees look for in choosing a place to live are consistent climate natural beauty and lowhousing costs The author must also provide better evidence that Clear views property taxesare lower than the those of cities in other areas The author must also explain how the city canmake its promised improvements without raising property taxes Finally to better assess theargument I would need to now how the per capita number of physicians in Clearview wouldcompare to the national average in the futureThe following appeared as a letter to the editor from a Central Plaza store owner

Over the past two years the number of shoppers in Central Plaza has been steadily decreasing while the popularity of skateboarding has increased dramatically Many Central Plaza store owners believe that the decrease in their business is due to the number of skateboard users in the plaza There has also been a dramatic increase in the amount of litter and vandalism throughout the plaza Thus we recommend that the city prohibit skateboarding in Central Plaza If skateboarding is prohibited here we predict that business in Central Plaza will return to its previously high levels

Write a response in which you discuss what questions would need to be answered in order to decide whether the recommendation is likely to have the predicted result Be sure to explain how the answers to these questions would help to evaluate the recommendation

Why two years ago What happened two years ago which started this declineIs the dramatic increase in the ldquopopularityrdquo of skateboarding the cause of the steady decline of shoppers Are there any malls nearby Were there any changes nearby which could affect the decline in customersmdasha big mall perhaps Could the decline be due to the shop ownersHow many skateboarders use the plazaWhere do they skateboardDo they shop and are they customersAre the increase in litter and vandalism due to skateboarders Could this be alleviated by installing CCTVs and hiring security

This editorial concludes that the city should ban skateboarding from its downtown CentralPlaza in order to attract visitors to that area to return the area to its former glory and to makeit a place where people can congregate for fun and relaxation To justify this conclusion theeditorial points out that skateboarders are nearly the only people one sees anymore at CentralPlaza and that the Plaza is littered and its property defaced The editorial also points out thatthe majority of downtown merchants support the skate boarding ban This argument is flawedin several critical respectsFirst the editorials author falsely assumes that a ban on skateboarding is both necessaryand sufficient to achieve the three stated objectives Perhaps the city can achieve thoseobjectives by other means as well--for example by creating a new mall that incorporates anattractive new skateboard park Even if banning skateboarders altogether is necessary to meetthe citys goals the author has not shown that this action by itself would suffice Assuming thatthe Plazas reputation is now tarnished restoring that reputation and in turn enticing peopleback to the Plaza might require additional measures--such as removing litter and graffiti

GRE AWA John박 박정어학원

promoting the Plaza to the public or enticing popular restaurant or retail chains to the PlazaSecondly the editorial assumes too hastily that the Plazas decline is attributable to theskateboarders--rather than to some other phenomenon Perhaps the Plazas primary appeal inits glory days had to do with particular shops or eateries which were eventually replaced byless appealing ones Or perhaps the crime rate in surrounding areas has risen dramatically forreasons unrelated to the skateboarders presence at the Plaza Without ruling out these andother alternative explanations for the Plazas decline the editorials author cannot convince methat a skateboard ban would reverse that declineThirdly the editorials author might be confusing cause with effect--by assuming that theskateboarders caused the abandonment of the Plaza rather than vice versa It is entirelypossible that skateboarders did not frequent the Plaza until it was largely abandoned--andbecause it had been abandoned In fact this scenario makes good sense since skateboardingis most enjoyable where there are few pedestrians or motorists to get in the wayFourth it is unreasonable to infer from the mere fact that most merchants favor the ban thatthe ban would be effective in achieving the citys objectives Admittedly perhaps thesemerchants would be more likely to help dean up the Plaza area and promote their businesseswere the city to act in accordance with their preference Yet lacking any supporting evidencethe author cannot convince me of this Thus the survey amounts to scant evidence at best thatthe proposed ban would carry the intended resultFinally the author recommends a course of action that might actually defeat the citysobjective of providing a fun and relaxing place for people to congregate In my experienceskateboarding contributes to an atmosphere of fun and relaxation for adults and children alikemore so than many other types of ambiance Without considering that continuing to allowskateboarding--or even encouraging this activity--might achieve the citys goal more effectivelythan banning the activity the author cannot convincingly conclude that the ban would be in thecitys best interestsIn sum the argument is a specious one To strengthen it the editorials author must providedear evidence that skateboarding and not some other factor is responsible for the conditionsmarking the Plazas decline The author must also convince me that no alternative means ofrestoring the Plaza are available to the city and that the proposed ban by itself would suffice toattract tourists and restore the Plaza to its former glory Finally to better assess the argument itwould be useful to know the circumstances under which the downtown merchants would bewilling to help the city achieve its objectives

6그룹 약한 비유 빈출

The following recommendation appeared in a memo from the mayor of the town of Hopewell

Two years ago the nearby town of Ocean View built a new municipal golf course and resort hotel During the past two years tourism in Ocean View has increased new businesses have opened there and Ocean Views tax revenues have risen by 30 percent Therefore the best way to improve Hopewells economymdashand generate additional tax revenuesmdashis to build a golf course and resort hotel similar to those in Ocean View

Write a response in which you examine the stated andor unstated assumptions of the argument Be sure to explain how the argument depends on these assumptions and what the implications are for the argument if the assumptions prove unwarranted

GRE AWA John박 박정어학원

Assumptions The author assumes that OVrsquos municipal golf course and resort hotel caused tourism new businesses and increased tax revenues There may be other reasons advertising promo He assumes that this will continueAssumes that Ocean View and Hopewell are similar in many waysmdashthe name suggests otherwise OV may have always been a tourist attractions for its beaches We need to know the topography

2년전 Ocean View 시는 시정 소유 골프 및 휴양지 호텔을 신축했다 그리고 지난 2년동안 이 시의 관광객이 증가했으며 새로운 사업들이 생겨났다 그에따라 시의 세수도 30나 증가했다 Hopewell의 경제를 향상시키고 아울러 세수를 늘릴 수 있는 가장 좋은 방법은 Ocean View에 세워진 것과 같은 골프 시설과 휴양지 호텔을 신축하는 것이다

1 다른 요인으로 관광 산업이 발전했을 수도 있다 문화 유적이 발견이 되었거나 도로의 정비등으로 여행자가 늘었을 수도 있다

2 관광 산업의증가가 늘어난 세수의 원인이 아니라 새로 유입된 인구의 증가나 다른 공장에서 발생한 것일 수 있다

3 2년동안 한참 골프가 붐을 이루었을 수 있다 경제상황이 나빠지거나 다른 레포츠가 인근 지역에 생겨난다면 골프하는 사람이 줄어들 수 있다

In this memo HopeweUs mayor recommends that in order to stimulate the towns economyand boost tax revenues HopeweU should build a new golf course and resort hotel just as thetown of Ocean View did two years ago To support this recommendation the mayor points outthat in Ocean View during the last two years tourism has increased new businesses haveopened and tax revenues have increased by 30 I find the mayors argument unconvincingin several important respectsFirst of all it is possible that the mayor has confused cause with effect respecting the recentdevelopments in Ocean View Perhaps Ocean Views construction of a new golf course andhotel was a response to previous increases in tourism and business development increasesthat have simply continued during the most recent two years Since the mayor has failed toaccount for this possibility the claim that Hopewell would boost its economy by alsoconstructing a golf course and hotel is completely unwarrantedSecondly the mayor fails to account for other possible causes of the trends in Ocean Viewduring the last two years The increase in tourism might have been due to improving economicconditions nationwide or to unusually pleasant weather in the region The new businessesthat have opened in Ocean View might have opened there irrespective of the new golf courseand hotel And the 30 increase in tax revenues might have been the result of an increase intax rates or the addition of a new type of municipal taxWithout ruling out these and other alternative explanations for the three recent trends inOcean View the mayor cannot reasonably infer based on those trends that Hopewellseconomy would benefit by following Ocean Views exampleThirdly even if the recent trends in Ocean View are attributable to the construction of the newgolf course and hotel there the mayor assumes too hastily that the golf course and hotel willcontinue to benefit that towns overall economy The mayor has not accounted for thepossibility that increased tourism will begin to drive residents away during tourist season orthat new business development will result in the towns losing its appeal as a place to visit or tolive Unless the mayor can convince me that these scenarios are unlikely I cannot accept themayors recommendation that Hopewell follow Ocean Views exampleFinally the mayors argument rests on the unsubstantiated assumption that Hopewell andOcean View are sufficiently alike in ways that might affect the economic impact of a new golfcourse and hotel Hopewell might lack the sort of natural environment that would attract moretourists and new businesses to the town--regardless of its new golf course and hotel For thatmatter perhaps Hopewell already contains several resort hotels and golf courses that are notutilized to their capacity If so building yet another golf course and hotel might amount to amisallocation of the towns resources--and actually harm the towns overall economyIn sum the mayors recommendation is not well supported To bolster it the mayor mustprovide better evidence that Ocean Views new golf course and hotel and not some otherphenomenon--has been responsible for boosting Ocean Views economy during the last twoyears To better assess the recommendation I would need to know why Ocean View decidedto construct its new golf course and hotel in the first place--specifically what events prior toconstruction might have prompted that decision I would also need to thoroughly compare

GRE AWA John박 박정어학원

HopeweU with Ocean View--especially in terms of their appeal to tourists and businesses--todetermine whether the same course of action that appears to have boosted Ocean Viewseconomy would also boost Hopewells economy

The following is part of a memorandum from the president of Humana University

Last year the number of students who enrolled in online degree programs offered by nearby Omni University increased by 50 percent During the same year Omni showed a significant decrease from prior years in expenditures for dormitory and classroom space most likely because instruction in the online programs takes place via the Internet In contrast over the past three years enrollment at Humana University has failed to grow and the cost of maintaining buildings has increased along with our budget deficit To address these problems Humana University will begin immediately to create and actively promote online degree programs like those at Omni We predict that instituting these online degree programs will help Humana both increase its total enrollment and solve its budget problems

Write a response in which you discuss what questions would need to be answered in order to decide whether the prediction and the argument on which it is based are reasonable Be sure to explain how the answers to these questions would help to evaluate the prediction

Is Omni University successful due to the online degree program 50 Is the decrease in expenditures for dormitory and classroom space due to the decrease in of on-campus students Which classes were successful Does HU have those classes

Even if the long-distance degree programs at Omni University benefited the school the presidentrsquos recommendation that Human College should emulate Omni University is too hasty First OUrsquos name implies that the school would have more majors than Humanahellip the president should examine which degrees were in the long-distance programhellip

지난해에는 Omni 대학에서 개강했던 원거리 학생 학점 취득 프로그램을 등록했던 학생들의 숫자가 50나 증가했다 같은해 기간동안 Omni 대학에서는 그 전년도부터 기숙사와 학급의 공간 확충을 위한 예산을 대폭 줄였는데 이는 이 원거리 학점 취득 프로그램이 양방향 비디오 컴퓨터 접속을 통해서만 가능한 수업지도 방식이기때문인 것으로 보인다 반면 지난 3개년 동안 Humana 대학에서의 수강률은 감소한데다가 건물

GRE AWA John박 박정어학원

유지비도 올랐다 따라서 Humana대학의 수강을 늘리고 예산손실을 회복하기 위해서는 Omni 대학에서 취한 조치와 같은 능동적인 프로그램을 추진해야 한다

결론 we should initiate and actively promote long-distance degree programs like those at Omni 반박 원거리 학생 취득 프로그램 숫자가 증가한거하고 예산이 줄어드는 것 사이에 연관이 약하다 (causal 학생의 증가로 관리비용 증가할수 있음 원거리 수업가능 장비도입에의한 비용발생)bad analogy(omni university 하고 같은 조건이 아니다 )-gt omni college 가 강좌내용이 좋아서 학생의 등록이 많을수 있다 Humana 대학에서 만들었다 하더라도 인기 없을수 있음다른 요인에 의해서 Humana 대학의 수강 인원이 증가할수 있음(비록 과거엔 인기가 없었을지라도)

The following appeared as part of a business plan developed by the manager of the Rialto Movie Theater

Despite its downtown location the Rialto Movie Theater a local institution for five decades must make big changes or close its doors forever It should follow the example of the new Apex Theater in the mall outside of town When the Apex opened last year it featured a video arcade plush carpeting and seats and a state-of-the-art sound system Furthermore in a recent survey over 85 percent of respondents reported that the high price of newly released movies prevents them from going to the movies more than five times per year Thus if the Rialto intends to hold on to its share of a decreasing pool of moviegoers it must offer the same features as Apex

Write a response in which you discuss what questions would need to be answered in order to decide whether the recommendation is likely to have the predicted result Be sure to explain how the answers to these questions would help to evaluate the recommendation

Before following through this business plan the manager should investigate the cause of Rialtorsquos unsuccessful business

The author provides no evidence that the surveyrsquos results are statistically reliable The surveyrsquos sample of 85 percent must be sufficient in size and representative of overall population of the city where Rialto and Apex is serving Lacking evidence of a sufficiently representative sample the author cannot justifiably rely on the survey to draw any conclusion whatsoever The author does not indicate that Apex is indeed currently successful However even if Apex is enjoying success the argument relies on what might be a false analogy between Rialto and Apex In order for Apex to serve as a model that Rialto should emulate the author must assume that all relevant circumstances are essentially the same However this assumption is unwarranted For example the argument overlooks the face that Apex is located in a strategic placemdashbeside a mall where customers can not only watch a movie but also enjoy shopping Therefore simply changing the facility to that of Apex may not lead to success

The author does not mention whether Apex is successful or not Nevertheless even if Apex is currently successful the argument relies on what might be a false analogy between Rialto and Apex In order for Apex to serve as a model that Rialto should emulate the author must assume that all relevant circumstances are essentially the same However this assumption is unwarranted For example the argument overlooks the fact that these two institutions are located in different locations Rialto in downtown and Apex in a mall outside of town Although Apex opened with state-of-the-art facilities the decisive factor in its success could be due to its strategic location of being in a mall People could enjoy both shopping and movies at one location thus they may prefer Apex over Rialto Furthermore the place where people enjoy leisure activities has shifted in the past decades for most cities from downtown to the suburbs Therefore Rialto may not be successful even if it emulates Apexrsquos facilities A better business plan may be relocating Apex to the thriving section of the downtown

Rialto 극장은 지난 50여년간 지역 회관으로써 시내에 위치해 있으면서도 이제 변화를 꾀하지 않으면 문을 닫을

GRE AWA John박 박정어학원

판이다 이 극장은 시외 쇼핑타운에 새로 들어선 Apex 극장의 사례를 본받아야 했다 Apex가 지난해 개업했을 당시 이 극장은 비디오 아케이드 플러쉬 카펫트 바닥과 좌석 그리고 최신 음향시설을 갖추었다 더군다나 최근 조사에서는 응답자의 85 이상이 새로 출시된 영화 입장료가 비싼 탓으로 지난해보다 5배이상의 관람객이 줄어들었다고 나타났다 따라서 Rialto 극장이 줄어들고 있는 관람객을 뺐기지 않고 유지하려면 Apex와 같은 시설들을 갖추어야 할 것이다주장 리알토 극장이 줄어들고 있는 관람객을 뺐기지 않고 유지하려면 Apex와 같은 시설들을 갖추어야 할 것이다

1 조사에서 응답자가 전체를 대표할 수 없다 2 apex 극장이 좋은 시설을 갖추고 있지만 그로 인해 수익이 많이 발생했다는 말이 없으므로 시설투자를

하고도 좋은 결과를 얻을 수 있을지 그 근거가 미흡하다3 좋은 영화가 출시된다면 입장료가 비싸도 영화관에서 꼭 보려고 할 수 있다 4 rialto 가 시설이 아닌 다른 요인에 의해 장사가 안될수도 있다( 우범 지역이라든지)

The following is a recommendation from the business manager of Monarch Books

Since its opening in Collegeville twenty years ago Monarch Books has developed a large customer base due to its reader-friendly atmosphere and wide selection of books on all subjects Last month Book and Bean a combination bookstore and coffee shop announced its intention to open a Collegeville store Monarch Books should open its own in-store cafeacute in the space currently devoted to childrens books Given recent national census data indicating a significant decline in the percentage of the population under age ten sales of childrens books are likely to decline By replacing its childrens books section with a cafeacute Monarch Books can increase profits and ward off competition from Book and Bean

Write a response in which you examine the stated andor unstated assumptions of the argument Be sure to explain how the argument depends on these assumptions and what the implications are for the argument if the assumptions prove unwarranted

The following is a recommendation from the business manager of Monarch Books

Since its opening in Collegeville twenty years ago Monarch Books has developed a large customer base due to its reader-friendly atmosphere and wide selection of books on all subjects Last month Book and Bean a combination bookstore and coffee shop announced its intention to open a Collegeville store Monarch Books should open its own in-store cafeacute in the space currently devoted to childrens books Given recent national census data indicating a significant decline in the percentage of the population under age ten sales of childrens books are likely to decline By replacing its childrens books section with a cafeacute Monarch Books can increase profits and ward off competition from Book and Bean

1 Write a response in which you discuss what questions would need to be answered in order to decide whether the recommendation is likely to have the predicted result Be sure to explain how the answers to these questions would help to evaluate the recommendation

2 Write a response in which you discuss what specific evidence is needed to evaluate the argument and explain how the evidence would weaken or strengthen the argument

No evidence regarding Monarch Bookrsquos successEven if Regal Bookrsquos is successful this may not be attributable to the cafeacute False analogy Emulating may not lead to success Other factors may be involvedInsufficient condition The national census is not enough evidence that childrenrsquos book sales will decline Can

GRE AWA John박 박정어학원

the national census represent the local child populationDid opening a cafeacute boost sales for Regal Books Even assuming Regal is successful by opening a cafeacute this may not be suitable for Monarch which plans to close the childrenrsquos book section to establish a cafe Imprecise language ldquorelatively little spacerdquo how smallThe managerrsquos recommendation contradicts what he says Since Monarch is popular for its wide selection of books closing a selection which targets a major group of readers may hurt Monarchrsquos salesIs this the best way to compete

When Stanley Park first opened it was the largest most heavily used public park in town It is still the largest park but it is no longer heavily used Video cameras mounted in the parks parking lots last month revealed the parks drop in popularity the recordings showed an average of only 50 cars per day In contrast tiny Carlton Park in the heart of the business district is visited by more than 150 people on a typical weekday An obvious difference is that Carlton Park unlike Stanley Park provides ample seating Thus if Stanley Park is ever to be as popular with our citizens as Carlton Park the town will obviously need to provide more benches thereby converting some of the unused open areas into spaces suitable for socializing

Write a response in which you examine the stated andor unstated assumptions of the argument Be sure to explain how the argument depends on these assumptions and what the implications are for the argument if the assumptions prove unwarranted

Stanley 파크가 처음 개장했을 당시 가장 크고 가장 많이 이용되는 공원이었다 아직도 공원중에서는 가장 크지만 이용률은 상당히 떨어졌다 지난달 공원 주차장에 설치해놓은 비디오 카메라를 통해 보면 drop(주차장으로 여겨짐) 이용률이 가장 높았다 수치상으로는 하루 평균 50대의 차량만이 이용하였다 반면 직장 중심거리에 위치한 작은 규모의 Carlton 파크는 주당 무려 150여명 이상이 이용하고 있다 Stanley 파크와는 달리 Carlton 파크에는 의자가 있다는 것이 가장 뚜렷한 차이점이다 따라서 Stanley 파크가 Carlton 파크처럼 시민들이 자주 이용하는 공원이 되기 위해서는 벤치를 설치할 필요가 있으며 이렇게 사용되지 않는 일부 공간을 활용해서 사교를 위한 공간으로 바꾸어야 한다 ===gtdrop 에 대한 첨부사항 (영영사전내용입니다)---- a place or central depository to which something (as mail money or stolen property) is brought for distribution or transmission also the act of depositing something at such a place dropgt

주장 if Stanley Park is ever to be as popular with our citizens as is Carlton Park the town will obviously need to provide more benches thereby converting some of the unused open areas into spaces suitable for socializing1 조사가 언제 이루어진 것인가 조사가 언제 실시되었느냐에 따라 결과가 다를 수있다 현재는 다시 스탠리 파크가 늘어났었을 수 있다 2 벤치를 많이 설치했다고 해서 많은 관광객이 오지 않을수 있다(사람들이 벤치나 사교 공간을 원한다는 어떠한 자료도 없다)3스탠리 파크 주변에 교통 상황이 악화가 되었거나 칼튼 파크에서 문화행사등을 많이 가져서 이용객이 줄어든것일 수도 있다 4 칼튼 파크가 중심지에 있어서 접근성이 좋을수 있다5 조사가 같은 시간을 기준으로 한게 아니다(하나는 주중이고 하나는 주말이다)6사람의 수와 차의 대수를 같은것으로 비교할수 없다 (차안에 몇 명이 타고 있는지 모르고 대중교통을 이용해서 왔을수도 있다)

Page 9: GRE writing argument brain storm

GRE AWA John박 박정어학원

ldquoBatavia 지역의 농림부의 보고에 따르면 전국적으로 낙농업자의 숫자가 과거 10 년전에 비해 25나 증가했다고 합니다 그러나 동기간 동안 이 지역에 있는 Excello Food Market 에서 우유의 가격은 갤런당 15 불에서 3 불로 증가했습니다 증가된 우유 공급량에 맞춘 낙농업자들의 수익률 증대를 보장하기 위해서는 주 정부가 소매 공급가에 대한 규제를 해야 합니다 소비자에게 보다 저렴한 가격에 안정된 물량을 공급할 수 있도록 하기 위해서는 이러한 조치가 필요합니다

결론 the Batavia government should begin to regulate retail milk prices

반박 ( Excello Food Market doesnrsquot reflect the whole of Batavia) 우유값 증가가 단순히 inflation 을 따라간것일수도 있다 가격상승이 수익을 보장하지 않는다( 원자재 상승등으로 인해서)우유값 규제만이 최선의 방법이 아니다( 물량 조절)낙농업자의 수익보호도 필요하다

A recent sales study indicates that consumption of seafood dishes in Bay City restaurants has increased by 30 percent during the past five years Yet there are no currently operating city restaurants whose specialty is seafood Moreover the majority of families in Bay City are two-income families and a nationwide study has shown that such families eat significantly fewer home-cooked meals than they did a decade ago but at the same time express more concern about healthful eating Therefore the new Captain Seafood restaurant that specializes in seafood should be quite popular and profitable

Write a response in which you discuss what questions would need to be addressed in order to decide whether the conclusion and the argument on which it is based are reasonable Be sure to explain how the answers to the questions would help to evaluate the conclusion

BAY CITY The name implies that this is a port city which would have traditionally consumed seafood 30--what type of seafood Unprocessed or processed cans of tuna and sardines If the latter increased Captain Seafood may not profit 30--natural inflation Are the non-seafood restaurants selling seafood If yes this may suffice since patrons would not easily dine at an unfamiliar restaurant Nationwide studyrsquos representativeness Desire does not lead to action

Nationwide study applied to Bay City Eating fewer home cooked meals than they did a decade agoHealthy food The author needs to show that 30 -gt population increaseOther foods may be more popularCurrent number of restaurants that serve seafood might sufficeWhat kind of healthy food are they interested in Surely not just seafoodThe locals may still patronize the current restaurants that serve seafood dishes

lt national-gt local에 적용 local-gtnational에 적용gt1) The author assumes that the nationwide trend reflects the general trend upon which the argument relies

Yet the author fails to provide evidence to substantiate this crucial assumption The nationwide trend of two income families eating fewer home-cooked and concern for health my not reflect the lifestyle and interests of Bay City citizens Therefore to make his argument stronger the author needs to provide evidence that two income families of Bay City maintains the nationwide trend

GRE AWA John박 박정어학원

최근의 매상에 관한 조사에서 Bay 시에 있는 레스토랑의 해산물 요리의 소비가 지난 5년동안 30 증가했다고 보고되었다 그러나 현재 해산물을 전문으로 취급하고 있는 레스토랑이 없다 더군다나 대다수의 가정이 맞벌이 가정인데다가 전국 조사에서도 나와있듯이 이러한 맞벌이 가정은 10년전의 가정에서 했듯이 집에서 식사를 챙겨먹는 것이 급격하게 줄어 들었고 동시에 건강식과 관련한 지출이 늘고 있다 따라서 해산물 전문 레스토랑이 아주 인기를 끌것이고 그만큼 수익도 많을 것이다

1 해산물이 건강식인지에 대한 언급이 없다2 외식하는데 해산물 요리만 먹지 않을 것이다 집에서 챙겨먹지 않는다고 해산물을 먹는다고 할 수 없다3 현재 해산물 취급하는 식당이 현재까지 없다고 해서 계속 없을 수만은 없다 수익이 만약 늘어난다면

Bay시 주변의 도시의 레스토랑이 체인점을 내거나 새로운 식당이 생길 수 있고 오히려 수익이 줄어들 수도 있다

Scientists studying historical weather patterns have discovered that in the mid-sixth century Earth suddenly became significantly cooler Although few historical records survive from that time some accounts found both in Asia and Europe mention a dimming of the sun and extremely cold temperatures Either a huge volcanic eruption or a large meteorite colliding with Earth could have created a large dust cloud throughout Earths atmosphere that would have been capable of blocking enough sunlight to lower global temperatures significantly A large meteorite collision however would probably create a sudden bright flash of light and no extant historical records of the time mention such a flash Some surviving Asian historical records of the time however mention a loud boom that would be consistent with a volcanic eruption Therefore the cooling was probably caused by a volcanic eruption

Write a response in which you discuss what questions would need to be addressed in order to decide whether the conclusion and the argument on which it is based are reasonable Be sure to explain how the answers to the questions would help to evaluate the conclusion

Historical records may not be enough to explain the global cooling phenomenonThere may be reasons other than the two hypothesis to explain the global coolingA volcanic eruption big enough to produce dust clouds that envelop the earth to cause global cooling would have countless recordsThe author argues that a large meteorite collision is not a feasible explanation for the global cooling because there are no historical records that indicate a flash The authorrsquos logic is flawed in eliminating the meteorite hypothesis by claiming that there was no historical record of a bright flash of light According to common sense a meteorite collision big enough to produce dust that could envelop the earth would result in catastrophe even to the extinction of major species of animals

Loud boom would be insufficient evidence to conclude it was a volcanic eruption If could hear a loud boom there should be records of a volcanic eruption because the author claims that the dust cloud from this gigantic eruption was significant enough to cause global coolingCooling could have been caused by factors besides volcanic eruption and meteor collisionThe absence of historical records that indicate a bright flash of light Collision could have occurred at a place where there no witnesses Could have fell out in the seaBoom might have been caused by things other than a volcanic eruptionHistorical records from Asia and Europe may not be applicable to earth as a whole

과거의 날씨 주기를 연구하는 과학자들은 6세기 중엽 지구가 갑자기 혹한기가 되었던 것을 알게되었다 이 당시의 몇몇 기록들이 아직도 남아있긴 하지만 아시아와 유럽에서 발견되는 몇가지 자료에서 태양 빛의 수축과 그에따른 엄청난 추위가 있었음을 언급하고 있다 거대한 화산 폭발이나 거대 운석의 지구 충돌중 어느것에

GRE AWA John박 박정어학원

의해서든 엄청난 먼지구름을 형성시켜 지구전체에 태양빛을 차단해서 지구의 기온을 뚝 떨어뜨릴수 있을 수도 있다 그러나 이러한 운석 충돌은 순간 섬광을 발산시키게 될 것인데 이 시기의 어느 기록에서도 이러한 섬광은 언급되지 않고 있다 그러나 이 시기에 남아 있는 일부 아시아의 기록문서에서는 연속적으로 화산폭발음일수 있는 엄청난 폭발음이 있었음을 기록하고 있다 따라서 이러한 혹한은 아마도 화산 폭발에 의한 것이었을 것이다

결론 the cooling was probably caused by a volcanic eruption

반박 자료의 부족( 몇몇 자료가지고 그 당시의 기후를 예상하기에는 무리가 있다)다른 원인에 의해서 기후가 떨어졌을수도 있다 (빙하기 다른 기상원인)기록되지 않았다고 해서 그 사실이 없었던 것은 아니다(기록은 했으나 자료가 없어졌을수도 있고 모든 사실이 전부다 기록되지는 않는다 )기록된 폭발음이 꼭 확산 폭발음이 아닐수도 있다( 화산 폭발음이 너무 먼거리여서 들리지 않을수도 있고 다른 소리가 기록된 것이였을수도 있다)부족한 자료를 신빙할수 있는가 기록의 소멸도 예상 할수 있다 실제로 운석이 떨어졌는데 안들렸을수 있다

The following appeared in a memo from the president of Bower Builders a company that constructs new homes

A nationwide survey reveals that the two most-desired home features are a large family room and a large well-appointed kitchen A number of homes in our area built by our competitor Domus Construction have such features and have sold much faster and at significantly higher prices than the national average To boost sales and profits we should increase the size of the family rooms and kitchens in all the homes we build and should make state-of-the-art kitchens a standard feature Moreover our larger family rooms and kitchens can come at the expense of the dining room since many of our recent buyers say they do not need a separate dining room for family meals

Write a response in which you examine the stated andor unstated assumptions of the argument Be sure to explain how the argument depends on these assumptions and what the implications are for the argument if the assumptions prove unwarranted

The presidentrsquos first unstated but apparent assumption is that a nationwide survey can be applied to local areas There is no guarantee that the area in which Bower Builders builds houses will follow the national consumer trend Since he only mentions the overall result of the nationwide survey it is difficult to determine more specific conditions such whether people in urban and rural areas have the same housing preferences Such uncertainty is further exacerbated by the nature of the survey question itselfmdash it does not deal with current trends in actual home purchases but with desired home features The naiumlve assumption that consumer desire will directly result in consumer action underlies the presidentrsquos interpretation and application of the survey results to his company plan Not everyone who wishes for a large family room and kitchen will or can actually buy a house with those features mainly due to financial reasons Furthermore the president also assumes for no evident reason that consumers will not hesitate to purchase houses with state-of-the-art kitchens as a standard rather than optional feature when it is clear that such a feature will raise the overall cost significantly He continues to blunder in his mistaken assumptions about consumer behavior in assuming that the taste of recent buyers can represent the concern of future prospective buyers The fact that recent buyers have claimed no need for separate dining rooms does not mean future buyers will feel the same way as well

The president of Bower Builders recommends that in order to raise company profits the company should build houses with large family rooms and large state-of-the-art kitchens He uses the results of a nationwide survey on desired home features and the example of Bower Buildersrsquo competitor Domus Construction to support his plan His memo manifests several assumptions about surveys consumer behavior and Domus Constructionrsquos houses that do not necessarily bolsterbuttress his argument

GRE AWA John박 박정어학원

The writer assumes 1 the nationwide survey results can be applied to the local area (Desire cannot reflect consumer trend) 2 Domus Construction was profitable because they had such features (The writer should investigate what other features the DC provides and the actual number of homes sold) 3 State-of-the-art kitchens will sell well (no evidence to buttress his assumption furthermore they would need to spend more money which would increase the price of the housesmdashtheir market will be limited to the affluent) 4 The recent buyers represent the concern of most buyersmdashthat they are ok with a house that is without a dining room

Nationwide survey might not be applicable to this regionDomus Construction could have other superior qualities than Bower Builders Ex DesignSelling fast doesnrsquot indicate quantitySmall yards complaints could be voiced in the future

First the author concludes that a nationwide survey reveals that the two most desired home features are a bathroom with a whirlpool tub and a large kitchen However this nationwide survey may not reflect the trends of the customers that Bower Builders target The author assumes that the nationwide trend reflects customer trends The national study would lead support only if the nationwide trend reflect the home-purchasing trends The nationwide trend may just be a trend of desire which does not result in the act of purchasing However the author does not provide credible evidence that this is the case

전국 조사에서 가장 인기있는 집안 구조 2가지는 훨풀 튜브가 마련된 욕실과 커다른 부엌으로 조사되었다 경쟁업체 Domus Construction이 신축한 완공예정인 주택은 이러한 욕실을 갖추고 있어서 분양이 조기에 이루어지고 있고 가격도 평균치보다 상당히 웃돌고 있다 우리도 집을 많이 팔도 그만큼 이윤을 많이 남기려면 신규 주택에는 위의 2가지 사항을 갖추어야 한다 최근 우리가 지은 주택 수요자들이 적은 평수에 대해서는 별다른 불만사항이 없었기 때문에 앞으로 평수를 더 줄여서 이익을 극대화 할 수 있을 것이다

주장 We should include whirlpool tubs and a large kitchen

1 경쟁업체 Domus Construction이 신축한 완공예정인 주택은 이러한 욕실을 갖추고 있어서라기 보다는 위치가 좋거나 다른 마감재(construction material)나 인테리어가 뛰어나서 분양이 조기에 이루어진것이라고 볼 수 있다 2 전국조사가 건물이 지어지는 지역에 항상 적용되리라고 볼 수 없다 3 평수를 줄이는 것에 불만이 없었다는 것은 editor의 견해일 수 있다 사람들이 표현하지 않은 것일 수도 있고 다른 장점이 많아서 그런 단점을 드러내지 않은 것일 수 있기 때문에 속단할 수 없다 4 다른것을 함으로써 더 잘 팔릴수도 있다

2011 7 19 화요일 GRE AWA 실전반이OO

Argument 94

The president of Bower Builders contends recommends that in order to raise company profits the company should build houses with large family rooms and large state-of-the-art kitchens He uses the results of a nationwide survey on desired home features and the example of Bower Buildersrsquo competitor Domus Construction to support his plan His memo manifests several assumptions about surveys consumer behavior and Domus Constructionrsquos houses that do not

GRE AWA John박 박정어학원

necessarily bolsterbuttress his argumentThe presidentrsquos first unstated but apparent assumption is that a nationwide survey can be

applied to local areas There is no guarantee that the area in which Bower Builders builds houses will follow the national consumer trend Since he only mentions the overall result of the nationwide survey it is difficult to determine more specific conditions such whether people in urban and rural areas have the same housing preferences Such uncertainty is further exacerbated by the nature of the survey question itselfmdash it does not deal with current trends in actual home purchases but with desired home features The naiumlve assumption that consumer desire will directly result in consumer action underlies the presidentrsquos interpretation and application of the survey results to his company plan Not everyone who wishes for a large family room and kitchen will or can actually buy a house with those features mainly due to financial reasons Furthermore the president also assumes for no evident reason that consumers will not hesitate to purchase houses with state-of-the-art kitchens as a standard rather than optional feature when it is clear that such a feature will raise the overall cost significantly He continues to blunder in his mistaken assumptions about consumer behavior in assuming that the taste of recent buyers can represent the concern of future prospective buyers The fact that recent buyers have claimed no need for separate dining rooms does not mean future buyers will feel the same way as well

In addition the president finds a real-life actualization of the nationwide survey results in the recent sales of Domus Construction However he easily assumes that large family rooms and kitchens are the only reasons the houses of Domus Construction sell well He does not take into account other features and selling points of the competitorrsquos houses There may well be other explanations for its houses selling more quickly and expensively including additional home features as well as external factors such as proximity to better schools or superior financial solvency of its clientele If Bower Builders merely added larger family rooms and kitchens without taking into consideration the other factors they may lose rather than gain profits

Clearly the presidentrsquos assertion that Bower Builders make houses with large family rooms and high-tech kitchens at the expense of dining rooms rests on a number of assumptions that are ill-informed and naiumlve If Bower Builders undertakes the proposed plan without further research into local consumer desires purchasing trends and the marketing and sales of competing companies the company will risk losing money by building big new houses that people cannot afford to or will not wish to buy

The following appeared in a letter to the editor of a journal on environmental issues

Over the past year the Crust Copper Company (CCC) has purchased over 10000 square miles of land in the tropical nation of West Fredonia Mining copper on this land will inevitably result in pollution and since West Fredonia is the home of several endangered animal species in environmental disaster But such disasters can be prevented if consumers simply refuse to purchase products that are made with CCCs copper unless the company abandons its mining plans

Write a response in which you examine the stated andor unstated assumptions of the argument Be sure to explain how the argument depends on these assumptions and what the implications are for the argument if the assumptions prove unwarranted

The writer assumes 1 The writer is trying to avoid an inevitability 2 Mining copper will result pollution (they could make preventative measures) 3 The writer may be too late from stopping CCC from developing the area into a copper mine 4 Since mining is an underground enterprise the surface may not be affected that much therefore endangered species may not be affected 4 Consumers wonrsquot buy CCC products if the journal publishes a negative review about CCC (How many readers) CCC could a company that has ties with many IT companies and industries in that their copper is almost ubiquitous in various products

GRE AWA John박 박정어학원

지난 한해동안 CCC(Consolidated Copper 회사)는 서부 플로리다의 열대 지역에 1백만 마일이 넘는 땅을 사들였다 이 곳에서의 채광 활동은 서부 플로리다가 몇몇 멸종 위기에 처한 동물의 서식지이기 때문에 분명 오염과 환경파괴를 가져올 것이다 그러나 이러한 파괴는 CCC 회사가 채광을 포기할 때까지 이 회사가 채굴한 구리로 제조된 제품을 구매하지 않으면 막을 수 있을 것이다

결론 such disaster can be prevented if consumers simply refuse to purchase products that are made with CCCs copper until the company abandons its mining plans

1 구리로 제조된 물건이 생활에 필수적인 것이 많은 만큼 불매가 쉽지 않을 수 있다(전선이나 각종 전자제품에 필수적으로 들어가기 때문에)

2 불매를 유도한다고 해서 소비자들이 구매를 안하는 것은 아니다3 적절한 채굴로 환경파괴를 가져 오지 않을 수 있다 (땅속에 있는 물질을 채굴하는 만큼 생물에 영향을 안

미칠 수도 있다)4 이미 채굴이 다 끝나서 더 이상의 채굴이 없을 수도 있다 5 땅을 구입한다고 해서 채광하는건 아니다 (다른 용도로 샀을수도 있다)

The following is a letter to the editor of an environmental magazine

In 1975 a wildlife census found that there were seven species of amphibians in Xanadu National Park with abundant numbers of each species However in 2002 only four species of amphibians were observed in the park and the numbers of each species were drastically reduced There has been a substantial decline in the numbers of amphibians worldwide and global pollution of water and air is clearly implicated The decline of amphibians in Xanadu National Park however almost certainly has a different cause in 1975 troutmdashwhich are known to eat amphibian eggsmdashwere introduced into the park

Write a response in which you discuss what specific evidence is needed to evaluate the argument and explain how the evidence would weaken or strengthen the argument

Evidence needed the identity of the sender and the census taker of rsquo75 and lsquo02mdasha scientist or an environmentalist credibility issue census methodology vs mere observationmdashthe absence of evidence is not an evidence of absence the season of when the census was taken time-shiftmdashconditions may have changed worldwide decline may include Xanadu other species of predators that prey on amphibians because trout is only one species that prey on amphibians the number of troutmdashhave they increased significantly since rsquo75 First the author needs to be more overt about the credibility of the census and observation There were to accounts that notes the population of amphibiansmdashthe first a census and the second an observation The author needs to bolster his conclusion with the evidence that indicate that the census and second observation were done using scientific methodologies This could either could strengthen or weaken his claim In addition he

GRE AWA John박 박정어학원

needs to provide the specific season of when the census and observation occurred In this letter the writer is informing an editor of an environmental magazine that the number of amphibians was greatly reduced since 1975 and he points out the introduction of trout as the only reason for the decline However the author fails to provide crucial evidences that could strengthen or weaken his conclusion

The following appeared in a memorandum from the president of Hyper-Go Toy Company

Last year sales of our Fierce Fighter toy airplane declined sharply even though the toy had been a top seller for three years Our customer surveys show that parents are now more worried about youthful violence and are concerned about better education for their children Therefore to maintain profits we should discontinue all our action toys and focus exclusively on a new line of educational toys Several other toy companies have already begun marketing educational toys and report sales increases last year of 200 percent And since the average family income is growing sales of new Hyper-Go toys should also increase

Write a response in which you discuss what specific evidence is needed to evaluate the argument and explain how the evidence would weaken or strengthen the argument

First the president indicates that the sales of Fierce Fighter toy airplane declined sharply However he fails to consider the fact that toys are a fad Since FFT enjoyed a three year success it may be natural that the trend would subsideSecond Customer survey is this representative of most toy consumers This trend is not newThird other companies may have profited not through educational but other toys Or educational toy profit may be small in proportion to their sale of other toys 200Parents are not the customers companies should concentrate on what the children want to increase profitLastly parents worry about youthful violence and concern for better education are not new trends

The presidentrsquos decision to discontinue all action toys and focus only on educational toys is too extreme If this decision is taken into effect its procedure may be an onerous task because it would require major shifts in human resources and company image Therefore further data should be considered before following up on this decision

우리 회사에서 지난 3년간 최고 매출을 일으켰던 Fierce Fighter 장난감 비행기가 지난해에는 매출이 급격히 떨어졌다 자체 고객 조사에서는 부모들이 현재 청소년 폭력에 걱정을 하고 있어서 아이들의 양질 교육에 더 관심을 가지고 있는 것으로 나타났다 따라서 회사의 수익을 유지하기 위해서는 모든 자사 전투용 장난감 생산을 중단하고 오로지 교육적인 장난감 생산에 집중해야 한다 몇몇 여타 장난감 회사들도 이미 교육용 장난감 마케팅을 시작해서 지난해에는 200의 매출신장을 가져왔다고 한다 그리고 평균 가계 수입이 점점 늘고 있기 때문에 신형 Hyper-Go 장난감의 매출도 늘어날 것이다

3그룹 Time-shift Error

Woven baskets characterized by a particular distinctive pattern have previously been found only in the immediate vicinity of the prehistoric village of Palea and therefore were believed to have been unique to the Palean people Recently however archaeologists discovered such a Palean basket in Lithos an ancient village across the Brim River from Palea The Brim River is very deep and broad and so the ancient Paleans could only have crossed it by boat but there is no evidence that the Paleans had boats And boats capable of carrying groups of people and cargo were not developed until thousands of years after the Palean people disappeared Moreover Paleans would have had no need to cross the rivermdashthe woods around Palea are full of nuts berries and small game It follows that the so-called Palean baskets were not unique to Palea

GRE AWA John박 박정어학원

Write a response in which you discuss what specific evidence is needed to evaluate the argument and explain how the evidence would weaken or strengthen the argument

RefutationPrehistoric time shift-gtbrim river could have been narrow and shallow or it might have not existed Indigenous patterns may exist in other disconnected remote placesNuts berries small game(hunting) may not have existed or the author should be proved these existed at that time Abundance of resources doesnrsquot support the reason for seclusionBoats not yet found baskets may have been carried across by the river current without the help of a boat Lithos might have crossed the river for commercial purposeOne Palean basket does not substantiate the authorrsquos claimThe absence of evidence is not an evidence of absence

The author assumes without justification that present conditions are the same as at the prehistoric era The author unfairly infers from the presence of Brim River which exist today that it would have existed in the past However the author fails to offer any evidence to substantiate this inference It is very likely that the Brim River might not have existed in prehistoric times or if it did exist may have been shallow and narrow enough for the Paleans to easily cross Any of these scenarios if true would serve to undermine the claim thathelliphelliphelliphelliphellip

First the author claims that the Brim River was very deep and broad so the Paleans could not have crossed it However the author fails to offer any evidence to substantiate that this was true in the prehistoric time of the Paleans For all we know the Brim River might not have existed in the prehistoric era or if it did exist could have been a shallow and narrow river For example scientists believe that thousands of years ago an ice-bridge existed on the Bering Sea connecting Eurasia to what is now North America to explain how the Eskimos and the Asian inhabitants of America came to migrate all over the Americas In this example the absence of evidence is not an evidence of absence Therefore to sufficiently support his claim the author needs to substantiate the fact that the Brim River really did exist and was broad and wide in the Prehistoric Era proving that the Paleans could not have influenced or traded with other groups of people

First the author claims that the Brim River was very deep and broad so the Paleans could not have crossed it However the author fails to offer any evidence to substantiate that this was true in the prehistoric times of the Paleans For all we know the Brim River might not have existed in the prehistoric era or if it did exist could have been a shallow and narrow river For example rivers are created by natural erosion over thousands of years Niagara Falls carved its way from the mouth of Lake Ottawa and created a long river Likewise the current Brim Riverrsquos physical features may not have been wide and deep Therefore to sufficiently support his claim the author needs to substantiate the fact that the Brim River really did exist and was broad and wide in the Prehistoric Era proving that the Paleans could not have influenced or traded with other groups of people

예전에는 실로짠 특이한 무늬 바구니가 Palea의 선사시대 지역의 인근마을에서만 발견되어왔기 때문에 Palea 마을 사람들의 특징이라고 여겨졌었다 그러나 최근들어 고고학자들이 Lithos지역에서 Palean 바구니를 발견하였는데 그 당시 지역은 Brim 강을 가로질러 Palea까지 닿아있었다 이 강은 수심이 아주 깊고 강폭이 넓었으며 때문에 고대의 Palea인들은 배를 이용해서 강을 건널수 있었을 것이다 그러나 이들이 배를 가지고 있었다는 증거는 발견되지 않고 있다 더군다나 이들이 멸명한 이후 수천년이 지난뒤에도 수많은 물자와 사람을 실어 나를수 있는 용적을 가진 배는 개발되지 않았다 이와더불어 Palea인들은 강을 건널필요가 없었는데 그것은 너트나무 장과열매 그리고 작은 사냥감들이 주변숲에 풍부했기때문이다 따라서 Palean 바구니라고 하는 것도 Palea인들만의 전유물이 아니라는 결론을 얻을 수 있다

결론 if follows that the so-called Palean baskets were not unique to Palea이번문제는 굿이 causal Bad analogy 로 구분해서 찾기가 힘드내요 배를 발견했다는 증거가 없는것이다 (앞으로도 발견될수 있음)

GRE AWA John박 박정어학원

계절의 영향으로 겨울에 얼음이 두껍게 언다든지 여름에 가뭄으로 인해서 건널수 있다물자가 풍부한 것이 이동하지 않을 조건이 아니다 다른 것에 의해서 이동가능(의약품등)

Thirteen years ago researchers studied a group of 25 infants who showed signs of mild distress when exposed to unfamiliar stimuli such as an unusual odor or a tape recording of an unknown voice They discovered that these infants were more likely than other infants to have been conceived in early autumn a time when their mothers production of melatonin hormone known to affect some brain functions would naturally increase in response to decreased daylight In a follow-up study conducted earlier this year more than half of these children now teenagers who had shown signs of distress identified themselves as shy Clearly increased levels of melatonin before birth cause shyness during infancy and this shyness continues into later life

Write a response in which you examine the stated andor unstated assumptions of the argument Be sure to explain how the argument depends on these assumptions and what the implications are for the argument if the assumptions prove unwarranted

Any baby exposed to unpleasant stimuli would react in such wayFirst of all the author states 25 infants as his evidence However this research sample is too small to prove his claimSecond the author states that 25 infants were conceived in early autumn which he claims lead to a shy disposition However this is faulty evidenceThird the research study was a long term study done in the span of 13 years However the author only writes about the initial and final stages of the study and leaves out evidences of what could have happened during the 13 years which could be more evidential factors of influenceFourth neither the infantsrsquo genetic predisposition nor their environment were taken into accountFinally the author concludes that his shyness continues into later life (Other factors could alter this disposition epigenetic theory)

13 년전 학자들은 25명의 유아를 대상으로 이상한 냄새나 특이한 소리를 녹음한 테잎등으로 낯선 자극을 주었을때 보이는 미미한 압박감 증상을 조사하였다 이들은 성숙기가 막 지났을 즈음에 보통의 유아들이 비슷한 증상을 보이는 정도 보다는 다소 민감한 반응을 보였는데 이 시기는 아이의 엄마가 뇌의 일부 기능에 영향을 미치는 것으로 알려진 멜라토니아 호르몬을 생산하는 시기로써 이 호르몬은 낯 시간이 짧을때 자연적으로 증가할 수도 있다 금년초에 실시된 추가연구에서 현재 10대로 성장한 당시 조사대상의 절반 이상의 아이들이 부끄럼을 잘타는 것으로 여기고 있었다 따라서 분명한 것은 출산전 멜라토닌 수치의 증가가 유아기에 수줍음 등의 영향을 미치게 되며 이러한 영향이 성장후에도 작용한다는 것이다

주장 Clearly increased levels of melatonin before birth cause shyness during infancy and this shyness continues into later life

1 25명의 아기로 결론 내리기에 샘플이 작다2 과학적 사실들에 대한 명확한 근거가 엇음3 다른 영향을 간과했다(다른 호르몬에 의한 영향 후천적인 성격형성의 영향)

GRE AWA John박 박정어학원

The following is a letter to the editor of the Atticus City newspaper

Former Mayor Durant owes an apology to the city of Atticus Both the damage to the River Bridge which connects Atticus to Hartley and the traffic problems we have long experienced on the bridge were actually caused 20 years ago by Durant After all he is the one who approved the construction of the bridge If he had approved a wider and better-designed bridge on which approximately the same amount of public money would have been spent none of the damage or problems would have occurred Instead the River Bridge has deteriorated far more rapidly over the past 20 years than has the much longer Derby Bridge up the river Even though the winters have been severe in the past several years this is no excuse for the negligence and wastefulness of Durant

Write a response in which you discuss what questions would need to be answered in order to decide whether the recommendation is likely to have the predicted result Be sure to explain how the answers to these questions would help to evaluate the recommendation

전임 시장인 Durant 씨는 Atticus 시에 대해 사과할 의무가 있습니다 Atticus와 Hartley를 잇는 River Bridge 교량에 대한 피해와 이 교량에서 오랫동안 주민들이 겪어오고 있는 교통 혼잡 문제들은 실제로 20년 전부터 시작된 것이었습니다 결정적으로 그가 교량 공사를 허가했던 바로 그 장본인입니다 당시 비슷한 공사비용으로 폭이 더 넓고 튼튼하게 설계된 교량을 허가했다면 이러한 문제나 피해는 발생하지 않았을 겁니다 더군다나 이 다리는 지난 20년 동안 상류에 건설된 훨씬 오래된 Derby 다리보다도 빠르게 부식되어 갔습니다 지난 수년동안 심지어 혹한이 있었다 하더라도 이러한 태만과 국고 손실에 대한 책임을 회피할 길이 없는 것입니다

결론 Former Mayor Durant owes an apology to the city of Atticus

1 디자인이 문제가 아닐수 있다 (디자인은 좋았으나 건설과정에 문제가 있었을 수 있다)2 그 당시의 시예산이 적어서 더 큰 다리를 짓기가 불가능했을 수도 있다3 교통량이 많거나 다른 상황으로 인해서 부식이 빨리 됐을 수 있다4 그 당시에는 최선의 선택이였지만 갑자기 변한 상황에 의해서 이런 문제점들이 발생했을 수 있다

GRE AWA John박 박정어학원

4그룹 거짓인과관계 오류 (False Cause) 빈출

Fifteen years ago Omega University implemented a new procedure that encouraged students to evaluate the teaching effectiveness of all their professors Since that time Omega professors have begun to assign higher grades in their classes and overall student grade averages at Omega have risen by 30 percent Potential employers looking at this dramatic rise in grades believe that grades at Omega are inflated and do not accurately reflect student achievement as a result Omega graduates have not been as successful at getting jobs as have graduates from nearby Alpha University To enable its graduates to secure better jobs Omega University should terminate student evaluation of professors

Write a response in which you discuss what specific evidence is needed to evaluate the argument and explain how the evidence would weaken or strengthen the argument

Omega professor evaluation implemented 15 years ago =gt Omega prof assign higher grades 30Employers believe therersquos grade inflation

Thus unsuccessful employment than AlphaTherefore to secure jobs Omega should end evaluating profs

Specific evidence neededRelationship between higher grades and evaluationRelationship between GPA and unsuccessful employmentAlpharsquos education could just be better than OmegaldquoFifteen years agordquo is a long time other factors could have influenced Why is the inflation a problem just now How much is Alpha better Is the comparison just How much gap is thereOmegarsquos student could just be doing better in their studiesComparison to other universities다른 대안 없나hellip Could Omega alleviate the employment problem by implementing a different procedure or program

15 년전 우리 대학은 학생들로 하여금 교수평가를 하도록 한 새로운 조치를 시행했었습니다 이후 교수들은 자신의 학과 학생들에게 높은 학점을 주었으며 그에따라 학생들의 전체 평점이 30나 올랐습니다 외부의 기업체들은 분명 점수가 지나치게 부풀려졌다고 믿고 있습니다 결국 본 대학 졸업생들이 인근 Alpha 대학의 졸업자들보다 구직률이 떨어지는 이유를 잘 보여주고 있는 것입니다 이를 해결하기 위해 이제부터는 학생들에 의한 교수평가제를 중단해야 합니다

결론 Omega University should now terminate student evaluation of professors

반박 교수 평가와 학점 인플레의 연관성이 적다( 교수 평가를 먼저하고 학점을 나중에 매길수도 있다)채용기준에 성적만 있는게 아니다 학업성취의 결과 일수도 있다 Alpha 가 원래 유능했다 Alpha 의 교육내용이 좋았다

GRE AWA John박 박정어학원

In this memo the dean of Omega University(OU) recommends OU to terminate professor evaluation to secure better jobs for the students To support this recommendation the dean offers several reasons However this argument contains several logical flaws which render it unconvincing

A threshold problem with the argument involves the voluntary nature of the evaluationprocedure The dean provides no evidence about the number or percentage of Omegastudents who participate in the procedure Lacking such evidence it is entirely possible thatthose numbers are insignificant in which case terminating the procedure is unlikely to haveany effect on the grade average of Omega students or their success in getting jobs aftergraduationThe argument also assumes unfairly that the grade-average increase is the result of theevaluation procedure--rather than some other phenomenon The dean ignores a host of otherpossible explanations for the increase--such as a trend at Omega toward higher admissionstandards or higher quality instruction or facilities Without ruling out all other possibleexplanations for the grade-average increase the dean cannot convince me that by terminatingthe evaluation procedure Omega would curb its perceived grade inflation let alone help itsgraduates get jobsEven if the evaluation procedure has resulted in grade inflation at Omega the deans claimthat grade inflation explains why Omega graduates are less successful than Alpha graduatesin getting jobs is unjustified The dean overlooks a myriad of other possible reasons forOmegas comparatively poor job-placement record Perhaps Omegas career services areinadequate or perhaps Omegas curriculum does not prepare students for the job market aseffectively as Alphas In short without accounting for other factors that might contribute toOmega graduates comparative lack of success in getting jobs the dean cannot justify theclaim that if Omega curbs its grade inflation employers will be more likely to hire OmegagraduatesFinally even if the dean can substantiate all of the foregoing assumptions the deansassertion that Omega must terminate its evaluation procedure to enable its graduates to findbetter jobs is still unwarranted in two respects First the dean ignores other possible ways bywhich Omega can increase its job-placement record--for example by improving its publicrelations or career-counseling services Second the dean unfairly equates more jobs withbetter jobs In other words even if more Omega graduates are able to find jobs as a result ofthe deans recommended course of action the kinds of jobs Omega graduates find would notnecessarily be better onesIn sum the deans argument is unpersuasive as it stands To strengthen it the dean mustprovide better evidence that the increase in grade average is attributable to Omegasprofessor-evaluation procedure and that the end result is a perception on the part ofemployers that Omega graduates are less qualified for jobs than Alpha graduates To betterassess the argument I would need to analyze 15-year trends in (l) the percentage of Omegastudents participating in the evaluation procedure (2) Omegas admission standards andquality of education and (3) Omegas emphasis on job training and career preparation I wouldalso need to know what other means are available to Omega for enabling its graduates to findbetter jobs

GRE AWA John박 박정어학원

The following appeared in a memo from a vice president of Quiot Manufacturing

During the past year Quiot Manufacturing had 30 percent more on-the-job accidents than at the nearby Panoply Industries plant where the work shifts are one hour shorter than ours Experts say that significant contributing factors in many on-the-job accidents are fatigue and sleep deprivation among workers Therefore to reduce the number of on-the-job accidents at Quiot and thereby increase productivity we should shorten each of our three work shifts by one hour so that employees will get adequate amounts of sleep

Write a response in which you examine the stated andor unstated assumptions of the argument Be sure to explain how the argument depends on these assumptions and what the implications are for the argument if the assumptions prove unwarranted

The following appeared in a memo from a vice president of Alta Manufacturing

During the past year Alta Manufacturing had thirty percent more on-the-job accidents than nearby Panoply Industries where the work shifts are one hour shorter than ours Experts believe that a significant contributing factor in many accidents is fatigue caused by sleep deprivation among workers Therefore to reduce the number of on-the-job accidents at Alta we recommend shortening each of our three work shifts by one hour If we do this our employees will get adequate amounts of sleep

Write a response in which you discuss what questions would need to be answered in order to decide whether the recommendation and the argument on which it is based are reasonable Be sure to explain how the answers to these questions would help to evaluate the recommendation

The following appeared in a memo from the vice president of Butler Manufacturing

During the past year workers at Butler Manufacturing reported 30 percent more on-the-job accidents than workers at nearby Panoply Industries where the work shifts are one hour shorter than ours A recent government study reports that fatigue and sleep deprivation among workers are significant contributing factors in many on-the-job accidents If we shorten each of our work shifts by one hour we can improve Butler Manufacturings safety record by ensuring that our employees are adequately rested

1 Write a response in which you discuss what specific evidence is needed to evaluate the argument and explain how the evidence would weaken or strengthen the argument

2 Write a response in which you discuss what questions would need to be answered in order to decide whether the recommendation is likely to have the predicted result Be sure to explain how the answers to these questions would help to evaluate the recommendation

4번 반복됨

Alta has 30 more job accidents than Panoply(work shifts one hour shorter)Experts Job accidents caused by fatigue and sleep deprivationTherefore to reduce job accidents and increase productivity shorten three work shifts by one hour for adequate sleep

지난해 우리 회사는 인근 Panoply Industries보다 업무상 재해가 30나 더 많았다 그 회사는 우리보다 근무 교대시간이 1시간 정도 짧았다 전문가들은 대부분의 업무상 재해에 있어서 가장 중요한 요인이 과로와 수면부족으로 보고있다 따라서 우리 회사에서 높은 산업재해를 줄이고 아울러 생산성을 높이기 위해서는 근로자들이 충분한 수면을 취할 수 있도록 1시간씩 3교대 시간을 줄여야 한다

In this memo the (author) vice president of Alta Manufacturing (AM) recommends that to reduce on-the-job accidents and increase productivity AM should shorten its three work shifts by one hour so that employees can

GRE AWA John박 박정어학원

get more sleep To support this recommendation the author provides several evidences However careful scrutiny of each of the facts reveals that it provides little credible support for the authorrsquos recommendation QuestionsThe number of accidents What kind of accidents The seriousness of the accidents is importantHow many employees are in each company What are their productsFalse cause Sleep may not be the reason for the on-the-job accidents What do Alta and Panoply manufacture

First of all the author believes that fatigue caused the on-the job accidents However there could be other reasons The author observes a correlation between sleep deprivation and on-the-job accidents then concludes that the former is the cause of the latter However the author fails to rule out other possible explanations For example it is entirely possible that Alta factories require more strenuous and dangerous labor than Panoply Without ruling out all other such factors it is unfair to conclude that fatigue is responsible for the accidents In addition the work-shifts may not be the cause of the sleep deprivation and fatigue It is possiblehellip Thus the author should provide what exactly Panoply and Alta manufacture and more precise data about their working conditions to be more convincing

Shortening the shift by one hour does not necessarily lead to more sleep And is one hour enoughLess accidents does not mean increased productivity

결론 We should shorten each of out three work shifts by one hour

반박 경쟁사에 비해서 시간당 하는 업무량이 많아서 더욱 피곤할 수도 있다 시간이 문제가 아니라 노후된 시설 설비 자체의 문제 작업 자체가 원래 위험한 것이여서 사고가 많을 수도 있다 다른 회사는 더욱 많은 작업시간에도 불구하고 안정한 작업여건으로 인해서 사고율이 오히려 더 작을 수도 있다비교사의 재해감소가 다른 요인일수 있다(안전 교육 철저)줄인 시간이 피로회복이나 수면으로 연결 안될수 있음(술을 마실 수도 있고 그 시간에 휴식을 취하지 않고 다른일을 함으로써 더욱 피로해질수 있다)

This editorial recommends that Alta Manufacturing reduce its work shifts by one hour each inorder to reduce its on-the-job accident rate and thereby increase Altas productivity To supportthis recommendation the author points out that last year the number of accidents at Alta was30 greater than at Panoply Industries where work shifts were one hour shorter The authoralso cites certain experts who believe that many on-the-job accidents are caused by fatigueand sleep deprivation I find this the argument unconvincing for several reasonsFirst and foremost the author provides absolutely no evidence that overall workerproductivity is attributable in part to the number of on-the-job accidents Although commonsense informs me that such a relationship exists the author must provide some evidence ofthis cause-and-effect relationship before I can accept the authors final conclusion that theproposed course of action would in fact increase Altas productivitySecondly the author assumes that some accidents at Alta are caused by fatigue or sleepdeprivation However the author overlooks other possible causes such as inadequateequipment maintenance or worker training or the inherent hazards of Altas manufacturingprocesses By the same token Panoplys comparatively low accident rate might be attributablenot to the length of its work shifts but rather to other factors such as superior equipmentmaintenance or worker training In other words without ruling out alternative causes ofon-the-job accidents at both companies the author cannot justifmbly conclude that merely byemulating Panoplys work-shift policy Alta would reduce the number of such accidentsThirdly even assuming that Altas workers are fatigued or sleep-deprived and that this is thecause of some of Altas on-the-job accidents in order to accept the authors solution to thisproblem we must assume that Altas workers would use the additional hour of free time tosleep or rest However the author provides no evidence that they would use the time in thismanner It is entirely possible that Altas workers would use that extra hour to engage in someother fatiguing activity Without ruling out this possibility the author cannot convincinglyconclude that reducing Altas work shifts by one hour would reduce Altas accident rateFinally a series of problems with the argument arise from the scant statistical information onwhich it relies In comparing the number of accidents at Alta and Panoply the author fails toconsider that the per-worker accident rate might reveal that Alta is actually safer than Panoplydepending on the total number of workers at each company Second perhaps accident rates

GRE AWA John박 박정어학원

at the two companies last year were aberrations and during other years Altas accident ratewas no greater or even lower than Panoplys rate Or perhaps Panoply is not representativeof industrial companies generally and that other companies with shorter work shifts have evenhigher accident rates In short since the argument relies on very limited statistical information Icannot take the authors recommendation seriouslyIn conclusion the recommendation for emulating Panoplys work-shift policy is not wellsupported To convince me that shorter work shifts would reduce Altas on-the-job accidentrate the author must provide clear evidence that work-shift length is responsible for some ofAltas accidents The author must also supply evidence to support her final conclusion that alower accident rate would in fact increase overall worker productivity

The following appeared in a memo from the vice president of marketing at Dura-Sock Inc

A recent study of our customers suggests that our company is wasting the money it spends on its patented Endure manufacturing process which ensures that our socks are strong enough to last for two years We have always advertised our use of the Endure process but the new study shows that despite our socks durability our average customer actually purchases new Dura-Socks every three months Furthermore our customers surveyed in our largest market northeastern United States cities say that they most value Dura-Socks stylish appearance and availability in many colors These findings suggest that we can increase our profits by discontinuing use of the Endure manufacturing process

1 Write a response in which you examine the stated andor unstated assumptions of the argument Be sure to explain how the argument depends on these assumptions and what the implications are for the argument if the assumptions prove unwarranted

2 Write a response in which you discuss what specific evidence is needed to evaluate the argument and explain how the evidence would weaken or strengthen the argument

3 Write a response in which you discuss what questions would need to be answered in order to decide whether the recommendation and the argument on which it is based are reasonable Be sure to explain how the answers to these questions would help to evaluate the recommendation

Intro The vice president of marketing at Dura-Sock Inc is offering a potentially harmful investment recommendation by claiming that Dura-Sock should discontinue its use of the ldquoEndurerdquo process To support his recommendation he points out a study that Dura-Sock customers actually purchase the socks every three months and a survey that reveals that Dura-Sock customers like the sockrsquos stylish appearance and availability in many colors The study and survey however are insufficient in supporting his proposal and the VP makes several unwarranted assumptionsIntro (simplified) The VP states that though Dura-Socks last for two years customers buy the socks every three months Therefore he assumes that the consumersrsquo motive for buying the produce is not its durabilityHowever the author fails to rule out other possible motivation for consumption

Even if the survey is reliable the author should consider the rest of the market Vague terms ldquowasting moneyrdquomdashprecisely how much are they wasting Studysurvey errorThe company must calculate the outcome of such momentous decisionStudy participantsrsquo comment that they prefer Dura-Sock for its stylishness and availability might take Dura-Sockrsquos enduring quality for granted

우리회사 제품 소비자들에 대한 최근 조사에서 지난 2년여간 양말의 내구성을 강하게 하는 필수공정이었던 자사 특허의 Endure 공정에 들어가는 비용이 낭비라고 말하고 있다 우리 회사는 항상 이 공정 처리에 대한 광고를 내보냈으나 이에 대한 시장 조사에서 실제로 고객들은 이 신제품을 평균 석달마다 구매하는 것으로 나타났다 더군다나 북동부지역에서 실시한 대규모 시장조사에 응답한 고객들은 양말의 모양과 색상등에 더

GRE AWA John박 박정어학원

관심을 나타냈다 이러한 결과는 우리회사가 신기술 공법을 중단하면 그에 따라 수익이 늘어날 것이라는 것을 말해주고 있는 것이다주장 These findings suggest that Dura0Sock can increase its profits by discontinuing its use of the ldquoEndurerdquo manufacturing process

1 survey가 정확한 소비자의 의견을 나타낸 것인가 다른 선택없이 양자택일과 같은 방법의 survey였는지2 북동부지역의 시장조사가 전체 의견을 대표할 수 있나3 사람들이 모양이나 색상에 앞서 내구성을 먼저 평가했을 수도 있다 내구성을 갖추었다는 전제하에 모양과 색상에 관심을 드러낸 것일 수 있다4 소비자가 도매상(retail)인지 소매상(whole)인지가 없다

The following appeared in a business magazine

As a result of numerous complaints of dizziness and nausea on the part of consumers of Promofoods tuna the company requested that eight million cans of its tuna be returned for testing Promofoods concluded that the canned tuna did not after all pose a health risk This conclusion is based on tests performed on samples of the recalled cans by chemists from Promofoods the chemists found that of the eight food chemicals most commonly blamed for causing symptoms of dizziness and nausea five were not found in any of the tested cans The chemists did find small amounts of the three remaining suspected chemicals but pointed out that these occur naturally in all canned foods

Write a response in which you discuss what questions would need to be addressed in order to decide whether the conclusion and the argument on which it is based are reasonable Be sure to explain how the answers to the questions would help to evaluate the conclusion

Representativeness of the tested cansThey should conduct a comparative studyThe testing could be biased because Promofoods employees conducted the testingHow much (quantity) of the five and three suspected chemicals were in the canned foodsFalse cause The substance that caused dizziness and nausea may not be one of the eight common chemicals

많은 소비자들의 현기증과 구역질 불만에 따라 Promofoods사는 지난해 참치 캔 8백만 개를 테스트하기 위해 반품시켰다 그 결과 캔에서는 건강에 위험이 될 수 있는 화합물질이 없었던 것으로 회사측은 결론지었다 이러한 결론은 회사측 화학연구자들이 회수된 캔의 샘플을 테스트해서 이들 증상의 원인이 되는 8가지 화합물 중에서 5가지가 실험된 캔에서 발견되지 않았다는 사실에 근거한 것이다 이들 화학자들은 나머지 3개가지 화합물이 모든 캔 식료품에서 흔히 발견되는 것이라고 언급했다 결론 Promofoods concluded that the cans did not after all contain chemicals that posed a health risk

1 공인된 기간에서 테스트를 한 것이 아니고 자사에서 직접 테스트를 했기에 신뢰성이 안간다 2 이런 증상을 일으키는 8개의 물질 말고 다른 물질들이 캔속에 많이 포함됬을수 있다 3 나머지 3개의 물질들의 함유량이 많아서 다른 종류의 캔들은 문제를 일으키지 않지만 참치캔은 문제를

일으킬 수 있다

This magazine article concludes that the 8 million cans of tuna Promofoods recalled due tocomplaints about nausea and dizziness do not after ail contain any chemicals that pose a

GRE AWA John박 박정어학원

health risk To support this conclusion the author cites the fact that five of eight chemicalscommonly causing these symptoms were not found in the recalled cans while the other threealso occur naturally in other canned foods For several reasons this evidence lends littlecredible support to the authors conclusionTo begin with the author relies partly on the fact that although three of the eight chemicalsmost commonly blamed for nausea and dizziness appeared in Promofoods recalled tunathese chemicals also occur naturally in other canned foods However this fact alone lends nosupport to the authors conclusion for two reasons First the author might be ignoring animportant distinction between naturally occurring chemicals and those not occurring naturallyIt is entirely possible that these three chemicals do not occur naturally in Promofoods tunaand that it is for this reason that the chemicals cause nausea and dizziness Secondly it isentirely possible that even when they occur naturally these chemicals cause the samesymptoms Unless the author rules out both possibilities he cannot reliably conclude that therecalled tuna would not cause these symptomsAnother problem with the argument is that the authors conclusion is too broad Based onevidence about certain chemicals that might cause two particular heath-related symptoms theauthor concludes that the recalled tuna contains no chemicals that pose a health risk Howeverthe author fails to account for the myriad of other possible health risks that the recalled tunamight potentially pose Without ruling out all other such risks the author cannot justifiablyreach his conclusionA third problem with the argument involves that fact that the eight particular chemicals withwhich the test was concerned are only the eight most commonly blamed for nausea anddizziness It is entirely possibly that other chemicals might also cause these symptoms andthat one or more of these other chemicals actually caused the symptoms Without ruling outthis possibility the author cannot jusufiably conclude that the recalled tuna would not causenausea and dizzinessA final problem with the argument involves thetesting procedure itself The author providesno information about the number of recaUed cans tested or the selection method used Unlessthe number of cans is a sufficiently large sample and is statistically repre sentative of all therecalled cans the studys results are not statistically reliableIn conclusion the article is unconvincing as it stands To strengthen the assertion that therecalled tuna would not cause nausea and dizziness the author must provide evidence thatthe three chemicals mentioned that occur naturally in other canned foods also appear naturallyin Promofoods tuna The author must also provide evidence that ingesting other canned foodscontaining these three chemicals does not cause these symptoms To better evaluate theargument we would need to know whether the sample used in the tests was statisticallysignificant and representative of all the recalled tuna We would also need to know what otherchemicals in the recalled tuna might pose any health risk at all

5그룹 불충분 조건오류 빈출

Natures Way a chain of stores selling health food and other health-related products is opening its next franchise in the town of Plainsville The store should prove to be very successful Natures Way franchises tend to be most profitable in areas where residents lead healthy lives and clearly Plainsville is such an area Plainsville merchants report that sales of running shoes and exercise clothing are at all-time highs The local health club has more members than ever and the weight training and aerobics classes are always full Finally Plainsvilles schoolchildren represent a new generation of potential customers these schoolchildren are required to participate in a fitness-for-life program which emphasizes the benefits of regular exercise at an early age

Write a response in which you examine the stated andor unstated assumptions of the argument Be sure to

GRE AWA John박 박정어학원

explain how the argument depends on these assumptions and what the implications are for the argument if the assumptions prove unwarranted

False cause

First of all the author believes that the Increased sales of running shoes and exercise clothing indicates

Plainesville residentsrsquo interest in leading healthy lives However this assumption is not logically convincing for

several reasons could be a fashion trendTime shift ldquoFitness for liferdquo might not have any influence on schoolchildren as they growFalse cause There could be other reasons for member increase in the health clubAll of the above are insufficient condition

The author has to prove that local residents are interested in leading healthy lives However he supports his conclusion with insufficient evidence Nevertheless even if the residents are concerned with health naturersquos way may not be successful First

그 동안의 경험을 토대로 볼 때 건강생활과 밀접히 관련되어 있는 거주 지역에서 본 상점들이 아주 호응을 얻고 있다 따라서 이러한 주민들이 많이 거주하고 있는 Plainsville 에 새로운 상점들을 계속 세워야 한다 이 지역 상인들은 런닝화와 운동복 판매가 가장 높다고 말한다 불과 5 년전에는 거의 전무하다시피하던 지역 헬스 클럽의 경우도 엄청나게 많은 회원을 확보하고 있으며 웨이트 트레이닝과 에어로빅 강좌들도 항상 만원이라고 한다 새로운 고객층을 예측해 보는 것도 가능하다 이 지역의 학생들의 경우 Fitness for Life프로그램을 받게 되는데 이러한 프로그램을 통해서 유년시절부터 정규적인 운동 습관을 들이게 하고 있는 것이 그것이다

결론 We should therefore build our next new store in Plainsville

반박 그동안의 경험에 의한 과거 통계가 꼭 여기에도 적용되는건 아니다 5 년전 헬스 클럽이 잘 안되었던게 다른 원인이였을수 있다(강사수준 미달 강좌미비)tourist 에 의한 원인 일수 있다 어렸을때부터 운동을 했다고 해서 커서도 관심이 있지는 않다 (오히려 반감이 있을수 있다 혹은 건강하기에 건강에 관심이 적을수도 있다)운동복이나 신발의 판매가 육체노동에 의한 것일수도 있다

IntroductionSupport1049896In this memorandum the author asserts that Naturersquos Way should build its next newstore in Plainsville To support this assertion the author states that Plainsvillesmerchantsrsquo sales of exercise clothing are going well the local health club has moremembers than ever and a new generation of customers will help to ensure NaturersquosWayrsquos success At first glance the authorrsquos assumption seems convincing but in-depth scrutiny revealsthat it lacks substantial evidence as it stands

Body 1-SamplingTopic Sentence 1To begin with the author assumes that the merchantsrsquo report indicates that the residentsare concerned about their health However this assumption is based on unsubstantiated

GRE AWA John박 박정어학원

data Example 1 (Rebuttal1) First if we do not know the total volume of items sold and the price of the goods exactly we cannot infer whether the residents are actually buying many goods Example 2 (Rebuttal2)In addition to that the report emphasizes the rising sales of running shoes and exerciseclothing however these may not be hot-selling items for Naturersquos Way or may not be theproducts the company is planning to sell Concluding Sentence Therefore in order to make the argument reliable the author should reconsider themerchantsrsquo report with more detailed data

Body 2-CausalTopic Sentence 2Second the author contends that the health clubs classes are full yet this does not meanthat many people actually use the health club other factors may be the real cause forthose closed classes Example 1 (Rebuttal1) To begin with if the health club is very small the number of people working out wouldnot be a large one In fact regular gym-going may just be a vogue among a smallunrepresentative segment of Plainsvilles population Example 2 (Rebuttal2) Moreover it is possible that most of the people who exercise in the health club do weight training and aerobics only to look good and to meet other singles not for their health In that case there would be little demand for health products Concluding SentenceThus the author should not hasten to presume what really caused people to be interested in a healthier lifestyle and enroll in the health club

Body 3-Time-ShiftTopic Sentence 3Finally the author highlights that Naturersquos Way can expect a new generation of customersin Plainsville that will help the company in the long term This notion is mistaken in that itassumes the conditions of the present will continue unchanged in the future Although theschool children are required to participate in the fitness for life program they may notnecessarily buy Naturersquos Ways products Example 1 (Rebuttal1) In the first instance they may suffer a fall in purchasing power arising from future economic difficulties this would cause reluctance to spend a considerable amount of money on health products which tend to be more expensive Example 2 (Rebuttal2)Another possibility is that there may emerge many competitor companies vying with Naturersquos Way so that in the future the school children may not feel the necessity to purchase one companyrsquos health products over anotherrsquosConcluding Sentence Thus the authorrsquos assumption is highly speculative since it relies heavily on unknowablefuture circumstances

ConclusionThesis In sum the author uses many assumptions that are insufficient in supporting his claimsSupportIn order for the authorrsquos claims to be convincing he needs to advance more persuasiveevidence that people in Plainsville really are concerned with their health and health foodThe following was written as a part of an application for a small-business loan by a group of developers in the city of Monroe

A jazz music club in Monroe would be a tremendously profitable enterprise Currently the nearest jazz club is 65 miles away thus the proposed new jazz club in Monroe the C-Note would have the local market all to itself Plus jazz is extremely popular in Monroe over 100000 people attended Monroes annual jazz festival last summer several well-known jazz musicians live in Monroe and the highest-rated radio program in Monroe is Jazz Nightly which airs every weeknight at 7 PM Finally a nationwide study indicates that the typical jazz fan spends close to $1000 per year on jazz entertainment

1 Write a response in which you discuss what specific evidence is needed to evaluate the argument and explain how the evidence would weaken or strengthen the argument

2 Write a response in which you examine the stated andor unstated assumptions of the argument Be

GRE AWA John박 박정어학원

sure to explain how the argument depends on these assumptions and what the implications are for the argument if the assumptions prove unwarranted

3 Write a response in which you discuss what questions would need to be answered in order to decide whether the prediction and the argument on which it is based are reasonable Be sure to explain how the answers to these questions would help to evaluate the prediction

Group error nationwide survey may not reflect local trends Is the nationwide jazz fan population substantialInsufficient non-residents of Monroe may have attended the jazz festival (Body alternative explanation last year may have been an anomaly The author should consider data from various years) The author should indicate how many out of 100000 were Monroe residentsNationwide study Does this reflect Insufficient Citizens of Monroe may continue to go to the jazz club 65 miles away

Are the people in Monroe really interested in jazzMajority of the people who attended the jazz festival might not be Monroe residentsSurvey error nationwide study may not be applicable to MonroeJazz musicians who live in MonroeMonopolyRadio station

In this business application the author claims that the proposed jazz club C Note will be very profitable in Monroe To support this claim the author argues for his case with several evidences At first glance the authorrsquos argument seems convincing however careful scrutiny reveals that his argument in specious

To begin with the author claims that Monroersquos citizens are interested in jazz He presents three evidences First Secondhellip Thirdhellip Howeverhellip

Monroe 시에 있는 재즈 음악 클럽은 수익성이 좋은 사업이다 현재 가장 가까이에 있는 클럽은 65 마일 정도 떨어져 있다 따라서 이번에 세우려고 하는 C Note 는 독보적인 위치를 점할것이다 더군다나 재즈는 이 시에서 가장 인기있는 음악이다 지난 여름 재즈 축제에서는 10 만명 이상의 Morone 시 주민이 참석하였고 몇몇 유명한 재즈 음악가들도 이곳에 살고 있으며 저녁때 방영되는 라디오 프로그램중에서 최고의 시청률을 보이고 있는 것도 Jazz Nightly 이다 전국조사에서도 전형적인 재즈 팬들은 재즈 분야에 년간 1천 달러 가까이 지출하고 있는 것으로 보고되고 있다 따라서 C Note 클럽이 돈을 벌 수 있는 사업이라는 것은 확실한 것이다

결과 It is clear that the C Note cannot help but make money반박 nearest jazz club 이 양질의 써비스로 여전히 손님을 끌수도 있다Festival 에 얼마나 참여하는지가 jazz 의 인기를 반영하지 않는다 뮤지션이 많이 사는거랑 jazz 의 인기가 상관없다라디오 프로그램이 다른 요인에 의해서 인기일수도 있다 (진행자때문)전국 통계 적용 불가화목 실전반_Ms Noh6In this application the author suggests that a jazz club in Monroe will make a number of profits To support this suggestion the author exemplifies the local condition popularity of jazz in Monroe and nationwide study However careful scrutiny of each of the facts reveals that it provides little credible support for the authorrsquos recommendation Good clear intro

First the author assumes that jazz is popular in Monroe because of several facts the jazz festival last year had high participation some famous jazz musicians live in Monroe and the high-rated radio program is lsquoJazz Nightlyrsquo However this assumption has many drawbacks that must be seriously considered(Good topic sentences) If many attendants in the last-yearrsquos festival came from other cities and not Monroe it is hard to conclude that Monroersquos people like jazz Therefore the author must examine how many Monroe residents actually attended the festival On top of that there is little relationship between habitation of famous jazz musician and the popularity of jazz in Monroe Although several well-known musicians live there if they do not take part in any jazz performance of Monroe this might have no effect to the interest of Monroersquos residents

GRE AWA John박 박정어학원

about jazz Finally in the case of radio program this is also not suitable reason why jazz is popular in Monroe It might be possible that people cannot help choosing lsquoJazz Nightlyrsquo because there are few radio programs at Night The fact that the radio program is the highest rating program is not a germane evidence The approximate number of listeners would be the more crucial evidence Therefore the author needs to seriously deliberate the correlation between jazzrsquos popularity in Monroe and his examples (Good logical flow and clarity)

Second the author uses as evidence the nationwide study that jazz fans spend much money on jazz entertainment to substantiate why starting a jazz club in Monroe will be profitable In other words the author assumes that the characteristics of a nationwide study can be applied to Monroe The national study would lend support to the applicantrsquos claim only if residents in Monroe typify national jazz fans However the author does not provide credible evidence that this is the case Moreover the populations of jazz fans nationwide may be insubstantial Thus the author should not infer hastily that Monroersquos residents will spend much money on enjoying jazz from the nationwide study

Lastly even if jazz is popular in Monroe C Note may not be successful It is entirely possible that residents might still prefer other clubs where they have always went In addition there is another possibility that the nearest jazz club will attract many of Monroersquos people because it serves fine performances and is equipped with favorite facilities Without considering these other possibilities the author cannot make his argument convincing In sum the author presents many reasons that are insufficient in supporting his or her claim In order for the authorrsquos claims to be convincing he needs to advance more persuasive evidence such as the total number of Monroe residents who attended the jazz festival the effects on the popularity of jazz by the musicians living in Monroe and the actual number of residents who would typify themselves to be jazz fans through a local survey Without substantial evidence that C Note will be successful in Monroe the businessmen may be overinvesting in what might lead to a business failureExcellent clarity Score 50

The following appeared in a newsletter offering advice to investors

Over 80 percent of the respondents to a recent survey indicated a desire to reduce their intake of foods containing fats and cholesterol and today low-fat products abound in many food stores Since many of the food products currently marketed by Old Dairy Industries are high in fat and cholesterol the companys sales are likely to diminish greatly and company profits will no doubt decrease We therefore advise Old Dairy stockholders to sell their shares and other investors not to purchase stock in this company

Write a response in which you discuss what questions would need to be answered in order to decide whether the advice and the argument on which it is based are reasonable Be sure to explain how the answers to these questions would help to evaluate the advice

Survey 80

GRE AWA John박 박정어학원

Old Dairy could change their products and manufacture low fat dairy foodsLess competing companies Old Dairy could eventually be the only company that produces hellipImprecise numbers and measurementsCustomers may still buy high fat dairy products

The author of the newsletter is offering potentially dangerous advice by recommending Old Dairy stockholders to withdraw investment and stop purchase What is more the authorrsquos prediction debases the reputation and business of Old Dairy and if false could devoid the investment opportunity of the newsletter readers Therefore investors should examine whether the authorrsquos evidences are substantial

To begin with the author states that 80 percent of the respondents in a survey indicated a desire to reduce their intake of foods He therefore argues that Old Dairyrsquos high fat and cholesterol products would decrease in sales However the author makes a crucial error in this argument First the author provides no evidence that the surveyrsquos results are statistically reliable Were they representative of all the customers Were they chosen for the survey randomly Furthermore the desire to reduce fat and cholesterol intake is a pervasive trend in todayrsquos opulent society however the author erroneously identifies this as a new phenomenon which will affect consumer trends Second having a desire to reduce fat and cholesterol intake does not necessarily indicate that people who have this desire will actually reduce consuming these types of products It is entirely possible that they may continue buying Old Dairy products for its quality and taste Accordingly the author cannot draw any firm conclusion that people will not buy Old Dairy products Therefore if any of these cases are true the author may be offering investors a detrimental investment advice

최근 조사에 대한 응답자중 80 이상이 자신이 먹는 음식에서 지방과 콜레스테롤의 함유량을 줄이고 싶다고 한다 아울러 요즘은 많은 식료품 가계에서 저지방 제품들을 많이 취급하고 있다 현재 Old Dairy Industries가 판매하고 있는 많은 음식제품들은 지방과 콜레스테롤이 높기 때문에 이 회사의 매출이 격감할 것으로 보이며 당연히 매출이익도 줄어들것이다 따라서 이 회사의 주주들은 주식을 매각하고 다른 주식 투자가들도 이 회사의 주식을 매입하지 않는 것이 좋다

결론 Old Dairy stockholders to sell their shares and other investors not to purchase stock in this company

반박 모든 상품이 다 고 지방 고 칼로리는 아니다(비록 많을지라도) 일부의 식품의 경우 기호에 맞어서 히트해서 전체적인 수입이 증가할 수도 있다국내시장만 생각할 수 없다( 외국시장에서 호황을 누릴수 있다 )입맛이라는게 즉각 바뀌는게 아니다

The following appeared in a letter to the editor of the Balmer Island Gazette

On Balmer Island where mopeds serve as a popular form of transportation the population increases to 100000 during the summer months To reduce the number of accidents involving mopeds and pedestrians the town council of Balmer Island should limit the number of mopeds rented by the islands moped rental companies from 50 per day to 25 per day during the summer season By limiting the number of rentals the town council will attain the 50 percent annual reduction in moped accidents that was achieved last year on the neighboring island of Seaville when Seavilles town council enforced similar limits on moped rentals

1 Write a response in which you discuss what questions would need to be answered in order to decide whether the recommendation is likely to have the predicted result Be sure to explain how the answers to these questions would help to evaluate the recommendation

2 Write a response in which you discuss what questions would need to be answered in order to decide whether the prediction and the argument on which it is based are reasonable Be sure to explain how the answers to these questions would help to evaluate the prediction

3 Write a response in which you examine the stated andor unstated assumptions of the argument Be sure to explain how the argument depends on these assumptions and what the implications are for the argument if the assumptions prove unwarranted

Whatrsquos the actual population of Balmer Island 100000mdashis this a significant increase What kind of accidents Skin abrasions or serious injury And compared to Seaville how serious are the accidents and the actual number of accidents Did Seaville enforce other restrictions like safety signsHow different are the conditions of Balmer

GRE AWA John박 박정어학원

and Seaville regarding population road (safety) conditions topography other town-government regulation How much will the economy of Balmer be affected do to this restriction Could it cause an economic recession due to the fact that these rental companiesrsquo chance to make money is only during the summer thereby weakening the economic infrastructure Are there any other ways that could better alleviate the accident rate

Statistics 50-impreciseAnalogy Balmer compared with TorseauFalse Cause Accidents might have occurred because of reasons other than mopeds False Cause population increase may not be part of the cause of the accidentsOther explanations for the accident pedestrians few road safety regulations narrow roadsThere could be other better solutionshellip

Balmer Island의 인구가 여름철에는 십만명으로 늘어난다 2륜차와 보행자간 사고를 줄이기 위해 시의회는 6개의 자전거를 포함한 2륜차 대여업체에게 이 기간동안에는 대여숫자를 일일 50에서 30으로 제한하도록 할 것이다 대여숫자를 줄임으로써 시 의회는 지난해 이웃한 Torseau섬에서 이와 동일한 규제를 시행해서 50나 줄인 결과를 보고 마찬가지로 50를 줄일수 있다고 확신하고 있다

결론 The town council of Balmer Island should linit the number

반박 보행자의 부실에 의해서 사고가 많이 일어날수도 있다렌탈수의 줄임만이 대책은 아니다(대부분의 사람들이 렌탈 보다는 소유하고 있을 수도 있다)옆섬과는 상황이 다를수도 있다(그 섬에서는 사고의 원인이 많은 자전거 수로 인한것일수있다) 하지만 이 섬은 좁은 도로가 원인일 수도 있고 도로 안전 장치의미비가 원일일수 있다

In this letter the author recommends that Balmer Island should limit the number moped rentals from 50 to 30 per day To support this recommendation the author points out several reasons However careful scrutiny of each of the facts reveals that it is filled with unanswered questions that could significantly weaken the authorrsquos recommendation with loops and holes which are answered

The recommendation depends on the assumption that no alternative means of reducing the number of accidents are available However the author fails to offer any evidence to substantiate this crucial assumption It is highly possible that means other than this would better solve the problem Perhaps they could widen the roads or put-up more safety signs Or perhaps the accidents were due to the lack of skills in which case proper safety training would significantly alleviate the problem Without considering and ruling out these and other alternative means of reducing accidetns the author cannot confidently conclude that merely emulating Torseau would suffice Moreover the author is advising a recommendation which could potentially harm the economy of Balmer Island sincehellip Moreover the Balmer Island should alternative means to reduce accidents because limiting moped rentals during the summer could harm the economy of Balmerhellip

First of all the author believes that increase in population and the number of moped rentals are responsible for the accidents It is entirely possible that other factors are responsible for the accidents Perhaps Balmer Islandrsquos lack of safety signs was a major factor Or maybe the roads are narrow and dangerous on the Island therefore the town council could enforce stricter traffic regulations to alleviate the problem Accordingly if either of these scenarios is true the author cannot draw any firm conclusion that increase in the number of population and moped rentals are the cause of the accidents

The author of this editorial recommends that to reduce accidents involving mopeds andpedestrians Balmer Islands city council should restrict moped rentals to 30 per day down from50 at each of the islands six rental outlets To support this recommendation the author citesthe fact that last year when nearby Torseau Islands town council enforced similar measuresTorseaus rate of moped accidents fell by 50 For several reasons this evidence providesscant support for the authors recommendationTo begin with the author assumes that all other conditions in Balmer that might affect therate of moped-pedestrian accidents will remain unchanged after the restrictions are enactedHowever with a restricted supply of rental mopeds people in Balmer might purchase mopedsinstead Also the number of pedestrians might increase in the future with more pedestriansespecially tourists the risk of moped-pedestrian accidents would probably increase For thatmatter the number of rental outlets might increase to make up for the artificial supplyrestriction per outlet--a likely scenario assuming moped rental demand does not declineWithout considering and ruling out these and other possible changes that might contribute to ahigh incidence of moped-pedestrian accidents the author cannot convince me that theproposed restrictions will necessarily have the desired effect

GRE AWA John박 박정어학원

Next the author fails to consider other possible explanations for the 50 decline inTorseaus moped accident rate last year Perhaps last year Torseau experienced unusually fairweather during which moped accidents are less likely Perhaps fewer tourists visited Tot seanlast year than during most years thereby diminishing the demand for rental mopeds to belowthe allowed limits Perhaps last year some of Torseaus moped rental outlets purchased newmopeds that are safer to drive Or perhaps the restrictions were already in effect but were notenforced until last year In any event a decline in Torseaus moped accident rate during onlyone year is scarcely sufficient to draw any reliable conclusions about what might have causedthe decline or about what the accident rate will be in years aheadAdditionally in asserting that the same phenomenon that caused a 50 decline in mopedaccidents in Torseau would cause a similar decline in Balmer the author relies on what mightamount to an unfair analogy between Balmer and Torseau Perhaps Balmers ability to enforcemoped-rental restrictions does not meet Torseaus ability if not then the mere enactment ofsimilar restrictions in Balmer is no guarantee of a similar result Or perhaps the demand formopeds in Torseau is always greater than in Balmer Specifically if fewer than all availablemopeds are currently rented per day from the average Balmer outlet while in Torseau everyavailable moped is rented each day then the proposed restriction is likely to have less impacton the accident rate in Balmer than in TorseauFinally the author provides no evidence that the same restrictions that served to reduce theincidence of all moped accidents by 50 would also serve to reduce the incidence ofaccidents involving mopeds and pedestrians by 50 Lacking such evidence it is entirelypossible that the number of moped accidents not involving pedestrians decreased by a greaterpercentage while the number of moped-pedestrian accidents decreased by a smallerpercentage or even increased Since the author has not accounted for these possibilities theeditorials recommendation cannot be taken seriouslyIn conclusion the recommendation is not well supported To convince me that the proposedrestriction would achieve the desired outcome the author would have to assure me that nochanges serving to increase Balmers moped-pedestrian accident rate will occur in theforeseeable future The author must also provide dear evidence that last years decline inmoped accidents in Torseau was attributable primarily to its moped rental restrictions ratherthan to one or more other factors In order to better evaluate the recommendation I wouldneed more information comparing the supply of and demand for moped rentals on the twoislands I would also need to know the rate of mopedpedestrian accidents in Torseau both priorto and after the restrictions were enforced in TorseauThe following appeared in a magazine article about planning for retirement

Clearview should be a top choice for anyone seeking a place to retire because it has spectacular natural beauty and a consistent climate Another advantage is that housing costs in Clearview have fallen significantly during the past year and taxes remain lower than those in neighboring towns Moreover Clearviews mayor promises many new programs to improve schools streets and public services And best of all retirees in Clearview can also expect excellent health care as they grow older since the number of physicians in the area is far greater than the national average

Write a response in which you discuss what specific evidence is needed to evaluate the argument and explain how the evidence would weaken or strengthen the argument

-Natural beauty and consistent climate may not be the most wanted qualities-Housing costs could have lowered on a national level wealthy retirees may not care about costs-Taxes may be high compared to the nationrsquos average tax rate-What about other qualities of Clearview Crime rate what qualities would retirees want -If schools streets and public services need improvement then this is proof that the current condition of Clearview is low Or due to budgetary reasons the mayor may not follow-up on his promise because of lowered tax rate -Schools and people who are retired no relationship-Physicians What kind of physicians Number is irrelevant Are these physicians capable of addressing the illnesses of old people

This author argues that anyone seeking a place to retire should choose Clearview To supportthis argument the article cites Clearviews consistent climate and natural beauty its fallinghousing costs its low property taxes compared to nearby towns and the mayors promise toimprove schools streets and services The article also claims that retirees can expectexcellent health care because the number of physicians in Clearview greatly exceeds thenational average This argument is flawed in several critical respectsTo begin with although consistent climate and natural beauty might be attractive to manyretirees these features are probably not important to all retirees For many retirees it isprobably more important to live near relatives or even to enjoy changing seasons Thus I

GRE AWA John박 박정어학원

cannot accept the authors sweeping recommendation for all retirees on this basisAlso Clearviews declining housing costs do not necessarily make Clearview the best placeto retire for two reasons First despite the decline Clearviews housing costs might be highcompared to housing costs in other cities Secondly for wealthier retirees housing costs arenot likely to be a factor in choosing a place to retire Thus the mere fact that housing costshave been in decline lends scant support to the recommendationThe articles reliance on Clearviews property-tax rates is also problematic in two respectsFirst retirees obviously have innumerable choices about where to retire besides Clear viewand nearby towns Secondly for retirees who are well-off financially property taxes are notlikely to be an important concern in choosing a place to retire Thus it is unfair to infer fromClearviews property-tax rates that retirees would prefer ClearviewYet another problem with the argument involves the mayors promises In light of Clearviewslow property-tax rates whether the mayor can follow through on those promises is highlyquestionable Absent any explanation of how the city can spend more money in the areas citedwithout raising property taxes I simply cannot accept the editorials recommendation on thebasis of those promises Besides even if the city makes the improvements promised thoseimprovements--particular the ones to schools--would not necessarily be important to retireesFinally although the number of physicians in Clearview is relatively high the per capitanumber might be relatively low Moreover it would be fairer to compare this per capita numberwith the per capita number for other attractive retirement towns--rather than the nationalaverage After all retirees are likely to place a relatively heavy burden on health-careresources Besides the article provides no assurances that the number of physicians inClearview will remain high in the foreseeable futureIn conclusion the recommendation is poorly supported To strengthen it the author mustconvince me--perhaps by way of a reliable survey--that the key features that the vast majorityof retirees look for in choosing a place to live are consistent climate natural beauty and lowhousing costs The author must also provide better evidence that Clear views property taxesare lower than the those of cities in other areas The author must also explain how the city canmake its promised improvements without raising property taxes Finally to better assess theargument I would need to now how the per capita number of physicians in Clearview wouldcompare to the national average in the futureThe following appeared as a letter to the editor from a Central Plaza store owner

Over the past two years the number of shoppers in Central Plaza has been steadily decreasing while the popularity of skateboarding has increased dramatically Many Central Plaza store owners believe that the decrease in their business is due to the number of skateboard users in the plaza There has also been a dramatic increase in the amount of litter and vandalism throughout the plaza Thus we recommend that the city prohibit skateboarding in Central Plaza If skateboarding is prohibited here we predict that business in Central Plaza will return to its previously high levels

Write a response in which you discuss what questions would need to be answered in order to decide whether the recommendation is likely to have the predicted result Be sure to explain how the answers to these questions would help to evaluate the recommendation

Why two years ago What happened two years ago which started this declineIs the dramatic increase in the ldquopopularityrdquo of skateboarding the cause of the steady decline of shoppers Are there any malls nearby Were there any changes nearby which could affect the decline in customersmdasha big mall perhaps Could the decline be due to the shop ownersHow many skateboarders use the plazaWhere do they skateboardDo they shop and are they customersAre the increase in litter and vandalism due to skateboarders Could this be alleviated by installing CCTVs and hiring security

This editorial concludes that the city should ban skateboarding from its downtown CentralPlaza in order to attract visitors to that area to return the area to its former glory and to makeit a place where people can congregate for fun and relaxation To justify this conclusion theeditorial points out that skateboarders are nearly the only people one sees anymore at CentralPlaza and that the Plaza is littered and its property defaced The editorial also points out thatthe majority of downtown merchants support the skate boarding ban This argument is flawedin several critical respectsFirst the editorials author falsely assumes that a ban on skateboarding is both necessaryand sufficient to achieve the three stated objectives Perhaps the city can achieve thoseobjectives by other means as well--for example by creating a new mall that incorporates anattractive new skateboard park Even if banning skateboarders altogether is necessary to meetthe citys goals the author has not shown that this action by itself would suffice Assuming thatthe Plazas reputation is now tarnished restoring that reputation and in turn enticing peopleback to the Plaza might require additional measures--such as removing litter and graffiti

GRE AWA John박 박정어학원

promoting the Plaza to the public or enticing popular restaurant or retail chains to the PlazaSecondly the editorial assumes too hastily that the Plazas decline is attributable to theskateboarders--rather than to some other phenomenon Perhaps the Plazas primary appeal inits glory days had to do with particular shops or eateries which were eventually replaced byless appealing ones Or perhaps the crime rate in surrounding areas has risen dramatically forreasons unrelated to the skateboarders presence at the Plaza Without ruling out these andother alternative explanations for the Plazas decline the editorials author cannot convince methat a skateboard ban would reverse that declineThirdly the editorials author might be confusing cause with effect--by assuming that theskateboarders caused the abandonment of the Plaza rather than vice versa It is entirelypossible that skateboarders did not frequent the Plaza until it was largely abandoned--andbecause it had been abandoned In fact this scenario makes good sense since skateboardingis most enjoyable where there are few pedestrians or motorists to get in the wayFourth it is unreasonable to infer from the mere fact that most merchants favor the ban thatthe ban would be effective in achieving the citys objectives Admittedly perhaps thesemerchants would be more likely to help dean up the Plaza area and promote their businesseswere the city to act in accordance with their preference Yet lacking any supporting evidencethe author cannot convince me of this Thus the survey amounts to scant evidence at best thatthe proposed ban would carry the intended resultFinally the author recommends a course of action that might actually defeat the citysobjective of providing a fun and relaxing place for people to congregate In my experienceskateboarding contributes to an atmosphere of fun and relaxation for adults and children alikemore so than many other types of ambiance Without considering that continuing to allowskateboarding--or even encouraging this activity--might achieve the citys goal more effectivelythan banning the activity the author cannot convincingly conclude that the ban would be in thecitys best interestsIn sum the argument is a specious one To strengthen it the editorials author must providedear evidence that skateboarding and not some other factor is responsible for the conditionsmarking the Plazas decline The author must also convince me that no alternative means ofrestoring the Plaza are available to the city and that the proposed ban by itself would suffice toattract tourists and restore the Plaza to its former glory Finally to better assess the argument itwould be useful to know the circumstances under which the downtown merchants would bewilling to help the city achieve its objectives

6그룹 약한 비유 빈출

The following recommendation appeared in a memo from the mayor of the town of Hopewell

Two years ago the nearby town of Ocean View built a new municipal golf course and resort hotel During the past two years tourism in Ocean View has increased new businesses have opened there and Ocean Views tax revenues have risen by 30 percent Therefore the best way to improve Hopewells economymdashand generate additional tax revenuesmdashis to build a golf course and resort hotel similar to those in Ocean View

Write a response in which you examine the stated andor unstated assumptions of the argument Be sure to explain how the argument depends on these assumptions and what the implications are for the argument if the assumptions prove unwarranted

GRE AWA John박 박정어학원

Assumptions The author assumes that OVrsquos municipal golf course and resort hotel caused tourism new businesses and increased tax revenues There may be other reasons advertising promo He assumes that this will continueAssumes that Ocean View and Hopewell are similar in many waysmdashthe name suggests otherwise OV may have always been a tourist attractions for its beaches We need to know the topography

2년전 Ocean View 시는 시정 소유 골프 및 휴양지 호텔을 신축했다 그리고 지난 2년동안 이 시의 관광객이 증가했으며 새로운 사업들이 생겨났다 그에따라 시의 세수도 30나 증가했다 Hopewell의 경제를 향상시키고 아울러 세수를 늘릴 수 있는 가장 좋은 방법은 Ocean View에 세워진 것과 같은 골프 시설과 휴양지 호텔을 신축하는 것이다

1 다른 요인으로 관광 산업이 발전했을 수도 있다 문화 유적이 발견이 되었거나 도로의 정비등으로 여행자가 늘었을 수도 있다

2 관광 산업의증가가 늘어난 세수의 원인이 아니라 새로 유입된 인구의 증가나 다른 공장에서 발생한 것일 수 있다

3 2년동안 한참 골프가 붐을 이루었을 수 있다 경제상황이 나빠지거나 다른 레포츠가 인근 지역에 생겨난다면 골프하는 사람이 줄어들 수 있다

In this memo HopeweUs mayor recommends that in order to stimulate the towns economyand boost tax revenues HopeweU should build a new golf course and resort hotel just as thetown of Ocean View did two years ago To support this recommendation the mayor points outthat in Ocean View during the last two years tourism has increased new businesses haveopened and tax revenues have increased by 30 I find the mayors argument unconvincingin several important respectsFirst of all it is possible that the mayor has confused cause with effect respecting the recentdevelopments in Ocean View Perhaps Ocean Views construction of a new golf course andhotel was a response to previous increases in tourism and business development increasesthat have simply continued during the most recent two years Since the mayor has failed toaccount for this possibility the claim that Hopewell would boost its economy by alsoconstructing a golf course and hotel is completely unwarrantedSecondly the mayor fails to account for other possible causes of the trends in Ocean Viewduring the last two years The increase in tourism might have been due to improving economicconditions nationwide or to unusually pleasant weather in the region The new businessesthat have opened in Ocean View might have opened there irrespective of the new golf courseand hotel And the 30 increase in tax revenues might have been the result of an increase intax rates or the addition of a new type of municipal taxWithout ruling out these and other alternative explanations for the three recent trends inOcean View the mayor cannot reasonably infer based on those trends that Hopewellseconomy would benefit by following Ocean Views exampleThirdly even if the recent trends in Ocean View are attributable to the construction of the newgolf course and hotel there the mayor assumes too hastily that the golf course and hotel willcontinue to benefit that towns overall economy The mayor has not accounted for thepossibility that increased tourism will begin to drive residents away during tourist season orthat new business development will result in the towns losing its appeal as a place to visit or tolive Unless the mayor can convince me that these scenarios are unlikely I cannot accept themayors recommendation that Hopewell follow Ocean Views exampleFinally the mayors argument rests on the unsubstantiated assumption that Hopewell andOcean View are sufficiently alike in ways that might affect the economic impact of a new golfcourse and hotel Hopewell might lack the sort of natural environment that would attract moretourists and new businesses to the town--regardless of its new golf course and hotel For thatmatter perhaps Hopewell already contains several resort hotels and golf courses that are notutilized to their capacity If so building yet another golf course and hotel might amount to amisallocation of the towns resources--and actually harm the towns overall economyIn sum the mayors recommendation is not well supported To bolster it the mayor mustprovide better evidence that Ocean Views new golf course and hotel and not some otherphenomenon--has been responsible for boosting Ocean Views economy during the last twoyears To better assess the recommendation I would need to know why Ocean View decidedto construct its new golf course and hotel in the first place--specifically what events prior toconstruction might have prompted that decision I would also need to thoroughly compare

GRE AWA John박 박정어학원

HopeweU with Ocean View--especially in terms of their appeal to tourists and businesses--todetermine whether the same course of action that appears to have boosted Ocean Viewseconomy would also boost Hopewells economy

The following is part of a memorandum from the president of Humana University

Last year the number of students who enrolled in online degree programs offered by nearby Omni University increased by 50 percent During the same year Omni showed a significant decrease from prior years in expenditures for dormitory and classroom space most likely because instruction in the online programs takes place via the Internet In contrast over the past three years enrollment at Humana University has failed to grow and the cost of maintaining buildings has increased along with our budget deficit To address these problems Humana University will begin immediately to create and actively promote online degree programs like those at Omni We predict that instituting these online degree programs will help Humana both increase its total enrollment and solve its budget problems

Write a response in which you discuss what questions would need to be answered in order to decide whether the prediction and the argument on which it is based are reasonable Be sure to explain how the answers to these questions would help to evaluate the prediction

Is Omni University successful due to the online degree program 50 Is the decrease in expenditures for dormitory and classroom space due to the decrease in of on-campus students Which classes were successful Does HU have those classes

Even if the long-distance degree programs at Omni University benefited the school the presidentrsquos recommendation that Human College should emulate Omni University is too hasty First OUrsquos name implies that the school would have more majors than Humanahellip the president should examine which degrees were in the long-distance programhellip

지난해에는 Omni 대학에서 개강했던 원거리 학생 학점 취득 프로그램을 등록했던 학생들의 숫자가 50나 증가했다 같은해 기간동안 Omni 대학에서는 그 전년도부터 기숙사와 학급의 공간 확충을 위한 예산을 대폭 줄였는데 이는 이 원거리 학점 취득 프로그램이 양방향 비디오 컴퓨터 접속을 통해서만 가능한 수업지도 방식이기때문인 것으로 보인다 반면 지난 3개년 동안 Humana 대학에서의 수강률은 감소한데다가 건물

GRE AWA John박 박정어학원

유지비도 올랐다 따라서 Humana대학의 수강을 늘리고 예산손실을 회복하기 위해서는 Omni 대학에서 취한 조치와 같은 능동적인 프로그램을 추진해야 한다

결론 we should initiate and actively promote long-distance degree programs like those at Omni 반박 원거리 학생 취득 프로그램 숫자가 증가한거하고 예산이 줄어드는 것 사이에 연관이 약하다 (causal 학생의 증가로 관리비용 증가할수 있음 원거리 수업가능 장비도입에의한 비용발생)bad analogy(omni university 하고 같은 조건이 아니다 )-gt omni college 가 강좌내용이 좋아서 학생의 등록이 많을수 있다 Humana 대학에서 만들었다 하더라도 인기 없을수 있음다른 요인에 의해서 Humana 대학의 수강 인원이 증가할수 있음(비록 과거엔 인기가 없었을지라도)

The following appeared as part of a business plan developed by the manager of the Rialto Movie Theater

Despite its downtown location the Rialto Movie Theater a local institution for five decades must make big changes or close its doors forever It should follow the example of the new Apex Theater in the mall outside of town When the Apex opened last year it featured a video arcade plush carpeting and seats and a state-of-the-art sound system Furthermore in a recent survey over 85 percent of respondents reported that the high price of newly released movies prevents them from going to the movies more than five times per year Thus if the Rialto intends to hold on to its share of a decreasing pool of moviegoers it must offer the same features as Apex

Write a response in which you discuss what questions would need to be answered in order to decide whether the recommendation is likely to have the predicted result Be sure to explain how the answers to these questions would help to evaluate the recommendation

Before following through this business plan the manager should investigate the cause of Rialtorsquos unsuccessful business

The author provides no evidence that the surveyrsquos results are statistically reliable The surveyrsquos sample of 85 percent must be sufficient in size and representative of overall population of the city where Rialto and Apex is serving Lacking evidence of a sufficiently representative sample the author cannot justifiably rely on the survey to draw any conclusion whatsoever The author does not indicate that Apex is indeed currently successful However even if Apex is enjoying success the argument relies on what might be a false analogy between Rialto and Apex In order for Apex to serve as a model that Rialto should emulate the author must assume that all relevant circumstances are essentially the same However this assumption is unwarranted For example the argument overlooks the face that Apex is located in a strategic placemdashbeside a mall where customers can not only watch a movie but also enjoy shopping Therefore simply changing the facility to that of Apex may not lead to success

The author does not mention whether Apex is successful or not Nevertheless even if Apex is currently successful the argument relies on what might be a false analogy between Rialto and Apex In order for Apex to serve as a model that Rialto should emulate the author must assume that all relevant circumstances are essentially the same However this assumption is unwarranted For example the argument overlooks the fact that these two institutions are located in different locations Rialto in downtown and Apex in a mall outside of town Although Apex opened with state-of-the-art facilities the decisive factor in its success could be due to its strategic location of being in a mall People could enjoy both shopping and movies at one location thus they may prefer Apex over Rialto Furthermore the place where people enjoy leisure activities has shifted in the past decades for most cities from downtown to the suburbs Therefore Rialto may not be successful even if it emulates Apexrsquos facilities A better business plan may be relocating Apex to the thriving section of the downtown

Rialto 극장은 지난 50여년간 지역 회관으로써 시내에 위치해 있으면서도 이제 변화를 꾀하지 않으면 문을 닫을

GRE AWA John박 박정어학원

판이다 이 극장은 시외 쇼핑타운에 새로 들어선 Apex 극장의 사례를 본받아야 했다 Apex가 지난해 개업했을 당시 이 극장은 비디오 아케이드 플러쉬 카펫트 바닥과 좌석 그리고 최신 음향시설을 갖추었다 더군다나 최근 조사에서는 응답자의 85 이상이 새로 출시된 영화 입장료가 비싼 탓으로 지난해보다 5배이상의 관람객이 줄어들었다고 나타났다 따라서 Rialto 극장이 줄어들고 있는 관람객을 뺐기지 않고 유지하려면 Apex와 같은 시설들을 갖추어야 할 것이다주장 리알토 극장이 줄어들고 있는 관람객을 뺐기지 않고 유지하려면 Apex와 같은 시설들을 갖추어야 할 것이다

1 조사에서 응답자가 전체를 대표할 수 없다 2 apex 극장이 좋은 시설을 갖추고 있지만 그로 인해 수익이 많이 발생했다는 말이 없으므로 시설투자를

하고도 좋은 결과를 얻을 수 있을지 그 근거가 미흡하다3 좋은 영화가 출시된다면 입장료가 비싸도 영화관에서 꼭 보려고 할 수 있다 4 rialto 가 시설이 아닌 다른 요인에 의해 장사가 안될수도 있다( 우범 지역이라든지)

The following is a recommendation from the business manager of Monarch Books

Since its opening in Collegeville twenty years ago Monarch Books has developed a large customer base due to its reader-friendly atmosphere and wide selection of books on all subjects Last month Book and Bean a combination bookstore and coffee shop announced its intention to open a Collegeville store Monarch Books should open its own in-store cafeacute in the space currently devoted to childrens books Given recent national census data indicating a significant decline in the percentage of the population under age ten sales of childrens books are likely to decline By replacing its childrens books section with a cafeacute Monarch Books can increase profits and ward off competition from Book and Bean

Write a response in which you examine the stated andor unstated assumptions of the argument Be sure to explain how the argument depends on these assumptions and what the implications are for the argument if the assumptions prove unwarranted

The following is a recommendation from the business manager of Monarch Books

Since its opening in Collegeville twenty years ago Monarch Books has developed a large customer base due to its reader-friendly atmosphere and wide selection of books on all subjects Last month Book and Bean a combination bookstore and coffee shop announced its intention to open a Collegeville store Monarch Books should open its own in-store cafeacute in the space currently devoted to childrens books Given recent national census data indicating a significant decline in the percentage of the population under age ten sales of childrens books are likely to decline By replacing its childrens books section with a cafeacute Monarch Books can increase profits and ward off competition from Book and Bean

1 Write a response in which you discuss what questions would need to be answered in order to decide whether the recommendation is likely to have the predicted result Be sure to explain how the answers to these questions would help to evaluate the recommendation

2 Write a response in which you discuss what specific evidence is needed to evaluate the argument and explain how the evidence would weaken or strengthen the argument

No evidence regarding Monarch Bookrsquos successEven if Regal Bookrsquos is successful this may not be attributable to the cafeacute False analogy Emulating may not lead to success Other factors may be involvedInsufficient condition The national census is not enough evidence that childrenrsquos book sales will decline Can

GRE AWA John박 박정어학원

the national census represent the local child populationDid opening a cafeacute boost sales for Regal Books Even assuming Regal is successful by opening a cafeacute this may not be suitable for Monarch which plans to close the childrenrsquos book section to establish a cafe Imprecise language ldquorelatively little spacerdquo how smallThe managerrsquos recommendation contradicts what he says Since Monarch is popular for its wide selection of books closing a selection which targets a major group of readers may hurt Monarchrsquos salesIs this the best way to compete

When Stanley Park first opened it was the largest most heavily used public park in town It is still the largest park but it is no longer heavily used Video cameras mounted in the parks parking lots last month revealed the parks drop in popularity the recordings showed an average of only 50 cars per day In contrast tiny Carlton Park in the heart of the business district is visited by more than 150 people on a typical weekday An obvious difference is that Carlton Park unlike Stanley Park provides ample seating Thus if Stanley Park is ever to be as popular with our citizens as Carlton Park the town will obviously need to provide more benches thereby converting some of the unused open areas into spaces suitable for socializing

Write a response in which you examine the stated andor unstated assumptions of the argument Be sure to explain how the argument depends on these assumptions and what the implications are for the argument if the assumptions prove unwarranted

Stanley 파크가 처음 개장했을 당시 가장 크고 가장 많이 이용되는 공원이었다 아직도 공원중에서는 가장 크지만 이용률은 상당히 떨어졌다 지난달 공원 주차장에 설치해놓은 비디오 카메라를 통해 보면 drop(주차장으로 여겨짐) 이용률이 가장 높았다 수치상으로는 하루 평균 50대의 차량만이 이용하였다 반면 직장 중심거리에 위치한 작은 규모의 Carlton 파크는 주당 무려 150여명 이상이 이용하고 있다 Stanley 파크와는 달리 Carlton 파크에는 의자가 있다는 것이 가장 뚜렷한 차이점이다 따라서 Stanley 파크가 Carlton 파크처럼 시민들이 자주 이용하는 공원이 되기 위해서는 벤치를 설치할 필요가 있으며 이렇게 사용되지 않는 일부 공간을 활용해서 사교를 위한 공간으로 바꾸어야 한다 ===gtdrop 에 대한 첨부사항 (영영사전내용입니다)---- a place or central depository to which something (as mail money or stolen property) is brought for distribution or transmission also the act of depositing something at such a place dropgt

주장 if Stanley Park is ever to be as popular with our citizens as is Carlton Park the town will obviously need to provide more benches thereby converting some of the unused open areas into spaces suitable for socializing1 조사가 언제 이루어진 것인가 조사가 언제 실시되었느냐에 따라 결과가 다를 수있다 현재는 다시 스탠리 파크가 늘어났었을 수 있다 2 벤치를 많이 설치했다고 해서 많은 관광객이 오지 않을수 있다(사람들이 벤치나 사교 공간을 원한다는 어떠한 자료도 없다)3스탠리 파크 주변에 교통 상황이 악화가 되었거나 칼튼 파크에서 문화행사등을 많이 가져서 이용객이 줄어든것일 수도 있다 4 칼튼 파크가 중심지에 있어서 접근성이 좋을수 있다5 조사가 같은 시간을 기준으로 한게 아니다(하나는 주중이고 하나는 주말이다)6사람의 수와 차의 대수를 같은것으로 비교할수 없다 (차안에 몇 명이 타고 있는지 모르고 대중교통을 이용해서 왔을수도 있다)

Page 10: GRE writing argument brain storm

GRE AWA John박 박정어학원

최근의 매상에 관한 조사에서 Bay 시에 있는 레스토랑의 해산물 요리의 소비가 지난 5년동안 30 증가했다고 보고되었다 그러나 현재 해산물을 전문으로 취급하고 있는 레스토랑이 없다 더군다나 대다수의 가정이 맞벌이 가정인데다가 전국 조사에서도 나와있듯이 이러한 맞벌이 가정은 10년전의 가정에서 했듯이 집에서 식사를 챙겨먹는 것이 급격하게 줄어 들었고 동시에 건강식과 관련한 지출이 늘고 있다 따라서 해산물 전문 레스토랑이 아주 인기를 끌것이고 그만큼 수익도 많을 것이다

1 해산물이 건강식인지에 대한 언급이 없다2 외식하는데 해산물 요리만 먹지 않을 것이다 집에서 챙겨먹지 않는다고 해산물을 먹는다고 할 수 없다3 현재 해산물 취급하는 식당이 현재까지 없다고 해서 계속 없을 수만은 없다 수익이 만약 늘어난다면

Bay시 주변의 도시의 레스토랑이 체인점을 내거나 새로운 식당이 생길 수 있고 오히려 수익이 줄어들 수도 있다

Scientists studying historical weather patterns have discovered that in the mid-sixth century Earth suddenly became significantly cooler Although few historical records survive from that time some accounts found both in Asia and Europe mention a dimming of the sun and extremely cold temperatures Either a huge volcanic eruption or a large meteorite colliding with Earth could have created a large dust cloud throughout Earths atmosphere that would have been capable of blocking enough sunlight to lower global temperatures significantly A large meteorite collision however would probably create a sudden bright flash of light and no extant historical records of the time mention such a flash Some surviving Asian historical records of the time however mention a loud boom that would be consistent with a volcanic eruption Therefore the cooling was probably caused by a volcanic eruption

Write a response in which you discuss what questions would need to be addressed in order to decide whether the conclusion and the argument on which it is based are reasonable Be sure to explain how the answers to the questions would help to evaluate the conclusion

Historical records may not be enough to explain the global cooling phenomenonThere may be reasons other than the two hypothesis to explain the global coolingA volcanic eruption big enough to produce dust clouds that envelop the earth to cause global cooling would have countless recordsThe author argues that a large meteorite collision is not a feasible explanation for the global cooling because there are no historical records that indicate a flash The authorrsquos logic is flawed in eliminating the meteorite hypothesis by claiming that there was no historical record of a bright flash of light According to common sense a meteorite collision big enough to produce dust that could envelop the earth would result in catastrophe even to the extinction of major species of animals

Loud boom would be insufficient evidence to conclude it was a volcanic eruption If could hear a loud boom there should be records of a volcanic eruption because the author claims that the dust cloud from this gigantic eruption was significant enough to cause global coolingCooling could have been caused by factors besides volcanic eruption and meteor collisionThe absence of historical records that indicate a bright flash of light Collision could have occurred at a place where there no witnesses Could have fell out in the seaBoom might have been caused by things other than a volcanic eruptionHistorical records from Asia and Europe may not be applicable to earth as a whole

과거의 날씨 주기를 연구하는 과학자들은 6세기 중엽 지구가 갑자기 혹한기가 되었던 것을 알게되었다 이 당시의 몇몇 기록들이 아직도 남아있긴 하지만 아시아와 유럽에서 발견되는 몇가지 자료에서 태양 빛의 수축과 그에따른 엄청난 추위가 있었음을 언급하고 있다 거대한 화산 폭발이나 거대 운석의 지구 충돌중 어느것에

GRE AWA John박 박정어학원

의해서든 엄청난 먼지구름을 형성시켜 지구전체에 태양빛을 차단해서 지구의 기온을 뚝 떨어뜨릴수 있을 수도 있다 그러나 이러한 운석 충돌은 순간 섬광을 발산시키게 될 것인데 이 시기의 어느 기록에서도 이러한 섬광은 언급되지 않고 있다 그러나 이 시기에 남아 있는 일부 아시아의 기록문서에서는 연속적으로 화산폭발음일수 있는 엄청난 폭발음이 있었음을 기록하고 있다 따라서 이러한 혹한은 아마도 화산 폭발에 의한 것이었을 것이다

결론 the cooling was probably caused by a volcanic eruption

반박 자료의 부족( 몇몇 자료가지고 그 당시의 기후를 예상하기에는 무리가 있다)다른 원인에 의해서 기후가 떨어졌을수도 있다 (빙하기 다른 기상원인)기록되지 않았다고 해서 그 사실이 없었던 것은 아니다(기록은 했으나 자료가 없어졌을수도 있고 모든 사실이 전부다 기록되지는 않는다 )기록된 폭발음이 꼭 확산 폭발음이 아닐수도 있다( 화산 폭발음이 너무 먼거리여서 들리지 않을수도 있고 다른 소리가 기록된 것이였을수도 있다)부족한 자료를 신빙할수 있는가 기록의 소멸도 예상 할수 있다 실제로 운석이 떨어졌는데 안들렸을수 있다

The following appeared in a memo from the president of Bower Builders a company that constructs new homes

A nationwide survey reveals that the two most-desired home features are a large family room and a large well-appointed kitchen A number of homes in our area built by our competitor Domus Construction have such features and have sold much faster and at significantly higher prices than the national average To boost sales and profits we should increase the size of the family rooms and kitchens in all the homes we build and should make state-of-the-art kitchens a standard feature Moreover our larger family rooms and kitchens can come at the expense of the dining room since many of our recent buyers say they do not need a separate dining room for family meals

Write a response in which you examine the stated andor unstated assumptions of the argument Be sure to explain how the argument depends on these assumptions and what the implications are for the argument if the assumptions prove unwarranted

The presidentrsquos first unstated but apparent assumption is that a nationwide survey can be applied to local areas There is no guarantee that the area in which Bower Builders builds houses will follow the national consumer trend Since he only mentions the overall result of the nationwide survey it is difficult to determine more specific conditions such whether people in urban and rural areas have the same housing preferences Such uncertainty is further exacerbated by the nature of the survey question itselfmdash it does not deal with current trends in actual home purchases but with desired home features The naiumlve assumption that consumer desire will directly result in consumer action underlies the presidentrsquos interpretation and application of the survey results to his company plan Not everyone who wishes for a large family room and kitchen will or can actually buy a house with those features mainly due to financial reasons Furthermore the president also assumes for no evident reason that consumers will not hesitate to purchase houses with state-of-the-art kitchens as a standard rather than optional feature when it is clear that such a feature will raise the overall cost significantly He continues to blunder in his mistaken assumptions about consumer behavior in assuming that the taste of recent buyers can represent the concern of future prospective buyers The fact that recent buyers have claimed no need for separate dining rooms does not mean future buyers will feel the same way as well

The president of Bower Builders recommends that in order to raise company profits the company should build houses with large family rooms and large state-of-the-art kitchens He uses the results of a nationwide survey on desired home features and the example of Bower Buildersrsquo competitor Domus Construction to support his plan His memo manifests several assumptions about surveys consumer behavior and Domus Constructionrsquos houses that do not necessarily bolsterbuttress his argument

GRE AWA John박 박정어학원

The writer assumes 1 the nationwide survey results can be applied to the local area (Desire cannot reflect consumer trend) 2 Domus Construction was profitable because they had such features (The writer should investigate what other features the DC provides and the actual number of homes sold) 3 State-of-the-art kitchens will sell well (no evidence to buttress his assumption furthermore they would need to spend more money which would increase the price of the housesmdashtheir market will be limited to the affluent) 4 The recent buyers represent the concern of most buyersmdashthat they are ok with a house that is without a dining room

Nationwide survey might not be applicable to this regionDomus Construction could have other superior qualities than Bower Builders Ex DesignSelling fast doesnrsquot indicate quantitySmall yards complaints could be voiced in the future

First the author concludes that a nationwide survey reveals that the two most desired home features are a bathroom with a whirlpool tub and a large kitchen However this nationwide survey may not reflect the trends of the customers that Bower Builders target The author assumes that the nationwide trend reflects customer trends The national study would lead support only if the nationwide trend reflect the home-purchasing trends The nationwide trend may just be a trend of desire which does not result in the act of purchasing However the author does not provide credible evidence that this is the case

전국 조사에서 가장 인기있는 집안 구조 2가지는 훨풀 튜브가 마련된 욕실과 커다른 부엌으로 조사되었다 경쟁업체 Domus Construction이 신축한 완공예정인 주택은 이러한 욕실을 갖추고 있어서 분양이 조기에 이루어지고 있고 가격도 평균치보다 상당히 웃돌고 있다 우리도 집을 많이 팔도 그만큼 이윤을 많이 남기려면 신규 주택에는 위의 2가지 사항을 갖추어야 한다 최근 우리가 지은 주택 수요자들이 적은 평수에 대해서는 별다른 불만사항이 없었기 때문에 앞으로 평수를 더 줄여서 이익을 극대화 할 수 있을 것이다

주장 We should include whirlpool tubs and a large kitchen

1 경쟁업체 Domus Construction이 신축한 완공예정인 주택은 이러한 욕실을 갖추고 있어서라기 보다는 위치가 좋거나 다른 마감재(construction material)나 인테리어가 뛰어나서 분양이 조기에 이루어진것이라고 볼 수 있다 2 전국조사가 건물이 지어지는 지역에 항상 적용되리라고 볼 수 없다 3 평수를 줄이는 것에 불만이 없었다는 것은 editor의 견해일 수 있다 사람들이 표현하지 않은 것일 수도 있고 다른 장점이 많아서 그런 단점을 드러내지 않은 것일 수 있기 때문에 속단할 수 없다 4 다른것을 함으로써 더 잘 팔릴수도 있다

2011 7 19 화요일 GRE AWA 실전반이OO

Argument 94

The president of Bower Builders contends recommends that in order to raise company profits the company should build houses with large family rooms and large state-of-the-art kitchens He uses the results of a nationwide survey on desired home features and the example of Bower Buildersrsquo competitor Domus Construction to support his plan His memo manifests several assumptions about surveys consumer behavior and Domus Constructionrsquos houses that do not

GRE AWA John박 박정어학원

necessarily bolsterbuttress his argumentThe presidentrsquos first unstated but apparent assumption is that a nationwide survey can be

applied to local areas There is no guarantee that the area in which Bower Builders builds houses will follow the national consumer trend Since he only mentions the overall result of the nationwide survey it is difficult to determine more specific conditions such whether people in urban and rural areas have the same housing preferences Such uncertainty is further exacerbated by the nature of the survey question itselfmdash it does not deal with current trends in actual home purchases but with desired home features The naiumlve assumption that consumer desire will directly result in consumer action underlies the presidentrsquos interpretation and application of the survey results to his company plan Not everyone who wishes for a large family room and kitchen will or can actually buy a house with those features mainly due to financial reasons Furthermore the president also assumes for no evident reason that consumers will not hesitate to purchase houses with state-of-the-art kitchens as a standard rather than optional feature when it is clear that such a feature will raise the overall cost significantly He continues to blunder in his mistaken assumptions about consumer behavior in assuming that the taste of recent buyers can represent the concern of future prospective buyers The fact that recent buyers have claimed no need for separate dining rooms does not mean future buyers will feel the same way as well

In addition the president finds a real-life actualization of the nationwide survey results in the recent sales of Domus Construction However he easily assumes that large family rooms and kitchens are the only reasons the houses of Domus Construction sell well He does not take into account other features and selling points of the competitorrsquos houses There may well be other explanations for its houses selling more quickly and expensively including additional home features as well as external factors such as proximity to better schools or superior financial solvency of its clientele If Bower Builders merely added larger family rooms and kitchens without taking into consideration the other factors they may lose rather than gain profits

Clearly the presidentrsquos assertion that Bower Builders make houses with large family rooms and high-tech kitchens at the expense of dining rooms rests on a number of assumptions that are ill-informed and naiumlve If Bower Builders undertakes the proposed plan without further research into local consumer desires purchasing trends and the marketing and sales of competing companies the company will risk losing money by building big new houses that people cannot afford to or will not wish to buy

The following appeared in a letter to the editor of a journal on environmental issues

Over the past year the Crust Copper Company (CCC) has purchased over 10000 square miles of land in the tropical nation of West Fredonia Mining copper on this land will inevitably result in pollution and since West Fredonia is the home of several endangered animal species in environmental disaster But such disasters can be prevented if consumers simply refuse to purchase products that are made with CCCs copper unless the company abandons its mining plans

Write a response in which you examine the stated andor unstated assumptions of the argument Be sure to explain how the argument depends on these assumptions and what the implications are for the argument if the assumptions prove unwarranted

The writer assumes 1 The writer is trying to avoid an inevitability 2 Mining copper will result pollution (they could make preventative measures) 3 The writer may be too late from stopping CCC from developing the area into a copper mine 4 Since mining is an underground enterprise the surface may not be affected that much therefore endangered species may not be affected 4 Consumers wonrsquot buy CCC products if the journal publishes a negative review about CCC (How many readers) CCC could a company that has ties with many IT companies and industries in that their copper is almost ubiquitous in various products

GRE AWA John박 박정어학원

지난 한해동안 CCC(Consolidated Copper 회사)는 서부 플로리다의 열대 지역에 1백만 마일이 넘는 땅을 사들였다 이 곳에서의 채광 활동은 서부 플로리다가 몇몇 멸종 위기에 처한 동물의 서식지이기 때문에 분명 오염과 환경파괴를 가져올 것이다 그러나 이러한 파괴는 CCC 회사가 채광을 포기할 때까지 이 회사가 채굴한 구리로 제조된 제품을 구매하지 않으면 막을 수 있을 것이다

결론 such disaster can be prevented if consumers simply refuse to purchase products that are made with CCCs copper until the company abandons its mining plans

1 구리로 제조된 물건이 생활에 필수적인 것이 많은 만큼 불매가 쉽지 않을 수 있다(전선이나 각종 전자제품에 필수적으로 들어가기 때문에)

2 불매를 유도한다고 해서 소비자들이 구매를 안하는 것은 아니다3 적절한 채굴로 환경파괴를 가져 오지 않을 수 있다 (땅속에 있는 물질을 채굴하는 만큼 생물에 영향을 안

미칠 수도 있다)4 이미 채굴이 다 끝나서 더 이상의 채굴이 없을 수도 있다 5 땅을 구입한다고 해서 채광하는건 아니다 (다른 용도로 샀을수도 있다)

The following is a letter to the editor of an environmental magazine

In 1975 a wildlife census found that there were seven species of amphibians in Xanadu National Park with abundant numbers of each species However in 2002 only four species of amphibians were observed in the park and the numbers of each species were drastically reduced There has been a substantial decline in the numbers of amphibians worldwide and global pollution of water and air is clearly implicated The decline of amphibians in Xanadu National Park however almost certainly has a different cause in 1975 troutmdashwhich are known to eat amphibian eggsmdashwere introduced into the park

Write a response in which you discuss what specific evidence is needed to evaluate the argument and explain how the evidence would weaken or strengthen the argument

Evidence needed the identity of the sender and the census taker of rsquo75 and lsquo02mdasha scientist or an environmentalist credibility issue census methodology vs mere observationmdashthe absence of evidence is not an evidence of absence the season of when the census was taken time-shiftmdashconditions may have changed worldwide decline may include Xanadu other species of predators that prey on amphibians because trout is only one species that prey on amphibians the number of troutmdashhave they increased significantly since rsquo75 First the author needs to be more overt about the credibility of the census and observation There were to accounts that notes the population of amphibiansmdashthe first a census and the second an observation The author needs to bolster his conclusion with the evidence that indicate that the census and second observation were done using scientific methodologies This could either could strengthen or weaken his claim In addition he

GRE AWA John박 박정어학원

needs to provide the specific season of when the census and observation occurred In this letter the writer is informing an editor of an environmental magazine that the number of amphibians was greatly reduced since 1975 and he points out the introduction of trout as the only reason for the decline However the author fails to provide crucial evidences that could strengthen or weaken his conclusion

The following appeared in a memorandum from the president of Hyper-Go Toy Company

Last year sales of our Fierce Fighter toy airplane declined sharply even though the toy had been a top seller for three years Our customer surveys show that parents are now more worried about youthful violence and are concerned about better education for their children Therefore to maintain profits we should discontinue all our action toys and focus exclusively on a new line of educational toys Several other toy companies have already begun marketing educational toys and report sales increases last year of 200 percent And since the average family income is growing sales of new Hyper-Go toys should also increase

Write a response in which you discuss what specific evidence is needed to evaluate the argument and explain how the evidence would weaken or strengthen the argument

First the president indicates that the sales of Fierce Fighter toy airplane declined sharply However he fails to consider the fact that toys are a fad Since FFT enjoyed a three year success it may be natural that the trend would subsideSecond Customer survey is this representative of most toy consumers This trend is not newThird other companies may have profited not through educational but other toys Or educational toy profit may be small in proportion to their sale of other toys 200Parents are not the customers companies should concentrate on what the children want to increase profitLastly parents worry about youthful violence and concern for better education are not new trends

The presidentrsquos decision to discontinue all action toys and focus only on educational toys is too extreme If this decision is taken into effect its procedure may be an onerous task because it would require major shifts in human resources and company image Therefore further data should be considered before following up on this decision

우리 회사에서 지난 3년간 최고 매출을 일으켰던 Fierce Fighter 장난감 비행기가 지난해에는 매출이 급격히 떨어졌다 자체 고객 조사에서는 부모들이 현재 청소년 폭력에 걱정을 하고 있어서 아이들의 양질 교육에 더 관심을 가지고 있는 것으로 나타났다 따라서 회사의 수익을 유지하기 위해서는 모든 자사 전투용 장난감 생산을 중단하고 오로지 교육적인 장난감 생산에 집중해야 한다 몇몇 여타 장난감 회사들도 이미 교육용 장난감 마케팅을 시작해서 지난해에는 200의 매출신장을 가져왔다고 한다 그리고 평균 가계 수입이 점점 늘고 있기 때문에 신형 Hyper-Go 장난감의 매출도 늘어날 것이다

3그룹 Time-shift Error

Woven baskets characterized by a particular distinctive pattern have previously been found only in the immediate vicinity of the prehistoric village of Palea and therefore were believed to have been unique to the Palean people Recently however archaeologists discovered such a Palean basket in Lithos an ancient village across the Brim River from Palea The Brim River is very deep and broad and so the ancient Paleans could only have crossed it by boat but there is no evidence that the Paleans had boats And boats capable of carrying groups of people and cargo were not developed until thousands of years after the Palean people disappeared Moreover Paleans would have had no need to cross the rivermdashthe woods around Palea are full of nuts berries and small game It follows that the so-called Palean baskets were not unique to Palea

GRE AWA John박 박정어학원

Write a response in which you discuss what specific evidence is needed to evaluate the argument and explain how the evidence would weaken or strengthen the argument

RefutationPrehistoric time shift-gtbrim river could have been narrow and shallow or it might have not existed Indigenous patterns may exist in other disconnected remote placesNuts berries small game(hunting) may not have existed or the author should be proved these existed at that time Abundance of resources doesnrsquot support the reason for seclusionBoats not yet found baskets may have been carried across by the river current without the help of a boat Lithos might have crossed the river for commercial purposeOne Palean basket does not substantiate the authorrsquos claimThe absence of evidence is not an evidence of absence

The author assumes without justification that present conditions are the same as at the prehistoric era The author unfairly infers from the presence of Brim River which exist today that it would have existed in the past However the author fails to offer any evidence to substantiate this inference It is very likely that the Brim River might not have existed in prehistoric times or if it did exist may have been shallow and narrow enough for the Paleans to easily cross Any of these scenarios if true would serve to undermine the claim thathelliphelliphelliphelliphellip

First the author claims that the Brim River was very deep and broad so the Paleans could not have crossed it However the author fails to offer any evidence to substantiate that this was true in the prehistoric time of the Paleans For all we know the Brim River might not have existed in the prehistoric era or if it did exist could have been a shallow and narrow river For example scientists believe that thousands of years ago an ice-bridge existed on the Bering Sea connecting Eurasia to what is now North America to explain how the Eskimos and the Asian inhabitants of America came to migrate all over the Americas In this example the absence of evidence is not an evidence of absence Therefore to sufficiently support his claim the author needs to substantiate the fact that the Brim River really did exist and was broad and wide in the Prehistoric Era proving that the Paleans could not have influenced or traded with other groups of people

First the author claims that the Brim River was very deep and broad so the Paleans could not have crossed it However the author fails to offer any evidence to substantiate that this was true in the prehistoric times of the Paleans For all we know the Brim River might not have existed in the prehistoric era or if it did exist could have been a shallow and narrow river For example rivers are created by natural erosion over thousands of years Niagara Falls carved its way from the mouth of Lake Ottawa and created a long river Likewise the current Brim Riverrsquos physical features may not have been wide and deep Therefore to sufficiently support his claim the author needs to substantiate the fact that the Brim River really did exist and was broad and wide in the Prehistoric Era proving that the Paleans could not have influenced or traded with other groups of people

예전에는 실로짠 특이한 무늬 바구니가 Palea의 선사시대 지역의 인근마을에서만 발견되어왔기 때문에 Palea 마을 사람들의 특징이라고 여겨졌었다 그러나 최근들어 고고학자들이 Lithos지역에서 Palean 바구니를 발견하였는데 그 당시 지역은 Brim 강을 가로질러 Palea까지 닿아있었다 이 강은 수심이 아주 깊고 강폭이 넓었으며 때문에 고대의 Palea인들은 배를 이용해서 강을 건널수 있었을 것이다 그러나 이들이 배를 가지고 있었다는 증거는 발견되지 않고 있다 더군다나 이들이 멸명한 이후 수천년이 지난뒤에도 수많은 물자와 사람을 실어 나를수 있는 용적을 가진 배는 개발되지 않았다 이와더불어 Palea인들은 강을 건널필요가 없었는데 그것은 너트나무 장과열매 그리고 작은 사냥감들이 주변숲에 풍부했기때문이다 따라서 Palean 바구니라고 하는 것도 Palea인들만의 전유물이 아니라는 결론을 얻을 수 있다

결론 if follows that the so-called Palean baskets were not unique to Palea이번문제는 굿이 causal Bad analogy 로 구분해서 찾기가 힘드내요 배를 발견했다는 증거가 없는것이다 (앞으로도 발견될수 있음)

GRE AWA John박 박정어학원

계절의 영향으로 겨울에 얼음이 두껍게 언다든지 여름에 가뭄으로 인해서 건널수 있다물자가 풍부한 것이 이동하지 않을 조건이 아니다 다른 것에 의해서 이동가능(의약품등)

Thirteen years ago researchers studied a group of 25 infants who showed signs of mild distress when exposed to unfamiliar stimuli such as an unusual odor or a tape recording of an unknown voice They discovered that these infants were more likely than other infants to have been conceived in early autumn a time when their mothers production of melatonin hormone known to affect some brain functions would naturally increase in response to decreased daylight In a follow-up study conducted earlier this year more than half of these children now teenagers who had shown signs of distress identified themselves as shy Clearly increased levels of melatonin before birth cause shyness during infancy and this shyness continues into later life

Write a response in which you examine the stated andor unstated assumptions of the argument Be sure to explain how the argument depends on these assumptions and what the implications are for the argument if the assumptions prove unwarranted

Any baby exposed to unpleasant stimuli would react in such wayFirst of all the author states 25 infants as his evidence However this research sample is too small to prove his claimSecond the author states that 25 infants were conceived in early autumn which he claims lead to a shy disposition However this is faulty evidenceThird the research study was a long term study done in the span of 13 years However the author only writes about the initial and final stages of the study and leaves out evidences of what could have happened during the 13 years which could be more evidential factors of influenceFourth neither the infantsrsquo genetic predisposition nor their environment were taken into accountFinally the author concludes that his shyness continues into later life (Other factors could alter this disposition epigenetic theory)

13 년전 학자들은 25명의 유아를 대상으로 이상한 냄새나 특이한 소리를 녹음한 테잎등으로 낯선 자극을 주었을때 보이는 미미한 압박감 증상을 조사하였다 이들은 성숙기가 막 지났을 즈음에 보통의 유아들이 비슷한 증상을 보이는 정도 보다는 다소 민감한 반응을 보였는데 이 시기는 아이의 엄마가 뇌의 일부 기능에 영향을 미치는 것으로 알려진 멜라토니아 호르몬을 생산하는 시기로써 이 호르몬은 낯 시간이 짧을때 자연적으로 증가할 수도 있다 금년초에 실시된 추가연구에서 현재 10대로 성장한 당시 조사대상의 절반 이상의 아이들이 부끄럼을 잘타는 것으로 여기고 있었다 따라서 분명한 것은 출산전 멜라토닌 수치의 증가가 유아기에 수줍음 등의 영향을 미치게 되며 이러한 영향이 성장후에도 작용한다는 것이다

주장 Clearly increased levels of melatonin before birth cause shyness during infancy and this shyness continues into later life

1 25명의 아기로 결론 내리기에 샘플이 작다2 과학적 사실들에 대한 명확한 근거가 엇음3 다른 영향을 간과했다(다른 호르몬에 의한 영향 후천적인 성격형성의 영향)

GRE AWA John박 박정어학원

The following is a letter to the editor of the Atticus City newspaper

Former Mayor Durant owes an apology to the city of Atticus Both the damage to the River Bridge which connects Atticus to Hartley and the traffic problems we have long experienced on the bridge were actually caused 20 years ago by Durant After all he is the one who approved the construction of the bridge If he had approved a wider and better-designed bridge on which approximately the same amount of public money would have been spent none of the damage or problems would have occurred Instead the River Bridge has deteriorated far more rapidly over the past 20 years than has the much longer Derby Bridge up the river Even though the winters have been severe in the past several years this is no excuse for the negligence and wastefulness of Durant

Write a response in which you discuss what questions would need to be answered in order to decide whether the recommendation is likely to have the predicted result Be sure to explain how the answers to these questions would help to evaluate the recommendation

전임 시장인 Durant 씨는 Atticus 시에 대해 사과할 의무가 있습니다 Atticus와 Hartley를 잇는 River Bridge 교량에 대한 피해와 이 교량에서 오랫동안 주민들이 겪어오고 있는 교통 혼잡 문제들은 실제로 20년 전부터 시작된 것이었습니다 결정적으로 그가 교량 공사를 허가했던 바로 그 장본인입니다 당시 비슷한 공사비용으로 폭이 더 넓고 튼튼하게 설계된 교량을 허가했다면 이러한 문제나 피해는 발생하지 않았을 겁니다 더군다나 이 다리는 지난 20년 동안 상류에 건설된 훨씬 오래된 Derby 다리보다도 빠르게 부식되어 갔습니다 지난 수년동안 심지어 혹한이 있었다 하더라도 이러한 태만과 국고 손실에 대한 책임을 회피할 길이 없는 것입니다

결론 Former Mayor Durant owes an apology to the city of Atticus

1 디자인이 문제가 아닐수 있다 (디자인은 좋았으나 건설과정에 문제가 있었을 수 있다)2 그 당시의 시예산이 적어서 더 큰 다리를 짓기가 불가능했을 수도 있다3 교통량이 많거나 다른 상황으로 인해서 부식이 빨리 됐을 수 있다4 그 당시에는 최선의 선택이였지만 갑자기 변한 상황에 의해서 이런 문제점들이 발생했을 수 있다

GRE AWA John박 박정어학원

4그룹 거짓인과관계 오류 (False Cause) 빈출

Fifteen years ago Omega University implemented a new procedure that encouraged students to evaluate the teaching effectiveness of all their professors Since that time Omega professors have begun to assign higher grades in their classes and overall student grade averages at Omega have risen by 30 percent Potential employers looking at this dramatic rise in grades believe that grades at Omega are inflated and do not accurately reflect student achievement as a result Omega graduates have not been as successful at getting jobs as have graduates from nearby Alpha University To enable its graduates to secure better jobs Omega University should terminate student evaluation of professors

Write a response in which you discuss what specific evidence is needed to evaluate the argument and explain how the evidence would weaken or strengthen the argument

Omega professor evaluation implemented 15 years ago =gt Omega prof assign higher grades 30Employers believe therersquos grade inflation

Thus unsuccessful employment than AlphaTherefore to secure jobs Omega should end evaluating profs

Specific evidence neededRelationship between higher grades and evaluationRelationship between GPA and unsuccessful employmentAlpharsquos education could just be better than OmegaldquoFifteen years agordquo is a long time other factors could have influenced Why is the inflation a problem just now How much is Alpha better Is the comparison just How much gap is thereOmegarsquos student could just be doing better in their studiesComparison to other universities다른 대안 없나hellip Could Omega alleviate the employment problem by implementing a different procedure or program

15 년전 우리 대학은 학생들로 하여금 교수평가를 하도록 한 새로운 조치를 시행했었습니다 이후 교수들은 자신의 학과 학생들에게 높은 학점을 주었으며 그에따라 학생들의 전체 평점이 30나 올랐습니다 외부의 기업체들은 분명 점수가 지나치게 부풀려졌다고 믿고 있습니다 결국 본 대학 졸업생들이 인근 Alpha 대학의 졸업자들보다 구직률이 떨어지는 이유를 잘 보여주고 있는 것입니다 이를 해결하기 위해 이제부터는 학생들에 의한 교수평가제를 중단해야 합니다

결론 Omega University should now terminate student evaluation of professors

반박 교수 평가와 학점 인플레의 연관성이 적다( 교수 평가를 먼저하고 학점을 나중에 매길수도 있다)채용기준에 성적만 있는게 아니다 학업성취의 결과 일수도 있다 Alpha 가 원래 유능했다 Alpha 의 교육내용이 좋았다

GRE AWA John박 박정어학원

In this memo the dean of Omega University(OU) recommends OU to terminate professor evaluation to secure better jobs for the students To support this recommendation the dean offers several reasons However this argument contains several logical flaws which render it unconvincing

A threshold problem with the argument involves the voluntary nature of the evaluationprocedure The dean provides no evidence about the number or percentage of Omegastudents who participate in the procedure Lacking such evidence it is entirely possible thatthose numbers are insignificant in which case terminating the procedure is unlikely to haveany effect on the grade average of Omega students or their success in getting jobs aftergraduationThe argument also assumes unfairly that the grade-average increase is the result of theevaluation procedure--rather than some other phenomenon The dean ignores a host of otherpossible explanations for the increase--such as a trend at Omega toward higher admissionstandards or higher quality instruction or facilities Without ruling out all other possibleexplanations for the grade-average increase the dean cannot convince me that by terminatingthe evaluation procedure Omega would curb its perceived grade inflation let alone help itsgraduates get jobsEven if the evaluation procedure has resulted in grade inflation at Omega the deans claimthat grade inflation explains why Omega graduates are less successful than Alpha graduatesin getting jobs is unjustified The dean overlooks a myriad of other possible reasons forOmegas comparatively poor job-placement record Perhaps Omegas career services areinadequate or perhaps Omegas curriculum does not prepare students for the job market aseffectively as Alphas In short without accounting for other factors that might contribute toOmega graduates comparative lack of success in getting jobs the dean cannot justify theclaim that if Omega curbs its grade inflation employers will be more likely to hire OmegagraduatesFinally even if the dean can substantiate all of the foregoing assumptions the deansassertion that Omega must terminate its evaluation procedure to enable its graduates to findbetter jobs is still unwarranted in two respects First the dean ignores other possible ways bywhich Omega can increase its job-placement record--for example by improving its publicrelations or career-counseling services Second the dean unfairly equates more jobs withbetter jobs In other words even if more Omega graduates are able to find jobs as a result ofthe deans recommended course of action the kinds of jobs Omega graduates find would notnecessarily be better onesIn sum the deans argument is unpersuasive as it stands To strengthen it the dean mustprovide better evidence that the increase in grade average is attributable to Omegasprofessor-evaluation procedure and that the end result is a perception on the part ofemployers that Omega graduates are less qualified for jobs than Alpha graduates To betterassess the argument I would need to analyze 15-year trends in (l) the percentage of Omegastudents participating in the evaluation procedure (2) Omegas admission standards andquality of education and (3) Omegas emphasis on job training and career preparation I wouldalso need to know what other means are available to Omega for enabling its graduates to findbetter jobs

GRE AWA John박 박정어학원

The following appeared in a memo from a vice president of Quiot Manufacturing

During the past year Quiot Manufacturing had 30 percent more on-the-job accidents than at the nearby Panoply Industries plant where the work shifts are one hour shorter than ours Experts say that significant contributing factors in many on-the-job accidents are fatigue and sleep deprivation among workers Therefore to reduce the number of on-the-job accidents at Quiot and thereby increase productivity we should shorten each of our three work shifts by one hour so that employees will get adequate amounts of sleep

Write a response in which you examine the stated andor unstated assumptions of the argument Be sure to explain how the argument depends on these assumptions and what the implications are for the argument if the assumptions prove unwarranted

The following appeared in a memo from a vice president of Alta Manufacturing

During the past year Alta Manufacturing had thirty percent more on-the-job accidents than nearby Panoply Industries where the work shifts are one hour shorter than ours Experts believe that a significant contributing factor in many accidents is fatigue caused by sleep deprivation among workers Therefore to reduce the number of on-the-job accidents at Alta we recommend shortening each of our three work shifts by one hour If we do this our employees will get adequate amounts of sleep

Write a response in which you discuss what questions would need to be answered in order to decide whether the recommendation and the argument on which it is based are reasonable Be sure to explain how the answers to these questions would help to evaluate the recommendation

The following appeared in a memo from the vice president of Butler Manufacturing

During the past year workers at Butler Manufacturing reported 30 percent more on-the-job accidents than workers at nearby Panoply Industries where the work shifts are one hour shorter than ours A recent government study reports that fatigue and sleep deprivation among workers are significant contributing factors in many on-the-job accidents If we shorten each of our work shifts by one hour we can improve Butler Manufacturings safety record by ensuring that our employees are adequately rested

1 Write a response in which you discuss what specific evidence is needed to evaluate the argument and explain how the evidence would weaken or strengthen the argument

2 Write a response in which you discuss what questions would need to be answered in order to decide whether the recommendation is likely to have the predicted result Be sure to explain how the answers to these questions would help to evaluate the recommendation

4번 반복됨

Alta has 30 more job accidents than Panoply(work shifts one hour shorter)Experts Job accidents caused by fatigue and sleep deprivationTherefore to reduce job accidents and increase productivity shorten three work shifts by one hour for adequate sleep

지난해 우리 회사는 인근 Panoply Industries보다 업무상 재해가 30나 더 많았다 그 회사는 우리보다 근무 교대시간이 1시간 정도 짧았다 전문가들은 대부분의 업무상 재해에 있어서 가장 중요한 요인이 과로와 수면부족으로 보고있다 따라서 우리 회사에서 높은 산업재해를 줄이고 아울러 생산성을 높이기 위해서는 근로자들이 충분한 수면을 취할 수 있도록 1시간씩 3교대 시간을 줄여야 한다

In this memo the (author) vice president of Alta Manufacturing (AM) recommends that to reduce on-the-job accidents and increase productivity AM should shorten its three work shifts by one hour so that employees can

GRE AWA John박 박정어학원

get more sleep To support this recommendation the author provides several evidences However careful scrutiny of each of the facts reveals that it provides little credible support for the authorrsquos recommendation QuestionsThe number of accidents What kind of accidents The seriousness of the accidents is importantHow many employees are in each company What are their productsFalse cause Sleep may not be the reason for the on-the-job accidents What do Alta and Panoply manufacture

First of all the author believes that fatigue caused the on-the job accidents However there could be other reasons The author observes a correlation between sleep deprivation and on-the-job accidents then concludes that the former is the cause of the latter However the author fails to rule out other possible explanations For example it is entirely possible that Alta factories require more strenuous and dangerous labor than Panoply Without ruling out all other such factors it is unfair to conclude that fatigue is responsible for the accidents In addition the work-shifts may not be the cause of the sleep deprivation and fatigue It is possiblehellip Thus the author should provide what exactly Panoply and Alta manufacture and more precise data about their working conditions to be more convincing

Shortening the shift by one hour does not necessarily lead to more sleep And is one hour enoughLess accidents does not mean increased productivity

결론 We should shorten each of out three work shifts by one hour

반박 경쟁사에 비해서 시간당 하는 업무량이 많아서 더욱 피곤할 수도 있다 시간이 문제가 아니라 노후된 시설 설비 자체의 문제 작업 자체가 원래 위험한 것이여서 사고가 많을 수도 있다 다른 회사는 더욱 많은 작업시간에도 불구하고 안정한 작업여건으로 인해서 사고율이 오히려 더 작을 수도 있다비교사의 재해감소가 다른 요인일수 있다(안전 교육 철저)줄인 시간이 피로회복이나 수면으로 연결 안될수 있음(술을 마실 수도 있고 그 시간에 휴식을 취하지 않고 다른일을 함으로써 더욱 피로해질수 있다)

This editorial recommends that Alta Manufacturing reduce its work shifts by one hour each inorder to reduce its on-the-job accident rate and thereby increase Altas productivity To supportthis recommendation the author points out that last year the number of accidents at Alta was30 greater than at Panoply Industries where work shifts were one hour shorter The authoralso cites certain experts who believe that many on-the-job accidents are caused by fatigueand sleep deprivation I find this the argument unconvincing for several reasonsFirst and foremost the author provides absolutely no evidence that overall workerproductivity is attributable in part to the number of on-the-job accidents Although commonsense informs me that such a relationship exists the author must provide some evidence ofthis cause-and-effect relationship before I can accept the authors final conclusion that theproposed course of action would in fact increase Altas productivitySecondly the author assumes that some accidents at Alta are caused by fatigue or sleepdeprivation However the author overlooks other possible causes such as inadequateequipment maintenance or worker training or the inherent hazards of Altas manufacturingprocesses By the same token Panoplys comparatively low accident rate might be attributablenot to the length of its work shifts but rather to other factors such as superior equipmentmaintenance or worker training In other words without ruling out alternative causes ofon-the-job accidents at both companies the author cannot justifmbly conclude that merely byemulating Panoplys work-shift policy Alta would reduce the number of such accidentsThirdly even assuming that Altas workers are fatigued or sleep-deprived and that this is thecause of some of Altas on-the-job accidents in order to accept the authors solution to thisproblem we must assume that Altas workers would use the additional hour of free time tosleep or rest However the author provides no evidence that they would use the time in thismanner It is entirely possible that Altas workers would use that extra hour to engage in someother fatiguing activity Without ruling out this possibility the author cannot convincinglyconclude that reducing Altas work shifts by one hour would reduce Altas accident rateFinally a series of problems with the argument arise from the scant statistical information onwhich it relies In comparing the number of accidents at Alta and Panoply the author fails toconsider that the per-worker accident rate might reveal that Alta is actually safer than Panoplydepending on the total number of workers at each company Second perhaps accident rates

GRE AWA John박 박정어학원

at the two companies last year were aberrations and during other years Altas accident ratewas no greater or even lower than Panoplys rate Or perhaps Panoply is not representativeof industrial companies generally and that other companies with shorter work shifts have evenhigher accident rates In short since the argument relies on very limited statistical information Icannot take the authors recommendation seriouslyIn conclusion the recommendation for emulating Panoplys work-shift policy is not wellsupported To convince me that shorter work shifts would reduce Altas on-the-job accidentrate the author must provide clear evidence that work-shift length is responsible for some ofAltas accidents The author must also supply evidence to support her final conclusion that alower accident rate would in fact increase overall worker productivity

The following appeared in a memo from the vice president of marketing at Dura-Sock Inc

A recent study of our customers suggests that our company is wasting the money it spends on its patented Endure manufacturing process which ensures that our socks are strong enough to last for two years We have always advertised our use of the Endure process but the new study shows that despite our socks durability our average customer actually purchases new Dura-Socks every three months Furthermore our customers surveyed in our largest market northeastern United States cities say that they most value Dura-Socks stylish appearance and availability in many colors These findings suggest that we can increase our profits by discontinuing use of the Endure manufacturing process

1 Write a response in which you examine the stated andor unstated assumptions of the argument Be sure to explain how the argument depends on these assumptions and what the implications are for the argument if the assumptions prove unwarranted

2 Write a response in which you discuss what specific evidence is needed to evaluate the argument and explain how the evidence would weaken or strengthen the argument

3 Write a response in which you discuss what questions would need to be answered in order to decide whether the recommendation and the argument on which it is based are reasonable Be sure to explain how the answers to these questions would help to evaluate the recommendation

Intro The vice president of marketing at Dura-Sock Inc is offering a potentially harmful investment recommendation by claiming that Dura-Sock should discontinue its use of the ldquoEndurerdquo process To support his recommendation he points out a study that Dura-Sock customers actually purchase the socks every three months and a survey that reveals that Dura-Sock customers like the sockrsquos stylish appearance and availability in many colors The study and survey however are insufficient in supporting his proposal and the VP makes several unwarranted assumptionsIntro (simplified) The VP states that though Dura-Socks last for two years customers buy the socks every three months Therefore he assumes that the consumersrsquo motive for buying the produce is not its durabilityHowever the author fails to rule out other possible motivation for consumption

Even if the survey is reliable the author should consider the rest of the market Vague terms ldquowasting moneyrdquomdashprecisely how much are they wasting Studysurvey errorThe company must calculate the outcome of such momentous decisionStudy participantsrsquo comment that they prefer Dura-Sock for its stylishness and availability might take Dura-Sockrsquos enduring quality for granted

우리회사 제품 소비자들에 대한 최근 조사에서 지난 2년여간 양말의 내구성을 강하게 하는 필수공정이었던 자사 특허의 Endure 공정에 들어가는 비용이 낭비라고 말하고 있다 우리 회사는 항상 이 공정 처리에 대한 광고를 내보냈으나 이에 대한 시장 조사에서 실제로 고객들은 이 신제품을 평균 석달마다 구매하는 것으로 나타났다 더군다나 북동부지역에서 실시한 대규모 시장조사에 응답한 고객들은 양말의 모양과 색상등에 더

GRE AWA John박 박정어학원

관심을 나타냈다 이러한 결과는 우리회사가 신기술 공법을 중단하면 그에 따라 수익이 늘어날 것이라는 것을 말해주고 있는 것이다주장 These findings suggest that Dura0Sock can increase its profits by discontinuing its use of the ldquoEndurerdquo manufacturing process

1 survey가 정확한 소비자의 의견을 나타낸 것인가 다른 선택없이 양자택일과 같은 방법의 survey였는지2 북동부지역의 시장조사가 전체 의견을 대표할 수 있나3 사람들이 모양이나 색상에 앞서 내구성을 먼저 평가했을 수도 있다 내구성을 갖추었다는 전제하에 모양과 색상에 관심을 드러낸 것일 수 있다4 소비자가 도매상(retail)인지 소매상(whole)인지가 없다

The following appeared in a business magazine

As a result of numerous complaints of dizziness and nausea on the part of consumers of Promofoods tuna the company requested that eight million cans of its tuna be returned for testing Promofoods concluded that the canned tuna did not after all pose a health risk This conclusion is based on tests performed on samples of the recalled cans by chemists from Promofoods the chemists found that of the eight food chemicals most commonly blamed for causing symptoms of dizziness and nausea five were not found in any of the tested cans The chemists did find small amounts of the three remaining suspected chemicals but pointed out that these occur naturally in all canned foods

Write a response in which you discuss what questions would need to be addressed in order to decide whether the conclusion and the argument on which it is based are reasonable Be sure to explain how the answers to the questions would help to evaluate the conclusion

Representativeness of the tested cansThey should conduct a comparative studyThe testing could be biased because Promofoods employees conducted the testingHow much (quantity) of the five and three suspected chemicals were in the canned foodsFalse cause The substance that caused dizziness and nausea may not be one of the eight common chemicals

많은 소비자들의 현기증과 구역질 불만에 따라 Promofoods사는 지난해 참치 캔 8백만 개를 테스트하기 위해 반품시켰다 그 결과 캔에서는 건강에 위험이 될 수 있는 화합물질이 없었던 것으로 회사측은 결론지었다 이러한 결론은 회사측 화학연구자들이 회수된 캔의 샘플을 테스트해서 이들 증상의 원인이 되는 8가지 화합물 중에서 5가지가 실험된 캔에서 발견되지 않았다는 사실에 근거한 것이다 이들 화학자들은 나머지 3개가지 화합물이 모든 캔 식료품에서 흔히 발견되는 것이라고 언급했다 결론 Promofoods concluded that the cans did not after all contain chemicals that posed a health risk

1 공인된 기간에서 테스트를 한 것이 아니고 자사에서 직접 테스트를 했기에 신뢰성이 안간다 2 이런 증상을 일으키는 8개의 물질 말고 다른 물질들이 캔속에 많이 포함됬을수 있다 3 나머지 3개의 물질들의 함유량이 많아서 다른 종류의 캔들은 문제를 일으키지 않지만 참치캔은 문제를

일으킬 수 있다

This magazine article concludes that the 8 million cans of tuna Promofoods recalled due tocomplaints about nausea and dizziness do not after ail contain any chemicals that pose a

GRE AWA John박 박정어학원

health risk To support this conclusion the author cites the fact that five of eight chemicalscommonly causing these symptoms were not found in the recalled cans while the other threealso occur naturally in other canned foods For several reasons this evidence lends littlecredible support to the authors conclusionTo begin with the author relies partly on the fact that although three of the eight chemicalsmost commonly blamed for nausea and dizziness appeared in Promofoods recalled tunathese chemicals also occur naturally in other canned foods However this fact alone lends nosupport to the authors conclusion for two reasons First the author might be ignoring animportant distinction between naturally occurring chemicals and those not occurring naturallyIt is entirely possible that these three chemicals do not occur naturally in Promofoods tunaand that it is for this reason that the chemicals cause nausea and dizziness Secondly it isentirely possible that even when they occur naturally these chemicals cause the samesymptoms Unless the author rules out both possibilities he cannot reliably conclude that therecalled tuna would not cause these symptomsAnother problem with the argument is that the authors conclusion is too broad Based onevidence about certain chemicals that might cause two particular heath-related symptoms theauthor concludes that the recalled tuna contains no chemicals that pose a health risk Howeverthe author fails to account for the myriad of other possible health risks that the recalled tunamight potentially pose Without ruling out all other such risks the author cannot justifiablyreach his conclusionA third problem with the argument involves that fact that the eight particular chemicals withwhich the test was concerned are only the eight most commonly blamed for nausea anddizziness It is entirely possibly that other chemicals might also cause these symptoms andthat one or more of these other chemicals actually caused the symptoms Without ruling outthis possibility the author cannot jusufiably conclude that the recalled tuna would not causenausea and dizzinessA final problem with the argument involves thetesting procedure itself The author providesno information about the number of recaUed cans tested or the selection method used Unlessthe number of cans is a sufficiently large sample and is statistically repre sentative of all therecalled cans the studys results are not statistically reliableIn conclusion the article is unconvincing as it stands To strengthen the assertion that therecalled tuna would not cause nausea and dizziness the author must provide evidence thatthe three chemicals mentioned that occur naturally in other canned foods also appear naturallyin Promofoods tuna The author must also provide evidence that ingesting other canned foodscontaining these three chemicals does not cause these symptoms To better evaluate theargument we would need to know whether the sample used in the tests was statisticallysignificant and representative of all the recalled tuna We would also need to know what otherchemicals in the recalled tuna might pose any health risk at all

5그룹 불충분 조건오류 빈출

Natures Way a chain of stores selling health food and other health-related products is opening its next franchise in the town of Plainsville The store should prove to be very successful Natures Way franchises tend to be most profitable in areas where residents lead healthy lives and clearly Plainsville is such an area Plainsville merchants report that sales of running shoes and exercise clothing are at all-time highs The local health club has more members than ever and the weight training and aerobics classes are always full Finally Plainsvilles schoolchildren represent a new generation of potential customers these schoolchildren are required to participate in a fitness-for-life program which emphasizes the benefits of regular exercise at an early age

Write a response in which you examine the stated andor unstated assumptions of the argument Be sure to

GRE AWA John박 박정어학원

explain how the argument depends on these assumptions and what the implications are for the argument if the assumptions prove unwarranted

False cause

First of all the author believes that the Increased sales of running shoes and exercise clothing indicates

Plainesville residentsrsquo interest in leading healthy lives However this assumption is not logically convincing for

several reasons could be a fashion trendTime shift ldquoFitness for liferdquo might not have any influence on schoolchildren as they growFalse cause There could be other reasons for member increase in the health clubAll of the above are insufficient condition

The author has to prove that local residents are interested in leading healthy lives However he supports his conclusion with insufficient evidence Nevertheless even if the residents are concerned with health naturersquos way may not be successful First

그 동안의 경험을 토대로 볼 때 건강생활과 밀접히 관련되어 있는 거주 지역에서 본 상점들이 아주 호응을 얻고 있다 따라서 이러한 주민들이 많이 거주하고 있는 Plainsville 에 새로운 상점들을 계속 세워야 한다 이 지역 상인들은 런닝화와 운동복 판매가 가장 높다고 말한다 불과 5 년전에는 거의 전무하다시피하던 지역 헬스 클럽의 경우도 엄청나게 많은 회원을 확보하고 있으며 웨이트 트레이닝과 에어로빅 강좌들도 항상 만원이라고 한다 새로운 고객층을 예측해 보는 것도 가능하다 이 지역의 학생들의 경우 Fitness for Life프로그램을 받게 되는데 이러한 프로그램을 통해서 유년시절부터 정규적인 운동 습관을 들이게 하고 있는 것이 그것이다

결론 We should therefore build our next new store in Plainsville

반박 그동안의 경험에 의한 과거 통계가 꼭 여기에도 적용되는건 아니다 5 년전 헬스 클럽이 잘 안되었던게 다른 원인이였을수 있다(강사수준 미달 강좌미비)tourist 에 의한 원인 일수 있다 어렸을때부터 운동을 했다고 해서 커서도 관심이 있지는 않다 (오히려 반감이 있을수 있다 혹은 건강하기에 건강에 관심이 적을수도 있다)운동복이나 신발의 판매가 육체노동에 의한 것일수도 있다

IntroductionSupport1049896In this memorandum the author asserts that Naturersquos Way should build its next newstore in Plainsville To support this assertion the author states that Plainsvillesmerchantsrsquo sales of exercise clothing are going well the local health club has moremembers than ever and a new generation of customers will help to ensure NaturersquosWayrsquos success At first glance the authorrsquos assumption seems convincing but in-depth scrutiny revealsthat it lacks substantial evidence as it stands

Body 1-SamplingTopic Sentence 1To begin with the author assumes that the merchantsrsquo report indicates that the residentsare concerned about their health However this assumption is based on unsubstantiated

GRE AWA John박 박정어학원

data Example 1 (Rebuttal1) First if we do not know the total volume of items sold and the price of the goods exactly we cannot infer whether the residents are actually buying many goods Example 2 (Rebuttal2)In addition to that the report emphasizes the rising sales of running shoes and exerciseclothing however these may not be hot-selling items for Naturersquos Way or may not be theproducts the company is planning to sell Concluding Sentence Therefore in order to make the argument reliable the author should reconsider themerchantsrsquo report with more detailed data

Body 2-CausalTopic Sentence 2Second the author contends that the health clubs classes are full yet this does not meanthat many people actually use the health club other factors may be the real cause forthose closed classes Example 1 (Rebuttal1) To begin with if the health club is very small the number of people working out wouldnot be a large one In fact regular gym-going may just be a vogue among a smallunrepresentative segment of Plainsvilles population Example 2 (Rebuttal2) Moreover it is possible that most of the people who exercise in the health club do weight training and aerobics only to look good and to meet other singles not for their health In that case there would be little demand for health products Concluding SentenceThus the author should not hasten to presume what really caused people to be interested in a healthier lifestyle and enroll in the health club

Body 3-Time-ShiftTopic Sentence 3Finally the author highlights that Naturersquos Way can expect a new generation of customersin Plainsville that will help the company in the long term This notion is mistaken in that itassumes the conditions of the present will continue unchanged in the future Although theschool children are required to participate in the fitness for life program they may notnecessarily buy Naturersquos Ways products Example 1 (Rebuttal1) In the first instance they may suffer a fall in purchasing power arising from future economic difficulties this would cause reluctance to spend a considerable amount of money on health products which tend to be more expensive Example 2 (Rebuttal2)Another possibility is that there may emerge many competitor companies vying with Naturersquos Way so that in the future the school children may not feel the necessity to purchase one companyrsquos health products over anotherrsquosConcluding Sentence Thus the authorrsquos assumption is highly speculative since it relies heavily on unknowablefuture circumstances

ConclusionThesis In sum the author uses many assumptions that are insufficient in supporting his claimsSupportIn order for the authorrsquos claims to be convincing he needs to advance more persuasiveevidence that people in Plainsville really are concerned with their health and health foodThe following was written as a part of an application for a small-business loan by a group of developers in the city of Monroe

A jazz music club in Monroe would be a tremendously profitable enterprise Currently the nearest jazz club is 65 miles away thus the proposed new jazz club in Monroe the C-Note would have the local market all to itself Plus jazz is extremely popular in Monroe over 100000 people attended Monroes annual jazz festival last summer several well-known jazz musicians live in Monroe and the highest-rated radio program in Monroe is Jazz Nightly which airs every weeknight at 7 PM Finally a nationwide study indicates that the typical jazz fan spends close to $1000 per year on jazz entertainment

1 Write a response in which you discuss what specific evidence is needed to evaluate the argument and explain how the evidence would weaken or strengthen the argument

2 Write a response in which you examine the stated andor unstated assumptions of the argument Be

GRE AWA John박 박정어학원

sure to explain how the argument depends on these assumptions and what the implications are for the argument if the assumptions prove unwarranted

3 Write a response in which you discuss what questions would need to be answered in order to decide whether the prediction and the argument on which it is based are reasonable Be sure to explain how the answers to these questions would help to evaluate the prediction

Group error nationwide survey may not reflect local trends Is the nationwide jazz fan population substantialInsufficient non-residents of Monroe may have attended the jazz festival (Body alternative explanation last year may have been an anomaly The author should consider data from various years) The author should indicate how many out of 100000 were Monroe residentsNationwide study Does this reflect Insufficient Citizens of Monroe may continue to go to the jazz club 65 miles away

Are the people in Monroe really interested in jazzMajority of the people who attended the jazz festival might not be Monroe residentsSurvey error nationwide study may not be applicable to MonroeJazz musicians who live in MonroeMonopolyRadio station

In this business application the author claims that the proposed jazz club C Note will be very profitable in Monroe To support this claim the author argues for his case with several evidences At first glance the authorrsquos argument seems convincing however careful scrutiny reveals that his argument in specious

To begin with the author claims that Monroersquos citizens are interested in jazz He presents three evidences First Secondhellip Thirdhellip Howeverhellip

Monroe 시에 있는 재즈 음악 클럽은 수익성이 좋은 사업이다 현재 가장 가까이에 있는 클럽은 65 마일 정도 떨어져 있다 따라서 이번에 세우려고 하는 C Note 는 독보적인 위치를 점할것이다 더군다나 재즈는 이 시에서 가장 인기있는 음악이다 지난 여름 재즈 축제에서는 10 만명 이상의 Morone 시 주민이 참석하였고 몇몇 유명한 재즈 음악가들도 이곳에 살고 있으며 저녁때 방영되는 라디오 프로그램중에서 최고의 시청률을 보이고 있는 것도 Jazz Nightly 이다 전국조사에서도 전형적인 재즈 팬들은 재즈 분야에 년간 1천 달러 가까이 지출하고 있는 것으로 보고되고 있다 따라서 C Note 클럽이 돈을 벌 수 있는 사업이라는 것은 확실한 것이다

결과 It is clear that the C Note cannot help but make money반박 nearest jazz club 이 양질의 써비스로 여전히 손님을 끌수도 있다Festival 에 얼마나 참여하는지가 jazz 의 인기를 반영하지 않는다 뮤지션이 많이 사는거랑 jazz 의 인기가 상관없다라디오 프로그램이 다른 요인에 의해서 인기일수도 있다 (진행자때문)전국 통계 적용 불가화목 실전반_Ms Noh6In this application the author suggests that a jazz club in Monroe will make a number of profits To support this suggestion the author exemplifies the local condition popularity of jazz in Monroe and nationwide study However careful scrutiny of each of the facts reveals that it provides little credible support for the authorrsquos recommendation Good clear intro

First the author assumes that jazz is popular in Monroe because of several facts the jazz festival last year had high participation some famous jazz musicians live in Monroe and the high-rated radio program is lsquoJazz Nightlyrsquo However this assumption has many drawbacks that must be seriously considered(Good topic sentences) If many attendants in the last-yearrsquos festival came from other cities and not Monroe it is hard to conclude that Monroersquos people like jazz Therefore the author must examine how many Monroe residents actually attended the festival On top of that there is little relationship between habitation of famous jazz musician and the popularity of jazz in Monroe Although several well-known musicians live there if they do not take part in any jazz performance of Monroe this might have no effect to the interest of Monroersquos residents

GRE AWA John박 박정어학원

about jazz Finally in the case of radio program this is also not suitable reason why jazz is popular in Monroe It might be possible that people cannot help choosing lsquoJazz Nightlyrsquo because there are few radio programs at Night The fact that the radio program is the highest rating program is not a germane evidence The approximate number of listeners would be the more crucial evidence Therefore the author needs to seriously deliberate the correlation between jazzrsquos popularity in Monroe and his examples (Good logical flow and clarity)

Second the author uses as evidence the nationwide study that jazz fans spend much money on jazz entertainment to substantiate why starting a jazz club in Monroe will be profitable In other words the author assumes that the characteristics of a nationwide study can be applied to Monroe The national study would lend support to the applicantrsquos claim only if residents in Monroe typify national jazz fans However the author does not provide credible evidence that this is the case Moreover the populations of jazz fans nationwide may be insubstantial Thus the author should not infer hastily that Monroersquos residents will spend much money on enjoying jazz from the nationwide study

Lastly even if jazz is popular in Monroe C Note may not be successful It is entirely possible that residents might still prefer other clubs where they have always went In addition there is another possibility that the nearest jazz club will attract many of Monroersquos people because it serves fine performances and is equipped with favorite facilities Without considering these other possibilities the author cannot make his argument convincing In sum the author presents many reasons that are insufficient in supporting his or her claim In order for the authorrsquos claims to be convincing he needs to advance more persuasive evidence such as the total number of Monroe residents who attended the jazz festival the effects on the popularity of jazz by the musicians living in Monroe and the actual number of residents who would typify themselves to be jazz fans through a local survey Without substantial evidence that C Note will be successful in Monroe the businessmen may be overinvesting in what might lead to a business failureExcellent clarity Score 50

The following appeared in a newsletter offering advice to investors

Over 80 percent of the respondents to a recent survey indicated a desire to reduce their intake of foods containing fats and cholesterol and today low-fat products abound in many food stores Since many of the food products currently marketed by Old Dairy Industries are high in fat and cholesterol the companys sales are likely to diminish greatly and company profits will no doubt decrease We therefore advise Old Dairy stockholders to sell their shares and other investors not to purchase stock in this company

Write a response in which you discuss what questions would need to be answered in order to decide whether the advice and the argument on which it is based are reasonable Be sure to explain how the answers to these questions would help to evaluate the advice

Survey 80

GRE AWA John박 박정어학원

Old Dairy could change their products and manufacture low fat dairy foodsLess competing companies Old Dairy could eventually be the only company that produces hellipImprecise numbers and measurementsCustomers may still buy high fat dairy products

The author of the newsletter is offering potentially dangerous advice by recommending Old Dairy stockholders to withdraw investment and stop purchase What is more the authorrsquos prediction debases the reputation and business of Old Dairy and if false could devoid the investment opportunity of the newsletter readers Therefore investors should examine whether the authorrsquos evidences are substantial

To begin with the author states that 80 percent of the respondents in a survey indicated a desire to reduce their intake of foods He therefore argues that Old Dairyrsquos high fat and cholesterol products would decrease in sales However the author makes a crucial error in this argument First the author provides no evidence that the surveyrsquos results are statistically reliable Were they representative of all the customers Were they chosen for the survey randomly Furthermore the desire to reduce fat and cholesterol intake is a pervasive trend in todayrsquos opulent society however the author erroneously identifies this as a new phenomenon which will affect consumer trends Second having a desire to reduce fat and cholesterol intake does not necessarily indicate that people who have this desire will actually reduce consuming these types of products It is entirely possible that they may continue buying Old Dairy products for its quality and taste Accordingly the author cannot draw any firm conclusion that people will not buy Old Dairy products Therefore if any of these cases are true the author may be offering investors a detrimental investment advice

최근 조사에 대한 응답자중 80 이상이 자신이 먹는 음식에서 지방과 콜레스테롤의 함유량을 줄이고 싶다고 한다 아울러 요즘은 많은 식료품 가계에서 저지방 제품들을 많이 취급하고 있다 현재 Old Dairy Industries가 판매하고 있는 많은 음식제품들은 지방과 콜레스테롤이 높기 때문에 이 회사의 매출이 격감할 것으로 보이며 당연히 매출이익도 줄어들것이다 따라서 이 회사의 주주들은 주식을 매각하고 다른 주식 투자가들도 이 회사의 주식을 매입하지 않는 것이 좋다

결론 Old Dairy stockholders to sell their shares and other investors not to purchase stock in this company

반박 모든 상품이 다 고 지방 고 칼로리는 아니다(비록 많을지라도) 일부의 식품의 경우 기호에 맞어서 히트해서 전체적인 수입이 증가할 수도 있다국내시장만 생각할 수 없다( 외국시장에서 호황을 누릴수 있다 )입맛이라는게 즉각 바뀌는게 아니다

The following appeared in a letter to the editor of the Balmer Island Gazette

On Balmer Island where mopeds serve as a popular form of transportation the population increases to 100000 during the summer months To reduce the number of accidents involving mopeds and pedestrians the town council of Balmer Island should limit the number of mopeds rented by the islands moped rental companies from 50 per day to 25 per day during the summer season By limiting the number of rentals the town council will attain the 50 percent annual reduction in moped accidents that was achieved last year on the neighboring island of Seaville when Seavilles town council enforced similar limits on moped rentals

1 Write a response in which you discuss what questions would need to be answered in order to decide whether the recommendation is likely to have the predicted result Be sure to explain how the answers to these questions would help to evaluate the recommendation

2 Write a response in which you discuss what questions would need to be answered in order to decide whether the prediction and the argument on which it is based are reasonable Be sure to explain how the answers to these questions would help to evaluate the prediction

3 Write a response in which you examine the stated andor unstated assumptions of the argument Be sure to explain how the argument depends on these assumptions and what the implications are for the argument if the assumptions prove unwarranted

Whatrsquos the actual population of Balmer Island 100000mdashis this a significant increase What kind of accidents Skin abrasions or serious injury And compared to Seaville how serious are the accidents and the actual number of accidents Did Seaville enforce other restrictions like safety signsHow different are the conditions of Balmer

GRE AWA John박 박정어학원

and Seaville regarding population road (safety) conditions topography other town-government regulation How much will the economy of Balmer be affected do to this restriction Could it cause an economic recession due to the fact that these rental companiesrsquo chance to make money is only during the summer thereby weakening the economic infrastructure Are there any other ways that could better alleviate the accident rate

Statistics 50-impreciseAnalogy Balmer compared with TorseauFalse Cause Accidents might have occurred because of reasons other than mopeds False Cause population increase may not be part of the cause of the accidentsOther explanations for the accident pedestrians few road safety regulations narrow roadsThere could be other better solutionshellip

Balmer Island의 인구가 여름철에는 십만명으로 늘어난다 2륜차와 보행자간 사고를 줄이기 위해 시의회는 6개의 자전거를 포함한 2륜차 대여업체에게 이 기간동안에는 대여숫자를 일일 50에서 30으로 제한하도록 할 것이다 대여숫자를 줄임으로써 시 의회는 지난해 이웃한 Torseau섬에서 이와 동일한 규제를 시행해서 50나 줄인 결과를 보고 마찬가지로 50를 줄일수 있다고 확신하고 있다

결론 The town council of Balmer Island should linit the number

반박 보행자의 부실에 의해서 사고가 많이 일어날수도 있다렌탈수의 줄임만이 대책은 아니다(대부분의 사람들이 렌탈 보다는 소유하고 있을 수도 있다)옆섬과는 상황이 다를수도 있다(그 섬에서는 사고의 원인이 많은 자전거 수로 인한것일수있다) 하지만 이 섬은 좁은 도로가 원인일 수도 있고 도로 안전 장치의미비가 원일일수 있다

In this letter the author recommends that Balmer Island should limit the number moped rentals from 50 to 30 per day To support this recommendation the author points out several reasons However careful scrutiny of each of the facts reveals that it is filled with unanswered questions that could significantly weaken the authorrsquos recommendation with loops and holes which are answered

The recommendation depends on the assumption that no alternative means of reducing the number of accidents are available However the author fails to offer any evidence to substantiate this crucial assumption It is highly possible that means other than this would better solve the problem Perhaps they could widen the roads or put-up more safety signs Or perhaps the accidents were due to the lack of skills in which case proper safety training would significantly alleviate the problem Without considering and ruling out these and other alternative means of reducing accidetns the author cannot confidently conclude that merely emulating Torseau would suffice Moreover the author is advising a recommendation which could potentially harm the economy of Balmer Island sincehellip Moreover the Balmer Island should alternative means to reduce accidents because limiting moped rentals during the summer could harm the economy of Balmerhellip

First of all the author believes that increase in population and the number of moped rentals are responsible for the accidents It is entirely possible that other factors are responsible for the accidents Perhaps Balmer Islandrsquos lack of safety signs was a major factor Or maybe the roads are narrow and dangerous on the Island therefore the town council could enforce stricter traffic regulations to alleviate the problem Accordingly if either of these scenarios is true the author cannot draw any firm conclusion that increase in the number of population and moped rentals are the cause of the accidents

The author of this editorial recommends that to reduce accidents involving mopeds andpedestrians Balmer Islands city council should restrict moped rentals to 30 per day down from50 at each of the islands six rental outlets To support this recommendation the author citesthe fact that last year when nearby Torseau Islands town council enforced similar measuresTorseaus rate of moped accidents fell by 50 For several reasons this evidence providesscant support for the authors recommendationTo begin with the author assumes that all other conditions in Balmer that might affect therate of moped-pedestrian accidents will remain unchanged after the restrictions are enactedHowever with a restricted supply of rental mopeds people in Balmer might purchase mopedsinstead Also the number of pedestrians might increase in the future with more pedestriansespecially tourists the risk of moped-pedestrian accidents would probably increase For thatmatter the number of rental outlets might increase to make up for the artificial supplyrestriction per outlet--a likely scenario assuming moped rental demand does not declineWithout considering and ruling out these and other possible changes that might contribute to ahigh incidence of moped-pedestrian accidents the author cannot convince me that theproposed restrictions will necessarily have the desired effect

GRE AWA John박 박정어학원

Next the author fails to consider other possible explanations for the 50 decline inTorseaus moped accident rate last year Perhaps last year Torseau experienced unusually fairweather during which moped accidents are less likely Perhaps fewer tourists visited Tot seanlast year than during most years thereby diminishing the demand for rental mopeds to belowthe allowed limits Perhaps last year some of Torseaus moped rental outlets purchased newmopeds that are safer to drive Or perhaps the restrictions were already in effect but were notenforced until last year In any event a decline in Torseaus moped accident rate during onlyone year is scarcely sufficient to draw any reliable conclusions about what might have causedthe decline or about what the accident rate will be in years aheadAdditionally in asserting that the same phenomenon that caused a 50 decline in mopedaccidents in Torseau would cause a similar decline in Balmer the author relies on what mightamount to an unfair analogy between Balmer and Torseau Perhaps Balmers ability to enforcemoped-rental restrictions does not meet Torseaus ability if not then the mere enactment ofsimilar restrictions in Balmer is no guarantee of a similar result Or perhaps the demand formopeds in Torseau is always greater than in Balmer Specifically if fewer than all availablemopeds are currently rented per day from the average Balmer outlet while in Torseau everyavailable moped is rented each day then the proposed restriction is likely to have less impacton the accident rate in Balmer than in TorseauFinally the author provides no evidence that the same restrictions that served to reduce theincidence of all moped accidents by 50 would also serve to reduce the incidence ofaccidents involving mopeds and pedestrians by 50 Lacking such evidence it is entirelypossible that the number of moped accidents not involving pedestrians decreased by a greaterpercentage while the number of moped-pedestrian accidents decreased by a smallerpercentage or even increased Since the author has not accounted for these possibilities theeditorials recommendation cannot be taken seriouslyIn conclusion the recommendation is not well supported To convince me that the proposedrestriction would achieve the desired outcome the author would have to assure me that nochanges serving to increase Balmers moped-pedestrian accident rate will occur in theforeseeable future The author must also provide dear evidence that last years decline inmoped accidents in Torseau was attributable primarily to its moped rental restrictions ratherthan to one or more other factors In order to better evaluate the recommendation I wouldneed more information comparing the supply of and demand for moped rentals on the twoislands I would also need to know the rate of mopedpedestrian accidents in Torseau both priorto and after the restrictions were enforced in TorseauThe following appeared in a magazine article about planning for retirement

Clearview should be a top choice for anyone seeking a place to retire because it has spectacular natural beauty and a consistent climate Another advantage is that housing costs in Clearview have fallen significantly during the past year and taxes remain lower than those in neighboring towns Moreover Clearviews mayor promises many new programs to improve schools streets and public services And best of all retirees in Clearview can also expect excellent health care as they grow older since the number of physicians in the area is far greater than the national average

Write a response in which you discuss what specific evidence is needed to evaluate the argument and explain how the evidence would weaken or strengthen the argument

-Natural beauty and consistent climate may not be the most wanted qualities-Housing costs could have lowered on a national level wealthy retirees may not care about costs-Taxes may be high compared to the nationrsquos average tax rate-What about other qualities of Clearview Crime rate what qualities would retirees want -If schools streets and public services need improvement then this is proof that the current condition of Clearview is low Or due to budgetary reasons the mayor may not follow-up on his promise because of lowered tax rate -Schools and people who are retired no relationship-Physicians What kind of physicians Number is irrelevant Are these physicians capable of addressing the illnesses of old people

This author argues that anyone seeking a place to retire should choose Clearview To supportthis argument the article cites Clearviews consistent climate and natural beauty its fallinghousing costs its low property taxes compared to nearby towns and the mayors promise toimprove schools streets and services The article also claims that retirees can expectexcellent health care because the number of physicians in Clearview greatly exceeds thenational average This argument is flawed in several critical respectsTo begin with although consistent climate and natural beauty might be attractive to manyretirees these features are probably not important to all retirees For many retirees it isprobably more important to live near relatives or even to enjoy changing seasons Thus I

GRE AWA John박 박정어학원

cannot accept the authors sweeping recommendation for all retirees on this basisAlso Clearviews declining housing costs do not necessarily make Clearview the best placeto retire for two reasons First despite the decline Clearviews housing costs might be highcompared to housing costs in other cities Secondly for wealthier retirees housing costs arenot likely to be a factor in choosing a place to retire Thus the mere fact that housing costshave been in decline lends scant support to the recommendationThe articles reliance on Clearviews property-tax rates is also problematic in two respectsFirst retirees obviously have innumerable choices about where to retire besides Clear viewand nearby towns Secondly for retirees who are well-off financially property taxes are notlikely to be an important concern in choosing a place to retire Thus it is unfair to infer fromClearviews property-tax rates that retirees would prefer ClearviewYet another problem with the argument involves the mayors promises In light of Clearviewslow property-tax rates whether the mayor can follow through on those promises is highlyquestionable Absent any explanation of how the city can spend more money in the areas citedwithout raising property taxes I simply cannot accept the editorials recommendation on thebasis of those promises Besides even if the city makes the improvements promised thoseimprovements--particular the ones to schools--would not necessarily be important to retireesFinally although the number of physicians in Clearview is relatively high the per capitanumber might be relatively low Moreover it would be fairer to compare this per capita numberwith the per capita number for other attractive retirement towns--rather than the nationalaverage After all retirees are likely to place a relatively heavy burden on health-careresources Besides the article provides no assurances that the number of physicians inClearview will remain high in the foreseeable futureIn conclusion the recommendation is poorly supported To strengthen it the author mustconvince me--perhaps by way of a reliable survey--that the key features that the vast majorityof retirees look for in choosing a place to live are consistent climate natural beauty and lowhousing costs The author must also provide better evidence that Clear views property taxesare lower than the those of cities in other areas The author must also explain how the city canmake its promised improvements without raising property taxes Finally to better assess theargument I would need to now how the per capita number of physicians in Clearview wouldcompare to the national average in the futureThe following appeared as a letter to the editor from a Central Plaza store owner

Over the past two years the number of shoppers in Central Plaza has been steadily decreasing while the popularity of skateboarding has increased dramatically Many Central Plaza store owners believe that the decrease in their business is due to the number of skateboard users in the plaza There has also been a dramatic increase in the amount of litter and vandalism throughout the plaza Thus we recommend that the city prohibit skateboarding in Central Plaza If skateboarding is prohibited here we predict that business in Central Plaza will return to its previously high levels

Write a response in which you discuss what questions would need to be answered in order to decide whether the recommendation is likely to have the predicted result Be sure to explain how the answers to these questions would help to evaluate the recommendation

Why two years ago What happened two years ago which started this declineIs the dramatic increase in the ldquopopularityrdquo of skateboarding the cause of the steady decline of shoppers Are there any malls nearby Were there any changes nearby which could affect the decline in customersmdasha big mall perhaps Could the decline be due to the shop ownersHow many skateboarders use the plazaWhere do they skateboardDo they shop and are they customersAre the increase in litter and vandalism due to skateboarders Could this be alleviated by installing CCTVs and hiring security

This editorial concludes that the city should ban skateboarding from its downtown CentralPlaza in order to attract visitors to that area to return the area to its former glory and to makeit a place where people can congregate for fun and relaxation To justify this conclusion theeditorial points out that skateboarders are nearly the only people one sees anymore at CentralPlaza and that the Plaza is littered and its property defaced The editorial also points out thatthe majority of downtown merchants support the skate boarding ban This argument is flawedin several critical respectsFirst the editorials author falsely assumes that a ban on skateboarding is both necessaryand sufficient to achieve the three stated objectives Perhaps the city can achieve thoseobjectives by other means as well--for example by creating a new mall that incorporates anattractive new skateboard park Even if banning skateboarders altogether is necessary to meetthe citys goals the author has not shown that this action by itself would suffice Assuming thatthe Plazas reputation is now tarnished restoring that reputation and in turn enticing peopleback to the Plaza might require additional measures--such as removing litter and graffiti

GRE AWA John박 박정어학원

promoting the Plaza to the public or enticing popular restaurant or retail chains to the PlazaSecondly the editorial assumes too hastily that the Plazas decline is attributable to theskateboarders--rather than to some other phenomenon Perhaps the Plazas primary appeal inits glory days had to do with particular shops or eateries which were eventually replaced byless appealing ones Or perhaps the crime rate in surrounding areas has risen dramatically forreasons unrelated to the skateboarders presence at the Plaza Without ruling out these andother alternative explanations for the Plazas decline the editorials author cannot convince methat a skateboard ban would reverse that declineThirdly the editorials author might be confusing cause with effect--by assuming that theskateboarders caused the abandonment of the Plaza rather than vice versa It is entirelypossible that skateboarders did not frequent the Plaza until it was largely abandoned--andbecause it had been abandoned In fact this scenario makes good sense since skateboardingis most enjoyable where there are few pedestrians or motorists to get in the wayFourth it is unreasonable to infer from the mere fact that most merchants favor the ban thatthe ban would be effective in achieving the citys objectives Admittedly perhaps thesemerchants would be more likely to help dean up the Plaza area and promote their businesseswere the city to act in accordance with their preference Yet lacking any supporting evidencethe author cannot convince me of this Thus the survey amounts to scant evidence at best thatthe proposed ban would carry the intended resultFinally the author recommends a course of action that might actually defeat the citysobjective of providing a fun and relaxing place for people to congregate In my experienceskateboarding contributes to an atmosphere of fun and relaxation for adults and children alikemore so than many other types of ambiance Without considering that continuing to allowskateboarding--or even encouraging this activity--might achieve the citys goal more effectivelythan banning the activity the author cannot convincingly conclude that the ban would be in thecitys best interestsIn sum the argument is a specious one To strengthen it the editorials author must providedear evidence that skateboarding and not some other factor is responsible for the conditionsmarking the Plazas decline The author must also convince me that no alternative means ofrestoring the Plaza are available to the city and that the proposed ban by itself would suffice toattract tourists and restore the Plaza to its former glory Finally to better assess the argument itwould be useful to know the circumstances under which the downtown merchants would bewilling to help the city achieve its objectives

6그룹 약한 비유 빈출

The following recommendation appeared in a memo from the mayor of the town of Hopewell

Two years ago the nearby town of Ocean View built a new municipal golf course and resort hotel During the past two years tourism in Ocean View has increased new businesses have opened there and Ocean Views tax revenues have risen by 30 percent Therefore the best way to improve Hopewells economymdashand generate additional tax revenuesmdashis to build a golf course and resort hotel similar to those in Ocean View

Write a response in which you examine the stated andor unstated assumptions of the argument Be sure to explain how the argument depends on these assumptions and what the implications are for the argument if the assumptions prove unwarranted

GRE AWA John박 박정어학원

Assumptions The author assumes that OVrsquos municipal golf course and resort hotel caused tourism new businesses and increased tax revenues There may be other reasons advertising promo He assumes that this will continueAssumes that Ocean View and Hopewell are similar in many waysmdashthe name suggests otherwise OV may have always been a tourist attractions for its beaches We need to know the topography

2년전 Ocean View 시는 시정 소유 골프 및 휴양지 호텔을 신축했다 그리고 지난 2년동안 이 시의 관광객이 증가했으며 새로운 사업들이 생겨났다 그에따라 시의 세수도 30나 증가했다 Hopewell의 경제를 향상시키고 아울러 세수를 늘릴 수 있는 가장 좋은 방법은 Ocean View에 세워진 것과 같은 골프 시설과 휴양지 호텔을 신축하는 것이다

1 다른 요인으로 관광 산업이 발전했을 수도 있다 문화 유적이 발견이 되었거나 도로의 정비등으로 여행자가 늘었을 수도 있다

2 관광 산업의증가가 늘어난 세수의 원인이 아니라 새로 유입된 인구의 증가나 다른 공장에서 발생한 것일 수 있다

3 2년동안 한참 골프가 붐을 이루었을 수 있다 경제상황이 나빠지거나 다른 레포츠가 인근 지역에 생겨난다면 골프하는 사람이 줄어들 수 있다

In this memo HopeweUs mayor recommends that in order to stimulate the towns economyand boost tax revenues HopeweU should build a new golf course and resort hotel just as thetown of Ocean View did two years ago To support this recommendation the mayor points outthat in Ocean View during the last two years tourism has increased new businesses haveopened and tax revenues have increased by 30 I find the mayors argument unconvincingin several important respectsFirst of all it is possible that the mayor has confused cause with effect respecting the recentdevelopments in Ocean View Perhaps Ocean Views construction of a new golf course andhotel was a response to previous increases in tourism and business development increasesthat have simply continued during the most recent two years Since the mayor has failed toaccount for this possibility the claim that Hopewell would boost its economy by alsoconstructing a golf course and hotel is completely unwarrantedSecondly the mayor fails to account for other possible causes of the trends in Ocean Viewduring the last two years The increase in tourism might have been due to improving economicconditions nationwide or to unusually pleasant weather in the region The new businessesthat have opened in Ocean View might have opened there irrespective of the new golf courseand hotel And the 30 increase in tax revenues might have been the result of an increase intax rates or the addition of a new type of municipal taxWithout ruling out these and other alternative explanations for the three recent trends inOcean View the mayor cannot reasonably infer based on those trends that Hopewellseconomy would benefit by following Ocean Views exampleThirdly even if the recent trends in Ocean View are attributable to the construction of the newgolf course and hotel there the mayor assumes too hastily that the golf course and hotel willcontinue to benefit that towns overall economy The mayor has not accounted for thepossibility that increased tourism will begin to drive residents away during tourist season orthat new business development will result in the towns losing its appeal as a place to visit or tolive Unless the mayor can convince me that these scenarios are unlikely I cannot accept themayors recommendation that Hopewell follow Ocean Views exampleFinally the mayors argument rests on the unsubstantiated assumption that Hopewell andOcean View are sufficiently alike in ways that might affect the economic impact of a new golfcourse and hotel Hopewell might lack the sort of natural environment that would attract moretourists and new businesses to the town--regardless of its new golf course and hotel For thatmatter perhaps Hopewell already contains several resort hotels and golf courses that are notutilized to their capacity If so building yet another golf course and hotel might amount to amisallocation of the towns resources--and actually harm the towns overall economyIn sum the mayors recommendation is not well supported To bolster it the mayor mustprovide better evidence that Ocean Views new golf course and hotel and not some otherphenomenon--has been responsible for boosting Ocean Views economy during the last twoyears To better assess the recommendation I would need to know why Ocean View decidedto construct its new golf course and hotel in the first place--specifically what events prior toconstruction might have prompted that decision I would also need to thoroughly compare

GRE AWA John박 박정어학원

HopeweU with Ocean View--especially in terms of their appeal to tourists and businesses--todetermine whether the same course of action that appears to have boosted Ocean Viewseconomy would also boost Hopewells economy

The following is part of a memorandum from the president of Humana University

Last year the number of students who enrolled in online degree programs offered by nearby Omni University increased by 50 percent During the same year Omni showed a significant decrease from prior years in expenditures for dormitory and classroom space most likely because instruction in the online programs takes place via the Internet In contrast over the past three years enrollment at Humana University has failed to grow and the cost of maintaining buildings has increased along with our budget deficit To address these problems Humana University will begin immediately to create and actively promote online degree programs like those at Omni We predict that instituting these online degree programs will help Humana both increase its total enrollment and solve its budget problems

Write a response in which you discuss what questions would need to be answered in order to decide whether the prediction and the argument on which it is based are reasonable Be sure to explain how the answers to these questions would help to evaluate the prediction

Is Omni University successful due to the online degree program 50 Is the decrease in expenditures for dormitory and classroom space due to the decrease in of on-campus students Which classes were successful Does HU have those classes

Even if the long-distance degree programs at Omni University benefited the school the presidentrsquos recommendation that Human College should emulate Omni University is too hasty First OUrsquos name implies that the school would have more majors than Humanahellip the president should examine which degrees were in the long-distance programhellip

지난해에는 Omni 대학에서 개강했던 원거리 학생 학점 취득 프로그램을 등록했던 학생들의 숫자가 50나 증가했다 같은해 기간동안 Omni 대학에서는 그 전년도부터 기숙사와 학급의 공간 확충을 위한 예산을 대폭 줄였는데 이는 이 원거리 학점 취득 프로그램이 양방향 비디오 컴퓨터 접속을 통해서만 가능한 수업지도 방식이기때문인 것으로 보인다 반면 지난 3개년 동안 Humana 대학에서의 수강률은 감소한데다가 건물

GRE AWA John박 박정어학원

유지비도 올랐다 따라서 Humana대학의 수강을 늘리고 예산손실을 회복하기 위해서는 Omni 대학에서 취한 조치와 같은 능동적인 프로그램을 추진해야 한다

결론 we should initiate and actively promote long-distance degree programs like those at Omni 반박 원거리 학생 취득 프로그램 숫자가 증가한거하고 예산이 줄어드는 것 사이에 연관이 약하다 (causal 학생의 증가로 관리비용 증가할수 있음 원거리 수업가능 장비도입에의한 비용발생)bad analogy(omni university 하고 같은 조건이 아니다 )-gt omni college 가 강좌내용이 좋아서 학생의 등록이 많을수 있다 Humana 대학에서 만들었다 하더라도 인기 없을수 있음다른 요인에 의해서 Humana 대학의 수강 인원이 증가할수 있음(비록 과거엔 인기가 없었을지라도)

The following appeared as part of a business plan developed by the manager of the Rialto Movie Theater

Despite its downtown location the Rialto Movie Theater a local institution for five decades must make big changes or close its doors forever It should follow the example of the new Apex Theater in the mall outside of town When the Apex opened last year it featured a video arcade plush carpeting and seats and a state-of-the-art sound system Furthermore in a recent survey over 85 percent of respondents reported that the high price of newly released movies prevents them from going to the movies more than five times per year Thus if the Rialto intends to hold on to its share of a decreasing pool of moviegoers it must offer the same features as Apex

Write a response in which you discuss what questions would need to be answered in order to decide whether the recommendation is likely to have the predicted result Be sure to explain how the answers to these questions would help to evaluate the recommendation

Before following through this business plan the manager should investigate the cause of Rialtorsquos unsuccessful business

The author provides no evidence that the surveyrsquos results are statistically reliable The surveyrsquos sample of 85 percent must be sufficient in size and representative of overall population of the city where Rialto and Apex is serving Lacking evidence of a sufficiently representative sample the author cannot justifiably rely on the survey to draw any conclusion whatsoever The author does not indicate that Apex is indeed currently successful However even if Apex is enjoying success the argument relies on what might be a false analogy between Rialto and Apex In order for Apex to serve as a model that Rialto should emulate the author must assume that all relevant circumstances are essentially the same However this assumption is unwarranted For example the argument overlooks the face that Apex is located in a strategic placemdashbeside a mall where customers can not only watch a movie but also enjoy shopping Therefore simply changing the facility to that of Apex may not lead to success

The author does not mention whether Apex is successful or not Nevertheless even if Apex is currently successful the argument relies on what might be a false analogy between Rialto and Apex In order for Apex to serve as a model that Rialto should emulate the author must assume that all relevant circumstances are essentially the same However this assumption is unwarranted For example the argument overlooks the fact that these two institutions are located in different locations Rialto in downtown and Apex in a mall outside of town Although Apex opened with state-of-the-art facilities the decisive factor in its success could be due to its strategic location of being in a mall People could enjoy both shopping and movies at one location thus they may prefer Apex over Rialto Furthermore the place where people enjoy leisure activities has shifted in the past decades for most cities from downtown to the suburbs Therefore Rialto may not be successful even if it emulates Apexrsquos facilities A better business plan may be relocating Apex to the thriving section of the downtown

Rialto 극장은 지난 50여년간 지역 회관으로써 시내에 위치해 있으면서도 이제 변화를 꾀하지 않으면 문을 닫을

GRE AWA John박 박정어학원

판이다 이 극장은 시외 쇼핑타운에 새로 들어선 Apex 극장의 사례를 본받아야 했다 Apex가 지난해 개업했을 당시 이 극장은 비디오 아케이드 플러쉬 카펫트 바닥과 좌석 그리고 최신 음향시설을 갖추었다 더군다나 최근 조사에서는 응답자의 85 이상이 새로 출시된 영화 입장료가 비싼 탓으로 지난해보다 5배이상의 관람객이 줄어들었다고 나타났다 따라서 Rialto 극장이 줄어들고 있는 관람객을 뺐기지 않고 유지하려면 Apex와 같은 시설들을 갖추어야 할 것이다주장 리알토 극장이 줄어들고 있는 관람객을 뺐기지 않고 유지하려면 Apex와 같은 시설들을 갖추어야 할 것이다

1 조사에서 응답자가 전체를 대표할 수 없다 2 apex 극장이 좋은 시설을 갖추고 있지만 그로 인해 수익이 많이 발생했다는 말이 없으므로 시설투자를

하고도 좋은 결과를 얻을 수 있을지 그 근거가 미흡하다3 좋은 영화가 출시된다면 입장료가 비싸도 영화관에서 꼭 보려고 할 수 있다 4 rialto 가 시설이 아닌 다른 요인에 의해 장사가 안될수도 있다( 우범 지역이라든지)

The following is a recommendation from the business manager of Monarch Books

Since its opening in Collegeville twenty years ago Monarch Books has developed a large customer base due to its reader-friendly atmosphere and wide selection of books on all subjects Last month Book and Bean a combination bookstore and coffee shop announced its intention to open a Collegeville store Monarch Books should open its own in-store cafeacute in the space currently devoted to childrens books Given recent national census data indicating a significant decline in the percentage of the population under age ten sales of childrens books are likely to decline By replacing its childrens books section with a cafeacute Monarch Books can increase profits and ward off competition from Book and Bean

Write a response in which you examine the stated andor unstated assumptions of the argument Be sure to explain how the argument depends on these assumptions and what the implications are for the argument if the assumptions prove unwarranted

The following is a recommendation from the business manager of Monarch Books

Since its opening in Collegeville twenty years ago Monarch Books has developed a large customer base due to its reader-friendly atmosphere and wide selection of books on all subjects Last month Book and Bean a combination bookstore and coffee shop announced its intention to open a Collegeville store Monarch Books should open its own in-store cafeacute in the space currently devoted to childrens books Given recent national census data indicating a significant decline in the percentage of the population under age ten sales of childrens books are likely to decline By replacing its childrens books section with a cafeacute Monarch Books can increase profits and ward off competition from Book and Bean

1 Write a response in which you discuss what questions would need to be answered in order to decide whether the recommendation is likely to have the predicted result Be sure to explain how the answers to these questions would help to evaluate the recommendation

2 Write a response in which you discuss what specific evidence is needed to evaluate the argument and explain how the evidence would weaken or strengthen the argument

No evidence regarding Monarch Bookrsquos successEven if Regal Bookrsquos is successful this may not be attributable to the cafeacute False analogy Emulating may not lead to success Other factors may be involvedInsufficient condition The national census is not enough evidence that childrenrsquos book sales will decline Can

GRE AWA John박 박정어학원

the national census represent the local child populationDid opening a cafeacute boost sales for Regal Books Even assuming Regal is successful by opening a cafeacute this may not be suitable for Monarch which plans to close the childrenrsquos book section to establish a cafe Imprecise language ldquorelatively little spacerdquo how smallThe managerrsquos recommendation contradicts what he says Since Monarch is popular for its wide selection of books closing a selection which targets a major group of readers may hurt Monarchrsquos salesIs this the best way to compete

When Stanley Park first opened it was the largest most heavily used public park in town It is still the largest park but it is no longer heavily used Video cameras mounted in the parks parking lots last month revealed the parks drop in popularity the recordings showed an average of only 50 cars per day In contrast tiny Carlton Park in the heart of the business district is visited by more than 150 people on a typical weekday An obvious difference is that Carlton Park unlike Stanley Park provides ample seating Thus if Stanley Park is ever to be as popular with our citizens as Carlton Park the town will obviously need to provide more benches thereby converting some of the unused open areas into spaces suitable for socializing

Write a response in which you examine the stated andor unstated assumptions of the argument Be sure to explain how the argument depends on these assumptions and what the implications are for the argument if the assumptions prove unwarranted

Stanley 파크가 처음 개장했을 당시 가장 크고 가장 많이 이용되는 공원이었다 아직도 공원중에서는 가장 크지만 이용률은 상당히 떨어졌다 지난달 공원 주차장에 설치해놓은 비디오 카메라를 통해 보면 drop(주차장으로 여겨짐) 이용률이 가장 높았다 수치상으로는 하루 평균 50대의 차량만이 이용하였다 반면 직장 중심거리에 위치한 작은 규모의 Carlton 파크는 주당 무려 150여명 이상이 이용하고 있다 Stanley 파크와는 달리 Carlton 파크에는 의자가 있다는 것이 가장 뚜렷한 차이점이다 따라서 Stanley 파크가 Carlton 파크처럼 시민들이 자주 이용하는 공원이 되기 위해서는 벤치를 설치할 필요가 있으며 이렇게 사용되지 않는 일부 공간을 활용해서 사교를 위한 공간으로 바꾸어야 한다 ===gtdrop 에 대한 첨부사항 (영영사전내용입니다)---- a place or central depository to which something (as mail money or stolen property) is brought for distribution or transmission also the act of depositing something at such a place dropgt

주장 if Stanley Park is ever to be as popular with our citizens as is Carlton Park the town will obviously need to provide more benches thereby converting some of the unused open areas into spaces suitable for socializing1 조사가 언제 이루어진 것인가 조사가 언제 실시되었느냐에 따라 결과가 다를 수있다 현재는 다시 스탠리 파크가 늘어났었을 수 있다 2 벤치를 많이 설치했다고 해서 많은 관광객이 오지 않을수 있다(사람들이 벤치나 사교 공간을 원한다는 어떠한 자료도 없다)3스탠리 파크 주변에 교통 상황이 악화가 되었거나 칼튼 파크에서 문화행사등을 많이 가져서 이용객이 줄어든것일 수도 있다 4 칼튼 파크가 중심지에 있어서 접근성이 좋을수 있다5 조사가 같은 시간을 기준으로 한게 아니다(하나는 주중이고 하나는 주말이다)6사람의 수와 차의 대수를 같은것으로 비교할수 없다 (차안에 몇 명이 타고 있는지 모르고 대중교통을 이용해서 왔을수도 있다)

Page 11: GRE writing argument brain storm

GRE AWA John박 박정어학원

의해서든 엄청난 먼지구름을 형성시켜 지구전체에 태양빛을 차단해서 지구의 기온을 뚝 떨어뜨릴수 있을 수도 있다 그러나 이러한 운석 충돌은 순간 섬광을 발산시키게 될 것인데 이 시기의 어느 기록에서도 이러한 섬광은 언급되지 않고 있다 그러나 이 시기에 남아 있는 일부 아시아의 기록문서에서는 연속적으로 화산폭발음일수 있는 엄청난 폭발음이 있었음을 기록하고 있다 따라서 이러한 혹한은 아마도 화산 폭발에 의한 것이었을 것이다

결론 the cooling was probably caused by a volcanic eruption

반박 자료의 부족( 몇몇 자료가지고 그 당시의 기후를 예상하기에는 무리가 있다)다른 원인에 의해서 기후가 떨어졌을수도 있다 (빙하기 다른 기상원인)기록되지 않았다고 해서 그 사실이 없었던 것은 아니다(기록은 했으나 자료가 없어졌을수도 있고 모든 사실이 전부다 기록되지는 않는다 )기록된 폭발음이 꼭 확산 폭발음이 아닐수도 있다( 화산 폭발음이 너무 먼거리여서 들리지 않을수도 있고 다른 소리가 기록된 것이였을수도 있다)부족한 자료를 신빙할수 있는가 기록의 소멸도 예상 할수 있다 실제로 운석이 떨어졌는데 안들렸을수 있다

The following appeared in a memo from the president of Bower Builders a company that constructs new homes

A nationwide survey reveals that the two most-desired home features are a large family room and a large well-appointed kitchen A number of homes in our area built by our competitor Domus Construction have such features and have sold much faster and at significantly higher prices than the national average To boost sales and profits we should increase the size of the family rooms and kitchens in all the homes we build and should make state-of-the-art kitchens a standard feature Moreover our larger family rooms and kitchens can come at the expense of the dining room since many of our recent buyers say they do not need a separate dining room for family meals

Write a response in which you examine the stated andor unstated assumptions of the argument Be sure to explain how the argument depends on these assumptions and what the implications are for the argument if the assumptions prove unwarranted

The presidentrsquos first unstated but apparent assumption is that a nationwide survey can be applied to local areas There is no guarantee that the area in which Bower Builders builds houses will follow the national consumer trend Since he only mentions the overall result of the nationwide survey it is difficult to determine more specific conditions such whether people in urban and rural areas have the same housing preferences Such uncertainty is further exacerbated by the nature of the survey question itselfmdash it does not deal with current trends in actual home purchases but with desired home features The naiumlve assumption that consumer desire will directly result in consumer action underlies the presidentrsquos interpretation and application of the survey results to his company plan Not everyone who wishes for a large family room and kitchen will or can actually buy a house with those features mainly due to financial reasons Furthermore the president also assumes for no evident reason that consumers will not hesitate to purchase houses with state-of-the-art kitchens as a standard rather than optional feature when it is clear that such a feature will raise the overall cost significantly He continues to blunder in his mistaken assumptions about consumer behavior in assuming that the taste of recent buyers can represent the concern of future prospective buyers The fact that recent buyers have claimed no need for separate dining rooms does not mean future buyers will feel the same way as well

The president of Bower Builders recommends that in order to raise company profits the company should build houses with large family rooms and large state-of-the-art kitchens He uses the results of a nationwide survey on desired home features and the example of Bower Buildersrsquo competitor Domus Construction to support his plan His memo manifests several assumptions about surveys consumer behavior and Domus Constructionrsquos houses that do not necessarily bolsterbuttress his argument

GRE AWA John박 박정어학원

The writer assumes 1 the nationwide survey results can be applied to the local area (Desire cannot reflect consumer trend) 2 Domus Construction was profitable because they had such features (The writer should investigate what other features the DC provides and the actual number of homes sold) 3 State-of-the-art kitchens will sell well (no evidence to buttress his assumption furthermore they would need to spend more money which would increase the price of the housesmdashtheir market will be limited to the affluent) 4 The recent buyers represent the concern of most buyersmdashthat they are ok with a house that is without a dining room

Nationwide survey might not be applicable to this regionDomus Construction could have other superior qualities than Bower Builders Ex DesignSelling fast doesnrsquot indicate quantitySmall yards complaints could be voiced in the future

First the author concludes that a nationwide survey reveals that the two most desired home features are a bathroom with a whirlpool tub and a large kitchen However this nationwide survey may not reflect the trends of the customers that Bower Builders target The author assumes that the nationwide trend reflects customer trends The national study would lead support only if the nationwide trend reflect the home-purchasing trends The nationwide trend may just be a trend of desire which does not result in the act of purchasing However the author does not provide credible evidence that this is the case

전국 조사에서 가장 인기있는 집안 구조 2가지는 훨풀 튜브가 마련된 욕실과 커다른 부엌으로 조사되었다 경쟁업체 Domus Construction이 신축한 완공예정인 주택은 이러한 욕실을 갖추고 있어서 분양이 조기에 이루어지고 있고 가격도 평균치보다 상당히 웃돌고 있다 우리도 집을 많이 팔도 그만큼 이윤을 많이 남기려면 신규 주택에는 위의 2가지 사항을 갖추어야 한다 최근 우리가 지은 주택 수요자들이 적은 평수에 대해서는 별다른 불만사항이 없었기 때문에 앞으로 평수를 더 줄여서 이익을 극대화 할 수 있을 것이다

주장 We should include whirlpool tubs and a large kitchen

1 경쟁업체 Domus Construction이 신축한 완공예정인 주택은 이러한 욕실을 갖추고 있어서라기 보다는 위치가 좋거나 다른 마감재(construction material)나 인테리어가 뛰어나서 분양이 조기에 이루어진것이라고 볼 수 있다 2 전국조사가 건물이 지어지는 지역에 항상 적용되리라고 볼 수 없다 3 평수를 줄이는 것에 불만이 없었다는 것은 editor의 견해일 수 있다 사람들이 표현하지 않은 것일 수도 있고 다른 장점이 많아서 그런 단점을 드러내지 않은 것일 수 있기 때문에 속단할 수 없다 4 다른것을 함으로써 더 잘 팔릴수도 있다

2011 7 19 화요일 GRE AWA 실전반이OO

Argument 94

The president of Bower Builders contends recommends that in order to raise company profits the company should build houses with large family rooms and large state-of-the-art kitchens He uses the results of a nationwide survey on desired home features and the example of Bower Buildersrsquo competitor Domus Construction to support his plan His memo manifests several assumptions about surveys consumer behavior and Domus Constructionrsquos houses that do not

GRE AWA John박 박정어학원

necessarily bolsterbuttress his argumentThe presidentrsquos first unstated but apparent assumption is that a nationwide survey can be

applied to local areas There is no guarantee that the area in which Bower Builders builds houses will follow the national consumer trend Since he only mentions the overall result of the nationwide survey it is difficult to determine more specific conditions such whether people in urban and rural areas have the same housing preferences Such uncertainty is further exacerbated by the nature of the survey question itselfmdash it does not deal with current trends in actual home purchases but with desired home features The naiumlve assumption that consumer desire will directly result in consumer action underlies the presidentrsquos interpretation and application of the survey results to his company plan Not everyone who wishes for a large family room and kitchen will or can actually buy a house with those features mainly due to financial reasons Furthermore the president also assumes for no evident reason that consumers will not hesitate to purchase houses with state-of-the-art kitchens as a standard rather than optional feature when it is clear that such a feature will raise the overall cost significantly He continues to blunder in his mistaken assumptions about consumer behavior in assuming that the taste of recent buyers can represent the concern of future prospective buyers The fact that recent buyers have claimed no need for separate dining rooms does not mean future buyers will feel the same way as well

In addition the president finds a real-life actualization of the nationwide survey results in the recent sales of Domus Construction However he easily assumes that large family rooms and kitchens are the only reasons the houses of Domus Construction sell well He does not take into account other features and selling points of the competitorrsquos houses There may well be other explanations for its houses selling more quickly and expensively including additional home features as well as external factors such as proximity to better schools or superior financial solvency of its clientele If Bower Builders merely added larger family rooms and kitchens without taking into consideration the other factors they may lose rather than gain profits

Clearly the presidentrsquos assertion that Bower Builders make houses with large family rooms and high-tech kitchens at the expense of dining rooms rests on a number of assumptions that are ill-informed and naiumlve If Bower Builders undertakes the proposed plan without further research into local consumer desires purchasing trends and the marketing and sales of competing companies the company will risk losing money by building big new houses that people cannot afford to or will not wish to buy

The following appeared in a letter to the editor of a journal on environmental issues

Over the past year the Crust Copper Company (CCC) has purchased over 10000 square miles of land in the tropical nation of West Fredonia Mining copper on this land will inevitably result in pollution and since West Fredonia is the home of several endangered animal species in environmental disaster But such disasters can be prevented if consumers simply refuse to purchase products that are made with CCCs copper unless the company abandons its mining plans

Write a response in which you examine the stated andor unstated assumptions of the argument Be sure to explain how the argument depends on these assumptions and what the implications are for the argument if the assumptions prove unwarranted

The writer assumes 1 The writer is trying to avoid an inevitability 2 Mining copper will result pollution (they could make preventative measures) 3 The writer may be too late from stopping CCC from developing the area into a copper mine 4 Since mining is an underground enterprise the surface may not be affected that much therefore endangered species may not be affected 4 Consumers wonrsquot buy CCC products if the journal publishes a negative review about CCC (How many readers) CCC could a company that has ties with many IT companies and industries in that their copper is almost ubiquitous in various products

GRE AWA John박 박정어학원

지난 한해동안 CCC(Consolidated Copper 회사)는 서부 플로리다의 열대 지역에 1백만 마일이 넘는 땅을 사들였다 이 곳에서의 채광 활동은 서부 플로리다가 몇몇 멸종 위기에 처한 동물의 서식지이기 때문에 분명 오염과 환경파괴를 가져올 것이다 그러나 이러한 파괴는 CCC 회사가 채광을 포기할 때까지 이 회사가 채굴한 구리로 제조된 제품을 구매하지 않으면 막을 수 있을 것이다

결론 such disaster can be prevented if consumers simply refuse to purchase products that are made with CCCs copper until the company abandons its mining plans

1 구리로 제조된 물건이 생활에 필수적인 것이 많은 만큼 불매가 쉽지 않을 수 있다(전선이나 각종 전자제품에 필수적으로 들어가기 때문에)

2 불매를 유도한다고 해서 소비자들이 구매를 안하는 것은 아니다3 적절한 채굴로 환경파괴를 가져 오지 않을 수 있다 (땅속에 있는 물질을 채굴하는 만큼 생물에 영향을 안

미칠 수도 있다)4 이미 채굴이 다 끝나서 더 이상의 채굴이 없을 수도 있다 5 땅을 구입한다고 해서 채광하는건 아니다 (다른 용도로 샀을수도 있다)

The following is a letter to the editor of an environmental magazine

In 1975 a wildlife census found that there were seven species of amphibians in Xanadu National Park with abundant numbers of each species However in 2002 only four species of amphibians were observed in the park and the numbers of each species were drastically reduced There has been a substantial decline in the numbers of amphibians worldwide and global pollution of water and air is clearly implicated The decline of amphibians in Xanadu National Park however almost certainly has a different cause in 1975 troutmdashwhich are known to eat amphibian eggsmdashwere introduced into the park

Write a response in which you discuss what specific evidence is needed to evaluate the argument and explain how the evidence would weaken or strengthen the argument

Evidence needed the identity of the sender and the census taker of rsquo75 and lsquo02mdasha scientist or an environmentalist credibility issue census methodology vs mere observationmdashthe absence of evidence is not an evidence of absence the season of when the census was taken time-shiftmdashconditions may have changed worldwide decline may include Xanadu other species of predators that prey on amphibians because trout is only one species that prey on amphibians the number of troutmdashhave they increased significantly since rsquo75 First the author needs to be more overt about the credibility of the census and observation There were to accounts that notes the population of amphibiansmdashthe first a census and the second an observation The author needs to bolster his conclusion with the evidence that indicate that the census and second observation were done using scientific methodologies This could either could strengthen or weaken his claim In addition he

GRE AWA John박 박정어학원

needs to provide the specific season of when the census and observation occurred In this letter the writer is informing an editor of an environmental magazine that the number of amphibians was greatly reduced since 1975 and he points out the introduction of trout as the only reason for the decline However the author fails to provide crucial evidences that could strengthen or weaken his conclusion

The following appeared in a memorandum from the president of Hyper-Go Toy Company

Last year sales of our Fierce Fighter toy airplane declined sharply even though the toy had been a top seller for three years Our customer surveys show that parents are now more worried about youthful violence and are concerned about better education for their children Therefore to maintain profits we should discontinue all our action toys and focus exclusively on a new line of educational toys Several other toy companies have already begun marketing educational toys and report sales increases last year of 200 percent And since the average family income is growing sales of new Hyper-Go toys should also increase

Write a response in which you discuss what specific evidence is needed to evaluate the argument and explain how the evidence would weaken or strengthen the argument

First the president indicates that the sales of Fierce Fighter toy airplane declined sharply However he fails to consider the fact that toys are a fad Since FFT enjoyed a three year success it may be natural that the trend would subsideSecond Customer survey is this representative of most toy consumers This trend is not newThird other companies may have profited not through educational but other toys Or educational toy profit may be small in proportion to their sale of other toys 200Parents are not the customers companies should concentrate on what the children want to increase profitLastly parents worry about youthful violence and concern for better education are not new trends

The presidentrsquos decision to discontinue all action toys and focus only on educational toys is too extreme If this decision is taken into effect its procedure may be an onerous task because it would require major shifts in human resources and company image Therefore further data should be considered before following up on this decision

우리 회사에서 지난 3년간 최고 매출을 일으켰던 Fierce Fighter 장난감 비행기가 지난해에는 매출이 급격히 떨어졌다 자체 고객 조사에서는 부모들이 현재 청소년 폭력에 걱정을 하고 있어서 아이들의 양질 교육에 더 관심을 가지고 있는 것으로 나타났다 따라서 회사의 수익을 유지하기 위해서는 모든 자사 전투용 장난감 생산을 중단하고 오로지 교육적인 장난감 생산에 집중해야 한다 몇몇 여타 장난감 회사들도 이미 교육용 장난감 마케팅을 시작해서 지난해에는 200의 매출신장을 가져왔다고 한다 그리고 평균 가계 수입이 점점 늘고 있기 때문에 신형 Hyper-Go 장난감의 매출도 늘어날 것이다

3그룹 Time-shift Error

Woven baskets characterized by a particular distinctive pattern have previously been found only in the immediate vicinity of the prehistoric village of Palea and therefore were believed to have been unique to the Palean people Recently however archaeologists discovered such a Palean basket in Lithos an ancient village across the Brim River from Palea The Brim River is very deep and broad and so the ancient Paleans could only have crossed it by boat but there is no evidence that the Paleans had boats And boats capable of carrying groups of people and cargo were not developed until thousands of years after the Palean people disappeared Moreover Paleans would have had no need to cross the rivermdashthe woods around Palea are full of nuts berries and small game It follows that the so-called Palean baskets were not unique to Palea

GRE AWA John박 박정어학원

Write a response in which you discuss what specific evidence is needed to evaluate the argument and explain how the evidence would weaken or strengthen the argument

RefutationPrehistoric time shift-gtbrim river could have been narrow and shallow or it might have not existed Indigenous patterns may exist in other disconnected remote placesNuts berries small game(hunting) may not have existed or the author should be proved these existed at that time Abundance of resources doesnrsquot support the reason for seclusionBoats not yet found baskets may have been carried across by the river current without the help of a boat Lithos might have crossed the river for commercial purposeOne Palean basket does not substantiate the authorrsquos claimThe absence of evidence is not an evidence of absence

The author assumes without justification that present conditions are the same as at the prehistoric era The author unfairly infers from the presence of Brim River which exist today that it would have existed in the past However the author fails to offer any evidence to substantiate this inference It is very likely that the Brim River might not have existed in prehistoric times or if it did exist may have been shallow and narrow enough for the Paleans to easily cross Any of these scenarios if true would serve to undermine the claim thathelliphelliphelliphelliphellip

First the author claims that the Brim River was very deep and broad so the Paleans could not have crossed it However the author fails to offer any evidence to substantiate that this was true in the prehistoric time of the Paleans For all we know the Brim River might not have existed in the prehistoric era or if it did exist could have been a shallow and narrow river For example scientists believe that thousands of years ago an ice-bridge existed on the Bering Sea connecting Eurasia to what is now North America to explain how the Eskimos and the Asian inhabitants of America came to migrate all over the Americas In this example the absence of evidence is not an evidence of absence Therefore to sufficiently support his claim the author needs to substantiate the fact that the Brim River really did exist and was broad and wide in the Prehistoric Era proving that the Paleans could not have influenced or traded with other groups of people

First the author claims that the Brim River was very deep and broad so the Paleans could not have crossed it However the author fails to offer any evidence to substantiate that this was true in the prehistoric times of the Paleans For all we know the Brim River might not have existed in the prehistoric era or if it did exist could have been a shallow and narrow river For example rivers are created by natural erosion over thousands of years Niagara Falls carved its way from the mouth of Lake Ottawa and created a long river Likewise the current Brim Riverrsquos physical features may not have been wide and deep Therefore to sufficiently support his claim the author needs to substantiate the fact that the Brim River really did exist and was broad and wide in the Prehistoric Era proving that the Paleans could not have influenced or traded with other groups of people

예전에는 실로짠 특이한 무늬 바구니가 Palea의 선사시대 지역의 인근마을에서만 발견되어왔기 때문에 Palea 마을 사람들의 특징이라고 여겨졌었다 그러나 최근들어 고고학자들이 Lithos지역에서 Palean 바구니를 발견하였는데 그 당시 지역은 Brim 강을 가로질러 Palea까지 닿아있었다 이 강은 수심이 아주 깊고 강폭이 넓었으며 때문에 고대의 Palea인들은 배를 이용해서 강을 건널수 있었을 것이다 그러나 이들이 배를 가지고 있었다는 증거는 발견되지 않고 있다 더군다나 이들이 멸명한 이후 수천년이 지난뒤에도 수많은 물자와 사람을 실어 나를수 있는 용적을 가진 배는 개발되지 않았다 이와더불어 Palea인들은 강을 건널필요가 없었는데 그것은 너트나무 장과열매 그리고 작은 사냥감들이 주변숲에 풍부했기때문이다 따라서 Palean 바구니라고 하는 것도 Palea인들만의 전유물이 아니라는 결론을 얻을 수 있다

결론 if follows that the so-called Palean baskets were not unique to Palea이번문제는 굿이 causal Bad analogy 로 구분해서 찾기가 힘드내요 배를 발견했다는 증거가 없는것이다 (앞으로도 발견될수 있음)

GRE AWA John박 박정어학원

계절의 영향으로 겨울에 얼음이 두껍게 언다든지 여름에 가뭄으로 인해서 건널수 있다물자가 풍부한 것이 이동하지 않을 조건이 아니다 다른 것에 의해서 이동가능(의약품등)

Thirteen years ago researchers studied a group of 25 infants who showed signs of mild distress when exposed to unfamiliar stimuli such as an unusual odor or a tape recording of an unknown voice They discovered that these infants were more likely than other infants to have been conceived in early autumn a time when their mothers production of melatonin hormone known to affect some brain functions would naturally increase in response to decreased daylight In a follow-up study conducted earlier this year more than half of these children now teenagers who had shown signs of distress identified themselves as shy Clearly increased levels of melatonin before birth cause shyness during infancy and this shyness continues into later life

Write a response in which you examine the stated andor unstated assumptions of the argument Be sure to explain how the argument depends on these assumptions and what the implications are for the argument if the assumptions prove unwarranted

Any baby exposed to unpleasant stimuli would react in such wayFirst of all the author states 25 infants as his evidence However this research sample is too small to prove his claimSecond the author states that 25 infants were conceived in early autumn which he claims lead to a shy disposition However this is faulty evidenceThird the research study was a long term study done in the span of 13 years However the author only writes about the initial and final stages of the study and leaves out evidences of what could have happened during the 13 years which could be more evidential factors of influenceFourth neither the infantsrsquo genetic predisposition nor their environment were taken into accountFinally the author concludes that his shyness continues into later life (Other factors could alter this disposition epigenetic theory)

13 년전 학자들은 25명의 유아를 대상으로 이상한 냄새나 특이한 소리를 녹음한 테잎등으로 낯선 자극을 주었을때 보이는 미미한 압박감 증상을 조사하였다 이들은 성숙기가 막 지났을 즈음에 보통의 유아들이 비슷한 증상을 보이는 정도 보다는 다소 민감한 반응을 보였는데 이 시기는 아이의 엄마가 뇌의 일부 기능에 영향을 미치는 것으로 알려진 멜라토니아 호르몬을 생산하는 시기로써 이 호르몬은 낯 시간이 짧을때 자연적으로 증가할 수도 있다 금년초에 실시된 추가연구에서 현재 10대로 성장한 당시 조사대상의 절반 이상의 아이들이 부끄럼을 잘타는 것으로 여기고 있었다 따라서 분명한 것은 출산전 멜라토닌 수치의 증가가 유아기에 수줍음 등의 영향을 미치게 되며 이러한 영향이 성장후에도 작용한다는 것이다

주장 Clearly increased levels of melatonin before birth cause shyness during infancy and this shyness continues into later life

1 25명의 아기로 결론 내리기에 샘플이 작다2 과학적 사실들에 대한 명확한 근거가 엇음3 다른 영향을 간과했다(다른 호르몬에 의한 영향 후천적인 성격형성의 영향)

GRE AWA John박 박정어학원

The following is a letter to the editor of the Atticus City newspaper

Former Mayor Durant owes an apology to the city of Atticus Both the damage to the River Bridge which connects Atticus to Hartley and the traffic problems we have long experienced on the bridge were actually caused 20 years ago by Durant After all he is the one who approved the construction of the bridge If he had approved a wider and better-designed bridge on which approximately the same amount of public money would have been spent none of the damage or problems would have occurred Instead the River Bridge has deteriorated far more rapidly over the past 20 years than has the much longer Derby Bridge up the river Even though the winters have been severe in the past several years this is no excuse for the negligence and wastefulness of Durant

Write a response in which you discuss what questions would need to be answered in order to decide whether the recommendation is likely to have the predicted result Be sure to explain how the answers to these questions would help to evaluate the recommendation

전임 시장인 Durant 씨는 Atticus 시에 대해 사과할 의무가 있습니다 Atticus와 Hartley를 잇는 River Bridge 교량에 대한 피해와 이 교량에서 오랫동안 주민들이 겪어오고 있는 교통 혼잡 문제들은 실제로 20년 전부터 시작된 것이었습니다 결정적으로 그가 교량 공사를 허가했던 바로 그 장본인입니다 당시 비슷한 공사비용으로 폭이 더 넓고 튼튼하게 설계된 교량을 허가했다면 이러한 문제나 피해는 발생하지 않았을 겁니다 더군다나 이 다리는 지난 20년 동안 상류에 건설된 훨씬 오래된 Derby 다리보다도 빠르게 부식되어 갔습니다 지난 수년동안 심지어 혹한이 있었다 하더라도 이러한 태만과 국고 손실에 대한 책임을 회피할 길이 없는 것입니다

결론 Former Mayor Durant owes an apology to the city of Atticus

1 디자인이 문제가 아닐수 있다 (디자인은 좋았으나 건설과정에 문제가 있었을 수 있다)2 그 당시의 시예산이 적어서 더 큰 다리를 짓기가 불가능했을 수도 있다3 교통량이 많거나 다른 상황으로 인해서 부식이 빨리 됐을 수 있다4 그 당시에는 최선의 선택이였지만 갑자기 변한 상황에 의해서 이런 문제점들이 발생했을 수 있다

GRE AWA John박 박정어학원

4그룹 거짓인과관계 오류 (False Cause) 빈출

Fifteen years ago Omega University implemented a new procedure that encouraged students to evaluate the teaching effectiveness of all their professors Since that time Omega professors have begun to assign higher grades in their classes and overall student grade averages at Omega have risen by 30 percent Potential employers looking at this dramatic rise in grades believe that grades at Omega are inflated and do not accurately reflect student achievement as a result Omega graduates have not been as successful at getting jobs as have graduates from nearby Alpha University To enable its graduates to secure better jobs Omega University should terminate student evaluation of professors

Write a response in which you discuss what specific evidence is needed to evaluate the argument and explain how the evidence would weaken or strengthen the argument

Omega professor evaluation implemented 15 years ago =gt Omega prof assign higher grades 30Employers believe therersquos grade inflation

Thus unsuccessful employment than AlphaTherefore to secure jobs Omega should end evaluating profs

Specific evidence neededRelationship between higher grades and evaluationRelationship between GPA and unsuccessful employmentAlpharsquos education could just be better than OmegaldquoFifteen years agordquo is a long time other factors could have influenced Why is the inflation a problem just now How much is Alpha better Is the comparison just How much gap is thereOmegarsquos student could just be doing better in their studiesComparison to other universities다른 대안 없나hellip Could Omega alleviate the employment problem by implementing a different procedure or program

15 년전 우리 대학은 학생들로 하여금 교수평가를 하도록 한 새로운 조치를 시행했었습니다 이후 교수들은 자신의 학과 학생들에게 높은 학점을 주었으며 그에따라 학생들의 전체 평점이 30나 올랐습니다 외부의 기업체들은 분명 점수가 지나치게 부풀려졌다고 믿고 있습니다 결국 본 대학 졸업생들이 인근 Alpha 대학의 졸업자들보다 구직률이 떨어지는 이유를 잘 보여주고 있는 것입니다 이를 해결하기 위해 이제부터는 학생들에 의한 교수평가제를 중단해야 합니다

결론 Omega University should now terminate student evaluation of professors

반박 교수 평가와 학점 인플레의 연관성이 적다( 교수 평가를 먼저하고 학점을 나중에 매길수도 있다)채용기준에 성적만 있는게 아니다 학업성취의 결과 일수도 있다 Alpha 가 원래 유능했다 Alpha 의 교육내용이 좋았다

GRE AWA John박 박정어학원

In this memo the dean of Omega University(OU) recommends OU to terminate professor evaluation to secure better jobs for the students To support this recommendation the dean offers several reasons However this argument contains several logical flaws which render it unconvincing

A threshold problem with the argument involves the voluntary nature of the evaluationprocedure The dean provides no evidence about the number or percentage of Omegastudents who participate in the procedure Lacking such evidence it is entirely possible thatthose numbers are insignificant in which case terminating the procedure is unlikely to haveany effect on the grade average of Omega students or their success in getting jobs aftergraduationThe argument also assumes unfairly that the grade-average increase is the result of theevaluation procedure--rather than some other phenomenon The dean ignores a host of otherpossible explanations for the increase--such as a trend at Omega toward higher admissionstandards or higher quality instruction or facilities Without ruling out all other possibleexplanations for the grade-average increase the dean cannot convince me that by terminatingthe evaluation procedure Omega would curb its perceived grade inflation let alone help itsgraduates get jobsEven if the evaluation procedure has resulted in grade inflation at Omega the deans claimthat grade inflation explains why Omega graduates are less successful than Alpha graduatesin getting jobs is unjustified The dean overlooks a myriad of other possible reasons forOmegas comparatively poor job-placement record Perhaps Omegas career services areinadequate or perhaps Omegas curriculum does not prepare students for the job market aseffectively as Alphas In short without accounting for other factors that might contribute toOmega graduates comparative lack of success in getting jobs the dean cannot justify theclaim that if Omega curbs its grade inflation employers will be more likely to hire OmegagraduatesFinally even if the dean can substantiate all of the foregoing assumptions the deansassertion that Omega must terminate its evaluation procedure to enable its graduates to findbetter jobs is still unwarranted in two respects First the dean ignores other possible ways bywhich Omega can increase its job-placement record--for example by improving its publicrelations or career-counseling services Second the dean unfairly equates more jobs withbetter jobs In other words even if more Omega graduates are able to find jobs as a result ofthe deans recommended course of action the kinds of jobs Omega graduates find would notnecessarily be better onesIn sum the deans argument is unpersuasive as it stands To strengthen it the dean mustprovide better evidence that the increase in grade average is attributable to Omegasprofessor-evaluation procedure and that the end result is a perception on the part ofemployers that Omega graduates are less qualified for jobs than Alpha graduates To betterassess the argument I would need to analyze 15-year trends in (l) the percentage of Omegastudents participating in the evaluation procedure (2) Omegas admission standards andquality of education and (3) Omegas emphasis on job training and career preparation I wouldalso need to know what other means are available to Omega for enabling its graduates to findbetter jobs

GRE AWA John박 박정어학원

The following appeared in a memo from a vice president of Quiot Manufacturing

During the past year Quiot Manufacturing had 30 percent more on-the-job accidents than at the nearby Panoply Industries plant where the work shifts are one hour shorter than ours Experts say that significant contributing factors in many on-the-job accidents are fatigue and sleep deprivation among workers Therefore to reduce the number of on-the-job accidents at Quiot and thereby increase productivity we should shorten each of our three work shifts by one hour so that employees will get adequate amounts of sleep

Write a response in which you examine the stated andor unstated assumptions of the argument Be sure to explain how the argument depends on these assumptions and what the implications are for the argument if the assumptions prove unwarranted

The following appeared in a memo from a vice president of Alta Manufacturing

During the past year Alta Manufacturing had thirty percent more on-the-job accidents than nearby Panoply Industries where the work shifts are one hour shorter than ours Experts believe that a significant contributing factor in many accidents is fatigue caused by sleep deprivation among workers Therefore to reduce the number of on-the-job accidents at Alta we recommend shortening each of our three work shifts by one hour If we do this our employees will get adequate amounts of sleep

Write a response in which you discuss what questions would need to be answered in order to decide whether the recommendation and the argument on which it is based are reasonable Be sure to explain how the answers to these questions would help to evaluate the recommendation

The following appeared in a memo from the vice president of Butler Manufacturing

During the past year workers at Butler Manufacturing reported 30 percent more on-the-job accidents than workers at nearby Panoply Industries where the work shifts are one hour shorter than ours A recent government study reports that fatigue and sleep deprivation among workers are significant contributing factors in many on-the-job accidents If we shorten each of our work shifts by one hour we can improve Butler Manufacturings safety record by ensuring that our employees are adequately rested

1 Write a response in which you discuss what specific evidence is needed to evaluate the argument and explain how the evidence would weaken or strengthen the argument

2 Write a response in which you discuss what questions would need to be answered in order to decide whether the recommendation is likely to have the predicted result Be sure to explain how the answers to these questions would help to evaluate the recommendation

4번 반복됨

Alta has 30 more job accidents than Panoply(work shifts one hour shorter)Experts Job accidents caused by fatigue and sleep deprivationTherefore to reduce job accidents and increase productivity shorten three work shifts by one hour for adequate sleep

지난해 우리 회사는 인근 Panoply Industries보다 업무상 재해가 30나 더 많았다 그 회사는 우리보다 근무 교대시간이 1시간 정도 짧았다 전문가들은 대부분의 업무상 재해에 있어서 가장 중요한 요인이 과로와 수면부족으로 보고있다 따라서 우리 회사에서 높은 산업재해를 줄이고 아울러 생산성을 높이기 위해서는 근로자들이 충분한 수면을 취할 수 있도록 1시간씩 3교대 시간을 줄여야 한다

In this memo the (author) vice president of Alta Manufacturing (AM) recommends that to reduce on-the-job accidents and increase productivity AM should shorten its three work shifts by one hour so that employees can

GRE AWA John박 박정어학원

get more sleep To support this recommendation the author provides several evidences However careful scrutiny of each of the facts reveals that it provides little credible support for the authorrsquos recommendation QuestionsThe number of accidents What kind of accidents The seriousness of the accidents is importantHow many employees are in each company What are their productsFalse cause Sleep may not be the reason for the on-the-job accidents What do Alta and Panoply manufacture

First of all the author believes that fatigue caused the on-the job accidents However there could be other reasons The author observes a correlation between sleep deprivation and on-the-job accidents then concludes that the former is the cause of the latter However the author fails to rule out other possible explanations For example it is entirely possible that Alta factories require more strenuous and dangerous labor than Panoply Without ruling out all other such factors it is unfair to conclude that fatigue is responsible for the accidents In addition the work-shifts may not be the cause of the sleep deprivation and fatigue It is possiblehellip Thus the author should provide what exactly Panoply and Alta manufacture and more precise data about their working conditions to be more convincing

Shortening the shift by one hour does not necessarily lead to more sleep And is one hour enoughLess accidents does not mean increased productivity

결론 We should shorten each of out three work shifts by one hour

반박 경쟁사에 비해서 시간당 하는 업무량이 많아서 더욱 피곤할 수도 있다 시간이 문제가 아니라 노후된 시설 설비 자체의 문제 작업 자체가 원래 위험한 것이여서 사고가 많을 수도 있다 다른 회사는 더욱 많은 작업시간에도 불구하고 안정한 작업여건으로 인해서 사고율이 오히려 더 작을 수도 있다비교사의 재해감소가 다른 요인일수 있다(안전 교육 철저)줄인 시간이 피로회복이나 수면으로 연결 안될수 있음(술을 마실 수도 있고 그 시간에 휴식을 취하지 않고 다른일을 함으로써 더욱 피로해질수 있다)

This editorial recommends that Alta Manufacturing reduce its work shifts by one hour each inorder to reduce its on-the-job accident rate and thereby increase Altas productivity To supportthis recommendation the author points out that last year the number of accidents at Alta was30 greater than at Panoply Industries where work shifts were one hour shorter The authoralso cites certain experts who believe that many on-the-job accidents are caused by fatigueand sleep deprivation I find this the argument unconvincing for several reasonsFirst and foremost the author provides absolutely no evidence that overall workerproductivity is attributable in part to the number of on-the-job accidents Although commonsense informs me that such a relationship exists the author must provide some evidence ofthis cause-and-effect relationship before I can accept the authors final conclusion that theproposed course of action would in fact increase Altas productivitySecondly the author assumes that some accidents at Alta are caused by fatigue or sleepdeprivation However the author overlooks other possible causes such as inadequateequipment maintenance or worker training or the inherent hazards of Altas manufacturingprocesses By the same token Panoplys comparatively low accident rate might be attributablenot to the length of its work shifts but rather to other factors such as superior equipmentmaintenance or worker training In other words without ruling out alternative causes ofon-the-job accidents at both companies the author cannot justifmbly conclude that merely byemulating Panoplys work-shift policy Alta would reduce the number of such accidentsThirdly even assuming that Altas workers are fatigued or sleep-deprived and that this is thecause of some of Altas on-the-job accidents in order to accept the authors solution to thisproblem we must assume that Altas workers would use the additional hour of free time tosleep or rest However the author provides no evidence that they would use the time in thismanner It is entirely possible that Altas workers would use that extra hour to engage in someother fatiguing activity Without ruling out this possibility the author cannot convincinglyconclude that reducing Altas work shifts by one hour would reduce Altas accident rateFinally a series of problems with the argument arise from the scant statistical information onwhich it relies In comparing the number of accidents at Alta and Panoply the author fails toconsider that the per-worker accident rate might reveal that Alta is actually safer than Panoplydepending on the total number of workers at each company Second perhaps accident rates

GRE AWA John박 박정어학원

at the two companies last year were aberrations and during other years Altas accident ratewas no greater or even lower than Panoplys rate Or perhaps Panoply is not representativeof industrial companies generally and that other companies with shorter work shifts have evenhigher accident rates In short since the argument relies on very limited statistical information Icannot take the authors recommendation seriouslyIn conclusion the recommendation for emulating Panoplys work-shift policy is not wellsupported To convince me that shorter work shifts would reduce Altas on-the-job accidentrate the author must provide clear evidence that work-shift length is responsible for some ofAltas accidents The author must also supply evidence to support her final conclusion that alower accident rate would in fact increase overall worker productivity

The following appeared in a memo from the vice president of marketing at Dura-Sock Inc

A recent study of our customers suggests that our company is wasting the money it spends on its patented Endure manufacturing process which ensures that our socks are strong enough to last for two years We have always advertised our use of the Endure process but the new study shows that despite our socks durability our average customer actually purchases new Dura-Socks every three months Furthermore our customers surveyed in our largest market northeastern United States cities say that they most value Dura-Socks stylish appearance and availability in many colors These findings suggest that we can increase our profits by discontinuing use of the Endure manufacturing process

1 Write a response in which you examine the stated andor unstated assumptions of the argument Be sure to explain how the argument depends on these assumptions and what the implications are for the argument if the assumptions prove unwarranted

2 Write a response in which you discuss what specific evidence is needed to evaluate the argument and explain how the evidence would weaken or strengthen the argument

3 Write a response in which you discuss what questions would need to be answered in order to decide whether the recommendation and the argument on which it is based are reasonable Be sure to explain how the answers to these questions would help to evaluate the recommendation

Intro The vice president of marketing at Dura-Sock Inc is offering a potentially harmful investment recommendation by claiming that Dura-Sock should discontinue its use of the ldquoEndurerdquo process To support his recommendation he points out a study that Dura-Sock customers actually purchase the socks every three months and a survey that reveals that Dura-Sock customers like the sockrsquos stylish appearance and availability in many colors The study and survey however are insufficient in supporting his proposal and the VP makes several unwarranted assumptionsIntro (simplified) The VP states that though Dura-Socks last for two years customers buy the socks every three months Therefore he assumes that the consumersrsquo motive for buying the produce is not its durabilityHowever the author fails to rule out other possible motivation for consumption

Even if the survey is reliable the author should consider the rest of the market Vague terms ldquowasting moneyrdquomdashprecisely how much are they wasting Studysurvey errorThe company must calculate the outcome of such momentous decisionStudy participantsrsquo comment that they prefer Dura-Sock for its stylishness and availability might take Dura-Sockrsquos enduring quality for granted

우리회사 제품 소비자들에 대한 최근 조사에서 지난 2년여간 양말의 내구성을 강하게 하는 필수공정이었던 자사 특허의 Endure 공정에 들어가는 비용이 낭비라고 말하고 있다 우리 회사는 항상 이 공정 처리에 대한 광고를 내보냈으나 이에 대한 시장 조사에서 실제로 고객들은 이 신제품을 평균 석달마다 구매하는 것으로 나타났다 더군다나 북동부지역에서 실시한 대규모 시장조사에 응답한 고객들은 양말의 모양과 색상등에 더

GRE AWA John박 박정어학원

관심을 나타냈다 이러한 결과는 우리회사가 신기술 공법을 중단하면 그에 따라 수익이 늘어날 것이라는 것을 말해주고 있는 것이다주장 These findings suggest that Dura0Sock can increase its profits by discontinuing its use of the ldquoEndurerdquo manufacturing process

1 survey가 정확한 소비자의 의견을 나타낸 것인가 다른 선택없이 양자택일과 같은 방법의 survey였는지2 북동부지역의 시장조사가 전체 의견을 대표할 수 있나3 사람들이 모양이나 색상에 앞서 내구성을 먼저 평가했을 수도 있다 내구성을 갖추었다는 전제하에 모양과 색상에 관심을 드러낸 것일 수 있다4 소비자가 도매상(retail)인지 소매상(whole)인지가 없다

The following appeared in a business magazine

As a result of numerous complaints of dizziness and nausea on the part of consumers of Promofoods tuna the company requested that eight million cans of its tuna be returned for testing Promofoods concluded that the canned tuna did not after all pose a health risk This conclusion is based on tests performed on samples of the recalled cans by chemists from Promofoods the chemists found that of the eight food chemicals most commonly blamed for causing symptoms of dizziness and nausea five were not found in any of the tested cans The chemists did find small amounts of the three remaining suspected chemicals but pointed out that these occur naturally in all canned foods

Write a response in which you discuss what questions would need to be addressed in order to decide whether the conclusion and the argument on which it is based are reasonable Be sure to explain how the answers to the questions would help to evaluate the conclusion

Representativeness of the tested cansThey should conduct a comparative studyThe testing could be biased because Promofoods employees conducted the testingHow much (quantity) of the five and three suspected chemicals were in the canned foodsFalse cause The substance that caused dizziness and nausea may not be one of the eight common chemicals

많은 소비자들의 현기증과 구역질 불만에 따라 Promofoods사는 지난해 참치 캔 8백만 개를 테스트하기 위해 반품시켰다 그 결과 캔에서는 건강에 위험이 될 수 있는 화합물질이 없었던 것으로 회사측은 결론지었다 이러한 결론은 회사측 화학연구자들이 회수된 캔의 샘플을 테스트해서 이들 증상의 원인이 되는 8가지 화합물 중에서 5가지가 실험된 캔에서 발견되지 않았다는 사실에 근거한 것이다 이들 화학자들은 나머지 3개가지 화합물이 모든 캔 식료품에서 흔히 발견되는 것이라고 언급했다 결론 Promofoods concluded that the cans did not after all contain chemicals that posed a health risk

1 공인된 기간에서 테스트를 한 것이 아니고 자사에서 직접 테스트를 했기에 신뢰성이 안간다 2 이런 증상을 일으키는 8개의 물질 말고 다른 물질들이 캔속에 많이 포함됬을수 있다 3 나머지 3개의 물질들의 함유량이 많아서 다른 종류의 캔들은 문제를 일으키지 않지만 참치캔은 문제를

일으킬 수 있다

This magazine article concludes that the 8 million cans of tuna Promofoods recalled due tocomplaints about nausea and dizziness do not after ail contain any chemicals that pose a

GRE AWA John박 박정어학원

health risk To support this conclusion the author cites the fact that five of eight chemicalscommonly causing these symptoms were not found in the recalled cans while the other threealso occur naturally in other canned foods For several reasons this evidence lends littlecredible support to the authors conclusionTo begin with the author relies partly on the fact that although three of the eight chemicalsmost commonly blamed for nausea and dizziness appeared in Promofoods recalled tunathese chemicals also occur naturally in other canned foods However this fact alone lends nosupport to the authors conclusion for two reasons First the author might be ignoring animportant distinction between naturally occurring chemicals and those not occurring naturallyIt is entirely possible that these three chemicals do not occur naturally in Promofoods tunaand that it is for this reason that the chemicals cause nausea and dizziness Secondly it isentirely possible that even when they occur naturally these chemicals cause the samesymptoms Unless the author rules out both possibilities he cannot reliably conclude that therecalled tuna would not cause these symptomsAnother problem with the argument is that the authors conclusion is too broad Based onevidence about certain chemicals that might cause two particular heath-related symptoms theauthor concludes that the recalled tuna contains no chemicals that pose a health risk Howeverthe author fails to account for the myriad of other possible health risks that the recalled tunamight potentially pose Without ruling out all other such risks the author cannot justifiablyreach his conclusionA third problem with the argument involves that fact that the eight particular chemicals withwhich the test was concerned are only the eight most commonly blamed for nausea anddizziness It is entirely possibly that other chemicals might also cause these symptoms andthat one or more of these other chemicals actually caused the symptoms Without ruling outthis possibility the author cannot jusufiably conclude that the recalled tuna would not causenausea and dizzinessA final problem with the argument involves thetesting procedure itself The author providesno information about the number of recaUed cans tested or the selection method used Unlessthe number of cans is a sufficiently large sample and is statistically repre sentative of all therecalled cans the studys results are not statistically reliableIn conclusion the article is unconvincing as it stands To strengthen the assertion that therecalled tuna would not cause nausea and dizziness the author must provide evidence thatthe three chemicals mentioned that occur naturally in other canned foods also appear naturallyin Promofoods tuna The author must also provide evidence that ingesting other canned foodscontaining these three chemicals does not cause these symptoms To better evaluate theargument we would need to know whether the sample used in the tests was statisticallysignificant and representative of all the recalled tuna We would also need to know what otherchemicals in the recalled tuna might pose any health risk at all

5그룹 불충분 조건오류 빈출

Natures Way a chain of stores selling health food and other health-related products is opening its next franchise in the town of Plainsville The store should prove to be very successful Natures Way franchises tend to be most profitable in areas where residents lead healthy lives and clearly Plainsville is such an area Plainsville merchants report that sales of running shoes and exercise clothing are at all-time highs The local health club has more members than ever and the weight training and aerobics classes are always full Finally Plainsvilles schoolchildren represent a new generation of potential customers these schoolchildren are required to participate in a fitness-for-life program which emphasizes the benefits of regular exercise at an early age

Write a response in which you examine the stated andor unstated assumptions of the argument Be sure to

GRE AWA John박 박정어학원

explain how the argument depends on these assumptions and what the implications are for the argument if the assumptions prove unwarranted

False cause

First of all the author believes that the Increased sales of running shoes and exercise clothing indicates

Plainesville residentsrsquo interest in leading healthy lives However this assumption is not logically convincing for

several reasons could be a fashion trendTime shift ldquoFitness for liferdquo might not have any influence on schoolchildren as they growFalse cause There could be other reasons for member increase in the health clubAll of the above are insufficient condition

The author has to prove that local residents are interested in leading healthy lives However he supports his conclusion with insufficient evidence Nevertheless even if the residents are concerned with health naturersquos way may not be successful First

그 동안의 경험을 토대로 볼 때 건강생활과 밀접히 관련되어 있는 거주 지역에서 본 상점들이 아주 호응을 얻고 있다 따라서 이러한 주민들이 많이 거주하고 있는 Plainsville 에 새로운 상점들을 계속 세워야 한다 이 지역 상인들은 런닝화와 운동복 판매가 가장 높다고 말한다 불과 5 년전에는 거의 전무하다시피하던 지역 헬스 클럽의 경우도 엄청나게 많은 회원을 확보하고 있으며 웨이트 트레이닝과 에어로빅 강좌들도 항상 만원이라고 한다 새로운 고객층을 예측해 보는 것도 가능하다 이 지역의 학생들의 경우 Fitness for Life프로그램을 받게 되는데 이러한 프로그램을 통해서 유년시절부터 정규적인 운동 습관을 들이게 하고 있는 것이 그것이다

결론 We should therefore build our next new store in Plainsville

반박 그동안의 경험에 의한 과거 통계가 꼭 여기에도 적용되는건 아니다 5 년전 헬스 클럽이 잘 안되었던게 다른 원인이였을수 있다(강사수준 미달 강좌미비)tourist 에 의한 원인 일수 있다 어렸을때부터 운동을 했다고 해서 커서도 관심이 있지는 않다 (오히려 반감이 있을수 있다 혹은 건강하기에 건강에 관심이 적을수도 있다)운동복이나 신발의 판매가 육체노동에 의한 것일수도 있다

IntroductionSupport1049896In this memorandum the author asserts that Naturersquos Way should build its next newstore in Plainsville To support this assertion the author states that Plainsvillesmerchantsrsquo sales of exercise clothing are going well the local health club has moremembers than ever and a new generation of customers will help to ensure NaturersquosWayrsquos success At first glance the authorrsquos assumption seems convincing but in-depth scrutiny revealsthat it lacks substantial evidence as it stands

Body 1-SamplingTopic Sentence 1To begin with the author assumes that the merchantsrsquo report indicates that the residentsare concerned about their health However this assumption is based on unsubstantiated

GRE AWA John박 박정어학원

data Example 1 (Rebuttal1) First if we do not know the total volume of items sold and the price of the goods exactly we cannot infer whether the residents are actually buying many goods Example 2 (Rebuttal2)In addition to that the report emphasizes the rising sales of running shoes and exerciseclothing however these may not be hot-selling items for Naturersquos Way or may not be theproducts the company is planning to sell Concluding Sentence Therefore in order to make the argument reliable the author should reconsider themerchantsrsquo report with more detailed data

Body 2-CausalTopic Sentence 2Second the author contends that the health clubs classes are full yet this does not meanthat many people actually use the health club other factors may be the real cause forthose closed classes Example 1 (Rebuttal1) To begin with if the health club is very small the number of people working out wouldnot be a large one In fact regular gym-going may just be a vogue among a smallunrepresentative segment of Plainsvilles population Example 2 (Rebuttal2) Moreover it is possible that most of the people who exercise in the health club do weight training and aerobics only to look good and to meet other singles not for their health In that case there would be little demand for health products Concluding SentenceThus the author should not hasten to presume what really caused people to be interested in a healthier lifestyle and enroll in the health club

Body 3-Time-ShiftTopic Sentence 3Finally the author highlights that Naturersquos Way can expect a new generation of customersin Plainsville that will help the company in the long term This notion is mistaken in that itassumes the conditions of the present will continue unchanged in the future Although theschool children are required to participate in the fitness for life program they may notnecessarily buy Naturersquos Ways products Example 1 (Rebuttal1) In the first instance they may suffer a fall in purchasing power arising from future economic difficulties this would cause reluctance to spend a considerable amount of money on health products which tend to be more expensive Example 2 (Rebuttal2)Another possibility is that there may emerge many competitor companies vying with Naturersquos Way so that in the future the school children may not feel the necessity to purchase one companyrsquos health products over anotherrsquosConcluding Sentence Thus the authorrsquos assumption is highly speculative since it relies heavily on unknowablefuture circumstances

ConclusionThesis In sum the author uses many assumptions that are insufficient in supporting his claimsSupportIn order for the authorrsquos claims to be convincing he needs to advance more persuasiveevidence that people in Plainsville really are concerned with their health and health foodThe following was written as a part of an application for a small-business loan by a group of developers in the city of Monroe

A jazz music club in Monroe would be a tremendously profitable enterprise Currently the nearest jazz club is 65 miles away thus the proposed new jazz club in Monroe the C-Note would have the local market all to itself Plus jazz is extremely popular in Monroe over 100000 people attended Monroes annual jazz festival last summer several well-known jazz musicians live in Monroe and the highest-rated radio program in Monroe is Jazz Nightly which airs every weeknight at 7 PM Finally a nationwide study indicates that the typical jazz fan spends close to $1000 per year on jazz entertainment

1 Write a response in which you discuss what specific evidence is needed to evaluate the argument and explain how the evidence would weaken or strengthen the argument

2 Write a response in which you examine the stated andor unstated assumptions of the argument Be

GRE AWA John박 박정어학원

sure to explain how the argument depends on these assumptions and what the implications are for the argument if the assumptions prove unwarranted

3 Write a response in which you discuss what questions would need to be answered in order to decide whether the prediction and the argument on which it is based are reasonable Be sure to explain how the answers to these questions would help to evaluate the prediction

Group error nationwide survey may not reflect local trends Is the nationwide jazz fan population substantialInsufficient non-residents of Monroe may have attended the jazz festival (Body alternative explanation last year may have been an anomaly The author should consider data from various years) The author should indicate how many out of 100000 were Monroe residentsNationwide study Does this reflect Insufficient Citizens of Monroe may continue to go to the jazz club 65 miles away

Are the people in Monroe really interested in jazzMajority of the people who attended the jazz festival might not be Monroe residentsSurvey error nationwide study may not be applicable to MonroeJazz musicians who live in MonroeMonopolyRadio station

In this business application the author claims that the proposed jazz club C Note will be very profitable in Monroe To support this claim the author argues for his case with several evidences At first glance the authorrsquos argument seems convincing however careful scrutiny reveals that his argument in specious

To begin with the author claims that Monroersquos citizens are interested in jazz He presents three evidences First Secondhellip Thirdhellip Howeverhellip

Monroe 시에 있는 재즈 음악 클럽은 수익성이 좋은 사업이다 현재 가장 가까이에 있는 클럽은 65 마일 정도 떨어져 있다 따라서 이번에 세우려고 하는 C Note 는 독보적인 위치를 점할것이다 더군다나 재즈는 이 시에서 가장 인기있는 음악이다 지난 여름 재즈 축제에서는 10 만명 이상의 Morone 시 주민이 참석하였고 몇몇 유명한 재즈 음악가들도 이곳에 살고 있으며 저녁때 방영되는 라디오 프로그램중에서 최고의 시청률을 보이고 있는 것도 Jazz Nightly 이다 전국조사에서도 전형적인 재즈 팬들은 재즈 분야에 년간 1천 달러 가까이 지출하고 있는 것으로 보고되고 있다 따라서 C Note 클럽이 돈을 벌 수 있는 사업이라는 것은 확실한 것이다

결과 It is clear that the C Note cannot help but make money반박 nearest jazz club 이 양질의 써비스로 여전히 손님을 끌수도 있다Festival 에 얼마나 참여하는지가 jazz 의 인기를 반영하지 않는다 뮤지션이 많이 사는거랑 jazz 의 인기가 상관없다라디오 프로그램이 다른 요인에 의해서 인기일수도 있다 (진행자때문)전국 통계 적용 불가화목 실전반_Ms Noh6In this application the author suggests that a jazz club in Monroe will make a number of profits To support this suggestion the author exemplifies the local condition popularity of jazz in Monroe and nationwide study However careful scrutiny of each of the facts reveals that it provides little credible support for the authorrsquos recommendation Good clear intro

First the author assumes that jazz is popular in Monroe because of several facts the jazz festival last year had high participation some famous jazz musicians live in Monroe and the high-rated radio program is lsquoJazz Nightlyrsquo However this assumption has many drawbacks that must be seriously considered(Good topic sentences) If many attendants in the last-yearrsquos festival came from other cities and not Monroe it is hard to conclude that Monroersquos people like jazz Therefore the author must examine how many Monroe residents actually attended the festival On top of that there is little relationship between habitation of famous jazz musician and the popularity of jazz in Monroe Although several well-known musicians live there if they do not take part in any jazz performance of Monroe this might have no effect to the interest of Monroersquos residents

GRE AWA John박 박정어학원

about jazz Finally in the case of radio program this is also not suitable reason why jazz is popular in Monroe It might be possible that people cannot help choosing lsquoJazz Nightlyrsquo because there are few radio programs at Night The fact that the radio program is the highest rating program is not a germane evidence The approximate number of listeners would be the more crucial evidence Therefore the author needs to seriously deliberate the correlation between jazzrsquos popularity in Monroe and his examples (Good logical flow and clarity)

Second the author uses as evidence the nationwide study that jazz fans spend much money on jazz entertainment to substantiate why starting a jazz club in Monroe will be profitable In other words the author assumes that the characteristics of a nationwide study can be applied to Monroe The national study would lend support to the applicantrsquos claim only if residents in Monroe typify national jazz fans However the author does not provide credible evidence that this is the case Moreover the populations of jazz fans nationwide may be insubstantial Thus the author should not infer hastily that Monroersquos residents will spend much money on enjoying jazz from the nationwide study

Lastly even if jazz is popular in Monroe C Note may not be successful It is entirely possible that residents might still prefer other clubs where they have always went In addition there is another possibility that the nearest jazz club will attract many of Monroersquos people because it serves fine performances and is equipped with favorite facilities Without considering these other possibilities the author cannot make his argument convincing In sum the author presents many reasons that are insufficient in supporting his or her claim In order for the authorrsquos claims to be convincing he needs to advance more persuasive evidence such as the total number of Monroe residents who attended the jazz festival the effects on the popularity of jazz by the musicians living in Monroe and the actual number of residents who would typify themselves to be jazz fans through a local survey Without substantial evidence that C Note will be successful in Monroe the businessmen may be overinvesting in what might lead to a business failureExcellent clarity Score 50

The following appeared in a newsletter offering advice to investors

Over 80 percent of the respondents to a recent survey indicated a desire to reduce their intake of foods containing fats and cholesterol and today low-fat products abound in many food stores Since many of the food products currently marketed by Old Dairy Industries are high in fat and cholesterol the companys sales are likely to diminish greatly and company profits will no doubt decrease We therefore advise Old Dairy stockholders to sell their shares and other investors not to purchase stock in this company

Write a response in which you discuss what questions would need to be answered in order to decide whether the advice and the argument on which it is based are reasonable Be sure to explain how the answers to these questions would help to evaluate the advice

Survey 80

GRE AWA John박 박정어학원

Old Dairy could change their products and manufacture low fat dairy foodsLess competing companies Old Dairy could eventually be the only company that produces hellipImprecise numbers and measurementsCustomers may still buy high fat dairy products

The author of the newsletter is offering potentially dangerous advice by recommending Old Dairy stockholders to withdraw investment and stop purchase What is more the authorrsquos prediction debases the reputation and business of Old Dairy and if false could devoid the investment opportunity of the newsletter readers Therefore investors should examine whether the authorrsquos evidences are substantial

To begin with the author states that 80 percent of the respondents in a survey indicated a desire to reduce their intake of foods He therefore argues that Old Dairyrsquos high fat and cholesterol products would decrease in sales However the author makes a crucial error in this argument First the author provides no evidence that the surveyrsquos results are statistically reliable Were they representative of all the customers Were they chosen for the survey randomly Furthermore the desire to reduce fat and cholesterol intake is a pervasive trend in todayrsquos opulent society however the author erroneously identifies this as a new phenomenon which will affect consumer trends Second having a desire to reduce fat and cholesterol intake does not necessarily indicate that people who have this desire will actually reduce consuming these types of products It is entirely possible that they may continue buying Old Dairy products for its quality and taste Accordingly the author cannot draw any firm conclusion that people will not buy Old Dairy products Therefore if any of these cases are true the author may be offering investors a detrimental investment advice

최근 조사에 대한 응답자중 80 이상이 자신이 먹는 음식에서 지방과 콜레스테롤의 함유량을 줄이고 싶다고 한다 아울러 요즘은 많은 식료품 가계에서 저지방 제품들을 많이 취급하고 있다 현재 Old Dairy Industries가 판매하고 있는 많은 음식제품들은 지방과 콜레스테롤이 높기 때문에 이 회사의 매출이 격감할 것으로 보이며 당연히 매출이익도 줄어들것이다 따라서 이 회사의 주주들은 주식을 매각하고 다른 주식 투자가들도 이 회사의 주식을 매입하지 않는 것이 좋다

결론 Old Dairy stockholders to sell their shares and other investors not to purchase stock in this company

반박 모든 상품이 다 고 지방 고 칼로리는 아니다(비록 많을지라도) 일부의 식품의 경우 기호에 맞어서 히트해서 전체적인 수입이 증가할 수도 있다국내시장만 생각할 수 없다( 외국시장에서 호황을 누릴수 있다 )입맛이라는게 즉각 바뀌는게 아니다

The following appeared in a letter to the editor of the Balmer Island Gazette

On Balmer Island where mopeds serve as a popular form of transportation the population increases to 100000 during the summer months To reduce the number of accidents involving mopeds and pedestrians the town council of Balmer Island should limit the number of mopeds rented by the islands moped rental companies from 50 per day to 25 per day during the summer season By limiting the number of rentals the town council will attain the 50 percent annual reduction in moped accidents that was achieved last year on the neighboring island of Seaville when Seavilles town council enforced similar limits on moped rentals

1 Write a response in which you discuss what questions would need to be answered in order to decide whether the recommendation is likely to have the predicted result Be sure to explain how the answers to these questions would help to evaluate the recommendation

2 Write a response in which you discuss what questions would need to be answered in order to decide whether the prediction and the argument on which it is based are reasonable Be sure to explain how the answers to these questions would help to evaluate the prediction

3 Write a response in which you examine the stated andor unstated assumptions of the argument Be sure to explain how the argument depends on these assumptions and what the implications are for the argument if the assumptions prove unwarranted

Whatrsquos the actual population of Balmer Island 100000mdashis this a significant increase What kind of accidents Skin abrasions or serious injury And compared to Seaville how serious are the accidents and the actual number of accidents Did Seaville enforce other restrictions like safety signsHow different are the conditions of Balmer

GRE AWA John박 박정어학원

and Seaville regarding population road (safety) conditions topography other town-government regulation How much will the economy of Balmer be affected do to this restriction Could it cause an economic recession due to the fact that these rental companiesrsquo chance to make money is only during the summer thereby weakening the economic infrastructure Are there any other ways that could better alleviate the accident rate

Statistics 50-impreciseAnalogy Balmer compared with TorseauFalse Cause Accidents might have occurred because of reasons other than mopeds False Cause population increase may not be part of the cause of the accidentsOther explanations for the accident pedestrians few road safety regulations narrow roadsThere could be other better solutionshellip

Balmer Island의 인구가 여름철에는 십만명으로 늘어난다 2륜차와 보행자간 사고를 줄이기 위해 시의회는 6개의 자전거를 포함한 2륜차 대여업체에게 이 기간동안에는 대여숫자를 일일 50에서 30으로 제한하도록 할 것이다 대여숫자를 줄임으로써 시 의회는 지난해 이웃한 Torseau섬에서 이와 동일한 규제를 시행해서 50나 줄인 결과를 보고 마찬가지로 50를 줄일수 있다고 확신하고 있다

결론 The town council of Balmer Island should linit the number

반박 보행자의 부실에 의해서 사고가 많이 일어날수도 있다렌탈수의 줄임만이 대책은 아니다(대부분의 사람들이 렌탈 보다는 소유하고 있을 수도 있다)옆섬과는 상황이 다를수도 있다(그 섬에서는 사고의 원인이 많은 자전거 수로 인한것일수있다) 하지만 이 섬은 좁은 도로가 원인일 수도 있고 도로 안전 장치의미비가 원일일수 있다

In this letter the author recommends that Balmer Island should limit the number moped rentals from 50 to 30 per day To support this recommendation the author points out several reasons However careful scrutiny of each of the facts reveals that it is filled with unanswered questions that could significantly weaken the authorrsquos recommendation with loops and holes which are answered

The recommendation depends on the assumption that no alternative means of reducing the number of accidents are available However the author fails to offer any evidence to substantiate this crucial assumption It is highly possible that means other than this would better solve the problem Perhaps they could widen the roads or put-up more safety signs Or perhaps the accidents were due to the lack of skills in which case proper safety training would significantly alleviate the problem Without considering and ruling out these and other alternative means of reducing accidetns the author cannot confidently conclude that merely emulating Torseau would suffice Moreover the author is advising a recommendation which could potentially harm the economy of Balmer Island sincehellip Moreover the Balmer Island should alternative means to reduce accidents because limiting moped rentals during the summer could harm the economy of Balmerhellip

First of all the author believes that increase in population and the number of moped rentals are responsible for the accidents It is entirely possible that other factors are responsible for the accidents Perhaps Balmer Islandrsquos lack of safety signs was a major factor Or maybe the roads are narrow and dangerous on the Island therefore the town council could enforce stricter traffic regulations to alleviate the problem Accordingly if either of these scenarios is true the author cannot draw any firm conclusion that increase in the number of population and moped rentals are the cause of the accidents

The author of this editorial recommends that to reduce accidents involving mopeds andpedestrians Balmer Islands city council should restrict moped rentals to 30 per day down from50 at each of the islands six rental outlets To support this recommendation the author citesthe fact that last year when nearby Torseau Islands town council enforced similar measuresTorseaus rate of moped accidents fell by 50 For several reasons this evidence providesscant support for the authors recommendationTo begin with the author assumes that all other conditions in Balmer that might affect therate of moped-pedestrian accidents will remain unchanged after the restrictions are enactedHowever with a restricted supply of rental mopeds people in Balmer might purchase mopedsinstead Also the number of pedestrians might increase in the future with more pedestriansespecially tourists the risk of moped-pedestrian accidents would probably increase For thatmatter the number of rental outlets might increase to make up for the artificial supplyrestriction per outlet--a likely scenario assuming moped rental demand does not declineWithout considering and ruling out these and other possible changes that might contribute to ahigh incidence of moped-pedestrian accidents the author cannot convince me that theproposed restrictions will necessarily have the desired effect

GRE AWA John박 박정어학원

Next the author fails to consider other possible explanations for the 50 decline inTorseaus moped accident rate last year Perhaps last year Torseau experienced unusually fairweather during which moped accidents are less likely Perhaps fewer tourists visited Tot seanlast year than during most years thereby diminishing the demand for rental mopeds to belowthe allowed limits Perhaps last year some of Torseaus moped rental outlets purchased newmopeds that are safer to drive Or perhaps the restrictions were already in effect but were notenforced until last year In any event a decline in Torseaus moped accident rate during onlyone year is scarcely sufficient to draw any reliable conclusions about what might have causedthe decline or about what the accident rate will be in years aheadAdditionally in asserting that the same phenomenon that caused a 50 decline in mopedaccidents in Torseau would cause a similar decline in Balmer the author relies on what mightamount to an unfair analogy between Balmer and Torseau Perhaps Balmers ability to enforcemoped-rental restrictions does not meet Torseaus ability if not then the mere enactment ofsimilar restrictions in Balmer is no guarantee of a similar result Or perhaps the demand formopeds in Torseau is always greater than in Balmer Specifically if fewer than all availablemopeds are currently rented per day from the average Balmer outlet while in Torseau everyavailable moped is rented each day then the proposed restriction is likely to have less impacton the accident rate in Balmer than in TorseauFinally the author provides no evidence that the same restrictions that served to reduce theincidence of all moped accidents by 50 would also serve to reduce the incidence ofaccidents involving mopeds and pedestrians by 50 Lacking such evidence it is entirelypossible that the number of moped accidents not involving pedestrians decreased by a greaterpercentage while the number of moped-pedestrian accidents decreased by a smallerpercentage or even increased Since the author has not accounted for these possibilities theeditorials recommendation cannot be taken seriouslyIn conclusion the recommendation is not well supported To convince me that the proposedrestriction would achieve the desired outcome the author would have to assure me that nochanges serving to increase Balmers moped-pedestrian accident rate will occur in theforeseeable future The author must also provide dear evidence that last years decline inmoped accidents in Torseau was attributable primarily to its moped rental restrictions ratherthan to one or more other factors In order to better evaluate the recommendation I wouldneed more information comparing the supply of and demand for moped rentals on the twoislands I would also need to know the rate of mopedpedestrian accidents in Torseau both priorto and after the restrictions were enforced in TorseauThe following appeared in a magazine article about planning for retirement

Clearview should be a top choice for anyone seeking a place to retire because it has spectacular natural beauty and a consistent climate Another advantage is that housing costs in Clearview have fallen significantly during the past year and taxes remain lower than those in neighboring towns Moreover Clearviews mayor promises many new programs to improve schools streets and public services And best of all retirees in Clearview can also expect excellent health care as they grow older since the number of physicians in the area is far greater than the national average

Write a response in which you discuss what specific evidence is needed to evaluate the argument and explain how the evidence would weaken or strengthen the argument

-Natural beauty and consistent climate may not be the most wanted qualities-Housing costs could have lowered on a national level wealthy retirees may not care about costs-Taxes may be high compared to the nationrsquos average tax rate-What about other qualities of Clearview Crime rate what qualities would retirees want -If schools streets and public services need improvement then this is proof that the current condition of Clearview is low Or due to budgetary reasons the mayor may not follow-up on his promise because of lowered tax rate -Schools and people who are retired no relationship-Physicians What kind of physicians Number is irrelevant Are these physicians capable of addressing the illnesses of old people

This author argues that anyone seeking a place to retire should choose Clearview To supportthis argument the article cites Clearviews consistent climate and natural beauty its fallinghousing costs its low property taxes compared to nearby towns and the mayors promise toimprove schools streets and services The article also claims that retirees can expectexcellent health care because the number of physicians in Clearview greatly exceeds thenational average This argument is flawed in several critical respectsTo begin with although consistent climate and natural beauty might be attractive to manyretirees these features are probably not important to all retirees For many retirees it isprobably more important to live near relatives or even to enjoy changing seasons Thus I

GRE AWA John박 박정어학원

cannot accept the authors sweeping recommendation for all retirees on this basisAlso Clearviews declining housing costs do not necessarily make Clearview the best placeto retire for two reasons First despite the decline Clearviews housing costs might be highcompared to housing costs in other cities Secondly for wealthier retirees housing costs arenot likely to be a factor in choosing a place to retire Thus the mere fact that housing costshave been in decline lends scant support to the recommendationThe articles reliance on Clearviews property-tax rates is also problematic in two respectsFirst retirees obviously have innumerable choices about where to retire besides Clear viewand nearby towns Secondly for retirees who are well-off financially property taxes are notlikely to be an important concern in choosing a place to retire Thus it is unfair to infer fromClearviews property-tax rates that retirees would prefer ClearviewYet another problem with the argument involves the mayors promises In light of Clearviewslow property-tax rates whether the mayor can follow through on those promises is highlyquestionable Absent any explanation of how the city can spend more money in the areas citedwithout raising property taxes I simply cannot accept the editorials recommendation on thebasis of those promises Besides even if the city makes the improvements promised thoseimprovements--particular the ones to schools--would not necessarily be important to retireesFinally although the number of physicians in Clearview is relatively high the per capitanumber might be relatively low Moreover it would be fairer to compare this per capita numberwith the per capita number for other attractive retirement towns--rather than the nationalaverage After all retirees are likely to place a relatively heavy burden on health-careresources Besides the article provides no assurances that the number of physicians inClearview will remain high in the foreseeable futureIn conclusion the recommendation is poorly supported To strengthen it the author mustconvince me--perhaps by way of a reliable survey--that the key features that the vast majorityof retirees look for in choosing a place to live are consistent climate natural beauty and lowhousing costs The author must also provide better evidence that Clear views property taxesare lower than the those of cities in other areas The author must also explain how the city canmake its promised improvements without raising property taxes Finally to better assess theargument I would need to now how the per capita number of physicians in Clearview wouldcompare to the national average in the futureThe following appeared as a letter to the editor from a Central Plaza store owner

Over the past two years the number of shoppers in Central Plaza has been steadily decreasing while the popularity of skateboarding has increased dramatically Many Central Plaza store owners believe that the decrease in their business is due to the number of skateboard users in the plaza There has also been a dramatic increase in the amount of litter and vandalism throughout the plaza Thus we recommend that the city prohibit skateboarding in Central Plaza If skateboarding is prohibited here we predict that business in Central Plaza will return to its previously high levels

Write a response in which you discuss what questions would need to be answered in order to decide whether the recommendation is likely to have the predicted result Be sure to explain how the answers to these questions would help to evaluate the recommendation

Why two years ago What happened two years ago which started this declineIs the dramatic increase in the ldquopopularityrdquo of skateboarding the cause of the steady decline of shoppers Are there any malls nearby Were there any changes nearby which could affect the decline in customersmdasha big mall perhaps Could the decline be due to the shop ownersHow many skateboarders use the plazaWhere do they skateboardDo they shop and are they customersAre the increase in litter and vandalism due to skateboarders Could this be alleviated by installing CCTVs and hiring security

This editorial concludes that the city should ban skateboarding from its downtown CentralPlaza in order to attract visitors to that area to return the area to its former glory and to makeit a place where people can congregate for fun and relaxation To justify this conclusion theeditorial points out that skateboarders are nearly the only people one sees anymore at CentralPlaza and that the Plaza is littered and its property defaced The editorial also points out thatthe majority of downtown merchants support the skate boarding ban This argument is flawedin several critical respectsFirst the editorials author falsely assumes that a ban on skateboarding is both necessaryand sufficient to achieve the three stated objectives Perhaps the city can achieve thoseobjectives by other means as well--for example by creating a new mall that incorporates anattractive new skateboard park Even if banning skateboarders altogether is necessary to meetthe citys goals the author has not shown that this action by itself would suffice Assuming thatthe Plazas reputation is now tarnished restoring that reputation and in turn enticing peopleback to the Plaza might require additional measures--such as removing litter and graffiti

GRE AWA John박 박정어학원

promoting the Plaza to the public or enticing popular restaurant or retail chains to the PlazaSecondly the editorial assumes too hastily that the Plazas decline is attributable to theskateboarders--rather than to some other phenomenon Perhaps the Plazas primary appeal inits glory days had to do with particular shops or eateries which were eventually replaced byless appealing ones Or perhaps the crime rate in surrounding areas has risen dramatically forreasons unrelated to the skateboarders presence at the Plaza Without ruling out these andother alternative explanations for the Plazas decline the editorials author cannot convince methat a skateboard ban would reverse that declineThirdly the editorials author might be confusing cause with effect--by assuming that theskateboarders caused the abandonment of the Plaza rather than vice versa It is entirelypossible that skateboarders did not frequent the Plaza until it was largely abandoned--andbecause it had been abandoned In fact this scenario makes good sense since skateboardingis most enjoyable where there are few pedestrians or motorists to get in the wayFourth it is unreasonable to infer from the mere fact that most merchants favor the ban thatthe ban would be effective in achieving the citys objectives Admittedly perhaps thesemerchants would be more likely to help dean up the Plaza area and promote their businesseswere the city to act in accordance with their preference Yet lacking any supporting evidencethe author cannot convince me of this Thus the survey amounts to scant evidence at best thatthe proposed ban would carry the intended resultFinally the author recommends a course of action that might actually defeat the citysobjective of providing a fun and relaxing place for people to congregate In my experienceskateboarding contributes to an atmosphere of fun and relaxation for adults and children alikemore so than many other types of ambiance Without considering that continuing to allowskateboarding--or even encouraging this activity--might achieve the citys goal more effectivelythan banning the activity the author cannot convincingly conclude that the ban would be in thecitys best interestsIn sum the argument is a specious one To strengthen it the editorials author must providedear evidence that skateboarding and not some other factor is responsible for the conditionsmarking the Plazas decline The author must also convince me that no alternative means ofrestoring the Plaza are available to the city and that the proposed ban by itself would suffice toattract tourists and restore the Plaza to its former glory Finally to better assess the argument itwould be useful to know the circumstances under which the downtown merchants would bewilling to help the city achieve its objectives

6그룹 약한 비유 빈출

The following recommendation appeared in a memo from the mayor of the town of Hopewell

Two years ago the nearby town of Ocean View built a new municipal golf course and resort hotel During the past two years tourism in Ocean View has increased new businesses have opened there and Ocean Views tax revenues have risen by 30 percent Therefore the best way to improve Hopewells economymdashand generate additional tax revenuesmdashis to build a golf course and resort hotel similar to those in Ocean View

Write a response in which you examine the stated andor unstated assumptions of the argument Be sure to explain how the argument depends on these assumptions and what the implications are for the argument if the assumptions prove unwarranted

GRE AWA John박 박정어학원

Assumptions The author assumes that OVrsquos municipal golf course and resort hotel caused tourism new businesses and increased tax revenues There may be other reasons advertising promo He assumes that this will continueAssumes that Ocean View and Hopewell are similar in many waysmdashthe name suggests otherwise OV may have always been a tourist attractions for its beaches We need to know the topography

2년전 Ocean View 시는 시정 소유 골프 및 휴양지 호텔을 신축했다 그리고 지난 2년동안 이 시의 관광객이 증가했으며 새로운 사업들이 생겨났다 그에따라 시의 세수도 30나 증가했다 Hopewell의 경제를 향상시키고 아울러 세수를 늘릴 수 있는 가장 좋은 방법은 Ocean View에 세워진 것과 같은 골프 시설과 휴양지 호텔을 신축하는 것이다

1 다른 요인으로 관광 산업이 발전했을 수도 있다 문화 유적이 발견이 되었거나 도로의 정비등으로 여행자가 늘었을 수도 있다

2 관광 산업의증가가 늘어난 세수의 원인이 아니라 새로 유입된 인구의 증가나 다른 공장에서 발생한 것일 수 있다

3 2년동안 한참 골프가 붐을 이루었을 수 있다 경제상황이 나빠지거나 다른 레포츠가 인근 지역에 생겨난다면 골프하는 사람이 줄어들 수 있다

In this memo HopeweUs mayor recommends that in order to stimulate the towns economyand boost tax revenues HopeweU should build a new golf course and resort hotel just as thetown of Ocean View did two years ago To support this recommendation the mayor points outthat in Ocean View during the last two years tourism has increased new businesses haveopened and tax revenues have increased by 30 I find the mayors argument unconvincingin several important respectsFirst of all it is possible that the mayor has confused cause with effect respecting the recentdevelopments in Ocean View Perhaps Ocean Views construction of a new golf course andhotel was a response to previous increases in tourism and business development increasesthat have simply continued during the most recent two years Since the mayor has failed toaccount for this possibility the claim that Hopewell would boost its economy by alsoconstructing a golf course and hotel is completely unwarrantedSecondly the mayor fails to account for other possible causes of the trends in Ocean Viewduring the last two years The increase in tourism might have been due to improving economicconditions nationwide or to unusually pleasant weather in the region The new businessesthat have opened in Ocean View might have opened there irrespective of the new golf courseand hotel And the 30 increase in tax revenues might have been the result of an increase intax rates or the addition of a new type of municipal taxWithout ruling out these and other alternative explanations for the three recent trends inOcean View the mayor cannot reasonably infer based on those trends that Hopewellseconomy would benefit by following Ocean Views exampleThirdly even if the recent trends in Ocean View are attributable to the construction of the newgolf course and hotel there the mayor assumes too hastily that the golf course and hotel willcontinue to benefit that towns overall economy The mayor has not accounted for thepossibility that increased tourism will begin to drive residents away during tourist season orthat new business development will result in the towns losing its appeal as a place to visit or tolive Unless the mayor can convince me that these scenarios are unlikely I cannot accept themayors recommendation that Hopewell follow Ocean Views exampleFinally the mayors argument rests on the unsubstantiated assumption that Hopewell andOcean View are sufficiently alike in ways that might affect the economic impact of a new golfcourse and hotel Hopewell might lack the sort of natural environment that would attract moretourists and new businesses to the town--regardless of its new golf course and hotel For thatmatter perhaps Hopewell already contains several resort hotels and golf courses that are notutilized to their capacity If so building yet another golf course and hotel might amount to amisallocation of the towns resources--and actually harm the towns overall economyIn sum the mayors recommendation is not well supported To bolster it the mayor mustprovide better evidence that Ocean Views new golf course and hotel and not some otherphenomenon--has been responsible for boosting Ocean Views economy during the last twoyears To better assess the recommendation I would need to know why Ocean View decidedto construct its new golf course and hotel in the first place--specifically what events prior toconstruction might have prompted that decision I would also need to thoroughly compare

GRE AWA John박 박정어학원

HopeweU with Ocean View--especially in terms of their appeal to tourists and businesses--todetermine whether the same course of action that appears to have boosted Ocean Viewseconomy would also boost Hopewells economy

The following is part of a memorandum from the president of Humana University

Last year the number of students who enrolled in online degree programs offered by nearby Omni University increased by 50 percent During the same year Omni showed a significant decrease from prior years in expenditures for dormitory and classroom space most likely because instruction in the online programs takes place via the Internet In contrast over the past three years enrollment at Humana University has failed to grow and the cost of maintaining buildings has increased along with our budget deficit To address these problems Humana University will begin immediately to create and actively promote online degree programs like those at Omni We predict that instituting these online degree programs will help Humana both increase its total enrollment and solve its budget problems

Write a response in which you discuss what questions would need to be answered in order to decide whether the prediction and the argument on which it is based are reasonable Be sure to explain how the answers to these questions would help to evaluate the prediction

Is Omni University successful due to the online degree program 50 Is the decrease in expenditures for dormitory and classroom space due to the decrease in of on-campus students Which classes were successful Does HU have those classes

Even if the long-distance degree programs at Omni University benefited the school the presidentrsquos recommendation that Human College should emulate Omni University is too hasty First OUrsquos name implies that the school would have more majors than Humanahellip the president should examine which degrees were in the long-distance programhellip

지난해에는 Omni 대학에서 개강했던 원거리 학생 학점 취득 프로그램을 등록했던 학생들의 숫자가 50나 증가했다 같은해 기간동안 Omni 대학에서는 그 전년도부터 기숙사와 학급의 공간 확충을 위한 예산을 대폭 줄였는데 이는 이 원거리 학점 취득 프로그램이 양방향 비디오 컴퓨터 접속을 통해서만 가능한 수업지도 방식이기때문인 것으로 보인다 반면 지난 3개년 동안 Humana 대학에서의 수강률은 감소한데다가 건물

GRE AWA John박 박정어학원

유지비도 올랐다 따라서 Humana대학의 수강을 늘리고 예산손실을 회복하기 위해서는 Omni 대학에서 취한 조치와 같은 능동적인 프로그램을 추진해야 한다

결론 we should initiate and actively promote long-distance degree programs like those at Omni 반박 원거리 학생 취득 프로그램 숫자가 증가한거하고 예산이 줄어드는 것 사이에 연관이 약하다 (causal 학생의 증가로 관리비용 증가할수 있음 원거리 수업가능 장비도입에의한 비용발생)bad analogy(omni university 하고 같은 조건이 아니다 )-gt omni college 가 강좌내용이 좋아서 학생의 등록이 많을수 있다 Humana 대학에서 만들었다 하더라도 인기 없을수 있음다른 요인에 의해서 Humana 대학의 수강 인원이 증가할수 있음(비록 과거엔 인기가 없었을지라도)

The following appeared as part of a business plan developed by the manager of the Rialto Movie Theater

Despite its downtown location the Rialto Movie Theater a local institution for five decades must make big changes or close its doors forever It should follow the example of the new Apex Theater in the mall outside of town When the Apex opened last year it featured a video arcade plush carpeting and seats and a state-of-the-art sound system Furthermore in a recent survey over 85 percent of respondents reported that the high price of newly released movies prevents them from going to the movies more than five times per year Thus if the Rialto intends to hold on to its share of a decreasing pool of moviegoers it must offer the same features as Apex

Write a response in which you discuss what questions would need to be answered in order to decide whether the recommendation is likely to have the predicted result Be sure to explain how the answers to these questions would help to evaluate the recommendation

Before following through this business plan the manager should investigate the cause of Rialtorsquos unsuccessful business

The author provides no evidence that the surveyrsquos results are statistically reliable The surveyrsquos sample of 85 percent must be sufficient in size and representative of overall population of the city where Rialto and Apex is serving Lacking evidence of a sufficiently representative sample the author cannot justifiably rely on the survey to draw any conclusion whatsoever The author does not indicate that Apex is indeed currently successful However even if Apex is enjoying success the argument relies on what might be a false analogy between Rialto and Apex In order for Apex to serve as a model that Rialto should emulate the author must assume that all relevant circumstances are essentially the same However this assumption is unwarranted For example the argument overlooks the face that Apex is located in a strategic placemdashbeside a mall where customers can not only watch a movie but also enjoy shopping Therefore simply changing the facility to that of Apex may not lead to success

The author does not mention whether Apex is successful or not Nevertheless even if Apex is currently successful the argument relies on what might be a false analogy between Rialto and Apex In order for Apex to serve as a model that Rialto should emulate the author must assume that all relevant circumstances are essentially the same However this assumption is unwarranted For example the argument overlooks the fact that these two institutions are located in different locations Rialto in downtown and Apex in a mall outside of town Although Apex opened with state-of-the-art facilities the decisive factor in its success could be due to its strategic location of being in a mall People could enjoy both shopping and movies at one location thus they may prefer Apex over Rialto Furthermore the place where people enjoy leisure activities has shifted in the past decades for most cities from downtown to the suburbs Therefore Rialto may not be successful even if it emulates Apexrsquos facilities A better business plan may be relocating Apex to the thriving section of the downtown

Rialto 극장은 지난 50여년간 지역 회관으로써 시내에 위치해 있으면서도 이제 변화를 꾀하지 않으면 문을 닫을

GRE AWA John박 박정어학원

판이다 이 극장은 시외 쇼핑타운에 새로 들어선 Apex 극장의 사례를 본받아야 했다 Apex가 지난해 개업했을 당시 이 극장은 비디오 아케이드 플러쉬 카펫트 바닥과 좌석 그리고 최신 음향시설을 갖추었다 더군다나 최근 조사에서는 응답자의 85 이상이 새로 출시된 영화 입장료가 비싼 탓으로 지난해보다 5배이상의 관람객이 줄어들었다고 나타났다 따라서 Rialto 극장이 줄어들고 있는 관람객을 뺐기지 않고 유지하려면 Apex와 같은 시설들을 갖추어야 할 것이다주장 리알토 극장이 줄어들고 있는 관람객을 뺐기지 않고 유지하려면 Apex와 같은 시설들을 갖추어야 할 것이다

1 조사에서 응답자가 전체를 대표할 수 없다 2 apex 극장이 좋은 시설을 갖추고 있지만 그로 인해 수익이 많이 발생했다는 말이 없으므로 시설투자를

하고도 좋은 결과를 얻을 수 있을지 그 근거가 미흡하다3 좋은 영화가 출시된다면 입장료가 비싸도 영화관에서 꼭 보려고 할 수 있다 4 rialto 가 시설이 아닌 다른 요인에 의해 장사가 안될수도 있다( 우범 지역이라든지)

The following is a recommendation from the business manager of Monarch Books

Since its opening in Collegeville twenty years ago Monarch Books has developed a large customer base due to its reader-friendly atmosphere and wide selection of books on all subjects Last month Book and Bean a combination bookstore and coffee shop announced its intention to open a Collegeville store Monarch Books should open its own in-store cafeacute in the space currently devoted to childrens books Given recent national census data indicating a significant decline in the percentage of the population under age ten sales of childrens books are likely to decline By replacing its childrens books section with a cafeacute Monarch Books can increase profits and ward off competition from Book and Bean

Write a response in which you examine the stated andor unstated assumptions of the argument Be sure to explain how the argument depends on these assumptions and what the implications are for the argument if the assumptions prove unwarranted

The following is a recommendation from the business manager of Monarch Books

Since its opening in Collegeville twenty years ago Monarch Books has developed a large customer base due to its reader-friendly atmosphere and wide selection of books on all subjects Last month Book and Bean a combination bookstore and coffee shop announced its intention to open a Collegeville store Monarch Books should open its own in-store cafeacute in the space currently devoted to childrens books Given recent national census data indicating a significant decline in the percentage of the population under age ten sales of childrens books are likely to decline By replacing its childrens books section with a cafeacute Monarch Books can increase profits and ward off competition from Book and Bean

1 Write a response in which you discuss what questions would need to be answered in order to decide whether the recommendation is likely to have the predicted result Be sure to explain how the answers to these questions would help to evaluate the recommendation

2 Write a response in which you discuss what specific evidence is needed to evaluate the argument and explain how the evidence would weaken or strengthen the argument

No evidence regarding Monarch Bookrsquos successEven if Regal Bookrsquos is successful this may not be attributable to the cafeacute False analogy Emulating may not lead to success Other factors may be involvedInsufficient condition The national census is not enough evidence that childrenrsquos book sales will decline Can

GRE AWA John박 박정어학원

the national census represent the local child populationDid opening a cafeacute boost sales for Regal Books Even assuming Regal is successful by opening a cafeacute this may not be suitable for Monarch which plans to close the childrenrsquos book section to establish a cafe Imprecise language ldquorelatively little spacerdquo how smallThe managerrsquos recommendation contradicts what he says Since Monarch is popular for its wide selection of books closing a selection which targets a major group of readers may hurt Monarchrsquos salesIs this the best way to compete

When Stanley Park first opened it was the largest most heavily used public park in town It is still the largest park but it is no longer heavily used Video cameras mounted in the parks parking lots last month revealed the parks drop in popularity the recordings showed an average of only 50 cars per day In contrast tiny Carlton Park in the heart of the business district is visited by more than 150 people on a typical weekday An obvious difference is that Carlton Park unlike Stanley Park provides ample seating Thus if Stanley Park is ever to be as popular with our citizens as Carlton Park the town will obviously need to provide more benches thereby converting some of the unused open areas into spaces suitable for socializing

Write a response in which you examine the stated andor unstated assumptions of the argument Be sure to explain how the argument depends on these assumptions and what the implications are for the argument if the assumptions prove unwarranted

Stanley 파크가 처음 개장했을 당시 가장 크고 가장 많이 이용되는 공원이었다 아직도 공원중에서는 가장 크지만 이용률은 상당히 떨어졌다 지난달 공원 주차장에 설치해놓은 비디오 카메라를 통해 보면 drop(주차장으로 여겨짐) 이용률이 가장 높았다 수치상으로는 하루 평균 50대의 차량만이 이용하였다 반면 직장 중심거리에 위치한 작은 규모의 Carlton 파크는 주당 무려 150여명 이상이 이용하고 있다 Stanley 파크와는 달리 Carlton 파크에는 의자가 있다는 것이 가장 뚜렷한 차이점이다 따라서 Stanley 파크가 Carlton 파크처럼 시민들이 자주 이용하는 공원이 되기 위해서는 벤치를 설치할 필요가 있으며 이렇게 사용되지 않는 일부 공간을 활용해서 사교를 위한 공간으로 바꾸어야 한다 ===gtdrop 에 대한 첨부사항 (영영사전내용입니다)---- a place or central depository to which something (as mail money or stolen property) is brought for distribution or transmission also the act of depositing something at such a place dropgt

주장 if Stanley Park is ever to be as popular with our citizens as is Carlton Park the town will obviously need to provide more benches thereby converting some of the unused open areas into spaces suitable for socializing1 조사가 언제 이루어진 것인가 조사가 언제 실시되었느냐에 따라 결과가 다를 수있다 현재는 다시 스탠리 파크가 늘어났었을 수 있다 2 벤치를 많이 설치했다고 해서 많은 관광객이 오지 않을수 있다(사람들이 벤치나 사교 공간을 원한다는 어떠한 자료도 없다)3스탠리 파크 주변에 교통 상황이 악화가 되었거나 칼튼 파크에서 문화행사등을 많이 가져서 이용객이 줄어든것일 수도 있다 4 칼튼 파크가 중심지에 있어서 접근성이 좋을수 있다5 조사가 같은 시간을 기준으로 한게 아니다(하나는 주중이고 하나는 주말이다)6사람의 수와 차의 대수를 같은것으로 비교할수 없다 (차안에 몇 명이 타고 있는지 모르고 대중교통을 이용해서 왔을수도 있다)

Page 12: GRE writing argument brain storm

GRE AWA John박 박정어학원

The writer assumes 1 the nationwide survey results can be applied to the local area (Desire cannot reflect consumer trend) 2 Domus Construction was profitable because they had such features (The writer should investigate what other features the DC provides and the actual number of homes sold) 3 State-of-the-art kitchens will sell well (no evidence to buttress his assumption furthermore they would need to spend more money which would increase the price of the housesmdashtheir market will be limited to the affluent) 4 The recent buyers represent the concern of most buyersmdashthat they are ok with a house that is without a dining room

Nationwide survey might not be applicable to this regionDomus Construction could have other superior qualities than Bower Builders Ex DesignSelling fast doesnrsquot indicate quantitySmall yards complaints could be voiced in the future

First the author concludes that a nationwide survey reveals that the two most desired home features are a bathroom with a whirlpool tub and a large kitchen However this nationwide survey may not reflect the trends of the customers that Bower Builders target The author assumes that the nationwide trend reflects customer trends The national study would lead support only if the nationwide trend reflect the home-purchasing trends The nationwide trend may just be a trend of desire which does not result in the act of purchasing However the author does not provide credible evidence that this is the case

전국 조사에서 가장 인기있는 집안 구조 2가지는 훨풀 튜브가 마련된 욕실과 커다른 부엌으로 조사되었다 경쟁업체 Domus Construction이 신축한 완공예정인 주택은 이러한 욕실을 갖추고 있어서 분양이 조기에 이루어지고 있고 가격도 평균치보다 상당히 웃돌고 있다 우리도 집을 많이 팔도 그만큼 이윤을 많이 남기려면 신규 주택에는 위의 2가지 사항을 갖추어야 한다 최근 우리가 지은 주택 수요자들이 적은 평수에 대해서는 별다른 불만사항이 없었기 때문에 앞으로 평수를 더 줄여서 이익을 극대화 할 수 있을 것이다

주장 We should include whirlpool tubs and a large kitchen

1 경쟁업체 Domus Construction이 신축한 완공예정인 주택은 이러한 욕실을 갖추고 있어서라기 보다는 위치가 좋거나 다른 마감재(construction material)나 인테리어가 뛰어나서 분양이 조기에 이루어진것이라고 볼 수 있다 2 전국조사가 건물이 지어지는 지역에 항상 적용되리라고 볼 수 없다 3 평수를 줄이는 것에 불만이 없었다는 것은 editor의 견해일 수 있다 사람들이 표현하지 않은 것일 수도 있고 다른 장점이 많아서 그런 단점을 드러내지 않은 것일 수 있기 때문에 속단할 수 없다 4 다른것을 함으로써 더 잘 팔릴수도 있다

2011 7 19 화요일 GRE AWA 실전반이OO

Argument 94

The president of Bower Builders contends recommends that in order to raise company profits the company should build houses with large family rooms and large state-of-the-art kitchens He uses the results of a nationwide survey on desired home features and the example of Bower Buildersrsquo competitor Domus Construction to support his plan His memo manifests several assumptions about surveys consumer behavior and Domus Constructionrsquos houses that do not

GRE AWA John박 박정어학원

necessarily bolsterbuttress his argumentThe presidentrsquos first unstated but apparent assumption is that a nationwide survey can be

applied to local areas There is no guarantee that the area in which Bower Builders builds houses will follow the national consumer trend Since he only mentions the overall result of the nationwide survey it is difficult to determine more specific conditions such whether people in urban and rural areas have the same housing preferences Such uncertainty is further exacerbated by the nature of the survey question itselfmdash it does not deal with current trends in actual home purchases but with desired home features The naiumlve assumption that consumer desire will directly result in consumer action underlies the presidentrsquos interpretation and application of the survey results to his company plan Not everyone who wishes for a large family room and kitchen will or can actually buy a house with those features mainly due to financial reasons Furthermore the president also assumes for no evident reason that consumers will not hesitate to purchase houses with state-of-the-art kitchens as a standard rather than optional feature when it is clear that such a feature will raise the overall cost significantly He continues to blunder in his mistaken assumptions about consumer behavior in assuming that the taste of recent buyers can represent the concern of future prospective buyers The fact that recent buyers have claimed no need for separate dining rooms does not mean future buyers will feel the same way as well

In addition the president finds a real-life actualization of the nationwide survey results in the recent sales of Domus Construction However he easily assumes that large family rooms and kitchens are the only reasons the houses of Domus Construction sell well He does not take into account other features and selling points of the competitorrsquos houses There may well be other explanations for its houses selling more quickly and expensively including additional home features as well as external factors such as proximity to better schools or superior financial solvency of its clientele If Bower Builders merely added larger family rooms and kitchens without taking into consideration the other factors they may lose rather than gain profits

Clearly the presidentrsquos assertion that Bower Builders make houses with large family rooms and high-tech kitchens at the expense of dining rooms rests on a number of assumptions that are ill-informed and naiumlve If Bower Builders undertakes the proposed plan without further research into local consumer desires purchasing trends and the marketing and sales of competing companies the company will risk losing money by building big new houses that people cannot afford to or will not wish to buy

The following appeared in a letter to the editor of a journal on environmental issues

Over the past year the Crust Copper Company (CCC) has purchased over 10000 square miles of land in the tropical nation of West Fredonia Mining copper on this land will inevitably result in pollution and since West Fredonia is the home of several endangered animal species in environmental disaster But such disasters can be prevented if consumers simply refuse to purchase products that are made with CCCs copper unless the company abandons its mining plans

Write a response in which you examine the stated andor unstated assumptions of the argument Be sure to explain how the argument depends on these assumptions and what the implications are for the argument if the assumptions prove unwarranted

The writer assumes 1 The writer is trying to avoid an inevitability 2 Mining copper will result pollution (they could make preventative measures) 3 The writer may be too late from stopping CCC from developing the area into a copper mine 4 Since mining is an underground enterprise the surface may not be affected that much therefore endangered species may not be affected 4 Consumers wonrsquot buy CCC products if the journal publishes a negative review about CCC (How many readers) CCC could a company that has ties with many IT companies and industries in that their copper is almost ubiquitous in various products

GRE AWA John박 박정어학원

지난 한해동안 CCC(Consolidated Copper 회사)는 서부 플로리다의 열대 지역에 1백만 마일이 넘는 땅을 사들였다 이 곳에서의 채광 활동은 서부 플로리다가 몇몇 멸종 위기에 처한 동물의 서식지이기 때문에 분명 오염과 환경파괴를 가져올 것이다 그러나 이러한 파괴는 CCC 회사가 채광을 포기할 때까지 이 회사가 채굴한 구리로 제조된 제품을 구매하지 않으면 막을 수 있을 것이다

결론 such disaster can be prevented if consumers simply refuse to purchase products that are made with CCCs copper until the company abandons its mining plans

1 구리로 제조된 물건이 생활에 필수적인 것이 많은 만큼 불매가 쉽지 않을 수 있다(전선이나 각종 전자제품에 필수적으로 들어가기 때문에)

2 불매를 유도한다고 해서 소비자들이 구매를 안하는 것은 아니다3 적절한 채굴로 환경파괴를 가져 오지 않을 수 있다 (땅속에 있는 물질을 채굴하는 만큼 생물에 영향을 안

미칠 수도 있다)4 이미 채굴이 다 끝나서 더 이상의 채굴이 없을 수도 있다 5 땅을 구입한다고 해서 채광하는건 아니다 (다른 용도로 샀을수도 있다)

The following is a letter to the editor of an environmental magazine

In 1975 a wildlife census found that there were seven species of amphibians in Xanadu National Park with abundant numbers of each species However in 2002 only four species of amphibians were observed in the park and the numbers of each species were drastically reduced There has been a substantial decline in the numbers of amphibians worldwide and global pollution of water and air is clearly implicated The decline of amphibians in Xanadu National Park however almost certainly has a different cause in 1975 troutmdashwhich are known to eat amphibian eggsmdashwere introduced into the park

Write a response in which you discuss what specific evidence is needed to evaluate the argument and explain how the evidence would weaken or strengthen the argument

Evidence needed the identity of the sender and the census taker of rsquo75 and lsquo02mdasha scientist or an environmentalist credibility issue census methodology vs mere observationmdashthe absence of evidence is not an evidence of absence the season of when the census was taken time-shiftmdashconditions may have changed worldwide decline may include Xanadu other species of predators that prey on amphibians because trout is only one species that prey on amphibians the number of troutmdashhave they increased significantly since rsquo75 First the author needs to be more overt about the credibility of the census and observation There were to accounts that notes the population of amphibiansmdashthe first a census and the second an observation The author needs to bolster his conclusion with the evidence that indicate that the census and second observation were done using scientific methodologies This could either could strengthen or weaken his claim In addition he

GRE AWA John박 박정어학원

needs to provide the specific season of when the census and observation occurred In this letter the writer is informing an editor of an environmental magazine that the number of amphibians was greatly reduced since 1975 and he points out the introduction of trout as the only reason for the decline However the author fails to provide crucial evidences that could strengthen or weaken his conclusion

The following appeared in a memorandum from the president of Hyper-Go Toy Company

Last year sales of our Fierce Fighter toy airplane declined sharply even though the toy had been a top seller for three years Our customer surveys show that parents are now more worried about youthful violence and are concerned about better education for their children Therefore to maintain profits we should discontinue all our action toys and focus exclusively on a new line of educational toys Several other toy companies have already begun marketing educational toys and report sales increases last year of 200 percent And since the average family income is growing sales of new Hyper-Go toys should also increase

Write a response in which you discuss what specific evidence is needed to evaluate the argument and explain how the evidence would weaken or strengthen the argument

First the president indicates that the sales of Fierce Fighter toy airplane declined sharply However he fails to consider the fact that toys are a fad Since FFT enjoyed a three year success it may be natural that the trend would subsideSecond Customer survey is this representative of most toy consumers This trend is not newThird other companies may have profited not through educational but other toys Or educational toy profit may be small in proportion to their sale of other toys 200Parents are not the customers companies should concentrate on what the children want to increase profitLastly parents worry about youthful violence and concern for better education are not new trends

The presidentrsquos decision to discontinue all action toys and focus only on educational toys is too extreme If this decision is taken into effect its procedure may be an onerous task because it would require major shifts in human resources and company image Therefore further data should be considered before following up on this decision

우리 회사에서 지난 3년간 최고 매출을 일으켰던 Fierce Fighter 장난감 비행기가 지난해에는 매출이 급격히 떨어졌다 자체 고객 조사에서는 부모들이 현재 청소년 폭력에 걱정을 하고 있어서 아이들의 양질 교육에 더 관심을 가지고 있는 것으로 나타났다 따라서 회사의 수익을 유지하기 위해서는 모든 자사 전투용 장난감 생산을 중단하고 오로지 교육적인 장난감 생산에 집중해야 한다 몇몇 여타 장난감 회사들도 이미 교육용 장난감 마케팅을 시작해서 지난해에는 200의 매출신장을 가져왔다고 한다 그리고 평균 가계 수입이 점점 늘고 있기 때문에 신형 Hyper-Go 장난감의 매출도 늘어날 것이다

3그룹 Time-shift Error

Woven baskets characterized by a particular distinctive pattern have previously been found only in the immediate vicinity of the prehistoric village of Palea and therefore were believed to have been unique to the Palean people Recently however archaeologists discovered such a Palean basket in Lithos an ancient village across the Brim River from Palea The Brim River is very deep and broad and so the ancient Paleans could only have crossed it by boat but there is no evidence that the Paleans had boats And boats capable of carrying groups of people and cargo were not developed until thousands of years after the Palean people disappeared Moreover Paleans would have had no need to cross the rivermdashthe woods around Palea are full of nuts berries and small game It follows that the so-called Palean baskets were not unique to Palea

GRE AWA John박 박정어학원

Write a response in which you discuss what specific evidence is needed to evaluate the argument and explain how the evidence would weaken or strengthen the argument

RefutationPrehistoric time shift-gtbrim river could have been narrow and shallow or it might have not existed Indigenous patterns may exist in other disconnected remote placesNuts berries small game(hunting) may not have existed or the author should be proved these existed at that time Abundance of resources doesnrsquot support the reason for seclusionBoats not yet found baskets may have been carried across by the river current without the help of a boat Lithos might have crossed the river for commercial purposeOne Palean basket does not substantiate the authorrsquos claimThe absence of evidence is not an evidence of absence

The author assumes without justification that present conditions are the same as at the prehistoric era The author unfairly infers from the presence of Brim River which exist today that it would have existed in the past However the author fails to offer any evidence to substantiate this inference It is very likely that the Brim River might not have existed in prehistoric times or if it did exist may have been shallow and narrow enough for the Paleans to easily cross Any of these scenarios if true would serve to undermine the claim thathelliphelliphelliphelliphellip

First the author claims that the Brim River was very deep and broad so the Paleans could not have crossed it However the author fails to offer any evidence to substantiate that this was true in the prehistoric time of the Paleans For all we know the Brim River might not have existed in the prehistoric era or if it did exist could have been a shallow and narrow river For example scientists believe that thousands of years ago an ice-bridge existed on the Bering Sea connecting Eurasia to what is now North America to explain how the Eskimos and the Asian inhabitants of America came to migrate all over the Americas In this example the absence of evidence is not an evidence of absence Therefore to sufficiently support his claim the author needs to substantiate the fact that the Brim River really did exist and was broad and wide in the Prehistoric Era proving that the Paleans could not have influenced or traded with other groups of people

First the author claims that the Brim River was very deep and broad so the Paleans could not have crossed it However the author fails to offer any evidence to substantiate that this was true in the prehistoric times of the Paleans For all we know the Brim River might not have existed in the prehistoric era or if it did exist could have been a shallow and narrow river For example rivers are created by natural erosion over thousands of years Niagara Falls carved its way from the mouth of Lake Ottawa and created a long river Likewise the current Brim Riverrsquos physical features may not have been wide and deep Therefore to sufficiently support his claim the author needs to substantiate the fact that the Brim River really did exist and was broad and wide in the Prehistoric Era proving that the Paleans could not have influenced or traded with other groups of people

예전에는 실로짠 특이한 무늬 바구니가 Palea의 선사시대 지역의 인근마을에서만 발견되어왔기 때문에 Palea 마을 사람들의 특징이라고 여겨졌었다 그러나 최근들어 고고학자들이 Lithos지역에서 Palean 바구니를 발견하였는데 그 당시 지역은 Brim 강을 가로질러 Palea까지 닿아있었다 이 강은 수심이 아주 깊고 강폭이 넓었으며 때문에 고대의 Palea인들은 배를 이용해서 강을 건널수 있었을 것이다 그러나 이들이 배를 가지고 있었다는 증거는 발견되지 않고 있다 더군다나 이들이 멸명한 이후 수천년이 지난뒤에도 수많은 물자와 사람을 실어 나를수 있는 용적을 가진 배는 개발되지 않았다 이와더불어 Palea인들은 강을 건널필요가 없었는데 그것은 너트나무 장과열매 그리고 작은 사냥감들이 주변숲에 풍부했기때문이다 따라서 Palean 바구니라고 하는 것도 Palea인들만의 전유물이 아니라는 결론을 얻을 수 있다

결론 if follows that the so-called Palean baskets were not unique to Palea이번문제는 굿이 causal Bad analogy 로 구분해서 찾기가 힘드내요 배를 발견했다는 증거가 없는것이다 (앞으로도 발견될수 있음)

GRE AWA John박 박정어학원

계절의 영향으로 겨울에 얼음이 두껍게 언다든지 여름에 가뭄으로 인해서 건널수 있다물자가 풍부한 것이 이동하지 않을 조건이 아니다 다른 것에 의해서 이동가능(의약품등)

Thirteen years ago researchers studied a group of 25 infants who showed signs of mild distress when exposed to unfamiliar stimuli such as an unusual odor or a tape recording of an unknown voice They discovered that these infants were more likely than other infants to have been conceived in early autumn a time when their mothers production of melatonin hormone known to affect some brain functions would naturally increase in response to decreased daylight In a follow-up study conducted earlier this year more than half of these children now teenagers who had shown signs of distress identified themselves as shy Clearly increased levels of melatonin before birth cause shyness during infancy and this shyness continues into later life

Write a response in which you examine the stated andor unstated assumptions of the argument Be sure to explain how the argument depends on these assumptions and what the implications are for the argument if the assumptions prove unwarranted

Any baby exposed to unpleasant stimuli would react in such wayFirst of all the author states 25 infants as his evidence However this research sample is too small to prove his claimSecond the author states that 25 infants were conceived in early autumn which he claims lead to a shy disposition However this is faulty evidenceThird the research study was a long term study done in the span of 13 years However the author only writes about the initial and final stages of the study and leaves out evidences of what could have happened during the 13 years which could be more evidential factors of influenceFourth neither the infantsrsquo genetic predisposition nor their environment were taken into accountFinally the author concludes that his shyness continues into later life (Other factors could alter this disposition epigenetic theory)

13 년전 학자들은 25명의 유아를 대상으로 이상한 냄새나 특이한 소리를 녹음한 테잎등으로 낯선 자극을 주었을때 보이는 미미한 압박감 증상을 조사하였다 이들은 성숙기가 막 지났을 즈음에 보통의 유아들이 비슷한 증상을 보이는 정도 보다는 다소 민감한 반응을 보였는데 이 시기는 아이의 엄마가 뇌의 일부 기능에 영향을 미치는 것으로 알려진 멜라토니아 호르몬을 생산하는 시기로써 이 호르몬은 낯 시간이 짧을때 자연적으로 증가할 수도 있다 금년초에 실시된 추가연구에서 현재 10대로 성장한 당시 조사대상의 절반 이상의 아이들이 부끄럼을 잘타는 것으로 여기고 있었다 따라서 분명한 것은 출산전 멜라토닌 수치의 증가가 유아기에 수줍음 등의 영향을 미치게 되며 이러한 영향이 성장후에도 작용한다는 것이다

주장 Clearly increased levels of melatonin before birth cause shyness during infancy and this shyness continues into later life

1 25명의 아기로 결론 내리기에 샘플이 작다2 과학적 사실들에 대한 명확한 근거가 엇음3 다른 영향을 간과했다(다른 호르몬에 의한 영향 후천적인 성격형성의 영향)

GRE AWA John박 박정어학원

The following is a letter to the editor of the Atticus City newspaper

Former Mayor Durant owes an apology to the city of Atticus Both the damage to the River Bridge which connects Atticus to Hartley and the traffic problems we have long experienced on the bridge were actually caused 20 years ago by Durant After all he is the one who approved the construction of the bridge If he had approved a wider and better-designed bridge on which approximately the same amount of public money would have been spent none of the damage or problems would have occurred Instead the River Bridge has deteriorated far more rapidly over the past 20 years than has the much longer Derby Bridge up the river Even though the winters have been severe in the past several years this is no excuse for the negligence and wastefulness of Durant

Write a response in which you discuss what questions would need to be answered in order to decide whether the recommendation is likely to have the predicted result Be sure to explain how the answers to these questions would help to evaluate the recommendation

전임 시장인 Durant 씨는 Atticus 시에 대해 사과할 의무가 있습니다 Atticus와 Hartley를 잇는 River Bridge 교량에 대한 피해와 이 교량에서 오랫동안 주민들이 겪어오고 있는 교통 혼잡 문제들은 실제로 20년 전부터 시작된 것이었습니다 결정적으로 그가 교량 공사를 허가했던 바로 그 장본인입니다 당시 비슷한 공사비용으로 폭이 더 넓고 튼튼하게 설계된 교량을 허가했다면 이러한 문제나 피해는 발생하지 않았을 겁니다 더군다나 이 다리는 지난 20년 동안 상류에 건설된 훨씬 오래된 Derby 다리보다도 빠르게 부식되어 갔습니다 지난 수년동안 심지어 혹한이 있었다 하더라도 이러한 태만과 국고 손실에 대한 책임을 회피할 길이 없는 것입니다

결론 Former Mayor Durant owes an apology to the city of Atticus

1 디자인이 문제가 아닐수 있다 (디자인은 좋았으나 건설과정에 문제가 있었을 수 있다)2 그 당시의 시예산이 적어서 더 큰 다리를 짓기가 불가능했을 수도 있다3 교통량이 많거나 다른 상황으로 인해서 부식이 빨리 됐을 수 있다4 그 당시에는 최선의 선택이였지만 갑자기 변한 상황에 의해서 이런 문제점들이 발생했을 수 있다

GRE AWA John박 박정어학원

4그룹 거짓인과관계 오류 (False Cause) 빈출

Fifteen years ago Omega University implemented a new procedure that encouraged students to evaluate the teaching effectiveness of all their professors Since that time Omega professors have begun to assign higher grades in their classes and overall student grade averages at Omega have risen by 30 percent Potential employers looking at this dramatic rise in grades believe that grades at Omega are inflated and do not accurately reflect student achievement as a result Omega graduates have not been as successful at getting jobs as have graduates from nearby Alpha University To enable its graduates to secure better jobs Omega University should terminate student evaluation of professors

Write a response in which you discuss what specific evidence is needed to evaluate the argument and explain how the evidence would weaken or strengthen the argument

Omega professor evaluation implemented 15 years ago =gt Omega prof assign higher grades 30Employers believe therersquos grade inflation

Thus unsuccessful employment than AlphaTherefore to secure jobs Omega should end evaluating profs

Specific evidence neededRelationship between higher grades and evaluationRelationship between GPA and unsuccessful employmentAlpharsquos education could just be better than OmegaldquoFifteen years agordquo is a long time other factors could have influenced Why is the inflation a problem just now How much is Alpha better Is the comparison just How much gap is thereOmegarsquos student could just be doing better in their studiesComparison to other universities다른 대안 없나hellip Could Omega alleviate the employment problem by implementing a different procedure or program

15 년전 우리 대학은 학생들로 하여금 교수평가를 하도록 한 새로운 조치를 시행했었습니다 이후 교수들은 자신의 학과 학생들에게 높은 학점을 주었으며 그에따라 학생들의 전체 평점이 30나 올랐습니다 외부의 기업체들은 분명 점수가 지나치게 부풀려졌다고 믿고 있습니다 결국 본 대학 졸업생들이 인근 Alpha 대학의 졸업자들보다 구직률이 떨어지는 이유를 잘 보여주고 있는 것입니다 이를 해결하기 위해 이제부터는 학생들에 의한 교수평가제를 중단해야 합니다

결론 Omega University should now terminate student evaluation of professors

반박 교수 평가와 학점 인플레의 연관성이 적다( 교수 평가를 먼저하고 학점을 나중에 매길수도 있다)채용기준에 성적만 있는게 아니다 학업성취의 결과 일수도 있다 Alpha 가 원래 유능했다 Alpha 의 교육내용이 좋았다

GRE AWA John박 박정어학원

In this memo the dean of Omega University(OU) recommends OU to terminate professor evaluation to secure better jobs for the students To support this recommendation the dean offers several reasons However this argument contains several logical flaws which render it unconvincing

A threshold problem with the argument involves the voluntary nature of the evaluationprocedure The dean provides no evidence about the number or percentage of Omegastudents who participate in the procedure Lacking such evidence it is entirely possible thatthose numbers are insignificant in which case terminating the procedure is unlikely to haveany effect on the grade average of Omega students or their success in getting jobs aftergraduationThe argument also assumes unfairly that the grade-average increase is the result of theevaluation procedure--rather than some other phenomenon The dean ignores a host of otherpossible explanations for the increase--such as a trend at Omega toward higher admissionstandards or higher quality instruction or facilities Without ruling out all other possibleexplanations for the grade-average increase the dean cannot convince me that by terminatingthe evaluation procedure Omega would curb its perceived grade inflation let alone help itsgraduates get jobsEven if the evaluation procedure has resulted in grade inflation at Omega the deans claimthat grade inflation explains why Omega graduates are less successful than Alpha graduatesin getting jobs is unjustified The dean overlooks a myriad of other possible reasons forOmegas comparatively poor job-placement record Perhaps Omegas career services areinadequate or perhaps Omegas curriculum does not prepare students for the job market aseffectively as Alphas In short without accounting for other factors that might contribute toOmega graduates comparative lack of success in getting jobs the dean cannot justify theclaim that if Omega curbs its grade inflation employers will be more likely to hire OmegagraduatesFinally even if the dean can substantiate all of the foregoing assumptions the deansassertion that Omega must terminate its evaluation procedure to enable its graduates to findbetter jobs is still unwarranted in two respects First the dean ignores other possible ways bywhich Omega can increase its job-placement record--for example by improving its publicrelations or career-counseling services Second the dean unfairly equates more jobs withbetter jobs In other words even if more Omega graduates are able to find jobs as a result ofthe deans recommended course of action the kinds of jobs Omega graduates find would notnecessarily be better onesIn sum the deans argument is unpersuasive as it stands To strengthen it the dean mustprovide better evidence that the increase in grade average is attributable to Omegasprofessor-evaluation procedure and that the end result is a perception on the part ofemployers that Omega graduates are less qualified for jobs than Alpha graduates To betterassess the argument I would need to analyze 15-year trends in (l) the percentage of Omegastudents participating in the evaluation procedure (2) Omegas admission standards andquality of education and (3) Omegas emphasis on job training and career preparation I wouldalso need to know what other means are available to Omega for enabling its graduates to findbetter jobs

GRE AWA John박 박정어학원

The following appeared in a memo from a vice president of Quiot Manufacturing

During the past year Quiot Manufacturing had 30 percent more on-the-job accidents than at the nearby Panoply Industries plant where the work shifts are one hour shorter than ours Experts say that significant contributing factors in many on-the-job accidents are fatigue and sleep deprivation among workers Therefore to reduce the number of on-the-job accidents at Quiot and thereby increase productivity we should shorten each of our three work shifts by one hour so that employees will get adequate amounts of sleep

Write a response in which you examine the stated andor unstated assumptions of the argument Be sure to explain how the argument depends on these assumptions and what the implications are for the argument if the assumptions prove unwarranted

The following appeared in a memo from a vice president of Alta Manufacturing

During the past year Alta Manufacturing had thirty percent more on-the-job accidents than nearby Panoply Industries where the work shifts are one hour shorter than ours Experts believe that a significant contributing factor in many accidents is fatigue caused by sleep deprivation among workers Therefore to reduce the number of on-the-job accidents at Alta we recommend shortening each of our three work shifts by one hour If we do this our employees will get adequate amounts of sleep

Write a response in which you discuss what questions would need to be answered in order to decide whether the recommendation and the argument on which it is based are reasonable Be sure to explain how the answers to these questions would help to evaluate the recommendation

The following appeared in a memo from the vice president of Butler Manufacturing

During the past year workers at Butler Manufacturing reported 30 percent more on-the-job accidents than workers at nearby Panoply Industries where the work shifts are one hour shorter than ours A recent government study reports that fatigue and sleep deprivation among workers are significant contributing factors in many on-the-job accidents If we shorten each of our work shifts by one hour we can improve Butler Manufacturings safety record by ensuring that our employees are adequately rested

1 Write a response in which you discuss what specific evidence is needed to evaluate the argument and explain how the evidence would weaken or strengthen the argument

2 Write a response in which you discuss what questions would need to be answered in order to decide whether the recommendation is likely to have the predicted result Be sure to explain how the answers to these questions would help to evaluate the recommendation

4번 반복됨

Alta has 30 more job accidents than Panoply(work shifts one hour shorter)Experts Job accidents caused by fatigue and sleep deprivationTherefore to reduce job accidents and increase productivity shorten three work shifts by one hour for adequate sleep

지난해 우리 회사는 인근 Panoply Industries보다 업무상 재해가 30나 더 많았다 그 회사는 우리보다 근무 교대시간이 1시간 정도 짧았다 전문가들은 대부분의 업무상 재해에 있어서 가장 중요한 요인이 과로와 수면부족으로 보고있다 따라서 우리 회사에서 높은 산업재해를 줄이고 아울러 생산성을 높이기 위해서는 근로자들이 충분한 수면을 취할 수 있도록 1시간씩 3교대 시간을 줄여야 한다

In this memo the (author) vice president of Alta Manufacturing (AM) recommends that to reduce on-the-job accidents and increase productivity AM should shorten its three work shifts by one hour so that employees can

GRE AWA John박 박정어학원

get more sleep To support this recommendation the author provides several evidences However careful scrutiny of each of the facts reveals that it provides little credible support for the authorrsquos recommendation QuestionsThe number of accidents What kind of accidents The seriousness of the accidents is importantHow many employees are in each company What are their productsFalse cause Sleep may not be the reason for the on-the-job accidents What do Alta and Panoply manufacture

First of all the author believes that fatigue caused the on-the job accidents However there could be other reasons The author observes a correlation between sleep deprivation and on-the-job accidents then concludes that the former is the cause of the latter However the author fails to rule out other possible explanations For example it is entirely possible that Alta factories require more strenuous and dangerous labor than Panoply Without ruling out all other such factors it is unfair to conclude that fatigue is responsible for the accidents In addition the work-shifts may not be the cause of the sleep deprivation and fatigue It is possiblehellip Thus the author should provide what exactly Panoply and Alta manufacture and more precise data about their working conditions to be more convincing

Shortening the shift by one hour does not necessarily lead to more sleep And is one hour enoughLess accidents does not mean increased productivity

결론 We should shorten each of out three work shifts by one hour

반박 경쟁사에 비해서 시간당 하는 업무량이 많아서 더욱 피곤할 수도 있다 시간이 문제가 아니라 노후된 시설 설비 자체의 문제 작업 자체가 원래 위험한 것이여서 사고가 많을 수도 있다 다른 회사는 더욱 많은 작업시간에도 불구하고 안정한 작업여건으로 인해서 사고율이 오히려 더 작을 수도 있다비교사의 재해감소가 다른 요인일수 있다(안전 교육 철저)줄인 시간이 피로회복이나 수면으로 연결 안될수 있음(술을 마실 수도 있고 그 시간에 휴식을 취하지 않고 다른일을 함으로써 더욱 피로해질수 있다)

This editorial recommends that Alta Manufacturing reduce its work shifts by one hour each inorder to reduce its on-the-job accident rate and thereby increase Altas productivity To supportthis recommendation the author points out that last year the number of accidents at Alta was30 greater than at Panoply Industries where work shifts were one hour shorter The authoralso cites certain experts who believe that many on-the-job accidents are caused by fatigueand sleep deprivation I find this the argument unconvincing for several reasonsFirst and foremost the author provides absolutely no evidence that overall workerproductivity is attributable in part to the number of on-the-job accidents Although commonsense informs me that such a relationship exists the author must provide some evidence ofthis cause-and-effect relationship before I can accept the authors final conclusion that theproposed course of action would in fact increase Altas productivitySecondly the author assumes that some accidents at Alta are caused by fatigue or sleepdeprivation However the author overlooks other possible causes such as inadequateequipment maintenance or worker training or the inherent hazards of Altas manufacturingprocesses By the same token Panoplys comparatively low accident rate might be attributablenot to the length of its work shifts but rather to other factors such as superior equipmentmaintenance or worker training In other words without ruling out alternative causes ofon-the-job accidents at both companies the author cannot justifmbly conclude that merely byemulating Panoplys work-shift policy Alta would reduce the number of such accidentsThirdly even assuming that Altas workers are fatigued or sleep-deprived and that this is thecause of some of Altas on-the-job accidents in order to accept the authors solution to thisproblem we must assume that Altas workers would use the additional hour of free time tosleep or rest However the author provides no evidence that they would use the time in thismanner It is entirely possible that Altas workers would use that extra hour to engage in someother fatiguing activity Without ruling out this possibility the author cannot convincinglyconclude that reducing Altas work shifts by one hour would reduce Altas accident rateFinally a series of problems with the argument arise from the scant statistical information onwhich it relies In comparing the number of accidents at Alta and Panoply the author fails toconsider that the per-worker accident rate might reveal that Alta is actually safer than Panoplydepending on the total number of workers at each company Second perhaps accident rates

GRE AWA John박 박정어학원

at the two companies last year were aberrations and during other years Altas accident ratewas no greater or even lower than Panoplys rate Or perhaps Panoply is not representativeof industrial companies generally and that other companies with shorter work shifts have evenhigher accident rates In short since the argument relies on very limited statistical information Icannot take the authors recommendation seriouslyIn conclusion the recommendation for emulating Panoplys work-shift policy is not wellsupported To convince me that shorter work shifts would reduce Altas on-the-job accidentrate the author must provide clear evidence that work-shift length is responsible for some ofAltas accidents The author must also supply evidence to support her final conclusion that alower accident rate would in fact increase overall worker productivity

The following appeared in a memo from the vice president of marketing at Dura-Sock Inc

A recent study of our customers suggests that our company is wasting the money it spends on its patented Endure manufacturing process which ensures that our socks are strong enough to last for two years We have always advertised our use of the Endure process but the new study shows that despite our socks durability our average customer actually purchases new Dura-Socks every three months Furthermore our customers surveyed in our largest market northeastern United States cities say that they most value Dura-Socks stylish appearance and availability in many colors These findings suggest that we can increase our profits by discontinuing use of the Endure manufacturing process

1 Write a response in which you examine the stated andor unstated assumptions of the argument Be sure to explain how the argument depends on these assumptions and what the implications are for the argument if the assumptions prove unwarranted

2 Write a response in which you discuss what specific evidence is needed to evaluate the argument and explain how the evidence would weaken or strengthen the argument

3 Write a response in which you discuss what questions would need to be answered in order to decide whether the recommendation and the argument on which it is based are reasonable Be sure to explain how the answers to these questions would help to evaluate the recommendation

Intro The vice president of marketing at Dura-Sock Inc is offering a potentially harmful investment recommendation by claiming that Dura-Sock should discontinue its use of the ldquoEndurerdquo process To support his recommendation he points out a study that Dura-Sock customers actually purchase the socks every three months and a survey that reveals that Dura-Sock customers like the sockrsquos stylish appearance and availability in many colors The study and survey however are insufficient in supporting his proposal and the VP makes several unwarranted assumptionsIntro (simplified) The VP states that though Dura-Socks last for two years customers buy the socks every three months Therefore he assumes that the consumersrsquo motive for buying the produce is not its durabilityHowever the author fails to rule out other possible motivation for consumption

Even if the survey is reliable the author should consider the rest of the market Vague terms ldquowasting moneyrdquomdashprecisely how much are they wasting Studysurvey errorThe company must calculate the outcome of such momentous decisionStudy participantsrsquo comment that they prefer Dura-Sock for its stylishness and availability might take Dura-Sockrsquos enduring quality for granted

우리회사 제품 소비자들에 대한 최근 조사에서 지난 2년여간 양말의 내구성을 강하게 하는 필수공정이었던 자사 특허의 Endure 공정에 들어가는 비용이 낭비라고 말하고 있다 우리 회사는 항상 이 공정 처리에 대한 광고를 내보냈으나 이에 대한 시장 조사에서 실제로 고객들은 이 신제품을 평균 석달마다 구매하는 것으로 나타났다 더군다나 북동부지역에서 실시한 대규모 시장조사에 응답한 고객들은 양말의 모양과 색상등에 더

GRE AWA John박 박정어학원

관심을 나타냈다 이러한 결과는 우리회사가 신기술 공법을 중단하면 그에 따라 수익이 늘어날 것이라는 것을 말해주고 있는 것이다주장 These findings suggest that Dura0Sock can increase its profits by discontinuing its use of the ldquoEndurerdquo manufacturing process

1 survey가 정확한 소비자의 의견을 나타낸 것인가 다른 선택없이 양자택일과 같은 방법의 survey였는지2 북동부지역의 시장조사가 전체 의견을 대표할 수 있나3 사람들이 모양이나 색상에 앞서 내구성을 먼저 평가했을 수도 있다 내구성을 갖추었다는 전제하에 모양과 색상에 관심을 드러낸 것일 수 있다4 소비자가 도매상(retail)인지 소매상(whole)인지가 없다

The following appeared in a business magazine

As a result of numerous complaints of dizziness and nausea on the part of consumers of Promofoods tuna the company requested that eight million cans of its tuna be returned for testing Promofoods concluded that the canned tuna did not after all pose a health risk This conclusion is based on tests performed on samples of the recalled cans by chemists from Promofoods the chemists found that of the eight food chemicals most commonly blamed for causing symptoms of dizziness and nausea five were not found in any of the tested cans The chemists did find small amounts of the three remaining suspected chemicals but pointed out that these occur naturally in all canned foods

Write a response in which you discuss what questions would need to be addressed in order to decide whether the conclusion and the argument on which it is based are reasonable Be sure to explain how the answers to the questions would help to evaluate the conclusion

Representativeness of the tested cansThey should conduct a comparative studyThe testing could be biased because Promofoods employees conducted the testingHow much (quantity) of the five and three suspected chemicals were in the canned foodsFalse cause The substance that caused dizziness and nausea may not be one of the eight common chemicals

많은 소비자들의 현기증과 구역질 불만에 따라 Promofoods사는 지난해 참치 캔 8백만 개를 테스트하기 위해 반품시켰다 그 결과 캔에서는 건강에 위험이 될 수 있는 화합물질이 없었던 것으로 회사측은 결론지었다 이러한 결론은 회사측 화학연구자들이 회수된 캔의 샘플을 테스트해서 이들 증상의 원인이 되는 8가지 화합물 중에서 5가지가 실험된 캔에서 발견되지 않았다는 사실에 근거한 것이다 이들 화학자들은 나머지 3개가지 화합물이 모든 캔 식료품에서 흔히 발견되는 것이라고 언급했다 결론 Promofoods concluded that the cans did not after all contain chemicals that posed a health risk

1 공인된 기간에서 테스트를 한 것이 아니고 자사에서 직접 테스트를 했기에 신뢰성이 안간다 2 이런 증상을 일으키는 8개의 물질 말고 다른 물질들이 캔속에 많이 포함됬을수 있다 3 나머지 3개의 물질들의 함유량이 많아서 다른 종류의 캔들은 문제를 일으키지 않지만 참치캔은 문제를

일으킬 수 있다

This magazine article concludes that the 8 million cans of tuna Promofoods recalled due tocomplaints about nausea and dizziness do not after ail contain any chemicals that pose a

GRE AWA John박 박정어학원

health risk To support this conclusion the author cites the fact that five of eight chemicalscommonly causing these symptoms were not found in the recalled cans while the other threealso occur naturally in other canned foods For several reasons this evidence lends littlecredible support to the authors conclusionTo begin with the author relies partly on the fact that although three of the eight chemicalsmost commonly blamed for nausea and dizziness appeared in Promofoods recalled tunathese chemicals also occur naturally in other canned foods However this fact alone lends nosupport to the authors conclusion for two reasons First the author might be ignoring animportant distinction between naturally occurring chemicals and those not occurring naturallyIt is entirely possible that these three chemicals do not occur naturally in Promofoods tunaand that it is for this reason that the chemicals cause nausea and dizziness Secondly it isentirely possible that even when they occur naturally these chemicals cause the samesymptoms Unless the author rules out both possibilities he cannot reliably conclude that therecalled tuna would not cause these symptomsAnother problem with the argument is that the authors conclusion is too broad Based onevidence about certain chemicals that might cause two particular heath-related symptoms theauthor concludes that the recalled tuna contains no chemicals that pose a health risk Howeverthe author fails to account for the myriad of other possible health risks that the recalled tunamight potentially pose Without ruling out all other such risks the author cannot justifiablyreach his conclusionA third problem with the argument involves that fact that the eight particular chemicals withwhich the test was concerned are only the eight most commonly blamed for nausea anddizziness It is entirely possibly that other chemicals might also cause these symptoms andthat one or more of these other chemicals actually caused the symptoms Without ruling outthis possibility the author cannot jusufiably conclude that the recalled tuna would not causenausea and dizzinessA final problem with the argument involves thetesting procedure itself The author providesno information about the number of recaUed cans tested or the selection method used Unlessthe number of cans is a sufficiently large sample and is statistically repre sentative of all therecalled cans the studys results are not statistically reliableIn conclusion the article is unconvincing as it stands To strengthen the assertion that therecalled tuna would not cause nausea and dizziness the author must provide evidence thatthe three chemicals mentioned that occur naturally in other canned foods also appear naturallyin Promofoods tuna The author must also provide evidence that ingesting other canned foodscontaining these three chemicals does not cause these symptoms To better evaluate theargument we would need to know whether the sample used in the tests was statisticallysignificant and representative of all the recalled tuna We would also need to know what otherchemicals in the recalled tuna might pose any health risk at all

5그룹 불충분 조건오류 빈출

Natures Way a chain of stores selling health food and other health-related products is opening its next franchise in the town of Plainsville The store should prove to be very successful Natures Way franchises tend to be most profitable in areas where residents lead healthy lives and clearly Plainsville is such an area Plainsville merchants report that sales of running shoes and exercise clothing are at all-time highs The local health club has more members than ever and the weight training and aerobics classes are always full Finally Plainsvilles schoolchildren represent a new generation of potential customers these schoolchildren are required to participate in a fitness-for-life program which emphasizes the benefits of regular exercise at an early age

Write a response in which you examine the stated andor unstated assumptions of the argument Be sure to

GRE AWA John박 박정어학원

explain how the argument depends on these assumptions and what the implications are for the argument if the assumptions prove unwarranted

False cause

First of all the author believes that the Increased sales of running shoes and exercise clothing indicates

Plainesville residentsrsquo interest in leading healthy lives However this assumption is not logically convincing for

several reasons could be a fashion trendTime shift ldquoFitness for liferdquo might not have any influence on schoolchildren as they growFalse cause There could be other reasons for member increase in the health clubAll of the above are insufficient condition

The author has to prove that local residents are interested in leading healthy lives However he supports his conclusion with insufficient evidence Nevertheless even if the residents are concerned with health naturersquos way may not be successful First

그 동안의 경험을 토대로 볼 때 건강생활과 밀접히 관련되어 있는 거주 지역에서 본 상점들이 아주 호응을 얻고 있다 따라서 이러한 주민들이 많이 거주하고 있는 Plainsville 에 새로운 상점들을 계속 세워야 한다 이 지역 상인들은 런닝화와 운동복 판매가 가장 높다고 말한다 불과 5 년전에는 거의 전무하다시피하던 지역 헬스 클럽의 경우도 엄청나게 많은 회원을 확보하고 있으며 웨이트 트레이닝과 에어로빅 강좌들도 항상 만원이라고 한다 새로운 고객층을 예측해 보는 것도 가능하다 이 지역의 학생들의 경우 Fitness for Life프로그램을 받게 되는데 이러한 프로그램을 통해서 유년시절부터 정규적인 운동 습관을 들이게 하고 있는 것이 그것이다

결론 We should therefore build our next new store in Plainsville

반박 그동안의 경험에 의한 과거 통계가 꼭 여기에도 적용되는건 아니다 5 년전 헬스 클럽이 잘 안되었던게 다른 원인이였을수 있다(강사수준 미달 강좌미비)tourist 에 의한 원인 일수 있다 어렸을때부터 운동을 했다고 해서 커서도 관심이 있지는 않다 (오히려 반감이 있을수 있다 혹은 건강하기에 건강에 관심이 적을수도 있다)운동복이나 신발의 판매가 육체노동에 의한 것일수도 있다

IntroductionSupport1049896In this memorandum the author asserts that Naturersquos Way should build its next newstore in Plainsville To support this assertion the author states that Plainsvillesmerchantsrsquo sales of exercise clothing are going well the local health club has moremembers than ever and a new generation of customers will help to ensure NaturersquosWayrsquos success At first glance the authorrsquos assumption seems convincing but in-depth scrutiny revealsthat it lacks substantial evidence as it stands

Body 1-SamplingTopic Sentence 1To begin with the author assumes that the merchantsrsquo report indicates that the residentsare concerned about their health However this assumption is based on unsubstantiated

GRE AWA John박 박정어학원

data Example 1 (Rebuttal1) First if we do not know the total volume of items sold and the price of the goods exactly we cannot infer whether the residents are actually buying many goods Example 2 (Rebuttal2)In addition to that the report emphasizes the rising sales of running shoes and exerciseclothing however these may not be hot-selling items for Naturersquos Way or may not be theproducts the company is planning to sell Concluding Sentence Therefore in order to make the argument reliable the author should reconsider themerchantsrsquo report with more detailed data

Body 2-CausalTopic Sentence 2Second the author contends that the health clubs classes are full yet this does not meanthat many people actually use the health club other factors may be the real cause forthose closed classes Example 1 (Rebuttal1) To begin with if the health club is very small the number of people working out wouldnot be a large one In fact regular gym-going may just be a vogue among a smallunrepresentative segment of Plainsvilles population Example 2 (Rebuttal2) Moreover it is possible that most of the people who exercise in the health club do weight training and aerobics only to look good and to meet other singles not for their health In that case there would be little demand for health products Concluding SentenceThus the author should not hasten to presume what really caused people to be interested in a healthier lifestyle and enroll in the health club

Body 3-Time-ShiftTopic Sentence 3Finally the author highlights that Naturersquos Way can expect a new generation of customersin Plainsville that will help the company in the long term This notion is mistaken in that itassumes the conditions of the present will continue unchanged in the future Although theschool children are required to participate in the fitness for life program they may notnecessarily buy Naturersquos Ways products Example 1 (Rebuttal1) In the first instance they may suffer a fall in purchasing power arising from future economic difficulties this would cause reluctance to spend a considerable amount of money on health products which tend to be more expensive Example 2 (Rebuttal2)Another possibility is that there may emerge many competitor companies vying with Naturersquos Way so that in the future the school children may not feel the necessity to purchase one companyrsquos health products over anotherrsquosConcluding Sentence Thus the authorrsquos assumption is highly speculative since it relies heavily on unknowablefuture circumstances

ConclusionThesis In sum the author uses many assumptions that are insufficient in supporting his claimsSupportIn order for the authorrsquos claims to be convincing he needs to advance more persuasiveevidence that people in Plainsville really are concerned with their health and health foodThe following was written as a part of an application for a small-business loan by a group of developers in the city of Monroe

A jazz music club in Monroe would be a tremendously profitable enterprise Currently the nearest jazz club is 65 miles away thus the proposed new jazz club in Monroe the C-Note would have the local market all to itself Plus jazz is extremely popular in Monroe over 100000 people attended Monroes annual jazz festival last summer several well-known jazz musicians live in Monroe and the highest-rated radio program in Monroe is Jazz Nightly which airs every weeknight at 7 PM Finally a nationwide study indicates that the typical jazz fan spends close to $1000 per year on jazz entertainment

1 Write a response in which you discuss what specific evidence is needed to evaluate the argument and explain how the evidence would weaken or strengthen the argument

2 Write a response in which you examine the stated andor unstated assumptions of the argument Be

GRE AWA John박 박정어학원

sure to explain how the argument depends on these assumptions and what the implications are for the argument if the assumptions prove unwarranted

3 Write a response in which you discuss what questions would need to be answered in order to decide whether the prediction and the argument on which it is based are reasonable Be sure to explain how the answers to these questions would help to evaluate the prediction

Group error nationwide survey may not reflect local trends Is the nationwide jazz fan population substantialInsufficient non-residents of Monroe may have attended the jazz festival (Body alternative explanation last year may have been an anomaly The author should consider data from various years) The author should indicate how many out of 100000 were Monroe residentsNationwide study Does this reflect Insufficient Citizens of Monroe may continue to go to the jazz club 65 miles away

Are the people in Monroe really interested in jazzMajority of the people who attended the jazz festival might not be Monroe residentsSurvey error nationwide study may not be applicable to MonroeJazz musicians who live in MonroeMonopolyRadio station

In this business application the author claims that the proposed jazz club C Note will be very profitable in Monroe To support this claim the author argues for his case with several evidences At first glance the authorrsquos argument seems convincing however careful scrutiny reveals that his argument in specious

To begin with the author claims that Monroersquos citizens are interested in jazz He presents three evidences First Secondhellip Thirdhellip Howeverhellip

Monroe 시에 있는 재즈 음악 클럽은 수익성이 좋은 사업이다 현재 가장 가까이에 있는 클럽은 65 마일 정도 떨어져 있다 따라서 이번에 세우려고 하는 C Note 는 독보적인 위치를 점할것이다 더군다나 재즈는 이 시에서 가장 인기있는 음악이다 지난 여름 재즈 축제에서는 10 만명 이상의 Morone 시 주민이 참석하였고 몇몇 유명한 재즈 음악가들도 이곳에 살고 있으며 저녁때 방영되는 라디오 프로그램중에서 최고의 시청률을 보이고 있는 것도 Jazz Nightly 이다 전국조사에서도 전형적인 재즈 팬들은 재즈 분야에 년간 1천 달러 가까이 지출하고 있는 것으로 보고되고 있다 따라서 C Note 클럽이 돈을 벌 수 있는 사업이라는 것은 확실한 것이다

결과 It is clear that the C Note cannot help but make money반박 nearest jazz club 이 양질의 써비스로 여전히 손님을 끌수도 있다Festival 에 얼마나 참여하는지가 jazz 의 인기를 반영하지 않는다 뮤지션이 많이 사는거랑 jazz 의 인기가 상관없다라디오 프로그램이 다른 요인에 의해서 인기일수도 있다 (진행자때문)전국 통계 적용 불가화목 실전반_Ms Noh6In this application the author suggests that a jazz club in Monroe will make a number of profits To support this suggestion the author exemplifies the local condition popularity of jazz in Monroe and nationwide study However careful scrutiny of each of the facts reveals that it provides little credible support for the authorrsquos recommendation Good clear intro

First the author assumes that jazz is popular in Monroe because of several facts the jazz festival last year had high participation some famous jazz musicians live in Monroe and the high-rated radio program is lsquoJazz Nightlyrsquo However this assumption has many drawbacks that must be seriously considered(Good topic sentences) If many attendants in the last-yearrsquos festival came from other cities and not Monroe it is hard to conclude that Monroersquos people like jazz Therefore the author must examine how many Monroe residents actually attended the festival On top of that there is little relationship between habitation of famous jazz musician and the popularity of jazz in Monroe Although several well-known musicians live there if they do not take part in any jazz performance of Monroe this might have no effect to the interest of Monroersquos residents

GRE AWA John박 박정어학원

about jazz Finally in the case of radio program this is also not suitable reason why jazz is popular in Monroe It might be possible that people cannot help choosing lsquoJazz Nightlyrsquo because there are few radio programs at Night The fact that the radio program is the highest rating program is not a germane evidence The approximate number of listeners would be the more crucial evidence Therefore the author needs to seriously deliberate the correlation between jazzrsquos popularity in Monroe and his examples (Good logical flow and clarity)

Second the author uses as evidence the nationwide study that jazz fans spend much money on jazz entertainment to substantiate why starting a jazz club in Monroe will be profitable In other words the author assumes that the characteristics of a nationwide study can be applied to Monroe The national study would lend support to the applicantrsquos claim only if residents in Monroe typify national jazz fans However the author does not provide credible evidence that this is the case Moreover the populations of jazz fans nationwide may be insubstantial Thus the author should not infer hastily that Monroersquos residents will spend much money on enjoying jazz from the nationwide study

Lastly even if jazz is popular in Monroe C Note may not be successful It is entirely possible that residents might still prefer other clubs where they have always went In addition there is another possibility that the nearest jazz club will attract many of Monroersquos people because it serves fine performances and is equipped with favorite facilities Without considering these other possibilities the author cannot make his argument convincing In sum the author presents many reasons that are insufficient in supporting his or her claim In order for the authorrsquos claims to be convincing he needs to advance more persuasive evidence such as the total number of Monroe residents who attended the jazz festival the effects on the popularity of jazz by the musicians living in Monroe and the actual number of residents who would typify themselves to be jazz fans through a local survey Without substantial evidence that C Note will be successful in Monroe the businessmen may be overinvesting in what might lead to a business failureExcellent clarity Score 50

The following appeared in a newsletter offering advice to investors

Over 80 percent of the respondents to a recent survey indicated a desire to reduce their intake of foods containing fats and cholesterol and today low-fat products abound in many food stores Since many of the food products currently marketed by Old Dairy Industries are high in fat and cholesterol the companys sales are likely to diminish greatly and company profits will no doubt decrease We therefore advise Old Dairy stockholders to sell their shares and other investors not to purchase stock in this company

Write a response in which you discuss what questions would need to be answered in order to decide whether the advice and the argument on which it is based are reasonable Be sure to explain how the answers to these questions would help to evaluate the advice

Survey 80

GRE AWA John박 박정어학원

Old Dairy could change their products and manufacture low fat dairy foodsLess competing companies Old Dairy could eventually be the only company that produces hellipImprecise numbers and measurementsCustomers may still buy high fat dairy products

The author of the newsletter is offering potentially dangerous advice by recommending Old Dairy stockholders to withdraw investment and stop purchase What is more the authorrsquos prediction debases the reputation and business of Old Dairy and if false could devoid the investment opportunity of the newsletter readers Therefore investors should examine whether the authorrsquos evidences are substantial

To begin with the author states that 80 percent of the respondents in a survey indicated a desire to reduce their intake of foods He therefore argues that Old Dairyrsquos high fat and cholesterol products would decrease in sales However the author makes a crucial error in this argument First the author provides no evidence that the surveyrsquos results are statistically reliable Were they representative of all the customers Were they chosen for the survey randomly Furthermore the desire to reduce fat and cholesterol intake is a pervasive trend in todayrsquos opulent society however the author erroneously identifies this as a new phenomenon which will affect consumer trends Second having a desire to reduce fat and cholesterol intake does not necessarily indicate that people who have this desire will actually reduce consuming these types of products It is entirely possible that they may continue buying Old Dairy products for its quality and taste Accordingly the author cannot draw any firm conclusion that people will not buy Old Dairy products Therefore if any of these cases are true the author may be offering investors a detrimental investment advice

최근 조사에 대한 응답자중 80 이상이 자신이 먹는 음식에서 지방과 콜레스테롤의 함유량을 줄이고 싶다고 한다 아울러 요즘은 많은 식료품 가계에서 저지방 제품들을 많이 취급하고 있다 현재 Old Dairy Industries가 판매하고 있는 많은 음식제품들은 지방과 콜레스테롤이 높기 때문에 이 회사의 매출이 격감할 것으로 보이며 당연히 매출이익도 줄어들것이다 따라서 이 회사의 주주들은 주식을 매각하고 다른 주식 투자가들도 이 회사의 주식을 매입하지 않는 것이 좋다

결론 Old Dairy stockholders to sell their shares and other investors not to purchase stock in this company

반박 모든 상품이 다 고 지방 고 칼로리는 아니다(비록 많을지라도) 일부의 식품의 경우 기호에 맞어서 히트해서 전체적인 수입이 증가할 수도 있다국내시장만 생각할 수 없다( 외국시장에서 호황을 누릴수 있다 )입맛이라는게 즉각 바뀌는게 아니다

The following appeared in a letter to the editor of the Balmer Island Gazette

On Balmer Island where mopeds serve as a popular form of transportation the population increases to 100000 during the summer months To reduce the number of accidents involving mopeds and pedestrians the town council of Balmer Island should limit the number of mopeds rented by the islands moped rental companies from 50 per day to 25 per day during the summer season By limiting the number of rentals the town council will attain the 50 percent annual reduction in moped accidents that was achieved last year on the neighboring island of Seaville when Seavilles town council enforced similar limits on moped rentals

1 Write a response in which you discuss what questions would need to be answered in order to decide whether the recommendation is likely to have the predicted result Be sure to explain how the answers to these questions would help to evaluate the recommendation

2 Write a response in which you discuss what questions would need to be answered in order to decide whether the prediction and the argument on which it is based are reasonable Be sure to explain how the answers to these questions would help to evaluate the prediction

3 Write a response in which you examine the stated andor unstated assumptions of the argument Be sure to explain how the argument depends on these assumptions and what the implications are for the argument if the assumptions prove unwarranted

Whatrsquos the actual population of Balmer Island 100000mdashis this a significant increase What kind of accidents Skin abrasions or serious injury And compared to Seaville how serious are the accidents and the actual number of accidents Did Seaville enforce other restrictions like safety signsHow different are the conditions of Balmer

GRE AWA John박 박정어학원

and Seaville regarding population road (safety) conditions topography other town-government regulation How much will the economy of Balmer be affected do to this restriction Could it cause an economic recession due to the fact that these rental companiesrsquo chance to make money is only during the summer thereby weakening the economic infrastructure Are there any other ways that could better alleviate the accident rate

Statistics 50-impreciseAnalogy Balmer compared with TorseauFalse Cause Accidents might have occurred because of reasons other than mopeds False Cause population increase may not be part of the cause of the accidentsOther explanations for the accident pedestrians few road safety regulations narrow roadsThere could be other better solutionshellip

Balmer Island의 인구가 여름철에는 십만명으로 늘어난다 2륜차와 보행자간 사고를 줄이기 위해 시의회는 6개의 자전거를 포함한 2륜차 대여업체에게 이 기간동안에는 대여숫자를 일일 50에서 30으로 제한하도록 할 것이다 대여숫자를 줄임으로써 시 의회는 지난해 이웃한 Torseau섬에서 이와 동일한 규제를 시행해서 50나 줄인 결과를 보고 마찬가지로 50를 줄일수 있다고 확신하고 있다

결론 The town council of Balmer Island should linit the number

반박 보행자의 부실에 의해서 사고가 많이 일어날수도 있다렌탈수의 줄임만이 대책은 아니다(대부분의 사람들이 렌탈 보다는 소유하고 있을 수도 있다)옆섬과는 상황이 다를수도 있다(그 섬에서는 사고의 원인이 많은 자전거 수로 인한것일수있다) 하지만 이 섬은 좁은 도로가 원인일 수도 있고 도로 안전 장치의미비가 원일일수 있다

In this letter the author recommends that Balmer Island should limit the number moped rentals from 50 to 30 per day To support this recommendation the author points out several reasons However careful scrutiny of each of the facts reveals that it is filled with unanswered questions that could significantly weaken the authorrsquos recommendation with loops and holes which are answered

The recommendation depends on the assumption that no alternative means of reducing the number of accidents are available However the author fails to offer any evidence to substantiate this crucial assumption It is highly possible that means other than this would better solve the problem Perhaps they could widen the roads or put-up more safety signs Or perhaps the accidents were due to the lack of skills in which case proper safety training would significantly alleviate the problem Without considering and ruling out these and other alternative means of reducing accidetns the author cannot confidently conclude that merely emulating Torseau would suffice Moreover the author is advising a recommendation which could potentially harm the economy of Balmer Island sincehellip Moreover the Balmer Island should alternative means to reduce accidents because limiting moped rentals during the summer could harm the economy of Balmerhellip

First of all the author believes that increase in population and the number of moped rentals are responsible for the accidents It is entirely possible that other factors are responsible for the accidents Perhaps Balmer Islandrsquos lack of safety signs was a major factor Or maybe the roads are narrow and dangerous on the Island therefore the town council could enforce stricter traffic regulations to alleviate the problem Accordingly if either of these scenarios is true the author cannot draw any firm conclusion that increase in the number of population and moped rentals are the cause of the accidents

The author of this editorial recommends that to reduce accidents involving mopeds andpedestrians Balmer Islands city council should restrict moped rentals to 30 per day down from50 at each of the islands six rental outlets To support this recommendation the author citesthe fact that last year when nearby Torseau Islands town council enforced similar measuresTorseaus rate of moped accidents fell by 50 For several reasons this evidence providesscant support for the authors recommendationTo begin with the author assumes that all other conditions in Balmer that might affect therate of moped-pedestrian accidents will remain unchanged after the restrictions are enactedHowever with a restricted supply of rental mopeds people in Balmer might purchase mopedsinstead Also the number of pedestrians might increase in the future with more pedestriansespecially tourists the risk of moped-pedestrian accidents would probably increase For thatmatter the number of rental outlets might increase to make up for the artificial supplyrestriction per outlet--a likely scenario assuming moped rental demand does not declineWithout considering and ruling out these and other possible changes that might contribute to ahigh incidence of moped-pedestrian accidents the author cannot convince me that theproposed restrictions will necessarily have the desired effect

GRE AWA John박 박정어학원

Next the author fails to consider other possible explanations for the 50 decline inTorseaus moped accident rate last year Perhaps last year Torseau experienced unusually fairweather during which moped accidents are less likely Perhaps fewer tourists visited Tot seanlast year than during most years thereby diminishing the demand for rental mopeds to belowthe allowed limits Perhaps last year some of Torseaus moped rental outlets purchased newmopeds that are safer to drive Or perhaps the restrictions were already in effect but were notenforced until last year In any event a decline in Torseaus moped accident rate during onlyone year is scarcely sufficient to draw any reliable conclusions about what might have causedthe decline or about what the accident rate will be in years aheadAdditionally in asserting that the same phenomenon that caused a 50 decline in mopedaccidents in Torseau would cause a similar decline in Balmer the author relies on what mightamount to an unfair analogy between Balmer and Torseau Perhaps Balmers ability to enforcemoped-rental restrictions does not meet Torseaus ability if not then the mere enactment ofsimilar restrictions in Balmer is no guarantee of a similar result Or perhaps the demand formopeds in Torseau is always greater than in Balmer Specifically if fewer than all availablemopeds are currently rented per day from the average Balmer outlet while in Torseau everyavailable moped is rented each day then the proposed restriction is likely to have less impacton the accident rate in Balmer than in TorseauFinally the author provides no evidence that the same restrictions that served to reduce theincidence of all moped accidents by 50 would also serve to reduce the incidence ofaccidents involving mopeds and pedestrians by 50 Lacking such evidence it is entirelypossible that the number of moped accidents not involving pedestrians decreased by a greaterpercentage while the number of moped-pedestrian accidents decreased by a smallerpercentage or even increased Since the author has not accounted for these possibilities theeditorials recommendation cannot be taken seriouslyIn conclusion the recommendation is not well supported To convince me that the proposedrestriction would achieve the desired outcome the author would have to assure me that nochanges serving to increase Balmers moped-pedestrian accident rate will occur in theforeseeable future The author must also provide dear evidence that last years decline inmoped accidents in Torseau was attributable primarily to its moped rental restrictions ratherthan to one or more other factors In order to better evaluate the recommendation I wouldneed more information comparing the supply of and demand for moped rentals on the twoislands I would also need to know the rate of mopedpedestrian accidents in Torseau both priorto and after the restrictions were enforced in TorseauThe following appeared in a magazine article about planning for retirement

Clearview should be a top choice for anyone seeking a place to retire because it has spectacular natural beauty and a consistent climate Another advantage is that housing costs in Clearview have fallen significantly during the past year and taxes remain lower than those in neighboring towns Moreover Clearviews mayor promises many new programs to improve schools streets and public services And best of all retirees in Clearview can also expect excellent health care as they grow older since the number of physicians in the area is far greater than the national average

Write a response in which you discuss what specific evidence is needed to evaluate the argument and explain how the evidence would weaken or strengthen the argument

-Natural beauty and consistent climate may not be the most wanted qualities-Housing costs could have lowered on a national level wealthy retirees may not care about costs-Taxes may be high compared to the nationrsquos average tax rate-What about other qualities of Clearview Crime rate what qualities would retirees want -If schools streets and public services need improvement then this is proof that the current condition of Clearview is low Or due to budgetary reasons the mayor may not follow-up on his promise because of lowered tax rate -Schools and people who are retired no relationship-Physicians What kind of physicians Number is irrelevant Are these physicians capable of addressing the illnesses of old people

This author argues that anyone seeking a place to retire should choose Clearview To supportthis argument the article cites Clearviews consistent climate and natural beauty its fallinghousing costs its low property taxes compared to nearby towns and the mayors promise toimprove schools streets and services The article also claims that retirees can expectexcellent health care because the number of physicians in Clearview greatly exceeds thenational average This argument is flawed in several critical respectsTo begin with although consistent climate and natural beauty might be attractive to manyretirees these features are probably not important to all retirees For many retirees it isprobably more important to live near relatives or even to enjoy changing seasons Thus I

GRE AWA John박 박정어학원

cannot accept the authors sweeping recommendation for all retirees on this basisAlso Clearviews declining housing costs do not necessarily make Clearview the best placeto retire for two reasons First despite the decline Clearviews housing costs might be highcompared to housing costs in other cities Secondly for wealthier retirees housing costs arenot likely to be a factor in choosing a place to retire Thus the mere fact that housing costshave been in decline lends scant support to the recommendationThe articles reliance on Clearviews property-tax rates is also problematic in two respectsFirst retirees obviously have innumerable choices about where to retire besides Clear viewand nearby towns Secondly for retirees who are well-off financially property taxes are notlikely to be an important concern in choosing a place to retire Thus it is unfair to infer fromClearviews property-tax rates that retirees would prefer ClearviewYet another problem with the argument involves the mayors promises In light of Clearviewslow property-tax rates whether the mayor can follow through on those promises is highlyquestionable Absent any explanation of how the city can spend more money in the areas citedwithout raising property taxes I simply cannot accept the editorials recommendation on thebasis of those promises Besides even if the city makes the improvements promised thoseimprovements--particular the ones to schools--would not necessarily be important to retireesFinally although the number of physicians in Clearview is relatively high the per capitanumber might be relatively low Moreover it would be fairer to compare this per capita numberwith the per capita number for other attractive retirement towns--rather than the nationalaverage After all retirees are likely to place a relatively heavy burden on health-careresources Besides the article provides no assurances that the number of physicians inClearview will remain high in the foreseeable futureIn conclusion the recommendation is poorly supported To strengthen it the author mustconvince me--perhaps by way of a reliable survey--that the key features that the vast majorityof retirees look for in choosing a place to live are consistent climate natural beauty and lowhousing costs The author must also provide better evidence that Clear views property taxesare lower than the those of cities in other areas The author must also explain how the city canmake its promised improvements without raising property taxes Finally to better assess theargument I would need to now how the per capita number of physicians in Clearview wouldcompare to the national average in the futureThe following appeared as a letter to the editor from a Central Plaza store owner

Over the past two years the number of shoppers in Central Plaza has been steadily decreasing while the popularity of skateboarding has increased dramatically Many Central Plaza store owners believe that the decrease in their business is due to the number of skateboard users in the plaza There has also been a dramatic increase in the amount of litter and vandalism throughout the plaza Thus we recommend that the city prohibit skateboarding in Central Plaza If skateboarding is prohibited here we predict that business in Central Plaza will return to its previously high levels

Write a response in which you discuss what questions would need to be answered in order to decide whether the recommendation is likely to have the predicted result Be sure to explain how the answers to these questions would help to evaluate the recommendation

Why two years ago What happened two years ago which started this declineIs the dramatic increase in the ldquopopularityrdquo of skateboarding the cause of the steady decline of shoppers Are there any malls nearby Were there any changes nearby which could affect the decline in customersmdasha big mall perhaps Could the decline be due to the shop ownersHow many skateboarders use the plazaWhere do they skateboardDo they shop and are they customersAre the increase in litter and vandalism due to skateboarders Could this be alleviated by installing CCTVs and hiring security

This editorial concludes that the city should ban skateboarding from its downtown CentralPlaza in order to attract visitors to that area to return the area to its former glory and to makeit a place where people can congregate for fun and relaxation To justify this conclusion theeditorial points out that skateboarders are nearly the only people one sees anymore at CentralPlaza and that the Plaza is littered and its property defaced The editorial also points out thatthe majority of downtown merchants support the skate boarding ban This argument is flawedin several critical respectsFirst the editorials author falsely assumes that a ban on skateboarding is both necessaryand sufficient to achieve the three stated objectives Perhaps the city can achieve thoseobjectives by other means as well--for example by creating a new mall that incorporates anattractive new skateboard park Even if banning skateboarders altogether is necessary to meetthe citys goals the author has not shown that this action by itself would suffice Assuming thatthe Plazas reputation is now tarnished restoring that reputation and in turn enticing peopleback to the Plaza might require additional measures--such as removing litter and graffiti

GRE AWA John박 박정어학원

promoting the Plaza to the public or enticing popular restaurant or retail chains to the PlazaSecondly the editorial assumes too hastily that the Plazas decline is attributable to theskateboarders--rather than to some other phenomenon Perhaps the Plazas primary appeal inits glory days had to do with particular shops or eateries which were eventually replaced byless appealing ones Or perhaps the crime rate in surrounding areas has risen dramatically forreasons unrelated to the skateboarders presence at the Plaza Without ruling out these andother alternative explanations for the Plazas decline the editorials author cannot convince methat a skateboard ban would reverse that declineThirdly the editorials author might be confusing cause with effect--by assuming that theskateboarders caused the abandonment of the Plaza rather than vice versa It is entirelypossible that skateboarders did not frequent the Plaza until it was largely abandoned--andbecause it had been abandoned In fact this scenario makes good sense since skateboardingis most enjoyable where there are few pedestrians or motorists to get in the wayFourth it is unreasonable to infer from the mere fact that most merchants favor the ban thatthe ban would be effective in achieving the citys objectives Admittedly perhaps thesemerchants would be more likely to help dean up the Plaza area and promote their businesseswere the city to act in accordance with their preference Yet lacking any supporting evidencethe author cannot convince me of this Thus the survey amounts to scant evidence at best thatthe proposed ban would carry the intended resultFinally the author recommends a course of action that might actually defeat the citysobjective of providing a fun and relaxing place for people to congregate In my experienceskateboarding contributes to an atmosphere of fun and relaxation for adults and children alikemore so than many other types of ambiance Without considering that continuing to allowskateboarding--or even encouraging this activity--might achieve the citys goal more effectivelythan banning the activity the author cannot convincingly conclude that the ban would be in thecitys best interestsIn sum the argument is a specious one To strengthen it the editorials author must providedear evidence that skateboarding and not some other factor is responsible for the conditionsmarking the Plazas decline The author must also convince me that no alternative means ofrestoring the Plaza are available to the city and that the proposed ban by itself would suffice toattract tourists and restore the Plaza to its former glory Finally to better assess the argument itwould be useful to know the circumstances under which the downtown merchants would bewilling to help the city achieve its objectives

6그룹 약한 비유 빈출

The following recommendation appeared in a memo from the mayor of the town of Hopewell

Two years ago the nearby town of Ocean View built a new municipal golf course and resort hotel During the past two years tourism in Ocean View has increased new businesses have opened there and Ocean Views tax revenues have risen by 30 percent Therefore the best way to improve Hopewells economymdashand generate additional tax revenuesmdashis to build a golf course and resort hotel similar to those in Ocean View

Write a response in which you examine the stated andor unstated assumptions of the argument Be sure to explain how the argument depends on these assumptions and what the implications are for the argument if the assumptions prove unwarranted

GRE AWA John박 박정어학원

Assumptions The author assumes that OVrsquos municipal golf course and resort hotel caused tourism new businesses and increased tax revenues There may be other reasons advertising promo He assumes that this will continueAssumes that Ocean View and Hopewell are similar in many waysmdashthe name suggests otherwise OV may have always been a tourist attractions for its beaches We need to know the topography

2년전 Ocean View 시는 시정 소유 골프 및 휴양지 호텔을 신축했다 그리고 지난 2년동안 이 시의 관광객이 증가했으며 새로운 사업들이 생겨났다 그에따라 시의 세수도 30나 증가했다 Hopewell의 경제를 향상시키고 아울러 세수를 늘릴 수 있는 가장 좋은 방법은 Ocean View에 세워진 것과 같은 골프 시설과 휴양지 호텔을 신축하는 것이다

1 다른 요인으로 관광 산업이 발전했을 수도 있다 문화 유적이 발견이 되었거나 도로의 정비등으로 여행자가 늘었을 수도 있다

2 관광 산업의증가가 늘어난 세수의 원인이 아니라 새로 유입된 인구의 증가나 다른 공장에서 발생한 것일 수 있다

3 2년동안 한참 골프가 붐을 이루었을 수 있다 경제상황이 나빠지거나 다른 레포츠가 인근 지역에 생겨난다면 골프하는 사람이 줄어들 수 있다

In this memo HopeweUs mayor recommends that in order to stimulate the towns economyand boost tax revenues HopeweU should build a new golf course and resort hotel just as thetown of Ocean View did two years ago To support this recommendation the mayor points outthat in Ocean View during the last two years tourism has increased new businesses haveopened and tax revenues have increased by 30 I find the mayors argument unconvincingin several important respectsFirst of all it is possible that the mayor has confused cause with effect respecting the recentdevelopments in Ocean View Perhaps Ocean Views construction of a new golf course andhotel was a response to previous increases in tourism and business development increasesthat have simply continued during the most recent two years Since the mayor has failed toaccount for this possibility the claim that Hopewell would boost its economy by alsoconstructing a golf course and hotel is completely unwarrantedSecondly the mayor fails to account for other possible causes of the trends in Ocean Viewduring the last two years The increase in tourism might have been due to improving economicconditions nationwide or to unusually pleasant weather in the region The new businessesthat have opened in Ocean View might have opened there irrespective of the new golf courseand hotel And the 30 increase in tax revenues might have been the result of an increase intax rates or the addition of a new type of municipal taxWithout ruling out these and other alternative explanations for the three recent trends inOcean View the mayor cannot reasonably infer based on those trends that Hopewellseconomy would benefit by following Ocean Views exampleThirdly even if the recent trends in Ocean View are attributable to the construction of the newgolf course and hotel there the mayor assumes too hastily that the golf course and hotel willcontinue to benefit that towns overall economy The mayor has not accounted for thepossibility that increased tourism will begin to drive residents away during tourist season orthat new business development will result in the towns losing its appeal as a place to visit or tolive Unless the mayor can convince me that these scenarios are unlikely I cannot accept themayors recommendation that Hopewell follow Ocean Views exampleFinally the mayors argument rests on the unsubstantiated assumption that Hopewell andOcean View are sufficiently alike in ways that might affect the economic impact of a new golfcourse and hotel Hopewell might lack the sort of natural environment that would attract moretourists and new businesses to the town--regardless of its new golf course and hotel For thatmatter perhaps Hopewell already contains several resort hotels and golf courses that are notutilized to their capacity If so building yet another golf course and hotel might amount to amisallocation of the towns resources--and actually harm the towns overall economyIn sum the mayors recommendation is not well supported To bolster it the mayor mustprovide better evidence that Ocean Views new golf course and hotel and not some otherphenomenon--has been responsible for boosting Ocean Views economy during the last twoyears To better assess the recommendation I would need to know why Ocean View decidedto construct its new golf course and hotel in the first place--specifically what events prior toconstruction might have prompted that decision I would also need to thoroughly compare

GRE AWA John박 박정어학원

HopeweU with Ocean View--especially in terms of their appeal to tourists and businesses--todetermine whether the same course of action that appears to have boosted Ocean Viewseconomy would also boost Hopewells economy

The following is part of a memorandum from the president of Humana University

Last year the number of students who enrolled in online degree programs offered by nearby Omni University increased by 50 percent During the same year Omni showed a significant decrease from prior years in expenditures for dormitory and classroom space most likely because instruction in the online programs takes place via the Internet In contrast over the past three years enrollment at Humana University has failed to grow and the cost of maintaining buildings has increased along with our budget deficit To address these problems Humana University will begin immediately to create and actively promote online degree programs like those at Omni We predict that instituting these online degree programs will help Humana both increase its total enrollment and solve its budget problems

Write a response in which you discuss what questions would need to be answered in order to decide whether the prediction and the argument on which it is based are reasonable Be sure to explain how the answers to these questions would help to evaluate the prediction

Is Omni University successful due to the online degree program 50 Is the decrease in expenditures for dormitory and classroom space due to the decrease in of on-campus students Which classes were successful Does HU have those classes

Even if the long-distance degree programs at Omni University benefited the school the presidentrsquos recommendation that Human College should emulate Omni University is too hasty First OUrsquos name implies that the school would have more majors than Humanahellip the president should examine which degrees were in the long-distance programhellip

지난해에는 Omni 대학에서 개강했던 원거리 학생 학점 취득 프로그램을 등록했던 학생들의 숫자가 50나 증가했다 같은해 기간동안 Omni 대학에서는 그 전년도부터 기숙사와 학급의 공간 확충을 위한 예산을 대폭 줄였는데 이는 이 원거리 학점 취득 프로그램이 양방향 비디오 컴퓨터 접속을 통해서만 가능한 수업지도 방식이기때문인 것으로 보인다 반면 지난 3개년 동안 Humana 대학에서의 수강률은 감소한데다가 건물

GRE AWA John박 박정어학원

유지비도 올랐다 따라서 Humana대학의 수강을 늘리고 예산손실을 회복하기 위해서는 Omni 대학에서 취한 조치와 같은 능동적인 프로그램을 추진해야 한다

결론 we should initiate and actively promote long-distance degree programs like those at Omni 반박 원거리 학생 취득 프로그램 숫자가 증가한거하고 예산이 줄어드는 것 사이에 연관이 약하다 (causal 학생의 증가로 관리비용 증가할수 있음 원거리 수업가능 장비도입에의한 비용발생)bad analogy(omni university 하고 같은 조건이 아니다 )-gt omni college 가 강좌내용이 좋아서 학생의 등록이 많을수 있다 Humana 대학에서 만들었다 하더라도 인기 없을수 있음다른 요인에 의해서 Humana 대학의 수강 인원이 증가할수 있음(비록 과거엔 인기가 없었을지라도)

The following appeared as part of a business plan developed by the manager of the Rialto Movie Theater

Despite its downtown location the Rialto Movie Theater a local institution for five decades must make big changes or close its doors forever It should follow the example of the new Apex Theater in the mall outside of town When the Apex opened last year it featured a video arcade plush carpeting and seats and a state-of-the-art sound system Furthermore in a recent survey over 85 percent of respondents reported that the high price of newly released movies prevents them from going to the movies more than five times per year Thus if the Rialto intends to hold on to its share of a decreasing pool of moviegoers it must offer the same features as Apex

Write a response in which you discuss what questions would need to be answered in order to decide whether the recommendation is likely to have the predicted result Be sure to explain how the answers to these questions would help to evaluate the recommendation

Before following through this business plan the manager should investigate the cause of Rialtorsquos unsuccessful business

The author provides no evidence that the surveyrsquos results are statistically reliable The surveyrsquos sample of 85 percent must be sufficient in size and representative of overall population of the city where Rialto and Apex is serving Lacking evidence of a sufficiently representative sample the author cannot justifiably rely on the survey to draw any conclusion whatsoever The author does not indicate that Apex is indeed currently successful However even if Apex is enjoying success the argument relies on what might be a false analogy between Rialto and Apex In order for Apex to serve as a model that Rialto should emulate the author must assume that all relevant circumstances are essentially the same However this assumption is unwarranted For example the argument overlooks the face that Apex is located in a strategic placemdashbeside a mall where customers can not only watch a movie but also enjoy shopping Therefore simply changing the facility to that of Apex may not lead to success

The author does not mention whether Apex is successful or not Nevertheless even if Apex is currently successful the argument relies on what might be a false analogy between Rialto and Apex In order for Apex to serve as a model that Rialto should emulate the author must assume that all relevant circumstances are essentially the same However this assumption is unwarranted For example the argument overlooks the fact that these two institutions are located in different locations Rialto in downtown and Apex in a mall outside of town Although Apex opened with state-of-the-art facilities the decisive factor in its success could be due to its strategic location of being in a mall People could enjoy both shopping and movies at one location thus they may prefer Apex over Rialto Furthermore the place where people enjoy leisure activities has shifted in the past decades for most cities from downtown to the suburbs Therefore Rialto may not be successful even if it emulates Apexrsquos facilities A better business plan may be relocating Apex to the thriving section of the downtown

Rialto 극장은 지난 50여년간 지역 회관으로써 시내에 위치해 있으면서도 이제 변화를 꾀하지 않으면 문을 닫을

GRE AWA John박 박정어학원

판이다 이 극장은 시외 쇼핑타운에 새로 들어선 Apex 극장의 사례를 본받아야 했다 Apex가 지난해 개업했을 당시 이 극장은 비디오 아케이드 플러쉬 카펫트 바닥과 좌석 그리고 최신 음향시설을 갖추었다 더군다나 최근 조사에서는 응답자의 85 이상이 새로 출시된 영화 입장료가 비싼 탓으로 지난해보다 5배이상의 관람객이 줄어들었다고 나타났다 따라서 Rialto 극장이 줄어들고 있는 관람객을 뺐기지 않고 유지하려면 Apex와 같은 시설들을 갖추어야 할 것이다주장 리알토 극장이 줄어들고 있는 관람객을 뺐기지 않고 유지하려면 Apex와 같은 시설들을 갖추어야 할 것이다

1 조사에서 응답자가 전체를 대표할 수 없다 2 apex 극장이 좋은 시설을 갖추고 있지만 그로 인해 수익이 많이 발생했다는 말이 없으므로 시설투자를

하고도 좋은 결과를 얻을 수 있을지 그 근거가 미흡하다3 좋은 영화가 출시된다면 입장료가 비싸도 영화관에서 꼭 보려고 할 수 있다 4 rialto 가 시설이 아닌 다른 요인에 의해 장사가 안될수도 있다( 우범 지역이라든지)

The following is a recommendation from the business manager of Monarch Books

Since its opening in Collegeville twenty years ago Monarch Books has developed a large customer base due to its reader-friendly atmosphere and wide selection of books on all subjects Last month Book and Bean a combination bookstore and coffee shop announced its intention to open a Collegeville store Monarch Books should open its own in-store cafeacute in the space currently devoted to childrens books Given recent national census data indicating a significant decline in the percentage of the population under age ten sales of childrens books are likely to decline By replacing its childrens books section with a cafeacute Monarch Books can increase profits and ward off competition from Book and Bean

Write a response in which you examine the stated andor unstated assumptions of the argument Be sure to explain how the argument depends on these assumptions and what the implications are for the argument if the assumptions prove unwarranted

The following is a recommendation from the business manager of Monarch Books

Since its opening in Collegeville twenty years ago Monarch Books has developed a large customer base due to its reader-friendly atmosphere and wide selection of books on all subjects Last month Book and Bean a combination bookstore and coffee shop announced its intention to open a Collegeville store Monarch Books should open its own in-store cafeacute in the space currently devoted to childrens books Given recent national census data indicating a significant decline in the percentage of the population under age ten sales of childrens books are likely to decline By replacing its childrens books section with a cafeacute Monarch Books can increase profits and ward off competition from Book and Bean

1 Write a response in which you discuss what questions would need to be answered in order to decide whether the recommendation is likely to have the predicted result Be sure to explain how the answers to these questions would help to evaluate the recommendation

2 Write a response in which you discuss what specific evidence is needed to evaluate the argument and explain how the evidence would weaken or strengthen the argument

No evidence regarding Monarch Bookrsquos successEven if Regal Bookrsquos is successful this may not be attributable to the cafeacute False analogy Emulating may not lead to success Other factors may be involvedInsufficient condition The national census is not enough evidence that childrenrsquos book sales will decline Can

GRE AWA John박 박정어학원

the national census represent the local child populationDid opening a cafeacute boost sales for Regal Books Even assuming Regal is successful by opening a cafeacute this may not be suitable for Monarch which plans to close the childrenrsquos book section to establish a cafe Imprecise language ldquorelatively little spacerdquo how smallThe managerrsquos recommendation contradicts what he says Since Monarch is popular for its wide selection of books closing a selection which targets a major group of readers may hurt Monarchrsquos salesIs this the best way to compete

When Stanley Park first opened it was the largest most heavily used public park in town It is still the largest park but it is no longer heavily used Video cameras mounted in the parks parking lots last month revealed the parks drop in popularity the recordings showed an average of only 50 cars per day In contrast tiny Carlton Park in the heart of the business district is visited by more than 150 people on a typical weekday An obvious difference is that Carlton Park unlike Stanley Park provides ample seating Thus if Stanley Park is ever to be as popular with our citizens as Carlton Park the town will obviously need to provide more benches thereby converting some of the unused open areas into spaces suitable for socializing

Write a response in which you examine the stated andor unstated assumptions of the argument Be sure to explain how the argument depends on these assumptions and what the implications are for the argument if the assumptions prove unwarranted

Stanley 파크가 처음 개장했을 당시 가장 크고 가장 많이 이용되는 공원이었다 아직도 공원중에서는 가장 크지만 이용률은 상당히 떨어졌다 지난달 공원 주차장에 설치해놓은 비디오 카메라를 통해 보면 drop(주차장으로 여겨짐) 이용률이 가장 높았다 수치상으로는 하루 평균 50대의 차량만이 이용하였다 반면 직장 중심거리에 위치한 작은 규모의 Carlton 파크는 주당 무려 150여명 이상이 이용하고 있다 Stanley 파크와는 달리 Carlton 파크에는 의자가 있다는 것이 가장 뚜렷한 차이점이다 따라서 Stanley 파크가 Carlton 파크처럼 시민들이 자주 이용하는 공원이 되기 위해서는 벤치를 설치할 필요가 있으며 이렇게 사용되지 않는 일부 공간을 활용해서 사교를 위한 공간으로 바꾸어야 한다 ===gtdrop 에 대한 첨부사항 (영영사전내용입니다)---- a place or central depository to which something (as mail money or stolen property) is brought for distribution or transmission also the act of depositing something at such a place dropgt

주장 if Stanley Park is ever to be as popular with our citizens as is Carlton Park the town will obviously need to provide more benches thereby converting some of the unused open areas into spaces suitable for socializing1 조사가 언제 이루어진 것인가 조사가 언제 실시되었느냐에 따라 결과가 다를 수있다 현재는 다시 스탠리 파크가 늘어났었을 수 있다 2 벤치를 많이 설치했다고 해서 많은 관광객이 오지 않을수 있다(사람들이 벤치나 사교 공간을 원한다는 어떠한 자료도 없다)3스탠리 파크 주변에 교통 상황이 악화가 되었거나 칼튼 파크에서 문화행사등을 많이 가져서 이용객이 줄어든것일 수도 있다 4 칼튼 파크가 중심지에 있어서 접근성이 좋을수 있다5 조사가 같은 시간을 기준으로 한게 아니다(하나는 주중이고 하나는 주말이다)6사람의 수와 차의 대수를 같은것으로 비교할수 없다 (차안에 몇 명이 타고 있는지 모르고 대중교통을 이용해서 왔을수도 있다)

Page 13: GRE writing argument brain storm

GRE AWA John박 박정어학원

necessarily bolsterbuttress his argumentThe presidentrsquos first unstated but apparent assumption is that a nationwide survey can be

applied to local areas There is no guarantee that the area in which Bower Builders builds houses will follow the national consumer trend Since he only mentions the overall result of the nationwide survey it is difficult to determine more specific conditions such whether people in urban and rural areas have the same housing preferences Such uncertainty is further exacerbated by the nature of the survey question itselfmdash it does not deal with current trends in actual home purchases but with desired home features The naiumlve assumption that consumer desire will directly result in consumer action underlies the presidentrsquos interpretation and application of the survey results to his company plan Not everyone who wishes for a large family room and kitchen will or can actually buy a house with those features mainly due to financial reasons Furthermore the president also assumes for no evident reason that consumers will not hesitate to purchase houses with state-of-the-art kitchens as a standard rather than optional feature when it is clear that such a feature will raise the overall cost significantly He continues to blunder in his mistaken assumptions about consumer behavior in assuming that the taste of recent buyers can represent the concern of future prospective buyers The fact that recent buyers have claimed no need for separate dining rooms does not mean future buyers will feel the same way as well

In addition the president finds a real-life actualization of the nationwide survey results in the recent sales of Domus Construction However he easily assumes that large family rooms and kitchens are the only reasons the houses of Domus Construction sell well He does not take into account other features and selling points of the competitorrsquos houses There may well be other explanations for its houses selling more quickly and expensively including additional home features as well as external factors such as proximity to better schools or superior financial solvency of its clientele If Bower Builders merely added larger family rooms and kitchens without taking into consideration the other factors they may lose rather than gain profits

Clearly the presidentrsquos assertion that Bower Builders make houses with large family rooms and high-tech kitchens at the expense of dining rooms rests on a number of assumptions that are ill-informed and naiumlve If Bower Builders undertakes the proposed plan without further research into local consumer desires purchasing trends and the marketing and sales of competing companies the company will risk losing money by building big new houses that people cannot afford to or will not wish to buy

The following appeared in a letter to the editor of a journal on environmental issues

Over the past year the Crust Copper Company (CCC) has purchased over 10000 square miles of land in the tropical nation of West Fredonia Mining copper on this land will inevitably result in pollution and since West Fredonia is the home of several endangered animal species in environmental disaster But such disasters can be prevented if consumers simply refuse to purchase products that are made with CCCs copper unless the company abandons its mining plans

Write a response in which you examine the stated andor unstated assumptions of the argument Be sure to explain how the argument depends on these assumptions and what the implications are for the argument if the assumptions prove unwarranted

The writer assumes 1 The writer is trying to avoid an inevitability 2 Mining copper will result pollution (they could make preventative measures) 3 The writer may be too late from stopping CCC from developing the area into a copper mine 4 Since mining is an underground enterprise the surface may not be affected that much therefore endangered species may not be affected 4 Consumers wonrsquot buy CCC products if the journal publishes a negative review about CCC (How many readers) CCC could a company that has ties with many IT companies and industries in that their copper is almost ubiquitous in various products

GRE AWA John박 박정어학원

지난 한해동안 CCC(Consolidated Copper 회사)는 서부 플로리다의 열대 지역에 1백만 마일이 넘는 땅을 사들였다 이 곳에서의 채광 활동은 서부 플로리다가 몇몇 멸종 위기에 처한 동물의 서식지이기 때문에 분명 오염과 환경파괴를 가져올 것이다 그러나 이러한 파괴는 CCC 회사가 채광을 포기할 때까지 이 회사가 채굴한 구리로 제조된 제품을 구매하지 않으면 막을 수 있을 것이다

결론 such disaster can be prevented if consumers simply refuse to purchase products that are made with CCCs copper until the company abandons its mining plans

1 구리로 제조된 물건이 생활에 필수적인 것이 많은 만큼 불매가 쉽지 않을 수 있다(전선이나 각종 전자제품에 필수적으로 들어가기 때문에)

2 불매를 유도한다고 해서 소비자들이 구매를 안하는 것은 아니다3 적절한 채굴로 환경파괴를 가져 오지 않을 수 있다 (땅속에 있는 물질을 채굴하는 만큼 생물에 영향을 안

미칠 수도 있다)4 이미 채굴이 다 끝나서 더 이상의 채굴이 없을 수도 있다 5 땅을 구입한다고 해서 채광하는건 아니다 (다른 용도로 샀을수도 있다)

The following is a letter to the editor of an environmental magazine

In 1975 a wildlife census found that there were seven species of amphibians in Xanadu National Park with abundant numbers of each species However in 2002 only four species of amphibians were observed in the park and the numbers of each species were drastically reduced There has been a substantial decline in the numbers of amphibians worldwide and global pollution of water and air is clearly implicated The decline of amphibians in Xanadu National Park however almost certainly has a different cause in 1975 troutmdashwhich are known to eat amphibian eggsmdashwere introduced into the park

Write a response in which you discuss what specific evidence is needed to evaluate the argument and explain how the evidence would weaken or strengthen the argument

Evidence needed the identity of the sender and the census taker of rsquo75 and lsquo02mdasha scientist or an environmentalist credibility issue census methodology vs mere observationmdashthe absence of evidence is not an evidence of absence the season of when the census was taken time-shiftmdashconditions may have changed worldwide decline may include Xanadu other species of predators that prey on amphibians because trout is only one species that prey on amphibians the number of troutmdashhave they increased significantly since rsquo75 First the author needs to be more overt about the credibility of the census and observation There were to accounts that notes the population of amphibiansmdashthe first a census and the second an observation The author needs to bolster his conclusion with the evidence that indicate that the census and second observation were done using scientific methodologies This could either could strengthen or weaken his claim In addition he

GRE AWA John박 박정어학원

needs to provide the specific season of when the census and observation occurred In this letter the writer is informing an editor of an environmental magazine that the number of amphibians was greatly reduced since 1975 and he points out the introduction of trout as the only reason for the decline However the author fails to provide crucial evidences that could strengthen or weaken his conclusion

The following appeared in a memorandum from the president of Hyper-Go Toy Company

Last year sales of our Fierce Fighter toy airplane declined sharply even though the toy had been a top seller for three years Our customer surveys show that parents are now more worried about youthful violence and are concerned about better education for their children Therefore to maintain profits we should discontinue all our action toys and focus exclusively on a new line of educational toys Several other toy companies have already begun marketing educational toys and report sales increases last year of 200 percent And since the average family income is growing sales of new Hyper-Go toys should also increase

Write a response in which you discuss what specific evidence is needed to evaluate the argument and explain how the evidence would weaken or strengthen the argument

First the president indicates that the sales of Fierce Fighter toy airplane declined sharply However he fails to consider the fact that toys are a fad Since FFT enjoyed a three year success it may be natural that the trend would subsideSecond Customer survey is this representative of most toy consumers This trend is not newThird other companies may have profited not through educational but other toys Or educational toy profit may be small in proportion to their sale of other toys 200Parents are not the customers companies should concentrate on what the children want to increase profitLastly parents worry about youthful violence and concern for better education are not new trends

The presidentrsquos decision to discontinue all action toys and focus only on educational toys is too extreme If this decision is taken into effect its procedure may be an onerous task because it would require major shifts in human resources and company image Therefore further data should be considered before following up on this decision

우리 회사에서 지난 3년간 최고 매출을 일으켰던 Fierce Fighter 장난감 비행기가 지난해에는 매출이 급격히 떨어졌다 자체 고객 조사에서는 부모들이 현재 청소년 폭력에 걱정을 하고 있어서 아이들의 양질 교육에 더 관심을 가지고 있는 것으로 나타났다 따라서 회사의 수익을 유지하기 위해서는 모든 자사 전투용 장난감 생산을 중단하고 오로지 교육적인 장난감 생산에 집중해야 한다 몇몇 여타 장난감 회사들도 이미 교육용 장난감 마케팅을 시작해서 지난해에는 200의 매출신장을 가져왔다고 한다 그리고 평균 가계 수입이 점점 늘고 있기 때문에 신형 Hyper-Go 장난감의 매출도 늘어날 것이다

3그룹 Time-shift Error

Woven baskets characterized by a particular distinctive pattern have previously been found only in the immediate vicinity of the prehistoric village of Palea and therefore were believed to have been unique to the Palean people Recently however archaeologists discovered such a Palean basket in Lithos an ancient village across the Brim River from Palea The Brim River is very deep and broad and so the ancient Paleans could only have crossed it by boat but there is no evidence that the Paleans had boats And boats capable of carrying groups of people and cargo were not developed until thousands of years after the Palean people disappeared Moreover Paleans would have had no need to cross the rivermdashthe woods around Palea are full of nuts berries and small game It follows that the so-called Palean baskets were not unique to Palea

GRE AWA John박 박정어학원

Write a response in which you discuss what specific evidence is needed to evaluate the argument and explain how the evidence would weaken or strengthen the argument

RefutationPrehistoric time shift-gtbrim river could have been narrow and shallow or it might have not existed Indigenous patterns may exist in other disconnected remote placesNuts berries small game(hunting) may not have existed or the author should be proved these existed at that time Abundance of resources doesnrsquot support the reason for seclusionBoats not yet found baskets may have been carried across by the river current without the help of a boat Lithos might have crossed the river for commercial purposeOne Palean basket does not substantiate the authorrsquos claimThe absence of evidence is not an evidence of absence

The author assumes without justification that present conditions are the same as at the prehistoric era The author unfairly infers from the presence of Brim River which exist today that it would have existed in the past However the author fails to offer any evidence to substantiate this inference It is very likely that the Brim River might not have existed in prehistoric times or if it did exist may have been shallow and narrow enough for the Paleans to easily cross Any of these scenarios if true would serve to undermine the claim thathelliphelliphelliphelliphellip

First the author claims that the Brim River was very deep and broad so the Paleans could not have crossed it However the author fails to offer any evidence to substantiate that this was true in the prehistoric time of the Paleans For all we know the Brim River might not have existed in the prehistoric era or if it did exist could have been a shallow and narrow river For example scientists believe that thousands of years ago an ice-bridge existed on the Bering Sea connecting Eurasia to what is now North America to explain how the Eskimos and the Asian inhabitants of America came to migrate all over the Americas In this example the absence of evidence is not an evidence of absence Therefore to sufficiently support his claim the author needs to substantiate the fact that the Brim River really did exist and was broad and wide in the Prehistoric Era proving that the Paleans could not have influenced or traded with other groups of people

First the author claims that the Brim River was very deep and broad so the Paleans could not have crossed it However the author fails to offer any evidence to substantiate that this was true in the prehistoric times of the Paleans For all we know the Brim River might not have existed in the prehistoric era or if it did exist could have been a shallow and narrow river For example rivers are created by natural erosion over thousands of years Niagara Falls carved its way from the mouth of Lake Ottawa and created a long river Likewise the current Brim Riverrsquos physical features may not have been wide and deep Therefore to sufficiently support his claim the author needs to substantiate the fact that the Brim River really did exist and was broad and wide in the Prehistoric Era proving that the Paleans could not have influenced or traded with other groups of people

예전에는 실로짠 특이한 무늬 바구니가 Palea의 선사시대 지역의 인근마을에서만 발견되어왔기 때문에 Palea 마을 사람들의 특징이라고 여겨졌었다 그러나 최근들어 고고학자들이 Lithos지역에서 Palean 바구니를 발견하였는데 그 당시 지역은 Brim 강을 가로질러 Palea까지 닿아있었다 이 강은 수심이 아주 깊고 강폭이 넓었으며 때문에 고대의 Palea인들은 배를 이용해서 강을 건널수 있었을 것이다 그러나 이들이 배를 가지고 있었다는 증거는 발견되지 않고 있다 더군다나 이들이 멸명한 이후 수천년이 지난뒤에도 수많은 물자와 사람을 실어 나를수 있는 용적을 가진 배는 개발되지 않았다 이와더불어 Palea인들은 강을 건널필요가 없었는데 그것은 너트나무 장과열매 그리고 작은 사냥감들이 주변숲에 풍부했기때문이다 따라서 Palean 바구니라고 하는 것도 Palea인들만의 전유물이 아니라는 결론을 얻을 수 있다

결론 if follows that the so-called Palean baskets were not unique to Palea이번문제는 굿이 causal Bad analogy 로 구분해서 찾기가 힘드내요 배를 발견했다는 증거가 없는것이다 (앞으로도 발견될수 있음)

GRE AWA John박 박정어학원

계절의 영향으로 겨울에 얼음이 두껍게 언다든지 여름에 가뭄으로 인해서 건널수 있다물자가 풍부한 것이 이동하지 않을 조건이 아니다 다른 것에 의해서 이동가능(의약품등)

Thirteen years ago researchers studied a group of 25 infants who showed signs of mild distress when exposed to unfamiliar stimuli such as an unusual odor or a tape recording of an unknown voice They discovered that these infants were more likely than other infants to have been conceived in early autumn a time when their mothers production of melatonin hormone known to affect some brain functions would naturally increase in response to decreased daylight In a follow-up study conducted earlier this year more than half of these children now teenagers who had shown signs of distress identified themselves as shy Clearly increased levels of melatonin before birth cause shyness during infancy and this shyness continues into later life

Write a response in which you examine the stated andor unstated assumptions of the argument Be sure to explain how the argument depends on these assumptions and what the implications are for the argument if the assumptions prove unwarranted

Any baby exposed to unpleasant stimuli would react in such wayFirst of all the author states 25 infants as his evidence However this research sample is too small to prove his claimSecond the author states that 25 infants were conceived in early autumn which he claims lead to a shy disposition However this is faulty evidenceThird the research study was a long term study done in the span of 13 years However the author only writes about the initial and final stages of the study and leaves out evidences of what could have happened during the 13 years which could be more evidential factors of influenceFourth neither the infantsrsquo genetic predisposition nor their environment were taken into accountFinally the author concludes that his shyness continues into later life (Other factors could alter this disposition epigenetic theory)

13 년전 학자들은 25명의 유아를 대상으로 이상한 냄새나 특이한 소리를 녹음한 테잎등으로 낯선 자극을 주었을때 보이는 미미한 압박감 증상을 조사하였다 이들은 성숙기가 막 지났을 즈음에 보통의 유아들이 비슷한 증상을 보이는 정도 보다는 다소 민감한 반응을 보였는데 이 시기는 아이의 엄마가 뇌의 일부 기능에 영향을 미치는 것으로 알려진 멜라토니아 호르몬을 생산하는 시기로써 이 호르몬은 낯 시간이 짧을때 자연적으로 증가할 수도 있다 금년초에 실시된 추가연구에서 현재 10대로 성장한 당시 조사대상의 절반 이상의 아이들이 부끄럼을 잘타는 것으로 여기고 있었다 따라서 분명한 것은 출산전 멜라토닌 수치의 증가가 유아기에 수줍음 등의 영향을 미치게 되며 이러한 영향이 성장후에도 작용한다는 것이다

주장 Clearly increased levels of melatonin before birth cause shyness during infancy and this shyness continues into later life

1 25명의 아기로 결론 내리기에 샘플이 작다2 과학적 사실들에 대한 명확한 근거가 엇음3 다른 영향을 간과했다(다른 호르몬에 의한 영향 후천적인 성격형성의 영향)

GRE AWA John박 박정어학원

The following is a letter to the editor of the Atticus City newspaper

Former Mayor Durant owes an apology to the city of Atticus Both the damage to the River Bridge which connects Atticus to Hartley and the traffic problems we have long experienced on the bridge were actually caused 20 years ago by Durant After all he is the one who approved the construction of the bridge If he had approved a wider and better-designed bridge on which approximately the same amount of public money would have been spent none of the damage or problems would have occurred Instead the River Bridge has deteriorated far more rapidly over the past 20 years than has the much longer Derby Bridge up the river Even though the winters have been severe in the past several years this is no excuse for the negligence and wastefulness of Durant

Write a response in which you discuss what questions would need to be answered in order to decide whether the recommendation is likely to have the predicted result Be sure to explain how the answers to these questions would help to evaluate the recommendation

전임 시장인 Durant 씨는 Atticus 시에 대해 사과할 의무가 있습니다 Atticus와 Hartley를 잇는 River Bridge 교량에 대한 피해와 이 교량에서 오랫동안 주민들이 겪어오고 있는 교통 혼잡 문제들은 실제로 20년 전부터 시작된 것이었습니다 결정적으로 그가 교량 공사를 허가했던 바로 그 장본인입니다 당시 비슷한 공사비용으로 폭이 더 넓고 튼튼하게 설계된 교량을 허가했다면 이러한 문제나 피해는 발생하지 않았을 겁니다 더군다나 이 다리는 지난 20년 동안 상류에 건설된 훨씬 오래된 Derby 다리보다도 빠르게 부식되어 갔습니다 지난 수년동안 심지어 혹한이 있었다 하더라도 이러한 태만과 국고 손실에 대한 책임을 회피할 길이 없는 것입니다

결론 Former Mayor Durant owes an apology to the city of Atticus

1 디자인이 문제가 아닐수 있다 (디자인은 좋았으나 건설과정에 문제가 있었을 수 있다)2 그 당시의 시예산이 적어서 더 큰 다리를 짓기가 불가능했을 수도 있다3 교통량이 많거나 다른 상황으로 인해서 부식이 빨리 됐을 수 있다4 그 당시에는 최선의 선택이였지만 갑자기 변한 상황에 의해서 이런 문제점들이 발생했을 수 있다

GRE AWA John박 박정어학원

4그룹 거짓인과관계 오류 (False Cause) 빈출

Fifteen years ago Omega University implemented a new procedure that encouraged students to evaluate the teaching effectiveness of all their professors Since that time Omega professors have begun to assign higher grades in their classes and overall student grade averages at Omega have risen by 30 percent Potential employers looking at this dramatic rise in grades believe that grades at Omega are inflated and do not accurately reflect student achievement as a result Omega graduates have not been as successful at getting jobs as have graduates from nearby Alpha University To enable its graduates to secure better jobs Omega University should terminate student evaluation of professors

Write a response in which you discuss what specific evidence is needed to evaluate the argument and explain how the evidence would weaken or strengthen the argument

Omega professor evaluation implemented 15 years ago =gt Omega prof assign higher grades 30Employers believe therersquos grade inflation

Thus unsuccessful employment than AlphaTherefore to secure jobs Omega should end evaluating profs

Specific evidence neededRelationship between higher grades and evaluationRelationship between GPA and unsuccessful employmentAlpharsquos education could just be better than OmegaldquoFifteen years agordquo is a long time other factors could have influenced Why is the inflation a problem just now How much is Alpha better Is the comparison just How much gap is thereOmegarsquos student could just be doing better in their studiesComparison to other universities다른 대안 없나hellip Could Omega alleviate the employment problem by implementing a different procedure or program

15 년전 우리 대학은 학생들로 하여금 교수평가를 하도록 한 새로운 조치를 시행했었습니다 이후 교수들은 자신의 학과 학생들에게 높은 학점을 주었으며 그에따라 학생들의 전체 평점이 30나 올랐습니다 외부의 기업체들은 분명 점수가 지나치게 부풀려졌다고 믿고 있습니다 결국 본 대학 졸업생들이 인근 Alpha 대학의 졸업자들보다 구직률이 떨어지는 이유를 잘 보여주고 있는 것입니다 이를 해결하기 위해 이제부터는 학생들에 의한 교수평가제를 중단해야 합니다

결론 Omega University should now terminate student evaluation of professors

반박 교수 평가와 학점 인플레의 연관성이 적다( 교수 평가를 먼저하고 학점을 나중에 매길수도 있다)채용기준에 성적만 있는게 아니다 학업성취의 결과 일수도 있다 Alpha 가 원래 유능했다 Alpha 의 교육내용이 좋았다

GRE AWA John박 박정어학원

In this memo the dean of Omega University(OU) recommends OU to terminate professor evaluation to secure better jobs for the students To support this recommendation the dean offers several reasons However this argument contains several logical flaws which render it unconvincing

A threshold problem with the argument involves the voluntary nature of the evaluationprocedure The dean provides no evidence about the number or percentage of Omegastudents who participate in the procedure Lacking such evidence it is entirely possible thatthose numbers are insignificant in which case terminating the procedure is unlikely to haveany effect on the grade average of Omega students or their success in getting jobs aftergraduationThe argument also assumes unfairly that the grade-average increase is the result of theevaluation procedure--rather than some other phenomenon The dean ignores a host of otherpossible explanations for the increase--such as a trend at Omega toward higher admissionstandards or higher quality instruction or facilities Without ruling out all other possibleexplanations for the grade-average increase the dean cannot convince me that by terminatingthe evaluation procedure Omega would curb its perceived grade inflation let alone help itsgraduates get jobsEven if the evaluation procedure has resulted in grade inflation at Omega the deans claimthat grade inflation explains why Omega graduates are less successful than Alpha graduatesin getting jobs is unjustified The dean overlooks a myriad of other possible reasons forOmegas comparatively poor job-placement record Perhaps Omegas career services areinadequate or perhaps Omegas curriculum does not prepare students for the job market aseffectively as Alphas In short without accounting for other factors that might contribute toOmega graduates comparative lack of success in getting jobs the dean cannot justify theclaim that if Omega curbs its grade inflation employers will be more likely to hire OmegagraduatesFinally even if the dean can substantiate all of the foregoing assumptions the deansassertion that Omega must terminate its evaluation procedure to enable its graduates to findbetter jobs is still unwarranted in two respects First the dean ignores other possible ways bywhich Omega can increase its job-placement record--for example by improving its publicrelations or career-counseling services Second the dean unfairly equates more jobs withbetter jobs In other words even if more Omega graduates are able to find jobs as a result ofthe deans recommended course of action the kinds of jobs Omega graduates find would notnecessarily be better onesIn sum the deans argument is unpersuasive as it stands To strengthen it the dean mustprovide better evidence that the increase in grade average is attributable to Omegasprofessor-evaluation procedure and that the end result is a perception on the part ofemployers that Omega graduates are less qualified for jobs than Alpha graduates To betterassess the argument I would need to analyze 15-year trends in (l) the percentage of Omegastudents participating in the evaluation procedure (2) Omegas admission standards andquality of education and (3) Omegas emphasis on job training and career preparation I wouldalso need to know what other means are available to Omega for enabling its graduates to findbetter jobs

GRE AWA John박 박정어학원

The following appeared in a memo from a vice president of Quiot Manufacturing

During the past year Quiot Manufacturing had 30 percent more on-the-job accidents than at the nearby Panoply Industries plant where the work shifts are one hour shorter than ours Experts say that significant contributing factors in many on-the-job accidents are fatigue and sleep deprivation among workers Therefore to reduce the number of on-the-job accidents at Quiot and thereby increase productivity we should shorten each of our three work shifts by one hour so that employees will get adequate amounts of sleep

Write a response in which you examine the stated andor unstated assumptions of the argument Be sure to explain how the argument depends on these assumptions and what the implications are for the argument if the assumptions prove unwarranted

The following appeared in a memo from a vice president of Alta Manufacturing

During the past year Alta Manufacturing had thirty percent more on-the-job accidents than nearby Panoply Industries where the work shifts are one hour shorter than ours Experts believe that a significant contributing factor in many accidents is fatigue caused by sleep deprivation among workers Therefore to reduce the number of on-the-job accidents at Alta we recommend shortening each of our three work shifts by one hour If we do this our employees will get adequate amounts of sleep

Write a response in which you discuss what questions would need to be answered in order to decide whether the recommendation and the argument on which it is based are reasonable Be sure to explain how the answers to these questions would help to evaluate the recommendation

The following appeared in a memo from the vice president of Butler Manufacturing

During the past year workers at Butler Manufacturing reported 30 percent more on-the-job accidents than workers at nearby Panoply Industries where the work shifts are one hour shorter than ours A recent government study reports that fatigue and sleep deprivation among workers are significant contributing factors in many on-the-job accidents If we shorten each of our work shifts by one hour we can improve Butler Manufacturings safety record by ensuring that our employees are adequately rested

1 Write a response in which you discuss what specific evidence is needed to evaluate the argument and explain how the evidence would weaken or strengthen the argument

2 Write a response in which you discuss what questions would need to be answered in order to decide whether the recommendation is likely to have the predicted result Be sure to explain how the answers to these questions would help to evaluate the recommendation

4번 반복됨

Alta has 30 more job accidents than Panoply(work shifts one hour shorter)Experts Job accidents caused by fatigue and sleep deprivationTherefore to reduce job accidents and increase productivity shorten three work shifts by one hour for adequate sleep

지난해 우리 회사는 인근 Panoply Industries보다 업무상 재해가 30나 더 많았다 그 회사는 우리보다 근무 교대시간이 1시간 정도 짧았다 전문가들은 대부분의 업무상 재해에 있어서 가장 중요한 요인이 과로와 수면부족으로 보고있다 따라서 우리 회사에서 높은 산업재해를 줄이고 아울러 생산성을 높이기 위해서는 근로자들이 충분한 수면을 취할 수 있도록 1시간씩 3교대 시간을 줄여야 한다

In this memo the (author) vice president of Alta Manufacturing (AM) recommends that to reduce on-the-job accidents and increase productivity AM should shorten its three work shifts by one hour so that employees can

GRE AWA John박 박정어학원

get more sleep To support this recommendation the author provides several evidences However careful scrutiny of each of the facts reveals that it provides little credible support for the authorrsquos recommendation QuestionsThe number of accidents What kind of accidents The seriousness of the accidents is importantHow many employees are in each company What are their productsFalse cause Sleep may not be the reason for the on-the-job accidents What do Alta and Panoply manufacture

First of all the author believes that fatigue caused the on-the job accidents However there could be other reasons The author observes a correlation between sleep deprivation and on-the-job accidents then concludes that the former is the cause of the latter However the author fails to rule out other possible explanations For example it is entirely possible that Alta factories require more strenuous and dangerous labor than Panoply Without ruling out all other such factors it is unfair to conclude that fatigue is responsible for the accidents In addition the work-shifts may not be the cause of the sleep deprivation and fatigue It is possiblehellip Thus the author should provide what exactly Panoply and Alta manufacture and more precise data about their working conditions to be more convincing

Shortening the shift by one hour does not necessarily lead to more sleep And is one hour enoughLess accidents does not mean increased productivity

결론 We should shorten each of out three work shifts by one hour

반박 경쟁사에 비해서 시간당 하는 업무량이 많아서 더욱 피곤할 수도 있다 시간이 문제가 아니라 노후된 시설 설비 자체의 문제 작업 자체가 원래 위험한 것이여서 사고가 많을 수도 있다 다른 회사는 더욱 많은 작업시간에도 불구하고 안정한 작업여건으로 인해서 사고율이 오히려 더 작을 수도 있다비교사의 재해감소가 다른 요인일수 있다(안전 교육 철저)줄인 시간이 피로회복이나 수면으로 연결 안될수 있음(술을 마실 수도 있고 그 시간에 휴식을 취하지 않고 다른일을 함으로써 더욱 피로해질수 있다)

This editorial recommends that Alta Manufacturing reduce its work shifts by one hour each inorder to reduce its on-the-job accident rate and thereby increase Altas productivity To supportthis recommendation the author points out that last year the number of accidents at Alta was30 greater than at Panoply Industries where work shifts were one hour shorter The authoralso cites certain experts who believe that many on-the-job accidents are caused by fatigueand sleep deprivation I find this the argument unconvincing for several reasonsFirst and foremost the author provides absolutely no evidence that overall workerproductivity is attributable in part to the number of on-the-job accidents Although commonsense informs me that such a relationship exists the author must provide some evidence ofthis cause-and-effect relationship before I can accept the authors final conclusion that theproposed course of action would in fact increase Altas productivitySecondly the author assumes that some accidents at Alta are caused by fatigue or sleepdeprivation However the author overlooks other possible causes such as inadequateequipment maintenance or worker training or the inherent hazards of Altas manufacturingprocesses By the same token Panoplys comparatively low accident rate might be attributablenot to the length of its work shifts but rather to other factors such as superior equipmentmaintenance or worker training In other words without ruling out alternative causes ofon-the-job accidents at both companies the author cannot justifmbly conclude that merely byemulating Panoplys work-shift policy Alta would reduce the number of such accidentsThirdly even assuming that Altas workers are fatigued or sleep-deprived and that this is thecause of some of Altas on-the-job accidents in order to accept the authors solution to thisproblem we must assume that Altas workers would use the additional hour of free time tosleep or rest However the author provides no evidence that they would use the time in thismanner It is entirely possible that Altas workers would use that extra hour to engage in someother fatiguing activity Without ruling out this possibility the author cannot convincinglyconclude that reducing Altas work shifts by one hour would reduce Altas accident rateFinally a series of problems with the argument arise from the scant statistical information onwhich it relies In comparing the number of accidents at Alta and Panoply the author fails toconsider that the per-worker accident rate might reveal that Alta is actually safer than Panoplydepending on the total number of workers at each company Second perhaps accident rates

GRE AWA John박 박정어학원

at the two companies last year were aberrations and during other years Altas accident ratewas no greater or even lower than Panoplys rate Or perhaps Panoply is not representativeof industrial companies generally and that other companies with shorter work shifts have evenhigher accident rates In short since the argument relies on very limited statistical information Icannot take the authors recommendation seriouslyIn conclusion the recommendation for emulating Panoplys work-shift policy is not wellsupported To convince me that shorter work shifts would reduce Altas on-the-job accidentrate the author must provide clear evidence that work-shift length is responsible for some ofAltas accidents The author must also supply evidence to support her final conclusion that alower accident rate would in fact increase overall worker productivity

The following appeared in a memo from the vice president of marketing at Dura-Sock Inc

A recent study of our customers suggests that our company is wasting the money it spends on its patented Endure manufacturing process which ensures that our socks are strong enough to last for two years We have always advertised our use of the Endure process but the new study shows that despite our socks durability our average customer actually purchases new Dura-Socks every three months Furthermore our customers surveyed in our largest market northeastern United States cities say that they most value Dura-Socks stylish appearance and availability in many colors These findings suggest that we can increase our profits by discontinuing use of the Endure manufacturing process

1 Write a response in which you examine the stated andor unstated assumptions of the argument Be sure to explain how the argument depends on these assumptions and what the implications are for the argument if the assumptions prove unwarranted

2 Write a response in which you discuss what specific evidence is needed to evaluate the argument and explain how the evidence would weaken or strengthen the argument

3 Write a response in which you discuss what questions would need to be answered in order to decide whether the recommendation and the argument on which it is based are reasonable Be sure to explain how the answers to these questions would help to evaluate the recommendation

Intro The vice president of marketing at Dura-Sock Inc is offering a potentially harmful investment recommendation by claiming that Dura-Sock should discontinue its use of the ldquoEndurerdquo process To support his recommendation he points out a study that Dura-Sock customers actually purchase the socks every three months and a survey that reveals that Dura-Sock customers like the sockrsquos stylish appearance and availability in many colors The study and survey however are insufficient in supporting his proposal and the VP makes several unwarranted assumptionsIntro (simplified) The VP states that though Dura-Socks last for two years customers buy the socks every three months Therefore he assumes that the consumersrsquo motive for buying the produce is not its durabilityHowever the author fails to rule out other possible motivation for consumption

Even if the survey is reliable the author should consider the rest of the market Vague terms ldquowasting moneyrdquomdashprecisely how much are they wasting Studysurvey errorThe company must calculate the outcome of such momentous decisionStudy participantsrsquo comment that they prefer Dura-Sock for its stylishness and availability might take Dura-Sockrsquos enduring quality for granted

우리회사 제품 소비자들에 대한 최근 조사에서 지난 2년여간 양말의 내구성을 강하게 하는 필수공정이었던 자사 특허의 Endure 공정에 들어가는 비용이 낭비라고 말하고 있다 우리 회사는 항상 이 공정 처리에 대한 광고를 내보냈으나 이에 대한 시장 조사에서 실제로 고객들은 이 신제품을 평균 석달마다 구매하는 것으로 나타났다 더군다나 북동부지역에서 실시한 대규모 시장조사에 응답한 고객들은 양말의 모양과 색상등에 더

GRE AWA John박 박정어학원

관심을 나타냈다 이러한 결과는 우리회사가 신기술 공법을 중단하면 그에 따라 수익이 늘어날 것이라는 것을 말해주고 있는 것이다주장 These findings suggest that Dura0Sock can increase its profits by discontinuing its use of the ldquoEndurerdquo manufacturing process

1 survey가 정확한 소비자의 의견을 나타낸 것인가 다른 선택없이 양자택일과 같은 방법의 survey였는지2 북동부지역의 시장조사가 전체 의견을 대표할 수 있나3 사람들이 모양이나 색상에 앞서 내구성을 먼저 평가했을 수도 있다 내구성을 갖추었다는 전제하에 모양과 색상에 관심을 드러낸 것일 수 있다4 소비자가 도매상(retail)인지 소매상(whole)인지가 없다

The following appeared in a business magazine

As a result of numerous complaints of dizziness and nausea on the part of consumers of Promofoods tuna the company requested that eight million cans of its tuna be returned for testing Promofoods concluded that the canned tuna did not after all pose a health risk This conclusion is based on tests performed on samples of the recalled cans by chemists from Promofoods the chemists found that of the eight food chemicals most commonly blamed for causing symptoms of dizziness and nausea five were not found in any of the tested cans The chemists did find small amounts of the three remaining suspected chemicals but pointed out that these occur naturally in all canned foods

Write a response in which you discuss what questions would need to be addressed in order to decide whether the conclusion and the argument on which it is based are reasonable Be sure to explain how the answers to the questions would help to evaluate the conclusion

Representativeness of the tested cansThey should conduct a comparative studyThe testing could be biased because Promofoods employees conducted the testingHow much (quantity) of the five and three suspected chemicals were in the canned foodsFalse cause The substance that caused dizziness and nausea may not be one of the eight common chemicals

많은 소비자들의 현기증과 구역질 불만에 따라 Promofoods사는 지난해 참치 캔 8백만 개를 테스트하기 위해 반품시켰다 그 결과 캔에서는 건강에 위험이 될 수 있는 화합물질이 없었던 것으로 회사측은 결론지었다 이러한 결론은 회사측 화학연구자들이 회수된 캔의 샘플을 테스트해서 이들 증상의 원인이 되는 8가지 화합물 중에서 5가지가 실험된 캔에서 발견되지 않았다는 사실에 근거한 것이다 이들 화학자들은 나머지 3개가지 화합물이 모든 캔 식료품에서 흔히 발견되는 것이라고 언급했다 결론 Promofoods concluded that the cans did not after all contain chemicals that posed a health risk

1 공인된 기간에서 테스트를 한 것이 아니고 자사에서 직접 테스트를 했기에 신뢰성이 안간다 2 이런 증상을 일으키는 8개의 물질 말고 다른 물질들이 캔속에 많이 포함됬을수 있다 3 나머지 3개의 물질들의 함유량이 많아서 다른 종류의 캔들은 문제를 일으키지 않지만 참치캔은 문제를

일으킬 수 있다

This magazine article concludes that the 8 million cans of tuna Promofoods recalled due tocomplaints about nausea and dizziness do not after ail contain any chemicals that pose a

GRE AWA John박 박정어학원

health risk To support this conclusion the author cites the fact that five of eight chemicalscommonly causing these symptoms were not found in the recalled cans while the other threealso occur naturally in other canned foods For several reasons this evidence lends littlecredible support to the authors conclusionTo begin with the author relies partly on the fact that although three of the eight chemicalsmost commonly blamed for nausea and dizziness appeared in Promofoods recalled tunathese chemicals also occur naturally in other canned foods However this fact alone lends nosupport to the authors conclusion for two reasons First the author might be ignoring animportant distinction between naturally occurring chemicals and those not occurring naturallyIt is entirely possible that these three chemicals do not occur naturally in Promofoods tunaand that it is for this reason that the chemicals cause nausea and dizziness Secondly it isentirely possible that even when they occur naturally these chemicals cause the samesymptoms Unless the author rules out both possibilities he cannot reliably conclude that therecalled tuna would not cause these symptomsAnother problem with the argument is that the authors conclusion is too broad Based onevidence about certain chemicals that might cause two particular heath-related symptoms theauthor concludes that the recalled tuna contains no chemicals that pose a health risk Howeverthe author fails to account for the myriad of other possible health risks that the recalled tunamight potentially pose Without ruling out all other such risks the author cannot justifiablyreach his conclusionA third problem with the argument involves that fact that the eight particular chemicals withwhich the test was concerned are only the eight most commonly blamed for nausea anddizziness It is entirely possibly that other chemicals might also cause these symptoms andthat one or more of these other chemicals actually caused the symptoms Without ruling outthis possibility the author cannot jusufiably conclude that the recalled tuna would not causenausea and dizzinessA final problem with the argument involves thetesting procedure itself The author providesno information about the number of recaUed cans tested or the selection method used Unlessthe number of cans is a sufficiently large sample and is statistically repre sentative of all therecalled cans the studys results are not statistically reliableIn conclusion the article is unconvincing as it stands To strengthen the assertion that therecalled tuna would not cause nausea and dizziness the author must provide evidence thatthe three chemicals mentioned that occur naturally in other canned foods also appear naturallyin Promofoods tuna The author must also provide evidence that ingesting other canned foodscontaining these three chemicals does not cause these symptoms To better evaluate theargument we would need to know whether the sample used in the tests was statisticallysignificant and representative of all the recalled tuna We would also need to know what otherchemicals in the recalled tuna might pose any health risk at all

5그룹 불충분 조건오류 빈출

Natures Way a chain of stores selling health food and other health-related products is opening its next franchise in the town of Plainsville The store should prove to be very successful Natures Way franchises tend to be most profitable in areas where residents lead healthy lives and clearly Plainsville is such an area Plainsville merchants report that sales of running shoes and exercise clothing are at all-time highs The local health club has more members than ever and the weight training and aerobics classes are always full Finally Plainsvilles schoolchildren represent a new generation of potential customers these schoolchildren are required to participate in a fitness-for-life program which emphasizes the benefits of regular exercise at an early age

Write a response in which you examine the stated andor unstated assumptions of the argument Be sure to

GRE AWA John박 박정어학원

explain how the argument depends on these assumptions and what the implications are for the argument if the assumptions prove unwarranted

False cause

First of all the author believes that the Increased sales of running shoes and exercise clothing indicates

Plainesville residentsrsquo interest in leading healthy lives However this assumption is not logically convincing for

several reasons could be a fashion trendTime shift ldquoFitness for liferdquo might not have any influence on schoolchildren as they growFalse cause There could be other reasons for member increase in the health clubAll of the above are insufficient condition

The author has to prove that local residents are interested in leading healthy lives However he supports his conclusion with insufficient evidence Nevertheless even if the residents are concerned with health naturersquos way may not be successful First

그 동안의 경험을 토대로 볼 때 건강생활과 밀접히 관련되어 있는 거주 지역에서 본 상점들이 아주 호응을 얻고 있다 따라서 이러한 주민들이 많이 거주하고 있는 Plainsville 에 새로운 상점들을 계속 세워야 한다 이 지역 상인들은 런닝화와 운동복 판매가 가장 높다고 말한다 불과 5 년전에는 거의 전무하다시피하던 지역 헬스 클럽의 경우도 엄청나게 많은 회원을 확보하고 있으며 웨이트 트레이닝과 에어로빅 강좌들도 항상 만원이라고 한다 새로운 고객층을 예측해 보는 것도 가능하다 이 지역의 학생들의 경우 Fitness for Life프로그램을 받게 되는데 이러한 프로그램을 통해서 유년시절부터 정규적인 운동 습관을 들이게 하고 있는 것이 그것이다

결론 We should therefore build our next new store in Plainsville

반박 그동안의 경험에 의한 과거 통계가 꼭 여기에도 적용되는건 아니다 5 년전 헬스 클럽이 잘 안되었던게 다른 원인이였을수 있다(강사수준 미달 강좌미비)tourist 에 의한 원인 일수 있다 어렸을때부터 운동을 했다고 해서 커서도 관심이 있지는 않다 (오히려 반감이 있을수 있다 혹은 건강하기에 건강에 관심이 적을수도 있다)운동복이나 신발의 판매가 육체노동에 의한 것일수도 있다

IntroductionSupport1049896In this memorandum the author asserts that Naturersquos Way should build its next newstore in Plainsville To support this assertion the author states that Plainsvillesmerchantsrsquo sales of exercise clothing are going well the local health club has moremembers than ever and a new generation of customers will help to ensure NaturersquosWayrsquos success At first glance the authorrsquos assumption seems convincing but in-depth scrutiny revealsthat it lacks substantial evidence as it stands

Body 1-SamplingTopic Sentence 1To begin with the author assumes that the merchantsrsquo report indicates that the residentsare concerned about their health However this assumption is based on unsubstantiated

GRE AWA John박 박정어학원

data Example 1 (Rebuttal1) First if we do not know the total volume of items sold and the price of the goods exactly we cannot infer whether the residents are actually buying many goods Example 2 (Rebuttal2)In addition to that the report emphasizes the rising sales of running shoes and exerciseclothing however these may not be hot-selling items for Naturersquos Way or may not be theproducts the company is planning to sell Concluding Sentence Therefore in order to make the argument reliable the author should reconsider themerchantsrsquo report with more detailed data

Body 2-CausalTopic Sentence 2Second the author contends that the health clubs classes are full yet this does not meanthat many people actually use the health club other factors may be the real cause forthose closed classes Example 1 (Rebuttal1) To begin with if the health club is very small the number of people working out wouldnot be a large one In fact regular gym-going may just be a vogue among a smallunrepresentative segment of Plainsvilles population Example 2 (Rebuttal2) Moreover it is possible that most of the people who exercise in the health club do weight training and aerobics only to look good and to meet other singles not for their health In that case there would be little demand for health products Concluding SentenceThus the author should not hasten to presume what really caused people to be interested in a healthier lifestyle and enroll in the health club

Body 3-Time-ShiftTopic Sentence 3Finally the author highlights that Naturersquos Way can expect a new generation of customersin Plainsville that will help the company in the long term This notion is mistaken in that itassumes the conditions of the present will continue unchanged in the future Although theschool children are required to participate in the fitness for life program they may notnecessarily buy Naturersquos Ways products Example 1 (Rebuttal1) In the first instance they may suffer a fall in purchasing power arising from future economic difficulties this would cause reluctance to spend a considerable amount of money on health products which tend to be more expensive Example 2 (Rebuttal2)Another possibility is that there may emerge many competitor companies vying with Naturersquos Way so that in the future the school children may not feel the necessity to purchase one companyrsquos health products over anotherrsquosConcluding Sentence Thus the authorrsquos assumption is highly speculative since it relies heavily on unknowablefuture circumstances

ConclusionThesis In sum the author uses many assumptions that are insufficient in supporting his claimsSupportIn order for the authorrsquos claims to be convincing he needs to advance more persuasiveevidence that people in Plainsville really are concerned with their health and health foodThe following was written as a part of an application for a small-business loan by a group of developers in the city of Monroe

A jazz music club in Monroe would be a tremendously profitable enterprise Currently the nearest jazz club is 65 miles away thus the proposed new jazz club in Monroe the C-Note would have the local market all to itself Plus jazz is extremely popular in Monroe over 100000 people attended Monroes annual jazz festival last summer several well-known jazz musicians live in Monroe and the highest-rated radio program in Monroe is Jazz Nightly which airs every weeknight at 7 PM Finally a nationwide study indicates that the typical jazz fan spends close to $1000 per year on jazz entertainment

1 Write a response in which you discuss what specific evidence is needed to evaluate the argument and explain how the evidence would weaken or strengthen the argument

2 Write a response in which you examine the stated andor unstated assumptions of the argument Be

GRE AWA John박 박정어학원

sure to explain how the argument depends on these assumptions and what the implications are for the argument if the assumptions prove unwarranted

3 Write a response in which you discuss what questions would need to be answered in order to decide whether the prediction and the argument on which it is based are reasonable Be sure to explain how the answers to these questions would help to evaluate the prediction

Group error nationwide survey may not reflect local trends Is the nationwide jazz fan population substantialInsufficient non-residents of Monroe may have attended the jazz festival (Body alternative explanation last year may have been an anomaly The author should consider data from various years) The author should indicate how many out of 100000 were Monroe residentsNationwide study Does this reflect Insufficient Citizens of Monroe may continue to go to the jazz club 65 miles away

Are the people in Monroe really interested in jazzMajority of the people who attended the jazz festival might not be Monroe residentsSurvey error nationwide study may not be applicable to MonroeJazz musicians who live in MonroeMonopolyRadio station

In this business application the author claims that the proposed jazz club C Note will be very profitable in Monroe To support this claim the author argues for his case with several evidences At first glance the authorrsquos argument seems convincing however careful scrutiny reveals that his argument in specious

To begin with the author claims that Monroersquos citizens are interested in jazz He presents three evidences First Secondhellip Thirdhellip Howeverhellip

Monroe 시에 있는 재즈 음악 클럽은 수익성이 좋은 사업이다 현재 가장 가까이에 있는 클럽은 65 마일 정도 떨어져 있다 따라서 이번에 세우려고 하는 C Note 는 독보적인 위치를 점할것이다 더군다나 재즈는 이 시에서 가장 인기있는 음악이다 지난 여름 재즈 축제에서는 10 만명 이상의 Morone 시 주민이 참석하였고 몇몇 유명한 재즈 음악가들도 이곳에 살고 있으며 저녁때 방영되는 라디오 프로그램중에서 최고의 시청률을 보이고 있는 것도 Jazz Nightly 이다 전국조사에서도 전형적인 재즈 팬들은 재즈 분야에 년간 1천 달러 가까이 지출하고 있는 것으로 보고되고 있다 따라서 C Note 클럽이 돈을 벌 수 있는 사업이라는 것은 확실한 것이다

결과 It is clear that the C Note cannot help but make money반박 nearest jazz club 이 양질의 써비스로 여전히 손님을 끌수도 있다Festival 에 얼마나 참여하는지가 jazz 의 인기를 반영하지 않는다 뮤지션이 많이 사는거랑 jazz 의 인기가 상관없다라디오 프로그램이 다른 요인에 의해서 인기일수도 있다 (진행자때문)전국 통계 적용 불가화목 실전반_Ms Noh6In this application the author suggests that a jazz club in Monroe will make a number of profits To support this suggestion the author exemplifies the local condition popularity of jazz in Monroe and nationwide study However careful scrutiny of each of the facts reveals that it provides little credible support for the authorrsquos recommendation Good clear intro

First the author assumes that jazz is popular in Monroe because of several facts the jazz festival last year had high participation some famous jazz musicians live in Monroe and the high-rated radio program is lsquoJazz Nightlyrsquo However this assumption has many drawbacks that must be seriously considered(Good topic sentences) If many attendants in the last-yearrsquos festival came from other cities and not Monroe it is hard to conclude that Monroersquos people like jazz Therefore the author must examine how many Monroe residents actually attended the festival On top of that there is little relationship between habitation of famous jazz musician and the popularity of jazz in Monroe Although several well-known musicians live there if they do not take part in any jazz performance of Monroe this might have no effect to the interest of Monroersquos residents

GRE AWA John박 박정어학원

about jazz Finally in the case of radio program this is also not suitable reason why jazz is popular in Monroe It might be possible that people cannot help choosing lsquoJazz Nightlyrsquo because there are few radio programs at Night The fact that the radio program is the highest rating program is not a germane evidence The approximate number of listeners would be the more crucial evidence Therefore the author needs to seriously deliberate the correlation between jazzrsquos popularity in Monroe and his examples (Good logical flow and clarity)

Second the author uses as evidence the nationwide study that jazz fans spend much money on jazz entertainment to substantiate why starting a jazz club in Monroe will be profitable In other words the author assumes that the characteristics of a nationwide study can be applied to Monroe The national study would lend support to the applicantrsquos claim only if residents in Monroe typify national jazz fans However the author does not provide credible evidence that this is the case Moreover the populations of jazz fans nationwide may be insubstantial Thus the author should not infer hastily that Monroersquos residents will spend much money on enjoying jazz from the nationwide study

Lastly even if jazz is popular in Monroe C Note may not be successful It is entirely possible that residents might still prefer other clubs where they have always went In addition there is another possibility that the nearest jazz club will attract many of Monroersquos people because it serves fine performances and is equipped with favorite facilities Without considering these other possibilities the author cannot make his argument convincing In sum the author presents many reasons that are insufficient in supporting his or her claim In order for the authorrsquos claims to be convincing he needs to advance more persuasive evidence such as the total number of Monroe residents who attended the jazz festival the effects on the popularity of jazz by the musicians living in Monroe and the actual number of residents who would typify themselves to be jazz fans through a local survey Without substantial evidence that C Note will be successful in Monroe the businessmen may be overinvesting in what might lead to a business failureExcellent clarity Score 50

The following appeared in a newsletter offering advice to investors

Over 80 percent of the respondents to a recent survey indicated a desire to reduce their intake of foods containing fats and cholesterol and today low-fat products abound in many food stores Since many of the food products currently marketed by Old Dairy Industries are high in fat and cholesterol the companys sales are likely to diminish greatly and company profits will no doubt decrease We therefore advise Old Dairy stockholders to sell their shares and other investors not to purchase stock in this company

Write a response in which you discuss what questions would need to be answered in order to decide whether the advice and the argument on which it is based are reasonable Be sure to explain how the answers to these questions would help to evaluate the advice

Survey 80

GRE AWA John박 박정어학원

Old Dairy could change their products and manufacture low fat dairy foodsLess competing companies Old Dairy could eventually be the only company that produces hellipImprecise numbers and measurementsCustomers may still buy high fat dairy products

The author of the newsletter is offering potentially dangerous advice by recommending Old Dairy stockholders to withdraw investment and stop purchase What is more the authorrsquos prediction debases the reputation and business of Old Dairy and if false could devoid the investment opportunity of the newsletter readers Therefore investors should examine whether the authorrsquos evidences are substantial

To begin with the author states that 80 percent of the respondents in a survey indicated a desire to reduce their intake of foods He therefore argues that Old Dairyrsquos high fat and cholesterol products would decrease in sales However the author makes a crucial error in this argument First the author provides no evidence that the surveyrsquos results are statistically reliable Were they representative of all the customers Were they chosen for the survey randomly Furthermore the desire to reduce fat and cholesterol intake is a pervasive trend in todayrsquos opulent society however the author erroneously identifies this as a new phenomenon which will affect consumer trends Second having a desire to reduce fat and cholesterol intake does not necessarily indicate that people who have this desire will actually reduce consuming these types of products It is entirely possible that they may continue buying Old Dairy products for its quality and taste Accordingly the author cannot draw any firm conclusion that people will not buy Old Dairy products Therefore if any of these cases are true the author may be offering investors a detrimental investment advice

최근 조사에 대한 응답자중 80 이상이 자신이 먹는 음식에서 지방과 콜레스테롤의 함유량을 줄이고 싶다고 한다 아울러 요즘은 많은 식료품 가계에서 저지방 제품들을 많이 취급하고 있다 현재 Old Dairy Industries가 판매하고 있는 많은 음식제품들은 지방과 콜레스테롤이 높기 때문에 이 회사의 매출이 격감할 것으로 보이며 당연히 매출이익도 줄어들것이다 따라서 이 회사의 주주들은 주식을 매각하고 다른 주식 투자가들도 이 회사의 주식을 매입하지 않는 것이 좋다

결론 Old Dairy stockholders to sell their shares and other investors not to purchase stock in this company

반박 모든 상품이 다 고 지방 고 칼로리는 아니다(비록 많을지라도) 일부의 식품의 경우 기호에 맞어서 히트해서 전체적인 수입이 증가할 수도 있다국내시장만 생각할 수 없다( 외국시장에서 호황을 누릴수 있다 )입맛이라는게 즉각 바뀌는게 아니다

The following appeared in a letter to the editor of the Balmer Island Gazette

On Balmer Island where mopeds serve as a popular form of transportation the population increases to 100000 during the summer months To reduce the number of accidents involving mopeds and pedestrians the town council of Balmer Island should limit the number of mopeds rented by the islands moped rental companies from 50 per day to 25 per day during the summer season By limiting the number of rentals the town council will attain the 50 percent annual reduction in moped accidents that was achieved last year on the neighboring island of Seaville when Seavilles town council enforced similar limits on moped rentals

1 Write a response in which you discuss what questions would need to be answered in order to decide whether the recommendation is likely to have the predicted result Be sure to explain how the answers to these questions would help to evaluate the recommendation

2 Write a response in which you discuss what questions would need to be answered in order to decide whether the prediction and the argument on which it is based are reasonable Be sure to explain how the answers to these questions would help to evaluate the prediction

3 Write a response in which you examine the stated andor unstated assumptions of the argument Be sure to explain how the argument depends on these assumptions and what the implications are for the argument if the assumptions prove unwarranted

Whatrsquos the actual population of Balmer Island 100000mdashis this a significant increase What kind of accidents Skin abrasions or serious injury And compared to Seaville how serious are the accidents and the actual number of accidents Did Seaville enforce other restrictions like safety signsHow different are the conditions of Balmer

GRE AWA John박 박정어학원

and Seaville regarding population road (safety) conditions topography other town-government regulation How much will the economy of Balmer be affected do to this restriction Could it cause an economic recession due to the fact that these rental companiesrsquo chance to make money is only during the summer thereby weakening the economic infrastructure Are there any other ways that could better alleviate the accident rate

Statistics 50-impreciseAnalogy Balmer compared with TorseauFalse Cause Accidents might have occurred because of reasons other than mopeds False Cause population increase may not be part of the cause of the accidentsOther explanations for the accident pedestrians few road safety regulations narrow roadsThere could be other better solutionshellip

Balmer Island의 인구가 여름철에는 십만명으로 늘어난다 2륜차와 보행자간 사고를 줄이기 위해 시의회는 6개의 자전거를 포함한 2륜차 대여업체에게 이 기간동안에는 대여숫자를 일일 50에서 30으로 제한하도록 할 것이다 대여숫자를 줄임으로써 시 의회는 지난해 이웃한 Torseau섬에서 이와 동일한 규제를 시행해서 50나 줄인 결과를 보고 마찬가지로 50를 줄일수 있다고 확신하고 있다

결론 The town council of Balmer Island should linit the number

반박 보행자의 부실에 의해서 사고가 많이 일어날수도 있다렌탈수의 줄임만이 대책은 아니다(대부분의 사람들이 렌탈 보다는 소유하고 있을 수도 있다)옆섬과는 상황이 다를수도 있다(그 섬에서는 사고의 원인이 많은 자전거 수로 인한것일수있다) 하지만 이 섬은 좁은 도로가 원인일 수도 있고 도로 안전 장치의미비가 원일일수 있다

In this letter the author recommends that Balmer Island should limit the number moped rentals from 50 to 30 per day To support this recommendation the author points out several reasons However careful scrutiny of each of the facts reveals that it is filled with unanswered questions that could significantly weaken the authorrsquos recommendation with loops and holes which are answered

The recommendation depends on the assumption that no alternative means of reducing the number of accidents are available However the author fails to offer any evidence to substantiate this crucial assumption It is highly possible that means other than this would better solve the problem Perhaps they could widen the roads or put-up more safety signs Or perhaps the accidents were due to the lack of skills in which case proper safety training would significantly alleviate the problem Without considering and ruling out these and other alternative means of reducing accidetns the author cannot confidently conclude that merely emulating Torseau would suffice Moreover the author is advising a recommendation which could potentially harm the economy of Balmer Island sincehellip Moreover the Balmer Island should alternative means to reduce accidents because limiting moped rentals during the summer could harm the economy of Balmerhellip

First of all the author believes that increase in population and the number of moped rentals are responsible for the accidents It is entirely possible that other factors are responsible for the accidents Perhaps Balmer Islandrsquos lack of safety signs was a major factor Or maybe the roads are narrow and dangerous on the Island therefore the town council could enforce stricter traffic regulations to alleviate the problem Accordingly if either of these scenarios is true the author cannot draw any firm conclusion that increase in the number of population and moped rentals are the cause of the accidents

The author of this editorial recommends that to reduce accidents involving mopeds andpedestrians Balmer Islands city council should restrict moped rentals to 30 per day down from50 at each of the islands six rental outlets To support this recommendation the author citesthe fact that last year when nearby Torseau Islands town council enforced similar measuresTorseaus rate of moped accidents fell by 50 For several reasons this evidence providesscant support for the authors recommendationTo begin with the author assumes that all other conditions in Balmer that might affect therate of moped-pedestrian accidents will remain unchanged after the restrictions are enactedHowever with a restricted supply of rental mopeds people in Balmer might purchase mopedsinstead Also the number of pedestrians might increase in the future with more pedestriansespecially tourists the risk of moped-pedestrian accidents would probably increase For thatmatter the number of rental outlets might increase to make up for the artificial supplyrestriction per outlet--a likely scenario assuming moped rental demand does not declineWithout considering and ruling out these and other possible changes that might contribute to ahigh incidence of moped-pedestrian accidents the author cannot convince me that theproposed restrictions will necessarily have the desired effect

GRE AWA John박 박정어학원

Next the author fails to consider other possible explanations for the 50 decline inTorseaus moped accident rate last year Perhaps last year Torseau experienced unusually fairweather during which moped accidents are less likely Perhaps fewer tourists visited Tot seanlast year than during most years thereby diminishing the demand for rental mopeds to belowthe allowed limits Perhaps last year some of Torseaus moped rental outlets purchased newmopeds that are safer to drive Or perhaps the restrictions were already in effect but were notenforced until last year In any event a decline in Torseaus moped accident rate during onlyone year is scarcely sufficient to draw any reliable conclusions about what might have causedthe decline or about what the accident rate will be in years aheadAdditionally in asserting that the same phenomenon that caused a 50 decline in mopedaccidents in Torseau would cause a similar decline in Balmer the author relies on what mightamount to an unfair analogy between Balmer and Torseau Perhaps Balmers ability to enforcemoped-rental restrictions does not meet Torseaus ability if not then the mere enactment ofsimilar restrictions in Balmer is no guarantee of a similar result Or perhaps the demand formopeds in Torseau is always greater than in Balmer Specifically if fewer than all availablemopeds are currently rented per day from the average Balmer outlet while in Torseau everyavailable moped is rented each day then the proposed restriction is likely to have less impacton the accident rate in Balmer than in TorseauFinally the author provides no evidence that the same restrictions that served to reduce theincidence of all moped accidents by 50 would also serve to reduce the incidence ofaccidents involving mopeds and pedestrians by 50 Lacking such evidence it is entirelypossible that the number of moped accidents not involving pedestrians decreased by a greaterpercentage while the number of moped-pedestrian accidents decreased by a smallerpercentage or even increased Since the author has not accounted for these possibilities theeditorials recommendation cannot be taken seriouslyIn conclusion the recommendation is not well supported To convince me that the proposedrestriction would achieve the desired outcome the author would have to assure me that nochanges serving to increase Balmers moped-pedestrian accident rate will occur in theforeseeable future The author must also provide dear evidence that last years decline inmoped accidents in Torseau was attributable primarily to its moped rental restrictions ratherthan to one or more other factors In order to better evaluate the recommendation I wouldneed more information comparing the supply of and demand for moped rentals on the twoislands I would also need to know the rate of mopedpedestrian accidents in Torseau both priorto and after the restrictions were enforced in TorseauThe following appeared in a magazine article about planning for retirement

Clearview should be a top choice for anyone seeking a place to retire because it has spectacular natural beauty and a consistent climate Another advantage is that housing costs in Clearview have fallen significantly during the past year and taxes remain lower than those in neighboring towns Moreover Clearviews mayor promises many new programs to improve schools streets and public services And best of all retirees in Clearview can also expect excellent health care as they grow older since the number of physicians in the area is far greater than the national average

Write a response in which you discuss what specific evidence is needed to evaluate the argument and explain how the evidence would weaken or strengthen the argument

-Natural beauty and consistent climate may not be the most wanted qualities-Housing costs could have lowered on a national level wealthy retirees may not care about costs-Taxes may be high compared to the nationrsquos average tax rate-What about other qualities of Clearview Crime rate what qualities would retirees want -If schools streets and public services need improvement then this is proof that the current condition of Clearview is low Or due to budgetary reasons the mayor may not follow-up on his promise because of lowered tax rate -Schools and people who are retired no relationship-Physicians What kind of physicians Number is irrelevant Are these physicians capable of addressing the illnesses of old people

This author argues that anyone seeking a place to retire should choose Clearview To supportthis argument the article cites Clearviews consistent climate and natural beauty its fallinghousing costs its low property taxes compared to nearby towns and the mayors promise toimprove schools streets and services The article also claims that retirees can expectexcellent health care because the number of physicians in Clearview greatly exceeds thenational average This argument is flawed in several critical respectsTo begin with although consistent climate and natural beauty might be attractive to manyretirees these features are probably not important to all retirees For many retirees it isprobably more important to live near relatives or even to enjoy changing seasons Thus I

GRE AWA John박 박정어학원

cannot accept the authors sweeping recommendation for all retirees on this basisAlso Clearviews declining housing costs do not necessarily make Clearview the best placeto retire for two reasons First despite the decline Clearviews housing costs might be highcompared to housing costs in other cities Secondly for wealthier retirees housing costs arenot likely to be a factor in choosing a place to retire Thus the mere fact that housing costshave been in decline lends scant support to the recommendationThe articles reliance on Clearviews property-tax rates is also problematic in two respectsFirst retirees obviously have innumerable choices about where to retire besides Clear viewand nearby towns Secondly for retirees who are well-off financially property taxes are notlikely to be an important concern in choosing a place to retire Thus it is unfair to infer fromClearviews property-tax rates that retirees would prefer ClearviewYet another problem with the argument involves the mayors promises In light of Clearviewslow property-tax rates whether the mayor can follow through on those promises is highlyquestionable Absent any explanation of how the city can spend more money in the areas citedwithout raising property taxes I simply cannot accept the editorials recommendation on thebasis of those promises Besides even if the city makes the improvements promised thoseimprovements--particular the ones to schools--would not necessarily be important to retireesFinally although the number of physicians in Clearview is relatively high the per capitanumber might be relatively low Moreover it would be fairer to compare this per capita numberwith the per capita number for other attractive retirement towns--rather than the nationalaverage After all retirees are likely to place a relatively heavy burden on health-careresources Besides the article provides no assurances that the number of physicians inClearview will remain high in the foreseeable futureIn conclusion the recommendation is poorly supported To strengthen it the author mustconvince me--perhaps by way of a reliable survey--that the key features that the vast majorityof retirees look for in choosing a place to live are consistent climate natural beauty and lowhousing costs The author must also provide better evidence that Clear views property taxesare lower than the those of cities in other areas The author must also explain how the city canmake its promised improvements without raising property taxes Finally to better assess theargument I would need to now how the per capita number of physicians in Clearview wouldcompare to the national average in the futureThe following appeared as a letter to the editor from a Central Plaza store owner

Over the past two years the number of shoppers in Central Plaza has been steadily decreasing while the popularity of skateboarding has increased dramatically Many Central Plaza store owners believe that the decrease in their business is due to the number of skateboard users in the plaza There has also been a dramatic increase in the amount of litter and vandalism throughout the plaza Thus we recommend that the city prohibit skateboarding in Central Plaza If skateboarding is prohibited here we predict that business in Central Plaza will return to its previously high levels

Write a response in which you discuss what questions would need to be answered in order to decide whether the recommendation is likely to have the predicted result Be sure to explain how the answers to these questions would help to evaluate the recommendation

Why two years ago What happened two years ago which started this declineIs the dramatic increase in the ldquopopularityrdquo of skateboarding the cause of the steady decline of shoppers Are there any malls nearby Were there any changes nearby which could affect the decline in customersmdasha big mall perhaps Could the decline be due to the shop ownersHow many skateboarders use the plazaWhere do they skateboardDo they shop and are they customersAre the increase in litter and vandalism due to skateboarders Could this be alleviated by installing CCTVs and hiring security

This editorial concludes that the city should ban skateboarding from its downtown CentralPlaza in order to attract visitors to that area to return the area to its former glory and to makeit a place where people can congregate for fun and relaxation To justify this conclusion theeditorial points out that skateboarders are nearly the only people one sees anymore at CentralPlaza and that the Plaza is littered and its property defaced The editorial also points out thatthe majority of downtown merchants support the skate boarding ban This argument is flawedin several critical respectsFirst the editorials author falsely assumes that a ban on skateboarding is both necessaryand sufficient to achieve the three stated objectives Perhaps the city can achieve thoseobjectives by other means as well--for example by creating a new mall that incorporates anattractive new skateboard park Even if banning skateboarders altogether is necessary to meetthe citys goals the author has not shown that this action by itself would suffice Assuming thatthe Plazas reputation is now tarnished restoring that reputation and in turn enticing peopleback to the Plaza might require additional measures--such as removing litter and graffiti

GRE AWA John박 박정어학원

promoting the Plaza to the public or enticing popular restaurant or retail chains to the PlazaSecondly the editorial assumes too hastily that the Plazas decline is attributable to theskateboarders--rather than to some other phenomenon Perhaps the Plazas primary appeal inits glory days had to do with particular shops or eateries which were eventually replaced byless appealing ones Or perhaps the crime rate in surrounding areas has risen dramatically forreasons unrelated to the skateboarders presence at the Plaza Without ruling out these andother alternative explanations for the Plazas decline the editorials author cannot convince methat a skateboard ban would reverse that declineThirdly the editorials author might be confusing cause with effect--by assuming that theskateboarders caused the abandonment of the Plaza rather than vice versa It is entirelypossible that skateboarders did not frequent the Plaza until it was largely abandoned--andbecause it had been abandoned In fact this scenario makes good sense since skateboardingis most enjoyable where there are few pedestrians or motorists to get in the wayFourth it is unreasonable to infer from the mere fact that most merchants favor the ban thatthe ban would be effective in achieving the citys objectives Admittedly perhaps thesemerchants would be more likely to help dean up the Plaza area and promote their businesseswere the city to act in accordance with their preference Yet lacking any supporting evidencethe author cannot convince me of this Thus the survey amounts to scant evidence at best thatthe proposed ban would carry the intended resultFinally the author recommends a course of action that might actually defeat the citysobjective of providing a fun and relaxing place for people to congregate In my experienceskateboarding contributes to an atmosphere of fun and relaxation for adults and children alikemore so than many other types of ambiance Without considering that continuing to allowskateboarding--or even encouraging this activity--might achieve the citys goal more effectivelythan banning the activity the author cannot convincingly conclude that the ban would be in thecitys best interestsIn sum the argument is a specious one To strengthen it the editorials author must providedear evidence that skateboarding and not some other factor is responsible for the conditionsmarking the Plazas decline The author must also convince me that no alternative means ofrestoring the Plaza are available to the city and that the proposed ban by itself would suffice toattract tourists and restore the Plaza to its former glory Finally to better assess the argument itwould be useful to know the circumstances under which the downtown merchants would bewilling to help the city achieve its objectives

6그룹 약한 비유 빈출

The following recommendation appeared in a memo from the mayor of the town of Hopewell

Two years ago the nearby town of Ocean View built a new municipal golf course and resort hotel During the past two years tourism in Ocean View has increased new businesses have opened there and Ocean Views tax revenues have risen by 30 percent Therefore the best way to improve Hopewells economymdashand generate additional tax revenuesmdashis to build a golf course and resort hotel similar to those in Ocean View

Write a response in which you examine the stated andor unstated assumptions of the argument Be sure to explain how the argument depends on these assumptions and what the implications are for the argument if the assumptions prove unwarranted

GRE AWA John박 박정어학원

Assumptions The author assumes that OVrsquos municipal golf course and resort hotel caused tourism new businesses and increased tax revenues There may be other reasons advertising promo He assumes that this will continueAssumes that Ocean View and Hopewell are similar in many waysmdashthe name suggests otherwise OV may have always been a tourist attractions for its beaches We need to know the topography

2년전 Ocean View 시는 시정 소유 골프 및 휴양지 호텔을 신축했다 그리고 지난 2년동안 이 시의 관광객이 증가했으며 새로운 사업들이 생겨났다 그에따라 시의 세수도 30나 증가했다 Hopewell의 경제를 향상시키고 아울러 세수를 늘릴 수 있는 가장 좋은 방법은 Ocean View에 세워진 것과 같은 골프 시설과 휴양지 호텔을 신축하는 것이다

1 다른 요인으로 관광 산업이 발전했을 수도 있다 문화 유적이 발견이 되었거나 도로의 정비등으로 여행자가 늘었을 수도 있다

2 관광 산업의증가가 늘어난 세수의 원인이 아니라 새로 유입된 인구의 증가나 다른 공장에서 발생한 것일 수 있다

3 2년동안 한참 골프가 붐을 이루었을 수 있다 경제상황이 나빠지거나 다른 레포츠가 인근 지역에 생겨난다면 골프하는 사람이 줄어들 수 있다

In this memo HopeweUs mayor recommends that in order to stimulate the towns economyand boost tax revenues HopeweU should build a new golf course and resort hotel just as thetown of Ocean View did two years ago To support this recommendation the mayor points outthat in Ocean View during the last two years tourism has increased new businesses haveopened and tax revenues have increased by 30 I find the mayors argument unconvincingin several important respectsFirst of all it is possible that the mayor has confused cause with effect respecting the recentdevelopments in Ocean View Perhaps Ocean Views construction of a new golf course andhotel was a response to previous increases in tourism and business development increasesthat have simply continued during the most recent two years Since the mayor has failed toaccount for this possibility the claim that Hopewell would boost its economy by alsoconstructing a golf course and hotel is completely unwarrantedSecondly the mayor fails to account for other possible causes of the trends in Ocean Viewduring the last two years The increase in tourism might have been due to improving economicconditions nationwide or to unusually pleasant weather in the region The new businessesthat have opened in Ocean View might have opened there irrespective of the new golf courseand hotel And the 30 increase in tax revenues might have been the result of an increase intax rates or the addition of a new type of municipal taxWithout ruling out these and other alternative explanations for the three recent trends inOcean View the mayor cannot reasonably infer based on those trends that Hopewellseconomy would benefit by following Ocean Views exampleThirdly even if the recent trends in Ocean View are attributable to the construction of the newgolf course and hotel there the mayor assumes too hastily that the golf course and hotel willcontinue to benefit that towns overall economy The mayor has not accounted for thepossibility that increased tourism will begin to drive residents away during tourist season orthat new business development will result in the towns losing its appeal as a place to visit or tolive Unless the mayor can convince me that these scenarios are unlikely I cannot accept themayors recommendation that Hopewell follow Ocean Views exampleFinally the mayors argument rests on the unsubstantiated assumption that Hopewell andOcean View are sufficiently alike in ways that might affect the economic impact of a new golfcourse and hotel Hopewell might lack the sort of natural environment that would attract moretourists and new businesses to the town--regardless of its new golf course and hotel For thatmatter perhaps Hopewell already contains several resort hotels and golf courses that are notutilized to their capacity If so building yet another golf course and hotel might amount to amisallocation of the towns resources--and actually harm the towns overall economyIn sum the mayors recommendation is not well supported To bolster it the mayor mustprovide better evidence that Ocean Views new golf course and hotel and not some otherphenomenon--has been responsible for boosting Ocean Views economy during the last twoyears To better assess the recommendation I would need to know why Ocean View decidedto construct its new golf course and hotel in the first place--specifically what events prior toconstruction might have prompted that decision I would also need to thoroughly compare

GRE AWA John박 박정어학원

HopeweU with Ocean View--especially in terms of their appeal to tourists and businesses--todetermine whether the same course of action that appears to have boosted Ocean Viewseconomy would also boost Hopewells economy

The following is part of a memorandum from the president of Humana University

Last year the number of students who enrolled in online degree programs offered by nearby Omni University increased by 50 percent During the same year Omni showed a significant decrease from prior years in expenditures for dormitory and classroom space most likely because instruction in the online programs takes place via the Internet In contrast over the past three years enrollment at Humana University has failed to grow and the cost of maintaining buildings has increased along with our budget deficit To address these problems Humana University will begin immediately to create and actively promote online degree programs like those at Omni We predict that instituting these online degree programs will help Humana both increase its total enrollment and solve its budget problems

Write a response in which you discuss what questions would need to be answered in order to decide whether the prediction and the argument on which it is based are reasonable Be sure to explain how the answers to these questions would help to evaluate the prediction

Is Omni University successful due to the online degree program 50 Is the decrease in expenditures for dormitory and classroom space due to the decrease in of on-campus students Which classes were successful Does HU have those classes

Even if the long-distance degree programs at Omni University benefited the school the presidentrsquos recommendation that Human College should emulate Omni University is too hasty First OUrsquos name implies that the school would have more majors than Humanahellip the president should examine which degrees were in the long-distance programhellip

지난해에는 Omni 대학에서 개강했던 원거리 학생 학점 취득 프로그램을 등록했던 학생들의 숫자가 50나 증가했다 같은해 기간동안 Omni 대학에서는 그 전년도부터 기숙사와 학급의 공간 확충을 위한 예산을 대폭 줄였는데 이는 이 원거리 학점 취득 프로그램이 양방향 비디오 컴퓨터 접속을 통해서만 가능한 수업지도 방식이기때문인 것으로 보인다 반면 지난 3개년 동안 Humana 대학에서의 수강률은 감소한데다가 건물

GRE AWA John박 박정어학원

유지비도 올랐다 따라서 Humana대학의 수강을 늘리고 예산손실을 회복하기 위해서는 Omni 대학에서 취한 조치와 같은 능동적인 프로그램을 추진해야 한다

결론 we should initiate and actively promote long-distance degree programs like those at Omni 반박 원거리 학생 취득 프로그램 숫자가 증가한거하고 예산이 줄어드는 것 사이에 연관이 약하다 (causal 학생의 증가로 관리비용 증가할수 있음 원거리 수업가능 장비도입에의한 비용발생)bad analogy(omni university 하고 같은 조건이 아니다 )-gt omni college 가 강좌내용이 좋아서 학생의 등록이 많을수 있다 Humana 대학에서 만들었다 하더라도 인기 없을수 있음다른 요인에 의해서 Humana 대학의 수강 인원이 증가할수 있음(비록 과거엔 인기가 없었을지라도)

The following appeared as part of a business plan developed by the manager of the Rialto Movie Theater

Despite its downtown location the Rialto Movie Theater a local institution for five decades must make big changes or close its doors forever It should follow the example of the new Apex Theater in the mall outside of town When the Apex opened last year it featured a video arcade plush carpeting and seats and a state-of-the-art sound system Furthermore in a recent survey over 85 percent of respondents reported that the high price of newly released movies prevents them from going to the movies more than five times per year Thus if the Rialto intends to hold on to its share of a decreasing pool of moviegoers it must offer the same features as Apex

Write a response in which you discuss what questions would need to be answered in order to decide whether the recommendation is likely to have the predicted result Be sure to explain how the answers to these questions would help to evaluate the recommendation

Before following through this business plan the manager should investigate the cause of Rialtorsquos unsuccessful business

The author provides no evidence that the surveyrsquos results are statistically reliable The surveyrsquos sample of 85 percent must be sufficient in size and representative of overall population of the city where Rialto and Apex is serving Lacking evidence of a sufficiently representative sample the author cannot justifiably rely on the survey to draw any conclusion whatsoever The author does not indicate that Apex is indeed currently successful However even if Apex is enjoying success the argument relies on what might be a false analogy between Rialto and Apex In order for Apex to serve as a model that Rialto should emulate the author must assume that all relevant circumstances are essentially the same However this assumption is unwarranted For example the argument overlooks the face that Apex is located in a strategic placemdashbeside a mall where customers can not only watch a movie but also enjoy shopping Therefore simply changing the facility to that of Apex may not lead to success

The author does not mention whether Apex is successful or not Nevertheless even if Apex is currently successful the argument relies on what might be a false analogy between Rialto and Apex In order for Apex to serve as a model that Rialto should emulate the author must assume that all relevant circumstances are essentially the same However this assumption is unwarranted For example the argument overlooks the fact that these two institutions are located in different locations Rialto in downtown and Apex in a mall outside of town Although Apex opened with state-of-the-art facilities the decisive factor in its success could be due to its strategic location of being in a mall People could enjoy both shopping and movies at one location thus they may prefer Apex over Rialto Furthermore the place where people enjoy leisure activities has shifted in the past decades for most cities from downtown to the suburbs Therefore Rialto may not be successful even if it emulates Apexrsquos facilities A better business plan may be relocating Apex to the thriving section of the downtown

Rialto 극장은 지난 50여년간 지역 회관으로써 시내에 위치해 있으면서도 이제 변화를 꾀하지 않으면 문을 닫을

GRE AWA John박 박정어학원

판이다 이 극장은 시외 쇼핑타운에 새로 들어선 Apex 극장의 사례를 본받아야 했다 Apex가 지난해 개업했을 당시 이 극장은 비디오 아케이드 플러쉬 카펫트 바닥과 좌석 그리고 최신 음향시설을 갖추었다 더군다나 최근 조사에서는 응답자의 85 이상이 새로 출시된 영화 입장료가 비싼 탓으로 지난해보다 5배이상의 관람객이 줄어들었다고 나타났다 따라서 Rialto 극장이 줄어들고 있는 관람객을 뺐기지 않고 유지하려면 Apex와 같은 시설들을 갖추어야 할 것이다주장 리알토 극장이 줄어들고 있는 관람객을 뺐기지 않고 유지하려면 Apex와 같은 시설들을 갖추어야 할 것이다

1 조사에서 응답자가 전체를 대표할 수 없다 2 apex 극장이 좋은 시설을 갖추고 있지만 그로 인해 수익이 많이 발생했다는 말이 없으므로 시설투자를

하고도 좋은 결과를 얻을 수 있을지 그 근거가 미흡하다3 좋은 영화가 출시된다면 입장료가 비싸도 영화관에서 꼭 보려고 할 수 있다 4 rialto 가 시설이 아닌 다른 요인에 의해 장사가 안될수도 있다( 우범 지역이라든지)

The following is a recommendation from the business manager of Monarch Books

Since its opening in Collegeville twenty years ago Monarch Books has developed a large customer base due to its reader-friendly atmosphere and wide selection of books on all subjects Last month Book and Bean a combination bookstore and coffee shop announced its intention to open a Collegeville store Monarch Books should open its own in-store cafeacute in the space currently devoted to childrens books Given recent national census data indicating a significant decline in the percentage of the population under age ten sales of childrens books are likely to decline By replacing its childrens books section with a cafeacute Monarch Books can increase profits and ward off competition from Book and Bean

Write a response in which you examine the stated andor unstated assumptions of the argument Be sure to explain how the argument depends on these assumptions and what the implications are for the argument if the assumptions prove unwarranted

The following is a recommendation from the business manager of Monarch Books

Since its opening in Collegeville twenty years ago Monarch Books has developed a large customer base due to its reader-friendly atmosphere and wide selection of books on all subjects Last month Book and Bean a combination bookstore and coffee shop announced its intention to open a Collegeville store Monarch Books should open its own in-store cafeacute in the space currently devoted to childrens books Given recent national census data indicating a significant decline in the percentage of the population under age ten sales of childrens books are likely to decline By replacing its childrens books section with a cafeacute Monarch Books can increase profits and ward off competition from Book and Bean

1 Write a response in which you discuss what questions would need to be answered in order to decide whether the recommendation is likely to have the predicted result Be sure to explain how the answers to these questions would help to evaluate the recommendation

2 Write a response in which you discuss what specific evidence is needed to evaluate the argument and explain how the evidence would weaken or strengthen the argument

No evidence regarding Monarch Bookrsquos successEven if Regal Bookrsquos is successful this may not be attributable to the cafeacute False analogy Emulating may not lead to success Other factors may be involvedInsufficient condition The national census is not enough evidence that childrenrsquos book sales will decline Can

GRE AWA John박 박정어학원

the national census represent the local child populationDid opening a cafeacute boost sales for Regal Books Even assuming Regal is successful by opening a cafeacute this may not be suitable for Monarch which plans to close the childrenrsquos book section to establish a cafe Imprecise language ldquorelatively little spacerdquo how smallThe managerrsquos recommendation contradicts what he says Since Monarch is popular for its wide selection of books closing a selection which targets a major group of readers may hurt Monarchrsquos salesIs this the best way to compete

When Stanley Park first opened it was the largest most heavily used public park in town It is still the largest park but it is no longer heavily used Video cameras mounted in the parks parking lots last month revealed the parks drop in popularity the recordings showed an average of only 50 cars per day In contrast tiny Carlton Park in the heart of the business district is visited by more than 150 people on a typical weekday An obvious difference is that Carlton Park unlike Stanley Park provides ample seating Thus if Stanley Park is ever to be as popular with our citizens as Carlton Park the town will obviously need to provide more benches thereby converting some of the unused open areas into spaces suitable for socializing

Write a response in which you examine the stated andor unstated assumptions of the argument Be sure to explain how the argument depends on these assumptions and what the implications are for the argument if the assumptions prove unwarranted

Stanley 파크가 처음 개장했을 당시 가장 크고 가장 많이 이용되는 공원이었다 아직도 공원중에서는 가장 크지만 이용률은 상당히 떨어졌다 지난달 공원 주차장에 설치해놓은 비디오 카메라를 통해 보면 drop(주차장으로 여겨짐) 이용률이 가장 높았다 수치상으로는 하루 평균 50대의 차량만이 이용하였다 반면 직장 중심거리에 위치한 작은 규모의 Carlton 파크는 주당 무려 150여명 이상이 이용하고 있다 Stanley 파크와는 달리 Carlton 파크에는 의자가 있다는 것이 가장 뚜렷한 차이점이다 따라서 Stanley 파크가 Carlton 파크처럼 시민들이 자주 이용하는 공원이 되기 위해서는 벤치를 설치할 필요가 있으며 이렇게 사용되지 않는 일부 공간을 활용해서 사교를 위한 공간으로 바꾸어야 한다 ===gtdrop 에 대한 첨부사항 (영영사전내용입니다)---- a place or central depository to which something (as mail money or stolen property) is brought for distribution or transmission also the act of depositing something at such a place dropgt

주장 if Stanley Park is ever to be as popular with our citizens as is Carlton Park the town will obviously need to provide more benches thereby converting some of the unused open areas into spaces suitable for socializing1 조사가 언제 이루어진 것인가 조사가 언제 실시되었느냐에 따라 결과가 다를 수있다 현재는 다시 스탠리 파크가 늘어났었을 수 있다 2 벤치를 많이 설치했다고 해서 많은 관광객이 오지 않을수 있다(사람들이 벤치나 사교 공간을 원한다는 어떠한 자료도 없다)3스탠리 파크 주변에 교통 상황이 악화가 되었거나 칼튼 파크에서 문화행사등을 많이 가져서 이용객이 줄어든것일 수도 있다 4 칼튼 파크가 중심지에 있어서 접근성이 좋을수 있다5 조사가 같은 시간을 기준으로 한게 아니다(하나는 주중이고 하나는 주말이다)6사람의 수와 차의 대수를 같은것으로 비교할수 없다 (차안에 몇 명이 타고 있는지 모르고 대중교통을 이용해서 왔을수도 있다)

Page 14: GRE writing argument brain storm

GRE AWA John박 박정어학원

지난 한해동안 CCC(Consolidated Copper 회사)는 서부 플로리다의 열대 지역에 1백만 마일이 넘는 땅을 사들였다 이 곳에서의 채광 활동은 서부 플로리다가 몇몇 멸종 위기에 처한 동물의 서식지이기 때문에 분명 오염과 환경파괴를 가져올 것이다 그러나 이러한 파괴는 CCC 회사가 채광을 포기할 때까지 이 회사가 채굴한 구리로 제조된 제품을 구매하지 않으면 막을 수 있을 것이다

결론 such disaster can be prevented if consumers simply refuse to purchase products that are made with CCCs copper until the company abandons its mining plans

1 구리로 제조된 물건이 생활에 필수적인 것이 많은 만큼 불매가 쉽지 않을 수 있다(전선이나 각종 전자제품에 필수적으로 들어가기 때문에)

2 불매를 유도한다고 해서 소비자들이 구매를 안하는 것은 아니다3 적절한 채굴로 환경파괴를 가져 오지 않을 수 있다 (땅속에 있는 물질을 채굴하는 만큼 생물에 영향을 안

미칠 수도 있다)4 이미 채굴이 다 끝나서 더 이상의 채굴이 없을 수도 있다 5 땅을 구입한다고 해서 채광하는건 아니다 (다른 용도로 샀을수도 있다)

The following is a letter to the editor of an environmental magazine

In 1975 a wildlife census found that there were seven species of amphibians in Xanadu National Park with abundant numbers of each species However in 2002 only four species of amphibians were observed in the park and the numbers of each species were drastically reduced There has been a substantial decline in the numbers of amphibians worldwide and global pollution of water and air is clearly implicated The decline of amphibians in Xanadu National Park however almost certainly has a different cause in 1975 troutmdashwhich are known to eat amphibian eggsmdashwere introduced into the park

Write a response in which you discuss what specific evidence is needed to evaluate the argument and explain how the evidence would weaken or strengthen the argument

Evidence needed the identity of the sender and the census taker of rsquo75 and lsquo02mdasha scientist or an environmentalist credibility issue census methodology vs mere observationmdashthe absence of evidence is not an evidence of absence the season of when the census was taken time-shiftmdashconditions may have changed worldwide decline may include Xanadu other species of predators that prey on amphibians because trout is only one species that prey on amphibians the number of troutmdashhave they increased significantly since rsquo75 First the author needs to be more overt about the credibility of the census and observation There were to accounts that notes the population of amphibiansmdashthe first a census and the second an observation The author needs to bolster his conclusion with the evidence that indicate that the census and second observation were done using scientific methodologies This could either could strengthen or weaken his claim In addition he

GRE AWA John박 박정어학원

needs to provide the specific season of when the census and observation occurred In this letter the writer is informing an editor of an environmental magazine that the number of amphibians was greatly reduced since 1975 and he points out the introduction of trout as the only reason for the decline However the author fails to provide crucial evidences that could strengthen or weaken his conclusion

The following appeared in a memorandum from the president of Hyper-Go Toy Company

Last year sales of our Fierce Fighter toy airplane declined sharply even though the toy had been a top seller for three years Our customer surveys show that parents are now more worried about youthful violence and are concerned about better education for their children Therefore to maintain profits we should discontinue all our action toys and focus exclusively on a new line of educational toys Several other toy companies have already begun marketing educational toys and report sales increases last year of 200 percent And since the average family income is growing sales of new Hyper-Go toys should also increase

Write a response in which you discuss what specific evidence is needed to evaluate the argument and explain how the evidence would weaken or strengthen the argument

First the president indicates that the sales of Fierce Fighter toy airplane declined sharply However he fails to consider the fact that toys are a fad Since FFT enjoyed a three year success it may be natural that the trend would subsideSecond Customer survey is this representative of most toy consumers This trend is not newThird other companies may have profited not through educational but other toys Or educational toy profit may be small in proportion to their sale of other toys 200Parents are not the customers companies should concentrate on what the children want to increase profitLastly parents worry about youthful violence and concern for better education are not new trends

The presidentrsquos decision to discontinue all action toys and focus only on educational toys is too extreme If this decision is taken into effect its procedure may be an onerous task because it would require major shifts in human resources and company image Therefore further data should be considered before following up on this decision

우리 회사에서 지난 3년간 최고 매출을 일으켰던 Fierce Fighter 장난감 비행기가 지난해에는 매출이 급격히 떨어졌다 자체 고객 조사에서는 부모들이 현재 청소년 폭력에 걱정을 하고 있어서 아이들의 양질 교육에 더 관심을 가지고 있는 것으로 나타났다 따라서 회사의 수익을 유지하기 위해서는 모든 자사 전투용 장난감 생산을 중단하고 오로지 교육적인 장난감 생산에 집중해야 한다 몇몇 여타 장난감 회사들도 이미 교육용 장난감 마케팅을 시작해서 지난해에는 200의 매출신장을 가져왔다고 한다 그리고 평균 가계 수입이 점점 늘고 있기 때문에 신형 Hyper-Go 장난감의 매출도 늘어날 것이다

3그룹 Time-shift Error

Woven baskets characterized by a particular distinctive pattern have previously been found only in the immediate vicinity of the prehistoric village of Palea and therefore were believed to have been unique to the Palean people Recently however archaeologists discovered such a Palean basket in Lithos an ancient village across the Brim River from Palea The Brim River is very deep and broad and so the ancient Paleans could only have crossed it by boat but there is no evidence that the Paleans had boats And boats capable of carrying groups of people and cargo were not developed until thousands of years after the Palean people disappeared Moreover Paleans would have had no need to cross the rivermdashthe woods around Palea are full of nuts berries and small game It follows that the so-called Palean baskets were not unique to Palea

GRE AWA John박 박정어학원

Write a response in which you discuss what specific evidence is needed to evaluate the argument and explain how the evidence would weaken or strengthen the argument

RefutationPrehistoric time shift-gtbrim river could have been narrow and shallow or it might have not existed Indigenous patterns may exist in other disconnected remote placesNuts berries small game(hunting) may not have existed or the author should be proved these existed at that time Abundance of resources doesnrsquot support the reason for seclusionBoats not yet found baskets may have been carried across by the river current without the help of a boat Lithos might have crossed the river for commercial purposeOne Palean basket does not substantiate the authorrsquos claimThe absence of evidence is not an evidence of absence

The author assumes without justification that present conditions are the same as at the prehistoric era The author unfairly infers from the presence of Brim River which exist today that it would have existed in the past However the author fails to offer any evidence to substantiate this inference It is very likely that the Brim River might not have existed in prehistoric times or if it did exist may have been shallow and narrow enough for the Paleans to easily cross Any of these scenarios if true would serve to undermine the claim thathelliphelliphelliphelliphellip

First the author claims that the Brim River was very deep and broad so the Paleans could not have crossed it However the author fails to offer any evidence to substantiate that this was true in the prehistoric time of the Paleans For all we know the Brim River might not have existed in the prehistoric era or if it did exist could have been a shallow and narrow river For example scientists believe that thousands of years ago an ice-bridge existed on the Bering Sea connecting Eurasia to what is now North America to explain how the Eskimos and the Asian inhabitants of America came to migrate all over the Americas In this example the absence of evidence is not an evidence of absence Therefore to sufficiently support his claim the author needs to substantiate the fact that the Brim River really did exist and was broad and wide in the Prehistoric Era proving that the Paleans could not have influenced or traded with other groups of people

First the author claims that the Brim River was very deep and broad so the Paleans could not have crossed it However the author fails to offer any evidence to substantiate that this was true in the prehistoric times of the Paleans For all we know the Brim River might not have existed in the prehistoric era or if it did exist could have been a shallow and narrow river For example rivers are created by natural erosion over thousands of years Niagara Falls carved its way from the mouth of Lake Ottawa and created a long river Likewise the current Brim Riverrsquos physical features may not have been wide and deep Therefore to sufficiently support his claim the author needs to substantiate the fact that the Brim River really did exist and was broad and wide in the Prehistoric Era proving that the Paleans could not have influenced or traded with other groups of people

예전에는 실로짠 특이한 무늬 바구니가 Palea의 선사시대 지역의 인근마을에서만 발견되어왔기 때문에 Palea 마을 사람들의 특징이라고 여겨졌었다 그러나 최근들어 고고학자들이 Lithos지역에서 Palean 바구니를 발견하였는데 그 당시 지역은 Brim 강을 가로질러 Palea까지 닿아있었다 이 강은 수심이 아주 깊고 강폭이 넓었으며 때문에 고대의 Palea인들은 배를 이용해서 강을 건널수 있었을 것이다 그러나 이들이 배를 가지고 있었다는 증거는 발견되지 않고 있다 더군다나 이들이 멸명한 이후 수천년이 지난뒤에도 수많은 물자와 사람을 실어 나를수 있는 용적을 가진 배는 개발되지 않았다 이와더불어 Palea인들은 강을 건널필요가 없었는데 그것은 너트나무 장과열매 그리고 작은 사냥감들이 주변숲에 풍부했기때문이다 따라서 Palean 바구니라고 하는 것도 Palea인들만의 전유물이 아니라는 결론을 얻을 수 있다

결론 if follows that the so-called Palean baskets were not unique to Palea이번문제는 굿이 causal Bad analogy 로 구분해서 찾기가 힘드내요 배를 발견했다는 증거가 없는것이다 (앞으로도 발견될수 있음)

GRE AWA John박 박정어학원

계절의 영향으로 겨울에 얼음이 두껍게 언다든지 여름에 가뭄으로 인해서 건널수 있다물자가 풍부한 것이 이동하지 않을 조건이 아니다 다른 것에 의해서 이동가능(의약품등)

Thirteen years ago researchers studied a group of 25 infants who showed signs of mild distress when exposed to unfamiliar stimuli such as an unusual odor or a tape recording of an unknown voice They discovered that these infants were more likely than other infants to have been conceived in early autumn a time when their mothers production of melatonin hormone known to affect some brain functions would naturally increase in response to decreased daylight In a follow-up study conducted earlier this year more than half of these children now teenagers who had shown signs of distress identified themselves as shy Clearly increased levels of melatonin before birth cause shyness during infancy and this shyness continues into later life

Write a response in which you examine the stated andor unstated assumptions of the argument Be sure to explain how the argument depends on these assumptions and what the implications are for the argument if the assumptions prove unwarranted

Any baby exposed to unpleasant stimuli would react in such wayFirst of all the author states 25 infants as his evidence However this research sample is too small to prove his claimSecond the author states that 25 infants were conceived in early autumn which he claims lead to a shy disposition However this is faulty evidenceThird the research study was a long term study done in the span of 13 years However the author only writes about the initial and final stages of the study and leaves out evidences of what could have happened during the 13 years which could be more evidential factors of influenceFourth neither the infantsrsquo genetic predisposition nor their environment were taken into accountFinally the author concludes that his shyness continues into later life (Other factors could alter this disposition epigenetic theory)

13 년전 학자들은 25명의 유아를 대상으로 이상한 냄새나 특이한 소리를 녹음한 테잎등으로 낯선 자극을 주었을때 보이는 미미한 압박감 증상을 조사하였다 이들은 성숙기가 막 지났을 즈음에 보통의 유아들이 비슷한 증상을 보이는 정도 보다는 다소 민감한 반응을 보였는데 이 시기는 아이의 엄마가 뇌의 일부 기능에 영향을 미치는 것으로 알려진 멜라토니아 호르몬을 생산하는 시기로써 이 호르몬은 낯 시간이 짧을때 자연적으로 증가할 수도 있다 금년초에 실시된 추가연구에서 현재 10대로 성장한 당시 조사대상의 절반 이상의 아이들이 부끄럼을 잘타는 것으로 여기고 있었다 따라서 분명한 것은 출산전 멜라토닌 수치의 증가가 유아기에 수줍음 등의 영향을 미치게 되며 이러한 영향이 성장후에도 작용한다는 것이다

주장 Clearly increased levels of melatonin before birth cause shyness during infancy and this shyness continues into later life

1 25명의 아기로 결론 내리기에 샘플이 작다2 과학적 사실들에 대한 명확한 근거가 엇음3 다른 영향을 간과했다(다른 호르몬에 의한 영향 후천적인 성격형성의 영향)

GRE AWA John박 박정어학원

The following is a letter to the editor of the Atticus City newspaper

Former Mayor Durant owes an apology to the city of Atticus Both the damage to the River Bridge which connects Atticus to Hartley and the traffic problems we have long experienced on the bridge were actually caused 20 years ago by Durant After all he is the one who approved the construction of the bridge If he had approved a wider and better-designed bridge on which approximately the same amount of public money would have been spent none of the damage or problems would have occurred Instead the River Bridge has deteriorated far more rapidly over the past 20 years than has the much longer Derby Bridge up the river Even though the winters have been severe in the past several years this is no excuse for the negligence and wastefulness of Durant

Write a response in which you discuss what questions would need to be answered in order to decide whether the recommendation is likely to have the predicted result Be sure to explain how the answers to these questions would help to evaluate the recommendation

전임 시장인 Durant 씨는 Atticus 시에 대해 사과할 의무가 있습니다 Atticus와 Hartley를 잇는 River Bridge 교량에 대한 피해와 이 교량에서 오랫동안 주민들이 겪어오고 있는 교통 혼잡 문제들은 실제로 20년 전부터 시작된 것이었습니다 결정적으로 그가 교량 공사를 허가했던 바로 그 장본인입니다 당시 비슷한 공사비용으로 폭이 더 넓고 튼튼하게 설계된 교량을 허가했다면 이러한 문제나 피해는 발생하지 않았을 겁니다 더군다나 이 다리는 지난 20년 동안 상류에 건설된 훨씬 오래된 Derby 다리보다도 빠르게 부식되어 갔습니다 지난 수년동안 심지어 혹한이 있었다 하더라도 이러한 태만과 국고 손실에 대한 책임을 회피할 길이 없는 것입니다

결론 Former Mayor Durant owes an apology to the city of Atticus

1 디자인이 문제가 아닐수 있다 (디자인은 좋았으나 건설과정에 문제가 있었을 수 있다)2 그 당시의 시예산이 적어서 더 큰 다리를 짓기가 불가능했을 수도 있다3 교통량이 많거나 다른 상황으로 인해서 부식이 빨리 됐을 수 있다4 그 당시에는 최선의 선택이였지만 갑자기 변한 상황에 의해서 이런 문제점들이 발생했을 수 있다

GRE AWA John박 박정어학원

4그룹 거짓인과관계 오류 (False Cause) 빈출

Fifteen years ago Omega University implemented a new procedure that encouraged students to evaluate the teaching effectiveness of all their professors Since that time Omega professors have begun to assign higher grades in their classes and overall student grade averages at Omega have risen by 30 percent Potential employers looking at this dramatic rise in grades believe that grades at Omega are inflated and do not accurately reflect student achievement as a result Omega graduates have not been as successful at getting jobs as have graduates from nearby Alpha University To enable its graduates to secure better jobs Omega University should terminate student evaluation of professors

Write a response in which you discuss what specific evidence is needed to evaluate the argument and explain how the evidence would weaken or strengthen the argument

Omega professor evaluation implemented 15 years ago =gt Omega prof assign higher grades 30Employers believe therersquos grade inflation

Thus unsuccessful employment than AlphaTherefore to secure jobs Omega should end evaluating profs

Specific evidence neededRelationship between higher grades and evaluationRelationship between GPA and unsuccessful employmentAlpharsquos education could just be better than OmegaldquoFifteen years agordquo is a long time other factors could have influenced Why is the inflation a problem just now How much is Alpha better Is the comparison just How much gap is thereOmegarsquos student could just be doing better in their studiesComparison to other universities다른 대안 없나hellip Could Omega alleviate the employment problem by implementing a different procedure or program

15 년전 우리 대학은 학생들로 하여금 교수평가를 하도록 한 새로운 조치를 시행했었습니다 이후 교수들은 자신의 학과 학생들에게 높은 학점을 주었으며 그에따라 학생들의 전체 평점이 30나 올랐습니다 외부의 기업체들은 분명 점수가 지나치게 부풀려졌다고 믿고 있습니다 결국 본 대학 졸업생들이 인근 Alpha 대학의 졸업자들보다 구직률이 떨어지는 이유를 잘 보여주고 있는 것입니다 이를 해결하기 위해 이제부터는 학생들에 의한 교수평가제를 중단해야 합니다

결론 Omega University should now terminate student evaluation of professors

반박 교수 평가와 학점 인플레의 연관성이 적다( 교수 평가를 먼저하고 학점을 나중에 매길수도 있다)채용기준에 성적만 있는게 아니다 학업성취의 결과 일수도 있다 Alpha 가 원래 유능했다 Alpha 의 교육내용이 좋았다

GRE AWA John박 박정어학원

In this memo the dean of Omega University(OU) recommends OU to terminate professor evaluation to secure better jobs for the students To support this recommendation the dean offers several reasons However this argument contains several logical flaws which render it unconvincing

A threshold problem with the argument involves the voluntary nature of the evaluationprocedure The dean provides no evidence about the number or percentage of Omegastudents who participate in the procedure Lacking such evidence it is entirely possible thatthose numbers are insignificant in which case terminating the procedure is unlikely to haveany effect on the grade average of Omega students or their success in getting jobs aftergraduationThe argument also assumes unfairly that the grade-average increase is the result of theevaluation procedure--rather than some other phenomenon The dean ignores a host of otherpossible explanations for the increase--such as a trend at Omega toward higher admissionstandards or higher quality instruction or facilities Without ruling out all other possibleexplanations for the grade-average increase the dean cannot convince me that by terminatingthe evaluation procedure Omega would curb its perceived grade inflation let alone help itsgraduates get jobsEven if the evaluation procedure has resulted in grade inflation at Omega the deans claimthat grade inflation explains why Omega graduates are less successful than Alpha graduatesin getting jobs is unjustified The dean overlooks a myriad of other possible reasons forOmegas comparatively poor job-placement record Perhaps Omegas career services areinadequate or perhaps Omegas curriculum does not prepare students for the job market aseffectively as Alphas In short without accounting for other factors that might contribute toOmega graduates comparative lack of success in getting jobs the dean cannot justify theclaim that if Omega curbs its grade inflation employers will be more likely to hire OmegagraduatesFinally even if the dean can substantiate all of the foregoing assumptions the deansassertion that Omega must terminate its evaluation procedure to enable its graduates to findbetter jobs is still unwarranted in two respects First the dean ignores other possible ways bywhich Omega can increase its job-placement record--for example by improving its publicrelations or career-counseling services Second the dean unfairly equates more jobs withbetter jobs In other words even if more Omega graduates are able to find jobs as a result ofthe deans recommended course of action the kinds of jobs Omega graduates find would notnecessarily be better onesIn sum the deans argument is unpersuasive as it stands To strengthen it the dean mustprovide better evidence that the increase in grade average is attributable to Omegasprofessor-evaluation procedure and that the end result is a perception on the part ofemployers that Omega graduates are less qualified for jobs than Alpha graduates To betterassess the argument I would need to analyze 15-year trends in (l) the percentage of Omegastudents participating in the evaluation procedure (2) Omegas admission standards andquality of education and (3) Omegas emphasis on job training and career preparation I wouldalso need to know what other means are available to Omega for enabling its graduates to findbetter jobs

GRE AWA John박 박정어학원

The following appeared in a memo from a vice president of Quiot Manufacturing

During the past year Quiot Manufacturing had 30 percent more on-the-job accidents than at the nearby Panoply Industries plant where the work shifts are one hour shorter than ours Experts say that significant contributing factors in many on-the-job accidents are fatigue and sleep deprivation among workers Therefore to reduce the number of on-the-job accidents at Quiot and thereby increase productivity we should shorten each of our three work shifts by one hour so that employees will get adequate amounts of sleep

Write a response in which you examine the stated andor unstated assumptions of the argument Be sure to explain how the argument depends on these assumptions and what the implications are for the argument if the assumptions prove unwarranted

The following appeared in a memo from a vice president of Alta Manufacturing

During the past year Alta Manufacturing had thirty percent more on-the-job accidents than nearby Panoply Industries where the work shifts are one hour shorter than ours Experts believe that a significant contributing factor in many accidents is fatigue caused by sleep deprivation among workers Therefore to reduce the number of on-the-job accidents at Alta we recommend shortening each of our three work shifts by one hour If we do this our employees will get adequate amounts of sleep

Write a response in which you discuss what questions would need to be answered in order to decide whether the recommendation and the argument on which it is based are reasonable Be sure to explain how the answers to these questions would help to evaluate the recommendation

The following appeared in a memo from the vice president of Butler Manufacturing

During the past year workers at Butler Manufacturing reported 30 percent more on-the-job accidents than workers at nearby Panoply Industries where the work shifts are one hour shorter than ours A recent government study reports that fatigue and sleep deprivation among workers are significant contributing factors in many on-the-job accidents If we shorten each of our work shifts by one hour we can improve Butler Manufacturings safety record by ensuring that our employees are adequately rested

1 Write a response in which you discuss what specific evidence is needed to evaluate the argument and explain how the evidence would weaken or strengthen the argument

2 Write a response in which you discuss what questions would need to be answered in order to decide whether the recommendation is likely to have the predicted result Be sure to explain how the answers to these questions would help to evaluate the recommendation

4번 반복됨

Alta has 30 more job accidents than Panoply(work shifts one hour shorter)Experts Job accidents caused by fatigue and sleep deprivationTherefore to reduce job accidents and increase productivity shorten three work shifts by one hour for adequate sleep

지난해 우리 회사는 인근 Panoply Industries보다 업무상 재해가 30나 더 많았다 그 회사는 우리보다 근무 교대시간이 1시간 정도 짧았다 전문가들은 대부분의 업무상 재해에 있어서 가장 중요한 요인이 과로와 수면부족으로 보고있다 따라서 우리 회사에서 높은 산업재해를 줄이고 아울러 생산성을 높이기 위해서는 근로자들이 충분한 수면을 취할 수 있도록 1시간씩 3교대 시간을 줄여야 한다

In this memo the (author) vice president of Alta Manufacturing (AM) recommends that to reduce on-the-job accidents and increase productivity AM should shorten its three work shifts by one hour so that employees can

GRE AWA John박 박정어학원

get more sleep To support this recommendation the author provides several evidences However careful scrutiny of each of the facts reveals that it provides little credible support for the authorrsquos recommendation QuestionsThe number of accidents What kind of accidents The seriousness of the accidents is importantHow many employees are in each company What are their productsFalse cause Sleep may not be the reason for the on-the-job accidents What do Alta and Panoply manufacture

First of all the author believes that fatigue caused the on-the job accidents However there could be other reasons The author observes a correlation between sleep deprivation and on-the-job accidents then concludes that the former is the cause of the latter However the author fails to rule out other possible explanations For example it is entirely possible that Alta factories require more strenuous and dangerous labor than Panoply Without ruling out all other such factors it is unfair to conclude that fatigue is responsible for the accidents In addition the work-shifts may not be the cause of the sleep deprivation and fatigue It is possiblehellip Thus the author should provide what exactly Panoply and Alta manufacture and more precise data about their working conditions to be more convincing

Shortening the shift by one hour does not necessarily lead to more sleep And is one hour enoughLess accidents does not mean increased productivity

결론 We should shorten each of out three work shifts by one hour

반박 경쟁사에 비해서 시간당 하는 업무량이 많아서 더욱 피곤할 수도 있다 시간이 문제가 아니라 노후된 시설 설비 자체의 문제 작업 자체가 원래 위험한 것이여서 사고가 많을 수도 있다 다른 회사는 더욱 많은 작업시간에도 불구하고 안정한 작업여건으로 인해서 사고율이 오히려 더 작을 수도 있다비교사의 재해감소가 다른 요인일수 있다(안전 교육 철저)줄인 시간이 피로회복이나 수면으로 연결 안될수 있음(술을 마실 수도 있고 그 시간에 휴식을 취하지 않고 다른일을 함으로써 더욱 피로해질수 있다)

This editorial recommends that Alta Manufacturing reduce its work shifts by one hour each inorder to reduce its on-the-job accident rate and thereby increase Altas productivity To supportthis recommendation the author points out that last year the number of accidents at Alta was30 greater than at Panoply Industries where work shifts were one hour shorter The authoralso cites certain experts who believe that many on-the-job accidents are caused by fatigueand sleep deprivation I find this the argument unconvincing for several reasonsFirst and foremost the author provides absolutely no evidence that overall workerproductivity is attributable in part to the number of on-the-job accidents Although commonsense informs me that such a relationship exists the author must provide some evidence ofthis cause-and-effect relationship before I can accept the authors final conclusion that theproposed course of action would in fact increase Altas productivitySecondly the author assumes that some accidents at Alta are caused by fatigue or sleepdeprivation However the author overlooks other possible causes such as inadequateequipment maintenance or worker training or the inherent hazards of Altas manufacturingprocesses By the same token Panoplys comparatively low accident rate might be attributablenot to the length of its work shifts but rather to other factors such as superior equipmentmaintenance or worker training In other words without ruling out alternative causes ofon-the-job accidents at both companies the author cannot justifmbly conclude that merely byemulating Panoplys work-shift policy Alta would reduce the number of such accidentsThirdly even assuming that Altas workers are fatigued or sleep-deprived and that this is thecause of some of Altas on-the-job accidents in order to accept the authors solution to thisproblem we must assume that Altas workers would use the additional hour of free time tosleep or rest However the author provides no evidence that they would use the time in thismanner It is entirely possible that Altas workers would use that extra hour to engage in someother fatiguing activity Without ruling out this possibility the author cannot convincinglyconclude that reducing Altas work shifts by one hour would reduce Altas accident rateFinally a series of problems with the argument arise from the scant statistical information onwhich it relies In comparing the number of accidents at Alta and Panoply the author fails toconsider that the per-worker accident rate might reveal that Alta is actually safer than Panoplydepending on the total number of workers at each company Second perhaps accident rates

GRE AWA John박 박정어학원

at the two companies last year were aberrations and during other years Altas accident ratewas no greater or even lower than Panoplys rate Or perhaps Panoply is not representativeof industrial companies generally and that other companies with shorter work shifts have evenhigher accident rates In short since the argument relies on very limited statistical information Icannot take the authors recommendation seriouslyIn conclusion the recommendation for emulating Panoplys work-shift policy is not wellsupported To convince me that shorter work shifts would reduce Altas on-the-job accidentrate the author must provide clear evidence that work-shift length is responsible for some ofAltas accidents The author must also supply evidence to support her final conclusion that alower accident rate would in fact increase overall worker productivity

The following appeared in a memo from the vice president of marketing at Dura-Sock Inc

A recent study of our customers suggests that our company is wasting the money it spends on its patented Endure manufacturing process which ensures that our socks are strong enough to last for two years We have always advertised our use of the Endure process but the new study shows that despite our socks durability our average customer actually purchases new Dura-Socks every three months Furthermore our customers surveyed in our largest market northeastern United States cities say that they most value Dura-Socks stylish appearance and availability in many colors These findings suggest that we can increase our profits by discontinuing use of the Endure manufacturing process

1 Write a response in which you examine the stated andor unstated assumptions of the argument Be sure to explain how the argument depends on these assumptions and what the implications are for the argument if the assumptions prove unwarranted

2 Write a response in which you discuss what specific evidence is needed to evaluate the argument and explain how the evidence would weaken or strengthen the argument

3 Write a response in which you discuss what questions would need to be answered in order to decide whether the recommendation and the argument on which it is based are reasonable Be sure to explain how the answers to these questions would help to evaluate the recommendation

Intro The vice president of marketing at Dura-Sock Inc is offering a potentially harmful investment recommendation by claiming that Dura-Sock should discontinue its use of the ldquoEndurerdquo process To support his recommendation he points out a study that Dura-Sock customers actually purchase the socks every three months and a survey that reveals that Dura-Sock customers like the sockrsquos stylish appearance and availability in many colors The study and survey however are insufficient in supporting his proposal and the VP makes several unwarranted assumptionsIntro (simplified) The VP states that though Dura-Socks last for two years customers buy the socks every three months Therefore he assumes that the consumersrsquo motive for buying the produce is not its durabilityHowever the author fails to rule out other possible motivation for consumption

Even if the survey is reliable the author should consider the rest of the market Vague terms ldquowasting moneyrdquomdashprecisely how much are they wasting Studysurvey errorThe company must calculate the outcome of such momentous decisionStudy participantsrsquo comment that they prefer Dura-Sock for its stylishness and availability might take Dura-Sockrsquos enduring quality for granted

우리회사 제품 소비자들에 대한 최근 조사에서 지난 2년여간 양말의 내구성을 강하게 하는 필수공정이었던 자사 특허의 Endure 공정에 들어가는 비용이 낭비라고 말하고 있다 우리 회사는 항상 이 공정 처리에 대한 광고를 내보냈으나 이에 대한 시장 조사에서 실제로 고객들은 이 신제품을 평균 석달마다 구매하는 것으로 나타났다 더군다나 북동부지역에서 실시한 대규모 시장조사에 응답한 고객들은 양말의 모양과 색상등에 더

GRE AWA John박 박정어학원

관심을 나타냈다 이러한 결과는 우리회사가 신기술 공법을 중단하면 그에 따라 수익이 늘어날 것이라는 것을 말해주고 있는 것이다주장 These findings suggest that Dura0Sock can increase its profits by discontinuing its use of the ldquoEndurerdquo manufacturing process

1 survey가 정확한 소비자의 의견을 나타낸 것인가 다른 선택없이 양자택일과 같은 방법의 survey였는지2 북동부지역의 시장조사가 전체 의견을 대표할 수 있나3 사람들이 모양이나 색상에 앞서 내구성을 먼저 평가했을 수도 있다 내구성을 갖추었다는 전제하에 모양과 색상에 관심을 드러낸 것일 수 있다4 소비자가 도매상(retail)인지 소매상(whole)인지가 없다

The following appeared in a business magazine

As a result of numerous complaints of dizziness and nausea on the part of consumers of Promofoods tuna the company requested that eight million cans of its tuna be returned for testing Promofoods concluded that the canned tuna did not after all pose a health risk This conclusion is based on tests performed on samples of the recalled cans by chemists from Promofoods the chemists found that of the eight food chemicals most commonly blamed for causing symptoms of dizziness and nausea five were not found in any of the tested cans The chemists did find small amounts of the three remaining suspected chemicals but pointed out that these occur naturally in all canned foods

Write a response in which you discuss what questions would need to be addressed in order to decide whether the conclusion and the argument on which it is based are reasonable Be sure to explain how the answers to the questions would help to evaluate the conclusion

Representativeness of the tested cansThey should conduct a comparative studyThe testing could be biased because Promofoods employees conducted the testingHow much (quantity) of the five and three suspected chemicals were in the canned foodsFalse cause The substance that caused dizziness and nausea may not be one of the eight common chemicals

많은 소비자들의 현기증과 구역질 불만에 따라 Promofoods사는 지난해 참치 캔 8백만 개를 테스트하기 위해 반품시켰다 그 결과 캔에서는 건강에 위험이 될 수 있는 화합물질이 없었던 것으로 회사측은 결론지었다 이러한 결론은 회사측 화학연구자들이 회수된 캔의 샘플을 테스트해서 이들 증상의 원인이 되는 8가지 화합물 중에서 5가지가 실험된 캔에서 발견되지 않았다는 사실에 근거한 것이다 이들 화학자들은 나머지 3개가지 화합물이 모든 캔 식료품에서 흔히 발견되는 것이라고 언급했다 결론 Promofoods concluded that the cans did not after all contain chemicals that posed a health risk

1 공인된 기간에서 테스트를 한 것이 아니고 자사에서 직접 테스트를 했기에 신뢰성이 안간다 2 이런 증상을 일으키는 8개의 물질 말고 다른 물질들이 캔속에 많이 포함됬을수 있다 3 나머지 3개의 물질들의 함유량이 많아서 다른 종류의 캔들은 문제를 일으키지 않지만 참치캔은 문제를

일으킬 수 있다

This magazine article concludes that the 8 million cans of tuna Promofoods recalled due tocomplaints about nausea and dizziness do not after ail contain any chemicals that pose a

GRE AWA John박 박정어학원

health risk To support this conclusion the author cites the fact that five of eight chemicalscommonly causing these symptoms were not found in the recalled cans while the other threealso occur naturally in other canned foods For several reasons this evidence lends littlecredible support to the authors conclusionTo begin with the author relies partly on the fact that although three of the eight chemicalsmost commonly blamed for nausea and dizziness appeared in Promofoods recalled tunathese chemicals also occur naturally in other canned foods However this fact alone lends nosupport to the authors conclusion for two reasons First the author might be ignoring animportant distinction between naturally occurring chemicals and those not occurring naturallyIt is entirely possible that these three chemicals do not occur naturally in Promofoods tunaand that it is for this reason that the chemicals cause nausea and dizziness Secondly it isentirely possible that even when they occur naturally these chemicals cause the samesymptoms Unless the author rules out both possibilities he cannot reliably conclude that therecalled tuna would not cause these symptomsAnother problem with the argument is that the authors conclusion is too broad Based onevidence about certain chemicals that might cause two particular heath-related symptoms theauthor concludes that the recalled tuna contains no chemicals that pose a health risk Howeverthe author fails to account for the myriad of other possible health risks that the recalled tunamight potentially pose Without ruling out all other such risks the author cannot justifiablyreach his conclusionA third problem with the argument involves that fact that the eight particular chemicals withwhich the test was concerned are only the eight most commonly blamed for nausea anddizziness It is entirely possibly that other chemicals might also cause these symptoms andthat one or more of these other chemicals actually caused the symptoms Without ruling outthis possibility the author cannot jusufiably conclude that the recalled tuna would not causenausea and dizzinessA final problem with the argument involves thetesting procedure itself The author providesno information about the number of recaUed cans tested or the selection method used Unlessthe number of cans is a sufficiently large sample and is statistically repre sentative of all therecalled cans the studys results are not statistically reliableIn conclusion the article is unconvincing as it stands To strengthen the assertion that therecalled tuna would not cause nausea and dizziness the author must provide evidence thatthe three chemicals mentioned that occur naturally in other canned foods also appear naturallyin Promofoods tuna The author must also provide evidence that ingesting other canned foodscontaining these three chemicals does not cause these symptoms To better evaluate theargument we would need to know whether the sample used in the tests was statisticallysignificant and representative of all the recalled tuna We would also need to know what otherchemicals in the recalled tuna might pose any health risk at all

5그룹 불충분 조건오류 빈출

Natures Way a chain of stores selling health food and other health-related products is opening its next franchise in the town of Plainsville The store should prove to be very successful Natures Way franchises tend to be most profitable in areas where residents lead healthy lives and clearly Plainsville is such an area Plainsville merchants report that sales of running shoes and exercise clothing are at all-time highs The local health club has more members than ever and the weight training and aerobics classes are always full Finally Plainsvilles schoolchildren represent a new generation of potential customers these schoolchildren are required to participate in a fitness-for-life program which emphasizes the benefits of regular exercise at an early age

Write a response in which you examine the stated andor unstated assumptions of the argument Be sure to

GRE AWA John박 박정어학원

explain how the argument depends on these assumptions and what the implications are for the argument if the assumptions prove unwarranted

False cause

First of all the author believes that the Increased sales of running shoes and exercise clothing indicates

Plainesville residentsrsquo interest in leading healthy lives However this assumption is not logically convincing for

several reasons could be a fashion trendTime shift ldquoFitness for liferdquo might not have any influence on schoolchildren as they growFalse cause There could be other reasons for member increase in the health clubAll of the above are insufficient condition

The author has to prove that local residents are interested in leading healthy lives However he supports his conclusion with insufficient evidence Nevertheless even if the residents are concerned with health naturersquos way may not be successful First

그 동안의 경험을 토대로 볼 때 건강생활과 밀접히 관련되어 있는 거주 지역에서 본 상점들이 아주 호응을 얻고 있다 따라서 이러한 주민들이 많이 거주하고 있는 Plainsville 에 새로운 상점들을 계속 세워야 한다 이 지역 상인들은 런닝화와 운동복 판매가 가장 높다고 말한다 불과 5 년전에는 거의 전무하다시피하던 지역 헬스 클럽의 경우도 엄청나게 많은 회원을 확보하고 있으며 웨이트 트레이닝과 에어로빅 강좌들도 항상 만원이라고 한다 새로운 고객층을 예측해 보는 것도 가능하다 이 지역의 학생들의 경우 Fitness for Life프로그램을 받게 되는데 이러한 프로그램을 통해서 유년시절부터 정규적인 운동 습관을 들이게 하고 있는 것이 그것이다

결론 We should therefore build our next new store in Plainsville

반박 그동안의 경험에 의한 과거 통계가 꼭 여기에도 적용되는건 아니다 5 년전 헬스 클럽이 잘 안되었던게 다른 원인이였을수 있다(강사수준 미달 강좌미비)tourist 에 의한 원인 일수 있다 어렸을때부터 운동을 했다고 해서 커서도 관심이 있지는 않다 (오히려 반감이 있을수 있다 혹은 건강하기에 건강에 관심이 적을수도 있다)운동복이나 신발의 판매가 육체노동에 의한 것일수도 있다

IntroductionSupport1049896In this memorandum the author asserts that Naturersquos Way should build its next newstore in Plainsville To support this assertion the author states that Plainsvillesmerchantsrsquo sales of exercise clothing are going well the local health club has moremembers than ever and a new generation of customers will help to ensure NaturersquosWayrsquos success At first glance the authorrsquos assumption seems convincing but in-depth scrutiny revealsthat it lacks substantial evidence as it stands

Body 1-SamplingTopic Sentence 1To begin with the author assumes that the merchantsrsquo report indicates that the residentsare concerned about their health However this assumption is based on unsubstantiated

GRE AWA John박 박정어학원

data Example 1 (Rebuttal1) First if we do not know the total volume of items sold and the price of the goods exactly we cannot infer whether the residents are actually buying many goods Example 2 (Rebuttal2)In addition to that the report emphasizes the rising sales of running shoes and exerciseclothing however these may not be hot-selling items for Naturersquos Way or may not be theproducts the company is planning to sell Concluding Sentence Therefore in order to make the argument reliable the author should reconsider themerchantsrsquo report with more detailed data

Body 2-CausalTopic Sentence 2Second the author contends that the health clubs classes are full yet this does not meanthat many people actually use the health club other factors may be the real cause forthose closed classes Example 1 (Rebuttal1) To begin with if the health club is very small the number of people working out wouldnot be a large one In fact regular gym-going may just be a vogue among a smallunrepresentative segment of Plainsvilles population Example 2 (Rebuttal2) Moreover it is possible that most of the people who exercise in the health club do weight training and aerobics only to look good and to meet other singles not for their health In that case there would be little demand for health products Concluding SentenceThus the author should not hasten to presume what really caused people to be interested in a healthier lifestyle and enroll in the health club

Body 3-Time-ShiftTopic Sentence 3Finally the author highlights that Naturersquos Way can expect a new generation of customersin Plainsville that will help the company in the long term This notion is mistaken in that itassumes the conditions of the present will continue unchanged in the future Although theschool children are required to participate in the fitness for life program they may notnecessarily buy Naturersquos Ways products Example 1 (Rebuttal1) In the first instance they may suffer a fall in purchasing power arising from future economic difficulties this would cause reluctance to spend a considerable amount of money on health products which tend to be more expensive Example 2 (Rebuttal2)Another possibility is that there may emerge many competitor companies vying with Naturersquos Way so that in the future the school children may not feel the necessity to purchase one companyrsquos health products over anotherrsquosConcluding Sentence Thus the authorrsquos assumption is highly speculative since it relies heavily on unknowablefuture circumstances

ConclusionThesis In sum the author uses many assumptions that are insufficient in supporting his claimsSupportIn order for the authorrsquos claims to be convincing he needs to advance more persuasiveevidence that people in Plainsville really are concerned with their health and health foodThe following was written as a part of an application for a small-business loan by a group of developers in the city of Monroe

A jazz music club in Monroe would be a tremendously profitable enterprise Currently the nearest jazz club is 65 miles away thus the proposed new jazz club in Monroe the C-Note would have the local market all to itself Plus jazz is extremely popular in Monroe over 100000 people attended Monroes annual jazz festival last summer several well-known jazz musicians live in Monroe and the highest-rated radio program in Monroe is Jazz Nightly which airs every weeknight at 7 PM Finally a nationwide study indicates that the typical jazz fan spends close to $1000 per year on jazz entertainment

1 Write a response in which you discuss what specific evidence is needed to evaluate the argument and explain how the evidence would weaken or strengthen the argument

2 Write a response in which you examine the stated andor unstated assumptions of the argument Be

GRE AWA John박 박정어학원

sure to explain how the argument depends on these assumptions and what the implications are for the argument if the assumptions prove unwarranted

3 Write a response in which you discuss what questions would need to be answered in order to decide whether the prediction and the argument on which it is based are reasonable Be sure to explain how the answers to these questions would help to evaluate the prediction

Group error nationwide survey may not reflect local trends Is the nationwide jazz fan population substantialInsufficient non-residents of Monroe may have attended the jazz festival (Body alternative explanation last year may have been an anomaly The author should consider data from various years) The author should indicate how many out of 100000 were Monroe residentsNationwide study Does this reflect Insufficient Citizens of Monroe may continue to go to the jazz club 65 miles away

Are the people in Monroe really interested in jazzMajority of the people who attended the jazz festival might not be Monroe residentsSurvey error nationwide study may not be applicable to MonroeJazz musicians who live in MonroeMonopolyRadio station

In this business application the author claims that the proposed jazz club C Note will be very profitable in Monroe To support this claim the author argues for his case with several evidences At first glance the authorrsquos argument seems convincing however careful scrutiny reveals that his argument in specious

To begin with the author claims that Monroersquos citizens are interested in jazz He presents three evidences First Secondhellip Thirdhellip Howeverhellip

Monroe 시에 있는 재즈 음악 클럽은 수익성이 좋은 사업이다 현재 가장 가까이에 있는 클럽은 65 마일 정도 떨어져 있다 따라서 이번에 세우려고 하는 C Note 는 독보적인 위치를 점할것이다 더군다나 재즈는 이 시에서 가장 인기있는 음악이다 지난 여름 재즈 축제에서는 10 만명 이상의 Morone 시 주민이 참석하였고 몇몇 유명한 재즈 음악가들도 이곳에 살고 있으며 저녁때 방영되는 라디오 프로그램중에서 최고의 시청률을 보이고 있는 것도 Jazz Nightly 이다 전국조사에서도 전형적인 재즈 팬들은 재즈 분야에 년간 1천 달러 가까이 지출하고 있는 것으로 보고되고 있다 따라서 C Note 클럽이 돈을 벌 수 있는 사업이라는 것은 확실한 것이다

결과 It is clear that the C Note cannot help but make money반박 nearest jazz club 이 양질의 써비스로 여전히 손님을 끌수도 있다Festival 에 얼마나 참여하는지가 jazz 의 인기를 반영하지 않는다 뮤지션이 많이 사는거랑 jazz 의 인기가 상관없다라디오 프로그램이 다른 요인에 의해서 인기일수도 있다 (진행자때문)전국 통계 적용 불가화목 실전반_Ms Noh6In this application the author suggests that a jazz club in Monroe will make a number of profits To support this suggestion the author exemplifies the local condition popularity of jazz in Monroe and nationwide study However careful scrutiny of each of the facts reveals that it provides little credible support for the authorrsquos recommendation Good clear intro

First the author assumes that jazz is popular in Monroe because of several facts the jazz festival last year had high participation some famous jazz musicians live in Monroe and the high-rated radio program is lsquoJazz Nightlyrsquo However this assumption has many drawbacks that must be seriously considered(Good topic sentences) If many attendants in the last-yearrsquos festival came from other cities and not Monroe it is hard to conclude that Monroersquos people like jazz Therefore the author must examine how many Monroe residents actually attended the festival On top of that there is little relationship between habitation of famous jazz musician and the popularity of jazz in Monroe Although several well-known musicians live there if they do not take part in any jazz performance of Monroe this might have no effect to the interest of Monroersquos residents

GRE AWA John박 박정어학원

about jazz Finally in the case of radio program this is also not suitable reason why jazz is popular in Monroe It might be possible that people cannot help choosing lsquoJazz Nightlyrsquo because there are few radio programs at Night The fact that the radio program is the highest rating program is not a germane evidence The approximate number of listeners would be the more crucial evidence Therefore the author needs to seriously deliberate the correlation between jazzrsquos popularity in Monroe and his examples (Good logical flow and clarity)

Second the author uses as evidence the nationwide study that jazz fans spend much money on jazz entertainment to substantiate why starting a jazz club in Monroe will be profitable In other words the author assumes that the characteristics of a nationwide study can be applied to Monroe The national study would lend support to the applicantrsquos claim only if residents in Monroe typify national jazz fans However the author does not provide credible evidence that this is the case Moreover the populations of jazz fans nationwide may be insubstantial Thus the author should not infer hastily that Monroersquos residents will spend much money on enjoying jazz from the nationwide study

Lastly even if jazz is popular in Monroe C Note may not be successful It is entirely possible that residents might still prefer other clubs where they have always went In addition there is another possibility that the nearest jazz club will attract many of Monroersquos people because it serves fine performances and is equipped with favorite facilities Without considering these other possibilities the author cannot make his argument convincing In sum the author presents many reasons that are insufficient in supporting his or her claim In order for the authorrsquos claims to be convincing he needs to advance more persuasive evidence such as the total number of Monroe residents who attended the jazz festival the effects on the popularity of jazz by the musicians living in Monroe and the actual number of residents who would typify themselves to be jazz fans through a local survey Without substantial evidence that C Note will be successful in Monroe the businessmen may be overinvesting in what might lead to a business failureExcellent clarity Score 50

The following appeared in a newsletter offering advice to investors

Over 80 percent of the respondents to a recent survey indicated a desire to reduce their intake of foods containing fats and cholesterol and today low-fat products abound in many food stores Since many of the food products currently marketed by Old Dairy Industries are high in fat and cholesterol the companys sales are likely to diminish greatly and company profits will no doubt decrease We therefore advise Old Dairy stockholders to sell their shares and other investors not to purchase stock in this company

Write a response in which you discuss what questions would need to be answered in order to decide whether the advice and the argument on which it is based are reasonable Be sure to explain how the answers to these questions would help to evaluate the advice

Survey 80

GRE AWA John박 박정어학원

Old Dairy could change their products and manufacture low fat dairy foodsLess competing companies Old Dairy could eventually be the only company that produces hellipImprecise numbers and measurementsCustomers may still buy high fat dairy products

The author of the newsletter is offering potentially dangerous advice by recommending Old Dairy stockholders to withdraw investment and stop purchase What is more the authorrsquos prediction debases the reputation and business of Old Dairy and if false could devoid the investment opportunity of the newsletter readers Therefore investors should examine whether the authorrsquos evidences are substantial

To begin with the author states that 80 percent of the respondents in a survey indicated a desire to reduce their intake of foods He therefore argues that Old Dairyrsquos high fat and cholesterol products would decrease in sales However the author makes a crucial error in this argument First the author provides no evidence that the surveyrsquos results are statistically reliable Were they representative of all the customers Were they chosen for the survey randomly Furthermore the desire to reduce fat and cholesterol intake is a pervasive trend in todayrsquos opulent society however the author erroneously identifies this as a new phenomenon which will affect consumer trends Second having a desire to reduce fat and cholesterol intake does not necessarily indicate that people who have this desire will actually reduce consuming these types of products It is entirely possible that they may continue buying Old Dairy products for its quality and taste Accordingly the author cannot draw any firm conclusion that people will not buy Old Dairy products Therefore if any of these cases are true the author may be offering investors a detrimental investment advice

최근 조사에 대한 응답자중 80 이상이 자신이 먹는 음식에서 지방과 콜레스테롤의 함유량을 줄이고 싶다고 한다 아울러 요즘은 많은 식료품 가계에서 저지방 제품들을 많이 취급하고 있다 현재 Old Dairy Industries가 판매하고 있는 많은 음식제품들은 지방과 콜레스테롤이 높기 때문에 이 회사의 매출이 격감할 것으로 보이며 당연히 매출이익도 줄어들것이다 따라서 이 회사의 주주들은 주식을 매각하고 다른 주식 투자가들도 이 회사의 주식을 매입하지 않는 것이 좋다

결론 Old Dairy stockholders to sell their shares and other investors not to purchase stock in this company

반박 모든 상품이 다 고 지방 고 칼로리는 아니다(비록 많을지라도) 일부의 식품의 경우 기호에 맞어서 히트해서 전체적인 수입이 증가할 수도 있다국내시장만 생각할 수 없다( 외국시장에서 호황을 누릴수 있다 )입맛이라는게 즉각 바뀌는게 아니다

The following appeared in a letter to the editor of the Balmer Island Gazette

On Balmer Island where mopeds serve as a popular form of transportation the population increases to 100000 during the summer months To reduce the number of accidents involving mopeds and pedestrians the town council of Balmer Island should limit the number of mopeds rented by the islands moped rental companies from 50 per day to 25 per day during the summer season By limiting the number of rentals the town council will attain the 50 percent annual reduction in moped accidents that was achieved last year on the neighboring island of Seaville when Seavilles town council enforced similar limits on moped rentals

1 Write a response in which you discuss what questions would need to be answered in order to decide whether the recommendation is likely to have the predicted result Be sure to explain how the answers to these questions would help to evaluate the recommendation

2 Write a response in which you discuss what questions would need to be answered in order to decide whether the prediction and the argument on which it is based are reasonable Be sure to explain how the answers to these questions would help to evaluate the prediction

3 Write a response in which you examine the stated andor unstated assumptions of the argument Be sure to explain how the argument depends on these assumptions and what the implications are for the argument if the assumptions prove unwarranted

Whatrsquos the actual population of Balmer Island 100000mdashis this a significant increase What kind of accidents Skin abrasions or serious injury And compared to Seaville how serious are the accidents and the actual number of accidents Did Seaville enforce other restrictions like safety signsHow different are the conditions of Balmer

GRE AWA John박 박정어학원

and Seaville regarding population road (safety) conditions topography other town-government regulation How much will the economy of Balmer be affected do to this restriction Could it cause an economic recession due to the fact that these rental companiesrsquo chance to make money is only during the summer thereby weakening the economic infrastructure Are there any other ways that could better alleviate the accident rate

Statistics 50-impreciseAnalogy Balmer compared with TorseauFalse Cause Accidents might have occurred because of reasons other than mopeds False Cause population increase may not be part of the cause of the accidentsOther explanations for the accident pedestrians few road safety regulations narrow roadsThere could be other better solutionshellip

Balmer Island의 인구가 여름철에는 십만명으로 늘어난다 2륜차와 보행자간 사고를 줄이기 위해 시의회는 6개의 자전거를 포함한 2륜차 대여업체에게 이 기간동안에는 대여숫자를 일일 50에서 30으로 제한하도록 할 것이다 대여숫자를 줄임으로써 시 의회는 지난해 이웃한 Torseau섬에서 이와 동일한 규제를 시행해서 50나 줄인 결과를 보고 마찬가지로 50를 줄일수 있다고 확신하고 있다

결론 The town council of Balmer Island should linit the number

반박 보행자의 부실에 의해서 사고가 많이 일어날수도 있다렌탈수의 줄임만이 대책은 아니다(대부분의 사람들이 렌탈 보다는 소유하고 있을 수도 있다)옆섬과는 상황이 다를수도 있다(그 섬에서는 사고의 원인이 많은 자전거 수로 인한것일수있다) 하지만 이 섬은 좁은 도로가 원인일 수도 있고 도로 안전 장치의미비가 원일일수 있다

In this letter the author recommends that Balmer Island should limit the number moped rentals from 50 to 30 per day To support this recommendation the author points out several reasons However careful scrutiny of each of the facts reveals that it is filled with unanswered questions that could significantly weaken the authorrsquos recommendation with loops and holes which are answered

The recommendation depends on the assumption that no alternative means of reducing the number of accidents are available However the author fails to offer any evidence to substantiate this crucial assumption It is highly possible that means other than this would better solve the problem Perhaps they could widen the roads or put-up more safety signs Or perhaps the accidents were due to the lack of skills in which case proper safety training would significantly alleviate the problem Without considering and ruling out these and other alternative means of reducing accidetns the author cannot confidently conclude that merely emulating Torseau would suffice Moreover the author is advising a recommendation which could potentially harm the economy of Balmer Island sincehellip Moreover the Balmer Island should alternative means to reduce accidents because limiting moped rentals during the summer could harm the economy of Balmerhellip

First of all the author believes that increase in population and the number of moped rentals are responsible for the accidents It is entirely possible that other factors are responsible for the accidents Perhaps Balmer Islandrsquos lack of safety signs was a major factor Or maybe the roads are narrow and dangerous on the Island therefore the town council could enforce stricter traffic regulations to alleviate the problem Accordingly if either of these scenarios is true the author cannot draw any firm conclusion that increase in the number of population and moped rentals are the cause of the accidents

The author of this editorial recommends that to reduce accidents involving mopeds andpedestrians Balmer Islands city council should restrict moped rentals to 30 per day down from50 at each of the islands six rental outlets To support this recommendation the author citesthe fact that last year when nearby Torseau Islands town council enforced similar measuresTorseaus rate of moped accidents fell by 50 For several reasons this evidence providesscant support for the authors recommendationTo begin with the author assumes that all other conditions in Balmer that might affect therate of moped-pedestrian accidents will remain unchanged after the restrictions are enactedHowever with a restricted supply of rental mopeds people in Balmer might purchase mopedsinstead Also the number of pedestrians might increase in the future with more pedestriansespecially tourists the risk of moped-pedestrian accidents would probably increase For thatmatter the number of rental outlets might increase to make up for the artificial supplyrestriction per outlet--a likely scenario assuming moped rental demand does not declineWithout considering and ruling out these and other possible changes that might contribute to ahigh incidence of moped-pedestrian accidents the author cannot convince me that theproposed restrictions will necessarily have the desired effect

GRE AWA John박 박정어학원

Next the author fails to consider other possible explanations for the 50 decline inTorseaus moped accident rate last year Perhaps last year Torseau experienced unusually fairweather during which moped accidents are less likely Perhaps fewer tourists visited Tot seanlast year than during most years thereby diminishing the demand for rental mopeds to belowthe allowed limits Perhaps last year some of Torseaus moped rental outlets purchased newmopeds that are safer to drive Or perhaps the restrictions were already in effect but were notenforced until last year In any event a decline in Torseaus moped accident rate during onlyone year is scarcely sufficient to draw any reliable conclusions about what might have causedthe decline or about what the accident rate will be in years aheadAdditionally in asserting that the same phenomenon that caused a 50 decline in mopedaccidents in Torseau would cause a similar decline in Balmer the author relies on what mightamount to an unfair analogy between Balmer and Torseau Perhaps Balmers ability to enforcemoped-rental restrictions does not meet Torseaus ability if not then the mere enactment ofsimilar restrictions in Balmer is no guarantee of a similar result Or perhaps the demand formopeds in Torseau is always greater than in Balmer Specifically if fewer than all availablemopeds are currently rented per day from the average Balmer outlet while in Torseau everyavailable moped is rented each day then the proposed restriction is likely to have less impacton the accident rate in Balmer than in TorseauFinally the author provides no evidence that the same restrictions that served to reduce theincidence of all moped accidents by 50 would also serve to reduce the incidence ofaccidents involving mopeds and pedestrians by 50 Lacking such evidence it is entirelypossible that the number of moped accidents not involving pedestrians decreased by a greaterpercentage while the number of moped-pedestrian accidents decreased by a smallerpercentage or even increased Since the author has not accounted for these possibilities theeditorials recommendation cannot be taken seriouslyIn conclusion the recommendation is not well supported To convince me that the proposedrestriction would achieve the desired outcome the author would have to assure me that nochanges serving to increase Balmers moped-pedestrian accident rate will occur in theforeseeable future The author must also provide dear evidence that last years decline inmoped accidents in Torseau was attributable primarily to its moped rental restrictions ratherthan to one or more other factors In order to better evaluate the recommendation I wouldneed more information comparing the supply of and demand for moped rentals on the twoislands I would also need to know the rate of mopedpedestrian accidents in Torseau both priorto and after the restrictions were enforced in TorseauThe following appeared in a magazine article about planning for retirement

Clearview should be a top choice for anyone seeking a place to retire because it has spectacular natural beauty and a consistent climate Another advantage is that housing costs in Clearview have fallen significantly during the past year and taxes remain lower than those in neighboring towns Moreover Clearviews mayor promises many new programs to improve schools streets and public services And best of all retirees in Clearview can also expect excellent health care as they grow older since the number of physicians in the area is far greater than the national average

Write a response in which you discuss what specific evidence is needed to evaluate the argument and explain how the evidence would weaken or strengthen the argument

-Natural beauty and consistent climate may not be the most wanted qualities-Housing costs could have lowered on a national level wealthy retirees may not care about costs-Taxes may be high compared to the nationrsquos average tax rate-What about other qualities of Clearview Crime rate what qualities would retirees want -If schools streets and public services need improvement then this is proof that the current condition of Clearview is low Or due to budgetary reasons the mayor may not follow-up on his promise because of lowered tax rate -Schools and people who are retired no relationship-Physicians What kind of physicians Number is irrelevant Are these physicians capable of addressing the illnesses of old people

This author argues that anyone seeking a place to retire should choose Clearview To supportthis argument the article cites Clearviews consistent climate and natural beauty its fallinghousing costs its low property taxes compared to nearby towns and the mayors promise toimprove schools streets and services The article also claims that retirees can expectexcellent health care because the number of physicians in Clearview greatly exceeds thenational average This argument is flawed in several critical respectsTo begin with although consistent climate and natural beauty might be attractive to manyretirees these features are probably not important to all retirees For many retirees it isprobably more important to live near relatives or even to enjoy changing seasons Thus I

GRE AWA John박 박정어학원

cannot accept the authors sweeping recommendation for all retirees on this basisAlso Clearviews declining housing costs do not necessarily make Clearview the best placeto retire for two reasons First despite the decline Clearviews housing costs might be highcompared to housing costs in other cities Secondly for wealthier retirees housing costs arenot likely to be a factor in choosing a place to retire Thus the mere fact that housing costshave been in decline lends scant support to the recommendationThe articles reliance on Clearviews property-tax rates is also problematic in two respectsFirst retirees obviously have innumerable choices about where to retire besides Clear viewand nearby towns Secondly for retirees who are well-off financially property taxes are notlikely to be an important concern in choosing a place to retire Thus it is unfair to infer fromClearviews property-tax rates that retirees would prefer ClearviewYet another problem with the argument involves the mayors promises In light of Clearviewslow property-tax rates whether the mayor can follow through on those promises is highlyquestionable Absent any explanation of how the city can spend more money in the areas citedwithout raising property taxes I simply cannot accept the editorials recommendation on thebasis of those promises Besides even if the city makes the improvements promised thoseimprovements--particular the ones to schools--would not necessarily be important to retireesFinally although the number of physicians in Clearview is relatively high the per capitanumber might be relatively low Moreover it would be fairer to compare this per capita numberwith the per capita number for other attractive retirement towns--rather than the nationalaverage After all retirees are likely to place a relatively heavy burden on health-careresources Besides the article provides no assurances that the number of physicians inClearview will remain high in the foreseeable futureIn conclusion the recommendation is poorly supported To strengthen it the author mustconvince me--perhaps by way of a reliable survey--that the key features that the vast majorityof retirees look for in choosing a place to live are consistent climate natural beauty and lowhousing costs The author must also provide better evidence that Clear views property taxesare lower than the those of cities in other areas The author must also explain how the city canmake its promised improvements without raising property taxes Finally to better assess theargument I would need to now how the per capita number of physicians in Clearview wouldcompare to the national average in the futureThe following appeared as a letter to the editor from a Central Plaza store owner

Over the past two years the number of shoppers in Central Plaza has been steadily decreasing while the popularity of skateboarding has increased dramatically Many Central Plaza store owners believe that the decrease in their business is due to the number of skateboard users in the plaza There has also been a dramatic increase in the amount of litter and vandalism throughout the plaza Thus we recommend that the city prohibit skateboarding in Central Plaza If skateboarding is prohibited here we predict that business in Central Plaza will return to its previously high levels

Write a response in which you discuss what questions would need to be answered in order to decide whether the recommendation is likely to have the predicted result Be sure to explain how the answers to these questions would help to evaluate the recommendation

Why two years ago What happened two years ago which started this declineIs the dramatic increase in the ldquopopularityrdquo of skateboarding the cause of the steady decline of shoppers Are there any malls nearby Were there any changes nearby which could affect the decline in customersmdasha big mall perhaps Could the decline be due to the shop ownersHow many skateboarders use the plazaWhere do they skateboardDo they shop and are they customersAre the increase in litter and vandalism due to skateboarders Could this be alleviated by installing CCTVs and hiring security

This editorial concludes that the city should ban skateboarding from its downtown CentralPlaza in order to attract visitors to that area to return the area to its former glory and to makeit a place where people can congregate for fun and relaxation To justify this conclusion theeditorial points out that skateboarders are nearly the only people one sees anymore at CentralPlaza and that the Plaza is littered and its property defaced The editorial also points out thatthe majority of downtown merchants support the skate boarding ban This argument is flawedin several critical respectsFirst the editorials author falsely assumes that a ban on skateboarding is both necessaryand sufficient to achieve the three stated objectives Perhaps the city can achieve thoseobjectives by other means as well--for example by creating a new mall that incorporates anattractive new skateboard park Even if banning skateboarders altogether is necessary to meetthe citys goals the author has not shown that this action by itself would suffice Assuming thatthe Plazas reputation is now tarnished restoring that reputation and in turn enticing peopleback to the Plaza might require additional measures--such as removing litter and graffiti

GRE AWA John박 박정어학원

promoting the Plaza to the public or enticing popular restaurant or retail chains to the PlazaSecondly the editorial assumes too hastily that the Plazas decline is attributable to theskateboarders--rather than to some other phenomenon Perhaps the Plazas primary appeal inits glory days had to do with particular shops or eateries which were eventually replaced byless appealing ones Or perhaps the crime rate in surrounding areas has risen dramatically forreasons unrelated to the skateboarders presence at the Plaza Without ruling out these andother alternative explanations for the Plazas decline the editorials author cannot convince methat a skateboard ban would reverse that declineThirdly the editorials author might be confusing cause with effect--by assuming that theskateboarders caused the abandonment of the Plaza rather than vice versa It is entirelypossible that skateboarders did not frequent the Plaza until it was largely abandoned--andbecause it had been abandoned In fact this scenario makes good sense since skateboardingis most enjoyable where there are few pedestrians or motorists to get in the wayFourth it is unreasonable to infer from the mere fact that most merchants favor the ban thatthe ban would be effective in achieving the citys objectives Admittedly perhaps thesemerchants would be more likely to help dean up the Plaza area and promote their businesseswere the city to act in accordance with their preference Yet lacking any supporting evidencethe author cannot convince me of this Thus the survey amounts to scant evidence at best thatthe proposed ban would carry the intended resultFinally the author recommends a course of action that might actually defeat the citysobjective of providing a fun and relaxing place for people to congregate In my experienceskateboarding contributes to an atmosphere of fun and relaxation for adults and children alikemore so than many other types of ambiance Without considering that continuing to allowskateboarding--or even encouraging this activity--might achieve the citys goal more effectivelythan banning the activity the author cannot convincingly conclude that the ban would be in thecitys best interestsIn sum the argument is a specious one To strengthen it the editorials author must providedear evidence that skateboarding and not some other factor is responsible for the conditionsmarking the Plazas decline The author must also convince me that no alternative means ofrestoring the Plaza are available to the city and that the proposed ban by itself would suffice toattract tourists and restore the Plaza to its former glory Finally to better assess the argument itwould be useful to know the circumstances under which the downtown merchants would bewilling to help the city achieve its objectives

6그룹 약한 비유 빈출

The following recommendation appeared in a memo from the mayor of the town of Hopewell

Two years ago the nearby town of Ocean View built a new municipal golf course and resort hotel During the past two years tourism in Ocean View has increased new businesses have opened there and Ocean Views tax revenues have risen by 30 percent Therefore the best way to improve Hopewells economymdashand generate additional tax revenuesmdashis to build a golf course and resort hotel similar to those in Ocean View

Write a response in which you examine the stated andor unstated assumptions of the argument Be sure to explain how the argument depends on these assumptions and what the implications are for the argument if the assumptions prove unwarranted

GRE AWA John박 박정어학원

Assumptions The author assumes that OVrsquos municipal golf course and resort hotel caused tourism new businesses and increased tax revenues There may be other reasons advertising promo He assumes that this will continueAssumes that Ocean View and Hopewell are similar in many waysmdashthe name suggests otherwise OV may have always been a tourist attractions for its beaches We need to know the topography

2년전 Ocean View 시는 시정 소유 골프 및 휴양지 호텔을 신축했다 그리고 지난 2년동안 이 시의 관광객이 증가했으며 새로운 사업들이 생겨났다 그에따라 시의 세수도 30나 증가했다 Hopewell의 경제를 향상시키고 아울러 세수를 늘릴 수 있는 가장 좋은 방법은 Ocean View에 세워진 것과 같은 골프 시설과 휴양지 호텔을 신축하는 것이다

1 다른 요인으로 관광 산업이 발전했을 수도 있다 문화 유적이 발견이 되었거나 도로의 정비등으로 여행자가 늘었을 수도 있다

2 관광 산업의증가가 늘어난 세수의 원인이 아니라 새로 유입된 인구의 증가나 다른 공장에서 발생한 것일 수 있다

3 2년동안 한참 골프가 붐을 이루었을 수 있다 경제상황이 나빠지거나 다른 레포츠가 인근 지역에 생겨난다면 골프하는 사람이 줄어들 수 있다

In this memo HopeweUs mayor recommends that in order to stimulate the towns economyand boost tax revenues HopeweU should build a new golf course and resort hotel just as thetown of Ocean View did two years ago To support this recommendation the mayor points outthat in Ocean View during the last two years tourism has increased new businesses haveopened and tax revenues have increased by 30 I find the mayors argument unconvincingin several important respectsFirst of all it is possible that the mayor has confused cause with effect respecting the recentdevelopments in Ocean View Perhaps Ocean Views construction of a new golf course andhotel was a response to previous increases in tourism and business development increasesthat have simply continued during the most recent two years Since the mayor has failed toaccount for this possibility the claim that Hopewell would boost its economy by alsoconstructing a golf course and hotel is completely unwarrantedSecondly the mayor fails to account for other possible causes of the trends in Ocean Viewduring the last two years The increase in tourism might have been due to improving economicconditions nationwide or to unusually pleasant weather in the region The new businessesthat have opened in Ocean View might have opened there irrespective of the new golf courseand hotel And the 30 increase in tax revenues might have been the result of an increase intax rates or the addition of a new type of municipal taxWithout ruling out these and other alternative explanations for the three recent trends inOcean View the mayor cannot reasonably infer based on those trends that Hopewellseconomy would benefit by following Ocean Views exampleThirdly even if the recent trends in Ocean View are attributable to the construction of the newgolf course and hotel there the mayor assumes too hastily that the golf course and hotel willcontinue to benefit that towns overall economy The mayor has not accounted for thepossibility that increased tourism will begin to drive residents away during tourist season orthat new business development will result in the towns losing its appeal as a place to visit or tolive Unless the mayor can convince me that these scenarios are unlikely I cannot accept themayors recommendation that Hopewell follow Ocean Views exampleFinally the mayors argument rests on the unsubstantiated assumption that Hopewell andOcean View are sufficiently alike in ways that might affect the economic impact of a new golfcourse and hotel Hopewell might lack the sort of natural environment that would attract moretourists and new businesses to the town--regardless of its new golf course and hotel For thatmatter perhaps Hopewell already contains several resort hotels and golf courses that are notutilized to their capacity If so building yet another golf course and hotel might amount to amisallocation of the towns resources--and actually harm the towns overall economyIn sum the mayors recommendation is not well supported To bolster it the mayor mustprovide better evidence that Ocean Views new golf course and hotel and not some otherphenomenon--has been responsible for boosting Ocean Views economy during the last twoyears To better assess the recommendation I would need to know why Ocean View decidedto construct its new golf course and hotel in the first place--specifically what events prior toconstruction might have prompted that decision I would also need to thoroughly compare

GRE AWA John박 박정어학원

HopeweU with Ocean View--especially in terms of their appeal to tourists and businesses--todetermine whether the same course of action that appears to have boosted Ocean Viewseconomy would also boost Hopewells economy

The following is part of a memorandum from the president of Humana University

Last year the number of students who enrolled in online degree programs offered by nearby Omni University increased by 50 percent During the same year Omni showed a significant decrease from prior years in expenditures for dormitory and classroom space most likely because instruction in the online programs takes place via the Internet In contrast over the past three years enrollment at Humana University has failed to grow and the cost of maintaining buildings has increased along with our budget deficit To address these problems Humana University will begin immediately to create and actively promote online degree programs like those at Omni We predict that instituting these online degree programs will help Humana both increase its total enrollment and solve its budget problems

Write a response in which you discuss what questions would need to be answered in order to decide whether the prediction and the argument on which it is based are reasonable Be sure to explain how the answers to these questions would help to evaluate the prediction

Is Omni University successful due to the online degree program 50 Is the decrease in expenditures for dormitory and classroom space due to the decrease in of on-campus students Which classes were successful Does HU have those classes

Even if the long-distance degree programs at Omni University benefited the school the presidentrsquos recommendation that Human College should emulate Omni University is too hasty First OUrsquos name implies that the school would have more majors than Humanahellip the president should examine which degrees were in the long-distance programhellip

지난해에는 Omni 대학에서 개강했던 원거리 학생 학점 취득 프로그램을 등록했던 학생들의 숫자가 50나 증가했다 같은해 기간동안 Omni 대학에서는 그 전년도부터 기숙사와 학급의 공간 확충을 위한 예산을 대폭 줄였는데 이는 이 원거리 학점 취득 프로그램이 양방향 비디오 컴퓨터 접속을 통해서만 가능한 수업지도 방식이기때문인 것으로 보인다 반면 지난 3개년 동안 Humana 대학에서의 수강률은 감소한데다가 건물

GRE AWA John박 박정어학원

유지비도 올랐다 따라서 Humana대학의 수강을 늘리고 예산손실을 회복하기 위해서는 Omni 대학에서 취한 조치와 같은 능동적인 프로그램을 추진해야 한다

결론 we should initiate and actively promote long-distance degree programs like those at Omni 반박 원거리 학생 취득 프로그램 숫자가 증가한거하고 예산이 줄어드는 것 사이에 연관이 약하다 (causal 학생의 증가로 관리비용 증가할수 있음 원거리 수업가능 장비도입에의한 비용발생)bad analogy(omni university 하고 같은 조건이 아니다 )-gt omni college 가 강좌내용이 좋아서 학생의 등록이 많을수 있다 Humana 대학에서 만들었다 하더라도 인기 없을수 있음다른 요인에 의해서 Humana 대학의 수강 인원이 증가할수 있음(비록 과거엔 인기가 없었을지라도)

The following appeared as part of a business plan developed by the manager of the Rialto Movie Theater

Despite its downtown location the Rialto Movie Theater a local institution for five decades must make big changes or close its doors forever It should follow the example of the new Apex Theater in the mall outside of town When the Apex opened last year it featured a video arcade plush carpeting and seats and a state-of-the-art sound system Furthermore in a recent survey over 85 percent of respondents reported that the high price of newly released movies prevents them from going to the movies more than five times per year Thus if the Rialto intends to hold on to its share of a decreasing pool of moviegoers it must offer the same features as Apex

Write a response in which you discuss what questions would need to be answered in order to decide whether the recommendation is likely to have the predicted result Be sure to explain how the answers to these questions would help to evaluate the recommendation

Before following through this business plan the manager should investigate the cause of Rialtorsquos unsuccessful business

The author provides no evidence that the surveyrsquos results are statistically reliable The surveyrsquos sample of 85 percent must be sufficient in size and representative of overall population of the city where Rialto and Apex is serving Lacking evidence of a sufficiently representative sample the author cannot justifiably rely on the survey to draw any conclusion whatsoever The author does not indicate that Apex is indeed currently successful However even if Apex is enjoying success the argument relies on what might be a false analogy between Rialto and Apex In order for Apex to serve as a model that Rialto should emulate the author must assume that all relevant circumstances are essentially the same However this assumption is unwarranted For example the argument overlooks the face that Apex is located in a strategic placemdashbeside a mall where customers can not only watch a movie but also enjoy shopping Therefore simply changing the facility to that of Apex may not lead to success

The author does not mention whether Apex is successful or not Nevertheless even if Apex is currently successful the argument relies on what might be a false analogy between Rialto and Apex In order for Apex to serve as a model that Rialto should emulate the author must assume that all relevant circumstances are essentially the same However this assumption is unwarranted For example the argument overlooks the fact that these two institutions are located in different locations Rialto in downtown and Apex in a mall outside of town Although Apex opened with state-of-the-art facilities the decisive factor in its success could be due to its strategic location of being in a mall People could enjoy both shopping and movies at one location thus they may prefer Apex over Rialto Furthermore the place where people enjoy leisure activities has shifted in the past decades for most cities from downtown to the suburbs Therefore Rialto may not be successful even if it emulates Apexrsquos facilities A better business plan may be relocating Apex to the thriving section of the downtown

Rialto 극장은 지난 50여년간 지역 회관으로써 시내에 위치해 있으면서도 이제 변화를 꾀하지 않으면 문을 닫을

GRE AWA John박 박정어학원

판이다 이 극장은 시외 쇼핑타운에 새로 들어선 Apex 극장의 사례를 본받아야 했다 Apex가 지난해 개업했을 당시 이 극장은 비디오 아케이드 플러쉬 카펫트 바닥과 좌석 그리고 최신 음향시설을 갖추었다 더군다나 최근 조사에서는 응답자의 85 이상이 새로 출시된 영화 입장료가 비싼 탓으로 지난해보다 5배이상의 관람객이 줄어들었다고 나타났다 따라서 Rialto 극장이 줄어들고 있는 관람객을 뺐기지 않고 유지하려면 Apex와 같은 시설들을 갖추어야 할 것이다주장 리알토 극장이 줄어들고 있는 관람객을 뺐기지 않고 유지하려면 Apex와 같은 시설들을 갖추어야 할 것이다

1 조사에서 응답자가 전체를 대표할 수 없다 2 apex 극장이 좋은 시설을 갖추고 있지만 그로 인해 수익이 많이 발생했다는 말이 없으므로 시설투자를

하고도 좋은 결과를 얻을 수 있을지 그 근거가 미흡하다3 좋은 영화가 출시된다면 입장료가 비싸도 영화관에서 꼭 보려고 할 수 있다 4 rialto 가 시설이 아닌 다른 요인에 의해 장사가 안될수도 있다( 우범 지역이라든지)

The following is a recommendation from the business manager of Monarch Books

Since its opening in Collegeville twenty years ago Monarch Books has developed a large customer base due to its reader-friendly atmosphere and wide selection of books on all subjects Last month Book and Bean a combination bookstore and coffee shop announced its intention to open a Collegeville store Monarch Books should open its own in-store cafeacute in the space currently devoted to childrens books Given recent national census data indicating a significant decline in the percentage of the population under age ten sales of childrens books are likely to decline By replacing its childrens books section with a cafeacute Monarch Books can increase profits and ward off competition from Book and Bean

Write a response in which you examine the stated andor unstated assumptions of the argument Be sure to explain how the argument depends on these assumptions and what the implications are for the argument if the assumptions prove unwarranted

The following is a recommendation from the business manager of Monarch Books

Since its opening in Collegeville twenty years ago Monarch Books has developed a large customer base due to its reader-friendly atmosphere and wide selection of books on all subjects Last month Book and Bean a combination bookstore and coffee shop announced its intention to open a Collegeville store Monarch Books should open its own in-store cafeacute in the space currently devoted to childrens books Given recent national census data indicating a significant decline in the percentage of the population under age ten sales of childrens books are likely to decline By replacing its childrens books section with a cafeacute Monarch Books can increase profits and ward off competition from Book and Bean

1 Write a response in which you discuss what questions would need to be answered in order to decide whether the recommendation is likely to have the predicted result Be sure to explain how the answers to these questions would help to evaluate the recommendation

2 Write a response in which you discuss what specific evidence is needed to evaluate the argument and explain how the evidence would weaken or strengthen the argument

No evidence regarding Monarch Bookrsquos successEven if Regal Bookrsquos is successful this may not be attributable to the cafeacute False analogy Emulating may not lead to success Other factors may be involvedInsufficient condition The national census is not enough evidence that childrenrsquos book sales will decline Can

GRE AWA John박 박정어학원

the national census represent the local child populationDid opening a cafeacute boost sales for Regal Books Even assuming Regal is successful by opening a cafeacute this may not be suitable for Monarch which plans to close the childrenrsquos book section to establish a cafe Imprecise language ldquorelatively little spacerdquo how smallThe managerrsquos recommendation contradicts what he says Since Monarch is popular for its wide selection of books closing a selection which targets a major group of readers may hurt Monarchrsquos salesIs this the best way to compete

When Stanley Park first opened it was the largest most heavily used public park in town It is still the largest park but it is no longer heavily used Video cameras mounted in the parks parking lots last month revealed the parks drop in popularity the recordings showed an average of only 50 cars per day In contrast tiny Carlton Park in the heart of the business district is visited by more than 150 people on a typical weekday An obvious difference is that Carlton Park unlike Stanley Park provides ample seating Thus if Stanley Park is ever to be as popular with our citizens as Carlton Park the town will obviously need to provide more benches thereby converting some of the unused open areas into spaces suitable for socializing

Write a response in which you examine the stated andor unstated assumptions of the argument Be sure to explain how the argument depends on these assumptions and what the implications are for the argument if the assumptions prove unwarranted

Stanley 파크가 처음 개장했을 당시 가장 크고 가장 많이 이용되는 공원이었다 아직도 공원중에서는 가장 크지만 이용률은 상당히 떨어졌다 지난달 공원 주차장에 설치해놓은 비디오 카메라를 통해 보면 drop(주차장으로 여겨짐) 이용률이 가장 높았다 수치상으로는 하루 평균 50대의 차량만이 이용하였다 반면 직장 중심거리에 위치한 작은 규모의 Carlton 파크는 주당 무려 150여명 이상이 이용하고 있다 Stanley 파크와는 달리 Carlton 파크에는 의자가 있다는 것이 가장 뚜렷한 차이점이다 따라서 Stanley 파크가 Carlton 파크처럼 시민들이 자주 이용하는 공원이 되기 위해서는 벤치를 설치할 필요가 있으며 이렇게 사용되지 않는 일부 공간을 활용해서 사교를 위한 공간으로 바꾸어야 한다 ===gtdrop 에 대한 첨부사항 (영영사전내용입니다)---- a place or central depository to which something (as mail money or stolen property) is brought for distribution or transmission also the act of depositing something at such a place dropgt

주장 if Stanley Park is ever to be as popular with our citizens as is Carlton Park the town will obviously need to provide more benches thereby converting some of the unused open areas into spaces suitable for socializing1 조사가 언제 이루어진 것인가 조사가 언제 실시되었느냐에 따라 결과가 다를 수있다 현재는 다시 스탠리 파크가 늘어났었을 수 있다 2 벤치를 많이 설치했다고 해서 많은 관광객이 오지 않을수 있다(사람들이 벤치나 사교 공간을 원한다는 어떠한 자료도 없다)3스탠리 파크 주변에 교통 상황이 악화가 되었거나 칼튼 파크에서 문화행사등을 많이 가져서 이용객이 줄어든것일 수도 있다 4 칼튼 파크가 중심지에 있어서 접근성이 좋을수 있다5 조사가 같은 시간을 기준으로 한게 아니다(하나는 주중이고 하나는 주말이다)6사람의 수와 차의 대수를 같은것으로 비교할수 없다 (차안에 몇 명이 타고 있는지 모르고 대중교통을 이용해서 왔을수도 있다)

Page 15: GRE writing argument brain storm

GRE AWA John박 박정어학원

needs to provide the specific season of when the census and observation occurred In this letter the writer is informing an editor of an environmental magazine that the number of amphibians was greatly reduced since 1975 and he points out the introduction of trout as the only reason for the decline However the author fails to provide crucial evidences that could strengthen or weaken his conclusion

The following appeared in a memorandum from the president of Hyper-Go Toy Company

Last year sales of our Fierce Fighter toy airplane declined sharply even though the toy had been a top seller for three years Our customer surveys show that parents are now more worried about youthful violence and are concerned about better education for their children Therefore to maintain profits we should discontinue all our action toys and focus exclusively on a new line of educational toys Several other toy companies have already begun marketing educational toys and report sales increases last year of 200 percent And since the average family income is growing sales of new Hyper-Go toys should also increase

Write a response in which you discuss what specific evidence is needed to evaluate the argument and explain how the evidence would weaken or strengthen the argument

First the president indicates that the sales of Fierce Fighter toy airplane declined sharply However he fails to consider the fact that toys are a fad Since FFT enjoyed a three year success it may be natural that the trend would subsideSecond Customer survey is this representative of most toy consumers This trend is not newThird other companies may have profited not through educational but other toys Or educational toy profit may be small in proportion to their sale of other toys 200Parents are not the customers companies should concentrate on what the children want to increase profitLastly parents worry about youthful violence and concern for better education are not new trends

The presidentrsquos decision to discontinue all action toys and focus only on educational toys is too extreme If this decision is taken into effect its procedure may be an onerous task because it would require major shifts in human resources and company image Therefore further data should be considered before following up on this decision

우리 회사에서 지난 3년간 최고 매출을 일으켰던 Fierce Fighter 장난감 비행기가 지난해에는 매출이 급격히 떨어졌다 자체 고객 조사에서는 부모들이 현재 청소년 폭력에 걱정을 하고 있어서 아이들의 양질 교육에 더 관심을 가지고 있는 것으로 나타났다 따라서 회사의 수익을 유지하기 위해서는 모든 자사 전투용 장난감 생산을 중단하고 오로지 교육적인 장난감 생산에 집중해야 한다 몇몇 여타 장난감 회사들도 이미 교육용 장난감 마케팅을 시작해서 지난해에는 200의 매출신장을 가져왔다고 한다 그리고 평균 가계 수입이 점점 늘고 있기 때문에 신형 Hyper-Go 장난감의 매출도 늘어날 것이다

3그룹 Time-shift Error

Woven baskets characterized by a particular distinctive pattern have previously been found only in the immediate vicinity of the prehistoric village of Palea and therefore were believed to have been unique to the Palean people Recently however archaeologists discovered such a Palean basket in Lithos an ancient village across the Brim River from Palea The Brim River is very deep and broad and so the ancient Paleans could only have crossed it by boat but there is no evidence that the Paleans had boats And boats capable of carrying groups of people and cargo were not developed until thousands of years after the Palean people disappeared Moreover Paleans would have had no need to cross the rivermdashthe woods around Palea are full of nuts berries and small game It follows that the so-called Palean baskets were not unique to Palea

GRE AWA John박 박정어학원

Write a response in which you discuss what specific evidence is needed to evaluate the argument and explain how the evidence would weaken or strengthen the argument

RefutationPrehistoric time shift-gtbrim river could have been narrow and shallow or it might have not existed Indigenous patterns may exist in other disconnected remote placesNuts berries small game(hunting) may not have existed or the author should be proved these existed at that time Abundance of resources doesnrsquot support the reason for seclusionBoats not yet found baskets may have been carried across by the river current without the help of a boat Lithos might have crossed the river for commercial purposeOne Palean basket does not substantiate the authorrsquos claimThe absence of evidence is not an evidence of absence

The author assumes without justification that present conditions are the same as at the prehistoric era The author unfairly infers from the presence of Brim River which exist today that it would have existed in the past However the author fails to offer any evidence to substantiate this inference It is very likely that the Brim River might not have existed in prehistoric times or if it did exist may have been shallow and narrow enough for the Paleans to easily cross Any of these scenarios if true would serve to undermine the claim thathelliphelliphelliphelliphellip

First the author claims that the Brim River was very deep and broad so the Paleans could not have crossed it However the author fails to offer any evidence to substantiate that this was true in the prehistoric time of the Paleans For all we know the Brim River might not have existed in the prehistoric era or if it did exist could have been a shallow and narrow river For example scientists believe that thousands of years ago an ice-bridge existed on the Bering Sea connecting Eurasia to what is now North America to explain how the Eskimos and the Asian inhabitants of America came to migrate all over the Americas In this example the absence of evidence is not an evidence of absence Therefore to sufficiently support his claim the author needs to substantiate the fact that the Brim River really did exist and was broad and wide in the Prehistoric Era proving that the Paleans could not have influenced or traded with other groups of people

First the author claims that the Brim River was very deep and broad so the Paleans could not have crossed it However the author fails to offer any evidence to substantiate that this was true in the prehistoric times of the Paleans For all we know the Brim River might not have existed in the prehistoric era or if it did exist could have been a shallow and narrow river For example rivers are created by natural erosion over thousands of years Niagara Falls carved its way from the mouth of Lake Ottawa and created a long river Likewise the current Brim Riverrsquos physical features may not have been wide and deep Therefore to sufficiently support his claim the author needs to substantiate the fact that the Brim River really did exist and was broad and wide in the Prehistoric Era proving that the Paleans could not have influenced or traded with other groups of people

예전에는 실로짠 특이한 무늬 바구니가 Palea의 선사시대 지역의 인근마을에서만 발견되어왔기 때문에 Palea 마을 사람들의 특징이라고 여겨졌었다 그러나 최근들어 고고학자들이 Lithos지역에서 Palean 바구니를 발견하였는데 그 당시 지역은 Brim 강을 가로질러 Palea까지 닿아있었다 이 강은 수심이 아주 깊고 강폭이 넓었으며 때문에 고대의 Palea인들은 배를 이용해서 강을 건널수 있었을 것이다 그러나 이들이 배를 가지고 있었다는 증거는 발견되지 않고 있다 더군다나 이들이 멸명한 이후 수천년이 지난뒤에도 수많은 물자와 사람을 실어 나를수 있는 용적을 가진 배는 개발되지 않았다 이와더불어 Palea인들은 강을 건널필요가 없었는데 그것은 너트나무 장과열매 그리고 작은 사냥감들이 주변숲에 풍부했기때문이다 따라서 Palean 바구니라고 하는 것도 Palea인들만의 전유물이 아니라는 결론을 얻을 수 있다

결론 if follows that the so-called Palean baskets were not unique to Palea이번문제는 굿이 causal Bad analogy 로 구분해서 찾기가 힘드내요 배를 발견했다는 증거가 없는것이다 (앞으로도 발견될수 있음)

GRE AWA John박 박정어학원

계절의 영향으로 겨울에 얼음이 두껍게 언다든지 여름에 가뭄으로 인해서 건널수 있다물자가 풍부한 것이 이동하지 않을 조건이 아니다 다른 것에 의해서 이동가능(의약품등)

Thirteen years ago researchers studied a group of 25 infants who showed signs of mild distress when exposed to unfamiliar stimuli such as an unusual odor or a tape recording of an unknown voice They discovered that these infants were more likely than other infants to have been conceived in early autumn a time when their mothers production of melatonin hormone known to affect some brain functions would naturally increase in response to decreased daylight In a follow-up study conducted earlier this year more than half of these children now teenagers who had shown signs of distress identified themselves as shy Clearly increased levels of melatonin before birth cause shyness during infancy and this shyness continues into later life

Write a response in which you examine the stated andor unstated assumptions of the argument Be sure to explain how the argument depends on these assumptions and what the implications are for the argument if the assumptions prove unwarranted

Any baby exposed to unpleasant stimuli would react in such wayFirst of all the author states 25 infants as his evidence However this research sample is too small to prove his claimSecond the author states that 25 infants were conceived in early autumn which he claims lead to a shy disposition However this is faulty evidenceThird the research study was a long term study done in the span of 13 years However the author only writes about the initial and final stages of the study and leaves out evidences of what could have happened during the 13 years which could be more evidential factors of influenceFourth neither the infantsrsquo genetic predisposition nor their environment were taken into accountFinally the author concludes that his shyness continues into later life (Other factors could alter this disposition epigenetic theory)

13 년전 학자들은 25명의 유아를 대상으로 이상한 냄새나 특이한 소리를 녹음한 테잎등으로 낯선 자극을 주었을때 보이는 미미한 압박감 증상을 조사하였다 이들은 성숙기가 막 지났을 즈음에 보통의 유아들이 비슷한 증상을 보이는 정도 보다는 다소 민감한 반응을 보였는데 이 시기는 아이의 엄마가 뇌의 일부 기능에 영향을 미치는 것으로 알려진 멜라토니아 호르몬을 생산하는 시기로써 이 호르몬은 낯 시간이 짧을때 자연적으로 증가할 수도 있다 금년초에 실시된 추가연구에서 현재 10대로 성장한 당시 조사대상의 절반 이상의 아이들이 부끄럼을 잘타는 것으로 여기고 있었다 따라서 분명한 것은 출산전 멜라토닌 수치의 증가가 유아기에 수줍음 등의 영향을 미치게 되며 이러한 영향이 성장후에도 작용한다는 것이다

주장 Clearly increased levels of melatonin before birth cause shyness during infancy and this shyness continues into later life

1 25명의 아기로 결론 내리기에 샘플이 작다2 과학적 사실들에 대한 명확한 근거가 엇음3 다른 영향을 간과했다(다른 호르몬에 의한 영향 후천적인 성격형성의 영향)

GRE AWA John박 박정어학원

The following is a letter to the editor of the Atticus City newspaper

Former Mayor Durant owes an apology to the city of Atticus Both the damage to the River Bridge which connects Atticus to Hartley and the traffic problems we have long experienced on the bridge were actually caused 20 years ago by Durant After all he is the one who approved the construction of the bridge If he had approved a wider and better-designed bridge on which approximately the same amount of public money would have been spent none of the damage or problems would have occurred Instead the River Bridge has deteriorated far more rapidly over the past 20 years than has the much longer Derby Bridge up the river Even though the winters have been severe in the past several years this is no excuse for the negligence and wastefulness of Durant

Write a response in which you discuss what questions would need to be answered in order to decide whether the recommendation is likely to have the predicted result Be sure to explain how the answers to these questions would help to evaluate the recommendation

전임 시장인 Durant 씨는 Atticus 시에 대해 사과할 의무가 있습니다 Atticus와 Hartley를 잇는 River Bridge 교량에 대한 피해와 이 교량에서 오랫동안 주민들이 겪어오고 있는 교통 혼잡 문제들은 실제로 20년 전부터 시작된 것이었습니다 결정적으로 그가 교량 공사를 허가했던 바로 그 장본인입니다 당시 비슷한 공사비용으로 폭이 더 넓고 튼튼하게 설계된 교량을 허가했다면 이러한 문제나 피해는 발생하지 않았을 겁니다 더군다나 이 다리는 지난 20년 동안 상류에 건설된 훨씬 오래된 Derby 다리보다도 빠르게 부식되어 갔습니다 지난 수년동안 심지어 혹한이 있었다 하더라도 이러한 태만과 국고 손실에 대한 책임을 회피할 길이 없는 것입니다

결론 Former Mayor Durant owes an apology to the city of Atticus

1 디자인이 문제가 아닐수 있다 (디자인은 좋았으나 건설과정에 문제가 있었을 수 있다)2 그 당시의 시예산이 적어서 더 큰 다리를 짓기가 불가능했을 수도 있다3 교통량이 많거나 다른 상황으로 인해서 부식이 빨리 됐을 수 있다4 그 당시에는 최선의 선택이였지만 갑자기 변한 상황에 의해서 이런 문제점들이 발생했을 수 있다

GRE AWA John박 박정어학원

4그룹 거짓인과관계 오류 (False Cause) 빈출

Fifteen years ago Omega University implemented a new procedure that encouraged students to evaluate the teaching effectiveness of all their professors Since that time Omega professors have begun to assign higher grades in their classes and overall student grade averages at Omega have risen by 30 percent Potential employers looking at this dramatic rise in grades believe that grades at Omega are inflated and do not accurately reflect student achievement as a result Omega graduates have not been as successful at getting jobs as have graduates from nearby Alpha University To enable its graduates to secure better jobs Omega University should terminate student evaluation of professors

Write a response in which you discuss what specific evidence is needed to evaluate the argument and explain how the evidence would weaken or strengthen the argument

Omega professor evaluation implemented 15 years ago =gt Omega prof assign higher grades 30Employers believe therersquos grade inflation

Thus unsuccessful employment than AlphaTherefore to secure jobs Omega should end evaluating profs

Specific evidence neededRelationship between higher grades and evaluationRelationship between GPA and unsuccessful employmentAlpharsquos education could just be better than OmegaldquoFifteen years agordquo is a long time other factors could have influenced Why is the inflation a problem just now How much is Alpha better Is the comparison just How much gap is thereOmegarsquos student could just be doing better in their studiesComparison to other universities다른 대안 없나hellip Could Omega alleviate the employment problem by implementing a different procedure or program

15 년전 우리 대학은 학생들로 하여금 교수평가를 하도록 한 새로운 조치를 시행했었습니다 이후 교수들은 자신의 학과 학생들에게 높은 학점을 주었으며 그에따라 학생들의 전체 평점이 30나 올랐습니다 외부의 기업체들은 분명 점수가 지나치게 부풀려졌다고 믿고 있습니다 결국 본 대학 졸업생들이 인근 Alpha 대학의 졸업자들보다 구직률이 떨어지는 이유를 잘 보여주고 있는 것입니다 이를 해결하기 위해 이제부터는 학생들에 의한 교수평가제를 중단해야 합니다

결론 Omega University should now terminate student evaluation of professors

반박 교수 평가와 학점 인플레의 연관성이 적다( 교수 평가를 먼저하고 학점을 나중에 매길수도 있다)채용기준에 성적만 있는게 아니다 학업성취의 결과 일수도 있다 Alpha 가 원래 유능했다 Alpha 의 교육내용이 좋았다

GRE AWA John박 박정어학원

In this memo the dean of Omega University(OU) recommends OU to terminate professor evaluation to secure better jobs for the students To support this recommendation the dean offers several reasons However this argument contains several logical flaws which render it unconvincing

A threshold problem with the argument involves the voluntary nature of the evaluationprocedure The dean provides no evidence about the number or percentage of Omegastudents who participate in the procedure Lacking such evidence it is entirely possible thatthose numbers are insignificant in which case terminating the procedure is unlikely to haveany effect on the grade average of Omega students or their success in getting jobs aftergraduationThe argument also assumes unfairly that the grade-average increase is the result of theevaluation procedure--rather than some other phenomenon The dean ignores a host of otherpossible explanations for the increase--such as a trend at Omega toward higher admissionstandards or higher quality instruction or facilities Without ruling out all other possibleexplanations for the grade-average increase the dean cannot convince me that by terminatingthe evaluation procedure Omega would curb its perceived grade inflation let alone help itsgraduates get jobsEven if the evaluation procedure has resulted in grade inflation at Omega the deans claimthat grade inflation explains why Omega graduates are less successful than Alpha graduatesin getting jobs is unjustified The dean overlooks a myriad of other possible reasons forOmegas comparatively poor job-placement record Perhaps Omegas career services areinadequate or perhaps Omegas curriculum does not prepare students for the job market aseffectively as Alphas In short without accounting for other factors that might contribute toOmega graduates comparative lack of success in getting jobs the dean cannot justify theclaim that if Omega curbs its grade inflation employers will be more likely to hire OmegagraduatesFinally even if the dean can substantiate all of the foregoing assumptions the deansassertion that Omega must terminate its evaluation procedure to enable its graduates to findbetter jobs is still unwarranted in two respects First the dean ignores other possible ways bywhich Omega can increase its job-placement record--for example by improving its publicrelations or career-counseling services Second the dean unfairly equates more jobs withbetter jobs In other words even if more Omega graduates are able to find jobs as a result ofthe deans recommended course of action the kinds of jobs Omega graduates find would notnecessarily be better onesIn sum the deans argument is unpersuasive as it stands To strengthen it the dean mustprovide better evidence that the increase in grade average is attributable to Omegasprofessor-evaluation procedure and that the end result is a perception on the part ofemployers that Omega graduates are less qualified for jobs than Alpha graduates To betterassess the argument I would need to analyze 15-year trends in (l) the percentage of Omegastudents participating in the evaluation procedure (2) Omegas admission standards andquality of education and (3) Omegas emphasis on job training and career preparation I wouldalso need to know what other means are available to Omega for enabling its graduates to findbetter jobs

GRE AWA John박 박정어학원

The following appeared in a memo from a vice president of Quiot Manufacturing

During the past year Quiot Manufacturing had 30 percent more on-the-job accidents than at the nearby Panoply Industries plant where the work shifts are one hour shorter than ours Experts say that significant contributing factors in many on-the-job accidents are fatigue and sleep deprivation among workers Therefore to reduce the number of on-the-job accidents at Quiot and thereby increase productivity we should shorten each of our three work shifts by one hour so that employees will get adequate amounts of sleep

Write a response in which you examine the stated andor unstated assumptions of the argument Be sure to explain how the argument depends on these assumptions and what the implications are for the argument if the assumptions prove unwarranted

The following appeared in a memo from a vice president of Alta Manufacturing

During the past year Alta Manufacturing had thirty percent more on-the-job accidents than nearby Panoply Industries where the work shifts are one hour shorter than ours Experts believe that a significant contributing factor in many accidents is fatigue caused by sleep deprivation among workers Therefore to reduce the number of on-the-job accidents at Alta we recommend shortening each of our three work shifts by one hour If we do this our employees will get adequate amounts of sleep

Write a response in which you discuss what questions would need to be answered in order to decide whether the recommendation and the argument on which it is based are reasonable Be sure to explain how the answers to these questions would help to evaluate the recommendation

The following appeared in a memo from the vice president of Butler Manufacturing

During the past year workers at Butler Manufacturing reported 30 percent more on-the-job accidents than workers at nearby Panoply Industries where the work shifts are one hour shorter than ours A recent government study reports that fatigue and sleep deprivation among workers are significant contributing factors in many on-the-job accidents If we shorten each of our work shifts by one hour we can improve Butler Manufacturings safety record by ensuring that our employees are adequately rested

1 Write a response in which you discuss what specific evidence is needed to evaluate the argument and explain how the evidence would weaken or strengthen the argument

2 Write a response in which you discuss what questions would need to be answered in order to decide whether the recommendation is likely to have the predicted result Be sure to explain how the answers to these questions would help to evaluate the recommendation

4번 반복됨

Alta has 30 more job accidents than Panoply(work shifts one hour shorter)Experts Job accidents caused by fatigue and sleep deprivationTherefore to reduce job accidents and increase productivity shorten three work shifts by one hour for adequate sleep

지난해 우리 회사는 인근 Panoply Industries보다 업무상 재해가 30나 더 많았다 그 회사는 우리보다 근무 교대시간이 1시간 정도 짧았다 전문가들은 대부분의 업무상 재해에 있어서 가장 중요한 요인이 과로와 수면부족으로 보고있다 따라서 우리 회사에서 높은 산업재해를 줄이고 아울러 생산성을 높이기 위해서는 근로자들이 충분한 수면을 취할 수 있도록 1시간씩 3교대 시간을 줄여야 한다

In this memo the (author) vice president of Alta Manufacturing (AM) recommends that to reduce on-the-job accidents and increase productivity AM should shorten its three work shifts by one hour so that employees can

GRE AWA John박 박정어학원

get more sleep To support this recommendation the author provides several evidences However careful scrutiny of each of the facts reveals that it provides little credible support for the authorrsquos recommendation QuestionsThe number of accidents What kind of accidents The seriousness of the accidents is importantHow many employees are in each company What are their productsFalse cause Sleep may not be the reason for the on-the-job accidents What do Alta and Panoply manufacture

First of all the author believes that fatigue caused the on-the job accidents However there could be other reasons The author observes a correlation between sleep deprivation and on-the-job accidents then concludes that the former is the cause of the latter However the author fails to rule out other possible explanations For example it is entirely possible that Alta factories require more strenuous and dangerous labor than Panoply Without ruling out all other such factors it is unfair to conclude that fatigue is responsible for the accidents In addition the work-shifts may not be the cause of the sleep deprivation and fatigue It is possiblehellip Thus the author should provide what exactly Panoply and Alta manufacture and more precise data about their working conditions to be more convincing

Shortening the shift by one hour does not necessarily lead to more sleep And is one hour enoughLess accidents does not mean increased productivity

결론 We should shorten each of out three work shifts by one hour

반박 경쟁사에 비해서 시간당 하는 업무량이 많아서 더욱 피곤할 수도 있다 시간이 문제가 아니라 노후된 시설 설비 자체의 문제 작업 자체가 원래 위험한 것이여서 사고가 많을 수도 있다 다른 회사는 더욱 많은 작업시간에도 불구하고 안정한 작업여건으로 인해서 사고율이 오히려 더 작을 수도 있다비교사의 재해감소가 다른 요인일수 있다(안전 교육 철저)줄인 시간이 피로회복이나 수면으로 연결 안될수 있음(술을 마실 수도 있고 그 시간에 휴식을 취하지 않고 다른일을 함으로써 더욱 피로해질수 있다)

This editorial recommends that Alta Manufacturing reduce its work shifts by one hour each inorder to reduce its on-the-job accident rate and thereby increase Altas productivity To supportthis recommendation the author points out that last year the number of accidents at Alta was30 greater than at Panoply Industries where work shifts were one hour shorter The authoralso cites certain experts who believe that many on-the-job accidents are caused by fatigueand sleep deprivation I find this the argument unconvincing for several reasonsFirst and foremost the author provides absolutely no evidence that overall workerproductivity is attributable in part to the number of on-the-job accidents Although commonsense informs me that such a relationship exists the author must provide some evidence ofthis cause-and-effect relationship before I can accept the authors final conclusion that theproposed course of action would in fact increase Altas productivitySecondly the author assumes that some accidents at Alta are caused by fatigue or sleepdeprivation However the author overlooks other possible causes such as inadequateequipment maintenance or worker training or the inherent hazards of Altas manufacturingprocesses By the same token Panoplys comparatively low accident rate might be attributablenot to the length of its work shifts but rather to other factors such as superior equipmentmaintenance or worker training In other words without ruling out alternative causes ofon-the-job accidents at both companies the author cannot justifmbly conclude that merely byemulating Panoplys work-shift policy Alta would reduce the number of such accidentsThirdly even assuming that Altas workers are fatigued or sleep-deprived and that this is thecause of some of Altas on-the-job accidents in order to accept the authors solution to thisproblem we must assume that Altas workers would use the additional hour of free time tosleep or rest However the author provides no evidence that they would use the time in thismanner It is entirely possible that Altas workers would use that extra hour to engage in someother fatiguing activity Without ruling out this possibility the author cannot convincinglyconclude that reducing Altas work shifts by one hour would reduce Altas accident rateFinally a series of problems with the argument arise from the scant statistical information onwhich it relies In comparing the number of accidents at Alta and Panoply the author fails toconsider that the per-worker accident rate might reveal that Alta is actually safer than Panoplydepending on the total number of workers at each company Second perhaps accident rates

GRE AWA John박 박정어학원

at the two companies last year were aberrations and during other years Altas accident ratewas no greater or even lower than Panoplys rate Or perhaps Panoply is not representativeof industrial companies generally and that other companies with shorter work shifts have evenhigher accident rates In short since the argument relies on very limited statistical information Icannot take the authors recommendation seriouslyIn conclusion the recommendation for emulating Panoplys work-shift policy is not wellsupported To convince me that shorter work shifts would reduce Altas on-the-job accidentrate the author must provide clear evidence that work-shift length is responsible for some ofAltas accidents The author must also supply evidence to support her final conclusion that alower accident rate would in fact increase overall worker productivity

The following appeared in a memo from the vice president of marketing at Dura-Sock Inc

A recent study of our customers suggests that our company is wasting the money it spends on its patented Endure manufacturing process which ensures that our socks are strong enough to last for two years We have always advertised our use of the Endure process but the new study shows that despite our socks durability our average customer actually purchases new Dura-Socks every three months Furthermore our customers surveyed in our largest market northeastern United States cities say that they most value Dura-Socks stylish appearance and availability in many colors These findings suggest that we can increase our profits by discontinuing use of the Endure manufacturing process

1 Write a response in which you examine the stated andor unstated assumptions of the argument Be sure to explain how the argument depends on these assumptions and what the implications are for the argument if the assumptions prove unwarranted

2 Write a response in which you discuss what specific evidence is needed to evaluate the argument and explain how the evidence would weaken or strengthen the argument

3 Write a response in which you discuss what questions would need to be answered in order to decide whether the recommendation and the argument on which it is based are reasonable Be sure to explain how the answers to these questions would help to evaluate the recommendation

Intro The vice president of marketing at Dura-Sock Inc is offering a potentially harmful investment recommendation by claiming that Dura-Sock should discontinue its use of the ldquoEndurerdquo process To support his recommendation he points out a study that Dura-Sock customers actually purchase the socks every three months and a survey that reveals that Dura-Sock customers like the sockrsquos stylish appearance and availability in many colors The study and survey however are insufficient in supporting his proposal and the VP makes several unwarranted assumptionsIntro (simplified) The VP states that though Dura-Socks last for two years customers buy the socks every three months Therefore he assumes that the consumersrsquo motive for buying the produce is not its durabilityHowever the author fails to rule out other possible motivation for consumption

Even if the survey is reliable the author should consider the rest of the market Vague terms ldquowasting moneyrdquomdashprecisely how much are they wasting Studysurvey errorThe company must calculate the outcome of such momentous decisionStudy participantsrsquo comment that they prefer Dura-Sock for its stylishness and availability might take Dura-Sockrsquos enduring quality for granted

우리회사 제품 소비자들에 대한 최근 조사에서 지난 2년여간 양말의 내구성을 강하게 하는 필수공정이었던 자사 특허의 Endure 공정에 들어가는 비용이 낭비라고 말하고 있다 우리 회사는 항상 이 공정 처리에 대한 광고를 내보냈으나 이에 대한 시장 조사에서 실제로 고객들은 이 신제품을 평균 석달마다 구매하는 것으로 나타났다 더군다나 북동부지역에서 실시한 대규모 시장조사에 응답한 고객들은 양말의 모양과 색상등에 더

GRE AWA John박 박정어학원

관심을 나타냈다 이러한 결과는 우리회사가 신기술 공법을 중단하면 그에 따라 수익이 늘어날 것이라는 것을 말해주고 있는 것이다주장 These findings suggest that Dura0Sock can increase its profits by discontinuing its use of the ldquoEndurerdquo manufacturing process

1 survey가 정확한 소비자의 의견을 나타낸 것인가 다른 선택없이 양자택일과 같은 방법의 survey였는지2 북동부지역의 시장조사가 전체 의견을 대표할 수 있나3 사람들이 모양이나 색상에 앞서 내구성을 먼저 평가했을 수도 있다 내구성을 갖추었다는 전제하에 모양과 색상에 관심을 드러낸 것일 수 있다4 소비자가 도매상(retail)인지 소매상(whole)인지가 없다

The following appeared in a business magazine

As a result of numerous complaints of dizziness and nausea on the part of consumers of Promofoods tuna the company requested that eight million cans of its tuna be returned for testing Promofoods concluded that the canned tuna did not after all pose a health risk This conclusion is based on tests performed on samples of the recalled cans by chemists from Promofoods the chemists found that of the eight food chemicals most commonly blamed for causing symptoms of dizziness and nausea five were not found in any of the tested cans The chemists did find small amounts of the three remaining suspected chemicals but pointed out that these occur naturally in all canned foods

Write a response in which you discuss what questions would need to be addressed in order to decide whether the conclusion and the argument on which it is based are reasonable Be sure to explain how the answers to the questions would help to evaluate the conclusion

Representativeness of the tested cansThey should conduct a comparative studyThe testing could be biased because Promofoods employees conducted the testingHow much (quantity) of the five and three suspected chemicals were in the canned foodsFalse cause The substance that caused dizziness and nausea may not be one of the eight common chemicals

많은 소비자들의 현기증과 구역질 불만에 따라 Promofoods사는 지난해 참치 캔 8백만 개를 테스트하기 위해 반품시켰다 그 결과 캔에서는 건강에 위험이 될 수 있는 화합물질이 없었던 것으로 회사측은 결론지었다 이러한 결론은 회사측 화학연구자들이 회수된 캔의 샘플을 테스트해서 이들 증상의 원인이 되는 8가지 화합물 중에서 5가지가 실험된 캔에서 발견되지 않았다는 사실에 근거한 것이다 이들 화학자들은 나머지 3개가지 화합물이 모든 캔 식료품에서 흔히 발견되는 것이라고 언급했다 결론 Promofoods concluded that the cans did not after all contain chemicals that posed a health risk

1 공인된 기간에서 테스트를 한 것이 아니고 자사에서 직접 테스트를 했기에 신뢰성이 안간다 2 이런 증상을 일으키는 8개의 물질 말고 다른 물질들이 캔속에 많이 포함됬을수 있다 3 나머지 3개의 물질들의 함유량이 많아서 다른 종류의 캔들은 문제를 일으키지 않지만 참치캔은 문제를

일으킬 수 있다

This magazine article concludes that the 8 million cans of tuna Promofoods recalled due tocomplaints about nausea and dizziness do not after ail contain any chemicals that pose a

GRE AWA John박 박정어학원

health risk To support this conclusion the author cites the fact that five of eight chemicalscommonly causing these symptoms were not found in the recalled cans while the other threealso occur naturally in other canned foods For several reasons this evidence lends littlecredible support to the authors conclusionTo begin with the author relies partly on the fact that although three of the eight chemicalsmost commonly blamed for nausea and dizziness appeared in Promofoods recalled tunathese chemicals also occur naturally in other canned foods However this fact alone lends nosupport to the authors conclusion for two reasons First the author might be ignoring animportant distinction between naturally occurring chemicals and those not occurring naturallyIt is entirely possible that these three chemicals do not occur naturally in Promofoods tunaand that it is for this reason that the chemicals cause nausea and dizziness Secondly it isentirely possible that even when they occur naturally these chemicals cause the samesymptoms Unless the author rules out both possibilities he cannot reliably conclude that therecalled tuna would not cause these symptomsAnother problem with the argument is that the authors conclusion is too broad Based onevidence about certain chemicals that might cause two particular heath-related symptoms theauthor concludes that the recalled tuna contains no chemicals that pose a health risk Howeverthe author fails to account for the myriad of other possible health risks that the recalled tunamight potentially pose Without ruling out all other such risks the author cannot justifiablyreach his conclusionA third problem with the argument involves that fact that the eight particular chemicals withwhich the test was concerned are only the eight most commonly blamed for nausea anddizziness It is entirely possibly that other chemicals might also cause these symptoms andthat one or more of these other chemicals actually caused the symptoms Without ruling outthis possibility the author cannot jusufiably conclude that the recalled tuna would not causenausea and dizzinessA final problem with the argument involves thetesting procedure itself The author providesno information about the number of recaUed cans tested or the selection method used Unlessthe number of cans is a sufficiently large sample and is statistically repre sentative of all therecalled cans the studys results are not statistically reliableIn conclusion the article is unconvincing as it stands To strengthen the assertion that therecalled tuna would not cause nausea and dizziness the author must provide evidence thatthe three chemicals mentioned that occur naturally in other canned foods also appear naturallyin Promofoods tuna The author must also provide evidence that ingesting other canned foodscontaining these three chemicals does not cause these symptoms To better evaluate theargument we would need to know whether the sample used in the tests was statisticallysignificant and representative of all the recalled tuna We would also need to know what otherchemicals in the recalled tuna might pose any health risk at all

5그룹 불충분 조건오류 빈출

Natures Way a chain of stores selling health food and other health-related products is opening its next franchise in the town of Plainsville The store should prove to be very successful Natures Way franchises tend to be most profitable in areas where residents lead healthy lives and clearly Plainsville is such an area Plainsville merchants report that sales of running shoes and exercise clothing are at all-time highs The local health club has more members than ever and the weight training and aerobics classes are always full Finally Plainsvilles schoolchildren represent a new generation of potential customers these schoolchildren are required to participate in a fitness-for-life program which emphasizes the benefits of regular exercise at an early age

Write a response in which you examine the stated andor unstated assumptions of the argument Be sure to

GRE AWA John박 박정어학원

explain how the argument depends on these assumptions and what the implications are for the argument if the assumptions prove unwarranted

False cause

First of all the author believes that the Increased sales of running shoes and exercise clothing indicates

Plainesville residentsrsquo interest in leading healthy lives However this assumption is not logically convincing for

several reasons could be a fashion trendTime shift ldquoFitness for liferdquo might not have any influence on schoolchildren as they growFalse cause There could be other reasons for member increase in the health clubAll of the above are insufficient condition

The author has to prove that local residents are interested in leading healthy lives However he supports his conclusion with insufficient evidence Nevertheless even if the residents are concerned with health naturersquos way may not be successful First

그 동안의 경험을 토대로 볼 때 건강생활과 밀접히 관련되어 있는 거주 지역에서 본 상점들이 아주 호응을 얻고 있다 따라서 이러한 주민들이 많이 거주하고 있는 Plainsville 에 새로운 상점들을 계속 세워야 한다 이 지역 상인들은 런닝화와 운동복 판매가 가장 높다고 말한다 불과 5 년전에는 거의 전무하다시피하던 지역 헬스 클럽의 경우도 엄청나게 많은 회원을 확보하고 있으며 웨이트 트레이닝과 에어로빅 강좌들도 항상 만원이라고 한다 새로운 고객층을 예측해 보는 것도 가능하다 이 지역의 학생들의 경우 Fitness for Life프로그램을 받게 되는데 이러한 프로그램을 통해서 유년시절부터 정규적인 운동 습관을 들이게 하고 있는 것이 그것이다

결론 We should therefore build our next new store in Plainsville

반박 그동안의 경험에 의한 과거 통계가 꼭 여기에도 적용되는건 아니다 5 년전 헬스 클럽이 잘 안되었던게 다른 원인이였을수 있다(강사수준 미달 강좌미비)tourist 에 의한 원인 일수 있다 어렸을때부터 운동을 했다고 해서 커서도 관심이 있지는 않다 (오히려 반감이 있을수 있다 혹은 건강하기에 건강에 관심이 적을수도 있다)운동복이나 신발의 판매가 육체노동에 의한 것일수도 있다

IntroductionSupport1049896In this memorandum the author asserts that Naturersquos Way should build its next newstore in Plainsville To support this assertion the author states that Plainsvillesmerchantsrsquo sales of exercise clothing are going well the local health club has moremembers than ever and a new generation of customers will help to ensure NaturersquosWayrsquos success At first glance the authorrsquos assumption seems convincing but in-depth scrutiny revealsthat it lacks substantial evidence as it stands

Body 1-SamplingTopic Sentence 1To begin with the author assumes that the merchantsrsquo report indicates that the residentsare concerned about their health However this assumption is based on unsubstantiated

GRE AWA John박 박정어학원

data Example 1 (Rebuttal1) First if we do not know the total volume of items sold and the price of the goods exactly we cannot infer whether the residents are actually buying many goods Example 2 (Rebuttal2)In addition to that the report emphasizes the rising sales of running shoes and exerciseclothing however these may not be hot-selling items for Naturersquos Way or may not be theproducts the company is planning to sell Concluding Sentence Therefore in order to make the argument reliable the author should reconsider themerchantsrsquo report with more detailed data

Body 2-CausalTopic Sentence 2Second the author contends that the health clubs classes are full yet this does not meanthat many people actually use the health club other factors may be the real cause forthose closed classes Example 1 (Rebuttal1) To begin with if the health club is very small the number of people working out wouldnot be a large one In fact regular gym-going may just be a vogue among a smallunrepresentative segment of Plainsvilles population Example 2 (Rebuttal2) Moreover it is possible that most of the people who exercise in the health club do weight training and aerobics only to look good and to meet other singles not for their health In that case there would be little demand for health products Concluding SentenceThus the author should not hasten to presume what really caused people to be interested in a healthier lifestyle and enroll in the health club

Body 3-Time-ShiftTopic Sentence 3Finally the author highlights that Naturersquos Way can expect a new generation of customersin Plainsville that will help the company in the long term This notion is mistaken in that itassumes the conditions of the present will continue unchanged in the future Although theschool children are required to participate in the fitness for life program they may notnecessarily buy Naturersquos Ways products Example 1 (Rebuttal1) In the first instance they may suffer a fall in purchasing power arising from future economic difficulties this would cause reluctance to spend a considerable amount of money on health products which tend to be more expensive Example 2 (Rebuttal2)Another possibility is that there may emerge many competitor companies vying with Naturersquos Way so that in the future the school children may not feel the necessity to purchase one companyrsquos health products over anotherrsquosConcluding Sentence Thus the authorrsquos assumption is highly speculative since it relies heavily on unknowablefuture circumstances

ConclusionThesis In sum the author uses many assumptions that are insufficient in supporting his claimsSupportIn order for the authorrsquos claims to be convincing he needs to advance more persuasiveevidence that people in Plainsville really are concerned with their health and health foodThe following was written as a part of an application for a small-business loan by a group of developers in the city of Monroe

A jazz music club in Monroe would be a tremendously profitable enterprise Currently the nearest jazz club is 65 miles away thus the proposed new jazz club in Monroe the C-Note would have the local market all to itself Plus jazz is extremely popular in Monroe over 100000 people attended Monroes annual jazz festival last summer several well-known jazz musicians live in Monroe and the highest-rated radio program in Monroe is Jazz Nightly which airs every weeknight at 7 PM Finally a nationwide study indicates that the typical jazz fan spends close to $1000 per year on jazz entertainment

1 Write a response in which you discuss what specific evidence is needed to evaluate the argument and explain how the evidence would weaken or strengthen the argument

2 Write a response in which you examine the stated andor unstated assumptions of the argument Be

GRE AWA John박 박정어학원

sure to explain how the argument depends on these assumptions and what the implications are for the argument if the assumptions prove unwarranted

3 Write a response in which you discuss what questions would need to be answered in order to decide whether the prediction and the argument on which it is based are reasonable Be sure to explain how the answers to these questions would help to evaluate the prediction

Group error nationwide survey may not reflect local trends Is the nationwide jazz fan population substantialInsufficient non-residents of Monroe may have attended the jazz festival (Body alternative explanation last year may have been an anomaly The author should consider data from various years) The author should indicate how many out of 100000 were Monroe residentsNationwide study Does this reflect Insufficient Citizens of Monroe may continue to go to the jazz club 65 miles away

Are the people in Monroe really interested in jazzMajority of the people who attended the jazz festival might not be Monroe residentsSurvey error nationwide study may not be applicable to MonroeJazz musicians who live in MonroeMonopolyRadio station

In this business application the author claims that the proposed jazz club C Note will be very profitable in Monroe To support this claim the author argues for his case with several evidences At first glance the authorrsquos argument seems convincing however careful scrutiny reveals that his argument in specious

To begin with the author claims that Monroersquos citizens are interested in jazz He presents three evidences First Secondhellip Thirdhellip Howeverhellip

Monroe 시에 있는 재즈 음악 클럽은 수익성이 좋은 사업이다 현재 가장 가까이에 있는 클럽은 65 마일 정도 떨어져 있다 따라서 이번에 세우려고 하는 C Note 는 독보적인 위치를 점할것이다 더군다나 재즈는 이 시에서 가장 인기있는 음악이다 지난 여름 재즈 축제에서는 10 만명 이상의 Morone 시 주민이 참석하였고 몇몇 유명한 재즈 음악가들도 이곳에 살고 있으며 저녁때 방영되는 라디오 프로그램중에서 최고의 시청률을 보이고 있는 것도 Jazz Nightly 이다 전국조사에서도 전형적인 재즈 팬들은 재즈 분야에 년간 1천 달러 가까이 지출하고 있는 것으로 보고되고 있다 따라서 C Note 클럽이 돈을 벌 수 있는 사업이라는 것은 확실한 것이다

결과 It is clear that the C Note cannot help but make money반박 nearest jazz club 이 양질의 써비스로 여전히 손님을 끌수도 있다Festival 에 얼마나 참여하는지가 jazz 의 인기를 반영하지 않는다 뮤지션이 많이 사는거랑 jazz 의 인기가 상관없다라디오 프로그램이 다른 요인에 의해서 인기일수도 있다 (진행자때문)전국 통계 적용 불가화목 실전반_Ms Noh6In this application the author suggests that a jazz club in Monroe will make a number of profits To support this suggestion the author exemplifies the local condition popularity of jazz in Monroe and nationwide study However careful scrutiny of each of the facts reveals that it provides little credible support for the authorrsquos recommendation Good clear intro

First the author assumes that jazz is popular in Monroe because of several facts the jazz festival last year had high participation some famous jazz musicians live in Monroe and the high-rated radio program is lsquoJazz Nightlyrsquo However this assumption has many drawbacks that must be seriously considered(Good topic sentences) If many attendants in the last-yearrsquos festival came from other cities and not Monroe it is hard to conclude that Monroersquos people like jazz Therefore the author must examine how many Monroe residents actually attended the festival On top of that there is little relationship between habitation of famous jazz musician and the popularity of jazz in Monroe Although several well-known musicians live there if they do not take part in any jazz performance of Monroe this might have no effect to the interest of Monroersquos residents

GRE AWA John박 박정어학원

about jazz Finally in the case of radio program this is also not suitable reason why jazz is popular in Monroe It might be possible that people cannot help choosing lsquoJazz Nightlyrsquo because there are few radio programs at Night The fact that the radio program is the highest rating program is not a germane evidence The approximate number of listeners would be the more crucial evidence Therefore the author needs to seriously deliberate the correlation between jazzrsquos popularity in Monroe and his examples (Good logical flow and clarity)

Second the author uses as evidence the nationwide study that jazz fans spend much money on jazz entertainment to substantiate why starting a jazz club in Monroe will be profitable In other words the author assumes that the characteristics of a nationwide study can be applied to Monroe The national study would lend support to the applicantrsquos claim only if residents in Monroe typify national jazz fans However the author does not provide credible evidence that this is the case Moreover the populations of jazz fans nationwide may be insubstantial Thus the author should not infer hastily that Monroersquos residents will spend much money on enjoying jazz from the nationwide study

Lastly even if jazz is popular in Monroe C Note may not be successful It is entirely possible that residents might still prefer other clubs where they have always went In addition there is another possibility that the nearest jazz club will attract many of Monroersquos people because it serves fine performances and is equipped with favorite facilities Without considering these other possibilities the author cannot make his argument convincing In sum the author presents many reasons that are insufficient in supporting his or her claim In order for the authorrsquos claims to be convincing he needs to advance more persuasive evidence such as the total number of Monroe residents who attended the jazz festival the effects on the popularity of jazz by the musicians living in Monroe and the actual number of residents who would typify themselves to be jazz fans through a local survey Without substantial evidence that C Note will be successful in Monroe the businessmen may be overinvesting in what might lead to a business failureExcellent clarity Score 50

The following appeared in a newsletter offering advice to investors

Over 80 percent of the respondents to a recent survey indicated a desire to reduce their intake of foods containing fats and cholesterol and today low-fat products abound in many food stores Since many of the food products currently marketed by Old Dairy Industries are high in fat and cholesterol the companys sales are likely to diminish greatly and company profits will no doubt decrease We therefore advise Old Dairy stockholders to sell their shares and other investors not to purchase stock in this company

Write a response in which you discuss what questions would need to be answered in order to decide whether the advice and the argument on which it is based are reasonable Be sure to explain how the answers to these questions would help to evaluate the advice

Survey 80

GRE AWA John박 박정어학원

Old Dairy could change their products and manufacture low fat dairy foodsLess competing companies Old Dairy could eventually be the only company that produces hellipImprecise numbers and measurementsCustomers may still buy high fat dairy products

The author of the newsletter is offering potentially dangerous advice by recommending Old Dairy stockholders to withdraw investment and stop purchase What is more the authorrsquos prediction debases the reputation and business of Old Dairy and if false could devoid the investment opportunity of the newsletter readers Therefore investors should examine whether the authorrsquos evidences are substantial

To begin with the author states that 80 percent of the respondents in a survey indicated a desire to reduce their intake of foods He therefore argues that Old Dairyrsquos high fat and cholesterol products would decrease in sales However the author makes a crucial error in this argument First the author provides no evidence that the surveyrsquos results are statistically reliable Were they representative of all the customers Were they chosen for the survey randomly Furthermore the desire to reduce fat and cholesterol intake is a pervasive trend in todayrsquos opulent society however the author erroneously identifies this as a new phenomenon which will affect consumer trends Second having a desire to reduce fat and cholesterol intake does not necessarily indicate that people who have this desire will actually reduce consuming these types of products It is entirely possible that they may continue buying Old Dairy products for its quality and taste Accordingly the author cannot draw any firm conclusion that people will not buy Old Dairy products Therefore if any of these cases are true the author may be offering investors a detrimental investment advice

최근 조사에 대한 응답자중 80 이상이 자신이 먹는 음식에서 지방과 콜레스테롤의 함유량을 줄이고 싶다고 한다 아울러 요즘은 많은 식료품 가계에서 저지방 제품들을 많이 취급하고 있다 현재 Old Dairy Industries가 판매하고 있는 많은 음식제품들은 지방과 콜레스테롤이 높기 때문에 이 회사의 매출이 격감할 것으로 보이며 당연히 매출이익도 줄어들것이다 따라서 이 회사의 주주들은 주식을 매각하고 다른 주식 투자가들도 이 회사의 주식을 매입하지 않는 것이 좋다

결론 Old Dairy stockholders to sell their shares and other investors not to purchase stock in this company

반박 모든 상품이 다 고 지방 고 칼로리는 아니다(비록 많을지라도) 일부의 식품의 경우 기호에 맞어서 히트해서 전체적인 수입이 증가할 수도 있다국내시장만 생각할 수 없다( 외국시장에서 호황을 누릴수 있다 )입맛이라는게 즉각 바뀌는게 아니다

The following appeared in a letter to the editor of the Balmer Island Gazette

On Balmer Island where mopeds serve as a popular form of transportation the population increases to 100000 during the summer months To reduce the number of accidents involving mopeds and pedestrians the town council of Balmer Island should limit the number of mopeds rented by the islands moped rental companies from 50 per day to 25 per day during the summer season By limiting the number of rentals the town council will attain the 50 percent annual reduction in moped accidents that was achieved last year on the neighboring island of Seaville when Seavilles town council enforced similar limits on moped rentals

1 Write a response in which you discuss what questions would need to be answered in order to decide whether the recommendation is likely to have the predicted result Be sure to explain how the answers to these questions would help to evaluate the recommendation

2 Write a response in which you discuss what questions would need to be answered in order to decide whether the prediction and the argument on which it is based are reasonable Be sure to explain how the answers to these questions would help to evaluate the prediction

3 Write a response in which you examine the stated andor unstated assumptions of the argument Be sure to explain how the argument depends on these assumptions and what the implications are for the argument if the assumptions prove unwarranted

Whatrsquos the actual population of Balmer Island 100000mdashis this a significant increase What kind of accidents Skin abrasions or serious injury And compared to Seaville how serious are the accidents and the actual number of accidents Did Seaville enforce other restrictions like safety signsHow different are the conditions of Balmer

GRE AWA John박 박정어학원

and Seaville regarding population road (safety) conditions topography other town-government regulation How much will the economy of Balmer be affected do to this restriction Could it cause an economic recession due to the fact that these rental companiesrsquo chance to make money is only during the summer thereby weakening the economic infrastructure Are there any other ways that could better alleviate the accident rate

Statistics 50-impreciseAnalogy Balmer compared with TorseauFalse Cause Accidents might have occurred because of reasons other than mopeds False Cause population increase may not be part of the cause of the accidentsOther explanations for the accident pedestrians few road safety regulations narrow roadsThere could be other better solutionshellip

Balmer Island의 인구가 여름철에는 십만명으로 늘어난다 2륜차와 보행자간 사고를 줄이기 위해 시의회는 6개의 자전거를 포함한 2륜차 대여업체에게 이 기간동안에는 대여숫자를 일일 50에서 30으로 제한하도록 할 것이다 대여숫자를 줄임으로써 시 의회는 지난해 이웃한 Torseau섬에서 이와 동일한 규제를 시행해서 50나 줄인 결과를 보고 마찬가지로 50를 줄일수 있다고 확신하고 있다

결론 The town council of Balmer Island should linit the number

반박 보행자의 부실에 의해서 사고가 많이 일어날수도 있다렌탈수의 줄임만이 대책은 아니다(대부분의 사람들이 렌탈 보다는 소유하고 있을 수도 있다)옆섬과는 상황이 다를수도 있다(그 섬에서는 사고의 원인이 많은 자전거 수로 인한것일수있다) 하지만 이 섬은 좁은 도로가 원인일 수도 있고 도로 안전 장치의미비가 원일일수 있다

In this letter the author recommends that Balmer Island should limit the number moped rentals from 50 to 30 per day To support this recommendation the author points out several reasons However careful scrutiny of each of the facts reveals that it is filled with unanswered questions that could significantly weaken the authorrsquos recommendation with loops and holes which are answered

The recommendation depends on the assumption that no alternative means of reducing the number of accidents are available However the author fails to offer any evidence to substantiate this crucial assumption It is highly possible that means other than this would better solve the problem Perhaps they could widen the roads or put-up more safety signs Or perhaps the accidents were due to the lack of skills in which case proper safety training would significantly alleviate the problem Without considering and ruling out these and other alternative means of reducing accidetns the author cannot confidently conclude that merely emulating Torseau would suffice Moreover the author is advising a recommendation which could potentially harm the economy of Balmer Island sincehellip Moreover the Balmer Island should alternative means to reduce accidents because limiting moped rentals during the summer could harm the economy of Balmerhellip

First of all the author believes that increase in population and the number of moped rentals are responsible for the accidents It is entirely possible that other factors are responsible for the accidents Perhaps Balmer Islandrsquos lack of safety signs was a major factor Or maybe the roads are narrow and dangerous on the Island therefore the town council could enforce stricter traffic regulations to alleviate the problem Accordingly if either of these scenarios is true the author cannot draw any firm conclusion that increase in the number of population and moped rentals are the cause of the accidents

The author of this editorial recommends that to reduce accidents involving mopeds andpedestrians Balmer Islands city council should restrict moped rentals to 30 per day down from50 at each of the islands six rental outlets To support this recommendation the author citesthe fact that last year when nearby Torseau Islands town council enforced similar measuresTorseaus rate of moped accidents fell by 50 For several reasons this evidence providesscant support for the authors recommendationTo begin with the author assumes that all other conditions in Balmer that might affect therate of moped-pedestrian accidents will remain unchanged after the restrictions are enactedHowever with a restricted supply of rental mopeds people in Balmer might purchase mopedsinstead Also the number of pedestrians might increase in the future with more pedestriansespecially tourists the risk of moped-pedestrian accidents would probably increase For thatmatter the number of rental outlets might increase to make up for the artificial supplyrestriction per outlet--a likely scenario assuming moped rental demand does not declineWithout considering and ruling out these and other possible changes that might contribute to ahigh incidence of moped-pedestrian accidents the author cannot convince me that theproposed restrictions will necessarily have the desired effect

GRE AWA John박 박정어학원

Next the author fails to consider other possible explanations for the 50 decline inTorseaus moped accident rate last year Perhaps last year Torseau experienced unusually fairweather during which moped accidents are less likely Perhaps fewer tourists visited Tot seanlast year than during most years thereby diminishing the demand for rental mopeds to belowthe allowed limits Perhaps last year some of Torseaus moped rental outlets purchased newmopeds that are safer to drive Or perhaps the restrictions were already in effect but were notenforced until last year In any event a decline in Torseaus moped accident rate during onlyone year is scarcely sufficient to draw any reliable conclusions about what might have causedthe decline or about what the accident rate will be in years aheadAdditionally in asserting that the same phenomenon that caused a 50 decline in mopedaccidents in Torseau would cause a similar decline in Balmer the author relies on what mightamount to an unfair analogy between Balmer and Torseau Perhaps Balmers ability to enforcemoped-rental restrictions does not meet Torseaus ability if not then the mere enactment ofsimilar restrictions in Balmer is no guarantee of a similar result Or perhaps the demand formopeds in Torseau is always greater than in Balmer Specifically if fewer than all availablemopeds are currently rented per day from the average Balmer outlet while in Torseau everyavailable moped is rented each day then the proposed restriction is likely to have less impacton the accident rate in Balmer than in TorseauFinally the author provides no evidence that the same restrictions that served to reduce theincidence of all moped accidents by 50 would also serve to reduce the incidence ofaccidents involving mopeds and pedestrians by 50 Lacking such evidence it is entirelypossible that the number of moped accidents not involving pedestrians decreased by a greaterpercentage while the number of moped-pedestrian accidents decreased by a smallerpercentage or even increased Since the author has not accounted for these possibilities theeditorials recommendation cannot be taken seriouslyIn conclusion the recommendation is not well supported To convince me that the proposedrestriction would achieve the desired outcome the author would have to assure me that nochanges serving to increase Balmers moped-pedestrian accident rate will occur in theforeseeable future The author must also provide dear evidence that last years decline inmoped accidents in Torseau was attributable primarily to its moped rental restrictions ratherthan to one or more other factors In order to better evaluate the recommendation I wouldneed more information comparing the supply of and demand for moped rentals on the twoislands I would also need to know the rate of mopedpedestrian accidents in Torseau both priorto and after the restrictions were enforced in TorseauThe following appeared in a magazine article about planning for retirement

Clearview should be a top choice for anyone seeking a place to retire because it has spectacular natural beauty and a consistent climate Another advantage is that housing costs in Clearview have fallen significantly during the past year and taxes remain lower than those in neighboring towns Moreover Clearviews mayor promises many new programs to improve schools streets and public services And best of all retirees in Clearview can also expect excellent health care as they grow older since the number of physicians in the area is far greater than the national average

Write a response in which you discuss what specific evidence is needed to evaluate the argument and explain how the evidence would weaken or strengthen the argument

-Natural beauty and consistent climate may not be the most wanted qualities-Housing costs could have lowered on a national level wealthy retirees may not care about costs-Taxes may be high compared to the nationrsquos average tax rate-What about other qualities of Clearview Crime rate what qualities would retirees want -If schools streets and public services need improvement then this is proof that the current condition of Clearview is low Or due to budgetary reasons the mayor may not follow-up on his promise because of lowered tax rate -Schools and people who are retired no relationship-Physicians What kind of physicians Number is irrelevant Are these physicians capable of addressing the illnesses of old people

This author argues that anyone seeking a place to retire should choose Clearview To supportthis argument the article cites Clearviews consistent climate and natural beauty its fallinghousing costs its low property taxes compared to nearby towns and the mayors promise toimprove schools streets and services The article also claims that retirees can expectexcellent health care because the number of physicians in Clearview greatly exceeds thenational average This argument is flawed in several critical respectsTo begin with although consistent climate and natural beauty might be attractive to manyretirees these features are probably not important to all retirees For many retirees it isprobably more important to live near relatives or even to enjoy changing seasons Thus I

GRE AWA John박 박정어학원

cannot accept the authors sweeping recommendation for all retirees on this basisAlso Clearviews declining housing costs do not necessarily make Clearview the best placeto retire for two reasons First despite the decline Clearviews housing costs might be highcompared to housing costs in other cities Secondly for wealthier retirees housing costs arenot likely to be a factor in choosing a place to retire Thus the mere fact that housing costshave been in decline lends scant support to the recommendationThe articles reliance on Clearviews property-tax rates is also problematic in two respectsFirst retirees obviously have innumerable choices about where to retire besides Clear viewand nearby towns Secondly for retirees who are well-off financially property taxes are notlikely to be an important concern in choosing a place to retire Thus it is unfair to infer fromClearviews property-tax rates that retirees would prefer ClearviewYet another problem with the argument involves the mayors promises In light of Clearviewslow property-tax rates whether the mayor can follow through on those promises is highlyquestionable Absent any explanation of how the city can spend more money in the areas citedwithout raising property taxes I simply cannot accept the editorials recommendation on thebasis of those promises Besides even if the city makes the improvements promised thoseimprovements--particular the ones to schools--would not necessarily be important to retireesFinally although the number of physicians in Clearview is relatively high the per capitanumber might be relatively low Moreover it would be fairer to compare this per capita numberwith the per capita number for other attractive retirement towns--rather than the nationalaverage After all retirees are likely to place a relatively heavy burden on health-careresources Besides the article provides no assurances that the number of physicians inClearview will remain high in the foreseeable futureIn conclusion the recommendation is poorly supported To strengthen it the author mustconvince me--perhaps by way of a reliable survey--that the key features that the vast majorityof retirees look for in choosing a place to live are consistent climate natural beauty and lowhousing costs The author must also provide better evidence that Clear views property taxesare lower than the those of cities in other areas The author must also explain how the city canmake its promised improvements without raising property taxes Finally to better assess theargument I would need to now how the per capita number of physicians in Clearview wouldcompare to the national average in the futureThe following appeared as a letter to the editor from a Central Plaza store owner

Over the past two years the number of shoppers in Central Plaza has been steadily decreasing while the popularity of skateboarding has increased dramatically Many Central Plaza store owners believe that the decrease in their business is due to the number of skateboard users in the plaza There has also been a dramatic increase in the amount of litter and vandalism throughout the plaza Thus we recommend that the city prohibit skateboarding in Central Plaza If skateboarding is prohibited here we predict that business in Central Plaza will return to its previously high levels

Write a response in which you discuss what questions would need to be answered in order to decide whether the recommendation is likely to have the predicted result Be sure to explain how the answers to these questions would help to evaluate the recommendation

Why two years ago What happened two years ago which started this declineIs the dramatic increase in the ldquopopularityrdquo of skateboarding the cause of the steady decline of shoppers Are there any malls nearby Were there any changes nearby which could affect the decline in customersmdasha big mall perhaps Could the decline be due to the shop ownersHow many skateboarders use the plazaWhere do they skateboardDo they shop and are they customersAre the increase in litter and vandalism due to skateboarders Could this be alleviated by installing CCTVs and hiring security

This editorial concludes that the city should ban skateboarding from its downtown CentralPlaza in order to attract visitors to that area to return the area to its former glory and to makeit a place where people can congregate for fun and relaxation To justify this conclusion theeditorial points out that skateboarders are nearly the only people one sees anymore at CentralPlaza and that the Plaza is littered and its property defaced The editorial also points out thatthe majority of downtown merchants support the skate boarding ban This argument is flawedin several critical respectsFirst the editorials author falsely assumes that a ban on skateboarding is both necessaryand sufficient to achieve the three stated objectives Perhaps the city can achieve thoseobjectives by other means as well--for example by creating a new mall that incorporates anattractive new skateboard park Even if banning skateboarders altogether is necessary to meetthe citys goals the author has not shown that this action by itself would suffice Assuming thatthe Plazas reputation is now tarnished restoring that reputation and in turn enticing peopleback to the Plaza might require additional measures--such as removing litter and graffiti

GRE AWA John박 박정어학원

promoting the Plaza to the public or enticing popular restaurant or retail chains to the PlazaSecondly the editorial assumes too hastily that the Plazas decline is attributable to theskateboarders--rather than to some other phenomenon Perhaps the Plazas primary appeal inits glory days had to do with particular shops or eateries which were eventually replaced byless appealing ones Or perhaps the crime rate in surrounding areas has risen dramatically forreasons unrelated to the skateboarders presence at the Plaza Without ruling out these andother alternative explanations for the Plazas decline the editorials author cannot convince methat a skateboard ban would reverse that declineThirdly the editorials author might be confusing cause with effect--by assuming that theskateboarders caused the abandonment of the Plaza rather than vice versa It is entirelypossible that skateboarders did not frequent the Plaza until it was largely abandoned--andbecause it had been abandoned In fact this scenario makes good sense since skateboardingis most enjoyable where there are few pedestrians or motorists to get in the wayFourth it is unreasonable to infer from the mere fact that most merchants favor the ban thatthe ban would be effective in achieving the citys objectives Admittedly perhaps thesemerchants would be more likely to help dean up the Plaza area and promote their businesseswere the city to act in accordance with their preference Yet lacking any supporting evidencethe author cannot convince me of this Thus the survey amounts to scant evidence at best thatthe proposed ban would carry the intended resultFinally the author recommends a course of action that might actually defeat the citysobjective of providing a fun and relaxing place for people to congregate In my experienceskateboarding contributes to an atmosphere of fun and relaxation for adults and children alikemore so than many other types of ambiance Without considering that continuing to allowskateboarding--or even encouraging this activity--might achieve the citys goal more effectivelythan banning the activity the author cannot convincingly conclude that the ban would be in thecitys best interestsIn sum the argument is a specious one To strengthen it the editorials author must providedear evidence that skateboarding and not some other factor is responsible for the conditionsmarking the Plazas decline The author must also convince me that no alternative means ofrestoring the Plaza are available to the city and that the proposed ban by itself would suffice toattract tourists and restore the Plaza to its former glory Finally to better assess the argument itwould be useful to know the circumstances under which the downtown merchants would bewilling to help the city achieve its objectives

6그룹 약한 비유 빈출

The following recommendation appeared in a memo from the mayor of the town of Hopewell

Two years ago the nearby town of Ocean View built a new municipal golf course and resort hotel During the past two years tourism in Ocean View has increased new businesses have opened there and Ocean Views tax revenues have risen by 30 percent Therefore the best way to improve Hopewells economymdashand generate additional tax revenuesmdashis to build a golf course and resort hotel similar to those in Ocean View

Write a response in which you examine the stated andor unstated assumptions of the argument Be sure to explain how the argument depends on these assumptions and what the implications are for the argument if the assumptions prove unwarranted

GRE AWA John박 박정어학원

Assumptions The author assumes that OVrsquos municipal golf course and resort hotel caused tourism new businesses and increased tax revenues There may be other reasons advertising promo He assumes that this will continueAssumes that Ocean View and Hopewell are similar in many waysmdashthe name suggests otherwise OV may have always been a tourist attractions for its beaches We need to know the topography

2년전 Ocean View 시는 시정 소유 골프 및 휴양지 호텔을 신축했다 그리고 지난 2년동안 이 시의 관광객이 증가했으며 새로운 사업들이 생겨났다 그에따라 시의 세수도 30나 증가했다 Hopewell의 경제를 향상시키고 아울러 세수를 늘릴 수 있는 가장 좋은 방법은 Ocean View에 세워진 것과 같은 골프 시설과 휴양지 호텔을 신축하는 것이다

1 다른 요인으로 관광 산업이 발전했을 수도 있다 문화 유적이 발견이 되었거나 도로의 정비등으로 여행자가 늘었을 수도 있다

2 관광 산업의증가가 늘어난 세수의 원인이 아니라 새로 유입된 인구의 증가나 다른 공장에서 발생한 것일 수 있다

3 2년동안 한참 골프가 붐을 이루었을 수 있다 경제상황이 나빠지거나 다른 레포츠가 인근 지역에 생겨난다면 골프하는 사람이 줄어들 수 있다

In this memo HopeweUs mayor recommends that in order to stimulate the towns economyand boost tax revenues HopeweU should build a new golf course and resort hotel just as thetown of Ocean View did two years ago To support this recommendation the mayor points outthat in Ocean View during the last two years tourism has increased new businesses haveopened and tax revenues have increased by 30 I find the mayors argument unconvincingin several important respectsFirst of all it is possible that the mayor has confused cause with effect respecting the recentdevelopments in Ocean View Perhaps Ocean Views construction of a new golf course andhotel was a response to previous increases in tourism and business development increasesthat have simply continued during the most recent two years Since the mayor has failed toaccount for this possibility the claim that Hopewell would boost its economy by alsoconstructing a golf course and hotel is completely unwarrantedSecondly the mayor fails to account for other possible causes of the trends in Ocean Viewduring the last two years The increase in tourism might have been due to improving economicconditions nationwide or to unusually pleasant weather in the region The new businessesthat have opened in Ocean View might have opened there irrespective of the new golf courseand hotel And the 30 increase in tax revenues might have been the result of an increase intax rates or the addition of a new type of municipal taxWithout ruling out these and other alternative explanations for the three recent trends inOcean View the mayor cannot reasonably infer based on those trends that Hopewellseconomy would benefit by following Ocean Views exampleThirdly even if the recent trends in Ocean View are attributable to the construction of the newgolf course and hotel there the mayor assumes too hastily that the golf course and hotel willcontinue to benefit that towns overall economy The mayor has not accounted for thepossibility that increased tourism will begin to drive residents away during tourist season orthat new business development will result in the towns losing its appeal as a place to visit or tolive Unless the mayor can convince me that these scenarios are unlikely I cannot accept themayors recommendation that Hopewell follow Ocean Views exampleFinally the mayors argument rests on the unsubstantiated assumption that Hopewell andOcean View are sufficiently alike in ways that might affect the economic impact of a new golfcourse and hotel Hopewell might lack the sort of natural environment that would attract moretourists and new businesses to the town--regardless of its new golf course and hotel For thatmatter perhaps Hopewell already contains several resort hotels and golf courses that are notutilized to their capacity If so building yet another golf course and hotel might amount to amisallocation of the towns resources--and actually harm the towns overall economyIn sum the mayors recommendation is not well supported To bolster it the mayor mustprovide better evidence that Ocean Views new golf course and hotel and not some otherphenomenon--has been responsible for boosting Ocean Views economy during the last twoyears To better assess the recommendation I would need to know why Ocean View decidedto construct its new golf course and hotel in the first place--specifically what events prior toconstruction might have prompted that decision I would also need to thoroughly compare

GRE AWA John박 박정어학원

HopeweU with Ocean View--especially in terms of their appeal to tourists and businesses--todetermine whether the same course of action that appears to have boosted Ocean Viewseconomy would also boost Hopewells economy

The following is part of a memorandum from the president of Humana University

Last year the number of students who enrolled in online degree programs offered by nearby Omni University increased by 50 percent During the same year Omni showed a significant decrease from prior years in expenditures for dormitory and classroom space most likely because instruction in the online programs takes place via the Internet In contrast over the past three years enrollment at Humana University has failed to grow and the cost of maintaining buildings has increased along with our budget deficit To address these problems Humana University will begin immediately to create and actively promote online degree programs like those at Omni We predict that instituting these online degree programs will help Humana both increase its total enrollment and solve its budget problems

Write a response in which you discuss what questions would need to be answered in order to decide whether the prediction and the argument on which it is based are reasonable Be sure to explain how the answers to these questions would help to evaluate the prediction

Is Omni University successful due to the online degree program 50 Is the decrease in expenditures for dormitory and classroom space due to the decrease in of on-campus students Which classes were successful Does HU have those classes

Even if the long-distance degree programs at Omni University benefited the school the presidentrsquos recommendation that Human College should emulate Omni University is too hasty First OUrsquos name implies that the school would have more majors than Humanahellip the president should examine which degrees were in the long-distance programhellip

지난해에는 Omni 대학에서 개강했던 원거리 학생 학점 취득 프로그램을 등록했던 학생들의 숫자가 50나 증가했다 같은해 기간동안 Omni 대학에서는 그 전년도부터 기숙사와 학급의 공간 확충을 위한 예산을 대폭 줄였는데 이는 이 원거리 학점 취득 프로그램이 양방향 비디오 컴퓨터 접속을 통해서만 가능한 수업지도 방식이기때문인 것으로 보인다 반면 지난 3개년 동안 Humana 대학에서의 수강률은 감소한데다가 건물

GRE AWA John박 박정어학원

유지비도 올랐다 따라서 Humana대학의 수강을 늘리고 예산손실을 회복하기 위해서는 Omni 대학에서 취한 조치와 같은 능동적인 프로그램을 추진해야 한다

결론 we should initiate and actively promote long-distance degree programs like those at Omni 반박 원거리 학생 취득 프로그램 숫자가 증가한거하고 예산이 줄어드는 것 사이에 연관이 약하다 (causal 학생의 증가로 관리비용 증가할수 있음 원거리 수업가능 장비도입에의한 비용발생)bad analogy(omni university 하고 같은 조건이 아니다 )-gt omni college 가 강좌내용이 좋아서 학생의 등록이 많을수 있다 Humana 대학에서 만들었다 하더라도 인기 없을수 있음다른 요인에 의해서 Humana 대학의 수강 인원이 증가할수 있음(비록 과거엔 인기가 없었을지라도)

The following appeared as part of a business plan developed by the manager of the Rialto Movie Theater

Despite its downtown location the Rialto Movie Theater a local institution for five decades must make big changes or close its doors forever It should follow the example of the new Apex Theater in the mall outside of town When the Apex opened last year it featured a video arcade plush carpeting and seats and a state-of-the-art sound system Furthermore in a recent survey over 85 percent of respondents reported that the high price of newly released movies prevents them from going to the movies more than five times per year Thus if the Rialto intends to hold on to its share of a decreasing pool of moviegoers it must offer the same features as Apex

Write a response in which you discuss what questions would need to be answered in order to decide whether the recommendation is likely to have the predicted result Be sure to explain how the answers to these questions would help to evaluate the recommendation

Before following through this business plan the manager should investigate the cause of Rialtorsquos unsuccessful business

The author provides no evidence that the surveyrsquos results are statistically reliable The surveyrsquos sample of 85 percent must be sufficient in size and representative of overall population of the city where Rialto and Apex is serving Lacking evidence of a sufficiently representative sample the author cannot justifiably rely on the survey to draw any conclusion whatsoever The author does not indicate that Apex is indeed currently successful However even if Apex is enjoying success the argument relies on what might be a false analogy between Rialto and Apex In order for Apex to serve as a model that Rialto should emulate the author must assume that all relevant circumstances are essentially the same However this assumption is unwarranted For example the argument overlooks the face that Apex is located in a strategic placemdashbeside a mall where customers can not only watch a movie but also enjoy shopping Therefore simply changing the facility to that of Apex may not lead to success

The author does not mention whether Apex is successful or not Nevertheless even if Apex is currently successful the argument relies on what might be a false analogy between Rialto and Apex In order for Apex to serve as a model that Rialto should emulate the author must assume that all relevant circumstances are essentially the same However this assumption is unwarranted For example the argument overlooks the fact that these two institutions are located in different locations Rialto in downtown and Apex in a mall outside of town Although Apex opened with state-of-the-art facilities the decisive factor in its success could be due to its strategic location of being in a mall People could enjoy both shopping and movies at one location thus they may prefer Apex over Rialto Furthermore the place where people enjoy leisure activities has shifted in the past decades for most cities from downtown to the suburbs Therefore Rialto may not be successful even if it emulates Apexrsquos facilities A better business plan may be relocating Apex to the thriving section of the downtown

Rialto 극장은 지난 50여년간 지역 회관으로써 시내에 위치해 있으면서도 이제 변화를 꾀하지 않으면 문을 닫을

GRE AWA John박 박정어학원

판이다 이 극장은 시외 쇼핑타운에 새로 들어선 Apex 극장의 사례를 본받아야 했다 Apex가 지난해 개업했을 당시 이 극장은 비디오 아케이드 플러쉬 카펫트 바닥과 좌석 그리고 최신 음향시설을 갖추었다 더군다나 최근 조사에서는 응답자의 85 이상이 새로 출시된 영화 입장료가 비싼 탓으로 지난해보다 5배이상의 관람객이 줄어들었다고 나타났다 따라서 Rialto 극장이 줄어들고 있는 관람객을 뺐기지 않고 유지하려면 Apex와 같은 시설들을 갖추어야 할 것이다주장 리알토 극장이 줄어들고 있는 관람객을 뺐기지 않고 유지하려면 Apex와 같은 시설들을 갖추어야 할 것이다

1 조사에서 응답자가 전체를 대표할 수 없다 2 apex 극장이 좋은 시설을 갖추고 있지만 그로 인해 수익이 많이 발생했다는 말이 없으므로 시설투자를

하고도 좋은 결과를 얻을 수 있을지 그 근거가 미흡하다3 좋은 영화가 출시된다면 입장료가 비싸도 영화관에서 꼭 보려고 할 수 있다 4 rialto 가 시설이 아닌 다른 요인에 의해 장사가 안될수도 있다( 우범 지역이라든지)

The following is a recommendation from the business manager of Monarch Books

Since its opening in Collegeville twenty years ago Monarch Books has developed a large customer base due to its reader-friendly atmosphere and wide selection of books on all subjects Last month Book and Bean a combination bookstore and coffee shop announced its intention to open a Collegeville store Monarch Books should open its own in-store cafeacute in the space currently devoted to childrens books Given recent national census data indicating a significant decline in the percentage of the population under age ten sales of childrens books are likely to decline By replacing its childrens books section with a cafeacute Monarch Books can increase profits and ward off competition from Book and Bean

Write a response in which you examine the stated andor unstated assumptions of the argument Be sure to explain how the argument depends on these assumptions and what the implications are for the argument if the assumptions prove unwarranted

The following is a recommendation from the business manager of Monarch Books

Since its opening in Collegeville twenty years ago Monarch Books has developed a large customer base due to its reader-friendly atmosphere and wide selection of books on all subjects Last month Book and Bean a combination bookstore and coffee shop announced its intention to open a Collegeville store Monarch Books should open its own in-store cafeacute in the space currently devoted to childrens books Given recent national census data indicating a significant decline in the percentage of the population under age ten sales of childrens books are likely to decline By replacing its childrens books section with a cafeacute Monarch Books can increase profits and ward off competition from Book and Bean

1 Write a response in which you discuss what questions would need to be answered in order to decide whether the recommendation is likely to have the predicted result Be sure to explain how the answers to these questions would help to evaluate the recommendation

2 Write a response in which you discuss what specific evidence is needed to evaluate the argument and explain how the evidence would weaken or strengthen the argument

No evidence regarding Monarch Bookrsquos successEven if Regal Bookrsquos is successful this may not be attributable to the cafeacute False analogy Emulating may not lead to success Other factors may be involvedInsufficient condition The national census is not enough evidence that childrenrsquos book sales will decline Can

GRE AWA John박 박정어학원

the national census represent the local child populationDid opening a cafeacute boost sales for Regal Books Even assuming Regal is successful by opening a cafeacute this may not be suitable for Monarch which plans to close the childrenrsquos book section to establish a cafe Imprecise language ldquorelatively little spacerdquo how smallThe managerrsquos recommendation contradicts what he says Since Monarch is popular for its wide selection of books closing a selection which targets a major group of readers may hurt Monarchrsquos salesIs this the best way to compete

When Stanley Park first opened it was the largest most heavily used public park in town It is still the largest park but it is no longer heavily used Video cameras mounted in the parks parking lots last month revealed the parks drop in popularity the recordings showed an average of only 50 cars per day In contrast tiny Carlton Park in the heart of the business district is visited by more than 150 people on a typical weekday An obvious difference is that Carlton Park unlike Stanley Park provides ample seating Thus if Stanley Park is ever to be as popular with our citizens as Carlton Park the town will obviously need to provide more benches thereby converting some of the unused open areas into spaces suitable for socializing

Write a response in which you examine the stated andor unstated assumptions of the argument Be sure to explain how the argument depends on these assumptions and what the implications are for the argument if the assumptions prove unwarranted

Stanley 파크가 처음 개장했을 당시 가장 크고 가장 많이 이용되는 공원이었다 아직도 공원중에서는 가장 크지만 이용률은 상당히 떨어졌다 지난달 공원 주차장에 설치해놓은 비디오 카메라를 통해 보면 drop(주차장으로 여겨짐) 이용률이 가장 높았다 수치상으로는 하루 평균 50대의 차량만이 이용하였다 반면 직장 중심거리에 위치한 작은 규모의 Carlton 파크는 주당 무려 150여명 이상이 이용하고 있다 Stanley 파크와는 달리 Carlton 파크에는 의자가 있다는 것이 가장 뚜렷한 차이점이다 따라서 Stanley 파크가 Carlton 파크처럼 시민들이 자주 이용하는 공원이 되기 위해서는 벤치를 설치할 필요가 있으며 이렇게 사용되지 않는 일부 공간을 활용해서 사교를 위한 공간으로 바꾸어야 한다 ===gtdrop 에 대한 첨부사항 (영영사전내용입니다)---- a place or central depository to which something (as mail money or stolen property) is brought for distribution or transmission also the act of depositing something at such a place dropgt

주장 if Stanley Park is ever to be as popular with our citizens as is Carlton Park the town will obviously need to provide more benches thereby converting some of the unused open areas into spaces suitable for socializing1 조사가 언제 이루어진 것인가 조사가 언제 실시되었느냐에 따라 결과가 다를 수있다 현재는 다시 스탠리 파크가 늘어났었을 수 있다 2 벤치를 많이 설치했다고 해서 많은 관광객이 오지 않을수 있다(사람들이 벤치나 사교 공간을 원한다는 어떠한 자료도 없다)3스탠리 파크 주변에 교통 상황이 악화가 되었거나 칼튼 파크에서 문화행사등을 많이 가져서 이용객이 줄어든것일 수도 있다 4 칼튼 파크가 중심지에 있어서 접근성이 좋을수 있다5 조사가 같은 시간을 기준으로 한게 아니다(하나는 주중이고 하나는 주말이다)6사람의 수와 차의 대수를 같은것으로 비교할수 없다 (차안에 몇 명이 타고 있는지 모르고 대중교통을 이용해서 왔을수도 있다)

Page 16: GRE writing argument brain storm

GRE AWA John박 박정어학원

Write a response in which you discuss what specific evidence is needed to evaluate the argument and explain how the evidence would weaken or strengthen the argument

RefutationPrehistoric time shift-gtbrim river could have been narrow and shallow or it might have not existed Indigenous patterns may exist in other disconnected remote placesNuts berries small game(hunting) may not have existed or the author should be proved these existed at that time Abundance of resources doesnrsquot support the reason for seclusionBoats not yet found baskets may have been carried across by the river current without the help of a boat Lithos might have crossed the river for commercial purposeOne Palean basket does not substantiate the authorrsquos claimThe absence of evidence is not an evidence of absence

The author assumes without justification that present conditions are the same as at the prehistoric era The author unfairly infers from the presence of Brim River which exist today that it would have existed in the past However the author fails to offer any evidence to substantiate this inference It is very likely that the Brim River might not have existed in prehistoric times or if it did exist may have been shallow and narrow enough for the Paleans to easily cross Any of these scenarios if true would serve to undermine the claim thathelliphelliphelliphelliphellip

First the author claims that the Brim River was very deep and broad so the Paleans could not have crossed it However the author fails to offer any evidence to substantiate that this was true in the prehistoric time of the Paleans For all we know the Brim River might not have existed in the prehistoric era or if it did exist could have been a shallow and narrow river For example scientists believe that thousands of years ago an ice-bridge existed on the Bering Sea connecting Eurasia to what is now North America to explain how the Eskimos and the Asian inhabitants of America came to migrate all over the Americas In this example the absence of evidence is not an evidence of absence Therefore to sufficiently support his claim the author needs to substantiate the fact that the Brim River really did exist and was broad and wide in the Prehistoric Era proving that the Paleans could not have influenced or traded with other groups of people

First the author claims that the Brim River was very deep and broad so the Paleans could not have crossed it However the author fails to offer any evidence to substantiate that this was true in the prehistoric times of the Paleans For all we know the Brim River might not have existed in the prehistoric era or if it did exist could have been a shallow and narrow river For example rivers are created by natural erosion over thousands of years Niagara Falls carved its way from the mouth of Lake Ottawa and created a long river Likewise the current Brim Riverrsquos physical features may not have been wide and deep Therefore to sufficiently support his claim the author needs to substantiate the fact that the Brim River really did exist and was broad and wide in the Prehistoric Era proving that the Paleans could not have influenced or traded with other groups of people

예전에는 실로짠 특이한 무늬 바구니가 Palea의 선사시대 지역의 인근마을에서만 발견되어왔기 때문에 Palea 마을 사람들의 특징이라고 여겨졌었다 그러나 최근들어 고고학자들이 Lithos지역에서 Palean 바구니를 발견하였는데 그 당시 지역은 Brim 강을 가로질러 Palea까지 닿아있었다 이 강은 수심이 아주 깊고 강폭이 넓었으며 때문에 고대의 Palea인들은 배를 이용해서 강을 건널수 있었을 것이다 그러나 이들이 배를 가지고 있었다는 증거는 발견되지 않고 있다 더군다나 이들이 멸명한 이후 수천년이 지난뒤에도 수많은 물자와 사람을 실어 나를수 있는 용적을 가진 배는 개발되지 않았다 이와더불어 Palea인들은 강을 건널필요가 없었는데 그것은 너트나무 장과열매 그리고 작은 사냥감들이 주변숲에 풍부했기때문이다 따라서 Palean 바구니라고 하는 것도 Palea인들만의 전유물이 아니라는 결론을 얻을 수 있다

결론 if follows that the so-called Palean baskets were not unique to Palea이번문제는 굿이 causal Bad analogy 로 구분해서 찾기가 힘드내요 배를 발견했다는 증거가 없는것이다 (앞으로도 발견될수 있음)

GRE AWA John박 박정어학원

계절의 영향으로 겨울에 얼음이 두껍게 언다든지 여름에 가뭄으로 인해서 건널수 있다물자가 풍부한 것이 이동하지 않을 조건이 아니다 다른 것에 의해서 이동가능(의약품등)

Thirteen years ago researchers studied a group of 25 infants who showed signs of mild distress when exposed to unfamiliar stimuli such as an unusual odor or a tape recording of an unknown voice They discovered that these infants were more likely than other infants to have been conceived in early autumn a time when their mothers production of melatonin hormone known to affect some brain functions would naturally increase in response to decreased daylight In a follow-up study conducted earlier this year more than half of these children now teenagers who had shown signs of distress identified themselves as shy Clearly increased levels of melatonin before birth cause shyness during infancy and this shyness continues into later life

Write a response in which you examine the stated andor unstated assumptions of the argument Be sure to explain how the argument depends on these assumptions and what the implications are for the argument if the assumptions prove unwarranted

Any baby exposed to unpleasant stimuli would react in such wayFirst of all the author states 25 infants as his evidence However this research sample is too small to prove his claimSecond the author states that 25 infants were conceived in early autumn which he claims lead to a shy disposition However this is faulty evidenceThird the research study was a long term study done in the span of 13 years However the author only writes about the initial and final stages of the study and leaves out evidences of what could have happened during the 13 years which could be more evidential factors of influenceFourth neither the infantsrsquo genetic predisposition nor their environment were taken into accountFinally the author concludes that his shyness continues into later life (Other factors could alter this disposition epigenetic theory)

13 년전 학자들은 25명의 유아를 대상으로 이상한 냄새나 특이한 소리를 녹음한 테잎등으로 낯선 자극을 주었을때 보이는 미미한 압박감 증상을 조사하였다 이들은 성숙기가 막 지났을 즈음에 보통의 유아들이 비슷한 증상을 보이는 정도 보다는 다소 민감한 반응을 보였는데 이 시기는 아이의 엄마가 뇌의 일부 기능에 영향을 미치는 것으로 알려진 멜라토니아 호르몬을 생산하는 시기로써 이 호르몬은 낯 시간이 짧을때 자연적으로 증가할 수도 있다 금년초에 실시된 추가연구에서 현재 10대로 성장한 당시 조사대상의 절반 이상의 아이들이 부끄럼을 잘타는 것으로 여기고 있었다 따라서 분명한 것은 출산전 멜라토닌 수치의 증가가 유아기에 수줍음 등의 영향을 미치게 되며 이러한 영향이 성장후에도 작용한다는 것이다

주장 Clearly increased levels of melatonin before birth cause shyness during infancy and this shyness continues into later life

1 25명의 아기로 결론 내리기에 샘플이 작다2 과학적 사실들에 대한 명확한 근거가 엇음3 다른 영향을 간과했다(다른 호르몬에 의한 영향 후천적인 성격형성의 영향)

GRE AWA John박 박정어학원

The following is a letter to the editor of the Atticus City newspaper

Former Mayor Durant owes an apology to the city of Atticus Both the damage to the River Bridge which connects Atticus to Hartley and the traffic problems we have long experienced on the bridge were actually caused 20 years ago by Durant After all he is the one who approved the construction of the bridge If he had approved a wider and better-designed bridge on which approximately the same amount of public money would have been spent none of the damage or problems would have occurred Instead the River Bridge has deteriorated far more rapidly over the past 20 years than has the much longer Derby Bridge up the river Even though the winters have been severe in the past several years this is no excuse for the negligence and wastefulness of Durant

Write a response in which you discuss what questions would need to be answered in order to decide whether the recommendation is likely to have the predicted result Be sure to explain how the answers to these questions would help to evaluate the recommendation

전임 시장인 Durant 씨는 Atticus 시에 대해 사과할 의무가 있습니다 Atticus와 Hartley를 잇는 River Bridge 교량에 대한 피해와 이 교량에서 오랫동안 주민들이 겪어오고 있는 교통 혼잡 문제들은 실제로 20년 전부터 시작된 것이었습니다 결정적으로 그가 교량 공사를 허가했던 바로 그 장본인입니다 당시 비슷한 공사비용으로 폭이 더 넓고 튼튼하게 설계된 교량을 허가했다면 이러한 문제나 피해는 발생하지 않았을 겁니다 더군다나 이 다리는 지난 20년 동안 상류에 건설된 훨씬 오래된 Derby 다리보다도 빠르게 부식되어 갔습니다 지난 수년동안 심지어 혹한이 있었다 하더라도 이러한 태만과 국고 손실에 대한 책임을 회피할 길이 없는 것입니다

결론 Former Mayor Durant owes an apology to the city of Atticus

1 디자인이 문제가 아닐수 있다 (디자인은 좋았으나 건설과정에 문제가 있었을 수 있다)2 그 당시의 시예산이 적어서 더 큰 다리를 짓기가 불가능했을 수도 있다3 교통량이 많거나 다른 상황으로 인해서 부식이 빨리 됐을 수 있다4 그 당시에는 최선의 선택이였지만 갑자기 변한 상황에 의해서 이런 문제점들이 발생했을 수 있다

GRE AWA John박 박정어학원

4그룹 거짓인과관계 오류 (False Cause) 빈출

Fifteen years ago Omega University implemented a new procedure that encouraged students to evaluate the teaching effectiveness of all their professors Since that time Omega professors have begun to assign higher grades in their classes and overall student grade averages at Omega have risen by 30 percent Potential employers looking at this dramatic rise in grades believe that grades at Omega are inflated and do not accurately reflect student achievement as a result Omega graduates have not been as successful at getting jobs as have graduates from nearby Alpha University To enable its graduates to secure better jobs Omega University should terminate student evaluation of professors

Write a response in which you discuss what specific evidence is needed to evaluate the argument and explain how the evidence would weaken or strengthen the argument

Omega professor evaluation implemented 15 years ago =gt Omega prof assign higher grades 30Employers believe therersquos grade inflation

Thus unsuccessful employment than AlphaTherefore to secure jobs Omega should end evaluating profs

Specific evidence neededRelationship between higher grades and evaluationRelationship between GPA and unsuccessful employmentAlpharsquos education could just be better than OmegaldquoFifteen years agordquo is a long time other factors could have influenced Why is the inflation a problem just now How much is Alpha better Is the comparison just How much gap is thereOmegarsquos student could just be doing better in their studiesComparison to other universities다른 대안 없나hellip Could Omega alleviate the employment problem by implementing a different procedure or program

15 년전 우리 대학은 학생들로 하여금 교수평가를 하도록 한 새로운 조치를 시행했었습니다 이후 교수들은 자신의 학과 학생들에게 높은 학점을 주었으며 그에따라 학생들의 전체 평점이 30나 올랐습니다 외부의 기업체들은 분명 점수가 지나치게 부풀려졌다고 믿고 있습니다 결국 본 대학 졸업생들이 인근 Alpha 대학의 졸업자들보다 구직률이 떨어지는 이유를 잘 보여주고 있는 것입니다 이를 해결하기 위해 이제부터는 학생들에 의한 교수평가제를 중단해야 합니다

결론 Omega University should now terminate student evaluation of professors

반박 교수 평가와 학점 인플레의 연관성이 적다( 교수 평가를 먼저하고 학점을 나중에 매길수도 있다)채용기준에 성적만 있는게 아니다 학업성취의 결과 일수도 있다 Alpha 가 원래 유능했다 Alpha 의 교육내용이 좋았다

GRE AWA John박 박정어학원

In this memo the dean of Omega University(OU) recommends OU to terminate professor evaluation to secure better jobs for the students To support this recommendation the dean offers several reasons However this argument contains several logical flaws which render it unconvincing

A threshold problem with the argument involves the voluntary nature of the evaluationprocedure The dean provides no evidence about the number or percentage of Omegastudents who participate in the procedure Lacking such evidence it is entirely possible thatthose numbers are insignificant in which case terminating the procedure is unlikely to haveany effect on the grade average of Omega students or their success in getting jobs aftergraduationThe argument also assumes unfairly that the grade-average increase is the result of theevaluation procedure--rather than some other phenomenon The dean ignores a host of otherpossible explanations for the increase--such as a trend at Omega toward higher admissionstandards or higher quality instruction or facilities Without ruling out all other possibleexplanations for the grade-average increase the dean cannot convince me that by terminatingthe evaluation procedure Omega would curb its perceived grade inflation let alone help itsgraduates get jobsEven if the evaluation procedure has resulted in grade inflation at Omega the deans claimthat grade inflation explains why Omega graduates are less successful than Alpha graduatesin getting jobs is unjustified The dean overlooks a myriad of other possible reasons forOmegas comparatively poor job-placement record Perhaps Omegas career services areinadequate or perhaps Omegas curriculum does not prepare students for the job market aseffectively as Alphas In short without accounting for other factors that might contribute toOmega graduates comparative lack of success in getting jobs the dean cannot justify theclaim that if Omega curbs its grade inflation employers will be more likely to hire OmegagraduatesFinally even if the dean can substantiate all of the foregoing assumptions the deansassertion that Omega must terminate its evaluation procedure to enable its graduates to findbetter jobs is still unwarranted in two respects First the dean ignores other possible ways bywhich Omega can increase its job-placement record--for example by improving its publicrelations or career-counseling services Second the dean unfairly equates more jobs withbetter jobs In other words even if more Omega graduates are able to find jobs as a result ofthe deans recommended course of action the kinds of jobs Omega graduates find would notnecessarily be better onesIn sum the deans argument is unpersuasive as it stands To strengthen it the dean mustprovide better evidence that the increase in grade average is attributable to Omegasprofessor-evaluation procedure and that the end result is a perception on the part ofemployers that Omega graduates are less qualified for jobs than Alpha graduates To betterassess the argument I would need to analyze 15-year trends in (l) the percentage of Omegastudents participating in the evaluation procedure (2) Omegas admission standards andquality of education and (3) Omegas emphasis on job training and career preparation I wouldalso need to know what other means are available to Omega for enabling its graduates to findbetter jobs

GRE AWA John박 박정어학원

The following appeared in a memo from a vice president of Quiot Manufacturing

During the past year Quiot Manufacturing had 30 percent more on-the-job accidents than at the nearby Panoply Industries plant where the work shifts are one hour shorter than ours Experts say that significant contributing factors in many on-the-job accidents are fatigue and sleep deprivation among workers Therefore to reduce the number of on-the-job accidents at Quiot and thereby increase productivity we should shorten each of our three work shifts by one hour so that employees will get adequate amounts of sleep

Write a response in which you examine the stated andor unstated assumptions of the argument Be sure to explain how the argument depends on these assumptions and what the implications are for the argument if the assumptions prove unwarranted

The following appeared in a memo from a vice president of Alta Manufacturing

During the past year Alta Manufacturing had thirty percent more on-the-job accidents than nearby Panoply Industries where the work shifts are one hour shorter than ours Experts believe that a significant contributing factor in many accidents is fatigue caused by sleep deprivation among workers Therefore to reduce the number of on-the-job accidents at Alta we recommend shortening each of our three work shifts by one hour If we do this our employees will get adequate amounts of sleep

Write a response in which you discuss what questions would need to be answered in order to decide whether the recommendation and the argument on which it is based are reasonable Be sure to explain how the answers to these questions would help to evaluate the recommendation

The following appeared in a memo from the vice president of Butler Manufacturing

During the past year workers at Butler Manufacturing reported 30 percent more on-the-job accidents than workers at nearby Panoply Industries where the work shifts are one hour shorter than ours A recent government study reports that fatigue and sleep deprivation among workers are significant contributing factors in many on-the-job accidents If we shorten each of our work shifts by one hour we can improve Butler Manufacturings safety record by ensuring that our employees are adequately rested

1 Write a response in which you discuss what specific evidence is needed to evaluate the argument and explain how the evidence would weaken or strengthen the argument

2 Write a response in which you discuss what questions would need to be answered in order to decide whether the recommendation is likely to have the predicted result Be sure to explain how the answers to these questions would help to evaluate the recommendation

4번 반복됨

Alta has 30 more job accidents than Panoply(work shifts one hour shorter)Experts Job accidents caused by fatigue and sleep deprivationTherefore to reduce job accidents and increase productivity shorten three work shifts by one hour for adequate sleep

지난해 우리 회사는 인근 Panoply Industries보다 업무상 재해가 30나 더 많았다 그 회사는 우리보다 근무 교대시간이 1시간 정도 짧았다 전문가들은 대부분의 업무상 재해에 있어서 가장 중요한 요인이 과로와 수면부족으로 보고있다 따라서 우리 회사에서 높은 산업재해를 줄이고 아울러 생산성을 높이기 위해서는 근로자들이 충분한 수면을 취할 수 있도록 1시간씩 3교대 시간을 줄여야 한다

In this memo the (author) vice president of Alta Manufacturing (AM) recommends that to reduce on-the-job accidents and increase productivity AM should shorten its three work shifts by one hour so that employees can

GRE AWA John박 박정어학원

get more sleep To support this recommendation the author provides several evidences However careful scrutiny of each of the facts reveals that it provides little credible support for the authorrsquos recommendation QuestionsThe number of accidents What kind of accidents The seriousness of the accidents is importantHow many employees are in each company What are their productsFalse cause Sleep may not be the reason for the on-the-job accidents What do Alta and Panoply manufacture

First of all the author believes that fatigue caused the on-the job accidents However there could be other reasons The author observes a correlation between sleep deprivation and on-the-job accidents then concludes that the former is the cause of the latter However the author fails to rule out other possible explanations For example it is entirely possible that Alta factories require more strenuous and dangerous labor than Panoply Without ruling out all other such factors it is unfair to conclude that fatigue is responsible for the accidents In addition the work-shifts may not be the cause of the sleep deprivation and fatigue It is possiblehellip Thus the author should provide what exactly Panoply and Alta manufacture and more precise data about their working conditions to be more convincing

Shortening the shift by one hour does not necessarily lead to more sleep And is one hour enoughLess accidents does not mean increased productivity

결론 We should shorten each of out three work shifts by one hour

반박 경쟁사에 비해서 시간당 하는 업무량이 많아서 더욱 피곤할 수도 있다 시간이 문제가 아니라 노후된 시설 설비 자체의 문제 작업 자체가 원래 위험한 것이여서 사고가 많을 수도 있다 다른 회사는 더욱 많은 작업시간에도 불구하고 안정한 작업여건으로 인해서 사고율이 오히려 더 작을 수도 있다비교사의 재해감소가 다른 요인일수 있다(안전 교육 철저)줄인 시간이 피로회복이나 수면으로 연결 안될수 있음(술을 마실 수도 있고 그 시간에 휴식을 취하지 않고 다른일을 함으로써 더욱 피로해질수 있다)

This editorial recommends that Alta Manufacturing reduce its work shifts by one hour each inorder to reduce its on-the-job accident rate and thereby increase Altas productivity To supportthis recommendation the author points out that last year the number of accidents at Alta was30 greater than at Panoply Industries where work shifts were one hour shorter The authoralso cites certain experts who believe that many on-the-job accidents are caused by fatigueand sleep deprivation I find this the argument unconvincing for several reasonsFirst and foremost the author provides absolutely no evidence that overall workerproductivity is attributable in part to the number of on-the-job accidents Although commonsense informs me that such a relationship exists the author must provide some evidence ofthis cause-and-effect relationship before I can accept the authors final conclusion that theproposed course of action would in fact increase Altas productivitySecondly the author assumes that some accidents at Alta are caused by fatigue or sleepdeprivation However the author overlooks other possible causes such as inadequateequipment maintenance or worker training or the inherent hazards of Altas manufacturingprocesses By the same token Panoplys comparatively low accident rate might be attributablenot to the length of its work shifts but rather to other factors such as superior equipmentmaintenance or worker training In other words without ruling out alternative causes ofon-the-job accidents at both companies the author cannot justifmbly conclude that merely byemulating Panoplys work-shift policy Alta would reduce the number of such accidentsThirdly even assuming that Altas workers are fatigued or sleep-deprived and that this is thecause of some of Altas on-the-job accidents in order to accept the authors solution to thisproblem we must assume that Altas workers would use the additional hour of free time tosleep or rest However the author provides no evidence that they would use the time in thismanner It is entirely possible that Altas workers would use that extra hour to engage in someother fatiguing activity Without ruling out this possibility the author cannot convincinglyconclude that reducing Altas work shifts by one hour would reduce Altas accident rateFinally a series of problems with the argument arise from the scant statistical information onwhich it relies In comparing the number of accidents at Alta and Panoply the author fails toconsider that the per-worker accident rate might reveal that Alta is actually safer than Panoplydepending on the total number of workers at each company Second perhaps accident rates

GRE AWA John박 박정어학원

at the two companies last year were aberrations and during other years Altas accident ratewas no greater or even lower than Panoplys rate Or perhaps Panoply is not representativeof industrial companies generally and that other companies with shorter work shifts have evenhigher accident rates In short since the argument relies on very limited statistical information Icannot take the authors recommendation seriouslyIn conclusion the recommendation for emulating Panoplys work-shift policy is not wellsupported To convince me that shorter work shifts would reduce Altas on-the-job accidentrate the author must provide clear evidence that work-shift length is responsible for some ofAltas accidents The author must also supply evidence to support her final conclusion that alower accident rate would in fact increase overall worker productivity

The following appeared in a memo from the vice president of marketing at Dura-Sock Inc

A recent study of our customers suggests that our company is wasting the money it spends on its patented Endure manufacturing process which ensures that our socks are strong enough to last for two years We have always advertised our use of the Endure process but the new study shows that despite our socks durability our average customer actually purchases new Dura-Socks every three months Furthermore our customers surveyed in our largest market northeastern United States cities say that they most value Dura-Socks stylish appearance and availability in many colors These findings suggest that we can increase our profits by discontinuing use of the Endure manufacturing process

1 Write a response in which you examine the stated andor unstated assumptions of the argument Be sure to explain how the argument depends on these assumptions and what the implications are for the argument if the assumptions prove unwarranted

2 Write a response in which you discuss what specific evidence is needed to evaluate the argument and explain how the evidence would weaken or strengthen the argument

3 Write a response in which you discuss what questions would need to be answered in order to decide whether the recommendation and the argument on which it is based are reasonable Be sure to explain how the answers to these questions would help to evaluate the recommendation

Intro The vice president of marketing at Dura-Sock Inc is offering a potentially harmful investment recommendation by claiming that Dura-Sock should discontinue its use of the ldquoEndurerdquo process To support his recommendation he points out a study that Dura-Sock customers actually purchase the socks every three months and a survey that reveals that Dura-Sock customers like the sockrsquos stylish appearance and availability in many colors The study and survey however are insufficient in supporting his proposal and the VP makes several unwarranted assumptionsIntro (simplified) The VP states that though Dura-Socks last for two years customers buy the socks every three months Therefore he assumes that the consumersrsquo motive for buying the produce is not its durabilityHowever the author fails to rule out other possible motivation for consumption

Even if the survey is reliable the author should consider the rest of the market Vague terms ldquowasting moneyrdquomdashprecisely how much are they wasting Studysurvey errorThe company must calculate the outcome of such momentous decisionStudy participantsrsquo comment that they prefer Dura-Sock for its stylishness and availability might take Dura-Sockrsquos enduring quality for granted

우리회사 제품 소비자들에 대한 최근 조사에서 지난 2년여간 양말의 내구성을 강하게 하는 필수공정이었던 자사 특허의 Endure 공정에 들어가는 비용이 낭비라고 말하고 있다 우리 회사는 항상 이 공정 처리에 대한 광고를 내보냈으나 이에 대한 시장 조사에서 실제로 고객들은 이 신제품을 평균 석달마다 구매하는 것으로 나타났다 더군다나 북동부지역에서 실시한 대규모 시장조사에 응답한 고객들은 양말의 모양과 색상등에 더

GRE AWA John박 박정어학원

관심을 나타냈다 이러한 결과는 우리회사가 신기술 공법을 중단하면 그에 따라 수익이 늘어날 것이라는 것을 말해주고 있는 것이다주장 These findings suggest that Dura0Sock can increase its profits by discontinuing its use of the ldquoEndurerdquo manufacturing process

1 survey가 정확한 소비자의 의견을 나타낸 것인가 다른 선택없이 양자택일과 같은 방법의 survey였는지2 북동부지역의 시장조사가 전체 의견을 대표할 수 있나3 사람들이 모양이나 색상에 앞서 내구성을 먼저 평가했을 수도 있다 내구성을 갖추었다는 전제하에 모양과 색상에 관심을 드러낸 것일 수 있다4 소비자가 도매상(retail)인지 소매상(whole)인지가 없다

The following appeared in a business magazine

As a result of numerous complaints of dizziness and nausea on the part of consumers of Promofoods tuna the company requested that eight million cans of its tuna be returned for testing Promofoods concluded that the canned tuna did not after all pose a health risk This conclusion is based on tests performed on samples of the recalled cans by chemists from Promofoods the chemists found that of the eight food chemicals most commonly blamed for causing symptoms of dizziness and nausea five were not found in any of the tested cans The chemists did find small amounts of the three remaining suspected chemicals but pointed out that these occur naturally in all canned foods

Write a response in which you discuss what questions would need to be addressed in order to decide whether the conclusion and the argument on which it is based are reasonable Be sure to explain how the answers to the questions would help to evaluate the conclusion

Representativeness of the tested cansThey should conduct a comparative studyThe testing could be biased because Promofoods employees conducted the testingHow much (quantity) of the five and three suspected chemicals were in the canned foodsFalse cause The substance that caused dizziness and nausea may not be one of the eight common chemicals

많은 소비자들의 현기증과 구역질 불만에 따라 Promofoods사는 지난해 참치 캔 8백만 개를 테스트하기 위해 반품시켰다 그 결과 캔에서는 건강에 위험이 될 수 있는 화합물질이 없었던 것으로 회사측은 결론지었다 이러한 결론은 회사측 화학연구자들이 회수된 캔의 샘플을 테스트해서 이들 증상의 원인이 되는 8가지 화합물 중에서 5가지가 실험된 캔에서 발견되지 않았다는 사실에 근거한 것이다 이들 화학자들은 나머지 3개가지 화합물이 모든 캔 식료품에서 흔히 발견되는 것이라고 언급했다 결론 Promofoods concluded that the cans did not after all contain chemicals that posed a health risk

1 공인된 기간에서 테스트를 한 것이 아니고 자사에서 직접 테스트를 했기에 신뢰성이 안간다 2 이런 증상을 일으키는 8개의 물질 말고 다른 물질들이 캔속에 많이 포함됬을수 있다 3 나머지 3개의 물질들의 함유량이 많아서 다른 종류의 캔들은 문제를 일으키지 않지만 참치캔은 문제를

일으킬 수 있다

This magazine article concludes that the 8 million cans of tuna Promofoods recalled due tocomplaints about nausea and dizziness do not after ail contain any chemicals that pose a

GRE AWA John박 박정어학원

health risk To support this conclusion the author cites the fact that five of eight chemicalscommonly causing these symptoms were not found in the recalled cans while the other threealso occur naturally in other canned foods For several reasons this evidence lends littlecredible support to the authors conclusionTo begin with the author relies partly on the fact that although three of the eight chemicalsmost commonly blamed for nausea and dizziness appeared in Promofoods recalled tunathese chemicals also occur naturally in other canned foods However this fact alone lends nosupport to the authors conclusion for two reasons First the author might be ignoring animportant distinction between naturally occurring chemicals and those not occurring naturallyIt is entirely possible that these three chemicals do not occur naturally in Promofoods tunaand that it is for this reason that the chemicals cause nausea and dizziness Secondly it isentirely possible that even when they occur naturally these chemicals cause the samesymptoms Unless the author rules out both possibilities he cannot reliably conclude that therecalled tuna would not cause these symptomsAnother problem with the argument is that the authors conclusion is too broad Based onevidence about certain chemicals that might cause two particular heath-related symptoms theauthor concludes that the recalled tuna contains no chemicals that pose a health risk Howeverthe author fails to account for the myriad of other possible health risks that the recalled tunamight potentially pose Without ruling out all other such risks the author cannot justifiablyreach his conclusionA third problem with the argument involves that fact that the eight particular chemicals withwhich the test was concerned are only the eight most commonly blamed for nausea anddizziness It is entirely possibly that other chemicals might also cause these symptoms andthat one or more of these other chemicals actually caused the symptoms Without ruling outthis possibility the author cannot jusufiably conclude that the recalled tuna would not causenausea and dizzinessA final problem with the argument involves thetesting procedure itself The author providesno information about the number of recaUed cans tested or the selection method used Unlessthe number of cans is a sufficiently large sample and is statistically repre sentative of all therecalled cans the studys results are not statistically reliableIn conclusion the article is unconvincing as it stands To strengthen the assertion that therecalled tuna would not cause nausea and dizziness the author must provide evidence thatthe three chemicals mentioned that occur naturally in other canned foods also appear naturallyin Promofoods tuna The author must also provide evidence that ingesting other canned foodscontaining these three chemicals does not cause these symptoms To better evaluate theargument we would need to know whether the sample used in the tests was statisticallysignificant and representative of all the recalled tuna We would also need to know what otherchemicals in the recalled tuna might pose any health risk at all

5그룹 불충분 조건오류 빈출

Natures Way a chain of stores selling health food and other health-related products is opening its next franchise in the town of Plainsville The store should prove to be very successful Natures Way franchises tend to be most profitable in areas where residents lead healthy lives and clearly Plainsville is such an area Plainsville merchants report that sales of running shoes and exercise clothing are at all-time highs The local health club has more members than ever and the weight training and aerobics classes are always full Finally Plainsvilles schoolchildren represent a new generation of potential customers these schoolchildren are required to participate in a fitness-for-life program which emphasizes the benefits of regular exercise at an early age

Write a response in which you examine the stated andor unstated assumptions of the argument Be sure to

GRE AWA John박 박정어학원

explain how the argument depends on these assumptions and what the implications are for the argument if the assumptions prove unwarranted

False cause

First of all the author believes that the Increased sales of running shoes and exercise clothing indicates

Plainesville residentsrsquo interest in leading healthy lives However this assumption is not logically convincing for

several reasons could be a fashion trendTime shift ldquoFitness for liferdquo might not have any influence on schoolchildren as they growFalse cause There could be other reasons for member increase in the health clubAll of the above are insufficient condition

The author has to prove that local residents are interested in leading healthy lives However he supports his conclusion with insufficient evidence Nevertheless even if the residents are concerned with health naturersquos way may not be successful First

그 동안의 경험을 토대로 볼 때 건강생활과 밀접히 관련되어 있는 거주 지역에서 본 상점들이 아주 호응을 얻고 있다 따라서 이러한 주민들이 많이 거주하고 있는 Plainsville 에 새로운 상점들을 계속 세워야 한다 이 지역 상인들은 런닝화와 운동복 판매가 가장 높다고 말한다 불과 5 년전에는 거의 전무하다시피하던 지역 헬스 클럽의 경우도 엄청나게 많은 회원을 확보하고 있으며 웨이트 트레이닝과 에어로빅 강좌들도 항상 만원이라고 한다 새로운 고객층을 예측해 보는 것도 가능하다 이 지역의 학생들의 경우 Fitness for Life프로그램을 받게 되는데 이러한 프로그램을 통해서 유년시절부터 정규적인 운동 습관을 들이게 하고 있는 것이 그것이다

결론 We should therefore build our next new store in Plainsville

반박 그동안의 경험에 의한 과거 통계가 꼭 여기에도 적용되는건 아니다 5 년전 헬스 클럽이 잘 안되었던게 다른 원인이였을수 있다(강사수준 미달 강좌미비)tourist 에 의한 원인 일수 있다 어렸을때부터 운동을 했다고 해서 커서도 관심이 있지는 않다 (오히려 반감이 있을수 있다 혹은 건강하기에 건강에 관심이 적을수도 있다)운동복이나 신발의 판매가 육체노동에 의한 것일수도 있다

IntroductionSupport1049896In this memorandum the author asserts that Naturersquos Way should build its next newstore in Plainsville To support this assertion the author states that Plainsvillesmerchantsrsquo sales of exercise clothing are going well the local health club has moremembers than ever and a new generation of customers will help to ensure NaturersquosWayrsquos success At first glance the authorrsquos assumption seems convincing but in-depth scrutiny revealsthat it lacks substantial evidence as it stands

Body 1-SamplingTopic Sentence 1To begin with the author assumes that the merchantsrsquo report indicates that the residentsare concerned about their health However this assumption is based on unsubstantiated

GRE AWA John박 박정어학원

data Example 1 (Rebuttal1) First if we do not know the total volume of items sold and the price of the goods exactly we cannot infer whether the residents are actually buying many goods Example 2 (Rebuttal2)In addition to that the report emphasizes the rising sales of running shoes and exerciseclothing however these may not be hot-selling items for Naturersquos Way or may not be theproducts the company is planning to sell Concluding Sentence Therefore in order to make the argument reliable the author should reconsider themerchantsrsquo report with more detailed data

Body 2-CausalTopic Sentence 2Second the author contends that the health clubs classes are full yet this does not meanthat many people actually use the health club other factors may be the real cause forthose closed classes Example 1 (Rebuttal1) To begin with if the health club is very small the number of people working out wouldnot be a large one In fact regular gym-going may just be a vogue among a smallunrepresentative segment of Plainsvilles population Example 2 (Rebuttal2) Moreover it is possible that most of the people who exercise in the health club do weight training and aerobics only to look good and to meet other singles not for their health In that case there would be little demand for health products Concluding SentenceThus the author should not hasten to presume what really caused people to be interested in a healthier lifestyle and enroll in the health club

Body 3-Time-ShiftTopic Sentence 3Finally the author highlights that Naturersquos Way can expect a new generation of customersin Plainsville that will help the company in the long term This notion is mistaken in that itassumes the conditions of the present will continue unchanged in the future Although theschool children are required to participate in the fitness for life program they may notnecessarily buy Naturersquos Ways products Example 1 (Rebuttal1) In the first instance they may suffer a fall in purchasing power arising from future economic difficulties this would cause reluctance to spend a considerable amount of money on health products which tend to be more expensive Example 2 (Rebuttal2)Another possibility is that there may emerge many competitor companies vying with Naturersquos Way so that in the future the school children may not feel the necessity to purchase one companyrsquos health products over anotherrsquosConcluding Sentence Thus the authorrsquos assumption is highly speculative since it relies heavily on unknowablefuture circumstances

ConclusionThesis In sum the author uses many assumptions that are insufficient in supporting his claimsSupportIn order for the authorrsquos claims to be convincing he needs to advance more persuasiveevidence that people in Plainsville really are concerned with their health and health foodThe following was written as a part of an application for a small-business loan by a group of developers in the city of Monroe

A jazz music club in Monroe would be a tremendously profitable enterprise Currently the nearest jazz club is 65 miles away thus the proposed new jazz club in Monroe the C-Note would have the local market all to itself Plus jazz is extremely popular in Monroe over 100000 people attended Monroes annual jazz festival last summer several well-known jazz musicians live in Monroe and the highest-rated radio program in Monroe is Jazz Nightly which airs every weeknight at 7 PM Finally a nationwide study indicates that the typical jazz fan spends close to $1000 per year on jazz entertainment

1 Write a response in which you discuss what specific evidence is needed to evaluate the argument and explain how the evidence would weaken or strengthen the argument

2 Write a response in which you examine the stated andor unstated assumptions of the argument Be

GRE AWA John박 박정어학원

sure to explain how the argument depends on these assumptions and what the implications are for the argument if the assumptions prove unwarranted

3 Write a response in which you discuss what questions would need to be answered in order to decide whether the prediction and the argument on which it is based are reasonable Be sure to explain how the answers to these questions would help to evaluate the prediction

Group error nationwide survey may not reflect local trends Is the nationwide jazz fan population substantialInsufficient non-residents of Monroe may have attended the jazz festival (Body alternative explanation last year may have been an anomaly The author should consider data from various years) The author should indicate how many out of 100000 were Monroe residentsNationwide study Does this reflect Insufficient Citizens of Monroe may continue to go to the jazz club 65 miles away

Are the people in Monroe really interested in jazzMajority of the people who attended the jazz festival might not be Monroe residentsSurvey error nationwide study may not be applicable to MonroeJazz musicians who live in MonroeMonopolyRadio station

In this business application the author claims that the proposed jazz club C Note will be very profitable in Monroe To support this claim the author argues for his case with several evidences At first glance the authorrsquos argument seems convincing however careful scrutiny reveals that his argument in specious

To begin with the author claims that Monroersquos citizens are interested in jazz He presents three evidences First Secondhellip Thirdhellip Howeverhellip

Monroe 시에 있는 재즈 음악 클럽은 수익성이 좋은 사업이다 현재 가장 가까이에 있는 클럽은 65 마일 정도 떨어져 있다 따라서 이번에 세우려고 하는 C Note 는 독보적인 위치를 점할것이다 더군다나 재즈는 이 시에서 가장 인기있는 음악이다 지난 여름 재즈 축제에서는 10 만명 이상의 Morone 시 주민이 참석하였고 몇몇 유명한 재즈 음악가들도 이곳에 살고 있으며 저녁때 방영되는 라디오 프로그램중에서 최고의 시청률을 보이고 있는 것도 Jazz Nightly 이다 전국조사에서도 전형적인 재즈 팬들은 재즈 분야에 년간 1천 달러 가까이 지출하고 있는 것으로 보고되고 있다 따라서 C Note 클럽이 돈을 벌 수 있는 사업이라는 것은 확실한 것이다

결과 It is clear that the C Note cannot help but make money반박 nearest jazz club 이 양질의 써비스로 여전히 손님을 끌수도 있다Festival 에 얼마나 참여하는지가 jazz 의 인기를 반영하지 않는다 뮤지션이 많이 사는거랑 jazz 의 인기가 상관없다라디오 프로그램이 다른 요인에 의해서 인기일수도 있다 (진행자때문)전국 통계 적용 불가화목 실전반_Ms Noh6In this application the author suggests that a jazz club in Monroe will make a number of profits To support this suggestion the author exemplifies the local condition popularity of jazz in Monroe and nationwide study However careful scrutiny of each of the facts reveals that it provides little credible support for the authorrsquos recommendation Good clear intro

First the author assumes that jazz is popular in Monroe because of several facts the jazz festival last year had high participation some famous jazz musicians live in Monroe and the high-rated radio program is lsquoJazz Nightlyrsquo However this assumption has many drawbacks that must be seriously considered(Good topic sentences) If many attendants in the last-yearrsquos festival came from other cities and not Monroe it is hard to conclude that Monroersquos people like jazz Therefore the author must examine how many Monroe residents actually attended the festival On top of that there is little relationship between habitation of famous jazz musician and the popularity of jazz in Monroe Although several well-known musicians live there if they do not take part in any jazz performance of Monroe this might have no effect to the interest of Monroersquos residents

GRE AWA John박 박정어학원

about jazz Finally in the case of radio program this is also not suitable reason why jazz is popular in Monroe It might be possible that people cannot help choosing lsquoJazz Nightlyrsquo because there are few radio programs at Night The fact that the radio program is the highest rating program is not a germane evidence The approximate number of listeners would be the more crucial evidence Therefore the author needs to seriously deliberate the correlation between jazzrsquos popularity in Monroe and his examples (Good logical flow and clarity)

Second the author uses as evidence the nationwide study that jazz fans spend much money on jazz entertainment to substantiate why starting a jazz club in Monroe will be profitable In other words the author assumes that the characteristics of a nationwide study can be applied to Monroe The national study would lend support to the applicantrsquos claim only if residents in Monroe typify national jazz fans However the author does not provide credible evidence that this is the case Moreover the populations of jazz fans nationwide may be insubstantial Thus the author should not infer hastily that Monroersquos residents will spend much money on enjoying jazz from the nationwide study

Lastly even if jazz is popular in Monroe C Note may not be successful It is entirely possible that residents might still prefer other clubs where they have always went In addition there is another possibility that the nearest jazz club will attract many of Monroersquos people because it serves fine performances and is equipped with favorite facilities Without considering these other possibilities the author cannot make his argument convincing In sum the author presents many reasons that are insufficient in supporting his or her claim In order for the authorrsquos claims to be convincing he needs to advance more persuasive evidence such as the total number of Monroe residents who attended the jazz festival the effects on the popularity of jazz by the musicians living in Monroe and the actual number of residents who would typify themselves to be jazz fans through a local survey Without substantial evidence that C Note will be successful in Monroe the businessmen may be overinvesting in what might lead to a business failureExcellent clarity Score 50

The following appeared in a newsletter offering advice to investors

Over 80 percent of the respondents to a recent survey indicated a desire to reduce their intake of foods containing fats and cholesterol and today low-fat products abound in many food stores Since many of the food products currently marketed by Old Dairy Industries are high in fat and cholesterol the companys sales are likely to diminish greatly and company profits will no doubt decrease We therefore advise Old Dairy stockholders to sell their shares and other investors not to purchase stock in this company

Write a response in which you discuss what questions would need to be answered in order to decide whether the advice and the argument on which it is based are reasonable Be sure to explain how the answers to these questions would help to evaluate the advice

Survey 80

GRE AWA John박 박정어학원

Old Dairy could change their products and manufacture low fat dairy foodsLess competing companies Old Dairy could eventually be the only company that produces hellipImprecise numbers and measurementsCustomers may still buy high fat dairy products

The author of the newsletter is offering potentially dangerous advice by recommending Old Dairy stockholders to withdraw investment and stop purchase What is more the authorrsquos prediction debases the reputation and business of Old Dairy and if false could devoid the investment opportunity of the newsletter readers Therefore investors should examine whether the authorrsquos evidences are substantial

To begin with the author states that 80 percent of the respondents in a survey indicated a desire to reduce their intake of foods He therefore argues that Old Dairyrsquos high fat and cholesterol products would decrease in sales However the author makes a crucial error in this argument First the author provides no evidence that the surveyrsquos results are statistically reliable Were they representative of all the customers Were they chosen for the survey randomly Furthermore the desire to reduce fat and cholesterol intake is a pervasive trend in todayrsquos opulent society however the author erroneously identifies this as a new phenomenon which will affect consumer trends Second having a desire to reduce fat and cholesterol intake does not necessarily indicate that people who have this desire will actually reduce consuming these types of products It is entirely possible that they may continue buying Old Dairy products for its quality and taste Accordingly the author cannot draw any firm conclusion that people will not buy Old Dairy products Therefore if any of these cases are true the author may be offering investors a detrimental investment advice

최근 조사에 대한 응답자중 80 이상이 자신이 먹는 음식에서 지방과 콜레스테롤의 함유량을 줄이고 싶다고 한다 아울러 요즘은 많은 식료품 가계에서 저지방 제품들을 많이 취급하고 있다 현재 Old Dairy Industries가 판매하고 있는 많은 음식제품들은 지방과 콜레스테롤이 높기 때문에 이 회사의 매출이 격감할 것으로 보이며 당연히 매출이익도 줄어들것이다 따라서 이 회사의 주주들은 주식을 매각하고 다른 주식 투자가들도 이 회사의 주식을 매입하지 않는 것이 좋다

결론 Old Dairy stockholders to sell their shares and other investors not to purchase stock in this company

반박 모든 상품이 다 고 지방 고 칼로리는 아니다(비록 많을지라도) 일부의 식품의 경우 기호에 맞어서 히트해서 전체적인 수입이 증가할 수도 있다국내시장만 생각할 수 없다( 외국시장에서 호황을 누릴수 있다 )입맛이라는게 즉각 바뀌는게 아니다

The following appeared in a letter to the editor of the Balmer Island Gazette

On Balmer Island where mopeds serve as a popular form of transportation the population increases to 100000 during the summer months To reduce the number of accidents involving mopeds and pedestrians the town council of Balmer Island should limit the number of mopeds rented by the islands moped rental companies from 50 per day to 25 per day during the summer season By limiting the number of rentals the town council will attain the 50 percent annual reduction in moped accidents that was achieved last year on the neighboring island of Seaville when Seavilles town council enforced similar limits on moped rentals

1 Write a response in which you discuss what questions would need to be answered in order to decide whether the recommendation is likely to have the predicted result Be sure to explain how the answers to these questions would help to evaluate the recommendation

2 Write a response in which you discuss what questions would need to be answered in order to decide whether the prediction and the argument on which it is based are reasonable Be sure to explain how the answers to these questions would help to evaluate the prediction

3 Write a response in which you examine the stated andor unstated assumptions of the argument Be sure to explain how the argument depends on these assumptions and what the implications are for the argument if the assumptions prove unwarranted

Whatrsquos the actual population of Balmer Island 100000mdashis this a significant increase What kind of accidents Skin abrasions or serious injury And compared to Seaville how serious are the accidents and the actual number of accidents Did Seaville enforce other restrictions like safety signsHow different are the conditions of Balmer

GRE AWA John박 박정어학원

and Seaville regarding population road (safety) conditions topography other town-government regulation How much will the economy of Balmer be affected do to this restriction Could it cause an economic recession due to the fact that these rental companiesrsquo chance to make money is only during the summer thereby weakening the economic infrastructure Are there any other ways that could better alleviate the accident rate

Statistics 50-impreciseAnalogy Balmer compared with TorseauFalse Cause Accidents might have occurred because of reasons other than mopeds False Cause population increase may not be part of the cause of the accidentsOther explanations for the accident pedestrians few road safety regulations narrow roadsThere could be other better solutionshellip

Balmer Island의 인구가 여름철에는 십만명으로 늘어난다 2륜차와 보행자간 사고를 줄이기 위해 시의회는 6개의 자전거를 포함한 2륜차 대여업체에게 이 기간동안에는 대여숫자를 일일 50에서 30으로 제한하도록 할 것이다 대여숫자를 줄임으로써 시 의회는 지난해 이웃한 Torseau섬에서 이와 동일한 규제를 시행해서 50나 줄인 결과를 보고 마찬가지로 50를 줄일수 있다고 확신하고 있다

결론 The town council of Balmer Island should linit the number

반박 보행자의 부실에 의해서 사고가 많이 일어날수도 있다렌탈수의 줄임만이 대책은 아니다(대부분의 사람들이 렌탈 보다는 소유하고 있을 수도 있다)옆섬과는 상황이 다를수도 있다(그 섬에서는 사고의 원인이 많은 자전거 수로 인한것일수있다) 하지만 이 섬은 좁은 도로가 원인일 수도 있고 도로 안전 장치의미비가 원일일수 있다

In this letter the author recommends that Balmer Island should limit the number moped rentals from 50 to 30 per day To support this recommendation the author points out several reasons However careful scrutiny of each of the facts reveals that it is filled with unanswered questions that could significantly weaken the authorrsquos recommendation with loops and holes which are answered

The recommendation depends on the assumption that no alternative means of reducing the number of accidents are available However the author fails to offer any evidence to substantiate this crucial assumption It is highly possible that means other than this would better solve the problem Perhaps they could widen the roads or put-up more safety signs Or perhaps the accidents were due to the lack of skills in which case proper safety training would significantly alleviate the problem Without considering and ruling out these and other alternative means of reducing accidetns the author cannot confidently conclude that merely emulating Torseau would suffice Moreover the author is advising a recommendation which could potentially harm the economy of Balmer Island sincehellip Moreover the Balmer Island should alternative means to reduce accidents because limiting moped rentals during the summer could harm the economy of Balmerhellip

First of all the author believes that increase in population and the number of moped rentals are responsible for the accidents It is entirely possible that other factors are responsible for the accidents Perhaps Balmer Islandrsquos lack of safety signs was a major factor Or maybe the roads are narrow and dangerous on the Island therefore the town council could enforce stricter traffic regulations to alleviate the problem Accordingly if either of these scenarios is true the author cannot draw any firm conclusion that increase in the number of population and moped rentals are the cause of the accidents

The author of this editorial recommends that to reduce accidents involving mopeds andpedestrians Balmer Islands city council should restrict moped rentals to 30 per day down from50 at each of the islands six rental outlets To support this recommendation the author citesthe fact that last year when nearby Torseau Islands town council enforced similar measuresTorseaus rate of moped accidents fell by 50 For several reasons this evidence providesscant support for the authors recommendationTo begin with the author assumes that all other conditions in Balmer that might affect therate of moped-pedestrian accidents will remain unchanged after the restrictions are enactedHowever with a restricted supply of rental mopeds people in Balmer might purchase mopedsinstead Also the number of pedestrians might increase in the future with more pedestriansespecially tourists the risk of moped-pedestrian accidents would probably increase For thatmatter the number of rental outlets might increase to make up for the artificial supplyrestriction per outlet--a likely scenario assuming moped rental demand does not declineWithout considering and ruling out these and other possible changes that might contribute to ahigh incidence of moped-pedestrian accidents the author cannot convince me that theproposed restrictions will necessarily have the desired effect

GRE AWA John박 박정어학원

Next the author fails to consider other possible explanations for the 50 decline inTorseaus moped accident rate last year Perhaps last year Torseau experienced unusually fairweather during which moped accidents are less likely Perhaps fewer tourists visited Tot seanlast year than during most years thereby diminishing the demand for rental mopeds to belowthe allowed limits Perhaps last year some of Torseaus moped rental outlets purchased newmopeds that are safer to drive Or perhaps the restrictions were already in effect but were notenforced until last year In any event a decline in Torseaus moped accident rate during onlyone year is scarcely sufficient to draw any reliable conclusions about what might have causedthe decline or about what the accident rate will be in years aheadAdditionally in asserting that the same phenomenon that caused a 50 decline in mopedaccidents in Torseau would cause a similar decline in Balmer the author relies on what mightamount to an unfair analogy between Balmer and Torseau Perhaps Balmers ability to enforcemoped-rental restrictions does not meet Torseaus ability if not then the mere enactment ofsimilar restrictions in Balmer is no guarantee of a similar result Or perhaps the demand formopeds in Torseau is always greater than in Balmer Specifically if fewer than all availablemopeds are currently rented per day from the average Balmer outlet while in Torseau everyavailable moped is rented each day then the proposed restriction is likely to have less impacton the accident rate in Balmer than in TorseauFinally the author provides no evidence that the same restrictions that served to reduce theincidence of all moped accidents by 50 would also serve to reduce the incidence ofaccidents involving mopeds and pedestrians by 50 Lacking such evidence it is entirelypossible that the number of moped accidents not involving pedestrians decreased by a greaterpercentage while the number of moped-pedestrian accidents decreased by a smallerpercentage or even increased Since the author has not accounted for these possibilities theeditorials recommendation cannot be taken seriouslyIn conclusion the recommendation is not well supported To convince me that the proposedrestriction would achieve the desired outcome the author would have to assure me that nochanges serving to increase Balmers moped-pedestrian accident rate will occur in theforeseeable future The author must also provide dear evidence that last years decline inmoped accidents in Torseau was attributable primarily to its moped rental restrictions ratherthan to one or more other factors In order to better evaluate the recommendation I wouldneed more information comparing the supply of and demand for moped rentals on the twoislands I would also need to know the rate of mopedpedestrian accidents in Torseau both priorto and after the restrictions were enforced in TorseauThe following appeared in a magazine article about planning for retirement

Clearview should be a top choice for anyone seeking a place to retire because it has spectacular natural beauty and a consistent climate Another advantage is that housing costs in Clearview have fallen significantly during the past year and taxes remain lower than those in neighboring towns Moreover Clearviews mayor promises many new programs to improve schools streets and public services And best of all retirees in Clearview can also expect excellent health care as they grow older since the number of physicians in the area is far greater than the national average

Write a response in which you discuss what specific evidence is needed to evaluate the argument and explain how the evidence would weaken or strengthen the argument

-Natural beauty and consistent climate may not be the most wanted qualities-Housing costs could have lowered on a national level wealthy retirees may not care about costs-Taxes may be high compared to the nationrsquos average tax rate-What about other qualities of Clearview Crime rate what qualities would retirees want -If schools streets and public services need improvement then this is proof that the current condition of Clearview is low Or due to budgetary reasons the mayor may not follow-up on his promise because of lowered tax rate -Schools and people who are retired no relationship-Physicians What kind of physicians Number is irrelevant Are these physicians capable of addressing the illnesses of old people

This author argues that anyone seeking a place to retire should choose Clearview To supportthis argument the article cites Clearviews consistent climate and natural beauty its fallinghousing costs its low property taxes compared to nearby towns and the mayors promise toimprove schools streets and services The article also claims that retirees can expectexcellent health care because the number of physicians in Clearview greatly exceeds thenational average This argument is flawed in several critical respectsTo begin with although consistent climate and natural beauty might be attractive to manyretirees these features are probably not important to all retirees For many retirees it isprobably more important to live near relatives or even to enjoy changing seasons Thus I

GRE AWA John박 박정어학원

cannot accept the authors sweeping recommendation for all retirees on this basisAlso Clearviews declining housing costs do not necessarily make Clearview the best placeto retire for two reasons First despite the decline Clearviews housing costs might be highcompared to housing costs in other cities Secondly for wealthier retirees housing costs arenot likely to be a factor in choosing a place to retire Thus the mere fact that housing costshave been in decline lends scant support to the recommendationThe articles reliance on Clearviews property-tax rates is also problematic in two respectsFirst retirees obviously have innumerable choices about where to retire besides Clear viewand nearby towns Secondly for retirees who are well-off financially property taxes are notlikely to be an important concern in choosing a place to retire Thus it is unfair to infer fromClearviews property-tax rates that retirees would prefer ClearviewYet another problem with the argument involves the mayors promises In light of Clearviewslow property-tax rates whether the mayor can follow through on those promises is highlyquestionable Absent any explanation of how the city can spend more money in the areas citedwithout raising property taxes I simply cannot accept the editorials recommendation on thebasis of those promises Besides even if the city makes the improvements promised thoseimprovements--particular the ones to schools--would not necessarily be important to retireesFinally although the number of physicians in Clearview is relatively high the per capitanumber might be relatively low Moreover it would be fairer to compare this per capita numberwith the per capita number for other attractive retirement towns--rather than the nationalaverage After all retirees are likely to place a relatively heavy burden on health-careresources Besides the article provides no assurances that the number of physicians inClearview will remain high in the foreseeable futureIn conclusion the recommendation is poorly supported To strengthen it the author mustconvince me--perhaps by way of a reliable survey--that the key features that the vast majorityof retirees look for in choosing a place to live are consistent climate natural beauty and lowhousing costs The author must also provide better evidence that Clear views property taxesare lower than the those of cities in other areas The author must also explain how the city canmake its promised improvements without raising property taxes Finally to better assess theargument I would need to now how the per capita number of physicians in Clearview wouldcompare to the national average in the futureThe following appeared as a letter to the editor from a Central Plaza store owner

Over the past two years the number of shoppers in Central Plaza has been steadily decreasing while the popularity of skateboarding has increased dramatically Many Central Plaza store owners believe that the decrease in their business is due to the number of skateboard users in the plaza There has also been a dramatic increase in the amount of litter and vandalism throughout the plaza Thus we recommend that the city prohibit skateboarding in Central Plaza If skateboarding is prohibited here we predict that business in Central Plaza will return to its previously high levels

Write a response in which you discuss what questions would need to be answered in order to decide whether the recommendation is likely to have the predicted result Be sure to explain how the answers to these questions would help to evaluate the recommendation

Why two years ago What happened two years ago which started this declineIs the dramatic increase in the ldquopopularityrdquo of skateboarding the cause of the steady decline of shoppers Are there any malls nearby Were there any changes nearby which could affect the decline in customersmdasha big mall perhaps Could the decline be due to the shop ownersHow many skateboarders use the plazaWhere do they skateboardDo they shop and are they customersAre the increase in litter and vandalism due to skateboarders Could this be alleviated by installing CCTVs and hiring security

This editorial concludes that the city should ban skateboarding from its downtown CentralPlaza in order to attract visitors to that area to return the area to its former glory and to makeit a place where people can congregate for fun and relaxation To justify this conclusion theeditorial points out that skateboarders are nearly the only people one sees anymore at CentralPlaza and that the Plaza is littered and its property defaced The editorial also points out thatthe majority of downtown merchants support the skate boarding ban This argument is flawedin several critical respectsFirst the editorials author falsely assumes that a ban on skateboarding is both necessaryand sufficient to achieve the three stated objectives Perhaps the city can achieve thoseobjectives by other means as well--for example by creating a new mall that incorporates anattractive new skateboard park Even if banning skateboarders altogether is necessary to meetthe citys goals the author has not shown that this action by itself would suffice Assuming thatthe Plazas reputation is now tarnished restoring that reputation and in turn enticing peopleback to the Plaza might require additional measures--such as removing litter and graffiti

GRE AWA John박 박정어학원

promoting the Plaza to the public or enticing popular restaurant or retail chains to the PlazaSecondly the editorial assumes too hastily that the Plazas decline is attributable to theskateboarders--rather than to some other phenomenon Perhaps the Plazas primary appeal inits glory days had to do with particular shops or eateries which were eventually replaced byless appealing ones Or perhaps the crime rate in surrounding areas has risen dramatically forreasons unrelated to the skateboarders presence at the Plaza Without ruling out these andother alternative explanations for the Plazas decline the editorials author cannot convince methat a skateboard ban would reverse that declineThirdly the editorials author might be confusing cause with effect--by assuming that theskateboarders caused the abandonment of the Plaza rather than vice versa It is entirelypossible that skateboarders did not frequent the Plaza until it was largely abandoned--andbecause it had been abandoned In fact this scenario makes good sense since skateboardingis most enjoyable where there are few pedestrians or motorists to get in the wayFourth it is unreasonable to infer from the mere fact that most merchants favor the ban thatthe ban would be effective in achieving the citys objectives Admittedly perhaps thesemerchants would be more likely to help dean up the Plaza area and promote their businesseswere the city to act in accordance with their preference Yet lacking any supporting evidencethe author cannot convince me of this Thus the survey amounts to scant evidence at best thatthe proposed ban would carry the intended resultFinally the author recommends a course of action that might actually defeat the citysobjective of providing a fun and relaxing place for people to congregate In my experienceskateboarding contributes to an atmosphere of fun and relaxation for adults and children alikemore so than many other types of ambiance Without considering that continuing to allowskateboarding--or even encouraging this activity--might achieve the citys goal more effectivelythan banning the activity the author cannot convincingly conclude that the ban would be in thecitys best interestsIn sum the argument is a specious one To strengthen it the editorials author must providedear evidence that skateboarding and not some other factor is responsible for the conditionsmarking the Plazas decline The author must also convince me that no alternative means ofrestoring the Plaza are available to the city and that the proposed ban by itself would suffice toattract tourists and restore the Plaza to its former glory Finally to better assess the argument itwould be useful to know the circumstances under which the downtown merchants would bewilling to help the city achieve its objectives

6그룹 약한 비유 빈출

The following recommendation appeared in a memo from the mayor of the town of Hopewell

Two years ago the nearby town of Ocean View built a new municipal golf course and resort hotel During the past two years tourism in Ocean View has increased new businesses have opened there and Ocean Views tax revenues have risen by 30 percent Therefore the best way to improve Hopewells economymdashand generate additional tax revenuesmdashis to build a golf course and resort hotel similar to those in Ocean View

Write a response in which you examine the stated andor unstated assumptions of the argument Be sure to explain how the argument depends on these assumptions and what the implications are for the argument if the assumptions prove unwarranted

GRE AWA John박 박정어학원

Assumptions The author assumes that OVrsquos municipal golf course and resort hotel caused tourism new businesses and increased tax revenues There may be other reasons advertising promo He assumes that this will continueAssumes that Ocean View and Hopewell are similar in many waysmdashthe name suggests otherwise OV may have always been a tourist attractions for its beaches We need to know the topography

2년전 Ocean View 시는 시정 소유 골프 및 휴양지 호텔을 신축했다 그리고 지난 2년동안 이 시의 관광객이 증가했으며 새로운 사업들이 생겨났다 그에따라 시의 세수도 30나 증가했다 Hopewell의 경제를 향상시키고 아울러 세수를 늘릴 수 있는 가장 좋은 방법은 Ocean View에 세워진 것과 같은 골프 시설과 휴양지 호텔을 신축하는 것이다

1 다른 요인으로 관광 산업이 발전했을 수도 있다 문화 유적이 발견이 되었거나 도로의 정비등으로 여행자가 늘었을 수도 있다

2 관광 산업의증가가 늘어난 세수의 원인이 아니라 새로 유입된 인구의 증가나 다른 공장에서 발생한 것일 수 있다

3 2년동안 한참 골프가 붐을 이루었을 수 있다 경제상황이 나빠지거나 다른 레포츠가 인근 지역에 생겨난다면 골프하는 사람이 줄어들 수 있다

In this memo HopeweUs mayor recommends that in order to stimulate the towns economyand boost tax revenues HopeweU should build a new golf course and resort hotel just as thetown of Ocean View did two years ago To support this recommendation the mayor points outthat in Ocean View during the last two years tourism has increased new businesses haveopened and tax revenues have increased by 30 I find the mayors argument unconvincingin several important respectsFirst of all it is possible that the mayor has confused cause with effect respecting the recentdevelopments in Ocean View Perhaps Ocean Views construction of a new golf course andhotel was a response to previous increases in tourism and business development increasesthat have simply continued during the most recent two years Since the mayor has failed toaccount for this possibility the claim that Hopewell would boost its economy by alsoconstructing a golf course and hotel is completely unwarrantedSecondly the mayor fails to account for other possible causes of the trends in Ocean Viewduring the last two years The increase in tourism might have been due to improving economicconditions nationwide or to unusually pleasant weather in the region The new businessesthat have opened in Ocean View might have opened there irrespective of the new golf courseand hotel And the 30 increase in tax revenues might have been the result of an increase intax rates or the addition of a new type of municipal taxWithout ruling out these and other alternative explanations for the three recent trends inOcean View the mayor cannot reasonably infer based on those trends that Hopewellseconomy would benefit by following Ocean Views exampleThirdly even if the recent trends in Ocean View are attributable to the construction of the newgolf course and hotel there the mayor assumes too hastily that the golf course and hotel willcontinue to benefit that towns overall economy The mayor has not accounted for thepossibility that increased tourism will begin to drive residents away during tourist season orthat new business development will result in the towns losing its appeal as a place to visit or tolive Unless the mayor can convince me that these scenarios are unlikely I cannot accept themayors recommendation that Hopewell follow Ocean Views exampleFinally the mayors argument rests on the unsubstantiated assumption that Hopewell andOcean View are sufficiently alike in ways that might affect the economic impact of a new golfcourse and hotel Hopewell might lack the sort of natural environment that would attract moretourists and new businesses to the town--regardless of its new golf course and hotel For thatmatter perhaps Hopewell already contains several resort hotels and golf courses that are notutilized to their capacity If so building yet another golf course and hotel might amount to amisallocation of the towns resources--and actually harm the towns overall economyIn sum the mayors recommendation is not well supported To bolster it the mayor mustprovide better evidence that Ocean Views new golf course and hotel and not some otherphenomenon--has been responsible for boosting Ocean Views economy during the last twoyears To better assess the recommendation I would need to know why Ocean View decidedto construct its new golf course and hotel in the first place--specifically what events prior toconstruction might have prompted that decision I would also need to thoroughly compare

GRE AWA John박 박정어학원

HopeweU with Ocean View--especially in terms of their appeal to tourists and businesses--todetermine whether the same course of action that appears to have boosted Ocean Viewseconomy would also boost Hopewells economy

The following is part of a memorandum from the president of Humana University

Last year the number of students who enrolled in online degree programs offered by nearby Omni University increased by 50 percent During the same year Omni showed a significant decrease from prior years in expenditures for dormitory and classroom space most likely because instruction in the online programs takes place via the Internet In contrast over the past three years enrollment at Humana University has failed to grow and the cost of maintaining buildings has increased along with our budget deficit To address these problems Humana University will begin immediately to create and actively promote online degree programs like those at Omni We predict that instituting these online degree programs will help Humana both increase its total enrollment and solve its budget problems

Write a response in which you discuss what questions would need to be answered in order to decide whether the prediction and the argument on which it is based are reasonable Be sure to explain how the answers to these questions would help to evaluate the prediction

Is Omni University successful due to the online degree program 50 Is the decrease in expenditures for dormitory and classroom space due to the decrease in of on-campus students Which classes were successful Does HU have those classes

Even if the long-distance degree programs at Omni University benefited the school the presidentrsquos recommendation that Human College should emulate Omni University is too hasty First OUrsquos name implies that the school would have more majors than Humanahellip the president should examine which degrees were in the long-distance programhellip

지난해에는 Omni 대학에서 개강했던 원거리 학생 학점 취득 프로그램을 등록했던 학생들의 숫자가 50나 증가했다 같은해 기간동안 Omni 대학에서는 그 전년도부터 기숙사와 학급의 공간 확충을 위한 예산을 대폭 줄였는데 이는 이 원거리 학점 취득 프로그램이 양방향 비디오 컴퓨터 접속을 통해서만 가능한 수업지도 방식이기때문인 것으로 보인다 반면 지난 3개년 동안 Humana 대학에서의 수강률은 감소한데다가 건물

GRE AWA John박 박정어학원

유지비도 올랐다 따라서 Humana대학의 수강을 늘리고 예산손실을 회복하기 위해서는 Omni 대학에서 취한 조치와 같은 능동적인 프로그램을 추진해야 한다

결론 we should initiate and actively promote long-distance degree programs like those at Omni 반박 원거리 학생 취득 프로그램 숫자가 증가한거하고 예산이 줄어드는 것 사이에 연관이 약하다 (causal 학생의 증가로 관리비용 증가할수 있음 원거리 수업가능 장비도입에의한 비용발생)bad analogy(omni university 하고 같은 조건이 아니다 )-gt omni college 가 강좌내용이 좋아서 학생의 등록이 많을수 있다 Humana 대학에서 만들었다 하더라도 인기 없을수 있음다른 요인에 의해서 Humana 대학의 수강 인원이 증가할수 있음(비록 과거엔 인기가 없었을지라도)

The following appeared as part of a business plan developed by the manager of the Rialto Movie Theater

Despite its downtown location the Rialto Movie Theater a local institution for five decades must make big changes or close its doors forever It should follow the example of the new Apex Theater in the mall outside of town When the Apex opened last year it featured a video arcade plush carpeting and seats and a state-of-the-art sound system Furthermore in a recent survey over 85 percent of respondents reported that the high price of newly released movies prevents them from going to the movies more than five times per year Thus if the Rialto intends to hold on to its share of a decreasing pool of moviegoers it must offer the same features as Apex

Write a response in which you discuss what questions would need to be answered in order to decide whether the recommendation is likely to have the predicted result Be sure to explain how the answers to these questions would help to evaluate the recommendation

Before following through this business plan the manager should investigate the cause of Rialtorsquos unsuccessful business

The author provides no evidence that the surveyrsquos results are statistically reliable The surveyrsquos sample of 85 percent must be sufficient in size and representative of overall population of the city where Rialto and Apex is serving Lacking evidence of a sufficiently representative sample the author cannot justifiably rely on the survey to draw any conclusion whatsoever The author does not indicate that Apex is indeed currently successful However even if Apex is enjoying success the argument relies on what might be a false analogy between Rialto and Apex In order for Apex to serve as a model that Rialto should emulate the author must assume that all relevant circumstances are essentially the same However this assumption is unwarranted For example the argument overlooks the face that Apex is located in a strategic placemdashbeside a mall where customers can not only watch a movie but also enjoy shopping Therefore simply changing the facility to that of Apex may not lead to success

The author does not mention whether Apex is successful or not Nevertheless even if Apex is currently successful the argument relies on what might be a false analogy between Rialto and Apex In order for Apex to serve as a model that Rialto should emulate the author must assume that all relevant circumstances are essentially the same However this assumption is unwarranted For example the argument overlooks the fact that these two institutions are located in different locations Rialto in downtown and Apex in a mall outside of town Although Apex opened with state-of-the-art facilities the decisive factor in its success could be due to its strategic location of being in a mall People could enjoy both shopping and movies at one location thus they may prefer Apex over Rialto Furthermore the place where people enjoy leisure activities has shifted in the past decades for most cities from downtown to the suburbs Therefore Rialto may not be successful even if it emulates Apexrsquos facilities A better business plan may be relocating Apex to the thriving section of the downtown

Rialto 극장은 지난 50여년간 지역 회관으로써 시내에 위치해 있으면서도 이제 변화를 꾀하지 않으면 문을 닫을

GRE AWA John박 박정어학원

판이다 이 극장은 시외 쇼핑타운에 새로 들어선 Apex 극장의 사례를 본받아야 했다 Apex가 지난해 개업했을 당시 이 극장은 비디오 아케이드 플러쉬 카펫트 바닥과 좌석 그리고 최신 음향시설을 갖추었다 더군다나 최근 조사에서는 응답자의 85 이상이 새로 출시된 영화 입장료가 비싼 탓으로 지난해보다 5배이상의 관람객이 줄어들었다고 나타났다 따라서 Rialto 극장이 줄어들고 있는 관람객을 뺐기지 않고 유지하려면 Apex와 같은 시설들을 갖추어야 할 것이다주장 리알토 극장이 줄어들고 있는 관람객을 뺐기지 않고 유지하려면 Apex와 같은 시설들을 갖추어야 할 것이다

1 조사에서 응답자가 전체를 대표할 수 없다 2 apex 극장이 좋은 시설을 갖추고 있지만 그로 인해 수익이 많이 발생했다는 말이 없으므로 시설투자를

하고도 좋은 결과를 얻을 수 있을지 그 근거가 미흡하다3 좋은 영화가 출시된다면 입장료가 비싸도 영화관에서 꼭 보려고 할 수 있다 4 rialto 가 시설이 아닌 다른 요인에 의해 장사가 안될수도 있다( 우범 지역이라든지)

The following is a recommendation from the business manager of Monarch Books

Since its opening in Collegeville twenty years ago Monarch Books has developed a large customer base due to its reader-friendly atmosphere and wide selection of books on all subjects Last month Book and Bean a combination bookstore and coffee shop announced its intention to open a Collegeville store Monarch Books should open its own in-store cafeacute in the space currently devoted to childrens books Given recent national census data indicating a significant decline in the percentage of the population under age ten sales of childrens books are likely to decline By replacing its childrens books section with a cafeacute Monarch Books can increase profits and ward off competition from Book and Bean

Write a response in which you examine the stated andor unstated assumptions of the argument Be sure to explain how the argument depends on these assumptions and what the implications are for the argument if the assumptions prove unwarranted

The following is a recommendation from the business manager of Monarch Books

Since its opening in Collegeville twenty years ago Monarch Books has developed a large customer base due to its reader-friendly atmosphere and wide selection of books on all subjects Last month Book and Bean a combination bookstore and coffee shop announced its intention to open a Collegeville store Monarch Books should open its own in-store cafeacute in the space currently devoted to childrens books Given recent national census data indicating a significant decline in the percentage of the population under age ten sales of childrens books are likely to decline By replacing its childrens books section with a cafeacute Monarch Books can increase profits and ward off competition from Book and Bean

1 Write a response in which you discuss what questions would need to be answered in order to decide whether the recommendation is likely to have the predicted result Be sure to explain how the answers to these questions would help to evaluate the recommendation

2 Write a response in which you discuss what specific evidence is needed to evaluate the argument and explain how the evidence would weaken or strengthen the argument

No evidence regarding Monarch Bookrsquos successEven if Regal Bookrsquos is successful this may not be attributable to the cafeacute False analogy Emulating may not lead to success Other factors may be involvedInsufficient condition The national census is not enough evidence that childrenrsquos book sales will decline Can

GRE AWA John박 박정어학원

the national census represent the local child populationDid opening a cafeacute boost sales for Regal Books Even assuming Regal is successful by opening a cafeacute this may not be suitable for Monarch which plans to close the childrenrsquos book section to establish a cafe Imprecise language ldquorelatively little spacerdquo how smallThe managerrsquos recommendation contradicts what he says Since Monarch is popular for its wide selection of books closing a selection which targets a major group of readers may hurt Monarchrsquos salesIs this the best way to compete

When Stanley Park first opened it was the largest most heavily used public park in town It is still the largest park but it is no longer heavily used Video cameras mounted in the parks parking lots last month revealed the parks drop in popularity the recordings showed an average of only 50 cars per day In contrast tiny Carlton Park in the heart of the business district is visited by more than 150 people on a typical weekday An obvious difference is that Carlton Park unlike Stanley Park provides ample seating Thus if Stanley Park is ever to be as popular with our citizens as Carlton Park the town will obviously need to provide more benches thereby converting some of the unused open areas into spaces suitable for socializing

Write a response in which you examine the stated andor unstated assumptions of the argument Be sure to explain how the argument depends on these assumptions and what the implications are for the argument if the assumptions prove unwarranted

Stanley 파크가 처음 개장했을 당시 가장 크고 가장 많이 이용되는 공원이었다 아직도 공원중에서는 가장 크지만 이용률은 상당히 떨어졌다 지난달 공원 주차장에 설치해놓은 비디오 카메라를 통해 보면 drop(주차장으로 여겨짐) 이용률이 가장 높았다 수치상으로는 하루 평균 50대의 차량만이 이용하였다 반면 직장 중심거리에 위치한 작은 규모의 Carlton 파크는 주당 무려 150여명 이상이 이용하고 있다 Stanley 파크와는 달리 Carlton 파크에는 의자가 있다는 것이 가장 뚜렷한 차이점이다 따라서 Stanley 파크가 Carlton 파크처럼 시민들이 자주 이용하는 공원이 되기 위해서는 벤치를 설치할 필요가 있으며 이렇게 사용되지 않는 일부 공간을 활용해서 사교를 위한 공간으로 바꾸어야 한다 ===gtdrop 에 대한 첨부사항 (영영사전내용입니다)---- a place or central depository to which something (as mail money or stolen property) is brought for distribution or transmission also the act of depositing something at such a place dropgt

주장 if Stanley Park is ever to be as popular with our citizens as is Carlton Park the town will obviously need to provide more benches thereby converting some of the unused open areas into spaces suitable for socializing1 조사가 언제 이루어진 것인가 조사가 언제 실시되었느냐에 따라 결과가 다를 수있다 현재는 다시 스탠리 파크가 늘어났었을 수 있다 2 벤치를 많이 설치했다고 해서 많은 관광객이 오지 않을수 있다(사람들이 벤치나 사교 공간을 원한다는 어떠한 자료도 없다)3스탠리 파크 주변에 교통 상황이 악화가 되었거나 칼튼 파크에서 문화행사등을 많이 가져서 이용객이 줄어든것일 수도 있다 4 칼튼 파크가 중심지에 있어서 접근성이 좋을수 있다5 조사가 같은 시간을 기준으로 한게 아니다(하나는 주중이고 하나는 주말이다)6사람의 수와 차의 대수를 같은것으로 비교할수 없다 (차안에 몇 명이 타고 있는지 모르고 대중교통을 이용해서 왔을수도 있다)

Page 17: GRE writing argument brain storm

GRE AWA John박 박정어학원

계절의 영향으로 겨울에 얼음이 두껍게 언다든지 여름에 가뭄으로 인해서 건널수 있다물자가 풍부한 것이 이동하지 않을 조건이 아니다 다른 것에 의해서 이동가능(의약품등)

Thirteen years ago researchers studied a group of 25 infants who showed signs of mild distress when exposed to unfamiliar stimuli such as an unusual odor or a tape recording of an unknown voice They discovered that these infants were more likely than other infants to have been conceived in early autumn a time when their mothers production of melatonin hormone known to affect some brain functions would naturally increase in response to decreased daylight In a follow-up study conducted earlier this year more than half of these children now teenagers who had shown signs of distress identified themselves as shy Clearly increased levels of melatonin before birth cause shyness during infancy and this shyness continues into later life

Write a response in which you examine the stated andor unstated assumptions of the argument Be sure to explain how the argument depends on these assumptions and what the implications are for the argument if the assumptions prove unwarranted

Any baby exposed to unpleasant stimuli would react in such wayFirst of all the author states 25 infants as his evidence However this research sample is too small to prove his claimSecond the author states that 25 infants were conceived in early autumn which he claims lead to a shy disposition However this is faulty evidenceThird the research study was a long term study done in the span of 13 years However the author only writes about the initial and final stages of the study and leaves out evidences of what could have happened during the 13 years which could be more evidential factors of influenceFourth neither the infantsrsquo genetic predisposition nor their environment were taken into accountFinally the author concludes that his shyness continues into later life (Other factors could alter this disposition epigenetic theory)

13 년전 학자들은 25명의 유아를 대상으로 이상한 냄새나 특이한 소리를 녹음한 테잎등으로 낯선 자극을 주었을때 보이는 미미한 압박감 증상을 조사하였다 이들은 성숙기가 막 지났을 즈음에 보통의 유아들이 비슷한 증상을 보이는 정도 보다는 다소 민감한 반응을 보였는데 이 시기는 아이의 엄마가 뇌의 일부 기능에 영향을 미치는 것으로 알려진 멜라토니아 호르몬을 생산하는 시기로써 이 호르몬은 낯 시간이 짧을때 자연적으로 증가할 수도 있다 금년초에 실시된 추가연구에서 현재 10대로 성장한 당시 조사대상의 절반 이상의 아이들이 부끄럼을 잘타는 것으로 여기고 있었다 따라서 분명한 것은 출산전 멜라토닌 수치의 증가가 유아기에 수줍음 등의 영향을 미치게 되며 이러한 영향이 성장후에도 작용한다는 것이다

주장 Clearly increased levels of melatonin before birth cause shyness during infancy and this shyness continues into later life

1 25명의 아기로 결론 내리기에 샘플이 작다2 과학적 사실들에 대한 명확한 근거가 엇음3 다른 영향을 간과했다(다른 호르몬에 의한 영향 후천적인 성격형성의 영향)

GRE AWA John박 박정어학원

The following is a letter to the editor of the Atticus City newspaper

Former Mayor Durant owes an apology to the city of Atticus Both the damage to the River Bridge which connects Atticus to Hartley and the traffic problems we have long experienced on the bridge were actually caused 20 years ago by Durant After all he is the one who approved the construction of the bridge If he had approved a wider and better-designed bridge on which approximately the same amount of public money would have been spent none of the damage or problems would have occurred Instead the River Bridge has deteriorated far more rapidly over the past 20 years than has the much longer Derby Bridge up the river Even though the winters have been severe in the past several years this is no excuse for the negligence and wastefulness of Durant

Write a response in which you discuss what questions would need to be answered in order to decide whether the recommendation is likely to have the predicted result Be sure to explain how the answers to these questions would help to evaluate the recommendation

전임 시장인 Durant 씨는 Atticus 시에 대해 사과할 의무가 있습니다 Atticus와 Hartley를 잇는 River Bridge 교량에 대한 피해와 이 교량에서 오랫동안 주민들이 겪어오고 있는 교통 혼잡 문제들은 실제로 20년 전부터 시작된 것이었습니다 결정적으로 그가 교량 공사를 허가했던 바로 그 장본인입니다 당시 비슷한 공사비용으로 폭이 더 넓고 튼튼하게 설계된 교량을 허가했다면 이러한 문제나 피해는 발생하지 않았을 겁니다 더군다나 이 다리는 지난 20년 동안 상류에 건설된 훨씬 오래된 Derby 다리보다도 빠르게 부식되어 갔습니다 지난 수년동안 심지어 혹한이 있었다 하더라도 이러한 태만과 국고 손실에 대한 책임을 회피할 길이 없는 것입니다

결론 Former Mayor Durant owes an apology to the city of Atticus

1 디자인이 문제가 아닐수 있다 (디자인은 좋았으나 건설과정에 문제가 있었을 수 있다)2 그 당시의 시예산이 적어서 더 큰 다리를 짓기가 불가능했을 수도 있다3 교통량이 많거나 다른 상황으로 인해서 부식이 빨리 됐을 수 있다4 그 당시에는 최선의 선택이였지만 갑자기 변한 상황에 의해서 이런 문제점들이 발생했을 수 있다

GRE AWA John박 박정어학원

4그룹 거짓인과관계 오류 (False Cause) 빈출

Fifteen years ago Omega University implemented a new procedure that encouraged students to evaluate the teaching effectiveness of all their professors Since that time Omega professors have begun to assign higher grades in their classes and overall student grade averages at Omega have risen by 30 percent Potential employers looking at this dramatic rise in grades believe that grades at Omega are inflated and do not accurately reflect student achievement as a result Omega graduates have not been as successful at getting jobs as have graduates from nearby Alpha University To enable its graduates to secure better jobs Omega University should terminate student evaluation of professors

Write a response in which you discuss what specific evidence is needed to evaluate the argument and explain how the evidence would weaken or strengthen the argument

Omega professor evaluation implemented 15 years ago =gt Omega prof assign higher grades 30Employers believe therersquos grade inflation

Thus unsuccessful employment than AlphaTherefore to secure jobs Omega should end evaluating profs

Specific evidence neededRelationship between higher grades and evaluationRelationship between GPA and unsuccessful employmentAlpharsquos education could just be better than OmegaldquoFifteen years agordquo is a long time other factors could have influenced Why is the inflation a problem just now How much is Alpha better Is the comparison just How much gap is thereOmegarsquos student could just be doing better in their studiesComparison to other universities다른 대안 없나hellip Could Omega alleviate the employment problem by implementing a different procedure or program

15 년전 우리 대학은 학생들로 하여금 교수평가를 하도록 한 새로운 조치를 시행했었습니다 이후 교수들은 자신의 학과 학생들에게 높은 학점을 주었으며 그에따라 학생들의 전체 평점이 30나 올랐습니다 외부의 기업체들은 분명 점수가 지나치게 부풀려졌다고 믿고 있습니다 결국 본 대학 졸업생들이 인근 Alpha 대학의 졸업자들보다 구직률이 떨어지는 이유를 잘 보여주고 있는 것입니다 이를 해결하기 위해 이제부터는 학생들에 의한 교수평가제를 중단해야 합니다

결론 Omega University should now terminate student evaluation of professors

반박 교수 평가와 학점 인플레의 연관성이 적다( 교수 평가를 먼저하고 학점을 나중에 매길수도 있다)채용기준에 성적만 있는게 아니다 학업성취의 결과 일수도 있다 Alpha 가 원래 유능했다 Alpha 의 교육내용이 좋았다

GRE AWA John박 박정어학원

In this memo the dean of Omega University(OU) recommends OU to terminate professor evaluation to secure better jobs for the students To support this recommendation the dean offers several reasons However this argument contains several logical flaws which render it unconvincing

A threshold problem with the argument involves the voluntary nature of the evaluationprocedure The dean provides no evidence about the number or percentage of Omegastudents who participate in the procedure Lacking such evidence it is entirely possible thatthose numbers are insignificant in which case terminating the procedure is unlikely to haveany effect on the grade average of Omega students or their success in getting jobs aftergraduationThe argument also assumes unfairly that the grade-average increase is the result of theevaluation procedure--rather than some other phenomenon The dean ignores a host of otherpossible explanations for the increase--such as a trend at Omega toward higher admissionstandards or higher quality instruction or facilities Without ruling out all other possibleexplanations for the grade-average increase the dean cannot convince me that by terminatingthe evaluation procedure Omega would curb its perceived grade inflation let alone help itsgraduates get jobsEven if the evaluation procedure has resulted in grade inflation at Omega the deans claimthat grade inflation explains why Omega graduates are less successful than Alpha graduatesin getting jobs is unjustified The dean overlooks a myriad of other possible reasons forOmegas comparatively poor job-placement record Perhaps Omegas career services areinadequate or perhaps Omegas curriculum does not prepare students for the job market aseffectively as Alphas In short without accounting for other factors that might contribute toOmega graduates comparative lack of success in getting jobs the dean cannot justify theclaim that if Omega curbs its grade inflation employers will be more likely to hire OmegagraduatesFinally even if the dean can substantiate all of the foregoing assumptions the deansassertion that Omega must terminate its evaluation procedure to enable its graduates to findbetter jobs is still unwarranted in two respects First the dean ignores other possible ways bywhich Omega can increase its job-placement record--for example by improving its publicrelations or career-counseling services Second the dean unfairly equates more jobs withbetter jobs In other words even if more Omega graduates are able to find jobs as a result ofthe deans recommended course of action the kinds of jobs Omega graduates find would notnecessarily be better onesIn sum the deans argument is unpersuasive as it stands To strengthen it the dean mustprovide better evidence that the increase in grade average is attributable to Omegasprofessor-evaluation procedure and that the end result is a perception on the part ofemployers that Omega graduates are less qualified for jobs than Alpha graduates To betterassess the argument I would need to analyze 15-year trends in (l) the percentage of Omegastudents participating in the evaluation procedure (2) Omegas admission standards andquality of education and (3) Omegas emphasis on job training and career preparation I wouldalso need to know what other means are available to Omega for enabling its graduates to findbetter jobs

GRE AWA John박 박정어학원

The following appeared in a memo from a vice president of Quiot Manufacturing

During the past year Quiot Manufacturing had 30 percent more on-the-job accidents than at the nearby Panoply Industries plant where the work shifts are one hour shorter than ours Experts say that significant contributing factors in many on-the-job accidents are fatigue and sleep deprivation among workers Therefore to reduce the number of on-the-job accidents at Quiot and thereby increase productivity we should shorten each of our three work shifts by one hour so that employees will get adequate amounts of sleep

Write a response in which you examine the stated andor unstated assumptions of the argument Be sure to explain how the argument depends on these assumptions and what the implications are for the argument if the assumptions prove unwarranted

The following appeared in a memo from a vice president of Alta Manufacturing

During the past year Alta Manufacturing had thirty percent more on-the-job accidents than nearby Panoply Industries where the work shifts are one hour shorter than ours Experts believe that a significant contributing factor in many accidents is fatigue caused by sleep deprivation among workers Therefore to reduce the number of on-the-job accidents at Alta we recommend shortening each of our three work shifts by one hour If we do this our employees will get adequate amounts of sleep

Write a response in which you discuss what questions would need to be answered in order to decide whether the recommendation and the argument on which it is based are reasonable Be sure to explain how the answers to these questions would help to evaluate the recommendation

The following appeared in a memo from the vice president of Butler Manufacturing

During the past year workers at Butler Manufacturing reported 30 percent more on-the-job accidents than workers at nearby Panoply Industries where the work shifts are one hour shorter than ours A recent government study reports that fatigue and sleep deprivation among workers are significant contributing factors in many on-the-job accidents If we shorten each of our work shifts by one hour we can improve Butler Manufacturings safety record by ensuring that our employees are adequately rested

1 Write a response in which you discuss what specific evidence is needed to evaluate the argument and explain how the evidence would weaken or strengthen the argument

2 Write a response in which you discuss what questions would need to be answered in order to decide whether the recommendation is likely to have the predicted result Be sure to explain how the answers to these questions would help to evaluate the recommendation

4번 반복됨

Alta has 30 more job accidents than Panoply(work shifts one hour shorter)Experts Job accidents caused by fatigue and sleep deprivationTherefore to reduce job accidents and increase productivity shorten three work shifts by one hour for adequate sleep

지난해 우리 회사는 인근 Panoply Industries보다 업무상 재해가 30나 더 많았다 그 회사는 우리보다 근무 교대시간이 1시간 정도 짧았다 전문가들은 대부분의 업무상 재해에 있어서 가장 중요한 요인이 과로와 수면부족으로 보고있다 따라서 우리 회사에서 높은 산업재해를 줄이고 아울러 생산성을 높이기 위해서는 근로자들이 충분한 수면을 취할 수 있도록 1시간씩 3교대 시간을 줄여야 한다

In this memo the (author) vice president of Alta Manufacturing (AM) recommends that to reduce on-the-job accidents and increase productivity AM should shorten its three work shifts by one hour so that employees can

GRE AWA John박 박정어학원

get more sleep To support this recommendation the author provides several evidences However careful scrutiny of each of the facts reveals that it provides little credible support for the authorrsquos recommendation QuestionsThe number of accidents What kind of accidents The seriousness of the accidents is importantHow many employees are in each company What are their productsFalse cause Sleep may not be the reason for the on-the-job accidents What do Alta and Panoply manufacture

First of all the author believes that fatigue caused the on-the job accidents However there could be other reasons The author observes a correlation between sleep deprivation and on-the-job accidents then concludes that the former is the cause of the latter However the author fails to rule out other possible explanations For example it is entirely possible that Alta factories require more strenuous and dangerous labor than Panoply Without ruling out all other such factors it is unfair to conclude that fatigue is responsible for the accidents In addition the work-shifts may not be the cause of the sleep deprivation and fatigue It is possiblehellip Thus the author should provide what exactly Panoply and Alta manufacture and more precise data about their working conditions to be more convincing

Shortening the shift by one hour does not necessarily lead to more sleep And is one hour enoughLess accidents does not mean increased productivity

결론 We should shorten each of out three work shifts by one hour

반박 경쟁사에 비해서 시간당 하는 업무량이 많아서 더욱 피곤할 수도 있다 시간이 문제가 아니라 노후된 시설 설비 자체의 문제 작업 자체가 원래 위험한 것이여서 사고가 많을 수도 있다 다른 회사는 더욱 많은 작업시간에도 불구하고 안정한 작업여건으로 인해서 사고율이 오히려 더 작을 수도 있다비교사의 재해감소가 다른 요인일수 있다(안전 교육 철저)줄인 시간이 피로회복이나 수면으로 연결 안될수 있음(술을 마실 수도 있고 그 시간에 휴식을 취하지 않고 다른일을 함으로써 더욱 피로해질수 있다)

This editorial recommends that Alta Manufacturing reduce its work shifts by one hour each inorder to reduce its on-the-job accident rate and thereby increase Altas productivity To supportthis recommendation the author points out that last year the number of accidents at Alta was30 greater than at Panoply Industries where work shifts were one hour shorter The authoralso cites certain experts who believe that many on-the-job accidents are caused by fatigueand sleep deprivation I find this the argument unconvincing for several reasonsFirst and foremost the author provides absolutely no evidence that overall workerproductivity is attributable in part to the number of on-the-job accidents Although commonsense informs me that such a relationship exists the author must provide some evidence ofthis cause-and-effect relationship before I can accept the authors final conclusion that theproposed course of action would in fact increase Altas productivitySecondly the author assumes that some accidents at Alta are caused by fatigue or sleepdeprivation However the author overlooks other possible causes such as inadequateequipment maintenance or worker training or the inherent hazards of Altas manufacturingprocesses By the same token Panoplys comparatively low accident rate might be attributablenot to the length of its work shifts but rather to other factors such as superior equipmentmaintenance or worker training In other words without ruling out alternative causes ofon-the-job accidents at both companies the author cannot justifmbly conclude that merely byemulating Panoplys work-shift policy Alta would reduce the number of such accidentsThirdly even assuming that Altas workers are fatigued or sleep-deprived and that this is thecause of some of Altas on-the-job accidents in order to accept the authors solution to thisproblem we must assume that Altas workers would use the additional hour of free time tosleep or rest However the author provides no evidence that they would use the time in thismanner It is entirely possible that Altas workers would use that extra hour to engage in someother fatiguing activity Without ruling out this possibility the author cannot convincinglyconclude that reducing Altas work shifts by one hour would reduce Altas accident rateFinally a series of problems with the argument arise from the scant statistical information onwhich it relies In comparing the number of accidents at Alta and Panoply the author fails toconsider that the per-worker accident rate might reveal that Alta is actually safer than Panoplydepending on the total number of workers at each company Second perhaps accident rates

GRE AWA John박 박정어학원

at the two companies last year were aberrations and during other years Altas accident ratewas no greater or even lower than Panoplys rate Or perhaps Panoply is not representativeof industrial companies generally and that other companies with shorter work shifts have evenhigher accident rates In short since the argument relies on very limited statistical information Icannot take the authors recommendation seriouslyIn conclusion the recommendation for emulating Panoplys work-shift policy is not wellsupported To convince me that shorter work shifts would reduce Altas on-the-job accidentrate the author must provide clear evidence that work-shift length is responsible for some ofAltas accidents The author must also supply evidence to support her final conclusion that alower accident rate would in fact increase overall worker productivity

The following appeared in a memo from the vice president of marketing at Dura-Sock Inc

A recent study of our customers suggests that our company is wasting the money it spends on its patented Endure manufacturing process which ensures that our socks are strong enough to last for two years We have always advertised our use of the Endure process but the new study shows that despite our socks durability our average customer actually purchases new Dura-Socks every three months Furthermore our customers surveyed in our largest market northeastern United States cities say that they most value Dura-Socks stylish appearance and availability in many colors These findings suggest that we can increase our profits by discontinuing use of the Endure manufacturing process

1 Write a response in which you examine the stated andor unstated assumptions of the argument Be sure to explain how the argument depends on these assumptions and what the implications are for the argument if the assumptions prove unwarranted

2 Write a response in which you discuss what specific evidence is needed to evaluate the argument and explain how the evidence would weaken or strengthen the argument

3 Write a response in which you discuss what questions would need to be answered in order to decide whether the recommendation and the argument on which it is based are reasonable Be sure to explain how the answers to these questions would help to evaluate the recommendation

Intro The vice president of marketing at Dura-Sock Inc is offering a potentially harmful investment recommendation by claiming that Dura-Sock should discontinue its use of the ldquoEndurerdquo process To support his recommendation he points out a study that Dura-Sock customers actually purchase the socks every three months and a survey that reveals that Dura-Sock customers like the sockrsquos stylish appearance and availability in many colors The study and survey however are insufficient in supporting his proposal and the VP makes several unwarranted assumptionsIntro (simplified) The VP states that though Dura-Socks last for two years customers buy the socks every three months Therefore he assumes that the consumersrsquo motive for buying the produce is not its durabilityHowever the author fails to rule out other possible motivation for consumption

Even if the survey is reliable the author should consider the rest of the market Vague terms ldquowasting moneyrdquomdashprecisely how much are they wasting Studysurvey errorThe company must calculate the outcome of such momentous decisionStudy participantsrsquo comment that they prefer Dura-Sock for its stylishness and availability might take Dura-Sockrsquos enduring quality for granted

우리회사 제품 소비자들에 대한 최근 조사에서 지난 2년여간 양말의 내구성을 강하게 하는 필수공정이었던 자사 특허의 Endure 공정에 들어가는 비용이 낭비라고 말하고 있다 우리 회사는 항상 이 공정 처리에 대한 광고를 내보냈으나 이에 대한 시장 조사에서 실제로 고객들은 이 신제품을 평균 석달마다 구매하는 것으로 나타났다 더군다나 북동부지역에서 실시한 대규모 시장조사에 응답한 고객들은 양말의 모양과 색상등에 더

GRE AWA John박 박정어학원

관심을 나타냈다 이러한 결과는 우리회사가 신기술 공법을 중단하면 그에 따라 수익이 늘어날 것이라는 것을 말해주고 있는 것이다주장 These findings suggest that Dura0Sock can increase its profits by discontinuing its use of the ldquoEndurerdquo manufacturing process

1 survey가 정확한 소비자의 의견을 나타낸 것인가 다른 선택없이 양자택일과 같은 방법의 survey였는지2 북동부지역의 시장조사가 전체 의견을 대표할 수 있나3 사람들이 모양이나 색상에 앞서 내구성을 먼저 평가했을 수도 있다 내구성을 갖추었다는 전제하에 모양과 색상에 관심을 드러낸 것일 수 있다4 소비자가 도매상(retail)인지 소매상(whole)인지가 없다

The following appeared in a business magazine

As a result of numerous complaints of dizziness and nausea on the part of consumers of Promofoods tuna the company requested that eight million cans of its tuna be returned for testing Promofoods concluded that the canned tuna did not after all pose a health risk This conclusion is based on tests performed on samples of the recalled cans by chemists from Promofoods the chemists found that of the eight food chemicals most commonly blamed for causing symptoms of dizziness and nausea five were not found in any of the tested cans The chemists did find small amounts of the three remaining suspected chemicals but pointed out that these occur naturally in all canned foods

Write a response in which you discuss what questions would need to be addressed in order to decide whether the conclusion and the argument on which it is based are reasonable Be sure to explain how the answers to the questions would help to evaluate the conclusion

Representativeness of the tested cansThey should conduct a comparative studyThe testing could be biased because Promofoods employees conducted the testingHow much (quantity) of the five and three suspected chemicals were in the canned foodsFalse cause The substance that caused dizziness and nausea may not be one of the eight common chemicals

많은 소비자들의 현기증과 구역질 불만에 따라 Promofoods사는 지난해 참치 캔 8백만 개를 테스트하기 위해 반품시켰다 그 결과 캔에서는 건강에 위험이 될 수 있는 화합물질이 없었던 것으로 회사측은 결론지었다 이러한 결론은 회사측 화학연구자들이 회수된 캔의 샘플을 테스트해서 이들 증상의 원인이 되는 8가지 화합물 중에서 5가지가 실험된 캔에서 발견되지 않았다는 사실에 근거한 것이다 이들 화학자들은 나머지 3개가지 화합물이 모든 캔 식료품에서 흔히 발견되는 것이라고 언급했다 결론 Promofoods concluded that the cans did not after all contain chemicals that posed a health risk

1 공인된 기간에서 테스트를 한 것이 아니고 자사에서 직접 테스트를 했기에 신뢰성이 안간다 2 이런 증상을 일으키는 8개의 물질 말고 다른 물질들이 캔속에 많이 포함됬을수 있다 3 나머지 3개의 물질들의 함유량이 많아서 다른 종류의 캔들은 문제를 일으키지 않지만 참치캔은 문제를

일으킬 수 있다

This magazine article concludes that the 8 million cans of tuna Promofoods recalled due tocomplaints about nausea and dizziness do not after ail contain any chemicals that pose a

GRE AWA John박 박정어학원

health risk To support this conclusion the author cites the fact that five of eight chemicalscommonly causing these symptoms were not found in the recalled cans while the other threealso occur naturally in other canned foods For several reasons this evidence lends littlecredible support to the authors conclusionTo begin with the author relies partly on the fact that although three of the eight chemicalsmost commonly blamed for nausea and dizziness appeared in Promofoods recalled tunathese chemicals also occur naturally in other canned foods However this fact alone lends nosupport to the authors conclusion for two reasons First the author might be ignoring animportant distinction between naturally occurring chemicals and those not occurring naturallyIt is entirely possible that these three chemicals do not occur naturally in Promofoods tunaand that it is for this reason that the chemicals cause nausea and dizziness Secondly it isentirely possible that even when they occur naturally these chemicals cause the samesymptoms Unless the author rules out both possibilities he cannot reliably conclude that therecalled tuna would not cause these symptomsAnother problem with the argument is that the authors conclusion is too broad Based onevidence about certain chemicals that might cause two particular heath-related symptoms theauthor concludes that the recalled tuna contains no chemicals that pose a health risk Howeverthe author fails to account for the myriad of other possible health risks that the recalled tunamight potentially pose Without ruling out all other such risks the author cannot justifiablyreach his conclusionA third problem with the argument involves that fact that the eight particular chemicals withwhich the test was concerned are only the eight most commonly blamed for nausea anddizziness It is entirely possibly that other chemicals might also cause these symptoms andthat one or more of these other chemicals actually caused the symptoms Without ruling outthis possibility the author cannot jusufiably conclude that the recalled tuna would not causenausea and dizzinessA final problem with the argument involves thetesting procedure itself The author providesno information about the number of recaUed cans tested or the selection method used Unlessthe number of cans is a sufficiently large sample and is statistically repre sentative of all therecalled cans the studys results are not statistically reliableIn conclusion the article is unconvincing as it stands To strengthen the assertion that therecalled tuna would not cause nausea and dizziness the author must provide evidence thatthe three chemicals mentioned that occur naturally in other canned foods also appear naturallyin Promofoods tuna The author must also provide evidence that ingesting other canned foodscontaining these three chemicals does not cause these symptoms To better evaluate theargument we would need to know whether the sample used in the tests was statisticallysignificant and representative of all the recalled tuna We would also need to know what otherchemicals in the recalled tuna might pose any health risk at all

5그룹 불충분 조건오류 빈출

Natures Way a chain of stores selling health food and other health-related products is opening its next franchise in the town of Plainsville The store should prove to be very successful Natures Way franchises tend to be most profitable in areas where residents lead healthy lives and clearly Plainsville is such an area Plainsville merchants report that sales of running shoes and exercise clothing are at all-time highs The local health club has more members than ever and the weight training and aerobics classes are always full Finally Plainsvilles schoolchildren represent a new generation of potential customers these schoolchildren are required to participate in a fitness-for-life program which emphasizes the benefits of regular exercise at an early age

Write a response in which you examine the stated andor unstated assumptions of the argument Be sure to

GRE AWA John박 박정어학원

explain how the argument depends on these assumptions and what the implications are for the argument if the assumptions prove unwarranted

False cause

First of all the author believes that the Increased sales of running shoes and exercise clothing indicates

Plainesville residentsrsquo interest in leading healthy lives However this assumption is not logically convincing for

several reasons could be a fashion trendTime shift ldquoFitness for liferdquo might not have any influence on schoolchildren as they growFalse cause There could be other reasons for member increase in the health clubAll of the above are insufficient condition

The author has to prove that local residents are interested in leading healthy lives However he supports his conclusion with insufficient evidence Nevertheless even if the residents are concerned with health naturersquos way may not be successful First

그 동안의 경험을 토대로 볼 때 건강생활과 밀접히 관련되어 있는 거주 지역에서 본 상점들이 아주 호응을 얻고 있다 따라서 이러한 주민들이 많이 거주하고 있는 Plainsville 에 새로운 상점들을 계속 세워야 한다 이 지역 상인들은 런닝화와 운동복 판매가 가장 높다고 말한다 불과 5 년전에는 거의 전무하다시피하던 지역 헬스 클럽의 경우도 엄청나게 많은 회원을 확보하고 있으며 웨이트 트레이닝과 에어로빅 강좌들도 항상 만원이라고 한다 새로운 고객층을 예측해 보는 것도 가능하다 이 지역의 학생들의 경우 Fitness for Life프로그램을 받게 되는데 이러한 프로그램을 통해서 유년시절부터 정규적인 운동 습관을 들이게 하고 있는 것이 그것이다

결론 We should therefore build our next new store in Plainsville

반박 그동안의 경험에 의한 과거 통계가 꼭 여기에도 적용되는건 아니다 5 년전 헬스 클럽이 잘 안되었던게 다른 원인이였을수 있다(강사수준 미달 강좌미비)tourist 에 의한 원인 일수 있다 어렸을때부터 운동을 했다고 해서 커서도 관심이 있지는 않다 (오히려 반감이 있을수 있다 혹은 건강하기에 건강에 관심이 적을수도 있다)운동복이나 신발의 판매가 육체노동에 의한 것일수도 있다

IntroductionSupport1049896In this memorandum the author asserts that Naturersquos Way should build its next newstore in Plainsville To support this assertion the author states that Plainsvillesmerchantsrsquo sales of exercise clothing are going well the local health club has moremembers than ever and a new generation of customers will help to ensure NaturersquosWayrsquos success At first glance the authorrsquos assumption seems convincing but in-depth scrutiny revealsthat it lacks substantial evidence as it stands

Body 1-SamplingTopic Sentence 1To begin with the author assumes that the merchantsrsquo report indicates that the residentsare concerned about their health However this assumption is based on unsubstantiated

GRE AWA John박 박정어학원

data Example 1 (Rebuttal1) First if we do not know the total volume of items sold and the price of the goods exactly we cannot infer whether the residents are actually buying many goods Example 2 (Rebuttal2)In addition to that the report emphasizes the rising sales of running shoes and exerciseclothing however these may not be hot-selling items for Naturersquos Way or may not be theproducts the company is planning to sell Concluding Sentence Therefore in order to make the argument reliable the author should reconsider themerchantsrsquo report with more detailed data

Body 2-CausalTopic Sentence 2Second the author contends that the health clubs classes are full yet this does not meanthat many people actually use the health club other factors may be the real cause forthose closed classes Example 1 (Rebuttal1) To begin with if the health club is very small the number of people working out wouldnot be a large one In fact regular gym-going may just be a vogue among a smallunrepresentative segment of Plainsvilles population Example 2 (Rebuttal2) Moreover it is possible that most of the people who exercise in the health club do weight training and aerobics only to look good and to meet other singles not for their health In that case there would be little demand for health products Concluding SentenceThus the author should not hasten to presume what really caused people to be interested in a healthier lifestyle and enroll in the health club

Body 3-Time-ShiftTopic Sentence 3Finally the author highlights that Naturersquos Way can expect a new generation of customersin Plainsville that will help the company in the long term This notion is mistaken in that itassumes the conditions of the present will continue unchanged in the future Although theschool children are required to participate in the fitness for life program they may notnecessarily buy Naturersquos Ways products Example 1 (Rebuttal1) In the first instance they may suffer a fall in purchasing power arising from future economic difficulties this would cause reluctance to spend a considerable amount of money on health products which tend to be more expensive Example 2 (Rebuttal2)Another possibility is that there may emerge many competitor companies vying with Naturersquos Way so that in the future the school children may not feel the necessity to purchase one companyrsquos health products over anotherrsquosConcluding Sentence Thus the authorrsquos assumption is highly speculative since it relies heavily on unknowablefuture circumstances

ConclusionThesis In sum the author uses many assumptions that are insufficient in supporting his claimsSupportIn order for the authorrsquos claims to be convincing he needs to advance more persuasiveevidence that people in Plainsville really are concerned with their health and health foodThe following was written as a part of an application for a small-business loan by a group of developers in the city of Monroe

A jazz music club in Monroe would be a tremendously profitable enterprise Currently the nearest jazz club is 65 miles away thus the proposed new jazz club in Monroe the C-Note would have the local market all to itself Plus jazz is extremely popular in Monroe over 100000 people attended Monroes annual jazz festival last summer several well-known jazz musicians live in Monroe and the highest-rated radio program in Monroe is Jazz Nightly which airs every weeknight at 7 PM Finally a nationwide study indicates that the typical jazz fan spends close to $1000 per year on jazz entertainment

1 Write a response in which you discuss what specific evidence is needed to evaluate the argument and explain how the evidence would weaken or strengthen the argument

2 Write a response in which you examine the stated andor unstated assumptions of the argument Be

GRE AWA John박 박정어학원

sure to explain how the argument depends on these assumptions and what the implications are for the argument if the assumptions prove unwarranted

3 Write a response in which you discuss what questions would need to be answered in order to decide whether the prediction and the argument on which it is based are reasonable Be sure to explain how the answers to these questions would help to evaluate the prediction

Group error nationwide survey may not reflect local trends Is the nationwide jazz fan population substantialInsufficient non-residents of Monroe may have attended the jazz festival (Body alternative explanation last year may have been an anomaly The author should consider data from various years) The author should indicate how many out of 100000 were Monroe residentsNationwide study Does this reflect Insufficient Citizens of Monroe may continue to go to the jazz club 65 miles away

Are the people in Monroe really interested in jazzMajority of the people who attended the jazz festival might not be Monroe residentsSurvey error nationwide study may not be applicable to MonroeJazz musicians who live in MonroeMonopolyRadio station

In this business application the author claims that the proposed jazz club C Note will be very profitable in Monroe To support this claim the author argues for his case with several evidences At first glance the authorrsquos argument seems convincing however careful scrutiny reveals that his argument in specious

To begin with the author claims that Monroersquos citizens are interested in jazz He presents three evidences First Secondhellip Thirdhellip Howeverhellip

Monroe 시에 있는 재즈 음악 클럽은 수익성이 좋은 사업이다 현재 가장 가까이에 있는 클럽은 65 마일 정도 떨어져 있다 따라서 이번에 세우려고 하는 C Note 는 독보적인 위치를 점할것이다 더군다나 재즈는 이 시에서 가장 인기있는 음악이다 지난 여름 재즈 축제에서는 10 만명 이상의 Morone 시 주민이 참석하였고 몇몇 유명한 재즈 음악가들도 이곳에 살고 있으며 저녁때 방영되는 라디오 프로그램중에서 최고의 시청률을 보이고 있는 것도 Jazz Nightly 이다 전국조사에서도 전형적인 재즈 팬들은 재즈 분야에 년간 1천 달러 가까이 지출하고 있는 것으로 보고되고 있다 따라서 C Note 클럽이 돈을 벌 수 있는 사업이라는 것은 확실한 것이다

결과 It is clear that the C Note cannot help but make money반박 nearest jazz club 이 양질의 써비스로 여전히 손님을 끌수도 있다Festival 에 얼마나 참여하는지가 jazz 의 인기를 반영하지 않는다 뮤지션이 많이 사는거랑 jazz 의 인기가 상관없다라디오 프로그램이 다른 요인에 의해서 인기일수도 있다 (진행자때문)전국 통계 적용 불가화목 실전반_Ms Noh6In this application the author suggests that a jazz club in Monroe will make a number of profits To support this suggestion the author exemplifies the local condition popularity of jazz in Monroe and nationwide study However careful scrutiny of each of the facts reveals that it provides little credible support for the authorrsquos recommendation Good clear intro

First the author assumes that jazz is popular in Monroe because of several facts the jazz festival last year had high participation some famous jazz musicians live in Monroe and the high-rated radio program is lsquoJazz Nightlyrsquo However this assumption has many drawbacks that must be seriously considered(Good topic sentences) If many attendants in the last-yearrsquos festival came from other cities and not Monroe it is hard to conclude that Monroersquos people like jazz Therefore the author must examine how many Monroe residents actually attended the festival On top of that there is little relationship between habitation of famous jazz musician and the popularity of jazz in Monroe Although several well-known musicians live there if they do not take part in any jazz performance of Monroe this might have no effect to the interest of Monroersquos residents

GRE AWA John박 박정어학원

about jazz Finally in the case of radio program this is also not suitable reason why jazz is popular in Monroe It might be possible that people cannot help choosing lsquoJazz Nightlyrsquo because there are few radio programs at Night The fact that the radio program is the highest rating program is not a germane evidence The approximate number of listeners would be the more crucial evidence Therefore the author needs to seriously deliberate the correlation between jazzrsquos popularity in Monroe and his examples (Good logical flow and clarity)

Second the author uses as evidence the nationwide study that jazz fans spend much money on jazz entertainment to substantiate why starting a jazz club in Monroe will be profitable In other words the author assumes that the characteristics of a nationwide study can be applied to Monroe The national study would lend support to the applicantrsquos claim only if residents in Monroe typify national jazz fans However the author does not provide credible evidence that this is the case Moreover the populations of jazz fans nationwide may be insubstantial Thus the author should not infer hastily that Monroersquos residents will spend much money on enjoying jazz from the nationwide study

Lastly even if jazz is popular in Monroe C Note may not be successful It is entirely possible that residents might still prefer other clubs where they have always went In addition there is another possibility that the nearest jazz club will attract many of Monroersquos people because it serves fine performances and is equipped with favorite facilities Without considering these other possibilities the author cannot make his argument convincing In sum the author presents many reasons that are insufficient in supporting his or her claim In order for the authorrsquos claims to be convincing he needs to advance more persuasive evidence such as the total number of Monroe residents who attended the jazz festival the effects on the popularity of jazz by the musicians living in Monroe and the actual number of residents who would typify themselves to be jazz fans through a local survey Without substantial evidence that C Note will be successful in Monroe the businessmen may be overinvesting in what might lead to a business failureExcellent clarity Score 50

The following appeared in a newsletter offering advice to investors

Over 80 percent of the respondents to a recent survey indicated a desire to reduce their intake of foods containing fats and cholesterol and today low-fat products abound in many food stores Since many of the food products currently marketed by Old Dairy Industries are high in fat and cholesterol the companys sales are likely to diminish greatly and company profits will no doubt decrease We therefore advise Old Dairy stockholders to sell their shares and other investors not to purchase stock in this company

Write a response in which you discuss what questions would need to be answered in order to decide whether the advice and the argument on which it is based are reasonable Be sure to explain how the answers to these questions would help to evaluate the advice

Survey 80

GRE AWA John박 박정어학원

Old Dairy could change their products and manufacture low fat dairy foodsLess competing companies Old Dairy could eventually be the only company that produces hellipImprecise numbers and measurementsCustomers may still buy high fat dairy products

The author of the newsletter is offering potentially dangerous advice by recommending Old Dairy stockholders to withdraw investment and stop purchase What is more the authorrsquos prediction debases the reputation and business of Old Dairy and if false could devoid the investment opportunity of the newsletter readers Therefore investors should examine whether the authorrsquos evidences are substantial

To begin with the author states that 80 percent of the respondents in a survey indicated a desire to reduce their intake of foods He therefore argues that Old Dairyrsquos high fat and cholesterol products would decrease in sales However the author makes a crucial error in this argument First the author provides no evidence that the surveyrsquos results are statistically reliable Were they representative of all the customers Were they chosen for the survey randomly Furthermore the desire to reduce fat and cholesterol intake is a pervasive trend in todayrsquos opulent society however the author erroneously identifies this as a new phenomenon which will affect consumer trends Second having a desire to reduce fat and cholesterol intake does not necessarily indicate that people who have this desire will actually reduce consuming these types of products It is entirely possible that they may continue buying Old Dairy products for its quality and taste Accordingly the author cannot draw any firm conclusion that people will not buy Old Dairy products Therefore if any of these cases are true the author may be offering investors a detrimental investment advice

최근 조사에 대한 응답자중 80 이상이 자신이 먹는 음식에서 지방과 콜레스테롤의 함유량을 줄이고 싶다고 한다 아울러 요즘은 많은 식료품 가계에서 저지방 제품들을 많이 취급하고 있다 현재 Old Dairy Industries가 판매하고 있는 많은 음식제품들은 지방과 콜레스테롤이 높기 때문에 이 회사의 매출이 격감할 것으로 보이며 당연히 매출이익도 줄어들것이다 따라서 이 회사의 주주들은 주식을 매각하고 다른 주식 투자가들도 이 회사의 주식을 매입하지 않는 것이 좋다

결론 Old Dairy stockholders to sell their shares and other investors not to purchase stock in this company

반박 모든 상품이 다 고 지방 고 칼로리는 아니다(비록 많을지라도) 일부의 식품의 경우 기호에 맞어서 히트해서 전체적인 수입이 증가할 수도 있다국내시장만 생각할 수 없다( 외국시장에서 호황을 누릴수 있다 )입맛이라는게 즉각 바뀌는게 아니다

The following appeared in a letter to the editor of the Balmer Island Gazette

On Balmer Island where mopeds serve as a popular form of transportation the population increases to 100000 during the summer months To reduce the number of accidents involving mopeds and pedestrians the town council of Balmer Island should limit the number of mopeds rented by the islands moped rental companies from 50 per day to 25 per day during the summer season By limiting the number of rentals the town council will attain the 50 percent annual reduction in moped accidents that was achieved last year on the neighboring island of Seaville when Seavilles town council enforced similar limits on moped rentals

1 Write a response in which you discuss what questions would need to be answered in order to decide whether the recommendation is likely to have the predicted result Be sure to explain how the answers to these questions would help to evaluate the recommendation

2 Write a response in which you discuss what questions would need to be answered in order to decide whether the prediction and the argument on which it is based are reasonable Be sure to explain how the answers to these questions would help to evaluate the prediction

3 Write a response in which you examine the stated andor unstated assumptions of the argument Be sure to explain how the argument depends on these assumptions and what the implications are for the argument if the assumptions prove unwarranted

Whatrsquos the actual population of Balmer Island 100000mdashis this a significant increase What kind of accidents Skin abrasions or serious injury And compared to Seaville how serious are the accidents and the actual number of accidents Did Seaville enforce other restrictions like safety signsHow different are the conditions of Balmer

GRE AWA John박 박정어학원

and Seaville regarding population road (safety) conditions topography other town-government regulation How much will the economy of Balmer be affected do to this restriction Could it cause an economic recession due to the fact that these rental companiesrsquo chance to make money is only during the summer thereby weakening the economic infrastructure Are there any other ways that could better alleviate the accident rate

Statistics 50-impreciseAnalogy Balmer compared with TorseauFalse Cause Accidents might have occurred because of reasons other than mopeds False Cause population increase may not be part of the cause of the accidentsOther explanations for the accident pedestrians few road safety regulations narrow roadsThere could be other better solutionshellip

Balmer Island의 인구가 여름철에는 십만명으로 늘어난다 2륜차와 보행자간 사고를 줄이기 위해 시의회는 6개의 자전거를 포함한 2륜차 대여업체에게 이 기간동안에는 대여숫자를 일일 50에서 30으로 제한하도록 할 것이다 대여숫자를 줄임으로써 시 의회는 지난해 이웃한 Torseau섬에서 이와 동일한 규제를 시행해서 50나 줄인 결과를 보고 마찬가지로 50를 줄일수 있다고 확신하고 있다

결론 The town council of Balmer Island should linit the number

반박 보행자의 부실에 의해서 사고가 많이 일어날수도 있다렌탈수의 줄임만이 대책은 아니다(대부분의 사람들이 렌탈 보다는 소유하고 있을 수도 있다)옆섬과는 상황이 다를수도 있다(그 섬에서는 사고의 원인이 많은 자전거 수로 인한것일수있다) 하지만 이 섬은 좁은 도로가 원인일 수도 있고 도로 안전 장치의미비가 원일일수 있다

In this letter the author recommends that Balmer Island should limit the number moped rentals from 50 to 30 per day To support this recommendation the author points out several reasons However careful scrutiny of each of the facts reveals that it is filled with unanswered questions that could significantly weaken the authorrsquos recommendation with loops and holes which are answered

The recommendation depends on the assumption that no alternative means of reducing the number of accidents are available However the author fails to offer any evidence to substantiate this crucial assumption It is highly possible that means other than this would better solve the problem Perhaps they could widen the roads or put-up more safety signs Or perhaps the accidents were due to the lack of skills in which case proper safety training would significantly alleviate the problem Without considering and ruling out these and other alternative means of reducing accidetns the author cannot confidently conclude that merely emulating Torseau would suffice Moreover the author is advising a recommendation which could potentially harm the economy of Balmer Island sincehellip Moreover the Balmer Island should alternative means to reduce accidents because limiting moped rentals during the summer could harm the economy of Balmerhellip

First of all the author believes that increase in population and the number of moped rentals are responsible for the accidents It is entirely possible that other factors are responsible for the accidents Perhaps Balmer Islandrsquos lack of safety signs was a major factor Or maybe the roads are narrow and dangerous on the Island therefore the town council could enforce stricter traffic regulations to alleviate the problem Accordingly if either of these scenarios is true the author cannot draw any firm conclusion that increase in the number of population and moped rentals are the cause of the accidents

The author of this editorial recommends that to reduce accidents involving mopeds andpedestrians Balmer Islands city council should restrict moped rentals to 30 per day down from50 at each of the islands six rental outlets To support this recommendation the author citesthe fact that last year when nearby Torseau Islands town council enforced similar measuresTorseaus rate of moped accidents fell by 50 For several reasons this evidence providesscant support for the authors recommendationTo begin with the author assumes that all other conditions in Balmer that might affect therate of moped-pedestrian accidents will remain unchanged after the restrictions are enactedHowever with a restricted supply of rental mopeds people in Balmer might purchase mopedsinstead Also the number of pedestrians might increase in the future with more pedestriansespecially tourists the risk of moped-pedestrian accidents would probably increase For thatmatter the number of rental outlets might increase to make up for the artificial supplyrestriction per outlet--a likely scenario assuming moped rental demand does not declineWithout considering and ruling out these and other possible changes that might contribute to ahigh incidence of moped-pedestrian accidents the author cannot convince me that theproposed restrictions will necessarily have the desired effect

GRE AWA John박 박정어학원

Next the author fails to consider other possible explanations for the 50 decline inTorseaus moped accident rate last year Perhaps last year Torseau experienced unusually fairweather during which moped accidents are less likely Perhaps fewer tourists visited Tot seanlast year than during most years thereby diminishing the demand for rental mopeds to belowthe allowed limits Perhaps last year some of Torseaus moped rental outlets purchased newmopeds that are safer to drive Or perhaps the restrictions were already in effect but were notenforced until last year In any event a decline in Torseaus moped accident rate during onlyone year is scarcely sufficient to draw any reliable conclusions about what might have causedthe decline or about what the accident rate will be in years aheadAdditionally in asserting that the same phenomenon that caused a 50 decline in mopedaccidents in Torseau would cause a similar decline in Balmer the author relies on what mightamount to an unfair analogy between Balmer and Torseau Perhaps Balmers ability to enforcemoped-rental restrictions does not meet Torseaus ability if not then the mere enactment ofsimilar restrictions in Balmer is no guarantee of a similar result Or perhaps the demand formopeds in Torseau is always greater than in Balmer Specifically if fewer than all availablemopeds are currently rented per day from the average Balmer outlet while in Torseau everyavailable moped is rented each day then the proposed restriction is likely to have less impacton the accident rate in Balmer than in TorseauFinally the author provides no evidence that the same restrictions that served to reduce theincidence of all moped accidents by 50 would also serve to reduce the incidence ofaccidents involving mopeds and pedestrians by 50 Lacking such evidence it is entirelypossible that the number of moped accidents not involving pedestrians decreased by a greaterpercentage while the number of moped-pedestrian accidents decreased by a smallerpercentage or even increased Since the author has not accounted for these possibilities theeditorials recommendation cannot be taken seriouslyIn conclusion the recommendation is not well supported To convince me that the proposedrestriction would achieve the desired outcome the author would have to assure me that nochanges serving to increase Balmers moped-pedestrian accident rate will occur in theforeseeable future The author must also provide dear evidence that last years decline inmoped accidents in Torseau was attributable primarily to its moped rental restrictions ratherthan to one or more other factors In order to better evaluate the recommendation I wouldneed more information comparing the supply of and demand for moped rentals on the twoislands I would also need to know the rate of mopedpedestrian accidents in Torseau both priorto and after the restrictions were enforced in TorseauThe following appeared in a magazine article about planning for retirement

Clearview should be a top choice for anyone seeking a place to retire because it has spectacular natural beauty and a consistent climate Another advantage is that housing costs in Clearview have fallen significantly during the past year and taxes remain lower than those in neighboring towns Moreover Clearviews mayor promises many new programs to improve schools streets and public services And best of all retirees in Clearview can also expect excellent health care as they grow older since the number of physicians in the area is far greater than the national average

Write a response in which you discuss what specific evidence is needed to evaluate the argument and explain how the evidence would weaken or strengthen the argument

-Natural beauty and consistent climate may not be the most wanted qualities-Housing costs could have lowered on a national level wealthy retirees may not care about costs-Taxes may be high compared to the nationrsquos average tax rate-What about other qualities of Clearview Crime rate what qualities would retirees want -If schools streets and public services need improvement then this is proof that the current condition of Clearview is low Or due to budgetary reasons the mayor may not follow-up on his promise because of lowered tax rate -Schools and people who are retired no relationship-Physicians What kind of physicians Number is irrelevant Are these physicians capable of addressing the illnesses of old people

This author argues that anyone seeking a place to retire should choose Clearview To supportthis argument the article cites Clearviews consistent climate and natural beauty its fallinghousing costs its low property taxes compared to nearby towns and the mayors promise toimprove schools streets and services The article also claims that retirees can expectexcellent health care because the number of physicians in Clearview greatly exceeds thenational average This argument is flawed in several critical respectsTo begin with although consistent climate and natural beauty might be attractive to manyretirees these features are probably not important to all retirees For many retirees it isprobably more important to live near relatives or even to enjoy changing seasons Thus I

GRE AWA John박 박정어학원

cannot accept the authors sweeping recommendation for all retirees on this basisAlso Clearviews declining housing costs do not necessarily make Clearview the best placeto retire for two reasons First despite the decline Clearviews housing costs might be highcompared to housing costs in other cities Secondly for wealthier retirees housing costs arenot likely to be a factor in choosing a place to retire Thus the mere fact that housing costshave been in decline lends scant support to the recommendationThe articles reliance on Clearviews property-tax rates is also problematic in two respectsFirst retirees obviously have innumerable choices about where to retire besides Clear viewand nearby towns Secondly for retirees who are well-off financially property taxes are notlikely to be an important concern in choosing a place to retire Thus it is unfair to infer fromClearviews property-tax rates that retirees would prefer ClearviewYet another problem with the argument involves the mayors promises In light of Clearviewslow property-tax rates whether the mayor can follow through on those promises is highlyquestionable Absent any explanation of how the city can spend more money in the areas citedwithout raising property taxes I simply cannot accept the editorials recommendation on thebasis of those promises Besides even if the city makes the improvements promised thoseimprovements--particular the ones to schools--would not necessarily be important to retireesFinally although the number of physicians in Clearview is relatively high the per capitanumber might be relatively low Moreover it would be fairer to compare this per capita numberwith the per capita number for other attractive retirement towns--rather than the nationalaverage After all retirees are likely to place a relatively heavy burden on health-careresources Besides the article provides no assurances that the number of physicians inClearview will remain high in the foreseeable futureIn conclusion the recommendation is poorly supported To strengthen it the author mustconvince me--perhaps by way of a reliable survey--that the key features that the vast majorityof retirees look for in choosing a place to live are consistent climate natural beauty and lowhousing costs The author must also provide better evidence that Clear views property taxesare lower than the those of cities in other areas The author must also explain how the city canmake its promised improvements without raising property taxes Finally to better assess theargument I would need to now how the per capita number of physicians in Clearview wouldcompare to the national average in the futureThe following appeared as a letter to the editor from a Central Plaza store owner

Over the past two years the number of shoppers in Central Plaza has been steadily decreasing while the popularity of skateboarding has increased dramatically Many Central Plaza store owners believe that the decrease in their business is due to the number of skateboard users in the plaza There has also been a dramatic increase in the amount of litter and vandalism throughout the plaza Thus we recommend that the city prohibit skateboarding in Central Plaza If skateboarding is prohibited here we predict that business in Central Plaza will return to its previously high levels

Write a response in which you discuss what questions would need to be answered in order to decide whether the recommendation is likely to have the predicted result Be sure to explain how the answers to these questions would help to evaluate the recommendation

Why two years ago What happened two years ago which started this declineIs the dramatic increase in the ldquopopularityrdquo of skateboarding the cause of the steady decline of shoppers Are there any malls nearby Were there any changes nearby which could affect the decline in customersmdasha big mall perhaps Could the decline be due to the shop ownersHow many skateboarders use the plazaWhere do they skateboardDo they shop and are they customersAre the increase in litter and vandalism due to skateboarders Could this be alleviated by installing CCTVs and hiring security

This editorial concludes that the city should ban skateboarding from its downtown CentralPlaza in order to attract visitors to that area to return the area to its former glory and to makeit a place where people can congregate for fun and relaxation To justify this conclusion theeditorial points out that skateboarders are nearly the only people one sees anymore at CentralPlaza and that the Plaza is littered and its property defaced The editorial also points out thatthe majority of downtown merchants support the skate boarding ban This argument is flawedin several critical respectsFirst the editorials author falsely assumes that a ban on skateboarding is both necessaryand sufficient to achieve the three stated objectives Perhaps the city can achieve thoseobjectives by other means as well--for example by creating a new mall that incorporates anattractive new skateboard park Even if banning skateboarders altogether is necessary to meetthe citys goals the author has not shown that this action by itself would suffice Assuming thatthe Plazas reputation is now tarnished restoring that reputation and in turn enticing peopleback to the Plaza might require additional measures--such as removing litter and graffiti

GRE AWA John박 박정어학원

promoting the Plaza to the public or enticing popular restaurant or retail chains to the PlazaSecondly the editorial assumes too hastily that the Plazas decline is attributable to theskateboarders--rather than to some other phenomenon Perhaps the Plazas primary appeal inits glory days had to do with particular shops or eateries which were eventually replaced byless appealing ones Or perhaps the crime rate in surrounding areas has risen dramatically forreasons unrelated to the skateboarders presence at the Plaza Without ruling out these andother alternative explanations for the Plazas decline the editorials author cannot convince methat a skateboard ban would reverse that declineThirdly the editorials author might be confusing cause with effect--by assuming that theskateboarders caused the abandonment of the Plaza rather than vice versa It is entirelypossible that skateboarders did not frequent the Plaza until it was largely abandoned--andbecause it had been abandoned In fact this scenario makes good sense since skateboardingis most enjoyable where there are few pedestrians or motorists to get in the wayFourth it is unreasonable to infer from the mere fact that most merchants favor the ban thatthe ban would be effective in achieving the citys objectives Admittedly perhaps thesemerchants would be more likely to help dean up the Plaza area and promote their businesseswere the city to act in accordance with their preference Yet lacking any supporting evidencethe author cannot convince me of this Thus the survey amounts to scant evidence at best thatthe proposed ban would carry the intended resultFinally the author recommends a course of action that might actually defeat the citysobjective of providing a fun and relaxing place for people to congregate In my experienceskateboarding contributes to an atmosphere of fun and relaxation for adults and children alikemore so than many other types of ambiance Without considering that continuing to allowskateboarding--or even encouraging this activity--might achieve the citys goal more effectivelythan banning the activity the author cannot convincingly conclude that the ban would be in thecitys best interestsIn sum the argument is a specious one To strengthen it the editorials author must providedear evidence that skateboarding and not some other factor is responsible for the conditionsmarking the Plazas decline The author must also convince me that no alternative means ofrestoring the Plaza are available to the city and that the proposed ban by itself would suffice toattract tourists and restore the Plaza to its former glory Finally to better assess the argument itwould be useful to know the circumstances under which the downtown merchants would bewilling to help the city achieve its objectives

6그룹 약한 비유 빈출

The following recommendation appeared in a memo from the mayor of the town of Hopewell

Two years ago the nearby town of Ocean View built a new municipal golf course and resort hotel During the past two years tourism in Ocean View has increased new businesses have opened there and Ocean Views tax revenues have risen by 30 percent Therefore the best way to improve Hopewells economymdashand generate additional tax revenuesmdashis to build a golf course and resort hotel similar to those in Ocean View

Write a response in which you examine the stated andor unstated assumptions of the argument Be sure to explain how the argument depends on these assumptions and what the implications are for the argument if the assumptions prove unwarranted

GRE AWA John박 박정어학원

Assumptions The author assumes that OVrsquos municipal golf course and resort hotel caused tourism new businesses and increased tax revenues There may be other reasons advertising promo He assumes that this will continueAssumes that Ocean View and Hopewell are similar in many waysmdashthe name suggests otherwise OV may have always been a tourist attractions for its beaches We need to know the topography

2년전 Ocean View 시는 시정 소유 골프 및 휴양지 호텔을 신축했다 그리고 지난 2년동안 이 시의 관광객이 증가했으며 새로운 사업들이 생겨났다 그에따라 시의 세수도 30나 증가했다 Hopewell의 경제를 향상시키고 아울러 세수를 늘릴 수 있는 가장 좋은 방법은 Ocean View에 세워진 것과 같은 골프 시설과 휴양지 호텔을 신축하는 것이다

1 다른 요인으로 관광 산업이 발전했을 수도 있다 문화 유적이 발견이 되었거나 도로의 정비등으로 여행자가 늘었을 수도 있다

2 관광 산업의증가가 늘어난 세수의 원인이 아니라 새로 유입된 인구의 증가나 다른 공장에서 발생한 것일 수 있다

3 2년동안 한참 골프가 붐을 이루었을 수 있다 경제상황이 나빠지거나 다른 레포츠가 인근 지역에 생겨난다면 골프하는 사람이 줄어들 수 있다

In this memo HopeweUs mayor recommends that in order to stimulate the towns economyand boost tax revenues HopeweU should build a new golf course and resort hotel just as thetown of Ocean View did two years ago To support this recommendation the mayor points outthat in Ocean View during the last two years tourism has increased new businesses haveopened and tax revenues have increased by 30 I find the mayors argument unconvincingin several important respectsFirst of all it is possible that the mayor has confused cause with effect respecting the recentdevelopments in Ocean View Perhaps Ocean Views construction of a new golf course andhotel was a response to previous increases in tourism and business development increasesthat have simply continued during the most recent two years Since the mayor has failed toaccount for this possibility the claim that Hopewell would boost its economy by alsoconstructing a golf course and hotel is completely unwarrantedSecondly the mayor fails to account for other possible causes of the trends in Ocean Viewduring the last two years The increase in tourism might have been due to improving economicconditions nationwide or to unusually pleasant weather in the region The new businessesthat have opened in Ocean View might have opened there irrespective of the new golf courseand hotel And the 30 increase in tax revenues might have been the result of an increase intax rates or the addition of a new type of municipal taxWithout ruling out these and other alternative explanations for the three recent trends inOcean View the mayor cannot reasonably infer based on those trends that Hopewellseconomy would benefit by following Ocean Views exampleThirdly even if the recent trends in Ocean View are attributable to the construction of the newgolf course and hotel there the mayor assumes too hastily that the golf course and hotel willcontinue to benefit that towns overall economy The mayor has not accounted for thepossibility that increased tourism will begin to drive residents away during tourist season orthat new business development will result in the towns losing its appeal as a place to visit or tolive Unless the mayor can convince me that these scenarios are unlikely I cannot accept themayors recommendation that Hopewell follow Ocean Views exampleFinally the mayors argument rests on the unsubstantiated assumption that Hopewell andOcean View are sufficiently alike in ways that might affect the economic impact of a new golfcourse and hotel Hopewell might lack the sort of natural environment that would attract moretourists and new businesses to the town--regardless of its new golf course and hotel For thatmatter perhaps Hopewell already contains several resort hotels and golf courses that are notutilized to their capacity If so building yet another golf course and hotel might amount to amisallocation of the towns resources--and actually harm the towns overall economyIn sum the mayors recommendation is not well supported To bolster it the mayor mustprovide better evidence that Ocean Views new golf course and hotel and not some otherphenomenon--has been responsible for boosting Ocean Views economy during the last twoyears To better assess the recommendation I would need to know why Ocean View decidedto construct its new golf course and hotel in the first place--specifically what events prior toconstruction might have prompted that decision I would also need to thoroughly compare

GRE AWA John박 박정어학원

HopeweU with Ocean View--especially in terms of their appeal to tourists and businesses--todetermine whether the same course of action that appears to have boosted Ocean Viewseconomy would also boost Hopewells economy

The following is part of a memorandum from the president of Humana University

Last year the number of students who enrolled in online degree programs offered by nearby Omni University increased by 50 percent During the same year Omni showed a significant decrease from prior years in expenditures for dormitory and classroom space most likely because instruction in the online programs takes place via the Internet In contrast over the past three years enrollment at Humana University has failed to grow and the cost of maintaining buildings has increased along with our budget deficit To address these problems Humana University will begin immediately to create and actively promote online degree programs like those at Omni We predict that instituting these online degree programs will help Humana both increase its total enrollment and solve its budget problems

Write a response in which you discuss what questions would need to be answered in order to decide whether the prediction and the argument on which it is based are reasonable Be sure to explain how the answers to these questions would help to evaluate the prediction

Is Omni University successful due to the online degree program 50 Is the decrease in expenditures for dormitory and classroom space due to the decrease in of on-campus students Which classes were successful Does HU have those classes

Even if the long-distance degree programs at Omni University benefited the school the presidentrsquos recommendation that Human College should emulate Omni University is too hasty First OUrsquos name implies that the school would have more majors than Humanahellip the president should examine which degrees were in the long-distance programhellip

지난해에는 Omni 대학에서 개강했던 원거리 학생 학점 취득 프로그램을 등록했던 학생들의 숫자가 50나 증가했다 같은해 기간동안 Omni 대학에서는 그 전년도부터 기숙사와 학급의 공간 확충을 위한 예산을 대폭 줄였는데 이는 이 원거리 학점 취득 프로그램이 양방향 비디오 컴퓨터 접속을 통해서만 가능한 수업지도 방식이기때문인 것으로 보인다 반면 지난 3개년 동안 Humana 대학에서의 수강률은 감소한데다가 건물

GRE AWA John박 박정어학원

유지비도 올랐다 따라서 Humana대학의 수강을 늘리고 예산손실을 회복하기 위해서는 Omni 대학에서 취한 조치와 같은 능동적인 프로그램을 추진해야 한다

결론 we should initiate and actively promote long-distance degree programs like those at Omni 반박 원거리 학생 취득 프로그램 숫자가 증가한거하고 예산이 줄어드는 것 사이에 연관이 약하다 (causal 학생의 증가로 관리비용 증가할수 있음 원거리 수업가능 장비도입에의한 비용발생)bad analogy(omni university 하고 같은 조건이 아니다 )-gt omni college 가 강좌내용이 좋아서 학생의 등록이 많을수 있다 Humana 대학에서 만들었다 하더라도 인기 없을수 있음다른 요인에 의해서 Humana 대학의 수강 인원이 증가할수 있음(비록 과거엔 인기가 없었을지라도)

The following appeared as part of a business plan developed by the manager of the Rialto Movie Theater

Despite its downtown location the Rialto Movie Theater a local institution for five decades must make big changes or close its doors forever It should follow the example of the new Apex Theater in the mall outside of town When the Apex opened last year it featured a video arcade plush carpeting and seats and a state-of-the-art sound system Furthermore in a recent survey over 85 percent of respondents reported that the high price of newly released movies prevents them from going to the movies more than five times per year Thus if the Rialto intends to hold on to its share of a decreasing pool of moviegoers it must offer the same features as Apex

Write a response in which you discuss what questions would need to be answered in order to decide whether the recommendation is likely to have the predicted result Be sure to explain how the answers to these questions would help to evaluate the recommendation

Before following through this business plan the manager should investigate the cause of Rialtorsquos unsuccessful business

The author provides no evidence that the surveyrsquos results are statistically reliable The surveyrsquos sample of 85 percent must be sufficient in size and representative of overall population of the city where Rialto and Apex is serving Lacking evidence of a sufficiently representative sample the author cannot justifiably rely on the survey to draw any conclusion whatsoever The author does not indicate that Apex is indeed currently successful However even if Apex is enjoying success the argument relies on what might be a false analogy between Rialto and Apex In order for Apex to serve as a model that Rialto should emulate the author must assume that all relevant circumstances are essentially the same However this assumption is unwarranted For example the argument overlooks the face that Apex is located in a strategic placemdashbeside a mall where customers can not only watch a movie but also enjoy shopping Therefore simply changing the facility to that of Apex may not lead to success

The author does not mention whether Apex is successful or not Nevertheless even if Apex is currently successful the argument relies on what might be a false analogy between Rialto and Apex In order for Apex to serve as a model that Rialto should emulate the author must assume that all relevant circumstances are essentially the same However this assumption is unwarranted For example the argument overlooks the fact that these two institutions are located in different locations Rialto in downtown and Apex in a mall outside of town Although Apex opened with state-of-the-art facilities the decisive factor in its success could be due to its strategic location of being in a mall People could enjoy both shopping and movies at one location thus they may prefer Apex over Rialto Furthermore the place where people enjoy leisure activities has shifted in the past decades for most cities from downtown to the suburbs Therefore Rialto may not be successful even if it emulates Apexrsquos facilities A better business plan may be relocating Apex to the thriving section of the downtown

Rialto 극장은 지난 50여년간 지역 회관으로써 시내에 위치해 있으면서도 이제 변화를 꾀하지 않으면 문을 닫을

GRE AWA John박 박정어학원

판이다 이 극장은 시외 쇼핑타운에 새로 들어선 Apex 극장의 사례를 본받아야 했다 Apex가 지난해 개업했을 당시 이 극장은 비디오 아케이드 플러쉬 카펫트 바닥과 좌석 그리고 최신 음향시설을 갖추었다 더군다나 최근 조사에서는 응답자의 85 이상이 새로 출시된 영화 입장료가 비싼 탓으로 지난해보다 5배이상의 관람객이 줄어들었다고 나타났다 따라서 Rialto 극장이 줄어들고 있는 관람객을 뺐기지 않고 유지하려면 Apex와 같은 시설들을 갖추어야 할 것이다주장 리알토 극장이 줄어들고 있는 관람객을 뺐기지 않고 유지하려면 Apex와 같은 시설들을 갖추어야 할 것이다

1 조사에서 응답자가 전체를 대표할 수 없다 2 apex 극장이 좋은 시설을 갖추고 있지만 그로 인해 수익이 많이 발생했다는 말이 없으므로 시설투자를

하고도 좋은 결과를 얻을 수 있을지 그 근거가 미흡하다3 좋은 영화가 출시된다면 입장료가 비싸도 영화관에서 꼭 보려고 할 수 있다 4 rialto 가 시설이 아닌 다른 요인에 의해 장사가 안될수도 있다( 우범 지역이라든지)

The following is a recommendation from the business manager of Monarch Books

Since its opening in Collegeville twenty years ago Monarch Books has developed a large customer base due to its reader-friendly atmosphere and wide selection of books on all subjects Last month Book and Bean a combination bookstore and coffee shop announced its intention to open a Collegeville store Monarch Books should open its own in-store cafeacute in the space currently devoted to childrens books Given recent national census data indicating a significant decline in the percentage of the population under age ten sales of childrens books are likely to decline By replacing its childrens books section with a cafeacute Monarch Books can increase profits and ward off competition from Book and Bean

Write a response in which you examine the stated andor unstated assumptions of the argument Be sure to explain how the argument depends on these assumptions and what the implications are for the argument if the assumptions prove unwarranted

The following is a recommendation from the business manager of Monarch Books

Since its opening in Collegeville twenty years ago Monarch Books has developed a large customer base due to its reader-friendly atmosphere and wide selection of books on all subjects Last month Book and Bean a combination bookstore and coffee shop announced its intention to open a Collegeville store Monarch Books should open its own in-store cafeacute in the space currently devoted to childrens books Given recent national census data indicating a significant decline in the percentage of the population under age ten sales of childrens books are likely to decline By replacing its childrens books section with a cafeacute Monarch Books can increase profits and ward off competition from Book and Bean

1 Write a response in which you discuss what questions would need to be answered in order to decide whether the recommendation is likely to have the predicted result Be sure to explain how the answers to these questions would help to evaluate the recommendation

2 Write a response in which you discuss what specific evidence is needed to evaluate the argument and explain how the evidence would weaken or strengthen the argument

No evidence regarding Monarch Bookrsquos successEven if Regal Bookrsquos is successful this may not be attributable to the cafeacute False analogy Emulating may not lead to success Other factors may be involvedInsufficient condition The national census is not enough evidence that childrenrsquos book sales will decline Can

GRE AWA John박 박정어학원

the national census represent the local child populationDid opening a cafeacute boost sales for Regal Books Even assuming Regal is successful by opening a cafeacute this may not be suitable for Monarch which plans to close the childrenrsquos book section to establish a cafe Imprecise language ldquorelatively little spacerdquo how smallThe managerrsquos recommendation contradicts what he says Since Monarch is popular for its wide selection of books closing a selection which targets a major group of readers may hurt Monarchrsquos salesIs this the best way to compete

When Stanley Park first opened it was the largest most heavily used public park in town It is still the largest park but it is no longer heavily used Video cameras mounted in the parks parking lots last month revealed the parks drop in popularity the recordings showed an average of only 50 cars per day In contrast tiny Carlton Park in the heart of the business district is visited by more than 150 people on a typical weekday An obvious difference is that Carlton Park unlike Stanley Park provides ample seating Thus if Stanley Park is ever to be as popular with our citizens as Carlton Park the town will obviously need to provide more benches thereby converting some of the unused open areas into spaces suitable for socializing

Write a response in which you examine the stated andor unstated assumptions of the argument Be sure to explain how the argument depends on these assumptions and what the implications are for the argument if the assumptions prove unwarranted

Stanley 파크가 처음 개장했을 당시 가장 크고 가장 많이 이용되는 공원이었다 아직도 공원중에서는 가장 크지만 이용률은 상당히 떨어졌다 지난달 공원 주차장에 설치해놓은 비디오 카메라를 통해 보면 drop(주차장으로 여겨짐) 이용률이 가장 높았다 수치상으로는 하루 평균 50대의 차량만이 이용하였다 반면 직장 중심거리에 위치한 작은 규모의 Carlton 파크는 주당 무려 150여명 이상이 이용하고 있다 Stanley 파크와는 달리 Carlton 파크에는 의자가 있다는 것이 가장 뚜렷한 차이점이다 따라서 Stanley 파크가 Carlton 파크처럼 시민들이 자주 이용하는 공원이 되기 위해서는 벤치를 설치할 필요가 있으며 이렇게 사용되지 않는 일부 공간을 활용해서 사교를 위한 공간으로 바꾸어야 한다 ===gtdrop 에 대한 첨부사항 (영영사전내용입니다)---- a place or central depository to which something (as mail money or stolen property) is brought for distribution or transmission also the act of depositing something at such a place dropgt

주장 if Stanley Park is ever to be as popular with our citizens as is Carlton Park the town will obviously need to provide more benches thereby converting some of the unused open areas into spaces suitable for socializing1 조사가 언제 이루어진 것인가 조사가 언제 실시되었느냐에 따라 결과가 다를 수있다 현재는 다시 스탠리 파크가 늘어났었을 수 있다 2 벤치를 많이 설치했다고 해서 많은 관광객이 오지 않을수 있다(사람들이 벤치나 사교 공간을 원한다는 어떠한 자료도 없다)3스탠리 파크 주변에 교통 상황이 악화가 되었거나 칼튼 파크에서 문화행사등을 많이 가져서 이용객이 줄어든것일 수도 있다 4 칼튼 파크가 중심지에 있어서 접근성이 좋을수 있다5 조사가 같은 시간을 기준으로 한게 아니다(하나는 주중이고 하나는 주말이다)6사람의 수와 차의 대수를 같은것으로 비교할수 없다 (차안에 몇 명이 타고 있는지 모르고 대중교통을 이용해서 왔을수도 있다)

Page 18: GRE writing argument brain storm

GRE AWA John박 박정어학원

The following is a letter to the editor of the Atticus City newspaper

Former Mayor Durant owes an apology to the city of Atticus Both the damage to the River Bridge which connects Atticus to Hartley and the traffic problems we have long experienced on the bridge were actually caused 20 years ago by Durant After all he is the one who approved the construction of the bridge If he had approved a wider and better-designed bridge on which approximately the same amount of public money would have been spent none of the damage or problems would have occurred Instead the River Bridge has deteriorated far more rapidly over the past 20 years than has the much longer Derby Bridge up the river Even though the winters have been severe in the past several years this is no excuse for the negligence and wastefulness of Durant

Write a response in which you discuss what questions would need to be answered in order to decide whether the recommendation is likely to have the predicted result Be sure to explain how the answers to these questions would help to evaluate the recommendation

전임 시장인 Durant 씨는 Atticus 시에 대해 사과할 의무가 있습니다 Atticus와 Hartley를 잇는 River Bridge 교량에 대한 피해와 이 교량에서 오랫동안 주민들이 겪어오고 있는 교통 혼잡 문제들은 실제로 20년 전부터 시작된 것이었습니다 결정적으로 그가 교량 공사를 허가했던 바로 그 장본인입니다 당시 비슷한 공사비용으로 폭이 더 넓고 튼튼하게 설계된 교량을 허가했다면 이러한 문제나 피해는 발생하지 않았을 겁니다 더군다나 이 다리는 지난 20년 동안 상류에 건설된 훨씬 오래된 Derby 다리보다도 빠르게 부식되어 갔습니다 지난 수년동안 심지어 혹한이 있었다 하더라도 이러한 태만과 국고 손실에 대한 책임을 회피할 길이 없는 것입니다

결론 Former Mayor Durant owes an apology to the city of Atticus

1 디자인이 문제가 아닐수 있다 (디자인은 좋았으나 건설과정에 문제가 있었을 수 있다)2 그 당시의 시예산이 적어서 더 큰 다리를 짓기가 불가능했을 수도 있다3 교통량이 많거나 다른 상황으로 인해서 부식이 빨리 됐을 수 있다4 그 당시에는 최선의 선택이였지만 갑자기 변한 상황에 의해서 이런 문제점들이 발생했을 수 있다

GRE AWA John박 박정어학원

4그룹 거짓인과관계 오류 (False Cause) 빈출

Fifteen years ago Omega University implemented a new procedure that encouraged students to evaluate the teaching effectiveness of all their professors Since that time Omega professors have begun to assign higher grades in their classes and overall student grade averages at Omega have risen by 30 percent Potential employers looking at this dramatic rise in grades believe that grades at Omega are inflated and do not accurately reflect student achievement as a result Omega graduates have not been as successful at getting jobs as have graduates from nearby Alpha University To enable its graduates to secure better jobs Omega University should terminate student evaluation of professors

Write a response in which you discuss what specific evidence is needed to evaluate the argument and explain how the evidence would weaken or strengthen the argument

Omega professor evaluation implemented 15 years ago =gt Omega prof assign higher grades 30Employers believe therersquos grade inflation

Thus unsuccessful employment than AlphaTherefore to secure jobs Omega should end evaluating profs

Specific evidence neededRelationship between higher grades and evaluationRelationship between GPA and unsuccessful employmentAlpharsquos education could just be better than OmegaldquoFifteen years agordquo is a long time other factors could have influenced Why is the inflation a problem just now How much is Alpha better Is the comparison just How much gap is thereOmegarsquos student could just be doing better in their studiesComparison to other universities다른 대안 없나hellip Could Omega alleviate the employment problem by implementing a different procedure or program

15 년전 우리 대학은 학생들로 하여금 교수평가를 하도록 한 새로운 조치를 시행했었습니다 이후 교수들은 자신의 학과 학생들에게 높은 학점을 주었으며 그에따라 학생들의 전체 평점이 30나 올랐습니다 외부의 기업체들은 분명 점수가 지나치게 부풀려졌다고 믿고 있습니다 결국 본 대학 졸업생들이 인근 Alpha 대학의 졸업자들보다 구직률이 떨어지는 이유를 잘 보여주고 있는 것입니다 이를 해결하기 위해 이제부터는 학생들에 의한 교수평가제를 중단해야 합니다

결론 Omega University should now terminate student evaluation of professors

반박 교수 평가와 학점 인플레의 연관성이 적다( 교수 평가를 먼저하고 학점을 나중에 매길수도 있다)채용기준에 성적만 있는게 아니다 학업성취의 결과 일수도 있다 Alpha 가 원래 유능했다 Alpha 의 교육내용이 좋았다

GRE AWA John박 박정어학원

In this memo the dean of Omega University(OU) recommends OU to terminate professor evaluation to secure better jobs for the students To support this recommendation the dean offers several reasons However this argument contains several logical flaws which render it unconvincing

A threshold problem with the argument involves the voluntary nature of the evaluationprocedure The dean provides no evidence about the number or percentage of Omegastudents who participate in the procedure Lacking such evidence it is entirely possible thatthose numbers are insignificant in which case terminating the procedure is unlikely to haveany effect on the grade average of Omega students or their success in getting jobs aftergraduationThe argument also assumes unfairly that the grade-average increase is the result of theevaluation procedure--rather than some other phenomenon The dean ignores a host of otherpossible explanations for the increase--such as a trend at Omega toward higher admissionstandards or higher quality instruction or facilities Without ruling out all other possibleexplanations for the grade-average increase the dean cannot convince me that by terminatingthe evaluation procedure Omega would curb its perceived grade inflation let alone help itsgraduates get jobsEven if the evaluation procedure has resulted in grade inflation at Omega the deans claimthat grade inflation explains why Omega graduates are less successful than Alpha graduatesin getting jobs is unjustified The dean overlooks a myriad of other possible reasons forOmegas comparatively poor job-placement record Perhaps Omegas career services areinadequate or perhaps Omegas curriculum does not prepare students for the job market aseffectively as Alphas In short without accounting for other factors that might contribute toOmega graduates comparative lack of success in getting jobs the dean cannot justify theclaim that if Omega curbs its grade inflation employers will be more likely to hire OmegagraduatesFinally even if the dean can substantiate all of the foregoing assumptions the deansassertion that Omega must terminate its evaluation procedure to enable its graduates to findbetter jobs is still unwarranted in two respects First the dean ignores other possible ways bywhich Omega can increase its job-placement record--for example by improving its publicrelations or career-counseling services Second the dean unfairly equates more jobs withbetter jobs In other words even if more Omega graduates are able to find jobs as a result ofthe deans recommended course of action the kinds of jobs Omega graduates find would notnecessarily be better onesIn sum the deans argument is unpersuasive as it stands To strengthen it the dean mustprovide better evidence that the increase in grade average is attributable to Omegasprofessor-evaluation procedure and that the end result is a perception on the part ofemployers that Omega graduates are less qualified for jobs than Alpha graduates To betterassess the argument I would need to analyze 15-year trends in (l) the percentage of Omegastudents participating in the evaluation procedure (2) Omegas admission standards andquality of education and (3) Omegas emphasis on job training and career preparation I wouldalso need to know what other means are available to Omega for enabling its graduates to findbetter jobs

GRE AWA John박 박정어학원

The following appeared in a memo from a vice president of Quiot Manufacturing

During the past year Quiot Manufacturing had 30 percent more on-the-job accidents than at the nearby Panoply Industries plant where the work shifts are one hour shorter than ours Experts say that significant contributing factors in many on-the-job accidents are fatigue and sleep deprivation among workers Therefore to reduce the number of on-the-job accidents at Quiot and thereby increase productivity we should shorten each of our three work shifts by one hour so that employees will get adequate amounts of sleep

Write a response in which you examine the stated andor unstated assumptions of the argument Be sure to explain how the argument depends on these assumptions and what the implications are for the argument if the assumptions prove unwarranted

The following appeared in a memo from a vice president of Alta Manufacturing

During the past year Alta Manufacturing had thirty percent more on-the-job accidents than nearby Panoply Industries where the work shifts are one hour shorter than ours Experts believe that a significant contributing factor in many accidents is fatigue caused by sleep deprivation among workers Therefore to reduce the number of on-the-job accidents at Alta we recommend shortening each of our three work shifts by one hour If we do this our employees will get adequate amounts of sleep

Write a response in which you discuss what questions would need to be answered in order to decide whether the recommendation and the argument on which it is based are reasonable Be sure to explain how the answers to these questions would help to evaluate the recommendation

The following appeared in a memo from the vice president of Butler Manufacturing

During the past year workers at Butler Manufacturing reported 30 percent more on-the-job accidents than workers at nearby Panoply Industries where the work shifts are one hour shorter than ours A recent government study reports that fatigue and sleep deprivation among workers are significant contributing factors in many on-the-job accidents If we shorten each of our work shifts by one hour we can improve Butler Manufacturings safety record by ensuring that our employees are adequately rested

1 Write a response in which you discuss what specific evidence is needed to evaluate the argument and explain how the evidence would weaken or strengthen the argument

2 Write a response in which you discuss what questions would need to be answered in order to decide whether the recommendation is likely to have the predicted result Be sure to explain how the answers to these questions would help to evaluate the recommendation

4번 반복됨

Alta has 30 more job accidents than Panoply(work shifts one hour shorter)Experts Job accidents caused by fatigue and sleep deprivationTherefore to reduce job accidents and increase productivity shorten three work shifts by one hour for adequate sleep

지난해 우리 회사는 인근 Panoply Industries보다 업무상 재해가 30나 더 많았다 그 회사는 우리보다 근무 교대시간이 1시간 정도 짧았다 전문가들은 대부분의 업무상 재해에 있어서 가장 중요한 요인이 과로와 수면부족으로 보고있다 따라서 우리 회사에서 높은 산업재해를 줄이고 아울러 생산성을 높이기 위해서는 근로자들이 충분한 수면을 취할 수 있도록 1시간씩 3교대 시간을 줄여야 한다

In this memo the (author) vice president of Alta Manufacturing (AM) recommends that to reduce on-the-job accidents and increase productivity AM should shorten its three work shifts by one hour so that employees can

GRE AWA John박 박정어학원

get more sleep To support this recommendation the author provides several evidences However careful scrutiny of each of the facts reveals that it provides little credible support for the authorrsquos recommendation QuestionsThe number of accidents What kind of accidents The seriousness of the accidents is importantHow many employees are in each company What are their productsFalse cause Sleep may not be the reason for the on-the-job accidents What do Alta and Panoply manufacture

First of all the author believes that fatigue caused the on-the job accidents However there could be other reasons The author observes a correlation between sleep deprivation and on-the-job accidents then concludes that the former is the cause of the latter However the author fails to rule out other possible explanations For example it is entirely possible that Alta factories require more strenuous and dangerous labor than Panoply Without ruling out all other such factors it is unfair to conclude that fatigue is responsible for the accidents In addition the work-shifts may not be the cause of the sleep deprivation and fatigue It is possiblehellip Thus the author should provide what exactly Panoply and Alta manufacture and more precise data about their working conditions to be more convincing

Shortening the shift by one hour does not necessarily lead to more sleep And is one hour enoughLess accidents does not mean increased productivity

결론 We should shorten each of out three work shifts by one hour

반박 경쟁사에 비해서 시간당 하는 업무량이 많아서 더욱 피곤할 수도 있다 시간이 문제가 아니라 노후된 시설 설비 자체의 문제 작업 자체가 원래 위험한 것이여서 사고가 많을 수도 있다 다른 회사는 더욱 많은 작업시간에도 불구하고 안정한 작업여건으로 인해서 사고율이 오히려 더 작을 수도 있다비교사의 재해감소가 다른 요인일수 있다(안전 교육 철저)줄인 시간이 피로회복이나 수면으로 연결 안될수 있음(술을 마실 수도 있고 그 시간에 휴식을 취하지 않고 다른일을 함으로써 더욱 피로해질수 있다)

This editorial recommends that Alta Manufacturing reduce its work shifts by one hour each inorder to reduce its on-the-job accident rate and thereby increase Altas productivity To supportthis recommendation the author points out that last year the number of accidents at Alta was30 greater than at Panoply Industries where work shifts were one hour shorter The authoralso cites certain experts who believe that many on-the-job accidents are caused by fatigueand sleep deprivation I find this the argument unconvincing for several reasonsFirst and foremost the author provides absolutely no evidence that overall workerproductivity is attributable in part to the number of on-the-job accidents Although commonsense informs me that such a relationship exists the author must provide some evidence ofthis cause-and-effect relationship before I can accept the authors final conclusion that theproposed course of action would in fact increase Altas productivitySecondly the author assumes that some accidents at Alta are caused by fatigue or sleepdeprivation However the author overlooks other possible causes such as inadequateequipment maintenance or worker training or the inherent hazards of Altas manufacturingprocesses By the same token Panoplys comparatively low accident rate might be attributablenot to the length of its work shifts but rather to other factors such as superior equipmentmaintenance or worker training In other words without ruling out alternative causes ofon-the-job accidents at both companies the author cannot justifmbly conclude that merely byemulating Panoplys work-shift policy Alta would reduce the number of such accidentsThirdly even assuming that Altas workers are fatigued or sleep-deprived and that this is thecause of some of Altas on-the-job accidents in order to accept the authors solution to thisproblem we must assume that Altas workers would use the additional hour of free time tosleep or rest However the author provides no evidence that they would use the time in thismanner It is entirely possible that Altas workers would use that extra hour to engage in someother fatiguing activity Without ruling out this possibility the author cannot convincinglyconclude that reducing Altas work shifts by one hour would reduce Altas accident rateFinally a series of problems with the argument arise from the scant statistical information onwhich it relies In comparing the number of accidents at Alta and Panoply the author fails toconsider that the per-worker accident rate might reveal that Alta is actually safer than Panoplydepending on the total number of workers at each company Second perhaps accident rates

GRE AWA John박 박정어학원

at the two companies last year were aberrations and during other years Altas accident ratewas no greater or even lower than Panoplys rate Or perhaps Panoply is not representativeof industrial companies generally and that other companies with shorter work shifts have evenhigher accident rates In short since the argument relies on very limited statistical information Icannot take the authors recommendation seriouslyIn conclusion the recommendation for emulating Panoplys work-shift policy is not wellsupported To convince me that shorter work shifts would reduce Altas on-the-job accidentrate the author must provide clear evidence that work-shift length is responsible for some ofAltas accidents The author must also supply evidence to support her final conclusion that alower accident rate would in fact increase overall worker productivity

The following appeared in a memo from the vice president of marketing at Dura-Sock Inc

A recent study of our customers suggests that our company is wasting the money it spends on its patented Endure manufacturing process which ensures that our socks are strong enough to last for two years We have always advertised our use of the Endure process but the new study shows that despite our socks durability our average customer actually purchases new Dura-Socks every three months Furthermore our customers surveyed in our largest market northeastern United States cities say that they most value Dura-Socks stylish appearance and availability in many colors These findings suggest that we can increase our profits by discontinuing use of the Endure manufacturing process

1 Write a response in which you examine the stated andor unstated assumptions of the argument Be sure to explain how the argument depends on these assumptions and what the implications are for the argument if the assumptions prove unwarranted

2 Write a response in which you discuss what specific evidence is needed to evaluate the argument and explain how the evidence would weaken or strengthen the argument

3 Write a response in which you discuss what questions would need to be answered in order to decide whether the recommendation and the argument on which it is based are reasonable Be sure to explain how the answers to these questions would help to evaluate the recommendation

Intro The vice president of marketing at Dura-Sock Inc is offering a potentially harmful investment recommendation by claiming that Dura-Sock should discontinue its use of the ldquoEndurerdquo process To support his recommendation he points out a study that Dura-Sock customers actually purchase the socks every three months and a survey that reveals that Dura-Sock customers like the sockrsquos stylish appearance and availability in many colors The study and survey however are insufficient in supporting his proposal and the VP makes several unwarranted assumptionsIntro (simplified) The VP states that though Dura-Socks last for two years customers buy the socks every three months Therefore he assumes that the consumersrsquo motive for buying the produce is not its durabilityHowever the author fails to rule out other possible motivation for consumption

Even if the survey is reliable the author should consider the rest of the market Vague terms ldquowasting moneyrdquomdashprecisely how much are they wasting Studysurvey errorThe company must calculate the outcome of such momentous decisionStudy participantsrsquo comment that they prefer Dura-Sock for its stylishness and availability might take Dura-Sockrsquos enduring quality for granted

우리회사 제품 소비자들에 대한 최근 조사에서 지난 2년여간 양말의 내구성을 강하게 하는 필수공정이었던 자사 특허의 Endure 공정에 들어가는 비용이 낭비라고 말하고 있다 우리 회사는 항상 이 공정 처리에 대한 광고를 내보냈으나 이에 대한 시장 조사에서 실제로 고객들은 이 신제품을 평균 석달마다 구매하는 것으로 나타났다 더군다나 북동부지역에서 실시한 대규모 시장조사에 응답한 고객들은 양말의 모양과 색상등에 더

GRE AWA John박 박정어학원

관심을 나타냈다 이러한 결과는 우리회사가 신기술 공법을 중단하면 그에 따라 수익이 늘어날 것이라는 것을 말해주고 있는 것이다주장 These findings suggest that Dura0Sock can increase its profits by discontinuing its use of the ldquoEndurerdquo manufacturing process

1 survey가 정확한 소비자의 의견을 나타낸 것인가 다른 선택없이 양자택일과 같은 방법의 survey였는지2 북동부지역의 시장조사가 전체 의견을 대표할 수 있나3 사람들이 모양이나 색상에 앞서 내구성을 먼저 평가했을 수도 있다 내구성을 갖추었다는 전제하에 모양과 색상에 관심을 드러낸 것일 수 있다4 소비자가 도매상(retail)인지 소매상(whole)인지가 없다

The following appeared in a business magazine

As a result of numerous complaints of dizziness and nausea on the part of consumers of Promofoods tuna the company requested that eight million cans of its tuna be returned for testing Promofoods concluded that the canned tuna did not after all pose a health risk This conclusion is based on tests performed on samples of the recalled cans by chemists from Promofoods the chemists found that of the eight food chemicals most commonly blamed for causing symptoms of dizziness and nausea five were not found in any of the tested cans The chemists did find small amounts of the three remaining suspected chemicals but pointed out that these occur naturally in all canned foods

Write a response in which you discuss what questions would need to be addressed in order to decide whether the conclusion and the argument on which it is based are reasonable Be sure to explain how the answers to the questions would help to evaluate the conclusion

Representativeness of the tested cansThey should conduct a comparative studyThe testing could be biased because Promofoods employees conducted the testingHow much (quantity) of the five and three suspected chemicals were in the canned foodsFalse cause The substance that caused dizziness and nausea may not be one of the eight common chemicals

많은 소비자들의 현기증과 구역질 불만에 따라 Promofoods사는 지난해 참치 캔 8백만 개를 테스트하기 위해 반품시켰다 그 결과 캔에서는 건강에 위험이 될 수 있는 화합물질이 없었던 것으로 회사측은 결론지었다 이러한 결론은 회사측 화학연구자들이 회수된 캔의 샘플을 테스트해서 이들 증상의 원인이 되는 8가지 화합물 중에서 5가지가 실험된 캔에서 발견되지 않았다는 사실에 근거한 것이다 이들 화학자들은 나머지 3개가지 화합물이 모든 캔 식료품에서 흔히 발견되는 것이라고 언급했다 결론 Promofoods concluded that the cans did not after all contain chemicals that posed a health risk

1 공인된 기간에서 테스트를 한 것이 아니고 자사에서 직접 테스트를 했기에 신뢰성이 안간다 2 이런 증상을 일으키는 8개의 물질 말고 다른 물질들이 캔속에 많이 포함됬을수 있다 3 나머지 3개의 물질들의 함유량이 많아서 다른 종류의 캔들은 문제를 일으키지 않지만 참치캔은 문제를

일으킬 수 있다

This magazine article concludes that the 8 million cans of tuna Promofoods recalled due tocomplaints about nausea and dizziness do not after ail contain any chemicals that pose a

GRE AWA John박 박정어학원

health risk To support this conclusion the author cites the fact that five of eight chemicalscommonly causing these symptoms were not found in the recalled cans while the other threealso occur naturally in other canned foods For several reasons this evidence lends littlecredible support to the authors conclusionTo begin with the author relies partly on the fact that although three of the eight chemicalsmost commonly blamed for nausea and dizziness appeared in Promofoods recalled tunathese chemicals also occur naturally in other canned foods However this fact alone lends nosupport to the authors conclusion for two reasons First the author might be ignoring animportant distinction between naturally occurring chemicals and those not occurring naturallyIt is entirely possible that these three chemicals do not occur naturally in Promofoods tunaand that it is for this reason that the chemicals cause nausea and dizziness Secondly it isentirely possible that even when they occur naturally these chemicals cause the samesymptoms Unless the author rules out both possibilities he cannot reliably conclude that therecalled tuna would not cause these symptomsAnother problem with the argument is that the authors conclusion is too broad Based onevidence about certain chemicals that might cause two particular heath-related symptoms theauthor concludes that the recalled tuna contains no chemicals that pose a health risk Howeverthe author fails to account for the myriad of other possible health risks that the recalled tunamight potentially pose Without ruling out all other such risks the author cannot justifiablyreach his conclusionA third problem with the argument involves that fact that the eight particular chemicals withwhich the test was concerned are only the eight most commonly blamed for nausea anddizziness It is entirely possibly that other chemicals might also cause these symptoms andthat one or more of these other chemicals actually caused the symptoms Without ruling outthis possibility the author cannot jusufiably conclude that the recalled tuna would not causenausea and dizzinessA final problem with the argument involves thetesting procedure itself The author providesno information about the number of recaUed cans tested or the selection method used Unlessthe number of cans is a sufficiently large sample and is statistically repre sentative of all therecalled cans the studys results are not statistically reliableIn conclusion the article is unconvincing as it stands To strengthen the assertion that therecalled tuna would not cause nausea and dizziness the author must provide evidence thatthe three chemicals mentioned that occur naturally in other canned foods also appear naturallyin Promofoods tuna The author must also provide evidence that ingesting other canned foodscontaining these three chemicals does not cause these symptoms To better evaluate theargument we would need to know whether the sample used in the tests was statisticallysignificant and representative of all the recalled tuna We would also need to know what otherchemicals in the recalled tuna might pose any health risk at all

5그룹 불충분 조건오류 빈출

Natures Way a chain of stores selling health food and other health-related products is opening its next franchise in the town of Plainsville The store should prove to be very successful Natures Way franchises tend to be most profitable in areas where residents lead healthy lives and clearly Plainsville is such an area Plainsville merchants report that sales of running shoes and exercise clothing are at all-time highs The local health club has more members than ever and the weight training and aerobics classes are always full Finally Plainsvilles schoolchildren represent a new generation of potential customers these schoolchildren are required to participate in a fitness-for-life program which emphasizes the benefits of regular exercise at an early age

Write a response in which you examine the stated andor unstated assumptions of the argument Be sure to

GRE AWA John박 박정어학원

explain how the argument depends on these assumptions and what the implications are for the argument if the assumptions prove unwarranted

False cause

First of all the author believes that the Increased sales of running shoes and exercise clothing indicates

Plainesville residentsrsquo interest in leading healthy lives However this assumption is not logically convincing for

several reasons could be a fashion trendTime shift ldquoFitness for liferdquo might not have any influence on schoolchildren as they growFalse cause There could be other reasons for member increase in the health clubAll of the above are insufficient condition

The author has to prove that local residents are interested in leading healthy lives However he supports his conclusion with insufficient evidence Nevertheless even if the residents are concerned with health naturersquos way may not be successful First

그 동안의 경험을 토대로 볼 때 건강생활과 밀접히 관련되어 있는 거주 지역에서 본 상점들이 아주 호응을 얻고 있다 따라서 이러한 주민들이 많이 거주하고 있는 Plainsville 에 새로운 상점들을 계속 세워야 한다 이 지역 상인들은 런닝화와 운동복 판매가 가장 높다고 말한다 불과 5 년전에는 거의 전무하다시피하던 지역 헬스 클럽의 경우도 엄청나게 많은 회원을 확보하고 있으며 웨이트 트레이닝과 에어로빅 강좌들도 항상 만원이라고 한다 새로운 고객층을 예측해 보는 것도 가능하다 이 지역의 학생들의 경우 Fitness for Life프로그램을 받게 되는데 이러한 프로그램을 통해서 유년시절부터 정규적인 운동 습관을 들이게 하고 있는 것이 그것이다

결론 We should therefore build our next new store in Plainsville

반박 그동안의 경험에 의한 과거 통계가 꼭 여기에도 적용되는건 아니다 5 년전 헬스 클럽이 잘 안되었던게 다른 원인이였을수 있다(강사수준 미달 강좌미비)tourist 에 의한 원인 일수 있다 어렸을때부터 운동을 했다고 해서 커서도 관심이 있지는 않다 (오히려 반감이 있을수 있다 혹은 건강하기에 건강에 관심이 적을수도 있다)운동복이나 신발의 판매가 육체노동에 의한 것일수도 있다

IntroductionSupport1049896In this memorandum the author asserts that Naturersquos Way should build its next newstore in Plainsville To support this assertion the author states that Plainsvillesmerchantsrsquo sales of exercise clothing are going well the local health club has moremembers than ever and a new generation of customers will help to ensure NaturersquosWayrsquos success At first glance the authorrsquos assumption seems convincing but in-depth scrutiny revealsthat it lacks substantial evidence as it stands

Body 1-SamplingTopic Sentence 1To begin with the author assumes that the merchantsrsquo report indicates that the residentsare concerned about their health However this assumption is based on unsubstantiated

GRE AWA John박 박정어학원

data Example 1 (Rebuttal1) First if we do not know the total volume of items sold and the price of the goods exactly we cannot infer whether the residents are actually buying many goods Example 2 (Rebuttal2)In addition to that the report emphasizes the rising sales of running shoes and exerciseclothing however these may not be hot-selling items for Naturersquos Way or may not be theproducts the company is planning to sell Concluding Sentence Therefore in order to make the argument reliable the author should reconsider themerchantsrsquo report with more detailed data

Body 2-CausalTopic Sentence 2Second the author contends that the health clubs classes are full yet this does not meanthat many people actually use the health club other factors may be the real cause forthose closed classes Example 1 (Rebuttal1) To begin with if the health club is very small the number of people working out wouldnot be a large one In fact regular gym-going may just be a vogue among a smallunrepresentative segment of Plainsvilles population Example 2 (Rebuttal2) Moreover it is possible that most of the people who exercise in the health club do weight training and aerobics only to look good and to meet other singles not for their health In that case there would be little demand for health products Concluding SentenceThus the author should not hasten to presume what really caused people to be interested in a healthier lifestyle and enroll in the health club

Body 3-Time-ShiftTopic Sentence 3Finally the author highlights that Naturersquos Way can expect a new generation of customersin Plainsville that will help the company in the long term This notion is mistaken in that itassumes the conditions of the present will continue unchanged in the future Although theschool children are required to participate in the fitness for life program they may notnecessarily buy Naturersquos Ways products Example 1 (Rebuttal1) In the first instance they may suffer a fall in purchasing power arising from future economic difficulties this would cause reluctance to spend a considerable amount of money on health products which tend to be more expensive Example 2 (Rebuttal2)Another possibility is that there may emerge many competitor companies vying with Naturersquos Way so that in the future the school children may not feel the necessity to purchase one companyrsquos health products over anotherrsquosConcluding Sentence Thus the authorrsquos assumption is highly speculative since it relies heavily on unknowablefuture circumstances

ConclusionThesis In sum the author uses many assumptions that are insufficient in supporting his claimsSupportIn order for the authorrsquos claims to be convincing he needs to advance more persuasiveevidence that people in Plainsville really are concerned with their health and health foodThe following was written as a part of an application for a small-business loan by a group of developers in the city of Monroe

A jazz music club in Monroe would be a tremendously profitable enterprise Currently the nearest jazz club is 65 miles away thus the proposed new jazz club in Monroe the C-Note would have the local market all to itself Plus jazz is extremely popular in Monroe over 100000 people attended Monroes annual jazz festival last summer several well-known jazz musicians live in Monroe and the highest-rated radio program in Monroe is Jazz Nightly which airs every weeknight at 7 PM Finally a nationwide study indicates that the typical jazz fan spends close to $1000 per year on jazz entertainment

1 Write a response in which you discuss what specific evidence is needed to evaluate the argument and explain how the evidence would weaken or strengthen the argument

2 Write a response in which you examine the stated andor unstated assumptions of the argument Be

GRE AWA John박 박정어학원

sure to explain how the argument depends on these assumptions and what the implications are for the argument if the assumptions prove unwarranted

3 Write a response in which you discuss what questions would need to be answered in order to decide whether the prediction and the argument on which it is based are reasonable Be sure to explain how the answers to these questions would help to evaluate the prediction

Group error nationwide survey may not reflect local trends Is the nationwide jazz fan population substantialInsufficient non-residents of Monroe may have attended the jazz festival (Body alternative explanation last year may have been an anomaly The author should consider data from various years) The author should indicate how many out of 100000 were Monroe residentsNationwide study Does this reflect Insufficient Citizens of Monroe may continue to go to the jazz club 65 miles away

Are the people in Monroe really interested in jazzMajority of the people who attended the jazz festival might not be Monroe residentsSurvey error nationwide study may not be applicable to MonroeJazz musicians who live in MonroeMonopolyRadio station

In this business application the author claims that the proposed jazz club C Note will be very profitable in Monroe To support this claim the author argues for his case with several evidences At first glance the authorrsquos argument seems convincing however careful scrutiny reveals that his argument in specious

To begin with the author claims that Monroersquos citizens are interested in jazz He presents three evidences First Secondhellip Thirdhellip Howeverhellip

Monroe 시에 있는 재즈 음악 클럽은 수익성이 좋은 사업이다 현재 가장 가까이에 있는 클럽은 65 마일 정도 떨어져 있다 따라서 이번에 세우려고 하는 C Note 는 독보적인 위치를 점할것이다 더군다나 재즈는 이 시에서 가장 인기있는 음악이다 지난 여름 재즈 축제에서는 10 만명 이상의 Morone 시 주민이 참석하였고 몇몇 유명한 재즈 음악가들도 이곳에 살고 있으며 저녁때 방영되는 라디오 프로그램중에서 최고의 시청률을 보이고 있는 것도 Jazz Nightly 이다 전국조사에서도 전형적인 재즈 팬들은 재즈 분야에 년간 1천 달러 가까이 지출하고 있는 것으로 보고되고 있다 따라서 C Note 클럽이 돈을 벌 수 있는 사업이라는 것은 확실한 것이다

결과 It is clear that the C Note cannot help but make money반박 nearest jazz club 이 양질의 써비스로 여전히 손님을 끌수도 있다Festival 에 얼마나 참여하는지가 jazz 의 인기를 반영하지 않는다 뮤지션이 많이 사는거랑 jazz 의 인기가 상관없다라디오 프로그램이 다른 요인에 의해서 인기일수도 있다 (진행자때문)전국 통계 적용 불가화목 실전반_Ms Noh6In this application the author suggests that a jazz club in Monroe will make a number of profits To support this suggestion the author exemplifies the local condition popularity of jazz in Monroe and nationwide study However careful scrutiny of each of the facts reveals that it provides little credible support for the authorrsquos recommendation Good clear intro

First the author assumes that jazz is popular in Monroe because of several facts the jazz festival last year had high participation some famous jazz musicians live in Monroe and the high-rated radio program is lsquoJazz Nightlyrsquo However this assumption has many drawbacks that must be seriously considered(Good topic sentences) If many attendants in the last-yearrsquos festival came from other cities and not Monroe it is hard to conclude that Monroersquos people like jazz Therefore the author must examine how many Monroe residents actually attended the festival On top of that there is little relationship between habitation of famous jazz musician and the popularity of jazz in Monroe Although several well-known musicians live there if they do not take part in any jazz performance of Monroe this might have no effect to the interest of Monroersquos residents

GRE AWA John박 박정어학원

about jazz Finally in the case of radio program this is also not suitable reason why jazz is popular in Monroe It might be possible that people cannot help choosing lsquoJazz Nightlyrsquo because there are few radio programs at Night The fact that the radio program is the highest rating program is not a germane evidence The approximate number of listeners would be the more crucial evidence Therefore the author needs to seriously deliberate the correlation between jazzrsquos popularity in Monroe and his examples (Good logical flow and clarity)

Second the author uses as evidence the nationwide study that jazz fans spend much money on jazz entertainment to substantiate why starting a jazz club in Monroe will be profitable In other words the author assumes that the characteristics of a nationwide study can be applied to Monroe The national study would lend support to the applicantrsquos claim only if residents in Monroe typify national jazz fans However the author does not provide credible evidence that this is the case Moreover the populations of jazz fans nationwide may be insubstantial Thus the author should not infer hastily that Monroersquos residents will spend much money on enjoying jazz from the nationwide study

Lastly even if jazz is popular in Monroe C Note may not be successful It is entirely possible that residents might still prefer other clubs where they have always went In addition there is another possibility that the nearest jazz club will attract many of Monroersquos people because it serves fine performances and is equipped with favorite facilities Without considering these other possibilities the author cannot make his argument convincing In sum the author presents many reasons that are insufficient in supporting his or her claim In order for the authorrsquos claims to be convincing he needs to advance more persuasive evidence such as the total number of Monroe residents who attended the jazz festival the effects on the popularity of jazz by the musicians living in Monroe and the actual number of residents who would typify themselves to be jazz fans through a local survey Without substantial evidence that C Note will be successful in Monroe the businessmen may be overinvesting in what might lead to a business failureExcellent clarity Score 50

The following appeared in a newsletter offering advice to investors

Over 80 percent of the respondents to a recent survey indicated a desire to reduce their intake of foods containing fats and cholesterol and today low-fat products abound in many food stores Since many of the food products currently marketed by Old Dairy Industries are high in fat and cholesterol the companys sales are likely to diminish greatly and company profits will no doubt decrease We therefore advise Old Dairy stockholders to sell their shares and other investors not to purchase stock in this company

Write a response in which you discuss what questions would need to be answered in order to decide whether the advice and the argument on which it is based are reasonable Be sure to explain how the answers to these questions would help to evaluate the advice

Survey 80

GRE AWA John박 박정어학원

Old Dairy could change their products and manufacture low fat dairy foodsLess competing companies Old Dairy could eventually be the only company that produces hellipImprecise numbers and measurementsCustomers may still buy high fat dairy products

The author of the newsletter is offering potentially dangerous advice by recommending Old Dairy stockholders to withdraw investment and stop purchase What is more the authorrsquos prediction debases the reputation and business of Old Dairy and if false could devoid the investment opportunity of the newsletter readers Therefore investors should examine whether the authorrsquos evidences are substantial

To begin with the author states that 80 percent of the respondents in a survey indicated a desire to reduce their intake of foods He therefore argues that Old Dairyrsquos high fat and cholesterol products would decrease in sales However the author makes a crucial error in this argument First the author provides no evidence that the surveyrsquos results are statistically reliable Were they representative of all the customers Were they chosen for the survey randomly Furthermore the desire to reduce fat and cholesterol intake is a pervasive trend in todayrsquos opulent society however the author erroneously identifies this as a new phenomenon which will affect consumer trends Second having a desire to reduce fat and cholesterol intake does not necessarily indicate that people who have this desire will actually reduce consuming these types of products It is entirely possible that they may continue buying Old Dairy products for its quality and taste Accordingly the author cannot draw any firm conclusion that people will not buy Old Dairy products Therefore if any of these cases are true the author may be offering investors a detrimental investment advice

최근 조사에 대한 응답자중 80 이상이 자신이 먹는 음식에서 지방과 콜레스테롤의 함유량을 줄이고 싶다고 한다 아울러 요즘은 많은 식료품 가계에서 저지방 제품들을 많이 취급하고 있다 현재 Old Dairy Industries가 판매하고 있는 많은 음식제품들은 지방과 콜레스테롤이 높기 때문에 이 회사의 매출이 격감할 것으로 보이며 당연히 매출이익도 줄어들것이다 따라서 이 회사의 주주들은 주식을 매각하고 다른 주식 투자가들도 이 회사의 주식을 매입하지 않는 것이 좋다

결론 Old Dairy stockholders to sell their shares and other investors not to purchase stock in this company

반박 모든 상품이 다 고 지방 고 칼로리는 아니다(비록 많을지라도) 일부의 식품의 경우 기호에 맞어서 히트해서 전체적인 수입이 증가할 수도 있다국내시장만 생각할 수 없다( 외국시장에서 호황을 누릴수 있다 )입맛이라는게 즉각 바뀌는게 아니다

The following appeared in a letter to the editor of the Balmer Island Gazette

On Balmer Island where mopeds serve as a popular form of transportation the population increases to 100000 during the summer months To reduce the number of accidents involving mopeds and pedestrians the town council of Balmer Island should limit the number of mopeds rented by the islands moped rental companies from 50 per day to 25 per day during the summer season By limiting the number of rentals the town council will attain the 50 percent annual reduction in moped accidents that was achieved last year on the neighboring island of Seaville when Seavilles town council enforced similar limits on moped rentals

1 Write a response in which you discuss what questions would need to be answered in order to decide whether the recommendation is likely to have the predicted result Be sure to explain how the answers to these questions would help to evaluate the recommendation

2 Write a response in which you discuss what questions would need to be answered in order to decide whether the prediction and the argument on which it is based are reasonable Be sure to explain how the answers to these questions would help to evaluate the prediction

3 Write a response in which you examine the stated andor unstated assumptions of the argument Be sure to explain how the argument depends on these assumptions and what the implications are for the argument if the assumptions prove unwarranted

Whatrsquos the actual population of Balmer Island 100000mdashis this a significant increase What kind of accidents Skin abrasions or serious injury And compared to Seaville how serious are the accidents and the actual number of accidents Did Seaville enforce other restrictions like safety signsHow different are the conditions of Balmer

GRE AWA John박 박정어학원

and Seaville regarding population road (safety) conditions topography other town-government regulation How much will the economy of Balmer be affected do to this restriction Could it cause an economic recession due to the fact that these rental companiesrsquo chance to make money is only during the summer thereby weakening the economic infrastructure Are there any other ways that could better alleviate the accident rate

Statistics 50-impreciseAnalogy Balmer compared with TorseauFalse Cause Accidents might have occurred because of reasons other than mopeds False Cause population increase may not be part of the cause of the accidentsOther explanations for the accident pedestrians few road safety regulations narrow roadsThere could be other better solutionshellip

Balmer Island의 인구가 여름철에는 십만명으로 늘어난다 2륜차와 보행자간 사고를 줄이기 위해 시의회는 6개의 자전거를 포함한 2륜차 대여업체에게 이 기간동안에는 대여숫자를 일일 50에서 30으로 제한하도록 할 것이다 대여숫자를 줄임으로써 시 의회는 지난해 이웃한 Torseau섬에서 이와 동일한 규제를 시행해서 50나 줄인 결과를 보고 마찬가지로 50를 줄일수 있다고 확신하고 있다

결론 The town council of Balmer Island should linit the number

반박 보행자의 부실에 의해서 사고가 많이 일어날수도 있다렌탈수의 줄임만이 대책은 아니다(대부분의 사람들이 렌탈 보다는 소유하고 있을 수도 있다)옆섬과는 상황이 다를수도 있다(그 섬에서는 사고의 원인이 많은 자전거 수로 인한것일수있다) 하지만 이 섬은 좁은 도로가 원인일 수도 있고 도로 안전 장치의미비가 원일일수 있다

In this letter the author recommends that Balmer Island should limit the number moped rentals from 50 to 30 per day To support this recommendation the author points out several reasons However careful scrutiny of each of the facts reveals that it is filled with unanswered questions that could significantly weaken the authorrsquos recommendation with loops and holes which are answered

The recommendation depends on the assumption that no alternative means of reducing the number of accidents are available However the author fails to offer any evidence to substantiate this crucial assumption It is highly possible that means other than this would better solve the problem Perhaps they could widen the roads or put-up more safety signs Or perhaps the accidents were due to the lack of skills in which case proper safety training would significantly alleviate the problem Without considering and ruling out these and other alternative means of reducing accidetns the author cannot confidently conclude that merely emulating Torseau would suffice Moreover the author is advising a recommendation which could potentially harm the economy of Balmer Island sincehellip Moreover the Balmer Island should alternative means to reduce accidents because limiting moped rentals during the summer could harm the economy of Balmerhellip

First of all the author believes that increase in population and the number of moped rentals are responsible for the accidents It is entirely possible that other factors are responsible for the accidents Perhaps Balmer Islandrsquos lack of safety signs was a major factor Or maybe the roads are narrow and dangerous on the Island therefore the town council could enforce stricter traffic regulations to alleviate the problem Accordingly if either of these scenarios is true the author cannot draw any firm conclusion that increase in the number of population and moped rentals are the cause of the accidents

The author of this editorial recommends that to reduce accidents involving mopeds andpedestrians Balmer Islands city council should restrict moped rentals to 30 per day down from50 at each of the islands six rental outlets To support this recommendation the author citesthe fact that last year when nearby Torseau Islands town council enforced similar measuresTorseaus rate of moped accidents fell by 50 For several reasons this evidence providesscant support for the authors recommendationTo begin with the author assumes that all other conditions in Balmer that might affect therate of moped-pedestrian accidents will remain unchanged after the restrictions are enactedHowever with a restricted supply of rental mopeds people in Balmer might purchase mopedsinstead Also the number of pedestrians might increase in the future with more pedestriansespecially tourists the risk of moped-pedestrian accidents would probably increase For thatmatter the number of rental outlets might increase to make up for the artificial supplyrestriction per outlet--a likely scenario assuming moped rental demand does not declineWithout considering and ruling out these and other possible changes that might contribute to ahigh incidence of moped-pedestrian accidents the author cannot convince me that theproposed restrictions will necessarily have the desired effect

GRE AWA John박 박정어학원

Next the author fails to consider other possible explanations for the 50 decline inTorseaus moped accident rate last year Perhaps last year Torseau experienced unusually fairweather during which moped accidents are less likely Perhaps fewer tourists visited Tot seanlast year than during most years thereby diminishing the demand for rental mopeds to belowthe allowed limits Perhaps last year some of Torseaus moped rental outlets purchased newmopeds that are safer to drive Or perhaps the restrictions were already in effect but were notenforced until last year In any event a decline in Torseaus moped accident rate during onlyone year is scarcely sufficient to draw any reliable conclusions about what might have causedthe decline or about what the accident rate will be in years aheadAdditionally in asserting that the same phenomenon that caused a 50 decline in mopedaccidents in Torseau would cause a similar decline in Balmer the author relies on what mightamount to an unfair analogy between Balmer and Torseau Perhaps Balmers ability to enforcemoped-rental restrictions does not meet Torseaus ability if not then the mere enactment ofsimilar restrictions in Balmer is no guarantee of a similar result Or perhaps the demand formopeds in Torseau is always greater than in Balmer Specifically if fewer than all availablemopeds are currently rented per day from the average Balmer outlet while in Torseau everyavailable moped is rented each day then the proposed restriction is likely to have less impacton the accident rate in Balmer than in TorseauFinally the author provides no evidence that the same restrictions that served to reduce theincidence of all moped accidents by 50 would also serve to reduce the incidence ofaccidents involving mopeds and pedestrians by 50 Lacking such evidence it is entirelypossible that the number of moped accidents not involving pedestrians decreased by a greaterpercentage while the number of moped-pedestrian accidents decreased by a smallerpercentage or even increased Since the author has not accounted for these possibilities theeditorials recommendation cannot be taken seriouslyIn conclusion the recommendation is not well supported To convince me that the proposedrestriction would achieve the desired outcome the author would have to assure me that nochanges serving to increase Balmers moped-pedestrian accident rate will occur in theforeseeable future The author must also provide dear evidence that last years decline inmoped accidents in Torseau was attributable primarily to its moped rental restrictions ratherthan to one or more other factors In order to better evaluate the recommendation I wouldneed more information comparing the supply of and demand for moped rentals on the twoislands I would also need to know the rate of mopedpedestrian accidents in Torseau both priorto and after the restrictions were enforced in TorseauThe following appeared in a magazine article about planning for retirement

Clearview should be a top choice for anyone seeking a place to retire because it has spectacular natural beauty and a consistent climate Another advantage is that housing costs in Clearview have fallen significantly during the past year and taxes remain lower than those in neighboring towns Moreover Clearviews mayor promises many new programs to improve schools streets and public services And best of all retirees in Clearview can also expect excellent health care as they grow older since the number of physicians in the area is far greater than the national average

Write a response in which you discuss what specific evidence is needed to evaluate the argument and explain how the evidence would weaken or strengthen the argument

-Natural beauty and consistent climate may not be the most wanted qualities-Housing costs could have lowered on a national level wealthy retirees may not care about costs-Taxes may be high compared to the nationrsquos average tax rate-What about other qualities of Clearview Crime rate what qualities would retirees want -If schools streets and public services need improvement then this is proof that the current condition of Clearview is low Or due to budgetary reasons the mayor may not follow-up on his promise because of lowered tax rate -Schools and people who are retired no relationship-Physicians What kind of physicians Number is irrelevant Are these physicians capable of addressing the illnesses of old people

This author argues that anyone seeking a place to retire should choose Clearview To supportthis argument the article cites Clearviews consistent climate and natural beauty its fallinghousing costs its low property taxes compared to nearby towns and the mayors promise toimprove schools streets and services The article also claims that retirees can expectexcellent health care because the number of physicians in Clearview greatly exceeds thenational average This argument is flawed in several critical respectsTo begin with although consistent climate and natural beauty might be attractive to manyretirees these features are probably not important to all retirees For many retirees it isprobably more important to live near relatives or even to enjoy changing seasons Thus I

GRE AWA John박 박정어학원

cannot accept the authors sweeping recommendation for all retirees on this basisAlso Clearviews declining housing costs do not necessarily make Clearview the best placeto retire for two reasons First despite the decline Clearviews housing costs might be highcompared to housing costs in other cities Secondly for wealthier retirees housing costs arenot likely to be a factor in choosing a place to retire Thus the mere fact that housing costshave been in decline lends scant support to the recommendationThe articles reliance on Clearviews property-tax rates is also problematic in two respectsFirst retirees obviously have innumerable choices about where to retire besides Clear viewand nearby towns Secondly for retirees who are well-off financially property taxes are notlikely to be an important concern in choosing a place to retire Thus it is unfair to infer fromClearviews property-tax rates that retirees would prefer ClearviewYet another problem with the argument involves the mayors promises In light of Clearviewslow property-tax rates whether the mayor can follow through on those promises is highlyquestionable Absent any explanation of how the city can spend more money in the areas citedwithout raising property taxes I simply cannot accept the editorials recommendation on thebasis of those promises Besides even if the city makes the improvements promised thoseimprovements--particular the ones to schools--would not necessarily be important to retireesFinally although the number of physicians in Clearview is relatively high the per capitanumber might be relatively low Moreover it would be fairer to compare this per capita numberwith the per capita number for other attractive retirement towns--rather than the nationalaverage After all retirees are likely to place a relatively heavy burden on health-careresources Besides the article provides no assurances that the number of physicians inClearview will remain high in the foreseeable futureIn conclusion the recommendation is poorly supported To strengthen it the author mustconvince me--perhaps by way of a reliable survey--that the key features that the vast majorityof retirees look for in choosing a place to live are consistent climate natural beauty and lowhousing costs The author must also provide better evidence that Clear views property taxesare lower than the those of cities in other areas The author must also explain how the city canmake its promised improvements without raising property taxes Finally to better assess theargument I would need to now how the per capita number of physicians in Clearview wouldcompare to the national average in the futureThe following appeared as a letter to the editor from a Central Plaza store owner

Over the past two years the number of shoppers in Central Plaza has been steadily decreasing while the popularity of skateboarding has increased dramatically Many Central Plaza store owners believe that the decrease in their business is due to the number of skateboard users in the plaza There has also been a dramatic increase in the amount of litter and vandalism throughout the plaza Thus we recommend that the city prohibit skateboarding in Central Plaza If skateboarding is prohibited here we predict that business in Central Plaza will return to its previously high levels

Write a response in which you discuss what questions would need to be answered in order to decide whether the recommendation is likely to have the predicted result Be sure to explain how the answers to these questions would help to evaluate the recommendation

Why two years ago What happened two years ago which started this declineIs the dramatic increase in the ldquopopularityrdquo of skateboarding the cause of the steady decline of shoppers Are there any malls nearby Were there any changes nearby which could affect the decline in customersmdasha big mall perhaps Could the decline be due to the shop ownersHow many skateboarders use the plazaWhere do they skateboardDo they shop and are they customersAre the increase in litter and vandalism due to skateboarders Could this be alleviated by installing CCTVs and hiring security

This editorial concludes that the city should ban skateboarding from its downtown CentralPlaza in order to attract visitors to that area to return the area to its former glory and to makeit a place where people can congregate for fun and relaxation To justify this conclusion theeditorial points out that skateboarders are nearly the only people one sees anymore at CentralPlaza and that the Plaza is littered and its property defaced The editorial also points out thatthe majority of downtown merchants support the skate boarding ban This argument is flawedin several critical respectsFirst the editorials author falsely assumes that a ban on skateboarding is both necessaryand sufficient to achieve the three stated objectives Perhaps the city can achieve thoseobjectives by other means as well--for example by creating a new mall that incorporates anattractive new skateboard park Even if banning skateboarders altogether is necessary to meetthe citys goals the author has not shown that this action by itself would suffice Assuming thatthe Plazas reputation is now tarnished restoring that reputation and in turn enticing peopleback to the Plaza might require additional measures--such as removing litter and graffiti

GRE AWA John박 박정어학원

promoting the Plaza to the public or enticing popular restaurant or retail chains to the PlazaSecondly the editorial assumes too hastily that the Plazas decline is attributable to theskateboarders--rather than to some other phenomenon Perhaps the Plazas primary appeal inits glory days had to do with particular shops or eateries which were eventually replaced byless appealing ones Or perhaps the crime rate in surrounding areas has risen dramatically forreasons unrelated to the skateboarders presence at the Plaza Without ruling out these andother alternative explanations for the Plazas decline the editorials author cannot convince methat a skateboard ban would reverse that declineThirdly the editorials author might be confusing cause with effect--by assuming that theskateboarders caused the abandonment of the Plaza rather than vice versa It is entirelypossible that skateboarders did not frequent the Plaza until it was largely abandoned--andbecause it had been abandoned In fact this scenario makes good sense since skateboardingis most enjoyable where there are few pedestrians or motorists to get in the wayFourth it is unreasonable to infer from the mere fact that most merchants favor the ban thatthe ban would be effective in achieving the citys objectives Admittedly perhaps thesemerchants would be more likely to help dean up the Plaza area and promote their businesseswere the city to act in accordance with their preference Yet lacking any supporting evidencethe author cannot convince me of this Thus the survey amounts to scant evidence at best thatthe proposed ban would carry the intended resultFinally the author recommends a course of action that might actually defeat the citysobjective of providing a fun and relaxing place for people to congregate In my experienceskateboarding contributes to an atmosphere of fun and relaxation for adults and children alikemore so than many other types of ambiance Without considering that continuing to allowskateboarding--or even encouraging this activity--might achieve the citys goal more effectivelythan banning the activity the author cannot convincingly conclude that the ban would be in thecitys best interestsIn sum the argument is a specious one To strengthen it the editorials author must providedear evidence that skateboarding and not some other factor is responsible for the conditionsmarking the Plazas decline The author must also convince me that no alternative means ofrestoring the Plaza are available to the city and that the proposed ban by itself would suffice toattract tourists and restore the Plaza to its former glory Finally to better assess the argument itwould be useful to know the circumstances under which the downtown merchants would bewilling to help the city achieve its objectives

6그룹 약한 비유 빈출

The following recommendation appeared in a memo from the mayor of the town of Hopewell

Two years ago the nearby town of Ocean View built a new municipal golf course and resort hotel During the past two years tourism in Ocean View has increased new businesses have opened there and Ocean Views tax revenues have risen by 30 percent Therefore the best way to improve Hopewells economymdashand generate additional tax revenuesmdashis to build a golf course and resort hotel similar to those in Ocean View

Write a response in which you examine the stated andor unstated assumptions of the argument Be sure to explain how the argument depends on these assumptions and what the implications are for the argument if the assumptions prove unwarranted

GRE AWA John박 박정어학원

Assumptions The author assumes that OVrsquos municipal golf course and resort hotel caused tourism new businesses and increased tax revenues There may be other reasons advertising promo He assumes that this will continueAssumes that Ocean View and Hopewell are similar in many waysmdashthe name suggests otherwise OV may have always been a tourist attractions for its beaches We need to know the topography

2년전 Ocean View 시는 시정 소유 골프 및 휴양지 호텔을 신축했다 그리고 지난 2년동안 이 시의 관광객이 증가했으며 새로운 사업들이 생겨났다 그에따라 시의 세수도 30나 증가했다 Hopewell의 경제를 향상시키고 아울러 세수를 늘릴 수 있는 가장 좋은 방법은 Ocean View에 세워진 것과 같은 골프 시설과 휴양지 호텔을 신축하는 것이다

1 다른 요인으로 관광 산업이 발전했을 수도 있다 문화 유적이 발견이 되었거나 도로의 정비등으로 여행자가 늘었을 수도 있다

2 관광 산업의증가가 늘어난 세수의 원인이 아니라 새로 유입된 인구의 증가나 다른 공장에서 발생한 것일 수 있다

3 2년동안 한참 골프가 붐을 이루었을 수 있다 경제상황이 나빠지거나 다른 레포츠가 인근 지역에 생겨난다면 골프하는 사람이 줄어들 수 있다

In this memo HopeweUs mayor recommends that in order to stimulate the towns economyand boost tax revenues HopeweU should build a new golf course and resort hotel just as thetown of Ocean View did two years ago To support this recommendation the mayor points outthat in Ocean View during the last two years tourism has increased new businesses haveopened and tax revenues have increased by 30 I find the mayors argument unconvincingin several important respectsFirst of all it is possible that the mayor has confused cause with effect respecting the recentdevelopments in Ocean View Perhaps Ocean Views construction of a new golf course andhotel was a response to previous increases in tourism and business development increasesthat have simply continued during the most recent two years Since the mayor has failed toaccount for this possibility the claim that Hopewell would boost its economy by alsoconstructing a golf course and hotel is completely unwarrantedSecondly the mayor fails to account for other possible causes of the trends in Ocean Viewduring the last two years The increase in tourism might have been due to improving economicconditions nationwide or to unusually pleasant weather in the region The new businessesthat have opened in Ocean View might have opened there irrespective of the new golf courseand hotel And the 30 increase in tax revenues might have been the result of an increase intax rates or the addition of a new type of municipal taxWithout ruling out these and other alternative explanations for the three recent trends inOcean View the mayor cannot reasonably infer based on those trends that Hopewellseconomy would benefit by following Ocean Views exampleThirdly even if the recent trends in Ocean View are attributable to the construction of the newgolf course and hotel there the mayor assumes too hastily that the golf course and hotel willcontinue to benefit that towns overall economy The mayor has not accounted for thepossibility that increased tourism will begin to drive residents away during tourist season orthat new business development will result in the towns losing its appeal as a place to visit or tolive Unless the mayor can convince me that these scenarios are unlikely I cannot accept themayors recommendation that Hopewell follow Ocean Views exampleFinally the mayors argument rests on the unsubstantiated assumption that Hopewell andOcean View are sufficiently alike in ways that might affect the economic impact of a new golfcourse and hotel Hopewell might lack the sort of natural environment that would attract moretourists and new businesses to the town--regardless of its new golf course and hotel For thatmatter perhaps Hopewell already contains several resort hotels and golf courses that are notutilized to their capacity If so building yet another golf course and hotel might amount to amisallocation of the towns resources--and actually harm the towns overall economyIn sum the mayors recommendation is not well supported To bolster it the mayor mustprovide better evidence that Ocean Views new golf course and hotel and not some otherphenomenon--has been responsible for boosting Ocean Views economy during the last twoyears To better assess the recommendation I would need to know why Ocean View decidedto construct its new golf course and hotel in the first place--specifically what events prior toconstruction might have prompted that decision I would also need to thoroughly compare

GRE AWA John박 박정어학원

HopeweU with Ocean View--especially in terms of their appeal to tourists and businesses--todetermine whether the same course of action that appears to have boosted Ocean Viewseconomy would also boost Hopewells economy

The following is part of a memorandum from the president of Humana University

Last year the number of students who enrolled in online degree programs offered by nearby Omni University increased by 50 percent During the same year Omni showed a significant decrease from prior years in expenditures for dormitory and classroom space most likely because instruction in the online programs takes place via the Internet In contrast over the past three years enrollment at Humana University has failed to grow and the cost of maintaining buildings has increased along with our budget deficit To address these problems Humana University will begin immediately to create and actively promote online degree programs like those at Omni We predict that instituting these online degree programs will help Humana both increase its total enrollment and solve its budget problems

Write a response in which you discuss what questions would need to be answered in order to decide whether the prediction and the argument on which it is based are reasonable Be sure to explain how the answers to these questions would help to evaluate the prediction

Is Omni University successful due to the online degree program 50 Is the decrease in expenditures for dormitory and classroom space due to the decrease in of on-campus students Which classes were successful Does HU have those classes

Even if the long-distance degree programs at Omni University benefited the school the presidentrsquos recommendation that Human College should emulate Omni University is too hasty First OUrsquos name implies that the school would have more majors than Humanahellip the president should examine which degrees were in the long-distance programhellip

지난해에는 Omni 대학에서 개강했던 원거리 학생 학점 취득 프로그램을 등록했던 학생들의 숫자가 50나 증가했다 같은해 기간동안 Omni 대학에서는 그 전년도부터 기숙사와 학급의 공간 확충을 위한 예산을 대폭 줄였는데 이는 이 원거리 학점 취득 프로그램이 양방향 비디오 컴퓨터 접속을 통해서만 가능한 수업지도 방식이기때문인 것으로 보인다 반면 지난 3개년 동안 Humana 대학에서의 수강률은 감소한데다가 건물

GRE AWA John박 박정어학원

유지비도 올랐다 따라서 Humana대학의 수강을 늘리고 예산손실을 회복하기 위해서는 Omni 대학에서 취한 조치와 같은 능동적인 프로그램을 추진해야 한다

결론 we should initiate and actively promote long-distance degree programs like those at Omni 반박 원거리 학생 취득 프로그램 숫자가 증가한거하고 예산이 줄어드는 것 사이에 연관이 약하다 (causal 학생의 증가로 관리비용 증가할수 있음 원거리 수업가능 장비도입에의한 비용발생)bad analogy(omni university 하고 같은 조건이 아니다 )-gt omni college 가 강좌내용이 좋아서 학생의 등록이 많을수 있다 Humana 대학에서 만들었다 하더라도 인기 없을수 있음다른 요인에 의해서 Humana 대학의 수강 인원이 증가할수 있음(비록 과거엔 인기가 없었을지라도)

The following appeared as part of a business plan developed by the manager of the Rialto Movie Theater

Despite its downtown location the Rialto Movie Theater a local institution for five decades must make big changes or close its doors forever It should follow the example of the new Apex Theater in the mall outside of town When the Apex opened last year it featured a video arcade plush carpeting and seats and a state-of-the-art sound system Furthermore in a recent survey over 85 percent of respondents reported that the high price of newly released movies prevents them from going to the movies more than five times per year Thus if the Rialto intends to hold on to its share of a decreasing pool of moviegoers it must offer the same features as Apex

Write a response in which you discuss what questions would need to be answered in order to decide whether the recommendation is likely to have the predicted result Be sure to explain how the answers to these questions would help to evaluate the recommendation

Before following through this business plan the manager should investigate the cause of Rialtorsquos unsuccessful business

The author provides no evidence that the surveyrsquos results are statistically reliable The surveyrsquos sample of 85 percent must be sufficient in size and representative of overall population of the city where Rialto and Apex is serving Lacking evidence of a sufficiently representative sample the author cannot justifiably rely on the survey to draw any conclusion whatsoever The author does not indicate that Apex is indeed currently successful However even if Apex is enjoying success the argument relies on what might be a false analogy between Rialto and Apex In order for Apex to serve as a model that Rialto should emulate the author must assume that all relevant circumstances are essentially the same However this assumption is unwarranted For example the argument overlooks the face that Apex is located in a strategic placemdashbeside a mall where customers can not only watch a movie but also enjoy shopping Therefore simply changing the facility to that of Apex may not lead to success

The author does not mention whether Apex is successful or not Nevertheless even if Apex is currently successful the argument relies on what might be a false analogy between Rialto and Apex In order for Apex to serve as a model that Rialto should emulate the author must assume that all relevant circumstances are essentially the same However this assumption is unwarranted For example the argument overlooks the fact that these two institutions are located in different locations Rialto in downtown and Apex in a mall outside of town Although Apex opened with state-of-the-art facilities the decisive factor in its success could be due to its strategic location of being in a mall People could enjoy both shopping and movies at one location thus they may prefer Apex over Rialto Furthermore the place where people enjoy leisure activities has shifted in the past decades for most cities from downtown to the suburbs Therefore Rialto may not be successful even if it emulates Apexrsquos facilities A better business plan may be relocating Apex to the thriving section of the downtown

Rialto 극장은 지난 50여년간 지역 회관으로써 시내에 위치해 있으면서도 이제 변화를 꾀하지 않으면 문을 닫을

GRE AWA John박 박정어학원

판이다 이 극장은 시외 쇼핑타운에 새로 들어선 Apex 극장의 사례를 본받아야 했다 Apex가 지난해 개업했을 당시 이 극장은 비디오 아케이드 플러쉬 카펫트 바닥과 좌석 그리고 최신 음향시설을 갖추었다 더군다나 최근 조사에서는 응답자의 85 이상이 새로 출시된 영화 입장료가 비싼 탓으로 지난해보다 5배이상의 관람객이 줄어들었다고 나타났다 따라서 Rialto 극장이 줄어들고 있는 관람객을 뺐기지 않고 유지하려면 Apex와 같은 시설들을 갖추어야 할 것이다주장 리알토 극장이 줄어들고 있는 관람객을 뺐기지 않고 유지하려면 Apex와 같은 시설들을 갖추어야 할 것이다

1 조사에서 응답자가 전체를 대표할 수 없다 2 apex 극장이 좋은 시설을 갖추고 있지만 그로 인해 수익이 많이 발생했다는 말이 없으므로 시설투자를

하고도 좋은 결과를 얻을 수 있을지 그 근거가 미흡하다3 좋은 영화가 출시된다면 입장료가 비싸도 영화관에서 꼭 보려고 할 수 있다 4 rialto 가 시설이 아닌 다른 요인에 의해 장사가 안될수도 있다( 우범 지역이라든지)

The following is a recommendation from the business manager of Monarch Books

Since its opening in Collegeville twenty years ago Monarch Books has developed a large customer base due to its reader-friendly atmosphere and wide selection of books on all subjects Last month Book and Bean a combination bookstore and coffee shop announced its intention to open a Collegeville store Monarch Books should open its own in-store cafeacute in the space currently devoted to childrens books Given recent national census data indicating a significant decline in the percentage of the population under age ten sales of childrens books are likely to decline By replacing its childrens books section with a cafeacute Monarch Books can increase profits and ward off competition from Book and Bean

Write a response in which you examine the stated andor unstated assumptions of the argument Be sure to explain how the argument depends on these assumptions and what the implications are for the argument if the assumptions prove unwarranted

The following is a recommendation from the business manager of Monarch Books

Since its opening in Collegeville twenty years ago Monarch Books has developed a large customer base due to its reader-friendly atmosphere and wide selection of books on all subjects Last month Book and Bean a combination bookstore and coffee shop announced its intention to open a Collegeville store Monarch Books should open its own in-store cafeacute in the space currently devoted to childrens books Given recent national census data indicating a significant decline in the percentage of the population under age ten sales of childrens books are likely to decline By replacing its childrens books section with a cafeacute Monarch Books can increase profits and ward off competition from Book and Bean

1 Write a response in which you discuss what questions would need to be answered in order to decide whether the recommendation is likely to have the predicted result Be sure to explain how the answers to these questions would help to evaluate the recommendation

2 Write a response in which you discuss what specific evidence is needed to evaluate the argument and explain how the evidence would weaken or strengthen the argument

No evidence regarding Monarch Bookrsquos successEven if Regal Bookrsquos is successful this may not be attributable to the cafeacute False analogy Emulating may not lead to success Other factors may be involvedInsufficient condition The national census is not enough evidence that childrenrsquos book sales will decline Can

GRE AWA John박 박정어학원

the national census represent the local child populationDid opening a cafeacute boost sales for Regal Books Even assuming Regal is successful by opening a cafeacute this may not be suitable for Monarch which plans to close the childrenrsquos book section to establish a cafe Imprecise language ldquorelatively little spacerdquo how smallThe managerrsquos recommendation contradicts what he says Since Monarch is popular for its wide selection of books closing a selection which targets a major group of readers may hurt Monarchrsquos salesIs this the best way to compete

When Stanley Park first opened it was the largest most heavily used public park in town It is still the largest park but it is no longer heavily used Video cameras mounted in the parks parking lots last month revealed the parks drop in popularity the recordings showed an average of only 50 cars per day In contrast tiny Carlton Park in the heart of the business district is visited by more than 150 people on a typical weekday An obvious difference is that Carlton Park unlike Stanley Park provides ample seating Thus if Stanley Park is ever to be as popular with our citizens as Carlton Park the town will obviously need to provide more benches thereby converting some of the unused open areas into spaces suitable for socializing

Write a response in which you examine the stated andor unstated assumptions of the argument Be sure to explain how the argument depends on these assumptions and what the implications are for the argument if the assumptions prove unwarranted

Stanley 파크가 처음 개장했을 당시 가장 크고 가장 많이 이용되는 공원이었다 아직도 공원중에서는 가장 크지만 이용률은 상당히 떨어졌다 지난달 공원 주차장에 설치해놓은 비디오 카메라를 통해 보면 drop(주차장으로 여겨짐) 이용률이 가장 높았다 수치상으로는 하루 평균 50대의 차량만이 이용하였다 반면 직장 중심거리에 위치한 작은 규모의 Carlton 파크는 주당 무려 150여명 이상이 이용하고 있다 Stanley 파크와는 달리 Carlton 파크에는 의자가 있다는 것이 가장 뚜렷한 차이점이다 따라서 Stanley 파크가 Carlton 파크처럼 시민들이 자주 이용하는 공원이 되기 위해서는 벤치를 설치할 필요가 있으며 이렇게 사용되지 않는 일부 공간을 활용해서 사교를 위한 공간으로 바꾸어야 한다 ===gtdrop 에 대한 첨부사항 (영영사전내용입니다)---- a place or central depository to which something (as mail money or stolen property) is brought for distribution or transmission also the act of depositing something at such a place dropgt

주장 if Stanley Park is ever to be as popular with our citizens as is Carlton Park the town will obviously need to provide more benches thereby converting some of the unused open areas into spaces suitable for socializing1 조사가 언제 이루어진 것인가 조사가 언제 실시되었느냐에 따라 결과가 다를 수있다 현재는 다시 스탠리 파크가 늘어났었을 수 있다 2 벤치를 많이 설치했다고 해서 많은 관광객이 오지 않을수 있다(사람들이 벤치나 사교 공간을 원한다는 어떠한 자료도 없다)3스탠리 파크 주변에 교통 상황이 악화가 되었거나 칼튼 파크에서 문화행사등을 많이 가져서 이용객이 줄어든것일 수도 있다 4 칼튼 파크가 중심지에 있어서 접근성이 좋을수 있다5 조사가 같은 시간을 기준으로 한게 아니다(하나는 주중이고 하나는 주말이다)6사람의 수와 차의 대수를 같은것으로 비교할수 없다 (차안에 몇 명이 타고 있는지 모르고 대중교통을 이용해서 왔을수도 있다)

Page 19: GRE writing argument brain storm

GRE AWA John박 박정어학원

4그룹 거짓인과관계 오류 (False Cause) 빈출

Fifteen years ago Omega University implemented a new procedure that encouraged students to evaluate the teaching effectiveness of all their professors Since that time Omega professors have begun to assign higher grades in their classes and overall student grade averages at Omega have risen by 30 percent Potential employers looking at this dramatic rise in grades believe that grades at Omega are inflated and do not accurately reflect student achievement as a result Omega graduates have not been as successful at getting jobs as have graduates from nearby Alpha University To enable its graduates to secure better jobs Omega University should terminate student evaluation of professors

Write a response in which you discuss what specific evidence is needed to evaluate the argument and explain how the evidence would weaken or strengthen the argument

Omega professor evaluation implemented 15 years ago =gt Omega prof assign higher grades 30Employers believe therersquos grade inflation

Thus unsuccessful employment than AlphaTherefore to secure jobs Omega should end evaluating profs

Specific evidence neededRelationship between higher grades and evaluationRelationship between GPA and unsuccessful employmentAlpharsquos education could just be better than OmegaldquoFifteen years agordquo is a long time other factors could have influenced Why is the inflation a problem just now How much is Alpha better Is the comparison just How much gap is thereOmegarsquos student could just be doing better in their studiesComparison to other universities다른 대안 없나hellip Could Omega alleviate the employment problem by implementing a different procedure or program

15 년전 우리 대학은 학생들로 하여금 교수평가를 하도록 한 새로운 조치를 시행했었습니다 이후 교수들은 자신의 학과 학생들에게 높은 학점을 주었으며 그에따라 학생들의 전체 평점이 30나 올랐습니다 외부의 기업체들은 분명 점수가 지나치게 부풀려졌다고 믿고 있습니다 결국 본 대학 졸업생들이 인근 Alpha 대학의 졸업자들보다 구직률이 떨어지는 이유를 잘 보여주고 있는 것입니다 이를 해결하기 위해 이제부터는 학생들에 의한 교수평가제를 중단해야 합니다

결론 Omega University should now terminate student evaluation of professors

반박 교수 평가와 학점 인플레의 연관성이 적다( 교수 평가를 먼저하고 학점을 나중에 매길수도 있다)채용기준에 성적만 있는게 아니다 학업성취의 결과 일수도 있다 Alpha 가 원래 유능했다 Alpha 의 교육내용이 좋았다

GRE AWA John박 박정어학원

In this memo the dean of Omega University(OU) recommends OU to terminate professor evaluation to secure better jobs for the students To support this recommendation the dean offers several reasons However this argument contains several logical flaws which render it unconvincing

A threshold problem with the argument involves the voluntary nature of the evaluationprocedure The dean provides no evidence about the number or percentage of Omegastudents who participate in the procedure Lacking such evidence it is entirely possible thatthose numbers are insignificant in which case terminating the procedure is unlikely to haveany effect on the grade average of Omega students or their success in getting jobs aftergraduationThe argument also assumes unfairly that the grade-average increase is the result of theevaluation procedure--rather than some other phenomenon The dean ignores a host of otherpossible explanations for the increase--such as a trend at Omega toward higher admissionstandards or higher quality instruction or facilities Without ruling out all other possibleexplanations for the grade-average increase the dean cannot convince me that by terminatingthe evaluation procedure Omega would curb its perceived grade inflation let alone help itsgraduates get jobsEven if the evaluation procedure has resulted in grade inflation at Omega the deans claimthat grade inflation explains why Omega graduates are less successful than Alpha graduatesin getting jobs is unjustified The dean overlooks a myriad of other possible reasons forOmegas comparatively poor job-placement record Perhaps Omegas career services areinadequate or perhaps Omegas curriculum does not prepare students for the job market aseffectively as Alphas In short without accounting for other factors that might contribute toOmega graduates comparative lack of success in getting jobs the dean cannot justify theclaim that if Omega curbs its grade inflation employers will be more likely to hire OmegagraduatesFinally even if the dean can substantiate all of the foregoing assumptions the deansassertion that Omega must terminate its evaluation procedure to enable its graduates to findbetter jobs is still unwarranted in two respects First the dean ignores other possible ways bywhich Omega can increase its job-placement record--for example by improving its publicrelations or career-counseling services Second the dean unfairly equates more jobs withbetter jobs In other words even if more Omega graduates are able to find jobs as a result ofthe deans recommended course of action the kinds of jobs Omega graduates find would notnecessarily be better onesIn sum the deans argument is unpersuasive as it stands To strengthen it the dean mustprovide better evidence that the increase in grade average is attributable to Omegasprofessor-evaluation procedure and that the end result is a perception on the part ofemployers that Omega graduates are less qualified for jobs than Alpha graduates To betterassess the argument I would need to analyze 15-year trends in (l) the percentage of Omegastudents participating in the evaluation procedure (2) Omegas admission standards andquality of education and (3) Omegas emphasis on job training and career preparation I wouldalso need to know what other means are available to Omega for enabling its graduates to findbetter jobs

GRE AWA John박 박정어학원

The following appeared in a memo from a vice president of Quiot Manufacturing

During the past year Quiot Manufacturing had 30 percent more on-the-job accidents than at the nearby Panoply Industries plant where the work shifts are one hour shorter than ours Experts say that significant contributing factors in many on-the-job accidents are fatigue and sleep deprivation among workers Therefore to reduce the number of on-the-job accidents at Quiot and thereby increase productivity we should shorten each of our three work shifts by one hour so that employees will get adequate amounts of sleep

Write a response in which you examine the stated andor unstated assumptions of the argument Be sure to explain how the argument depends on these assumptions and what the implications are for the argument if the assumptions prove unwarranted

The following appeared in a memo from a vice president of Alta Manufacturing

During the past year Alta Manufacturing had thirty percent more on-the-job accidents than nearby Panoply Industries where the work shifts are one hour shorter than ours Experts believe that a significant contributing factor in many accidents is fatigue caused by sleep deprivation among workers Therefore to reduce the number of on-the-job accidents at Alta we recommend shortening each of our three work shifts by one hour If we do this our employees will get adequate amounts of sleep

Write a response in which you discuss what questions would need to be answered in order to decide whether the recommendation and the argument on which it is based are reasonable Be sure to explain how the answers to these questions would help to evaluate the recommendation

The following appeared in a memo from the vice president of Butler Manufacturing

During the past year workers at Butler Manufacturing reported 30 percent more on-the-job accidents than workers at nearby Panoply Industries where the work shifts are one hour shorter than ours A recent government study reports that fatigue and sleep deprivation among workers are significant contributing factors in many on-the-job accidents If we shorten each of our work shifts by one hour we can improve Butler Manufacturings safety record by ensuring that our employees are adequately rested

1 Write a response in which you discuss what specific evidence is needed to evaluate the argument and explain how the evidence would weaken or strengthen the argument

2 Write a response in which you discuss what questions would need to be answered in order to decide whether the recommendation is likely to have the predicted result Be sure to explain how the answers to these questions would help to evaluate the recommendation

4번 반복됨

Alta has 30 more job accidents than Panoply(work shifts one hour shorter)Experts Job accidents caused by fatigue and sleep deprivationTherefore to reduce job accidents and increase productivity shorten three work shifts by one hour for adequate sleep

지난해 우리 회사는 인근 Panoply Industries보다 업무상 재해가 30나 더 많았다 그 회사는 우리보다 근무 교대시간이 1시간 정도 짧았다 전문가들은 대부분의 업무상 재해에 있어서 가장 중요한 요인이 과로와 수면부족으로 보고있다 따라서 우리 회사에서 높은 산업재해를 줄이고 아울러 생산성을 높이기 위해서는 근로자들이 충분한 수면을 취할 수 있도록 1시간씩 3교대 시간을 줄여야 한다

In this memo the (author) vice president of Alta Manufacturing (AM) recommends that to reduce on-the-job accidents and increase productivity AM should shorten its three work shifts by one hour so that employees can

GRE AWA John박 박정어학원

get more sleep To support this recommendation the author provides several evidences However careful scrutiny of each of the facts reveals that it provides little credible support for the authorrsquos recommendation QuestionsThe number of accidents What kind of accidents The seriousness of the accidents is importantHow many employees are in each company What are their productsFalse cause Sleep may not be the reason for the on-the-job accidents What do Alta and Panoply manufacture

First of all the author believes that fatigue caused the on-the job accidents However there could be other reasons The author observes a correlation between sleep deprivation and on-the-job accidents then concludes that the former is the cause of the latter However the author fails to rule out other possible explanations For example it is entirely possible that Alta factories require more strenuous and dangerous labor than Panoply Without ruling out all other such factors it is unfair to conclude that fatigue is responsible for the accidents In addition the work-shifts may not be the cause of the sleep deprivation and fatigue It is possiblehellip Thus the author should provide what exactly Panoply and Alta manufacture and more precise data about their working conditions to be more convincing

Shortening the shift by one hour does not necessarily lead to more sleep And is one hour enoughLess accidents does not mean increased productivity

결론 We should shorten each of out three work shifts by one hour

반박 경쟁사에 비해서 시간당 하는 업무량이 많아서 더욱 피곤할 수도 있다 시간이 문제가 아니라 노후된 시설 설비 자체의 문제 작업 자체가 원래 위험한 것이여서 사고가 많을 수도 있다 다른 회사는 더욱 많은 작업시간에도 불구하고 안정한 작업여건으로 인해서 사고율이 오히려 더 작을 수도 있다비교사의 재해감소가 다른 요인일수 있다(안전 교육 철저)줄인 시간이 피로회복이나 수면으로 연결 안될수 있음(술을 마실 수도 있고 그 시간에 휴식을 취하지 않고 다른일을 함으로써 더욱 피로해질수 있다)

This editorial recommends that Alta Manufacturing reduce its work shifts by one hour each inorder to reduce its on-the-job accident rate and thereby increase Altas productivity To supportthis recommendation the author points out that last year the number of accidents at Alta was30 greater than at Panoply Industries where work shifts were one hour shorter The authoralso cites certain experts who believe that many on-the-job accidents are caused by fatigueand sleep deprivation I find this the argument unconvincing for several reasonsFirst and foremost the author provides absolutely no evidence that overall workerproductivity is attributable in part to the number of on-the-job accidents Although commonsense informs me that such a relationship exists the author must provide some evidence ofthis cause-and-effect relationship before I can accept the authors final conclusion that theproposed course of action would in fact increase Altas productivitySecondly the author assumes that some accidents at Alta are caused by fatigue or sleepdeprivation However the author overlooks other possible causes such as inadequateequipment maintenance or worker training or the inherent hazards of Altas manufacturingprocesses By the same token Panoplys comparatively low accident rate might be attributablenot to the length of its work shifts but rather to other factors such as superior equipmentmaintenance or worker training In other words without ruling out alternative causes ofon-the-job accidents at both companies the author cannot justifmbly conclude that merely byemulating Panoplys work-shift policy Alta would reduce the number of such accidentsThirdly even assuming that Altas workers are fatigued or sleep-deprived and that this is thecause of some of Altas on-the-job accidents in order to accept the authors solution to thisproblem we must assume that Altas workers would use the additional hour of free time tosleep or rest However the author provides no evidence that they would use the time in thismanner It is entirely possible that Altas workers would use that extra hour to engage in someother fatiguing activity Without ruling out this possibility the author cannot convincinglyconclude that reducing Altas work shifts by one hour would reduce Altas accident rateFinally a series of problems with the argument arise from the scant statistical information onwhich it relies In comparing the number of accidents at Alta and Panoply the author fails toconsider that the per-worker accident rate might reveal that Alta is actually safer than Panoplydepending on the total number of workers at each company Second perhaps accident rates

GRE AWA John박 박정어학원

at the two companies last year were aberrations and during other years Altas accident ratewas no greater or even lower than Panoplys rate Or perhaps Panoply is not representativeof industrial companies generally and that other companies with shorter work shifts have evenhigher accident rates In short since the argument relies on very limited statistical information Icannot take the authors recommendation seriouslyIn conclusion the recommendation for emulating Panoplys work-shift policy is not wellsupported To convince me that shorter work shifts would reduce Altas on-the-job accidentrate the author must provide clear evidence that work-shift length is responsible for some ofAltas accidents The author must also supply evidence to support her final conclusion that alower accident rate would in fact increase overall worker productivity

The following appeared in a memo from the vice president of marketing at Dura-Sock Inc

A recent study of our customers suggests that our company is wasting the money it spends on its patented Endure manufacturing process which ensures that our socks are strong enough to last for two years We have always advertised our use of the Endure process but the new study shows that despite our socks durability our average customer actually purchases new Dura-Socks every three months Furthermore our customers surveyed in our largest market northeastern United States cities say that they most value Dura-Socks stylish appearance and availability in many colors These findings suggest that we can increase our profits by discontinuing use of the Endure manufacturing process

1 Write a response in which you examine the stated andor unstated assumptions of the argument Be sure to explain how the argument depends on these assumptions and what the implications are for the argument if the assumptions prove unwarranted

2 Write a response in which you discuss what specific evidence is needed to evaluate the argument and explain how the evidence would weaken or strengthen the argument

3 Write a response in which you discuss what questions would need to be answered in order to decide whether the recommendation and the argument on which it is based are reasonable Be sure to explain how the answers to these questions would help to evaluate the recommendation

Intro The vice president of marketing at Dura-Sock Inc is offering a potentially harmful investment recommendation by claiming that Dura-Sock should discontinue its use of the ldquoEndurerdquo process To support his recommendation he points out a study that Dura-Sock customers actually purchase the socks every three months and a survey that reveals that Dura-Sock customers like the sockrsquos stylish appearance and availability in many colors The study and survey however are insufficient in supporting his proposal and the VP makes several unwarranted assumptionsIntro (simplified) The VP states that though Dura-Socks last for two years customers buy the socks every three months Therefore he assumes that the consumersrsquo motive for buying the produce is not its durabilityHowever the author fails to rule out other possible motivation for consumption

Even if the survey is reliable the author should consider the rest of the market Vague terms ldquowasting moneyrdquomdashprecisely how much are they wasting Studysurvey errorThe company must calculate the outcome of such momentous decisionStudy participantsrsquo comment that they prefer Dura-Sock for its stylishness and availability might take Dura-Sockrsquos enduring quality for granted

우리회사 제품 소비자들에 대한 최근 조사에서 지난 2년여간 양말의 내구성을 강하게 하는 필수공정이었던 자사 특허의 Endure 공정에 들어가는 비용이 낭비라고 말하고 있다 우리 회사는 항상 이 공정 처리에 대한 광고를 내보냈으나 이에 대한 시장 조사에서 실제로 고객들은 이 신제품을 평균 석달마다 구매하는 것으로 나타났다 더군다나 북동부지역에서 실시한 대규모 시장조사에 응답한 고객들은 양말의 모양과 색상등에 더

GRE AWA John박 박정어학원

관심을 나타냈다 이러한 결과는 우리회사가 신기술 공법을 중단하면 그에 따라 수익이 늘어날 것이라는 것을 말해주고 있는 것이다주장 These findings suggest that Dura0Sock can increase its profits by discontinuing its use of the ldquoEndurerdquo manufacturing process

1 survey가 정확한 소비자의 의견을 나타낸 것인가 다른 선택없이 양자택일과 같은 방법의 survey였는지2 북동부지역의 시장조사가 전체 의견을 대표할 수 있나3 사람들이 모양이나 색상에 앞서 내구성을 먼저 평가했을 수도 있다 내구성을 갖추었다는 전제하에 모양과 색상에 관심을 드러낸 것일 수 있다4 소비자가 도매상(retail)인지 소매상(whole)인지가 없다

The following appeared in a business magazine

As a result of numerous complaints of dizziness and nausea on the part of consumers of Promofoods tuna the company requested that eight million cans of its tuna be returned for testing Promofoods concluded that the canned tuna did not after all pose a health risk This conclusion is based on tests performed on samples of the recalled cans by chemists from Promofoods the chemists found that of the eight food chemicals most commonly blamed for causing symptoms of dizziness and nausea five were not found in any of the tested cans The chemists did find small amounts of the three remaining suspected chemicals but pointed out that these occur naturally in all canned foods

Write a response in which you discuss what questions would need to be addressed in order to decide whether the conclusion and the argument on which it is based are reasonable Be sure to explain how the answers to the questions would help to evaluate the conclusion

Representativeness of the tested cansThey should conduct a comparative studyThe testing could be biased because Promofoods employees conducted the testingHow much (quantity) of the five and three suspected chemicals were in the canned foodsFalse cause The substance that caused dizziness and nausea may not be one of the eight common chemicals

많은 소비자들의 현기증과 구역질 불만에 따라 Promofoods사는 지난해 참치 캔 8백만 개를 테스트하기 위해 반품시켰다 그 결과 캔에서는 건강에 위험이 될 수 있는 화합물질이 없었던 것으로 회사측은 결론지었다 이러한 결론은 회사측 화학연구자들이 회수된 캔의 샘플을 테스트해서 이들 증상의 원인이 되는 8가지 화합물 중에서 5가지가 실험된 캔에서 발견되지 않았다는 사실에 근거한 것이다 이들 화학자들은 나머지 3개가지 화합물이 모든 캔 식료품에서 흔히 발견되는 것이라고 언급했다 결론 Promofoods concluded that the cans did not after all contain chemicals that posed a health risk

1 공인된 기간에서 테스트를 한 것이 아니고 자사에서 직접 테스트를 했기에 신뢰성이 안간다 2 이런 증상을 일으키는 8개의 물질 말고 다른 물질들이 캔속에 많이 포함됬을수 있다 3 나머지 3개의 물질들의 함유량이 많아서 다른 종류의 캔들은 문제를 일으키지 않지만 참치캔은 문제를

일으킬 수 있다

This magazine article concludes that the 8 million cans of tuna Promofoods recalled due tocomplaints about nausea and dizziness do not after ail contain any chemicals that pose a

GRE AWA John박 박정어학원

health risk To support this conclusion the author cites the fact that five of eight chemicalscommonly causing these symptoms were not found in the recalled cans while the other threealso occur naturally in other canned foods For several reasons this evidence lends littlecredible support to the authors conclusionTo begin with the author relies partly on the fact that although three of the eight chemicalsmost commonly blamed for nausea and dizziness appeared in Promofoods recalled tunathese chemicals also occur naturally in other canned foods However this fact alone lends nosupport to the authors conclusion for two reasons First the author might be ignoring animportant distinction between naturally occurring chemicals and those not occurring naturallyIt is entirely possible that these three chemicals do not occur naturally in Promofoods tunaand that it is for this reason that the chemicals cause nausea and dizziness Secondly it isentirely possible that even when they occur naturally these chemicals cause the samesymptoms Unless the author rules out both possibilities he cannot reliably conclude that therecalled tuna would not cause these symptomsAnother problem with the argument is that the authors conclusion is too broad Based onevidence about certain chemicals that might cause two particular heath-related symptoms theauthor concludes that the recalled tuna contains no chemicals that pose a health risk Howeverthe author fails to account for the myriad of other possible health risks that the recalled tunamight potentially pose Without ruling out all other such risks the author cannot justifiablyreach his conclusionA third problem with the argument involves that fact that the eight particular chemicals withwhich the test was concerned are only the eight most commonly blamed for nausea anddizziness It is entirely possibly that other chemicals might also cause these symptoms andthat one or more of these other chemicals actually caused the symptoms Without ruling outthis possibility the author cannot jusufiably conclude that the recalled tuna would not causenausea and dizzinessA final problem with the argument involves thetesting procedure itself The author providesno information about the number of recaUed cans tested or the selection method used Unlessthe number of cans is a sufficiently large sample and is statistically repre sentative of all therecalled cans the studys results are not statistically reliableIn conclusion the article is unconvincing as it stands To strengthen the assertion that therecalled tuna would not cause nausea and dizziness the author must provide evidence thatthe three chemicals mentioned that occur naturally in other canned foods also appear naturallyin Promofoods tuna The author must also provide evidence that ingesting other canned foodscontaining these three chemicals does not cause these symptoms To better evaluate theargument we would need to know whether the sample used in the tests was statisticallysignificant and representative of all the recalled tuna We would also need to know what otherchemicals in the recalled tuna might pose any health risk at all

5그룹 불충분 조건오류 빈출

Natures Way a chain of stores selling health food and other health-related products is opening its next franchise in the town of Plainsville The store should prove to be very successful Natures Way franchises tend to be most profitable in areas where residents lead healthy lives and clearly Plainsville is such an area Plainsville merchants report that sales of running shoes and exercise clothing are at all-time highs The local health club has more members than ever and the weight training and aerobics classes are always full Finally Plainsvilles schoolchildren represent a new generation of potential customers these schoolchildren are required to participate in a fitness-for-life program which emphasizes the benefits of regular exercise at an early age

Write a response in which you examine the stated andor unstated assumptions of the argument Be sure to

GRE AWA John박 박정어학원

explain how the argument depends on these assumptions and what the implications are for the argument if the assumptions prove unwarranted

False cause

First of all the author believes that the Increased sales of running shoes and exercise clothing indicates

Plainesville residentsrsquo interest in leading healthy lives However this assumption is not logically convincing for

several reasons could be a fashion trendTime shift ldquoFitness for liferdquo might not have any influence on schoolchildren as they growFalse cause There could be other reasons for member increase in the health clubAll of the above are insufficient condition

The author has to prove that local residents are interested in leading healthy lives However he supports his conclusion with insufficient evidence Nevertheless even if the residents are concerned with health naturersquos way may not be successful First

그 동안의 경험을 토대로 볼 때 건강생활과 밀접히 관련되어 있는 거주 지역에서 본 상점들이 아주 호응을 얻고 있다 따라서 이러한 주민들이 많이 거주하고 있는 Plainsville 에 새로운 상점들을 계속 세워야 한다 이 지역 상인들은 런닝화와 운동복 판매가 가장 높다고 말한다 불과 5 년전에는 거의 전무하다시피하던 지역 헬스 클럽의 경우도 엄청나게 많은 회원을 확보하고 있으며 웨이트 트레이닝과 에어로빅 강좌들도 항상 만원이라고 한다 새로운 고객층을 예측해 보는 것도 가능하다 이 지역의 학생들의 경우 Fitness for Life프로그램을 받게 되는데 이러한 프로그램을 통해서 유년시절부터 정규적인 운동 습관을 들이게 하고 있는 것이 그것이다

결론 We should therefore build our next new store in Plainsville

반박 그동안의 경험에 의한 과거 통계가 꼭 여기에도 적용되는건 아니다 5 년전 헬스 클럽이 잘 안되었던게 다른 원인이였을수 있다(강사수준 미달 강좌미비)tourist 에 의한 원인 일수 있다 어렸을때부터 운동을 했다고 해서 커서도 관심이 있지는 않다 (오히려 반감이 있을수 있다 혹은 건강하기에 건강에 관심이 적을수도 있다)운동복이나 신발의 판매가 육체노동에 의한 것일수도 있다

IntroductionSupport1049896In this memorandum the author asserts that Naturersquos Way should build its next newstore in Plainsville To support this assertion the author states that Plainsvillesmerchantsrsquo sales of exercise clothing are going well the local health club has moremembers than ever and a new generation of customers will help to ensure NaturersquosWayrsquos success At first glance the authorrsquos assumption seems convincing but in-depth scrutiny revealsthat it lacks substantial evidence as it stands

Body 1-SamplingTopic Sentence 1To begin with the author assumes that the merchantsrsquo report indicates that the residentsare concerned about their health However this assumption is based on unsubstantiated

GRE AWA John박 박정어학원

data Example 1 (Rebuttal1) First if we do not know the total volume of items sold and the price of the goods exactly we cannot infer whether the residents are actually buying many goods Example 2 (Rebuttal2)In addition to that the report emphasizes the rising sales of running shoes and exerciseclothing however these may not be hot-selling items for Naturersquos Way or may not be theproducts the company is planning to sell Concluding Sentence Therefore in order to make the argument reliable the author should reconsider themerchantsrsquo report with more detailed data

Body 2-CausalTopic Sentence 2Second the author contends that the health clubs classes are full yet this does not meanthat many people actually use the health club other factors may be the real cause forthose closed classes Example 1 (Rebuttal1) To begin with if the health club is very small the number of people working out wouldnot be a large one In fact regular gym-going may just be a vogue among a smallunrepresentative segment of Plainsvilles population Example 2 (Rebuttal2) Moreover it is possible that most of the people who exercise in the health club do weight training and aerobics only to look good and to meet other singles not for their health In that case there would be little demand for health products Concluding SentenceThus the author should not hasten to presume what really caused people to be interested in a healthier lifestyle and enroll in the health club

Body 3-Time-ShiftTopic Sentence 3Finally the author highlights that Naturersquos Way can expect a new generation of customersin Plainsville that will help the company in the long term This notion is mistaken in that itassumes the conditions of the present will continue unchanged in the future Although theschool children are required to participate in the fitness for life program they may notnecessarily buy Naturersquos Ways products Example 1 (Rebuttal1) In the first instance they may suffer a fall in purchasing power arising from future economic difficulties this would cause reluctance to spend a considerable amount of money on health products which tend to be more expensive Example 2 (Rebuttal2)Another possibility is that there may emerge many competitor companies vying with Naturersquos Way so that in the future the school children may not feel the necessity to purchase one companyrsquos health products over anotherrsquosConcluding Sentence Thus the authorrsquos assumption is highly speculative since it relies heavily on unknowablefuture circumstances

ConclusionThesis In sum the author uses many assumptions that are insufficient in supporting his claimsSupportIn order for the authorrsquos claims to be convincing he needs to advance more persuasiveevidence that people in Plainsville really are concerned with their health and health foodThe following was written as a part of an application for a small-business loan by a group of developers in the city of Monroe

A jazz music club in Monroe would be a tremendously profitable enterprise Currently the nearest jazz club is 65 miles away thus the proposed new jazz club in Monroe the C-Note would have the local market all to itself Plus jazz is extremely popular in Monroe over 100000 people attended Monroes annual jazz festival last summer several well-known jazz musicians live in Monroe and the highest-rated radio program in Monroe is Jazz Nightly which airs every weeknight at 7 PM Finally a nationwide study indicates that the typical jazz fan spends close to $1000 per year on jazz entertainment

1 Write a response in which you discuss what specific evidence is needed to evaluate the argument and explain how the evidence would weaken or strengthen the argument

2 Write a response in which you examine the stated andor unstated assumptions of the argument Be

GRE AWA John박 박정어학원

sure to explain how the argument depends on these assumptions and what the implications are for the argument if the assumptions prove unwarranted

3 Write a response in which you discuss what questions would need to be answered in order to decide whether the prediction and the argument on which it is based are reasonable Be sure to explain how the answers to these questions would help to evaluate the prediction

Group error nationwide survey may not reflect local trends Is the nationwide jazz fan population substantialInsufficient non-residents of Monroe may have attended the jazz festival (Body alternative explanation last year may have been an anomaly The author should consider data from various years) The author should indicate how many out of 100000 were Monroe residentsNationwide study Does this reflect Insufficient Citizens of Monroe may continue to go to the jazz club 65 miles away

Are the people in Monroe really interested in jazzMajority of the people who attended the jazz festival might not be Monroe residentsSurvey error nationwide study may not be applicable to MonroeJazz musicians who live in MonroeMonopolyRadio station

In this business application the author claims that the proposed jazz club C Note will be very profitable in Monroe To support this claim the author argues for his case with several evidences At first glance the authorrsquos argument seems convincing however careful scrutiny reveals that his argument in specious

To begin with the author claims that Monroersquos citizens are interested in jazz He presents three evidences First Secondhellip Thirdhellip Howeverhellip

Monroe 시에 있는 재즈 음악 클럽은 수익성이 좋은 사업이다 현재 가장 가까이에 있는 클럽은 65 마일 정도 떨어져 있다 따라서 이번에 세우려고 하는 C Note 는 독보적인 위치를 점할것이다 더군다나 재즈는 이 시에서 가장 인기있는 음악이다 지난 여름 재즈 축제에서는 10 만명 이상의 Morone 시 주민이 참석하였고 몇몇 유명한 재즈 음악가들도 이곳에 살고 있으며 저녁때 방영되는 라디오 프로그램중에서 최고의 시청률을 보이고 있는 것도 Jazz Nightly 이다 전국조사에서도 전형적인 재즈 팬들은 재즈 분야에 년간 1천 달러 가까이 지출하고 있는 것으로 보고되고 있다 따라서 C Note 클럽이 돈을 벌 수 있는 사업이라는 것은 확실한 것이다

결과 It is clear that the C Note cannot help but make money반박 nearest jazz club 이 양질의 써비스로 여전히 손님을 끌수도 있다Festival 에 얼마나 참여하는지가 jazz 의 인기를 반영하지 않는다 뮤지션이 많이 사는거랑 jazz 의 인기가 상관없다라디오 프로그램이 다른 요인에 의해서 인기일수도 있다 (진행자때문)전국 통계 적용 불가화목 실전반_Ms Noh6In this application the author suggests that a jazz club in Monroe will make a number of profits To support this suggestion the author exemplifies the local condition popularity of jazz in Monroe and nationwide study However careful scrutiny of each of the facts reveals that it provides little credible support for the authorrsquos recommendation Good clear intro

First the author assumes that jazz is popular in Monroe because of several facts the jazz festival last year had high participation some famous jazz musicians live in Monroe and the high-rated radio program is lsquoJazz Nightlyrsquo However this assumption has many drawbacks that must be seriously considered(Good topic sentences) If many attendants in the last-yearrsquos festival came from other cities and not Monroe it is hard to conclude that Monroersquos people like jazz Therefore the author must examine how many Monroe residents actually attended the festival On top of that there is little relationship between habitation of famous jazz musician and the popularity of jazz in Monroe Although several well-known musicians live there if they do not take part in any jazz performance of Monroe this might have no effect to the interest of Monroersquos residents

GRE AWA John박 박정어학원

about jazz Finally in the case of radio program this is also not suitable reason why jazz is popular in Monroe It might be possible that people cannot help choosing lsquoJazz Nightlyrsquo because there are few radio programs at Night The fact that the radio program is the highest rating program is not a germane evidence The approximate number of listeners would be the more crucial evidence Therefore the author needs to seriously deliberate the correlation between jazzrsquos popularity in Monroe and his examples (Good logical flow and clarity)

Second the author uses as evidence the nationwide study that jazz fans spend much money on jazz entertainment to substantiate why starting a jazz club in Monroe will be profitable In other words the author assumes that the characteristics of a nationwide study can be applied to Monroe The national study would lend support to the applicantrsquos claim only if residents in Monroe typify national jazz fans However the author does not provide credible evidence that this is the case Moreover the populations of jazz fans nationwide may be insubstantial Thus the author should not infer hastily that Monroersquos residents will spend much money on enjoying jazz from the nationwide study

Lastly even if jazz is popular in Monroe C Note may not be successful It is entirely possible that residents might still prefer other clubs where they have always went In addition there is another possibility that the nearest jazz club will attract many of Monroersquos people because it serves fine performances and is equipped with favorite facilities Without considering these other possibilities the author cannot make his argument convincing In sum the author presents many reasons that are insufficient in supporting his or her claim In order for the authorrsquos claims to be convincing he needs to advance more persuasive evidence such as the total number of Monroe residents who attended the jazz festival the effects on the popularity of jazz by the musicians living in Monroe and the actual number of residents who would typify themselves to be jazz fans through a local survey Without substantial evidence that C Note will be successful in Monroe the businessmen may be overinvesting in what might lead to a business failureExcellent clarity Score 50

The following appeared in a newsletter offering advice to investors

Over 80 percent of the respondents to a recent survey indicated a desire to reduce their intake of foods containing fats and cholesterol and today low-fat products abound in many food stores Since many of the food products currently marketed by Old Dairy Industries are high in fat and cholesterol the companys sales are likely to diminish greatly and company profits will no doubt decrease We therefore advise Old Dairy stockholders to sell their shares and other investors not to purchase stock in this company

Write a response in which you discuss what questions would need to be answered in order to decide whether the advice and the argument on which it is based are reasonable Be sure to explain how the answers to these questions would help to evaluate the advice

Survey 80

GRE AWA John박 박정어학원

Old Dairy could change their products and manufacture low fat dairy foodsLess competing companies Old Dairy could eventually be the only company that produces hellipImprecise numbers and measurementsCustomers may still buy high fat dairy products

The author of the newsletter is offering potentially dangerous advice by recommending Old Dairy stockholders to withdraw investment and stop purchase What is more the authorrsquos prediction debases the reputation and business of Old Dairy and if false could devoid the investment opportunity of the newsletter readers Therefore investors should examine whether the authorrsquos evidences are substantial

To begin with the author states that 80 percent of the respondents in a survey indicated a desire to reduce their intake of foods He therefore argues that Old Dairyrsquos high fat and cholesterol products would decrease in sales However the author makes a crucial error in this argument First the author provides no evidence that the surveyrsquos results are statistically reliable Were they representative of all the customers Were they chosen for the survey randomly Furthermore the desire to reduce fat and cholesterol intake is a pervasive trend in todayrsquos opulent society however the author erroneously identifies this as a new phenomenon which will affect consumer trends Second having a desire to reduce fat and cholesterol intake does not necessarily indicate that people who have this desire will actually reduce consuming these types of products It is entirely possible that they may continue buying Old Dairy products for its quality and taste Accordingly the author cannot draw any firm conclusion that people will not buy Old Dairy products Therefore if any of these cases are true the author may be offering investors a detrimental investment advice

최근 조사에 대한 응답자중 80 이상이 자신이 먹는 음식에서 지방과 콜레스테롤의 함유량을 줄이고 싶다고 한다 아울러 요즘은 많은 식료품 가계에서 저지방 제품들을 많이 취급하고 있다 현재 Old Dairy Industries가 판매하고 있는 많은 음식제품들은 지방과 콜레스테롤이 높기 때문에 이 회사의 매출이 격감할 것으로 보이며 당연히 매출이익도 줄어들것이다 따라서 이 회사의 주주들은 주식을 매각하고 다른 주식 투자가들도 이 회사의 주식을 매입하지 않는 것이 좋다

결론 Old Dairy stockholders to sell their shares and other investors not to purchase stock in this company

반박 모든 상품이 다 고 지방 고 칼로리는 아니다(비록 많을지라도) 일부의 식품의 경우 기호에 맞어서 히트해서 전체적인 수입이 증가할 수도 있다국내시장만 생각할 수 없다( 외국시장에서 호황을 누릴수 있다 )입맛이라는게 즉각 바뀌는게 아니다

The following appeared in a letter to the editor of the Balmer Island Gazette

On Balmer Island where mopeds serve as a popular form of transportation the population increases to 100000 during the summer months To reduce the number of accidents involving mopeds and pedestrians the town council of Balmer Island should limit the number of mopeds rented by the islands moped rental companies from 50 per day to 25 per day during the summer season By limiting the number of rentals the town council will attain the 50 percent annual reduction in moped accidents that was achieved last year on the neighboring island of Seaville when Seavilles town council enforced similar limits on moped rentals

1 Write a response in which you discuss what questions would need to be answered in order to decide whether the recommendation is likely to have the predicted result Be sure to explain how the answers to these questions would help to evaluate the recommendation

2 Write a response in which you discuss what questions would need to be answered in order to decide whether the prediction and the argument on which it is based are reasonable Be sure to explain how the answers to these questions would help to evaluate the prediction

3 Write a response in which you examine the stated andor unstated assumptions of the argument Be sure to explain how the argument depends on these assumptions and what the implications are for the argument if the assumptions prove unwarranted

Whatrsquos the actual population of Balmer Island 100000mdashis this a significant increase What kind of accidents Skin abrasions or serious injury And compared to Seaville how serious are the accidents and the actual number of accidents Did Seaville enforce other restrictions like safety signsHow different are the conditions of Balmer

GRE AWA John박 박정어학원

and Seaville regarding population road (safety) conditions topography other town-government regulation How much will the economy of Balmer be affected do to this restriction Could it cause an economic recession due to the fact that these rental companiesrsquo chance to make money is only during the summer thereby weakening the economic infrastructure Are there any other ways that could better alleviate the accident rate

Statistics 50-impreciseAnalogy Balmer compared with TorseauFalse Cause Accidents might have occurred because of reasons other than mopeds False Cause population increase may not be part of the cause of the accidentsOther explanations for the accident pedestrians few road safety regulations narrow roadsThere could be other better solutionshellip

Balmer Island의 인구가 여름철에는 십만명으로 늘어난다 2륜차와 보행자간 사고를 줄이기 위해 시의회는 6개의 자전거를 포함한 2륜차 대여업체에게 이 기간동안에는 대여숫자를 일일 50에서 30으로 제한하도록 할 것이다 대여숫자를 줄임으로써 시 의회는 지난해 이웃한 Torseau섬에서 이와 동일한 규제를 시행해서 50나 줄인 결과를 보고 마찬가지로 50를 줄일수 있다고 확신하고 있다

결론 The town council of Balmer Island should linit the number

반박 보행자의 부실에 의해서 사고가 많이 일어날수도 있다렌탈수의 줄임만이 대책은 아니다(대부분의 사람들이 렌탈 보다는 소유하고 있을 수도 있다)옆섬과는 상황이 다를수도 있다(그 섬에서는 사고의 원인이 많은 자전거 수로 인한것일수있다) 하지만 이 섬은 좁은 도로가 원인일 수도 있고 도로 안전 장치의미비가 원일일수 있다

In this letter the author recommends that Balmer Island should limit the number moped rentals from 50 to 30 per day To support this recommendation the author points out several reasons However careful scrutiny of each of the facts reveals that it is filled with unanswered questions that could significantly weaken the authorrsquos recommendation with loops and holes which are answered

The recommendation depends on the assumption that no alternative means of reducing the number of accidents are available However the author fails to offer any evidence to substantiate this crucial assumption It is highly possible that means other than this would better solve the problem Perhaps they could widen the roads or put-up more safety signs Or perhaps the accidents were due to the lack of skills in which case proper safety training would significantly alleviate the problem Without considering and ruling out these and other alternative means of reducing accidetns the author cannot confidently conclude that merely emulating Torseau would suffice Moreover the author is advising a recommendation which could potentially harm the economy of Balmer Island sincehellip Moreover the Balmer Island should alternative means to reduce accidents because limiting moped rentals during the summer could harm the economy of Balmerhellip

First of all the author believes that increase in population and the number of moped rentals are responsible for the accidents It is entirely possible that other factors are responsible for the accidents Perhaps Balmer Islandrsquos lack of safety signs was a major factor Or maybe the roads are narrow and dangerous on the Island therefore the town council could enforce stricter traffic regulations to alleviate the problem Accordingly if either of these scenarios is true the author cannot draw any firm conclusion that increase in the number of population and moped rentals are the cause of the accidents

The author of this editorial recommends that to reduce accidents involving mopeds andpedestrians Balmer Islands city council should restrict moped rentals to 30 per day down from50 at each of the islands six rental outlets To support this recommendation the author citesthe fact that last year when nearby Torseau Islands town council enforced similar measuresTorseaus rate of moped accidents fell by 50 For several reasons this evidence providesscant support for the authors recommendationTo begin with the author assumes that all other conditions in Balmer that might affect therate of moped-pedestrian accidents will remain unchanged after the restrictions are enactedHowever with a restricted supply of rental mopeds people in Balmer might purchase mopedsinstead Also the number of pedestrians might increase in the future with more pedestriansespecially tourists the risk of moped-pedestrian accidents would probably increase For thatmatter the number of rental outlets might increase to make up for the artificial supplyrestriction per outlet--a likely scenario assuming moped rental demand does not declineWithout considering and ruling out these and other possible changes that might contribute to ahigh incidence of moped-pedestrian accidents the author cannot convince me that theproposed restrictions will necessarily have the desired effect

GRE AWA John박 박정어학원

Next the author fails to consider other possible explanations for the 50 decline inTorseaus moped accident rate last year Perhaps last year Torseau experienced unusually fairweather during which moped accidents are less likely Perhaps fewer tourists visited Tot seanlast year than during most years thereby diminishing the demand for rental mopeds to belowthe allowed limits Perhaps last year some of Torseaus moped rental outlets purchased newmopeds that are safer to drive Or perhaps the restrictions were already in effect but were notenforced until last year In any event a decline in Torseaus moped accident rate during onlyone year is scarcely sufficient to draw any reliable conclusions about what might have causedthe decline or about what the accident rate will be in years aheadAdditionally in asserting that the same phenomenon that caused a 50 decline in mopedaccidents in Torseau would cause a similar decline in Balmer the author relies on what mightamount to an unfair analogy between Balmer and Torseau Perhaps Balmers ability to enforcemoped-rental restrictions does not meet Torseaus ability if not then the mere enactment ofsimilar restrictions in Balmer is no guarantee of a similar result Or perhaps the demand formopeds in Torseau is always greater than in Balmer Specifically if fewer than all availablemopeds are currently rented per day from the average Balmer outlet while in Torseau everyavailable moped is rented each day then the proposed restriction is likely to have less impacton the accident rate in Balmer than in TorseauFinally the author provides no evidence that the same restrictions that served to reduce theincidence of all moped accidents by 50 would also serve to reduce the incidence ofaccidents involving mopeds and pedestrians by 50 Lacking such evidence it is entirelypossible that the number of moped accidents not involving pedestrians decreased by a greaterpercentage while the number of moped-pedestrian accidents decreased by a smallerpercentage or even increased Since the author has not accounted for these possibilities theeditorials recommendation cannot be taken seriouslyIn conclusion the recommendation is not well supported To convince me that the proposedrestriction would achieve the desired outcome the author would have to assure me that nochanges serving to increase Balmers moped-pedestrian accident rate will occur in theforeseeable future The author must also provide dear evidence that last years decline inmoped accidents in Torseau was attributable primarily to its moped rental restrictions ratherthan to one or more other factors In order to better evaluate the recommendation I wouldneed more information comparing the supply of and demand for moped rentals on the twoislands I would also need to know the rate of mopedpedestrian accidents in Torseau both priorto and after the restrictions were enforced in TorseauThe following appeared in a magazine article about planning for retirement

Clearview should be a top choice for anyone seeking a place to retire because it has spectacular natural beauty and a consistent climate Another advantage is that housing costs in Clearview have fallen significantly during the past year and taxes remain lower than those in neighboring towns Moreover Clearviews mayor promises many new programs to improve schools streets and public services And best of all retirees in Clearview can also expect excellent health care as they grow older since the number of physicians in the area is far greater than the national average

Write a response in which you discuss what specific evidence is needed to evaluate the argument and explain how the evidence would weaken or strengthen the argument

-Natural beauty and consistent climate may not be the most wanted qualities-Housing costs could have lowered on a national level wealthy retirees may not care about costs-Taxes may be high compared to the nationrsquos average tax rate-What about other qualities of Clearview Crime rate what qualities would retirees want -If schools streets and public services need improvement then this is proof that the current condition of Clearview is low Or due to budgetary reasons the mayor may not follow-up on his promise because of lowered tax rate -Schools and people who are retired no relationship-Physicians What kind of physicians Number is irrelevant Are these physicians capable of addressing the illnesses of old people

This author argues that anyone seeking a place to retire should choose Clearview To supportthis argument the article cites Clearviews consistent climate and natural beauty its fallinghousing costs its low property taxes compared to nearby towns and the mayors promise toimprove schools streets and services The article also claims that retirees can expectexcellent health care because the number of physicians in Clearview greatly exceeds thenational average This argument is flawed in several critical respectsTo begin with although consistent climate and natural beauty might be attractive to manyretirees these features are probably not important to all retirees For many retirees it isprobably more important to live near relatives or even to enjoy changing seasons Thus I

GRE AWA John박 박정어학원

cannot accept the authors sweeping recommendation for all retirees on this basisAlso Clearviews declining housing costs do not necessarily make Clearview the best placeto retire for two reasons First despite the decline Clearviews housing costs might be highcompared to housing costs in other cities Secondly for wealthier retirees housing costs arenot likely to be a factor in choosing a place to retire Thus the mere fact that housing costshave been in decline lends scant support to the recommendationThe articles reliance on Clearviews property-tax rates is also problematic in two respectsFirst retirees obviously have innumerable choices about where to retire besides Clear viewand nearby towns Secondly for retirees who are well-off financially property taxes are notlikely to be an important concern in choosing a place to retire Thus it is unfair to infer fromClearviews property-tax rates that retirees would prefer ClearviewYet another problem with the argument involves the mayors promises In light of Clearviewslow property-tax rates whether the mayor can follow through on those promises is highlyquestionable Absent any explanation of how the city can spend more money in the areas citedwithout raising property taxes I simply cannot accept the editorials recommendation on thebasis of those promises Besides even if the city makes the improvements promised thoseimprovements--particular the ones to schools--would not necessarily be important to retireesFinally although the number of physicians in Clearview is relatively high the per capitanumber might be relatively low Moreover it would be fairer to compare this per capita numberwith the per capita number for other attractive retirement towns--rather than the nationalaverage After all retirees are likely to place a relatively heavy burden on health-careresources Besides the article provides no assurances that the number of physicians inClearview will remain high in the foreseeable futureIn conclusion the recommendation is poorly supported To strengthen it the author mustconvince me--perhaps by way of a reliable survey--that the key features that the vast majorityof retirees look for in choosing a place to live are consistent climate natural beauty and lowhousing costs The author must also provide better evidence that Clear views property taxesare lower than the those of cities in other areas The author must also explain how the city canmake its promised improvements without raising property taxes Finally to better assess theargument I would need to now how the per capita number of physicians in Clearview wouldcompare to the national average in the futureThe following appeared as a letter to the editor from a Central Plaza store owner

Over the past two years the number of shoppers in Central Plaza has been steadily decreasing while the popularity of skateboarding has increased dramatically Many Central Plaza store owners believe that the decrease in their business is due to the number of skateboard users in the plaza There has also been a dramatic increase in the amount of litter and vandalism throughout the plaza Thus we recommend that the city prohibit skateboarding in Central Plaza If skateboarding is prohibited here we predict that business in Central Plaza will return to its previously high levels

Write a response in which you discuss what questions would need to be answered in order to decide whether the recommendation is likely to have the predicted result Be sure to explain how the answers to these questions would help to evaluate the recommendation

Why two years ago What happened two years ago which started this declineIs the dramatic increase in the ldquopopularityrdquo of skateboarding the cause of the steady decline of shoppers Are there any malls nearby Were there any changes nearby which could affect the decline in customersmdasha big mall perhaps Could the decline be due to the shop ownersHow many skateboarders use the plazaWhere do they skateboardDo they shop and are they customersAre the increase in litter and vandalism due to skateboarders Could this be alleviated by installing CCTVs and hiring security

This editorial concludes that the city should ban skateboarding from its downtown CentralPlaza in order to attract visitors to that area to return the area to its former glory and to makeit a place where people can congregate for fun and relaxation To justify this conclusion theeditorial points out that skateboarders are nearly the only people one sees anymore at CentralPlaza and that the Plaza is littered and its property defaced The editorial also points out thatthe majority of downtown merchants support the skate boarding ban This argument is flawedin several critical respectsFirst the editorials author falsely assumes that a ban on skateboarding is both necessaryand sufficient to achieve the three stated objectives Perhaps the city can achieve thoseobjectives by other means as well--for example by creating a new mall that incorporates anattractive new skateboard park Even if banning skateboarders altogether is necessary to meetthe citys goals the author has not shown that this action by itself would suffice Assuming thatthe Plazas reputation is now tarnished restoring that reputation and in turn enticing peopleback to the Plaza might require additional measures--such as removing litter and graffiti

GRE AWA John박 박정어학원

promoting the Plaza to the public or enticing popular restaurant or retail chains to the PlazaSecondly the editorial assumes too hastily that the Plazas decline is attributable to theskateboarders--rather than to some other phenomenon Perhaps the Plazas primary appeal inits glory days had to do with particular shops or eateries which were eventually replaced byless appealing ones Or perhaps the crime rate in surrounding areas has risen dramatically forreasons unrelated to the skateboarders presence at the Plaza Without ruling out these andother alternative explanations for the Plazas decline the editorials author cannot convince methat a skateboard ban would reverse that declineThirdly the editorials author might be confusing cause with effect--by assuming that theskateboarders caused the abandonment of the Plaza rather than vice versa It is entirelypossible that skateboarders did not frequent the Plaza until it was largely abandoned--andbecause it had been abandoned In fact this scenario makes good sense since skateboardingis most enjoyable where there are few pedestrians or motorists to get in the wayFourth it is unreasonable to infer from the mere fact that most merchants favor the ban thatthe ban would be effective in achieving the citys objectives Admittedly perhaps thesemerchants would be more likely to help dean up the Plaza area and promote their businesseswere the city to act in accordance with their preference Yet lacking any supporting evidencethe author cannot convince me of this Thus the survey amounts to scant evidence at best thatthe proposed ban would carry the intended resultFinally the author recommends a course of action that might actually defeat the citysobjective of providing a fun and relaxing place for people to congregate In my experienceskateboarding contributes to an atmosphere of fun and relaxation for adults and children alikemore so than many other types of ambiance Without considering that continuing to allowskateboarding--or even encouraging this activity--might achieve the citys goal more effectivelythan banning the activity the author cannot convincingly conclude that the ban would be in thecitys best interestsIn sum the argument is a specious one To strengthen it the editorials author must providedear evidence that skateboarding and not some other factor is responsible for the conditionsmarking the Plazas decline The author must also convince me that no alternative means ofrestoring the Plaza are available to the city and that the proposed ban by itself would suffice toattract tourists and restore the Plaza to its former glory Finally to better assess the argument itwould be useful to know the circumstances under which the downtown merchants would bewilling to help the city achieve its objectives

6그룹 약한 비유 빈출

The following recommendation appeared in a memo from the mayor of the town of Hopewell

Two years ago the nearby town of Ocean View built a new municipal golf course and resort hotel During the past two years tourism in Ocean View has increased new businesses have opened there and Ocean Views tax revenues have risen by 30 percent Therefore the best way to improve Hopewells economymdashand generate additional tax revenuesmdashis to build a golf course and resort hotel similar to those in Ocean View

Write a response in which you examine the stated andor unstated assumptions of the argument Be sure to explain how the argument depends on these assumptions and what the implications are for the argument if the assumptions prove unwarranted

GRE AWA John박 박정어학원

Assumptions The author assumes that OVrsquos municipal golf course and resort hotel caused tourism new businesses and increased tax revenues There may be other reasons advertising promo He assumes that this will continueAssumes that Ocean View and Hopewell are similar in many waysmdashthe name suggests otherwise OV may have always been a tourist attractions for its beaches We need to know the topography

2년전 Ocean View 시는 시정 소유 골프 및 휴양지 호텔을 신축했다 그리고 지난 2년동안 이 시의 관광객이 증가했으며 새로운 사업들이 생겨났다 그에따라 시의 세수도 30나 증가했다 Hopewell의 경제를 향상시키고 아울러 세수를 늘릴 수 있는 가장 좋은 방법은 Ocean View에 세워진 것과 같은 골프 시설과 휴양지 호텔을 신축하는 것이다

1 다른 요인으로 관광 산업이 발전했을 수도 있다 문화 유적이 발견이 되었거나 도로의 정비등으로 여행자가 늘었을 수도 있다

2 관광 산업의증가가 늘어난 세수의 원인이 아니라 새로 유입된 인구의 증가나 다른 공장에서 발생한 것일 수 있다

3 2년동안 한참 골프가 붐을 이루었을 수 있다 경제상황이 나빠지거나 다른 레포츠가 인근 지역에 생겨난다면 골프하는 사람이 줄어들 수 있다

In this memo HopeweUs mayor recommends that in order to stimulate the towns economyand boost tax revenues HopeweU should build a new golf course and resort hotel just as thetown of Ocean View did two years ago To support this recommendation the mayor points outthat in Ocean View during the last two years tourism has increased new businesses haveopened and tax revenues have increased by 30 I find the mayors argument unconvincingin several important respectsFirst of all it is possible that the mayor has confused cause with effect respecting the recentdevelopments in Ocean View Perhaps Ocean Views construction of a new golf course andhotel was a response to previous increases in tourism and business development increasesthat have simply continued during the most recent two years Since the mayor has failed toaccount for this possibility the claim that Hopewell would boost its economy by alsoconstructing a golf course and hotel is completely unwarrantedSecondly the mayor fails to account for other possible causes of the trends in Ocean Viewduring the last two years The increase in tourism might have been due to improving economicconditions nationwide or to unusually pleasant weather in the region The new businessesthat have opened in Ocean View might have opened there irrespective of the new golf courseand hotel And the 30 increase in tax revenues might have been the result of an increase intax rates or the addition of a new type of municipal taxWithout ruling out these and other alternative explanations for the three recent trends inOcean View the mayor cannot reasonably infer based on those trends that Hopewellseconomy would benefit by following Ocean Views exampleThirdly even if the recent trends in Ocean View are attributable to the construction of the newgolf course and hotel there the mayor assumes too hastily that the golf course and hotel willcontinue to benefit that towns overall economy The mayor has not accounted for thepossibility that increased tourism will begin to drive residents away during tourist season orthat new business development will result in the towns losing its appeal as a place to visit or tolive Unless the mayor can convince me that these scenarios are unlikely I cannot accept themayors recommendation that Hopewell follow Ocean Views exampleFinally the mayors argument rests on the unsubstantiated assumption that Hopewell andOcean View are sufficiently alike in ways that might affect the economic impact of a new golfcourse and hotel Hopewell might lack the sort of natural environment that would attract moretourists and new businesses to the town--regardless of its new golf course and hotel For thatmatter perhaps Hopewell already contains several resort hotels and golf courses that are notutilized to their capacity If so building yet another golf course and hotel might amount to amisallocation of the towns resources--and actually harm the towns overall economyIn sum the mayors recommendation is not well supported To bolster it the mayor mustprovide better evidence that Ocean Views new golf course and hotel and not some otherphenomenon--has been responsible for boosting Ocean Views economy during the last twoyears To better assess the recommendation I would need to know why Ocean View decidedto construct its new golf course and hotel in the first place--specifically what events prior toconstruction might have prompted that decision I would also need to thoroughly compare

GRE AWA John박 박정어학원

HopeweU with Ocean View--especially in terms of their appeal to tourists and businesses--todetermine whether the same course of action that appears to have boosted Ocean Viewseconomy would also boost Hopewells economy

The following is part of a memorandum from the president of Humana University

Last year the number of students who enrolled in online degree programs offered by nearby Omni University increased by 50 percent During the same year Omni showed a significant decrease from prior years in expenditures for dormitory and classroom space most likely because instruction in the online programs takes place via the Internet In contrast over the past three years enrollment at Humana University has failed to grow and the cost of maintaining buildings has increased along with our budget deficit To address these problems Humana University will begin immediately to create and actively promote online degree programs like those at Omni We predict that instituting these online degree programs will help Humana both increase its total enrollment and solve its budget problems

Write a response in which you discuss what questions would need to be answered in order to decide whether the prediction and the argument on which it is based are reasonable Be sure to explain how the answers to these questions would help to evaluate the prediction

Is Omni University successful due to the online degree program 50 Is the decrease in expenditures for dormitory and classroom space due to the decrease in of on-campus students Which classes were successful Does HU have those classes

Even if the long-distance degree programs at Omni University benefited the school the presidentrsquos recommendation that Human College should emulate Omni University is too hasty First OUrsquos name implies that the school would have more majors than Humanahellip the president should examine which degrees were in the long-distance programhellip

지난해에는 Omni 대학에서 개강했던 원거리 학생 학점 취득 프로그램을 등록했던 학생들의 숫자가 50나 증가했다 같은해 기간동안 Omni 대학에서는 그 전년도부터 기숙사와 학급의 공간 확충을 위한 예산을 대폭 줄였는데 이는 이 원거리 학점 취득 프로그램이 양방향 비디오 컴퓨터 접속을 통해서만 가능한 수업지도 방식이기때문인 것으로 보인다 반면 지난 3개년 동안 Humana 대학에서의 수강률은 감소한데다가 건물

GRE AWA John박 박정어학원

유지비도 올랐다 따라서 Humana대학의 수강을 늘리고 예산손실을 회복하기 위해서는 Omni 대학에서 취한 조치와 같은 능동적인 프로그램을 추진해야 한다

결론 we should initiate and actively promote long-distance degree programs like those at Omni 반박 원거리 학생 취득 프로그램 숫자가 증가한거하고 예산이 줄어드는 것 사이에 연관이 약하다 (causal 학생의 증가로 관리비용 증가할수 있음 원거리 수업가능 장비도입에의한 비용발생)bad analogy(omni university 하고 같은 조건이 아니다 )-gt omni college 가 강좌내용이 좋아서 학생의 등록이 많을수 있다 Humana 대학에서 만들었다 하더라도 인기 없을수 있음다른 요인에 의해서 Humana 대학의 수강 인원이 증가할수 있음(비록 과거엔 인기가 없었을지라도)

The following appeared as part of a business plan developed by the manager of the Rialto Movie Theater

Despite its downtown location the Rialto Movie Theater a local institution for five decades must make big changes or close its doors forever It should follow the example of the new Apex Theater in the mall outside of town When the Apex opened last year it featured a video arcade plush carpeting and seats and a state-of-the-art sound system Furthermore in a recent survey over 85 percent of respondents reported that the high price of newly released movies prevents them from going to the movies more than five times per year Thus if the Rialto intends to hold on to its share of a decreasing pool of moviegoers it must offer the same features as Apex

Write a response in which you discuss what questions would need to be answered in order to decide whether the recommendation is likely to have the predicted result Be sure to explain how the answers to these questions would help to evaluate the recommendation

Before following through this business plan the manager should investigate the cause of Rialtorsquos unsuccessful business

The author provides no evidence that the surveyrsquos results are statistically reliable The surveyrsquos sample of 85 percent must be sufficient in size and representative of overall population of the city where Rialto and Apex is serving Lacking evidence of a sufficiently representative sample the author cannot justifiably rely on the survey to draw any conclusion whatsoever The author does not indicate that Apex is indeed currently successful However even if Apex is enjoying success the argument relies on what might be a false analogy between Rialto and Apex In order for Apex to serve as a model that Rialto should emulate the author must assume that all relevant circumstances are essentially the same However this assumption is unwarranted For example the argument overlooks the face that Apex is located in a strategic placemdashbeside a mall where customers can not only watch a movie but also enjoy shopping Therefore simply changing the facility to that of Apex may not lead to success

The author does not mention whether Apex is successful or not Nevertheless even if Apex is currently successful the argument relies on what might be a false analogy between Rialto and Apex In order for Apex to serve as a model that Rialto should emulate the author must assume that all relevant circumstances are essentially the same However this assumption is unwarranted For example the argument overlooks the fact that these two institutions are located in different locations Rialto in downtown and Apex in a mall outside of town Although Apex opened with state-of-the-art facilities the decisive factor in its success could be due to its strategic location of being in a mall People could enjoy both shopping and movies at one location thus they may prefer Apex over Rialto Furthermore the place where people enjoy leisure activities has shifted in the past decades for most cities from downtown to the suburbs Therefore Rialto may not be successful even if it emulates Apexrsquos facilities A better business plan may be relocating Apex to the thriving section of the downtown

Rialto 극장은 지난 50여년간 지역 회관으로써 시내에 위치해 있으면서도 이제 변화를 꾀하지 않으면 문을 닫을

GRE AWA John박 박정어학원

판이다 이 극장은 시외 쇼핑타운에 새로 들어선 Apex 극장의 사례를 본받아야 했다 Apex가 지난해 개업했을 당시 이 극장은 비디오 아케이드 플러쉬 카펫트 바닥과 좌석 그리고 최신 음향시설을 갖추었다 더군다나 최근 조사에서는 응답자의 85 이상이 새로 출시된 영화 입장료가 비싼 탓으로 지난해보다 5배이상의 관람객이 줄어들었다고 나타났다 따라서 Rialto 극장이 줄어들고 있는 관람객을 뺐기지 않고 유지하려면 Apex와 같은 시설들을 갖추어야 할 것이다주장 리알토 극장이 줄어들고 있는 관람객을 뺐기지 않고 유지하려면 Apex와 같은 시설들을 갖추어야 할 것이다

1 조사에서 응답자가 전체를 대표할 수 없다 2 apex 극장이 좋은 시설을 갖추고 있지만 그로 인해 수익이 많이 발생했다는 말이 없으므로 시설투자를

하고도 좋은 결과를 얻을 수 있을지 그 근거가 미흡하다3 좋은 영화가 출시된다면 입장료가 비싸도 영화관에서 꼭 보려고 할 수 있다 4 rialto 가 시설이 아닌 다른 요인에 의해 장사가 안될수도 있다( 우범 지역이라든지)

The following is a recommendation from the business manager of Monarch Books

Since its opening in Collegeville twenty years ago Monarch Books has developed a large customer base due to its reader-friendly atmosphere and wide selection of books on all subjects Last month Book and Bean a combination bookstore and coffee shop announced its intention to open a Collegeville store Monarch Books should open its own in-store cafeacute in the space currently devoted to childrens books Given recent national census data indicating a significant decline in the percentage of the population under age ten sales of childrens books are likely to decline By replacing its childrens books section with a cafeacute Monarch Books can increase profits and ward off competition from Book and Bean

Write a response in which you examine the stated andor unstated assumptions of the argument Be sure to explain how the argument depends on these assumptions and what the implications are for the argument if the assumptions prove unwarranted

The following is a recommendation from the business manager of Monarch Books

Since its opening in Collegeville twenty years ago Monarch Books has developed a large customer base due to its reader-friendly atmosphere and wide selection of books on all subjects Last month Book and Bean a combination bookstore and coffee shop announced its intention to open a Collegeville store Monarch Books should open its own in-store cafeacute in the space currently devoted to childrens books Given recent national census data indicating a significant decline in the percentage of the population under age ten sales of childrens books are likely to decline By replacing its childrens books section with a cafeacute Monarch Books can increase profits and ward off competition from Book and Bean

1 Write a response in which you discuss what questions would need to be answered in order to decide whether the recommendation is likely to have the predicted result Be sure to explain how the answers to these questions would help to evaluate the recommendation

2 Write a response in which you discuss what specific evidence is needed to evaluate the argument and explain how the evidence would weaken or strengthen the argument

No evidence regarding Monarch Bookrsquos successEven if Regal Bookrsquos is successful this may not be attributable to the cafeacute False analogy Emulating may not lead to success Other factors may be involvedInsufficient condition The national census is not enough evidence that childrenrsquos book sales will decline Can

GRE AWA John박 박정어학원

the national census represent the local child populationDid opening a cafeacute boost sales for Regal Books Even assuming Regal is successful by opening a cafeacute this may not be suitable for Monarch which plans to close the childrenrsquos book section to establish a cafe Imprecise language ldquorelatively little spacerdquo how smallThe managerrsquos recommendation contradicts what he says Since Monarch is popular for its wide selection of books closing a selection which targets a major group of readers may hurt Monarchrsquos salesIs this the best way to compete

When Stanley Park first opened it was the largest most heavily used public park in town It is still the largest park but it is no longer heavily used Video cameras mounted in the parks parking lots last month revealed the parks drop in popularity the recordings showed an average of only 50 cars per day In contrast tiny Carlton Park in the heart of the business district is visited by more than 150 people on a typical weekday An obvious difference is that Carlton Park unlike Stanley Park provides ample seating Thus if Stanley Park is ever to be as popular with our citizens as Carlton Park the town will obviously need to provide more benches thereby converting some of the unused open areas into spaces suitable for socializing

Write a response in which you examine the stated andor unstated assumptions of the argument Be sure to explain how the argument depends on these assumptions and what the implications are for the argument if the assumptions prove unwarranted

Stanley 파크가 처음 개장했을 당시 가장 크고 가장 많이 이용되는 공원이었다 아직도 공원중에서는 가장 크지만 이용률은 상당히 떨어졌다 지난달 공원 주차장에 설치해놓은 비디오 카메라를 통해 보면 drop(주차장으로 여겨짐) 이용률이 가장 높았다 수치상으로는 하루 평균 50대의 차량만이 이용하였다 반면 직장 중심거리에 위치한 작은 규모의 Carlton 파크는 주당 무려 150여명 이상이 이용하고 있다 Stanley 파크와는 달리 Carlton 파크에는 의자가 있다는 것이 가장 뚜렷한 차이점이다 따라서 Stanley 파크가 Carlton 파크처럼 시민들이 자주 이용하는 공원이 되기 위해서는 벤치를 설치할 필요가 있으며 이렇게 사용되지 않는 일부 공간을 활용해서 사교를 위한 공간으로 바꾸어야 한다 ===gtdrop 에 대한 첨부사항 (영영사전내용입니다)---- a place or central depository to which something (as mail money or stolen property) is brought for distribution or transmission also the act of depositing something at such a place dropgt

주장 if Stanley Park is ever to be as popular with our citizens as is Carlton Park the town will obviously need to provide more benches thereby converting some of the unused open areas into spaces suitable for socializing1 조사가 언제 이루어진 것인가 조사가 언제 실시되었느냐에 따라 결과가 다를 수있다 현재는 다시 스탠리 파크가 늘어났었을 수 있다 2 벤치를 많이 설치했다고 해서 많은 관광객이 오지 않을수 있다(사람들이 벤치나 사교 공간을 원한다는 어떠한 자료도 없다)3스탠리 파크 주변에 교통 상황이 악화가 되었거나 칼튼 파크에서 문화행사등을 많이 가져서 이용객이 줄어든것일 수도 있다 4 칼튼 파크가 중심지에 있어서 접근성이 좋을수 있다5 조사가 같은 시간을 기준으로 한게 아니다(하나는 주중이고 하나는 주말이다)6사람의 수와 차의 대수를 같은것으로 비교할수 없다 (차안에 몇 명이 타고 있는지 모르고 대중교통을 이용해서 왔을수도 있다)

Page 20: GRE writing argument brain storm

GRE AWA John박 박정어학원

In this memo the dean of Omega University(OU) recommends OU to terminate professor evaluation to secure better jobs for the students To support this recommendation the dean offers several reasons However this argument contains several logical flaws which render it unconvincing

A threshold problem with the argument involves the voluntary nature of the evaluationprocedure The dean provides no evidence about the number or percentage of Omegastudents who participate in the procedure Lacking such evidence it is entirely possible thatthose numbers are insignificant in which case terminating the procedure is unlikely to haveany effect on the grade average of Omega students or their success in getting jobs aftergraduationThe argument also assumes unfairly that the grade-average increase is the result of theevaluation procedure--rather than some other phenomenon The dean ignores a host of otherpossible explanations for the increase--such as a trend at Omega toward higher admissionstandards or higher quality instruction or facilities Without ruling out all other possibleexplanations for the grade-average increase the dean cannot convince me that by terminatingthe evaluation procedure Omega would curb its perceived grade inflation let alone help itsgraduates get jobsEven if the evaluation procedure has resulted in grade inflation at Omega the deans claimthat grade inflation explains why Omega graduates are less successful than Alpha graduatesin getting jobs is unjustified The dean overlooks a myriad of other possible reasons forOmegas comparatively poor job-placement record Perhaps Omegas career services areinadequate or perhaps Omegas curriculum does not prepare students for the job market aseffectively as Alphas In short without accounting for other factors that might contribute toOmega graduates comparative lack of success in getting jobs the dean cannot justify theclaim that if Omega curbs its grade inflation employers will be more likely to hire OmegagraduatesFinally even if the dean can substantiate all of the foregoing assumptions the deansassertion that Omega must terminate its evaluation procedure to enable its graduates to findbetter jobs is still unwarranted in two respects First the dean ignores other possible ways bywhich Omega can increase its job-placement record--for example by improving its publicrelations or career-counseling services Second the dean unfairly equates more jobs withbetter jobs In other words even if more Omega graduates are able to find jobs as a result ofthe deans recommended course of action the kinds of jobs Omega graduates find would notnecessarily be better onesIn sum the deans argument is unpersuasive as it stands To strengthen it the dean mustprovide better evidence that the increase in grade average is attributable to Omegasprofessor-evaluation procedure and that the end result is a perception on the part ofemployers that Omega graduates are less qualified for jobs than Alpha graduates To betterassess the argument I would need to analyze 15-year trends in (l) the percentage of Omegastudents participating in the evaluation procedure (2) Omegas admission standards andquality of education and (3) Omegas emphasis on job training and career preparation I wouldalso need to know what other means are available to Omega for enabling its graduates to findbetter jobs

GRE AWA John박 박정어학원

The following appeared in a memo from a vice president of Quiot Manufacturing

During the past year Quiot Manufacturing had 30 percent more on-the-job accidents than at the nearby Panoply Industries plant where the work shifts are one hour shorter than ours Experts say that significant contributing factors in many on-the-job accidents are fatigue and sleep deprivation among workers Therefore to reduce the number of on-the-job accidents at Quiot and thereby increase productivity we should shorten each of our three work shifts by one hour so that employees will get adequate amounts of sleep

Write a response in which you examine the stated andor unstated assumptions of the argument Be sure to explain how the argument depends on these assumptions and what the implications are for the argument if the assumptions prove unwarranted

The following appeared in a memo from a vice president of Alta Manufacturing

During the past year Alta Manufacturing had thirty percent more on-the-job accidents than nearby Panoply Industries where the work shifts are one hour shorter than ours Experts believe that a significant contributing factor in many accidents is fatigue caused by sleep deprivation among workers Therefore to reduce the number of on-the-job accidents at Alta we recommend shortening each of our three work shifts by one hour If we do this our employees will get adequate amounts of sleep

Write a response in which you discuss what questions would need to be answered in order to decide whether the recommendation and the argument on which it is based are reasonable Be sure to explain how the answers to these questions would help to evaluate the recommendation

The following appeared in a memo from the vice president of Butler Manufacturing

During the past year workers at Butler Manufacturing reported 30 percent more on-the-job accidents than workers at nearby Panoply Industries where the work shifts are one hour shorter than ours A recent government study reports that fatigue and sleep deprivation among workers are significant contributing factors in many on-the-job accidents If we shorten each of our work shifts by one hour we can improve Butler Manufacturings safety record by ensuring that our employees are adequately rested

1 Write a response in which you discuss what specific evidence is needed to evaluate the argument and explain how the evidence would weaken or strengthen the argument

2 Write a response in which you discuss what questions would need to be answered in order to decide whether the recommendation is likely to have the predicted result Be sure to explain how the answers to these questions would help to evaluate the recommendation

4번 반복됨

Alta has 30 more job accidents than Panoply(work shifts one hour shorter)Experts Job accidents caused by fatigue and sleep deprivationTherefore to reduce job accidents and increase productivity shorten three work shifts by one hour for adequate sleep

지난해 우리 회사는 인근 Panoply Industries보다 업무상 재해가 30나 더 많았다 그 회사는 우리보다 근무 교대시간이 1시간 정도 짧았다 전문가들은 대부분의 업무상 재해에 있어서 가장 중요한 요인이 과로와 수면부족으로 보고있다 따라서 우리 회사에서 높은 산업재해를 줄이고 아울러 생산성을 높이기 위해서는 근로자들이 충분한 수면을 취할 수 있도록 1시간씩 3교대 시간을 줄여야 한다

In this memo the (author) vice president of Alta Manufacturing (AM) recommends that to reduce on-the-job accidents and increase productivity AM should shorten its three work shifts by one hour so that employees can

GRE AWA John박 박정어학원

get more sleep To support this recommendation the author provides several evidences However careful scrutiny of each of the facts reveals that it provides little credible support for the authorrsquos recommendation QuestionsThe number of accidents What kind of accidents The seriousness of the accidents is importantHow many employees are in each company What are their productsFalse cause Sleep may not be the reason for the on-the-job accidents What do Alta and Panoply manufacture

First of all the author believes that fatigue caused the on-the job accidents However there could be other reasons The author observes a correlation between sleep deprivation and on-the-job accidents then concludes that the former is the cause of the latter However the author fails to rule out other possible explanations For example it is entirely possible that Alta factories require more strenuous and dangerous labor than Panoply Without ruling out all other such factors it is unfair to conclude that fatigue is responsible for the accidents In addition the work-shifts may not be the cause of the sleep deprivation and fatigue It is possiblehellip Thus the author should provide what exactly Panoply and Alta manufacture and more precise data about their working conditions to be more convincing

Shortening the shift by one hour does not necessarily lead to more sleep And is one hour enoughLess accidents does not mean increased productivity

결론 We should shorten each of out three work shifts by one hour

반박 경쟁사에 비해서 시간당 하는 업무량이 많아서 더욱 피곤할 수도 있다 시간이 문제가 아니라 노후된 시설 설비 자체의 문제 작업 자체가 원래 위험한 것이여서 사고가 많을 수도 있다 다른 회사는 더욱 많은 작업시간에도 불구하고 안정한 작업여건으로 인해서 사고율이 오히려 더 작을 수도 있다비교사의 재해감소가 다른 요인일수 있다(안전 교육 철저)줄인 시간이 피로회복이나 수면으로 연결 안될수 있음(술을 마실 수도 있고 그 시간에 휴식을 취하지 않고 다른일을 함으로써 더욱 피로해질수 있다)

This editorial recommends that Alta Manufacturing reduce its work shifts by one hour each inorder to reduce its on-the-job accident rate and thereby increase Altas productivity To supportthis recommendation the author points out that last year the number of accidents at Alta was30 greater than at Panoply Industries where work shifts were one hour shorter The authoralso cites certain experts who believe that many on-the-job accidents are caused by fatigueand sleep deprivation I find this the argument unconvincing for several reasonsFirst and foremost the author provides absolutely no evidence that overall workerproductivity is attributable in part to the number of on-the-job accidents Although commonsense informs me that such a relationship exists the author must provide some evidence ofthis cause-and-effect relationship before I can accept the authors final conclusion that theproposed course of action would in fact increase Altas productivitySecondly the author assumes that some accidents at Alta are caused by fatigue or sleepdeprivation However the author overlooks other possible causes such as inadequateequipment maintenance or worker training or the inherent hazards of Altas manufacturingprocesses By the same token Panoplys comparatively low accident rate might be attributablenot to the length of its work shifts but rather to other factors such as superior equipmentmaintenance or worker training In other words without ruling out alternative causes ofon-the-job accidents at both companies the author cannot justifmbly conclude that merely byemulating Panoplys work-shift policy Alta would reduce the number of such accidentsThirdly even assuming that Altas workers are fatigued or sleep-deprived and that this is thecause of some of Altas on-the-job accidents in order to accept the authors solution to thisproblem we must assume that Altas workers would use the additional hour of free time tosleep or rest However the author provides no evidence that they would use the time in thismanner It is entirely possible that Altas workers would use that extra hour to engage in someother fatiguing activity Without ruling out this possibility the author cannot convincinglyconclude that reducing Altas work shifts by one hour would reduce Altas accident rateFinally a series of problems with the argument arise from the scant statistical information onwhich it relies In comparing the number of accidents at Alta and Panoply the author fails toconsider that the per-worker accident rate might reveal that Alta is actually safer than Panoplydepending on the total number of workers at each company Second perhaps accident rates

GRE AWA John박 박정어학원

at the two companies last year were aberrations and during other years Altas accident ratewas no greater or even lower than Panoplys rate Or perhaps Panoply is not representativeof industrial companies generally and that other companies with shorter work shifts have evenhigher accident rates In short since the argument relies on very limited statistical information Icannot take the authors recommendation seriouslyIn conclusion the recommendation for emulating Panoplys work-shift policy is not wellsupported To convince me that shorter work shifts would reduce Altas on-the-job accidentrate the author must provide clear evidence that work-shift length is responsible for some ofAltas accidents The author must also supply evidence to support her final conclusion that alower accident rate would in fact increase overall worker productivity

The following appeared in a memo from the vice president of marketing at Dura-Sock Inc

A recent study of our customers suggests that our company is wasting the money it spends on its patented Endure manufacturing process which ensures that our socks are strong enough to last for two years We have always advertised our use of the Endure process but the new study shows that despite our socks durability our average customer actually purchases new Dura-Socks every three months Furthermore our customers surveyed in our largest market northeastern United States cities say that they most value Dura-Socks stylish appearance and availability in many colors These findings suggest that we can increase our profits by discontinuing use of the Endure manufacturing process

1 Write a response in which you examine the stated andor unstated assumptions of the argument Be sure to explain how the argument depends on these assumptions and what the implications are for the argument if the assumptions prove unwarranted

2 Write a response in which you discuss what specific evidence is needed to evaluate the argument and explain how the evidence would weaken or strengthen the argument

3 Write a response in which you discuss what questions would need to be answered in order to decide whether the recommendation and the argument on which it is based are reasonable Be sure to explain how the answers to these questions would help to evaluate the recommendation

Intro The vice president of marketing at Dura-Sock Inc is offering a potentially harmful investment recommendation by claiming that Dura-Sock should discontinue its use of the ldquoEndurerdquo process To support his recommendation he points out a study that Dura-Sock customers actually purchase the socks every three months and a survey that reveals that Dura-Sock customers like the sockrsquos stylish appearance and availability in many colors The study and survey however are insufficient in supporting his proposal and the VP makes several unwarranted assumptionsIntro (simplified) The VP states that though Dura-Socks last for two years customers buy the socks every three months Therefore he assumes that the consumersrsquo motive for buying the produce is not its durabilityHowever the author fails to rule out other possible motivation for consumption

Even if the survey is reliable the author should consider the rest of the market Vague terms ldquowasting moneyrdquomdashprecisely how much are they wasting Studysurvey errorThe company must calculate the outcome of such momentous decisionStudy participantsrsquo comment that they prefer Dura-Sock for its stylishness and availability might take Dura-Sockrsquos enduring quality for granted

우리회사 제품 소비자들에 대한 최근 조사에서 지난 2년여간 양말의 내구성을 강하게 하는 필수공정이었던 자사 특허의 Endure 공정에 들어가는 비용이 낭비라고 말하고 있다 우리 회사는 항상 이 공정 처리에 대한 광고를 내보냈으나 이에 대한 시장 조사에서 실제로 고객들은 이 신제품을 평균 석달마다 구매하는 것으로 나타났다 더군다나 북동부지역에서 실시한 대규모 시장조사에 응답한 고객들은 양말의 모양과 색상등에 더

GRE AWA John박 박정어학원

관심을 나타냈다 이러한 결과는 우리회사가 신기술 공법을 중단하면 그에 따라 수익이 늘어날 것이라는 것을 말해주고 있는 것이다주장 These findings suggest that Dura0Sock can increase its profits by discontinuing its use of the ldquoEndurerdquo manufacturing process

1 survey가 정확한 소비자의 의견을 나타낸 것인가 다른 선택없이 양자택일과 같은 방법의 survey였는지2 북동부지역의 시장조사가 전체 의견을 대표할 수 있나3 사람들이 모양이나 색상에 앞서 내구성을 먼저 평가했을 수도 있다 내구성을 갖추었다는 전제하에 모양과 색상에 관심을 드러낸 것일 수 있다4 소비자가 도매상(retail)인지 소매상(whole)인지가 없다

The following appeared in a business magazine

As a result of numerous complaints of dizziness and nausea on the part of consumers of Promofoods tuna the company requested that eight million cans of its tuna be returned for testing Promofoods concluded that the canned tuna did not after all pose a health risk This conclusion is based on tests performed on samples of the recalled cans by chemists from Promofoods the chemists found that of the eight food chemicals most commonly blamed for causing symptoms of dizziness and nausea five were not found in any of the tested cans The chemists did find small amounts of the three remaining suspected chemicals but pointed out that these occur naturally in all canned foods

Write a response in which you discuss what questions would need to be addressed in order to decide whether the conclusion and the argument on which it is based are reasonable Be sure to explain how the answers to the questions would help to evaluate the conclusion

Representativeness of the tested cansThey should conduct a comparative studyThe testing could be biased because Promofoods employees conducted the testingHow much (quantity) of the five and three suspected chemicals were in the canned foodsFalse cause The substance that caused dizziness and nausea may not be one of the eight common chemicals

많은 소비자들의 현기증과 구역질 불만에 따라 Promofoods사는 지난해 참치 캔 8백만 개를 테스트하기 위해 반품시켰다 그 결과 캔에서는 건강에 위험이 될 수 있는 화합물질이 없었던 것으로 회사측은 결론지었다 이러한 결론은 회사측 화학연구자들이 회수된 캔의 샘플을 테스트해서 이들 증상의 원인이 되는 8가지 화합물 중에서 5가지가 실험된 캔에서 발견되지 않았다는 사실에 근거한 것이다 이들 화학자들은 나머지 3개가지 화합물이 모든 캔 식료품에서 흔히 발견되는 것이라고 언급했다 결론 Promofoods concluded that the cans did not after all contain chemicals that posed a health risk

1 공인된 기간에서 테스트를 한 것이 아니고 자사에서 직접 테스트를 했기에 신뢰성이 안간다 2 이런 증상을 일으키는 8개의 물질 말고 다른 물질들이 캔속에 많이 포함됬을수 있다 3 나머지 3개의 물질들의 함유량이 많아서 다른 종류의 캔들은 문제를 일으키지 않지만 참치캔은 문제를

일으킬 수 있다

This magazine article concludes that the 8 million cans of tuna Promofoods recalled due tocomplaints about nausea and dizziness do not after ail contain any chemicals that pose a

GRE AWA John박 박정어학원

health risk To support this conclusion the author cites the fact that five of eight chemicalscommonly causing these symptoms were not found in the recalled cans while the other threealso occur naturally in other canned foods For several reasons this evidence lends littlecredible support to the authors conclusionTo begin with the author relies partly on the fact that although three of the eight chemicalsmost commonly blamed for nausea and dizziness appeared in Promofoods recalled tunathese chemicals also occur naturally in other canned foods However this fact alone lends nosupport to the authors conclusion for two reasons First the author might be ignoring animportant distinction between naturally occurring chemicals and those not occurring naturallyIt is entirely possible that these three chemicals do not occur naturally in Promofoods tunaand that it is for this reason that the chemicals cause nausea and dizziness Secondly it isentirely possible that even when they occur naturally these chemicals cause the samesymptoms Unless the author rules out both possibilities he cannot reliably conclude that therecalled tuna would not cause these symptomsAnother problem with the argument is that the authors conclusion is too broad Based onevidence about certain chemicals that might cause two particular heath-related symptoms theauthor concludes that the recalled tuna contains no chemicals that pose a health risk Howeverthe author fails to account for the myriad of other possible health risks that the recalled tunamight potentially pose Without ruling out all other such risks the author cannot justifiablyreach his conclusionA third problem with the argument involves that fact that the eight particular chemicals withwhich the test was concerned are only the eight most commonly blamed for nausea anddizziness It is entirely possibly that other chemicals might also cause these symptoms andthat one or more of these other chemicals actually caused the symptoms Without ruling outthis possibility the author cannot jusufiably conclude that the recalled tuna would not causenausea and dizzinessA final problem with the argument involves thetesting procedure itself The author providesno information about the number of recaUed cans tested or the selection method used Unlessthe number of cans is a sufficiently large sample and is statistically repre sentative of all therecalled cans the studys results are not statistically reliableIn conclusion the article is unconvincing as it stands To strengthen the assertion that therecalled tuna would not cause nausea and dizziness the author must provide evidence thatthe three chemicals mentioned that occur naturally in other canned foods also appear naturallyin Promofoods tuna The author must also provide evidence that ingesting other canned foodscontaining these three chemicals does not cause these symptoms To better evaluate theargument we would need to know whether the sample used in the tests was statisticallysignificant and representative of all the recalled tuna We would also need to know what otherchemicals in the recalled tuna might pose any health risk at all

5그룹 불충분 조건오류 빈출

Natures Way a chain of stores selling health food and other health-related products is opening its next franchise in the town of Plainsville The store should prove to be very successful Natures Way franchises tend to be most profitable in areas where residents lead healthy lives and clearly Plainsville is such an area Plainsville merchants report that sales of running shoes and exercise clothing are at all-time highs The local health club has more members than ever and the weight training and aerobics classes are always full Finally Plainsvilles schoolchildren represent a new generation of potential customers these schoolchildren are required to participate in a fitness-for-life program which emphasizes the benefits of regular exercise at an early age

Write a response in which you examine the stated andor unstated assumptions of the argument Be sure to

GRE AWA John박 박정어학원

explain how the argument depends on these assumptions and what the implications are for the argument if the assumptions prove unwarranted

False cause

First of all the author believes that the Increased sales of running shoes and exercise clothing indicates

Plainesville residentsrsquo interest in leading healthy lives However this assumption is not logically convincing for

several reasons could be a fashion trendTime shift ldquoFitness for liferdquo might not have any influence on schoolchildren as they growFalse cause There could be other reasons for member increase in the health clubAll of the above are insufficient condition

The author has to prove that local residents are interested in leading healthy lives However he supports his conclusion with insufficient evidence Nevertheless even if the residents are concerned with health naturersquos way may not be successful First

그 동안의 경험을 토대로 볼 때 건강생활과 밀접히 관련되어 있는 거주 지역에서 본 상점들이 아주 호응을 얻고 있다 따라서 이러한 주민들이 많이 거주하고 있는 Plainsville 에 새로운 상점들을 계속 세워야 한다 이 지역 상인들은 런닝화와 운동복 판매가 가장 높다고 말한다 불과 5 년전에는 거의 전무하다시피하던 지역 헬스 클럽의 경우도 엄청나게 많은 회원을 확보하고 있으며 웨이트 트레이닝과 에어로빅 강좌들도 항상 만원이라고 한다 새로운 고객층을 예측해 보는 것도 가능하다 이 지역의 학생들의 경우 Fitness for Life프로그램을 받게 되는데 이러한 프로그램을 통해서 유년시절부터 정규적인 운동 습관을 들이게 하고 있는 것이 그것이다

결론 We should therefore build our next new store in Plainsville

반박 그동안의 경험에 의한 과거 통계가 꼭 여기에도 적용되는건 아니다 5 년전 헬스 클럽이 잘 안되었던게 다른 원인이였을수 있다(강사수준 미달 강좌미비)tourist 에 의한 원인 일수 있다 어렸을때부터 운동을 했다고 해서 커서도 관심이 있지는 않다 (오히려 반감이 있을수 있다 혹은 건강하기에 건강에 관심이 적을수도 있다)운동복이나 신발의 판매가 육체노동에 의한 것일수도 있다

IntroductionSupport1049896In this memorandum the author asserts that Naturersquos Way should build its next newstore in Plainsville To support this assertion the author states that Plainsvillesmerchantsrsquo sales of exercise clothing are going well the local health club has moremembers than ever and a new generation of customers will help to ensure NaturersquosWayrsquos success At first glance the authorrsquos assumption seems convincing but in-depth scrutiny revealsthat it lacks substantial evidence as it stands

Body 1-SamplingTopic Sentence 1To begin with the author assumes that the merchantsrsquo report indicates that the residentsare concerned about their health However this assumption is based on unsubstantiated

GRE AWA John박 박정어학원

data Example 1 (Rebuttal1) First if we do not know the total volume of items sold and the price of the goods exactly we cannot infer whether the residents are actually buying many goods Example 2 (Rebuttal2)In addition to that the report emphasizes the rising sales of running shoes and exerciseclothing however these may not be hot-selling items for Naturersquos Way or may not be theproducts the company is planning to sell Concluding Sentence Therefore in order to make the argument reliable the author should reconsider themerchantsrsquo report with more detailed data

Body 2-CausalTopic Sentence 2Second the author contends that the health clubs classes are full yet this does not meanthat many people actually use the health club other factors may be the real cause forthose closed classes Example 1 (Rebuttal1) To begin with if the health club is very small the number of people working out wouldnot be a large one In fact regular gym-going may just be a vogue among a smallunrepresentative segment of Plainsvilles population Example 2 (Rebuttal2) Moreover it is possible that most of the people who exercise in the health club do weight training and aerobics only to look good and to meet other singles not for their health In that case there would be little demand for health products Concluding SentenceThus the author should not hasten to presume what really caused people to be interested in a healthier lifestyle and enroll in the health club

Body 3-Time-ShiftTopic Sentence 3Finally the author highlights that Naturersquos Way can expect a new generation of customersin Plainsville that will help the company in the long term This notion is mistaken in that itassumes the conditions of the present will continue unchanged in the future Although theschool children are required to participate in the fitness for life program they may notnecessarily buy Naturersquos Ways products Example 1 (Rebuttal1) In the first instance they may suffer a fall in purchasing power arising from future economic difficulties this would cause reluctance to spend a considerable amount of money on health products which tend to be more expensive Example 2 (Rebuttal2)Another possibility is that there may emerge many competitor companies vying with Naturersquos Way so that in the future the school children may not feel the necessity to purchase one companyrsquos health products over anotherrsquosConcluding Sentence Thus the authorrsquos assumption is highly speculative since it relies heavily on unknowablefuture circumstances

ConclusionThesis In sum the author uses many assumptions that are insufficient in supporting his claimsSupportIn order for the authorrsquos claims to be convincing he needs to advance more persuasiveevidence that people in Plainsville really are concerned with their health and health foodThe following was written as a part of an application for a small-business loan by a group of developers in the city of Monroe

A jazz music club in Monroe would be a tremendously profitable enterprise Currently the nearest jazz club is 65 miles away thus the proposed new jazz club in Monroe the C-Note would have the local market all to itself Plus jazz is extremely popular in Monroe over 100000 people attended Monroes annual jazz festival last summer several well-known jazz musicians live in Monroe and the highest-rated radio program in Monroe is Jazz Nightly which airs every weeknight at 7 PM Finally a nationwide study indicates that the typical jazz fan spends close to $1000 per year on jazz entertainment

1 Write a response in which you discuss what specific evidence is needed to evaluate the argument and explain how the evidence would weaken or strengthen the argument

2 Write a response in which you examine the stated andor unstated assumptions of the argument Be

GRE AWA John박 박정어학원

sure to explain how the argument depends on these assumptions and what the implications are for the argument if the assumptions prove unwarranted

3 Write a response in which you discuss what questions would need to be answered in order to decide whether the prediction and the argument on which it is based are reasonable Be sure to explain how the answers to these questions would help to evaluate the prediction

Group error nationwide survey may not reflect local trends Is the nationwide jazz fan population substantialInsufficient non-residents of Monroe may have attended the jazz festival (Body alternative explanation last year may have been an anomaly The author should consider data from various years) The author should indicate how many out of 100000 were Monroe residentsNationwide study Does this reflect Insufficient Citizens of Monroe may continue to go to the jazz club 65 miles away

Are the people in Monroe really interested in jazzMajority of the people who attended the jazz festival might not be Monroe residentsSurvey error nationwide study may not be applicable to MonroeJazz musicians who live in MonroeMonopolyRadio station

In this business application the author claims that the proposed jazz club C Note will be very profitable in Monroe To support this claim the author argues for his case with several evidences At first glance the authorrsquos argument seems convincing however careful scrutiny reveals that his argument in specious

To begin with the author claims that Monroersquos citizens are interested in jazz He presents three evidences First Secondhellip Thirdhellip Howeverhellip

Monroe 시에 있는 재즈 음악 클럽은 수익성이 좋은 사업이다 현재 가장 가까이에 있는 클럽은 65 마일 정도 떨어져 있다 따라서 이번에 세우려고 하는 C Note 는 독보적인 위치를 점할것이다 더군다나 재즈는 이 시에서 가장 인기있는 음악이다 지난 여름 재즈 축제에서는 10 만명 이상의 Morone 시 주민이 참석하였고 몇몇 유명한 재즈 음악가들도 이곳에 살고 있으며 저녁때 방영되는 라디오 프로그램중에서 최고의 시청률을 보이고 있는 것도 Jazz Nightly 이다 전국조사에서도 전형적인 재즈 팬들은 재즈 분야에 년간 1천 달러 가까이 지출하고 있는 것으로 보고되고 있다 따라서 C Note 클럽이 돈을 벌 수 있는 사업이라는 것은 확실한 것이다

결과 It is clear that the C Note cannot help but make money반박 nearest jazz club 이 양질의 써비스로 여전히 손님을 끌수도 있다Festival 에 얼마나 참여하는지가 jazz 의 인기를 반영하지 않는다 뮤지션이 많이 사는거랑 jazz 의 인기가 상관없다라디오 프로그램이 다른 요인에 의해서 인기일수도 있다 (진행자때문)전국 통계 적용 불가화목 실전반_Ms Noh6In this application the author suggests that a jazz club in Monroe will make a number of profits To support this suggestion the author exemplifies the local condition popularity of jazz in Monroe and nationwide study However careful scrutiny of each of the facts reveals that it provides little credible support for the authorrsquos recommendation Good clear intro

First the author assumes that jazz is popular in Monroe because of several facts the jazz festival last year had high participation some famous jazz musicians live in Monroe and the high-rated radio program is lsquoJazz Nightlyrsquo However this assumption has many drawbacks that must be seriously considered(Good topic sentences) If many attendants in the last-yearrsquos festival came from other cities and not Monroe it is hard to conclude that Monroersquos people like jazz Therefore the author must examine how many Monroe residents actually attended the festival On top of that there is little relationship between habitation of famous jazz musician and the popularity of jazz in Monroe Although several well-known musicians live there if they do not take part in any jazz performance of Monroe this might have no effect to the interest of Monroersquos residents

GRE AWA John박 박정어학원

about jazz Finally in the case of radio program this is also not suitable reason why jazz is popular in Monroe It might be possible that people cannot help choosing lsquoJazz Nightlyrsquo because there are few radio programs at Night The fact that the radio program is the highest rating program is not a germane evidence The approximate number of listeners would be the more crucial evidence Therefore the author needs to seriously deliberate the correlation between jazzrsquos popularity in Monroe and his examples (Good logical flow and clarity)

Second the author uses as evidence the nationwide study that jazz fans spend much money on jazz entertainment to substantiate why starting a jazz club in Monroe will be profitable In other words the author assumes that the characteristics of a nationwide study can be applied to Monroe The national study would lend support to the applicantrsquos claim only if residents in Monroe typify national jazz fans However the author does not provide credible evidence that this is the case Moreover the populations of jazz fans nationwide may be insubstantial Thus the author should not infer hastily that Monroersquos residents will spend much money on enjoying jazz from the nationwide study

Lastly even if jazz is popular in Monroe C Note may not be successful It is entirely possible that residents might still prefer other clubs where they have always went In addition there is another possibility that the nearest jazz club will attract many of Monroersquos people because it serves fine performances and is equipped with favorite facilities Without considering these other possibilities the author cannot make his argument convincing In sum the author presents many reasons that are insufficient in supporting his or her claim In order for the authorrsquos claims to be convincing he needs to advance more persuasive evidence such as the total number of Monroe residents who attended the jazz festival the effects on the popularity of jazz by the musicians living in Monroe and the actual number of residents who would typify themselves to be jazz fans through a local survey Without substantial evidence that C Note will be successful in Monroe the businessmen may be overinvesting in what might lead to a business failureExcellent clarity Score 50

The following appeared in a newsletter offering advice to investors

Over 80 percent of the respondents to a recent survey indicated a desire to reduce their intake of foods containing fats and cholesterol and today low-fat products abound in many food stores Since many of the food products currently marketed by Old Dairy Industries are high in fat and cholesterol the companys sales are likely to diminish greatly and company profits will no doubt decrease We therefore advise Old Dairy stockholders to sell their shares and other investors not to purchase stock in this company

Write a response in which you discuss what questions would need to be answered in order to decide whether the advice and the argument on which it is based are reasonable Be sure to explain how the answers to these questions would help to evaluate the advice

Survey 80

GRE AWA John박 박정어학원

Old Dairy could change their products and manufacture low fat dairy foodsLess competing companies Old Dairy could eventually be the only company that produces hellipImprecise numbers and measurementsCustomers may still buy high fat dairy products

The author of the newsletter is offering potentially dangerous advice by recommending Old Dairy stockholders to withdraw investment and stop purchase What is more the authorrsquos prediction debases the reputation and business of Old Dairy and if false could devoid the investment opportunity of the newsletter readers Therefore investors should examine whether the authorrsquos evidences are substantial

To begin with the author states that 80 percent of the respondents in a survey indicated a desire to reduce their intake of foods He therefore argues that Old Dairyrsquos high fat and cholesterol products would decrease in sales However the author makes a crucial error in this argument First the author provides no evidence that the surveyrsquos results are statistically reliable Were they representative of all the customers Were they chosen for the survey randomly Furthermore the desire to reduce fat and cholesterol intake is a pervasive trend in todayrsquos opulent society however the author erroneously identifies this as a new phenomenon which will affect consumer trends Second having a desire to reduce fat and cholesterol intake does not necessarily indicate that people who have this desire will actually reduce consuming these types of products It is entirely possible that they may continue buying Old Dairy products for its quality and taste Accordingly the author cannot draw any firm conclusion that people will not buy Old Dairy products Therefore if any of these cases are true the author may be offering investors a detrimental investment advice

최근 조사에 대한 응답자중 80 이상이 자신이 먹는 음식에서 지방과 콜레스테롤의 함유량을 줄이고 싶다고 한다 아울러 요즘은 많은 식료품 가계에서 저지방 제품들을 많이 취급하고 있다 현재 Old Dairy Industries가 판매하고 있는 많은 음식제품들은 지방과 콜레스테롤이 높기 때문에 이 회사의 매출이 격감할 것으로 보이며 당연히 매출이익도 줄어들것이다 따라서 이 회사의 주주들은 주식을 매각하고 다른 주식 투자가들도 이 회사의 주식을 매입하지 않는 것이 좋다

결론 Old Dairy stockholders to sell their shares and other investors not to purchase stock in this company

반박 모든 상품이 다 고 지방 고 칼로리는 아니다(비록 많을지라도) 일부의 식품의 경우 기호에 맞어서 히트해서 전체적인 수입이 증가할 수도 있다국내시장만 생각할 수 없다( 외국시장에서 호황을 누릴수 있다 )입맛이라는게 즉각 바뀌는게 아니다

The following appeared in a letter to the editor of the Balmer Island Gazette

On Balmer Island where mopeds serve as a popular form of transportation the population increases to 100000 during the summer months To reduce the number of accidents involving mopeds and pedestrians the town council of Balmer Island should limit the number of mopeds rented by the islands moped rental companies from 50 per day to 25 per day during the summer season By limiting the number of rentals the town council will attain the 50 percent annual reduction in moped accidents that was achieved last year on the neighboring island of Seaville when Seavilles town council enforced similar limits on moped rentals

1 Write a response in which you discuss what questions would need to be answered in order to decide whether the recommendation is likely to have the predicted result Be sure to explain how the answers to these questions would help to evaluate the recommendation

2 Write a response in which you discuss what questions would need to be answered in order to decide whether the prediction and the argument on which it is based are reasonable Be sure to explain how the answers to these questions would help to evaluate the prediction

3 Write a response in which you examine the stated andor unstated assumptions of the argument Be sure to explain how the argument depends on these assumptions and what the implications are for the argument if the assumptions prove unwarranted

Whatrsquos the actual population of Balmer Island 100000mdashis this a significant increase What kind of accidents Skin abrasions or serious injury And compared to Seaville how serious are the accidents and the actual number of accidents Did Seaville enforce other restrictions like safety signsHow different are the conditions of Balmer

GRE AWA John박 박정어학원

and Seaville regarding population road (safety) conditions topography other town-government regulation How much will the economy of Balmer be affected do to this restriction Could it cause an economic recession due to the fact that these rental companiesrsquo chance to make money is only during the summer thereby weakening the economic infrastructure Are there any other ways that could better alleviate the accident rate

Statistics 50-impreciseAnalogy Balmer compared with TorseauFalse Cause Accidents might have occurred because of reasons other than mopeds False Cause population increase may not be part of the cause of the accidentsOther explanations for the accident pedestrians few road safety regulations narrow roadsThere could be other better solutionshellip

Balmer Island의 인구가 여름철에는 십만명으로 늘어난다 2륜차와 보행자간 사고를 줄이기 위해 시의회는 6개의 자전거를 포함한 2륜차 대여업체에게 이 기간동안에는 대여숫자를 일일 50에서 30으로 제한하도록 할 것이다 대여숫자를 줄임으로써 시 의회는 지난해 이웃한 Torseau섬에서 이와 동일한 규제를 시행해서 50나 줄인 결과를 보고 마찬가지로 50를 줄일수 있다고 확신하고 있다

결론 The town council of Balmer Island should linit the number

반박 보행자의 부실에 의해서 사고가 많이 일어날수도 있다렌탈수의 줄임만이 대책은 아니다(대부분의 사람들이 렌탈 보다는 소유하고 있을 수도 있다)옆섬과는 상황이 다를수도 있다(그 섬에서는 사고의 원인이 많은 자전거 수로 인한것일수있다) 하지만 이 섬은 좁은 도로가 원인일 수도 있고 도로 안전 장치의미비가 원일일수 있다

In this letter the author recommends that Balmer Island should limit the number moped rentals from 50 to 30 per day To support this recommendation the author points out several reasons However careful scrutiny of each of the facts reveals that it is filled with unanswered questions that could significantly weaken the authorrsquos recommendation with loops and holes which are answered

The recommendation depends on the assumption that no alternative means of reducing the number of accidents are available However the author fails to offer any evidence to substantiate this crucial assumption It is highly possible that means other than this would better solve the problem Perhaps they could widen the roads or put-up more safety signs Or perhaps the accidents were due to the lack of skills in which case proper safety training would significantly alleviate the problem Without considering and ruling out these and other alternative means of reducing accidetns the author cannot confidently conclude that merely emulating Torseau would suffice Moreover the author is advising a recommendation which could potentially harm the economy of Balmer Island sincehellip Moreover the Balmer Island should alternative means to reduce accidents because limiting moped rentals during the summer could harm the economy of Balmerhellip

First of all the author believes that increase in population and the number of moped rentals are responsible for the accidents It is entirely possible that other factors are responsible for the accidents Perhaps Balmer Islandrsquos lack of safety signs was a major factor Or maybe the roads are narrow and dangerous on the Island therefore the town council could enforce stricter traffic regulations to alleviate the problem Accordingly if either of these scenarios is true the author cannot draw any firm conclusion that increase in the number of population and moped rentals are the cause of the accidents

The author of this editorial recommends that to reduce accidents involving mopeds andpedestrians Balmer Islands city council should restrict moped rentals to 30 per day down from50 at each of the islands six rental outlets To support this recommendation the author citesthe fact that last year when nearby Torseau Islands town council enforced similar measuresTorseaus rate of moped accidents fell by 50 For several reasons this evidence providesscant support for the authors recommendationTo begin with the author assumes that all other conditions in Balmer that might affect therate of moped-pedestrian accidents will remain unchanged after the restrictions are enactedHowever with a restricted supply of rental mopeds people in Balmer might purchase mopedsinstead Also the number of pedestrians might increase in the future with more pedestriansespecially tourists the risk of moped-pedestrian accidents would probably increase For thatmatter the number of rental outlets might increase to make up for the artificial supplyrestriction per outlet--a likely scenario assuming moped rental demand does not declineWithout considering and ruling out these and other possible changes that might contribute to ahigh incidence of moped-pedestrian accidents the author cannot convince me that theproposed restrictions will necessarily have the desired effect

GRE AWA John박 박정어학원

Next the author fails to consider other possible explanations for the 50 decline inTorseaus moped accident rate last year Perhaps last year Torseau experienced unusually fairweather during which moped accidents are less likely Perhaps fewer tourists visited Tot seanlast year than during most years thereby diminishing the demand for rental mopeds to belowthe allowed limits Perhaps last year some of Torseaus moped rental outlets purchased newmopeds that are safer to drive Or perhaps the restrictions were already in effect but were notenforced until last year In any event a decline in Torseaus moped accident rate during onlyone year is scarcely sufficient to draw any reliable conclusions about what might have causedthe decline or about what the accident rate will be in years aheadAdditionally in asserting that the same phenomenon that caused a 50 decline in mopedaccidents in Torseau would cause a similar decline in Balmer the author relies on what mightamount to an unfair analogy between Balmer and Torseau Perhaps Balmers ability to enforcemoped-rental restrictions does not meet Torseaus ability if not then the mere enactment ofsimilar restrictions in Balmer is no guarantee of a similar result Or perhaps the demand formopeds in Torseau is always greater than in Balmer Specifically if fewer than all availablemopeds are currently rented per day from the average Balmer outlet while in Torseau everyavailable moped is rented each day then the proposed restriction is likely to have less impacton the accident rate in Balmer than in TorseauFinally the author provides no evidence that the same restrictions that served to reduce theincidence of all moped accidents by 50 would also serve to reduce the incidence ofaccidents involving mopeds and pedestrians by 50 Lacking such evidence it is entirelypossible that the number of moped accidents not involving pedestrians decreased by a greaterpercentage while the number of moped-pedestrian accidents decreased by a smallerpercentage or even increased Since the author has not accounted for these possibilities theeditorials recommendation cannot be taken seriouslyIn conclusion the recommendation is not well supported To convince me that the proposedrestriction would achieve the desired outcome the author would have to assure me that nochanges serving to increase Balmers moped-pedestrian accident rate will occur in theforeseeable future The author must also provide dear evidence that last years decline inmoped accidents in Torseau was attributable primarily to its moped rental restrictions ratherthan to one or more other factors In order to better evaluate the recommendation I wouldneed more information comparing the supply of and demand for moped rentals on the twoislands I would also need to know the rate of mopedpedestrian accidents in Torseau both priorto and after the restrictions were enforced in TorseauThe following appeared in a magazine article about planning for retirement

Clearview should be a top choice for anyone seeking a place to retire because it has spectacular natural beauty and a consistent climate Another advantage is that housing costs in Clearview have fallen significantly during the past year and taxes remain lower than those in neighboring towns Moreover Clearviews mayor promises many new programs to improve schools streets and public services And best of all retirees in Clearview can also expect excellent health care as they grow older since the number of physicians in the area is far greater than the national average

Write a response in which you discuss what specific evidence is needed to evaluate the argument and explain how the evidence would weaken or strengthen the argument

-Natural beauty and consistent climate may not be the most wanted qualities-Housing costs could have lowered on a national level wealthy retirees may not care about costs-Taxes may be high compared to the nationrsquos average tax rate-What about other qualities of Clearview Crime rate what qualities would retirees want -If schools streets and public services need improvement then this is proof that the current condition of Clearview is low Or due to budgetary reasons the mayor may not follow-up on his promise because of lowered tax rate -Schools and people who are retired no relationship-Physicians What kind of physicians Number is irrelevant Are these physicians capable of addressing the illnesses of old people

This author argues that anyone seeking a place to retire should choose Clearview To supportthis argument the article cites Clearviews consistent climate and natural beauty its fallinghousing costs its low property taxes compared to nearby towns and the mayors promise toimprove schools streets and services The article also claims that retirees can expectexcellent health care because the number of physicians in Clearview greatly exceeds thenational average This argument is flawed in several critical respectsTo begin with although consistent climate and natural beauty might be attractive to manyretirees these features are probably not important to all retirees For many retirees it isprobably more important to live near relatives or even to enjoy changing seasons Thus I

GRE AWA John박 박정어학원

cannot accept the authors sweeping recommendation for all retirees on this basisAlso Clearviews declining housing costs do not necessarily make Clearview the best placeto retire for two reasons First despite the decline Clearviews housing costs might be highcompared to housing costs in other cities Secondly for wealthier retirees housing costs arenot likely to be a factor in choosing a place to retire Thus the mere fact that housing costshave been in decline lends scant support to the recommendationThe articles reliance on Clearviews property-tax rates is also problematic in two respectsFirst retirees obviously have innumerable choices about where to retire besides Clear viewand nearby towns Secondly for retirees who are well-off financially property taxes are notlikely to be an important concern in choosing a place to retire Thus it is unfair to infer fromClearviews property-tax rates that retirees would prefer ClearviewYet another problem with the argument involves the mayors promises In light of Clearviewslow property-tax rates whether the mayor can follow through on those promises is highlyquestionable Absent any explanation of how the city can spend more money in the areas citedwithout raising property taxes I simply cannot accept the editorials recommendation on thebasis of those promises Besides even if the city makes the improvements promised thoseimprovements--particular the ones to schools--would not necessarily be important to retireesFinally although the number of physicians in Clearview is relatively high the per capitanumber might be relatively low Moreover it would be fairer to compare this per capita numberwith the per capita number for other attractive retirement towns--rather than the nationalaverage After all retirees are likely to place a relatively heavy burden on health-careresources Besides the article provides no assurances that the number of physicians inClearview will remain high in the foreseeable futureIn conclusion the recommendation is poorly supported To strengthen it the author mustconvince me--perhaps by way of a reliable survey--that the key features that the vast majorityof retirees look for in choosing a place to live are consistent climate natural beauty and lowhousing costs The author must also provide better evidence that Clear views property taxesare lower than the those of cities in other areas The author must also explain how the city canmake its promised improvements without raising property taxes Finally to better assess theargument I would need to now how the per capita number of physicians in Clearview wouldcompare to the national average in the futureThe following appeared as a letter to the editor from a Central Plaza store owner

Over the past two years the number of shoppers in Central Plaza has been steadily decreasing while the popularity of skateboarding has increased dramatically Many Central Plaza store owners believe that the decrease in their business is due to the number of skateboard users in the plaza There has also been a dramatic increase in the amount of litter and vandalism throughout the plaza Thus we recommend that the city prohibit skateboarding in Central Plaza If skateboarding is prohibited here we predict that business in Central Plaza will return to its previously high levels

Write a response in which you discuss what questions would need to be answered in order to decide whether the recommendation is likely to have the predicted result Be sure to explain how the answers to these questions would help to evaluate the recommendation

Why two years ago What happened two years ago which started this declineIs the dramatic increase in the ldquopopularityrdquo of skateboarding the cause of the steady decline of shoppers Are there any malls nearby Were there any changes nearby which could affect the decline in customersmdasha big mall perhaps Could the decline be due to the shop ownersHow many skateboarders use the plazaWhere do they skateboardDo they shop and are they customersAre the increase in litter and vandalism due to skateboarders Could this be alleviated by installing CCTVs and hiring security

This editorial concludes that the city should ban skateboarding from its downtown CentralPlaza in order to attract visitors to that area to return the area to its former glory and to makeit a place where people can congregate for fun and relaxation To justify this conclusion theeditorial points out that skateboarders are nearly the only people one sees anymore at CentralPlaza and that the Plaza is littered and its property defaced The editorial also points out thatthe majority of downtown merchants support the skate boarding ban This argument is flawedin several critical respectsFirst the editorials author falsely assumes that a ban on skateboarding is both necessaryand sufficient to achieve the three stated objectives Perhaps the city can achieve thoseobjectives by other means as well--for example by creating a new mall that incorporates anattractive new skateboard park Even if banning skateboarders altogether is necessary to meetthe citys goals the author has not shown that this action by itself would suffice Assuming thatthe Plazas reputation is now tarnished restoring that reputation and in turn enticing peopleback to the Plaza might require additional measures--such as removing litter and graffiti

GRE AWA John박 박정어학원

promoting the Plaza to the public or enticing popular restaurant or retail chains to the PlazaSecondly the editorial assumes too hastily that the Plazas decline is attributable to theskateboarders--rather than to some other phenomenon Perhaps the Plazas primary appeal inits glory days had to do with particular shops or eateries which were eventually replaced byless appealing ones Or perhaps the crime rate in surrounding areas has risen dramatically forreasons unrelated to the skateboarders presence at the Plaza Without ruling out these andother alternative explanations for the Plazas decline the editorials author cannot convince methat a skateboard ban would reverse that declineThirdly the editorials author might be confusing cause with effect--by assuming that theskateboarders caused the abandonment of the Plaza rather than vice versa It is entirelypossible that skateboarders did not frequent the Plaza until it was largely abandoned--andbecause it had been abandoned In fact this scenario makes good sense since skateboardingis most enjoyable where there are few pedestrians or motorists to get in the wayFourth it is unreasonable to infer from the mere fact that most merchants favor the ban thatthe ban would be effective in achieving the citys objectives Admittedly perhaps thesemerchants would be more likely to help dean up the Plaza area and promote their businesseswere the city to act in accordance with their preference Yet lacking any supporting evidencethe author cannot convince me of this Thus the survey amounts to scant evidence at best thatthe proposed ban would carry the intended resultFinally the author recommends a course of action that might actually defeat the citysobjective of providing a fun and relaxing place for people to congregate In my experienceskateboarding contributes to an atmosphere of fun and relaxation for adults and children alikemore so than many other types of ambiance Without considering that continuing to allowskateboarding--or even encouraging this activity--might achieve the citys goal more effectivelythan banning the activity the author cannot convincingly conclude that the ban would be in thecitys best interestsIn sum the argument is a specious one To strengthen it the editorials author must providedear evidence that skateboarding and not some other factor is responsible for the conditionsmarking the Plazas decline The author must also convince me that no alternative means ofrestoring the Plaza are available to the city and that the proposed ban by itself would suffice toattract tourists and restore the Plaza to its former glory Finally to better assess the argument itwould be useful to know the circumstances under which the downtown merchants would bewilling to help the city achieve its objectives

6그룹 약한 비유 빈출

The following recommendation appeared in a memo from the mayor of the town of Hopewell

Two years ago the nearby town of Ocean View built a new municipal golf course and resort hotel During the past two years tourism in Ocean View has increased new businesses have opened there and Ocean Views tax revenues have risen by 30 percent Therefore the best way to improve Hopewells economymdashand generate additional tax revenuesmdashis to build a golf course and resort hotel similar to those in Ocean View

Write a response in which you examine the stated andor unstated assumptions of the argument Be sure to explain how the argument depends on these assumptions and what the implications are for the argument if the assumptions prove unwarranted

GRE AWA John박 박정어학원

Assumptions The author assumes that OVrsquos municipal golf course and resort hotel caused tourism new businesses and increased tax revenues There may be other reasons advertising promo He assumes that this will continueAssumes that Ocean View and Hopewell are similar in many waysmdashthe name suggests otherwise OV may have always been a tourist attractions for its beaches We need to know the topography

2년전 Ocean View 시는 시정 소유 골프 및 휴양지 호텔을 신축했다 그리고 지난 2년동안 이 시의 관광객이 증가했으며 새로운 사업들이 생겨났다 그에따라 시의 세수도 30나 증가했다 Hopewell의 경제를 향상시키고 아울러 세수를 늘릴 수 있는 가장 좋은 방법은 Ocean View에 세워진 것과 같은 골프 시설과 휴양지 호텔을 신축하는 것이다

1 다른 요인으로 관광 산업이 발전했을 수도 있다 문화 유적이 발견이 되었거나 도로의 정비등으로 여행자가 늘었을 수도 있다

2 관광 산업의증가가 늘어난 세수의 원인이 아니라 새로 유입된 인구의 증가나 다른 공장에서 발생한 것일 수 있다

3 2년동안 한참 골프가 붐을 이루었을 수 있다 경제상황이 나빠지거나 다른 레포츠가 인근 지역에 생겨난다면 골프하는 사람이 줄어들 수 있다

In this memo HopeweUs mayor recommends that in order to stimulate the towns economyand boost tax revenues HopeweU should build a new golf course and resort hotel just as thetown of Ocean View did two years ago To support this recommendation the mayor points outthat in Ocean View during the last two years tourism has increased new businesses haveopened and tax revenues have increased by 30 I find the mayors argument unconvincingin several important respectsFirst of all it is possible that the mayor has confused cause with effect respecting the recentdevelopments in Ocean View Perhaps Ocean Views construction of a new golf course andhotel was a response to previous increases in tourism and business development increasesthat have simply continued during the most recent two years Since the mayor has failed toaccount for this possibility the claim that Hopewell would boost its economy by alsoconstructing a golf course and hotel is completely unwarrantedSecondly the mayor fails to account for other possible causes of the trends in Ocean Viewduring the last two years The increase in tourism might have been due to improving economicconditions nationwide or to unusually pleasant weather in the region The new businessesthat have opened in Ocean View might have opened there irrespective of the new golf courseand hotel And the 30 increase in tax revenues might have been the result of an increase intax rates or the addition of a new type of municipal taxWithout ruling out these and other alternative explanations for the three recent trends inOcean View the mayor cannot reasonably infer based on those trends that Hopewellseconomy would benefit by following Ocean Views exampleThirdly even if the recent trends in Ocean View are attributable to the construction of the newgolf course and hotel there the mayor assumes too hastily that the golf course and hotel willcontinue to benefit that towns overall economy The mayor has not accounted for thepossibility that increased tourism will begin to drive residents away during tourist season orthat new business development will result in the towns losing its appeal as a place to visit or tolive Unless the mayor can convince me that these scenarios are unlikely I cannot accept themayors recommendation that Hopewell follow Ocean Views exampleFinally the mayors argument rests on the unsubstantiated assumption that Hopewell andOcean View are sufficiently alike in ways that might affect the economic impact of a new golfcourse and hotel Hopewell might lack the sort of natural environment that would attract moretourists and new businesses to the town--regardless of its new golf course and hotel For thatmatter perhaps Hopewell already contains several resort hotels and golf courses that are notutilized to their capacity If so building yet another golf course and hotel might amount to amisallocation of the towns resources--and actually harm the towns overall economyIn sum the mayors recommendation is not well supported To bolster it the mayor mustprovide better evidence that Ocean Views new golf course and hotel and not some otherphenomenon--has been responsible for boosting Ocean Views economy during the last twoyears To better assess the recommendation I would need to know why Ocean View decidedto construct its new golf course and hotel in the first place--specifically what events prior toconstruction might have prompted that decision I would also need to thoroughly compare

GRE AWA John박 박정어학원

HopeweU with Ocean View--especially in terms of their appeal to tourists and businesses--todetermine whether the same course of action that appears to have boosted Ocean Viewseconomy would also boost Hopewells economy

The following is part of a memorandum from the president of Humana University

Last year the number of students who enrolled in online degree programs offered by nearby Omni University increased by 50 percent During the same year Omni showed a significant decrease from prior years in expenditures for dormitory and classroom space most likely because instruction in the online programs takes place via the Internet In contrast over the past three years enrollment at Humana University has failed to grow and the cost of maintaining buildings has increased along with our budget deficit To address these problems Humana University will begin immediately to create and actively promote online degree programs like those at Omni We predict that instituting these online degree programs will help Humana both increase its total enrollment and solve its budget problems

Write a response in which you discuss what questions would need to be answered in order to decide whether the prediction and the argument on which it is based are reasonable Be sure to explain how the answers to these questions would help to evaluate the prediction

Is Omni University successful due to the online degree program 50 Is the decrease in expenditures for dormitory and classroom space due to the decrease in of on-campus students Which classes were successful Does HU have those classes

Even if the long-distance degree programs at Omni University benefited the school the presidentrsquos recommendation that Human College should emulate Omni University is too hasty First OUrsquos name implies that the school would have more majors than Humanahellip the president should examine which degrees were in the long-distance programhellip

지난해에는 Omni 대학에서 개강했던 원거리 학생 학점 취득 프로그램을 등록했던 학생들의 숫자가 50나 증가했다 같은해 기간동안 Omni 대학에서는 그 전년도부터 기숙사와 학급의 공간 확충을 위한 예산을 대폭 줄였는데 이는 이 원거리 학점 취득 프로그램이 양방향 비디오 컴퓨터 접속을 통해서만 가능한 수업지도 방식이기때문인 것으로 보인다 반면 지난 3개년 동안 Humana 대학에서의 수강률은 감소한데다가 건물

GRE AWA John박 박정어학원

유지비도 올랐다 따라서 Humana대학의 수강을 늘리고 예산손실을 회복하기 위해서는 Omni 대학에서 취한 조치와 같은 능동적인 프로그램을 추진해야 한다

결론 we should initiate and actively promote long-distance degree programs like those at Omni 반박 원거리 학생 취득 프로그램 숫자가 증가한거하고 예산이 줄어드는 것 사이에 연관이 약하다 (causal 학생의 증가로 관리비용 증가할수 있음 원거리 수업가능 장비도입에의한 비용발생)bad analogy(omni university 하고 같은 조건이 아니다 )-gt omni college 가 강좌내용이 좋아서 학생의 등록이 많을수 있다 Humana 대학에서 만들었다 하더라도 인기 없을수 있음다른 요인에 의해서 Humana 대학의 수강 인원이 증가할수 있음(비록 과거엔 인기가 없었을지라도)

The following appeared as part of a business plan developed by the manager of the Rialto Movie Theater

Despite its downtown location the Rialto Movie Theater a local institution for five decades must make big changes or close its doors forever It should follow the example of the new Apex Theater in the mall outside of town When the Apex opened last year it featured a video arcade plush carpeting and seats and a state-of-the-art sound system Furthermore in a recent survey over 85 percent of respondents reported that the high price of newly released movies prevents them from going to the movies more than five times per year Thus if the Rialto intends to hold on to its share of a decreasing pool of moviegoers it must offer the same features as Apex

Write a response in which you discuss what questions would need to be answered in order to decide whether the recommendation is likely to have the predicted result Be sure to explain how the answers to these questions would help to evaluate the recommendation

Before following through this business plan the manager should investigate the cause of Rialtorsquos unsuccessful business

The author provides no evidence that the surveyrsquos results are statistically reliable The surveyrsquos sample of 85 percent must be sufficient in size and representative of overall population of the city where Rialto and Apex is serving Lacking evidence of a sufficiently representative sample the author cannot justifiably rely on the survey to draw any conclusion whatsoever The author does not indicate that Apex is indeed currently successful However even if Apex is enjoying success the argument relies on what might be a false analogy between Rialto and Apex In order for Apex to serve as a model that Rialto should emulate the author must assume that all relevant circumstances are essentially the same However this assumption is unwarranted For example the argument overlooks the face that Apex is located in a strategic placemdashbeside a mall where customers can not only watch a movie but also enjoy shopping Therefore simply changing the facility to that of Apex may not lead to success

The author does not mention whether Apex is successful or not Nevertheless even if Apex is currently successful the argument relies on what might be a false analogy between Rialto and Apex In order for Apex to serve as a model that Rialto should emulate the author must assume that all relevant circumstances are essentially the same However this assumption is unwarranted For example the argument overlooks the fact that these two institutions are located in different locations Rialto in downtown and Apex in a mall outside of town Although Apex opened with state-of-the-art facilities the decisive factor in its success could be due to its strategic location of being in a mall People could enjoy both shopping and movies at one location thus they may prefer Apex over Rialto Furthermore the place where people enjoy leisure activities has shifted in the past decades for most cities from downtown to the suburbs Therefore Rialto may not be successful even if it emulates Apexrsquos facilities A better business plan may be relocating Apex to the thriving section of the downtown

Rialto 극장은 지난 50여년간 지역 회관으로써 시내에 위치해 있으면서도 이제 변화를 꾀하지 않으면 문을 닫을

GRE AWA John박 박정어학원

판이다 이 극장은 시외 쇼핑타운에 새로 들어선 Apex 극장의 사례를 본받아야 했다 Apex가 지난해 개업했을 당시 이 극장은 비디오 아케이드 플러쉬 카펫트 바닥과 좌석 그리고 최신 음향시설을 갖추었다 더군다나 최근 조사에서는 응답자의 85 이상이 새로 출시된 영화 입장료가 비싼 탓으로 지난해보다 5배이상의 관람객이 줄어들었다고 나타났다 따라서 Rialto 극장이 줄어들고 있는 관람객을 뺐기지 않고 유지하려면 Apex와 같은 시설들을 갖추어야 할 것이다주장 리알토 극장이 줄어들고 있는 관람객을 뺐기지 않고 유지하려면 Apex와 같은 시설들을 갖추어야 할 것이다

1 조사에서 응답자가 전체를 대표할 수 없다 2 apex 극장이 좋은 시설을 갖추고 있지만 그로 인해 수익이 많이 발생했다는 말이 없으므로 시설투자를

하고도 좋은 결과를 얻을 수 있을지 그 근거가 미흡하다3 좋은 영화가 출시된다면 입장료가 비싸도 영화관에서 꼭 보려고 할 수 있다 4 rialto 가 시설이 아닌 다른 요인에 의해 장사가 안될수도 있다( 우범 지역이라든지)

The following is a recommendation from the business manager of Monarch Books

Since its opening in Collegeville twenty years ago Monarch Books has developed a large customer base due to its reader-friendly atmosphere and wide selection of books on all subjects Last month Book and Bean a combination bookstore and coffee shop announced its intention to open a Collegeville store Monarch Books should open its own in-store cafeacute in the space currently devoted to childrens books Given recent national census data indicating a significant decline in the percentage of the population under age ten sales of childrens books are likely to decline By replacing its childrens books section with a cafeacute Monarch Books can increase profits and ward off competition from Book and Bean

Write a response in which you examine the stated andor unstated assumptions of the argument Be sure to explain how the argument depends on these assumptions and what the implications are for the argument if the assumptions prove unwarranted

The following is a recommendation from the business manager of Monarch Books

Since its opening in Collegeville twenty years ago Monarch Books has developed a large customer base due to its reader-friendly atmosphere and wide selection of books on all subjects Last month Book and Bean a combination bookstore and coffee shop announced its intention to open a Collegeville store Monarch Books should open its own in-store cafeacute in the space currently devoted to childrens books Given recent national census data indicating a significant decline in the percentage of the population under age ten sales of childrens books are likely to decline By replacing its childrens books section with a cafeacute Monarch Books can increase profits and ward off competition from Book and Bean

1 Write a response in which you discuss what questions would need to be answered in order to decide whether the recommendation is likely to have the predicted result Be sure to explain how the answers to these questions would help to evaluate the recommendation

2 Write a response in which you discuss what specific evidence is needed to evaluate the argument and explain how the evidence would weaken or strengthen the argument

No evidence regarding Monarch Bookrsquos successEven if Regal Bookrsquos is successful this may not be attributable to the cafeacute False analogy Emulating may not lead to success Other factors may be involvedInsufficient condition The national census is not enough evidence that childrenrsquos book sales will decline Can

GRE AWA John박 박정어학원

the national census represent the local child populationDid opening a cafeacute boost sales for Regal Books Even assuming Regal is successful by opening a cafeacute this may not be suitable for Monarch which plans to close the childrenrsquos book section to establish a cafe Imprecise language ldquorelatively little spacerdquo how smallThe managerrsquos recommendation contradicts what he says Since Monarch is popular for its wide selection of books closing a selection which targets a major group of readers may hurt Monarchrsquos salesIs this the best way to compete

When Stanley Park first opened it was the largest most heavily used public park in town It is still the largest park but it is no longer heavily used Video cameras mounted in the parks parking lots last month revealed the parks drop in popularity the recordings showed an average of only 50 cars per day In contrast tiny Carlton Park in the heart of the business district is visited by more than 150 people on a typical weekday An obvious difference is that Carlton Park unlike Stanley Park provides ample seating Thus if Stanley Park is ever to be as popular with our citizens as Carlton Park the town will obviously need to provide more benches thereby converting some of the unused open areas into spaces suitable for socializing

Write a response in which you examine the stated andor unstated assumptions of the argument Be sure to explain how the argument depends on these assumptions and what the implications are for the argument if the assumptions prove unwarranted

Stanley 파크가 처음 개장했을 당시 가장 크고 가장 많이 이용되는 공원이었다 아직도 공원중에서는 가장 크지만 이용률은 상당히 떨어졌다 지난달 공원 주차장에 설치해놓은 비디오 카메라를 통해 보면 drop(주차장으로 여겨짐) 이용률이 가장 높았다 수치상으로는 하루 평균 50대의 차량만이 이용하였다 반면 직장 중심거리에 위치한 작은 규모의 Carlton 파크는 주당 무려 150여명 이상이 이용하고 있다 Stanley 파크와는 달리 Carlton 파크에는 의자가 있다는 것이 가장 뚜렷한 차이점이다 따라서 Stanley 파크가 Carlton 파크처럼 시민들이 자주 이용하는 공원이 되기 위해서는 벤치를 설치할 필요가 있으며 이렇게 사용되지 않는 일부 공간을 활용해서 사교를 위한 공간으로 바꾸어야 한다 ===gtdrop 에 대한 첨부사항 (영영사전내용입니다)---- a place or central depository to which something (as mail money or stolen property) is brought for distribution or transmission also the act of depositing something at such a place dropgt

주장 if Stanley Park is ever to be as popular with our citizens as is Carlton Park the town will obviously need to provide more benches thereby converting some of the unused open areas into spaces suitable for socializing1 조사가 언제 이루어진 것인가 조사가 언제 실시되었느냐에 따라 결과가 다를 수있다 현재는 다시 스탠리 파크가 늘어났었을 수 있다 2 벤치를 많이 설치했다고 해서 많은 관광객이 오지 않을수 있다(사람들이 벤치나 사교 공간을 원한다는 어떠한 자료도 없다)3스탠리 파크 주변에 교통 상황이 악화가 되었거나 칼튼 파크에서 문화행사등을 많이 가져서 이용객이 줄어든것일 수도 있다 4 칼튼 파크가 중심지에 있어서 접근성이 좋을수 있다5 조사가 같은 시간을 기준으로 한게 아니다(하나는 주중이고 하나는 주말이다)6사람의 수와 차의 대수를 같은것으로 비교할수 없다 (차안에 몇 명이 타고 있는지 모르고 대중교통을 이용해서 왔을수도 있다)

Page 21: GRE writing argument brain storm

GRE AWA John박 박정어학원

The following appeared in a memo from a vice president of Quiot Manufacturing

During the past year Quiot Manufacturing had 30 percent more on-the-job accidents than at the nearby Panoply Industries plant where the work shifts are one hour shorter than ours Experts say that significant contributing factors in many on-the-job accidents are fatigue and sleep deprivation among workers Therefore to reduce the number of on-the-job accidents at Quiot and thereby increase productivity we should shorten each of our three work shifts by one hour so that employees will get adequate amounts of sleep

Write a response in which you examine the stated andor unstated assumptions of the argument Be sure to explain how the argument depends on these assumptions and what the implications are for the argument if the assumptions prove unwarranted

The following appeared in a memo from a vice president of Alta Manufacturing

During the past year Alta Manufacturing had thirty percent more on-the-job accidents than nearby Panoply Industries where the work shifts are one hour shorter than ours Experts believe that a significant contributing factor in many accidents is fatigue caused by sleep deprivation among workers Therefore to reduce the number of on-the-job accidents at Alta we recommend shortening each of our three work shifts by one hour If we do this our employees will get adequate amounts of sleep

Write a response in which you discuss what questions would need to be answered in order to decide whether the recommendation and the argument on which it is based are reasonable Be sure to explain how the answers to these questions would help to evaluate the recommendation

The following appeared in a memo from the vice president of Butler Manufacturing

During the past year workers at Butler Manufacturing reported 30 percent more on-the-job accidents than workers at nearby Panoply Industries where the work shifts are one hour shorter than ours A recent government study reports that fatigue and sleep deprivation among workers are significant contributing factors in many on-the-job accidents If we shorten each of our work shifts by one hour we can improve Butler Manufacturings safety record by ensuring that our employees are adequately rested

1 Write a response in which you discuss what specific evidence is needed to evaluate the argument and explain how the evidence would weaken or strengthen the argument

2 Write a response in which you discuss what questions would need to be answered in order to decide whether the recommendation is likely to have the predicted result Be sure to explain how the answers to these questions would help to evaluate the recommendation

4번 반복됨

Alta has 30 more job accidents than Panoply(work shifts one hour shorter)Experts Job accidents caused by fatigue and sleep deprivationTherefore to reduce job accidents and increase productivity shorten three work shifts by one hour for adequate sleep

지난해 우리 회사는 인근 Panoply Industries보다 업무상 재해가 30나 더 많았다 그 회사는 우리보다 근무 교대시간이 1시간 정도 짧았다 전문가들은 대부분의 업무상 재해에 있어서 가장 중요한 요인이 과로와 수면부족으로 보고있다 따라서 우리 회사에서 높은 산업재해를 줄이고 아울러 생산성을 높이기 위해서는 근로자들이 충분한 수면을 취할 수 있도록 1시간씩 3교대 시간을 줄여야 한다

In this memo the (author) vice president of Alta Manufacturing (AM) recommends that to reduce on-the-job accidents and increase productivity AM should shorten its three work shifts by one hour so that employees can

GRE AWA John박 박정어학원

get more sleep To support this recommendation the author provides several evidences However careful scrutiny of each of the facts reveals that it provides little credible support for the authorrsquos recommendation QuestionsThe number of accidents What kind of accidents The seriousness of the accidents is importantHow many employees are in each company What are their productsFalse cause Sleep may not be the reason for the on-the-job accidents What do Alta and Panoply manufacture

First of all the author believes that fatigue caused the on-the job accidents However there could be other reasons The author observes a correlation between sleep deprivation and on-the-job accidents then concludes that the former is the cause of the latter However the author fails to rule out other possible explanations For example it is entirely possible that Alta factories require more strenuous and dangerous labor than Panoply Without ruling out all other such factors it is unfair to conclude that fatigue is responsible for the accidents In addition the work-shifts may not be the cause of the sleep deprivation and fatigue It is possiblehellip Thus the author should provide what exactly Panoply and Alta manufacture and more precise data about their working conditions to be more convincing

Shortening the shift by one hour does not necessarily lead to more sleep And is one hour enoughLess accidents does not mean increased productivity

결론 We should shorten each of out three work shifts by one hour

반박 경쟁사에 비해서 시간당 하는 업무량이 많아서 더욱 피곤할 수도 있다 시간이 문제가 아니라 노후된 시설 설비 자체의 문제 작업 자체가 원래 위험한 것이여서 사고가 많을 수도 있다 다른 회사는 더욱 많은 작업시간에도 불구하고 안정한 작업여건으로 인해서 사고율이 오히려 더 작을 수도 있다비교사의 재해감소가 다른 요인일수 있다(안전 교육 철저)줄인 시간이 피로회복이나 수면으로 연결 안될수 있음(술을 마실 수도 있고 그 시간에 휴식을 취하지 않고 다른일을 함으로써 더욱 피로해질수 있다)

This editorial recommends that Alta Manufacturing reduce its work shifts by one hour each inorder to reduce its on-the-job accident rate and thereby increase Altas productivity To supportthis recommendation the author points out that last year the number of accidents at Alta was30 greater than at Panoply Industries where work shifts were one hour shorter The authoralso cites certain experts who believe that many on-the-job accidents are caused by fatigueand sleep deprivation I find this the argument unconvincing for several reasonsFirst and foremost the author provides absolutely no evidence that overall workerproductivity is attributable in part to the number of on-the-job accidents Although commonsense informs me that such a relationship exists the author must provide some evidence ofthis cause-and-effect relationship before I can accept the authors final conclusion that theproposed course of action would in fact increase Altas productivitySecondly the author assumes that some accidents at Alta are caused by fatigue or sleepdeprivation However the author overlooks other possible causes such as inadequateequipment maintenance or worker training or the inherent hazards of Altas manufacturingprocesses By the same token Panoplys comparatively low accident rate might be attributablenot to the length of its work shifts but rather to other factors such as superior equipmentmaintenance or worker training In other words without ruling out alternative causes ofon-the-job accidents at both companies the author cannot justifmbly conclude that merely byemulating Panoplys work-shift policy Alta would reduce the number of such accidentsThirdly even assuming that Altas workers are fatigued or sleep-deprived and that this is thecause of some of Altas on-the-job accidents in order to accept the authors solution to thisproblem we must assume that Altas workers would use the additional hour of free time tosleep or rest However the author provides no evidence that they would use the time in thismanner It is entirely possible that Altas workers would use that extra hour to engage in someother fatiguing activity Without ruling out this possibility the author cannot convincinglyconclude that reducing Altas work shifts by one hour would reduce Altas accident rateFinally a series of problems with the argument arise from the scant statistical information onwhich it relies In comparing the number of accidents at Alta and Panoply the author fails toconsider that the per-worker accident rate might reveal that Alta is actually safer than Panoplydepending on the total number of workers at each company Second perhaps accident rates

GRE AWA John박 박정어학원

at the two companies last year were aberrations and during other years Altas accident ratewas no greater or even lower than Panoplys rate Or perhaps Panoply is not representativeof industrial companies generally and that other companies with shorter work shifts have evenhigher accident rates In short since the argument relies on very limited statistical information Icannot take the authors recommendation seriouslyIn conclusion the recommendation for emulating Panoplys work-shift policy is not wellsupported To convince me that shorter work shifts would reduce Altas on-the-job accidentrate the author must provide clear evidence that work-shift length is responsible for some ofAltas accidents The author must also supply evidence to support her final conclusion that alower accident rate would in fact increase overall worker productivity

The following appeared in a memo from the vice president of marketing at Dura-Sock Inc

A recent study of our customers suggests that our company is wasting the money it spends on its patented Endure manufacturing process which ensures that our socks are strong enough to last for two years We have always advertised our use of the Endure process but the new study shows that despite our socks durability our average customer actually purchases new Dura-Socks every three months Furthermore our customers surveyed in our largest market northeastern United States cities say that they most value Dura-Socks stylish appearance and availability in many colors These findings suggest that we can increase our profits by discontinuing use of the Endure manufacturing process

1 Write a response in which you examine the stated andor unstated assumptions of the argument Be sure to explain how the argument depends on these assumptions and what the implications are for the argument if the assumptions prove unwarranted

2 Write a response in which you discuss what specific evidence is needed to evaluate the argument and explain how the evidence would weaken or strengthen the argument

3 Write a response in which you discuss what questions would need to be answered in order to decide whether the recommendation and the argument on which it is based are reasonable Be sure to explain how the answers to these questions would help to evaluate the recommendation

Intro The vice president of marketing at Dura-Sock Inc is offering a potentially harmful investment recommendation by claiming that Dura-Sock should discontinue its use of the ldquoEndurerdquo process To support his recommendation he points out a study that Dura-Sock customers actually purchase the socks every three months and a survey that reveals that Dura-Sock customers like the sockrsquos stylish appearance and availability in many colors The study and survey however are insufficient in supporting his proposal and the VP makes several unwarranted assumptionsIntro (simplified) The VP states that though Dura-Socks last for two years customers buy the socks every three months Therefore he assumes that the consumersrsquo motive for buying the produce is not its durabilityHowever the author fails to rule out other possible motivation for consumption

Even if the survey is reliable the author should consider the rest of the market Vague terms ldquowasting moneyrdquomdashprecisely how much are they wasting Studysurvey errorThe company must calculate the outcome of such momentous decisionStudy participantsrsquo comment that they prefer Dura-Sock for its stylishness and availability might take Dura-Sockrsquos enduring quality for granted

우리회사 제품 소비자들에 대한 최근 조사에서 지난 2년여간 양말의 내구성을 강하게 하는 필수공정이었던 자사 특허의 Endure 공정에 들어가는 비용이 낭비라고 말하고 있다 우리 회사는 항상 이 공정 처리에 대한 광고를 내보냈으나 이에 대한 시장 조사에서 실제로 고객들은 이 신제품을 평균 석달마다 구매하는 것으로 나타났다 더군다나 북동부지역에서 실시한 대규모 시장조사에 응답한 고객들은 양말의 모양과 색상등에 더

GRE AWA John박 박정어학원

관심을 나타냈다 이러한 결과는 우리회사가 신기술 공법을 중단하면 그에 따라 수익이 늘어날 것이라는 것을 말해주고 있는 것이다주장 These findings suggest that Dura0Sock can increase its profits by discontinuing its use of the ldquoEndurerdquo manufacturing process

1 survey가 정확한 소비자의 의견을 나타낸 것인가 다른 선택없이 양자택일과 같은 방법의 survey였는지2 북동부지역의 시장조사가 전체 의견을 대표할 수 있나3 사람들이 모양이나 색상에 앞서 내구성을 먼저 평가했을 수도 있다 내구성을 갖추었다는 전제하에 모양과 색상에 관심을 드러낸 것일 수 있다4 소비자가 도매상(retail)인지 소매상(whole)인지가 없다

The following appeared in a business magazine

As a result of numerous complaints of dizziness and nausea on the part of consumers of Promofoods tuna the company requested that eight million cans of its tuna be returned for testing Promofoods concluded that the canned tuna did not after all pose a health risk This conclusion is based on tests performed on samples of the recalled cans by chemists from Promofoods the chemists found that of the eight food chemicals most commonly blamed for causing symptoms of dizziness and nausea five were not found in any of the tested cans The chemists did find small amounts of the three remaining suspected chemicals but pointed out that these occur naturally in all canned foods

Write a response in which you discuss what questions would need to be addressed in order to decide whether the conclusion and the argument on which it is based are reasonable Be sure to explain how the answers to the questions would help to evaluate the conclusion

Representativeness of the tested cansThey should conduct a comparative studyThe testing could be biased because Promofoods employees conducted the testingHow much (quantity) of the five and three suspected chemicals were in the canned foodsFalse cause The substance that caused dizziness and nausea may not be one of the eight common chemicals

많은 소비자들의 현기증과 구역질 불만에 따라 Promofoods사는 지난해 참치 캔 8백만 개를 테스트하기 위해 반품시켰다 그 결과 캔에서는 건강에 위험이 될 수 있는 화합물질이 없었던 것으로 회사측은 결론지었다 이러한 결론은 회사측 화학연구자들이 회수된 캔의 샘플을 테스트해서 이들 증상의 원인이 되는 8가지 화합물 중에서 5가지가 실험된 캔에서 발견되지 않았다는 사실에 근거한 것이다 이들 화학자들은 나머지 3개가지 화합물이 모든 캔 식료품에서 흔히 발견되는 것이라고 언급했다 결론 Promofoods concluded that the cans did not after all contain chemicals that posed a health risk

1 공인된 기간에서 테스트를 한 것이 아니고 자사에서 직접 테스트를 했기에 신뢰성이 안간다 2 이런 증상을 일으키는 8개의 물질 말고 다른 물질들이 캔속에 많이 포함됬을수 있다 3 나머지 3개의 물질들의 함유량이 많아서 다른 종류의 캔들은 문제를 일으키지 않지만 참치캔은 문제를

일으킬 수 있다

This magazine article concludes that the 8 million cans of tuna Promofoods recalled due tocomplaints about nausea and dizziness do not after ail contain any chemicals that pose a

GRE AWA John박 박정어학원

health risk To support this conclusion the author cites the fact that five of eight chemicalscommonly causing these symptoms were not found in the recalled cans while the other threealso occur naturally in other canned foods For several reasons this evidence lends littlecredible support to the authors conclusionTo begin with the author relies partly on the fact that although three of the eight chemicalsmost commonly blamed for nausea and dizziness appeared in Promofoods recalled tunathese chemicals also occur naturally in other canned foods However this fact alone lends nosupport to the authors conclusion for two reasons First the author might be ignoring animportant distinction between naturally occurring chemicals and those not occurring naturallyIt is entirely possible that these three chemicals do not occur naturally in Promofoods tunaand that it is for this reason that the chemicals cause nausea and dizziness Secondly it isentirely possible that even when they occur naturally these chemicals cause the samesymptoms Unless the author rules out both possibilities he cannot reliably conclude that therecalled tuna would not cause these symptomsAnother problem with the argument is that the authors conclusion is too broad Based onevidence about certain chemicals that might cause two particular heath-related symptoms theauthor concludes that the recalled tuna contains no chemicals that pose a health risk Howeverthe author fails to account for the myriad of other possible health risks that the recalled tunamight potentially pose Without ruling out all other such risks the author cannot justifiablyreach his conclusionA third problem with the argument involves that fact that the eight particular chemicals withwhich the test was concerned are only the eight most commonly blamed for nausea anddizziness It is entirely possibly that other chemicals might also cause these symptoms andthat one or more of these other chemicals actually caused the symptoms Without ruling outthis possibility the author cannot jusufiably conclude that the recalled tuna would not causenausea and dizzinessA final problem with the argument involves thetesting procedure itself The author providesno information about the number of recaUed cans tested or the selection method used Unlessthe number of cans is a sufficiently large sample and is statistically repre sentative of all therecalled cans the studys results are not statistically reliableIn conclusion the article is unconvincing as it stands To strengthen the assertion that therecalled tuna would not cause nausea and dizziness the author must provide evidence thatthe three chemicals mentioned that occur naturally in other canned foods also appear naturallyin Promofoods tuna The author must also provide evidence that ingesting other canned foodscontaining these three chemicals does not cause these symptoms To better evaluate theargument we would need to know whether the sample used in the tests was statisticallysignificant and representative of all the recalled tuna We would also need to know what otherchemicals in the recalled tuna might pose any health risk at all

5그룹 불충분 조건오류 빈출

Natures Way a chain of stores selling health food and other health-related products is opening its next franchise in the town of Plainsville The store should prove to be very successful Natures Way franchises tend to be most profitable in areas where residents lead healthy lives and clearly Plainsville is such an area Plainsville merchants report that sales of running shoes and exercise clothing are at all-time highs The local health club has more members than ever and the weight training and aerobics classes are always full Finally Plainsvilles schoolchildren represent a new generation of potential customers these schoolchildren are required to participate in a fitness-for-life program which emphasizes the benefits of regular exercise at an early age

Write a response in which you examine the stated andor unstated assumptions of the argument Be sure to

GRE AWA John박 박정어학원

explain how the argument depends on these assumptions and what the implications are for the argument if the assumptions prove unwarranted

False cause

First of all the author believes that the Increased sales of running shoes and exercise clothing indicates

Plainesville residentsrsquo interest in leading healthy lives However this assumption is not logically convincing for

several reasons could be a fashion trendTime shift ldquoFitness for liferdquo might not have any influence on schoolchildren as they growFalse cause There could be other reasons for member increase in the health clubAll of the above are insufficient condition

The author has to prove that local residents are interested in leading healthy lives However he supports his conclusion with insufficient evidence Nevertheless even if the residents are concerned with health naturersquos way may not be successful First

그 동안의 경험을 토대로 볼 때 건강생활과 밀접히 관련되어 있는 거주 지역에서 본 상점들이 아주 호응을 얻고 있다 따라서 이러한 주민들이 많이 거주하고 있는 Plainsville 에 새로운 상점들을 계속 세워야 한다 이 지역 상인들은 런닝화와 운동복 판매가 가장 높다고 말한다 불과 5 년전에는 거의 전무하다시피하던 지역 헬스 클럽의 경우도 엄청나게 많은 회원을 확보하고 있으며 웨이트 트레이닝과 에어로빅 강좌들도 항상 만원이라고 한다 새로운 고객층을 예측해 보는 것도 가능하다 이 지역의 학생들의 경우 Fitness for Life프로그램을 받게 되는데 이러한 프로그램을 통해서 유년시절부터 정규적인 운동 습관을 들이게 하고 있는 것이 그것이다

결론 We should therefore build our next new store in Plainsville

반박 그동안의 경험에 의한 과거 통계가 꼭 여기에도 적용되는건 아니다 5 년전 헬스 클럽이 잘 안되었던게 다른 원인이였을수 있다(강사수준 미달 강좌미비)tourist 에 의한 원인 일수 있다 어렸을때부터 운동을 했다고 해서 커서도 관심이 있지는 않다 (오히려 반감이 있을수 있다 혹은 건강하기에 건강에 관심이 적을수도 있다)운동복이나 신발의 판매가 육체노동에 의한 것일수도 있다

IntroductionSupport1049896In this memorandum the author asserts that Naturersquos Way should build its next newstore in Plainsville To support this assertion the author states that Plainsvillesmerchantsrsquo sales of exercise clothing are going well the local health club has moremembers than ever and a new generation of customers will help to ensure NaturersquosWayrsquos success At first glance the authorrsquos assumption seems convincing but in-depth scrutiny revealsthat it lacks substantial evidence as it stands

Body 1-SamplingTopic Sentence 1To begin with the author assumes that the merchantsrsquo report indicates that the residentsare concerned about their health However this assumption is based on unsubstantiated

GRE AWA John박 박정어학원

data Example 1 (Rebuttal1) First if we do not know the total volume of items sold and the price of the goods exactly we cannot infer whether the residents are actually buying many goods Example 2 (Rebuttal2)In addition to that the report emphasizes the rising sales of running shoes and exerciseclothing however these may not be hot-selling items for Naturersquos Way or may not be theproducts the company is planning to sell Concluding Sentence Therefore in order to make the argument reliable the author should reconsider themerchantsrsquo report with more detailed data

Body 2-CausalTopic Sentence 2Second the author contends that the health clubs classes are full yet this does not meanthat many people actually use the health club other factors may be the real cause forthose closed classes Example 1 (Rebuttal1) To begin with if the health club is very small the number of people working out wouldnot be a large one In fact regular gym-going may just be a vogue among a smallunrepresentative segment of Plainsvilles population Example 2 (Rebuttal2) Moreover it is possible that most of the people who exercise in the health club do weight training and aerobics only to look good and to meet other singles not for their health In that case there would be little demand for health products Concluding SentenceThus the author should not hasten to presume what really caused people to be interested in a healthier lifestyle and enroll in the health club

Body 3-Time-ShiftTopic Sentence 3Finally the author highlights that Naturersquos Way can expect a new generation of customersin Plainsville that will help the company in the long term This notion is mistaken in that itassumes the conditions of the present will continue unchanged in the future Although theschool children are required to participate in the fitness for life program they may notnecessarily buy Naturersquos Ways products Example 1 (Rebuttal1) In the first instance they may suffer a fall in purchasing power arising from future economic difficulties this would cause reluctance to spend a considerable amount of money on health products which tend to be more expensive Example 2 (Rebuttal2)Another possibility is that there may emerge many competitor companies vying with Naturersquos Way so that in the future the school children may not feel the necessity to purchase one companyrsquos health products over anotherrsquosConcluding Sentence Thus the authorrsquos assumption is highly speculative since it relies heavily on unknowablefuture circumstances

ConclusionThesis In sum the author uses many assumptions that are insufficient in supporting his claimsSupportIn order for the authorrsquos claims to be convincing he needs to advance more persuasiveevidence that people in Plainsville really are concerned with their health and health foodThe following was written as a part of an application for a small-business loan by a group of developers in the city of Monroe

A jazz music club in Monroe would be a tremendously profitable enterprise Currently the nearest jazz club is 65 miles away thus the proposed new jazz club in Monroe the C-Note would have the local market all to itself Plus jazz is extremely popular in Monroe over 100000 people attended Monroes annual jazz festival last summer several well-known jazz musicians live in Monroe and the highest-rated radio program in Monroe is Jazz Nightly which airs every weeknight at 7 PM Finally a nationwide study indicates that the typical jazz fan spends close to $1000 per year on jazz entertainment

1 Write a response in which you discuss what specific evidence is needed to evaluate the argument and explain how the evidence would weaken or strengthen the argument

2 Write a response in which you examine the stated andor unstated assumptions of the argument Be

GRE AWA John박 박정어학원

sure to explain how the argument depends on these assumptions and what the implications are for the argument if the assumptions prove unwarranted

3 Write a response in which you discuss what questions would need to be answered in order to decide whether the prediction and the argument on which it is based are reasonable Be sure to explain how the answers to these questions would help to evaluate the prediction

Group error nationwide survey may not reflect local trends Is the nationwide jazz fan population substantialInsufficient non-residents of Monroe may have attended the jazz festival (Body alternative explanation last year may have been an anomaly The author should consider data from various years) The author should indicate how many out of 100000 were Monroe residentsNationwide study Does this reflect Insufficient Citizens of Monroe may continue to go to the jazz club 65 miles away

Are the people in Monroe really interested in jazzMajority of the people who attended the jazz festival might not be Monroe residentsSurvey error nationwide study may not be applicable to MonroeJazz musicians who live in MonroeMonopolyRadio station

In this business application the author claims that the proposed jazz club C Note will be very profitable in Monroe To support this claim the author argues for his case with several evidences At first glance the authorrsquos argument seems convincing however careful scrutiny reveals that his argument in specious

To begin with the author claims that Monroersquos citizens are interested in jazz He presents three evidences First Secondhellip Thirdhellip Howeverhellip

Monroe 시에 있는 재즈 음악 클럽은 수익성이 좋은 사업이다 현재 가장 가까이에 있는 클럽은 65 마일 정도 떨어져 있다 따라서 이번에 세우려고 하는 C Note 는 독보적인 위치를 점할것이다 더군다나 재즈는 이 시에서 가장 인기있는 음악이다 지난 여름 재즈 축제에서는 10 만명 이상의 Morone 시 주민이 참석하였고 몇몇 유명한 재즈 음악가들도 이곳에 살고 있으며 저녁때 방영되는 라디오 프로그램중에서 최고의 시청률을 보이고 있는 것도 Jazz Nightly 이다 전국조사에서도 전형적인 재즈 팬들은 재즈 분야에 년간 1천 달러 가까이 지출하고 있는 것으로 보고되고 있다 따라서 C Note 클럽이 돈을 벌 수 있는 사업이라는 것은 확실한 것이다

결과 It is clear that the C Note cannot help but make money반박 nearest jazz club 이 양질의 써비스로 여전히 손님을 끌수도 있다Festival 에 얼마나 참여하는지가 jazz 의 인기를 반영하지 않는다 뮤지션이 많이 사는거랑 jazz 의 인기가 상관없다라디오 프로그램이 다른 요인에 의해서 인기일수도 있다 (진행자때문)전국 통계 적용 불가화목 실전반_Ms Noh6In this application the author suggests that a jazz club in Monroe will make a number of profits To support this suggestion the author exemplifies the local condition popularity of jazz in Monroe and nationwide study However careful scrutiny of each of the facts reveals that it provides little credible support for the authorrsquos recommendation Good clear intro

First the author assumes that jazz is popular in Monroe because of several facts the jazz festival last year had high participation some famous jazz musicians live in Monroe and the high-rated radio program is lsquoJazz Nightlyrsquo However this assumption has many drawbacks that must be seriously considered(Good topic sentences) If many attendants in the last-yearrsquos festival came from other cities and not Monroe it is hard to conclude that Monroersquos people like jazz Therefore the author must examine how many Monroe residents actually attended the festival On top of that there is little relationship between habitation of famous jazz musician and the popularity of jazz in Monroe Although several well-known musicians live there if they do not take part in any jazz performance of Monroe this might have no effect to the interest of Monroersquos residents

GRE AWA John박 박정어학원

about jazz Finally in the case of radio program this is also not suitable reason why jazz is popular in Monroe It might be possible that people cannot help choosing lsquoJazz Nightlyrsquo because there are few radio programs at Night The fact that the radio program is the highest rating program is not a germane evidence The approximate number of listeners would be the more crucial evidence Therefore the author needs to seriously deliberate the correlation between jazzrsquos popularity in Monroe and his examples (Good logical flow and clarity)

Second the author uses as evidence the nationwide study that jazz fans spend much money on jazz entertainment to substantiate why starting a jazz club in Monroe will be profitable In other words the author assumes that the characteristics of a nationwide study can be applied to Monroe The national study would lend support to the applicantrsquos claim only if residents in Monroe typify national jazz fans However the author does not provide credible evidence that this is the case Moreover the populations of jazz fans nationwide may be insubstantial Thus the author should not infer hastily that Monroersquos residents will spend much money on enjoying jazz from the nationwide study

Lastly even if jazz is popular in Monroe C Note may not be successful It is entirely possible that residents might still prefer other clubs where they have always went In addition there is another possibility that the nearest jazz club will attract many of Monroersquos people because it serves fine performances and is equipped with favorite facilities Without considering these other possibilities the author cannot make his argument convincing In sum the author presents many reasons that are insufficient in supporting his or her claim In order for the authorrsquos claims to be convincing he needs to advance more persuasive evidence such as the total number of Monroe residents who attended the jazz festival the effects on the popularity of jazz by the musicians living in Monroe and the actual number of residents who would typify themselves to be jazz fans through a local survey Without substantial evidence that C Note will be successful in Monroe the businessmen may be overinvesting in what might lead to a business failureExcellent clarity Score 50

The following appeared in a newsletter offering advice to investors

Over 80 percent of the respondents to a recent survey indicated a desire to reduce their intake of foods containing fats and cholesterol and today low-fat products abound in many food stores Since many of the food products currently marketed by Old Dairy Industries are high in fat and cholesterol the companys sales are likely to diminish greatly and company profits will no doubt decrease We therefore advise Old Dairy stockholders to sell their shares and other investors not to purchase stock in this company

Write a response in which you discuss what questions would need to be answered in order to decide whether the advice and the argument on which it is based are reasonable Be sure to explain how the answers to these questions would help to evaluate the advice

Survey 80

GRE AWA John박 박정어학원

Old Dairy could change their products and manufacture low fat dairy foodsLess competing companies Old Dairy could eventually be the only company that produces hellipImprecise numbers and measurementsCustomers may still buy high fat dairy products

The author of the newsletter is offering potentially dangerous advice by recommending Old Dairy stockholders to withdraw investment and stop purchase What is more the authorrsquos prediction debases the reputation and business of Old Dairy and if false could devoid the investment opportunity of the newsletter readers Therefore investors should examine whether the authorrsquos evidences are substantial

To begin with the author states that 80 percent of the respondents in a survey indicated a desire to reduce their intake of foods He therefore argues that Old Dairyrsquos high fat and cholesterol products would decrease in sales However the author makes a crucial error in this argument First the author provides no evidence that the surveyrsquos results are statistically reliable Were they representative of all the customers Were they chosen for the survey randomly Furthermore the desire to reduce fat and cholesterol intake is a pervasive trend in todayrsquos opulent society however the author erroneously identifies this as a new phenomenon which will affect consumer trends Second having a desire to reduce fat and cholesterol intake does not necessarily indicate that people who have this desire will actually reduce consuming these types of products It is entirely possible that they may continue buying Old Dairy products for its quality and taste Accordingly the author cannot draw any firm conclusion that people will not buy Old Dairy products Therefore if any of these cases are true the author may be offering investors a detrimental investment advice

최근 조사에 대한 응답자중 80 이상이 자신이 먹는 음식에서 지방과 콜레스테롤의 함유량을 줄이고 싶다고 한다 아울러 요즘은 많은 식료품 가계에서 저지방 제품들을 많이 취급하고 있다 현재 Old Dairy Industries가 판매하고 있는 많은 음식제품들은 지방과 콜레스테롤이 높기 때문에 이 회사의 매출이 격감할 것으로 보이며 당연히 매출이익도 줄어들것이다 따라서 이 회사의 주주들은 주식을 매각하고 다른 주식 투자가들도 이 회사의 주식을 매입하지 않는 것이 좋다

결론 Old Dairy stockholders to sell their shares and other investors not to purchase stock in this company

반박 모든 상품이 다 고 지방 고 칼로리는 아니다(비록 많을지라도) 일부의 식품의 경우 기호에 맞어서 히트해서 전체적인 수입이 증가할 수도 있다국내시장만 생각할 수 없다( 외국시장에서 호황을 누릴수 있다 )입맛이라는게 즉각 바뀌는게 아니다

The following appeared in a letter to the editor of the Balmer Island Gazette

On Balmer Island where mopeds serve as a popular form of transportation the population increases to 100000 during the summer months To reduce the number of accidents involving mopeds and pedestrians the town council of Balmer Island should limit the number of mopeds rented by the islands moped rental companies from 50 per day to 25 per day during the summer season By limiting the number of rentals the town council will attain the 50 percent annual reduction in moped accidents that was achieved last year on the neighboring island of Seaville when Seavilles town council enforced similar limits on moped rentals

1 Write a response in which you discuss what questions would need to be answered in order to decide whether the recommendation is likely to have the predicted result Be sure to explain how the answers to these questions would help to evaluate the recommendation

2 Write a response in which you discuss what questions would need to be answered in order to decide whether the prediction and the argument on which it is based are reasonable Be sure to explain how the answers to these questions would help to evaluate the prediction

3 Write a response in which you examine the stated andor unstated assumptions of the argument Be sure to explain how the argument depends on these assumptions and what the implications are for the argument if the assumptions prove unwarranted

Whatrsquos the actual population of Balmer Island 100000mdashis this a significant increase What kind of accidents Skin abrasions or serious injury And compared to Seaville how serious are the accidents and the actual number of accidents Did Seaville enforce other restrictions like safety signsHow different are the conditions of Balmer

GRE AWA John박 박정어학원

and Seaville regarding population road (safety) conditions topography other town-government regulation How much will the economy of Balmer be affected do to this restriction Could it cause an economic recession due to the fact that these rental companiesrsquo chance to make money is only during the summer thereby weakening the economic infrastructure Are there any other ways that could better alleviate the accident rate

Statistics 50-impreciseAnalogy Balmer compared with TorseauFalse Cause Accidents might have occurred because of reasons other than mopeds False Cause population increase may not be part of the cause of the accidentsOther explanations for the accident pedestrians few road safety regulations narrow roadsThere could be other better solutionshellip

Balmer Island의 인구가 여름철에는 십만명으로 늘어난다 2륜차와 보행자간 사고를 줄이기 위해 시의회는 6개의 자전거를 포함한 2륜차 대여업체에게 이 기간동안에는 대여숫자를 일일 50에서 30으로 제한하도록 할 것이다 대여숫자를 줄임으로써 시 의회는 지난해 이웃한 Torseau섬에서 이와 동일한 규제를 시행해서 50나 줄인 결과를 보고 마찬가지로 50를 줄일수 있다고 확신하고 있다

결론 The town council of Balmer Island should linit the number

반박 보행자의 부실에 의해서 사고가 많이 일어날수도 있다렌탈수의 줄임만이 대책은 아니다(대부분의 사람들이 렌탈 보다는 소유하고 있을 수도 있다)옆섬과는 상황이 다를수도 있다(그 섬에서는 사고의 원인이 많은 자전거 수로 인한것일수있다) 하지만 이 섬은 좁은 도로가 원인일 수도 있고 도로 안전 장치의미비가 원일일수 있다

In this letter the author recommends that Balmer Island should limit the number moped rentals from 50 to 30 per day To support this recommendation the author points out several reasons However careful scrutiny of each of the facts reveals that it is filled with unanswered questions that could significantly weaken the authorrsquos recommendation with loops and holes which are answered

The recommendation depends on the assumption that no alternative means of reducing the number of accidents are available However the author fails to offer any evidence to substantiate this crucial assumption It is highly possible that means other than this would better solve the problem Perhaps they could widen the roads or put-up more safety signs Or perhaps the accidents were due to the lack of skills in which case proper safety training would significantly alleviate the problem Without considering and ruling out these and other alternative means of reducing accidetns the author cannot confidently conclude that merely emulating Torseau would suffice Moreover the author is advising a recommendation which could potentially harm the economy of Balmer Island sincehellip Moreover the Balmer Island should alternative means to reduce accidents because limiting moped rentals during the summer could harm the economy of Balmerhellip

First of all the author believes that increase in population and the number of moped rentals are responsible for the accidents It is entirely possible that other factors are responsible for the accidents Perhaps Balmer Islandrsquos lack of safety signs was a major factor Or maybe the roads are narrow and dangerous on the Island therefore the town council could enforce stricter traffic regulations to alleviate the problem Accordingly if either of these scenarios is true the author cannot draw any firm conclusion that increase in the number of population and moped rentals are the cause of the accidents

The author of this editorial recommends that to reduce accidents involving mopeds andpedestrians Balmer Islands city council should restrict moped rentals to 30 per day down from50 at each of the islands six rental outlets To support this recommendation the author citesthe fact that last year when nearby Torseau Islands town council enforced similar measuresTorseaus rate of moped accidents fell by 50 For several reasons this evidence providesscant support for the authors recommendationTo begin with the author assumes that all other conditions in Balmer that might affect therate of moped-pedestrian accidents will remain unchanged after the restrictions are enactedHowever with a restricted supply of rental mopeds people in Balmer might purchase mopedsinstead Also the number of pedestrians might increase in the future with more pedestriansespecially tourists the risk of moped-pedestrian accidents would probably increase For thatmatter the number of rental outlets might increase to make up for the artificial supplyrestriction per outlet--a likely scenario assuming moped rental demand does not declineWithout considering and ruling out these and other possible changes that might contribute to ahigh incidence of moped-pedestrian accidents the author cannot convince me that theproposed restrictions will necessarily have the desired effect

GRE AWA John박 박정어학원

Next the author fails to consider other possible explanations for the 50 decline inTorseaus moped accident rate last year Perhaps last year Torseau experienced unusually fairweather during which moped accidents are less likely Perhaps fewer tourists visited Tot seanlast year than during most years thereby diminishing the demand for rental mopeds to belowthe allowed limits Perhaps last year some of Torseaus moped rental outlets purchased newmopeds that are safer to drive Or perhaps the restrictions were already in effect but were notenforced until last year In any event a decline in Torseaus moped accident rate during onlyone year is scarcely sufficient to draw any reliable conclusions about what might have causedthe decline or about what the accident rate will be in years aheadAdditionally in asserting that the same phenomenon that caused a 50 decline in mopedaccidents in Torseau would cause a similar decline in Balmer the author relies on what mightamount to an unfair analogy between Balmer and Torseau Perhaps Balmers ability to enforcemoped-rental restrictions does not meet Torseaus ability if not then the mere enactment ofsimilar restrictions in Balmer is no guarantee of a similar result Or perhaps the demand formopeds in Torseau is always greater than in Balmer Specifically if fewer than all availablemopeds are currently rented per day from the average Balmer outlet while in Torseau everyavailable moped is rented each day then the proposed restriction is likely to have less impacton the accident rate in Balmer than in TorseauFinally the author provides no evidence that the same restrictions that served to reduce theincidence of all moped accidents by 50 would also serve to reduce the incidence ofaccidents involving mopeds and pedestrians by 50 Lacking such evidence it is entirelypossible that the number of moped accidents not involving pedestrians decreased by a greaterpercentage while the number of moped-pedestrian accidents decreased by a smallerpercentage or even increased Since the author has not accounted for these possibilities theeditorials recommendation cannot be taken seriouslyIn conclusion the recommendation is not well supported To convince me that the proposedrestriction would achieve the desired outcome the author would have to assure me that nochanges serving to increase Balmers moped-pedestrian accident rate will occur in theforeseeable future The author must also provide dear evidence that last years decline inmoped accidents in Torseau was attributable primarily to its moped rental restrictions ratherthan to one or more other factors In order to better evaluate the recommendation I wouldneed more information comparing the supply of and demand for moped rentals on the twoislands I would also need to know the rate of mopedpedestrian accidents in Torseau both priorto and after the restrictions were enforced in TorseauThe following appeared in a magazine article about planning for retirement

Clearview should be a top choice for anyone seeking a place to retire because it has spectacular natural beauty and a consistent climate Another advantage is that housing costs in Clearview have fallen significantly during the past year and taxes remain lower than those in neighboring towns Moreover Clearviews mayor promises many new programs to improve schools streets and public services And best of all retirees in Clearview can also expect excellent health care as they grow older since the number of physicians in the area is far greater than the national average

Write a response in which you discuss what specific evidence is needed to evaluate the argument and explain how the evidence would weaken or strengthen the argument

-Natural beauty and consistent climate may not be the most wanted qualities-Housing costs could have lowered on a national level wealthy retirees may not care about costs-Taxes may be high compared to the nationrsquos average tax rate-What about other qualities of Clearview Crime rate what qualities would retirees want -If schools streets and public services need improvement then this is proof that the current condition of Clearview is low Or due to budgetary reasons the mayor may not follow-up on his promise because of lowered tax rate -Schools and people who are retired no relationship-Physicians What kind of physicians Number is irrelevant Are these physicians capable of addressing the illnesses of old people

This author argues that anyone seeking a place to retire should choose Clearview To supportthis argument the article cites Clearviews consistent climate and natural beauty its fallinghousing costs its low property taxes compared to nearby towns and the mayors promise toimprove schools streets and services The article also claims that retirees can expectexcellent health care because the number of physicians in Clearview greatly exceeds thenational average This argument is flawed in several critical respectsTo begin with although consistent climate and natural beauty might be attractive to manyretirees these features are probably not important to all retirees For many retirees it isprobably more important to live near relatives or even to enjoy changing seasons Thus I

GRE AWA John박 박정어학원

cannot accept the authors sweeping recommendation for all retirees on this basisAlso Clearviews declining housing costs do not necessarily make Clearview the best placeto retire for two reasons First despite the decline Clearviews housing costs might be highcompared to housing costs in other cities Secondly for wealthier retirees housing costs arenot likely to be a factor in choosing a place to retire Thus the mere fact that housing costshave been in decline lends scant support to the recommendationThe articles reliance on Clearviews property-tax rates is also problematic in two respectsFirst retirees obviously have innumerable choices about where to retire besides Clear viewand nearby towns Secondly for retirees who are well-off financially property taxes are notlikely to be an important concern in choosing a place to retire Thus it is unfair to infer fromClearviews property-tax rates that retirees would prefer ClearviewYet another problem with the argument involves the mayors promises In light of Clearviewslow property-tax rates whether the mayor can follow through on those promises is highlyquestionable Absent any explanation of how the city can spend more money in the areas citedwithout raising property taxes I simply cannot accept the editorials recommendation on thebasis of those promises Besides even if the city makes the improvements promised thoseimprovements--particular the ones to schools--would not necessarily be important to retireesFinally although the number of physicians in Clearview is relatively high the per capitanumber might be relatively low Moreover it would be fairer to compare this per capita numberwith the per capita number for other attractive retirement towns--rather than the nationalaverage After all retirees are likely to place a relatively heavy burden on health-careresources Besides the article provides no assurances that the number of physicians inClearview will remain high in the foreseeable futureIn conclusion the recommendation is poorly supported To strengthen it the author mustconvince me--perhaps by way of a reliable survey--that the key features that the vast majorityof retirees look for in choosing a place to live are consistent climate natural beauty and lowhousing costs The author must also provide better evidence that Clear views property taxesare lower than the those of cities in other areas The author must also explain how the city canmake its promised improvements without raising property taxes Finally to better assess theargument I would need to now how the per capita number of physicians in Clearview wouldcompare to the national average in the futureThe following appeared as a letter to the editor from a Central Plaza store owner

Over the past two years the number of shoppers in Central Plaza has been steadily decreasing while the popularity of skateboarding has increased dramatically Many Central Plaza store owners believe that the decrease in their business is due to the number of skateboard users in the plaza There has also been a dramatic increase in the amount of litter and vandalism throughout the plaza Thus we recommend that the city prohibit skateboarding in Central Plaza If skateboarding is prohibited here we predict that business in Central Plaza will return to its previously high levels

Write a response in which you discuss what questions would need to be answered in order to decide whether the recommendation is likely to have the predicted result Be sure to explain how the answers to these questions would help to evaluate the recommendation

Why two years ago What happened two years ago which started this declineIs the dramatic increase in the ldquopopularityrdquo of skateboarding the cause of the steady decline of shoppers Are there any malls nearby Were there any changes nearby which could affect the decline in customersmdasha big mall perhaps Could the decline be due to the shop ownersHow many skateboarders use the plazaWhere do they skateboardDo they shop and are they customersAre the increase in litter and vandalism due to skateboarders Could this be alleviated by installing CCTVs and hiring security

This editorial concludes that the city should ban skateboarding from its downtown CentralPlaza in order to attract visitors to that area to return the area to its former glory and to makeit a place where people can congregate for fun and relaxation To justify this conclusion theeditorial points out that skateboarders are nearly the only people one sees anymore at CentralPlaza and that the Plaza is littered and its property defaced The editorial also points out thatthe majority of downtown merchants support the skate boarding ban This argument is flawedin several critical respectsFirst the editorials author falsely assumes that a ban on skateboarding is both necessaryand sufficient to achieve the three stated objectives Perhaps the city can achieve thoseobjectives by other means as well--for example by creating a new mall that incorporates anattractive new skateboard park Even if banning skateboarders altogether is necessary to meetthe citys goals the author has not shown that this action by itself would suffice Assuming thatthe Plazas reputation is now tarnished restoring that reputation and in turn enticing peopleback to the Plaza might require additional measures--such as removing litter and graffiti

GRE AWA John박 박정어학원

promoting the Plaza to the public or enticing popular restaurant or retail chains to the PlazaSecondly the editorial assumes too hastily that the Plazas decline is attributable to theskateboarders--rather than to some other phenomenon Perhaps the Plazas primary appeal inits glory days had to do with particular shops or eateries which were eventually replaced byless appealing ones Or perhaps the crime rate in surrounding areas has risen dramatically forreasons unrelated to the skateboarders presence at the Plaza Without ruling out these andother alternative explanations for the Plazas decline the editorials author cannot convince methat a skateboard ban would reverse that declineThirdly the editorials author might be confusing cause with effect--by assuming that theskateboarders caused the abandonment of the Plaza rather than vice versa It is entirelypossible that skateboarders did not frequent the Plaza until it was largely abandoned--andbecause it had been abandoned In fact this scenario makes good sense since skateboardingis most enjoyable where there are few pedestrians or motorists to get in the wayFourth it is unreasonable to infer from the mere fact that most merchants favor the ban thatthe ban would be effective in achieving the citys objectives Admittedly perhaps thesemerchants would be more likely to help dean up the Plaza area and promote their businesseswere the city to act in accordance with their preference Yet lacking any supporting evidencethe author cannot convince me of this Thus the survey amounts to scant evidence at best thatthe proposed ban would carry the intended resultFinally the author recommends a course of action that might actually defeat the citysobjective of providing a fun and relaxing place for people to congregate In my experienceskateboarding contributes to an atmosphere of fun and relaxation for adults and children alikemore so than many other types of ambiance Without considering that continuing to allowskateboarding--or even encouraging this activity--might achieve the citys goal more effectivelythan banning the activity the author cannot convincingly conclude that the ban would be in thecitys best interestsIn sum the argument is a specious one To strengthen it the editorials author must providedear evidence that skateboarding and not some other factor is responsible for the conditionsmarking the Plazas decline The author must also convince me that no alternative means ofrestoring the Plaza are available to the city and that the proposed ban by itself would suffice toattract tourists and restore the Plaza to its former glory Finally to better assess the argument itwould be useful to know the circumstances under which the downtown merchants would bewilling to help the city achieve its objectives

6그룹 약한 비유 빈출

The following recommendation appeared in a memo from the mayor of the town of Hopewell

Two years ago the nearby town of Ocean View built a new municipal golf course and resort hotel During the past two years tourism in Ocean View has increased new businesses have opened there and Ocean Views tax revenues have risen by 30 percent Therefore the best way to improve Hopewells economymdashand generate additional tax revenuesmdashis to build a golf course and resort hotel similar to those in Ocean View

Write a response in which you examine the stated andor unstated assumptions of the argument Be sure to explain how the argument depends on these assumptions and what the implications are for the argument if the assumptions prove unwarranted

GRE AWA John박 박정어학원

Assumptions The author assumes that OVrsquos municipal golf course and resort hotel caused tourism new businesses and increased tax revenues There may be other reasons advertising promo He assumes that this will continueAssumes that Ocean View and Hopewell are similar in many waysmdashthe name suggests otherwise OV may have always been a tourist attractions for its beaches We need to know the topography

2년전 Ocean View 시는 시정 소유 골프 및 휴양지 호텔을 신축했다 그리고 지난 2년동안 이 시의 관광객이 증가했으며 새로운 사업들이 생겨났다 그에따라 시의 세수도 30나 증가했다 Hopewell의 경제를 향상시키고 아울러 세수를 늘릴 수 있는 가장 좋은 방법은 Ocean View에 세워진 것과 같은 골프 시설과 휴양지 호텔을 신축하는 것이다

1 다른 요인으로 관광 산업이 발전했을 수도 있다 문화 유적이 발견이 되었거나 도로의 정비등으로 여행자가 늘었을 수도 있다

2 관광 산업의증가가 늘어난 세수의 원인이 아니라 새로 유입된 인구의 증가나 다른 공장에서 발생한 것일 수 있다

3 2년동안 한참 골프가 붐을 이루었을 수 있다 경제상황이 나빠지거나 다른 레포츠가 인근 지역에 생겨난다면 골프하는 사람이 줄어들 수 있다

In this memo HopeweUs mayor recommends that in order to stimulate the towns economyand boost tax revenues HopeweU should build a new golf course and resort hotel just as thetown of Ocean View did two years ago To support this recommendation the mayor points outthat in Ocean View during the last two years tourism has increased new businesses haveopened and tax revenues have increased by 30 I find the mayors argument unconvincingin several important respectsFirst of all it is possible that the mayor has confused cause with effect respecting the recentdevelopments in Ocean View Perhaps Ocean Views construction of a new golf course andhotel was a response to previous increases in tourism and business development increasesthat have simply continued during the most recent two years Since the mayor has failed toaccount for this possibility the claim that Hopewell would boost its economy by alsoconstructing a golf course and hotel is completely unwarrantedSecondly the mayor fails to account for other possible causes of the trends in Ocean Viewduring the last two years The increase in tourism might have been due to improving economicconditions nationwide or to unusually pleasant weather in the region The new businessesthat have opened in Ocean View might have opened there irrespective of the new golf courseand hotel And the 30 increase in tax revenues might have been the result of an increase intax rates or the addition of a new type of municipal taxWithout ruling out these and other alternative explanations for the three recent trends inOcean View the mayor cannot reasonably infer based on those trends that Hopewellseconomy would benefit by following Ocean Views exampleThirdly even if the recent trends in Ocean View are attributable to the construction of the newgolf course and hotel there the mayor assumes too hastily that the golf course and hotel willcontinue to benefit that towns overall economy The mayor has not accounted for thepossibility that increased tourism will begin to drive residents away during tourist season orthat new business development will result in the towns losing its appeal as a place to visit or tolive Unless the mayor can convince me that these scenarios are unlikely I cannot accept themayors recommendation that Hopewell follow Ocean Views exampleFinally the mayors argument rests on the unsubstantiated assumption that Hopewell andOcean View are sufficiently alike in ways that might affect the economic impact of a new golfcourse and hotel Hopewell might lack the sort of natural environment that would attract moretourists and new businesses to the town--regardless of its new golf course and hotel For thatmatter perhaps Hopewell already contains several resort hotels and golf courses that are notutilized to their capacity If so building yet another golf course and hotel might amount to amisallocation of the towns resources--and actually harm the towns overall economyIn sum the mayors recommendation is not well supported To bolster it the mayor mustprovide better evidence that Ocean Views new golf course and hotel and not some otherphenomenon--has been responsible for boosting Ocean Views economy during the last twoyears To better assess the recommendation I would need to know why Ocean View decidedto construct its new golf course and hotel in the first place--specifically what events prior toconstruction might have prompted that decision I would also need to thoroughly compare

GRE AWA John박 박정어학원

HopeweU with Ocean View--especially in terms of their appeal to tourists and businesses--todetermine whether the same course of action that appears to have boosted Ocean Viewseconomy would also boost Hopewells economy

The following is part of a memorandum from the president of Humana University

Last year the number of students who enrolled in online degree programs offered by nearby Omni University increased by 50 percent During the same year Omni showed a significant decrease from prior years in expenditures for dormitory and classroom space most likely because instruction in the online programs takes place via the Internet In contrast over the past three years enrollment at Humana University has failed to grow and the cost of maintaining buildings has increased along with our budget deficit To address these problems Humana University will begin immediately to create and actively promote online degree programs like those at Omni We predict that instituting these online degree programs will help Humana both increase its total enrollment and solve its budget problems

Write a response in which you discuss what questions would need to be answered in order to decide whether the prediction and the argument on which it is based are reasonable Be sure to explain how the answers to these questions would help to evaluate the prediction

Is Omni University successful due to the online degree program 50 Is the decrease in expenditures for dormitory and classroom space due to the decrease in of on-campus students Which classes were successful Does HU have those classes

Even if the long-distance degree programs at Omni University benefited the school the presidentrsquos recommendation that Human College should emulate Omni University is too hasty First OUrsquos name implies that the school would have more majors than Humanahellip the president should examine which degrees were in the long-distance programhellip

지난해에는 Omni 대학에서 개강했던 원거리 학생 학점 취득 프로그램을 등록했던 학생들의 숫자가 50나 증가했다 같은해 기간동안 Omni 대학에서는 그 전년도부터 기숙사와 학급의 공간 확충을 위한 예산을 대폭 줄였는데 이는 이 원거리 학점 취득 프로그램이 양방향 비디오 컴퓨터 접속을 통해서만 가능한 수업지도 방식이기때문인 것으로 보인다 반면 지난 3개년 동안 Humana 대학에서의 수강률은 감소한데다가 건물

GRE AWA John박 박정어학원

유지비도 올랐다 따라서 Humana대학의 수강을 늘리고 예산손실을 회복하기 위해서는 Omni 대학에서 취한 조치와 같은 능동적인 프로그램을 추진해야 한다

결론 we should initiate and actively promote long-distance degree programs like those at Omni 반박 원거리 학생 취득 프로그램 숫자가 증가한거하고 예산이 줄어드는 것 사이에 연관이 약하다 (causal 학생의 증가로 관리비용 증가할수 있음 원거리 수업가능 장비도입에의한 비용발생)bad analogy(omni university 하고 같은 조건이 아니다 )-gt omni college 가 강좌내용이 좋아서 학생의 등록이 많을수 있다 Humana 대학에서 만들었다 하더라도 인기 없을수 있음다른 요인에 의해서 Humana 대학의 수강 인원이 증가할수 있음(비록 과거엔 인기가 없었을지라도)

The following appeared as part of a business plan developed by the manager of the Rialto Movie Theater

Despite its downtown location the Rialto Movie Theater a local institution for five decades must make big changes or close its doors forever It should follow the example of the new Apex Theater in the mall outside of town When the Apex opened last year it featured a video arcade plush carpeting and seats and a state-of-the-art sound system Furthermore in a recent survey over 85 percent of respondents reported that the high price of newly released movies prevents them from going to the movies more than five times per year Thus if the Rialto intends to hold on to its share of a decreasing pool of moviegoers it must offer the same features as Apex

Write a response in which you discuss what questions would need to be answered in order to decide whether the recommendation is likely to have the predicted result Be sure to explain how the answers to these questions would help to evaluate the recommendation

Before following through this business plan the manager should investigate the cause of Rialtorsquos unsuccessful business

The author provides no evidence that the surveyrsquos results are statistically reliable The surveyrsquos sample of 85 percent must be sufficient in size and representative of overall population of the city where Rialto and Apex is serving Lacking evidence of a sufficiently representative sample the author cannot justifiably rely on the survey to draw any conclusion whatsoever The author does not indicate that Apex is indeed currently successful However even if Apex is enjoying success the argument relies on what might be a false analogy between Rialto and Apex In order for Apex to serve as a model that Rialto should emulate the author must assume that all relevant circumstances are essentially the same However this assumption is unwarranted For example the argument overlooks the face that Apex is located in a strategic placemdashbeside a mall where customers can not only watch a movie but also enjoy shopping Therefore simply changing the facility to that of Apex may not lead to success

The author does not mention whether Apex is successful or not Nevertheless even if Apex is currently successful the argument relies on what might be a false analogy between Rialto and Apex In order for Apex to serve as a model that Rialto should emulate the author must assume that all relevant circumstances are essentially the same However this assumption is unwarranted For example the argument overlooks the fact that these two institutions are located in different locations Rialto in downtown and Apex in a mall outside of town Although Apex opened with state-of-the-art facilities the decisive factor in its success could be due to its strategic location of being in a mall People could enjoy both shopping and movies at one location thus they may prefer Apex over Rialto Furthermore the place where people enjoy leisure activities has shifted in the past decades for most cities from downtown to the suburbs Therefore Rialto may not be successful even if it emulates Apexrsquos facilities A better business plan may be relocating Apex to the thriving section of the downtown

Rialto 극장은 지난 50여년간 지역 회관으로써 시내에 위치해 있으면서도 이제 변화를 꾀하지 않으면 문을 닫을

GRE AWA John박 박정어학원

판이다 이 극장은 시외 쇼핑타운에 새로 들어선 Apex 극장의 사례를 본받아야 했다 Apex가 지난해 개업했을 당시 이 극장은 비디오 아케이드 플러쉬 카펫트 바닥과 좌석 그리고 최신 음향시설을 갖추었다 더군다나 최근 조사에서는 응답자의 85 이상이 새로 출시된 영화 입장료가 비싼 탓으로 지난해보다 5배이상의 관람객이 줄어들었다고 나타났다 따라서 Rialto 극장이 줄어들고 있는 관람객을 뺐기지 않고 유지하려면 Apex와 같은 시설들을 갖추어야 할 것이다주장 리알토 극장이 줄어들고 있는 관람객을 뺐기지 않고 유지하려면 Apex와 같은 시설들을 갖추어야 할 것이다

1 조사에서 응답자가 전체를 대표할 수 없다 2 apex 극장이 좋은 시설을 갖추고 있지만 그로 인해 수익이 많이 발생했다는 말이 없으므로 시설투자를

하고도 좋은 결과를 얻을 수 있을지 그 근거가 미흡하다3 좋은 영화가 출시된다면 입장료가 비싸도 영화관에서 꼭 보려고 할 수 있다 4 rialto 가 시설이 아닌 다른 요인에 의해 장사가 안될수도 있다( 우범 지역이라든지)

The following is a recommendation from the business manager of Monarch Books

Since its opening in Collegeville twenty years ago Monarch Books has developed a large customer base due to its reader-friendly atmosphere and wide selection of books on all subjects Last month Book and Bean a combination bookstore and coffee shop announced its intention to open a Collegeville store Monarch Books should open its own in-store cafeacute in the space currently devoted to childrens books Given recent national census data indicating a significant decline in the percentage of the population under age ten sales of childrens books are likely to decline By replacing its childrens books section with a cafeacute Monarch Books can increase profits and ward off competition from Book and Bean

Write a response in which you examine the stated andor unstated assumptions of the argument Be sure to explain how the argument depends on these assumptions and what the implications are for the argument if the assumptions prove unwarranted

The following is a recommendation from the business manager of Monarch Books

Since its opening in Collegeville twenty years ago Monarch Books has developed a large customer base due to its reader-friendly atmosphere and wide selection of books on all subjects Last month Book and Bean a combination bookstore and coffee shop announced its intention to open a Collegeville store Monarch Books should open its own in-store cafeacute in the space currently devoted to childrens books Given recent national census data indicating a significant decline in the percentage of the population under age ten sales of childrens books are likely to decline By replacing its childrens books section with a cafeacute Monarch Books can increase profits and ward off competition from Book and Bean

1 Write a response in which you discuss what questions would need to be answered in order to decide whether the recommendation is likely to have the predicted result Be sure to explain how the answers to these questions would help to evaluate the recommendation

2 Write a response in which you discuss what specific evidence is needed to evaluate the argument and explain how the evidence would weaken or strengthen the argument

No evidence regarding Monarch Bookrsquos successEven if Regal Bookrsquos is successful this may not be attributable to the cafeacute False analogy Emulating may not lead to success Other factors may be involvedInsufficient condition The national census is not enough evidence that childrenrsquos book sales will decline Can

GRE AWA John박 박정어학원

the national census represent the local child populationDid opening a cafeacute boost sales for Regal Books Even assuming Regal is successful by opening a cafeacute this may not be suitable for Monarch which plans to close the childrenrsquos book section to establish a cafe Imprecise language ldquorelatively little spacerdquo how smallThe managerrsquos recommendation contradicts what he says Since Monarch is popular for its wide selection of books closing a selection which targets a major group of readers may hurt Monarchrsquos salesIs this the best way to compete

When Stanley Park first opened it was the largest most heavily used public park in town It is still the largest park but it is no longer heavily used Video cameras mounted in the parks parking lots last month revealed the parks drop in popularity the recordings showed an average of only 50 cars per day In contrast tiny Carlton Park in the heart of the business district is visited by more than 150 people on a typical weekday An obvious difference is that Carlton Park unlike Stanley Park provides ample seating Thus if Stanley Park is ever to be as popular with our citizens as Carlton Park the town will obviously need to provide more benches thereby converting some of the unused open areas into spaces suitable for socializing

Write a response in which you examine the stated andor unstated assumptions of the argument Be sure to explain how the argument depends on these assumptions and what the implications are for the argument if the assumptions prove unwarranted

Stanley 파크가 처음 개장했을 당시 가장 크고 가장 많이 이용되는 공원이었다 아직도 공원중에서는 가장 크지만 이용률은 상당히 떨어졌다 지난달 공원 주차장에 설치해놓은 비디오 카메라를 통해 보면 drop(주차장으로 여겨짐) 이용률이 가장 높았다 수치상으로는 하루 평균 50대의 차량만이 이용하였다 반면 직장 중심거리에 위치한 작은 규모의 Carlton 파크는 주당 무려 150여명 이상이 이용하고 있다 Stanley 파크와는 달리 Carlton 파크에는 의자가 있다는 것이 가장 뚜렷한 차이점이다 따라서 Stanley 파크가 Carlton 파크처럼 시민들이 자주 이용하는 공원이 되기 위해서는 벤치를 설치할 필요가 있으며 이렇게 사용되지 않는 일부 공간을 활용해서 사교를 위한 공간으로 바꾸어야 한다 ===gtdrop 에 대한 첨부사항 (영영사전내용입니다)---- a place or central depository to which something (as mail money or stolen property) is brought for distribution or transmission also the act of depositing something at such a place dropgt

주장 if Stanley Park is ever to be as popular with our citizens as is Carlton Park the town will obviously need to provide more benches thereby converting some of the unused open areas into spaces suitable for socializing1 조사가 언제 이루어진 것인가 조사가 언제 실시되었느냐에 따라 결과가 다를 수있다 현재는 다시 스탠리 파크가 늘어났었을 수 있다 2 벤치를 많이 설치했다고 해서 많은 관광객이 오지 않을수 있다(사람들이 벤치나 사교 공간을 원한다는 어떠한 자료도 없다)3스탠리 파크 주변에 교통 상황이 악화가 되었거나 칼튼 파크에서 문화행사등을 많이 가져서 이용객이 줄어든것일 수도 있다 4 칼튼 파크가 중심지에 있어서 접근성이 좋을수 있다5 조사가 같은 시간을 기준으로 한게 아니다(하나는 주중이고 하나는 주말이다)6사람의 수와 차의 대수를 같은것으로 비교할수 없다 (차안에 몇 명이 타고 있는지 모르고 대중교통을 이용해서 왔을수도 있다)

Page 22: GRE writing argument brain storm

GRE AWA John박 박정어학원

get more sleep To support this recommendation the author provides several evidences However careful scrutiny of each of the facts reveals that it provides little credible support for the authorrsquos recommendation QuestionsThe number of accidents What kind of accidents The seriousness of the accidents is importantHow many employees are in each company What are their productsFalse cause Sleep may not be the reason for the on-the-job accidents What do Alta and Panoply manufacture

First of all the author believes that fatigue caused the on-the job accidents However there could be other reasons The author observes a correlation between sleep deprivation and on-the-job accidents then concludes that the former is the cause of the latter However the author fails to rule out other possible explanations For example it is entirely possible that Alta factories require more strenuous and dangerous labor than Panoply Without ruling out all other such factors it is unfair to conclude that fatigue is responsible for the accidents In addition the work-shifts may not be the cause of the sleep deprivation and fatigue It is possiblehellip Thus the author should provide what exactly Panoply and Alta manufacture and more precise data about their working conditions to be more convincing

Shortening the shift by one hour does not necessarily lead to more sleep And is one hour enoughLess accidents does not mean increased productivity

결론 We should shorten each of out three work shifts by one hour

반박 경쟁사에 비해서 시간당 하는 업무량이 많아서 더욱 피곤할 수도 있다 시간이 문제가 아니라 노후된 시설 설비 자체의 문제 작업 자체가 원래 위험한 것이여서 사고가 많을 수도 있다 다른 회사는 더욱 많은 작업시간에도 불구하고 안정한 작업여건으로 인해서 사고율이 오히려 더 작을 수도 있다비교사의 재해감소가 다른 요인일수 있다(안전 교육 철저)줄인 시간이 피로회복이나 수면으로 연결 안될수 있음(술을 마실 수도 있고 그 시간에 휴식을 취하지 않고 다른일을 함으로써 더욱 피로해질수 있다)

This editorial recommends that Alta Manufacturing reduce its work shifts by one hour each inorder to reduce its on-the-job accident rate and thereby increase Altas productivity To supportthis recommendation the author points out that last year the number of accidents at Alta was30 greater than at Panoply Industries where work shifts were one hour shorter The authoralso cites certain experts who believe that many on-the-job accidents are caused by fatigueand sleep deprivation I find this the argument unconvincing for several reasonsFirst and foremost the author provides absolutely no evidence that overall workerproductivity is attributable in part to the number of on-the-job accidents Although commonsense informs me that such a relationship exists the author must provide some evidence ofthis cause-and-effect relationship before I can accept the authors final conclusion that theproposed course of action would in fact increase Altas productivitySecondly the author assumes that some accidents at Alta are caused by fatigue or sleepdeprivation However the author overlooks other possible causes such as inadequateequipment maintenance or worker training or the inherent hazards of Altas manufacturingprocesses By the same token Panoplys comparatively low accident rate might be attributablenot to the length of its work shifts but rather to other factors such as superior equipmentmaintenance or worker training In other words without ruling out alternative causes ofon-the-job accidents at both companies the author cannot justifmbly conclude that merely byemulating Panoplys work-shift policy Alta would reduce the number of such accidentsThirdly even assuming that Altas workers are fatigued or sleep-deprived and that this is thecause of some of Altas on-the-job accidents in order to accept the authors solution to thisproblem we must assume that Altas workers would use the additional hour of free time tosleep or rest However the author provides no evidence that they would use the time in thismanner It is entirely possible that Altas workers would use that extra hour to engage in someother fatiguing activity Without ruling out this possibility the author cannot convincinglyconclude that reducing Altas work shifts by one hour would reduce Altas accident rateFinally a series of problems with the argument arise from the scant statistical information onwhich it relies In comparing the number of accidents at Alta and Panoply the author fails toconsider that the per-worker accident rate might reveal that Alta is actually safer than Panoplydepending on the total number of workers at each company Second perhaps accident rates

GRE AWA John박 박정어학원

at the two companies last year were aberrations and during other years Altas accident ratewas no greater or even lower than Panoplys rate Or perhaps Panoply is not representativeof industrial companies generally and that other companies with shorter work shifts have evenhigher accident rates In short since the argument relies on very limited statistical information Icannot take the authors recommendation seriouslyIn conclusion the recommendation for emulating Panoplys work-shift policy is not wellsupported To convince me that shorter work shifts would reduce Altas on-the-job accidentrate the author must provide clear evidence that work-shift length is responsible for some ofAltas accidents The author must also supply evidence to support her final conclusion that alower accident rate would in fact increase overall worker productivity

The following appeared in a memo from the vice president of marketing at Dura-Sock Inc

A recent study of our customers suggests that our company is wasting the money it spends on its patented Endure manufacturing process which ensures that our socks are strong enough to last for two years We have always advertised our use of the Endure process but the new study shows that despite our socks durability our average customer actually purchases new Dura-Socks every three months Furthermore our customers surveyed in our largest market northeastern United States cities say that they most value Dura-Socks stylish appearance and availability in many colors These findings suggest that we can increase our profits by discontinuing use of the Endure manufacturing process

1 Write a response in which you examine the stated andor unstated assumptions of the argument Be sure to explain how the argument depends on these assumptions and what the implications are for the argument if the assumptions prove unwarranted

2 Write a response in which you discuss what specific evidence is needed to evaluate the argument and explain how the evidence would weaken or strengthen the argument

3 Write a response in which you discuss what questions would need to be answered in order to decide whether the recommendation and the argument on which it is based are reasonable Be sure to explain how the answers to these questions would help to evaluate the recommendation

Intro The vice president of marketing at Dura-Sock Inc is offering a potentially harmful investment recommendation by claiming that Dura-Sock should discontinue its use of the ldquoEndurerdquo process To support his recommendation he points out a study that Dura-Sock customers actually purchase the socks every three months and a survey that reveals that Dura-Sock customers like the sockrsquos stylish appearance and availability in many colors The study and survey however are insufficient in supporting his proposal and the VP makes several unwarranted assumptionsIntro (simplified) The VP states that though Dura-Socks last for two years customers buy the socks every three months Therefore he assumes that the consumersrsquo motive for buying the produce is not its durabilityHowever the author fails to rule out other possible motivation for consumption

Even if the survey is reliable the author should consider the rest of the market Vague terms ldquowasting moneyrdquomdashprecisely how much are they wasting Studysurvey errorThe company must calculate the outcome of such momentous decisionStudy participantsrsquo comment that they prefer Dura-Sock for its stylishness and availability might take Dura-Sockrsquos enduring quality for granted

우리회사 제품 소비자들에 대한 최근 조사에서 지난 2년여간 양말의 내구성을 강하게 하는 필수공정이었던 자사 특허의 Endure 공정에 들어가는 비용이 낭비라고 말하고 있다 우리 회사는 항상 이 공정 처리에 대한 광고를 내보냈으나 이에 대한 시장 조사에서 실제로 고객들은 이 신제품을 평균 석달마다 구매하는 것으로 나타났다 더군다나 북동부지역에서 실시한 대규모 시장조사에 응답한 고객들은 양말의 모양과 색상등에 더

GRE AWA John박 박정어학원

관심을 나타냈다 이러한 결과는 우리회사가 신기술 공법을 중단하면 그에 따라 수익이 늘어날 것이라는 것을 말해주고 있는 것이다주장 These findings suggest that Dura0Sock can increase its profits by discontinuing its use of the ldquoEndurerdquo manufacturing process

1 survey가 정확한 소비자의 의견을 나타낸 것인가 다른 선택없이 양자택일과 같은 방법의 survey였는지2 북동부지역의 시장조사가 전체 의견을 대표할 수 있나3 사람들이 모양이나 색상에 앞서 내구성을 먼저 평가했을 수도 있다 내구성을 갖추었다는 전제하에 모양과 색상에 관심을 드러낸 것일 수 있다4 소비자가 도매상(retail)인지 소매상(whole)인지가 없다

The following appeared in a business magazine

As a result of numerous complaints of dizziness and nausea on the part of consumers of Promofoods tuna the company requested that eight million cans of its tuna be returned for testing Promofoods concluded that the canned tuna did not after all pose a health risk This conclusion is based on tests performed on samples of the recalled cans by chemists from Promofoods the chemists found that of the eight food chemicals most commonly blamed for causing symptoms of dizziness and nausea five were not found in any of the tested cans The chemists did find small amounts of the three remaining suspected chemicals but pointed out that these occur naturally in all canned foods

Write a response in which you discuss what questions would need to be addressed in order to decide whether the conclusion and the argument on which it is based are reasonable Be sure to explain how the answers to the questions would help to evaluate the conclusion

Representativeness of the tested cansThey should conduct a comparative studyThe testing could be biased because Promofoods employees conducted the testingHow much (quantity) of the five and three suspected chemicals were in the canned foodsFalse cause The substance that caused dizziness and nausea may not be one of the eight common chemicals

많은 소비자들의 현기증과 구역질 불만에 따라 Promofoods사는 지난해 참치 캔 8백만 개를 테스트하기 위해 반품시켰다 그 결과 캔에서는 건강에 위험이 될 수 있는 화합물질이 없었던 것으로 회사측은 결론지었다 이러한 결론은 회사측 화학연구자들이 회수된 캔의 샘플을 테스트해서 이들 증상의 원인이 되는 8가지 화합물 중에서 5가지가 실험된 캔에서 발견되지 않았다는 사실에 근거한 것이다 이들 화학자들은 나머지 3개가지 화합물이 모든 캔 식료품에서 흔히 발견되는 것이라고 언급했다 결론 Promofoods concluded that the cans did not after all contain chemicals that posed a health risk

1 공인된 기간에서 테스트를 한 것이 아니고 자사에서 직접 테스트를 했기에 신뢰성이 안간다 2 이런 증상을 일으키는 8개의 물질 말고 다른 물질들이 캔속에 많이 포함됬을수 있다 3 나머지 3개의 물질들의 함유량이 많아서 다른 종류의 캔들은 문제를 일으키지 않지만 참치캔은 문제를

일으킬 수 있다

This magazine article concludes that the 8 million cans of tuna Promofoods recalled due tocomplaints about nausea and dizziness do not after ail contain any chemicals that pose a

GRE AWA John박 박정어학원

health risk To support this conclusion the author cites the fact that five of eight chemicalscommonly causing these symptoms were not found in the recalled cans while the other threealso occur naturally in other canned foods For several reasons this evidence lends littlecredible support to the authors conclusionTo begin with the author relies partly on the fact that although three of the eight chemicalsmost commonly blamed for nausea and dizziness appeared in Promofoods recalled tunathese chemicals also occur naturally in other canned foods However this fact alone lends nosupport to the authors conclusion for two reasons First the author might be ignoring animportant distinction between naturally occurring chemicals and those not occurring naturallyIt is entirely possible that these three chemicals do not occur naturally in Promofoods tunaand that it is for this reason that the chemicals cause nausea and dizziness Secondly it isentirely possible that even when they occur naturally these chemicals cause the samesymptoms Unless the author rules out both possibilities he cannot reliably conclude that therecalled tuna would not cause these symptomsAnother problem with the argument is that the authors conclusion is too broad Based onevidence about certain chemicals that might cause two particular heath-related symptoms theauthor concludes that the recalled tuna contains no chemicals that pose a health risk Howeverthe author fails to account for the myriad of other possible health risks that the recalled tunamight potentially pose Without ruling out all other such risks the author cannot justifiablyreach his conclusionA third problem with the argument involves that fact that the eight particular chemicals withwhich the test was concerned are only the eight most commonly blamed for nausea anddizziness It is entirely possibly that other chemicals might also cause these symptoms andthat one or more of these other chemicals actually caused the symptoms Without ruling outthis possibility the author cannot jusufiably conclude that the recalled tuna would not causenausea and dizzinessA final problem with the argument involves thetesting procedure itself The author providesno information about the number of recaUed cans tested or the selection method used Unlessthe number of cans is a sufficiently large sample and is statistically repre sentative of all therecalled cans the studys results are not statistically reliableIn conclusion the article is unconvincing as it stands To strengthen the assertion that therecalled tuna would not cause nausea and dizziness the author must provide evidence thatthe three chemicals mentioned that occur naturally in other canned foods also appear naturallyin Promofoods tuna The author must also provide evidence that ingesting other canned foodscontaining these three chemicals does not cause these symptoms To better evaluate theargument we would need to know whether the sample used in the tests was statisticallysignificant and representative of all the recalled tuna We would also need to know what otherchemicals in the recalled tuna might pose any health risk at all

5그룹 불충분 조건오류 빈출

Natures Way a chain of stores selling health food and other health-related products is opening its next franchise in the town of Plainsville The store should prove to be very successful Natures Way franchises tend to be most profitable in areas where residents lead healthy lives and clearly Plainsville is such an area Plainsville merchants report that sales of running shoes and exercise clothing are at all-time highs The local health club has more members than ever and the weight training and aerobics classes are always full Finally Plainsvilles schoolchildren represent a new generation of potential customers these schoolchildren are required to participate in a fitness-for-life program which emphasizes the benefits of regular exercise at an early age

Write a response in which you examine the stated andor unstated assumptions of the argument Be sure to

GRE AWA John박 박정어학원

explain how the argument depends on these assumptions and what the implications are for the argument if the assumptions prove unwarranted

False cause

First of all the author believes that the Increased sales of running shoes and exercise clothing indicates

Plainesville residentsrsquo interest in leading healthy lives However this assumption is not logically convincing for

several reasons could be a fashion trendTime shift ldquoFitness for liferdquo might not have any influence on schoolchildren as they growFalse cause There could be other reasons for member increase in the health clubAll of the above are insufficient condition

The author has to prove that local residents are interested in leading healthy lives However he supports his conclusion with insufficient evidence Nevertheless even if the residents are concerned with health naturersquos way may not be successful First

그 동안의 경험을 토대로 볼 때 건강생활과 밀접히 관련되어 있는 거주 지역에서 본 상점들이 아주 호응을 얻고 있다 따라서 이러한 주민들이 많이 거주하고 있는 Plainsville 에 새로운 상점들을 계속 세워야 한다 이 지역 상인들은 런닝화와 운동복 판매가 가장 높다고 말한다 불과 5 년전에는 거의 전무하다시피하던 지역 헬스 클럽의 경우도 엄청나게 많은 회원을 확보하고 있으며 웨이트 트레이닝과 에어로빅 강좌들도 항상 만원이라고 한다 새로운 고객층을 예측해 보는 것도 가능하다 이 지역의 학생들의 경우 Fitness for Life프로그램을 받게 되는데 이러한 프로그램을 통해서 유년시절부터 정규적인 운동 습관을 들이게 하고 있는 것이 그것이다

결론 We should therefore build our next new store in Plainsville

반박 그동안의 경험에 의한 과거 통계가 꼭 여기에도 적용되는건 아니다 5 년전 헬스 클럽이 잘 안되었던게 다른 원인이였을수 있다(강사수준 미달 강좌미비)tourist 에 의한 원인 일수 있다 어렸을때부터 운동을 했다고 해서 커서도 관심이 있지는 않다 (오히려 반감이 있을수 있다 혹은 건강하기에 건강에 관심이 적을수도 있다)운동복이나 신발의 판매가 육체노동에 의한 것일수도 있다

IntroductionSupport1049896In this memorandum the author asserts that Naturersquos Way should build its next newstore in Plainsville To support this assertion the author states that Plainsvillesmerchantsrsquo sales of exercise clothing are going well the local health club has moremembers than ever and a new generation of customers will help to ensure NaturersquosWayrsquos success At first glance the authorrsquos assumption seems convincing but in-depth scrutiny revealsthat it lacks substantial evidence as it stands

Body 1-SamplingTopic Sentence 1To begin with the author assumes that the merchantsrsquo report indicates that the residentsare concerned about their health However this assumption is based on unsubstantiated

GRE AWA John박 박정어학원

data Example 1 (Rebuttal1) First if we do not know the total volume of items sold and the price of the goods exactly we cannot infer whether the residents are actually buying many goods Example 2 (Rebuttal2)In addition to that the report emphasizes the rising sales of running shoes and exerciseclothing however these may not be hot-selling items for Naturersquos Way or may not be theproducts the company is planning to sell Concluding Sentence Therefore in order to make the argument reliable the author should reconsider themerchantsrsquo report with more detailed data

Body 2-CausalTopic Sentence 2Second the author contends that the health clubs classes are full yet this does not meanthat many people actually use the health club other factors may be the real cause forthose closed classes Example 1 (Rebuttal1) To begin with if the health club is very small the number of people working out wouldnot be a large one In fact regular gym-going may just be a vogue among a smallunrepresentative segment of Plainsvilles population Example 2 (Rebuttal2) Moreover it is possible that most of the people who exercise in the health club do weight training and aerobics only to look good and to meet other singles not for their health In that case there would be little demand for health products Concluding SentenceThus the author should not hasten to presume what really caused people to be interested in a healthier lifestyle and enroll in the health club

Body 3-Time-ShiftTopic Sentence 3Finally the author highlights that Naturersquos Way can expect a new generation of customersin Plainsville that will help the company in the long term This notion is mistaken in that itassumes the conditions of the present will continue unchanged in the future Although theschool children are required to participate in the fitness for life program they may notnecessarily buy Naturersquos Ways products Example 1 (Rebuttal1) In the first instance they may suffer a fall in purchasing power arising from future economic difficulties this would cause reluctance to spend a considerable amount of money on health products which tend to be more expensive Example 2 (Rebuttal2)Another possibility is that there may emerge many competitor companies vying with Naturersquos Way so that in the future the school children may not feel the necessity to purchase one companyrsquos health products over anotherrsquosConcluding Sentence Thus the authorrsquos assumption is highly speculative since it relies heavily on unknowablefuture circumstances

ConclusionThesis In sum the author uses many assumptions that are insufficient in supporting his claimsSupportIn order for the authorrsquos claims to be convincing he needs to advance more persuasiveevidence that people in Plainsville really are concerned with their health and health foodThe following was written as a part of an application for a small-business loan by a group of developers in the city of Monroe

A jazz music club in Monroe would be a tremendously profitable enterprise Currently the nearest jazz club is 65 miles away thus the proposed new jazz club in Monroe the C-Note would have the local market all to itself Plus jazz is extremely popular in Monroe over 100000 people attended Monroes annual jazz festival last summer several well-known jazz musicians live in Monroe and the highest-rated radio program in Monroe is Jazz Nightly which airs every weeknight at 7 PM Finally a nationwide study indicates that the typical jazz fan spends close to $1000 per year on jazz entertainment

1 Write a response in which you discuss what specific evidence is needed to evaluate the argument and explain how the evidence would weaken or strengthen the argument

2 Write a response in which you examine the stated andor unstated assumptions of the argument Be

GRE AWA John박 박정어학원

sure to explain how the argument depends on these assumptions and what the implications are for the argument if the assumptions prove unwarranted

3 Write a response in which you discuss what questions would need to be answered in order to decide whether the prediction and the argument on which it is based are reasonable Be sure to explain how the answers to these questions would help to evaluate the prediction

Group error nationwide survey may not reflect local trends Is the nationwide jazz fan population substantialInsufficient non-residents of Monroe may have attended the jazz festival (Body alternative explanation last year may have been an anomaly The author should consider data from various years) The author should indicate how many out of 100000 were Monroe residentsNationwide study Does this reflect Insufficient Citizens of Monroe may continue to go to the jazz club 65 miles away

Are the people in Monroe really interested in jazzMajority of the people who attended the jazz festival might not be Monroe residentsSurvey error nationwide study may not be applicable to MonroeJazz musicians who live in MonroeMonopolyRadio station

In this business application the author claims that the proposed jazz club C Note will be very profitable in Monroe To support this claim the author argues for his case with several evidences At first glance the authorrsquos argument seems convincing however careful scrutiny reveals that his argument in specious

To begin with the author claims that Monroersquos citizens are interested in jazz He presents three evidences First Secondhellip Thirdhellip Howeverhellip

Monroe 시에 있는 재즈 음악 클럽은 수익성이 좋은 사업이다 현재 가장 가까이에 있는 클럽은 65 마일 정도 떨어져 있다 따라서 이번에 세우려고 하는 C Note 는 독보적인 위치를 점할것이다 더군다나 재즈는 이 시에서 가장 인기있는 음악이다 지난 여름 재즈 축제에서는 10 만명 이상의 Morone 시 주민이 참석하였고 몇몇 유명한 재즈 음악가들도 이곳에 살고 있으며 저녁때 방영되는 라디오 프로그램중에서 최고의 시청률을 보이고 있는 것도 Jazz Nightly 이다 전국조사에서도 전형적인 재즈 팬들은 재즈 분야에 년간 1천 달러 가까이 지출하고 있는 것으로 보고되고 있다 따라서 C Note 클럽이 돈을 벌 수 있는 사업이라는 것은 확실한 것이다

결과 It is clear that the C Note cannot help but make money반박 nearest jazz club 이 양질의 써비스로 여전히 손님을 끌수도 있다Festival 에 얼마나 참여하는지가 jazz 의 인기를 반영하지 않는다 뮤지션이 많이 사는거랑 jazz 의 인기가 상관없다라디오 프로그램이 다른 요인에 의해서 인기일수도 있다 (진행자때문)전국 통계 적용 불가화목 실전반_Ms Noh6In this application the author suggests that a jazz club in Monroe will make a number of profits To support this suggestion the author exemplifies the local condition popularity of jazz in Monroe and nationwide study However careful scrutiny of each of the facts reveals that it provides little credible support for the authorrsquos recommendation Good clear intro

First the author assumes that jazz is popular in Monroe because of several facts the jazz festival last year had high participation some famous jazz musicians live in Monroe and the high-rated radio program is lsquoJazz Nightlyrsquo However this assumption has many drawbacks that must be seriously considered(Good topic sentences) If many attendants in the last-yearrsquos festival came from other cities and not Monroe it is hard to conclude that Monroersquos people like jazz Therefore the author must examine how many Monroe residents actually attended the festival On top of that there is little relationship between habitation of famous jazz musician and the popularity of jazz in Monroe Although several well-known musicians live there if they do not take part in any jazz performance of Monroe this might have no effect to the interest of Monroersquos residents

GRE AWA John박 박정어학원

about jazz Finally in the case of radio program this is also not suitable reason why jazz is popular in Monroe It might be possible that people cannot help choosing lsquoJazz Nightlyrsquo because there are few radio programs at Night The fact that the radio program is the highest rating program is not a germane evidence The approximate number of listeners would be the more crucial evidence Therefore the author needs to seriously deliberate the correlation between jazzrsquos popularity in Monroe and his examples (Good logical flow and clarity)

Second the author uses as evidence the nationwide study that jazz fans spend much money on jazz entertainment to substantiate why starting a jazz club in Monroe will be profitable In other words the author assumes that the characteristics of a nationwide study can be applied to Monroe The national study would lend support to the applicantrsquos claim only if residents in Monroe typify national jazz fans However the author does not provide credible evidence that this is the case Moreover the populations of jazz fans nationwide may be insubstantial Thus the author should not infer hastily that Monroersquos residents will spend much money on enjoying jazz from the nationwide study

Lastly even if jazz is popular in Monroe C Note may not be successful It is entirely possible that residents might still prefer other clubs where they have always went In addition there is another possibility that the nearest jazz club will attract many of Monroersquos people because it serves fine performances and is equipped with favorite facilities Without considering these other possibilities the author cannot make his argument convincing In sum the author presents many reasons that are insufficient in supporting his or her claim In order for the authorrsquos claims to be convincing he needs to advance more persuasive evidence such as the total number of Monroe residents who attended the jazz festival the effects on the popularity of jazz by the musicians living in Monroe and the actual number of residents who would typify themselves to be jazz fans through a local survey Without substantial evidence that C Note will be successful in Monroe the businessmen may be overinvesting in what might lead to a business failureExcellent clarity Score 50

The following appeared in a newsletter offering advice to investors

Over 80 percent of the respondents to a recent survey indicated a desire to reduce their intake of foods containing fats and cholesterol and today low-fat products abound in many food stores Since many of the food products currently marketed by Old Dairy Industries are high in fat and cholesterol the companys sales are likely to diminish greatly and company profits will no doubt decrease We therefore advise Old Dairy stockholders to sell their shares and other investors not to purchase stock in this company

Write a response in which you discuss what questions would need to be answered in order to decide whether the advice and the argument on which it is based are reasonable Be sure to explain how the answers to these questions would help to evaluate the advice

Survey 80

GRE AWA John박 박정어학원

Old Dairy could change their products and manufacture low fat dairy foodsLess competing companies Old Dairy could eventually be the only company that produces hellipImprecise numbers and measurementsCustomers may still buy high fat dairy products

The author of the newsletter is offering potentially dangerous advice by recommending Old Dairy stockholders to withdraw investment and stop purchase What is more the authorrsquos prediction debases the reputation and business of Old Dairy and if false could devoid the investment opportunity of the newsletter readers Therefore investors should examine whether the authorrsquos evidences are substantial

To begin with the author states that 80 percent of the respondents in a survey indicated a desire to reduce their intake of foods He therefore argues that Old Dairyrsquos high fat and cholesterol products would decrease in sales However the author makes a crucial error in this argument First the author provides no evidence that the surveyrsquos results are statistically reliable Were they representative of all the customers Were they chosen for the survey randomly Furthermore the desire to reduce fat and cholesterol intake is a pervasive trend in todayrsquos opulent society however the author erroneously identifies this as a new phenomenon which will affect consumer trends Second having a desire to reduce fat and cholesterol intake does not necessarily indicate that people who have this desire will actually reduce consuming these types of products It is entirely possible that they may continue buying Old Dairy products for its quality and taste Accordingly the author cannot draw any firm conclusion that people will not buy Old Dairy products Therefore if any of these cases are true the author may be offering investors a detrimental investment advice

최근 조사에 대한 응답자중 80 이상이 자신이 먹는 음식에서 지방과 콜레스테롤의 함유량을 줄이고 싶다고 한다 아울러 요즘은 많은 식료품 가계에서 저지방 제품들을 많이 취급하고 있다 현재 Old Dairy Industries가 판매하고 있는 많은 음식제품들은 지방과 콜레스테롤이 높기 때문에 이 회사의 매출이 격감할 것으로 보이며 당연히 매출이익도 줄어들것이다 따라서 이 회사의 주주들은 주식을 매각하고 다른 주식 투자가들도 이 회사의 주식을 매입하지 않는 것이 좋다

결론 Old Dairy stockholders to sell their shares and other investors not to purchase stock in this company

반박 모든 상품이 다 고 지방 고 칼로리는 아니다(비록 많을지라도) 일부의 식품의 경우 기호에 맞어서 히트해서 전체적인 수입이 증가할 수도 있다국내시장만 생각할 수 없다( 외국시장에서 호황을 누릴수 있다 )입맛이라는게 즉각 바뀌는게 아니다

The following appeared in a letter to the editor of the Balmer Island Gazette

On Balmer Island where mopeds serve as a popular form of transportation the population increases to 100000 during the summer months To reduce the number of accidents involving mopeds and pedestrians the town council of Balmer Island should limit the number of mopeds rented by the islands moped rental companies from 50 per day to 25 per day during the summer season By limiting the number of rentals the town council will attain the 50 percent annual reduction in moped accidents that was achieved last year on the neighboring island of Seaville when Seavilles town council enforced similar limits on moped rentals

1 Write a response in which you discuss what questions would need to be answered in order to decide whether the recommendation is likely to have the predicted result Be sure to explain how the answers to these questions would help to evaluate the recommendation

2 Write a response in which you discuss what questions would need to be answered in order to decide whether the prediction and the argument on which it is based are reasonable Be sure to explain how the answers to these questions would help to evaluate the prediction

3 Write a response in which you examine the stated andor unstated assumptions of the argument Be sure to explain how the argument depends on these assumptions and what the implications are for the argument if the assumptions prove unwarranted

Whatrsquos the actual population of Balmer Island 100000mdashis this a significant increase What kind of accidents Skin abrasions or serious injury And compared to Seaville how serious are the accidents and the actual number of accidents Did Seaville enforce other restrictions like safety signsHow different are the conditions of Balmer

GRE AWA John박 박정어학원

and Seaville regarding population road (safety) conditions topography other town-government regulation How much will the economy of Balmer be affected do to this restriction Could it cause an economic recession due to the fact that these rental companiesrsquo chance to make money is only during the summer thereby weakening the economic infrastructure Are there any other ways that could better alleviate the accident rate

Statistics 50-impreciseAnalogy Balmer compared with TorseauFalse Cause Accidents might have occurred because of reasons other than mopeds False Cause population increase may not be part of the cause of the accidentsOther explanations for the accident pedestrians few road safety regulations narrow roadsThere could be other better solutionshellip

Balmer Island의 인구가 여름철에는 십만명으로 늘어난다 2륜차와 보행자간 사고를 줄이기 위해 시의회는 6개의 자전거를 포함한 2륜차 대여업체에게 이 기간동안에는 대여숫자를 일일 50에서 30으로 제한하도록 할 것이다 대여숫자를 줄임으로써 시 의회는 지난해 이웃한 Torseau섬에서 이와 동일한 규제를 시행해서 50나 줄인 결과를 보고 마찬가지로 50를 줄일수 있다고 확신하고 있다

결론 The town council of Balmer Island should linit the number

반박 보행자의 부실에 의해서 사고가 많이 일어날수도 있다렌탈수의 줄임만이 대책은 아니다(대부분의 사람들이 렌탈 보다는 소유하고 있을 수도 있다)옆섬과는 상황이 다를수도 있다(그 섬에서는 사고의 원인이 많은 자전거 수로 인한것일수있다) 하지만 이 섬은 좁은 도로가 원인일 수도 있고 도로 안전 장치의미비가 원일일수 있다

In this letter the author recommends that Balmer Island should limit the number moped rentals from 50 to 30 per day To support this recommendation the author points out several reasons However careful scrutiny of each of the facts reveals that it is filled with unanswered questions that could significantly weaken the authorrsquos recommendation with loops and holes which are answered

The recommendation depends on the assumption that no alternative means of reducing the number of accidents are available However the author fails to offer any evidence to substantiate this crucial assumption It is highly possible that means other than this would better solve the problem Perhaps they could widen the roads or put-up more safety signs Or perhaps the accidents were due to the lack of skills in which case proper safety training would significantly alleviate the problem Without considering and ruling out these and other alternative means of reducing accidetns the author cannot confidently conclude that merely emulating Torseau would suffice Moreover the author is advising a recommendation which could potentially harm the economy of Balmer Island sincehellip Moreover the Balmer Island should alternative means to reduce accidents because limiting moped rentals during the summer could harm the economy of Balmerhellip

First of all the author believes that increase in population and the number of moped rentals are responsible for the accidents It is entirely possible that other factors are responsible for the accidents Perhaps Balmer Islandrsquos lack of safety signs was a major factor Or maybe the roads are narrow and dangerous on the Island therefore the town council could enforce stricter traffic regulations to alleviate the problem Accordingly if either of these scenarios is true the author cannot draw any firm conclusion that increase in the number of population and moped rentals are the cause of the accidents

The author of this editorial recommends that to reduce accidents involving mopeds andpedestrians Balmer Islands city council should restrict moped rentals to 30 per day down from50 at each of the islands six rental outlets To support this recommendation the author citesthe fact that last year when nearby Torseau Islands town council enforced similar measuresTorseaus rate of moped accidents fell by 50 For several reasons this evidence providesscant support for the authors recommendationTo begin with the author assumes that all other conditions in Balmer that might affect therate of moped-pedestrian accidents will remain unchanged after the restrictions are enactedHowever with a restricted supply of rental mopeds people in Balmer might purchase mopedsinstead Also the number of pedestrians might increase in the future with more pedestriansespecially tourists the risk of moped-pedestrian accidents would probably increase For thatmatter the number of rental outlets might increase to make up for the artificial supplyrestriction per outlet--a likely scenario assuming moped rental demand does not declineWithout considering and ruling out these and other possible changes that might contribute to ahigh incidence of moped-pedestrian accidents the author cannot convince me that theproposed restrictions will necessarily have the desired effect

GRE AWA John박 박정어학원

Next the author fails to consider other possible explanations for the 50 decline inTorseaus moped accident rate last year Perhaps last year Torseau experienced unusually fairweather during which moped accidents are less likely Perhaps fewer tourists visited Tot seanlast year than during most years thereby diminishing the demand for rental mopeds to belowthe allowed limits Perhaps last year some of Torseaus moped rental outlets purchased newmopeds that are safer to drive Or perhaps the restrictions were already in effect but were notenforced until last year In any event a decline in Torseaus moped accident rate during onlyone year is scarcely sufficient to draw any reliable conclusions about what might have causedthe decline or about what the accident rate will be in years aheadAdditionally in asserting that the same phenomenon that caused a 50 decline in mopedaccidents in Torseau would cause a similar decline in Balmer the author relies on what mightamount to an unfair analogy between Balmer and Torseau Perhaps Balmers ability to enforcemoped-rental restrictions does not meet Torseaus ability if not then the mere enactment ofsimilar restrictions in Balmer is no guarantee of a similar result Or perhaps the demand formopeds in Torseau is always greater than in Balmer Specifically if fewer than all availablemopeds are currently rented per day from the average Balmer outlet while in Torseau everyavailable moped is rented each day then the proposed restriction is likely to have less impacton the accident rate in Balmer than in TorseauFinally the author provides no evidence that the same restrictions that served to reduce theincidence of all moped accidents by 50 would also serve to reduce the incidence ofaccidents involving mopeds and pedestrians by 50 Lacking such evidence it is entirelypossible that the number of moped accidents not involving pedestrians decreased by a greaterpercentage while the number of moped-pedestrian accidents decreased by a smallerpercentage or even increased Since the author has not accounted for these possibilities theeditorials recommendation cannot be taken seriouslyIn conclusion the recommendation is not well supported To convince me that the proposedrestriction would achieve the desired outcome the author would have to assure me that nochanges serving to increase Balmers moped-pedestrian accident rate will occur in theforeseeable future The author must also provide dear evidence that last years decline inmoped accidents in Torseau was attributable primarily to its moped rental restrictions ratherthan to one or more other factors In order to better evaluate the recommendation I wouldneed more information comparing the supply of and demand for moped rentals on the twoislands I would also need to know the rate of mopedpedestrian accidents in Torseau both priorto and after the restrictions were enforced in TorseauThe following appeared in a magazine article about planning for retirement

Clearview should be a top choice for anyone seeking a place to retire because it has spectacular natural beauty and a consistent climate Another advantage is that housing costs in Clearview have fallen significantly during the past year and taxes remain lower than those in neighboring towns Moreover Clearviews mayor promises many new programs to improve schools streets and public services And best of all retirees in Clearview can also expect excellent health care as they grow older since the number of physicians in the area is far greater than the national average

Write a response in which you discuss what specific evidence is needed to evaluate the argument and explain how the evidence would weaken or strengthen the argument

-Natural beauty and consistent climate may not be the most wanted qualities-Housing costs could have lowered on a national level wealthy retirees may not care about costs-Taxes may be high compared to the nationrsquos average tax rate-What about other qualities of Clearview Crime rate what qualities would retirees want -If schools streets and public services need improvement then this is proof that the current condition of Clearview is low Or due to budgetary reasons the mayor may not follow-up on his promise because of lowered tax rate -Schools and people who are retired no relationship-Physicians What kind of physicians Number is irrelevant Are these physicians capable of addressing the illnesses of old people

This author argues that anyone seeking a place to retire should choose Clearview To supportthis argument the article cites Clearviews consistent climate and natural beauty its fallinghousing costs its low property taxes compared to nearby towns and the mayors promise toimprove schools streets and services The article also claims that retirees can expectexcellent health care because the number of physicians in Clearview greatly exceeds thenational average This argument is flawed in several critical respectsTo begin with although consistent climate and natural beauty might be attractive to manyretirees these features are probably not important to all retirees For many retirees it isprobably more important to live near relatives or even to enjoy changing seasons Thus I

GRE AWA John박 박정어학원

cannot accept the authors sweeping recommendation for all retirees on this basisAlso Clearviews declining housing costs do not necessarily make Clearview the best placeto retire for two reasons First despite the decline Clearviews housing costs might be highcompared to housing costs in other cities Secondly for wealthier retirees housing costs arenot likely to be a factor in choosing a place to retire Thus the mere fact that housing costshave been in decline lends scant support to the recommendationThe articles reliance on Clearviews property-tax rates is also problematic in two respectsFirst retirees obviously have innumerable choices about where to retire besides Clear viewand nearby towns Secondly for retirees who are well-off financially property taxes are notlikely to be an important concern in choosing a place to retire Thus it is unfair to infer fromClearviews property-tax rates that retirees would prefer ClearviewYet another problem with the argument involves the mayors promises In light of Clearviewslow property-tax rates whether the mayor can follow through on those promises is highlyquestionable Absent any explanation of how the city can spend more money in the areas citedwithout raising property taxes I simply cannot accept the editorials recommendation on thebasis of those promises Besides even if the city makes the improvements promised thoseimprovements--particular the ones to schools--would not necessarily be important to retireesFinally although the number of physicians in Clearview is relatively high the per capitanumber might be relatively low Moreover it would be fairer to compare this per capita numberwith the per capita number for other attractive retirement towns--rather than the nationalaverage After all retirees are likely to place a relatively heavy burden on health-careresources Besides the article provides no assurances that the number of physicians inClearview will remain high in the foreseeable futureIn conclusion the recommendation is poorly supported To strengthen it the author mustconvince me--perhaps by way of a reliable survey--that the key features that the vast majorityof retirees look for in choosing a place to live are consistent climate natural beauty and lowhousing costs The author must also provide better evidence that Clear views property taxesare lower than the those of cities in other areas The author must also explain how the city canmake its promised improvements without raising property taxes Finally to better assess theargument I would need to now how the per capita number of physicians in Clearview wouldcompare to the national average in the futureThe following appeared as a letter to the editor from a Central Plaza store owner

Over the past two years the number of shoppers in Central Plaza has been steadily decreasing while the popularity of skateboarding has increased dramatically Many Central Plaza store owners believe that the decrease in their business is due to the number of skateboard users in the plaza There has also been a dramatic increase in the amount of litter and vandalism throughout the plaza Thus we recommend that the city prohibit skateboarding in Central Plaza If skateboarding is prohibited here we predict that business in Central Plaza will return to its previously high levels

Write a response in which you discuss what questions would need to be answered in order to decide whether the recommendation is likely to have the predicted result Be sure to explain how the answers to these questions would help to evaluate the recommendation

Why two years ago What happened two years ago which started this declineIs the dramatic increase in the ldquopopularityrdquo of skateboarding the cause of the steady decline of shoppers Are there any malls nearby Were there any changes nearby which could affect the decline in customersmdasha big mall perhaps Could the decline be due to the shop ownersHow many skateboarders use the plazaWhere do they skateboardDo they shop and are they customersAre the increase in litter and vandalism due to skateboarders Could this be alleviated by installing CCTVs and hiring security

This editorial concludes that the city should ban skateboarding from its downtown CentralPlaza in order to attract visitors to that area to return the area to its former glory and to makeit a place where people can congregate for fun and relaxation To justify this conclusion theeditorial points out that skateboarders are nearly the only people one sees anymore at CentralPlaza and that the Plaza is littered and its property defaced The editorial also points out thatthe majority of downtown merchants support the skate boarding ban This argument is flawedin several critical respectsFirst the editorials author falsely assumes that a ban on skateboarding is both necessaryand sufficient to achieve the three stated objectives Perhaps the city can achieve thoseobjectives by other means as well--for example by creating a new mall that incorporates anattractive new skateboard park Even if banning skateboarders altogether is necessary to meetthe citys goals the author has not shown that this action by itself would suffice Assuming thatthe Plazas reputation is now tarnished restoring that reputation and in turn enticing peopleback to the Plaza might require additional measures--such as removing litter and graffiti

GRE AWA John박 박정어학원

promoting the Plaza to the public or enticing popular restaurant or retail chains to the PlazaSecondly the editorial assumes too hastily that the Plazas decline is attributable to theskateboarders--rather than to some other phenomenon Perhaps the Plazas primary appeal inits glory days had to do with particular shops or eateries which were eventually replaced byless appealing ones Or perhaps the crime rate in surrounding areas has risen dramatically forreasons unrelated to the skateboarders presence at the Plaza Without ruling out these andother alternative explanations for the Plazas decline the editorials author cannot convince methat a skateboard ban would reverse that declineThirdly the editorials author might be confusing cause with effect--by assuming that theskateboarders caused the abandonment of the Plaza rather than vice versa It is entirelypossible that skateboarders did not frequent the Plaza until it was largely abandoned--andbecause it had been abandoned In fact this scenario makes good sense since skateboardingis most enjoyable where there are few pedestrians or motorists to get in the wayFourth it is unreasonable to infer from the mere fact that most merchants favor the ban thatthe ban would be effective in achieving the citys objectives Admittedly perhaps thesemerchants would be more likely to help dean up the Plaza area and promote their businesseswere the city to act in accordance with their preference Yet lacking any supporting evidencethe author cannot convince me of this Thus the survey amounts to scant evidence at best thatthe proposed ban would carry the intended resultFinally the author recommends a course of action that might actually defeat the citysobjective of providing a fun and relaxing place for people to congregate In my experienceskateboarding contributes to an atmosphere of fun and relaxation for adults and children alikemore so than many other types of ambiance Without considering that continuing to allowskateboarding--or even encouraging this activity--might achieve the citys goal more effectivelythan banning the activity the author cannot convincingly conclude that the ban would be in thecitys best interestsIn sum the argument is a specious one To strengthen it the editorials author must providedear evidence that skateboarding and not some other factor is responsible for the conditionsmarking the Plazas decline The author must also convince me that no alternative means ofrestoring the Plaza are available to the city and that the proposed ban by itself would suffice toattract tourists and restore the Plaza to its former glory Finally to better assess the argument itwould be useful to know the circumstances under which the downtown merchants would bewilling to help the city achieve its objectives

6그룹 약한 비유 빈출

The following recommendation appeared in a memo from the mayor of the town of Hopewell

Two years ago the nearby town of Ocean View built a new municipal golf course and resort hotel During the past two years tourism in Ocean View has increased new businesses have opened there and Ocean Views tax revenues have risen by 30 percent Therefore the best way to improve Hopewells economymdashand generate additional tax revenuesmdashis to build a golf course and resort hotel similar to those in Ocean View

Write a response in which you examine the stated andor unstated assumptions of the argument Be sure to explain how the argument depends on these assumptions and what the implications are for the argument if the assumptions prove unwarranted

GRE AWA John박 박정어학원

Assumptions The author assumes that OVrsquos municipal golf course and resort hotel caused tourism new businesses and increased tax revenues There may be other reasons advertising promo He assumes that this will continueAssumes that Ocean View and Hopewell are similar in many waysmdashthe name suggests otherwise OV may have always been a tourist attractions for its beaches We need to know the topography

2년전 Ocean View 시는 시정 소유 골프 및 휴양지 호텔을 신축했다 그리고 지난 2년동안 이 시의 관광객이 증가했으며 새로운 사업들이 생겨났다 그에따라 시의 세수도 30나 증가했다 Hopewell의 경제를 향상시키고 아울러 세수를 늘릴 수 있는 가장 좋은 방법은 Ocean View에 세워진 것과 같은 골프 시설과 휴양지 호텔을 신축하는 것이다

1 다른 요인으로 관광 산업이 발전했을 수도 있다 문화 유적이 발견이 되었거나 도로의 정비등으로 여행자가 늘었을 수도 있다

2 관광 산업의증가가 늘어난 세수의 원인이 아니라 새로 유입된 인구의 증가나 다른 공장에서 발생한 것일 수 있다

3 2년동안 한참 골프가 붐을 이루었을 수 있다 경제상황이 나빠지거나 다른 레포츠가 인근 지역에 생겨난다면 골프하는 사람이 줄어들 수 있다

In this memo HopeweUs mayor recommends that in order to stimulate the towns economyand boost tax revenues HopeweU should build a new golf course and resort hotel just as thetown of Ocean View did two years ago To support this recommendation the mayor points outthat in Ocean View during the last two years tourism has increased new businesses haveopened and tax revenues have increased by 30 I find the mayors argument unconvincingin several important respectsFirst of all it is possible that the mayor has confused cause with effect respecting the recentdevelopments in Ocean View Perhaps Ocean Views construction of a new golf course andhotel was a response to previous increases in tourism and business development increasesthat have simply continued during the most recent two years Since the mayor has failed toaccount for this possibility the claim that Hopewell would boost its economy by alsoconstructing a golf course and hotel is completely unwarrantedSecondly the mayor fails to account for other possible causes of the trends in Ocean Viewduring the last two years The increase in tourism might have been due to improving economicconditions nationwide or to unusually pleasant weather in the region The new businessesthat have opened in Ocean View might have opened there irrespective of the new golf courseand hotel And the 30 increase in tax revenues might have been the result of an increase intax rates or the addition of a new type of municipal taxWithout ruling out these and other alternative explanations for the three recent trends inOcean View the mayor cannot reasonably infer based on those trends that Hopewellseconomy would benefit by following Ocean Views exampleThirdly even if the recent trends in Ocean View are attributable to the construction of the newgolf course and hotel there the mayor assumes too hastily that the golf course and hotel willcontinue to benefit that towns overall economy The mayor has not accounted for thepossibility that increased tourism will begin to drive residents away during tourist season orthat new business development will result in the towns losing its appeal as a place to visit or tolive Unless the mayor can convince me that these scenarios are unlikely I cannot accept themayors recommendation that Hopewell follow Ocean Views exampleFinally the mayors argument rests on the unsubstantiated assumption that Hopewell andOcean View are sufficiently alike in ways that might affect the economic impact of a new golfcourse and hotel Hopewell might lack the sort of natural environment that would attract moretourists and new businesses to the town--regardless of its new golf course and hotel For thatmatter perhaps Hopewell already contains several resort hotels and golf courses that are notutilized to their capacity If so building yet another golf course and hotel might amount to amisallocation of the towns resources--and actually harm the towns overall economyIn sum the mayors recommendation is not well supported To bolster it the mayor mustprovide better evidence that Ocean Views new golf course and hotel and not some otherphenomenon--has been responsible for boosting Ocean Views economy during the last twoyears To better assess the recommendation I would need to know why Ocean View decidedto construct its new golf course and hotel in the first place--specifically what events prior toconstruction might have prompted that decision I would also need to thoroughly compare

GRE AWA John박 박정어학원

HopeweU with Ocean View--especially in terms of their appeal to tourists and businesses--todetermine whether the same course of action that appears to have boosted Ocean Viewseconomy would also boost Hopewells economy

The following is part of a memorandum from the president of Humana University

Last year the number of students who enrolled in online degree programs offered by nearby Omni University increased by 50 percent During the same year Omni showed a significant decrease from prior years in expenditures for dormitory and classroom space most likely because instruction in the online programs takes place via the Internet In contrast over the past three years enrollment at Humana University has failed to grow and the cost of maintaining buildings has increased along with our budget deficit To address these problems Humana University will begin immediately to create and actively promote online degree programs like those at Omni We predict that instituting these online degree programs will help Humana both increase its total enrollment and solve its budget problems

Write a response in which you discuss what questions would need to be answered in order to decide whether the prediction and the argument on which it is based are reasonable Be sure to explain how the answers to these questions would help to evaluate the prediction

Is Omni University successful due to the online degree program 50 Is the decrease in expenditures for dormitory and classroom space due to the decrease in of on-campus students Which classes were successful Does HU have those classes

Even if the long-distance degree programs at Omni University benefited the school the presidentrsquos recommendation that Human College should emulate Omni University is too hasty First OUrsquos name implies that the school would have more majors than Humanahellip the president should examine which degrees were in the long-distance programhellip

지난해에는 Omni 대학에서 개강했던 원거리 학생 학점 취득 프로그램을 등록했던 학생들의 숫자가 50나 증가했다 같은해 기간동안 Omni 대학에서는 그 전년도부터 기숙사와 학급의 공간 확충을 위한 예산을 대폭 줄였는데 이는 이 원거리 학점 취득 프로그램이 양방향 비디오 컴퓨터 접속을 통해서만 가능한 수업지도 방식이기때문인 것으로 보인다 반면 지난 3개년 동안 Humana 대학에서의 수강률은 감소한데다가 건물

GRE AWA John박 박정어학원

유지비도 올랐다 따라서 Humana대학의 수강을 늘리고 예산손실을 회복하기 위해서는 Omni 대학에서 취한 조치와 같은 능동적인 프로그램을 추진해야 한다

결론 we should initiate and actively promote long-distance degree programs like those at Omni 반박 원거리 학생 취득 프로그램 숫자가 증가한거하고 예산이 줄어드는 것 사이에 연관이 약하다 (causal 학생의 증가로 관리비용 증가할수 있음 원거리 수업가능 장비도입에의한 비용발생)bad analogy(omni university 하고 같은 조건이 아니다 )-gt omni college 가 강좌내용이 좋아서 학생의 등록이 많을수 있다 Humana 대학에서 만들었다 하더라도 인기 없을수 있음다른 요인에 의해서 Humana 대학의 수강 인원이 증가할수 있음(비록 과거엔 인기가 없었을지라도)

The following appeared as part of a business plan developed by the manager of the Rialto Movie Theater

Despite its downtown location the Rialto Movie Theater a local institution for five decades must make big changes or close its doors forever It should follow the example of the new Apex Theater in the mall outside of town When the Apex opened last year it featured a video arcade plush carpeting and seats and a state-of-the-art sound system Furthermore in a recent survey over 85 percent of respondents reported that the high price of newly released movies prevents them from going to the movies more than five times per year Thus if the Rialto intends to hold on to its share of a decreasing pool of moviegoers it must offer the same features as Apex

Write a response in which you discuss what questions would need to be answered in order to decide whether the recommendation is likely to have the predicted result Be sure to explain how the answers to these questions would help to evaluate the recommendation

Before following through this business plan the manager should investigate the cause of Rialtorsquos unsuccessful business

The author provides no evidence that the surveyrsquos results are statistically reliable The surveyrsquos sample of 85 percent must be sufficient in size and representative of overall population of the city where Rialto and Apex is serving Lacking evidence of a sufficiently representative sample the author cannot justifiably rely on the survey to draw any conclusion whatsoever The author does not indicate that Apex is indeed currently successful However even if Apex is enjoying success the argument relies on what might be a false analogy between Rialto and Apex In order for Apex to serve as a model that Rialto should emulate the author must assume that all relevant circumstances are essentially the same However this assumption is unwarranted For example the argument overlooks the face that Apex is located in a strategic placemdashbeside a mall where customers can not only watch a movie but also enjoy shopping Therefore simply changing the facility to that of Apex may not lead to success

The author does not mention whether Apex is successful or not Nevertheless even if Apex is currently successful the argument relies on what might be a false analogy between Rialto and Apex In order for Apex to serve as a model that Rialto should emulate the author must assume that all relevant circumstances are essentially the same However this assumption is unwarranted For example the argument overlooks the fact that these two institutions are located in different locations Rialto in downtown and Apex in a mall outside of town Although Apex opened with state-of-the-art facilities the decisive factor in its success could be due to its strategic location of being in a mall People could enjoy both shopping and movies at one location thus they may prefer Apex over Rialto Furthermore the place where people enjoy leisure activities has shifted in the past decades for most cities from downtown to the suburbs Therefore Rialto may not be successful even if it emulates Apexrsquos facilities A better business plan may be relocating Apex to the thriving section of the downtown

Rialto 극장은 지난 50여년간 지역 회관으로써 시내에 위치해 있으면서도 이제 변화를 꾀하지 않으면 문을 닫을

GRE AWA John박 박정어학원

판이다 이 극장은 시외 쇼핑타운에 새로 들어선 Apex 극장의 사례를 본받아야 했다 Apex가 지난해 개업했을 당시 이 극장은 비디오 아케이드 플러쉬 카펫트 바닥과 좌석 그리고 최신 음향시설을 갖추었다 더군다나 최근 조사에서는 응답자의 85 이상이 새로 출시된 영화 입장료가 비싼 탓으로 지난해보다 5배이상의 관람객이 줄어들었다고 나타났다 따라서 Rialto 극장이 줄어들고 있는 관람객을 뺐기지 않고 유지하려면 Apex와 같은 시설들을 갖추어야 할 것이다주장 리알토 극장이 줄어들고 있는 관람객을 뺐기지 않고 유지하려면 Apex와 같은 시설들을 갖추어야 할 것이다

1 조사에서 응답자가 전체를 대표할 수 없다 2 apex 극장이 좋은 시설을 갖추고 있지만 그로 인해 수익이 많이 발생했다는 말이 없으므로 시설투자를

하고도 좋은 결과를 얻을 수 있을지 그 근거가 미흡하다3 좋은 영화가 출시된다면 입장료가 비싸도 영화관에서 꼭 보려고 할 수 있다 4 rialto 가 시설이 아닌 다른 요인에 의해 장사가 안될수도 있다( 우범 지역이라든지)

The following is a recommendation from the business manager of Monarch Books

Since its opening in Collegeville twenty years ago Monarch Books has developed a large customer base due to its reader-friendly atmosphere and wide selection of books on all subjects Last month Book and Bean a combination bookstore and coffee shop announced its intention to open a Collegeville store Monarch Books should open its own in-store cafeacute in the space currently devoted to childrens books Given recent national census data indicating a significant decline in the percentage of the population under age ten sales of childrens books are likely to decline By replacing its childrens books section with a cafeacute Monarch Books can increase profits and ward off competition from Book and Bean

Write a response in which you examine the stated andor unstated assumptions of the argument Be sure to explain how the argument depends on these assumptions and what the implications are for the argument if the assumptions prove unwarranted

The following is a recommendation from the business manager of Monarch Books

Since its opening in Collegeville twenty years ago Monarch Books has developed a large customer base due to its reader-friendly atmosphere and wide selection of books on all subjects Last month Book and Bean a combination bookstore and coffee shop announced its intention to open a Collegeville store Monarch Books should open its own in-store cafeacute in the space currently devoted to childrens books Given recent national census data indicating a significant decline in the percentage of the population under age ten sales of childrens books are likely to decline By replacing its childrens books section with a cafeacute Monarch Books can increase profits and ward off competition from Book and Bean

1 Write a response in which you discuss what questions would need to be answered in order to decide whether the recommendation is likely to have the predicted result Be sure to explain how the answers to these questions would help to evaluate the recommendation

2 Write a response in which you discuss what specific evidence is needed to evaluate the argument and explain how the evidence would weaken or strengthen the argument

No evidence regarding Monarch Bookrsquos successEven if Regal Bookrsquos is successful this may not be attributable to the cafeacute False analogy Emulating may not lead to success Other factors may be involvedInsufficient condition The national census is not enough evidence that childrenrsquos book sales will decline Can

GRE AWA John박 박정어학원

the national census represent the local child populationDid opening a cafeacute boost sales for Regal Books Even assuming Regal is successful by opening a cafeacute this may not be suitable for Monarch which plans to close the childrenrsquos book section to establish a cafe Imprecise language ldquorelatively little spacerdquo how smallThe managerrsquos recommendation contradicts what he says Since Monarch is popular for its wide selection of books closing a selection which targets a major group of readers may hurt Monarchrsquos salesIs this the best way to compete

When Stanley Park first opened it was the largest most heavily used public park in town It is still the largest park but it is no longer heavily used Video cameras mounted in the parks parking lots last month revealed the parks drop in popularity the recordings showed an average of only 50 cars per day In contrast tiny Carlton Park in the heart of the business district is visited by more than 150 people on a typical weekday An obvious difference is that Carlton Park unlike Stanley Park provides ample seating Thus if Stanley Park is ever to be as popular with our citizens as Carlton Park the town will obviously need to provide more benches thereby converting some of the unused open areas into spaces suitable for socializing

Write a response in which you examine the stated andor unstated assumptions of the argument Be sure to explain how the argument depends on these assumptions and what the implications are for the argument if the assumptions prove unwarranted

Stanley 파크가 처음 개장했을 당시 가장 크고 가장 많이 이용되는 공원이었다 아직도 공원중에서는 가장 크지만 이용률은 상당히 떨어졌다 지난달 공원 주차장에 설치해놓은 비디오 카메라를 통해 보면 drop(주차장으로 여겨짐) 이용률이 가장 높았다 수치상으로는 하루 평균 50대의 차량만이 이용하였다 반면 직장 중심거리에 위치한 작은 규모의 Carlton 파크는 주당 무려 150여명 이상이 이용하고 있다 Stanley 파크와는 달리 Carlton 파크에는 의자가 있다는 것이 가장 뚜렷한 차이점이다 따라서 Stanley 파크가 Carlton 파크처럼 시민들이 자주 이용하는 공원이 되기 위해서는 벤치를 설치할 필요가 있으며 이렇게 사용되지 않는 일부 공간을 활용해서 사교를 위한 공간으로 바꾸어야 한다 ===gtdrop 에 대한 첨부사항 (영영사전내용입니다)---- a place or central depository to which something (as mail money or stolen property) is brought for distribution or transmission also the act of depositing something at such a place dropgt

주장 if Stanley Park is ever to be as popular with our citizens as is Carlton Park the town will obviously need to provide more benches thereby converting some of the unused open areas into spaces suitable for socializing1 조사가 언제 이루어진 것인가 조사가 언제 실시되었느냐에 따라 결과가 다를 수있다 현재는 다시 스탠리 파크가 늘어났었을 수 있다 2 벤치를 많이 설치했다고 해서 많은 관광객이 오지 않을수 있다(사람들이 벤치나 사교 공간을 원한다는 어떠한 자료도 없다)3스탠리 파크 주변에 교통 상황이 악화가 되었거나 칼튼 파크에서 문화행사등을 많이 가져서 이용객이 줄어든것일 수도 있다 4 칼튼 파크가 중심지에 있어서 접근성이 좋을수 있다5 조사가 같은 시간을 기준으로 한게 아니다(하나는 주중이고 하나는 주말이다)6사람의 수와 차의 대수를 같은것으로 비교할수 없다 (차안에 몇 명이 타고 있는지 모르고 대중교통을 이용해서 왔을수도 있다)

Page 23: GRE writing argument brain storm

GRE AWA John박 박정어학원

at the two companies last year were aberrations and during other years Altas accident ratewas no greater or even lower than Panoplys rate Or perhaps Panoply is not representativeof industrial companies generally and that other companies with shorter work shifts have evenhigher accident rates In short since the argument relies on very limited statistical information Icannot take the authors recommendation seriouslyIn conclusion the recommendation for emulating Panoplys work-shift policy is not wellsupported To convince me that shorter work shifts would reduce Altas on-the-job accidentrate the author must provide clear evidence that work-shift length is responsible for some ofAltas accidents The author must also supply evidence to support her final conclusion that alower accident rate would in fact increase overall worker productivity

The following appeared in a memo from the vice president of marketing at Dura-Sock Inc

A recent study of our customers suggests that our company is wasting the money it spends on its patented Endure manufacturing process which ensures that our socks are strong enough to last for two years We have always advertised our use of the Endure process but the new study shows that despite our socks durability our average customer actually purchases new Dura-Socks every three months Furthermore our customers surveyed in our largest market northeastern United States cities say that they most value Dura-Socks stylish appearance and availability in many colors These findings suggest that we can increase our profits by discontinuing use of the Endure manufacturing process

1 Write a response in which you examine the stated andor unstated assumptions of the argument Be sure to explain how the argument depends on these assumptions and what the implications are for the argument if the assumptions prove unwarranted

2 Write a response in which you discuss what specific evidence is needed to evaluate the argument and explain how the evidence would weaken or strengthen the argument

3 Write a response in which you discuss what questions would need to be answered in order to decide whether the recommendation and the argument on which it is based are reasonable Be sure to explain how the answers to these questions would help to evaluate the recommendation

Intro The vice president of marketing at Dura-Sock Inc is offering a potentially harmful investment recommendation by claiming that Dura-Sock should discontinue its use of the ldquoEndurerdquo process To support his recommendation he points out a study that Dura-Sock customers actually purchase the socks every three months and a survey that reveals that Dura-Sock customers like the sockrsquos stylish appearance and availability in many colors The study and survey however are insufficient in supporting his proposal and the VP makes several unwarranted assumptionsIntro (simplified) The VP states that though Dura-Socks last for two years customers buy the socks every three months Therefore he assumes that the consumersrsquo motive for buying the produce is not its durabilityHowever the author fails to rule out other possible motivation for consumption

Even if the survey is reliable the author should consider the rest of the market Vague terms ldquowasting moneyrdquomdashprecisely how much are they wasting Studysurvey errorThe company must calculate the outcome of such momentous decisionStudy participantsrsquo comment that they prefer Dura-Sock for its stylishness and availability might take Dura-Sockrsquos enduring quality for granted

우리회사 제품 소비자들에 대한 최근 조사에서 지난 2년여간 양말의 내구성을 강하게 하는 필수공정이었던 자사 특허의 Endure 공정에 들어가는 비용이 낭비라고 말하고 있다 우리 회사는 항상 이 공정 처리에 대한 광고를 내보냈으나 이에 대한 시장 조사에서 실제로 고객들은 이 신제품을 평균 석달마다 구매하는 것으로 나타났다 더군다나 북동부지역에서 실시한 대규모 시장조사에 응답한 고객들은 양말의 모양과 색상등에 더

GRE AWA John박 박정어학원

관심을 나타냈다 이러한 결과는 우리회사가 신기술 공법을 중단하면 그에 따라 수익이 늘어날 것이라는 것을 말해주고 있는 것이다주장 These findings suggest that Dura0Sock can increase its profits by discontinuing its use of the ldquoEndurerdquo manufacturing process

1 survey가 정확한 소비자의 의견을 나타낸 것인가 다른 선택없이 양자택일과 같은 방법의 survey였는지2 북동부지역의 시장조사가 전체 의견을 대표할 수 있나3 사람들이 모양이나 색상에 앞서 내구성을 먼저 평가했을 수도 있다 내구성을 갖추었다는 전제하에 모양과 색상에 관심을 드러낸 것일 수 있다4 소비자가 도매상(retail)인지 소매상(whole)인지가 없다

The following appeared in a business magazine

As a result of numerous complaints of dizziness and nausea on the part of consumers of Promofoods tuna the company requested that eight million cans of its tuna be returned for testing Promofoods concluded that the canned tuna did not after all pose a health risk This conclusion is based on tests performed on samples of the recalled cans by chemists from Promofoods the chemists found that of the eight food chemicals most commonly blamed for causing symptoms of dizziness and nausea five were not found in any of the tested cans The chemists did find small amounts of the three remaining suspected chemicals but pointed out that these occur naturally in all canned foods

Write a response in which you discuss what questions would need to be addressed in order to decide whether the conclusion and the argument on which it is based are reasonable Be sure to explain how the answers to the questions would help to evaluate the conclusion

Representativeness of the tested cansThey should conduct a comparative studyThe testing could be biased because Promofoods employees conducted the testingHow much (quantity) of the five and three suspected chemicals were in the canned foodsFalse cause The substance that caused dizziness and nausea may not be one of the eight common chemicals

많은 소비자들의 현기증과 구역질 불만에 따라 Promofoods사는 지난해 참치 캔 8백만 개를 테스트하기 위해 반품시켰다 그 결과 캔에서는 건강에 위험이 될 수 있는 화합물질이 없었던 것으로 회사측은 결론지었다 이러한 결론은 회사측 화학연구자들이 회수된 캔의 샘플을 테스트해서 이들 증상의 원인이 되는 8가지 화합물 중에서 5가지가 실험된 캔에서 발견되지 않았다는 사실에 근거한 것이다 이들 화학자들은 나머지 3개가지 화합물이 모든 캔 식료품에서 흔히 발견되는 것이라고 언급했다 결론 Promofoods concluded that the cans did not after all contain chemicals that posed a health risk

1 공인된 기간에서 테스트를 한 것이 아니고 자사에서 직접 테스트를 했기에 신뢰성이 안간다 2 이런 증상을 일으키는 8개의 물질 말고 다른 물질들이 캔속에 많이 포함됬을수 있다 3 나머지 3개의 물질들의 함유량이 많아서 다른 종류의 캔들은 문제를 일으키지 않지만 참치캔은 문제를

일으킬 수 있다

This magazine article concludes that the 8 million cans of tuna Promofoods recalled due tocomplaints about nausea and dizziness do not after ail contain any chemicals that pose a

GRE AWA John박 박정어학원

health risk To support this conclusion the author cites the fact that five of eight chemicalscommonly causing these symptoms were not found in the recalled cans while the other threealso occur naturally in other canned foods For several reasons this evidence lends littlecredible support to the authors conclusionTo begin with the author relies partly on the fact that although three of the eight chemicalsmost commonly blamed for nausea and dizziness appeared in Promofoods recalled tunathese chemicals also occur naturally in other canned foods However this fact alone lends nosupport to the authors conclusion for two reasons First the author might be ignoring animportant distinction between naturally occurring chemicals and those not occurring naturallyIt is entirely possible that these three chemicals do not occur naturally in Promofoods tunaand that it is for this reason that the chemicals cause nausea and dizziness Secondly it isentirely possible that even when they occur naturally these chemicals cause the samesymptoms Unless the author rules out both possibilities he cannot reliably conclude that therecalled tuna would not cause these symptomsAnother problem with the argument is that the authors conclusion is too broad Based onevidence about certain chemicals that might cause two particular heath-related symptoms theauthor concludes that the recalled tuna contains no chemicals that pose a health risk Howeverthe author fails to account for the myriad of other possible health risks that the recalled tunamight potentially pose Without ruling out all other such risks the author cannot justifiablyreach his conclusionA third problem with the argument involves that fact that the eight particular chemicals withwhich the test was concerned are only the eight most commonly blamed for nausea anddizziness It is entirely possibly that other chemicals might also cause these symptoms andthat one or more of these other chemicals actually caused the symptoms Without ruling outthis possibility the author cannot jusufiably conclude that the recalled tuna would not causenausea and dizzinessA final problem with the argument involves thetesting procedure itself The author providesno information about the number of recaUed cans tested or the selection method used Unlessthe number of cans is a sufficiently large sample and is statistically repre sentative of all therecalled cans the studys results are not statistically reliableIn conclusion the article is unconvincing as it stands To strengthen the assertion that therecalled tuna would not cause nausea and dizziness the author must provide evidence thatthe three chemicals mentioned that occur naturally in other canned foods also appear naturallyin Promofoods tuna The author must also provide evidence that ingesting other canned foodscontaining these three chemicals does not cause these symptoms To better evaluate theargument we would need to know whether the sample used in the tests was statisticallysignificant and representative of all the recalled tuna We would also need to know what otherchemicals in the recalled tuna might pose any health risk at all

5그룹 불충분 조건오류 빈출

Natures Way a chain of stores selling health food and other health-related products is opening its next franchise in the town of Plainsville The store should prove to be very successful Natures Way franchises tend to be most profitable in areas where residents lead healthy lives and clearly Plainsville is such an area Plainsville merchants report that sales of running shoes and exercise clothing are at all-time highs The local health club has more members than ever and the weight training and aerobics classes are always full Finally Plainsvilles schoolchildren represent a new generation of potential customers these schoolchildren are required to participate in a fitness-for-life program which emphasizes the benefits of regular exercise at an early age

Write a response in which you examine the stated andor unstated assumptions of the argument Be sure to

GRE AWA John박 박정어학원

explain how the argument depends on these assumptions and what the implications are for the argument if the assumptions prove unwarranted

False cause

First of all the author believes that the Increased sales of running shoes and exercise clothing indicates

Plainesville residentsrsquo interest in leading healthy lives However this assumption is not logically convincing for

several reasons could be a fashion trendTime shift ldquoFitness for liferdquo might not have any influence on schoolchildren as they growFalse cause There could be other reasons for member increase in the health clubAll of the above are insufficient condition

The author has to prove that local residents are interested in leading healthy lives However he supports his conclusion with insufficient evidence Nevertheless even if the residents are concerned with health naturersquos way may not be successful First

그 동안의 경험을 토대로 볼 때 건강생활과 밀접히 관련되어 있는 거주 지역에서 본 상점들이 아주 호응을 얻고 있다 따라서 이러한 주민들이 많이 거주하고 있는 Plainsville 에 새로운 상점들을 계속 세워야 한다 이 지역 상인들은 런닝화와 운동복 판매가 가장 높다고 말한다 불과 5 년전에는 거의 전무하다시피하던 지역 헬스 클럽의 경우도 엄청나게 많은 회원을 확보하고 있으며 웨이트 트레이닝과 에어로빅 강좌들도 항상 만원이라고 한다 새로운 고객층을 예측해 보는 것도 가능하다 이 지역의 학생들의 경우 Fitness for Life프로그램을 받게 되는데 이러한 프로그램을 통해서 유년시절부터 정규적인 운동 습관을 들이게 하고 있는 것이 그것이다

결론 We should therefore build our next new store in Plainsville

반박 그동안의 경험에 의한 과거 통계가 꼭 여기에도 적용되는건 아니다 5 년전 헬스 클럽이 잘 안되었던게 다른 원인이였을수 있다(강사수준 미달 강좌미비)tourist 에 의한 원인 일수 있다 어렸을때부터 운동을 했다고 해서 커서도 관심이 있지는 않다 (오히려 반감이 있을수 있다 혹은 건강하기에 건강에 관심이 적을수도 있다)운동복이나 신발의 판매가 육체노동에 의한 것일수도 있다

IntroductionSupport1049896In this memorandum the author asserts that Naturersquos Way should build its next newstore in Plainsville To support this assertion the author states that Plainsvillesmerchantsrsquo sales of exercise clothing are going well the local health club has moremembers than ever and a new generation of customers will help to ensure NaturersquosWayrsquos success At first glance the authorrsquos assumption seems convincing but in-depth scrutiny revealsthat it lacks substantial evidence as it stands

Body 1-SamplingTopic Sentence 1To begin with the author assumes that the merchantsrsquo report indicates that the residentsare concerned about their health However this assumption is based on unsubstantiated

GRE AWA John박 박정어학원

data Example 1 (Rebuttal1) First if we do not know the total volume of items sold and the price of the goods exactly we cannot infer whether the residents are actually buying many goods Example 2 (Rebuttal2)In addition to that the report emphasizes the rising sales of running shoes and exerciseclothing however these may not be hot-selling items for Naturersquos Way or may not be theproducts the company is planning to sell Concluding Sentence Therefore in order to make the argument reliable the author should reconsider themerchantsrsquo report with more detailed data

Body 2-CausalTopic Sentence 2Second the author contends that the health clubs classes are full yet this does not meanthat many people actually use the health club other factors may be the real cause forthose closed classes Example 1 (Rebuttal1) To begin with if the health club is very small the number of people working out wouldnot be a large one In fact regular gym-going may just be a vogue among a smallunrepresentative segment of Plainsvilles population Example 2 (Rebuttal2) Moreover it is possible that most of the people who exercise in the health club do weight training and aerobics only to look good and to meet other singles not for their health In that case there would be little demand for health products Concluding SentenceThus the author should not hasten to presume what really caused people to be interested in a healthier lifestyle and enroll in the health club

Body 3-Time-ShiftTopic Sentence 3Finally the author highlights that Naturersquos Way can expect a new generation of customersin Plainsville that will help the company in the long term This notion is mistaken in that itassumes the conditions of the present will continue unchanged in the future Although theschool children are required to participate in the fitness for life program they may notnecessarily buy Naturersquos Ways products Example 1 (Rebuttal1) In the first instance they may suffer a fall in purchasing power arising from future economic difficulties this would cause reluctance to spend a considerable amount of money on health products which tend to be more expensive Example 2 (Rebuttal2)Another possibility is that there may emerge many competitor companies vying with Naturersquos Way so that in the future the school children may not feel the necessity to purchase one companyrsquos health products over anotherrsquosConcluding Sentence Thus the authorrsquos assumption is highly speculative since it relies heavily on unknowablefuture circumstances

ConclusionThesis In sum the author uses many assumptions that are insufficient in supporting his claimsSupportIn order for the authorrsquos claims to be convincing he needs to advance more persuasiveevidence that people in Plainsville really are concerned with their health and health foodThe following was written as a part of an application for a small-business loan by a group of developers in the city of Monroe

A jazz music club in Monroe would be a tremendously profitable enterprise Currently the nearest jazz club is 65 miles away thus the proposed new jazz club in Monroe the C-Note would have the local market all to itself Plus jazz is extremely popular in Monroe over 100000 people attended Monroes annual jazz festival last summer several well-known jazz musicians live in Monroe and the highest-rated radio program in Monroe is Jazz Nightly which airs every weeknight at 7 PM Finally a nationwide study indicates that the typical jazz fan spends close to $1000 per year on jazz entertainment

1 Write a response in which you discuss what specific evidence is needed to evaluate the argument and explain how the evidence would weaken or strengthen the argument

2 Write a response in which you examine the stated andor unstated assumptions of the argument Be

GRE AWA John박 박정어학원

sure to explain how the argument depends on these assumptions and what the implications are for the argument if the assumptions prove unwarranted

3 Write a response in which you discuss what questions would need to be answered in order to decide whether the prediction and the argument on which it is based are reasonable Be sure to explain how the answers to these questions would help to evaluate the prediction

Group error nationwide survey may not reflect local trends Is the nationwide jazz fan population substantialInsufficient non-residents of Monroe may have attended the jazz festival (Body alternative explanation last year may have been an anomaly The author should consider data from various years) The author should indicate how many out of 100000 were Monroe residentsNationwide study Does this reflect Insufficient Citizens of Monroe may continue to go to the jazz club 65 miles away

Are the people in Monroe really interested in jazzMajority of the people who attended the jazz festival might not be Monroe residentsSurvey error nationwide study may not be applicable to MonroeJazz musicians who live in MonroeMonopolyRadio station

In this business application the author claims that the proposed jazz club C Note will be very profitable in Monroe To support this claim the author argues for his case with several evidences At first glance the authorrsquos argument seems convincing however careful scrutiny reveals that his argument in specious

To begin with the author claims that Monroersquos citizens are interested in jazz He presents three evidences First Secondhellip Thirdhellip Howeverhellip

Monroe 시에 있는 재즈 음악 클럽은 수익성이 좋은 사업이다 현재 가장 가까이에 있는 클럽은 65 마일 정도 떨어져 있다 따라서 이번에 세우려고 하는 C Note 는 독보적인 위치를 점할것이다 더군다나 재즈는 이 시에서 가장 인기있는 음악이다 지난 여름 재즈 축제에서는 10 만명 이상의 Morone 시 주민이 참석하였고 몇몇 유명한 재즈 음악가들도 이곳에 살고 있으며 저녁때 방영되는 라디오 프로그램중에서 최고의 시청률을 보이고 있는 것도 Jazz Nightly 이다 전국조사에서도 전형적인 재즈 팬들은 재즈 분야에 년간 1천 달러 가까이 지출하고 있는 것으로 보고되고 있다 따라서 C Note 클럽이 돈을 벌 수 있는 사업이라는 것은 확실한 것이다

결과 It is clear that the C Note cannot help but make money반박 nearest jazz club 이 양질의 써비스로 여전히 손님을 끌수도 있다Festival 에 얼마나 참여하는지가 jazz 의 인기를 반영하지 않는다 뮤지션이 많이 사는거랑 jazz 의 인기가 상관없다라디오 프로그램이 다른 요인에 의해서 인기일수도 있다 (진행자때문)전국 통계 적용 불가화목 실전반_Ms Noh6In this application the author suggests that a jazz club in Monroe will make a number of profits To support this suggestion the author exemplifies the local condition popularity of jazz in Monroe and nationwide study However careful scrutiny of each of the facts reveals that it provides little credible support for the authorrsquos recommendation Good clear intro

First the author assumes that jazz is popular in Monroe because of several facts the jazz festival last year had high participation some famous jazz musicians live in Monroe and the high-rated radio program is lsquoJazz Nightlyrsquo However this assumption has many drawbacks that must be seriously considered(Good topic sentences) If many attendants in the last-yearrsquos festival came from other cities and not Monroe it is hard to conclude that Monroersquos people like jazz Therefore the author must examine how many Monroe residents actually attended the festival On top of that there is little relationship between habitation of famous jazz musician and the popularity of jazz in Monroe Although several well-known musicians live there if they do not take part in any jazz performance of Monroe this might have no effect to the interest of Monroersquos residents

GRE AWA John박 박정어학원

about jazz Finally in the case of radio program this is also not suitable reason why jazz is popular in Monroe It might be possible that people cannot help choosing lsquoJazz Nightlyrsquo because there are few radio programs at Night The fact that the radio program is the highest rating program is not a germane evidence The approximate number of listeners would be the more crucial evidence Therefore the author needs to seriously deliberate the correlation between jazzrsquos popularity in Monroe and his examples (Good logical flow and clarity)

Second the author uses as evidence the nationwide study that jazz fans spend much money on jazz entertainment to substantiate why starting a jazz club in Monroe will be profitable In other words the author assumes that the characteristics of a nationwide study can be applied to Monroe The national study would lend support to the applicantrsquos claim only if residents in Monroe typify national jazz fans However the author does not provide credible evidence that this is the case Moreover the populations of jazz fans nationwide may be insubstantial Thus the author should not infer hastily that Monroersquos residents will spend much money on enjoying jazz from the nationwide study

Lastly even if jazz is popular in Monroe C Note may not be successful It is entirely possible that residents might still prefer other clubs where they have always went In addition there is another possibility that the nearest jazz club will attract many of Monroersquos people because it serves fine performances and is equipped with favorite facilities Without considering these other possibilities the author cannot make his argument convincing In sum the author presents many reasons that are insufficient in supporting his or her claim In order for the authorrsquos claims to be convincing he needs to advance more persuasive evidence such as the total number of Monroe residents who attended the jazz festival the effects on the popularity of jazz by the musicians living in Monroe and the actual number of residents who would typify themselves to be jazz fans through a local survey Without substantial evidence that C Note will be successful in Monroe the businessmen may be overinvesting in what might lead to a business failureExcellent clarity Score 50

The following appeared in a newsletter offering advice to investors

Over 80 percent of the respondents to a recent survey indicated a desire to reduce their intake of foods containing fats and cholesterol and today low-fat products abound in many food stores Since many of the food products currently marketed by Old Dairy Industries are high in fat and cholesterol the companys sales are likely to diminish greatly and company profits will no doubt decrease We therefore advise Old Dairy stockholders to sell their shares and other investors not to purchase stock in this company

Write a response in which you discuss what questions would need to be answered in order to decide whether the advice and the argument on which it is based are reasonable Be sure to explain how the answers to these questions would help to evaluate the advice

Survey 80

GRE AWA John박 박정어학원

Old Dairy could change their products and manufacture low fat dairy foodsLess competing companies Old Dairy could eventually be the only company that produces hellipImprecise numbers and measurementsCustomers may still buy high fat dairy products

The author of the newsletter is offering potentially dangerous advice by recommending Old Dairy stockholders to withdraw investment and stop purchase What is more the authorrsquos prediction debases the reputation and business of Old Dairy and if false could devoid the investment opportunity of the newsletter readers Therefore investors should examine whether the authorrsquos evidences are substantial

To begin with the author states that 80 percent of the respondents in a survey indicated a desire to reduce their intake of foods He therefore argues that Old Dairyrsquos high fat and cholesterol products would decrease in sales However the author makes a crucial error in this argument First the author provides no evidence that the surveyrsquos results are statistically reliable Were they representative of all the customers Were they chosen for the survey randomly Furthermore the desire to reduce fat and cholesterol intake is a pervasive trend in todayrsquos opulent society however the author erroneously identifies this as a new phenomenon which will affect consumer trends Second having a desire to reduce fat and cholesterol intake does not necessarily indicate that people who have this desire will actually reduce consuming these types of products It is entirely possible that they may continue buying Old Dairy products for its quality and taste Accordingly the author cannot draw any firm conclusion that people will not buy Old Dairy products Therefore if any of these cases are true the author may be offering investors a detrimental investment advice

최근 조사에 대한 응답자중 80 이상이 자신이 먹는 음식에서 지방과 콜레스테롤의 함유량을 줄이고 싶다고 한다 아울러 요즘은 많은 식료품 가계에서 저지방 제품들을 많이 취급하고 있다 현재 Old Dairy Industries가 판매하고 있는 많은 음식제품들은 지방과 콜레스테롤이 높기 때문에 이 회사의 매출이 격감할 것으로 보이며 당연히 매출이익도 줄어들것이다 따라서 이 회사의 주주들은 주식을 매각하고 다른 주식 투자가들도 이 회사의 주식을 매입하지 않는 것이 좋다

결론 Old Dairy stockholders to sell their shares and other investors not to purchase stock in this company

반박 모든 상품이 다 고 지방 고 칼로리는 아니다(비록 많을지라도) 일부의 식품의 경우 기호에 맞어서 히트해서 전체적인 수입이 증가할 수도 있다국내시장만 생각할 수 없다( 외국시장에서 호황을 누릴수 있다 )입맛이라는게 즉각 바뀌는게 아니다

The following appeared in a letter to the editor of the Balmer Island Gazette

On Balmer Island where mopeds serve as a popular form of transportation the population increases to 100000 during the summer months To reduce the number of accidents involving mopeds and pedestrians the town council of Balmer Island should limit the number of mopeds rented by the islands moped rental companies from 50 per day to 25 per day during the summer season By limiting the number of rentals the town council will attain the 50 percent annual reduction in moped accidents that was achieved last year on the neighboring island of Seaville when Seavilles town council enforced similar limits on moped rentals

1 Write a response in which you discuss what questions would need to be answered in order to decide whether the recommendation is likely to have the predicted result Be sure to explain how the answers to these questions would help to evaluate the recommendation

2 Write a response in which you discuss what questions would need to be answered in order to decide whether the prediction and the argument on which it is based are reasonable Be sure to explain how the answers to these questions would help to evaluate the prediction

3 Write a response in which you examine the stated andor unstated assumptions of the argument Be sure to explain how the argument depends on these assumptions and what the implications are for the argument if the assumptions prove unwarranted

Whatrsquos the actual population of Balmer Island 100000mdashis this a significant increase What kind of accidents Skin abrasions or serious injury And compared to Seaville how serious are the accidents and the actual number of accidents Did Seaville enforce other restrictions like safety signsHow different are the conditions of Balmer

GRE AWA John박 박정어학원

and Seaville regarding population road (safety) conditions topography other town-government regulation How much will the economy of Balmer be affected do to this restriction Could it cause an economic recession due to the fact that these rental companiesrsquo chance to make money is only during the summer thereby weakening the economic infrastructure Are there any other ways that could better alleviate the accident rate

Statistics 50-impreciseAnalogy Balmer compared with TorseauFalse Cause Accidents might have occurred because of reasons other than mopeds False Cause population increase may not be part of the cause of the accidentsOther explanations for the accident pedestrians few road safety regulations narrow roadsThere could be other better solutionshellip

Balmer Island의 인구가 여름철에는 십만명으로 늘어난다 2륜차와 보행자간 사고를 줄이기 위해 시의회는 6개의 자전거를 포함한 2륜차 대여업체에게 이 기간동안에는 대여숫자를 일일 50에서 30으로 제한하도록 할 것이다 대여숫자를 줄임으로써 시 의회는 지난해 이웃한 Torseau섬에서 이와 동일한 규제를 시행해서 50나 줄인 결과를 보고 마찬가지로 50를 줄일수 있다고 확신하고 있다

결론 The town council of Balmer Island should linit the number

반박 보행자의 부실에 의해서 사고가 많이 일어날수도 있다렌탈수의 줄임만이 대책은 아니다(대부분의 사람들이 렌탈 보다는 소유하고 있을 수도 있다)옆섬과는 상황이 다를수도 있다(그 섬에서는 사고의 원인이 많은 자전거 수로 인한것일수있다) 하지만 이 섬은 좁은 도로가 원인일 수도 있고 도로 안전 장치의미비가 원일일수 있다

In this letter the author recommends that Balmer Island should limit the number moped rentals from 50 to 30 per day To support this recommendation the author points out several reasons However careful scrutiny of each of the facts reveals that it is filled with unanswered questions that could significantly weaken the authorrsquos recommendation with loops and holes which are answered

The recommendation depends on the assumption that no alternative means of reducing the number of accidents are available However the author fails to offer any evidence to substantiate this crucial assumption It is highly possible that means other than this would better solve the problem Perhaps they could widen the roads or put-up more safety signs Or perhaps the accidents were due to the lack of skills in which case proper safety training would significantly alleviate the problem Without considering and ruling out these and other alternative means of reducing accidetns the author cannot confidently conclude that merely emulating Torseau would suffice Moreover the author is advising a recommendation which could potentially harm the economy of Balmer Island sincehellip Moreover the Balmer Island should alternative means to reduce accidents because limiting moped rentals during the summer could harm the economy of Balmerhellip

First of all the author believes that increase in population and the number of moped rentals are responsible for the accidents It is entirely possible that other factors are responsible for the accidents Perhaps Balmer Islandrsquos lack of safety signs was a major factor Or maybe the roads are narrow and dangerous on the Island therefore the town council could enforce stricter traffic regulations to alleviate the problem Accordingly if either of these scenarios is true the author cannot draw any firm conclusion that increase in the number of population and moped rentals are the cause of the accidents

The author of this editorial recommends that to reduce accidents involving mopeds andpedestrians Balmer Islands city council should restrict moped rentals to 30 per day down from50 at each of the islands six rental outlets To support this recommendation the author citesthe fact that last year when nearby Torseau Islands town council enforced similar measuresTorseaus rate of moped accidents fell by 50 For several reasons this evidence providesscant support for the authors recommendationTo begin with the author assumes that all other conditions in Balmer that might affect therate of moped-pedestrian accidents will remain unchanged after the restrictions are enactedHowever with a restricted supply of rental mopeds people in Balmer might purchase mopedsinstead Also the number of pedestrians might increase in the future with more pedestriansespecially tourists the risk of moped-pedestrian accidents would probably increase For thatmatter the number of rental outlets might increase to make up for the artificial supplyrestriction per outlet--a likely scenario assuming moped rental demand does not declineWithout considering and ruling out these and other possible changes that might contribute to ahigh incidence of moped-pedestrian accidents the author cannot convince me that theproposed restrictions will necessarily have the desired effect

GRE AWA John박 박정어학원

Next the author fails to consider other possible explanations for the 50 decline inTorseaus moped accident rate last year Perhaps last year Torseau experienced unusually fairweather during which moped accidents are less likely Perhaps fewer tourists visited Tot seanlast year than during most years thereby diminishing the demand for rental mopeds to belowthe allowed limits Perhaps last year some of Torseaus moped rental outlets purchased newmopeds that are safer to drive Or perhaps the restrictions were already in effect but were notenforced until last year In any event a decline in Torseaus moped accident rate during onlyone year is scarcely sufficient to draw any reliable conclusions about what might have causedthe decline or about what the accident rate will be in years aheadAdditionally in asserting that the same phenomenon that caused a 50 decline in mopedaccidents in Torseau would cause a similar decline in Balmer the author relies on what mightamount to an unfair analogy between Balmer and Torseau Perhaps Balmers ability to enforcemoped-rental restrictions does not meet Torseaus ability if not then the mere enactment ofsimilar restrictions in Balmer is no guarantee of a similar result Or perhaps the demand formopeds in Torseau is always greater than in Balmer Specifically if fewer than all availablemopeds are currently rented per day from the average Balmer outlet while in Torseau everyavailable moped is rented each day then the proposed restriction is likely to have less impacton the accident rate in Balmer than in TorseauFinally the author provides no evidence that the same restrictions that served to reduce theincidence of all moped accidents by 50 would also serve to reduce the incidence ofaccidents involving mopeds and pedestrians by 50 Lacking such evidence it is entirelypossible that the number of moped accidents not involving pedestrians decreased by a greaterpercentage while the number of moped-pedestrian accidents decreased by a smallerpercentage or even increased Since the author has not accounted for these possibilities theeditorials recommendation cannot be taken seriouslyIn conclusion the recommendation is not well supported To convince me that the proposedrestriction would achieve the desired outcome the author would have to assure me that nochanges serving to increase Balmers moped-pedestrian accident rate will occur in theforeseeable future The author must also provide dear evidence that last years decline inmoped accidents in Torseau was attributable primarily to its moped rental restrictions ratherthan to one or more other factors In order to better evaluate the recommendation I wouldneed more information comparing the supply of and demand for moped rentals on the twoislands I would also need to know the rate of mopedpedestrian accidents in Torseau both priorto and after the restrictions were enforced in TorseauThe following appeared in a magazine article about planning for retirement

Clearview should be a top choice for anyone seeking a place to retire because it has spectacular natural beauty and a consistent climate Another advantage is that housing costs in Clearview have fallen significantly during the past year and taxes remain lower than those in neighboring towns Moreover Clearviews mayor promises many new programs to improve schools streets and public services And best of all retirees in Clearview can also expect excellent health care as they grow older since the number of physicians in the area is far greater than the national average

Write a response in which you discuss what specific evidence is needed to evaluate the argument and explain how the evidence would weaken or strengthen the argument

-Natural beauty and consistent climate may not be the most wanted qualities-Housing costs could have lowered on a national level wealthy retirees may not care about costs-Taxes may be high compared to the nationrsquos average tax rate-What about other qualities of Clearview Crime rate what qualities would retirees want -If schools streets and public services need improvement then this is proof that the current condition of Clearview is low Or due to budgetary reasons the mayor may not follow-up on his promise because of lowered tax rate -Schools and people who are retired no relationship-Physicians What kind of physicians Number is irrelevant Are these physicians capable of addressing the illnesses of old people

This author argues that anyone seeking a place to retire should choose Clearview To supportthis argument the article cites Clearviews consistent climate and natural beauty its fallinghousing costs its low property taxes compared to nearby towns and the mayors promise toimprove schools streets and services The article also claims that retirees can expectexcellent health care because the number of physicians in Clearview greatly exceeds thenational average This argument is flawed in several critical respectsTo begin with although consistent climate and natural beauty might be attractive to manyretirees these features are probably not important to all retirees For many retirees it isprobably more important to live near relatives or even to enjoy changing seasons Thus I

GRE AWA John박 박정어학원

cannot accept the authors sweeping recommendation for all retirees on this basisAlso Clearviews declining housing costs do not necessarily make Clearview the best placeto retire for two reasons First despite the decline Clearviews housing costs might be highcompared to housing costs in other cities Secondly for wealthier retirees housing costs arenot likely to be a factor in choosing a place to retire Thus the mere fact that housing costshave been in decline lends scant support to the recommendationThe articles reliance on Clearviews property-tax rates is also problematic in two respectsFirst retirees obviously have innumerable choices about where to retire besides Clear viewand nearby towns Secondly for retirees who are well-off financially property taxes are notlikely to be an important concern in choosing a place to retire Thus it is unfair to infer fromClearviews property-tax rates that retirees would prefer ClearviewYet another problem with the argument involves the mayors promises In light of Clearviewslow property-tax rates whether the mayor can follow through on those promises is highlyquestionable Absent any explanation of how the city can spend more money in the areas citedwithout raising property taxes I simply cannot accept the editorials recommendation on thebasis of those promises Besides even if the city makes the improvements promised thoseimprovements--particular the ones to schools--would not necessarily be important to retireesFinally although the number of physicians in Clearview is relatively high the per capitanumber might be relatively low Moreover it would be fairer to compare this per capita numberwith the per capita number for other attractive retirement towns--rather than the nationalaverage After all retirees are likely to place a relatively heavy burden on health-careresources Besides the article provides no assurances that the number of physicians inClearview will remain high in the foreseeable futureIn conclusion the recommendation is poorly supported To strengthen it the author mustconvince me--perhaps by way of a reliable survey--that the key features that the vast majorityof retirees look for in choosing a place to live are consistent climate natural beauty and lowhousing costs The author must also provide better evidence that Clear views property taxesare lower than the those of cities in other areas The author must also explain how the city canmake its promised improvements without raising property taxes Finally to better assess theargument I would need to now how the per capita number of physicians in Clearview wouldcompare to the national average in the futureThe following appeared as a letter to the editor from a Central Plaza store owner

Over the past two years the number of shoppers in Central Plaza has been steadily decreasing while the popularity of skateboarding has increased dramatically Many Central Plaza store owners believe that the decrease in their business is due to the number of skateboard users in the plaza There has also been a dramatic increase in the amount of litter and vandalism throughout the plaza Thus we recommend that the city prohibit skateboarding in Central Plaza If skateboarding is prohibited here we predict that business in Central Plaza will return to its previously high levels

Write a response in which you discuss what questions would need to be answered in order to decide whether the recommendation is likely to have the predicted result Be sure to explain how the answers to these questions would help to evaluate the recommendation

Why two years ago What happened two years ago which started this declineIs the dramatic increase in the ldquopopularityrdquo of skateboarding the cause of the steady decline of shoppers Are there any malls nearby Were there any changes nearby which could affect the decline in customersmdasha big mall perhaps Could the decline be due to the shop ownersHow many skateboarders use the plazaWhere do they skateboardDo they shop and are they customersAre the increase in litter and vandalism due to skateboarders Could this be alleviated by installing CCTVs and hiring security

This editorial concludes that the city should ban skateboarding from its downtown CentralPlaza in order to attract visitors to that area to return the area to its former glory and to makeit a place where people can congregate for fun and relaxation To justify this conclusion theeditorial points out that skateboarders are nearly the only people one sees anymore at CentralPlaza and that the Plaza is littered and its property defaced The editorial also points out thatthe majority of downtown merchants support the skate boarding ban This argument is flawedin several critical respectsFirst the editorials author falsely assumes that a ban on skateboarding is both necessaryand sufficient to achieve the three stated objectives Perhaps the city can achieve thoseobjectives by other means as well--for example by creating a new mall that incorporates anattractive new skateboard park Even if banning skateboarders altogether is necessary to meetthe citys goals the author has not shown that this action by itself would suffice Assuming thatthe Plazas reputation is now tarnished restoring that reputation and in turn enticing peopleback to the Plaza might require additional measures--such as removing litter and graffiti

GRE AWA John박 박정어학원

promoting the Plaza to the public or enticing popular restaurant or retail chains to the PlazaSecondly the editorial assumes too hastily that the Plazas decline is attributable to theskateboarders--rather than to some other phenomenon Perhaps the Plazas primary appeal inits glory days had to do with particular shops or eateries which were eventually replaced byless appealing ones Or perhaps the crime rate in surrounding areas has risen dramatically forreasons unrelated to the skateboarders presence at the Plaza Without ruling out these andother alternative explanations for the Plazas decline the editorials author cannot convince methat a skateboard ban would reverse that declineThirdly the editorials author might be confusing cause with effect--by assuming that theskateboarders caused the abandonment of the Plaza rather than vice versa It is entirelypossible that skateboarders did not frequent the Plaza until it was largely abandoned--andbecause it had been abandoned In fact this scenario makes good sense since skateboardingis most enjoyable where there are few pedestrians or motorists to get in the wayFourth it is unreasonable to infer from the mere fact that most merchants favor the ban thatthe ban would be effective in achieving the citys objectives Admittedly perhaps thesemerchants would be more likely to help dean up the Plaza area and promote their businesseswere the city to act in accordance with their preference Yet lacking any supporting evidencethe author cannot convince me of this Thus the survey amounts to scant evidence at best thatthe proposed ban would carry the intended resultFinally the author recommends a course of action that might actually defeat the citysobjective of providing a fun and relaxing place for people to congregate In my experienceskateboarding contributes to an atmosphere of fun and relaxation for adults and children alikemore so than many other types of ambiance Without considering that continuing to allowskateboarding--or even encouraging this activity--might achieve the citys goal more effectivelythan banning the activity the author cannot convincingly conclude that the ban would be in thecitys best interestsIn sum the argument is a specious one To strengthen it the editorials author must providedear evidence that skateboarding and not some other factor is responsible for the conditionsmarking the Plazas decline The author must also convince me that no alternative means ofrestoring the Plaza are available to the city and that the proposed ban by itself would suffice toattract tourists and restore the Plaza to its former glory Finally to better assess the argument itwould be useful to know the circumstances under which the downtown merchants would bewilling to help the city achieve its objectives

6그룹 약한 비유 빈출

The following recommendation appeared in a memo from the mayor of the town of Hopewell

Two years ago the nearby town of Ocean View built a new municipal golf course and resort hotel During the past two years tourism in Ocean View has increased new businesses have opened there and Ocean Views tax revenues have risen by 30 percent Therefore the best way to improve Hopewells economymdashand generate additional tax revenuesmdashis to build a golf course and resort hotel similar to those in Ocean View

Write a response in which you examine the stated andor unstated assumptions of the argument Be sure to explain how the argument depends on these assumptions and what the implications are for the argument if the assumptions prove unwarranted

GRE AWA John박 박정어학원

Assumptions The author assumes that OVrsquos municipal golf course and resort hotel caused tourism new businesses and increased tax revenues There may be other reasons advertising promo He assumes that this will continueAssumes that Ocean View and Hopewell are similar in many waysmdashthe name suggests otherwise OV may have always been a tourist attractions for its beaches We need to know the topography

2년전 Ocean View 시는 시정 소유 골프 및 휴양지 호텔을 신축했다 그리고 지난 2년동안 이 시의 관광객이 증가했으며 새로운 사업들이 생겨났다 그에따라 시의 세수도 30나 증가했다 Hopewell의 경제를 향상시키고 아울러 세수를 늘릴 수 있는 가장 좋은 방법은 Ocean View에 세워진 것과 같은 골프 시설과 휴양지 호텔을 신축하는 것이다

1 다른 요인으로 관광 산업이 발전했을 수도 있다 문화 유적이 발견이 되었거나 도로의 정비등으로 여행자가 늘었을 수도 있다

2 관광 산업의증가가 늘어난 세수의 원인이 아니라 새로 유입된 인구의 증가나 다른 공장에서 발생한 것일 수 있다

3 2년동안 한참 골프가 붐을 이루었을 수 있다 경제상황이 나빠지거나 다른 레포츠가 인근 지역에 생겨난다면 골프하는 사람이 줄어들 수 있다

In this memo HopeweUs mayor recommends that in order to stimulate the towns economyand boost tax revenues HopeweU should build a new golf course and resort hotel just as thetown of Ocean View did two years ago To support this recommendation the mayor points outthat in Ocean View during the last two years tourism has increased new businesses haveopened and tax revenues have increased by 30 I find the mayors argument unconvincingin several important respectsFirst of all it is possible that the mayor has confused cause with effect respecting the recentdevelopments in Ocean View Perhaps Ocean Views construction of a new golf course andhotel was a response to previous increases in tourism and business development increasesthat have simply continued during the most recent two years Since the mayor has failed toaccount for this possibility the claim that Hopewell would boost its economy by alsoconstructing a golf course and hotel is completely unwarrantedSecondly the mayor fails to account for other possible causes of the trends in Ocean Viewduring the last two years The increase in tourism might have been due to improving economicconditions nationwide or to unusually pleasant weather in the region The new businessesthat have opened in Ocean View might have opened there irrespective of the new golf courseand hotel And the 30 increase in tax revenues might have been the result of an increase intax rates or the addition of a new type of municipal taxWithout ruling out these and other alternative explanations for the three recent trends inOcean View the mayor cannot reasonably infer based on those trends that Hopewellseconomy would benefit by following Ocean Views exampleThirdly even if the recent trends in Ocean View are attributable to the construction of the newgolf course and hotel there the mayor assumes too hastily that the golf course and hotel willcontinue to benefit that towns overall economy The mayor has not accounted for thepossibility that increased tourism will begin to drive residents away during tourist season orthat new business development will result in the towns losing its appeal as a place to visit or tolive Unless the mayor can convince me that these scenarios are unlikely I cannot accept themayors recommendation that Hopewell follow Ocean Views exampleFinally the mayors argument rests on the unsubstantiated assumption that Hopewell andOcean View are sufficiently alike in ways that might affect the economic impact of a new golfcourse and hotel Hopewell might lack the sort of natural environment that would attract moretourists and new businesses to the town--regardless of its new golf course and hotel For thatmatter perhaps Hopewell already contains several resort hotels and golf courses that are notutilized to their capacity If so building yet another golf course and hotel might amount to amisallocation of the towns resources--and actually harm the towns overall economyIn sum the mayors recommendation is not well supported To bolster it the mayor mustprovide better evidence that Ocean Views new golf course and hotel and not some otherphenomenon--has been responsible for boosting Ocean Views economy during the last twoyears To better assess the recommendation I would need to know why Ocean View decidedto construct its new golf course and hotel in the first place--specifically what events prior toconstruction might have prompted that decision I would also need to thoroughly compare

GRE AWA John박 박정어학원

HopeweU with Ocean View--especially in terms of their appeal to tourists and businesses--todetermine whether the same course of action that appears to have boosted Ocean Viewseconomy would also boost Hopewells economy

The following is part of a memorandum from the president of Humana University

Last year the number of students who enrolled in online degree programs offered by nearby Omni University increased by 50 percent During the same year Omni showed a significant decrease from prior years in expenditures for dormitory and classroom space most likely because instruction in the online programs takes place via the Internet In contrast over the past three years enrollment at Humana University has failed to grow and the cost of maintaining buildings has increased along with our budget deficit To address these problems Humana University will begin immediately to create and actively promote online degree programs like those at Omni We predict that instituting these online degree programs will help Humana both increase its total enrollment and solve its budget problems

Write a response in which you discuss what questions would need to be answered in order to decide whether the prediction and the argument on which it is based are reasonable Be sure to explain how the answers to these questions would help to evaluate the prediction

Is Omni University successful due to the online degree program 50 Is the decrease in expenditures for dormitory and classroom space due to the decrease in of on-campus students Which classes were successful Does HU have those classes

Even if the long-distance degree programs at Omni University benefited the school the presidentrsquos recommendation that Human College should emulate Omni University is too hasty First OUrsquos name implies that the school would have more majors than Humanahellip the president should examine which degrees were in the long-distance programhellip

지난해에는 Omni 대학에서 개강했던 원거리 학생 학점 취득 프로그램을 등록했던 학생들의 숫자가 50나 증가했다 같은해 기간동안 Omni 대학에서는 그 전년도부터 기숙사와 학급의 공간 확충을 위한 예산을 대폭 줄였는데 이는 이 원거리 학점 취득 프로그램이 양방향 비디오 컴퓨터 접속을 통해서만 가능한 수업지도 방식이기때문인 것으로 보인다 반면 지난 3개년 동안 Humana 대학에서의 수강률은 감소한데다가 건물

GRE AWA John박 박정어학원

유지비도 올랐다 따라서 Humana대학의 수강을 늘리고 예산손실을 회복하기 위해서는 Omni 대학에서 취한 조치와 같은 능동적인 프로그램을 추진해야 한다

결론 we should initiate and actively promote long-distance degree programs like those at Omni 반박 원거리 학생 취득 프로그램 숫자가 증가한거하고 예산이 줄어드는 것 사이에 연관이 약하다 (causal 학생의 증가로 관리비용 증가할수 있음 원거리 수업가능 장비도입에의한 비용발생)bad analogy(omni university 하고 같은 조건이 아니다 )-gt omni college 가 강좌내용이 좋아서 학생의 등록이 많을수 있다 Humana 대학에서 만들었다 하더라도 인기 없을수 있음다른 요인에 의해서 Humana 대학의 수강 인원이 증가할수 있음(비록 과거엔 인기가 없었을지라도)

The following appeared as part of a business plan developed by the manager of the Rialto Movie Theater

Despite its downtown location the Rialto Movie Theater a local institution for five decades must make big changes or close its doors forever It should follow the example of the new Apex Theater in the mall outside of town When the Apex opened last year it featured a video arcade plush carpeting and seats and a state-of-the-art sound system Furthermore in a recent survey over 85 percent of respondents reported that the high price of newly released movies prevents them from going to the movies more than five times per year Thus if the Rialto intends to hold on to its share of a decreasing pool of moviegoers it must offer the same features as Apex

Write a response in which you discuss what questions would need to be answered in order to decide whether the recommendation is likely to have the predicted result Be sure to explain how the answers to these questions would help to evaluate the recommendation

Before following through this business plan the manager should investigate the cause of Rialtorsquos unsuccessful business

The author provides no evidence that the surveyrsquos results are statistically reliable The surveyrsquos sample of 85 percent must be sufficient in size and representative of overall population of the city where Rialto and Apex is serving Lacking evidence of a sufficiently representative sample the author cannot justifiably rely on the survey to draw any conclusion whatsoever The author does not indicate that Apex is indeed currently successful However even if Apex is enjoying success the argument relies on what might be a false analogy between Rialto and Apex In order for Apex to serve as a model that Rialto should emulate the author must assume that all relevant circumstances are essentially the same However this assumption is unwarranted For example the argument overlooks the face that Apex is located in a strategic placemdashbeside a mall where customers can not only watch a movie but also enjoy shopping Therefore simply changing the facility to that of Apex may not lead to success

The author does not mention whether Apex is successful or not Nevertheless even if Apex is currently successful the argument relies on what might be a false analogy between Rialto and Apex In order for Apex to serve as a model that Rialto should emulate the author must assume that all relevant circumstances are essentially the same However this assumption is unwarranted For example the argument overlooks the fact that these two institutions are located in different locations Rialto in downtown and Apex in a mall outside of town Although Apex opened with state-of-the-art facilities the decisive factor in its success could be due to its strategic location of being in a mall People could enjoy both shopping and movies at one location thus they may prefer Apex over Rialto Furthermore the place where people enjoy leisure activities has shifted in the past decades for most cities from downtown to the suburbs Therefore Rialto may not be successful even if it emulates Apexrsquos facilities A better business plan may be relocating Apex to the thriving section of the downtown

Rialto 극장은 지난 50여년간 지역 회관으로써 시내에 위치해 있으면서도 이제 변화를 꾀하지 않으면 문을 닫을

GRE AWA John박 박정어학원

판이다 이 극장은 시외 쇼핑타운에 새로 들어선 Apex 극장의 사례를 본받아야 했다 Apex가 지난해 개업했을 당시 이 극장은 비디오 아케이드 플러쉬 카펫트 바닥과 좌석 그리고 최신 음향시설을 갖추었다 더군다나 최근 조사에서는 응답자의 85 이상이 새로 출시된 영화 입장료가 비싼 탓으로 지난해보다 5배이상의 관람객이 줄어들었다고 나타났다 따라서 Rialto 극장이 줄어들고 있는 관람객을 뺐기지 않고 유지하려면 Apex와 같은 시설들을 갖추어야 할 것이다주장 리알토 극장이 줄어들고 있는 관람객을 뺐기지 않고 유지하려면 Apex와 같은 시설들을 갖추어야 할 것이다

1 조사에서 응답자가 전체를 대표할 수 없다 2 apex 극장이 좋은 시설을 갖추고 있지만 그로 인해 수익이 많이 발생했다는 말이 없으므로 시설투자를

하고도 좋은 결과를 얻을 수 있을지 그 근거가 미흡하다3 좋은 영화가 출시된다면 입장료가 비싸도 영화관에서 꼭 보려고 할 수 있다 4 rialto 가 시설이 아닌 다른 요인에 의해 장사가 안될수도 있다( 우범 지역이라든지)

The following is a recommendation from the business manager of Monarch Books

Since its opening in Collegeville twenty years ago Monarch Books has developed a large customer base due to its reader-friendly atmosphere and wide selection of books on all subjects Last month Book and Bean a combination bookstore and coffee shop announced its intention to open a Collegeville store Monarch Books should open its own in-store cafeacute in the space currently devoted to childrens books Given recent national census data indicating a significant decline in the percentage of the population under age ten sales of childrens books are likely to decline By replacing its childrens books section with a cafeacute Monarch Books can increase profits and ward off competition from Book and Bean

Write a response in which you examine the stated andor unstated assumptions of the argument Be sure to explain how the argument depends on these assumptions and what the implications are for the argument if the assumptions prove unwarranted

The following is a recommendation from the business manager of Monarch Books

Since its opening in Collegeville twenty years ago Monarch Books has developed a large customer base due to its reader-friendly atmosphere and wide selection of books on all subjects Last month Book and Bean a combination bookstore and coffee shop announced its intention to open a Collegeville store Monarch Books should open its own in-store cafeacute in the space currently devoted to childrens books Given recent national census data indicating a significant decline in the percentage of the population under age ten sales of childrens books are likely to decline By replacing its childrens books section with a cafeacute Monarch Books can increase profits and ward off competition from Book and Bean

1 Write a response in which you discuss what questions would need to be answered in order to decide whether the recommendation is likely to have the predicted result Be sure to explain how the answers to these questions would help to evaluate the recommendation

2 Write a response in which you discuss what specific evidence is needed to evaluate the argument and explain how the evidence would weaken or strengthen the argument

No evidence regarding Monarch Bookrsquos successEven if Regal Bookrsquos is successful this may not be attributable to the cafeacute False analogy Emulating may not lead to success Other factors may be involvedInsufficient condition The national census is not enough evidence that childrenrsquos book sales will decline Can

GRE AWA John박 박정어학원

the national census represent the local child populationDid opening a cafeacute boost sales for Regal Books Even assuming Regal is successful by opening a cafeacute this may not be suitable for Monarch which plans to close the childrenrsquos book section to establish a cafe Imprecise language ldquorelatively little spacerdquo how smallThe managerrsquos recommendation contradicts what he says Since Monarch is popular for its wide selection of books closing a selection which targets a major group of readers may hurt Monarchrsquos salesIs this the best way to compete

When Stanley Park first opened it was the largest most heavily used public park in town It is still the largest park but it is no longer heavily used Video cameras mounted in the parks parking lots last month revealed the parks drop in popularity the recordings showed an average of only 50 cars per day In contrast tiny Carlton Park in the heart of the business district is visited by more than 150 people on a typical weekday An obvious difference is that Carlton Park unlike Stanley Park provides ample seating Thus if Stanley Park is ever to be as popular with our citizens as Carlton Park the town will obviously need to provide more benches thereby converting some of the unused open areas into spaces suitable for socializing

Write a response in which you examine the stated andor unstated assumptions of the argument Be sure to explain how the argument depends on these assumptions and what the implications are for the argument if the assumptions prove unwarranted

Stanley 파크가 처음 개장했을 당시 가장 크고 가장 많이 이용되는 공원이었다 아직도 공원중에서는 가장 크지만 이용률은 상당히 떨어졌다 지난달 공원 주차장에 설치해놓은 비디오 카메라를 통해 보면 drop(주차장으로 여겨짐) 이용률이 가장 높았다 수치상으로는 하루 평균 50대의 차량만이 이용하였다 반면 직장 중심거리에 위치한 작은 규모의 Carlton 파크는 주당 무려 150여명 이상이 이용하고 있다 Stanley 파크와는 달리 Carlton 파크에는 의자가 있다는 것이 가장 뚜렷한 차이점이다 따라서 Stanley 파크가 Carlton 파크처럼 시민들이 자주 이용하는 공원이 되기 위해서는 벤치를 설치할 필요가 있으며 이렇게 사용되지 않는 일부 공간을 활용해서 사교를 위한 공간으로 바꾸어야 한다 ===gtdrop 에 대한 첨부사항 (영영사전내용입니다)---- a place or central depository to which something (as mail money or stolen property) is brought for distribution or transmission also the act of depositing something at such a place dropgt

주장 if Stanley Park is ever to be as popular with our citizens as is Carlton Park the town will obviously need to provide more benches thereby converting some of the unused open areas into spaces suitable for socializing1 조사가 언제 이루어진 것인가 조사가 언제 실시되었느냐에 따라 결과가 다를 수있다 현재는 다시 스탠리 파크가 늘어났었을 수 있다 2 벤치를 많이 설치했다고 해서 많은 관광객이 오지 않을수 있다(사람들이 벤치나 사교 공간을 원한다는 어떠한 자료도 없다)3스탠리 파크 주변에 교통 상황이 악화가 되었거나 칼튼 파크에서 문화행사등을 많이 가져서 이용객이 줄어든것일 수도 있다 4 칼튼 파크가 중심지에 있어서 접근성이 좋을수 있다5 조사가 같은 시간을 기준으로 한게 아니다(하나는 주중이고 하나는 주말이다)6사람의 수와 차의 대수를 같은것으로 비교할수 없다 (차안에 몇 명이 타고 있는지 모르고 대중교통을 이용해서 왔을수도 있다)

Page 24: GRE writing argument brain storm

GRE AWA John박 박정어학원

관심을 나타냈다 이러한 결과는 우리회사가 신기술 공법을 중단하면 그에 따라 수익이 늘어날 것이라는 것을 말해주고 있는 것이다주장 These findings suggest that Dura0Sock can increase its profits by discontinuing its use of the ldquoEndurerdquo manufacturing process

1 survey가 정확한 소비자의 의견을 나타낸 것인가 다른 선택없이 양자택일과 같은 방법의 survey였는지2 북동부지역의 시장조사가 전체 의견을 대표할 수 있나3 사람들이 모양이나 색상에 앞서 내구성을 먼저 평가했을 수도 있다 내구성을 갖추었다는 전제하에 모양과 색상에 관심을 드러낸 것일 수 있다4 소비자가 도매상(retail)인지 소매상(whole)인지가 없다

The following appeared in a business magazine

As a result of numerous complaints of dizziness and nausea on the part of consumers of Promofoods tuna the company requested that eight million cans of its tuna be returned for testing Promofoods concluded that the canned tuna did not after all pose a health risk This conclusion is based on tests performed on samples of the recalled cans by chemists from Promofoods the chemists found that of the eight food chemicals most commonly blamed for causing symptoms of dizziness and nausea five were not found in any of the tested cans The chemists did find small amounts of the three remaining suspected chemicals but pointed out that these occur naturally in all canned foods

Write a response in which you discuss what questions would need to be addressed in order to decide whether the conclusion and the argument on which it is based are reasonable Be sure to explain how the answers to the questions would help to evaluate the conclusion

Representativeness of the tested cansThey should conduct a comparative studyThe testing could be biased because Promofoods employees conducted the testingHow much (quantity) of the five and three suspected chemicals were in the canned foodsFalse cause The substance that caused dizziness and nausea may not be one of the eight common chemicals

많은 소비자들의 현기증과 구역질 불만에 따라 Promofoods사는 지난해 참치 캔 8백만 개를 테스트하기 위해 반품시켰다 그 결과 캔에서는 건강에 위험이 될 수 있는 화합물질이 없었던 것으로 회사측은 결론지었다 이러한 결론은 회사측 화학연구자들이 회수된 캔의 샘플을 테스트해서 이들 증상의 원인이 되는 8가지 화합물 중에서 5가지가 실험된 캔에서 발견되지 않았다는 사실에 근거한 것이다 이들 화학자들은 나머지 3개가지 화합물이 모든 캔 식료품에서 흔히 발견되는 것이라고 언급했다 결론 Promofoods concluded that the cans did not after all contain chemicals that posed a health risk

1 공인된 기간에서 테스트를 한 것이 아니고 자사에서 직접 테스트를 했기에 신뢰성이 안간다 2 이런 증상을 일으키는 8개의 물질 말고 다른 물질들이 캔속에 많이 포함됬을수 있다 3 나머지 3개의 물질들의 함유량이 많아서 다른 종류의 캔들은 문제를 일으키지 않지만 참치캔은 문제를

일으킬 수 있다

This magazine article concludes that the 8 million cans of tuna Promofoods recalled due tocomplaints about nausea and dizziness do not after ail contain any chemicals that pose a

GRE AWA John박 박정어학원

health risk To support this conclusion the author cites the fact that five of eight chemicalscommonly causing these symptoms were not found in the recalled cans while the other threealso occur naturally in other canned foods For several reasons this evidence lends littlecredible support to the authors conclusionTo begin with the author relies partly on the fact that although three of the eight chemicalsmost commonly blamed for nausea and dizziness appeared in Promofoods recalled tunathese chemicals also occur naturally in other canned foods However this fact alone lends nosupport to the authors conclusion for two reasons First the author might be ignoring animportant distinction between naturally occurring chemicals and those not occurring naturallyIt is entirely possible that these three chemicals do not occur naturally in Promofoods tunaand that it is for this reason that the chemicals cause nausea and dizziness Secondly it isentirely possible that even when they occur naturally these chemicals cause the samesymptoms Unless the author rules out both possibilities he cannot reliably conclude that therecalled tuna would not cause these symptomsAnother problem with the argument is that the authors conclusion is too broad Based onevidence about certain chemicals that might cause two particular heath-related symptoms theauthor concludes that the recalled tuna contains no chemicals that pose a health risk Howeverthe author fails to account for the myriad of other possible health risks that the recalled tunamight potentially pose Without ruling out all other such risks the author cannot justifiablyreach his conclusionA third problem with the argument involves that fact that the eight particular chemicals withwhich the test was concerned are only the eight most commonly blamed for nausea anddizziness It is entirely possibly that other chemicals might also cause these symptoms andthat one or more of these other chemicals actually caused the symptoms Without ruling outthis possibility the author cannot jusufiably conclude that the recalled tuna would not causenausea and dizzinessA final problem with the argument involves thetesting procedure itself The author providesno information about the number of recaUed cans tested or the selection method used Unlessthe number of cans is a sufficiently large sample and is statistically repre sentative of all therecalled cans the studys results are not statistically reliableIn conclusion the article is unconvincing as it stands To strengthen the assertion that therecalled tuna would not cause nausea and dizziness the author must provide evidence thatthe three chemicals mentioned that occur naturally in other canned foods also appear naturallyin Promofoods tuna The author must also provide evidence that ingesting other canned foodscontaining these three chemicals does not cause these symptoms To better evaluate theargument we would need to know whether the sample used in the tests was statisticallysignificant and representative of all the recalled tuna We would also need to know what otherchemicals in the recalled tuna might pose any health risk at all

5그룹 불충분 조건오류 빈출

Natures Way a chain of stores selling health food and other health-related products is opening its next franchise in the town of Plainsville The store should prove to be very successful Natures Way franchises tend to be most profitable in areas where residents lead healthy lives and clearly Plainsville is such an area Plainsville merchants report that sales of running shoes and exercise clothing are at all-time highs The local health club has more members than ever and the weight training and aerobics classes are always full Finally Plainsvilles schoolchildren represent a new generation of potential customers these schoolchildren are required to participate in a fitness-for-life program which emphasizes the benefits of regular exercise at an early age

Write a response in which you examine the stated andor unstated assumptions of the argument Be sure to

GRE AWA John박 박정어학원

explain how the argument depends on these assumptions and what the implications are for the argument if the assumptions prove unwarranted

False cause

First of all the author believes that the Increased sales of running shoes and exercise clothing indicates

Plainesville residentsrsquo interest in leading healthy lives However this assumption is not logically convincing for

several reasons could be a fashion trendTime shift ldquoFitness for liferdquo might not have any influence on schoolchildren as they growFalse cause There could be other reasons for member increase in the health clubAll of the above are insufficient condition

The author has to prove that local residents are interested in leading healthy lives However he supports his conclusion with insufficient evidence Nevertheless even if the residents are concerned with health naturersquos way may not be successful First

그 동안의 경험을 토대로 볼 때 건강생활과 밀접히 관련되어 있는 거주 지역에서 본 상점들이 아주 호응을 얻고 있다 따라서 이러한 주민들이 많이 거주하고 있는 Plainsville 에 새로운 상점들을 계속 세워야 한다 이 지역 상인들은 런닝화와 운동복 판매가 가장 높다고 말한다 불과 5 년전에는 거의 전무하다시피하던 지역 헬스 클럽의 경우도 엄청나게 많은 회원을 확보하고 있으며 웨이트 트레이닝과 에어로빅 강좌들도 항상 만원이라고 한다 새로운 고객층을 예측해 보는 것도 가능하다 이 지역의 학생들의 경우 Fitness for Life프로그램을 받게 되는데 이러한 프로그램을 통해서 유년시절부터 정규적인 운동 습관을 들이게 하고 있는 것이 그것이다

결론 We should therefore build our next new store in Plainsville

반박 그동안의 경험에 의한 과거 통계가 꼭 여기에도 적용되는건 아니다 5 년전 헬스 클럽이 잘 안되었던게 다른 원인이였을수 있다(강사수준 미달 강좌미비)tourist 에 의한 원인 일수 있다 어렸을때부터 운동을 했다고 해서 커서도 관심이 있지는 않다 (오히려 반감이 있을수 있다 혹은 건강하기에 건강에 관심이 적을수도 있다)운동복이나 신발의 판매가 육체노동에 의한 것일수도 있다

IntroductionSupport1049896In this memorandum the author asserts that Naturersquos Way should build its next newstore in Plainsville To support this assertion the author states that Plainsvillesmerchantsrsquo sales of exercise clothing are going well the local health club has moremembers than ever and a new generation of customers will help to ensure NaturersquosWayrsquos success At first glance the authorrsquos assumption seems convincing but in-depth scrutiny revealsthat it lacks substantial evidence as it stands

Body 1-SamplingTopic Sentence 1To begin with the author assumes that the merchantsrsquo report indicates that the residentsare concerned about their health However this assumption is based on unsubstantiated

GRE AWA John박 박정어학원

data Example 1 (Rebuttal1) First if we do not know the total volume of items sold and the price of the goods exactly we cannot infer whether the residents are actually buying many goods Example 2 (Rebuttal2)In addition to that the report emphasizes the rising sales of running shoes and exerciseclothing however these may not be hot-selling items for Naturersquos Way or may not be theproducts the company is planning to sell Concluding Sentence Therefore in order to make the argument reliable the author should reconsider themerchantsrsquo report with more detailed data

Body 2-CausalTopic Sentence 2Second the author contends that the health clubs classes are full yet this does not meanthat many people actually use the health club other factors may be the real cause forthose closed classes Example 1 (Rebuttal1) To begin with if the health club is very small the number of people working out wouldnot be a large one In fact regular gym-going may just be a vogue among a smallunrepresentative segment of Plainsvilles population Example 2 (Rebuttal2) Moreover it is possible that most of the people who exercise in the health club do weight training and aerobics only to look good and to meet other singles not for their health In that case there would be little demand for health products Concluding SentenceThus the author should not hasten to presume what really caused people to be interested in a healthier lifestyle and enroll in the health club

Body 3-Time-ShiftTopic Sentence 3Finally the author highlights that Naturersquos Way can expect a new generation of customersin Plainsville that will help the company in the long term This notion is mistaken in that itassumes the conditions of the present will continue unchanged in the future Although theschool children are required to participate in the fitness for life program they may notnecessarily buy Naturersquos Ways products Example 1 (Rebuttal1) In the first instance they may suffer a fall in purchasing power arising from future economic difficulties this would cause reluctance to spend a considerable amount of money on health products which tend to be more expensive Example 2 (Rebuttal2)Another possibility is that there may emerge many competitor companies vying with Naturersquos Way so that in the future the school children may not feel the necessity to purchase one companyrsquos health products over anotherrsquosConcluding Sentence Thus the authorrsquos assumption is highly speculative since it relies heavily on unknowablefuture circumstances

ConclusionThesis In sum the author uses many assumptions that are insufficient in supporting his claimsSupportIn order for the authorrsquos claims to be convincing he needs to advance more persuasiveevidence that people in Plainsville really are concerned with their health and health foodThe following was written as a part of an application for a small-business loan by a group of developers in the city of Monroe

A jazz music club in Monroe would be a tremendously profitable enterprise Currently the nearest jazz club is 65 miles away thus the proposed new jazz club in Monroe the C-Note would have the local market all to itself Plus jazz is extremely popular in Monroe over 100000 people attended Monroes annual jazz festival last summer several well-known jazz musicians live in Monroe and the highest-rated radio program in Monroe is Jazz Nightly which airs every weeknight at 7 PM Finally a nationwide study indicates that the typical jazz fan spends close to $1000 per year on jazz entertainment

1 Write a response in which you discuss what specific evidence is needed to evaluate the argument and explain how the evidence would weaken or strengthen the argument

2 Write a response in which you examine the stated andor unstated assumptions of the argument Be

GRE AWA John박 박정어학원

sure to explain how the argument depends on these assumptions and what the implications are for the argument if the assumptions prove unwarranted

3 Write a response in which you discuss what questions would need to be answered in order to decide whether the prediction and the argument on which it is based are reasonable Be sure to explain how the answers to these questions would help to evaluate the prediction

Group error nationwide survey may not reflect local trends Is the nationwide jazz fan population substantialInsufficient non-residents of Monroe may have attended the jazz festival (Body alternative explanation last year may have been an anomaly The author should consider data from various years) The author should indicate how many out of 100000 were Monroe residentsNationwide study Does this reflect Insufficient Citizens of Monroe may continue to go to the jazz club 65 miles away

Are the people in Monroe really interested in jazzMajority of the people who attended the jazz festival might not be Monroe residentsSurvey error nationwide study may not be applicable to MonroeJazz musicians who live in MonroeMonopolyRadio station

In this business application the author claims that the proposed jazz club C Note will be very profitable in Monroe To support this claim the author argues for his case with several evidences At first glance the authorrsquos argument seems convincing however careful scrutiny reveals that his argument in specious

To begin with the author claims that Monroersquos citizens are interested in jazz He presents three evidences First Secondhellip Thirdhellip Howeverhellip

Monroe 시에 있는 재즈 음악 클럽은 수익성이 좋은 사업이다 현재 가장 가까이에 있는 클럽은 65 마일 정도 떨어져 있다 따라서 이번에 세우려고 하는 C Note 는 독보적인 위치를 점할것이다 더군다나 재즈는 이 시에서 가장 인기있는 음악이다 지난 여름 재즈 축제에서는 10 만명 이상의 Morone 시 주민이 참석하였고 몇몇 유명한 재즈 음악가들도 이곳에 살고 있으며 저녁때 방영되는 라디오 프로그램중에서 최고의 시청률을 보이고 있는 것도 Jazz Nightly 이다 전국조사에서도 전형적인 재즈 팬들은 재즈 분야에 년간 1천 달러 가까이 지출하고 있는 것으로 보고되고 있다 따라서 C Note 클럽이 돈을 벌 수 있는 사업이라는 것은 확실한 것이다

결과 It is clear that the C Note cannot help but make money반박 nearest jazz club 이 양질의 써비스로 여전히 손님을 끌수도 있다Festival 에 얼마나 참여하는지가 jazz 의 인기를 반영하지 않는다 뮤지션이 많이 사는거랑 jazz 의 인기가 상관없다라디오 프로그램이 다른 요인에 의해서 인기일수도 있다 (진행자때문)전국 통계 적용 불가화목 실전반_Ms Noh6In this application the author suggests that a jazz club in Monroe will make a number of profits To support this suggestion the author exemplifies the local condition popularity of jazz in Monroe and nationwide study However careful scrutiny of each of the facts reveals that it provides little credible support for the authorrsquos recommendation Good clear intro

First the author assumes that jazz is popular in Monroe because of several facts the jazz festival last year had high participation some famous jazz musicians live in Monroe and the high-rated radio program is lsquoJazz Nightlyrsquo However this assumption has many drawbacks that must be seriously considered(Good topic sentences) If many attendants in the last-yearrsquos festival came from other cities and not Monroe it is hard to conclude that Monroersquos people like jazz Therefore the author must examine how many Monroe residents actually attended the festival On top of that there is little relationship between habitation of famous jazz musician and the popularity of jazz in Monroe Although several well-known musicians live there if they do not take part in any jazz performance of Monroe this might have no effect to the interest of Monroersquos residents

GRE AWA John박 박정어학원

about jazz Finally in the case of radio program this is also not suitable reason why jazz is popular in Monroe It might be possible that people cannot help choosing lsquoJazz Nightlyrsquo because there are few radio programs at Night The fact that the radio program is the highest rating program is not a germane evidence The approximate number of listeners would be the more crucial evidence Therefore the author needs to seriously deliberate the correlation between jazzrsquos popularity in Monroe and his examples (Good logical flow and clarity)

Second the author uses as evidence the nationwide study that jazz fans spend much money on jazz entertainment to substantiate why starting a jazz club in Monroe will be profitable In other words the author assumes that the characteristics of a nationwide study can be applied to Monroe The national study would lend support to the applicantrsquos claim only if residents in Monroe typify national jazz fans However the author does not provide credible evidence that this is the case Moreover the populations of jazz fans nationwide may be insubstantial Thus the author should not infer hastily that Monroersquos residents will spend much money on enjoying jazz from the nationwide study

Lastly even if jazz is popular in Monroe C Note may not be successful It is entirely possible that residents might still prefer other clubs where they have always went In addition there is another possibility that the nearest jazz club will attract many of Monroersquos people because it serves fine performances and is equipped with favorite facilities Without considering these other possibilities the author cannot make his argument convincing In sum the author presents many reasons that are insufficient in supporting his or her claim In order for the authorrsquos claims to be convincing he needs to advance more persuasive evidence such as the total number of Monroe residents who attended the jazz festival the effects on the popularity of jazz by the musicians living in Monroe and the actual number of residents who would typify themselves to be jazz fans through a local survey Without substantial evidence that C Note will be successful in Monroe the businessmen may be overinvesting in what might lead to a business failureExcellent clarity Score 50

The following appeared in a newsletter offering advice to investors

Over 80 percent of the respondents to a recent survey indicated a desire to reduce their intake of foods containing fats and cholesterol and today low-fat products abound in many food stores Since many of the food products currently marketed by Old Dairy Industries are high in fat and cholesterol the companys sales are likely to diminish greatly and company profits will no doubt decrease We therefore advise Old Dairy stockholders to sell their shares and other investors not to purchase stock in this company

Write a response in which you discuss what questions would need to be answered in order to decide whether the advice and the argument on which it is based are reasonable Be sure to explain how the answers to these questions would help to evaluate the advice

Survey 80

GRE AWA John박 박정어학원

Old Dairy could change their products and manufacture low fat dairy foodsLess competing companies Old Dairy could eventually be the only company that produces hellipImprecise numbers and measurementsCustomers may still buy high fat dairy products

The author of the newsletter is offering potentially dangerous advice by recommending Old Dairy stockholders to withdraw investment and stop purchase What is more the authorrsquos prediction debases the reputation and business of Old Dairy and if false could devoid the investment opportunity of the newsletter readers Therefore investors should examine whether the authorrsquos evidences are substantial

To begin with the author states that 80 percent of the respondents in a survey indicated a desire to reduce their intake of foods He therefore argues that Old Dairyrsquos high fat and cholesterol products would decrease in sales However the author makes a crucial error in this argument First the author provides no evidence that the surveyrsquos results are statistically reliable Were they representative of all the customers Were they chosen for the survey randomly Furthermore the desire to reduce fat and cholesterol intake is a pervasive trend in todayrsquos opulent society however the author erroneously identifies this as a new phenomenon which will affect consumer trends Second having a desire to reduce fat and cholesterol intake does not necessarily indicate that people who have this desire will actually reduce consuming these types of products It is entirely possible that they may continue buying Old Dairy products for its quality and taste Accordingly the author cannot draw any firm conclusion that people will not buy Old Dairy products Therefore if any of these cases are true the author may be offering investors a detrimental investment advice

최근 조사에 대한 응답자중 80 이상이 자신이 먹는 음식에서 지방과 콜레스테롤의 함유량을 줄이고 싶다고 한다 아울러 요즘은 많은 식료품 가계에서 저지방 제품들을 많이 취급하고 있다 현재 Old Dairy Industries가 판매하고 있는 많은 음식제품들은 지방과 콜레스테롤이 높기 때문에 이 회사의 매출이 격감할 것으로 보이며 당연히 매출이익도 줄어들것이다 따라서 이 회사의 주주들은 주식을 매각하고 다른 주식 투자가들도 이 회사의 주식을 매입하지 않는 것이 좋다

결론 Old Dairy stockholders to sell their shares and other investors not to purchase stock in this company

반박 모든 상품이 다 고 지방 고 칼로리는 아니다(비록 많을지라도) 일부의 식품의 경우 기호에 맞어서 히트해서 전체적인 수입이 증가할 수도 있다국내시장만 생각할 수 없다( 외국시장에서 호황을 누릴수 있다 )입맛이라는게 즉각 바뀌는게 아니다

The following appeared in a letter to the editor of the Balmer Island Gazette

On Balmer Island where mopeds serve as a popular form of transportation the population increases to 100000 during the summer months To reduce the number of accidents involving mopeds and pedestrians the town council of Balmer Island should limit the number of mopeds rented by the islands moped rental companies from 50 per day to 25 per day during the summer season By limiting the number of rentals the town council will attain the 50 percent annual reduction in moped accidents that was achieved last year on the neighboring island of Seaville when Seavilles town council enforced similar limits on moped rentals

1 Write a response in which you discuss what questions would need to be answered in order to decide whether the recommendation is likely to have the predicted result Be sure to explain how the answers to these questions would help to evaluate the recommendation

2 Write a response in which you discuss what questions would need to be answered in order to decide whether the prediction and the argument on which it is based are reasonable Be sure to explain how the answers to these questions would help to evaluate the prediction

3 Write a response in which you examine the stated andor unstated assumptions of the argument Be sure to explain how the argument depends on these assumptions and what the implications are for the argument if the assumptions prove unwarranted

Whatrsquos the actual population of Balmer Island 100000mdashis this a significant increase What kind of accidents Skin abrasions or serious injury And compared to Seaville how serious are the accidents and the actual number of accidents Did Seaville enforce other restrictions like safety signsHow different are the conditions of Balmer

GRE AWA John박 박정어학원

and Seaville regarding population road (safety) conditions topography other town-government regulation How much will the economy of Balmer be affected do to this restriction Could it cause an economic recession due to the fact that these rental companiesrsquo chance to make money is only during the summer thereby weakening the economic infrastructure Are there any other ways that could better alleviate the accident rate

Statistics 50-impreciseAnalogy Balmer compared with TorseauFalse Cause Accidents might have occurred because of reasons other than mopeds False Cause population increase may not be part of the cause of the accidentsOther explanations for the accident pedestrians few road safety regulations narrow roadsThere could be other better solutionshellip

Balmer Island의 인구가 여름철에는 십만명으로 늘어난다 2륜차와 보행자간 사고를 줄이기 위해 시의회는 6개의 자전거를 포함한 2륜차 대여업체에게 이 기간동안에는 대여숫자를 일일 50에서 30으로 제한하도록 할 것이다 대여숫자를 줄임으로써 시 의회는 지난해 이웃한 Torseau섬에서 이와 동일한 규제를 시행해서 50나 줄인 결과를 보고 마찬가지로 50를 줄일수 있다고 확신하고 있다

결론 The town council of Balmer Island should linit the number

반박 보행자의 부실에 의해서 사고가 많이 일어날수도 있다렌탈수의 줄임만이 대책은 아니다(대부분의 사람들이 렌탈 보다는 소유하고 있을 수도 있다)옆섬과는 상황이 다를수도 있다(그 섬에서는 사고의 원인이 많은 자전거 수로 인한것일수있다) 하지만 이 섬은 좁은 도로가 원인일 수도 있고 도로 안전 장치의미비가 원일일수 있다

In this letter the author recommends that Balmer Island should limit the number moped rentals from 50 to 30 per day To support this recommendation the author points out several reasons However careful scrutiny of each of the facts reveals that it is filled with unanswered questions that could significantly weaken the authorrsquos recommendation with loops and holes which are answered

The recommendation depends on the assumption that no alternative means of reducing the number of accidents are available However the author fails to offer any evidence to substantiate this crucial assumption It is highly possible that means other than this would better solve the problem Perhaps they could widen the roads or put-up more safety signs Or perhaps the accidents were due to the lack of skills in which case proper safety training would significantly alleviate the problem Without considering and ruling out these and other alternative means of reducing accidetns the author cannot confidently conclude that merely emulating Torseau would suffice Moreover the author is advising a recommendation which could potentially harm the economy of Balmer Island sincehellip Moreover the Balmer Island should alternative means to reduce accidents because limiting moped rentals during the summer could harm the economy of Balmerhellip

First of all the author believes that increase in population and the number of moped rentals are responsible for the accidents It is entirely possible that other factors are responsible for the accidents Perhaps Balmer Islandrsquos lack of safety signs was a major factor Or maybe the roads are narrow and dangerous on the Island therefore the town council could enforce stricter traffic regulations to alleviate the problem Accordingly if either of these scenarios is true the author cannot draw any firm conclusion that increase in the number of population and moped rentals are the cause of the accidents

The author of this editorial recommends that to reduce accidents involving mopeds andpedestrians Balmer Islands city council should restrict moped rentals to 30 per day down from50 at each of the islands six rental outlets To support this recommendation the author citesthe fact that last year when nearby Torseau Islands town council enforced similar measuresTorseaus rate of moped accidents fell by 50 For several reasons this evidence providesscant support for the authors recommendationTo begin with the author assumes that all other conditions in Balmer that might affect therate of moped-pedestrian accidents will remain unchanged after the restrictions are enactedHowever with a restricted supply of rental mopeds people in Balmer might purchase mopedsinstead Also the number of pedestrians might increase in the future with more pedestriansespecially tourists the risk of moped-pedestrian accidents would probably increase For thatmatter the number of rental outlets might increase to make up for the artificial supplyrestriction per outlet--a likely scenario assuming moped rental demand does not declineWithout considering and ruling out these and other possible changes that might contribute to ahigh incidence of moped-pedestrian accidents the author cannot convince me that theproposed restrictions will necessarily have the desired effect

GRE AWA John박 박정어학원

Next the author fails to consider other possible explanations for the 50 decline inTorseaus moped accident rate last year Perhaps last year Torseau experienced unusually fairweather during which moped accidents are less likely Perhaps fewer tourists visited Tot seanlast year than during most years thereby diminishing the demand for rental mopeds to belowthe allowed limits Perhaps last year some of Torseaus moped rental outlets purchased newmopeds that are safer to drive Or perhaps the restrictions were already in effect but were notenforced until last year In any event a decline in Torseaus moped accident rate during onlyone year is scarcely sufficient to draw any reliable conclusions about what might have causedthe decline or about what the accident rate will be in years aheadAdditionally in asserting that the same phenomenon that caused a 50 decline in mopedaccidents in Torseau would cause a similar decline in Balmer the author relies on what mightamount to an unfair analogy between Balmer and Torseau Perhaps Balmers ability to enforcemoped-rental restrictions does not meet Torseaus ability if not then the mere enactment ofsimilar restrictions in Balmer is no guarantee of a similar result Or perhaps the demand formopeds in Torseau is always greater than in Balmer Specifically if fewer than all availablemopeds are currently rented per day from the average Balmer outlet while in Torseau everyavailable moped is rented each day then the proposed restriction is likely to have less impacton the accident rate in Balmer than in TorseauFinally the author provides no evidence that the same restrictions that served to reduce theincidence of all moped accidents by 50 would also serve to reduce the incidence ofaccidents involving mopeds and pedestrians by 50 Lacking such evidence it is entirelypossible that the number of moped accidents not involving pedestrians decreased by a greaterpercentage while the number of moped-pedestrian accidents decreased by a smallerpercentage or even increased Since the author has not accounted for these possibilities theeditorials recommendation cannot be taken seriouslyIn conclusion the recommendation is not well supported To convince me that the proposedrestriction would achieve the desired outcome the author would have to assure me that nochanges serving to increase Balmers moped-pedestrian accident rate will occur in theforeseeable future The author must also provide dear evidence that last years decline inmoped accidents in Torseau was attributable primarily to its moped rental restrictions ratherthan to one or more other factors In order to better evaluate the recommendation I wouldneed more information comparing the supply of and demand for moped rentals on the twoislands I would also need to know the rate of mopedpedestrian accidents in Torseau both priorto and after the restrictions were enforced in TorseauThe following appeared in a magazine article about planning for retirement

Clearview should be a top choice for anyone seeking a place to retire because it has spectacular natural beauty and a consistent climate Another advantage is that housing costs in Clearview have fallen significantly during the past year and taxes remain lower than those in neighboring towns Moreover Clearviews mayor promises many new programs to improve schools streets and public services And best of all retirees in Clearview can also expect excellent health care as they grow older since the number of physicians in the area is far greater than the national average

Write a response in which you discuss what specific evidence is needed to evaluate the argument and explain how the evidence would weaken or strengthen the argument

-Natural beauty and consistent climate may not be the most wanted qualities-Housing costs could have lowered on a national level wealthy retirees may not care about costs-Taxes may be high compared to the nationrsquos average tax rate-What about other qualities of Clearview Crime rate what qualities would retirees want -If schools streets and public services need improvement then this is proof that the current condition of Clearview is low Or due to budgetary reasons the mayor may not follow-up on his promise because of lowered tax rate -Schools and people who are retired no relationship-Physicians What kind of physicians Number is irrelevant Are these physicians capable of addressing the illnesses of old people

This author argues that anyone seeking a place to retire should choose Clearview To supportthis argument the article cites Clearviews consistent climate and natural beauty its fallinghousing costs its low property taxes compared to nearby towns and the mayors promise toimprove schools streets and services The article also claims that retirees can expectexcellent health care because the number of physicians in Clearview greatly exceeds thenational average This argument is flawed in several critical respectsTo begin with although consistent climate and natural beauty might be attractive to manyretirees these features are probably not important to all retirees For many retirees it isprobably more important to live near relatives or even to enjoy changing seasons Thus I

GRE AWA John박 박정어학원

cannot accept the authors sweeping recommendation for all retirees on this basisAlso Clearviews declining housing costs do not necessarily make Clearview the best placeto retire for two reasons First despite the decline Clearviews housing costs might be highcompared to housing costs in other cities Secondly for wealthier retirees housing costs arenot likely to be a factor in choosing a place to retire Thus the mere fact that housing costshave been in decline lends scant support to the recommendationThe articles reliance on Clearviews property-tax rates is also problematic in two respectsFirst retirees obviously have innumerable choices about where to retire besides Clear viewand nearby towns Secondly for retirees who are well-off financially property taxes are notlikely to be an important concern in choosing a place to retire Thus it is unfair to infer fromClearviews property-tax rates that retirees would prefer ClearviewYet another problem with the argument involves the mayors promises In light of Clearviewslow property-tax rates whether the mayor can follow through on those promises is highlyquestionable Absent any explanation of how the city can spend more money in the areas citedwithout raising property taxes I simply cannot accept the editorials recommendation on thebasis of those promises Besides even if the city makes the improvements promised thoseimprovements--particular the ones to schools--would not necessarily be important to retireesFinally although the number of physicians in Clearview is relatively high the per capitanumber might be relatively low Moreover it would be fairer to compare this per capita numberwith the per capita number for other attractive retirement towns--rather than the nationalaverage After all retirees are likely to place a relatively heavy burden on health-careresources Besides the article provides no assurances that the number of physicians inClearview will remain high in the foreseeable futureIn conclusion the recommendation is poorly supported To strengthen it the author mustconvince me--perhaps by way of a reliable survey--that the key features that the vast majorityof retirees look for in choosing a place to live are consistent climate natural beauty and lowhousing costs The author must also provide better evidence that Clear views property taxesare lower than the those of cities in other areas The author must also explain how the city canmake its promised improvements without raising property taxes Finally to better assess theargument I would need to now how the per capita number of physicians in Clearview wouldcompare to the national average in the futureThe following appeared as a letter to the editor from a Central Plaza store owner

Over the past two years the number of shoppers in Central Plaza has been steadily decreasing while the popularity of skateboarding has increased dramatically Many Central Plaza store owners believe that the decrease in their business is due to the number of skateboard users in the plaza There has also been a dramatic increase in the amount of litter and vandalism throughout the plaza Thus we recommend that the city prohibit skateboarding in Central Plaza If skateboarding is prohibited here we predict that business in Central Plaza will return to its previously high levels

Write a response in which you discuss what questions would need to be answered in order to decide whether the recommendation is likely to have the predicted result Be sure to explain how the answers to these questions would help to evaluate the recommendation

Why two years ago What happened two years ago which started this declineIs the dramatic increase in the ldquopopularityrdquo of skateboarding the cause of the steady decline of shoppers Are there any malls nearby Were there any changes nearby which could affect the decline in customersmdasha big mall perhaps Could the decline be due to the shop ownersHow many skateboarders use the plazaWhere do they skateboardDo they shop and are they customersAre the increase in litter and vandalism due to skateboarders Could this be alleviated by installing CCTVs and hiring security

This editorial concludes that the city should ban skateboarding from its downtown CentralPlaza in order to attract visitors to that area to return the area to its former glory and to makeit a place where people can congregate for fun and relaxation To justify this conclusion theeditorial points out that skateboarders are nearly the only people one sees anymore at CentralPlaza and that the Plaza is littered and its property defaced The editorial also points out thatthe majority of downtown merchants support the skate boarding ban This argument is flawedin several critical respectsFirst the editorials author falsely assumes that a ban on skateboarding is both necessaryand sufficient to achieve the three stated objectives Perhaps the city can achieve thoseobjectives by other means as well--for example by creating a new mall that incorporates anattractive new skateboard park Even if banning skateboarders altogether is necessary to meetthe citys goals the author has not shown that this action by itself would suffice Assuming thatthe Plazas reputation is now tarnished restoring that reputation and in turn enticing peopleback to the Plaza might require additional measures--such as removing litter and graffiti

GRE AWA John박 박정어학원

promoting the Plaza to the public or enticing popular restaurant or retail chains to the PlazaSecondly the editorial assumes too hastily that the Plazas decline is attributable to theskateboarders--rather than to some other phenomenon Perhaps the Plazas primary appeal inits glory days had to do with particular shops or eateries which were eventually replaced byless appealing ones Or perhaps the crime rate in surrounding areas has risen dramatically forreasons unrelated to the skateboarders presence at the Plaza Without ruling out these andother alternative explanations for the Plazas decline the editorials author cannot convince methat a skateboard ban would reverse that declineThirdly the editorials author might be confusing cause with effect--by assuming that theskateboarders caused the abandonment of the Plaza rather than vice versa It is entirelypossible that skateboarders did not frequent the Plaza until it was largely abandoned--andbecause it had been abandoned In fact this scenario makes good sense since skateboardingis most enjoyable where there are few pedestrians or motorists to get in the wayFourth it is unreasonable to infer from the mere fact that most merchants favor the ban thatthe ban would be effective in achieving the citys objectives Admittedly perhaps thesemerchants would be more likely to help dean up the Plaza area and promote their businesseswere the city to act in accordance with their preference Yet lacking any supporting evidencethe author cannot convince me of this Thus the survey amounts to scant evidence at best thatthe proposed ban would carry the intended resultFinally the author recommends a course of action that might actually defeat the citysobjective of providing a fun and relaxing place for people to congregate In my experienceskateboarding contributes to an atmosphere of fun and relaxation for adults and children alikemore so than many other types of ambiance Without considering that continuing to allowskateboarding--or even encouraging this activity--might achieve the citys goal more effectivelythan banning the activity the author cannot convincingly conclude that the ban would be in thecitys best interestsIn sum the argument is a specious one To strengthen it the editorials author must providedear evidence that skateboarding and not some other factor is responsible for the conditionsmarking the Plazas decline The author must also convince me that no alternative means ofrestoring the Plaza are available to the city and that the proposed ban by itself would suffice toattract tourists and restore the Plaza to its former glory Finally to better assess the argument itwould be useful to know the circumstances under which the downtown merchants would bewilling to help the city achieve its objectives

6그룹 약한 비유 빈출

The following recommendation appeared in a memo from the mayor of the town of Hopewell

Two years ago the nearby town of Ocean View built a new municipal golf course and resort hotel During the past two years tourism in Ocean View has increased new businesses have opened there and Ocean Views tax revenues have risen by 30 percent Therefore the best way to improve Hopewells economymdashand generate additional tax revenuesmdashis to build a golf course and resort hotel similar to those in Ocean View

Write a response in which you examine the stated andor unstated assumptions of the argument Be sure to explain how the argument depends on these assumptions and what the implications are for the argument if the assumptions prove unwarranted

GRE AWA John박 박정어학원

Assumptions The author assumes that OVrsquos municipal golf course and resort hotel caused tourism new businesses and increased tax revenues There may be other reasons advertising promo He assumes that this will continueAssumes that Ocean View and Hopewell are similar in many waysmdashthe name suggests otherwise OV may have always been a tourist attractions for its beaches We need to know the topography

2년전 Ocean View 시는 시정 소유 골프 및 휴양지 호텔을 신축했다 그리고 지난 2년동안 이 시의 관광객이 증가했으며 새로운 사업들이 생겨났다 그에따라 시의 세수도 30나 증가했다 Hopewell의 경제를 향상시키고 아울러 세수를 늘릴 수 있는 가장 좋은 방법은 Ocean View에 세워진 것과 같은 골프 시설과 휴양지 호텔을 신축하는 것이다

1 다른 요인으로 관광 산업이 발전했을 수도 있다 문화 유적이 발견이 되었거나 도로의 정비등으로 여행자가 늘었을 수도 있다

2 관광 산업의증가가 늘어난 세수의 원인이 아니라 새로 유입된 인구의 증가나 다른 공장에서 발생한 것일 수 있다

3 2년동안 한참 골프가 붐을 이루었을 수 있다 경제상황이 나빠지거나 다른 레포츠가 인근 지역에 생겨난다면 골프하는 사람이 줄어들 수 있다

In this memo HopeweUs mayor recommends that in order to stimulate the towns economyand boost tax revenues HopeweU should build a new golf course and resort hotel just as thetown of Ocean View did two years ago To support this recommendation the mayor points outthat in Ocean View during the last two years tourism has increased new businesses haveopened and tax revenues have increased by 30 I find the mayors argument unconvincingin several important respectsFirst of all it is possible that the mayor has confused cause with effect respecting the recentdevelopments in Ocean View Perhaps Ocean Views construction of a new golf course andhotel was a response to previous increases in tourism and business development increasesthat have simply continued during the most recent two years Since the mayor has failed toaccount for this possibility the claim that Hopewell would boost its economy by alsoconstructing a golf course and hotel is completely unwarrantedSecondly the mayor fails to account for other possible causes of the trends in Ocean Viewduring the last two years The increase in tourism might have been due to improving economicconditions nationwide or to unusually pleasant weather in the region The new businessesthat have opened in Ocean View might have opened there irrespective of the new golf courseand hotel And the 30 increase in tax revenues might have been the result of an increase intax rates or the addition of a new type of municipal taxWithout ruling out these and other alternative explanations for the three recent trends inOcean View the mayor cannot reasonably infer based on those trends that Hopewellseconomy would benefit by following Ocean Views exampleThirdly even if the recent trends in Ocean View are attributable to the construction of the newgolf course and hotel there the mayor assumes too hastily that the golf course and hotel willcontinue to benefit that towns overall economy The mayor has not accounted for thepossibility that increased tourism will begin to drive residents away during tourist season orthat new business development will result in the towns losing its appeal as a place to visit or tolive Unless the mayor can convince me that these scenarios are unlikely I cannot accept themayors recommendation that Hopewell follow Ocean Views exampleFinally the mayors argument rests on the unsubstantiated assumption that Hopewell andOcean View are sufficiently alike in ways that might affect the economic impact of a new golfcourse and hotel Hopewell might lack the sort of natural environment that would attract moretourists and new businesses to the town--regardless of its new golf course and hotel For thatmatter perhaps Hopewell already contains several resort hotels and golf courses that are notutilized to their capacity If so building yet another golf course and hotel might amount to amisallocation of the towns resources--and actually harm the towns overall economyIn sum the mayors recommendation is not well supported To bolster it the mayor mustprovide better evidence that Ocean Views new golf course and hotel and not some otherphenomenon--has been responsible for boosting Ocean Views economy during the last twoyears To better assess the recommendation I would need to know why Ocean View decidedto construct its new golf course and hotel in the first place--specifically what events prior toconstruction might have prompted that decision I would also need to thoroughly compare

GRE AWA John박 박정어학원

HopeweU with Ocean View--especially in terms of their appeal to tourists and businesses--todetermine whether the same course of action that appears to have boosted Ocean Viewseconomy would also boost Hopewells economy

The following is part of a memorandum from the president of Humana University

Last year the number of students who enrolled in online degree programs offered by nearby Omni University increased by 50 percent During the same year Omni showed a significant decrease from prior years in expenditures for dormitory and classroom space most likely because instruction in the online programs takes place via the Internet In contrast over the past three years enrollment at Humana University has failed to grow and the cost of maintaining buildings has increased along with our budget deficit To address these problems Humana University will begin immediately to create and actively promote online degree programs like those at Omni We predict that instituting these online degree programs will help Humana both increase its total enrollment and solve its budget problems

Write a response in which you discuss what questions would need to be answered in order to decide whether the prediction and the argument on which it is based are reasonable Be sure to explain how the answers to these questions would help to evaluate the prediction

Is Omni University successful due to the online degree program 50 Is the decrease in expenditures for dormitory and classroom space due to the decrease in of on-campus students Which classes were successful Does HU have those classes

Even if the long-distance degree programs at Omni University benefited the school the presidentrsquos recommendation that Human College should emulate Omni University is too hasty First OUrsquos name implies that the school would have more majors than Humanahellip the president should examine which degrees were in the long-distance programhellip

지난해에는 Omni 대학에서 개강했던 원거리 학생 학점 취득 프로그램을 등록했던 학생들의 숫자가 50나 증가했다 같은해 기간동안 Omni 대학에서는 그 전년도부터 기숙사와 학급의 공간 확충을 위한 예산을 대폭 줄였는데 이는 이 원거리 학점 취득 프로그램이 양방향 비디오 컴퓨터 접속을 통해서만 가능한 수업지도 방식이기때문인 것으로 보인다 반면 지난 3개년 동안 Humana 대학에서의 수강률은 감소한데다가 건물

GRE AWA John박 박정어학원

유지비도 올랐다 따라서 Humana대학의 수강을 늘리고 예산손실을 회복하기 위해서는 Omni 대학에서 취한 조치와 같은 능동적인 프로그램을 추진해야 한다

결론 we should initiate and actively promote long-distance degree programs like those at Omni 반박 원거리 학생 취득 프로그램 숫자가 증가한거하고 예산이 줄어드는 것 사이에 연관이 약하다 (causal 학생의 증가로 관리비용 증가할수 있음 원거리 수업가능 장비도입에의한 비용발생)bad analogy(omni university 하고 같은 조건이 아니다 )-gt omni college 가 강좌내용이 좋아서 학생의 등록이 많을수 있다 Humana 대학에서 만들었다 하더라도 인기 없을수 있음다른 요인에 의해서 Humana 대학의 수강 인원이 증가할수 있음(비록 과거엔 인기가 없었을지라도)

The following appeared as part of a business plan developed by the manager of the Rialto Movie Theater

Despite its downtown location the Rialto Movie Theater a local institution for five decades must make big changes or close its doors forever It should follow the example of the new Apex Theater in the mall outside of town When the Apex opened last year it featured a video arcade plush carpeting and seats and a state-of-the-art sound system Furthermore in a recent survey over 85 percent of respondents reported that the high price of newly released movies prevents them from going to the movies more than five times per year Thus if the Rialto intends to hold on to its share of a decreasing pool of moviegoers it must offer the same features as Apex

Write a response in which you discuss what questions would need to be answered in order to decide whether the recommendation is likely to have the predicted result Be sure to explain how the answers to these questions would help to evaluate the recommendation

Before following through this business plan the manager should investigate the cause of Rialtorsquos unsuccessful business

The author provides no evidence that the surveyrsquos results are statistically reliable The surveyrsquos sample of 85 percent must be sufficient in size and representative of overall population of the city where Rialto and Apex is serving Lacking evidence of a sufficiently representative sample the author cannot justifiably rely on the survey to draw any conclusion whatsoever The author does not indicate that Apex is indeed currently successful However even if Apex is enjoying success the argument relies on what might be a false analogy between Rialto and Apex In order for Apex to serve as a model that Rialto should emulate the author must assume that all relevant circumstances are essentially the same However this assumption is unwarranted For example the argument overlooks the face that Apex is located in a strategic placemdashbeside a mall where customers can not only watch a movie but also enjoy shopping Therefore simply changing the facility to that of Apex may not lead to success

The author does not mention whether Apex is successful or not Nevertheless even if Apex is currently successful the argument relies on what might be a false analogy between Rialto and Apex In order for Apex to serve as a model that Rialto should emulate the author must assume that all relevant circumstances are essentially the same However this assumption is unwarranted For example the argument overlooks the fact that these two institutions are located in different locations Rialto in downtown and Apex in a mall outside of town Although Apex opened with state-of-the-art facilities the decisive factor in its success could be due to its strategic location of being in a mall People could enjoy both shopping and movies at one location thus they may prefer Apex over Rialto Furthermore the place where people enjoy leisure activities has shifted in the past decades for most cities from downtown to the suburbs Therefore Rialto may not be successful even if it emulates Apexrsquos facilities A better business plan may be relocating Apex to the thriving section of the downtown

Rialto 극장은 지난 50여년간 지역 회관으로써 시내에 위치해 있으면서도 이제 변화를 꾀하지 않으면 문을 닫을

GRE AWA John박 박정어학원

판이다 이 극장은 시외 쇼핑타운에 새로 들어선 Apex 극장의 사례를 본받아야 했다 Apex가 지난해 개업했을 당시 이 극장은 비디오 아케이드 플러쉬 카펫트 바닥과 좌석 그리고 최신 음향시설을 갖추었다 더군다나 최근 조사에서는 응답자의 85 이상이 새로 출시된 영화 입장료가 비싼 탓으로 지난해보다 5배이상의 관람객이 줄어들었다고 나타났다 따라서 Rialto 극장이 줄어들고 있는 관람객을 뺐기지 않고 유지하려면 Apex와 같은 시설들을 갖추어야 할 것이다주장 리알토 극장이 줄어들고 있는 관람객을 뺐기지 않고 유지하려면 Apex와 같은 시설들을 갖추어야 할 것이다

1 조사에서 응답자가 전체를 대표할 수 없다 2 apex 극장이 좋은 시설을 갖추고 있지만 그로 인해 수익이 많이 발생했다는 말이 없으므로 시설투자를

하고도 좋은 결과를 얻을 수 있을지 그 근거가 미흡하다3 좋은 영화가 출시된다면 입장료가 비싸도 영화관에서 꼭 보려고 할 수 있다 4 rialto 가 시설이 아닌 다른 요인에 의해 장사가 안될수도 있다( 우범 지역이라든지)

The following is a recommendation from the business manager of Monarch Books

Since its opening in Collegeville twenty years ago Monarch Books has developed a large customer base due to its reader-friendly atmosphere and wide selection of books on all subjects Last month Book and Bean a combination bookstore and coffee shop announced its intention to open a Collegeville store Monarch Books should open its own in-store cafeacute in the space currently devoted to childrens books Given recent national census data indicating a significant decline in the percentage of the population under age ten sales of childrens books are likely to decline By replacing its childrens books section with a cafeacute Monarch Books can increase profits and ward off competition from Book and Bean

Write a response in which you examine the stated andor unstated assumptions of the argument Be sure to explain how the argument depends on these assumptions and what the implications are for the argument if the assumptions prove unwarranted

The following is a recommendation from the business manager of Monarch Books

Since its opening in Collegeville twenty years ago Monarch Books has developed a large customer base due to its reader-friendly atmosphere and wide selection of books on all subjects Last month Book and Bean a combination bookstore and coffee shop announced its intention to open a Collegeville store Monarch Books should open its own in-store cafeacute in the space currently devoted to childrens books Given recent national census data indicating a significant decline in the percentage of the population under age ten sales of childrens books are likely to decline By replacing its childrens books section with a cafeacute Monarch Books can increase profits and ward off competition from Book and Bean

1 Write a response in which you discuss what questions would need to be answered in order to decide whether the recommendation is likely to have the predicted result Be sure to explain how the answers to these questions would help to evaluate the recommendation

2 Write a response in which you discuss what specific evidence is needed to evaluate the argument and explain how the evidence would weaken or strengthen the argument

No evidence regarding Monarch Bookrsquos successEven if Regal Bookrsquos is successful this may not be attributable to the cafeacute False analogy Emulating may not lead to success Other factors may be involvedInsufficient condition The national census is not enough evidence that childrenrsquos book sales will decline Can

GRE AWA John박 박정어학원

the national census represent the local child populationDid opening a cafeacute boost sales for Regal Books Even assuming Regal is successful by opening a cafeacute this may not be suitable for Monarch which plans to close the childrenrsquos book section to establish a cafe Imprecise language ldquorelatively little spacerdquo how smallThe managerrsquos recommendation contradicts what he says Since Monarch is popular for its wide selection of books closing a selection which targets a major group of readers may hurt Monarchrsquos salesIs this the best way to compete

When Stanley Park first opened it was the largest most heavily used public park in town It is still the largest park but it is no longer heavily used Video cameras mounted in the parks parking lots last month revealed the parks drop in popularity the recordings showed an average of only 50 cars per day In contrast tiny Carlton Park in the heart of the business district is visited by more than 150 people on a typical weekday An obvious difference is that Carlton Park unlike Stanley Park provides ample seating Thus if Stanley Park is ever to be as popular with our citizens as Carlton Park the town will obviously need to provide more benches thereby converting some of the unused open areas into spaces suitable for socializing

Write a response in which you examine the stated andor unstated assumptions of the argument Be sure to explain how the argument depends on these assumptions and what the implications are for the argument if the assumptions prove unwarranted

Stanley 파크가 처음 개장했을 당시 가장 크고 가장 많이 이용되는 공원이었다 아직도 공원중에서는 가장 크지만 이용률은 상당히 떨어졌다 지난달 공원 주차장에 설치해놓은 비디오 카메라를 통해 보면 drop(주차장으로 여겨짐) 이용률이 가장 높았다 수치상으로는 하루 평균 50대의 차량만이 이용하였다 반면 직장 중심거리에 위치한 작은 규모의 Carlton 파크는 주당 무려 150여명 이상이 이용하고 있다 Stanley 파크와는 달리 Carlton 파크에는 의자가 있다는 것이 가장 뚜렷한 차이점이다 따라서 Stanley 파크가 Carlton 파크처럼 시민들이 자주 이용하는 공원이 되기 위해서는 벤치를 설치할 필요가 있으며 이렇게 사용되지 않는 일부 공간을 활용해서 사교를 위한 공간으로 바꾸어야 한다 ===gtdrop 에 대한 첨부사항 (영영사전내용입니다)---- a place or central depository to which something (as mail money or stolen property) is brought for distribution or transmission also the act of depositing something at such a place dropgt

주장 if Stanley Park is ever to be as popular with our citizens as is Carlton Park the town will obviously need to provide more benches thereby converting some of the unused open areas into spaces suitable for socializing1 조사가 언제 이루어진 것인가 조사가 언제 실시되었느냐에 따라 결과가 다를 수있다 현재는 다시 스탠리 파크가 늘어났었을 수 있다 2 벤치를 많이 설치했다고 해서 많은 관광객이 오지 않을수 있다(사람들이 벤치나 사교 공간을 원한다는 어떠한 자료도 없다)3스탠리 파크 주변에 교통 상황이 악화가 되었거나 칼튼 파크에서 문화행사등을 많이 가져서 이용객이 줄어든것일 수도 있다 4 칼튼 파크가 중심지에 있어서 접근성이 좋을수 있다5 조사가 같은 시간을 기준으로 한게 아니다(하나는 주중이고 하나는 주말이다)6사람의 수와 차의 대수를 같은것으로 비교할수 없다 (차안에 몇 명이 타고 있는지 모르고 대중교통을 이용해서 왔을수도 있다)

Page 25: GRE writing argument brain storm

GRE AWA John박 박정어학원

health risk To support this conclusion the author cites the fact that five of eight chemicalscommonly causing these symptoms were not found in the recalled cans while the other threealso occur naturally in other canned foods For several reasons this evidence lends littlecredible support to the authors conclusionTo begin with the author relies partly on the fact that although three of the eight chemicalsmost commonly blamed for nausea and dizziness appeared in Promofoods recalled tunathese chemicals also occur naturally in other canned foods However this fact alone lends nosupport to the authors conclusion for two reasons First the author might be ignoring animportant distinction between naturally occurring chemicals and those not occurring naturallyIt is entirely possible that these three chemicals do not occur naturally in Promofoods tunaand that it is for this reason that the chemicals cause nausea and dizziness Secondly it isentirely possible that even when they occur naturally these chemicals cause the samesymptoms Unless the author rules out both possibilities he cannot reliably conclude that therecalled tuna would not cause these symptomsAnother problem with the argument is that the authors conclusion is too broad Based onevidence about certain chemicals that might cause two particular heath-related symptoms theauthor concludes that the recalled tuna contains no chemicals that pose a health risk Howeverthe author fails to account for the myriad of other possible health risks that the recalled tunamight potentially pose Without ruling out all other such risks the author cannot justifiablyreach his conclusionA third problem with the argument involves that fact that the eight particular chemicals withwhich the test was concerned are only the eight most commonly blamed for nausea anddizziness It is entirely possibly that other chemicals might also cause these symptoms andthat one or more of these other chemicals actually caused the symptoms Without ruling outthis possibility the author cannot jusufiably conclude that the recalled tuna would not causenausea and dizzinessA final problem with the argument involves thetesting procedure itself The author providesno information about the number of recaUed cans tested or the selection method used Unlessthe number of cans is a sufficiently large sample and is statistically repre sentative of all therecalled cans the studys results are not statistically reliableIn conclusion the article is unconvincing as it stands To strengthen the assertion that therecalled tuna would not cause nausea and dizziness the author must provide evidence thatthe three chemicals mentioned that occur naturally in other canned foods also appear naturallyin Promofoods tuna The author must also provide evidence that ingesting other canned foodscontaining these three chemicals does not cause these symptoms To better evaluate theargument we would need to know whether the sample used in the tests was statisticallysignificant and representative of all the recalled tuna We would also need to know what otherchemicals in the recalled tuna might pose any health risk at all

5그룹 불충분 조건오류 빈출

Natures Way a chain of stores selling health food and other health-related products is opening its next franchise in the town of Plainsville The store should prove to be very successful Natures Way franchises tend to be most profitable in areas where residents lead healthy lives and clearly Plainsville is such an area Plainsville merchants report that sales of running shoes and exercise clothing are at all-time highs The local health club has more members than ever and the weight training and aerobics classes are always full Finally Plainsvilles schoolchildren represent a new generation of potential customers these schoolchildren are required to participate in a fitness-for-life program which emphasizes the benefits of regular exercise at an early age

Write a response in which you examine the stated andor unstated assumptions of the argument Be sure to

GRE AWA John박 박정어학원

explain how the argument depends on these assumptions and what the implications are for the argument if the assumptions prove unwarranted

False cause

First of all the author believes that the Increased sales of running shoes and exercise clothing indicates

Plainesville residentsrsquo interest in leading healthy lives However this assumption is not logically convincing for

several reasons could be a fashion trendTime shift ldquoFitness for liferdquo might not have any influence on schoolchildren as they growFalse cause There could be other reasons for member increase in the health clubAll of the above are insufficient condition

The author has to prove that local residents are interested in leading healthy lives However he supports his conclusion with insufficient evidence Nevertheless even if the residents are concerned with health naturersquos way may not be successful First

그 동안의 경험을 토대로 볼 때 건강생활과 밀접히 관련되어 있는 거주 지역에서 본 상점들이 아주 호응을 얻고 있다 따라서 이러한 주민들이 많이 거주하고 있는 Plainsville 에 새로운 상점들을 계속 세워야 한다 이 지역 상인들은 런닝화와 운동복 판매가 가장 높다고 말한다 불과 5 년전에는 거의 전무하다시피하던 지역 헬스 클럽의 경우도 엄청나게 많은 회원을 확보하고 있으며 웨이트 트레이닝과 에어로빅 강좌들도 항상 만원이라고 한다 새로운 고객층을 예측해 보는 것도 가능하다 이 지역의 학생들의 경우 Fitness for Life프로그램을 받게 되는데 이러한 프로그램을 통해서 유년시절부터 정규적인 운동 습관을 들이게 하고 있는 것이 그것이다

결론 We should therefore build our next new store in Plainsville

반박 그동안의 경험에 의한 과거 통계가 꼭 여기에도 적용되는건 아니다 5 년전 헬스 클럽이 잘 안되었던게 다른 원인이였을수 있다(강사수준 미달 강좌미비)tourist 에 의한 원인 일수 있다 어렸을때부터 운동을 했다고 해서 커서도 관심이 있지는 않다 (오히려 반감이 있을수 있다 혹은 건강하기에 건강에 관심이 적을수도 있다)운동복이나 신발의 판매가 육체노동에 의한 것일수도 있다

IntroductionSupport1049896In this memorandum the author asserts that Naturersquos Way should build its next newstore in Plainsville To support this assertion the author states that Plainsvillesmerchantsrsquo sales of exercise clothing are going well the local health club has moremembers than ever and a new generation of customers will help to ensure NaturersquosWayrsquos success At first glance the authorrsquos assumption seems convincing but in-depth scrutiny revealsthat it lacks substantial evidence as it stands

Body 1-SamplingTopic Sentence 1To begin with the author assumes that the merchantsrsquo report indicates that the residentsare concerned about their health However this assumption is based on unsubstantiated

GRE AWA John박 박정어학원

data Example 1 (Rebuttal1) First if we do not know the total volume of items sold and the price of the goods exactly we cannot infer whether the residents are actually buying many goods Example 2 (Rebuttal2)In addition to that the report emphasizes the rising sales of running shoes and exerciseclothing however these may not be hot-selling items for Naturersquos Way or may not be theproducts the company is planning to sell Concluding Sentence Therefore in order to make the argument reliable the author should reconsider themerchantsrsquo report with more detailed data

Body 2-CausalTopic Sentence 2Second the author contends that the health clubs classes are full yet this does not meanthat many people actually use the health club other factors may be the real cause forthose closed classes Example 1 (Rebuttal1) To begin with if the health club is very small the number of people working out wouldnot be a large one In fact regular gym-going may just be a vogue among a smallunrepresentative segment of Plainsvilles population Example 2 (Rebuttal2) Moreover it is possible that most of the people who exercise in the health club do weight training and aerobics only to look good and to meet other singles not for their health In that case there would be little demand for health products Concluding SentenceThus the author should not hasten to presume what really caused people to be interested in a healthier lifestyle and enroll in the health club

Body 3-Time-ShiftTopic Sentence 3Finally the author highlights that Naturersquos Way can expect a new generation of customersin Plainsville that will help the company in the long term This notion is mistaken in that itassumes the conditions of the present will continue unchanged in the future Although theschool children are required to participate in the fitness for life program they may notnecessarily buy Naturersquos Ways products Example 1 (Rebuttal1) In the first instance they may suffer a fall in purchasing power arising from future economic difficulties this would cause reluctance to spend a considerable amount of money on health products which tend to be more expensive Example 2 (Rebuttal2)Another possibility is that there may emerge many competitor companies vying with Naturersquos Way so that in the future the school children may not feel the necessity to purchase one companyrsquos health products over anotherrsquosConcluding Sentence Thus the authorrsquos assumption is highly speculative since it relies heavily on unknowablefuture circumstances

ConclusionThesis In sum the author uses many assumptions that are insufficient in supporting his claimsSupportIn order for the authorrsquos claims to be convincing he needs to advance more persuasiveevidence that people in Plainsville really are concerned with their health and health foodThe following was written as a part of an application for a small-business loan by a group of developers in the city of Monroe

A jazz music club in Monroe would be a tremendously profitable enterprise Currently the nearest jazz club is 65 miles away thus the proposed new jazz club in Monroe the C-Note would have the local market all to itself Plus jazz is extremely popular in Monroe over 100000 people attended Monroes annual jazz festival last summer several well-known jazz musicians live in Monroe and the highest-rated radio program in Monroe is Jazz Nightly which airs every weeknight at 7 PM Finally a nationwide study indicates that the typical jazz fan spends close to $1000 per year on jazz entertainment

1 Write a response in which you discuss what specific evidence is needed to evaluate the argument and explain how the evidence would weaken or strengthen the argument

2 Write a response in which you examine the stated andor unstated assumptions of the argument Be

GRE AWA John박 박정어학원

sure to explain how the argument depends on these assumptions and what the implications are for the argument if the assumptions prove unwarranted

3 Write a response in which you discuss what questions would need to be answered in order to decide whether the prediction and the argument on which it is based are reasonable Be sure to explain how the answers to these questions would help to evaluate the prediction

Group error nationwide survey may not reflect local trends Is the nationwide jazz fan population substantialInsufficient non-residents of Monroe may have attended the jazz festival (Body alternative explanation last year may have been an anomaly The author should consider data from various years) The author should indicate how many out of 100000 were Monroe residentsNationwide study Does this reflect Insufficient Citizens of Monroe may continue to go to the jazz club 65 miles away

Are the people in Monroe really interested in jazzMajority of the people who attended the jazz festival might not be Monroe residentsSurvey error nationwide study may not be applicable to MonroeJazz musicians who live in MonroeMonopolyRadio station

In this business application the author claims that the proposed jazz club C Note will be very profitable in Monroe To support this claim the author argues for his case with several evidences At first glance the authorrsquos argument seems convincing however careful scrutiny reveals that his argument in specious

To begin with the author claims that Monroersquos citizens are interested in jazz He presents three evidences First Secondhellip Thirdhellip Howeverhellip

Monroe 시에 있는 재즈 음악 클럽은 수익성이 좋은 사업이다 현재 가장 가까이에 있는 클럽은 65 마일 정도 떨어져 있다 따라서 이번에 세우려고 하는 C Note 는 독보적인 위치를 점할것이다 더군다나 재즈는 이 시에서 가장 인기있는 음악이다 지난 여름 재즈 축제에서는 10 만명 이상의 Morone 시 주민이 참석하였고 몇몇 유명한 재즈 음악가들도 이곳에 살고 있으며 저녁때 방영되는 라디오 프로그램중에서 최고의 시청률을 보이고 있는 것도 Jazz Nightly 이다 전국조사에서도 전형적인 재즈 팬들은 재즈 분야에 년간 1천 달러 가까이 지출하고 있는 것으로 보고되고 있다 따라서 C Note 클럽이 돈을 벌 수 있는 사업이라는 것은 확실한 것이다

결과 It is clear that the C Note cannot help but make money반박 nearest jazz club 이 양질의 써비스로 여전히 손님을 끌수도 있다Festival 에 얼마나 참여하는지가 jazz 의 인기를 반영하지 않는다 뮤지션이 많이 사는거랑 jazz 의 인기가 상관없다라디오 프로그램이 다른 요인에 의해서 인기일수도 있다 (진행자때문)전국 통계 적용 불가화목 실전반_Ms Noh6In this application the author suggests that a jazz club in Monroe will make a number of profits To support this suggestion the author exemplifies the local condition popularity of jazz in Monroe and nationwide study However careful scrutiny of each of the facts reveals that it provides little credible support for the authorrsquos recommendation Good clear intro

First the author assumes that jazz is popular in Monroe because of several facts the jazz festival last year had high participation some famous jazz musicians live in Monroe and the high-rated radio program is lsquoJazz Nightlyrsquo However this assumption has many drawbacks that must be seriously considered(Good topic sentences) If many attendants in the last-yearrsquos festival came from other cities and not Monroe it is hard to conclude that Monroersquos people like jazz Therefore the author must examine how many Monroe residents actually attended the festival On top of that there is little relationship between habitation of famous jazz musician and the popularity of jazz in Monroe Although several well-known musicians live there if they do not take part in any jazz performance of Monroe this might have no effect to the interest of Monroersquos residents

GRE AWA John박 박정어학원

about jazz Finally in the case of radio program this is also not suitable reason why jazz is popular in Monroe It might be possible that people cannot help choosing lsquoJazz Nightlyrsquo because there are few radio programs at Night The fact that the radio program is the highest rating program is not a germane evidence The approximate number of listeners would be the more crucial evidence Therefore the author needs to seriously deliberate the correlation between jazzrsquos popularity in Monroe and his examples (Good logical flow and clarity)

Second the author uses as evidence the nationwide study that jazz fans spend much money on jazz entertainment to substantiate why starting a jazz club in Monroe will be profitable In other words the author assumes that the characteristics of a nationwide study can be applied to Monroe The national study would lend support to the applicantrsquos claim only if residents in Monroe typify national jazz fans However the author does not provide credible evidence that this is the case Moreover the populations of jazz fans nationwide may be insubstantial Thus the author should not infer hastily that Monroersquos residents will spend much money on enjoying jazz from the nationwide study

Lastly even if jazz is popular in Monroe C Note may not be successful It is entirely possible that residents might still prefer other clubs where they have always went In addition there is another possibility that the nearest jazz club will attract many of Monroersquos people because it serves fine performances and is equipped with favorite facilities Without considering these other possibilities the author cannot make his argument convincing In sum the author presents many reasons that are insufficient in supporting his or her claim In order for the authorrsquos claims to be convincing he needs to advance more persuasive evidence such as the total number of Monroe residents who attended the jazz festival the effects on the popularity of jazz by the musicians living in Monroe and the actual number of residents who would typify themselves to be jazz fans through a local survey Without substantial evidence that C Note will be successful in Monroe the businessmen may be overinvesting in what might lead to a business failureExcellent clarity Score 50

The following appeared in a newsletter offering advice to investors

Over 80 percent of the respondents to a recent survey indicated a desire to reduce their intake of foods containing fats and cholesterol and today low-fat products abound in many food stores Since many of the food products currently marketed by Old Dairy Industries are high in fat and cholesterol the companys sales are likely to diminish greatly and company profits will no doubt decrease We therefore advise Old Dairy stockholders to sell their shares and other investors not to purchase stock in this company

Write a response in which you discuss what questions would need to be answered in order to decide whether the advice and the argument on which it is based are reasonable Be sure to explain how the answers to these questions would help to evaluate the advice

Survey 80

GRE AWA John박 박정어학원

Old Dairy could change their products and manufacture low fat dairy foodsLess competing companies Old Dairy could eventually be the only company that produces hellipImprecise numbers and measurementsCustomers may still buy high fat dairy products

The author of the newsletter is offering potentially dangerous advice by recommending Old Dairy stockholders to withdraw investment and stop purchase What is more the authorrsquos prediction debases the reputation and business of Old Dairy and if false could devoid the investment opportunity of the newsletter readers Therefore investors should examine whether the authorrsquos evidences are substantial

To begin with the author states that 80 percent of the respondents in a survey indicated a desire to reduce their intake of foods He therefore argues that Old Dairyrsquos high fat and cholesterol products would decrease in sales However the author makes a crucial error in this argument First the author provides no evidence that the surveyrsquos results are statistically reliable Were they representative of all the customers Were they chosen for the survey randomly Furthermore the desire to reduce fat and cholesterol intake is a pervasive trend in todayrsquos opulent society however the author erroneously identifies this as a new phenomenon which will affect consumer trends Second having a desire to reduce fat and cholesterol intake does not necessarily indicate that people who have this desire will actually reduce consuming these types of products It is entirely possible that they may continue buying Old Dairy products for its quality and taste Accordingly the author cannot draw any firm conclusion that people will not buy Old Dairy products Therefore if any of these cases are true the author may be offering investors a detrimental investment advice

최근 조사에 대한 응답자중 80 이상이 자신이 먹는 음식에서 지방과 콜레스테롤의 함유량을 줄이고 싶다고 한다 아울러 요즘은 많은 식료품 가계에서 저지방 제품들을 많이 취급하고 있다 현재 Old Dairy Industries가 판매하고 있는 많은 음식제품들은 지방과 콜레스테롤이 높기 때문에 이 회사의 매출이 격감할 것으로 보이며 당연히 매출이익도 줄어들것이다 따라서 이 회사의 주주들은 주식을 매각하고 다른 주식 투자가들도 이 회사의 주식을 매입하지 않는 것이 좋다

결론 Old Dairy stockholders to sell their shares and other investors not to purchase stock in this company

반박 모든 상품이 다 고 지방 고 칼로리는 아니다(비록 많을지라도) 일부의 식품의 경우 기호에 맞어서 히트해서 전체적인 수입이 증가할 수도 있다국내시장만 생각할 수 없다( 외국시장에서 호황을 누릴수 있다 )입맛이라는게 즉각 바뀌는게 아니다

The following appeared in a letter to the editor of the Balmer Island Gazette

On Balmer Island where mopeds serve as a popular form of transportation the population increases to 100000 during the summer months To reduce the number of accidents involving mopeds and pedestrians the town council of Balmer Island should limit the number of mopeds rented by the islands moped rental companies from 50 per day to 25 per day during the summer season By limiting the number of rentals the town council will attain the 50 percent annual reduction in moped accidents that was achieved last year on the neighboring island of Seaville when Seavilles town council enforced similar limits on moped rentals

1 Write a response in which you discuss what questions would need to be answered in order to decide whether the recommendation is likely to have the predicted result Be sure to explain how the answers to these questions would help to evaluate the recommendation

2 Write a response in which you discuss what questions would need to be answered in order to decide whether the prediction and the argument on which it is based are reasonable Be sure to explain how the answers to these questions would help to evaluate the prediction

3 Write a response in which you examine the stated andor unstated assumptions of the argument Be sure to explain how the argument depends on these assumptions and what the implications are for the argument if the assumptions prove unwarranted

Whatrsquos the actual population of Balmer Island 100000mdashis this a significant increase What kind of accidents Skin abrasions or serious injury And compared to Seaville how serious are the accidents and the actual number of accidents Did Seaville enforce other restrictions like safety signsHow different are the conditions of Balmer

GRE AWA John박 박정어학원

and Seaville regarding population road (safety) conditions topography other town-government regulation How much will the economy of Balmer be affected do to this restriction Could it cause an economic recession due to the fact that these rental companiesrsquo chance to make money is only during the summer thereby weakening the economic infrastructure Are there any other ways that could better alleviate the accident rate

Statistics 50-impreciseAnalogy Balmer compared with TorseauFalse Cause Accidents might have occurred because of reasons other than mopeds False Cause population increase may not be part of the cause of the accidentsOther explanations for the accident pedestrians few road safety regulations narrow roadsThere could be other better solutionshellip

Balmer Island의 인구가 여름철에는 십만명으로 늘어난다 2륜차와 보행자간 사고를 줄이기 위해 시의회는 6개의 자전거를 포함한 2륜차 대여업체에게 이 기간동안에는 대여숫자를 일일 50에서 30으로 제한하도록 할 것이다 대여숫자를 줄임으로써 시 의회는 지난해 이웃한 Torseau섬에서 이와 동일한 규제를 시행해서 50나 줄인 결과를 보고 마찬가지로 50를 줄일수 있다고 확신하고 있다

결론 The town council of Balmer Island should linit the number

반박 보행자의 부실에 의해서 사고가 많이 일어날수도 있다렌탈수의 줄임만이 대책은 아니다(대부분의 사람들이 렌탈 보다는 소유하고 있을 수도 있다)옆섬과는 상황이 다를수도 있다(그 섬에서는 사고의 원인이 많은 자전거 수로 인한것일수있다) 하지만 이 섬은 좁은 도로가 원인일 수도 있고 도로 안전 장치의미비가 원일일수 있다

In this letter the author recommends that Balmer Island should limit the number moped rentals from 50 to 30 per day To support this recommendation the author points out several reasons However careful scrutiny of each of the facts reveals that it is filled with unanswered questions that could significantly weaken the authorrsquos recommendation with loops and holes which are answered

The recommendation depends on the assumption that no alternative means of reducing the number of accidents are available However the author fails to offer any evidence to substantiate this crucial assumption It is highly possible that means other than this would better solve the problem Perhaps they could widen the roads or put-up more safety signs Or perhaps the accidents were due to the lack of skills in which case proper safety training would significantly alleviate the problem Without considering and ruling out these and other alternative means of reducing accidetns the author cannot confidently conclude that merely emulating Torseau would suffice Moreover the author is advising a recommendation which could potentially harm the economy of Balmer Island sincehellip Moreover the Balmer Island should alternative means to reduce accidents because limiting moped rentals during the summer could harm the economy of Balmerhellip

First of all the author believes that increase in population and the number of moped rentals are responsible for the accidents It is entirely possible that other factors are responsible for the accidents Perhaps Balmer Islandrsquos lack of safety signs was a major factor Or maybe the roads are narrow and dangerous on the Island therefore the town council could enforce stricter traffic regulations to alleviate the problem Accordingly if either of these scenarios is true the author cannot draw any firm conclusion that increase in the number of population and moped rentals are the cause of the accidents

The author of this editorial recommends that to reduce accidents involving mopeds andpedestrians Balmer Islands city council should restrict moped rentals to 30 per day down from50 at each of the islands six rental outlets To support this recommendation the author citesthe fact that last year when nearby Torseau Islands town council enforced similar measuresTorseaus rate of moped accidents fell by 50 For several reasons this evidence providesscant support for the authors recommendationTo begin with the author assumes that all other conditions in Balmer that might affect therate of moped-pedestrian accidents will remain unchanged after the restrictions are enactedHowever with a restricted supply of rental mopeds people in Balmer might purchase mopedsinstead Also the number of pedestrians might increase in the future with more pedestriansespecially tourists the risk of moped-pedestrian accidents would probably increase For thatmatter the number of rental outlets might increase to make up for the artificial supplyrestriction per outlet--a likely scenario assuming moped rental demand does not declineWithout considering and ruling out these and other possible changes that might contribute to ahigh incidence of moped-pedestrian accidents the author cannot convince me that theproposed restrictions will necessarily have the desired effect

GRE AWA John박 박정어학원

Next the author fails to consider other possible explanations for the 50 decline inTorseaus moped accident rate last year Perhaps last year Torseau experienced unusually fairweather during which moped accidents are less likely Perhaps fewer tourists visited Tot seanlast year than during most years thereby diminishing the demand for rental mopeds to belowthe allowed limits Perhaps last year some of Torseaus moped rental outlets purchased newmopeds that are safer to drive Or perhaps the restrictions were already in effect but were notenforced until last year In any event a decline in Torseaus moped accident rate during onlyone year is scarcely sufficient to draw any reliable conclusions about what might have causedthe decline or about what the accident rate will be in years aheadAdditionally in asserting that the same phenomenon that caused a 50 decline in mopedaccidents in Torseau would cause a similar decline in Balmer the author relies on what mightamount to an unfair analogy between Balmer and Torseau Perhaps Balmers ability to enforcemoped-rental restrictions does not meet Torseaus ability if not then the mere enactment ofsimilar restrictions in Balmer is no guarantee of a similar result Or perhaps the demand formopeds in Torseau is always greater than in Balmer Specifically if fewer than all availablemopeds are currently rented per day from the average Balmer outlet while in Torseau everyavailable moped is rented each day then the proposed restriction is likely to have less impacton the accident rate in Balmer than in TorseauFinally the author provides no evidence that the same restrictions that served to reduce theincidence of all moped accidents by 50 would also serve to reduce the incidence ofaccidents involving mopeds and pedestrians by 50 Lacking such evidence it is entirelypossible that the number of moped accidents not involving pedestrians decreased by a greaterpercentage while the number of moped-pedestrian accidents decreased by a smallerpercentage or even increased Since the author has not accounted for these possibilities theeditorials recommendation cannot be taken seriouslyIn conclusion the recommendation is not well supported To convince me that the proposedrestriction would achieve the desired outcome the author would have to assure me that nochanges serving to increase Balmers moped-pedestrian accident rate will occur in theforeseeable future The author must also provide dear evidence that last years decline inmoped accidents in Torseau was attributable primarily to its moped rental restrictions ratherthan to one or more other factors In order to better evaluate the recommendation I wouldneed more information comparing the supply of and demand for moped rentals on the twoislands I would also need to know the rate of mopedpedestrian accidents in Torseau both priorto and after the restrictions were enforced in TorseauThe following appeared in a magazine article about planning for retirement

Clearview should be a top choice for anyone seeking a place to retire because it has spectacular natural beauty and a consistent climate Another advantage is that housing costs in Clearview have fallen significantly during the past year and taxes remain lower than those in neighboring towns Moreover Clearviews mayor promises many new programs to improve schools streets and public services And best of all retirees in Clearview can also expect excellent health care as they grow older since the number of physicians in the area is far greater than the national average

Write a response in which you discuss what specific evidence is needed to evaluate the argument and explain how the evidence would weaken or strengthen the argument

-Natural beauty and consistent climate may not be the most wanted qualities-Housing costs could have lowered on a national level wealthy retirees may not care about costs-Taxes may be high compared to the nationrsquos average tax rate-What about other qualities of Clearview Crime rate what qualities would retirees want -If schools streets and public services need improvement then this is proof that the current condition of Clearview is low Or due to budgetary reasons the mayor may not follow-up on his promise because of lowered tax rate -Schools and people who are retired no relationship-Physicians What kind of physicians Number is irrelevant Are these physicians capable of addressing the illnesses of old people

This author argues that anyone seeking a place to retire should choose Clearview To supportthis argument the article cites Clearviews consistent climate and natural beauty its fallinghousing costs its low property taxes compared to nearby towns and the mayors promise toimprove schools streets and services The article also claims that retirees can expectexcellent health care because the number of physicians in Clearview greatly exceeds thenational average This argument is flawed in several critical respectsTo begin with although consistent climate and natural beauty might be attractive to manyretirees these features are probably not important to all retirees For many retirees it isprobably more important to live near relatives or even to enjoy changing seasons Thus I

GRE AWA John박 박정어학원

cannot accept the authors sweeping recommendation for all retirees on this basisAlso Clearviews declining housing costs do not necessarily make Clearview the best placeto retire for two reasons First despite the decline Clearviews housing costs might be highcompared to housing costs in other cities Secondly for wealthier retirees housing costs arenot likely to be a factor in choosing a place to retire Thus the mere fact that housing costshave been in decline lends scant support to the recommendationThe articles reliance on Clearviews property-tax rates is also problematic in two respectsFirst retirees obviously have innumerable choices about where to retire besides Clear viewand nearby towns Secondly for retirees who are well-off financially property taxes are notlikely to be an important concern in choosing a place to retire Thus it is unfair to infer fromClearviews property-tax rates that retirees would prefer ClearviewYet another problem with the argument involves the mayors promises In light of Clearviewslow property-tax rates whether the mayor can follow through on those promises is highlyquestionable Absent any explanation of how the city can spend more money in the areas citedwithout raising property taxes I simply cannot accept the editorials recommendation on thebasis of those promises Besides even if the city makes the improvements promised thoseimprovements--particular the ones to schools--would not necessarily be important to retireesFinally although the number of physicians in Clearview is relatively high the per capitanumber might be relatively low Moreover it would be fairer to compare this per capita numberwith the per capita number for other attractive retirement towns--rather than the nationalaverage After all retirees are likely to place a relatively heavy burden on health-careresources Besides the article provides no assurances that the number of physicians inClearview will remain high in the foreseeable futureIn conclusion the recommendation is poorly supported To strengthen it the author mustconvince me--perhaps by way of a reliable survey--that the key features that the vast majorityof retirees look for in choosing a place to live are consistent climate natural beauty and lowhousing costs The author must also provide better evidence that Clear views property taxesare lower than the those of cities in other areas The author must also explain how the city canmake its promised improvements without raising property taxes Finally to better assess theargument I would need to now how the per capita number of physicians in Clearview wouldcompare to the national average in the futureThe following appeared as a letter to the editor from a Central Plaza store owner

Over the past two years the number of shoppers in Central Plaza has been steadily decreasing while the popularity of skateboarding has increased dramatically Many Central Plaza store owners believe that the decrease in their business is due to the number of skateboard users in the plaza There has also been a dramatic increase in the amount of litter and vandalism throughout the plaza Thus we recommend that the city prohibit skateboarding in Central Plaza If skateboarding is prohibited here we predict that business in Central Plaza will return to its previously high levels

Write a response in which you discuss what questions would need to be answered in order to decide whether the recommendation is likely to have the predicted result Be sure to explain how the answers to these questions would help to evaluate the recommendation

Why two years ago What happened two years ago which started this declineIs the dramatic increase in the ldquopopularityrdquo of skateboarding the cause of the steady decline of shoppers Are there any malls nearby Were there any changes nearby which could affect the decline in customersmdasha big mall perhaps Could the decline be due to the shop ownersHow many skateboarders use the plazaWhere do they skateboardDo they shop and are they customersAre the increase in litter and vandalism due to skateboarders Could this be alleviated by installing CCTVs and hiring security

This editorial concludes that the city should ban skateboarding from its downtown CentralPlaza in order to attract visitors to that area to return the area to its former glory and to makeit a place where people can congregate for fun and relaxation To justify this conclusion theeditorial points out that skateboarders are nearly the only people one sees anymore at CentralPlaza and that the Plaza is littered and its property defaced The editorial also points out thatthe majority of downtown merchants support the skate boarding ban This argument is flawedin several critical respectsFirst the editorials author falsely assumes that a ban on skateboarding is both necessaryand sufficient to achieve the three stated objectives Perhaps the city can achieve thoseobjectives by other means as well--for example by creating a new mall that incorporates anattractive new skateboard park Even if banning skateboarders altogether is necessary to meetthe citys goals the author has not shown that this action by itself would suffice Assuming thatthe Plazas reputation is now tarnished restoring that reputation and in turn enticing peopleback to the Plaza might require additional measures--such as removing litter and graffiti

GRE AWA John박 박정어학원

promoting the Plaza to the public or enticing popular restaurant or retail chains to the PlazaSecondly the editorial assumes too hastily that the Plazas decline is attributable to theskateboarders--rather than to some other phenomenon Perhaps the Plazas primary appeal inits glory days had to do with particular shops or eateries which were eventually replaced byless appealing ones Or perhaps the crime rate in surrounding areas has risen dramatically forreasons unrelated to the skateboarders presence at the Plaza Without ruling out these andother alternative explanations for the Plazas decline the editorials author cannot convince methat a skateboard ban would reverse that declineThirdly the editorials author might be confusing cause with effect--by assuming that theskateboarders caused the abandonment of the Plaza rather than vice versa It is entirelypossible that skateboarders did not frequent the Plaza until it was largely abandoned--andbecause it had been abandoned In fact this scenario makes good sense since skateboardingis most enjoyable where there are few pedestrians or motorists to get in the wayFourth it is unreasonable to infer from the mere fact that most merchants favor the ban thatthe ban would be effective in achieving the citys objectives Admittedly perhaps thesemerchants would be more likely to help dean up the Plaza area and promote their businesseswere the city to act in accordance with their preference Yet lacking any supporting evidencethe author cannot convince me of this Thus the survey amounts to scant evidence at best thatthe proposed ban would carry the intended resultFinally the author recommends a course of action that might actually defeat the citysobjective of providing a fun and relaxing place for people to congregate In my experienceskateboarding contributes to an atmosphere of fun and relaxation for adults and children alikemore so than many other types of ambiance Without considering that continuing to allowskateboarding--or even encouraging this activity--might achieve the citys goal more effectivelythan banning the activity the author cannot convincingly conclude that the ban would be in thecitys best interestsIn sum the argument is a specious one To strengthen it the editorials author must providedear evidence that skateboarding and not some other factor is responsible for the conditionsmarking the Plazas decline The author must also convince me that no alternative means ofrestoring the Plaza are available to the city and that the proposed ban by itself would suffice toattract tourists and restore the Plaza to its former glory Finally to better assess the argument itwould be useful to know the circumstances under which the downtown merchants would bewilling to help the city achieve its objectives

6그룹 약한 비유 빈출

The following recommendation appeared in a memo from the mayor of the town of Hopewell

Two years ago the nearby town of Ocean View built a new municipal golf course and resort hotel During the past two years tourism in Ocean View has increased new businesses have opened there and Ocean Views tax revenues have risen by 30 percent Therefore the best way to improve Hopewells economymdashand generate additional tax revenuesmdashis to build a golf course and resort hotel similar to those in Ocean View

Write a response in which you examine the stated andor unstated assumptions of the argument Be sure to explain how the argument depends on these assumptions and what the implications are for the argument if the assumptions prove unwarranted

GRE AWA John박 박정어학원

Assumptions The author assumes that OVrsquos municipal golf course and resort hotel caused tourism new businesses and increased tax revenues There may be other reasons advertising promo He assumes that this will continueAssumes that Ocean View and Hopewell are similar in many waysmdashthe name suggests otherwise OV may have always been a tourist attractions for its beaches We need to know the topography

2년전 Ocean View 시는 시정 소유 골프 및 휴양지 호텔을 신축했다 그리고 지난 2년동안 이 시의 관광객이 증가했으며 새로운 사업들이 생겨났다 그에따라 시의 세수도 30나 증가했다 Hopewell의 경제를 향상시키고 아울러 세수를 늘릴 수 있는 가장 좋은 방법은 Ocean View에 세워진 것과 같은 골프 시설과 휴양지 호텔을 신축하는 것이다

1 다른 요인으로 관광 산업이 발전했을 수도 있다 문화 유적이 발견이 되었거나 도로의 정비등으로 여행자가 늘었을 수도 있다

2 관광 산업의증가가 늘어난 세수의 원인이 아니라 새로 유입된 인구의 증가나 다른 공장에서 발생한 것일 수 있다

3 2년동안 한참 골프가 붐을 이루었을 수 있다 경제상황이 나빠지거나 다른 레포츠가 인근 지역에 생겨난다면 골프하는 사람이 줄어들 수 있다

In this memo HopeweUs mayor recommends that in order to stimulate the towns economyand boost tax revenues HopeweU should build a new golf course and resort hotel just as thetown of Ocean View did two years ago To support this recommendation the mayor points outthat in Ocean View during the last two years tourism has increased new businesses haveopened and tax revenues have increased by 30 I find the mayors argument unconvincingin several important respectsFirst of all it is possible that the mayor has confused cause with effect respecting the recentdevelopments in Ocean View Perhaps Ocean Views construction of a new golf course andhotel was a response to previous increases in tourism and business development increasesthat have simply continued during the most recent two years Since the mayor has failed toaccount for this possibility the claim that Hopewell would boost its economy by alsoconstructing a golf course and hotel is completely unwarrantedSecondly the mayor fails to account for other possible causes of the trends in Ocean Viewduring the last two years The increase in tourism might have been due to improving economicconditions nationwide or to unusually pleasant weather in the region The new businessesthat have opened in Ocean View might have opened there irrespective of the new golf courseand hotel And the 30 increase in tax revenues might have been the result of an increase intax rates or the addition of a new type of municipal taxWithout ruling out these and other alternative explanations for the three recent trends inOcean View the mayor cannot reasonably infer based on those trends that Hopewellseconomy would benefit by following Ocean Views exampleThirdly even if the recent trends in Ocean View are attributable to the construction of the newgolf course and hotel there the mayor assumes too hastily that the golf course and hotel willcontinue to benefit that towns overall economy The mayor has not accounted for thepossibility that increased tourism will begin to drive residents away during tourist season orthat new business development will result in the towns losing its appeal as a place to visit or tolive Unless the mayor can convince me that these scenarios are unlikely I cannot accept themayors recommendation that Hopewell follow Ocean Views exampleFinally the mayors argument rests on the unsubstantiated assumption that Hopewell andOcean View are sufficiently alike in ways that might affect the economic impact of a new golfcourse and hotel Hopewell might lack the sort of natural environment that would attract moretourists and new businesses to the town--regardless of its new golf course and hotel For thatmatter perhaps Hopewell already contains several resort hotels and golf courses that are notutilized to their capacity If so building yet another golf course and hotel might amount to amisallocation of the towns resources--and actually harm the towns overall economyIn sum the mayors recommendation is not well supported To bolster it the mayor mustprovide better evidence that Ocean Views new golf course and hotel and not some otherphenomenon--has been responsible for boosting Ocean Views economy during the last twoyears To better assess the recommendation I would need to know why Ocean View decidedto construct its new golf course and hotel in the first place--specifically what events prior toconstruction might have prompted that decision I would also need to thoroughly compare

GRE AWA John박 박정어학원

HopeweU with Ocean View--especially in terms of their appeal to tourists and businesses--todetermine whether the same course of action that appears to have boosted Ocean Viewseconomy would also boost Hopewells economy

The following is part of a memorandum from the president of Humana University

Last year the number of students who enrolled in online degree programs offered by nearby Omni University increased by 50 percent During the same year Omni showed a significant decrease from prior years in expenditures for dormitory and classroom space most likely because instruction in the online programs takes place via the Internet In contrast over the past three years enrollment at Humana University has failed to grow and the cost of maintaining buildings has increased along with our budget deficit To address these problems Humana University will begin immediately to create and actively promote online degree programs like those at Omni We predict that instituting these online degree programs will help Humana both increase its total enrollment and solve its budget problems

Write a response in which you discuss what questions would need to be answered in order to decide whether the prediction and the argument on which it is based are reasonable Be sure to explain how the answers to these questions would help to evaluate the prediction

Is Omni University successful due to the online degree program 50 Is the decrease in expenditures for dormitory and classroom space due to the decrease in of on-campus students Which classes were successful Does HU have those classes

Even if the long-distance degree programs at Omni University benefited the school the presidentrsquos recommendation that Human College should emulate Omni University is too hasty First OUrsquos name implies that the school would have more majors than Humanahellip the president should examine which degrees were in the long-distance programhellip

지난해에는 Omni 대학에서 개강했던 원거리 학생 학점 취득 프로그램을 등록했던 학생들의 숫자가 50나 증가했다 같은해 기간동안 Omni 대학에서는 그 전년도부터 기숙사와 학급의 공간 확충을 위한 예산을 대폭 줄였는데 이는 이 원거리 학점 취득 프로그램이 양방향 비디오 컴퓨터 접속을 통해서만 가능한 수업지도 방식이기때문인 것으로 보인다 반면 지난 3개년 동안 Humana 대학에서의 수강률은 감소한데다가 건물

GRE AWA John박 박정어학원

유지비도 올랐다 따라서 Humana대학의 수강을 늘리고 예산손실을 회복하기 위해서는 Omni 대학에서 취한 조치와 같은 능동적인 프로그램을 추진해야 한다

결론 we should initiate and actively promote long-distance degree programs like those at Omni 반박 원거리 학생 취득 프로그램 숫자가 증가한거하고 예산이 줄어드는 것 사이에 연관이 약하다 (causal 학생의 증가로 관리비용 증가할수 있음 원거리 수업가능 장비도입에의한 비용발생)bad analogy(omni university 하고 같은 조건이 아니다 )-gt omni college 가 강좌내용이 좋아서 학생의 등록이 많을수 있다 Humana 대학에서 만들었다 하더라도 인기 없을수 있음다른 요인에 의해서 Humana 대학의 수강 인원이 증가할수 있음(비록 과거엔 인기가 없었을지라도)

The following appeared as part of a business plan developed by the manager of the Rialto Movie Theater

Despite its downtown location the Rialto Movie Theater a local institution for five decades must make big changes or close its doors forever It should follow the example of the new Apex Theater in the mall outside of town When the Apex opened last year it featured a video arcade plush carpeting and seats and a state-of-the-art sound system Furthermore in a recent survey over 85 percent of respondents reported that the high price of newly released movies prevents them from going to the movies more than five times per year Thus if the Rialto intends to hold on to its share of a decreasing pool of moviegoers it must offer the same features as Apex

Write a response in which you discuss what questions would need to be answered in order to decide whether the recommendation is likely to have the predicted result Be sure to explain how the answers to these questions would help to evaluate the recommendation

Before following through this business plan the manager should investigate the cause of Rialtorsquos unsuccessful business

The author provides no evidence that the surveyrsquos results are statistically reliable The surveyrsquos sample of 85 percent must be sufficient in size and representative of overall population of the city where Rialto and Apex is serving Lacking evidence of a sufficiently representative sample the author cannot justifiably rely on the survey to draw any conclusion whatsoever The author does not indicate that Apex is indeed currently successful However even if Apex is enjoying success the argument relies on what might be a false analogy between Rialto and Apex In order for Apex to serve as a model that Rialto should emulate the author must assume that all relevant circumstances are essentially the same However this assumption is unwarranted For example the argument overlooks the face that Apex is located in a strategic placemdashbeside a mall where customers can not only watch a movie but also enjoy shopping Therefore simply changing the facility to that of Apex may not lead to success

The author does not mention whether Apex is successful or not Nevertheless even if Apex is currently successful the argument relies on what might be a false analogy between Rialto and Apex In order for Apex to serve as a model that Rialto should emulate the author must assume that all relevant circumstances are essentially the same However this assumption is unwarranted For example the argument overlooks the fact that these two institutions are located in different locations Rialto in downtown and Apex in a mall outside of town Although Apex opened with state-of-the-art facilities the decisive factor in its success could be due to its strategic location of being in a mall People could enjoy both shopping and movies at one location thus they may prefer Apex over Rialto Furthermore the place where people enjoy leisure activities has shifted in the past decades for most cities from downtown to the suburbs Therefore Rialto may not be successful even if it emulates Apexrsquos facilities A better business plan may be relocating Apex to the thriving section of the downtown

Rialto 극장은 지난 50여년간 지역 회관으로써 시내에 위치해 있으면서도 이제 변화를 꾀하지 않으면 문을 닫을

GRE AWA John박 박정어학원

판이다 이 극장은 시외 쇼핑타운에 새로 들어선 Apex 극장의 사례를 본받아야 했다 Apex가 지난해 개업했을 당시 이 극장은 비디오 아케이드 플러쉬 카펫트 바닥과 좌석 그리고 최신 음향시설을 갖추었다 더군다나 최근 조사에서는 응답자의 85 이상이 새로 출시된 영화 입장료가 비싼 탓으로 지난해보다 5배이상의 관람객이 줄어들었다고 나타났다 따라서 Rialto 극장이 줄어들고 있는 관람객을 뺐기지 않고 유지하려면 Apex와 같은 시설들을 갖추어야 할 것이다주장 리알토 극장이 줄어들고 있는 관람객을 뺐기지 않고 유지하려면 Apex와 같은 시설들을 갖추어야 할 것이다

1 조사에서 응답자가 전체를 대표할 수 없다 2 apex 극장이 좋은 시설을 갖추고 있지만 그로 인해 수익이 많이 발생했다는 말이 없으므로 시설투자를

하고도 좋은 결과를 얻을 수 있을지 그 근거가 미흡하다3 좋은 영화가 출시된다면 입장료가 비싸도 영화관에서 꼭 보려고 할 수 있다 4 rialto 가 시설이 아닌 다른 요인에 의해 장사가 안될수도 있다( 우범 지역이라든지)

The following is a recommendation from the business manager of Monarch Books

Since its opening in Collegeville twenty years ago Monarch Books has developed a large customer base due to its reader-friendly atmosphere and wide selection of books on all subjects Last month Book and Bean a combination bookstore and coffee shop announced its intention to open a Collegeville store Monarch Books should open its own in-store cafeacute in the space currently devoted to childrens books Given recent national census data indicating a significant decline in the percentage of the population under age ten sales of childrens books are likely to decline By replacing its childrens books section with a cafeacute Monarch Books can increase profits and ward off competition from Book and Bean

Write a response in which you examine the stated andor unstated assumptions of the argument Be sure to explain how the argument depends on these assumptions and what the implications are for the argument if the assumptions prove unwarranted

The following is a recommendation from the business manager of Monarch Books

Since its opening in Collegeville twenty years ago Monarch Books has developed a large customer base due to its reader-friendly atmosphere and wide selection of books on all subjects Last month Book and Bean a combination bookstore and coffee shop announced its intention to open a Collegeville store Monarch Books should open its own in-store cafeacute in the space currently devoted to childrens books Given recent national census data indicating a significant decline in the percentage of the population under age ten sales of childrens books are likely to decline By replacing its childrens books section with a cafeacute Monarch Books can increase profits and ward off competition from Book and Bean

1 Write a response in which you discuss what questions would need to be answered in order to decide whether the recommendation is likely to have the predicted result Be sure to explain how the answers to these questions would help to evaluate the recommendation

2 Write a response in which you discuss what specific evidence is needed to evaluate the argument and explain how the evidence would weaken or strengthen the argument

No evidence regarding Monarch Bookrsquos successEven if Regal Bookrsquos is successful this may not be attributable to the cafeacute False analogy Emulating may not lead to success Other factors may be involvedInsufficient condition The national census is not enough evidence that childrenrsquos book sales will decline Can

GRE AWA John박 박정어학원

the national census represent the local child populationDid opening a cafeacute boost sales for Regal Books Even assuming Regal is successful by opening a cafeacute this may not be suitable for Monarch which plans to close the childrenrsquos book section to establish a cafe Imprecise language ldquorelatively little spacerdquo how smallThe managerrsquos recommendation contradicts what he says Since Monarch is popular for its wide selection of books closing a selection which targets a major group of readers may hurt Monarchrsquos salesIs this the best way to compete

When Stanley Park first opened it was the largest most heavily used public park in town It is still the largest park but it is no longer heavily used Video cameras mounted in the parks parking lots last month revealed the parks drop in popularity the recordings showed an average of only 50 cars per day In contrast tiny Carlton Park in the heart of the business district is visited by more than 150 people on a typical weekday An obvious difference is that Carlton Park unlike Stanley Park provides ample seating Thus if Stanley Park is ever to be as popular with our citizens as Carlton Park the town will obviously need to provide more benches thereby converting some of the unused open areas into spaces suitable for socializing

Write a response in which you examine the stated andor unstated assumptions of the argument Be sure to explain how the argument depends on these assumptions and what the implications are for the argument if the assumptions prove unwarranted

Stanley 파크가 처음 개장했을 당시 가장 크고 가장 많이 이용되는 공원이었다 아직도 공원중에서는 가장 크지만 이용률은 상당히 떨어졌다 지난달 공원 주차장에 설치해놓은 비디오 카메라를 통해 보면 drop(주차장으로 여겨짐) 이용률이 가장 높았다 수치상으로는 하루 평균 50대의 차량만이 이용하였다 반면 직장 중심거리에 위치한 작은 규모의 Carlton 파크는 주당 무려 150여명 이상이 이용하고 있다 Stanley 파크와는 달리 Carlton 파크에는 의자가 있다는 것이 가장 뚜렷한 차이점이다 따라서 Stanley 파크가 Carlton 파크처럼 시민들이 자주 이용하는 공원이 되기 위해서는 벤치를 설치할 필요가 있으며 이렇게 사용되지 않는 일부 공간을 활용해서 사교를 위한 공간으로 바꾸어야 한다 ===gtdrop 에 대한 첨부사항 (영영사전내용입니다)---- a place or central depository to which something (as mail money or stolen property) is brought for distribution or transmission also the act of depositing something at such a place dropgt

주장 if Stanley Park is ever to be as popular with our citizens as is Carlton Park the town will obviously need to provide more benches thereby converting some of the unused open areas into spaces suitable for socializing1 조사가 언제 이루어진 것인가 조사가 언제 실시되었느냐에 따라 결과가 다를 수있다 현재는 다시 스탠리 파크가 늘어났었을 수 있다 2 벤치를 많이 설치했다고 해서 많은 관광객이 오지 않을수 있다(사람들이 벤치나 사교 공간을 원한다는 어떠한 자료도 없다)3스탠리 파크 주변에 교통 상황이 악화가 되었거나 칼튼 파크에서 문화행사등을 많이 가져서 이용객이 줄어든것일 수도 있다 4 칼튼 파크가 중심지에 있어서 접근성이 좋을수 있다5 조사가 같은 시간을 기준으로 한게 아니다(하나는 주중이고 하나는 주말이다)6사람의 수와 차의 대수를 같은것으로 비교할수 없다 (차안에 몇 명이 타고 있는지 모르고 대중교통을 이용해서 왔을수도 있다)

Page 26: GRE writing argument brain storm

GRE AWA John박 박정어학원

explain how the argument depends on these assumptions and what the implications are for the argument if the assumptions prove unwarranted

False cause

First of all the author believes that the Increased sales of running shoes and exercise clothing indicates

Plainesville residentsrsquo interest in leading healthy lives However this assumption is not logically convincing for

several reasons could be a fashion trendTime shift ldquoFitness for liferdquo might not have any influence on schoolchildren as they growFalse cause There could be other reasons for member increase in the health clubAll of the above are insufficient condition

The author has to prove that local residents are interested in leading healthy lives However he supports his conclusion with insufficient evidence Nevertheless even if the residents are concerned with health naturersquos way may not be successful First

그 동안의 경험을 토대로 볼 때 건강생활과 밀접히 관련되어 있는 거주 지역에서 본 상점들이 아주 호응을 얻고 있다 따라서 이러한 주민들이 많이 거주하고 있는 Plainsville 에 새로운 상점들을 계속 세워야 한다 이 지역 상인들은 런닝화와 운동복 판매가 가장 높다고 말한다 불과 5 년전에는 거의 전무하다시피하던 지역 헬스 클럽의 경우도 엄청나게 많은 회원을 확보하고 있으며 웨이트 트레이닝과 에어로빅 강좌들도 항상 만원이라고 한다 새로운 고객층을 예측해 보는 것도 가능하다 이 지역의 학생들의 경우 Fitness for Life프로그램을 받게 되는데 이러한 프로그램을 통해서 유년시절부터 정규적인 운동 습관을 들이게 하고 있는 것이 그것이다

결론 We should therefore build our next new store in Plainsville

반박 그동안의 경험에 의한 과거 통계가 꼭 여기에도 적용되는건 아니다 5 년전 헬스 클럽이 잘 안되었던게 다른 원인이였을수 있다(강사수준 미달 강좌미비)tourist 에 의한 원인 일수 있다 어렸을때부터 운동을 했다고 해서 커서도 관심이 있지는 않다 (오히려 반감이 있을수 있다 혹은 건강하기에 건강에 관심이 적을수도 있다)운동복이나 신발의 판매가 육체노동에 의한 것일수도 있다

IntroductionSupport1049896In this memorandum the author asserts that Naturersquos Way should build its next newstore in Plainsville To support this assertion the author states that Plainsvillesmerchantsrsquo sales of exercise clothing are going well the local health club has moremembers than ever and a new generation of customers will help to ensure NaturersquosWayrsquos success At first glance the authorrsquos assumption seems convincing but in-depth scrutiny revealsthat it lacks substantial evidence as it stands

Body 1-SamplingTopic Sentence 1To begin with the author assumes that the merchantsrsquo report indicates that the residentsare concerned about their health However this assumption is based on unsubstantiated

GRE AWA John박 박정어학원

data Example 1 (Rebuttal1) First if we do not know the total volume of items sold and the price of the goods exactly we cannot infer whether the residents are actually buying many goods Example 2 (Rebuttal2)In addition to that the report emphasizes the rising sales of running shoes and exerciseclothing however these may not be hot-selling items for Naturersquos Way or may not be theproducts the company is planning to sell Concluding Sentence Therefore in order to make the argument reliable the author should reconsider themerchantsrsquo report with more detailed data

Body 2-CausalTopic Sentence 2Second the author contends that the health clubs classes are full yet this does not meanthat many people actually use the health club other factors may be the real cause forthose closed classes Example 1 (Rebuttal1) To begin with if the health club is very small the number of people working out wouldnot be a large one In fact regular gym-going may just be a vogue among a smallunrepresentative segment of Plainsvilles population Example 2 (Rebuttal2) Moreover it is possible that most of the people who exercise in the health club do weight training and aerobics only to look good and to meet other singles not for their health In that case there would be little demand for health products Concluding SentenceThus the author should not hasten to presume what really caused people to be interested in a healthier lifestyle and enroll in the health club

Body 3-Time-ShiftTopic Sentence 3Finally the author highlights that Naturersquos Way can expect a new generation of customersin Plainsville that will help the company in the long term This notion is mistaken in that itassumes the conditions of the present will continue unchanged in the future Although theschool children are required to participate in the fitness for life program they may notnecessarily buy Naturersquos Ways products Example 1 (Rebuttal1) In the first instance they may suffer a fall in purchasing power arising from future economic difficulties this would cause reluctance to spend a considerable amount of money on health products which tend to be more expensive Example 2 (Rebuttal2)Another possibility is that there may emerge many competitor companies vying with Naturersquos Way so that in the future the school children may not feel the necessity to purchase one companyrsquos health products over anotherrsquosConcluding Sentence Thus the authorrsquos assumption is highly speculative since it relies heavily on unknowablefuture circumstances

ConclusionThesis In sum the author uses many assumptions that are insufficient in supporting his claimsSupportIn order for the authorrsquos claims to be convincing he needs to advance more persuasiveevidence that people in Plainsville really are concerned with their health and health foodThe following was written as a part of an application for a small-business loan by a group of developers in the city of Monroe

A jazz music club in Monroe would be a tremendously profitable enterprise Currently the nearest jazz club is 65 miles away thus the proposed new jazz club in Monroe the C-Note would have the local market all to itself Plus jazz is extremely popular in Monroe over 100000 people attended Monroes annual jazz festival last summer several well-known jazz musicians live in Monroe and the highest-rated radio program in Monroe is Jazz Nightly which airs every weeknight at 7 PM Finally a nationwide study indicates that the typical jazz fan spends close to $1000 per year on jazz entertainment

1 Write a response in which you discuss what specific evidence is needed to evaluate the argument and explain how the evidence would weaken or strengthen the argument

2 Write a response in which you examine the stated andor unstated assumptions of the argument Be

GRE AWA John박 박정어학원

sure to explain how the argument depends on these assumptions and what the implications are for the argument if the assumptions prove unwarranted

3 Write a response in which you discuss what questions would need to be answered in order to decide whether the prediction and the argument on which it is based are reasonable Be sure to explain how the answers to these questions would help to evaluate the prediction

Group error nationwide survey may not reflect local trends Is the nationwide jazz fan population substantialInsufficient non-residents of Monroe may have attended the jazz festival (Body alternative explanation last year may have been an anomaly The author should consider data from various years) The author should indicate how many out of 100000 were Monroe residentsNationwide study Does this reflect Insufficient Citizens of Monroe may continue to go to the jazz club 65 miles away

Are the people in Monroe really interested in jazzMajority of the people who attended the jazz festival might not be Monroe residentsSurvey error nationwide study may not be applicable to MonroeJazz musicians who live in MonroeMonopolyRadio station

In this business application the author claims that the proposed jazz club C Note will be very profitable in Monroe To support this claim the author argues for his case with several evidences At first glance the authorrsquos argument seems convincing however careful scrutiny reveals that his argument in specious

To begin with the author claims that Monroersquos citizens are interested in jazz He presents three evidences First Secondhellip Thirdhellip Howeverhellip

Monroe 시에 있는 재즈 음악 클럽은 수익성이 좋은 사업이다 현재 가장 가까이에 있는 클럽은 65 마일 정도 떨어져 있다 따라서 이번에 세우려고 하는 C Note 는 독보적인 위치를 점할것이다 더군다나 재즈는 이 시에서 가장 인기있는 음악이다 지난 여름 재즈 축제에서는 10 만명 이상의 Morone 시 주민이 참석하였고 몇몇 유명한 재즈 음악가들도 이곳에 살고 있으며 저녁때 방영되는 라디오 프로그램중에서 최고의 시청률을 보이고 있는 것도 Jazz Nightly 이다 전국조사에서도 전형적인 재즈 팬들은 재즈 분야에 년간 1천 달러 가까이 지출하고 있는 것으로 보고되고 있다 따라서 C Note 클럽이 돈을 벌 수 있는 사업이라는 것은 확실한 것이다

결과 It is clear that the C Note cannot help but make money반박 nearest jazz club 이 양질의 써비스로 여전히 손님을 끌수도 있다Festival 에 얼마나 참여하는지가 jazz 의 인기를 반영하지 않는다 뮤지션이 많이 사는거랑 jazz 의 인기가 상관없다라디오 프로그램이 다른 요인에 의해서 인기일수도 있다 (진행자때문)전국 통계 적용 불가화목 실전반_Ms Noh6In this application the author suggests that a jazz club in Monroe will make a number of profits To support this suggestion the author exemplifies the local condition popularity of jazz in Monroe and nationwide study However careful scrutiny of each of the facts reveals that it provides little credible support for the authorrsquos recommendation Good clear intro

First the author assumes that jazz is popular in Monroe because of several facts the jazz festival last year had high participation some famous jazz musicians live in Monroe and the high-rated radio program is lsquoJazz Nightlyrsquo However this assumption has many drawbacks that must be seriously considered(Good topic sentences) If many attendants in the last-yearrsquos festival came from other cities and not Monroe it is hard to conclude that Monroersquos people like jazz Therefore the author must examine how many Monroe residents actually attended the festival On top of that there is little relationship between habitation of famous jazz musician and the popularity of jazz in Monroe Although several well-known musicians live there if they do not take part in any jazz performance of Monroe this might have no effect to the interest of Monroersquos residents

GRE AWA John박 박정어학원

about jazz Finally in the case of radio program this is also not suitable reason why jazz is popular in Monroe It might be possible that people cannot help choosing lsquoJazz Nightlyrsquo because there are few radio programs at Night The fact that the radio program is the highest rating program is not a germane evidence The approximate number of listeners would be the more crucial evidence Therefore the author needs to seriously deliberate the correlation between jazzrsquos popularity in Monroe and his examples (Good logical flow and clarity)

Second the author uses as evidence the nationwide study that jazz fans spend much money on jazz entertainment to substantiate why starting a jazz club in Monroe will be profitable In other words the author assumes that the characteristics of a nationwide study can be applied to Monroe The national study would lend support to the applicantrsquos claim only if residents in Monroe typify national jazz fans However the author does not provide credible evidence that this is the case Moreover the populations of jazz fans nationwide may be insubstantial Thus the author should not infer hastily that Monroersquos residents will spend much money on enjoying jazz from the nationwide study

Lastly even if jazz is popular in Monroe C Note may not be successful It is entirely possible that residents might still prefer other clubs where they have always went In addition there is another possibility that the nearest jazz club will attract many of Monroersquos people because it serves fine performances and is equipped with favorite facilities Without considering these other possibilities the author cannot make his argument convincing In sum the author presents many reasons that are insufficient in supporting his or her claim In order for the authorrsquos claims to be convincing he needs to advance more persuasive evidence such as the total number of Monroe residents who attended the jazz festival the effects on the popularity of jazz by the musicians living in Monroe and the actual number of residents who would typify themselves to be jazz fans through a local survey Without substantial evidence that C Note will be successful in Monroe the businessmen may be overinvesting in what might lead to a business failureExcellent clarity Score 50

The following appeared in a newsletter offering advice to investors

Over 80 percent of the respondents to a recent survey indicated a desire to reduce their intake of foods containing fats and cholesterol and today low-fat products abound in many food stores Since many of the food products currently marketed by Old Dairy Industries are high in fat and cholesterol the companys sales are likely to diminish greatly and company profits will no doubt decrease We therefore advise Old Dairy stockholders to sell their shares and other investors not to purchase stock in this company

Write a response in which you discuss what questions would need to be answered in order to decide whether the advice and the argument on which it is based are reasonable Be sure to explain how the answers to these questions would help to evaluate the advice

Survey 80

GRE AWA John박 박정어학원

Old Dairy could change their products and manufacture low fat dairy foodsLess competing companies Old Dairy could eventually be the only company that produces hellipImprecise numbers and measurementsCustomers may still buy high fat dairy products

The author of the newsletter is offering potentially dangerous advice by recommending Old Dairy stockholders to withdraw investment and stop purchase What is more the authorrsquos prediction debases the reputation and business of Old Dairy and if false could devoid the investment opportunity of the newsletter readers Therefore investors should examine whether the authorrsquos evidences are substantial

To begin with the author states that 80 percent of the respondents in a survey indicated a desire to reduce their intake of foods He therefore argues that Old Dairyrsquos high fat and cholesterol products would decrease in sales However the author makes a crucial error in this argument First the author provides no evidence that the surveyrsquos results are statistically reliable Were they representative of all the customers Were they chosen for the survey randomly Furthermore the desire to reduce fat and cholesterol intake is a pervasive trend in todayrsquos opulent society however the author erroneously identifies this as a new phenomenon which will affect consumer trends Second having a desire to reduce fat and cholesterol intake does not necessarily indicate that people who have this desire will actually reduce consuming these types of products It is entirely possible that they may continue buying Old Dairy products for its quality and taste Accordingly the author cannot draw any firm conclusion that people will not buy Old Dairy products Therefore if any of these cases are true the author may be offering investors a detrimental investment advice

최근 조사에 대한 응답자중 80 이상이 자신이 먹는 음식에서 지방과 콜레스테롤의 함유량을 줄이고 싶다고 한다 아울러 요즘은 많은 식료품 가계에서 저지방 제품들을 많이 취급하고 있다 현재 Old Dairy Industries가 판매하고 있는 많은 음식제품들은 지방과 콜레스테롤이 높기 때문에 이 회사의 매출이 격감할 것으로 보이며 당연히 매출이익도 줄어들것이다 따라서 이 회사의 주주들은 주식을 매각하고 다른 주식 투자가들도 이 회사의 주식을 매입하지 않는 것이 좋다

결론 Old Dairy stockholders to sell their shares and other investors not to purchase stock in this company

반박 모든 상품이 다 고 지방 고 칼로리는 아니다(비록 많을지라도) 일부의 식품의 경우 기호에 맞어서 히트해서 전체적인 수입이 증가할 수도 있다국내시장만 생각할 수 없다( 외국시장에서 호황을 누릴수 있다 )입맛이라는게 즉각 바뀌는게 아니다

The following appeared in a letter to the editor of the Balmer Island Gazette

On Balmer Island where mopeds serve as a popular form of transportation the population increases to 100000 during the summer months To reduce the number of accidents involving mopeds and pedestrians the town council of Balmer Island should limit the number of mopeds rented by the islands moped rental companies from 50 per day to 25 per day during the summer season By limiting the number of rentals the town council will attain the 50 percent annual reduction in moped accidents that was achieved last year on the neighboring island of Seaville when Seavilles town council enforced similar limits on moped rentals

1 Write a response in which you discuss what questions would need to be answered in order to decide whether the recommendation is likely to have the predicted result Be sure to explain how the answers to these questions would help to evaluate the recommendation

2 Write a response in which you discuss what questions would need to be answered in order to decide whether the prediction and the argument on which it is based are reasonable Be sure to explain how the answers to these questions would help to evaluate the prediction

3 Write a response in which you examine the stated andor unstated assumptions of the argument Be sure to explain how the argument depends on these assumptions and what the implications are for the argument if the assumptions prove unwarranted

Whatrsquos the actual population of Balmer Island 100000mdashis this a significant increase What kind of accidents Skin abrasions or serious injury And compared to Seaville how serious are the accidents and the actual number of accidents Did Seaville enforce other restrictions like safety signsHow different are the conditions of Balmer

GRE AWA John박 박정어학원

and Seaville regarding population road (safety) conditions topography other town-government regulation How much will the economy of Balmer be affected do to this restriction Could it cause an economic recession due to the fact that these rental companiesrsquo chance to make money is only during the summer thereby weakening the economic infrastructure Are there any other ways that could better alleviate the accident rate

Statistics 50-impreciseAnalogy Balmer compared with TorseauFalse Cause Accidents might have occurred because of reasons other than mopeds False Cause population increase may not be part of the cause of the accidentsOther explanations for the accident pedestrians few road safety regulations narrow roadsThere could be other better solutionshellip

Balmer Island의 인구가 여름철에는 십만명으로 늘어난다 2륜차와 보행자간 사고를 줄이기 위해 시의회는 6개의 자전거를 포함한 2륜차 대여업체에게 이 기간동안에는 대여숫자를 일일 50에서 30으로 제한하도록 할 것이다 대여숫자를 줄임으로써 시 의회는 지난해 이웃한 Torseau섬에서 이와 동일한 규제를 시행해서 50나 줄인 결과를 보고 마찬가지로 50를 줄일수 있다고 확신하고 있다

결론 The town council of Balmer Island should linit the number

반박 보행자의 부실에 의해서 사고가 많이 일어날수도 있다렌탈수의 줄임만이 대책은 아니다(대부분의 사람들이 렌탈 보다는 소유하고 있을 수도 있다)옆섬과는 상황이 다를수도 있다(그 섬에서는 사고의 원인이 많은 자전거 수로 인한것일수있다) 하지만 이 섬은 좁은 도로가 원인일 수도 있고 도로 안전 장치의미비가 원일일수 있다

In this letter the author recommends that Balmer Island should limit the number moped rentals from 50 to 30 per day To support this recommendation the author points out several reasons However careful scrutiny of each of the facts reveals that it is filled with unanswered questions that could significantly weaken the authorrsquos recommendation with loops and holes which are answered

The recommendation depends on the assumption that no alternative means of reducing the number of accidents are available However the author fails to offer any evidence to substantiate this crucial assumption It is highly possible that means other than this would better solve the problem Perhaps they could widen the roads or put-up more safety signs Or perhaps the accidents were due to the lack of skills in which case proper safety training would significantly alleviate the problem Without considering and ruling out these and other alternative means of reducing accidetns the author cannot confidently conclude that merely emulating Torseau would suffice Moreover the author is advising a recommendation which could potentially harm the economy of Balmer Island sincehellip Moreover the Balmer Island should alternative means to reduce accidents because limiting moped rentals during the summer could harm the economy of Balmerhellip

First of all the author believes that increase in population and the number of moped rentals are responsible for the accidents It is entirely possible that other factors are responsible for the accidents Perhaps Balmer Islandrsquos lack of safety signs was a major factor Or maybe the roads are narrow and dangerous on the Island therefore the town council could enforce stricter traffic regulations to alleviate the problem Accordingly if either of these scenarios is true the author cannot draw any firm conclusion that increase in the number of population and moped rentals are the cause of the accidents

The author of this editorial recommends that to reduce accidents involving mopeds andpedestrians Balmer Islands city council should restrict moped rentals to 30 per day down from50 at each of the islands six rental outlets To support this recommendation the author citesthe fact that last year when nearby Torseau Islands town council enforced similar measuresTorseaus rate of moped accidents fell by 50 For several reasons this evidence providesscant support for the authors recommendationTo begin with the author assumes that all other conditions in Balmer that might affect therate of moped-pedestrian accidents will remain unchanged after the restrictions are enactedHowever with a restricted supply of rental mopeds people in Balmer might purchase mopedsinstead Also the number of pedestrians might increase in the future with more pedestriansespecially tourists the risk of moped-pedestrian accidents would probably increase For thatmatter the number of rental outlets might increase to make up for the artificial supplyrestriction per outlet--a likely scenario assuming moped rental demand does not declineWithout considering and ruling out these and other possible changes that might contribute to ahigh incidence of moped-pedestrian accidents the author cannot convince me that theproposed restrictions will necessarily have the desired effect

GRE AWA John박 박정어학원

Next the author fails to consider other possible explanations for the 50 decline inTorseaus moped accident rate last year Perhaps last year Torseau experienced unusually fairweather during which moped accidents are less likely Perhaps fewer tourists visited Tot seanlast year than during most years thereby diminishing the demand for rental mopeds to belowthe allowed limits Perhaps last year some of Torseaus moped rental outlets purchased newmopeds that are safer to drive Or perhaps the restrictions were already in effect but were notenforced until last year In any event a decline in Torseaus moped accident rate during onlyone year is scarcely sufficient to draw any reliable conclusions about what might have causedthe decline or about what the accident rate will be in years aheadAdditionally in asserting that the same phenomenon that caused a 50 decline in mopedaccidents in Torseau would cause a similar decline in Balmer the author relies on what mightamount to an unfair analogy between Balmer and Torseau Perhaps Balmers ability to enforcemoped-rental restrictions does not meet Torseaus ability if not then the mere enactment ofsimilar restrictions in Balmer is no guarantee of a similar result Or perhaps the demand formopeds in Torseau is always greater than in Balmer Specifically if fewer than all availablemopeds are currently rented per day from the average Balmer outlet while in Torseau everyavailable moped is rented each day then the proposed restriction is likely to have less impacton the accident rate in Balmer than in TorseauFinally the author provides no evidence that the same restrictions that served to reduce theincidence of all moped accidents by 50 would also serve to reduce the incidence ofaccidents involving mopeds and pedestrians by 50 Lacking such evidence it is entirelypossible that the number of moped accidents not involving pedestrians decreased by a greaterpercentage while the number of moped-pedestrian accidents decreased by a smallerpercentage or even increased Since the author has not accounted for these possibilities theeditorials recommendation cannot be taken seriouslyIn conclusion the recommendation is not well supported To convince me that the proposedrestriction would achieve the desired outcome the author would have to assure me that nochanges serving to increase Balmers moped-pedestrian accident rate will occur in theforeseeable future The author must also provide dear evidence that last years decline inmoped accidents in Torseau was attributable primarily to its moped rental restrictions ratherthan to one or more other factors In order to better evaluate the recommendation I wouldneed more information comparing the supply of and demand for moped rentals on the twoislands I would also need to know the rate of mopedpedestrian accidents in Torseau both priorto and after the restrictions were enforced in TorseauThe following appeared in a magazine article about planning for retirement

Clearview should be a top choice for anyone seeking a place to retire because it has spectacular natural beauty and a consistent climate Another advantage is that housing costs in Clearview have fallen significantly during the past year and taxes remain lower than those in neighboring towns Moreover Clearviews mayor promises many new programs to improve schools streets and public services And best of all retirees in Clearview can also expect excellent health care as they grow older since the number of physicians in the area is far greater than the national average

Write a response in which you discuss what specific evidence is needed to evaluate the argument and explain how the evidence would weaken or strengthen the argument

-Natural beauty and consistent climate may not be the most wanted qualities-Housing costs could have lowered on a national level wealthy retirees may not care about costs-Taxes may be high compared to the nationrsquos average tax rate-What about other qualities of Clearview Crime rate what qualities would retirees want -If schools streets and public services need improvement then this is proof that the current condition of Clearview is low Or due to budgetary reasons the mayor may not follow-up on his promise because of lowered tax rate -Schools and people who are retired no relationship-Physicians What kind of physicians Number is irrelevant Are these physicians capable of addressing the illnesses of old people

This author argues that anyone seeking a place to retire should choose Clearview To supportthis argument the article cites Clearviews consistent climate and natural beauty its fallinghousing costs its low property taxes compared to nearby towns and the mayors promise toimprove schools streets and services The article also claims that retirees can expectexcellent health care because the number of physicians in Clearview greatly exceeds thenational average This argument is flawed in several critical respectsTo begin with although consistent climate and natural beauty might be attractive to manyretirees these features are probably not important to all retirees For many retirees it isprobably more important to live near relatives or even to enjoy changing seasons Thus I

GRE AWA John박 박정어학원

cannot accept the authors sweeping recommendation for all retirees on this basisAlso Clearviews declining housing costs do not necessarily make Clearview the best placeto retire for two reasons First despite the decline Clearviews housing costs might be highcompared to housing costs in other cities Secondly for wealthier retirees housing costs arenot likely to be a factor in choosing a place to retire Thus the mere fact that housing costshave been in decline lends scant support to the recommendationThe articles reliance on Clearviews property-tax rates is also problematic in two respectsFirst retirees obviously have innumerable choices about where to retire besides Clear viewand nearby towns Secondly for retirees who are well-off financially property taxes are notlikely to be an important concern in choosing a place to retire Thus it is unfair to infer fromClearviews property-tax rates that retirees would prefer ClearviewYet another problem with the argument involves the mayors promises In light of Clearviewslow property-tax rates whether the mayor can follow through on those promises is highlyquestionable Absent any explanation of how the city can spend more money in the areas citedwithout raising property taxes I simply cannot accept the editorials recommendation on thebasis of those promises Besides even if the city makes the improvements promised thoseimprovements--particular the ones to schools--would not necessarily be important to retireesFinally although the number of physicians in Clearview is relatively high the per capitanumber might be relatively low Moreover it would be fairer to compare this per capita numberwith the per capita number for other attractive retirement towns--rather than the nationalaverage After all retirees are likely to place a relatively heavy burden on health-careresources Besides the article provides no assurances that the number of physicians inClearview will remain high in the foreseeable futureIn conclusion the recommendation is poorly supported To strengthen it the author mustconvince me--perhaps by way of a reliable survey--that the key features that the vast majorityof retirees look for in choosing a place to live are consistent climate natural beauty and lowhousing costs The author must also provide better evidence that Clear views property taxesare lower than the those of cities in other areas The author must also explain how the city canmake its promised improvements without raising property taxes Finally to better assess theargument I would need to now how the per capita number of physicians in Clearview wouldcompare to the national average in the futureThe following appeared as a letter to the editor from a Central Plaza store owner

Over the past two years the number of shoppers in Central Plaza has been steadily decreasing while the popularity of skateboarding has increased dramatically Many Central Plaza store owners believe that the decrease in their business is due to the number of skateboard users in the plaza There has also been a dramatic increase in the amount of litter and vandalism throughout the plaza Thus we recommend that the city prohibit skateboarding in Central Plaza If skateboarding is prohibited here we predict that business in Central Plaza will return to its previously high levels

Write a response in which you discuss what questions would need to be answered in order to decide whether the recommendation is likely to have the predicted result Be sure to explain how the answers to these questions would help to evaluate the recommendation

Why two years ago What happened two years ago which started this declineIs the dramatic increase in the ldquopopularityrdquo of skateboarding the cause of the steady decline of shoppers Are there any malls nearby Were there any changes nearby which could affect the decline in customersmdasha big mall perhaps Could the decline be due to the shop ownersHow many skateboarders use the plazaWhere do they skateboardDo they shop and are they customersAre the increase in litter and vandalism due to skateboarders Could this be alleviated by installing CCTVs and hiring security

This editorial concludes that the city should ban skateboarding from its downtown CentralPlaza in order to attract visitors to that area to return the area to its former glory and to makeit a place where people can congregate for fun and relaxation To justify this conclusion theeditorial points out that skateboarders are nearly the only people one sees anymore at CentralPlaza and that the Plaza is littered and its property defaced The editorial also points out thatthe majority of downtown merchants support the skate boarding ban This argument is flawedin several critical respectsFirst the editorials author falsely assumes that a ban on skateboarding is both necessaryand sufficient to achieve the three stated objectives Perhaps the city can achieve thoseobjectives by other means as well--for example by creating a new mall that incorporates anattractive new skateboard park Even if banning skateboarders altogether is necessary to meetthe citys goals the author has not shown that this action by itself would suffice Assuming thatthe Plazas reputation is now tarnished restoring that reputation and in turn enticing peopleback to the Plaza might require additional measures--such as removing litter and graffiti

GRE AWA John박 박정어학원

promoting the Plaza to the public or enticing popular restaurant or retail chains to the PlazaSecondly the editorial assumes too hastily that the Plazas decline is attributable to theskateboarders--rather than to some other phenomenon Perhaps the Plazas primary appeal inits glory days had to do with particular shops or eateries which were eventually replaced byless appealing ones Or perhaps the crime rate in surrounding areas has risen dramatically forreasons unrelated to the skateboarders presence at the Plaza Without ruling out these andother alternative explanations for the Plazas decline the editorials author cannot convince methat a skateboard ban would reverse that declineThirdly the editorials author might be confusing cause with effect--by assuming that theskateboarders caused the abandonment of the Plaza rather than vice versa It is entirelypossible that skateboarders did not frequent the Plaza until it was largely abandoned--andbecause it had been abandoned In fact this scenario makes good sense since skateboardingis most enjoyable where there are few pedestrians or motorists to get in the wayFourth it is unreasonable to infer from the mere fact that most merchants favor the ban thatthe ban would be effective in achieving the citys objectives Admittedly perhaps thesemerchants would be more likely to help dean up the Plaza area and promote their businesseswere the city to act in accordance with their preference Yet lacking any supporting evidencethe author cannot convince me of this Thus the survey amounts to scant evidence at best thatthe proposed ban would carry the intended resultFinally the author recommends a course of action that might actually defeat the citysobjective of providing a fun and relaxing place for people to congregate In my experienceskateboarding contributes to an atmosphere of fun and relaxation for adults and children alikemore so than many other types of ambiance Without considering that continuing to allowskateboarding--or even encouraging this activity--might achieve the citys goal more effectivelythan banning the activity the author cannot convincingly conclude that the ban would be in thecitys best interestsIn sum the argument is a specious one To strengthen it the editorials author must providedear evidence that skateboarding and not some other factor is responsible for the conditionsmarking the Plazas decline The author must also convince me that no alternative means ofrestoring the Plaza are available to the city and that the proposed ban by itself would suffice toattract tourists and restore the Plaza to its former glory Finally to better assess the argument itwould be useful to know the circumstances under which the downtown merchants would bewilling to help the city achieve its objectives

6그룹 약한 비유 빈출

The following recommendation appeared in a memo from the mayor of the town of Hopewell

Two years ago the nearby town of Ocean View built a new municipal golf course and resort hotel During the past two years tourism in Ocean View has increased new businesses have opened there and Ocean Views tax revenues have risen by 30 percent Therefore the best way to improve Hopewells economymdashand generate additional tax revenuesmdashis to build a golf course and resort hotel similar to those in Ocean View

Write a response in which you examine the stated andor unstated assumptions of the argument Be sure to explain how the argument depends on these assumptions and what the implications are for the argument if the assumptions prove unwarranted

GRE AWA John박 박정어학원

Assumptions The author assumes that OVrsquos municipal golf course and resort hotel caused tourism new businesses and increased tax revenues There may be other reasons advertising promo He assumes that this will continueAssumes that Ocean View and Hopewell are similar in many waysmdashthe name suggests otherwise OV may have always been a tourist attractions for its beaches We need to know the topography

2년전 Ocean View 시는 시정 소유 골프 및 휴양지 호텔을 신축했다 그리고 지난 2년동안 이 시의 관광객이 증가했으며 새로운 사업들이 생겨났다 그에따라 시의 세수도 30나 증가했다 Hopewell의 경제를 향상시키고 아울러 세수를 늘릴 수 있는 가장 좋은 방법은 Ocean View에 세워진 것과 같은 골프 시설과 휴양지 호텔을 신축하는 것이다

1 다른 요인으로 관광 산업이 발전했을 수도 있다 문화 유적이 발견이 되었거나 도로의 정비등으로 여행자가 늘었을 수도 있다

2 관광 산업의증가가 늘어난 세수의 원인이 아니라 새로 유입된 인구의 증가나 다른 공장에서 발생한 것일 수 있다

3 2년동안 한참 골프가 붐을 이루었을 수 있다 경제상황이 나빠지거나 다른 레포츠가 인근 지역에 생겨난다면 골프하는 사람이 줄어들 수 있다

In this memo HopeweUs mayor recommends that in order to stimulate the towns economyand boost tax revenues HopeweU should build a new golf course and resort hotel just as thetown of Ocean View did two years ago To support this recommendation the mayor points outthat in Ocean View during the last two years tourism has increased new businesses haveopened and tax revenues have increased by 30 I find the mayors argument unconvincingin several important respectsFirst of all it is possible that the mayor has confused cause with effect respecting the recentdevelopments in Ocean View Perhaps Ocean Views construction of a new golf course andhotel was a response to previous increases in tourism and business development increasesthat have simply continued during the most recent two years Since the mayor has failed toaccount for this possibility the claim that Hopewell would boost its economy by alsoconstructing a golf course and hotel is completely unwarrantedSecondly the mayor fails to account for other possible causes of the trends in Ocean Viewduring the last two years The increase in tourism might have been due to improving economicconditions nationwide or to unusually pleasant weather in the region The new businessesthat have opened in Ocean View might have opened there irrespective of the new golf courseand hotel And the 30 increase in tax revenues might have been the result of an increase intax rates or the addition of a new type of municipal taxWithout ruling out these and other alternative explanations for the three recent trends inOcean View the mayor cannot reasonably infer based on those trends that Hopewellseconomy would benefit by following Ocean Views exampleThirdly even if the recent trends in Ocean View are attributable to the construction of the newgolf course and hotel there the mayor assumes too hastily that the golf course and hotel willcontinue to benefit that towns overall economy The mayor has not accounted for thepossibility that increased tourism will begin to drive residents away during tourist season orthat new business development will result in the towns losing its appeal as a place to visit or tolive Unless the mayor can convince me that these scenarios are unlikely I cannot accept themayors recommendation that Hopewell follow Ocean Views exampleFinally the mayors argument rests on the unsubstantiated assumption that Hopewell andOcean View are sufficiently alike in ways that might affect the economic impact of a new golfcourse and hotel Hopewell might lack the sort of natural environment that would attract moretourists and new businesses to the town--regardless of its new golf course and hotel For thatmatter perhaps Hopewell already contains several resort hotels and golf courses that are notutilized to their capacity If so building yet another golf course and hotel might amount to amisallocation of the towns resources--and actually harm the towns overall economyIn sum the mayors recommendation is not well supported To bolster it the mayor mustprovide better evidence that Ocean Views new golf course and hotel and not some otherphenomenon--has been responsible for boosting Ocean Views economy during the last twoyears To better assess the recommendation I would need to know why Ocean View decidedto construct its new golf course and hotel in the first place--specifically what events prior toconstruction might have prompted that decision I would also need to thoroughly compare

GRE AWA John박 박정어학원

HopeweU with Ocean View--especially in terms of their appeal to tourists and businesses--todetermine whether the same course of action that appears to have boosted Ocean Viewseconomy would also boost Hopewells economy

The following is part of a memorandum from the president of Humana University

Last year the number of students who enrolled in online degree programs offered by nearby Omni University increased by 50 percent During the same year Omni showed a significant decrease from prior years in expenditures for dormitory and classroom space most likely because instruction in the online programs takes place via the Internet In contrast over the past three years enrollment at Humana University has failed to grow and the cost of maintaining buildings has increased along with our budget deficit To address these problems Humana University will begin immediately to create and actively promote online degree programs like those at Omni We predict that instituting these online degree programs will help Humana both increase its total enrollment and solve its budget problems

Write a response in which you discuss what questions would need to be answered in order to decide whether the prediction and the argument on which it is based are reasonable Be sure to explain how the answers to these questions would help to evaluate the prediction

Is Omni University successful due to the online degree program 50 Is the decrease in expenditures for dormitory and classroom space due to the decrease in of on-campus students Which classes were successful Does HU have those classes

Even if the long-distance degree programs at Omni University benefited the school the presidentrsquos recommendation that Human College should emulate Omni University is too hasty First OUrsquos name implies that the school would have more majors than Humanahellip the president should examine which degrees were in the long-distance programhellip

지난해에는 Omni 대학에서 개강했던 원거리 학생 학점 취득 프로그램을 등록했던 학생들의 숫자가 50나 증가했다 같은해 기간동안 Omni 대학에서는 그 전년도부터 기숙사와 학급의 공간 확충을 위한 예산을 대폭 줄였는데 이는 이 원거리 학점 취득 프로그램이 양방향 비디오 컴퓨터 접속을 통해서만 가능한 수업지도 방식이기때문인 것으로 보인다 반면 지난 3개년 동안 Humana 대학에서의 수강률은 감소한데다가 건물

GRE AWA John박 박정어학원

유지비도 올랐다 따라서 Humana대학의 수강을 늘리고 예산손실을 회복하기 위해서는 Omni 대학에서 취한 조치와 같은 능동적인 프로그램을 추진해야 한다

결론 we should initiate and actively promote long-distance degree programs like those at Omni 반박 원거리 학생 취득 프로그램 숫자가 증가한거하고 예산이 줄어드는 것 사이에 연관이 약하다 (causal 학생의 증가로 관리비용 증가할수 있음 원거리 수업가능 장비도입에의한 비용발생)bad analogy(omni university 하고 같은 조건이 아니다 )-gt omni college 가 강좌내용이 좋아서 학생의 등록이 많을수 있다 Humana 대학에서 만들었다 하더라도 인기 없을수 있음다른 요인에 의해서 Humana 대학의 수강 인원이 증가할수 있음(비록 과거엔 인기가 없었을지라도)

The following appeared as part of a business plan developed by the manager of the Rialto Movie Theater

Despite its downtown location the Rialto Movie Theater a local institution for five decades must make big changes or close its doors forever It should follow the example of the new Apex Theater in the mall outside of town When the Apex opened last year it featured a video arcade plush carpeting and seats and a state-of-the-art sound system Furthermore in a recent survey over 85 percent of respondents reported that the high price of newly released movies prevents them from going to the movies more than five times per year Thus if the Rialto intends to hold on to its share of a decreasing pool of moviegoers it must offer the same features as Apex

Write a response in which you discuss what questions would need to be answered in order to decide whether the recommendation is likely to have the predicted result Be sure to explain how the answers to these questions would help to evaluate the recommendation

Before following through this business plan the manager should investigate the cause of Rialtorsquos unsuccessful business

The author provides no evidence that the surveyrsquos results are statistically reliable The surveyrsquos sample of 85 percent must be sufficient in size and representative of overall population of the city where Rialto and Apex is serving Lacking evidence of a sufficiently representative sample the author cannot justifiably rely on the survey to draw any conclusion whatsoever The author does not indicate that Apex is indeed currently successful However even if Apex is enjoying success the argument relies on what might be a false analogy between Rialto and Apex In order for Apex to serve as a model that Rialto should emulate the author must assume that all relevant circumstances are essentially the same However this assumption is unwarranted For example the argument overlooks the face that Apex is located in a strategic placemdashbeside a mall where customers can not only watch a movie but also enjoy shopping Therefore simply changing the facility to that of Apex may not lead to success

The author does not mention whether Apex is successful or not Nevertheless even if Apex is currently successful the argument relies on what might be a false analogy between Rialto and Apex In order for Apex to serve as a model that Rialto should emulate the author must assume that all relevant circumstances are essentially the same However this assumption is unwarranted For example the argument overlooks the fact that these two institutions are located in different locations Rialto in downtown and Apex in a mall outside of town Although Apex opened with state-of-the-art facilities the decisive factor in its success could be due to its strategic location of being in a mall People could enjoy both shopping and movies at one location thus they may prefer Apex over Rialto Furthermore the place where people enjoy leisure activities has shifted in the past decades for most cities from downtown to the suburbs Therefore Rialto may not be successful even if it emulates Apexrsquos facilities A better business plan may be relocating Apex to the thriving section of the downtown

Rialto 극장은 지난 50여년간 지역 회관으로써 시내에 위치해 있으면서도 이제 변화를 꾀하지 않으면 문을 닫을

GRE AWA John박 박정어학원

판이다 이 극장은 시외 쇼핑타운에 새로 들어선 Apex 극장의 사례를 본받아야 했다 Apex가 지난해 개업했을 당시 이 극장은 비디오 아케이드 플러쉬 카펫트 바닥과 좌석 그리고 최신 음향시설을 갖추었다 더군다나 최근 조사에서는 응답자의 85 이상이 새로 출시된 영화 입장료가 비싼 탓으로 지난해보다 5배이상의 관람객이 줄어들었다고 나타났다 따라서 Rialto 극장이 줄어들고 있는 관람객을 뺐기지 않고 유지하려면 Apex와 같은 시설들을 갖추어야 할 것이다주장 리알토 극장이 줄어들고 있는 관람객을 뺐기지 않고 유지하려면 Apex와 같은 시설들을 갖추어야 할 것이다

1 조사에서 응답자가 전체를 대표할 수 없다 2 apex 극장이 좋은 시설을 갖추고 있지만 그로 인해 수익이 많이 발생했다는 말이 없으므로 시설투자를

하고도 좋은 결과를 얻을 수 있을지 그 근거가 미흡하다3 좋은 영화가 출시된다면 입장료가 비싸도 영화관에서 꼭 보려고 할 수 있다 4 rialto 가 시설이 아닌 다른 요인에 의해 장사가 안될수도 있다( 우범 지역이라든지)

The following is a recommendation from the business manager of Monarch Books

Since its opening in Collegeville twenty years ago Monarch Books has developed a large customer base due to its reader-friendly atmosphere and wide selection of books on all subjects Last month Book and Bean a combination bookstore and coffee shop announced its intention to open a Collegeville store Monarch Books should open its own in-store cafeacute in the space currently devoted to childrens books Given recent national census data indicating a significant decline in the percentage of the population under age ten sales of childrens books are likely to decline By replacing its childrens books section with a cafeacute Monarch Books can increase profits and ward off competition from Book and Bean

Write a response in which you examine the stated andor unstated assumptions of the argument Be sure to explain how the argument depends on these assumptions and what the implications are for the argument if the assumptions prove unwarranted

The following is a recommendation from the business manager of Monarch Books

Since its opening in Collegeville twenty years ago Monarch Books has developed a large customer base due to its reader-friendly atmosphere and wide selection of books on all subjects Last month Book and Bean a combination bookstore and coffee shop announced its intention to open a Collegeville store Monarch Books should open its own in-store cafeacute in the space currently devoted to childrens books Given recent national census data indicating a significant decline in the percentage of the population under age ten sales of childrens books are likely to decline By replacing its childrens books section with a cafeacute Monarch Books can increase profits and ward off competition from Book and Bean

1 Write a response in which you discuss what questions would need to be answered in order to decide whether the recommendation is likely to have the predicted result Be sure to explain how the answers to these questions would help to evaluate the recommendation

2 Write a response in which you discuss what specific evidence is needed to evaluate the argument and explain how the evidence would weaken or strengthen the argument

No evidence regarding Monarch Bookrsquos successEven if Regal Bookrsquos is successful this may not be attributable to the cafeacute False analogy Emulating may not lead to success Other factors may be involvedInsufficient condition The national census is not enough evidence that childrenrsquos book sales will decline Can

GRE AWA John박 박정어학원

the national census represent the local child populationDid opening a cafeacute boost sales for Regal Books Even assuming Regal is successful by opening a cafeacute this may not be suitable for Monarch which plans to close the childrenrsquos book section to establish a cafe Imprecise language ldquorelatively little spacerdquo how smallThe managerrsquos recommendation contradicts what he says Since Monarch is popular for its wide selection of books closing a selection which targets a major group of readers may hurt Monarchrsquos salesIs this the best way to compete

When Stanley Park first opened it was the largest most heavily used public park in town It is still the largest park but it is no longer heavily used Video cameras mounted in the parks parking lots last month revealed the parks drop in popularity the recordings showed an average of only 50 cars per day In contrast tiny Carlton Park in the heart of the business district is visited by more than 150 people on a typical weekday An obvious difference is that Carlton Park unlike Stanley Park provides ample seating Thus if Stanley Park is ever to be as popular with our citizens as Carlton Park the town will obviously need to provide more benches thereby converting some of the unused open areas into spaces suitable for socializing

Write a response in which you examine the stated andor unstated assumptions of the argument Be sure to explain how the argument depends on these assumptions and what the implications are for the argument if the assumptions prove unwarranted

Stanley 파크가 처음 개장했을 당시 가장 크고 가장 많이 이용되는 공원이었다 아직도 공원중에서는 가장 크지만 이용률은 상당히 떨어졌다 지난달 공원 주차장에 설치해놓은 비디오 카메라를 통해 보면 drop(주차장으로 여겨짐) 이용률이 가장 높았다 수치상으로는 하루 평균 50대의 차량만이 이용하였다 반면 직장 중심거리에 위치한 작은 규모의 Carlton 파크는 주당 무려 150여명 이상이 이용하고 있다 Stanley 파크와는 달리 Carlton 파크에는 의자가 있다는 것이 가장 뚜렷한 차이점이다 따라서 Stanley 파크가 Carlton 파크처럼 시민들이 자주 이용하는 공원이 되기 위해서는 벤치를 설치할 필요가 있으며 이렇게 사용되지 않는 일부 공간을 활용해서 사교를 위한 공간으로 바꾸어야 한다 ===gtdrop 에 대한 첨부사항 (영영사전내용입니다)---- a place or central depository to which something (as mail money or stolen property) is brought for distribution or transmission also the act of depositing something at such a place dropgt

주장 if Stanley Park is ever to be as popular with our citizens as is Carlton Park the town will obviously need to provide more benches thereby converting some of the unused open areas into spaces suitable for socializing1 조사가 언제 이루어진 것인가 조사가 언제 실시되었느냐에 따라 결과가 다를 수있다 현재는 다시 스탠리 파크가 늘어났었을 수 있다 2 벤치를 많이 설치했다고 해서 많은 관광객이 오지 않을수 있다(사람들이 벤치나 사교 공간을 원한다는 어떠한 자료도 없다)3스탠리 파크 주변에 교통 상황이 악화가 되었거나 칼튼 파크에서 문화행사등을 많이 가져서 이용객이 줄어든것일 수도 있다 4 칼튼 파크가 중심지에 있어서 접근성이 좋을수 있다5 조사가 같은 시간을 기준으로 한게 아니다(하나는 주중이고 하나는 주말이다)6사람의 수와 차의 대수를 같은것으로 비교할수 없다 (차안에 몇 명이 타고 있는지 모르고 대중교통을 이용해서 왔을수도 있다)

Page 27: GRE writing argument brain storm

GRE AWA John박 박정어학원

data Example 1 (Rebuttal1) First if we do not know the total volume of items sold and the price of the goods exactly we cannot infer whether the residents are actually buying many goods Example 2 (Rebuttal2)In addition to that the report emphasizes the rising sales of running shoes and exerciseclothing however these may not be hot-selling items for Naturersquos Way or may not be theproducts the company is planning to sell Concluding Sentence Therefore in order to make the argument reliable the author should reconsider themerchantsrsquo report with more detailed data

Body 2-CausalTopic Sentence 2Second the author contends that the health clubs classes are full yet this does not meanthat many people actually use the health club other factors may be the real cause forthose closed classes Example 1 (Rebuttal1) To begin with if the health club is very small the number of people working out wouldnot be a large one In fact regular gym-going may just be a vogue among a smallunrepresentative segment of Plainsvilles population Example 2 (Rebuttal2) Moreover it is possible that most of the people who exercise in the health club do weight training and aerobics only to look good and to meet other singles not for their health In that case there would be little demand for health products Concluding SentenceThus the author should not hasten to presume what really caused people to be interested in a healthier lifestyle and enroll in the health club

Body 3-Time-ShiftTopic Sentence 3Finally the author highlights that Naturersquos Way can expect a new generation of customersin Plainsville that will help the company in the long term This notion is mistaken in that itassumes the conditions of the present will continue unchanged in the future Although theschool children are required to participate in the fitness for life program they may notnecessarily buy Naturersquos Ways products Example 1 (Rebuttal1) In the first instance they may suffer a fall in purchasing power arising from future economic difficulties this would cause reluctance to spend a considerable amount of money on health products which tend to be more expensive Example 2 (Rebuttal2)Another possibility is that there may emerge many competitor companies vying with Naturersquos Way so that in the future the school children may not feel the necessity to purchase one companyrsquos health products over anotherrsquosConcluding Sentence Thus the authorrsquos assumption is highly speculative since it relies heavily on unknowablefuture circumstances

ConclusionThesis In sum the author uses many assumptions that are insufficient in supporting his claimsSupportIn order for the authorrsquos claims to be convincing he needs to advance more persuasiveevidence that people in Plainsville really are concerned with their health and health foodThe following was written as a part of an application for a small-business loan by a group of developers in the city of Monroe

A jazz music club in Monroe would be a tremendously profitable enterprise Currently the nearest jazz club is 65 miles away thus the proposed new jazz club in Monroe the C-Note would have the local market all to itself Plus jazz is extremely popular in Monroe over 100000 people attended Monroes annual jazz festival last summer several well-known jazz musicians live in Monroe and the highest-rated radio program in Monroe is Jazz Nightly which airs every weeknight at 7 PM Finally a nationwide study indicates that the typical jazz fan spends close to $1000 per year on jazz entertainment

1 Write a response in which you discuss what specific evidence is needed to evaluate the argument and explain how the evidence would weaken or strengthen the argument

2 Write a response in which you examine the stated andor unstated assumptions of the argument Be

GRE AWA John박 박정어학원

sure to explain how the argument depends on these assumptions and what the implications are for the argument if the assumptions prove unwarranted

3 Write a response in which you discuss what questions would need to be answered in order to decide whether the prediction and the argument on which it is based are reasonable Be sure to explain how the answers to these questions would help to evaluate the prediction

Group error nationwide survey may not reflect local trends Is the nationwide jazz fan population substantialInsufficient non-residents of Monroe may have attended the jazz festival (Body alternative explanation last year may have been an anomaly The author should consider data from various years) The author should indicate how many out of 100000 were Monroe residentsNationwide study Does this reflect Insufficient Citizens of Monroe may continue to go to the jazz club 65 miles away

Are the people in Monroe really interested in jazzMajority of the people who attended the jazz festival might not be Monroe residentsSurvey error nationwide study may not be applicable to MonroeJazz musicians who live in MonroeMonopolyRadio station

In this business application the author claims that the proposed jazz club C Note will be very profitable in Monroe To support this claim the author argues for his case with several evidences At first glance the authorrsquos argument seems convincing however careful scrutiny reveals that his argument in specious

To begin with the author claims that Monroersquos citizens are interested in jazz He presents three evidences First Secondhellip Thirdhellip Howeverhellip

Monroe 시에 있는 재즈 음악 클럽은 수익성이 좋은 사업이다 현재 가장 가까이에 있는 클럽은 65 마일 정도 떨어져 있다 따라서 이번에 세우려고 하는 C Note 는 독보적인 위치를 점할것이다 더군다나 재즈는 이 시에서 가장 인기있는 음악이다 지난 여름 재즈 축제에서는 10 만명 이상의 Morone 시 주민이 참석하였고 몇몇 유명한 재즈 음악가들도 이곳에 살고 있으며 저녁때 방영되는 라디오 프로그램중에서 최고의 시청률을 보이고 있는 것도 Jazz Nightly 이다 전국조사에서도 전형적인 재즈 팬들은 재즈 분야에 년간 1천 달러 가까이 지출하고 있는 것으로 보고되고 있다 따라서 C Note 클럽이 돈을 벌 수 있는 사업이라는 것은 확실한 것이다

결과 It is clear that the C Note cannot help but make money반박 nearest jazz club 이 양질의 써비스로 여전히 손님을 끌수도 있다Festival 에 얼마나 참여하는지가 jazz 의 인기를 반영하지 않는다 뮤지션이 많이 사는거랑 jazz 의 인기가 상관없다라디오 프로그램이 다른 요인에 의해서 인기일수도 있다 (진행자때문)전국 통계 적용 불가화목 실전반_Ms Noh6In this application the author suggests that a jazz club in Monroe will make a number of profits To support this suggestion the author exemplifies the local condition popularity of jazz in Monroe and nationwide study However careful scrutiny of each of the facts reveals that it provides little credible support for the authorrsquos recommendation Good clear intro

First the author assumes that jazz is popular in Monroe because of several facts the jazz festival last year had high participation some famous jazz musicians live in Monroe and the high-rated radio program is lsquoJazz Nightlyrsquo However this assumption has many drawbacks that must be seriously considered(Good topic sentences) If many attendants in the last-yearrsquos festival came from other cities and not Monroe it is hard to conclude that Monroersquos people like jazz Therefore the author must examine how many Monroe residents actually attended the festival On top of that there is little relationship between habitation of famous jazz musician and the popularity of jazz in Monroe Although several well-known musicians live there if they do not take part in any jazz performance of Monroe this might have no effect to the interest of Monroersquos residents

GRE AWA John박 박정어학원

about jazz Finally in the case of radio program this is also not suitable reason why jazz is popular in Monroe It might be possible that people cannot help choosing lsquoJazz Nightlyrsquo because there are few radio programs at Night The fact that the radio program is the highest rating program is not a germane evidence The approximate number of listeners would be the more crucial evidence Therefore the author needs to seriously deliberate the correlation between jazzrsquos popularity in Monroe and his examples (Good logical flow and clarity)

Second the author uses as evidence the nationwide study that jazz fans spend much money on jazz entertainment to substantiate why starting a jazz club in Monroe will be profitable In other words the author assumes that the characteristics of a nationwide study can be applied to Monroe The national study would lend support to the applicantrsquos claim only if residents in Monroe typify national jazz fans However the author does not provide credible evidence that this is the case Moreover the populations of jazz fans nationwide may be insubstantial Thus the author should not infer hastily that Monroersquos residents will spend much money on enjoying jazz from the nationwide study

Lastly even if jazz is popular in Monroe C Note may not be successful It is entirely possible that residents might still prefer other clubs where they have always went In addition there is another possibility that the nearest jazz club will attract many of Monroersquos people because it serves fine performances and is equipped with favorite facilities Without considering these other possibilities the author cannot make his argument convincing In sum the author presents many reasons that are insufficient in supporting his or her claim In order for the authorrsquos claims to be convincing he needs to advance more persuasive evidence such as the total number of Monroe residents who attended the jazz festival the effects on the popularity of jazz by the musicians living in Monroe and the actual number of residents who would typify themselves to be jazz fans through a local survey Without substantial evidence that C Note will be successful in Monroe the businessmen may be overinvesting in what might lead to a business failureExcellent clarity Score 50

The following appeared in a newsletter offering advice to investors

Over 80 percent of the respondents to a recent survey indicated a desire to reduce their intake of foods containing fats and cholesterol and today low-fat products abound in many food stores Since many of the food products currently marketed by Old Dairy Industries are high in fat and cholesterol the companys sales are likely to diminish greatly and company profits will no doubt decrease We therefore advise Old Dairy stockholders to sell their shares and other investors not to purchase stock in this company

Write a response in which you discuss what questions would need to be answered in order to decide whether the advice and the argument on which it is based are reasonable Be sure to explain how the answers to these questions would help to evaluate the advice

Survey 80

GRE AWA John박 박정어학원

Old Dairy could change their products and manufacture low fat dairy foodsLess competing companies Old Dairy could eventually be the only company that produces hellipImprecise numbers and measurementsCustomers may still buy high fat dairy products

The author of the newsletter is offering potentially dangerous advice by recommending Old Dairy stockholders to withdraw investment and stop purchase What is more the authorrsquos prediction debases the reputation and business of Old Dairy and if false could devoid the investment opportunity of the newsletter readers Therefore investors should examine whether the authorrsquos evidences are substantial

To begin with the author states that 80 percent of the respondents in a survey indicated a desire to reduce their intake of foods He therefore argues that Old Dairyrsquos high fat and cholesterol products would decrease in sales However the author makes a crucial error in this argument First the author provides no evidence that the surveyrsquos results are statistically reliable Were they representative of all the customers Were they chosen for the survey randomly Furthermore the desire to reduce fat and cholesterol intake is a pervasive trend in todayrsquos opulent society however the author erroneously identifies this as a new phenomenon which will affect consumer trends Second having a desire to reduce fat and cholesterol intake does not necessarily indicate that people who have this desire will actually reduce consuming these types of products It is entirely possible that they may continue buying Old Dairy products for its quality and taste Accordingly the author cannot draw any firm conclusion that people will not buy Old Dairy products Therefore if any of these cases are true the author may be offering investors a detrimental investment advice

최근 조사에 대한 응답자중 80 이상이 자신이 먹는 음식에서 지방과 콜레스테롤의 함유량을 줄이고 싶다고 한다 아울러 요즘은 많은 식료품 가계에서 저지방 제품들을 많이 취급하고 있다 현재 Old Dairy Industries가 판매하고 있는 많은 음식제품들은 지방과 콜레스테롤이 높기 때문에 이 회사의 매출이 격감할 것으로 보이며 당연히 매출이익도 줄어들것이다 따라서 이 회사의 주주들은 주식을 매각하고 다른 주식 투자가들도 이 회사의 주식을 매입하지 않는 것이 좋다

결론 Old Dairy stockholders to sell their shares and other investors not to purchase stock in this company

반박 모든 상품이 다 고 지방 고 칼로리는 아니다(비록 많을지라도) 일부의 식품의 경우 기호에 맞어서 히트해서 전체적인 수입이 증가할 수도 있다국내시장만 생각할 수 없다( 외국시장에서 호황을 누릴수 있다 )입맛이라는게 즉각 바뀌는게 아니다

The following appeared in a letter to the editor of the Balmer Island Gazette

On Balmer Island where mopeds serve as a popular form of transportation the population increases to 100000 during the summer months To reduce the number of accidents involving mopeds and pedestrians the town council of Balmer Island should limit the number of mopeds rented by the islands moped rental companies from 50 per day to 25 per day during the summer season By limiting the number of rentals the town council will attain the 50 percent annual reduction in moped accidents that was achieved last year on the neighboring island of Seaville when Seavilles town council enforced similar limits on moped rentals

1 Write a response in which you discuss what questions would need to be answered in order to decide whether the recommendation is likely to have the predicted result Be sure to explain how the answers to these questions would help to evaluate the recommendation

2 Write a response in which you discuss what questions would need to be answered in order to decide whether the prediction and the argument on which it is based are reasonable Be sure to explain how the answers to these questions would help to evaluate the prediction

3 Write a response in which you examine the stated andor unstated assumptions of the argument Be sure to explain how the argument depends on these assumptions and what the implications are for the argument if the assumptions prove unwarranted

Whatrsquos the actual population of Balmer Island 100000mdashis this a significant increase What kind of accidents Skin abrasions or serious injury And compared to Seaville how serious are the accidents and the actual number of accidents Did Seaville enforce other restrictions like safety signsHow different are the conditions of Balmer

GRE AWA John박 박정어학원

and Seaville regarding population road (safety) conditions topography other town-government regulation How much will the economy of Balmer be affected do to this restriction Could it cause an economic recession due to the fact that these rental companiesrsquo chance to make money is only during the summer thereby weakening the economic infrastructure Are there any other ways that could better alleviate the accident rate

Statistics 50-impreciseAnalogy Balmer compared with TorseauFalse Cause Accidents might have occurred because of reasons other than mopeds False Cause population increase may not be part of the cause of the accidentsOther explanations for the accident pedestrians few road safety regulations narrow roadsThere could be other better solutionshellip

Balmer Island의 인구가 여름철에는 십만명으로 늘어난다 2륜차와 보행자간 사고를 줄이기 위해 시의회는 6개의 자전거를 포함한 2륜차 대여업체에게 이 기간동안에는 대여숫자를 일일 50에서 30으로 제한하도록 할 것이다 대여숫자를 줄임으로써 시 의회는 지난해 이웃한 Torseau섬에서 이와 동일한 규제를 시행해서 50나 줄인 결과를 보고 마찬가지로 50를 줄일수 있다고 확신하고 있다

결론 The town council of Balmer Island should linit the number

반박 보행자의 부실에 의해서 사고가 많이 일어날수도 있다렌탈수의 줄임만이 대책은 아니다(대부분의 사람들이 렌탈 보다는 소유하고 있을 수도 있다)옆섬과는 상황이 다를수도 있다(그 섬에서는 사고의 원인이 많은 자전거 수로 인한것일수있다) 하지만 이 섬은 좁은 도로가 원인일 수도 있고 도로 안전 장치의미비가 원일일수 있다

In this letter the author recommends that Balmer Island should limit the number moped rentals from 50 to 30 per day To support this recommendation the author points out several reasons However careful scrutiny of each of the facts reveals that it is filled with unanswered questions that could significantly weaken the authorrsquos recommendation with loops and holes which are answered

The recommendation depends on the assumption that no alternative means of reducing the number of accidents are available However the author fails to offer any evidence to substantiate this crucial assumption It is highly possible that means other than this would better solve the problem Perhaps they could widen the roads or put-up more safety signs Or perhaps the accidents were due to the lack of skills in which case proper safety training would significantly alleviate the problem Without considering and ruling out these and other alternative means of reducing accidetns the author cannot confidently conclude that merely emulating Torseau would suffice Moreover the author is advising a recommendation which could potentially harm the economy of Balmer Island sincehellip Moreover the Balmer Island should alternative means to reduce accidents because limiting moped rentals during the summer could harm the economy of Balmerhellip

First of all the author believes that increase in population and the number of moped rentals are responsible for the accidents It is entirely possible that other factors are responsible for the accidents Perhaps Balmer Islandrsquos lack of safety signs was a major factor Or maybe the roads are narrow and dangerous on the Island therefore the town council could enforce stricter traffic regulations to alleviate the problem Accordingly if either of these scenarios is true the author cannot draw any firm conclusion that increase in the number of population and moped rentals are the cause of the accidents

The author of this editorial recommends that to reduce accidents involving mopeds andpedestrians Balmer Islands city council should restrict moped rentals to 30 per day down from50 at each of the islands six rental outlets To support this recommendation the author citesthe fact that last year when nearby Torseau Islands town council enforced similar measuresTorseaus rate of moped accidents fell by 50 For several reasons this evidence providesscant support for the authors recommendationTo begin with the author assumes that all other conditions in Balmer that might affect therate of moped-pedestrian accidents will remain unchanged after the restrictions are enactedHowever with a restricted supply of rental mopeds people in Balmer might purchase mopedsinstead Also the number of pedestrians might increase in the future with more pedestriansespecially tourists the risk of moped-pedestrian accidents would probably increase For thatmatter the number of rental outlets might increase to make up for the artificial supplyrestriction per outlet--a likely scenario assuming moped rental demand does not declineWithout considering and ruling out these and other possible changes that might contribute to ahigh incidence of moped-pedestrian accidents the author cannot convince me that theproposed restrictions will necessarily have the desired effect

GRE AWA John박 박정어학원

Next the author fails to consider other possible explanations for the 50 decline inTorseaus moped accident rate last year Perhaps last year Torseau experienced unusually fairweather during which moped accidents are less likely Perhaps fewer tourists visited Tot seanlast year than during most years thereby diminishing the demand for rental mopeds to belowthe allowed limits Perhaps last year some of Torseaus moped rental outlets purchased newmopeds that are safer to drive Or perhaps the restrictions were already in effect but were notenforced until last year In any event a decline in Torseaus moped accident rate during onlyone year is scarcely sufficient to draw any reliable conclusions about what might have causedthe decline or about what the accident rate will be in years aheadAdditionally in asserting that the same phenomenon that caused a 50 decline in mopedaccidents in Torseau would cause a similar decline in Balmer the author relies on what mightamount to an unfair analogy between Balmer and Torseau Perhaps Balmers ability to enforcemoped-rental restrictions does not meet Torseaus ability if not then the mere enactment ofsimilar restrictions in Balmer is no guarantee of a similar result Or perhaps the demand formopeds in Torseau is always greater than in Balmer Specifically if fewer than all availablemopeds are currently rented per day from the average Balmer outlet while in Torseau everyavailable moped is rented each day then the proposed restriction is likely to have less impacton the accident rate in Balmer than in TorseauFinally the author provides no evidence that the same restrictions that served to reduce theincidence of all moped accidents by 50 would also serve to reduce the incidence ofaccidents involving mopeds and pedestrians by 50 Lacking such evidence it is entirelypossible that the number of moped accidents not involving pedestrians decreased by a greaterpercentage while the number of moped-pedestrian accidents decreased by a smallerpercentage or even increased Since the author has not accounted for these possibilities theeditorials recommendation cannot be taken seriouslyIn conclusion the recommendation is not well supported To convince me that the proposedrestriction would achieve the desired outcome the author would have to assure me that nochanges serving to increase Balmers moped-pedestrian accident rate will occur in theforeseeable future The author must also provide dear evidence that last years decline inmoped accidents in Torseau was attributable primarily to its moped rental restrictions ratherthan to one or more other factors In order to better evaluate the recommendation I wouldneed more information comparing the supply of and demand for moped rentals on the twoislands I would also need to know the rate of mopedpedestrian accidents in Torseau both priorto and after the restrictions were enforced in TorseauThe following appeared in a magazine article about planning for retirement

Clearview should be a top choice for anyone seeking a place to retire because it has spectacular natural beauty and a consistent climate Another advantage is that housing costs in Clearview have fallen significantly during the past year and taxes remain lower than those in neighboring towns Moreover Clearviews mayor promises many new programs to improve schools streets and public services And best of all retirees in Clearview can also expect excellent health care as they grow older since the number of physicians in the area is far greater than the national average

Write a response in which you discuss what specific evidence is needed to evaluate the argument and explain how the evidence would weaken or strengthen the argument

-Natural beauty and consistent climate may not be the most wanted qualities-Housing costs could have lowered on a national level wealthy retirees may not care about costs-Taxes may be high compared to the nationrsquos average tax rate-What about other qualities of Clearview Crime rate what qualities would retirees want -If schools streets and public services need improvement then this is proof that the current condition of Clearview is low Or due to budgetary reasons the mayor may not follow-up on his promise because of lowered tax rate -Schools and people who are retired no relationship-Physicians What kind of physicians Number is irrelevant Are these physicians capable of addressing the illnesses of old people

This author argues that anyone seeking a place to retire should choose Clearview To supportthis argument the article cites Clearviews consistent climate and natural beauty its fallinghousing costs its low property taxes compared to nearby towns and the mayors promise toimprove schools streets and services The article also claims that retirees can expectexcellent health care because the number of physicians in Clearview greatly exceeds thenational average This argument is flawed in several critical respectsTo begin with although consistent climate and natural beauty might be attractive to manyretirees these features are probably not important to all retirees For many retirees it isprobably more important to live near relatives or even to enjoy changing seasons Thus I

GRE AWA John박 박정어학원

cannot accept the authors sweeping recommendation for all retirees on this basisAlso Clearviews declining housing costs do not necessarily make Clearview the best placeto retire for two reasons First despite the decline Clearviews housing costs might be highcompared to housing costs in other cities Secondly for wealthier retirees housing costs arenot likely to be a factor in choosing a place to retire Thus the mere fact that housing costshave been in decline lends scant support to the recommendationThe articles reliance on Clearviews property-tax rates is also problematic in two respectsFirst retirees obviously have innumerable choices about where to retire besides Clear viewand nearby towns Secondly for retirees who are well-off financially property taxes are notlikely to be an important concern in choosing a place to retire Thus it is unfair to infer fromClearviews property-tax rates that retirees would prefer ClearviewYet another problem with the argument involves the mayors promises In light of Clearviewslow property-tax rates whether the mayor can follow through on those promises is highlyquestionable Absent any explanation of how the city can spend more money in the areas citedwithout raising property taxes I simply cannot accept the editorials recommendation on thebasis of those promises Besides even if the city makes the improvements promised thoseimprovements--particular the ones to schools--would not necessarily be important to retireesFinally although the number of physicians in Clearview is relatively high the per capitanumber might be relatively low Moreover it would be fairer to compare this per capita numberwith the per capita number for other attractive retirement towns--rather than the nationalaverage After all retirees are likely to place a relatively heavy burden on health-careresources Besides the article provides no assurances that the number of physicians inClearview will remain high in the foreseeable futureIn conclusion the recommendation is poorly supported To strengthen it the author mustconvince me--perhaps by way of a reliable survey--that the key features that the vast majorityof retirees look for in choosing a place to live are consistent climate natural beauty and lowhousing costs The author must also provide better evidence that Clear views property taxesare lower than the those of cities in other areas The author must also explain how the city canmake its promised improvements without raising property taxes Finally to better assess theargument I would need to now how the per capita number of physicians in Clearview wouldcompare to the national average in the futureThe following appeared as a letter to the editor from a Central Plaza store owner

Over the past two years the number of shoppers in Central Plaza has been steadily decreasing while the popularity of skateboarding has increased dramatically Many Central Plaza store owners believe that the decrease in their business is due to the number of skateboard users in the plaza There has also been a dramatic increase in the amount of litter and vandalism throughout the plaza Thus we recommend that the city prohibit skateboarding in Central Plaza If skateboarding is prohibited here we predict that business in Central Plaza will return to its previously high levels

Write a response in which you discuss what questions would need to be answered in order to decide whether the recommendation is likely to have the predicted result Be sure to explain how the answers to these questions would help to evaluate the recommendation

Why two years ago What happened two years ago which started this declineIs the dramatic increase in the ldquopopularityrdquo of skateboarding the cause of the steady decline of shoppers Are there any malls nearby Were there any changes nearby which could affect the decline in customersmdasha big mall perhaps Could the decline be due to the shop ownersHow many skateboarders use the plazaWhere do they skateboardDo they shop and are they customersAre the increase in litter and vandalism due to skateboarders Could this be alleviated by installing CCTVs and hiring security

This editorial concludes that the city should ban skateboarding from its downtown CentralPlaza in order to attract visitors to that area to return the area to its former glory and to makeit a place where people can congregate for fun and relaxation To justify this conclusion theeditorial points out that skateboarders are nearly the only people one sees anymore at CentralPlaza and that the Plaza is littered and its property defaced The editorial also points out thatthe majority of downtown merchants support the skate boarding ban This argument is flawedin several critical respectsFirst the editorials author falsely assumes that a ban on skateboarding is both necessaryand sufficient to achieve the three stated objectives Perhaps the city can achieve thoseobjectives by other means as well--for example by creating a new mall that incorporates anattractive new skateboard park Even if banning skateboarders altogether is necessary to meetthe citys goals the author has not shown that this action by itself would suffice Assuming thatthe Plazas reputation is now tarnished restoring that reputation and in turn enticing peopleback to the Plaza might require additional measures--such as removing litter and graffiti

GRE AWA John박 박정어학원

promoting the Plaza to the public or enticing popular restaurant or retail chains to the PlazaSecondly the editorial assumes too hastily that the Plazas decline is attributable to theskateboarders--rather than to some other phenomenon Perhaps the Plazas primary appeal inits glory days had to do with particular shops or eateries which were eventually replaced byless appealing ones Or perhaps the crime rate in surrounding areas has risen dramatically forreasons unrelated to the skateboarders presence at the Plaza Without ruling out these andother alternative explanations for the Plazas decline the editorials author cannot convince methat a skateboard ban would reverse that declineThirdly the editorials author might be confusing cause with effect--by assuming that theskateboarders caused the abandonment of the Plaza rather than vice versa It is entirelypossible that skateboarders did not frequent the Plaza until it was largely abandoned--andbecause it had been abandoned In fact this scenario makes good sense since skateboardingis most enjoyable where there are few pedestrians or motorists to get in the wayFourth it is unreasonable to infer from the mere fact that most merchants favor the ban thatthe ban would be effective in achieving the citys objectives Admittedly perhaps thesemerchants would be more likely to help dean up the Plaza area and promote their businesseswere the city to act in accordance with their preference Yet lacking any supporting evidencethe author cannot convince me of this Thus the survey amounts to scant evidence at best thatthe proposed ban would carry the intended resultFinally the author recommends a course of action that might actually defeat the citysobjective of providing a fun and relaxing place for people to congregate In my experienceskateboarding contributes to an atmosphere of fun and relaxation for adults and children alikemore so than many other types of ambiance Without considering that continuing to allowskateboarding--or even encouraging this activity--might achieve the citys goal more effectivelythan banning the activity the author cannot convincingly conclude that the ban would be in thecitys best interestsIn sum the argument is a specious one To strengthen it the editorials author must providedear evidence that skateboarding and not some other factor is responsible for the conditionsmarking the Plazas decline The author must also convince me that no alternative means ofrestoring the Plaza are available to the city and that the proposed ban by itself would suffice toattract tourists and restore the Plaza to its former glory Finally to better assess the argument itwould be useful to know the circumstances under which the downtown merchants would bewilling to help the city achieve its objectives

6그룹 약한 비유 빈출

The following recommendation appeared in a memo from the mayor of the town of Hopewell

Two years ago the nearby town of Ocean View built a new municipal golf course and resort hotel During the past two years tourism in Ocean View has increased new businesses have opened there and Ocean Views tax revenues have risen by 30 percent Therefore the best way to improve Hopewells economymdashand generate additional tax revenuesmdashis to build a golf course and resort hotel similar to those in Ocean View

Write a response in which you examine the stated andor unstated assumptions of the argument Be sure to explain how the argument depends on these assumptions and what the implications are for the argument if the assumptions prove unwarranted

GRE AWA John박 박정어학원

Assumptions The author assumes that OVrsquos municipal golf course and resort hotel caused tourism new businesses and increased tax revenues There may be other reasons advertising promo He assumes that this will continueAssumes that Ocean View and Hopewell are similar in many waysmdashthe name suggests otherwise OV may have always been a tourist attractions for its beaches We need to know the topography

2년전 Ocean View 시는 시정 소유 골프 및 휴양지 호텔을 신축했다 그리고 지난 2년동안 이 시의 관광객이 증가했으며 새로운 사업들이 생겨났다 그에따라 시의 세수도 30나 증가했다 Hopewell의 경제를 향상시키고 아울러 세수를 늘릴 수 있는 가장 좋은 방법은 Ocean View에 세워진 것과 같은 골프 시설과 휴양지 호텔을 신축하는 것이다

1 다른 요인으로 관광 산업이 발전했을 수도 있다 문화 유적이 발견이 되었거나 도로의 정비등으로 여행자가 늘었을 수도 있다

2 관광 산업의증가가 늘어난 세수의 원인이 아니라 새로 유입된 인구의 증가나 다른 공장에서 발생한 것일 수 있다

3 2년동안 한참 골프가 붐을 이루었을 수 있다 경제상황이 나빠지거나 다른 레포츠가 인근 지역에 생겨난다면 골프하는 사람이 줄어들 수 있다

In this memo HopeweUs mayor recommends that in order to stimulate the towns economyand boost tax revenues HopeweU should build a new golf course and resort hotel just as thetown of Ocean View did two years ago To support this recommendation the mayor points outthat in Ocean View during the last two years tourism has increased new businesses haveopened and tax revenues have increased by 30 I find the mayors argument unconvincingin several important respectsFirst of all it is possible that the mayor has confused cause with effect respecting the recentdevelopments in Ocean View Perhaps Ocean Views construction of a new golf course andhotel was a response to previous increases in tourism and business development increasesthat have simply continued during the most recent two years Since the mayor has failed toaccount for this possibility the claim that Hopewell would boost its economy by alsoconstructing a golf course and hotel is completely unwarrantedSecondly the mayor fails to account for other possible causes of the trends in Ocean Viewduring the last two years The increase in tourism might have been due to improving economicconditions nationwide or to unusually pleasant weather in the region The new businessesthat have opened in Ocean View might have opened there irrespective of the new golf courseand hotel And the 30 increase in tax revenues might have been the result of an increase intax rates or the addition of a new type of municipal taxWithout ruling out these and other alternative explanations for the three recent trends inOcean View the mayor cannot reasonably infer based on those trends that Hopewellseconomy would benefit by following Ocean Views exampleThirdly even if the recent trends in Ocean View are attributable to the construction of the newgolf course and hotel there the mayor assumes too hastily that the golf course and hotel willcontinue to benefit that towns overall economy The mayor has not accounted for thepossibility that increased tourism will begin to drive residents away during tourist season orthat new business development will result in the towns losing its appeal as a place to visit or tolive Unless the mayor can convince me that these scenarios are unlikely I cannot accept themayors recommendation that Hopewell follow Ocean Views exampleFinally the mayors argument rests on the unsubstantiated assumption that Hopewell andOcean View are sufficiently alike in ways that might affect the economic impact of a new golfcourse and hotel Hopewell might lack the sort of natural environment that would attract moretourists and new businesses to the town--regardless of its new golf course and hotel For thatmatter perhaps Hopewell already contains several resort hotels and golf courses that are notutilized to their capacity If so building yet another golf course and hotel might amount to amisallocation of the towns resources--and actually harm the towns overall economyIn sum the mayors recommendation is not well supported To bolster it the mayor mustprovide better evidence that Ocean Views new golf course and hotel and not some otherphenomenon--has been responsible for boosting Ocean Views economy during the last twoyears To better assess the recommendation I would need to know why Ocean View decidedto construct its new golf course and hotel in the first place--specifically what events prior toconstruction might have prompted that decision I would also need to thoroughly compare

GRE AWA John박 박정어학원

HopeweU with Ocean View--especially in terms of their appeal to tourists and businesses--todetermine whether the same course of action that appears to have boosted Ocean Viewseconomy would also boost Hopewells economy

The following is part of a memorandum from the president of Humana University

Last year the number of students who enrolled in online degree programs offered by nearby Omni University increased by 50 percent During the same year Omni showed a significant decrease from prior years in expenditures for dormitory and classroom space most likely because instruction in the online programs takes place via the Internet In contrast over the past three years enrollment at Humana University has failed to grow and the cost of maintaining buildings has increased along with our budget deficit To address these problems Humana University will begin immediately to create and actively promote online degree programs like those at Omni We predict that instituting these online degree programs will help Humana both increase its total enrollment and solve its budget problems

Write a response in which you discuss what questions would need to be answered in order to decide whether the prediction and the argument on which it is based are reasonable Be sure to explain how the answers to these questions would help to evaluate the prediction

Is Omni University successful due to the online degree program 50 Is the decrease in expenditures for dormitory and classroom space due to the decrease in of on-campus students Which classes were successful Does HU have those classes

Even if the long-distance degree programs at Omni University benefited the school the presidentrsquos recommendation that Human College should emulate Omni University is too hasty First OUrsquos name implies that the school would have more majors than Humanahellip the president should examine which degrees were in the long-distance programhellip

지난해에는 Omni 대학에서 개강했던 원거리 학생 학점 취득 프로그램을 등록했던 학생들의 숫자가 50나 증가했다 같은해 기간동안 Omni 대학에서는 그 전년도부터 기숙사와 학급의 공간 확충을 위한 예산을 대폭 줄였는데 이는 이 원거리 학점 취득 프로그램이 양방향 비디오 컴퓨터 접속을 통해서만 가능한 수업지도 방식이기때문인 것으로 보인다 반면 지난 3개년 동안 Humana 대학에서의 수강률은 감소한데다가 건물

GRE AWA John박 박정어학원

유지비도 올랐다 따라서 Humana대학의 수강을 늘리고 예산손실을 회복하기 위해서는 Omni 대학에서 취한 조치와 같은 능동적인 프로그램을 추진해야 한다

결론 we should initiate and actively promote long-distance degree programs like those at Omni 반박 원거리 학생 취득 프로그램 숫자가 증가한거하고 예산이 줄어드는 것 사이에 연관이 약하다 (causal 학생의 증가로 관리비용 증가할수 있음 원거리 수업가능 장비도입에의한 비용발생)bad analogy(omni university 하고 같은 조건이 아니다 )-gt omni college 가 강좌내용이 좋아서 학생의 등록이 많을수 있다 Humana 대학에서 만들었다 하더라도 인기 없을수 있음다른 요인에 의해서 Humana 대학의 수강 인원이 증가할수 있음(비록 과거엔 인기가 없었을지라도)

The following appeared as part of a business plan developed by the manager of the Rialto Movie Theater

Despite its downtown location the Rialto Movie Theater a local institution for five decades must make big changes or close its doors forever It should follow the example of the new Apex Theater in the mall outside of town When the Apex opened last year it featured a video arcade plush carpeting and seats and a state-of-the-art sound system Furthermore in a recent survey over 85 percent of respondents reported that the high price of newly released movies prevents them from going to the movies more than five times per year Thus if the Rialto intends to hold on to its share of a decreasing pool of moviegoers it must offer the same features as Apex

Write a response in which you discuss what questions would need to be answered in order to decide whether the recommendation is likely to have the predicted result Be sure to explain how the answers to these questions would help to evaluate the recommendation

Before following through this business plan the manager should investigate the cause of Rialtorsquos unsuccessful business

The author provides no evidence that the surveyrsquos results are statistically reliable The surveyrsquos sample of 85 percent must be sufficient in size and representative of overall population of the city where Rialto and Apex is serving Lacking evidence of a sufficiently representative sample the author cannot justifiably rely on the survey to draw any conclusion whatsoever The author does not indicate that Apex is indeed currently successful However even if Apex is enjoying success the argument relies on what might be a false analogy between Rialto and Apex In order for Apex to serve as a model that Rialto should emulate the author must assume that all relevant circumstances are essentially the same However this assumption is unwarranted For example the argument overlooks the face that Apex is located in a strategic placemdashbeside a mall where customers can not only watch a movie but also enjoy shopping Therefore simply changing the facility to that of Apex may not lead to success

The author does not mention whether Apex is successful or not Nevertheless even if Apex is currently successful the argument relies on what might be a false analogy between Rialto and Apex In order for Apex to serve as a model that Rialto should emulate the author must assume that all relevant circumstances are essentially the same However this assumption is unwarranted For example the argument overlooks the fact that these two institutions are located in different locations Rialto in downtown and Apex in a mall outside of town Although Apex opened with state-of-the-art facilities the decisive factor in its success could be due to its strategic location of being in a mall People could enjoy both shopping and movies at one location thus they may prefer Apex over Rialto Furthermore the place where people enjoy leisure activities has shifted in the past decades for most cities from downtown to the suburbs Therefore Rialto may not be successful even if it emulates Apexrsquos facilities A better business plan may be relocating Apex to the thriving section of the downtown

Rialto 극장은 지난 50여년간 지역 회관으로써 시내에 위치해 있으면서도 이제 변화를 꾀하지 않으면 문을 닫을

GRE AWA John박 박정어학원

판이다 이 극장은 시외 쇼핑타운에 새로 들어선 Apex 극장의 사례를 본받아야 했다 Apex가 지난해 개업했을 당시 이 극장은 비디오 아케이드 플러쉬 카펫트 바닥과 좌석 그리고 최신 음향시설을 갖추었다 더군다나 최근 조사에서는 응답자의 85 이상이 새로 출시된 영화 입장료가 비싼 탓으로 지난해보다 5배이상의 관람객이 줄어들었다고 나타났다 따라서 Rialto 극장이 줄어들고 있는 관람객을 뺐기지 않고 유지하려면 Apex와 같은 시설들을 갖추어야 할 것이다주장 리알토 극장이 줄어들고 있는 관람객을 뺐기지 않고 유지하려면 Apex와 같은 시설들을 갖추어야 할 것이다

1 조사에서 응답자가 전체를 대표할 수 없다 2 apex 극장이 좋은 시설을 갖추고 있지만 그로 인해 수익이 많이 발생했다는 말이 없으므로 시설투자를

하고도 좋은 결과를 얻을 수 있을지 그 근거가 미흡하다3 좋은 영화가 출시된다면 입장료가 비싸도 영화관에서 꼭 보려고 할 수 있다 4 rialto 가 시설이 아닌 다른 요인에 의해 장사가 안될수도 있다( 우범 지역이라든지)

The following is a recommendation from the business manager of Monarch Books

Since its opening in Collegeville twenty years ago Monarch Books has developed a large customer base due to its reader-friendly atmosphere and wide selection of books on all subjects Last month Book and Bean a combination bookstore and coffee shop announced its intention to open a Collegeville store Monarch Books should open its own in-store cafeacute in the space currently devoted to childrens books Given recent national census data indicating a significant decline in the percentage of the population under age ten sales of childrens books are likely to decline By replacing its childrens books section with a cafeacute Monarch Books can increase profits and ward off competition from Book and Bean

Write a response in which you examine the stated andor unstated assumptions of the argument Be sure to explain how the argument depends on these assumptions and what the implications are for the argument if the assumptions prove unwarranted

The following is a recommendation from the business manager of Monarch Books

Since its opening in Collegeville twenty years ago Monarch Books has developed a large customer base due to its reader-friendly atmosphere and wide selection of books on all subjects Last month Book and Bean a combination bookstore and coffee shop announced its intention to open a Collegeville store Monarch Books should open its own in-store cafeacute in the space currently devoted to childrens books Given recent national census data indicating a significant decline in the percentage of the population under age ten sales of childrens books are likely to decline By replacing its childrens books section with a cafeacute Monarch Books can increase profits and ward off competition from Book and Bean

1 Write a response in which you discuss what questions would need to be answered in order to decide whether the recommendation is likely to have the predicted result Be sure to explain how the answers to these questions would help to evaluate the recommendation

2 Write a response in which you discuss what specific evidence is needed to evaluate the argument and explain how the evidence would weaken or strengthen the argument

No evidence regarding Monarch Bookrsquos successEven if Regal Bookrsquos is successful this may not be attributable to the cafeacute False analogy Emulating may not lead to success Other factors may be involvedInsufficient condition The national census is not enough evidence that childrenrsquos book sales will decline Can

GRE AWA John박 박정어학원

the national census represent the local child populationDid opening a cafeacute boost sales for Regal Books Even assuming Regal is successful by opening a cafeacute this may not be suitable for Monarch which plans to close the childrenrsquos book section to establish a cafe Imprecise language ldquorelatively little spacerdquo how smallThe managerrsquos recommendation contradicts what he says Since Monarch is popular for its wide selection of books closing a selection which targets a major group of readers may hurt Monarchrsquos salesIs this the best way to compete

When Stanley Park first opened it was the largest most heavily used public park in town It is still the largest park but it is no longer heavily used Video cameras mounted in the parks parking lots last month revealed the parks drop in popularity the recordings showed an average of only 50 cars per day In contrast tiny Carlton Park in the heart of the business district is visited by more than 150 people on a typical weekday An obvious difference is that Carlton Park unlike Stanley Park provides ample seating Thus if Stanley Park is ever to be as popular with our citizens as Carlton Park the town will obviously need to provide more benches thereby converting some of the unused open areas into spaces suitable for socializing

Write a response in which you examine the stated andor unstated assumptions of the argument Be sure to explain how the argument depends on these assumptions and what the implications are for the argument if the assumptions prove unwarranted

Stanley 파크가 처음 개장했을 당시 가장 크고 가장 많이 이용되는 공원이었다 아직도 공원중에서는 가장 크지만 이용률은 상당히 떨어졌다 지난달 공원 주차장에 설치해놓은 비디오 카메라를 통해 보면 drop(주차장으로 여겨짐) 이용률이 가장 높았다 수치상으로는 하루 평균 50대의 차량만이 이용하였다 반면 직장 중심거리에 위치한 작은 규모의 Carlton 파크는 주당 무려 150여명 이상이 이용하고 있다 Stanley 파크와는 달리 Carlton 파크에는 의자가 있다는 것이 가장 뚜렷한 차이점이다 따라서 Stanley 파크가 Carlton 파크처럼 시민들이 자주 이용하는 공원이 되기 위해서는 벤치를 설치할 필요가 있으며 이렇게 사용되지 않는 일부 공간을 활용해서 사교를 위한 공간으로 바꾸어야 한다 ===gtdrop 에 대한 첨부사항 (영영사전내용입니다)---- a place or central depository to which something (as mail money or stolen property) is brought for distribution or transmission also the act of depositing something at such a place dropgt

주장 if Stanley Park is ever to be as popular with our citizens as is Carlton Park the town will obviously need to provide more benches thereby converting some of the unused open areas into spaces suitable for socializing1 조사가 언제 이루어진 것인가 조사가 언제 실시되었느냐에 따라 결과가 다를 수있다 현재는 다시 스탠리 파크가 늘어났었을 수 있다 2 벤치를 많이 설치했다고 해서 많은 관광객이 오지 않을수 있다(사람들이 벤치나 사교 공간을 원한다는 어떠한 자료도 없다)3스탠리 파크 주변에 교통 상황이 악화가 되었거나 칼튼 파크에서 문화행사등을 많이 가져서 이용객이 줄어든것일 수도 있다 4 칼튼 파크가 중심지에 있어서 접근성이 좋을수 있다5 조사가 같은 시간을 기준으로 한게 아니다(하나는 주중이고 하나는 주말이다)6사람의 수와 차의 대수를 같은것으로 비교할수 없다 (차안에 몇 명이 타고 있는지 모르고 대중교통을 이용해서 왔을수도 있다)

Page 28: GRE writing argument brain storm

GRE AWA John박 박정어학원

sure to explain how the argument depends on these assumptions and what the implications are for the argument if the assumptions prove unwarranted

3 Write a response in which you discuss what questions would need to be answered in order to decide whether the prediction and the argument on which it is based are reasonable Be sure to explain how the answers to these questions would help to evaluate the prediction

Group error nationwide survey may not reflect local trends Is the nationwide jazz fan population substantialInsufficient non-residents of Monroe may have attended the jazz festival (Body alternative explanation last year may have been an anomaly The author should consider data from various years) The author should indicate how many out of 100000 were Monroe residentsNationwide study Does this reflect Insufficient Citizens of Monroe may continue to go to the jazz club 65 miles away

Are the people in Monroe really interested in jazzMajority of the people who attended the jazz festival might not be Monroe residentsSurvey error nationwide study may not be applicable to MonroeJazz musicians who live in MonroeMonopolyRadio station

In this business application the author claims that the proposed jazz club C Note will be very profitable in Monroe To support this claim the author argues for his case with several evidences At first glance the authorrsquos argument seems convincing however careful scrutiny reveals that his argument in specious

To begin with the author claims that Monroersquos citizens are interested in jazz He presents three evidences First Secondhellip Thirdhellip Howeverhellip

Monroe 시에 있는 재즈 음악 클럽은 수익성이 좋은 사업이다 현재 가장 가까이에 있는 클럽은 65 마일 정도 떨어져 있다 따라서 이번에 세우려고 하는 C Note 는 독보적인 위치를 점할것이다 더군다나 재즈는 이 시에서 가장 인기있는 음악이다 지난 여름 재즈 축제에서는 10 만명 이상의 Morone 시 주민이 참석하였고 몇몇 유명한 재즈 음악가들도 이곳에 살고 있으며 저녁때 방영되는 라디오 프로그램중에서 최고의 시청률을 보이고 있는 것도 Jazz Nightly 이다 전국조사에서도 전형적인 재즈 팬들은 재즈 분야에 년간 1천 달러 가까이 지출하고 있는 것으로 보고되고 있다 따라서 C Note 클럽이 돈을 벌 수 있는 사업이라는 것은 확실한 것이다

결과 It is clear that the C Note cannot help but make money반박 nearest jazz club 이 양질의 써비스로 여전히 손님을 끌수도 있다Festival 에 얼마나 참여하는지가 jazz 의 인기를 반영하지 않는다 뮤지션이 많이 사는거랑 jazz 의 인기가 상관없다라디오 프로그램이 다른 요인에 의해서 인기일수도 있다 (진행자때문)전국 통계 적용 불가화목 실전반_Ms Noh6In this application the author suggests that a jazz club in Monroe will make a number of profits To support this suggestion the author exemplifies the local condition popularity of jazz in Monroe and nationwide study However careful scrutiny of each of the facts reveals that it provides little credible support for the authorrsquos recommendation Good clear intro

First the author assumes that jazz is popular in Monroe because of several facts the jazz festival last year had high participation some famous jazz musicians live in Monroe and the high-rated radio program is lsquoJazz Nightlyrsquo However this assumption has many drawbacks that must be seriously considered(Good topic sentences) If many attendants in the last-yearrsquos festival came from other cities and not Monroe it is hard to conclude that Monroersquos people like jazz Therefore the author must examine how many Monroe residents actually attended the festival On top of that there is little relationship between habitation of famous jazz musician and the popularity of jazz in Monroe Although several well-known musicians live there if they do not take part in any jazz performance of Monroe this might have no effect to the interest of Monroersquos residents

GRE AWA John박 박정어학원

about jazz Finally in the case of radio program this is also not suitable reason why jazz is popular in Monroe It might be possible that people cannot help choosing lsquoJazz Nightlyrsquo because there are few radio programs at Night The fact that the radio program is the highest rating program is not a germane evidence The approximate number of listeners would be the more crucial evidence Therefore the author needs to seriously deliberate the correlation between jazzrsquos popularity in Monroe and his examples (Good logical flow and clarity)

Second the author uses as evidence the nationwide study that jazz fans spend much money on jazz entertainment to substantiate why starting a jazz club in Monroe will be profitable In other words the author assumes that the characteristics of a nationwide study can be applied to Monroe The national study would lend support to the applicantrsquos claim only if residents in Monroe typify national jazz fans However the author does not provide credible evidence that this is the case Moreover the populations of jazz fans nationwide may be insubstantial Thus the author should not infer hastily that Monroersquos residents will spend much money on enjoying jazz from the nationwide study

Lastly even if jazz is popular in Monroe C Note may not be successful It is entirely possible that residents might still prefer other clubs where they have always went In addition there is another possibility that the nearest jazz club will attract many of Monroersquos people because it serves fine performances and is equipped with favorite facilities Without considering these other possibilities the author cannot make his argument convincing In sum the author presents many reasons that are insufficient in supporting his or her claim In order for the authorrsquos claims to be convincing he needs to advance more persuasive evidence such as the total number of Monroe residents who attended the jazz festival the effects on the popularity of jazz by the musicians living in Monroe and the actual number of residents who would typify themselves to be jazz fans through a local survey Without substantial evidence that C Note will be successful in Monroe the businessmen may be overinvesting in what might lead to a business failureExcellent clarity Score 50

The following appeared in a newsletter offering advice to investors

Over 80 percent of the respondents to a recent survey indicated a desire to reduce their intake of foods containing fats and cholesterol and today low-fat products abound in many food stores Since many of the food products currently marketed by Old Dairy Industries are high in fat and cholesterol the companys sales are likely to diminish greatly and company profits will no doubt decrease We therefore advise Old Dairy stockholders to sell their shares and other investors not to purchase stock in this company

Write a response in which you discuss what questions would need to be answered in order to decide whether the advice and the argument on which it is based are reasonable Be sure to explain how the answers to these questions would help to evaluate the advice

Survey 80

GRE AWA John박 박정어학원

Old Dairy could change their products and manufacture low fat dairy foodsLess competing companies Old Dairy could eventually be the only company that produces hellipImprecise numbers and measurementsCustomers may still buy high fat dairy products

The author of the newsletter is offering potentially dangerous advice by recommending Old Dairy stockholders to withdraw investment and stop purchase What is more the authorrsquos prediction debases the reputation and business of Old Dairy and if false could devoid the investment opportunity of the newsletter readers Therefore investors should examine whether the authorrsquos evidences are substantial

To begin with the author states that 80 percent of the respondents in a survey indicated a desire to reduce their intake of foods He therefore argues that Old Dairyrsquos high fat and cholesterol products would decrease in sales However the author makes a crucial error in this argument First the author provides no evidence that the surveyrsquos results are statistically reliable Were they representative of all the customers Were they chosen for the survey randomly Furthermore the desire to reduce fat and cholesterol intake is a pervasive trend in todayrsquos opulent society however the author erroneously identifies this as a new phenomenon which will affect consumer trends Second having a desire to reduce fat and cholesterol intake does not necessarily indicate that people who have this desire will actually reduce consuming these types of products It is entirely possible that they may continue buying Old Dairy products for its quality and taste Accordingly the author cannot draw any firm conclusion that people will not buy Old Dairy products Therefore if any of these cases are true the author may be offering investors a detrimental investment advice

최근 조사에 대한 응답자중 80 이상이 자신이 먹는 음식에서 지방과 콜레스테롤의 함유량을 줄이고 싶다고 한다 아울러 요즘은 많은 식료품 가계에서 저지방 제품들을 많이 취급하고 있다 현재 Old Dairy Industries가 판매하고 있는 많은 음식제품들은 지방과 콜레스테롤이 높기 때문에 이 회사의 매출이 격감할 것으로 보이며 당연히 매출이익도 줄어들것이다 따라서 이 회사의 주주들은 주식을 매각하고 다른 주식 투자가들도 이 회사의 주식을 매입하지 않는 것이 좋다

결론 Old Dairy stockholders to sell their shares and other investors not to purchase stock in this company

반박 모든 상품이 다 고 지방 고 칼로리는 아니다(비록 많을지라도) 일부의 식품의 경우 기호에 맞어서 히트해서 전체적인 수입이 증가할 수도 있다국내시장만 생각할 수 없다( 외국시장에서 호황을 누릴수 있다 )입맛이라는게 즉각 바뀌는게 아니다

The following appeared in a letter to the editor of the Balmer Island Gazette

On Balmer Island where mopeds serve as a popular form of transportation the population increases to 100000 during the summer months To reduce the number of accidents involving mopeds and pedestrians the town council of Balmer Island should limit the number of mopeds rented by the islands moped rental companies from 50 per day to 25 per day during the summer season By limiting the number of rentals the town council will attain the 50 percent annual reduction in moped accidents that was achieved last year on the neighboring island of Seaville when Seavilles town council enforced similar limits on moped rentals

1 Write a response in which you discuss what questions would need to be answered in order to decide whether the recommendation is likely to have the predicted result Be sure to explain how the answers to these questions would help to evaluate the recommendation

2 Write a response in which you discuss what questions would need to be answered in order to decide whether the prediction and the argument on which it is based are reasonable Be sure to explain how the answers to these questions would help to evaluate the prediction

3 Write a response in which you examine the stated andor unstated assumptions of the argument Be sure to explain how the argument depends on these assumptions and what the implications are for the argument if the assumptions prove unwarranted

Whatrsquos the actual population of Balmer Island 100000mdashis this a significant increase What kind of accidents Skin abrasions or serious injury And compared to Seaville how serious are the accidents and the actual number of accidents Did Seaville enforce other restrictions like safety signsHow different are the conditions of Balmer

GRE AWA John박 박정어학원

and Seaville regarding population road (safety) conditions topography other town-government regulation How much will the economy of Balmer be affected do to this restriction Could it cause an economic recession due to the fact that these rental companiesrsquo chance to make money is only during the summer thereby weakening the economic infrastructure Are there any other ways that could better alleviate the accident rate

Statistics 50-impreciseAnalogy Balmer compared with TorseauFalse Cause Accidents might have occurred because of reasons other than mopeds False Cause population increase may not be part of the cause of the accidentsOther explanations for the accident pedestrians few road safety regulations narrow roadsThere could be other better solutionshellip

Balmer Island의 인구가 여름철에는 십만명으로 늘어난다 2륜차와 보행자간 사고를 줄이기 위해 시의회는 6개의 자전거를 포함한 2륜차 대여업체에게 이 기간동안에는 대여숫자를 일일 50에서 30으로 제한하도록 할 것이다 대여숫자를 줄임으로써 시 의회는 지난해 이웃한 Torseau섬에서 이와 동일한 규제를 시행해서 50나 줄인 결과를 보고 마찬가지로 50를 줄일수 있다고 확신하고 있다

결론 The town council of Balmer Island should linit the number

반박 보행자의 부실에 의해서 사고가 많이 일어날수도 있다렌탈수의 줄임만이 대책은 아니다(대부분의 사람들이 렌탈 보다는 소유하고 있을 수도 있다)옆섬과는 상황이 다를수도 있다(그 섬에서는 사고의 원인이 많은 자전거 수로 인한것일수있다) 하지만 이 섬은 좁은 도로가 원인일 수도 있고 도로 안전 장치의미비가 원일일수 있다

In this letter the author recommends that Balmer Island should limit the number moped rentals from 50 to 30 per day To support this recommendation the author points out several reasons However careful scrutiny of each of the facts reveals that it is filled with unanswered questions that could significantly weaken the authorrsquos recommendation with loops and holes which are answered

The recommendation depends on the assumption that no alternative means of reducing the number of accidents are available However the author fails to offer any evidence to substantiate this crucial assumption It is highly possible that means other than this would better solve the problem Perhaps they could widen the roads or put-up more safety signs Or perhaps the accidents were due to the lack of skills in which case proper safety training would significantly alleviate the problem Without considering and ruling out these and other alternative means of reducing accidetns the author cannot confidently conclude that merely emulating Torseau would suffice Moreover the author is advising a recommendation which could potentially harm the economy of Balmer Island sincehellip Moreover the Balmer Island should alternative means to reduce accidents because limiting moped rentals during the summer could harm the economy of Balmerhellip

First of all the author believes that increase in population and the number of moped rentals are responsible for the accidents It is entirely possible that other factors are responsible for the accidents Perhaps Balmer Islandrsquos lack of safety signs was a major factor Or maybe the roads are narrow and dangerous on the Island therefore the town council could enforce stricter traffic regulations to alleviate the problem Accordingly if either of these scenarios is true the author cannot draw any firm conclusion that increase in the number of population and moped rentals are the cause of the accidents

The author of this editorial recommends that to reduce accidents involving mopeds andpedestrians Balmer Islands city council should restrict moped rentals to 30 per day down from50 at each of the islands six rental outlets To support this recommendation the author citesthe fact that last year when nearby Torseau Islands town council enforced similar measuresTorseaus rate of moped accidents fell by 50 For several reasons this evidence providesscant support for the authors recommendationTo begin with the author assumes that all other conditions in Balmer that might affect therate of moped-pedestrian accidents will remain unchanged after the restrictions are enactedHowever with a restricted supply of rental mopeds people in Balmer might purchase mopedsinstead Also the number of pedestrians might increase in the future with more pedestriansespecially tourists the risk of moped-pedestrian accidents would probably increase For thatmatter the number of rental outlets might increase to make up for the artificial supplyrestriction per outlet--a likely scenario assuming moped rental demand does not declineWithout considering and ruling out these and other possible changes that might contribute to ahigh incidence of moped-pedestrian accidents the author cannot convince me that theproposed restrictions will necessarily have the desired effect

GRE AWA John박 박정어학원

Next the author fails to consider other possible explanations for the 50 decline inTorseaus moped accident rate last year Perhaps last year Torseau experienced unusually fairweather during which moped accidents are less likely Perhaps fewer tourists visited Tot seanlast year than during most years thereby diminishing the demand for rental mopeds to belowthe allowed limits Perhaps last year some of Torseaus moped rental outlets purchased newmopeds that are safer to drive Or perhaps the restrictions were already in effect but were notenforced until last year In any event a decline in Torseaus moped accident rate during onlyone year is scarcely sufficient to draw any reliable conclusions about what might have causedthe decline or about what the accident rate will be in years aheadAdditionally in asserting that the same phenomenon that caused a 50 decline in mopedaccidents in Torseau would cause a similar decline in Balmer the author relies on what mightamount to an unfair analogy between Balmer and Torseau Perhaps Balmers ability to enforcemoped-rental restrictions does not meet Torseaus ability if not then the mere enactment ofsimilar restrictions in Balmer is no guarantee of a similar result Or perhaps the demand formopeds in Torseau is always greater than in Balmer Specifically if fewer than all availablemopeds are currently rented per day from the average Balmer outlet while in Torseau everyavailable moped is rented each day then the proposed restriction is likely to have less impacton the accident rate in Balmer than in TorseauFinally the author provides no evidence that the same restrictions that served to reduce theincidence of all moped accidents by 50 would also serve to reduce the incidence ofaccidents involving mopeds and pedestrians by 50 Lacking such evidence it is entirelypossible that the number of moped accidents not involving pedestrians decreased by a greaterpercentage while the number of moped-pedestrian accidents decreased by a smallerpercentage or even increased Since the author has not accounted for these possibilities theeditorials recommendation cannot be taken seriouslyIn conclusion the recommendation is not well supported To convince me that the proposedrestriction would achieve the desired outcome the author would have to assure me that nochanges serving to increase Balmers moped-pedestrian accident rate will occur in theforeseeable future The author must also provide dear evidence that last years decline inmoped accidents in Torseau was attributable primarily to its moped rental restrictions ratherthan to one or more other factors In order to better evaluate the recommendation I wouldneed more information comparing the supply of and demand for moped rentals on the twoislands I would also need to know the rate of mopedpedestrian accidents in Torseau both priorto and after the restrictions were enforced in TorseauThe following appeared in a magazine article about planning for retirement

Clearview should be a top choice for anyone seeking a place to retire because it has spectacular natural beauty and a consistent climate Another advantage is that housing costs in Clearview have fallen significantly during the past year and taxes remain lower than those in neighboring towns Moreover Clearviews mayor promises many new programs to improve schools streets and public services And best of all retirees in Clearview can also expect excellent health care as they grow older since the number of physicians in the area is far greater than the national average

Write a response in which you discuss what specific evidence is needed to evaluate the argument and explain how the evidence would weaken or strengthen the argument

-Natural beauty and consistent climate may not be the most wanted qualities-Housing costs could have lowered on a national level wealthy retirees may not care about costs-Taxes may be high compared to the nationrsquos average tax rate-What about other qualities of Clearview Crime rate what qualities would retirees want -If schools streets and public services need improvement then this is proof that the current condition of Clearview is low Or due to budgetary reasons the mayor may not follow-up on his promise because of lowered tax rate -Schools and people who are retired no relationship-Physicians What kind of physicians Number is irrelevant Are these physicians capable of addressing the illnesses of old people

This author argues that anyone seeking a place to retire should choose Clearview To supportthis argument the article cites Clearviews consistent climate and natural beauty its fallinghousing costs its low property taxes compared to nearby towns and the mayors promise toimprove schools streets and services The article also claims that retirees can expectexcellent health care because the number of physicians in Clearview greatly exceeds thenational average This argument is flawed in several critical respectsTo begin with although consistent climate and natural beauty might be attractive to manyretirees these features are probably not important to all retirees For many retirees it isprobably more important to live near relatives or even to enjoy changing seasons Thus I

GRE AWA John박 박정어학원

cannot accept the authors sweeping recommendation for all retirees on this basisAlso Clearviews declining housing costs do not necessarily make Clearview the best placeto retire for two reasons First despite the decline Clearviews housing costs might be highcompared to housing costs in other cities Secondly for wealthier retirees housing costs arenot likely to be a factor in choosing a place to retire Thus the mere fact that housing costshave been in decline lends scant support to the recommendationThe articles reliance on Clearviews property-tax rates is also problematic in two respectsFirst retirees obviously have innumerable choices about where to retire besides Clear viewand nearby towns Secondly for retirees who are well-off financially property taxes are notlikely to be an important concern in choosing a place to retire Thus it is unfair to infer fromClearviews property-tax rates that retirees would prefer ClearviewYet another problem with the argument involves the mayors promises In light of Clearviewslow property-tax rates whether the mayor can follow through on those promises is highlyquestionable Absent any explanation of how the city can spend more money in the areas citedwithout raising property taxes I simply cannot accept the editorials recommendation on thebasis of those promises Besides even if the city makes the improvements promised thoseimprovements--particular the ones to schools--would not necessarily be important to retireesFinally although the number of physicians in Clearview is relatively high the per capitanumber might be relatively low Moreover it would be fairer to compare this per capita numberwith the per capita number for other attractive retirement towns--rather than the nationalaverage After all retirees are likely to place a relatively heavy burden on health-careresources Besides the article provides no assurances that the number of physicians inClearview will remain high in the foreseeable futureIn conclusion the recommendation is poorly supported To strengthen it the author mustconvince me--perhaps by way of a reliable survey--that the key features that the vast majorityof retirees look for in choosing a place to live are consistent climate natural beauty and lowhousing costs The author must also provide better evidence that Clear views property taxesare lower than the those of cities in other areas The author must also explain how the city canmake its promised improvements without raising property taxes Finally to better assess theargument I would need to now how the per capita number of physicians in Clearview wouldcompare to the national average in the futureThe following appeared as a letter to the editor from a Central Plaza store owner

Over the past two years the number of shoppers in Central Plaza has been steadily decreasing while the popularity of skateboarding has increased dramatically Many Central Plaza store owners believe that the decrease in their business is due to the number of skateboard users in the plaza There has also been a dramatic increase in the amount of litter and vandalism throughout the plaza Thus we recommend that the city prohibit skateboarding in Central Plaza If skateboarding is prohibited here we predict that business in Central Plaza will return to its previously high levels

Write a response in which you discuss what questions would need to be answered in order to decide whether the recommendation is likely to have the predicted result Be sure to explain how the answers to these questions would help to evaluate the recommendation

Why two years ago What happened two years ago which started this declineIs the dramatic increase in the ldquopopularityrdquo of skateboarding the cause of the steady decline of shoppers Are there any malls nearby Were there any changes nearby which could affect the decline in customersmdasha big mall perhaps Could the decline be due to the shop ownersHow many skateboarders use the plazaWhere do they skateboardDo they shop and are they customersAre the increase in litter and vandalism due to skateboarders Could this be alleviated by installing CCTVs and hiring security

This editorial concludes that the city should ban skateboarding from its downtown CentralPlaza in order to attract visitors to that area to return the area to its former glory and to makeit a place where people can congregate for fun and relaxation To justify this conclusion theeditorial points out that skateboarders are nearly the only people one sees anymore at CentralPlaza and that the Plaza is littered and its property defaced The editorial also points out thatthe majority of downtown merchants support the skate boarding ban This argument is flawedin several critical respectsFirst the editorials author falsely assumes that a ban on skateboarding is both necessaryand sufficient to achieve the three stated objectives Perhaps the city can achieve thoseobjectives by other means as well--for example by creating a new mall that incorporates anattractive new skateboard park Even if banning skateboarders altogether is necessary to meetthe citys goals the author has not shown that this action by itself would suffice Assuming thatthe Plazas reputation is now tarnished restoring that reputation and in turn enticing peopleback to the Plaza might require additional measures--such as removing litter and graffiti

GRE AWA John박 박정어학원

promoting the Plaza to the public or enticing popular restaurant or retail chains to the PlazaSecondly the editorial assumes too hastily that the Plazas decline is attributable to theskateboarders--rather than to some other phenomenon Perhaps the Plazas primary appeal inits glory days had to do with particular shops or eateries which were eventually replaced byless appealing ones Or perhaps the crime rate in surrounding areas has risen dramatically forreasons unrelated to the skateboarders presence at the Plaza Without ruling out these andother alternative explanations for the Plazas decline the editorials author cannot convince methat a skateboard ban would reverse that declineThirdly the editorials author might be confusing cause with effect--by assuming that theskateboarders caused the abandonment of the Plaza rather than vice versa It is entirelypossible that skateboarders did not frequent the Plaza until it was largely abandoned--andbecause it had been abandoned In fact this scenario makes good sense since skateboardingis most enjoyable where there are few pedestrians or motorists to get in the wayFourth it is unreasonable to infer from the mere fact that most merchants favor the ban thatthe ban would be effective in achieving the citys objectives Admittedly perhaps thesemerchants would be more likely to help dean up the Plaza area and promote their businesseswere the city to act in accordance with their preference Yet lacking any supporting evidencethe author cannot convince me of this Thus the survey amounts to scant evidence at best thatthe proposed ban would carry the intended resultFinally the author recommends a course of action that might actually defeat the citysobjective of providing a fun and relaxing place for people to congregate In my experienceskateboarding contributes to an atmosphere of fun and relaxation for adults and children alikemore so than many other types of ambiance Without considering that continuing to allowskateboarding--or even encouraging this activity--might achieve the citys goal more effectivelythan banning the activity the author cannot convincingly conclude that the ban would be in thecitys best interestsIn sum the argument is a specious one To strengthen it the editorials author must providedear evidence that skateboarding and not some other factor is responsible for the conditionsmarking the Plazas decline The author must also convince me that no alternative means ofrestoring the Plaza are available to the city and that the proposed ban by itself would suffice toattract tourists and restore the Plaza to its former glory Finally to better assess the argument itwould be useful to know the circumstances under which the downtown merchants would bewilling to help the city achieve its objectives

6그룹 약한 비유 빈출

The following recommendation appeared in a memo from the mayor of the town of Hopewell

Two years ago the nearby town of Ocean View built a new municipal golf course and resort hotel During the past two years tourism in Ocean View has increased new businesses have opened there and Ocean Views tax revenues have risen by 30 percent Therefore the best way to improve Hopewells economymdashand generate additional tax revenuesmdashis to build a golf course and resort hotel similar to those in Ocean View

Write a response in which you examine the stated andor unstated assumptions of the argument Be sure to explain how the argument depends on these assumptions and what the implications are for the argument if the assumptions prove unwarranted

GRE AWA John박 박정어학원

Assumptions The author assumes that OVrsquos municipal golf course and resort hotel caused tourism new businesses and increased tax revenues There may be other reasons advertising promo He assumes that this will continueAssumes that Ocean View and Hopewell are similar in many waysmdashthe name suggests otherwise OV may have always been a tourist attractions for its beaches We need to know the topography

2년전 Ocean View 시는 시정 소유 골프 및 휴양지 호텔을 신축했다 그리고 지난 2년동안 이 시의 관광객이 증가했으며 새로운 사업들이 생겨났다 그에따라 시의 세수도 30나 증가했다 Hopewell의 경제를 향상시키고 아울러 세수를 늘릴 수 있는 가장 좋은 방법은 Ocean View에 세워진 것과 같은 골프 시설과 휴양지 호텔을 신축하는 것이다

1 다른 요인으로 관광 산업이 발전했을 수도 있다 문화 유적이 발견이 되었거나 도로의 정비등으로 여행자가 늘었을 수도 있다

2 관광 산업의증가가 늘어난 세수의 원인이 아니라 새로 유입된 인구의 증가나 다른 공장에서 발생한 것일 수 있다

3 2년동안 한참 골프가 붐을 이루었을 수 있다 경제상황이 나빠지거나 다른 레포츠가 인근 지역에 생겨난다면 골프하는 사람이 줄어들 수 있다

In this memo HopeweUs mayor recommends that in order to stimulate the towns economyand boost tax revenues HopeweU should build a new golf course and resort hotel just as thetown of Ocean View did two years ago To support this recommendation the mayor points outthat in Ocean View during the last two years tourism has increased new businesses haveopened and tax revenues have increased by 30 I find the mayors argument unconvincingin several important respectsFirst of all it is possible that the mayor has confused cause with effect respecting the recentdevelopments in Ocean View Perhaps Ocean Views construction of a new golf course andhotel was a response to previous increases in tourism and business development increasesthat have simply continued during the most recent two years Since the mayor has failed toaccount for this possibility the claim that Hopewell would boost its economy by alsoconstructing a golf course and hotel is completely unwarrantedSecondly the mayor fails to account for other possible causes of the trends in Ocean Viewduring the last two years The increase in tourism might have been due to improving economicconditions nationwide or to unusually pleasant weather in the region The new businessesthat have opened in Ocean View might have opened there irrespective of the new golf courseand hotel And the 30 increase in tax revenues might have been the result of an increase intax rates or the addition of a new type of municipal taxWithout ruling out these and other alternative explanations for the three recent trends inOcean View the mayor cannot reasonably infer based on those trends that Hopewellseconomy would benefit by following Ocean Views exampleThirdly even if the recent trends in Ocean View are attributable to the construction of the newgolf course and hotel there the mayor assumes too hastily that the golf course and hotel willcontinue to benefit that towns overall economy The mayor has not accounted for thepossibility that increased tourism will begin to drive residents away during tourist season orthat new business development will result in the towns losing its appeal as a place to visit or tolive Unless the mayor can convince me that these scenarios are unlikely I cannot accept themayors recommendation that Hopewell follow Ocean Views exampleFinally the mayors argument rests on the unsubstantiated assumption that Hopewell andOcean View are sufficiently alike in ways that might affect the economic impact of a new golfcourse and hotel Hopewell might lack the sort of natural environment that would attract moretourists and new businesses to the town--regardless of its new golf course and hotel For thatmatter perhaps Hopewell already contains several resort hotels and golf courses that are notutilized to their capacity If so building yet another golf course and hotel might amount to amisallocation of the towns resources--and actually harm the towns overall economyIn sum the mayors recommendation is not well supported To bolster it the mayor mustprovide better evidence that Ocean Views new golf course and hotel and not some otherphenomenon--has been responsible for boosting Ocean Views economy during the last twoyears To better assess the recommendation I would need to know why Ocean View decidedto construct its new golf course and hotel in the first place--specifically what events prior toconstruction might have prompted that decision I would also need to thoroughly compare

GRE AWA John박 박정어학원

HopeweU with Ocean View--especially in terms of their appeal to tourists and businesses--todetermine whether the same course of action that appears to have boosted Ocean Viewseconomy would also boost Hopewells economy

The following is part of a memorandum from the president of Humana University

Last year the number of students who enrolled in online degree programs offered by nearby Omni University increased by 50 percent During the same year Omni showed a significant decrease from prior years in expenditures for dormitory and classroom space most likely because instruction in the online programs takes place via the Internet In contrast over the past three years enrollment at Humana University has failed to grow and the cost of maintaining buildings has increased along with our budget deficit To address these problems Humana University will begin immediately to create and actively promote online degree programs like those at Omni We predict that instituting these online degree programs will help Humana both increase its total enrollment and solve its budget problems

Write a response in which you discuss what questions would need to be answered in order to decide whether the prediction and the argument on which it is based are reasonable Be sure to explain how the answers to these questions would help to evaluate the prediction

Is Omni University successful due to the online degree program 50 Is the decrease in expenditures for dormitory and classroom space due to the decrease in of on-campus students Which classes were successful Does HU have those classes

Even if the long-distance degree programs at Omni University benefited the school the presidentrsquos recommendation that Human College should emulate Omni University is too hasty First OUrsquos name implies that the school would have more majors than Humanahellip the president should examine which degrees were in the long-distance programhellip

지난해에는 Omni 대학에서 개강했던 원거리 학생 학점 취득 프로그램을 등록했던 학생들의 숫자가 50나 증가했다 같은해 기간동안 Omni 대학에서는 그 전년도부터 기숙사와 학급의 공간 확충을 위한 예산을 대폭 줄였는데 이는 이 원거리 학점 취득 프로그램이 양방향 비디오 컴퓨터 접속을 통해서만 가능한 수업지도 방식이기때문인 것으로 보인다 반면 지난 3개년 동안 Humana 대학에서의 수강률은 감소한데다가 건물

GRE AWA John박 박정어학원

유지비도 올랐다 따라서 Humana대학의 수강을 늘리고 예산손실을 회복하기 위해서는 Omni 대학에서 취한 조치와 같은 능동적인 프로그램을 추진해야 한다

결론 we should initiate and actively promote long-distance degree programs like those at Omni 반박 원거리 학생 취득 프로그램 숫자가 증가한거하고 예산이 줄어드는 것 사이에 연관이 약하다 (causal 학생의 증가로 관리비용 증가할수 있음 원거리 수업가능 장비도입에의한 비용발생)bad analogy(omni university 하고 같은 조건이 아니다 )-gt omni college 가 강좌내용이 좋아서 학생의 등록이 많을수 있다 Humana 대학에서 만들었다 하더라도 인기 없을수 있음다른 요인에 의해서 Humana 대학의 수강 인원이 증가할수 있음(비록 과거엔 인기가 없었을지라도)

The following appeared as part of a business plan developed by the manager of the Rialto Movie Theater

Despite its downtown location the Rialto Movie Theater a local institution for five decades must make big changes or close its doors forever It should follow the example of the new Apex Theater in the mall outside of town When the Apex opened last year it featured a video arcade plush carpeting and seats and a state-of-the-art sound system Furthermore in a recent survey over 85 percent of respondents reported that the high price of newly released movies prevents them from going to the movies more than five times per year Thus if the Rialto intends to hold on to its share of a decreasing pool of moviegoers it must offer the same features as Apex

Write a response in which you discuss what questions would need to be answered in order to decide whether the recommendation is likely to have the predicted result Be sure to explain how the answers to these questions would help to evaluate the recommendation

Before following through this business plan the manager should investigate the cause of Rialtorsquos unsuccessful business

The author provides no evidence that the surveyrsquos results are statistically reliable The surveyrsquos sample of 85 percent must be sufficient in size and representative of overall population of the city where Rialto and Apex is serving Lacking evidence of a sufficiently representative sample the author cannot justifiably rely on the survey to draw any conclusion whatsoever The author does not indicate that Apex is indeed currently successful However even if Apex is enjoying success the argument relies on what might be a false analogy between Rialto and Apex In order for Apex to serve as a model that Rialto should emulate the author must assume that all relevant circumstances are essentially the same However this assumption is unwarranted For example the argument overlooks the face that Apex is located in a strategic placemdashbeside a mall where customers can not only watch a movie but also enjoy shopping Therefore simply changing the facility to that of Apex may not lead to success

The author does not mention whether Apex is successful or not Nevertheless even if Apex is currently successful the argument relies on what might be a false analogy between Rialto and Apex In order for Apex to serve as a model that Rialto should emulate the author must assume that all relevant circumstances are essentially the same However this assumption is unwarranted For example the argument overlooks the fact that these two institutions are located in different locations Rialto in downtown and Apex in a mall outside of town Although Apex opened with state-of-the-art facilities the decisive factor in its success could be due to its strategic location of being in a mall People could enjoy both shopping and movies at one location thus they may prefer Apex over Rialto Furthermore the place where people enjoy leisure activities has shifted in the past decades for most cities from downtown to the suburbs Therefore Rialto may not be successful even if it emulates Apexrsquos facilities A better business plan may be relocating Apex to the thriving section of the downtown

Rialto 극장은 지난 50여년간 지역 회관으로써 시내에 위치해 있으면서도 이제 변화를 꾀하지 않으면 문을 닫을

GRE AWA John박 박정어학원

판이다 이 극장은 시외 쇼핑타운에 새로 들어선 Apex 극장의 사례를 본받아야 했다 Apex가 지난해 개업했을 당시 이 극장은 비디오 아케이드 플러쉬 카펫트 바닥과 좌석 그리고 최신 음향시설을 갖추었다 더군다나 최근 조사에서는 응답자의 85 이상이 새로 출시된 영화 입장료가 비싼 탓으로 지난해보다 5배이상의 관람객이 줄어들었다고 나타났다 따라서 Rialto 극장이 줄어들고 있는 관람객을 뺐기지 않고 유지하려면 Apex와 같은 시설들을 갖추어야 할 것이다주장 리알토 극장이 줄어들고 있는 관람객을 뺐기지 않고 유지하려면 Apex와 같은 시설들을 갖추어야 할 것이다

1 조사에서 응답자가 전체를 대표할 수 없다 2 apex 극장이 좋은 시설을 갖추고 있지만 그로 인해 수익이 많이 발생했다는 말이 없으므로 시설투자를

하고도 좋은 결과를 얻을 수 있을지 그 근거가 미흡하다3 좋은 영화가 출시된다면 입장료가 비싸도 영화관에서 꼭 보려고 할 수 있다 4 rialto 가 시설이 아닌 다른 요인에 의해 장사가 안될수도 있다( 우범 지역이라든지)

The following is a recommendation from the business manager of Monarch Books

Since its opening in Collegeville twenty years ago Monarch Books has developed a large customer base due to its reader-friendly atmosphere and wide selection of books on all subjects Last month Book and Bean a combination bookstore and coffee shop announced its intention to open a Collegeville store Monarch Books should open its own in-store cafeacute in the space currently devoted to childrens books Given recent national census data indicating a significant decline in the percentage of the population under age ten sales of childrens books are likely to decline By replacing its childrens books section with a cafeacute Monarch Books can increase profits and ward off competition from Book and Bean

Write a response in which you examine the stated andor unstated assumptions of the argument Be sure to explain how the argument depends on these assumptions and what the implications are for the argument if the assumptions prove unwarranted

The following is a recommendation from the business manager of Monarch Books

Since its opening in Collegeville twenty years ago Monarch Books has developed a large customer base due to its reader-friendly atmosphere and wide selection of books on all subjects Last month Book and Bean a combination bookstore and coffee shop announced its intention to open a Collegeville store Monarch Books should open its own in-store cafeacute in the space currently devoted to childrens books Given recent national census data indicating a significant decline in the percentage of the population under age ten sales of childrens books are likely to decline By replacing its childrens books section with a cafeacute Monarch Books can increase profits and ward off competition from Book and Bean

1 Write a response in which you discuss what questions would need to be answered in order to decide whether the recommendation is likely to have the predicted result Be sure to explain how the answers to these questions would help to evaluate the recommendation

2 Write a response in which you discuss what specific evidence is needed to evaluate the argument and explain how the evidence would weaken or strengthen the argument

No evidence regarding Monarch Bookrsquos successEven if Regal Bookrsquos is successful this may not be attributable to the cafeacute False analogy Emulating may not lead to success Other factors may be involvedInsufficient condition The national census is not enough evidence that childrenrsquos book sales will decline Can

GRE AWA John박 박정어학원

the national census represent the local child populationDid opening a cafeacute boost sales for Regal Books Even assuming Regal is successful by opening a cafeacute this may not be suitable for Monarch which plans to close the childrenrsquos book section to establish a cafe Imprecise language ldquorelatively little spacerdquo how smallThe managerrsquos recommendation contradicts what he says Since Monarch is popular for its wide selection of books closing a selection which targets a major group of readers may hurt Monarchrsquos salesIs this the best way to compete

When Stanley Park first opened it was the largest most heavily used public park in town It is still the largest park but it is no longer heavily used Video cameras mounted in the parks parking lots last month revealed the parks drop in popularity the recordings showed an average of only 50 cars per day In contrast tiny Carlton Park in the heart of the business district is visited by more than 150 people on a typical weekday An obvious difference is that Carlton Park unlike Stanley Park provides ample seating Thus if Stanley Park is ever to be as popular with our citizens as Carlton Park the town will obviously need to provide more benches thereby converting some of the unused open areas into spaces suitable for socializing

Write a response in which you examine the stated andor unstated assumptions of the argument Be sure to explain how the argument depends on these assumptions and what the implications are for the argument if the assumptions prove unwarranted

Stanley 파크가 처음 개장했을 당시 가장 크고 가장 많이 이용되는 공원이었다 아직도 공원중에서는 가장 크지만 이용률은 상당히 떨어졌다 지난달 공원 주차장에 설치해놓은 비디오 카메라를 통해 보면 drop(주차장으로 여겨짐) 이용률이 가장 높았다 수치상으로는 하루 평균 50대의 차량만이 이용하였다 반면 직장 중심거리에 위치한 작은 규모의 Carlton 파크는 주당 무려 150여명 이상이 이용하고 있다 Stanley 파크와는 달리 Carlton 파크에는 의자가 있다는 것이 가장 뚜렷한 차이점이다 따라서 Stanley 파크가 Carlton 파크처럼 시민들이 자주 이용하는 공원이 되기 위해서는 벤치를 설치할 필요가 있으며 이렇게 사용되지 않는 일부 공간을 활용해서 사교를 위한 공간으로 바꾸어야 한다 ===gtdrop 에 대한 첨부사항 (영영사전내용입니다)---- a place or central depository to which something (as mail money or stolen property) is brought for distribution or transmission also the act of depositing something at such a place dropgt

주장 if Stanley Park is ever to be as popular with our citizens as is Carlton Park the town will obviously need to provide more benches thereby converting some of the unused open areas into spaces suitable for socializing1 조사가 언제 이루어진 것인가 조사가 언제 실시되었느냐에 따라 결과가 다를 수있다 현재는 다시 스탠리 파크가 늘어났었을 수 있다 2 벤치를 많이 설치했다고 해서 많은 관광객이 오지 않을수 있다(사람들이 벤치나 사교 공간을 원한다는 어떠한 자료도 없다)3스탠리 파크 주변에 교통 상황이 악화가 되었거나 칼튼 파크에서 문화행사등을 많이 가져서 이용객이 줄어든것일 수도 있다 4 칼튼 파크가 중심지에 있어서 접근성이 좋을수 있다5 조사가 같은 시간을 기준으로 한게 아니다(하나는 주중이고 하나는 주말이다)6사람의 수와 차의 대수를 같은것으로 비교할수 없다 (차안에 몇 명이 타고 있는지 모르고 대중교통을 이용해서 왔을수도 있다)

Page 29: GRE writing argument brain storm

GRE AWA John박 박정어학원

about jazz Finally in the case of radio program this is also not suitable reason why jazz is popular in Monroe It might be possible that people cannot help choosing lsquoJazz Nightlyrsquo because there are few radio programs at Night The fact that the radio program is the highest rating program is not a germane evidence The approximate number of listeners would be the more crucial evidence Therefore the author needs to seriously deliberate the correlation between jazzrsquos popularity in Monroe and his examples (Good logical flow and clarity)

Second the author uses as evidence the nationwide study that jazz fans spend much money on jazz entertainment to substantiate why starting a jazz club in Monroe will be profitable In other words the author assumes that the characteristics of a nationwide study can be applied to Monroe The national study would lend support to the applicantrsquos claim only if residents in Monroe typify national jazz fans However the author does not provide credible evidence that this is the case Moreover the populations of jazz fans nationwide may be insubstantial Thus the author should not infer hastily that Monroersquos residents will spend much money on enjoying jazz from the nationwide study

Lastly even if jazz is popular in Monroe C Note may not be successful It is entirely possible that residents might still prefer other clubs where they have always went In addition there is another possibility that the nearest jazz club will attract many of Monroersquos people because it serves fine performances and is equipped with favorite facilities Without considering these other possibilities the author cannot make his argument convincing In sum the author presents many reasons that are insufficient in supporting his or her claim In order for the authorrsquos claims to be convincing he needs to advance more persuasive evidence such as the total number of Monroe residents who attended the jazz festival the effects on the popularity of jazz by the musicians living in Monroe and the actual number of residents who would typify themselves to be jazz fans through a local survey Without substantial evidence that C Note will be successful in Monroe the businessmen may be overinvesting in what might lead to a business failureExcellent clarity Score 50

The following appeared in a newsletter offering advice to investors

Over 80 percent of the respondents to a recent survey indicated a desire to reduce their intake of foods containing fats and cholesterol and today low-fat products abound in many food stores Since many of the food products currently marketed by Old Dairy Industries are high in fat and cholesterol the companys sales are likely to diminish greatly and company profits will no doubt decrease We therefore advise Old Dairy stockholders to sell their shares and other investors not to purchase stock in this company

Write a response in which you discuss what questions would need to be answered in order to decide whether the advice and the argument on which it is based are reasonable Be sure to explain how the answers to these questions would help to evaluate the advice

Survey 80

GRE AWA John박 박정어학원

Old Dairy could change their products and manufacture low fat dairy foodsLess competing companies Old Dairy could eventually be the only company that produces hellipImprecise numbers and measurementsCustomers may still buy high fat dairy products

The author of the newsletter is offering potentially dangerous advice by recommending Old Dairy stockholders to withdraw investment and stop purchase What is more the authorrsquos prediction debases the reputation and business of Old Dairy and if false could devoid the investment opportunity of the newsletter readers Therefore investors should examine whether the authorrsquos evidences are substantial

To begin with the author states that 80 percent of the respondents in a survey indicated a desire to reduce their intake of foods He therefore argues that Old Dairyrsquos high fat and cholesterol products would decrease in sales However the author makes a crucial error in this argument First the author provides no evidence that the surveyrsquos results are statistically reliable Were they representative of all the customers Were they chosen for the survey randomly Furthermore the desire to reduce fat and cholesterol intake is a pervasive trend in todayrsquos opulent society however the author erroneously identifies this as a new phenomenon which will affect consumer trends Second having a desire to reduce fat and cholesterol intake does not necessarily indicate that people who have this desire will actually reduce consuming these types of products It is entirely possible that they may continue buying Old Dairy products for its quality and taste Accordingly the author cannot draw any firm conclusion that people will not buy Old Dairy products Therefore if any of these cases are true the author may be offering investors a detrimental investment advice

최근 조사에 대한 응답자중 80 이상이 자신이 먹는 음식에서 지방과 콜레스테롤의 함유량을 줄이고 싶다고 한다 아울러 요즘은 많은 식료품 가계에서 저지방 제품들을 많이 취급하고 있다 현재 Old Dairy Industries가 판매하고 있는 많은 음식제품들은 지방과 콜레스테롤이 높기 때문에 이 회사의 매출이 격감할 것으로 보이며 당연히 매출이익도 줄어들것이다 따라서 이 회사의 주주들은 주식을 매각하고 다른 주식 투자가들도 이 회사의 주식을 매입하지 않는 것이 좋다

결론 Old Dairy stockholders to sell their shares and other investors not to purchase stock in this company

반박 모든 상품이 다 고 지방 고 칼로리는 아니다(비록 많을지라도) 일부의 식품의 경우 기호에 맞어서 히트해서 전체적인 수입이 증가할 수도 있다국내시장만 생각할 수 없다( 외국시장에서 호황을 누릴수 있다 )입맛이라는게 즉각 바뀌는게 아니다

The following appeared in a letter to the editor of the Balmer Island Gazette

On Balmer Island where mopeds serve as a popular form of transportation the population increases to 100000 during the summer months To reduce the number of accidents involving mopeds and pedestrians the town council of Balmer Island should limit the number of mopeds rented by the islands moped rental companies from 50 per day to 25 per day during the summer season By limiting the number of rentals the town council will attain the 50 percent annual reduction in moped accidents that was achieved last year on the neighboring island of Seaville when Seavilles town council enforced similar limits on moped rentals

1 Write a response in which you discuss what questions would need to be answered in order to decide whether the recommendation is likely to have the predicted result Be sure to explain how the answers to these questions would help to evaluate the recommendation

2 Write a response in which you discuss what questions would need to be answered in order to decide whether the prediction and the argument on which it is based are reasonable Be sure to explain how the answers to these questions would help to evaluate the prediction

3 Write a response in which you examine the stated andor unstated assumptions of the argument Be sure to explain how the argument depends on these assumptions and what the implications are for the argument if the assumptions prove unwarranted

Whatrsquos the actual population of Balmer Island 100000mdashis this a significant increase What kind of accidents Skin abrasions or serious injury And compared to Seaville how serious are the accidents and the actual number of accidents Did Seaville enforce other restrictions like safety signsHow different are the conditions of Balmer

GRE AWA John박 박정어학원

and Seaville regarding population road (safety) conditions topography other town-government regulation How much will the economy of Balmer be affected do to this restriction Could it cause an economic recession due to the fact that these rental companiesrsquo chance to make money is only during the summer thereby weakening the economic infrastructure Are there any other ways that could better alleviate the accident rate

Statistics 50-impreciseAnalogy Balmer compared with TorseauFalse Cause Accidents might have occurred because of reasons other than mopeds False Cause population increase may not be part of the cause of the accidentsOther explanations for the accident pedestrians few road safety regulations narrow roadsThere could be other better solutionshellip

Balmer Island의 인구가 여름철에는 십만명으로 늘어난다 2륜차와 보행자간 사고를 줄이기 위해 시의회는 6개의 자전거를 포함한 2륜차 대여업체에게 이 기간동안에는 대여숫자를 일일 50에서 30으로 제한하도록 할 것이다 대여숫자를 줄임으로써 시 의회는 지난해 이웃한 Torseau섬에서 이와 동일한 규제를 시행해서 50나 줄인 결과를 보고 마찬가지로 50를 줄일수 있다고 확신하고 있다

결론 The town council of Balmer Island should linit the number

반박 보행자의 부실에 의해서 사고가 많이 일어날수도 있다렌탈수의 줄임만이 대책은 아니다(대부분의 사람들이 렌탈 보다는 소유하고 있을 수도 있다)옆섬과는 상황이 다를수도 있다(그 섬에서는 사고의 원인이 많은 자전거 수로 인한것일수있다) 하지만 이 섬은 좁은 도로가 원인일 수도 있고 도로 안전 장치의미비가 원일일수 있다

In this letter the author recommends that Balmer Island should limit the number moped rentals from 50 to 30 per day To support this recommendation the author points out several reasons However careful scrutiny of each of the facts reveals that it is filled with unanswered questions that could significantly weaken the authorrsquos recommendation with loops and holes which are answered

The recommendation depends on the assumption that no alternative means of reducing the number of accidents are available However the author fails to offer any evidence to substantiate this crucial assumption It is highly possible that means other than this would better solve the problem Perhaps they could widen the roads or put-up more safety signs Or perhaps the accidents were due to the lack of skills in which case proper safety training would significantly alleviate the problem Without considering and ruling out these and other alternative means of reducing accidetns the author cannot confidently conclude that merely emulating Torseau would suffice Moreover the author is advising a recommendation which could potentially harm the economy of Balmer Island sincehellip Moreover the Balmer Island should alternative means to reduce accidents because limiting moped rentals during the summer could harm the economy of Balmerhellip

First of all the author believes that increase in population and the number of moped rentals are responsible for the accidents It is entirely possible that other factors are responsible for the accidents Perhaps Balmer Islandrsquos lack of safety signs was a major factor Or maybe the roads are narrow and dangerous on the Island therefore the town council could enforce stricter traffic regulations to alleviate the problem Accordingly if either of these scenarios is true the author cannot draw any firm conclusion that increase in the number of population and moped rentals are the cause of the accidents

The author of this editorial recommends that to reduce accidents involving mopeds andpedestrians Balmer Islands city council should restrict moped rentals to 30 per day down from50 at each of the islands six rental outlets To support this recommendation the author citesthe fact that last year when nearby Torseau Islands town council enforced similar measuresTorseaus rate of moped accidents fell by 50 For several reasons this evidence providesscant support for the authors recommendationTo begin with the author assumes that all other conditions in Balmer that might affect therate of moped-pedestrian accidents will remain unchanged after the restrictions are enactedHowever with a restricted supply of rental mopeds people in Balmer might purchase mopedsinstead Also the number of pedestrians might increase in the future with more pedestriansespecially tourists the risk of moped-pedestrian accidents would probably increase For thatmatter the number of rental outlets might increase to make up for the artificial supplyrestriction per outlet--a likely scenario assuming moped rental demand does not declineWithout considering and ruling out these and other possible changes that might contribute to ahigh incidence of moped-pedestrian accidents the author cannot convince me that theproposed restrictions will necessarily have the desired effect

GRE AWA John박 박정어학원

Next the author fails to consider other possible explanations for the 50 decline inTorseaus moped accident rate last year Perhaps last year Torseau experienced unusually fairweather during which moped accidents are less likely Perhaps fewer tourists visited Tot seanlast year than during most years thereby diminishing the demand for rental mopeds to belowthe allowed limits Perhaps last year some of Torseaus moped rental outlets purchased newmopeds that are safer to drive Or perhaps the restrictions were already in effect but were notenforced until last year In any event a decline in Torseaus moped accident rate during onlyone year is scarcely sufficient to draw any reliable conclusions about what might have causedthe decline or about what the accident rate will be in years aheadAdditionally in asserting that the same phenomenon that caused a 50 decline in mopedaccidents in Torseau would cause a similar decline in Balmer the author relies on what mightamount to an unfair analogy between Balmer and Torseau Perhaps Balmers ability to enforcemoped-rental restrictions does not meet Torseaus ability if not then the mere enactment ofsimilar restrictions in Balmer is no guarantee of a similar result Or perhaps the demand formopeds in Torseau is always greater than in Balmer Specifically if fewer than all availablemopeds are currently rented per day from the average Balmer outlet while in Torseau everyavailable moped is rented each day then the proposed restriction is likely to have less impacton the accident rate in Balmer than in TorseauFinally the author provides no evidence that the same restrictions that served to reduce theincidence of all moped accidents by 50 would also serve to reduce the incidence ofaccidents involving mopeds and pedestrians by 50 Lacking such evidence it is entirelypossible that the number of moped accidents not involving pedestrians decreased by a greaterpercentage while the number of moped-pedestrian accidents decreased by a smallerpercentage or even increased Since the author has not accounted for these possibilities theeditorials recommendation cannot be taken seriouslyIn conclusion the recommendation is not well supported To convince me that the proposedrestriction would achieve the desired outcome the author would have to assure me that nochanges serving to increase Balmers moped-pedestrian accident rate will occur in theforeseeable future The author must also provide dear evidence that last years decline inmoped accidents in Torseau was attributable primarily to its moped rental restrictions ratherthan to one or more other factors In order to better evaluate the recommendation I wouldneed more information comparing the supply of and demand for moped rentals on the twoislands I would also need to know the rate of mopedpedestrian accidents in Torseau both priorto and after the restrictions were enforced in TorseauThe following appeared in a magazine article about planning for retirement

Clearview should be a top choice for anyone seeking a place to retire because it has spectacular natural beauty and a consistent climate Another advantage is that housing costs in Clearview have fallen significantly during the past year and taxes remain lower than those in neighboring towns Moreover Clearviews mayor promises many new programs to improve schools streets and public services And best of all retirees in Clearview can also expect excellent health care as they grow older since the number of physicians in the area is far greater than the national average

Write a response in which you discuss what specific evidence is needed to evaluate the argument and explain how the evidence would weaken or strengthen the argument

-Natural beauty and consistent climate may not be the most wanted qualities-Housing costs could have lowered on a national level wealthy retirees may not care about costs-Taxes may be high compared to the nationrsquos average tax rate-What about other qualities of Clearview Crime rate what qualities would retirees want -If schools streets and public services need improvement then this is proof that the current condition of Clearview is low Or due to budgetary reasons the mayor may not follow-up on his promise because of lowered tax rate -Schools and people who are retired no relationship-Physicians What kind of physicians Number is irrelevant Are these physicians capable of addressing the illnesses of old people

This author argues that anyone seeking a place to retire should choose Clearview To supportthis argument the article cites Clearviews consistent climate and natural beauty its fallinghousing costs its low property taxes compared to nearby towns and the mayors promise toimprove schools streets and services The article also claims that retirees can expectexcellent health care because the number of physicians in Clearview greatly exceeds thenational average This argument is flawed in several critical respectsTo begin with although consistent climate and natural beauty might be attractive to manyretirees these features are probably not important to all retirees For many retirees it isprobably more important to live near relatives or even to enjoy changing seasons Thus I

GRE AWA John박 박정어학원

cannot accept the authors sweeping recommendation for all retirees on this basisAlso Clearviews declining housing costs do not necessarily make Clearview the best placeto retire for two reasons First despite the decline Clearviews housing costs might be highcompared to housing costs in other cities Secondly for wealthier retirees housing costs arenot likely to be a factor in choosing a place to retire Thus the mere fact that housing costshave been in decline lends scant support to the recommendationThe articles reliance on Clearviews property-tax rates is also problematic in two respectsFirst retirees obviously have innumerable choices about where to retire besides Clear viewand nearby towns Secondly for retirees who are well-off financially property taxes are notlikely to be an important concern in choosing a place to retire Thus it is unfair to infer fromClearviews property-tax rates that retirees would prefer ClearviewYet another problem with the argument involves the mayors promises In light of Clearviewslow property-tax rates whether the mayor can follow through on those promises is highlyquestionable Absent any explanation of how the city can spend more money in the areas citedwithout raising property taxes I simply cannot accept the editorials recommendation on thebasis of those promises Besides even if the city makes the improvements promised thoseimprovements--particular the ones to schools--would not necessarily be important to retireesFinally although the number of physicians in Clearview is relatively high the per capitanumber might be relatively low Moreover it would be fairer to compare this per capita numberwith the per capita number for other attractive retirement towns--rather than the nationalaverage After all retirees are likely to place a relatively heavy burden on health-careresources Besides the article provides no assurances that the number of physicians inClearview will remain high in the foreseeable futureIn conclusion the recommendation is poorly supported To strengthen it the author mustconvince me--perhaps by way of a reliable survey--that the key features that the vast majorityof retirees look for in choosing a place to live are consistent climate natural beauty and lowhousing costs The author must also provide better evidence that Clear views property taxesare lower than the those of cities in other areas The author must also explain how the city canmake its promised improvements without raising property taxes Finally to better assess theargument I would need to now how the per capita number of physicians in Clearview wouldcompare to the national average in the futureThe following appeared as a letter to the editor from a Central Plaza store owner

Over the past two years the number of shoppers in Central Plaza has been steadily decreasing while the popularity of skateboarding has increased dramatically Many Central Plaza store owners believe that the decrease in their business is due to the number of skateboard users in the plaza There has also been a dramatic increase in the amount of litter and vandalism throughout the plaza Thus we recommend that the city prohibit skateboarding in Central Plaza If skateboarding is prohibited here we predict that business in Central Plaza will return to its previously high levels

Write a response in which you discuss what questions would need to be answered in order to decide whether the recommendation is likely to have the predicted result Be sure to explain how the answers to these questions would help to evaluate the recommendation

Why two years ago What happened two years ago which started this declineIs the dramatic increase in the ldquopopularityrdquo of skateboarding the cause of the steady decline of shoppers Are there any malls nearby Were there any changes nearby which could affect the decline in customersmdasha big mall perhaps Could the decline be due to the shop ownersHow many skateboarders use the plazaWhere do they skateboardDo they shop and are they customersAre the increase in litter and vandalism due to skateboarders Could this be alleviated by installing CCTVs and hiring security

This editorial concludes that the city should ban skateboarding from its downtown CentralPlaza in order to attract visitors to that area to return the area to its former glory and to makeit a place where people can congregate for fun and relaxation To justify this conclusion theeditorial points out that skateboarders are nearly the only people one sees anymore at CentralPlaza and that the Plaza is littered and its property defaced The editorial also points out thatthe majority of downtown merchants support the skate boarding ban This argument is flawedin several critical respectsFirst the editorials author falsely assumes that a ban on skateboarding is both necessaryand sufficient to achieve the three stated objectives Perhaps the city can achieve thoseobjectives by other means as well--for example by creating a new mall that incorporates anattractive new skateboard park Even if banning skateboarders altogether is necessary to meetthe citys goals the author has not shown that this action by itself would suffice Assuming thatthe Plazas reputation is now tarnished restoring that reputation and in turn enticing peopleback to the Plaza might require additional measures--such as removing litter and graffiti

GRE AWA John박 박정어학원

promoting the Plaza to the public or enticing popular restaurant or retail chains to the PlazaSecondly the editorial assumes too hastily that the Plazas decline is attributable to theskateboarders--rather than to some other phenomenon Perhaps the Plazas primary appeal inits glory days had to do with particular shops or eateries which were eventually replaced byless appealing ones Or perhaps the crime rate in surrounding areas has risen dramatically forreasons unrelated to the skateboarders presence at the Plaza Without ruling out these andother alternative explanations for the Plazas decline the editorials author cannot convince methat a skateboard ban would reverse that declineThirdly the editorials author might be confusing cause with effect--by assuming that theskateboarders caused the abandonment of the Plaza rather than vice versa It is entirelypossible that skateboarders did not frequent the Plaza until it was largely abandoned--andbecause it had been abandoned In fact this scenario makes good sense since skateboardingis most enjoyable where there are few pedestrians or motorists to get in the wayFourth it is unreasonable to infer from the mere fact that most merchants favor the ban thatthe ban would be effective in achieving the citys objectives Admittedly perhaps thesemerchants would be more likely to help dean up the Plaza area and promote their businesseswere the city to act in accordance with their preference Yet lacking any supporting evidencethe author cannot convince me of this Thus the survey amounts to scant evidence at best thatthe proposed ban would carry the intended resultFinally the author recommends a course of action that might actually defeat the citysobjective of providing a fun and relaxing place for people to congregate In my experienceskateboarding contributes to an atmosphere of fun and relaxation for adults and children alikemore so than many other types of ambiance Without considering that continuing to allowskateboarding--or even encouraging this activity--might achieve the citys goal more effectivelythan banning the activity the author cannot convincingly conclude that the ban would be in thecitys best interestsIn sum the argument is a specious one To strengthen it the editorials author must providedear evidence that skateboarding and not some other factor is responsible for the conditionsmarking the Plazas decline The author must also convince me that no alternative means ofrestoring the Plaza are available to the city and that the proposed ban by itself would suffice toattract tourists and restore the Plaza to its former glory Finally to better assess the argument itwould be useful to know the circumstances under which the downtown merchants would bewilling to help the city achieve its objectives

6그룹 약한 비유 빈출

The following recommendation appeared in a memo from the mayor of the town of Hopewell

Two years ago the nearby town of Ocean View built a new municipal golf course and resort hotel During the past two years tourism in Ocean View has increased new businesses have opened there and Ocean Views tax revenues have risen by 30 percent Therefore the best way to improve Hopewells economymdashand generate additional tax revenuesmdashis to build a golf course and resort hotel similar to those in Ocean View

Write a response in which you examine the stated andor unstated assumptions of the argument Be sure to explain how the argument depends on these assumptions and what the implications are for the argument if the assumptions prove unwarranted

GRE AWA John박 박정어학원

Assumptions The author assumes that OVrsquos municipal golf course and resort hotel caused tourism new businesses and increased tax revenues There may be other reasons advertising promo He assumes that this will continueAssumes that Ocean View and Hopewell are similar in many waysmdashthe name suggests otherwise OV may have always been a tourist attractions for its beaches We need to know the topography

2년전 Ocean View 시는 시정 소유 골프 및 휴양지 호텔을 신축했다 그리고 지난 2년동안 이 시의 관광객이 증가했으며 새로운 사업들이 생겨났다 그에따라 시의 세수도 30나 증가했다 Hopewell의 경제를 향상시키고 아울러 세수를 늘릴 수 있는 가장 좋은 방법은 Ocean View에 세워진 것과 같은 골프 시설과 휴양지 호텔을 신축하는 것이다

1 다른 요인으로 관광 산업이 발전했을 수도 있다 문화 유적이 발견이 되었거나 도로의 정비등으로 여행자가 늘었을 수도 있다

2 관광 산업의증가가 늘어난 세수의 원인이 아니라 새로 유입된 인구의 증가나 다른 공장에서 발생한 것일 수 있다

3 2년동안 한참 골프가 붐을 이루었을 수 있다 경제상황이 나빠지거나 다른 레포츠가 인근 지역에 생겨난다면 골프하는 사람이 줄어들 수 있다

In this memo HopeweUs mayor recommends that in order to stimulate the towns economyand boost tax revenues HopeweU should build a new golf course and resort hotel just as thetown of Ocean View did two years ago To support this recommendation the mayor points outthat in Ocean View during the last two years tourism has increased new businesses haveopened and tax revenues have increased by 30 I find the mayors argument unconvincingin several important respectsFirst of all it is possible that the mayor has confused cause with effect respecting the recentdevelopments in Ocean View Perhaps Ocean Views construction of a new golf course andhotel was a response to previous increases in tourism and business development increasesthat have simply continued during the most recent two years Since the mayor has failed toaccount for this possibility the claim that Hopewell would boost its economy by alsoconstructing a golf course and hotel is completely unwarrantedSecondly the mayor fails to account for other possible causes of the trends in Ocean Viewduring the last two years The increase in tourism might have been due to improving economicconditions nationwide or to unusually pleasant weather in the region The new businessesthat have opened in Ocean View might have opened there irrespective of the new golf courseand hotel And the 30 increase in tax revenues might have been the result of an increase intax rates or the addition of a new type of municipal taxWithout ruling out these and other alternative explanations for the three recent trends inOcean View the mayor cannot reasonably infer based on those trends that Hopewellseconomy would benefit by following Ocean Views exampleThirdly even if the recent trends in Ocean View are attributable to the construction of the newgolf course and hotel there the mayor assumes too hastily that the golf course and hotel willcontinue to benefit that towns overall economy The mayor has not accounted for thepossibility that increased tourism will begin to drive residents away during tourist season orthat new business development will result in the towns losing its appeal as a place to visit or tolive Unless the mayor can convince me that these scenarios are unlikely I cannot accept themayors recommendation that Hopewell follow Ocean Views exampleFinally the mayors argument rests on the unsubstantiated assumption that Hopewell andOcean View are sufficiently alike in ways that might affect the economic impact of a new golfcourse and hotel Hopewell might lack the sort of natural environment that would attract moretourists and new businesses to the town--regardless of its new golf course and hotel For thatmatter perhaps Hopewell already contains several resort hotels and golf courses that are notutilized to their capacity If so building yet another golf course and hotel might amount to amisallocation of the towns resources--and actually harm the towns overall economyIn sum the mayors recommendation is not well supported To bolster it the mayor mustprovide better evidence that Ocean Views new golf course and hotel and not some otherphenomenon--has been responsible for boosting Ocean Views economy during the last twoyears To better assess the recommendation I would need to know why Ocean View decidedto construct its new golf course and hotel in the first place--specifically what events prior toconstruction might have prompted that decision I would also need to thoroughly compare

GRE AWA John박 박정어학원

HopeweU with Ocean View--especially in terms of their appeal to tourists and businesses--todetermine whether the same course of action that appears to have boosted Ocean Viewseconomy would also boost Hopewells economy

The following is part of a memorandum from the president of Humana University

Last year the number of students who enrolled in online degree programs offered by nearby Omni University increased by 50 percent During the same year Omni showed a significant decrease from prior years in expenditures for dormitory and classroom space most likely because instruction in the online programs takes place via the Internet In contrast over the past three years enrollment at Humana University has failed to grow and the cost of maintaining buildings has increased along with our budget deficit To address these problems Humana University will begin immediately to create and actively promote online degree programs like those at Omni We predict that instituting these online degree programs will help Humana both increase its total enrollment and solve its budget problems

Write a response in which you discuss what questions would need to be answered in order to decide whether the prediction and the argument on which it is based are reasonable Be sure to explain how the answers to these questions would help to evaluate the prediction

Is Omni University successful due to the online degree program 50 Is the decrease in expenditures for dormitory and classroom space due to the decrease in of on-campus students Which classes were successful Does HU have those classes

Even if the long-distance degree programs at Omni University benefited the school the presidentrsquos recommendation that Human College should emulate Omni University is too hasty First OUrsquos name implies that the school would have more majors than Humanahellip the president should examine which degrees were in the long-distance programhellip

지난해에는 Omni 대학에서 개강했던 원거리 학생 학점 취득 프로그램을 등록했던 학생들의 숫자가 50나 증가했다 같은해 기간동안 Omni 대학에서는 그 전년도부터 기숙사와 학급의 공간 확충을 위한 예산을 대폭 줄였는데 이는 이 원거리 학점 취득 프로그램이 양방향 비디오 컴퓨터 접속을 통해서만 가능한 수업지도 방식이기때문인 것으로 보인다 반면 지난 3개년 동안 Humana 대학에서의 수강률은 감소한데다가 건물

GRE AWA John박 박정어학원

유지비도 올랐다 따라서 Humana대학의 수강을 늘리고 예산손실을 회복하기 위해서는 Omni 대학에서 취한 조치와 같은 능동적인 프로그램을 추진해야 한다

결론 we should initiate and actively promote long-distance degree programs like those at Omni 반박 원거리 학생 취득 프로그램 숫자가 증가한거하고 예산이 줄어드는 것 사이에 연관이 약하다 (causal 학생의 증가로 관리비용 증가할수 있음 원거리 수업가능 장비도입에의한 비용발생)bad analogy(omni university 하고 같은 조건이 아니다 )-gt omni college 가 강좌내용이 좋아서 학생의 등록이 많을수 있다 Humana 대학에서 만들었다 하더라도 인기 없을수 있음다른 요인에 의해서 Humana 대학의 수강 인원이 증가할수 있음(비록 과거엔 인기가 없었을지라도)

The following appeared as part of a business plan developed by the manager of the Rialto Movie Theater

Despite its downtown location the Rialto Movie Theater a local institution for five decades must make big changes or close its doors forever It should follow the example of the new Apex Theater in the mall outside of town When the Apex opened last year it featured a video arcade plush carpeting and seats and a state-of-the-art sound system Furthermore in a recent survey over 85 percent of respondents reported that the high price of newly released movies prevents them from going to the movies more than five times per year Thus if the Rialto intends to hold on to its share of a decreasing pool of moviegoers it must offer the same features as Apex

Write a response in which you discuss what questions would need to be answered in order to decide whether the recommendation is likely to have the predicted result Be sure to explain how the answers to these questions would help to evaluate the recommendation

Before following through this business plan the manager should investigate the cause of Rialtorsquos unsuccessful business

The author provides no evidence that the surveyrsquos results are statistically reliable The surveyrsquos sample of 85 percent must be sufficient in size and representative of overall population of the city where Rialto and Apex is serving Lacking evidence of a sufficiently representative sample the author cannot justifiably rely on the survey to draw any conclusion whatsoever The author does not indicate that Apex is indeed currently successful However even if Apex is enjoying success the argument relies on what might be a false analogy between Rialto and Apex In order for Apex to serve as a model that Rialto should emulate the author must assume that all relevant circumstances are essentially the same However this assumption is unwarranted For example the argument overlooks the face that Apex is located in a strategic placemdashbeside a mall where customers can not only watch a movie but also enjoy shopping Therefore simply changing the facility to that of Apex may not lead to success

The author does not mention whether Apex is successful or not Nevertheless even if Apex is currently successful the argument relies on what might be a false analogy between Rialto and Apex In order for Apex to serve as a model that Rialto should emulate the author must assume that all relevant circumstances are essentially the same However this assumption is unwarranted For example the argument overlooks the fact that these two institutions are located in different locations Rialto in downtown and Apex in a mall outside of town Although Apex opened with state-of-the-art facilities the decisive factor in its success could be due to its strategic location of being in a mall People could enjoy both shopping and movies at one location thus they may prefer Apex over Rialto Furthermore the place where people enjoy leisure activities has shifted in the past decades for most cities from downtown to the suburbs Therefore Rialto may not be successful even if it emulates Apexrsquos facilities A better business plan may be relocating Apex to the thriving section of the downtown

Rialto 극장은 지난 50여년간 지역 회관으로써 시내에 위치해 있으면서도 이제 변화를 꾀하지 않으면 문을 닫을

GRE AWA John박 박정어학원

판이다 이 극장은 시외 쇼핑타운에 새로 들어선 Apex 극장의 사례를 본받아야 했다 Apex가 지난해 개업했을 당시 이 극장은 비디오 아케이드 플러쉬 카펫트 바닥과 좌석 그리고 최신 음향시설을 갖추었다 더군다나 최근 조사에서는 응답자의 85 이상이 새로 출시된 영화 입장료가 비싼 탓으로 지난해보다 5배이상의 관람객이 줄어들었다고 나타났다 따라서 Rialto 극장이 줄어들고 있는 관람객을 뺐기지 않고 유지하려면 Apex와 같은 시설들을 갖추어야 할 것이다주장 리알토 극장이 줄어들고 있는 관람객을 뺐기지 않고 유지하려면 Apex와 같은 시설들을 갖추어야 할 것이다

1 조사에서 응답자가 전체를 대표할 수 없다 2 apex 극장이 좋은 시설을 갖추고 있지만 그로 인해 수익이 많이 발생했다는 말이 없으므로 시설투자를

하고도 좋은 결과를 얻을 수 있을지 그 근거가 미흡하다3 좋은 영화가 출시된다면 입장료가 비싸도 영화관에서 꼭 보려고 할 수 있다 4 rialto 가 시설이 아닌 다른 요인에 의해 장사가 안될수도 있다( 우범 지역이라든지)

The following is a recommendation from the business manager of Monarch Books

Since its opening in Collegeville twenty years ago Monarch Books has developed a large customer base due to its reader-friendly atmosphere and wide selection of books on all subjects Last month Book and Bean a combination bookstore and coffee shop announced its intention to open a Collegeville store Monarch Books should open its own in-store cafeacute in the space currently devoted to childrens books Given recent national census data indicating a significant decline in the percentage of the population under age ten sales of childrens books are likely to decline By replacing its childrens books section with a cafeacute Monarch Books can increase profits and ward off competition from Book and Bean

Write a response in which you examine the stated andor unstated assumptions of the argument Be sure to explain how the argument depends on these assumptions and what the implications are for the argument if the assumptions prove unwarranted

The following is a recommendation from the business manager of Monarch Books

Since its opening in Collegeville twenty years ago Monarch Books has developed a large customer base due to its reader-friendly atmosphere and wide selection of books on all subjects Last month Book and Bean a combination bookstore and coffee shop announced its intention to open a Collegeville store Monarch Books should open its own in-store cafeacute in the space currently devoted to childrens books Given recent national census data indicating a significant decline in the percentage of the population under age ten sales of childrens books are likely to decline By replacing its childrens books section with a cafeacute Monarch Books can increase profits and ward off competition from Book and Bean

1 Write a response in which you discuss what questions would need to be answered in order to decide whether the recommendation is likely to have the predicted result Be sure to explain how the answers to these questions would help to evaluate the recommendation

2 Write a response in which you discuss what specific evidence is needed to evaluate the argument and explain how the evidence would weaken or strengthen the argument

No evidence regarding Monarch Bookrsquos successEven if Regal Bookrsquos is successful this may not be attributable to the cafeacute False analogy Emulating may not lead to success Other factors may be involvedInsufficient condition The national census is not enough evidence that childrenrsquos book sales will decline Can

GRE AWA John박 박정어학원

the national census represent the local child populationDid opening a cafeacute boost sales for Regal Books Even assuming Regal is successful by opening a cafeacute this may not be suitable for Monarch which plans to close the childrenrsquos book section to establish a cafe Imprecise language ldquorelatively little spacerdquo how smallThe managerrsquos recommendation contradicts what he says Since Monarch is popular for its wide selection of books closing a selection which targets a major group of readers may hurt Monarchrsquos salesIs this the best way to compete

When Stanley Park first opened it was the largest most heavily used public park in town It is still the largest park but it is no longer heavily used Video cameras mounted in the parks parking lots last month revealed the parks drop in popularity the recordings showed an average of only 50 cars per day In contrast tiny Carlton Park in the heart of the business district is visited by more than 150 people on a typical weekday An obvious difference is that Carlton Park unlike Stanley Park provides ample seating Thus if Stanley Park is ever to be as popular with our citizens as Carlton Park the town will obviously need to provide more benches thereby converting some of the unused open areas into spaces suitable for socializing

Write a response in which you examine the stated andor unstated assumptions of the argument Be sure to explain how the argument depends on these assumptions and what the implications are for the argument if the assumptions prove unwarranted

Stanley 파크가 처음 개장했을 당시 가장 크고 가장 많이 이용되는 공원이었다 아직도 공원중에서는 가장 크지만 이용률은 상당히 떨어졌다 지난달 공원 주차장에 설치해놓은 비디오 카메라를 통해 보면 drop(주차장으로 여겨짐) 이용률이 가장 높았다 수치상으로는 하루 평균 50대의 차량만이 이용하였다 반면 직장 중심거리에 위치한 작은 규모의 Carlton 파크는 주당 무려 150여명 이상이 이용하고 있다 Stanley 파크와는 달리 Carlton 파크에는 의자가 있다는 것이 가장 뚜렷한 차이점이다 따라서 Stanley 파크가 Carlton 파크처럼 시민들이 자주 이용하는 공원이 되기 위해서는 벤치를 설치할 필요가 있으며 이렇게 사용되지 않는 일부 공간을 활용해서 사교를 위한 공간으로 바꾸어야 한다 ===gtdrop 에 대한 첨부사항 (영영사전내용입니다)---- a place or central depository to which something (as mail money or stolen property) is brought for distribution or transmission also the act of depositing something at such a place dropgt

주장 if Stanley Park is ever to be as popular with our citizens as is Carlton Park the town will obviously need to provide more benches thereby converting some of the unused open areas into spaces suitable for socializing1 조사가 언제 이루어진 것인가 조사가 언제 실시되었느냐에 따라 결과가 다를 수있다 현재는 다시 스탠리 파크가 늘어났었을 수 있다 2 벤치를 많이 설치했다고 해서 많은 관광객이 오지 않을수 있다(사람들이 벤치나 사교 공간을 원한다는 어떠한 자료도 없다)3스탠리 파크 주변에 교통 상황이 악화가 되었거나 칼튼 파크에서 문화행사등을 많이 가져서 이용객이 줄어든것일 수도 있다 4 칼튼 파크가 중심지에 있어서 접근성이 좋을수 있다5 조사가 같은 시간을 기준으로 한게 아니다(하나는 주중이고 하나는 주말이다)6사람의 수와 차의 대수를 같은것으로 비교할수 없다 (차안에 몇 명이 타고 있는지 모르고 대중교통을 이용해서 왔을수도 있다)

Page 30: GRE writing argument brain storm

GRE AWA John박 박정어학원

Old Dairy could change their products and manufacture low fat dairy foodsLess competing companies Old Dairy could eventually be the only company that produces hellipImprecise numbers and measurementsCustomers may still buy high fat dairy products

The author of the newsletter is offering potentially dangerous advice by recommending Old Dairy stockholders to withdraw investment and stop purchase What is more the authorrsquos prediction debases the reputation and business of Old Dairy and if false could devoid the investment opportunity of the newsletter readers Therefore investors should examine whether the authorrsquos evidences are substantial

To begin with the author states that 80 percent of the respondents in a survey indicated a desire to reduce their intake of foods He therefore argues that Old Dairyrsquos high fat and cholesterol products would decrease in sales However the author makes a crucial error in this argument First the author provides no evidence that the surveyrsquos results are statistically reliable Were they representative of all the customers Were they chosen for the survey randomly Furthermore the desire to reduce fat and cholesterol intake is a pervasive trend in todayrsquos opulent society however the author erroneously identifies this as a new phenomenon which will affect consumer trends Second having a desire to reduce fat and cholesterol intake does not necessarily indicate that people who have this desire will actually reduce consuming these types of products It is entirely possible that they may continue buying Old Dairy products for its quality and taste Accordingly the author cannot draw any firm conclusion that people will not buy Old Dairy products Therefore if any of these cases are true the author may be offering investors a detrimental investment advice

최근 조사에 대한 응답자중 80 이상이 자신이 먹는 음식에서 지방과 콜레스테롤의 함유량을 줄이고 싶다고 한다 아울러 요즘은 많은 식료품 가계에서 저지방 제품들을 많이 취급하고 있다 현재 Old Dairy Industries가 판매하고 있는 많은 음식제품들은 지방과 콜레스테롤이 높기 때문에 이 회사의 매출이 격감할 것으로 보이며 당연히 매출이익도 줄어들것이다 따라서 이 회사의 주주들은 주식을 매각하고 다른 주식 투자가들도 이 회사의 주식을 매입하지 않는 것이 좋다

결론 Old Dairy stockholders to sell their shares and other investors not to purchase stock in this company

반박 모든 상품이 다 고 지방 고 칼로리는 아니다(비록 많을지라도) 일부의 식품의 경우 기호에 맞어서 히트해서 전체적인 수입이 증가할 수도 있다국내시장만 생각할 수 없다( 외국시장에서 호황을 누릴수 있다 )입맛이라는게 즉각 바뀌는게 아니다

The following appeared in a letter to the editor of the Balmer Island Gazette

On Balmer Island where mopeds serve as a popular form of transportation the population increases to 100000 during the summer months To reduce the number of accidents involving mopeds and pedestrians the town council of Balmer Island should limit the number of mopeds rented by the islands moped rental companies from 50 per day to 25 per day during the summer season By limiting the number of rentals the town council will attain the 50 percent annual reduction in moped accidents that was achieved last year on the neighboring island of Seaville when Seavilles town council enforced similar limits on moped rentals

1 Write a response in which you discuss what questions would need to be answered in order to decide whether the recommendation is likely to have the predicted result Be sure to explain how the answers to these questions would help to evaluate the recommendation

2 Write a response in which you discuss what questions would need to be answered in order to decide whether the prediction and the argument on which it is based are reasonable Be sure to explain how the answers to these questions would help to evaluate the prediction

3 Write a response in which you examine the stated andor unstated assumptions of the argument Be sure to explain how the argument depends on these assumptions and what the implications are for the argument if the assumptions prove unwarranted

Whatrsquos the actual population of Balmer Island 100000mdashis this a significant increase What kind of accidents Skin abrasions or serious injury And compared to Seaville how serious are the accidents and the actual number of accidents Did Seaville enforce other restrictions like safety signsHow different are the conditions of Balmer

GRE AWA John박 박정어학원

and Seaville regarding population road (safety) conditions topography other town-government regulation How much will the economy of Balmer be affected do to this restriction Could it cause an economic recession due to the fact that these rental companiesrsquo chance to make money is only during the summer thereby weakening the economic infrastructure Are there any other ways that could better alleviate the accident rate

Statistics 50-impreciseAnalogy Balmer compared with TorseauFalse Cause Accidents might have occurred because of reasons other than mopeds False Cause population increase may not be part of the cause of the accidentsOther explanations for the accident pedestrians few road safety regulations narrow roadsThere could be other better solutionshellip

Balmer Island의 인구가 여름철에는 십만명으로 늘어난다 2륜차와 보행자간 사고를 줄이기 위해 시의회는 6개의 자전거를 포함한 2륜차 대여업체에게 이 기간동안에는 대여숫자를 일일 50에서 30으로 제한하도록 할 것이다 대여숫자를 줄임으로써 시 의회는 지난해 이웃한 Torseau섬에서 이와 동일한 규제를 시행해서 50나 줄인 결과를 보고 마찬가지로 50를 줄일수 있다고 확신하고 있다

결론 The town council of Balmer Island should linit the number

반박 보행자의 부실에 의해서 사고가 많이 일어날수도 있다렌탈수의 줄임만이 대책은 아니다(대부분의 사람들이 렌탈 보다는 소유하고 있을 수도 있다)옆섬과는 상황이 다를수도 있다(그 섬에서는 사고의 원인이 많은 자전거 수로 인한것일수있다) 하지만 이 섬은 좁은 도로가 원인일 수도 있고 도로 안전 장치의미비가 원일일수 있다

In this letter the author recommends that Balmer Island should limit the number moped rentals from 50 to 30 per day To support this recommendation the author points out several reasons However careful scrutiny of each of the facts reveals that it is filled with unanswered questions that could significantly weaken the authorrsquos recommendation with loops and holes which are answered

The recommendation depends on the assumption that no alternative means of reducing the number of accidents are available However the author fails to offer any evidence to substantiate this crucial assumption It is highly possible that means other than this would better solve the problem Perhaps they could widen the roads or put-up more safety signs Or perhaps the accidents were due to the lack of skills in which case proper safety training would significantly alleviate the problem Without considering and ruling out these and other alternative means of reducing accidetns the author cannot confidently conclude that merely emulating Torseau would suffice Moreover the author is advising a recommendation which could potentially harm the economy of Balmer Island sincehellip Moreover the Balmer Island should alternative means to reduce accidents because limiting moped rentals during the summer could harm the economy of Balmerhellip

First of all the author believes that increase in population and the number of moped rentals are responsible for the accidents It is entirely possible that other factors are responsible for the accidents Perhaps Balmer Islandrsquos lack of safety signs was a major factor Or maybe the roads are narrow and dangerous on the Island therefore the town council could enforce stricter traffic regulations to alleviate the problem Accordingly if either of these scenarios is true the author cannot draw any firm conclusion that increase in the number of population and moped rentals are the cause of the accidents

The author of this editorial recommends that to reduce accidents involving mopeds andpedestrians Balmer Islands city council should restrict moped rentals to 30 per day down from50 at each of the islands six rental outlets To support this recommendation the author citesthe fact that last year when nearby Torseau Islands town council enforced similar measuresTorseaus rate of moped accidents fell by 50 For several reasons this evidence providesscant support for the authors recommendationTo begin with the author assumes that all other conditions in Balmer that might affect therate of moped-pedestrian accidents will remain unchanged after the restrictions are enactedHowever with a restricted supply of rental mopeds people in Balmer might purchase mopedsinstead Also the number of pedestrians might increase in the future with more pedestriansespecially tourists the risk of moped-pedestrian accidents would probably increase For thatmatter the number of rental outlets might increase to make up for the artificial supplyrestriction per outlet--a likely scenario assuming moped rental demand does not declineWithout considering and ruling out these and other possible changes that might contribute to ahigh incidence of moped-pedestrian accidents the author cannot convince me that theproposed restrictions will necessarily have the desired effect

GRE AWA John박 박정어학원

Next the author fails to consider other possible explanations for the 50 decline inTorseaus moped accident rate last year Perhaps last year Torseau experienced unusually fairweather during which moped accidents are less likely Perhaps fewer tourists visited Tot seanlast year than during most years thereby diminishing the demand for rental mopeds to belowthe allowed limits Perhaps last year some of Torseaus moped rental outlets purchased newmopeds that are safer to drive Or perhaps the restrictions were already in effect but were notenforced until last year In any event a decline in Torseaus moped accident rate during onlyone year is scarcely sufficient to draw any reliable conclusions about what might have causedthe decline or about what the accident rate will be in years aheadAdditionally in asserting that the same phenomenon that caused a 50 decline in mopedaccidents in Torseau would cause a similar decline in Balmer the author relies on what mightamount to an unfair analogy between Balmer and Torseau Perhaps Balmers ability to enforcemoped-rental restrictions does not meet Torseaus ability if not then the mere enactment ofsimilar restrictions in Balmer is no guarantee of a similar result Or perhaps the demand formopeds in Torseau is always greater than in Balmer Specifically if fewer than all availablemopeds are currently rented per day from the average Balmer outlet while in Torseau everyavailable moped is rented each day then the proposed restriction is likely to have less impacton the accident rate in Balmer than in TorseauFinally the author provides no evidence that the same restrictions that served to reduce theincidence of all moped accidents by 50 would also serve to reduce the incidence ofaccidents involving mopeds and pedestrians by 50 Lacking such evidence it is entirelypossible that the number of moped accidents not involving pedestrians decreased by a greaterpercentage while the number of moped-pedestrian accidents decreased by a smallerpercentage or even increased Since the author has not accounted for these possibilities theeditorials recommendation cannot be taken seriouslyIn conclusion the recommendation is not well supported To convince me that the proposedrestriction would achieve the desired outcome the author would have to assure me that nochanges serving to increase Balmers moped-pedestrian accident rate will occur in theforeseeable future The author must also provide dear evidence that last years decline inmoped accidents in Torseau was attributable primarily to its moped rental restrictions ratherthan to one or more other factors In order to better evaluate the recommendation I wouldneed more information comparing the supply of and demand for moped rentals on the twoislands I would also need to know the rate of mopedpedestrian accidents in Torseau both priorto and after the restrictions were enforced in TorseauThe following appeared in a magazine article about planning for retirement

Clearview should be a top choice for anyone seeking a place to retire because it has spectacular natural beauty and a consistent climate Another advantage is that housing costs in Clearview have fallen significantly during the past year and taxes remain lower than those in neighboring towns Moreover Clearviews mayor promises many new programs to improve schools streets and public services And best of all retirees in Clearview can also expect excellent health care as they grow older since the number of physicians in the area is far greater than the national average

Write a response in which you discuss what specific evidence is needed to evaluate the argument and explain how the evidence would weaken or strengthen the argument

-Natural beauty and consistent climate may not be the most wanted qualities-Housing costs could have lowered on a national level wealthy retirees may not care about costs-Taxes may be high compared to the nationrsquos average tax rate-What about other qualities of Clearview Crime rate what qualities would retirees want -If schools streets and public services need improvement then this is proof that the current condition of Clearview is low Or due to budgetary reasons the mayor may not follow-up on his promise because of lowered tax rate -Schools and people who are retired no relationship-Physicians What kind of physicians Number is irrelevant Are these physicians capable of addressing the illnesses of old people

This author argues that anyone seeking a place to retire should choose Clearview To supportthis argument the article cites Clearviews consistent climate and natural beauty its fallinghousing costs its low property taxes compared to nearby towns and the mayors promise toimprove schools streets and services The article also claims that retirees can expectexcellent health care because the number of physicians in Clearview greatly exceeds thenational average This argument is flawed in several critical respectsTo begin with although consistent climate and natural beauty might be attractive to manyretirees these features are probably not important to all retirees For many retirees it isprobably more important to live near relatives or even to enjoy changing seasons Thus I

GRE AWA John박 박정어학원

cannot accept the authors sweeping recommendation for all retirees on this basisAlso Clearviews declining housing costs do not necessarily make Clearview the best placeto retire for two reasons First despite the decline Clearviews housing costs might be highcompared to housing costs in other cities Secondly for wealthier retirees housing costs arenot likely to be a factor in choosing a place to retire Thus the mere fact that housing costshave been in decline lends scant support to the recommendationThe articles reliance on Clearviews property-tax rates is also problematic in two respectsFirst retirees obviously have innumerable choices about where to retire besides Clear viewand nearby towns Secondly for retirees who are well-off financially property taxes are notlikely to be an important concern in choosing a place to retire Thus it is unfair to infer fromClearviews property-tax rates that retirees would prefer ClearviewYet another problem with the argument involves the mayors promises In light of Clearviewslow property-tax rates whether the mayor can follow through on those promises is highlyquestionable Absent any explanation of how the city can spend more money in the areas citedwithout raising property taxes I simply cannot accept the editorials recommendation on thebasis of those promises Besides even if the city makes the improvements promised thoseimprovements--particular the ones to schools--would not necessarily be important to retireesFinally although the number of physicians in Clearview is relatively high the per capitanumber might be relatively low Moreover it would be fairer to compare this per capita numberwith the per capita number for other attractive retirement towns--rather than the nationalaverage After all retirees are likely to place a relatively heavy burden on health-careresources Besides the article provides no assurances that the number of physicians inClearview will remain high in the foreseeable futureIn conclusion the recommendation is poorly supported To strengthen it the author mustconvince me--perhaps by way of a reliable survey--that the key features that the vast majorityof retirees look for in choosing a place to live are consistent climate natural beauty and lowhousing costs The author must also provide better evidence that Clear views property taxesare lower than the those of cities in other areas The author must also explain how the city canmake its promised improvements without raising property taxes Finally to better assess theargument I would need to now how the per capita number of physicians in Clearview wouldcompare to the national average in the futureThe following appeared as a letter to the editor from a Central Plaza store owner

Over the past two years the number of shoppers in Central Plaza has been steadily decreasing while the popularity of skateboarding has increased dramatically Many Central Plaza store owners believe that the decrease in their business is due to the number of skateboard users in the plaza There has also been a dramatic increase in the amount of litter and vandalism throughout the plaza Thus we recommend that the city prohibit skateboarding in Central Plaza If skateboarding is prohibited here we predict that business in Central Plaza will return to its previously high levels

Write a response in which you discuss what questions would need to be answered in order to decide whether the recommendation is likely to have the predicted result Be sure to explain how the answers to these questions would help to evaluate the recommendation

Why two years ago What happened two years ago which started this declineIs the dramatic increase in the ldquopopularityrdquo of skateboarding the cause of the steady decline of shoppers Are there any malls nearby Were there any changes nearby which could affect the decline in customersmdasha big mall perhaps Could the decline be due to the shop ownersHow many skateboarders use the plazaWhere do they skateboardDo they shop and are they customersAre the increase in litter and vandalism due to skateboarders Could this be alleviated by installing CCTVs and hiring security

This editorial concludes that the city should ban skateboarding from its downtown CentralPlaza in order to attract visitors to that area to return the area to its former glory and to makeit a place where people can congregate for fun and relaxation To justify this conclusion theeditorial points out that skateboarders are nearly the only people one sees anymore at CentralPlaza and that the Plaza is littered and its property defaced The editorial also points out thatthe majority of downtown merchants support the skate boarding ban This argument is flawedin several critical respectsFirst the editorials author falsely assumes that a ban on skateboarding is both necessaryand sufficient to achieve the three stated objectives Perhaps the city can achieve thoseobjectives by other means as well--for example by creating a new mall that incorporates anattractive new skateboard park Even if banning skateboarders altogether is necessary to meetthe citys goals the author has not shown that this action by itself would suffice Assuming thatthe Plazas reputation is now tarnished restoring that reputation and in turn enticing peopleback to the Plaza might require additional measures--such as removing litter and graffiti

GRE AWA John박 박정어학원

promoting the Plaza to the public or enticing popular restaurant or retail chains to the PlazaSecondly the editorial assumes too hastily that the Plazas decline is attributable to theskateboarders--rather than to some other phenomenon Perhaps the Plazas primary appeal inits glory days had to do with particular shops or eateries which were eventually replaced byless appealing ones Or perhaps the crime rate in surrounding areas has risen dramatically forreasons unrelated to the skateboarders presence at the Plaza Without ruling out these andother alternative explanations for the Plazas decline the editorials author cannot convince methat a skateboard ban would reverse that declineThirdly the editorials author might be confusing cause with effect--by assuming that theskateboarders caused the abandonment of the Plaza rather than vice versa It is entirelypossible that skateboarders did not frequent the Plaza until it was largely abandoned--andbecause it had been abandoned In fact this scenario makes good sense since skateboardingis most enjoyable where there are few pedestrians or motorists to get in the wayFourth it is unreasonable to infer from the mere fact that most merchants favor the ban thatthe ban would be effective in achieving the citys objectives Admittedly perhaps thesemerchants would be more likely to help dean up the Plaza area and promote their businesseswere the city to act in accordance with their preference Yet lacking any supporting evidencethe author cannot convince me of this Thus the survey amounts to scant evidence at best thatthe proposed ban would carry the intended resultFinally the author recommends a course of action that might actually defeat the citysobjective of providing a fun and relaxing place for people to congregate In my experienceskateboarding contributes to an atmosphere of fun and relaxation for adults and children alikemore so than many other types of ambiance Without considering that continuing to allowskateboarding--or even encouraging this activity--might achieve the citys goal more effectivelythan banning the activity the author cannot convincingly conclude that the ban would be in thecitys best interestsIn sum the argument is a specious one To strengthen it the editorials author must providedear evidence that skateboarding and not some other factor is responsible for the conditionsmarking the Plazas decline The author must also convince me that no alternative means ofrestoring the Plaza are available to the city and that the proposed ban by itself would suffice toattract tourists and restore the Plaza to its former glory Finally to better assess the argument itwould be useful to know the circumstances under which the downtown merchants would bewilling to help the city achieve its objectives

6그룹 약한 비유 빈출

The following recommendation appeared in a memo from the mayor of the town of Hopewell

Two years ago the nearby town of Ocean View built a new municipal golf course and resort hotel During the past two years tourism in Ocean View has increased new businesses have opened there and Ocean Views tax revenues have risen by 30 percent Therefore the best way to improve Hopewells economymdashand generate additional tax revenuesmdashis to build a golf course and resort hotel similar to those in Ocean View

Write a response in which you examine the stated andor unstated assumptions of the argument Be sure to explain how the argument depends on these assumptions and what the implications are for the argument if the assumptions prove unwarranted

GRE AWA John박 박정어학원

Assumptions The author assumes that OVrsquos municipal golf course and resort hotel caused tourism new businesses and increased tax revenues There may be other reasons advertising promo He assumes that this will continueAssumes that Ocean View and Hopewell are similar in many waysmdashthe name suggests otherwise OV may have always been a tourist attractions for its beaches We need to know the topography

2년전 Ocean View 시는 시정 소유 골프 및 휴양지 호텔을 신축했다 그리고 지난 2년동안 이 시의 관광객이 증가했으며 새로운 사업들이 생겨났다 그에따라 시의 세수도 30나 증가했다 Hopewell의 경제를 향상시키고 아울러 세수를 늘릴 수 있는 가장 좋은 방법은 Ocean View에 세워진 것과 같은 골프 시설과 휴양지 호텔을 신축하는 것이다

1 다른 요인으로 관광 산업이 발전했을 수도 있다 문화 유적이 발견이 되었거나 도로의 정비등으로 여행자가 늘었을 수도 있다

2 관광 산업의증가가 늘어난 세수의 원인이 아니라 새로 유입된 인구의 증가나 다른 공장에서 발생한 것일 수 있다

3 2년동안 한참 골프가 붐을 이루었을 수 있다 경제상황이 나빠지거나 다른 레포츠가 인근 지역에 생겨난다면 골프하는 사람이 줄어들 수 있다

In this memo HopeweUs mayor recommends that in order to stimulate the towns economyand boost tax revenues HopeweU should build a new golf course and resort hotel just as thetown of Ocean View did two years ago To support this recommendation the mayor points outthat in Ocean View during the last two years tourism has increased new businesses haveopened and tax revenues have increased by 30 I find the mayors argument unconvincingin several important respectsFirst of all it is possible that the mayor has confused cause with effect respecting the recentdevelopments in Ocean View Perhaps Ocean Views construction of a new golf course andhotel was a response to previous increases in tourism and business development increasesthat have simply continued during the most recent two years Since the mayor has failed toaccount for this possibility the claim that Hopewell would boost its economy by alsoconstructing a golf course and hotel is completely unwarrantedSecondly the mayor fails to account for other possible causes of the trends in Ocean Viewduring the last two years The increase in tourism might have been due to improving economicconditions nationwide or to unusually pleasant weather in the region The new businessesthat have opened in Ocean View might have opened there irrespective of the new golf courseand hotel And the 30 increase in tax revenues might have been the result of an increase intax rates or the addition of a new type of municipal taxWithout ruling out these and other alternative explanations for the three recent trends inOcean View the mayor cannot reasonably infer based on those trends that Hopewellseconomy would benefit by following Ocean Views exampleThirdly even if the recent trends in Ocean View are attributable to the construction of the newgolf course and hotel there the mayor assumes too hastily that the golf course and hotel willcontinue to benefit that towns overall economy The mayor has not accounted for thepossibility that increased tourism will begin to drive residents away during tourist season orthat new business development will result in the towns losing its appeal as a place to visit or tolive Unless the mayor can convince me that these scenarios are unlikely I cannot accept themayors recommendation that Hopewell follow Ocean Views exampleFinally the mayors argument rests on the unsubstantiated assumption that Hopewell andOcean View are sufficiently alike in ways that might affect the economic impact of a new golfcourse and hotel Hopewell might lack the sort of natural environment that would attract moretourists and new businesses to the town--regardless of its new golf course and hotel For thatmatter perhaps Hopewell already contains several resort hotels and golf courses that are notutilized to their capacity If so building yet another golf course and hotel might amount to amisallocation of the towns resources--and actually harm the towns overall economyIn sum the mayors recommendation is not well supported To bolster it the mayor mustprovide better evidence that Ocean Views new golf course and hotel and not some otherphenomenon--has been responsible for boosting Ocean Views economy during the last twoyears To better assess the recommendation I would need to know why Ocean View decidedto construct its new golf course and hotel in the first place--specifically what events prior toconstruction might have prompted that decision I would also need to thoroughly compare

GRE AWA John박 박정어학원

HopeweU with Ocean View--especially in terms of their appeal to tourists and businesses--todetermine whether the same course of action that appears to have boosted Ocean Viewseconomy would also boost Hopewells economy

The following is part of a memorandum from the president of Humana University

Last year the number of students who enrolled in online degree programs offered by nearby Omni University increased by 50 percent During the same year Omni showed a significant decrease from prior years in expenditures for dormitory and classroom space most likely because instruction in the online programs takes place via the Internet In contrast over the past three years enrollment at Humana University has failed to grow and the cost of maintaining buildings has increased along with our budget deficit To address these problems Humana University will begin immediately to create and actively promote online degree programs like those at Omni We predict that instituting these online degree programs will help Humana both increase its total enrollment and solve its budget problems

Write a response in which you discuss what questions would need to be answered in order to decide whether the prediction and the argument on which it is based are reasonable Be sure to explain how the answers to these questions would help to evaluate the prediction

Is Omni University successful due to the online degree program 50 Is the decrease in expenditures for dormitory and classroom space due to the decrease in of on-campus students Which classes were successful Does HU have those classes

Even if the long-distance degree programs at Omni University benefited the school the presidentrsquos recommendation that Human College should emulate Omni University is too hasty First OUrsquos name implies that the school would have more majors than Humanahellip the president should examine which degrees were in the long-distance programhellip

지난해에는 Omni 대학에서 개강했던 원거리 학생 학점 취득 프로그램을 등록했던 학생들의 숫자가 50나 증가했다 같은해 기간동안 Omni 대학에서는 그 전년도부터 기숙사와 학급의 공간 확충을 위한 예산을 대폭 줄였는데 이는 이 원거리 학점 취득 프로그램이 양방향 비디오 컴퓨터 접속을 통해서만 가능한 수업지도 방식이기때문인 것으로 보인다 반면 지난 3개년 동안 Humana 대학에서의 수강률은 감소한데다가 건물

GRE AWA John박 박정어학원

유지비도 올랐다 따라서 Humana대학의 수강을 늘리고 예산손실을 회복하기 위해서는 Omni 대학에서 취한 조치와 같은 능동적인 프로그램을 추진해야 한다

결론 we should initiate and actively promote long-distance degree programs like those at Omni 반박 원거리 학생 취득 프로그램 숫자가 증가한거하고 예산이 줄어드는 것 사이에 연관이 약하다 (causal 학생의 증가로 관리비용 증가할수 있음 원거리 수업가능 장비도입에의한 비용발생)bad analogy(omni university 하고 같은 조건이 아니다 )-gt omni college 가 강좌내용이 좋아서 학생의 등록이 많을수 있다 Humana 대학에서 만들었다 하더라도 인기 없을수 있음다른 요인에 의해서 Humana 대학의 수강 인원이 증가할수 있음(비록 과거엔 인기가 없었을지라도)

The following appeared as part of a business plan developed by the manager of the Rialto Movie Theater

Despite its downtown location the Rialto Movie Theater a local institution for five decades must make big changes or close its doors forever It should follow the example of the new Apex Theater in the mall outside of town When the Apex opened last year it featured a video arcade plush carpeting and seats and a state-of-the-art sound system Furthermore in a recent survey over 85 percent of respondents reported that the high price of newly released movies prevents them from going to the movies more than five times per year Thus if the Rialto intends to hold on to its share of a decreasing pool of moviegoers it must offer the same features as Apex

Write a response in which you discuss what questions would need to be answered in order to decide whether the recommendation is likely to have the predicted result Be sure to explain how the answers to these questions would help to evaluate the recommendation

Before following through this business plan the manager should investigate the cause of Rialtorsquos unsuccessful business

The author provides no evidence that the surveyrsquos results are statistically reliable The surveyrsquos sample of 85 percent must be sufficient in size and representative of overall population of the city where Rialto and Apex is serving Lacking evidence of a sufficiently representative sample the author cannot justifiably rely on the survey to draw any conclusion whatsoever The author does not indicate that Apex is indeed currently successful However even if Apex is enjoying success the argument relies on what might be a false analogy between Rialto and Apex In order for Apex to serve as a model that Rialto should emulate the author must assume that all relevant circumstances are essentially the same However this assumption is unwarranted For example the argument overlooks the face that Apex is located in a strategic placemdashbeside a mall where customers can not only watch a movie but also enjoy shopping Therefore simply changing the facility to that of Apex may not lead to success

The author does not mention whether Apex is successful or not Nevertheless even if Apex is currently successful the argument relies on what might be a false analogy between Rialto and Apex In order for Apex to serve as a model that Rialto should emulate the author must assume that all relevant circumstances are essentially the same However this assumption is unwarranted For example the argument overlooks the fact that these two institutions are located in different locations Rialto in downtown and Apex in a mall outside of town Although Apex opened with state-of-the-art facilities the decisive factor in its success could be due to its strategic location of being in a mall People could enjoy both shopping and movies at one location thus they may prefer Apex over Rialto Furthermore the place where people enjoy leisure activities has shifted in the past decades for most cities from downtown to the suburbs Therefore Rialto may not be successful even if it emulates Apexrsquos facilities A better business plan may be relocating Apex to the thriving section of the downtown

Rialto 극장은 지난 50여년간 지역 회관으로써 시내에 위치해 있으면서도 이제 변화를 꾀하지 않으면 문을 닫을

GRE AWA John박 박정어학원

판이다 이 극장은 시외 쇼핑타운에 새로 들어선 Apex 극장의 사례를 본받아야 했다 Apex가 지난해 개업했을 당시 이 극장은 비디오 아케이드 플러쉬 카펫트 바닥과 좌석 그리고 최신 음향시설을 갖추었다 더군다나 최근 조사에서는 응답자의 85 이상이 새로 출시된 영화 입장료가 비싼 탓으로 지난해보다 5배이상의 관람객이 줄어들었다고 나타났다 따라서 Rialto 극장이 줄어들고 있는 관람객을 뺐기지 않고 유지하려면 Apex와 같은 시설들을 갖추어야 할 것이다주장 리알토 극장이 줄어들고 있는 관람객을 뺐기지 않고 유지하려면 Apex와 같은 시설들을 갖추어야 할 것이다

1 조사에서 응답자가 전체를 대표할 수 없다 2 apex 극장이 좋은 시설을 갖추고 있지만 그로 인해 수익이 많이 발생했다는 말이 없으므로 시설투자를

하고도 좋은 결과를 얻을 수 있을지 그 근거가 미흡하다3 좋은 영화가 출시된다면 입장료가 비싸도 영화관에서 꼭 보려고 할 수 있다 4 rialto 가 시설이 아닌 다른 요인에 의해 장사가 안될수도 있다( 우범 지역이라든지)

The following is a recommendation from the business manager of Monarch Books

Since its opening in Collegeville twenty years ago Monarch Books has developed a large customer base due to its reader-friendly atmosphere and wide selection of books on all subjects Last month Book and Bean a combination bookstore and coffee shop announced its intention to open a Collegeville store Monarch Books should open its own in-store cafeacute in the space currently devoted to childrens books Given recent national census data indicating a significant decline in the percentage of the population under age ten sales of childrens books are likely to decline By replacing its childrens books section with a cafeacute Monarch Books can increase profits and ward off competition from Book and Bean

Write a response in which you examine the stated andor unstated assumptions of the argument Be sure to explain how the argument depends on these assumptions and what the implications are for the argument if the assumptions prove unwarranted

The following is a recommendation from the business manager of Monarch Books

Since its opening in Collegeville twenty years ago Monarch Books has developed a large customer base due to its reader-friendly atmosphere and wide selection of books on all subjects Last month Book and Bean a combination bookstore and coffee shop announced its intention to open a Collegeville store Monarch Books should open its own in-store cafeacute in the space currently devoted to childrens books Given recent national census data indicating a significant decline in the percentage of the population under age ten sales of childrens books are likely to decline By replacing its childrens books section with a cafeacute Monarch Books can increase profits and ward off competition from Book and Bean

1 Write a response in which you discuss what questions would need to be answered in order to decide whether the recommendation is likely to have the predicted result Be sure to explain how the answers to these questions would help to evaluate the recommendation

2 Write a response in which you discuss what specific evidence is needed to evaluate the argument and explain how the evidence would weaken or strengthen the argument

No evidence regarding Monarch Bookrsquos successEven if Regal Bookrsquos is successful this may not be attributable to the cafeacute False analogy Emulating may not lead to success Other factors may be involvedInsufficient condition The national census is not enough evidence that childrenrsquos book sales will decline Can

GRE AWA John박 박정어학원

the national census represent the local child populationDid opening a cafeacute boost sales for Regal Books Even assuming Regal is successful by opening a cafeacute this may not be suitable for Monarch which plans to close the childrenrsquos book section to establish a cafe Imprecise language ldquorelatively little spacerdquo how smallThe managerrsquos recommendation contradicts what he says Since Monarch is popular for its wide selection of books closing a selection which targets a major group of readers may hurt Monarchrsquos salesIs this the best way to compete

When Stanley Park first opened it was the largest most heavily used public park in town It is still the largest park but it is no longer heavily used Video cameras mounted in the parks parking lots last month revealed the parks drop in popularity the recordings showed an average of only 50 cars per day In contrast tiny Carlton Park in the heart of the business district is visited by more than 150 people on a typical weekday An obvious difference is that Carlton Park unlike Stanley Park provides ample seating Thus if Stanley Park is ever to be as popular with our citizens as Carlton Park the town will obviously need to provide more benches thereby converting some of the unused open areas into spaces suitable for socializing

Write a response in which you examine the stated andor unstated assumptions of the argument Be sure to explain how the argument depends on these assumptions and what the implications are for the argument if the assumptions prove unwarranted

Stanley 파크가 처음 개장했을 당시 가장 크고 가장 많이 이용되는 공원이었다 아직도 공원중에서는 가장 크지만 이용률은 상당히 떨어졌다 지난달 공원 주차장에 설치해놓은 비디오 카메라를 통해 보면 drop(주차장으로 여겨짐) 이용률이 가장 높았다 수치상으로는 하루 평균 50대의 차량만이 이용하였다 반면 직장 중심거리에 위치한 작은 규모의 Carlton 파크는 주당 무려 150여명 이상이 이용하고 있다 Stanley 파크와는 달리 Carlton 파크에는 의자가 있다는 것이 가장 뚜렷한 차이점이다 따라서 Stanley 파크가 Carlton 파크처럼 시민들이 자주 이용하는 공원이 되기 위해서는 벤치를 설치할 필요가 있으며 이렇게 사용되지 않는 일부 공간을 활용해서 사교를 위한 공간으로 바꾸어야 한다 ===gtdrop 에 대한 첨부사항 (영영사전내용입니다)---- a place or central depository to which something (as mail money or stolen property) is brought for distribution or transmission also the act of depositing something at such a place dropgt

주장 if Stanley Park is ever to be as popular with our citizens as is Carlton Park the town will obviously need to provide more benches thereby converting some of the unused open areas into spaces suitable for socializing1 조사가 언제 이루어진 것인가 조사가 언제 실시되었느냐에 따라 결과가 다를 수있다 현재는 다시 스탠리 파크가 늘어났었을 수 있다 2 벤치를 많이 설치했다고 해서 많은 관광객이 오지 않을수 있다(사람들이 벤치나 사교 공간을 원한다는 어떠한 자료도 없다)3스탠리 파크 주변에 교통 상황이 악화가 되었거나 칼튼 파크에서 문화행사등을 많이 가져서 이용객이 줄어든것일 수도 있다 4 칼튼 파크가 중심지에 있어서 접근성이 좋을수 있다5 조사가 같은 시간을 기준으로 한게 아니다(하나는 주중이고 하나는 주말이다)6사람의 수와 차의 대수를 같은것으로 비교할수 없다 (차안에 몇 명이 타고 있는지 모르고 대중교통을 이용해서 왔을수도 있다)

Page 31: GRE writing argument brain storm

GRE AWA John박 박정어학원

and Seaville regarding population road (safety) conditions topography other town-government regulation How much will the economy of Balmer be affected do to this restriction Could it cause an economic recession due to the fact that these rental companiesrsquo chance to make money is only during the summer thereby weakening the economic infrastructure Are there any other ways that could better alleviate the accident rate

Statistics 50-impreciseAnalogy Balmer compared with TorseauFalse Cause Accidents might have occurred because of reasons other than mopeds False Cause population increase may not be part of the cause of the accidentsOther explanations for the accident pedestrians few road safety regulations narrow roadsThere could be other better solutionshellip

Balmer Island의 인구가 여름철에는 십만명으로 늘어난다 2륜차와 보행자간 사고를 줄이기 위해 시의회는 6개의 자전거를 포함한 2륜차 대여업체에게 이 기간동안에는 대여숫자를 일일 50에서 30으로 제한하도록 할 것이다 대여숫자를 줄임으로써 시 의회는 지난해 이웃한 Torseau섬에서 이와 동일한 규제를 시행해서 50나 줄인 결과를 보고 마찬가지로 50를 줄일수 있다고 확신하고 있다

결론 The town council of Balmer Island should linit the number

반박 보행자의 부실에 의해서 사고가 많이 일어날수도 있다렌탈수의 줄임만이 대책은 아니다(대부분의 사람들이 렌탈 보다는 소유하고 있을 수도 있다)옆섬과는 상황이 다를수도 있다(그 섬에서는 사고의 원인이 많은 자전거 수로 인한것일수있다) 하지만 이 섬은 좁은 도로가 원인일 수도 있고 도로 안전 장치의미비가 원일일수 있다

In this letter the author recommends that Balmer Island should limit the number moped rentals from 50 to 30 per day To support this recommendation the author points out several reasons However careful scrutiny of each of the facts reveals that it is filled with unanswered questions that could significantly weaken the authorrsquos recommendation with loops and holes which are answered

The recommendation depends on the assumption that no alternative means of reducing the number of accidents are available However the author fails to offer any evidence to substantiate this crucial assumption It is highly possible that means other than this would better solve the problem Perhaps they could widen the roads or put-up more safety signs Or perhaps the accidents were due to the lack of skills in which case proper safety training would significantly alleviate the problem Without considering and ruling out these and other alternative means of reducing accidetns the author cannot confidently conclude that merely emulating Torseau would suffice Moreover the author is advising a recommendation which could potentially harm the economy of Balmer Island sincehellip Moreover the Balmer Island should alternative means to reduce accidents because limiting moped rentals during the summer could harm the economy of Balmerhellip

First of all the author believes that increase in population and the number of moped rentals are responsible for the accidents It is entirely possible that other factors are responsible for the accidents Perhaps Balmer Islandrsquos lack of safety signs was a major factor Or maybe the roads are narrow and dangerous on the Island therefore the town council could enforce stricter traffic regulations to alleviate the problem Accordingly if either of these scenarios is true the author cannot draw any firm conclusion that increase in the number of population and moped rentals are the cause of the accidents

The author of this editorial recommends that to reduce accidents involving mopeds andpedestrians Balmer Islands city council should restrict moped rentals to 30 per day down from50 at each of the islands six rental outlets To support this recommendation the author citesthe fact that last year when nearby Torseau Islands town council enforced similar measuresTorseaus rate of moped accidents fell by 50 For several reasons this evidence providesscant support for the authors recommendationTo begin with the author assumes that all other conditions in Balmer that might affect therate of moped-pedestrian accidents will remain unchanged after the restrictions are enactedHowever with a restricted supply of rental mopeds people in Balmer might purchase mopedsinstead Also the number of pedestrians might increase in the future with more pedestriansespecially tourists the risk of moped-pedestrian accidents would probably increase For thatmatter the number of rental outlets might increase to make up for the artificial supplyrestriction per outlet--a likely scenario assuming moped rental demand does not declineWithout considering and ruling out these and other possible changes that might contribute to ahigh incidence of moped-pedestrian accidents the author cannot convince me that theproposed restrictions will necessarily have the desired effect

GRE AWA John박 박정어학원

Next the author fails to consider other possible explanations for the 50 decline inTorseaus moped accident rate last year Perhaps last year Torseau experienced unusually fairweather during which moped accidents are less likely Perhaps fewer tourists visited Tot seanlast year than during most years thereby diminishing the demand for rental mopeds to belowthe allowed limits Perhaps last year some of Torseaus moped rental outlets purchased newmopeds that are safer to drive Or perhaps the restrictions were already in effect but were notenforced until last year In any event a decline in Torseaus moped accident rate during onlyone year is scarcely sufficient to draw any reliable conclusions about what might have causedthe decline or about what the accident rate will be in years aheadAdditionally in asserting that the same phenomenon that caused a 50 decline in mopedaccidents in Torseau would cause a similar decline in Balmer the author relies on what mightamount to an unfair analogy between Balmer and Torseau Perhaps Balmers ability to enforcemoped-rental restrictions does not meet Torseaus ability if not then the mere enactment ofsimilar restrictions in Balmer is no guarantee of a similar result Or perhaps the demand formopeds in Torseau is always greater than in Balmer Specifically if fewer than all availablemopeds are currently rented per day from the average Balmer outlet while in Torseau everyavailable moped is rented each day then the proposed restriction is likely to have less impacton the accident rate in Balmer than in TorseauFinally the author provides no evidence that the same restrictions that served to reduce theincidence of all moped accidents by 50 would also serve to reduce the incidence ofaccidents involving mopeds and pedestrians by 50 Lacking such evidence it is entirelypossible that the number of moped accidents not involving pedestrians decreased by a greaterpercentage while the number of moped-pedestrian accidents decreased by a smallerpercentage or even increased Since the author has not accounted for these possibilities theeditorials recommendation cannot be taken seriouslyIn conclusion the recommendation is not well supported To convince me that the proposedrestriction would achieve the desired outcome the author would have to assure me that nochanges serving to increase Balmers moped-pedestrian accident rate will occur in theforeseeable future The author must also provide dear evidence that last years decline inmoped accidents in Torseau was attributable primarily to its moped rental restrictions ratherthan to one or more other factors In order to better evaluate the recommendation I wouldneed more information comparing the supply of and demand for moped rentals on the twoislands I would also need to know the rate of mopedpedestrian accidents in Torseau both priorto and after the restrictions were enforced in TorseauThe following appeared in a magazine article about planning for retirement

Clearview should be a top choice for anyone seeking a place to retire because it has spectacular natural beauty and a consistent climate Another advantage is that housing costs in Clearview have fallen significantly during the past year and taxes remain lower than those in neighboring towns Moreover Clearviews mayor promises many new programs to improve schools streets and public services And best of all retirees in Clearview can also expect excellent health care as they grow older since the number of physicians in the area is far greater than the national average

Write a response in which you discuss what specific evidence is needed to evaluate the argument and explain how the evidence would weaken or strengthen the argument

-Natural beauty and consistent climate may not be the most wanted qualities-Housing costs could have lowered on a national level wealthy retirees may not care about costs-Taxes may be high compared to the nationrsquos average tax rate-What about other qualities of Clearview Crime rate what qualities would retirees want -If schools streets and public services need improvement then this is proof that the current condition of Clearview is low Or due to budgetary reasons the mayor may not follow-up on his promise because of lowered tax rate -Schools and people who are retired no relationship-Physicians What kind of physicians Number is irrelevant Are these physicians capable of addressing the illnesses of old people

This author argues that anyone seeking a place to retire should choose Clearview To supportthis argument the article cites Clearviews consistent climate and natural beauty its fallinghousing costs its low property taxes compared to nearby towns and the mayors promise toimprove schools streets and services The article also claims that retirees can expectexcellent health care because the number of physicians in Clearview greatly exceeds thenational average This argument is flawed in several critical respectsTo begin with although consistent climate and natural beauty might be attractive to manyretirees these features are probably not important to all retirees For many retirees it isprobably more important to live near relatives or even to enjoy changing seasons Thus I

GRE AWA John박 박정어학원

cannot accept the authors sweeping recommendation for all retirees on this basisAlso Clearviews declining housing costs do not necessarily make Clearview the best placeto retire for two reasons First despite the decline Clearviews housing costs might be highcompared to housing costs in other cities Secondly for wealthier retirees housing costs arenot likely to be a factor in choosing a place to retire Thus the mere fact that housing costshave been in decline lends scant support to the recommendationThe articles reliance on Clearviews property-tax rates is also problematic in two respectsFirst retirees obviously have innumerable choices about where to retire besides Clear viewand nearby towns Secondly for retirees who are well-off financially property taxes are notlikely to be an important concern in choosing a place to retire Thus it is unfair to infer fromClearviews property-tax rates that retirees would prefer ClearviewYet another problem with the argument involves the mayors promises In light of Clearviewslow property-tax rates whether the mayor can follow through on those promises is highlyquestionable Absent any explanation of how the city can spend more money in the areas citedwithout raising property taxes I simply cannot accept the editorials recommendation on thebasis of those promises Besides even if the city makes the improvements promised thoseimprovements--particular the ones to schools--would not necessarily be important to retireesFinally although the number of physicians in Clearview is relatively high the per capitanumber might be relatively low Moreover it would be fairer to compare this per capita numberwith the per capita number for other attractive retirement towns--rather than the nationalaverage After all retirees are likely to place a relatively heavy burden on health-careresources Besides the article provides no assurances that the number of physicians inClearview will remain high in the foreseeable futureIn conclusion the recommendation is poorly supported To strengthen it the author mustconvince me--perhaps by way of a reliable survey--that the key features that the vast majorityof retirees look for in choosing a place to live are consistent climate natural beauty and lowhousing costs The author must also provide better evidence that Clear views property taxesare lower than the those of cities in other areas The author must also explain how the city canmake its promised improvements without raising property taxes Finally to better assess theargument I would need to now how the per capita number of physicians in Clearview wouldcompare to the national average in the futureThe following appeared as a letter to the editor from a Central Plaza store owner

Over the past two years the number of shoppers in Central Plaza has been steadily decreasing while the popularity of skateboarding has increased dramatically Many Central Plaza store owners believe that the decrease in their business is due to the number of skateboard users in the plaza There has also been a dramatic increase in the amount of litter and vandalism throughout the plaza Thus we recommend that the city prohibit skateboarding in Central Plaza If skateboarding is prohibited here we predict that business in Central Plaza will return to its previously high levels

Write a response in which you discuss what questions would need to be answered in order to decide whether the recommendation is likely to have the predicted result Be sure to explain how the answers to these questions would help to evaluate the recommendation

Why two years ago What happened two years ago which started this declineIs the dramatic increase in the ldquopopularityrdquo of skateboarding the cause of the steady decline of shoppers Are there any malls nearby Were there any changes nearby which could affect the decline in customersmdasha big mall perhaps Could the decline be due to the shop ownersHow many skateboarders use the plazaWhere do they skateboardDo they shop and are they customersAre the increase in litter and vandalism due to skateboarders Could this be alleviated by installing CCTVs and hiring security

This editorial concludes that the city should ban skateboarding from its downtown CentralPlaza in order to attract visitors to that area to return the area to its former glory and to makeit a place where people can congregate for fun and relaxation To justify this conclusion theeditorial points out that skateboarders are nearly the only people one sees anymore at CentralPlaza and that the Plaza is littered and its property defaced The editorial also points out thatthe majority of downtown merchants support the skate boarding ban This argument is flawedin several critical respectsFirst the editorials author falsely assumes that a ban on skateboarding is both necessaryand sufficient to achieve the three stated objectives Perhaps the city can achieve thoseobjectives by other means as well--for example by creating a new mall that incorporates anattractive new skateboard park Even if banning skateboarders altogether is necessary to meetthe citys goals the author has not shown that this action by itself would suffice Assuming thatthe Plazas reputation is now tarnished restoring that reputation and in turn enticing peopleback to the Plaza might require additional measures--such as removing litter and graffiti

GRE AWA John박 박정어학원

promoting the Plaza to the public or enticing popular restaurant or retail chains to the PlazaSecondly the editorial assumes too hastily that the Plazas decline is attributable to theskateboarders--rather than to some other phenomenon Perhaps the Plazas primary appeal inits glory days had to do with particular shops or eateries which were eventually replaced byless appealing ones Or perhaps the crime rate in surrounding areas has risen dramatically forreasons unrelated to the skateboarders presence at the Plaza Without ruling out these andother alternative explanations for the Plazas decline the editorials author cannot convince methat a skateboard ban would reverse that declineThirdly the editorials author might be confusing cause with effect--by assuming that theskateboarders caused the abandonment of the Plaza rather than vice versa It is entirelypossible that skateboarders did not frequent the Plaza until it was largely abandoned--andbecause it had been abandoned In fact this scenario makes good sense since skateboardingis most enjoyable where there are few pedestrians or motorists to get in the wayFourth it is unreasonable to infer from the mere fact that most merchants favor the ban thatthe ban would be effective in achieving the citys objectives Admittedly perhaps thesemerchants would be more likely to help dean up the Plaza area and promote their businesseswere the city to act in accordance with their preference Yet lacking any supporting evidencethe author cannot convince me of this Thus the survey amounts to scant evidence at best thatthe proposed ban would carry the intended resultFinally the author recommends a course of action that might actually defeat the citysobjective of providing a fun and relaxing place for people to congregate In my experienceskateboarding contributes to an atmosphere of fun and relaxation for adults and children alikemore so than many other types of ambiance Without considering that continuing to allowskateboarding--or even encouraging this activity--might achieve the citys goal more effectivelythan banning the activity the author cannot convincingly conclude that the ban would be in thecitys best interestsIn sum the argument is a specious one To strengthen it the editorials author must providedear evidence that skateboarding and not some other factor is responsible for the conditionsmarking the Plazas decline The author must also convince me that no alternative means ofrestoring the Plaza are available to the city and that the proposed ban by itself would suffice toattract tourists and restore the Plaza to its former glory Finally to better assess the argument itwould be useful to know the circumstances under which the downtown merchants would bewilling to help the city achieve its objectives

6그룹 약한 비유 빈출

The following recommendation appeared in a memo from the mayor of the town of Hopewell

Two years ago the nearby town of Ocean View built a new municipal golf course and resort hotel During the past two years tourism in Ocean View has increased new businesses have opened there and Ocean Views tax revenues have risen by 30 percent Therefore the best way to improve Hopewells economymdashand generate additional tax revenuesmdashis to build a golf course and resort hotel similar to those in Ocean View

Write a response in which you examine the stated andor unstated assumptions of the argument Be sure to explain how the argument depends on these assumptions and what the implications are for the argument if the assumptions prove unwarranted

GRE AWA John박 박정어학원

Assumptions The author assumes that OVrsquos municipal golf course and resort hotel caused tourism new businesses and increased tax revenues There may be other reasons advertising promo He assumes that this will continueAssumes that Ocean View and Hopewell are similar in many waysmdashthe name suggests otherwise OV may have always been a tourist attractions for its beaches We need to know the topography

2년전 Ocean View 시는 시정 소유 골프 및 휴양지 호텔을 신축했다 그리고 지난 2년동안 이 시의 관광객이 증가했으며 새로운 사업들이 생겨났다 그에따라 시의 세수도 30나 증가했다 Hopewell의 경제를 향상시키고 아울러 세수를 늘릴 수 있는 가장 좋은 방법은 Ocean View에 세워진 것과 같은 골프 시설과 휴양지 호텔을 신축하는 것이다

1 다른 요인으로 관광 산업이 발전했을 수도 있다 문화 유적이 발견이 되었거나 도로의 정비등으로 여행자가 늘었을 수도 있다

2 관광 산업의증가가 늘어난 세수의 원인이 아니라 새로 유입된 인구의 증가나 다른 공장에서 발생한 것일 수 있다

3 2년동안 한참 골프가 붐을 이루었을 수 있다 경제상황이 나빠지거나 다른 레포츠가 인근 지역에 생겨난다면 골프하는 사람이 줄어들 수 있다

In this memo HopeweUs mayor recommends that in order to stimulate the towns economyand boost tax revenues HopeweU should build a new golf course and resort hotel just as thetown of Ocean View did two years ago To support this recommendation the mayor points outthat in Ocean View during the last two years tourism has increased new businesses haveopened and tax revenues have increased by 30 I find the mayors argument unconvincingin several important respectsFirst of all it is possible that the mayor has confused cause with effect respecting the recentdevelopments in Ocean View Perhaps Ocean Views construction of a new golf course andhotel was a response to previous increases in tourism and business development increasesthat have simply continued during the most recent two years Since the mayor has failed toaccount for this possibility the claim that Hopewell would boost its economy by alsoconstructing a golf course and hotel is completely unwarrantedSecondly the mayor fails to account for other possible causes of the trends in Ocean Viewduring the last two years The increase in tourism might have been due to improving economicconditions nationwide or to unusually pleasant weather in the region The new businessesthat have opened in Ocean View might have opened there irrespective of the new golf courseand hotel And the 30 increase in tax revenues might have been the result of an increase intax rates or the addition of a new type of municipal taxWithout ruling out these and other alternative explanations for the three recent trends inOcean View the mayor cannot reasonably infer based on those trends that Hopewellseconomy would benefit by following Ocean Views exampleThirdly even if the recent trends in Ocean View are attributable to the construction of the newgolf course and hotel there the mayor assumes too hastily that the golf course and hotel willcontinue to benefit that towns overall economy The mayor has not accounted for thepossibility that increased tourism will begin to drive residents away during tourist season orthat new business development will result in the towns losing its appeal as a place to visit or tolive Unless the mayor can convince me that these scenarios are unlikely I cannot accept themayors recommendation that Hopewell follow Ocean Views exampleFinally the mayors argument rests on the unsubstantiated assumption that Hopewell andOcean View are sufficiently alike in ways that might affect the economic impact of a new golfcourse and hotel Hopewell might lack the sort of natural environment that would attract moretourists and new businesses to the town--regardless of its new golf course and hotel For thatmatter perhaps Hopewell already contains several resort hotels and golf courses that are notutilized to their capacity If so building yet another golf course and hotel might amount to amisallocation of the towns resources--and actually harm the towns overall economyIn sum the mayors recommendation is not well supported To bolster it the mayor mustprovide better evidence that Ocean Views new golf course and hotel and not some otherphenomenon--has been responsible for boosting Ocean Views economy during the last twoyears To better assess the recommendation I would need to know why Ocean View decidedto construct its new golf course and hotel in the first place--specifically what events prior toconstruction might have prompted that decision I would also need to thoroughly compare

GRE AWA John박 박정어학원

HopeweU with Ocean View--especially in terms of their appeal to tourists and businesses--todetermine whether the same course of action that appears to have boosted Ocean Viewseconomy would also boost Hopewells economy

The following is part of a memorandum from the president of Humana University

Last year the number of students who enrolled in online degree programs offered by nearby Omni University increased by 50 percent During the same year Omni showed a significant decrease from prior years in expenditures for dormitory and classroom space most likely because instruction in the online programs takes place via the Internet In contrast over the past three years enrollment at Humana University has failed to grow and the cost of maintaining buildings has increased along with our budget deficit To address these problems Humana University will begin immediately to create and actively promote online degree programs like those at Omni We predict that instituting these online degree programs will help Humana both increase its total enrollment and solve its budget problems

Write a response in which you discuss what questions would need to be answered in order to decide whether the prediction and the argument on which it is based are reasonable Be sure to explain how the answers to these questions would help to evaluate the prediction

Is Omni University successful due to the online degree program 50 Is the decrease in expenditures for dormitory and classroom space due to the decrease in of on-campus students Which classes were successful Does HU have those classes

Even if the long-distance degree programs at Omni University benefited the school the presidentrsquos recommendation that Human College should emulate Omni University is too hasty First OUrsquos name implies that the school would have more majors than Humanahellip the president should examine which degrees were in the long-distance programhellip

지난해에는 Omni 대학에서 개강했던 원거리 학생 학점 취득 프로그램을 등록했던 학생들의 숫자가 50나 증가했다 같은해 기간동안 Omni 대학에서는 그 전년도부터 기숙사와 학급의 공간 확충을 위한 예산을 대폭 줄였는데 이는 이 원거리 학점 취득 프로그램이 양방향 비디오 컴퓨터 접속을 통해서만 가능한 수업지도 방식이기때문인 것으로 보인다 반면 지난 3개년 동안 Humana 대학에서의 수강률은 감소한데다가 건물

GRE AWA John박 박정어학원

유지비도 올랐다 따라서 Humana대학의 수강을 늘리고 예산손실을 회복하기 위해서는 Omni 대학에서 취한 조치와 같은 능동적인 프로그램을 추진해야 한다

결론 we should initiate and actively promote long-distance degree programs like those at Omni 반박 원거리 학생 취득 프로그램 숫자가 증가한거하고 예산이 줄어드는 것 사이에 연관이 약하다 (causal 학생의 증가로 관리비용 증가할수 있음 원거리 수업가능 장비도입에의한 비용발생)bad analogy(omni university 하고 같은 조건이 아니다 )-gt omni college 가 강좌내용이 좋아서 학생의 등록이 많을수 있다 Humana 대학에서 만들었다 하더라도 인기 없을수 있음다른 요인에 의해서 Humana 대학의 수강 인원이 증가할수 있음(비록 과거엔 인기가 없었을지라도)

The following appeared as part of a business plan developed by the manager of the Rialto Movie Theater

Despite its downtown location the Rialto Movie Theater a local institution for five decades must make big changes or close its doors forever It should follow the example of the new Apex Theater in the mall outside of town When the Apex opened last year it featured a video arcade plush carpeting and seats and a state-of-the-art sound system Furthermore in a recent survey over 85 percent of respondents reported that the high price of newly released movies prevents them from going to the movies more than five times per year Thus if the Rialto intends to hold on to its share of a decreasing pool of moviegoers it must offer the same features as Apex

Write a response in which you discuss what questions would need to be answered in order to decide whether the recommendation is likely to have the predicted result Be sure to explain how the answers to these questions would help to evaluate the recommendation

Before following through this business plan the manager should investigate the cause of Rialtorsquos unsuccessful business

The author provides no evidence that the surveyrsquos results are statistically reliable The surveyrsquos sample of 85 percent must be sufficient in size and representative of overall population of the city where Rialto and Apex is serving Lacking evidence of a sufficiently representative sample the author cannot justifiably rely on the survey to draw any conclusion whatsoever The author does not indicate that Apex is indeed currently successful However even if Apex is enjoying success the argument relies on what might be a false analogy between Rialto and Apex In order for Apex to serve as a model that Rialto should emulate the author must assume that all relevant circumstances are essentially the same However this assumption is unwarranted For example the argument overlooks the face that Apex is located in a strategic placemdashbeside a mall where customers can not only watch a movie but also enjoy shopping Therefore simply changing the facility to that of Apex may not lead to success

The author does not mention whether Apex is successful or not Nevertheless even if Apex is currently successful the argument relies on what might be a false analogy between Rialto and Apex In order for Apex to serve as a model that Rialto should emulate the author must assume that all relevant circumstances are essentially the same However this assumption is unwarranted For example the argument overlooks the fact that these two institutions are located in different locations Rialto in downtown and Apex in a mall outside of town Although Apex opened with state-of-the-art facilities the decisive factor in its success could be due to its strategic location of being in a mall People could enjoy both shopping and movies at one location thus they may prefer Apex over Rialto Furthermore the place where people enjoy leisure activities has shifted in the past decades for most cities from downtown to the suburbs Therefore Rialto may not be successful even if it emulates Apexrsquos facilities A better business plan may be relocating Apex to the thriving section of the downtown

Rialto 극장은 지난 50여년간 지역 회관으로써 시내에 위치해 있으면서도 이제 변화를 꾀하지 않으면 문을 닫을

GRE AWA John박 박정어학원

판이다 이 극장은 시외 쇼핑타운에 새로 들어선 Apex 극장의 사례를 본받아야 했다 Apex가 지난해 개업했을 당시 이 극장은 비디오 아케이드 플러쉬 카펫트 바닥과 좌석 그리고 최신 음향시설을 갖추었다 더군다나 최근 조사에서는 응답자의 85 이상이 새로 출시된 영화 입장료가 비싼 탓으로 지난해보다 5배이상의 관람객이 줄어들었다고 나타났다 따라서 Rialto 극장이 줄어들고 있는 관람객을 뺐기지 않고 유지하려면 Apex와 같은 시설들을 갖추어야 할 것이다주장 리알토 극장이 줄어들고 있는 관람객을 뺐기지 않고 유지하려면 Apex와 같은 시설들을 갖추어야 할 것이다

1 조사에서 응답자가 전체를 대표할 수 없다 2 apex 극장이 좋은 시설을 갖추고 있지만 그로 인해 수익이 많이 발생했다는 말이 없으므로 시설투자를

하고도 좋은 결과를 얻을 수 있을지 그 근거가 미흡하다3 좋은 영화가 출시된다면 입장료가 비싸도 영화관에서 꼭 보려고 할 수 있다 4 rialto 가 시설이 아닌 다른 요인에 의해 장사가 안될수도 있다( 우범 지역이라든지)

The following is a recommendation from the business manager of Monarch Books

Since its opening in Collegeville twenty years ago Monarch Books has developed a large customer base due to its reader-friendly atmosphere and wide selection of books on all subjects Last month Book and Bean a combination bookstore and coffee shop announced its intention to open a Collegeville store Monarch Books should open its own in-store cafeacute in the space currently devoted to childrens books Given recent national census data indicating a significant decline in the percentage of the population under age ten sales of childrens books are likely to decline By replacing its childrens books section with a cafeacute Monarch Books can increase profits and ward off competition from Book and Bean

Write a response in which you examine the stated andor unstated assumptions of the argument Be sure to explain how the argument depends on these assumptions and what the implications are for the argument if the assumptions prove unwarranted

The following is a recommendation from the business manager of Monarch Books

Since its opening in Collegeville twenty years ago Monarch Books has developed a large customer base due to its reader-friendly atmosphere and wide selection of books on all subjects Last month Book and Bean a combination bookstore and coffee shop announced its intention to open a Collegeville store Monarch Books should open its own in-store cafeacute in the space currently devoted to childrens books Given recent national census data indicating a significant decline in the percentage of the population under age ten sales of childrens books are likely to decline By replacing its childrens books section with a cafeacute Monarch Books can increase profits and ward off competition from Book and Bean

1 Write a response in which you discuss what questions would need to be answered in order to decide whether the recommendation is likely to have the predicted result Be sure to explain how the answers to these questions would help to evaluate the recommendation

2 Write a response in which you discuss what specific evidence is needed to evaluate the argument and explain how the evidence would weaken or strengthen the argument

No evidence regarding Monarch Bookrsquos successEven if Regal Bookrsquos is successful this may not be attributable to the cafeacute False analogy Emulating may not lead to success Other factors may be involvedInsufficient condition The national census is not enough evidence that childrenrsquos book sales will decline Can

GRE AWA John박 박정어학원

the national census represent the local child populationDid opening a cafeacute boost sales for Regal Books Even assuming Regal is successful by opening a cafeacute this may not be suitable for Monarch which plans to close the childrenrsquos book section to establish a cafe Imprecise language ldquorelatively little spacerdquo how smallThe managerrsquos recommendation contradicts what he says Since Monarch is popular for its wide selection of books closing a selection which targets a major group of readers may hurt Monarchrsquos salesIs this the best way to compete

When Stanley Park first opened it was the largest most heavily used public park in town It is still the largest park but it is no longer heavily used Video cameras mounted in the parks parking lots last month revealed the parks drop in popularity the recordings showed an average of only 50 cars per day In contrast tiny Carlton Park in the heart of the business district is visited by more than 150 people on a typical weekday An obvious difference is that Carlton Park unlike Stanley Park provides ample seating Thus if Stanley Park is ever to be as popular with our citizens as Carlton Park the town will obviously need to provide more benches thereby converting some of the unused open areas into spaces suitable for socializing

Write a response in which you examine the stated andor unstated assumptions of the argument Be sure to explain how the argument depends on these assumptions and what the implications are for the argument if the assumptions prove unwarranted

Stanley 파크가 처음 개장했을 당시 가장 크고 가장 많이 이용되는 공원이었다 아직도 공원중에서는 가장 크지만 이용률은 상당히 떨어졌다 지난달 공원 주차장에 설치해놓은 비디오 카메라를 통해 보면 drop(주차장으로 여겨짐) 이용률이 가장 높았다 수치상으로는 하루 평균 50대의 차량만이 이용하였다 반면 직장 중심거리에 위치한 작은 규모의 Carlton 파크는 주당 무려 150여명 이상이 이용하고 있다 Stanley 파크와는 달리 Carlton 파크에는 의자가 있다는 것이 가장 뚜렷한 차이점이다 따라서 Stanley 파크가 Carlton 파크처럼 시민들이 자주 이용하는 공원이 되기 위해서는 벤치를 설치할 필요가 있으며 이렇게 사용되지 않는 일부 공간을 활용해서 사교를 위한 공간으로 바꾸어야 한다 ===gtdrop 에 대한 첨부사항 (영영사전내용입니다)---- a place or central depository to which something (as mail money or stolen property) is brought for distribution or transmission also the act of depositing something at such a place dropgt

주장 if Stanley Park is ever to be as popular with our citizens as is Carlton Park the town will obviously need to provide more benches thereby converting some of the unused open areas into spaces suitable for socializing1 조사가 언제 이루어진 것인가 조사가 언제 실시되었느냐에 따라 결과가 다를 수있다 현재는 다시 스탠리 파크가 늘어났었을 수 있다 2 벤치를 많이 설치했다고 해서 많은 관광객이 오지 않을수 있다(사람들이 벤치나 사교 공간을 원한다는 어떠한 자료도 없다)3스탠리 파크 주변에 교통 상황이 악화가 되었거나 칼튼 파크에서 문화행사등을 많이 가져서 이용객이 줄어든것일 수도 있다 4 칼튼 파크가 중심지에 있어서 접근성이 좋을수 있다5 조사가 같은 시간을 기준으로 한게 아니다(하나는 주중이고 하나는 주말이다)6사람의 수와 차의 대수를 같은것으로 비교할수 없다 (차안에 몇 명이 타고 있는지 모르고 대중교통을 이용해서 왔을수도 있다)

Page 32: GRE writing argument brain storm

GRE AWA John박 박정어학원

Next the author fails to consider other possible explanations for the 50 decline inTorseaus moped accident rate last year Perhaps last year Torseau experienced unusually fairweather during which moped accidents are less likely Perhaps fewer tourists visited Tot seanlast year than during most years thereby diminishing the demand for rental mopeds to belowthe allowed limits Perhaps last year some of Torseaus moped rental outlets purchased newmopeds that are safer to drive Or perhaps the restrictions were already in effect but were notenforced until last year In any event a decline in Torseaus moped accident rate during onlyone year is scarcely sufficient to draw any reliable conclusions about what might have causedthe decline or about what the accident rate will be in years aheadAdditionally in asserting that the same phenomenon that caused a 50 decline in mopedaccidents in Torseau would cause a similar decline in Balmer the author relies on what mightamount to an unfair analogy between Balmer and Torseau Perhaps Balmers ability to enforcemoped-rental restrictions does not meet Torseaus ability if not then the mere enactment ofsimilar restrictions in Balmer is no guarantee of a similar result Or perhaps the demand formopeds in Torseau is always greater than in Balmer Specifically if fewer than all availablemopeds are currently rented per day from the average Balmer outlet while in Torseau everyavailable moped is rented each day then the proposed restriction is likely to have less impacton the accident rate in Balmer than in TorseauFinally the author provides no evidence that the same restrictions that served to reduce theincidence of all moped accidents by 50 would also serve to reduce the incidence ofaccidents involving mopeds and pedestrians by 50 Lacking such evidence it is entirelypossible that the number of moped accidents not involving pedestrians decreased by a greaterpercentage while the number of moped-pedestrian accidents decreased by a smallerpercentage or even increased Since the author has not accounted for these possibilities theeditorials recommendation cannot be taken seriouslyIn conclusion the recommendation is not well supported To convince me that the proposedrestriction would achieve the desired outcome the author would have to assure me that nochanges serving to increase Balmers moped-pedestrian accident rate will occur in theforeseeable future The author must also provide dear evidence that last years decline inmoped accidents in Torseau was attributable primarily to its moped rental restrictions ratherthan to one or more other factors In order to better evaluate the recommendation I wouldneed more information comparing the supply of and demand for moped rentals on the twoislands I would also need to know the rate of mopedpedestrian accidents in Torseau both priorto and after the restrictions were enforced in TorseauThe following appeared in a magazine article about planning for retirement

Clearview should be a top choice for anyone seeking a place to retire because it has spectacular natural beauty and a consistent climate Another advantage is that housing costs in Clearview have fallen significantly during the past year and taxes remain lower than those in neighboring towns Moreover Clearviews mayor promises many new programs to improve schools streets and public services And best of all retirees in Clearview can also expect excellent health care as they grow older since the number of physicians in the area is far greater than the national average

Write a response in which you discuss what specific evidence is needed to evaluate the argument and explain how the evidence would weaken or strengthen the argument

-Natural beauty and consistent climate may not be the most wanted qualities-Housing costs could have lowered on a national level wealthy retirees may not care about costs-Taxes may be high compared to the nationrsquos average tax rate-What about other qualities of Clearview Crime rate what qualities would retirees want -If schools streets and public services need improvement then this is proof that the current condition of Clearview is low Or due to budgetary reasons the mayor may not follow-up on his promise because of lowered tax rate -Schools and people who are retired no relationship-Physicians What kind of physicians Number is irrelevant Are these physicians capable of addressing the illnesses of old people

This author argues that anyone seeking a place to retire should choose Clearview To supportthis argument the article cites Clearviews consistent climate and natural beauty its fallinghousing costs its low property taxes compared to nearby towns and the mayors promise toimprove schools streets and services The article also claims that retirees can expectexcellent health care because the number of physicians in Clearview greatly exceeds thenational average This argument is flawed in several critical respectsTo begin with although consistent climate and natural beauty might be attractive to manyretirees these features are probably not important to all retirees For many retirees it isprobably more important to live near relatives or even to enjoy changing seasons Thus I

GRE AWA John박 박정어학원

cannot accept the authors sweeping recommendation for all retirees on this basisAlso Clearviews declining housing costs do not necessarily make Clearview the best placeto retire for two reasons First despite the decline Clearviews housing costs might be highcompared to housing costs in other cities Secondly for wealthier retirees housing costs arenot likely to be a factor in choosing a place to retire Thus the mere fact that housing costshave been in decline lends scant support to the recommendationThe articles reliance on Clearviews property-tax rates is also problematic in two respectsFirst retirees obviously have innumerable choices about where to retire besides Clear viewand nearby towns Secondly for retirees who are well-off financially property taxes are notlikely to be an important concern in choosing a place to retire Thus it is unfair to infer fromClearviews property-tax rates that retirees would prefer ClearviewYet another problem with the argument involves the mayors promises In light of Clearviewslow property-tax rates whether the mayor can follow through on those promises is highlyquestionable Absent any explanation of how the city can spend more money in the areas citedwithout raising property taxes I simply cannot accept the editorials recommendation on thebasis of those promises Besides even if the city makes the improvements promised thoseimprovements--particular the ones to schools--would not necessarily be important to retireesFinally although the number of physicians in Clearview is relatively high the per capitanumber might be relatively low Moreover it would be fairer to compare this per capita numberwith the per capita number for other attractive retirement towns--rather than the nationalaverage After all retirees are likely to place a relatively heavy burden on health-careresources Besides the article provides no assurances that the number of physicians inClearview will remain high in the foreseeable futureIn conclusion the recommendation is poorly supported To strengthen it the author mustconvince me--perhaps by way of a reliable survey--that the key features that the vast majorityof retirees look for in choosing a place to live are consistent climate natural beauty and lowhousing costs The author must also provide better evidence that Clear views property taxesare lower than the those of cities in other areas The author must also explain how the city canmake its promised improvements without raising property taxes Finally to better assess theargument I would need to now how the per capita number of physicians in Clearview wouldcompare to the national average in the futureThe following appeared as a letter to the editor from a Central Plaza store owner

Over the past two years the number of shoppers in Central Plaza has been steadily decreasing while the popularity of skateboarding has increased dramatically Many Central Plaza store owners believe that the decrease in their business is due to the number of skateboard users in the plaza There has also been a dramatic increase in the amount of litter and vandalism throughout the plaza Thus we recommend that the city prohibit skateboarding in Central Plaza If skateboarding is prohibited here we predict that business in Central Plaza will return to its previously high levels

Write a response in which you discuss what questions would need to be answered in order to decide whether the recommendation is likely to have the predicted result Be sure to explain how the answers to these questions would help to evaluate the recommendation

Why two years ago What happened two years ago which started this declineIs the dramatic increase in the ldquopopularityrdquo of skateboarding the cause of the steady decline of shoppers Are there any malls nearby Were there any changes nearby which could affect the decline in customersmdasha big mall perhaps Could the decline be due to the shop ownersHow many skateboarders use the plazaWhere do they skateboardDo they shop and are they customersAre the increase in litter and vandalism due to skateboarders Could this be alleviated by installing CCTVs and hiring security

This editorial concludes that the city should ban skateboarding from its downtown CentralPlaza in order to attract visitors to that area to return the area to its former glory and to makeit a place where people can congregate for fun and relaxation To justify this conclusion theeditorial points out that skateboarders are nearly the only people one sees anymore at CentralPlaza and that the Plaza is littered and its property defaced The editorial also points out thatthe majority of downtown merchants support the skate boarding ban This argument is flawedin several critical respectsFirst the editorials author falsely assumes that a ban on skateboarding is both necessaryand sufficient to achieve the three stated objectives Perhaps the city can achieve thoseobjectives by other means as well--for example by creating a new mall that incorporates anattractive new skateboard park Even if banning skateboarders altogether is necessary to meetthe citys goals the author has not shown that this action by itself would suffice Assuming thatthe Plazas reputation is now tarnished restoring that reputation and in turn enticing peopleback to the Plaza might require additional measures--such as removing litter and graffiti

GRE AWA John박 박정어학원

promoting the Plaza to the public or enticing popular restaurant or retail chains to the PlazaSecondly the editorial assumes too hastily that the Plazas decline is attributable to theskateboarders--rather than to some other phenomenon Perhaps the Plazas primary appeal inits glory days had to do with particular shops or eateries which were eventually replaced byless appealing ones Or perhaps the crime rate in surrounding areas has risen dramatically forreasons unrelated to the skateboarders presence at the Plaza Without ruling out these andother alternative explanations for the Plazas decline the editorials author cannot convince methat a skateboard ban would reverse that declineThirdly the editorials author might be confusing cause with effect--by assuming that theskateboarders caused the abandonment of the Plaza rather than vice versa It is entirelypossible that skateboarders did not frequent the Plaza until it was largely abandoned--andbecause it had been abandoned In fact this scenario makes good sense since skateboardingis most enjoyable where there are few pedestrians or motorists to get in the wayFourth it is unreasonable to infer from the mere fact that most merchants favor the ban thatthe ban would be effective in achieving the citys objectives Admittedly perhaps thesemerchants would be more likely to help dean up the Plaza area and promote their businesseswere the city to act in accordance with their preference Yet lacking any supporting evidencethe author cannot convince me of this Thus the survey amounts to scant evidence at best thatthe proposed ban would carry the intended resultFinally the author recommends a course of action that might actually defeat the citysobjective of providing a fun and relaxing place for people to congregate In my experienceskateboarding contributes to an atmosphere of fun and relaxation for adults and children alikemore so than many other types of ambiance Without considering that continuing to allowskateboarding--or even encouraging this activity--might achieve the citys goal more effectivelythan banning the activity the author cannot convincingly conclude that the ban would be in thecitys best interestsIn sum the argument is a specious one To strengthen it the editorials author must providedear evidence that skateboarding and not some other factor is responsible for the conditionsmarking the Plazas decline The author must also convince me that no alternative means ofrestoring the Plaza are available to the city and that the proposed ban by itself would suffice toattract tourists and restore the Plaza to its former glory Finally to better assess the argument itwould be useful to know the circumstances under which the downtown merchants would bewilling to help the city achieve its objectives

6그룹 약한 비유 빈출

The following recommendation appeared in a memo from the mayor of the town of Hopewell

Two years ago the nearby town of Ocean View built a new municipal golf course and resort hotel During the past two years tourism in Ocean View has increased new businesses have opened there and Ocean Views tax revenues have risen by 30 percent Therefore the best way to improve Hopewells economymdashand generate additional tax revenuesmdashis to build a golf course and resort hotel similar to those in Ocean View

Write a response in which you examine the stated andor unstated assumptions of the argument Be sure to explain how the argument depends on these assumptions and what the implications are for the argument if the assumptions prove unwarranted

GRE AWA John박 박정어학원

Assumptions The author assumes that OVrsquos municipal golf course and resort hotel caused tourism new businesses and increased tax revenues There may be other reasons advertising promo He assumes that this will continueAssumes that Ocean View and Hopewell are similar in many waysmdashthe name suggests otherwise OV may have always been a tourist attractions for its beaches We need to know the topography

2년전 Ocean View 시는 시정 소유 골프 및 휴양지 호텔을 신축했다 그리고 지난 2년동안 이 시의 관광객이 증가했으며 새로운 사업들이 생겨났다 그에따라 시의 세수도 30나 증가했다 Hopewell의 경제를 향상시키고 아울러 세수를 늘릴 수 있는 가장 좋은 방법은 Ocean View에 세워진 것과 같은 골프 시설과 휴양지 호텔을 신축하는 것이다

1 다른 요인으로 관광 산업이 발전했을 수도 있다 문화 유적이 발견이 되었거나 도로의 정비등으로 여행자가 늘었을 수도 있다

2 관광 산업의증가가 늘어난 세수의 원인이 아니라 새로 유입된 인구의 증가나 다른 공장에서 발생한 것일 수 있다

3 2년동안 한참 골프가 붐을 이루었을 수 있다 경제상황이 나빠지거나 다른 레포츠가 인근 지역에 생겨난다면 골프하는 사람이 줄어들 수 있다

In this memo HopeweUs mayor recommends that in order to stimulate the towns economyand boost tax revenues HopeweU should build a new golf course and resort hotel just as thetown of Ocean View did two years ago To support this recommendation the mayor points outthat in Ocean View during the last two years tourism has increased new businesses haveopened and tax revenues have increased by 30 I find the mayors argument unconvincingin several important respectsFirst of all it is possible that the mayor has confused cause with effect respecting the recentdevelopments in Ocean View Perhaps Ocean Views construction of a new golf course andhotel was a response to previous increases in tourism and business development increasesthat have simply continued during the most recent two years Since the mayor has failed toaccount for this possibility the claim that Hopewell would boost its economy by alsoconstructing a golf course and hotel is completely unwarrantedSecondly the mayor fails to account for other possible causes of the trends in Ocean Viewduring the last two years The increase in tourism might have been due to improving economicconditions nationwide or to unusually pleasant weather in the region The new businessesthat have opened in Ocean View might have opened there irrespective of the new golf courseand hotel And the 30 increase in tax revenues might have been the result of an increase intax rates or the addition of a new type of municipal taxWithout ruling out these and other alternative explanations for the three recent trends inOcean View the mayor cannot reasonably infer based on those trends that Hopewellseconomy would benefit by following Ocean Views exampleThirdly even if the recent trends in Ocean View are attributable to the construction of the newgolf course and hotel there the mayor assumes too hastily that the golf course and hotel willcontinue to benefit that towns overall economy The mayor has not accounted for thepossibility that increased tourism will begin to drive residents away during tourist season orthat new business development will result in the towns losing its appeal as a place to visit or tolive Unless the mayor can convince me that these scenarios are unlikely I cannot accept themayors recommendation that Hopewell follow Ocean Views exampleFinally the mayors argument rests on the unsubstantiated assumption that Hopewell andOcean View are sufficiently alike in ways that might affect the economic impact of a new golfcourse and hotel Hopewell might lack the sort of natural environment that would attract moretourists and new businesses to the town--regardless of its new golf course and hotel For thatmatter perhaps Hopewell already contains several resort hotels and golf courses that are notutilized to their capacity If so building yet another golf course and hotel might amount to amisallocation of the towns resources--and actually harm the towns overall economyIn sum the mayors recommendation is not well supported To bolster it the mayor mustprovide better evidence that Ocean Views new golf course and hotel and not some otherphenomenon--has been responsible for boosting Ocean Views economy during the last twoyears To better assess the recommendation I would need to know why Ocean View decidedto construct its new golf course and hotel in the first place--specifically what events prior toconstruction might have prompted that decision I would also need to thoroughly compare

GRE AWA John박 박정어학원

HopeweU with Ocean View--especially in terms of their appeal to tourists and businesses--todetermine whether the same course of action that appears to have boosted Ocean Viewseconomy would also boost Hopewells economy

The following is part of a memorandum from the president of Humana University

Last year the number of students who enrolled in online degree programs offered by nearby Omni University increased by 50 percent During the same year Omni showed a significant decrease from prior years in expenditures for dormitory and classroom space most likely because instruction in the online programs takes place via the Internet In contrast over the past three years enrollment at Humana University has failed to grow and the cost of maintaining buildings has increased along with our budget deficit To address these problems Humana University will begin immediately to create and actively promote online degree programs like those at Omni We predict that instituting these online degree programs will help Humana both increase its total enrollment and solve its budget problems

Write a response in which you discuss what questions would need to be answered in order to decide whether the prediction and the argument on which it is based are reasonable Be sure to explain how the answers to these questions would help to evaluate the prediction

Is Omni University successful due to the online degree program 50 Is the decrease in expenditures for dormitory and classroom space due to the decrease in of on-campus students Which classes were successful Does HU have those classes

Even if the long-distance degree programs at Omni University benefited the school the presidentrsquos recommendation that Human College should emulate Omni University is too hasty First OUrsquos name implies that the school would have more majors than Humanahellip the president should examine which degrees were in the long-distance programhellip

지난해에는 Omni 대학에서 개강했던 원거리 학생 학점 취득 프로그램을 등록했던 학생들의 숫자가 50나 증가했다 같은해 기간동안 Omni 대학에서는 그 전년도부터 기숙사와 학급의 공간 확충을 위한 예산을 대폭 줄였는데 이는 이 원거리 학점 취득 프로그램이 양방향 비디오 컴퓨터 접속을 통해서만 가능한 수업지도 방식이기때문인 것으로 보인다 반면 지난 3개년 동안 Humana 대학에서의 수강률은 감소한데다가 건물

GRE AWA John박 박정어학원

유지비도 올랐다 따라서 Humana대학의 수강을 늘리고 예산손실을 회복하기 위해서는 Omni 대학에서 취한 조치와 같은 능동적인 프로그램을 추진해야 한다

결론 we should initiate and actively promote long-distance degree programs like those at Omni 반박 원거리 학생 취득 프로그램 숫자가 증가한거하고 예산이 줄어드는 것 사이에 연관이 약하다 (causal 학생의 증가로 관리비용 증가할수 있음 원거리 수업가능 장비도입에의한 비용발생)bad analogy(omni university 하고 같은 조건이 아니다 )-gt omni college 가 강좌내용이 좋아서 학생의 등록이 많을수 있다 Humana 대학에서 만들었다 하더라도 인기 없을수 있음다른 요인에 의해서 Humana 대학의 수강 인원이 증가할수 있음(비록 과거엔 인기가 없었을지라도)

The following appeared as part of a business plan developed by the manager of the Rialto Movie Theater

Despite its downtown location the Rialto Movie Theater a local institution for five decades must make big changes or close its doors forever It should follow the example of the new Apex Theater in the mall outside of town When the Apex opened last year it featured a video arcade plush carpeting and seats and a state-of-the-art sound system Furthermore in a recent survey over 85 percent of respondents reported that the high price of newly released movies prevents them from going to the movies more than five times per year Thus if the Rialto intends to hold on to its share of a decreasing pool of moviegoers it must offer the same features as Apex

Write a response in which you discuss what questions would need to be answered in order to decide whether the recommendation is likely to have the predicted result Be sure to explain how the answers to these questions would help to evaluate the recommendation

Before following through this business plan the manager should investigate the cause of Rialtorsquos unsuccessful business

The author provides no evidence that the surveyrsquos results are statistically reliable The surveyrsquos sample of 85 percent must be sufficient in size and representative of overall population of the city where Rialto and Apex is serving Lacking evidence of a sufficiently representative sample the author cannot justifiably rely on the survey to draw any conclusion whatsoever The author does not indicate that Apex is indeed currently successful However even if Apex is enjoying success the argument relies on what might be a false analogy between Rialto and Apex In order for Apex to serve as a model that Rialto should emulate the author must assume that all relevant circumstances are essentially the same However this assumption is unwarranted For example the argument overlooks the face that Apex is located in a strategic placemdashbeside a mall where customers can not only watch a movie but also enjoy shopping Therefore simply changing the facility to that of Apex may not lead to success

The author does not mention whether Apex is successful or not Nevertheless even if Apex is currently successful the argument relies on what might be a false analogy between Rialto and Apex In order for Apex to serve as a model that Rialto should emulate the author must assume that all relevant circumstances are essentially the same However this assumption is unwarranted For example the argument overlooks the fact that these two institutions are located in different locations Rialto in downtown and Apex in a mall outside of town Although Apex opened with state-of-the-art facilities the decisive factor in its success could be due to its strategic location of being in a mall People could enjoy both shopping and movies at one location thus they may prefer Apex over Rialto Furthermore the place where people enjoy leisure activities has shifted in the past decades for most cities from downtown to the suburbs Therefore Rialto may not be successful even if it emulates Apexrsquos facilities A better business plan may be relocating Apex to the thriving section of the downtown

Rialto 극장은 지난 50여년간 지역 회관으로써 시내에 위치해 있으면서도 이제 변화를 꾀하지 않으면 문을 닫을

GRE AWA John박 박정어학원

판이다 이 극장은 시외 쇼핑타운에 새로 들어선 Apex 극장의 사례를 본받아야 했다 Apex가 지난해 개업했을 당시 이 극장은 비디오 아케이드 플러쉬 카펫트 바닥과 좌석 그리고 최신 음향시설을 갖추었다 더군다나 최근 조사에서는 응답자의 85 이상이 새로 출시된 영화 입장료가 비싼 탓으로 지난해보다 5배이상의 관람객이 줄어들었다고 나타났다 따라서 Rialto 극장이 줄어들고 있는 관람객을 뺐기지 않고 유지하려면 Apex와 같은 시설들을 갖추어야 할 것이다주장 리알토 극장이 줄어들고 있는 관람객을 뺐기지 않고 유지하려면 Apex와 같은 시설들을 갖추어야 할 것이다

1 조사에서 응답자가 전체를 대표할 수 없다 2 apex 극장이 좋은 시설을 갖추고 있지만 그로 인해 수익이 많이 발생했다는 말이 없으므로 시설투자를

하고도 좋은 결과를 얻을 수 있을지 그 근거가 미흡하다3 좋은 영화가 출시된다면 입장료가 비싸도 영화관에서 꼭 보려고 할 수 있다 4 rialto 가 시설이 아닌 다른 요인에 의해 장사가 안될수도 있다( 우범 지역이라든지)

The following is a recommendation from the business manager of Monarch Books

Since its opening in Collegeville twenty years ago Monarch Books has developed a large customer base due to its reader-friendly atmosphere and wide selection of books on all subjects Last month Book and Bean a combination bookstore and coffee shop announced its intention to open a Collegeville store Monarch Books should open its own in-store cafeacute in the space currently devoted to childrens books Given recent national census data indicating a significant decline in the percentage of the population under age ten sales of childrens books are likely to decline By replacing its childrens books section with a cafeacute Monarch Books can increase profits and ward off competition from Book and Bean

Write a response in which you examine the stated andor unstated assumptions of the argument Be sure to explain how the argument depends on these assumptions and what the implications are for the argument if the assumptions prove unwarranted

The following is a recommendation from the business manager of Monarch Books

Since its opening in Collegeville twenty years ago Monarch Books has developed a large customer base due to its reader-friendly atmosphere and wide selection of books on all subjects Last month Book and Bean a combination bookstore and coffee shop announced its intention to open a Collegeville store Monarch Books should open its own in-store cafeacute in the space currently devoted to childrens books Given recent national census data indicating a significant decline in the percentage of the population under age ten sales of childrens books are likely to decline By replacing its childrens books section with a cafeacute Monarch Books can increase profits and ward off competition from Book and Bean

1 Write a response in which you discuss what questions would need to be answered in order to decide whether the recommendation is likely to have the predicted result Be sure to explain how the answers to these questions would help to evaluate the recommendation

2 Write a response in which you discuss what specific evidence is needed to evaluate the argument and explain how the evidence would weaken or strengthen the argument

No evidence regarding Monarch Bookrsquos successEven if Regal Bookrsquos is successful this may not be attributable to the cafeacute False analogy Emulating may not lead to success Other factors may be involvedInsufficient condition The national census is not enough evidence that childrenrsquos book sales will decline Can

GRE AWA John박 박정어학원

the national census represent the local child populationDid opening a cafeacute boost sales for Regal Books Even assuming Regal is successful by opening a cafeacute this may not be suitable for Monarch which plans to close the childrenrsquos book section to establish a cafe Imprecise language ldquorelatively little spacerdquo how smallThe managerrsquos recommendation contradicts what he says Since Monarch is popular for its wide selection of books closing a selection which targets a major group of readers may hurt Monarchrsquos salesIs this the best way to compete

When Stanley Park first opened it was the largest most heavily used public park in town It is still the largest park but it is no longer heavily used Video cameras mounted in the parks parking lots last month revealed the parks drop in popularity the recordings showed an average of only 50 cars per day In contrast tiny Carlton Park in the heart of the business district is visited by more than 150 people on a typical weekday An obvious difference is that Carlton Park unlike Stanley Park provides ample seating Thus if Stanley Park is ever to be as popular with our citizens as Carlton Park the town will obviously need to provide more benches thereby converting some of the unused open areas into spaces suitable for socializing

Write a response in which you examine the stated andor unstated assumptions of the argument Be sure to explain how the argument depends on these assumptions and what the implications are for the argument if the assumptions prove unwarranted

Stanley 파크가 처음 개장했을 당시 가장 크고 가장 많이 이용되는 공원이었다 아직도 공원중에서는 가장 크지만 이용률은 상당히 떨어졌다 지난달 공원 주차장에 설치해놓은 비디오 카메라를 통해 보면 drop(주차장으로 여겨짐) 이용률이 가장 높았다 수치상으로는 하루 평균 50대의 차량만이 이용하였다 반면 직장 중심거리에 위치한 작은 규모의 Carlton 파크는 주당 무려 150여명 이상이 이용하고 있다 Stanley 파크와는 달리 Carlton 파크에는 의자가 있다는 것이 가장 뚜렷한 차이점이다 따라서 Stanley 파크가 Carlton 파크처럼 시민들이 자주 이용하는 공원이 되기 위해서는 벤치를 설치할 필요가 있으며 이렇게 사용되지 않는 일부 공간을 활용해서 사교를 위한 공간으로 바꾸어야 한다 ===gtdrop 에 대한 첨부사항 (영영사전내용입니다)---- a place or central depository to which something (as mail money or stolen property) is brought for distribution or transmission also the act of depositing something at such a place dropgt

주장 if Stanley Park is ever to be as popular with our citizens as is Carlton Park the town will obviously need to provide more benches thereby converting some of the unused open areas into spaces suitable for socializing1 조사가 언제 이루어진 것인가 조사가 언제 실시되었느냐에 따라 결과가 다를 수있다 현재는 다시 스탠리 파크가 늘어났었을 수 있다 2 벤치를 많이 설치했다고 해서 많은 관광객이 오지 않을수 있다(사람들이 벤치나 사교 공간을 원한다는 어떠한 자료도 없다)3스탠리 파크 주변에 교통 상황이 악화가 되었거나 칼튼 파크에서 문화행사등을 많이 가져서 이용객이 줄어든것일 수도 있다 4 칼튼 파크가 중심지에 있어서 접근성이 좋을수 있다5 조사가 같은 시간을 기준으로 한게 아니다(하나는 주중이고 하나는 주말이다)6사람의 수와 차의 대수를 같은것으로 비교할수 없다 (차안에 몇 명이 타고 있는지 모르고 대중교통을 이용해서 왔을수도 있다)

Page 33: GRE writing argument brain storm

GRE AWA John박 박정어학원

cannot accept the authors sweeping recommendation for all retirees on this basisAlso Clearviews declining housing costs do not necessarily make Clearview the best placeto retire for two reasons First despite the decline Clearviews housing costs might be highcompared to housing costs in other cities Secondly for wealthier retirees housing costs arenot likely to be a factor in choosing a place to retire Thus the mere fact that housing costshave been in decline lends scant support to the recommendationThe articles reliance on Clearviews property-tax rates is also problematic in two respectsFirst retirees obviously have innumerable choices about where to retire besides Clear viewand nearby towns Secondly for retirees who are well-off financially property taxes are notlikely to be an important concern in choosing a place to retire Thus it is unfair to infer fromClearviews property-tax rates that retirees would prefer ClearviewYet another problem with the argument involves the mayors promises In light of Clearviewslow property-tax rates whether the mayor can follow through on those promises is highlyquestionable Absent any explanation of how the city can spend more money in the areas citedwithout raising property taxes I simply cannot accept the editorials recommendation on thebasis of those promises Besides even if the city makes the improvements promised thoseimprovements--particular the ones to schools--would not necessarily be important to retireesFinally although the number of physicians in Clearview is relatively high the per capitanumber might be relatively low Moreover it would be fairer to compare this per capita numberwith the per capita number for other attractive retirement towns--rather than the nationalaverage After all retirees are likely to place a relatively heavy burden on health-careresources Besides the article provides no assurances that the number of physicians inClearview will remain high in the foreseeable futureIn conclusion the recommendation is poorly supported To strengthen it the author mustconvince me--perhaps by way of a reliable survey--that the key features that the vast majorityof retirees look for in choosing a place to live are consistent climate natural beauty and lowhousing costs The author must also provide better evidence that Clear views property taxesare lower than the those of cities in other areas The author must also explain how the city canmake its promised improvements without raising property taxes Finally to better assess theargument I would need to now how the per capita number of physicians in Clearview wouldcompare to the national average in the futureThe following appeared as a letter to the editor from a Central Plaza store owner

Over the past two years the number of shoppers in Central Plaza has been steadily decreasing while the popularity of skateboarding has increased dramatically Many Central Plaza store owners believe that the decrease in their business is due to the number of skateboard users in the plaza There has also been a dramatic increase in the amount of litter and vandalism throughout the plaza Thus we recommend that the city prohibit skateboarding in Central Plaza If skateboarding is prohibited here we predict that business in Central Plaza will return to its previously high levels

Write a response in which you discuss what questions would need to be answered in order to decide whether the recommendation is likely to have the predicted result Be sure to explain how the answers to these questions would help to evaluate the recommendation

Why two years ago What happened two years ago which started this declineIs the dramatic increase in the ldquopopularityrdquo of skateboarding the cause of the steady decline of shoppers Are there any malls nearby Were there any changes nearby which could affect the decline in customersmdasha big mall perhaps Could the decline be due to the shop ownersHow many skateboarders use the plazaWhere do they skateboardDo they shop and are they customersAre the increase in litter and vandalism due to skateboarders Could this be alleviated by installing CCTVs and hiring security

This editorial concludes that the city should ban skateboarding from its downtown CentralPlaza in order to attract visitors to that area to return the area to its former glory and to makeit a place where people can congregate for fun and relaxation To justify this conclusion theeditorial points out that skateboarders are nearly the only people one sees anymore at CentralPlaza and that the Plaza is littered and its property defaced The editorial also points out thatthe majority of downtown merchants support the skate boarding ban This argument is flawedin several critical respectsFirst the editorials author falsely assumes that a ban on skateboarding is both necessaryand sufficient to achieve the three stated objectives Perhaps the city can achieve thoseobjectives by other means as well--for example by creating a new mall that incorporates anattractive new skateboard park Even if banning skateboarders altogether is necessary to meetthe citys goals the author has not shown that this action by itself would suffice Assuming thatthe Plazas reputation is now tarnished restoring that reputation and in turn enticing peopleback to the Plaza might require additional measures--such as removing litter and graffiti

GRE AWA John박 박정어학원

promoting the Plaza to the public or enticing popular restaurant or retail chains to the PlazaSecondly the editorial assumes too hastily that the Plazas decline is attributable to theskateboarders--rather than to some other phenomenon Perhaps the Plazas primary appeal inits glory days had to do with particular shops or eateries which were eventually replaced byless appealing ones Or perhaps the crime rate in surrounding areas has risen dramatically forreasons unrelated to the skateboarders presence at the Plaza Without ruling out these andother alternative explanations for the Plazas decline the editorials author cannot convince methat a skateboard ban would reverse that declineThirdly the editorials author might be confusing cause with effect--by assuming that theskateboarders caused the abandonment of the Plaza rather than vice versa It is entirelypossible that skateboarders did not frequent the Plaza until it was largely abandoned--andbecause it had been abandoned In fact this scenario makes good sense since skateboardingis most enjoyable where there are few pedestrians or motorists to get in the wayFourth it is unreasonable to infer from the mere fact that most merchants favor the ban thatthe ban would be effective in achieving the citys objectives Admittedly perhaps thesemerchants would be more likely to help dean up the Plaza area and promote their businesseswere the city to act in accordance with their preference Yet lacking any supporting evidencethe author cannot convince me of this Thus the survey amounts to scant evidence at best thatthe proposed ban would carry the intended resultFinally the author recommends a course of action that might actually defeat the citysobjective of providing a fun and relaxing place for people to congregate In my experienceskateboarding contributes to an atmosphere of fun and relaxation for adults and children alikemore so than many other types of ambiance Without considering that continuing to allowskateboarding--or even encouraging this activity--might achieve the citys goal more effectivelythan banning the activity the author cannot convincingly conclude that the ban would be in thecitys best interestsIn sum the argument is a specious one To strengthen it the editorials author must providedear evidence that skateboarding and not some other factor is responsible for the conditionsmarking the Plazas decline The author must also convince me that no alternative means ofrestoring the Plaza are available to the city and that the proposed ban by itself would suffice toattract tourists and restore the Plaza to its former glory Finally to better assess the argument itwould be useful to know the circumstances under which the downtown merchants would bewilling to help the city achieve its objectives

6그룹 약한 비유 빈출

The following recommendation appeared in a memo from the mayor of the town of Hopewell

Two years ago the nearby town of Ocean View built a new municipal golf course and resort hotel During the past two years tourism in Ocean View has increased new businesses have opened there and Ocean Views tax revenues have risen by 30 percent Therefore the best way to improve Hopewells economymdashand generate additional tax revenuesmdashis to build a golf course and resort hotel similar to those in Ocean View

Write a response in which you examine the stated andor unstated assumptions of the argument Be sure to explain how the argument depends on these assumptions and what the implications are for the argument if the assumptions prove unwarranted

GRE AWA John박 박정어학원

Assumptions The author assumes that OVrsquos municipal golf course and resort hotel caused tourism new businesses and increased tax revenues There may be other reasons advertising promo He assumes that this will continueAssumes that Ocean View and Hopewell are similar in many waysmdashthe name suggests otherwise OV may have always been a tourist attractions for its beaches We need to know the topography

2년전 Ocean View 시는 시정 소유 골프 및 휴양지 호텔을 신축했다 그리고 지난 2년동안 이 시의 관광객이 증가했으며 새로운 사업들이 생겨났다 그에따라 시의 세수도 30나 증가했다 Hopewell의 경제를 향상시키고 아울러 세수를 늘릴 수 있는 가장 좋은 방법은 Ocean View에 세워진 것과 같은 골프 시설과 휴양지 호텔을 신축하는 것이다

1 다른 요인으로 관광 산업이 발전했을 수도 있다 문화 유적이 발견이 되었거나 도로의 정비등으로 여행자가 늘었을 수도 있다

2 관광 산업의증가가 늘어난 세수의 원인이 아니라 새로 유입된 인구의 증가나 다른 공장에서 발생한 것일 수 있다

3 2년동안 한참 골프가 붐을 이루었을 수 있다 경제상황이 나빠지거나 다른 레포츠가 인근 지역에 생겨난다면 골프하는 사람이 줄어들 수 있다

In this memo HopeweUs mayor recommends that in order to stimulate the towns economyand boost tax revenues HopeweU should build a new golf course and resort hotel just as thetown of Ocean View did two years ago To support this recommendation the mayor points outthat in Ocean View during the last two years tourism has increased new businesses haveopened and tax revenues have increased by 30 I find the mayors argument unconvincingin several important respectsFirst of all it is possible that the mayor has confused cause with effect respecting the recentdevelopments in Ocean View Perhaps Ocean Views construction of a new golf course andhotel was a response to previous increases in tourism and business development increasesthat have simply continued during the most recent two years Since the mayor has failed toaccount for this possibility the claim that Hopewell would boost its economy by alsoconstructing a golf course and hotel is completely unwarrantedSecondly the mayor fails to account for other possible causes of the trends in Ocean Viewduring the last two years The increase in tourism might have been due to improving economicconditions nationwide or to unusually pleasant weather in the region The new businessesthat have opened in Ocean View might have opened there irrespective of the new golf courseand hotel And the 30 increase in tax revenues might have been the result of an increase intax rates or the addition of a new type of municipal taxWithout ruling out these and other alternative explanations for the three recent trends inOcean View the mayor cannot reasonably infer based on those trends that Hopewellseconomy would benefit by following Ocean Views exampleThirdly even if the recent trends in Ocean View are attributable to the construction of the newgolf course and hotel there the mayor assumes too hastily that the golf course and hotel willcontinue to benefit that towns overall economy The mayor has not accounted for thepossibility that increased tourism will begin to drive residents away during tourist season orthat new business development will result in the towns losing its appeal as a place to visit or tolive Unless the mayor can convince me that these scenarios are unlikely I cannot accept themayors recommendation that Hopewell follow Ocean Views exampleFinally the mayors argument rests on the unsubstantiated assumption that Hopewell andOcean View are sufficiently alike in ways that might affect the economic impact of a new golfcourse and hotel Hopewell might lack the sort of natural environment that would attract moretourists and new businesses to the town--regardless of its new golf course and hotel For thatmatter perhaps Hopewell already contains several resort hotels and golf courses that are notutilized to their capacity If so building yet another golf course and hotel might amount to amisallocation of the towns resources--and actually harm the towns overall economyIn sum the mayors recommendation is not well supported To bolster it the mayor mustprovide better evidence that Ocean Views new golf course and hotel and not some otherphenomenon--has been responsible for boosting Ocean Views economy during the last twoyears To better assess the recommendation I would need to know why Ocean View decidedto construct its new golf course and hotel in the first place--specifically what events prior toconstruction might have prompted that decision I would also need to thoroughly compare

GRE AWA John박 박정어학원

HopeweU with Ocean View--especially in terms of their appeal to tourists and businesses--todetermine whether the same course of action that appears to have boosted Ocean Viewseconomy would also boost Hopewells economy

The following is part of a memorandum from the president of Humana University

Last year the number of students who enrolled in online degree programs offered by nearby Omni University increased by 50 percent During the same year Omni showed a significant decrease from prior years in expenditures for dormitory and classroom space most likely because instruction in the online programs takes place via the Internet In contrast over the past three years enrollment at Humana University has failed to grow and the cost of maintaining buildings has increased along with our budget deficit To address these problems Humana University will begin immediately to create and actively promote online degree programs like those at Omni We predict that instituting these online degree programs will help Humana both increase its total enrollment and solve its budget problems

Write a response in which you discuss what questions would need to be answered in order to decide whether the prediction and the argument on which it is based are reasonable Be sure to explain how the answers to these questions would help to evaluate the prediction

Is Omni University successful due to the online degree program 50 Is the decrease in expenditures for dormitory and classroom space due to the decrease in of on-campus students Which classes were successful Does HU have those classes

Even if the long-distance degree programs at Omni University benefited the school the presidentrsquos recommendation that Human College should emulate Omni University is too hasty First OUrsquos name implies that the school would have more majors than Humanahellip the president should examine which degrees were in the long-distance programhellip

지난해에는 Omni 대학에서 개강했던 원거리 학생 학점 취득 프로그램을 등록했던 학생들의 숫자가 50나 증가했다 같은해 기간동안 Omni 대학에서는 그 전년도부터 기숙사와 학급의 공간 확충을 위한 예산을 대폭 줄였는데 이는 이 원거리 학점 취득 프로그램이 양방향 비디오 컴퓨터 접속을 통해서만 가능한 수업지도 방식이기때문인 것으로 보인다 반면 지난 3개년 동안 Humana 대학에서의 수강률은 감소한데다가 건물

GRE AWA John박 박정어학원

유지비도 올랐다 따라서 Humana대학의 수강을 늘리고 예산손실을 회복하기 위해서는 Omni 대학에서 취한 조치와 같은 능동적인 프로그램을 추진해야 한다

결론 we should initiate and actively promote long-distance degree programs like those at Omni 반박 원거리 학생 취득 프로그램 숫자가 증가한거하고 예산이 줄어드는 것 사이에 연관이 약하다 (causal 학생의 증가로 관리비용 증가할수 있음 원거리 수업가능 장비도입에의한 비용발생)bad analogy(omni university 하고 같은 조건이 아니다 )-gt omni college 가 강좌내용이 좋아서 학생의 등록이 많을수 있다 Humana 대학에서 만들었다 하더라도 인기 없을수 있음다른 요인에 의해서 Humana 대학의 수강 인원이 증가할수 있음(비록 과거엔 인기가 없었을지라도)

The following appeared as part of a business plan developed by the manager of the Rialto Movie Theater

Despite its downtown location the Rialto Movie Theater a local institution for five decades must make big changes or close its doors forever It should follow the example of the new Apex Theater in the mall outside of town When the Apex opened last year it featured a video arcade plush carpeting and seats and a state-of-the-art sound system Furthermore in a recent survey over 85 percent of respondents reported that the high price of newly released movies prevents them from going to the movies more than five times per year Thus if the Rialto intends to hold on to its share of a decreasing pool of moviegoers it must offer the same features as Apex

Write a response in which you discuss what questions would need to be answered in order to decide whether the recommendation is likely to have the predicted result Be sure to explain how the answers to these questions would help to evaluate the recommendation

Before following through this business plan the manager should investigate the cause of Rialtorsquos unsuccessful business

The author provides no evidence that the surveyrsquos results are statistically reliable The surveyrsquos sample of 85 percent must be sufficient in size and representative of overall population of the city where Rialto and Apex is serving Lacking evidence of a sufficiently representative sample the author cannot justifiably rely on the survey to draw any conclusion whatsoever The author does not indicate that Apex is indeed currently successful However even if Apex is enjoying success the argument relies on what might be a false analogy between Rialto and Apex In order for Apex to serve as a model that Rialto should emulate the author must assume that all relevant circumstances are essentially the same However this assumption is unwarranted For example the argument overlooks the face that Apex is located in a strategic placemdashbeside a mall where customers can not only watch a movie but also enjoy shopping Therefore simply changing the facility to that of Apex may not lead to success

The author does not mention whether Apex is successful or not Nevertheless even if Apex is currently successful the argument relies on what might be a false analogy between Rialto and Apex In order for Apex to serve as a model that Rialto should emulate the author must assume that all relevant circumstances are essentially the same However this assumption is unwarranted For example the argument overlooks the fact that these two institutions are located in different locations Rialto in downtown and Apex in a mall outside of town Although Apex opened with state-of-the-art facilities the decisive factor in its success could be due to its strategic location of being in a mall People could enjoy both shopping and movies at one location thus they may prefer Apex over Rialto Furthermore the place where people enjoy leisure activities has shifted in the past decades for most cities from downtown to the suburbs Therefore Rialto may not be successful even if it emulates Apexrsquos facilities A better business plan may be relocating Apex to the thriving section of the downtown

Rialto 극장은 지난 50여년간 지역 회관으로써 시내에 위치해 있으면서도 이제 변화를 꾀하지 않으면 문을 닫을

GRE AWA John박 박정어학원

판이다 이 극장은 시외 쇼핑타운에 새로 들어선 Apex 극장의 사례를 본받아야 했다 Apex가 지난해 개업했을 당시 이 극장은 비디오 아케이드 플러쉬 카펫트 바닥과 좌석 그리고 최신 음향시설을 갖추었다 더군다나 최근 조사에서는 응답자의 85 이상이 새로 출시된 영화 입장료가 비싼 탓으로 지난해보다 5배이상의 관람객이 줄어들었다고 나타났다 따라서 Rialto 극장이 줄어들고 있는 관람객을 뺐기지 않고 유지하려면 Apex와 같은 시설들을 갖추어야 할 것이다주장 리알토 극장이 줄어들고 있는 관람객을 뺐기지 않고 유지하려면 Apex와 같은 시설들을 갖추어야 할 것이다

1 조사에서 응답자가 전체를 대표할 수 없다 2 apex 극장이 좋은 시설을 갖추고 있지만 그로 인해 수익이 많이 발생했다는 말이 없으므로 시설투자를

하고도 좋은 결과를 얻을 수 있을지 그 근거가 미흡하다3 좋은 영화가 출시된다면 입장료가 비싸도 영화관에서 꼭 보려고 할 수 있다 4 rialto 가 시설이 아닌 다른 요인에 의해 장사가 안될수도 있다( 우범 지역이라든지)

The following is a recommendation from the business manager of Monarch Books

Since its opening in Collegeville twenty years ago Monarch Books has developed a large customer base due to its reader-friendly atmosphere and wide selection of books on all subjects Last month Book and Bean a combination bookstore and coffee shop announced its intention to open a Collegeville store Monarch Books should open its own in-store cafeacute in the space currently devoted to childrens books Given recent national census data indicating a significant decline in the percentage of the population under age ten sales of childrens books are likely to decline By replacing its childrens books section with a cafeacute Monarch Books can increase profits and ward off competition from Book and Bean

Write a response in which you examine the stated andor unstated assumptions of the argument Be sure to explain how the argument depends on these assumptions and what the implications are for the argument if the assumptions prove unwarranted

The following is a recommendation from the business manager of Monarch Books

Since its opening in Collegeville twenty years ago Monarch Books has developed a large customer base due to its reader-friendly atmosphere and wide selection of books on all subjects Last month Book and Bean a combination bookstore and coffee shop announced its intention to open a Collegeville store Monarch Books should open its own in-store cafeacute in the space currently devoted to childrens books Given recent national census data indicating a significant decline in the percentage of the population under age ten sales of childrens books are likely to decline By replacing its childrens books section with a cafeacute Monarch Books can increase profits and ward off competition from Book and Bean

1 Write a response in which you discuss what questions would need to be answered in order to decide whether the recommendation is likely to have the predicted result Be sure to explain how the answers to these questions would help to evaluate the recommendation

2 Write a response in which you discuss what specific evidence is needed to evaluate the argument and explain how the evidence would weaken or strengthen the argument

No evidence regarding Monarch Bookrsquos successEven if Regal Bookrsquos is successful this may not be attributable to the cafeacute False analogy Emulating may not lead to success Other factors may be involvedInsufficient condition The national census is not enough evidence that childrenrsquos book sales will decline Can

GRE AWA John박 박정어학원

the national census represent the local child populationDid opening a cafeacute boost sales for Regal Books Even assuming Regal is successful by opening a cafeacute this may not be suitable for Monarch which plans to close the childrenrsquos book section to establish a cafe Imprecise language ldquorelatively little spacerdquo how smallThe managerrsquos recommendation contradicts what he says Since Monarch is popular for its wide selection of books closing a selection which targets a major group of readers may hurt Monarchrsquos salesIs this the best way to compete

When Stanley Park first opened it was the largest most heavily used public park in town It is still the largest park but it is no longer heavily used Video cameras mounted in the parks parking lots last month revealed the parks drop in popularity the recordings showed an average of only 50 cars per day In contrast tiny Carlton Park in the heart of the business district is visited by more than 150 people on a typical weekday An obvious difference is that Carlton Park unlike Stanley Park provides ample seating Thus if Stanley Park is ever to be as popular with our citizens as Carlton Park the town will obviously need to provide more benches thereby converting some of the unused open areas into spaces suitable for socializing

Write a response in which you examine the stated andor unstated assumptions of the argument Be sure to explain how the argument depends on these assumptions and what the implications are for the argument if the assumptions prove unwarranted

Stanley 파크가 처음 개장했을 당시 가장 크고 가장 많이 이용되는 공원이었다 아직도 공원중에서는 가장 크지만 이용률은 상당히 떨어졌다 지난달 공원 주차장에 설치해놓은 비디오 카메라를 통해 보면 drop(주차장으로 여겨짐) 이용률이 가장 높았다 수치상으로는 하루 평균 50대의 차량만이 이용하였다 반면 직장 중심거리에 위치한 작은 규모의 Carlton 파크는 주당 무려 150여명 이상이 이용하고 있다 Stanley 파크와는 달리 Carlton 파크에는 의자가 있다는 것이 가장 뚜렷한 차이점이다 따라서 Stanley 파크가 Carlton 파크처럼 시민들이 자주 이용하는 공원이 되기 위해서는 벤치를 설치할 필요가 있으며 이렇게 사용되지 않는 일부 공간을 활용해서 사교를 위한 공간으로 바꾸어야 한다 ===gtdrop 에 대한 첨부사항 (영영사전내용입니다)---- a place or central depository to which something (as mail money or stolen property) is brought for distribution or transmission also the act of depositing something at such a place dropgt

주장 if Stanley Park is ever to be as popular with our citizens as is Carlton Park the town will obviously need to provide more benches thereby converting some of the unused open areas into spaces suitable for socializing1 조사가 언제 이루어진 것인가 조사가 언제 실시되었느냐에 따라 결과가 다를 수있다 현재는 다시 스탠리 파크가 늘어났었을 수 있다 2 벤치를 많이 설치했다고 해서 많은 관광객이 오지 않을수 있다(사람들이 벤치나 사교 공간을 원한다는 어떠한 자료도 없다)3스탠리 파크 주변에 교통 상황이 악화가 되었거나 칼튼 파크에서 문화행사등을 많이 가져서 이용객이 줄어든것일 수도 있다 4 칼튼 파크가 중심지에 있어서 접근성이 좋을수 있다5 조사가 같은 시간을 기준으로 한게 아니다(하나는 주중이고 하나는 주말이다)6사람의 수와 차의 대수를 같은것으로 비교할수 없다 (차안에 몇 명이 타고 있는지 모르고 대중교통을 이용해서 왔을수도 있다)

Page 34: GRE writing argument brain storm

GRE AWA John박 박정어학원

promoting the Plaza to the public or enticing popular restaurant or retail chains to the PlazaSecondly the editorial assumes too hastily that the Plazas decline is attributable to theskateboarders--rather than to some other phenomenon Perhaps the Plazas primary appeal inits glory days had to do with particular shops or eateries which were eventually replaced byless appealing ones Or perhaps the crime rate in surrounding areas has risen dramatically forreasons unrelated to the skateboarders presence at the Plaza Without ruling out these andother alternative explanations for the Plazas decline the editorials author cannot convince methat a skateboard ban would reverse that declineThirdly the editorials author might be confusing cause with effect--by assuming that theskateboarders caused the abandonment of the Plaza rather than vice versa It is entirelypossible that skateboarders did not frequent the Plaza until it was largely abandoned--andbecause it had been abandoned In fact this scenario makes good sense since skateboardingis most enjoyable where there are few pedestrians or motorists to get in the wayFourth it is unreasonable to infer from the mere fact that most merchants favor the ban thatthe ban would be effective in achieving the citys objectives Admittedly perhaps thesemerchants would be more likely to help dean up the Plaza area and promote their businesseswere the city to act in accordance with their preference Yet lacking any supporting evidencethe author cannot convince me of this Thus the survey amounts to scant evidence at best thatthe proposed ban would carry the intended resultFinally the author recommends a course of action that might actually defeat the citysobjective of providing a fun and relaxing place for people to congregate In my experienceskateboarding contributes to an atmosphere of fun and relaxation for adults and children alikemore so than many other types of ambiance Without considering that continuing to allowskateboarding--or even encouraging this activity--might achieve the citys goal more effectivelythan banning the activity the author cannot convincingly conclude that the ban would be in thecitys best interestsIn sum the argument is a specious one To strengthen it the editorials author must providedear evidence that skateboarding and not some other factor is responsible for the conditionsmarking the Plazas decline The author must also convince me that no alternative means ofrestoring the Plaza are available to the city and that the proposed ban by itself would suffice toattract tourists and restore the Plaza to its former glory Finally to better assess the argument itwould be useful to know the circumstances under which the downtown merchants would bewilling to help the city achieve its objectives

6그룹 약한 비유 빈출

The following recommendation appeared in a memo from the mayor of the town of Hopewell

Two years ago the nearby town of Ocean View built a new municipal golf course and resort hotel During the past two years tourism in Ocean View has increased new businesses have opened there and Ocean Views tax revenues have risen by 30 percent Therefore the best way to improve Hopewells economymdashand generate additional tax revenuesmdashis to build a golf course and resort hotel similar to those in Ocean View

Write a response in which you examine the stated andor unstated assumptions of the argument Be sure to explain how the argument depends on these assumptions and what the implications are for the argument if the assumptions prove unwarranted

GRE AWA John박 박정어학원

Assumptions The author assumes that OVrsquos municipal golf course and resort hotel caused tourism new businesses and increased tax revenues There may be other reasons advertising promo He assumes that this will continueAssumes that Ocean View and Hopewell are similar in many waysmdashthe name suggests otherwise OV may have always been a tourist attractions for its beaches We need to know the topography

2년전 Ocean View 시는 시정 소유 골프 및 휴양지 호텔을 신축했다 그리고 지난 2년동안 이 시의 관광객이 증가했으며 새로운 사업들이 생겨났다 그에따라 시의 세수도 30나 증가했다 Hopewell의 경제를 향상시키고 아울러 세수를 늘릴 수 있는 가장 좋은 방법은 Ocean View에 세워진 것과 같은 골프 시설과 휴양지 호텔을 신축하는 것이다

1 다른 요인으로 관광 산업이 발전했을 수도 있다 문화 유적이 발견이 되었거나 도로의 정비등으로 여행자가 늘었을 수도 있다

2 관광 산업의증가가 늘어난 세수의 원인이 아니라 새로 유입된 인구의 증가나 다른 공장에서 발생한 것일 수 있다

3 2년동안 한참 골프가 붐을 이루었을 수 있다 경제상황이 나빠지거나 다른 레포츠가 인근 지역에 생겨난다면 골프하는 사람이 줄어들 수 있다

In this memo HopeweUs mayor recommends that in order to stimulate the towns economyand boost tax revenues HopeweU should build a new golf course and resort hotel just as thetown of Ocean View did two years ago To support this recommendation the mayor points outthat in Ocean View during the last two years tourism has increased new businesses haveopened and tax revenues have increased by 30 I find the mayors argument unconvincingin several important respectsFirst of all it is possible that the mayor has confused cause with effect respecting the recentdevelopments in Ocean View Perhaps Ocean Views construction of a new golf course andhotel was a response to previous increases in tourism and business development increasesthat have simply continued during the most recent two years Since the mayor has failed toaccount for this possibility the claim that Hopewell would boost its economy by alsoconstructing a golf course and hotel is completely unwarrantedSecondly the mayor fails to account for other possible causes of the trends in Ocean Viewduring the last two years The increase in tourism might have been due to improving economicconditions nationwide or to unusually pleasant weather in the region The new businessesthat have opened in Ocean View might have opened there irrespective of the new golf courseand hotel And the 30 increase in tax revenues might have been the result of an increase intax rates or the addition of a new type of municipal taxWithout ruling out these and other alternative explanations for the three recent trends inOcean View the mayor cannot reasonably infer based on those trends that Hopewellseconomy would benefit by following Ocean Views exampleThirdly even if the recent trends in Ocean View are attributable to the construction of the newgolf course and hotel there the mayor assumes too hastily that the golf course and hotel willcontinue to benefit that towns overall economy The mayor has not accounted for thepossibility that increased tourism will begin to drive residents away during tourist season orthat new business development will result in the towns losing its appeal as a place to visit or tolive Unless the mayor can convince me that these scenarios are unlikely I cannot accept themayors recommendation that Hopewell follow Ocean Views exampleFinally the mayors argument rests on the unsubstantiated assumption that Hopewell andOcean View are sufficiently alike in ways that might affect the economic impact of a new golfcourse and hotel Hopewell might lack the sort of natural environment that would attract moretourists and new businesses to the town--regardless of its new golf course and hotel For thatmatter perhaps Hopewell already contains several resort hotels and golf courses that are notutilized to their capacity If so building yet another golf course and hotel might amount to amisallocation of the towns resources--and actually harm the towns overall economyIn sum the mayors recommendation is not well supported To bolster it the mayor mustprovide better evidence that Ocean Views new golf course and hotel and not some otherphenomenon--has been responsible for boosting Ocean Views economy during the last twoyears To better assess the recommendation I would need to know why Ocean View decidedto construct its new golf course and hotel in the first place--specifically what events prior toconstruction might have prompted that decision I would also need to thoroughly compare

GRE AWA John박 박정어학원

HopeweU with Ocean View--especially in terms of their appeal to tourists and businesses--todetermine whether the same course of action that appears to have boosted Ocean Viewseconomy would also boost Hopewells economy

The following is part of a memorandum from the president of Humana University

Last year the number of students who enrolled in online degree programs offered by nearby Omni University increased by 50 percent During the same year Omni showed a significant decrease from prior years in expenditures for dormitory and classroom space most likely because instruction in the online programs takes place via the Internet In contrast over the past three years enrollment at Humana University has failed to grow and the cost of maintaining buildings has increased along with our budget deficit To address these problems Humana University will begin immediately to create and actively promote online degree programs like those at Omni We predict that instituting these online degree programs will help Humana both increase its total enrollment and solve its budget problems

Write a response in which you discuss what questions would need to be answered in order to decide whether the prediction and the argument on which it is based are reasonable Be sure to explain how the answers to these questions would help to evaluate the prediction

Is Omni University successful due to the online degree program 50 Is the decrease in expenditures for dormitory and classroom space due to the decrease in of on-campus students Which classes were successful Does HU have those classes

Even if the long-distance degree programs at Omni University benefited the school the presidentrsquos recommendation that Human College should emulate Omni University is too hasty First OUrsquos name implies that the school would have more majors than Humanahellip the president should examine which degrees were in the long-distance programhellip

지난해에는 Omni 대학에서 개강했던 원거리 학생 학점 취득 프로그램을 등록했던 학생들의 숫자가 50나 증가했다 같은해 기간동안 Omni 대학에서는 그 전년도부터 기숙사와 학급의 공간 확충을 위한 예산을 대폭 줄였는데 이는 이 원거리 학점 취득 프로그램이 양방향 비디오 컴퓨터 접속을 통해서만 가능한 수업지도 방식이기때문인 것으로 보인다 반면 지난 3개년 동안 Humana 대학에서의 수강률은 감소한데다가 건물

GRE AWA John박 박정어학원

유지비도 올랐다 따라서 Humana대학의 수강을 늘리고 예산손실을 회복하기 위해서는 Omni 대학에서 취한 조치와 같은 능동적인 프로그램을 추진해야 한다

결론 we should initiate and actively promote long-distance degree programs like those at Omni 반박 원거리 학생 취득 프로그램 숫자가 증가한거하고 예산이 줄어드는 것 사이에 연관이 약하다 (causal 학생의 증가로 관리비용 증가할수 있음 원거리 수업가능 장비도입에의한 비용발생)bad analogy(omni university 하고 같은 조건이 아니다 )-gt omni college 가 강좌내용이 좋아서 학생의 등록이 많을수 있다 Humana 대학에서 만들었다 하더라도 인기 없을수 있음다른 요인에 의해서 Humana 대학의 수강 인원이 증가할수 있음(비록 과거엔 인기가 없었을지라도)

The following appeared as part of a business plan developed by the manager of the Rialto Movie Theater

Despite its downtown location the Rialto Movie Theater a local institution for five decades must make big changes or close its doors forever It should follow the example of the new Apex Theater in the mall outside of town When the Apex opened last year it featured a video arcade plush carpeting and seats and a state-of-the-art sound system Furthermore in a recent survey over 85 percent of respondents reported that the high price of newly released movies prevents them from going to the movies more than five times per year Thus if the Rialto intends to hold on to its share of a decreasing pool of moviegoers it must offer the same features as Apex

Write a response in which you discuss what questions would need to be answered in order to decide whether the recommendation is likely to have the predicted result Be sure to explain how the answers to these questions would help to evaluate the recommendation

Before following through this business plan the manager should investigate the cause of Rialtorsquos unsuccessful business

The author provides no evidence that the surveyrsquos results are statistically reliable The surveyrsquos sample of 85 percent must be sufficient in size and representative of overall population of the city where Rialto and Apex is serving Lacking evidence of a sufficiently representative sample the author cannot justifiably rely on the survey to draw any conclusion whatsoever The author does not indicate that Apex is indeed currently successful However even if Apex is enjoying success the argument relies on what might be a false analogy between Rialto and Apex In order for Apex to serve as a model that Rialto should emulate the author must assume that all relevant circumstances are essentially the same However this assumption is unwarranted For example the argument overlooks the face that Apex is located in a strategic placemdashbeside a mall where customers can not only watch a movie but also enjoy shopping Therefore simply changing the facility to that of Apex may not lead to success

The author does not mention whether Apex is successful or not Nevertheless even if Apex is currently successful the argument relies on what might be a false analogy between Rialto and Apex In order for Apex to serve as a model that Rialto should emulate the author must assume that all relevant circumstances are essentially the same However this assumption is unwarranted For example the argument overlooks the fact that these two institutions are located in different locations Rialto in downtown and Apex in a mall outside of town Although Apex opened with state-of-the-art facilities the decisive factor in its success could be due to its strategic location of being in a mall People could enjoy both shopping and movies at one location thus they may prefer Apex over Rialto Furthermore the place where people enjoy leisure activities has shifted in the past decades for most cities from downtown to the suburbs Therefore Rialto may not be successful even if it emulates Apexrsquos facilities A better business plan may be relocating Apex to the thriving section of the downtown

Rialto 극장은 지난 50여년간 지역 회관으로써 시내에 위치해 있으면서도 이제 변화를 꾀하지 않으면 문을 닫을

GRE AWA John박 박정어학원

판이다 이 극장은 시외 쇼핑타운에 새로 들어선 Apex 극장의 사례를 본받아야 했다 Apex가 지난해 개업했을 당시 이 극장은 비디오 아케이드 플러쉬 카펫트 바닥과 좌석 그리고 최신 음향시설을 갖추었다 더군다나 최근 조사에서는 응답자의 85 이상이 새로 출시된 영화 입장료가 비싼 탓으로 지난해보다 5배이상의 관람객이 줄어들었다고 나타났다 따라서 Rialto 극장이 줄어들고 있는 관람객을 뺐기지 않고 유지하려면 Apex와 같은 시설들을 갖추어야 할 것이다주장 리알토 극장이 줄어들고 있는 관람객을 뺐기지 않고 유지하려면 Apex와 같은 시설들을 갖추어야 할 것이다

1 조사에서 응답자가 전체를 대표할 수 없다 2 apex 극장이 좋은 시설을 갖추고 있지만 그로 인해 수익이 많이 발생했다는 말이 없으므로 시설투자를

하고도 좋은 결과를 얻을 수 있을지 그 근거가 미흡하다3 좋은 영화가 출시된다면 입장료가 비싸도 영화관에서 꼭 보려고 할 수 있다 4 rialto 가 시설이 아닌 다른 요인에 의해 장사가 안될수도 있다( 우범 지역이라든지)

The following is a recommendation from the business manager of Monarch Books

Since its opening in Collegeville twenty years ago Monarch Books has developed a large customer base due to its reader-friendly atmosphere and wide selection of books on all subjects Last month Book and Bean a combination bookstore and coffee shop announced its intention to open a Collegeville store Monarch Books should open its own in-store cafeacute in the space currently devoted to childrens books Given recent national census data indicating a significant decline in the percentage of the population under age ten sales of childrens books are likely to decline By replacing its childrens books section with a cafeacute Monarch Books can increase profits and ward off competition from Book and Bean

Write a response in which you examine the stated andor unstated assumptions of the argument Be sure to explain how the argument depends on these assumptions and what the implications are for the argument if the assumptions prove unwarranted

The following is a recommendation from the business manager of Monarch Books

Since its opening in Collegeville twenty years ago Monarch Books has developed a large customer base due to its reader-friendly atmosphere and wide selection of books on all subjects Last month Book and Bean a combination bookstore and coffee shop announced its intention to open a Collegeville store Monarch Books should open its own in-store cafeacute in the space currently devoted to childrens books Given recent national census data indicating a significant decline in the percentage of the population under age ten sales of childrens books are likely to decline By replacing its childrens books section with a cafeacute Monarch Books can increase profits and ward off competition from Book and Bean

1 Write a response in which you discuss what questions would need to be answered in order to decide whether the recommendation is likely to have the predicted result Be sure to explain how the answers to these questions would help to evaluate the recommendation

2 Write a response in which you discuss what specific evidence is needed to evaluate the argument and explain how the evidence would weaken or strengthen the argument

No evidence regarding Monarch Bookrsquos successEven if Regal Bookrsquos is successful this may not be attributable to the cafeacute False analogy Emulating may not lead to success Other factors may be involvedInsufficient condition The national census is not enough evidence that childrenrsquos book sales will decline Can

GRE AWA John박 박정어학원

the national census represent the local child populationDid opening a cafeacute boost sales for Regal Books Even assuming Regal is successful by opening a cafeacute this may not be suitable for Monarch which plans to close the childrenrsquos book section to establish a cafe Imprecise language ldquorelatively little spacerdquo how smallThe managerrsquos recommendation contradicts what he says Since Monarch is popular for its wide selection of books closing a selection which targets a major group of readers may hurt Monarchrsquos salesIs this the best way to compete

When Stanley Park first opened it was the largest most heavily used public park in town It is still the largest park but it is no longer heavily used Video cameras mounted in the parks parking lots last month revealed the parks drop in popularity the recordings showed an average of only 50 cars per day In contrast tiny Carlton Park in the heart of the business district is visited by more than 150 people on a typical weekday An obvious difference is that Carlton Park unlike Stanley Park provides ample seating Thus if Stanley Park is ever to be as popular with our citizens as Carlton Park the town will obviously need to provide more benches thereby converting some of the unused open areas into spaces suitable for socializing

Write a response in which you examine the stated andor unstated assumptions of the argument Be sure to explain how the argument depends on these assumptions and what the implications are for the argument if the assumptions prove unwarranted

Stanley 파크가 처음 개장했을 당시 가장 크고 가장 많이 이용되는 공원이었다 아직도 공원중에서는 가장 크지만 이용률은 상당히 떨어졌다 지난달 공원 주차장에 설치해놓은 비디오 카메라를 통해 보면 drop(주차장으로 여겨짐) 이용률이 가장 높았다 수치상으로는 하루 평균 50대의 차량만이 이용하였다 반면 직장 중심거리에 위치한 작은 규모의 Carlton 파크는 주당 무려 150여명 이상이 이용하고 있다 Stanley 파크와는 달리 Carlton 파크에는 의자가 있다는 것이 가장 뚜렷한 차이점이다 따라서 Stanley 파크가 Carlton 파크처럼 시민들이 자주 이용하는 공원이 되기 위해서는 벤치를 설치할 필요가 있으며 이렇게 사용되지 않는 일부 공간을 활용해서 사교를 위한 공간으로 바꾸어야 한다 ===gtdrop 에 대한 첨부사항 (영영사전내용입니다)---- a place or central depository to which something (as mail money or stolen property) is brought for distribution or transmission also the act of depositing something at such a place dropgt

주장 if Stanley Park is ever to be as popular with our citizens as is Carlton Park the town will obviously need to provide more benches thereby converting some of the unused open areas into spaces suitable for socializing1 조사가 언제 이루어진 것인가 조사가 언제 실시되었느냐에 따라 결과가 다를 수있다 현재는 다시 스탠리 파크가 늘어났었을 수 있다 2 벤치를 많이 설치했다고 해서 많은 관광객이 오지 않을수 있다(사람들이 벤치나 사교 공간을 원한다는 어떠한 자료도 없다)3스탠리 파크 주변에 교통 상황이 악화가 되었거나 칼튼 파크에서 문화행사등을 많이 가져서 이용객이 줄어든것일 수도 있다 4 칼튼 파크가 중심지에 있어서 접근성이 좋을수 있다5 조사가 같은 시간을 기준으로 한게 아니다(하나는 주중이고 하나는 주말이다)6사람의 수와 차의 대수를 같은것으로 비교할수 없다 (차안에 몇 명이 타고 있는지 모르고 대중교통을 이용해서 왔을수도 있다)

Page 35: GRE writing argument brain storm

GRE AWA John박 박정어학원

Assumptions The author assumes that OVrsquos municipal golf course and resort hotel caused tourism new businesses and increased tax revenues There may be other reasons advertising promo He assumes that this will continueAssumes that Ocean View and Hopewell are similar in many waysmdashthe name suggests otherwise OV may have always been a tourist attractions for its beaches We need to know the topography

2년전 Ocean View 시는 시정 소유 골프 및 휴양지 호텔을 신축했다 그리고 지난 2년동안 이 시의 관광객이 증가했으며 새로운 사업들이 생겨났다 그에따라 시의 세수도 30나 증가했다 Hopewell의 경제를 향상시키고 아울러 세수를 늘릴 수 있는 가장 좋은 방법은 Ocean View에 세워진 것과 같은 골프 시설과 휴양지 호텔을 신축하는 것이다

1 다른 요인으로 관광 산업이 발전했을 수도 있다 문화 유적이 발견이 되었거나 도로의 정비등으로 여행자가 늘었을 수도 있다

2 관광 산업의증가가 늘어난 세수의 원인이 아니라 새로 유입된 인구의 증가나 다른 공장에서 발생한 것일 수 있다

3 2년동안 한참 골프가 붐을 이루었을 수 있다 경제상황이 나빠지거나 다른 레포츠가 인근 지역에 생겨난다면 골프하는 사람이 줄어들 수 있다

In this memo HopeweUs mayor recommends that in order to stimulate the towns economyand boost tax revenues HopeweU should build a new golf course and resort hotel just as thetown of Ocean View did two years ago To support this recommendation the mayor points outthat in Ocean View during the last two years tourism has increased new businesses haveopened and tax revenues have increased by 30 I find the mayors argument unconvincingin several important respectsFirst of all it is possible that the mayor has confused cause with effect respecting the recentdevelopments in Ocean View Perhaps Ocean Views construction of a new golf course andhotel was a response to previous increases in tourism and business development increasesthat have simply continued during the most recent two years Since the mayor has failed toaccount for this possibility the claim that Hopewell would boost its economy by alsoconstructing a golf course and hotel is completely unwarrantedSecondly the mayor fails to account for other possible causes of the trends in Ocean Viewduring the last two years The increase in tourism might have been due to improving economicconditions nationwide or to unusually pleasant weather in the region The new businessesthat have opened in Ocean View might have opened there irrespective of the new golf courseand hotel And the 30 increase in tax revenues might have been the result of an increase intax rates or the addition of a new type of municipal taxWithout ruling out these and other alternative explanations for the three recent trends inOcean View the mayor cannot reasonably infer based on those trends that Hopewellseconomy would benefit by following Ocean Views exampleThirdly even if the recent trends in Ocean View are attributable to the construction of the newgolf course and hotel there the mayor assumes too hastily that the golf course and hotel willcontinue to benefit that towns overall economy The mayor has not accounted for thepossibility that increased tourism will begin to drive residents away during tourist season orthat new business development will result in the towns losing its appeal as a place to visit or tolive Unless the mayor can convince me that these scenarios are unlikely I cannot accept themayors recommendation that Hopewell follow Ocean Views exampleFinally the mayors argument rests on the unsubstantiated assumption that Hopewell andOcean View are sufficiently alike in ways that might affect the economic impact of a new golfcourse and hotel Hopewell might lack the sort of natural environment that would attract moretourists and new businesses to the town--regardless of its new golf course and hotel For thatmatter perhaps Hopewell already contains several resort hotels and golf courses that are notutilized to their capacity If so building yet another golf course and hotel might amount to amisallocation of the towns resources--and actually harm the towns overall economyIn sum the mayors recommendation is not well supported To bolster it the mayor mustprovide better evidence that Ocean Views new golf course and hotel and not some otherphenomenon--has been responsible for boosting Ocean Views economy during the last twoyears To better assess the recommendation I would need to know why Ocean View decidedto construct its new golf course and hotel in the first place--specifically what events prior toconstruction might have prompted that decision I would also need to thoroughly compare

GRE AWA John박 박정어학원

HopeweU with Ocean View--especially in terms of their appeal to tourists and businesses--todetermine whether the same course of action that appears to have boosted Ocean Viewseconomy would also boost Hopewells economy

The following is part of a memorandum from the president of Humana University

Last year the number of students who enrolled in online degree programs offered by nearby Omni University increased by 50 percent During the same year Omni showed a significant decrease from prior years in expenditures for dormitory and classroom space most likely because instruction in the online programs takes place via the Internet In contrast over the past three years enrollment at Humana University has failed to grow and the cost of maintaining buildings has increased along with our budget deficit To address these problems Humana University will begin immediately to create and actively promote online degree programs like those at Omni We predict that instituting these online degree programs will help Humana both increase its total enrollment and solve its budget problems

Write a response in which you discuss what questions would need to be answered in order to decide whether the prediction and the argument on which it is based are reasonable Be sure to explain how the answers to these questions would help to evaluate the prediction

Is Omni University successful due to the online degree program 50 Is the decrease in expenditures for dormitory and classroom space due to the decrease in of on-campus students Which classes were successful Does HU have those classes

Even if the long-distance degree programs at Omni University benefited the school the presidentrsquos recommendation that Human College should emulate Omni University is too hasty First OUrsquos name implies that the school would have more majors than Humanahellip the president should examine which degrees were in the long-distance programhellip

지난해에는 Omni 대학에서 개강했던 원거리 학생 학점 취득 프로그램을 등록했던 학생들의 숫자가 50나 증가했다 같은해 기간동안 Omni 대학에서는 그 전년도부터 기숙사와 학급의 공간 확충을 위한 예산을 대폭 줄였는데 이는 이 원거리 학점 취득 프로그램이 양방향 비디오 컴퓨터 접속을 통해서만 가능한 수업지도 방식이기때문인 것으로 보인다 반면 지난 3개년 동안 Humana 대학에서의 수강률은 감소한데다가 건물

GRE AWA John박 박정어학원

유지비도 올랐다 따라서 Humana대학의 수강을 늘리고 예산손실을 회복하기 위해서는 Omni 대학에서 취한 조치와 같은 능동적인 프로그램을 추진해야 한다

결론 we should initiate and actively promote long-distance degree programs like those at Omni 반박 원거리 학생 취득 프로그램 숫자가 증가한거하고 예산이 줄어드는 것 사이에 연관이 약하다 (causal 학생의 증가로 관리비용 증가할수 있음 원거리 수업가능 장비도입에의한 비용발생)bad analogy(omni university 하고 같은 조건이 아니다 )-gt omni college 가 강좌내용이 좋아서 학생의 등록이 많을수 있다 Humana 대학에서 만들었다 하더라도 인기 없을수 있음다른 요인에 의해서 Humana 대학의 수강 인원이 증가할수 있음(비록 과거엔 인기가 없었을지라도)

The following appeared as part of a business plan developed by the manager of the Rialto Movie Theater

Despite its downtown location the Rialto Movie Theater a local institution for five decades must make big changes or close its doors forever It should follow the example of the new Apex Theater in the mall outside of town When the Apex opened last year it featured a video arcade plush carpeting and seats and a state-of-the-art sound system Furthermore in a recent survey over 85 percent of respondents reported that the high price of newly released movies prevents them from going to the movies more than five times per year Thus if the Rialto intends to hold on to its share of a decreasing pool of moviegoers it must offer the same features as Apex

Write a response in which you discuss what questions would need to be answered in order to decide whether the recommendation is likely to have the predicted result Be sure to explain how the answers to these questions would help to evaluate the recommendation

Before following through this business plan the manager should investigate the cause of Rialtorsquos unsuccessful business

The author provides no evidence that the surveyrsquos results are statistically reliable The surveyrsquos sample of 85 percent must be sufficient in size and representative of overall population of the city where Rialto and Apex is serving Lacking evidence of a sufficiently representative sample the author cannot justifiably rely on the survey to draw any conclusion whatsoever The author does not indicate that Apex is indeed currently successful However even if Apex is enjoying success the argument relies on what might be a false analogy between Rialto and Apex In order for Apex to serve as a model that Rialto should emulate the author must assume that all relevant circumstances are essentially the same However this assumption is unwarranted For example the argument overlooks the face that Apex is located in a strategic placemdashbeside a mall where customers can not only watch a movie but also enjoy shopping Therefore simply changing the facility to that of Apex may not lead to success

The author does not mention whether Apex is successful or not Nevertheless even if Apex is currently successful the argument relies on what might be a false analogy between Rialto and Apex In order for Apex to serve as a model that Rialto should emulate the author must assume that all relevant circumstances are essentially the same However this assumption is unwarranted For example the argument overlooks the fact that these two institutions are located in different locations Rialto in downtown and Apex in a mall outside of town Although Apex opened with state-of-the-art facilities the decisive factor in its success could be due to its strategic location of being in a mall People could enjoy both shopping and movies at one location thus they may prefer Apex over Rialto Furthermore the place where people enjoy leisure activities has shifted in the past decades for most cities from downtown to the suburbs Therefore Rialto may not be successful even if it emulates Apexrsquos facilities A better business plan may be relocating Apex to the thriving section of the downtown

Rialto 극장은 지난 50여년간 지역 회관으로써 시내에 위치해 있으면서도 이제 변화를 꾀하지 않으면 문을 닫을

GRE AWA John박 박정어학원

판이다 이 극장은 시외 쇼핑타운에 새로 들어선 Apex 극장의 사례를 본받아야 했다 Apex가 지난해 개업했을 당시 이 극장은 비디오 아케이드 플러쉬 카펫트 바닥과 좌석 그리고 최신 음향시설을 갖추었다 더군다나 최근 조사에서는 응답자의 85 이상이 새로 출시된 영화 입장료가 비싼 탓으로 지난해보다 5배이상의 관람객이 줄어들었다고 나타났다 따라서 Rialto 극장이 줄어들고 있는 관람객을 뺐기지 않고 유지하려면 Apex와 같은 시설들을 갖추어야 할 것이다주장 리알토 극장이 줄어들고 있는 관람객을 뺐기지 않고 유지하려면 Apex와 같은 시설들을 갖추어야 할 것이다

1 조사에서 응답자가 전체를 대표할 수 없다 2 apex 극장이 좋은 시설을 갖추고 있지만 그로 인해 수익이 많이 발생했다는 말이 없으므로 시설투자를

하고도 좋은 결과를 얻을 수 있을지 그 근거가 미흡하다3 좋은 영화가 출시된다면 입장료가 비싸도 영화관에서 꼭 보려고 할 수 있다 4 rialto 가 시설이 아닌 다른 요인에 의해 장사가 안될수도 있다( 우범 지역이라든지)

The following is a recommendation from the business manager of Monarch Books

Since its opening in Collegeville twenty years ago Monarch Books has developed a large customer base due to its reader-friendly atmosphere and wide selection of books on all subjects Last month Book and Bean a combination bookstore and coffee shop announced its intention to open a Collegeville store Monarch Books should open its own in-store cafeacute in the space currently devoted to childrens books Given recent national census data indicating a significant decline in the percentage of the population under age ten sales of childrens books are likely to decline By replacing its childrens books section with a cafeacute Monarch Books can increase profits and ward off competition from Book and Bean

Write a response in which you examine the stated andor unstated assumptions of the argument Be sure to explain how the argument depends on these assumptions and what the implications are for the argument if the assumptions prove unwarranted

The following is a recommendation from the business manager of Monarch Books

Since its opening in Collegeville twenty years ago Monarch Books has developed a large customer base due to its reader-friendly atmosphere and wide selection of books on all subjects Last month Book and Bean a combination bookstore and coffee shop announced its intention to open a Collegeville store Monarch Books should open its own in-store cafeacute in the space currently devoted to childrens books Given recent national census data indicating a significant decline in the percentage of the population under age ten sales of childrens books are likely to decline By replacing its childrens books section with a cafeacute Monarch Books can increase profits and ward off competition from Book and Bean

1 Write a response in which you discuss what questions would need to be answered in order to decide whether the recommendation is likely to have the predicted result Be sure to explain how the answers to these questions would help to evaluate the recommendation

2 Write a response in which you discuss what specific evidence is needed to evaluate the argument and explain how the evidence would weaken or strengthen the argument

No evidence regarding Monarch Bookrsquos successEven if Regal Bookrsquos is successful this may not be attributable to the cafeacute False analogy Emulating may not lead to success Other factors may be involvedInsufficient condition The national census is not enough evidence that childrenrsquos book sales will decline Can

GRE AWA John박 박정어학원

the national census represent the local child populationDid opening a cafeacute boost sales for Regal Books Even assuming Regal is successful by opening a cafeacute this may not be suitable for Monarch which plans to close the childrenrsquos book section to establish a cafe Imprecise language ldquorelatively little spacerdquo how smallThe managerrsquos recommendation contradicts what he says Since Monarch is popular for its wide selection of books closing a selection which targets a major group of readers may hurt Monarchrsquos salesIs this the best way to compete

When Stanley Park first opened it was the largest most heavily used public park in town It is still the largest park but it is no longer heavily used Video cameras mounted in the parks parking lots last month revealed the parks drop in popularity the recordings showed an average of only 50 cars per day In contrast tiny Carlton Park in the heart of the business district is visited by more than 150 people on a typical weekday An obvious difference is that Carlton Park unlike Stanley Park provides ample seating Thus if Stanley Park is ever to be as popular with our citizens as Carlton Park the town will obviously need to provide more benches thereby converting some of the unused open areas into spaces suitable for socializing

Write a response in which you examine the stated andor unstated assumptions of the argument Be sure to explain how the argument depends on these assumptions and what the implications are for the argument if the assumptions prove unwarranted

Stanley 파크가 처음 개장했을 당시 가장 크고 가장 많이 이용되는 공원이었다 아직도 공원중에서는 가장 크지만 이용률은 상당히 떨어졌다 지난달 공원 주차장에 설치해놓은 비디오 카메라를 통해 보면 drop(주차장으로 여겨짐) 이용률이 가장 높았다 수치상으로는 하루 평균 50대의 차량만이 이용하였다 반면 직장 중심거리에 위치한 작은 규모의 Carlton 파크는 주당 무려 150여명 이상이 이용하고 있다 Stanley 파크와는 달리 Carlton 파크에는 의자가 있다는 것이 가장 뚜렷한 차이점이다 따라서 Stanley 파크가 Carlton 파크처럼 시민들이 자주 이용하는 공원이 되기 위해서는 벤치를 설치할 필요가 있으며 이렇게 사용되지 않는 일부 공간을 활용해서 사교를 위한 공간으로 바꾸어야 한다 ===gtdrop 에 대한 첨부사항 (영영사전내용입니다)---- a place or central depository to which something (as mail money or stolen property) is brought for distribution or transmission also the act of depositing something at such a place dropgt

주장 if Stanley Park is ever to be as popular with our citizens as is Carlton Park the town will obviously need to provide more benches thereby converting some of the unused open areas into spaces suitable for socializing1 조사가 언제 이루어진 것인가 조사가 언제 실시되었느냐에 따라 결과가 다를 수있다 현재는 다시 스탠리 파크가 늘어났었을 수 있다 2 벤치를 많이 설치했다고 해서 많은 관광객이 오지 않을수 있다(사람들이 벤치나 사교 공간을 원한다는 어떠한 자료도 없다)3스탠리 파크 주변에 교통 상황이 악화가 되었거나 칼튼 파크에서 문화행사등을 많이 가져서 이용객이 줄어든것일 수도 있다 4 칼튼 파크가 중심지에 있어서 접근성이 좋을수 있다5 조사가 같은 시간을 기준으로 한게 아니다(하나는 주중이고 하나는 주말이다)6사람의 수와 차의 대수를 같은것으로 비교할수 없다 (차안에 몇 명이 타고 있는지 모르고 대중교통을 이용해서 왔을수도 있다)

Page 36: GRE writing argument brain storm

GRE AWA John박 박정어학원

HopeweU with Ocean View--especially in terms of their appeal to tourists and businesses--todetermine whether the same course of action that appears to have boosted Ocean Viewseconomy would also boost Hopewells economy

The following is part of a memorandum from the president of Humana University

Last year the number of students who enrolled in online degree programs offered by nearby Omni University increased by 50 percent During the same year Omni showed a significant decrease from prior years in expenditures for dormitory and classroom space most likely because instruction in the online programs takes place via the Internet In contrast over the past three years enrollment at Humana University has failed to grow and the cost of maintaining buildings has increased along with our budget deficit To address these problems Humana University will begin immediately to create and actively promote online degree programs like those at Omni We predict that instituting these online degree programs will help Humana both increase its total enrollment and solve its budget problems

Write a response in which you discuss what questions would need to be answered in order to decide whether the prediction and the argument on which it is based are reasonable Be sure to explain how the answers to these questions would help to evaluate the prediction

Is Omni University successful due to the online degree program 50 Is the decrease in expenditures for dormitory and classroom space due to the decrease in of on-campus students Which classes were successful Does HU have those classes

Even if the long-distance degree programs at Omni University benefited the school the presidentrsquos recommendation that Human College should emulate Omni University is too hasty First OUrsquos name implies that the school would have more majors than Humanahellip the president should examine which degrees were in the long-distance programhellip

지난해에는 Omni 대학에서 개강했던 원거리 학생 학점 취득 프로그램을 등록했던 학생들의 숫자가 50나 증가했다 같은해 기간동안 Omni 대학에서는 그 전년도부터 기숙사와 학급의 공간 확충을 위한 예산을 대폭 줄였는데 이는 이 원거리 학점 취득 프로그램이 양방향 비디오 컴퓨터 접속을 통해서만 가능한 수업지도 방식이기때문인 것으로 보인다 반면 지난 3개년 동안 Humana 대학에서의 수강률은 감소한데다가 건물

GRE AWA John박 박정어학원

유지비도 올랐다 따라서 Humana대학의 수강을 늘리고 예산손실을 회복하기 위해서는 Omni 대학에서 취한 조치와 같은 능동적인 프로그램을 추진해야 한다

결론 we should initiate and actively promote long-distance degree programs like those at Omni 반박 원거리 학생 취득 프로그램 숫자가 증가한거하고 예산이 줄어드는 것 사이에 연관이 약하다 (causal 학생의 증가로 관리비용 증가할수 있음 원거리 수업가능 장비도입에의한 비용발생)bad analogy(omni university 하고 같은 조건이 아니다 )-gt omni college 가 강좌내용이 좋아서 학생의 등록이 많을수 있다 Humana 대학에서 만들었다 하더라도 인기 없을수 있음다른 요인에 의해서 Humana 대학의 수강 인원이 증가할수 있음(비록 과거엔 인기가 없었을지라도)

The following appeared as part of a business plan developed by the manager of the Rialto Movie Theater

Despite its downtown location the Rialto Movie Theater a local institution for five decades must make big changes or close its doors forever It should follow the example of the new Apex Theater in the mall outside of town When the Apex opened last year it featured a video arcade plush carpeting and seats and a state-of-the-art sound system Furthermore in a recent survey over 85 percent of respondents reported that the high price of newly released movies prevents them from going to the movies more than five times per year Thus if the Rialto intends to hold on to its share of a decreasing pool of moviegoers it must offer the same features as Apex

Write a response in which you discuss what questions would need to be answered in order to decide whether the recommendation is likely to have the predicted result Be sure to explain how the answers to these questions would help to evaluate the recommendation

Before following through this business plan the manager should investigate the cause of Rialtorsquos unsuccessful business

The author provides no evidence that the surveyrsquos results are statistically reliable The surveyrsquos sample of 85 percent must be sufficient in size and representative of overall population of the city where Rialto and Apex is serving Lacking evidence of a sufficiently representative sample the author cannot justifiably rely on the survey to draw any conclusion whatsoever The author does not indicate that Apex is indeed currently successful However even if Apex is enjoying success the argument relies on what might be a false analogy between Rialto and Apex In order for Apex to serve as a model that Rialto should emulate the author must assume that all relevant circumstances are essentially the same However this assumption is unwarranted For example the argument overlooks the face that Apex is located in a strategic placemdashbeside a mall where customers can not only watch a movie but also enjoy shopping Therefore simply changing the facility to that of Apex may not lead to success

The author does not mention whether Apex is successful or not Nevertheless even if Apex is currently successful the argument relies on what might be a false analogy between Rialto and Apex In order for Apex to serve as a model that Rialto should emulate the author must assume that all relevant circumstances are essentially the same However this assumption is unwarranted For example the argument overlooks the fact that these two institutions are located in different locations Rialto in downtown and Apex in a mall outside of town Although Apex opened with state-of-the-art facilities the decisive factor in its success could be due to its strategic location of being in a mall People could enjoy both shopping and movies at one location thus they may prefer Apex over Rialto Furthermore the place where people enjoy leisure activities has shifted in the past decades for most cities from downtown to the suburbs Therefore Rialto may not be successful even if it emulates Apexrsquos facilities A better business plan may be relocating Apex to the thriving section of the downtown

Rialto 극장은 지난 50여년간 지역 회관으로써 시내에 위치해 있으면서도 이제 변화를 꾀하지 않으면 문을 닫을

GRE AWA John박 박정어학원

판이다 이 극장은 시외 쇼핑타운에 새로 들어선 Apex 극장의 사례를 본받아야 했다 Apex가 지난해 개업했을 당시 이 극장은 비디오 아케이드 플러쉬 카펫트 바닥과 좌석 그리고 최신 음향시설을 갖추었다 더군다나 최근 조사에서는 응답자의 85 이상이 새로 출시된 영화 입장료가 비싼 탓으로 지난해보다 5배이상의 관람객이 줄어들었다고 나타났다 따라서 Rialto 극장이 줄어들고 있는 관람객을 뺐기지 않고 유지하려면 Apex와 같은 시설들을 갖추어야 할 것이다주장 리알토 극장이 줄어들고 있는 관람객을 뺐기지 않고 유지하려면 Apex와 같은 시설들을 갖추어야 할 것이다

1 조사에서 응답자가 전체를 대표할 수 없다 2 apex 극장이 좋은 시설을 갖추고 있지만 그로 인해 수익이 많이 발생했다는 말이 없으므로 시설투자를

하고도 좋은 결과를 얻을 수 있을지 그 근거가 미흡하다3 좋은 영화가 출시된다면 입장료가 비싸도 영화관에서 꼭 보려고 할 수 있다 4 rialto 가 시설이 아닌 다른 요인에 의해 장사가 안될수도 있다( 우범 지역이라든지)

The following is a recommendation from the business manager of Monarch Books

Since its opening in Collegeville twenty years ago Monarch Books has developed a large customer base due to its reader-friendly atmosphere and wide selection of books on all subjects Last month Book and Bean a combination bookstore and coffee shop announced its intention to open a Collegeville store Monarch Books should open its own in-store cafeacute in the space currently devoted to childrens books Given recent national census data indicating a significant decline in the percentage of the population under age ten sales of childrens books are likely to decline By replacing its childrens books section with a cafeacute Monarch Books can increase profits and ward off competition from Book and Bean

Write a response in which you examine the stated andor unstated assumptions of the argument Be sure to explain how the argument depends on these assumptions and what the implications are for the argument if the assumptions prove unwarranted

The following is a recommendation from the business manager of Monarch Books

Since its opening in Collegeville twenty years ago Monarch Books has developed a large customer base due to its reader-friendly atmosphere and wide selection of books on all subjects Last month Book and Bean a combination bookstore and coffee shop announced its intention to open a Collegeville store Monarch Books should open its own in-store cafeacute in the space currently devoted to childrens books Given recent national census data indicating a significant decline in the percentage of the population under age ten sales of childrens books are likely to decline By replacing its childrens books section with a cafeacute Monarch Books can increase profits and ward off competition from Book and Bean

1 Write a response in which you discuss what questions would need to be answered in order to decide whether the recommendation is likely to have the predicted result Be sure to explain how the answers to these questions would help to evaluate the recommendation

2 Write a response in which you discuss what specific evidence is needed to evaluate the argument and explain how the evidence would weaken or strengthen the argument

No evidence regarding Monarch Bookrsquos successEven if Regal Bookrsquos is successful this may not be attributable to the cafeacute False analogy Emulating may not lead to success Other factors may be involvedInsufficient condition The national census is not enough evidence that childrenrsquos book sales will decline Can

GRE AWA John박 박정어학원

the national census represent the local child populationDid opening a cafeacute boost sales for Regal Books Even assuming Regal is successful by opening a cafeacute this may not be suitable for Monarch which plans to close the childrenrsquos book section to establish a cafe Imprecise language ldquorelatively little spacerdquo how smallThe managerrsquos recommendation contradicts what he says Since Monarch is popular for its wide selection of books closing a selection which targets a major group of readers may hurt Monarchrsquos salesIs this the best way to compete

When Stanley Park first opened it was the largest most heavily used public park in town It is still the largest park but it is no longer heavily used Video cameras mounted in the parks parking lots last month revealed the parks drop in popularity the recordings showed an average of only 50 cars per day In contrast tiny Carlton Park in the heart of the business district is visited by more than 150 people on a typical weekday An obvious difference is that Carlton Park unlike Stanley Park provides ample seating Thus if Stanley Park is ever to be as popular with our citizens as Carlton Park the town will obviously need to provide more benches thereby converting some of the unused open areas into spaces suitable for socializing

Write a response in which you examine the stated andor unstated assumptions of the argument Be sure to explain how the argument depends on these assumptions and what the implications are for the argument if the assumptions prove unwarranted

Stanley 파크가 처음 개장했을 당시 가장 크고 가장 많이 이용되는 공원이었다 아직도 공원중에서는 가장 크지만 이용률은 상당히 떨어졌다 지난달 공원 주차장에 설치해놓은 비디오 카메라를 통해 보면 drop(주차장으로 여겨짐) 이용률이 가장 높았다 수치상으로는 하루 평균 50대의 차량만이 이용하였다 반면 직장 중심거리에 위치한 작은 규모의 Carlton 파크는 주당 무려 150여명 이상이 이용하고 있다 Stanley 파크와는 달리 Carlton 파크에는 의자가 있다는 것이 가장 뚜렷한 차이점이다 따라서 Stanley 파크가 Carlton 파크처럼 시민들이 자주 이용하는 공원이 되기 위해서는 벤치를 설치할 필요가 있으며 이렇게 사용되지 않는 일부 공간을 활용해서 사교를 위한 공간으로 바꾸어야 한다 ===gtdrop 에 대한 첨부사항 (영영사전내용입니다)---- a place or central depository to which something (as mail money or stolen property) is brought for distribution or transmission also the act of depositing something at such a place dropgt

주장 if Stanley Park is ever to be as popular with our citizens as is Carlton Park the town will obviously need to provide more benches thereby converting some of the unused open areas into spaces suitable for socializing1 조사가 언제 이루어진 것인가 조사가 언제 실시되었느냐에 따라 결과가 다를 수있다 현재는 다시 스탠리 파크가 늘어났었을 수 있다 2 벤치를 많이 설치했다고 해서 많은 관광객이 오지 않을수 있다(사람들이 벤치나 사교 공간을 원한다는 어떠한 자료도 없다)3스탠리 파크 주변에 교통 상황이 악화가 되었거나 칼튼 파크에서 문화행사등을 많이 가져서 이용객이 줄어든것일 수도 있다 4 칼튼 파크가 중심지에 있어서 접근성이 좋을수 있다5 조사가 같은 시간을 기준으로 한게 아니다(하나는 주중이고 하나는 주말이다)6사람의 수와 차의 대수를 같은것으로 비교할수 없다 (차안에 몇 명이 타고 있는지 모르고 대중교통을 이용해서 왔을수도 있다)

Page 37: GRE writing argument brain storm

GRE AWA John박 박정어학원

유지비도 올랐다 따라서 Humana대학의 수강을 늘리고 예산손실을 회복하기 위해서는 Omni 대학에서 취한 조치와 같은 능동적인 프로그램을 추진해야 한다

결론 we should initiate and actively promote long-distance degree programs like those at Omni 반박 원거리 학생 취득 프로그램 숫자가 증가한거하고 예산이 줄어드는 것 사이에 연관이 약하다 (causal 학생의 증가로 관리비용 증가할수 있음 원거리 수업가능 장비도입에의한 비용발생)bad analogy(omni university 하고 같은 조건이 아니다 )-gt omni college 가 강좌내용이 좋아서 학생의 등록이 많을수 있다 Humana 대학에서 만들었다 하더라도 인기 없을수 있음다른 요인에 의해서 Humana 대학의 수강 인원이 증가할수 있음(비록 과거엔 인기가 없었을지라도)

The following appeared as part of a business plan developed by the manager of the Rialto Movie Theater

Despite its downtown location the Rialto Movie Theater a local institution for five decades must make big changes or close its doors forever It should follow the example of the new Apex Theater in the mall outside of town When the Apex opened last year it featured a video arcade plush carpeting and seats and a state-of-the-art sound system Furthermore in a recent survey over 85 percent of respondents reported that the high price of newly released movies prevents them from going to the movies more than five times per year Thus if the Rialto intends to hold on to its share of a decreasing pool of moviegoers it must offer the same features as Apex

Write a response in which you discuss what questions would need to be answered in order to decide whether the recommendation is likely to have the predicted result Be sure to explain how the answers to these questions would help to evaluate the recommendation

Before following through this business plan the manager should investigate the cause of Rialtorsquos unsuccessful business

The author provides no evidence that the surveyrsquos results are statistically reliable The surveyrsquos sample of 85 percent must be sufficient in size and representative of overall population of the city where Rialto and Apex is serving Lacking evidence of a sufficiently representative sample the author cannot justifiably rely on the survey to draw any conclusion whatsoever The author does not indicate that Apex is indeed currently successful However even if Apex is enjoying success the argument relies on what might be a false analogy between Rialto and Apex In order for Apex to serve as a model that Rialto should emulate the author must assume that all relevant circumstances are essentially the same However this assumption is unwarranted For example the argument overlooks the face that Apex is located in a strategic placemdashbeside a mall where customers can not only watch a movie but also enjoy shopping Therefore simply changing the facility to that of Apex may not lead to success

The author does not mention whether Apex is successful or not Nevertheless even if Apex is currently successful the argument relies on what might be a false analogy between Rialto and Apex In order for Apex to serve as a model that Rialto should emulate the author must assume that all relevant circumstances are essentially the same However this assumption is unwarranted For example the argument overlooks the fact that these two institutions are located in different locations Rialto in downtown and Apex in a mall outside of town Although Apex opened with state-of-the-art facilities the decisive factor in its success could be due to its strategic location of being in a mall People could enjoy both shopping and movies at one location thus they may prefer Apex over Rialto Furthermore the place where people enjoy leisure activities has shifted in the past decades for most cities from downtown to the suburbs Therefore Rialto may not be successful even if it emulates Apexrsquos facilities A better business plan may be relocating Apex to the thriving section of the downtown

Rialto 극장은 지난 50여년간 지역 회관으로써 시내에 위치해 있으면서도 이제 변화를 꾀하지 않으면 문을 닫을

GRE AWA John박 박정어학원

판이다 이 극장은 시외 쇼핑타운에 새로 들어선 Apex 극장의 사례를 본받아야 했다 Apex가 지난해 개업했을 당시 이 극장은 비디오 아케이드 플러쉬 카펫트 바닥과 좌석 그리고 최신 음향시설을 갖추었다 더군다나 최근 조사에서는 응답자의 85 이상이 새로 출시된 영화 입장료가 비싼 탓으로 지난해보다 5배이상의 관람객이 줄어들었다고 나타났다 따라서 Rialto 극장이 줄어들고 있는 관람객을 뺐기지 않고 유지하려면 Apex와 같은 시설들을 갖추어야 할 것이다주장 리알토 극장이 줄어들고 있는 관람객을 뺐기지 않고 유지하려면 Apex와 같은 시설들을 갖추어야 할 것이다

1 조사에서 응답자가 전체를 대표할 수 없다 2 apex 극장이 좋은 시설을 갖추고 있지만 그로 인해 수익이 많이 발생했다는 말이 없으므로 시설투자를

하고도 좋은 결과를 얻을 수 있을지 그 근거가 미흡하다3 좋은 영화가 출시된다면 입장료가 비싸도 영화관에서 꼭 보려고 할 수 있다 4 rialto 가 시설이 아닌 다른 요인에 의해 장사가 안될수도 있다( 우범 지역이라든지)

The following is a recommendation from the business manager of Monarch Books

Since its opening in Collegeville twenty years ago Monarch Books has developed a large customer base due to its reader-friendly atmosphere and wide selection of books on all subjects Last month Book and Bean a combination bookstore and coffee shop announced its intention to open a Collegeville store Monarch Books should open its own in-store cafeacute in the space currently devoted to childrens books Given recent national census data indicating a significant decline in the percentage of the population under age ten sales of childrens books are likely to decline By replacing its childrens books section with a cafeacute Monarch Books can increase profits and ward off competition from Book and Bean

Write a response in which you examine the stated andor unstated assumptions of the argument Be sure to explain how the argument depends on these assumptions and what the implications are for the argument if the assumptions prove unwarranted

The following is a recommendation from the business manager of Monarch Books

Since its opening in Collegeville twenty years ago Monarch Books has developed a large customer base due to its reader-friendly atmosphere and wide selection of books on all subjects Last month Book and Bean a combination bookstore and coffee shop announced its intention to open a Collegeville store Monarch Books should open its own in-store cafeacute in the space currently devoted to childrens books Given recent national census data indicating a significant decline in the percentage of the population under age ten sales of childrens books are likely to decline By replacing its childrens books section with a cafeacute Monarch Books can increase profits and ward off competition from Book and Bean

1 Write a response in which you discuss what questions would need to be answered in order to decide whether the recommendation is likely to have the predicted result Be sure to explain how the answers to these questions would help to evaluate the recommendation

2 Write a response in which you discuss what specific evidence is needed to evaluate the argument and explain how the evidence would weaken or strengthen the argument

No evidence regarding Monarch Bookrsquos successEven if Regal Bookrsquos is successful this may not be attributable to the cafeacute False analogy Emulating may not lead to success Other factors may be involvedInsufficient condition The national census is not enough evidence that childrenrsquos book sales will decline Can

GRE AWA John박 박정어학원

the national census represent the local child populationDid opening a cafeacute boost sales for Regal Books Even assuming Regal is successful by opening a cafeacute this may not be suitable for Monarch which plans to close the childrenrsquos book section to establish a cafe Imprecise language ldquorelatively little spacerdquo how smallThe managerrsquos recommendation contradicts what he says Since Monarch is popular for its wide selection of books closing a selection which targets a major group of readers may hurt Monarchrsquos salesIs this the best way to compete

When Stanley Park first opened it was the largest most heavily used public park in town It is still the largest park but it is no longer heavily used Video cameras mounted in the parks parking lots last month revealed the parks drop in popularity the recordings showed an average of only 50 cars per day In contrast tiny Carlton Park in the heart of the business district is visited by more than 150 people on a typical weekday An obvious difference is that Carlton Park unlike Stanley Park provides ample seating Thus if Stanley Park is ever to be as popular with our citizens as Carlton Park the town will obviously need to provide more benches thereby converting some of the unused open areas into spaces suitable for socializing

Write a response in which you examine the stated andor unstated assumptions of the argument Be sure to explain how the argument depends on these assumptions and what the implications are for the argument if the assumptions prove unwarranted

Stanley 파크가 처음 개장했을 당시 가장 크고 가장 많이 이용되는 공원이었다 아직도 공원중에서는 가장 크지만 이용률은 상당히 떨어졌다 지난달 공원 주차장에 설치해놓은 비디오 카메라를 통해 보면 drop(주차장으로 여겨짐) 이용률이 가장 높았다 수치상으로는 하루 평균 50대의 차량만이 이용하였다 반면 직장 중심거리에 위치한 작은 규모의 Carlton 파크는 주당 무려 150여명 이상이 이용하고 있다 Stanley 파크와는 달리 Carlton 파크에는 의자가 있다는 것이 가장 뚜렷한 차이점이다 따라서 Stanley 파크가 Carlton 파크처럼 시민들이 자주 이용하는 공원이 되기 위해서는 벤치를 설치할 필요가 있으며 이렇게 사용되지 않는 일부 공간을 활용해서 사교를 위한 공간으로 바꾸어야 한다 ===gtdrop 에 대한 첨부사항 (영영사전내용입니다)---- a place or central depository to which something (as mail money or stolen property) is brought for distribution or transmission also the act of depositing something at such a place dropgt

주장 if Stanley Park is ever to be as popular with our citizens as is Carlton Park the town will obviously need to provide more benches thereby converting some of the unused open areas into spaces suitable for socializing1 조사가 언제 이루어진 것인가 조사가 언제 실시되었느냐에 따라 결과가 다를 수있다 현재는 다시 스탠리 파크가 늘어났었을 수 있다 2 벤치를 많이 설치했다고 해서 많은 관광객이 오지 않을수 있다(사람들이 벤치나 사교 공간을 원한다는 어떠한 자료도 없다)3스탠리 파크 주변에 교통 상황이 악화가 되었거나 칼튼 파크에서 문화행사등을 많이 가져서 이용객이 줄어든것일 수도 있다 4 칼튼 파크가 중심지에 있어서 접근성이 좋을수 있다5 조사가 같은 시간을 기준으로 한게 아니다(하나는 주중이고 하나는 주말이다)6사람의 수와 차의 대수를 같은것으로 비교할수 없다 (차안에 몇 명이 타고 있는지 모르고 대중교통을 이용해서 왔을수도 있다)

Page 38: GRE writing argument brain storm

GRE AWA John박 박정어학원

판이다 이 극장은 시외 쇼핑타운에 새로 들어선 Apex 극장의 사례를 본받아야 했다 Apex가 지난해 개업했을 당시 이 극장은 비디오 아케이드 플러쉬 카펫트 바닥과 좌석 그리고 최신 음향시설을 갖추었다 더군다나 최근 조사에서는 응답자의 85 이상이 새로 출시된 영화 입장료가 비싼 탓으로 지난해보다 5배이상의 관람객이 줄어들었다고 나타났다 따라서 Rialto 극장이 줄어들고 있는 관람객을 뺐기지 않고 유지하려면 Apex와 같은 시설들을 갖추어야 할 것이다주장 리알토 극장이 줄어들고 있는 관람객을 뺐기지 않고 유지하려면 Apex와 같은 시설들을 갖추어야 할 것이다

1 조사에서 응답자가 전체를 대표할 수 없다 2 apex 극장이 좋은 시설을 갖추고 있지만 그로 인해 수익이 많이 발생했다는 말이 없으므로 시설투자를

하고도 좋은 결과를 얻을 수 있을지 그 근거가 미흡하다3 좋은 영화가 출시된다면 입장료가 비싸도 영화관에서 꼭 보려고 할 수 있다 4 rialto 가 시설이 아닌 다른 요인에 의해 장사가 안될수도 있다( 우범 지역이라든지)

The following is a recommendation from the business manager of Monarch Books

Since its opening in Collegeville twenty years ago Monarch Books has developed a large customer base due to its reader-friendly atmosphere and wide selection of books on all subjects Last month Book and Bean a combination bookstore and coffee shop announced its intention to open a Collegeville store Monarch Books should open its own in-store cafeacute in the space currently devoted to childrens books Given recent national census data indicating a significant decline in the percentage of the population under age ten sales of childrens books are likely to decline By replacing its childrens books section with a cafeacute Monarch Books can increase profits and ward off competition from Book and Bean

Write a response in which you examine the stated andor unstated assumptions of the argument Be sure to explain how the argument depends on these assumptions and what the implications are for the argument if the assumptions prove unwarranted

The following is a recommendation from the business manager of Monarch Books

Since its opening in Collegeville twenty years ago Monarch Books has developed a large customer base due to its reader-friendly atmosphere and wide selection of books on all subjects Last month Book and Bean a combination bookstore and coffee shop announced its intention to open a Collegeville store Monarch Books should open its own in-store cafeacute in the space currently devoted to childrens books Given recent national census data indicating a significant decline in the percentage of the population under age ten sales of childrens books are likely to decline By replacing its childrens books section with a cafeacute Monarch Books can increase profits and ward off competition from Book and Bean

1 Write a response in which you discuss what questions would need to be answered in order to decide whether the recommendation is likely to have the predicted result Be sure to explain how the answers to these questions would help to evaluate the recommendation

2 Write a response in which you discuss what specific evidence is needed to evaluate the argument and explain how the evidence would weaken or strengthen the argument

No evidence regarding Monarch Bookrsquos successEven if Regal Bookrsquos is successful this may not be attributable to the cafeacute False analogy Emulating may not lead to success Other factors may be involvedInsufficient condition The national census is not enough evidence that childrenrsquos book sales will decline Can

GRE AWA John박 박정어학원

the national census represent the local child populationDid opening a cafeacute boost sales for Regal Books Even assuming Regal is successful by opening a cafeacute this may not be suitable for Monarch which plans to close the childrenrsquos book section to establish a cafe Imprecise language ldquorelatively little spacerdquo how smallThe managerrsquos recommendation contradicts what he says Since Monarch is popular for its wide selection of books closing a selection which targets a major group of readers may hurt Monarchrsquos salesIs this the best way to compete

When Stanley Park first opened it was the largest most heavily used public park in town It is still the largest park but it is no longer heavily used Video cameras mounted in the parks parking lots last month revealed the parks drop in popularity the recordings showed an average of only 50 cars per day In contrast tiny Carlton Park in the heart of the business district is visited by more than 150 people on a typical weekday An obvious difference is that Carlton Park unlike Stanley Park provides ample seating Thus if Stanley Park is ever to be as popular with our citizens as Carlton Park the town will obviously need to provide more benches thereby converting some of the unused open areas into spaces suitable for socializing

Write a response in which you examine the stated andor unstated assumptions of the argument Be sure to explain how the argument depends on these assumptions and what the implications are for the argument if the assumptions prove unwarranted

Stanley 파크가 처음 개장했을 당시 가장 크고 가장 많이 이용되는 공원이었다 아직도 공원중에서는 가장 크지만 이용률은 상당히 떨어졌다 지난달 공원 주차장에 설치해놓은 비디오 카메라를 통해 보면 drop(주차장으로 여겨짐) 이용률이 가장 높았다 수치상으로는 하루 평균 50대의 차량만이 이용하였다 반면 직장 중심거리에 위치한 작은 규모의 Carlton 파크는 주당 무려 150여명 이상이 이용하고 있다 Stanley 파크와는 달리 Carlton 파크에는 의자가 있다는 것이 가장 뚜렷한 차이점이다 따라서 Stanley 파크가 Carlton 파크처럼 시민들이 자주 이용하는 공원이 되기 위해서는 벤치를 설치할 필요가 있으며 이렇게 사용되지 않는 일부 공간을 활용해서 사교를 위한 공간으로 바꾸어야 한다 ===gtdrop 에 대한 첨부사항 (영영사전내용입니다)---- a place or central depository to which something (as mail money or stolen property) is brought for distribution or transmission also the act of depositing something at such a place dropgt

주장 if Stanley Park is ever to be as popular with our citizens as is Carlton Park the town will obviously need to provide more benches thereby converting some of the unused open areas into spaces suitable for socializing1 조사가 언제 이루어진 것인가 조사가 언제 실시되었느냐에 따라 결과가 다를 수있다 현재는 다시 스탠리 파크가 늘어났었을 수 있다 2 벤치를 많이 설치했다고 해서 많은 관광객이 오지 않을수 있다(사람들이 벤치나 사교 공간을 원한다는 어떠한 자료도 없다)3스탠리 파크 주변에 교통 상황이 악화가 되었거나 칼튼 파크에서 문화행사등을 많이 가져서 이용객이 줄어든것일 수도 있다 4 칼튼 파크가 중심지에 있어서 접근성이 좋을수 있다5 조사가 같은 시간을 기준으로 한게 아니다(하나는 주중이고 하나는 주말이다)6사람의 수와 차의 대수를 같은것으로 비교할수 없다 (차안에 몇 명이 타고 있는지 모르고 대중교통을 이용해서 왔을수도 있다)

Page 39: GRE writing argument brain storm

GRE AWA John박 박정어학원

the national census represent the local child populationDid opening a cafeacute boost sales for Regal Books Even assuming Regal is successful by opening a cafeacute this may not be suitable for Monarch which plans to close the childrenrsquos book section to establish a cafe Imprecise language ldquorelatively little spacerdquo how smallThe managerrsquos recommendation contradicts what he says Since Monarch is popular for its wide selection of books closing a selection which targets a major group of readers may hurt Monarchrsquos salesIs this the best way to compete

When Stanley Park first opened it was the largest most heavily used public park in town It is still the largest park but it is no longer heavily used Video cameras mounted in the parks parking lots last month revealed the parks drop in popularity the recordings showed an average of only 50 cars per day In contrast tiny Carlton Park in the heart of the business district is visited by more than 150 people on a typical weekday An obvious difference is that Carlton Park unlike Stanley Park provides ample seating Thus if Stanley Park is ever to be as popular with our citizens as Carlton Park the town will obviously need to provide more benches thereby converting some of the unused open areas into spaces suitable for socializing

Write a response in which you examine the stated andor unstated assumptions of the argument Be sure to explain how the argument depends on these assumptions and what the implications are for the argument if the assumptions prove unwarranted

Stanley 파크가 처음 개장했을 당시 가장 크고 가장 많이 이용되는 공원이었다 아직도 공원중에서는 가장 크지만 이용률은 상당히 떨어졌다 지난달 공원 주차장에 설치해놓은 비디오 카메라를 통해 보면 drop(주차장으로 여겨짐) 이용률이 가장 높았다 수치상으로는 하루 평균 50대의 차량만이 이용하였다 반면 직장 중심거리에 위치한 작은 규모의 Carlton 파크는 주당 무려 150여명 이상이 이용하고 있다 Stanley 파크와는 달리 Carlton 파크에는 의자가 있다는 것이 가장 뚜렷한 차이점이다 따라서 Stanley 파크가 Carlton 파크처럼 시민들이 자주 이용하는 공원이 되기 위해서는 벤치를 설치할 필요가 있으며 이렇게 사용되지 않는 일부 공간을 활용해서 사교를 위한 공간으로 바꾸어야 한다 ===gtdrop 에 대한 첨부사항 (영영사전내용입니다)---- a place or central depository to which something (as mail money or stolen property) is brought for distribution or transmission also the act of depositing something at such a place dropgt

주장 if Stanley Park is ever to be as popular with our citizens as is Carlton Park the town will obviously need to provide more benches thereby converting some of the unused open areas into spaces suitable for socializing1 조사가 언제 이루어진 것인가 조사가 언제 실시되었느냐에 따라 결과가 다를 수있다 현재는 다시 스탠리 파크가 늘어났었을 수 있다 2 벤치를 많이 설치했다고 해서 많은 관광객이 오지 않을수 있다(사람들이 벤치나 사교 공간을 원한다는 어떠한 자료도 없다)3스탠리 파크 주변에 교통 상황이 악화가 되었거나 칼튼 파크에서 문화행사등을 많이 가져서 이용객이 줄어든것일 수도 있다 4 칼튼 파크가 중심지에 있어서 접근성이 좋을수 있다5 조사가 같은 시간을 기준으로 한게 아니다(하나는 주중이고 하나는 주말이다)6사람의 수와 차의 대수를 같은것으로 비교할수 없다 (차안에 몇 명이 타고 있는지 모르고 대중교통을 이용해서 왔을수도 있다)